You are on page 1of 351

http://www.xtremepapers.

net

About Thomson Petersons


Thomson Petersons (www.petersons.com) is a leading provider of education information and advice, with books
and online resources focusing on education search, test preparation, and financial aid. Its Web site offers
searchable databases and interactive tools for contacting educational institutions, online practice tests and
instruction, and planning tools for securing financial aid. Thomson Petersons serves 110 million education
consumers annually.

Petersons.com/publishing
Check out our Web site at www.petersons.com/publishing to see if there is any new information regarding the test
and any revisions or corrections to the content of this book. Weve made sure the information in this book is
accurate and up-to-date; however, the test format or content may have changed since the time of publication.

For more information, contact Thomson Petersons, 2000 Lenox Drive, Lawrenceville, NJ 08648;
800-338-3282; or find us on the World Wide Web at www.petersons.com/about.
2004 Thomson Petersons, a part of The Thomson Corporation
Thomson Learning is a trademark used herein under license.
Chapter 5: GRE Word List is excerpted from The Insiders Guide to the GRE CAT.
2nd Edition, by Karl Weber. Petersons, 2002.
Editor: Mandie Rosenberg; Production Editor: Megan Hellerman; Manufacturing
Manager: Ray Golaszewski; Composition Manager: Gary Rozmierski; Cover and
Interior Design: Allison Sullivan; CD Producer: Carol Aickley; CD Quality
Assurance: Jeff Pagano.
Contributing Writer: Keith Cox
ALL RIGHTS RESERVED. No part of this work covered by the copyright herein
may be reproduced or used in any form or by any meansgraphic, electronic, or
mechanical, including photocopying, recording, taping, Web distribution, or
information storage and retrieval systemswithout the prior written permission of
the publisher.
For permission to use material from this text or product, submit a request online
at www.thomsonrights.com
Any additional questions about permissions can be submitted by e-mail to
thomsonrights@thomson.com
ISBN 0-7689-1432-9
Printed in the United States of America
10

6 5

06

05

04

First Edition

http://www.xtremepapers.net

Contents
123456789012345678901234567890121234567890123456789012
12345678901234567890123456789012123456789012345678901 2
2
12345678901234567890123456789012123456789012345678901
12345678901234567890123456789012123456789012345678901 2
2345678901234567890123456789012123456789012345678901
1
ABOUT
THIS TOOL KIT. . . . . . . . . . . . . . . . . . . . . . .
V 2
2
12345678901234567890123456789012123456789012345678901
2
12345678901234567890123456789012123456789012345678901
Your Tools . . . . . . . . . . . . . . . . . . . . . . . . . . . . . . . . . .
v 2
12345678901234567890123456789012123456789012345678901
2345678901234567890123456789012123456789012345678901
2
1
12345678901234567890123456789012123456789012345678901
Structure of the GRE CAT . . . . . . . . . . . . . . . . . . . . .
vii 2
2
12345678901234567890123456789012123456789012345678901
2
12345678901234567890123456789012123456789012345678901
2
12345678901234567890123456789012123456789012345678901
2345678901234567890123456789012123456789012345678901
1
INTRODUCTION
..............................
1 2
2
12345678901234567890123456789012123456789012345678901
2
12345678901234567890123456789012123456789012345678901
Does This CAT Have Claws? . . . . . . . . . . . . . . . . . . . . . .
1 2
12345678901234567890123456789012123456789012345678901
2
12345678901234567890123456789012123456789012345678901
The Unique Petersons ApproachAdaptive
2
12345678901234567890123456789012123456789012345678901
2
12345678901234567890123456789012123456789012345678901
Preparation for an Adaptive Test. . . . . . . . . . . . . . . .
3 2
12345678901234567890123456789012123456789012345678901
2
12345678901234567890123456789012123456789012345678901
2
12345678901234567890123456789012123456789012345678901
2345678901234567890123456789012123456789012345678901
2
1
2345678901234567890123456789012123456789012345678901
1
HOW
TO USE THIS BOOK . . . . . . . . . . . . . . . . . . . .
4 2
2
12345678901234567890123456789012123456789012345678901
2
12345678901234567890123456789012123456789012345678901
2
12345678901234567890123456789012123456789012345678901
2345678901234567890123456789012123456789012345678901
12345678901234567890123456789012123456789012345678901 22
1
PART
I: UNDERSTANDING THE GRE
2
12345678901234567890123456789012123456789012345678901
2
12345678901234567890123456789012123456789012345678901
Chapter
1:
Everything
You
Need
to
Know
about
2
12345678901234567890123456789012123456789012345678901
2345678901234567890123456789012123456789012345678901
12345678901234567890123456789012123456789012345678901 22
1
the
GRE
CAT
.
.
.
.
.
.
.
.
.
.
.
.
.
.
.
.
.
.
.
.
.
.
.
.
.
.
.
.
6
2
12345678901234567890123456789012123456789012345678901
2
12345678901234567890123456789012123456789012345678901
Chapter
2:
Section
Strategies
.
.
.
.
.
.
.
.
.
.
.
.
.
.
.
.
.
19
2
12345678901234567890123456789012123456789012345678901
2345678901234567890123456789012123456789012345678901
12345678901234567890123456789012123456789012345678901 22
12345678901234567890123456789012123456789012345678901 2
12345678901234567890123456789012123456789012345678901 2
12345678901234567890123456789012123456789012345678901
PART
II: DIAGNOSING STRENGTHS AND
2
12345678901234567890123456789012123456789012345678901
2
12345678901234567890123456789012123456789012345678901
WEAKNESSES
2
12345678901234567890123456789012123456789012345678901
2
12345678901234567890123456789012123456789012345678901
Chapter
3:
Diagnostic:
Dipping
Your
Big
Toe
in
12345678901234567890123456789012123456789012345678901 22
1
12345678901234567890123456789012123456789012345678901
the GRE Waters . . . . . . . . . . . . . . . . . . . . . . . . . .
32 2
2
12345678901234567890123456789012123456789012345678901
12345678901234567890123456789012123456789012345678901 2
12345678901234567890123456789012123456789012345678901 2
2
12345678901234567890123456789012123456789012345678901
PART
III: SECTION REVIEW
2
12345678901234567890123456789012123456789012345678901
2
12345678901234567890123456789012123456789012345678901
2
12345678901234567890123456789012123456789012345678901
Chapter
4:
Quantitative
Review.
.
.
.
.
.
.
.
.
.
.
.
.
.
.
48
2
12345678901234567890123456789012123456789012345678901
2
12345678901234567890123456789012123456789012345678901
Take
It
to
the
Next
Level.
.
.
.
.
.
.
.
.
.
.
.
.
.
.
.
.
.
.
106
2345678901234567890123456789012123456789012345678901
12345678901234567890123456789012123456789012345678901 22
12345678901234567890123456789012123456789012345678901
1 Chapter 5: Verbal Review . . . . . . . . . . . . . . . . . . . . 134 2
12345678901234567890123456789012123456789012345678901 2
12345678901234567890123456789012123456789012345678901
Take It to the Next Level. . . . . . . . . . . . . . . . . . . 161 2
2
12345678901234567890123456789012123456789012345678901
2
12345678901234567890123456789012123456789012345678901
Chapter
6:
Analytical
Writing
Review.
.
.
.
.
.
.
.
.
.
227
2
12345678901234567890123456789012123456789012345678901
12345678901234567890123456789012123456789012345678901
Take It to the Next Level. . . . . . . . . . . . . . . . . . . 245 2
2
12345678901234567890123456789012123456789012345678901
2345678901234567890123456789012123456789012345678901
12345678901234567890123456789012123456789012345678901 22
12345678901234567890123456789012123456789012345678901 2
12345678901234567890123456789012123456789012345678901 2
12345678901234567890123456789012123456789012345678901
PART
IV: THREE PRACTICE GREs
2
1
2
12345678901234567890123456789012123456789012345678901
Chapter 7: Preface to Practice Tests. . . . . . . . . . . . 256 2
12345678901234567890123456789012123456789012345678901
2
12345678901234567890123456789012123456789012345678901
2
12345678901234567890123456789012123456789012345678901
Practice
Test
1
.
.
.
.
.
.
.
.
.
.
.
.
.
.
.
.
.
.
.
.
.
.
.
.
.
.
.
.
.
.
258
12345678901234567890123456789012123456789012345678901 2
2345678901234567890123456789012123456789012345678901
12345678901234567890123456789012123456789012345678901
Answers and Explanations . . . . . . . . . . . . . . . . . 271 2
2
1
2
12345678901234567890123456789012123456789012345678901
2
12345678901234567890123456789012123456789012345678901
Practice
Test
2
.
.
.
.
.
.
.
.
.
.
.
.
.
.
.
.
.
.
.
.
.
.
.
.
.
.
.
.
.
.
282
2
12345678901234567890123456789012123456789012345678901
2345678901234567890123456789012123456789012345678901
1
Answers and Explanations . . . . . . . . . . . . . . . . . 294 2
2
12345678901234567890123456789012123456789012345678901
12345678901234567890123456789012123456789012345678901 22
12345678901234567890123456789012123456789012345678901
123456789012345678901234567890121234567890123456789012
iii

http://www.xtremepapers.net

123456789012345678901234567890121234567890123456789012
12345678901234567890123456789012123456789012345678901 2
2
12345678901234567890123456789012123456789012345678901
12345678901234567890123456789012123456789012345678901
Practice Test 3 . . . . . . . . . . . . . . . . . . . . . . . . . . . . . . 305 2
2
12345678901234567890123456789012123456789012345678901
2
12345678901234567890123456789012123456789012345678901
Answers
and
Explanations
.
.
.
.
.
.
.
.
.
.
.
.
.
.
.
.
.
318
12345678901234567890123456789012123456789012345678901 2
2
12345678901234567890123456789012123456789012345678901
12345678901234567890123456789012123456789012345678901
Chapter 8: Closing Remarks. . . . . . . . . . . . . . . . . . 329 2
2
12345678901234567890123456789012123456789012345678901
2
12345678901234567890123456789012123456789012345678901
2
12345678901234567890123456789012123456789012345678901
2
12345678901234567890123456789012123456789012345678901
PART
V:
APPENDIX
2345678901234567890123456789012123456789012345678901
2
1
2
12345678901234567890123456789012123456789012345678901
2
12345678901234567890123456789012123456789012345678901
About
the
CAT
.
.
.
.
.
.
.
.
.
.
.
.
.
.
.
.
.
.
.
.
.
.
.
.
.
.
.
.
.
332
2
12345678901234567890123456789012123456789012345678901
2
12345678901234567890123456789012123456789012345678901
2345678901234567890123456789012123456789012345678901
2
1
2
12345678901234567890123456789012123456789012345678901
2
12345678901234567890123456789012123456789012345678901
2
12345678901234567890123456789012123456789012345678901
2
12345678901234567890123456789012123456789012345678901
2345678901234567890123456789012123456789012345678901
2
1
2
12345678901234567890123456789012123456789012345678901
2
12345678901234567890123456789012123456789012345678901
2
12345678901234567890123456789012123456789012345678901
2
12345678901234567890123456789012123456789012345678901
2345678901234567890123456789012123456789012345678901
12345678901234567890123456789012123456789012345678901 2
12345678901234567890123456789012123456789012345678901 2
12345678901234567890123456789012123456789012345678901 2
12345678901234567890123456789012123456789012345678901 2
12345678901234567890123456789012123456789012345678901 2
12345678901234567890123456789012123456789012345678901 2
2
1
2
12345678901234567890123456789012123456789012345678901
2
12345678901234567890123456789012123456789012345678901
2
12345678901234567890123456789012123456789012345678901
2345678901234567890123456789012123456789012345678901
12345678901234567890123456789012123456789012345678901 2
12345678901234567890123456789012123456789012345678901 2
12345678901234567890123456789012123456789012345678901 2
12345678901234567890123456789012123456789012345678901 2
12345678901234567890123456789012123456789012345678901 2
12345678901234567890123456789012123456789012345678901 2
12345678901234567890123456789012123456789012345678901 2
12345678901234567890123456789012123456789012345678901 2
12345678901234567890123456789012123456789012345678901 2
2
1
2
12345678901234567890123456789012123456789012345678901
2
12345678901234567890123456789012123456789012345678901
12345678901234567890123456789012123456789012345678901 2
12345678901234567890123456789012123456789012345678901 2
2
12345678901234567890123456789012123456789012345678901
2
12345678901234567890123456789012123456789012345678901
2
12345678901234567890123456789012123456789012345678901
12345678901234567890123456789012123456789012345678901 2
2
12345678901234567890123456789012123456789012345678901
2
12345678901234567890123456789012123456789012345678901
2345678901234567890123456789012123456789012345678901
12345678901234567890123456789012123456789012345678901 2
12345678901234567890123456789012123456789012345678901 2
2
1
12345678901234567890123456789012123456789012345678901 2
12345678901234567890123456789012123456789012345678901 2
2
12345678901234567890123456789012123456789012345678901
2
12345678901234567890123456789012123456789012345678901
2
12345678901234567890123456789012123456789012345678901
2
12345678901234567890123456789012123456789012345678901
12345678901234567890123456789012123456789012345678901 2
2345678901234567890123456789012123456789012345678901
12345678901234567890123456789012123456789012345678901 2
12345678901234567890123456789012123456789012345678901 2
12345678901234567890123456789012123456789012345678901 2
12345678901234567890123456789012123456789012345678901 2
2
1
2
12345678901234567890123456789012123456789012345678901
2
12345678901234567890123456789012123456789012345678901
2
12345678901234567890123456789012123456789012345678901
2
12345678901234567890123456789012123456789012345678901
12345678901234567890123456789012123456789012345678901 2
2345678901234567890123456789012123456789012345678901
12345678901234567890123456789012123456789012345678901 2
2
1
2
12345678901234567890123456789012123456789012345678901
12345678901234567890123456789012123456789012345678901 2
2
12345678901234567890123456789012123456789012345678901
12345678901234567890123456789012123456789012345678901 2
12345678901234567890123456789012123456789012345678901 2
12345678901234567890123456789012123456789012345678901 2
12345678901234567890123456789012123456789012345678901 2
123456789012345678901234567890121234567890123456789012
iv
www.petersons.com

http://www.xtremepapers.net

About This Tool Kit


123456789012345678901234567890121234567890123456789012
2
12345678901234567890123456789012123456789012345678901
2
12345678901234567890123456789012123456789012345678901
2345678901234567890123456789012123456789012345678901
2
1 Tools
Your
2
12345678901234567890123456789012123456789012345678901
2
12345678901234567890123456789012123456789012345678901
Petersons
Ultimate
GRE
Tool
Kit
provides
the
complete
package
you
2
12345678901234567890123456789012123456789012345678901
2
12345678901234567890123456789012123456789012345678901
need
to
score
your
personal
best
on
the
GRE
and
get
into
your
top-choice
2345678901234567890123456789012123456789012345678901
12345678901234567890123456789012123456789012345678901 22
12345678901234567890123456789012123456789012345678901
program. Unlike any book previously published, this tool kit contains 2
12345678901234567890123456789012123456789012345678901
12345678901234567890123456789012123456789012345678901
many features that used to be available only to those who purchased 2
2
12345678901234567890123456789012123456789012345678901
2
12345678901234567890123456789012123456789012345678901
2
1 expensive test-prep classes.
2
12345678901234567890123456789012123456789012345678901
2
12345678901234567890123456789012123456789012345678901
2
12345678901234567890123456789012123456789012345678901
2345678901234567890123456789012123456789012345678901
2
12345678901234567890123456789012123456789012345678901
e-Tutoring
2
12345678901234567890123456789012123456789012345678901
2
12345678901234567890123456789012123456789012345678901
Use the CD to go on line to register for one-on-one math help from a live 2
12345678901234567890123456789012123456789012345678901
2
12345678901234567890123456789012123456789012345678901
expert whenever you need it. Tutoring is offered using an on-line 2
12345678901234567890123456789012123456789012345678901
2
12345678901234567890123456789012123456789012345678901
whiteboard shared by you and the tutor, which allows you to communi- 2
12345678901234567890123456789012123456789012345678901
2
12345678901234567890123456789012123456789012345678901
1 cate with one another in real time. However, if you prefer, you can submit 2
2345678901234567890123456789012123456789012345678901
12345678901234567890123456789012123456789012345678901
your math question in writing instead and receive a written answer within 2
2
12345678901234567890123456789012123456789012345678901
2
1 24 hours.
12345678901234567890123456789012123456789012345678901 2
2
12345678901234567890123456789012123456789012345678901
12345678901234567890123456789012123456789012345678901
The free tutoring offered with this product is limited to 30 minutes, 2
2
12345678901234567890123456789012123456789012345678901
2
12345678901234567890123456789012123456789012345678901
although
you
may
purchase
more
time
if
you
need
it.
A
written
response
to
2
12345678901234567890123456789012123456789012345678901
2
12345678901234567890123456789012123456789012345678901
a
submitted
question
counts
as
20
minutes.
The
tutoring
service
is
2345678901234567890123456789012123456789012345678901
12345678901234567890123456789012123456789012345678901 22
12345678901234567890123456789012123456789012345678901
1 available for six months from the date you register on line. Note: The 2
12345678901234567890123456789012123456789012345678901
e-tutoring service offered to purchasers of this 2005 edition will expire on 2
2
12345678901234567890123456789012123456789012345678901
2345678901234567890123456789012123456789012345678901
2
12345678901234567890123456789012123456789012345678901
January 1, 2006.
2
12345678901234567890123456789012123456789012345678901
2
12345678901234567890123456789012123456789012345678901
This service is available 24 hours a day, seven days a week for most of the 2
12345678901234567890123456789012123456789012345678901
1 year. During the summer the service is available from 9 a.m. to 1 a.m. 2
2
12345678901234567890123456789012123456789012345678901
2
12345678901234567890123456789012123456789012345678901
Eastern Standard Time. Due to low demand, the service is not available 2
12345678901234567890123456789012123456789012345678901
12345678901234567890123456789012123456789012345678901
during several holiday periods, including Thanksgiving, Christmas, Easter, 2
2
12345678901234567890123456789012123456789012345678901
2345678901234567890123456789012123456789012345678901
2
12345678901234567890123456789012123456789012345678901
Labor Day, and Memorial Day.
2
12345678901234567890123456789012123456789012345678901
12345678901234567890123456789012123456789012345678901 22
12345678901234567890123456789012123456789012345678901
1 Remember, to register for this service, you will need the CD that 2
12345678901234567890123456789012123456789012345678901
accompanies this book. You will also need to refer to this book to provide 2
2
12345678901234567890123456789012123456789012345678901
2
12345678901234567890123456789012123456789012345678901
the
access
code
when
prompted.
12345678901234567890123456789012123456789012345678901 2
2
12345678901234567890123456789012123456789012345678901
12345678901234567890123456789012123456789012345678901 2
12345678901234567890123456789012123456789012345678901 2
12345678901234567890123456789012123456789012345678901 2
12345678901234567890123456789012123456789012345678901 2
123456789012345678901234567890121234567890123456789012
v

http://www.xtremepapers.net

About This Tool Kit

123456789012345678901234567890121234567890123456789012
12345678901234567890123456789012123456789012345678901 2
2
12345678901234567890123456789012123456789012345678901
2
12345678901234567890123456789012123456789012345678901
Essay Scoring
2
12345678901234567890123456789012123456789012345678901
2
12345678901234567890123456789012123456789012345678901
The
CD
that
accompanies
this
book
allows
you
to
write
2
practice
test
12345678901234567890123456789012123456789012345678901 2
12345678901234567890123456789012123456789012345678901
essays on line and receive a score for each of them, which approximates 2
2
12345678901234567890123456789012123456789012345678901
12345678901234567890123456789012123456789012345678901
your performance on the essays in the actual GRE. In addition to a score, 2
2
12345678901234567890123456789012123456789012345678901
12345678901234567890123456789012123456789012345678901
you will also receive constructive feedback on your essays, including tips 2
2
12345678901234567890123456789012123456789012345678901
2
12345678901234567890123456789012123456789012345678901
to improve your score.
2
12345678901234567890123456789012123456789012345678901
2
12345678901234567890123456789012123456789012345678901
With this tool kit, you get scoring for 2 practice test essaysone analysis 2
12345678901234567890123456789012123456789012345678901
2
12345678901234567890123456789012123456789012345678901
of an issue and one analysis of an argument. If you wish, for a fee, you may 2
12345678901234567890123456789012123456789012345678901
2
12345678901234567890123456789012123456789012345678901
obtain scoring information and feedback for additional practice essays 2
12345678901234567890123456789012123456789012345678901
2
12345678901234567890123456789012123456789012345678901
you write on line.
2
12345678901234567890123456789012123456789012345678901
2345678901234567890123456789012123456789012345678901
1 Note: Your practice test essays will be scored by a computer, but on the 2
2
12345678901234567890123456789012123456789012345678901
2
12345678901234567890123456789012123456789012345678901
actual
GRE
test,
the
essays
are
scored
by
a
combination
of
a
computer
2
12345678901234567890123456789012123456789012345678901
2
12345678901234567890123456789012123456789012345678901
program
and
a
human
reader.
2345678901234567890123456789012123456789012345678901
12345678901234567890123456789012123456789012345678901 2
12345678901234567890123456789012123456789012345678901 2
12345678901234567890123456789012123456789012345678901
To register for the essay-scoring service you need the CD that accompanies 2
2
12345678901234567890123456789012123456789012345678901
12345678901234567890123456789012123456789012345678901
this book. In addition, you will need to refer to this book to provide the 2
12345678901234567890123456789012123456789012345678901 2
1 access code when prompted. Note: The scoring of 2 essays is offered free 2
2
12345678901234567890123456789012123456789012345678901
12345678901234567890123456789012123456789012345678901
to purchasers of the 2005 edition and this offer will expire on December 2
2
12345678901234567890123456789012123456789012345678901
2
12345678901234567890123456789012123456789012345678901
31, 2005.
2
12345678901234567890123456789012123456789012345678901
2
12345678901234567890123456789012123456789012345678901
2
12345678901234567890123456789012123456789012345678901
2
12345678901234567890123456789012123456789012345678901
2345678901234567890123456789012123456789012345678901
2
12345678901234567890123456789012123456789012345678901
Computer-Adaptive Tests on CD
2
12345678901234567890123456789012123456789012345678901
2
12345678901234567890123456789012123456789012345678901
The
actual
GRE
is
a
computer-adaptive
test
(CAT)the
test
questions
you
12345678901234567890123456789012123456789012345678901 2
1 see depend on your performance on previous questions. On the CD-ROM, 2
2
12345678901234567890123456789012123456789012345678901
2
12345678901234567890123456789012123456789012345678901
which
accompanies
this
book,
you
receive
3
practice
GMAT
CATs
that
12345678901234567890123456789012123456789012345678901 2
12345678901234567890123456789012123456789012345678901
carefully reproduce the experience you can expect on test day. The 2
2
12345678901234567890123456789012123456789012345678901
2345678901234567890123456789012123456789012345678901
2
12345678901234567890123456789012123456789012345678901
additional 3 practice tests in the book can be used for further practice.
2
12345678901234567890123456789012123456789012345678901
2
1
2
12345678901234567890123456789012123456789012345678901
2
12345678901234567890123456789012123456789012345678901
2345678901234567890123456789012123456789012345678901
12345678901234567890123456789012123456789012345678901 2
12345678901234567890123456789012123456789012345678901 2
2
1
12345678901234567890123456789012123456789012345678901 2
12345678901234567890123456789012123456789012345678901 2
2
12345678901234567890123456789012123456789012345678901
2
12345678901234567890123456789012123456789012345678901
2
12345678901234567890123456789012123456789012345678901
2
12345678901234567890123456789012123456789012345678901
12345678901234567890123456789012123456789012345678901 2
2345678901234567890123456789012123456789012345678901
12345678901234567890123456789012123456789012345678901 2
12345678901234567890123456789012123456789012345678901 2
12345678901234567890123456789012123456789012345678901 2
12345678901234567890123456789012123456789012345678901 2
2
1
2
12345678901234567890123456789012123456789012345678901
2
12345678901234567890123456789012123456789012345678901
2
12345678901234567890123456789012123456789012345678901
2
12345678901234567890123456789012123456789012345678901
12345678901234567890123456789012123456789012345678901 2
2345678901234567890123456789012123456789012345678901
12345678901234567890123456789012123456789012345678901 2
2
1
2
12345678901234567890123456789012123456789012345678901
12345678901234567890123456789012123456789012345678901 2
2
12345678901234567890123456789012123456789012345678901
12345678901234567890123456789012123456789012345678901 2
12345678901234567890123456789012123456789012345678901 2
12345678901234567890123456789012123456789012345678901 2
2
vi 12345678901234567890123456789012123456789012345678901
123456789012345678901234567890121234567890123456789012

www.petersons.com

http://www.xtremepapers.net

About This Tool Kit

123456789012345678901234567890121234567890123456789012
12345678901234567890123456789012123456789012345678901 2
2
12345678901234567890123456789012123456789012345678901
2
12345678901234567890123456789012123456789012345678901
STRUCTURE OF THE GRE CAT
2
12345678901234567890123456789012123456789012345678901
2
12345678901234567890123456789012123456789012345678901
2
12345678901234567890123456789012123456789012345678901
Test
Section
Number
of
Questions
Time
Allotted
2
12345678901234567890123456789012123456789012345678901
2345678901234567890123456789012123456789012345678901
2
1
2
12345678901234567890123456789012123456789012345678901
2
12345678901234567890123456789012123456789012345678901
2
12345678901234567890123456789012123456789012345678901
Computer-Based General Test
2
12345678901234567890123456789012123456789012345678901
2345678901234567890123456789012123456789012345678901
2
1 Analytical Writing 2 writing tasks
75-minute section
2
12345678901234567890123456789012123456789012345678901
2
12345678901234567890123456789012123456789012345678901
2
12345678901234567890123456789012123456789012345678901
Present Your Perspective 45 minutes
2
12345678901234567890123456789012123456789012345678901
2345678901234567890123456789012123456789012345678901
2
1
on an Issue
2
12345678901234567890123456789012123456789012345678901
2
12345678901234567890123456789012123456789012345678901
Analyze
an
Argument
30
minutes
2
12345678901234567890123456789012123456789012345678901
2
12345678901234567890123456789012123456789012345678901
2345678901234567890123456789012123456789012345678901
2
1 Verbal
30 questions
30-minute section
2
12345678901234567890123456789012123456789012345678901
2
12345678901234567890123456789012123456789012345678901
Quantitative
28
questions
45-minute
section
2
12345678901234567890123456789012123456789012345678901
2
12345678901234567890123456789012123456789012345678901
2345678901234567890123456789012123456789012345678901
12345678901234567890123456789012123456789012345678901 22
12345678901234567890123456789012123456789012345678901 2
12345678901234567890123456789012123456789012345678901
Paper-Based General Test
2
12345678901234567890123456789012123456789012345678901
2
12345678901234567890123456789012123456789012345678901
Analytical
Writing
2
writing
tasks
75-minute
section
12345678901234567890123456789012123456789012345678901 22
1
2
12345678901234567890123456789012123456789012345678901
Present Your Perspective 45 minutes
2
12345678901234567890123456789012123456789012345678901
2
12345678901234567890123456789012123456789012345678901
on
an
Issue
2345678901234567890123456789012123456789012345678901
12345678901234567890123456789012123456789012345678901 22
12345678901234567890123456789012123456789012345678901
Analyze an Argument
30 minutes
2
12345678901234567890123456789012123456789012345678901
12345678901234567890123456789012123456789012345678901 22
12345678901234567890123456789012123456789012345678901
Verbal
30 questions
30-minute section
2
12345678901234567890123456789012123456789012345678901
2
12345678901234567890123456789012123456789012345678901
Quantitative
28 questions
45-minute section
2
12345678901234567890123456789012123456789012345678901
12345678901234567890123456789012123456789012345678901 22
1
2
12345678901234567890123456789012123456789012345678901
2
12345678901234567890123456789012123456789012345678901
12345678901234567890123456789012123456789012345678901 2
12345678901234567890123456789012123456789012345678901 2
2
12345678901234567890123456789012123456789012345678901
2
12345678901234567890123456789012123456789012345678901
2
12345678901234567890123456789012123456789012345678901
12345678901234567890123456789012123456789012345678901 2
2
12345678901234567890123456789012123456789012345678901
2
12345678901234567890123456789012123456789012345678901
2345678901234567890123456789012123456789012345678901
12345678901234567890123456789012123456789012345678901 22
12345678901234567890123456789012123456789012345678901 2
1
12345678901234567890123456789012123456789012345678901 2
12345678901234567890123456789012123456789012345678901 2
2
12345678901234567890123456789012123456789012345678901
2
12345678901234567890123456789012123456789012345678901
2
12345678901234567890123456789012123456789012345678901
2
12345678901234567890123456789012123456789012345678901
12345678901234567890123456789012123456789012345678901 2
2345678901234567890123456789012123456789012345678901
12345678901234567890123456789012123456789012345678901 22
12345678901234567890123456789012123456789012345678901 2
12345678901234567890123456789012123456789012345678901 2
12345678901234567890123456789012123456789012345678901 2
1
2
12345678901234567890123456789012123456789012345678901
2
12345678901234567890123456789012123456789012345678901
2
12345678901234567890123456789012123456789012345678901
2
12345678901234567890123456789012123456789012345678901
12345678901234567890123456789012123456789012345678901 2
2345678901234567890123456789012123456789012345678901
12345678901234567890123456789012123456789012345678901 22
1
2
12345678901234567890123456789012123456789012345678901
12345678901234567890123456789012123456789012345678901 2
2
12345678901234567890123456789012123456789012345678901
12345678901234567890123456789012123456789012345678901 2
12345678901234567890123456789012123456789012345678901 2
12345678901234567890123456789012123456789012345678901 2
12345678901234567890123456789012123456789012345678901 2 vii
123456789012345678901234567890121234567890123456789012

http://www.xtremepapers.net

http://www.xtremepapers.net

Introduction
123456789012345678901234567890121234567890123456789012
2
12345678901234567890123456789012123456789012345678901
2
12345678901234567890123456789012123456789012345678901
2345678901234567890123456789012123456789012345678901
2
1
Does
This CAT Have Claws?
2
12345678901234567890123456789012123456789012345678901
2
12345678901234567890123456789012123456789012345678901
Unless
youve
been
living
as
a
hermit
in
a
hut
for
the
past
twenty
years,
2
12345678901234567890123456789012123456789012345678901
2
12345678901234567890123456789012123456789012345678901
chances
are
good
you
have
already
taken
at
least
one
standardized
test
in
2345678901234567890123456789012123456789012345678901
12345678901234567890123456789012123456789012345678901 22
12345678901234567890123456789012123456789012345678901
your lifetime. And we bet the list of tests you might have taken reads like a 2
12345678901234567890123456789012123456789012345678901
12345678901234567890123456789012123456789012345678901
bowl of alphabet soup: SAT, ACT, ITBS, TAAS, MAT, and the PSAT/ 2
2
12345678901234567890123456789012123456789012345678901
2
12345678901234567890123456789012123456789012345678901
1 NMSQT, to name just a few. Standardized tests such as these have become 2
12345678901234567890123456789012123456789012345678901
a fixture of Americas educational system, and like them or not, they are 2
2
12345678901234567890123456789012123456789012345678901
12345678901234567890123456789012123456789012345678901
not going away anytime soon. Students encounter them at every stage of 2
2345678901234567890123456789012123456789012345678901
12345678901234567890123456789012123456789012345678901 22
12345678901234567890123456789012123456789012345678901
their educationlike toll booths on the academic highway. Students in
2
12345678901234567890123456789012123456789012345678901
12345678901234567890123456789012123456789012345678901
grade school get the ITBS or a similar exam. Teenagers hoping to enter 2
12345678901234567890123456789012123456789012345678901 22
12345678901234567890123456789012123456789012345678901
college must pass through the SAT or ACT Assessment toll booth. For
2
12345678901234567890123456789012123456789012345678901
those brave folks who wish to enter graduate school, the exit is labeled 2
12345678901234567890123456789012123456789012345678901
2
12345678901234567890123456789012123456789012345678901
2
1 GRE (Graduate Record Examination).
2345678901234567890123456789012123456789012345678901
12345678901234567890123456789012123456789012345678901 22
12345678901234567890123456789012123456789012345678901
1 If you have taken the SAT, you will already be somewhat familiar with the 2
12345678901234567890123456789012123456789012345678901
GRE. Both tests are administered by the same company. Each test is a 2
2
12345678901234567890123456789012123456789012345678901
2345678901234567890123456789012123456789012345678901
12345678901234567890123456789012123456789012345678901
timed, multiple-choice exam that schools use as a factor when making 2
2
12345678901234567890123456789012123456789012345678901
1 enrollment decisions. Like the SAT, the GRE has a Quantitative (math) 2
2
12345678901234567890123456789012123456789012345678901
2
12345678901234567890123456789012123456789012345678901
section
and
a
Verbal
section,
and
many
of
the
questions
in
these
SAT
and
2345678901234567890123456789012123456789012345678901
12345678901234567890123456789012123456789012345678901 22
12345678901234567890123456789012123456789012345678901
1 GRE sections are similar in style. For example, there are analogy, sentence 2
12345678901234567890123456789012123456789012345678901
completions, and reading comprehension problems in the Verbal section of 2
2
12345678901234567890123456789012123456789012345678901
2345678901234567890123456789012123456789012345678901
12345678901234567890123456789012123456789012345678901
both tests. Many of the math topics covered on the SATgeometry, 2
2
12345678901234567890123456789012123456789012345678901
12345678901234567890123456789012123456789012345678901
algebra, understanding charts and graphsmake an encore appearance on 2
12345678901234567890123456789012123456789012345678901 22
1 the GRE.
2
12345678901234567890123456789012123456789012345678901
2
12345678901234567890123456789012123456789012345678901
While these similarities exist, dont be fooled into thinking the GRE is 2
12345678901234567890123456789012123456789012345678901
2
12345678901234567890123456789012123456789012345678901
simply another SAT. The GRE is more like the SATs weird older brother. 2
12345678901234567890123456789012123456789012345678901
2345678901234567890123456789012123456789012345678901
2
12345678901234567890123456789012123456789012345678901
Although you can see some family resemblance, some big differences are 2
12345678901234567890123456789012123456789012345678901
12345678901234567890123456789012123456789012345678901
pretty obvious as well. For example, both tests have analogy and sentence 2
2
12345678901234567890123456789012123456789012345678901
1 completions problems, but GRE questions are typically much harder, since 2
2345678901234567890123456789012123456789012345678901
12345678901234567890123456789012123456789012345678901 22
1 they employ a more advanced vocabulary. This is because the GRE
2
12345678901234567890123456789012123456789012345678901
2
12345678901234567890123456789012123456789012345678901
problems
assume
a
college-level
vocabulary,
while
the
SAT
questions
are
2
12345678901234567890123456789012123456789012345678901
2345678901234567890123456789012123456789012345678901
1 set at a lower, high school level. This is just one reason why the GRE can be 2
12345678901234567890123456789012123456789012345678901 2
12345678901234567890123456789012123456789012345678901 2
12345678901234567890123456789012123456789012345678901 2
123456789012345678901234567890121234567890123456789012
1

http://www.xtremepapers.net

Introduction

www.petersons.com

http://www.xtremepapers.net

X-Ref

123456789012345678901234567890121234567890123456789012
12345678901234567890123456789012123456789012345678901 2
2
12345678901234567890123456789012123456789012345678901
seen as an older brother, since it is geared toward an older, college- 2
12345678901234567890123456789012123456789012345678901
2
12345678901234567890123456789012123456789012345678901
educated audience. The GREs weirdness comes from its unusual format. 2
12345678901234567890123456789012123456789012345678901
2
12345678901234567890123456789012123456789012345678901
The SAT remains a pencil-and-paper test, meaning students across the 2
12345678901234567890123456789012123456789012345678901
2345678901234567890123456789012123456789012345678901
1 country are still using chewed-up number 2 pencils to fiercely darken ovals 2
2
12345678901234567890123456789012123456789012345678901
2
12345678901234567890123456789012123456789012345678901
on
a
score
sheet.
In
contrast,
the
GRE
is
now
primarily
a
Computer
2
12345678901234567890123456789012123456789012345678901
2
12345678901234567890123456789012123456789012345678901
Adaptive
Test
(CAT),
so
all
questions
appear
onscreen
and
are
answered
2345678901234567890123456789012123456789012345678901
2
1
2
12345678901234567890123456789012123456789012345678901
by
using
a
mouse
to
click
on
an
answer
choice.
2
12345678901234567890123456789012123456789012345678901
2
12345678901234567890123456789012123456789012345678901
2
12345678901234567890123456789012123456789012345678901
2345678901234567890123456789012123456789012345678901
2
1
2
12345678901234567890123456789012123456789012345678901
There
are
still
some
places,
mostly
outside
of
the
United
States,
that
offer
2
12345678901234567890123456789012123456789012345678901
2
12345678901234567890123456789012123456789012345678901
pencil-and-paper
versions
of
the
GRE.
For
more
information
about
these
2
12345678901234567890123456789012123456789012345678901
2345678901234567890123456789012123456789012345678901
1 exams, and about registering for the GRE in general, turn to the 2
2
12345678901234567890123456789012123456789012345678901
2
12345678901234567890123456789012123456789012345678901
Appendix.
2
12345678901234567890123456789012123456789012345678901
2
12345678901234567890123456789012123456789012345678901
2345678901234567890123456789012123456789012345678901
12345678901234567890123456789012123456789012345678901 2
12345678901234567890123456789012123456789012345678901 2
12345678901234567890123456789012123456789012345678901
The GRE CAT is more than just a pencil-and-paper test transposed onto a 2
2
12345678901234567890123456789012123456789012345678901
12345678901234567890123456789012123456789012345678901
computer screen, and unless you have taken the GRE previously, it is 2
12345678901234567890123456789012123456789012345678901 2
1 unlikely you have ever experienced an exam like it. For one thing, the 2
2
12345678901234567890123456789012123456789012345678901
12345678901234567890123456789012123456789012345678901
method used to score the GRE CAT is radically different from most tests. 2
2
12345678901234567890123456789012123456789012345678901
12345678901234567890123456789012123456789012345678901
Most exams start with easy questions, and then gradually progress to 2
2
12345678901234567890123456789012123456789012345678901
medium and harder problems. People taking this type of test might get the 2
12345678901234567890123456789012123456789012345678901
2
12345678901234567890123456789012123456789012345678901
easiest problem wrong but still manage to answer the toughest problem 2
12345678901234567890123456789012123456789012345678901
2345678901234567890123456789012123456789012345678901
2
12345678901234567890123456789012123456789012345678901
correctly. In the end, the number of right answers is added up (regardless 2
12345678901234567890123456789012123456789012345678901
2
12345678901234567890123456789012123456789012345678901
of which question was answered right or wrong), and a score is eventually 2
12345678901234567890123456789012123456789012345678901
2
1 determined.
2
12345678901234567890123456789012123456789012345678901
2
12345678901234567890123456789012123456789012345678901
2
12345678901234567890123456789012123456789012345678901
On
the
GRE
CAT,
things
are
a
bit
different.
How
well
(or
not
well)
you
12345678901234567890123456789012123456789012345678901 2
12345678901234567890123456789012123456789012345678901
answer each question determines the next question. In each section of the 2
2
12345678901234567890123456789012123456789012345678901
2
12345678901234567890123456789012123456789012345678901
test,
you
start
off
with
a
question
that
has
a
medium
difficulty
rating.
If
12345678901234567890123456789012123456789012345678901 2
12345678901234567890123456789012123456789012345678901
you get it right, the computer says, Okay, Brainiac, you handled that one, 2
2
12345678901234567890123456789012123456789012345678901
2345678901234567890123456789012123456789012345678901
12345678901234567890123456789012123456789012345678901
now try this tougher problem. If you get the next problem right, you get 2
2
12345678901234567890123456789012123456789012345678901
1 a harder question, and if you get that one correct, an even tougher 2
12345678901234567890123456789012123456789012345678901 2
12345678901234567890123456789012123456789012345678901
question comes next. Depending on how you answer each question, the 2
2
12345678901234567890123456789012123456789012345678901
12345678901234567890123456789012123456789012345678901
computer adapts the next question to make it either harder or easier than 2
2
12345678901234567890123456789012123456789012345678901
12345678901234567890123456789012123456789012345678901
the next one. The way the computer adapts your GRE experience will be 2
2
12345678901234567890123456789012123456789012345678901
different from what other GRE test-takers experience, so even though 2
12345678901234567890123456789012123456789012345678901
2
12345678901234567890123456789012123456789012345678901
youre all taking the same GRE CAT, youre not seeing the same problems 2
12345678901234567890123456789012123456789012345678901
2
12345678901234567890123456789012123456789012345678901
as everyone else. In the same way, if you get the first question wrong, youll 2
12345678901234567890123456789012123456789012345678901
2345678901234567890123456789012123456789012345678901
2
12345678901234567890123456789012123456789012345678901
get an easier one. And if you get the easier questions wrong, youll see an 2
12345678901234567890123456789012123456789012345678901
2
12345678901234567890123456789012123456789012345678901
easier question.
2
12345678901234567890123456789012123456789012345678901
2
1
2345678901234567890123456789012123456789012345678901
2
12345678901234567890123456789012123456789012345678901
1 One consequence of the computer adaptive format is that getting a high 2
12345678901234567890123456789012123456789012345678901
score means handling only medium and hard questions. The more difficult 2
2
12345678901234567890123456789012123456789012345678901
2
12345678901234567890123456789012123456789012345678901
your
questions
are,
the
higher
your
score
potential,
and
vice
versa.
And
2345678901234567890123456789012123456789012345678901
2
1
2
12345678901234567890123456789012123456789012345678901
there
are
no
easy
questions
at
the
start
of
a
section
to
help
you
get
your
feet
12345678901234567890123456789012123456789012345678901 2
12345678901234567890123456789012123456789012345678901 2
123456789012345678901234567890121234567890123456789012

Introduction

123456789012345678901234567890121234567890123456789012
12345678901234567890123456789012123456789012345678901 2
2
12345678901234567890123456789012123456789012345678901
wet. The first question will be somewhat difficult, and it may well be the 2
12345678901234567890123456789012123456789012345678901
2
12345678901234567890123456789012123456789012345678901
easiest problem you see on that section. In terms of your score, seeing only 2
12345678901234567890123456789012123456789012345678901
2
12345678901234567890123456789012123456789012345678901
medium and tough questions tells you that you are getting problems right, 2
12345678901234567890123456789012123456789012345678901
2345678901234567890123456789012123456789012345678901
1 which is good news. However, having to answer one challenging question 2
2
12345678901234567890123456789012123456789012345678901
2
12345678901234567890123456789012123456789012345678901
after
another
makes
the
testing
experience
a
really
nerve-wracking,
2
12345678901234567890123456789012123456789012345678901
2
12345678901234567890123456789012123456789012345678901
unpleasant
affair,
kind
of
like
visiting
a
dentist
who
holds
a
personal
2345678901234567890123456789012123456789012345678901
2
1
2
12345678901234567890123456789012123456789012345678901
grudge
against
your
gums.
2
12345678901234567890123456789012123456789012345678901
2
12345678901234567890123456789012123456789012345678901
2
12345678901234567890123456789012123456789012345678901
2345678901234567890123456789012123456789012345678901
2
1
2
12345678901234567890123456789012123456789012345678901
2
12345678901234567890123456789012123456789012345678901
The
Unique
Petersons
ApproachAdaptive
2
12345678901234567890123456789012123456789012345678901
2
12345678901234567890123456789012123456789012345678901
Preparation
for
an
Adaptive
Test
2345678901234567890123456789012123456789012345678901
2
1
2
12345678901234567890123456789012123456789012345678901
Test preparation is not a one-size-fits-all proposition. Each person has 2
12345678901234567890123456789012123456789012345678901
2
12345678901234567890123456789012123456789012345678901
different strengths and weaknesses, different quirks and anxieties. We 2
12345678901234567890123456789012123456789012345678901
2345678901234567890123456789012123456789012345678901
2
12345678901234567890123456789012123456789012345678901
designed Petersons Ultimate GRE Tool Kit with those differences in 2
12345678901234567890123456789012123456789012345678901
12345678901234567890123456789012123456789012345678901
mind. We provide a two-tiered approach that lets you decide how best to 2
2
12345678901234567890123456789012123456789012345678901
2
12345678901234567890123456789012123456789012345678901
organize
your
program
of
study
to
suit
your
needs.
The
first
tier
is
the
12345678901234567890123456789012123456789012345678901 22
1 general instruction in this book, and it provides skills and strategies you
2
12345678901234567890123456789012123456789012345678901
2
12345678901234567890123456789012123456789012345678901
will
need
to
handle
medium-level
questions
on
each
section.
You
then
have
2
12345678901234567890123456789012123456789012345678901
2345678901234567890123456789012123456789012345678901
12345678901234567890123456789012123456789012345678901
an opportunity to Take It to the Next Level, and learn skills and strategies 2
2
12345678901234567890123456789012123456789012345678901
2
12345678901234567890123456789012123456789012345678901
for
the
hardest
questions
on
the
GRE.
No
other
GRE
test-preparation
12345678901234567890123456789012123456789012345678901 22
12345678901234567890123456789012123456789012345678901
book does this. Its the next best thing to having a personal tutor.
2
12345678901234567890123456789012123456789012345678901
12345678901234567890123456789012123456789012345678901 22
12345678901234567890123456789012123456789012345678901 2
12345678901234567890123456789012123456789012345678901 2
1
2
12345678901234567890123456789012123456789012345678901
2
12345678901234567890123456789012123456789012345678901
12345678901234567890123456789012123456789012345678901 2
12345678901234567890123456789012123456789012345678901 2
2
12345678901234567890123456789012123456789012345678901
2
12345678901234567890123456789012123456789012345678901
2
12345678901234567890123456789012123456789012345678901
12345678901234567890123456789012123456789012345678901 2
2
12345678901234567890123456789012123456789012345678901
2
12345678901234567890123456789012123456789012345678901
2345678901234567890123456789012123456789012345678901
12345678901234567890123456789012123456789012345678901 22
12345678901234567890123456789012123456789012345678901 2
1
12345678901234567890123456789012123456789012345678901 2
12345678901234567890123456789012123456789012345678901 2
2
12345678901234567890123456789012123456789012345678901
2
12345678901234567890123456789012123456789012345678901
2
12345678901234567890123456789012123456789012345678901
2
12345678901234567890123456789012123456789012345678901
12345678901234567890123456789012123456789012345678901 2
2345678901234567890123456789012123456789012345678901
12345678901234567890123456789012123456789012345678901 22
12345678901234567890123456789012123456789012345678901 2
12345678901234567890123456789012123456789012345678901 2
12345678901234567890123456789012123456789012345678901 2
1
2
12345678901234567890123456789012123456789012345678901
2
12345678901234567890123456789012123456789012345678901
2
12345678901234567890123456789012123456789012345678901
2
12345678901234567890123456789012123456789012345678901
12345678901234567890123456789012123456789012345678901 2
2345678901234567890123456789012123456789012345678901
12345678901234567890123456789012123456789012345678901 22
1
2
12345678901234567890123456789012123456789012345678901
12345678901234567890123456789012123456789012345678901 2
2
12345678901234567890123456789012123456789012345678901
12345678901234567890123456789012123456789012345678901 2
12345678901234567890123456789012123456789012345678901 2
12345678901234567890123456789012123456789012345678901 2
12345678901234567890123456789012123456789012345678901 2
123456789012345678901234567890121234567890123456789012

http://www.xtremepapers.net

How to Use This Book


123456789012345678901234567890121234567890123456789012
2
12345678901234567890123456789012123456789012345678901
2
12345678901234567890123456789012123456789012345678901
How
do
you
know
where
to
begin?
We
give
you
a
roadmap.
After
you
read
2345678901234567890123456789012123456789012345678901
2
1
2
12345678901234567890123456789012123456789012345678901
Chapters 1 and 2 required reading for all GRE CAT takers, no matter 2
12345678901234567890123456789012123456789012345678901
2
12345678901234567890123456789012123456789012345678901
whattake the diagnostic pretest in Chapter 3. Use the scoring discussion 2
12345678901234567890123456789012123456789012345678901
2345678901234567890123456789012123456789012345678901
2
12345678901234567890123456789012123456789012345678901
provided on pages 3233 to assess your current GRE CAT testing level. 2
12345678901234567890123456789012123456789012345678901
12345678901234567890123456789012123456789012345678901
You can then either follow the suggested course of study we provide, or 2
2
12345678901234567890123456789012123456789012345678901
2
12345678901234567890123456789012123456789012345678901
design
your
own.
12345678901234567890123456789012123456789012345678901 2
2
1
2
12345678901234567890123456789012123456789012345678901
Your
personalized
Petersons
GRE
course
of
study
will
help
you
2
12345678901234567890123456789012123456789012345678901
2
12345678901234567890123456789012123456789012345678901
accomplish
four
things:
2345678901234567890123456789012123456789012345678901
12345678901234567890123456789012123456789012345678901 2
12345678901234567890123456789012123456789012345678901 2
12345678901234567890123456789012123456789012345678901
1. You will become thoroughly familiar with the computer adaptive 2
2
12345678901234567890123456789012123456789012345678901
2
12345678901234567890123456789012123456789012345678901
test format.
12345678901234567890123456789012123456789012345678901 2
2
12345678901234567890123456789012123456789012345678901
2. You will be comfortable with all question types you will 2
12345678901234567890123456789012123456789012345678901
2
12345678901234567890123456789012123456789012345678901
encounter on the three sections of the test.
2
1
2345678901234567890123456789012123456789012345678901
12345678901234567890123456789012123456789012345678901 2
2
12345678901234567890123456789012123456789012345678901
3. You will brush up on any weak subject areas.
2
1
12345678901234567890123456789012123456789012345678901 2
12345678901234567890123456789012123456789012345678901
4. You will learn and practice specific test-taking strategies to help 2
2
12345678901234567890123456789012123456789012345678901
2
12345678901234567890123456789012123456789012345678901
you
achieve
your
target
score.
12345678901234567890123456789012123456789012345678901 2
2
12345678901234567890123456789012123456789012345678901
12345678901234567890123456789012123456789012345678901
If you give yourself enough time to work through your personalized course 2
2345678901234567890123456789012123456789012345678901
2
12345678901234567890123456789012123456789012345678901
of study at a measured pace, you will accomplish all of these things.
2
12345678901234567890123456789012123456789012345678901
2
1
2
12345678901234567890123456789012123456789012345678901
Combining
relevant
factual
knowledge
with
a
sound
strategic
approach
is
12345678901234567890123456789012123456789012345678901 2
2345678901234567890123456789012123456789012345678901
12345678901234567890123456789012123456789012345678901
the best way to score well on any standardized test, not just the GRE. This 2
2
12345678901234567890123456789012123456789012345678901
2
12345678901234567890123456789012123456789012345678901
book
will
provide
you
with
the
facts
and
strategies
you
need
to
succeed
on
12345678901234567890123456789012123456789012345678901 2
1 the GRE. The first step is to learn about the computer adaptive format, 2
2
12345678901234567890123456789012123456789012345678901
2
12345678901234567890123456789012123456789012345678901
covered
in
the
next
chapter.
So,
lets
get
started.
2
12345678901234567890123456789012123456789012345678901
2
12345678901234567890123456789012123456789012345678901
12345678901234567890123456789012123456789012345678901 2
2345678901234567890123456789012123456789012345678901
12345678901234567890123456789012123456789012345678901 2
12345678901234567890123456789012123456789012345678901 2
12345678901234567890123456789012123456789012345678901 2
12345678901234567890123456789012123456789012345678901 2
2
1
2
12345678901234567890123456789012123456789012345678901
12345678901234567890123456789012123456789012345678901 2
2
12345678901234567890123456789012123456789012345678901
12345678901234567890123456789012123456789012345678901 2
2
12345678901234567890123456789012123456789012345678901
12345678901234567890123456789012123456789012345678901 2
12345678901234567890123456789012123456789012345678901 2
12345678901234567890123456789012123456789012345678901 2
12345678901234567890123456789012123456789012345678901 2
123456789012345678901234567890121234567890123456789012
4

http://www.xtremepapers.net

PART

Understanding the GRE


Everything You Need to Know
about the GRE CAT 6
Section Strategies

19

PART I
http://www.xtremepapers.net

Chapter

1
Everything You Need to
Know about the GRE CAT

123456789012345678901234567890121234567890123456789012
2
12345678901234567890123456789012123456789012345678901
2
12345678901234567890123456789012123456789012345678901
Once
upon
a
time,
people
who
wanted
to
take
the
GRE
(Graduate
Record
2
12345678901234567890123456789012123456789012345678901
2345678901234567890123456789012123456789012345678901
2
12345678901234567890123456789012123456789012345678901
Examination) and go to graduate school had to take a paper-and-pencil 2
12345678901234567890123456789012123456789012345678901
12345678901234567890123456789012123456789012345678901
test much like the SAT they had to take to get into college. They had to 2
2
12345678901234567890123456789012123456789012345678901
2
12345678901234567890123456789012123456789012345678901
show
up
at
a
test
center
early
on
a
Saturday
morning
with
dozens
of
other
12345678901234567890123456789012123456789012345678901 2
1 prospective grad students, all with their number 2 pencils sharpened, ready 2
2
12345678901234567890123456789012123456789012345678901
2
12345678901234567890123456789012123456789012345678901
to
fill
in
a
lot
of
answer
sheet
bubbles
and
spend
pretty
much
the
entire
2
12345678901234567890123456789012123456789012345678901
2345678901234567890123456789012123456789012345678901
2
12345678901234567890123456789012123456789012345678901
morning slogging through the test.
2
12345678901234567890123456789012123456789012345678901
12345678901234567890123456789012123456789012345678901 2
12345678901234567890123456789012123456789012345678901
That was before the CAT (Computer Adaptive Test) came along and 2
2
12345678901234567890123456789012123456789012345678901
2
12345678901234567890123456789012123456789012345678901
changed everything. . . .
2
12345678901234567890123456789012123456789012345678901
2
12345678901234567890123456789012123456789012345678901
No more number 2 pencils, no more answer bubbles; the GRE world 2
12345678901234567890123456789012123456789012345678901
1 changed. Like anything new, of course, the GRE CAT takes some getting 2
2
12345678901234567890123456789012123456789012345678901
2
12345678901234567890123456789012123456789012345678901
used to. Most students have taken countless paper-and-pencil tests, but 2
12345678901234567890123456789012123456789012345678901
2
12345678901234567890123456789012123456789012345678901
few have taken a high-stakes standardized test in a computerized format. 2
12345678901234567890123456789012123456789012345678901
2345678901234567890123456789012123456789012345678901
12345678901234567890123456789012123456789012345678901
In this chapter, well help you get a grip on the world of computer adaptive 2
2
12345678901234567890123456789012123456789012345678901
2
1 testing.
2
12345678901234567890123456789012123456789012345678901
2
12345678901234567890123456789012123456789012345678901
2345678901234567890123456789012123456789012345678901
12345678901234567890123456789012123456789012345678901 2
12345678901234567890123456789012123456789012345678901 2
2
1
2
12345678901234567890123456789012123456789012345678901
Why
the
GRE
CAT
Rocks!
12345678901234567890123456789012123456789012345678901 2
2345678901234567890123456789012123456789012345678901
12345678901234567890123456789012123456789012345678901
No one likes taking a big test and having to wait, and wait, and wait for 2
2
12345678901234567890123456789012123456789012345678901
2
12345678901234567890123456789012123456789012345678901
the
results.
The
time
between
finishing
the
test
and
getting
your
score
fills
12345678901234567890123456789012123456789012345678901 2
1 you with conflicting emotions. You might be confident one moment and 2
2
12345678901234567890123456789012123456789012345678901
2
12345678901234567890123456789012123456789012345678901
filled
with
despair
the
next.
The
doubt
and
uncertainty
wont
go
away
2
12345678901234567890123456789012123456789012345678901
2
12345678901234567890123456789012123456789012345678901
until
you
finally
get
your
scores
in
the
mail,
so
the
issue
goes
unresolved
for
12345678901234567890123456789012123456789012345678901 2
2345678901234567890123456789012123456789012345678901
2
12345678901234567890123456789012123456789012345678901
about a month.
2
12345678901234567890123456789012123456789012345678901
12345678901234567890123456789012123456789012345678901 2
12345678901234567890123456789012123456789012345678901
the GRE CAT, waiting for your score is a thing of the past. Once you 2
2
1 With
finish
your test, you have the option to cancel your score. If you decide not 2
2345678901234567890123456789012123456789012345678901
12345678901234567890123456789012123456789012345678901
1 to cancel, your Quantitative and Verbal scores appear on the screen. Voila! 2
2
12345678901234567890123456789012123456789012345678901
2
12345678901234567890123456789012123456789012345678901
A
wait
of
six
weeks
reduced
to
the
time
it
takes
to
click
one
button.
2
12345678901234567890123456789012123456789012345678901
2345678901234567890123456789012123456789012345678901
2
1
12345678901234567890123456789012123456789012345678901 2
12345678901234567890123456789012123456789012345678901 2
12345678901234567890123456789012123456789012345678901 2
123456789012345678901234567890121234567890123456789012
6

http://www.xtremepapers.net

Tip

Note

Chapter 1: Everything You Need to Know about the GRE CAT

123456789012345678901234567890121234567890123456789012
12345678901234567890123456789012123456789012345678901 2
2
12345678901234567890123456789012123456789012345678901
12345678901234567890123456789012123456789012345678901 2
2
12345678901234567890123456789012123456789012345678901
2
12345678901234567890123456789012123456789012345678901
You
dont
receive
your
Analytical
writing
score
immediately,
since
the
12345678901234567890123456789012123456789012345678901 2
2
12345678901234567890123456789012123456789012345678901
two essays must be graded by professional readers. You should get your
2
12345678901234567890123456789012123456789012345678901
2
12345678901234567890123456789012123456789012345678901
Analytical
results
back
in
about
three
to
four
weeks.
2
12345678901234567890123456789012123456789012345678901
2
12345678901234567890123456789012123456789012345678901
2
12345678901234567890123456789012123456789012345678901
2345678901234567890123456789012123456789012345678901
2
1
2
12345678901234567890123456789012123456789012345678901
This
immediacy
is
the
prime
advantage
of
the
GRE
CAT.
It
lets
you
know
2
12345678901234567890123456789012123456789012345678901
2
12345678901234567890123456789012123456789012345678901
where
you
stand
in
terms
of
the
application
process
and
allows
you
to
plan
2
12345678901234567890123456789012123456789012345678901
2345678901234567890123456789012123456789012345678901
1 accordingly. Most schools release information about the average GRE 2
2
12345678901234567890123456789012123456789012345678901
2
12345678901234567890123456789012123456789012345678901
scores
of
their
incoming
graduate
students,
and
some
also
include
an
idea
2
12345678901234567890123456789012123456789012345678901
2
12345678901234567890123456789012123456789012345678901
about
the
range
of
scores
by
releasing
factoids
like,
Over
90%
of
the
2345678901234567890123456789012123456789012345678901
2
1
12345678901234567890123456789012123456789012345678901
incoming class of psychiatry students scored more than 600 on the GRE. 2
2
12345678901234567890123456789012123456789012345678901
12345678901234567890123456789012123456789012345678901
If you wanted to go to that school, and you just received a combined 2
2
12345678901234567890123456789012123456789012345678901
12345678901234567890123456789012123456789012345678901
Quantitative and Verbal score of 750, then you know your GRE score is 2
2
12345678901234567890123456789012123456789012345678901
2
12345678901234567890123456789012123456789012345678901
good enough to get into that program.
2
12345678901234567890123456789012123456789012345678901
2
12345678901234567890123456789012123456789012345678901
If
your
GRE
scores
are
not
as
high
as
you
wanted,
the
immediate
feedback
2345678901234567890123456789012123456789012345678901
12345678901234567890123456789012123456789012345678901 22
1 gives you a chance to shift to Plan B right then and there. You might decide
2
12345678901234567890123456789012123456789012345678901
2
12345678901234567890123456789012123456789012345678901
to
take
the
exam
again,
although
you
wont
be
able
to
do
so
for
at
least
one
2
12345678901234567890123456789012123456789012345678901
2345678901234567890123456789012123456789012345678901
12345678901234567890123456789012123456789012345678901
calendar month. (You can take the GRE up to five times in a single year.) If 2
2
12345678901234567890123456789012123456789012345678901
2
12345678901234567890123456789012123456789012345678901
you
were
only
considering
extremely
competitive
schools,
you
might
12345678901234567890123456789012123456789012345678901 22
12345678901234567890123456789012123456789012345678901
decide to broaden your search and look at some schools with more 2
12345678901234567890123456789012123456789012345678901
12345678901234567890123456789012123456789012345678901
inclusive admissions standards. When your Analytical scores arrive, you 2
2
12345678901234567890123456789012123456789012345678901
2
12345678901234567890123456789012123456789012345678901
will
also
receive
a
form
you
can
use
to
report
your
scores
to
additional
2
1
2345678901234567890123456789012123456789012345678901
12345678901234567890123456789012123456789012345678901
schools. (You can download this from the GRE Web site.) Knowing your 2
2
12345678901234567890123456789012123456789012345678901
2
1 scores right away gives you a chance to react to them right away.
12345678901234567890123456789012123456789012345678901 2
2
12345678901234567890123456789012123456789012345678901
12345678901234567890123456789012123456789012345678901
The second big advantage to the GRE CAT is that you can schedule it for 2
2
12345678901234567890123456789012123456789012345678901
a convenient time. Other big standardized tests, like the business school 2
12345678901234567890123456789012123456789012345678901
2
12345678901234567890123456789012123456789012345678901
exam (GMAT) and the law school entrance exam (LSAT), are given at 2
12345678901234567890123456789012123456789012345678901
2345678901234567890123456789012123456789012345678901
2
12345678901234567890123456789012123456789012345678901
fixed times on fixed dates at fixed locations. If any one of these three 2
12345678901234567890123456789012123456789012345678901
2
1 factors poses a problem for you, tough luck.
12345678901234567890123456789012123456789012345678901 2
12345678901234567890123456789012123456789012345678901 2
12345678901234567890123456789012123456789012345678901
This rigidity is not a problem for the GRE CAT. Most test centers are open 2
2
12345678901234567890123456789012123456789012345678901
2
12345678901234567890123456789012123456789012345678901
during
regular
business
hours,
and
all
you
have
to
do
is
call
one
up
and
2
12345678901234567890123456789012123456789012345678901
2
12345678901234567890123456789012123456789012345678901
decide
on
a
time
that
works
well
for
you.
2345678901234567890123456789012123456789012345678901
12345678901234567890123456789012123456789012345678901 22
12345678901234567890123456789012123456789012345678901 2
12345678901234567890123456789012123456789012345678901 2
12345678901234567890123456789012123456789012345678901
2
1 Before you schedule your actual GRE CAT, make sure you complete your
2
12345678901234567890123456789012123456789012345678901
personalized course of study with this book and take all the practice exams
2
12345678901234567890123456789012123456789012345678901
2
12345678901234567890123456789012123456789012345678901
we
provide.
At
that
point,
schedule
an
exam
for
two
weeks
in
the
future.
2
12345678901234567890123456789012123456789012345678901
2
12345678901234567890123456789012123456789012345678901
Use
the
two-week
interval
to
take
practice
CAT
exams
and
write
practice
2345678901234567890123456789012123456789012345678901
12345678901234567890123456789012123456789012345678901 22
1 esays using the CD-ROM included. Your skills will stay fresh and you will
2
12345678901234567890123456789012123456789012345678901
2
12345678901234567890123456789012123456789012345678901
be accustomed to taking the test on a computer.
2
12345678901234567890123456789012123456789012345678901
2345678901234567890123456789012123456789012345678901
2
1
12345678901234567890123456789012123456789012345678901 2
12345678901234567890123456789012123456789012345678901 2
12345678901234567890123456789012123456789012345678901 2
123456789012345678901234567890121234567890123456789012

http://www.xtremepapers.net

Part I: Understanding the GRE

123456789012345678901234567890121234567890123456789012
12345678901234567890123456789012123456789012345678901 2
2
12345678901234567890123456789012123456789012345678901
In addition to the flexible schedule and immediacy of the test results, you 2
12345678901234567890123456789012123456789012345678901
2
12345678901234567890123456789012123456789012345678901
can also send your scores to schools automatically by filling out the proper 2
12345678901234567890123456789012123456789012345678901
2
12345678901234567890123456789012123456789012345678901
computer form before the test.
2
12345678901234567890123456789012123456789012345678901
2345678901234567890123456789012123456789012345678901
2
1
2
12345678901234567890123456789012123456789012345678901
2
12345678901234567890123456789012123456789012345678901
2
12345678901234567890123456789012123456789012345678901
2
12345678901234567890123456789012123456789012345678901
Why
the
GRE
CAT
Doesnt
Rock
2345678901234567890123456789012123456789012345678901
2
1
2
12345678901234567890123456789012123456789012345678901
No matter what the standardized test is, one fundamental truth always 2
12345678901234567890123456789012123456789012345678901
12345678901234567890123456789012123456789012345678901
holds true: The more familiar you are with the test format, the better your 2
2
12345678901234567890123456789012123456789012345678901
2345678901234567890123456789012123456789012345678901
1 score will be. Familiarity with the test design allows you to focus more 2
2
12345678901234567890123456789012123456789012345678901
2
12345678901234567890123456789012123456789012345678901
attention
on
the
questions
themselves
and
expend
less
mental
effort
just
2
12345678901234567890123456789012123456789012345678901
2
12345678901234567890123456789012123456789012345678901
figuring
out
the
test
itself.
This
axiom
holds
true
whether
you
are
taking
2345678901234567890123456789012123456789012345678901
2
1
2
12345678901234567890123456789012123456789012345678901
the
MCAT
(medical
school
exam)
or
a
third
grade
math
and
reading
exam.
2
12345678901234567890123456789012123456789012345678901
2
12345678901234567890123456789012123456789012345678901
12345678901234567890123456789012123456789012345678901
Now, answer this question: How many computer adaptive tests have you 2
2345678901234567890123456789012123456789012345678901
12345678901234567890123456789012123456789012345678901 2
2
12345678901234567890123456789012123456789012345678901
taken in your lifetime?
2
12345678901234567890123456789012123456789012345678901
2
12345678901234567890123456789012123456789012345678901
Most people would answer none or one or two to the above question, 2
12345678901234567890123456789012123456789012345678901
2
12345678901234567890123456789012123456789012345678901
1 but the real answer is, not enough to be comfortable with them. The 2
2
12345678901234567890123456789012123456789012345678901
adaptive format is unusual, and this can work against your score. Most 2
12345678901234567890123456789012123456789012345678901
2
12345678901234567890123456789012123456789012345678901
people are already a little nervous about taking the GRE, and the added 2
12345678901234567890123456789012123456789012345678901
12345678901234567890123456789012123456789012345678901
stress of having to tap dance with the three-legged wonder that is the GRE 2
2
12345678901234567890123456789012123456789012345678901
2
12345678901234567890123456789012123456789012345678901
CAT
only
makes
matters
worse.
2
12345678901234567890123456789012123456789012345678901
2345678901234567890123456789012123456789012345678901
12345678901234567890123456789012123456789012345678901 2
12345678901234567890123456789012123456789012345678901
In addition to being stranger than a paper-and-pencil test, the GRE CAT is 2
2
12345678901234567890123456789012123456789012345678901
2
12345678901234567890123456789012123456789012345678901
riskier.
Heres
why:
2
1
2345678901234567890123456789012123456789012345678901
12345678901234567890123456789012123456789012345678901 2
12345678901234567890123456789012123456789012345678901 2
2
1
Typical Test
GRE CAT
What It Means
2
12345678901234567890123456789012123456789012345678901
2
12345678901234567890123456789012123456789012345678901
2345678901234567890123456789012123456789012345678901
2
12345678901234567890123456789012123456789012345678901
Every question
Earlier questions are Minor mistakes at
2
12345678901234567890123456789012123456789012345678901
2
1
counts
the
same
weighted
more
the
beginning
have
2
12345678901234567890123456789012123456789012345678901
2
12345678901234567890123456789012123456789012345678901
toward
your
score.
than
later
ones.
greater
negative
2345678901234567890123456789012123456789012345678901
12345678901234567890123456789012123456789012345678901 2
2
12345678901234567890123456789012123456789012345678901
impact.
2
1
2
12345678901234567890123456789012123456789012345678901
You
can
skip
a
Must
answer
each
Limits
your
options
12345678901234567890123456789012123456789012345678901 2
2345678901234567890123456789012123456789012345678901
2
12345678901234567890123456789012123456789012345678901
question that you
question presented.
regarding a tough
2
12345678901234567890123456789012123456789012345678901
2
12345678901234567890123456789012123456789012345678901
dont
understand.
You
cant
proceed
problem
you
dont
12345678901234567890123456789012123456789012345678901 2
2
1
to the next question understandyou
2
12345678901234567890123456789012123456789012345678901
2
12345678901234567890123456789012123456789012345678901
until
you
have
submust
guess.
2
12345678901234567890123456789012123456789012345678901
2
12345678901234567890123456789012123456789012345678901
mitted
an
answer
to
12345678901234567890123456789012123456789012345678901 2
2345678901234567890123456789012123456789012345678901
2
12345678901234567890123456789012123456789012345678901
the current one.
2
12345678901234567890123456789012123456789012345678901
2
12345678901234567890123456789012123456789012345678901
Can go back and
Once the question is Adaptive format
2
12345678901234567890123456789012123456789012345678901
2
1
check
work
for
answered
and
verieliminates
the
abil2345678901234567890123456789012123456789012345678901
12345678901234567890123456789012123456789012345678901 2
2
1
careless errors.
fied, its out of your ity to go back and
2
12345678901234567890123456789012123456789012345678901
2
12345678901234567890123456789012123456789012345678901
hands.
catch
any
mistakes.
2
12345678901234567890123456789012123456789012345678901
2345678901234567890123456789012123456789012345678901
2
1
12345678901234567890123456789012123456789012345678901 2
12345678901234567890123456789012123456789012345678901 2
12345678901234567890123456789012123456789012345678901 2
123456789012345678901234567890121234567890123456789012

www.petersons.com

http://www.xtremepapers.net

Chapter 1: Everything You Need to Know about the GRE CAT

123456789012345678901234567890121234567890123456789012
12345678901234567890123456789012123456789012345678901 2
2
12345678901234567890123456789012123456789012345678901
If the GRE CAT were a pencil-and-paper test, you could begin with the 2
12345678901234567890123456789012123456789012345678901
2
12345678901234567890123456789012123456789012345678901
easy questions and skip over any questions that were too tough for you on 2
12345678901234567890123456789012123456789012345678901
2
12345678901234567890123456789012123456789012345678901
the first pass. On a second pass, you could review your work to catch any 2
12345678901234567890123456789012123456789012345678901
2345678901234567890123456789012123456789012345678901
1 careless mistakes, and devote time on those problems you skipped. This 2
2
12345678901234567890123456789012123456789012345678901
2
12345678901234567890123456789012123456789012345678901
sound
strategy
is
no
longer
available
to
you,
because
of
the
adaptive
nature
2
12345678901234567890123456789012123456789012345678901
2
12345678901234567890123456789012123456789012345678901
of
the
test.
Instead,
you
are
thrown
into
the
middle
of
the
fray,
and
how
2345678901234567890123456789012123456789012345678901
2
1
2
12345678901234567890123456789012123456789012345678901
well
you
answer
the
first
half
of
the
questions
posed
to
you
goes
a
long
way
2
12345678901234567890123456789012123456789012345678901
2
12345678901234567890123456789012123456789012345678901
toward
determining
your
score.
2
12345678901234567890123456789012123456789012345678901
2345678901234567890123456789012123456789012345678901
2
1
2
12345678901234567890123456789012123456789012345678901
This
doesnt
mean
the
adaptive
nature
will
invariably
lead
to
an
unfairly
2
12345678901234567890123456789012123456789012345678901
2
12345678901234567890123456789012123456789012345678901
low
score.
You
could
be
given
an
initial
set
of
questions
that
plays
right
2
12345678901234567890123456789012123456789012345678901
2345678901234567890123456789012123456789012345678901
1 into your hands, like a hard antonym that you just happen to know the 2
2
12345678901234567890123456789012123456789012345678901
12345678901234567890123456789012123456789012345678901
definition of or a particular type of math problem youve always been 2
2
12345678901234567890123456789012123456789012345678901
12345678901234567890123456789012123456789012345678901
good at. If thats the case, the weighted nature of the GRE CAT works to 2
2345678901234567890123456789012123456789012345678901
12345678901234567890123456789012123456789012345678901 22
12345678901234567890123456789012123456789012345678901
your advantage. Either way, the GRE CAT is a riskier test because many of
2
12345678901234567890123456789012123456789012345678901
the fail-safes you have on other standardized testsgetting easy problems 2
12345678901234567890123456789012123456789012345678901
2
12345678901234567890123456789012123456789012345678901
first, being able to go back over the entire section and not just one 2
12345678901234567890123456789012123456789012345678901
2
1 questionare not present on the GRE CAT.
2
12345678901234567890123456789012123456789012345678901
2
12345678901234567890123456789012123456789012345678901
2
12345678901234567890123456789012123456789012345678901
So,
yes,
you
get
your
score
back
super
early,
but
if
youre
not
ready
for
the
2345678901234567890123456789012123456789012345678901
12345678901234567890123456789012123456789012345678901 22
12345678901234567890123456789012123456789012345678901
adaptive format, it might not be the kind of score you want to get back 2
12345678901234567890123456789012123456789012345678901
2
12345678901234567890123456789012123456789012345678901
super early.
2
12345678901234567890123456789012123456789012345678901
12345678901234567890123456789012123456789012345678901 22
12345678901234567890123456789012123456789012345678901
Learning about the computer adaptive format will give you the opportu- 2
12345678901234567890123456789012123456789012345678901
2
12345678901234567890123456789012123456789012345678901
1 nity to eliminate many of the negative aspects of the CAT while keeping 2
2345678901234567890123456789012123456789012345678901
2
12345678901234567890123456789012123456789012345678901
the good ones, so the rest of this chapter is devoted to just that.
2
12345678901234567890123456789012123456789012345678901
2
1
12345678901234567890123456789012123456789012345678901 2
2
12345678901234567890123456789012123456789012345678901
2
12345678901234567890123456789012123456789012345678901
Putting
the Computer into the Computer
2
12345678901234567890123456789012123456789012345678901
12345678901234567890123456789012123456789012345678901 2
2
12345678901234567890123456789012123456789012345678901
Adaptive
Test
2
12345678901234567890123456789012123456789012345678901
2345678901234567890123456789012123456789012345678901
2
12345678901234567890123456789012123456789012345678901
Take a moment and give an honest answer to the following question:
2
12345678901234567890123456789012123456789012345678901
2
1
2
12345678901234567890123456789012123456789012345678901
How would you describe your comfort level with computers?
12345678901234567890123456789012123456789012345678901 2
2345678901234567890123456789012123456789012345678901
2
12345678901234567890123456789012123456789012345678901
In other words, do you feel your trusty laptop is like an extension of your 2
12345678901234567890123456789012123456789012345678901
2
12345678901234567890123456789012123456789012345678901
body, or do you believe that computers are little more than mutant 2
12345678901234567890123456789012123456789012345678901
2
1 typewriters determined to destroy the world? Take your time.
2
12345678901234567890123456789012123456789012345678901
2
12345678901234567890123456789012123456789012345678901
2
12345678901234567890123456789012123456789012345678901
Whichever
category
you
start
out
in,
by
the
time
you
take
the
GRE
CAT
2
12345678901234567890123456789012123456789012345678901
2
12345678901234567890123456789012123456789012345678901
you
need
to
be
completely
comfortable
navigating
through
a
computer
2345678901234567890123456789012123456789012345678901
12345678901234567890123456789012123456789012345678901 22
12345678901234567890123456789012123456789012345678901
test. If you start out already feeling good about computers, then it wont 2
12345678901234567890123456789012123456789012345678901
2
12345678901234567890123456789012123456789012345678901
1 take much time to get used to the commands used when taking the GRE 2
2345678901234567890123456789012123456789012345678901
2
12345678901234567890123456789012123456789012345678901
1 CAT. If youre closer to the mutant typewriter faction, then the amount 2
12345678901234567890123456789012123456789012345678901
of time needed to acclimate yourself will probably be much longer. Even if 2
2
12345678901234567890123456789012123456789012345678901
2
12345678901234567890123456789012123456789012345678901
that
is
the
case,
it
will
be
time
well
spent,
since
the
day
you
need
to
take
the
2345678901234567890123456789012123456789012345678901
2
1
12345678901234567890123456789012123456789012345678901 2
12345678901234567890123456789012123456789012345678901 2
12345678901234567890123456789012123456789012345678901 2
123456789012345678901234567890121234567890123456789012

http://www.xtremepapers.net

Part I: Understanding the GRE

10

123456789012345678901234567890121234567890123456789012
12345678901234567890123456789012123456789012345678901 2
2
12345678901234567890123456789012123456789012345678901
test should be exclusively about answering GRE questions correctly and 2
12345678901234567890123456789012123456789012345678901
2
12345678901234567890123456789012123456789012345678901
not about learning how to move through a computer adaptive test.
2
12345678901234567890123456789012123456789012345678901
12345678901234567890123456789012123456789012345678901 2
12345678901234567890123456789012123456789012345678901
Before starting the GRE CAT, you go through a computer tutorial, which 2
2
12345678901234567890123456789012123456789012345678901
2
12345678901234567890123456789012123456789012345678901
goes
over
every
facet
of
the
onscreen
instructions
in
excruciating
detail.
2
12345678901234567890123456789012123456789012345678901
2
12345678901234567890123456789012123456789012345678901
You
could
wait
until
test
day
and
then
spend
1545
minutes
in
the
tutorial
2
12345678901234567890123456789012123456789012345678901
2345678901234567890123456789012123456789012345678901
1 familiarizing yourself with all the different instructions, but wouldnt you 2
2
12345678901234567890123456789012123456789012345678901
2
12345678901234567890123456789012123456789012345678901
rather
learn
the
instructions
beforehand
so
you
can
skip
the
boring
tutorial
2
12345678901234567890123456789012123456789012345678901
2
12345678901234567890123456789012123456789012345678901
and
get
right
to
the
actual
test?
The
goal
on
that
day
is
to
use
all
your
2345678901234567890123456789012123456789012345678901
2
1
2
12345678901234567890123456789012123456789012345678901
brainpower
to
take
a
GRE
CAT,
not
waste
time
and
mental
energy
2
12345678901234567890123456789012123456789012345678901
2
12345678901234567890123456789012123456789012345678901
learning
about
how
to
take
a
GRE
CAT.
2
12345678901234567890123456789012123456789012345678901
2345678901234567890123456789012123456789012345678901
2
1
12345678901234567890123456789012123456789012345678901
Be sure you go over this tutorial before taking any tests on that disk. 2
2
12345678901234567890123456789012123456789012345678901
Learning about the computer commands ahead of time is a valuable skill. 2
12345678901234567890123456789012123456789012345678901
2
12345678901234567890123456789012123456789012345678901
A tutorial similar to the one youll see on test day is contained on the CD 2
12345678901234567890123456789012123456789012345678901
2
12345678901234567890123456789012123456789012345678901
that came with this book. Or, check out the Appendix on page 332, which 2
12345678901234567890123456789012123456789012345678901
2
12345678901234567890123456789012123456789012345678901
goes over all the major screen commands, what they look like, what each 2
12345678901234567890123456789012123456789012345678901
2345678901234567890123456789012123456789012345678901
2
12345678901234567890123456789012123456789012345678901
1 command means, and when they will appear. Be sure you understand all 2
2
12345678901234567890123456789012123456789012345678901
the different commands before you take the actual GRE CAT.
2
12345678901234567890123456789012123456789012345678901
2
12345678901234567890123456789012123456789012345678901
2345678901234567890123456789012123456789012345678901
12345678901234567890123456789012123456789012345678901
Now that weve talked about the instructions youll see on the computer, 2
2
12345678901234567890123456789012123456789012345678901
2
12345678901234567890123456789012123456789012345678901
lets
talk
about
the
computers
themselves.
You
know
those
computers
you
12345678901234567890123456789012123456789012345678901 2
12345678901234567890123456789012123456789012345678901
see on every TV show, the ones with monstrous flat screens, dynamic 2
2
12345678901234567890123456789012123456789012345678901
2
12345678901234567890123456789012123456789012345678901
graphics,
and
blazing
computing
speed?
Those
are
NOT
the
computers
12345678901234567890123456789012123456789012345678901 2
2
12345678901234567890123456789012123456789012345678901
1 you will use to take the GRE CAT. Instead, the computer you will probably 2
2345678901234567890123456789012123456789012345678901
12345678901234567890123456789012123456789012345678901
use resembles the kind you find in a public library. It will be a no-frills, 2
2
12345678901234567890123456789012123456789012345678901
1 functional PC with an average keyboard, average mouse, average screen, 2
12345678901234567890123456789012123456789012345678901 2
2
12345678901234567890123456789012123456789012345678901
average everything, really.
2
12345678901234567890123456789012123456789012345678901
2
12345678901234567890123456789012123456789012345678901
Many people who work on computers for a long time make modifications 2
12345678901234567890123456789012123456789012345678901
2
12345678901234567890123456789012123456789012345678901
to their computers to maximize comfort. They might replace the regular 2
12345678901234567890123456789012123456789012345678901
2345678901234567890123456789012123456789012345678901
12345678901234567890123456789012123456789012345678901
mouse with a wireless trackball version designed to reduce carpal tunnel 2
2
12345678901234567890123456789012123456789012345678901
1 problems and place ultra-fine netting over their screen to reduce the glare. 2
12345678901234567890123456789012123456789012345678901 2
12345678901234567890123456789012123456789012345678901
None of these modifications will be in place on the GRE computer, so if 2
2
12345678901234567890123456789012123456789012345678901
2
12345678901234567890123456789012123456789012345678901
you
are
someone
who
uses
modified
accessories,
you
have
two
options:
2
12345678901234567890123456789012123456789012345678901
2
12345678901234567890123456789012123456789012345678901
2
12345678901234567890123456789012123456789012345678901
1.
You
can
replace
your
nice
gear
with
the
basic
equipment
in
order
2345678901234567890123456789012123456789012345678901
12345678901234567890123456789012123456789012345678901 2
2
12345678901234567890123456789012123456789012345678901
to simulate the real GRE experience.
2
12345678901234567890123456789012123456789012345678901
12345678901234567890123456789012123456789012345678901 2
1
2. You can use your nice gear at home but be aware that during the 2
2
12345678901234567890123456789012123456789012345678901
real test none of it will be there for you.
2
12345678901234567890123456789012123456789012345678901
2
12345678901234567890123456789012123456789012345678901
2
12345678901234567890123456789012123456789012345678901
Both
options
have
advantages.
Option
1
gives
you
greater
verisimilitude
(a
12345678901234567890123456789012123456789012345678901 2
2345678901234567890123456789012123456789012345678901
2
12345678901234567890123456789012123456789012345678901
1 good GRE vocabulary word, by the way). If you want your computer 2
12345678901234567890123456789012123456789012345678901
practice to be as close to the real thing as possible, this is the plan for you. 2
2
12345678901234567890123456789012123456789012345678901
2
12345678901234567890123456789012123456789012345678901
On
the
other
hand,
if
you
understand
ahead
of
time
what
the
real
thing
2345678901234567890123456789012123456789012345678901
2
1
2
12345678901234567890123456789012123456789012345678901
will
be
like,
theres
no
reason
not
to
practice
in
a
way
thats
more
12345678901234567890123456789012123456789012345678901 2
12345678901234567890123456789012123456789012345678901 2
123456789012345678901234567890121234567890123456789012

www.petersons.com

http://www.xtremepapers.net

Chapter 1: Everything You Need to Know about the GRE CAT

Alert!

123456789012345678901234567890121234567890123456789012
12345678901234567890123456789012123456789012345678901 2
2
12345678901234567890123456789012123456789012345678901
physically or aesthetically comfortable. Ultimately, the type of computer 2
12345678901234567890123456789012123456789012345678901
2
12345678901234567890123456789012123456789012345678901
mouse you use wont be the deciding factor in your score, so this is not a 2
12345678901234567890123456789012123456789012345678901
2
12345678901234567890123456789012123456789012345678901
major point, but every little thing counts. Knowing what to expect and 2
12345678901234567890123456789012123456789012345678901
2345678901234567890123456789012123456789012345678901
1 planning for it is a good way to boost your confidence level going into 2
2
12345678901234567890123456789012123456789012345678901
2
12345678901234567890123456789012123456789012345678901
the
exam.
2
12345678901234567890123456789012123456789012345678901
2
12345678901234567890123456789012123456789012345678901
2345678901234567890123456789012123456789012345678901
2
1
2
12345678901234567890123456789012123456789012345678901
Before you register for the test by phone or computer, try to visit the
2
12345678901234567890123456789012123456789012345678901
2
12345678901234567890123456789012123456789012345678901
actual
testing
site
ahead
of
timebefore
you
plunk
down
your
money.
2
12345678901234567890123456789012123456789012345678901
2345678901234567890123456789012123456789012345678901
2
1 Most testing sites are fine, but what if the testing site gives you a bad vibe
2
12345678901234567890123456789012123456789012345678901
2
12345678901234567890123456789012123456789012345678901
or
something?
Dont
go
there!
You
need
to
be
at
your
best
on
test
day,
so
2
12345678901234567890123456789012123456789012345678901
2
12345678901234567890123456789012123456789012345678901
make
sure
the
site
is
adequate
before
you
sign
up.
Once
you
call
and
get
a
2345678901234567890123456789012123456789012345678901
2
1
2
12345678901234567890123456789012123456789012345678901
time,
youre
locked
in,
and
even
if
you
cancel
the
appointment,
you
lose
a
2
12345678901234567890123456789012123456789012345678901
2
12345678901234567890123456789012123456789012345678901
large
chunk
of
change.
2
12345678901234567890123456789012123456789012345678901
2345678901234567890123456789012123456789012345678901
12345678901234567890123456789012123456789012345678901 22
12345678901234567890123456789012123456789012345678901 2
12345678901234567890123456789012123456789012345678901
The
Adaptive
2
12345678901234567890123456789012123456789012345678901
12345678901234567890123456789012123456789012345678901 22
12345678901234567890123456789012123456789012345678901
1 To get a taste of what the computer adaptive format is like (without taking 2
12345678901234567890123456789012123456789012345678901
a test), rent a movie you know nothing about. Dont look at the cover or 2
2
12345678901234567890123456789012123456789012345678901
read any summary of the events of the film. Fast forward to the middle of 2
12345678901234567890123456789012123456789012345678901
2345678901234567890123456789012123456789012345678901
2
12345678901234567890123456789012123456789012345678901
the film and begin watching at that point. After a few minutes, if you think 2
12345678901234567890123456789012123456789012345678901
2
12345678901234567890123456789012123456789012345678901
you understand whats going on, keep watching. If you start watching the 2
12345678901234567890123456789012123456789012345678901
2
12345678901234567890123456789012123456789012345678901
middle of the film and have no idea whats going on, go back about 20 2
12345678901234567890123456789012123456789012345678901
12345678901234567890123456789012123456789012345678901
minutes and start watching from that point. Move forward when the 2
2
12345678901234567890123456789012123456789012345678901
2
12345678901234567890123456789012123456789012345678901
movie
makes
sense,
and
rewind
when
it
doesnt.
Keep
watching
until
you
2
1
2345678901234567890123456789012123456789012345678901
2
12345678901234567890123456789012123456789012345678901
reach the end of the film.
2
12345678901234567890123456789012123456789012345678901
2
1
2
12345678901234567890123456789012123456789012345678901
You
have
just
watched
your
first
adaptive
movie.
Was
it
fun?
How
was
the
2
12345678901234567890123456789012123456789012345678901
2345678901234567890123456789012123456789012345678901
2
12345678901234567890123456789012123456789012345678901
popcorn?
2
12345678901234567890123456789012123456789012345678901
2
1
12345678901234567890123456789012123456789012345678901
You probably dont watch movies this way on a regular basis, and you 2
2
12345678901234567890123456789012123456789012345678901
12345678901234567890123456789012123456789012345678901
dont take computer adaptive tests like the GRE every day, either. The 2
2
12345678901234567890123456789012123456789012345678901
danger is that the unusual format leads you to take the wrong approach to 2
12345678901234567890123456789012123456789012345678901
2
12345678901234567890123456789012123456789012345678901
the test. The chart on the following page lists some common mistakes 2
12345678901234567890123456789012123456789012345678901
2345678901234567890123456789012123456789012345678901
2
12345678901234567890123456789012123456789012345678901
people make when taking the CAT.
2
12345678901234567890123456789012123456789012345678901
12345678901234567890123456789012123456789012345678901 22
12345678901234567890123456789012123456789012345678901 2
1
2
12345678901234567890123456789012123456789012345678901
2
12345678901234567890123456789012123456789012345678901
2
12345678901234567890123456789012123456789012345678901
2
12345678901234567890123456789012123456789012345678901
12345678901234567890123456789012123456789012345678901 2
2345678901234567890123456789012123456789012345678901
12345678901234567890123456789012123456789012345678901 22
12345678901234567890123456789012123456789012345678901 2
12345678901234567890123456789012123456789012345678901 2
12345678901234567890123456789012123456789012345678901 2
1
2
12345678901234567890123456789012123456789012345678901
12345678901234567890123456789012123456789012345678901 2
2
12345678901234567890123456789012123456789012345678901
12345678901234567890123456789012123456789012345678901 2
2
12345678901234567890123456789012123456789012345678901
12345678901234567890123456789012123456789012345678901 2
12345678901234567890123456789012123456789012345678901 2
12345678901234567890123456789012123456789012345678901 2
12345678901234567890123456789012123456789012345678901 2 11
123456789012345678901234567890121234567890123456789012

http://www.xtremepapers.net

Part I: Understanding the GRE

12

123456789012345678901234567890121234567890123456789012
12345678901234567890123456789012123456789012345678901 2
2
12345678901234567890123456789012123456789012345678901
2
12345678901234567890123456789012123456789012345678901
Common Thinking
But Really . . .
2
12345678901234567890123456789012123456789012345678901
2
12345678901234567890123456789012123456789012345678901
2
12345678901234567890123456789012123456789012345678901
Tests
go
from
easy
to
GRE
CAT
problems
start
in
the
middle,
2
12345678901234567890123456789012123456789012345678901
2345678901234567890123456789012123456789012345678901
2
1
hard problems.
and then go all over the place from there
2
12345678901234567890123456789012123456789012345678901
2
12345678901234567890123456789012123456789012345678901
depending on how you score.
2
12345678901234567890123456789012123456789012345678901
2
12345678901234567890123456789012123456789012345678901
Early
problems
on
the
GRE
play
a
greater
Work
at
a
consistent
2345678901234567890123456789012123456789012345678901
2
1
2
12345678901234567890123456789012123456789012345678901
role
in
determining
your
score
than
later
pace
to
have
a
2
12345678901234567890123456789012123456789012345678901
2
12345678901234567890123456789012123456789012345678901
ones,
so
you
should
make
sure
to
spend
chance
to
answer
2
12345678901234567890123456789012123456789012345678901
2345678901234567890123456789012123456789012345678901
2
1
more time on the first 1015 problems.
every problem.
2
12345678901234567890123456789012123456789012345678901
2
12345678901234567890123456789012123456789012345678901
Even if this means you end up guessing on
2
12345678901234567890123456789012123456789012345678901
2
12345678901234567890123456789012123456789012345678901
the last five problems in a section because
2345678901234567890123456789012123456789012345678901
2
1
you have no time left, your score will still
2
12345678901234567890123456789012123456789012345678901
2
12345678901234567890123456789012123456789012345678901
be better than if you rushed through the
2
12345678901234567890123456789012123456789012345678901
2
12345678901234567890123456789012123456789012345678901
first
questions.
Remember,
even
if
you
2345678901234567890123456789012123456789012345678901
12345678901234567890123456789012123456789012345678901 2
2
12345678901234567890123456789012123456789012345678901
dont have time for the last few problems,
2
12345678901234567890123456789012123456789012345678901
2
12345678901234567890123456789012123456789012345678901
be
sure
to
guess.
12345678901234567890123456789012123456789012345678901 2
2
12345678901234567890123456789012123456789012345678901
If you have the necessary knowledge, then
If I study hard
2
1
2
12345678901234567890123456789012123456789012345678901
youll quickly find yourself mired in the
enough, the test
2
12345678901234567890123456789012123456789012345678901
2
12345678901234567890123456789012123456789012345678901
hardest
math
and
verbal
questions
the
should
be
easy.
2345678901234567890123456789012123456789012345678901
12345678901234567890123456789012123456789012345678901 2
2
12345678901234567890123456789012123456789012345678901
computer can dredge up. This means youre
2
12345678901234567890123456789012123456789012345678901
2
12345678901234567890123456789012123456789012345678901
heading
toward
a
good
score,
but
it
doesnt
12345678901234567890123456789012123456789012345678901 2
2
12345678901234567890123456789012123456789012345678901
mean the test will be easy. Unlike a pencil2
12345678901234567890123456789012123456789012345678901
2
12345678901234567890123456789012123456789012345678901
and-paper test with set easy, medium, and
12345678901234567890123456789012123456789012345678901 2
2
1
hard problems, the computer will adapt to
2
12345678901234567890123456789012123456789012345678901
throw you the nastiest, toughest, most
2
12345678901234567890123456789012123456789012345678901
2
12345678901234567890123456789012123456789012345678901
convoluted
questions
it
has
to
offer.
12345678901234567890123456789012123456789012345678901 2
2
12345678901234567890123456789012123456789012345678901
2
12345678901234567890123456789012123456789012345678901
2
12345678901234567890123456789012123456789012345678901
On
many
standardized
tests,
people
rush
through
the
initial
easy
questions
12345678901234567890123456789012123456789012345678901 2
12345678901234567890123456789012123456789012345678901
because they want to answer every problem. This impatience often leads to 2
2
12345678901234567890123456789012123456789012345678901
2345678901234567890123456789012123456789012345678901
12345678901234567890123456789012123456789012345678901
careless errors at the beginning. This impatience is harmful on a regular test, 2
2
12345678901234567890123456789012123456789012345678901
1 but its deadly on the GRE CAT. The first 1015 questions you face on each 2
12345678901234567890123456789012123456789012345678901 2
12345678901234567890123456789012123456789012345678901
section are the most crucial, and you should slow down and take as much as 2
2
12345678901234567890123456789012123456789012345678901
12345678901234567890123456789012123456789012345678901
2 minutes apiece if it will help you answer them correctly. As for the last 5 2
2
12345678901234567890123456789012123456789012345678901
12345678901234567890123456789012123456789012345678901
questions? If you dont have time for them, just guess. They are not as critical 2
12345678901234567890123456789012123456789012345678901 2
12345678901234567890123456789012123456789012345678901
to your score. If you can get to them, its a bonus, but the real way to improve 2
2
12345678901234567890123456789012123456789012345678901
your score is to focus on the earlier questions to the best of your ability.
2
12345678901234567890123456789012123456789012345678901
2
12345678901234567890123456789012123456789012345678901
2
12345678901234567890123456789012123456789012345678901
Now
that
the
Slow
down
and
get
the
early
question
right
mantra
has
2345678901234567890123456789012123456789012345678901
12345678901234567890123456789012123456789012345678901 2
12345678901234567890123456789012123456789012345678901
been beaten into your skull yet again, it might be a good time to clarify just 2
2
12345678901234567890123456789012123456789012345678901
2
12345678901234567890123456789012123456789012345678901
what
an
easy,
medium,
and
hard
question
looks
like.
Officially,
a
hard
2
1
2345678901234567890123456789012123456789012345678901
2
12345678901234567890123456789012123456789012345678901
1 problem is one that previous test-takers have seen and found to be very 2
12345678901234567890123456789012123456789012345678901
difficult to answer correctly. The official derivation of easy and 2
2
12345678901234567890123456789012123456789012345678901
2
12345678901234567890123456789012123456789012345678901
medium
also
comes
from
how
well
previous
test-takers
did
when
trying
2345678901234567890123456789012123456789012345678901
2
1
2
12345678901234567890123456789012123456789012345678901
to
answer
these
problems.
12345678901234567890123456789012123456789012345678901 2
12345678901234567890123456789012123456789012345678901 2
123456789012345678901234567890121234567890123456789012

www.petersons.com

http://www.xtremepapers.net

X-Ref

Chapter 1: Everything You Need to Know about the GRE CAT

123456789012345678901234567890121234567890123456789012
12345678901234567890123456789012123456789012345678901 2
2
12345678901234567890123456789012123456789012345678901
Yet these labels are all subjective, since everyone carries around a unique 2
12345678901234567890123456789012123456789012345678901
2
12345678901234567890123456789012123456789012345678901
knowledge set. You might have a knack for geometry, so a medium 2
12345678901234567890123456789012123456789012345678901
2
12345678901234567890123456789012123456789012345678901
geometry problem appears easy to you. The best way to get a feel for a 2
12345678901234567890123456789012123456789012345678901
2345678901234567890123456789012123456789012345678901
1 problems difficulty level is to look over samples of easy, medium, and 2
2
12345678901234567890123456789012123456789012345678901
2
12345678901234567890123456789012123456789012345678901
hard
questions.
You
can
find
these
throughout
this
book,
and
you
can
also
2
12345678901234567890123456789012123456789012345678901
2
12345678901234567890123456789012123456789012345678901
th
go
right
to
the
source
and
purchase
The
Official
Guide
to
the
GRE,
10
2345678901234567890123456789012123456789012345678901
2
1
2
12345678901234567890123456789012123456789012345678901
Edition,
by
the
Educational
Testing
Service.
This
roughly
$20
book
2
12345678901234567890123456789012123456789012345678901
2
12345678901234567890123456789012123456789012345678901
contains
a
host
of
old
pencil-and-paper
GREs
with
questions
arranged
in
2
12345678901234567890123456789012123456789012345678901
2345678901234567890123456789012123456789012345678901
1 old-timey easy/medium/hard order of difficulty. The exams are all over a 2
2
12345678901234567890123456789012123456789012345678901
2
12345678901234567890123456789012123456789012345678901
decade
old,
but
they
do
give
you
an
idea
of
what
constitutes
an
easy,
2
12345678901234567890123456789012123456789012345678901
2
12345678901234567890123456789012123456789012345678901
medium,
and
hard
problem.
2345678901234567890123456789012123456789012345678901
2
1
2
12345678901234567890123456789012123456789012345678901
2
12345678901234567890123456789012123456789012345678901
2
12345678901234567890123456789012123456789012345678901
2
12345678901234567890123456789012123456789012345678901
The
Official
Guide
is
a
good
place
to
work
on
sample
problems,
but
the
2345678901234567890123456789012123456789012345678901
12345678901234567890123456789012123456789012345678901 22
12345678901234567890123456789012123456789012345678901
format has changed a lot since these test were issued, so dont bother
2
12345678901234567890123456789012123456789012345678901
2
12345678901234567890123456789012123456789012345678901
working the sections under timed conditions. Use the sample tests at the
12345678901234567890123456789012123456789012345678901 22
12345678901234567890123456789012123456789012345678901
2
1 back of this book for that, as they are geared toward the new exam.
2
12345678901234567890123456789012123456789012345678901
2
12345678901234567890123456789012123456789012345678901
2
12345678901234567890123456789012123456789012345678901
2345678901234567890123456789012123456789012345678901
12345678901234567890123456789012123456789012345678901
The ability to determine easy, medium, and hard problems is a nice skill to 2
2
12345678901234567890123456789012123456789012345678901
have on the CAT, but it is not a crucial one. Dont spend any amount of 2
12345678901234567890123456789012123456789012345678901
2
12345678901234567890123456789012123456789012345678901
time looking at a problem and debating whether or not its a medium or 2
12345678901234567890123456789012123456789012345678901
2
12345678901234567890123456789012123456789012345678901
hard one. Either way, its in front of you, so you have to work it one way or 2
12345678901234567890123456789012123456789012345678901
2
12345678901234567890123456789012123456789012345678901
another. But understanding what kind of problem you are looking at can 2
12345678901234567890123456789012123456789012345678901
1 help you avoid incorrect answer choices. For example, if you are taking the 2
2
12345678901234567890123456789012123456789012345678901
12345678901234567890123456789012123456789012345678901
math section and doing pretty well, youll notice the problems are getting 2
2
12345678901234567890123456789012123456789012345678901
2
12345678901234567890123456789012123456789012345678901
more
difficult.
Suddenly,
you
get
the
following
probability
question:
2
12345678901234567890123456789012123456789012345678901
2345678901234567890123456789012123456789012345678901
12345678901234567890123456789012123456789012345678901 22
12345678901234567890123456789012123456789012345678901 2
1
There are seven tokens in a bag, labeled 8 through 14 consecutively. The
2
12345678901234567890123456789012123456789012345678901
12345678901234567890123456789012123456789012345678901
first number pulled out of the bag is 10. What are the odds that the next 2
2345678901234567890123456789012123456789012345678901
12345678901234567890123456789012123456789012345678901 22
12345678901234567890123456789012123456789012345678901
two numbers pulled out of the bag will BOTH be less than 10?
2
1
2
12345678901234567890123456789012123456789012345678901
1
2
12345678901234567890123456789012123456789012345678901
A.
2345678901234567890123456789012123456789012345678901
2
12345678901234567890123456789012123456789012345678901
15
2
12345678901234567890123456789012123456789012345678901
12345678901234567890123456789012123456789012345678901 22
12345678901234567890123456789012123456789012345678901
3
2
1
B.
2345678901234567890123456789012123456789012345678901
2
12345678901234567890123456789012123456789012345678901
14
2
12345678901234567890123456789012123456789012345678901
2
12345678901234567890123456789012123456789012345678901
2
2
12345678901234567890123456789012123456789012345678901
C.
2
1
7
2345678901234567890123456789012123456789012345678901
12345678901234567890123456789012123456789012345678901 22
12345678901234567890123456789012123456789012345678901 2
12345678901234567890123456789012123456789012345678901
1
2
12345678901234567890123456789012123456789012345678901
D.
2
1
3
2345678901234567890123456789012123456789012345678901
12345678901234567890123456789012123456789012345678901 22
1
2
2
12345678901234567890123456789012123456789012345678901
E.
2
12345678901234567890123456789012123456789012345678901
5
2
12345678901234567890123456789012123456789012345678901
2345678901234567890123456789012123456789012345678901
2
1
12345678901234567890123456789012123456789012345678901 2
12345678901234567890123456789012123456789012345678901 2
12345678901234567890123456789012123456789012345678901 2 13
123456789012345678901234567890121234567890123456789012

http://www.xtremepapers.net

Part I: Understanding the GRE

http://www.xtremepapers.net

X-Ref

www.petersons.com

Note

14

123456789012345678901234567890121234567890123456789012
12345678901234567890123456789012123456789012345678901 2
2
12345678901234567890123456789012123456789012345678901
12345678901234567890123456789012123456789012345678901 2
12345678901234567890123456789012123456789012345678901
On the CAT exam, letters will not be used to designate choices, as each 2
2
12345678901234567890123456789012123456789012345678901
2
12345678901234567890123456789012123456789012345678901
choice
will
simply
appear
on
the
screen
with
a
hollow
oval
next
to
it.
2
12345678901234567890123456789012123456789012345678901
2345678901234567890123456789012123456789012345678901
1 Obviously, the test-maker did not care that this lack of lettering would 2
2
12345678901234567890123456789012123456789012345678901
12345678901234567890123456789012123456789012345678901
make it hard on the poor souls who make a living writing test-prep 2
2
12345678901234567890123456789012123456789012345678901
12345678901234567890123456789012123456789012345678901
material. Throughout this book, all answer choices will have letters next 2
2345678901234567890123456789012123456789012345678901
2
1
12345678901234567890123456789012123456789012345678901
to them so that they can be discussed without having to resort to phrases 2
2
12345678901234567890123456789012123456789012345678901
12345678901234567890123456789012123456789012345678901
like the second oval from the top or you know, the answer thats got 2
2
12345678901234567890123456789012123456789012345678901
the word hamster in it.
2
12345678901234567890123456789012123456789012345678901
2
12345678901234567890123456789012123456789012345678901
2
12345678901234567890123456789012123456789012345678901
2
12345678901234567890123456789012123456789012345678901
This is a medium difficult problem, something to keep in mind when 2
12345678901234567890123456789012123456789012345678901
2345678901234567890123456789012123456789012345678901
1 you look over the answer choices. If you try to figure this problem in your 2
2
12345678901234567890123456789012123456789012345678901
2
12345678901234567890123456789012123456789012345678901
head, you might think that there are two numbers less than 10 (8 and 9) 2
12345678901234567890123456789012123456789012345678901
2
12345678901234567890123456789012123456789012345678901
and 7 tokens total, or 6 tokens now that the 10 has been removed. Choice 2
12345678901234567890123456789012123456789012345678901
2
12345678901234567890123456789012123456789012345678901
2
2
12345678901234567890123456789012123456789012345678901
(C) is (which is two tokens over 7 tokens, right), while choice (D) has 2
12345678901234567890123456789012123456789012345678901
7
2
12345678901234567890123456789012123456789012345678901
2
12345678901234567890123456789012123456789012345678901
2 tokens 2 1
2
12345678901234567890123456789012123456789012345678901
5
5
.
2
12345678901234567890123456789012123456789012345678901
6
tokens
6
3
2
12345678901234567890123456789012123456789012345678901
2
12345678901234567890123456789012123456789012345678901
Both
of
these
answers
use
the
wrong
math,
but
they
are
also
too
easy.
If
2345678901234567890123456789012123456789012345678901
12345678901234567890123456789012123456789012345678901 2
12345678901234567890123456789012123456789012345678901
you know it is a medium difficult problem, how could the answer be so 2
2
12345678901234567890123456789012123456789012345678901
2
12345678901234567890123456789012123456789012345678901
simple
that
you
could
just
do
the
math
in
your
head?
Finding
choice
(C)
12345678901234567890123456789012123456789012345678901 2
12345678901234567890123456789012123456789012345678901
takes only 15 seconds and choice (D) takes only a bit longer than that. 2
2
12345678901234567890123456789012123456789012345678901
2
12345678901234567890123456789012123456789012345678901
When
you
find
the
answer
to
a
tough
problem
that
easily,
your
GRE
sixth
12345678901234567890123456789012123456789012345678901 2
1 sense should start tingling wildly, telling you that you are picking an 2
2
12345678901234567890123456789012123456789012345678901
2
12345678901234567890123456789012123456789012345678901
answer
that
is
so
easy
it
is
probably
incorrect.
12345678901234567890123456789012123456789012345678901 2
12345678901234567890123456789012123456789012345678901 2
12345678901234567890123456789012123456789012345678901
For this particular problem, eliminating incorrect answers is just as 2
2
12345678901234567890123456789012123456789012345678901
12345678901234567890123456789012123456789012345678901
important as finding the correct choice. If you dont know how to do the 2
12345678901234567890123456789012123456789012345678901 2
math, eliminate choices (C) and (D) and take a guess. This gives you a 1 in 2
12345678901234567890123456789012123456789012345678901
2
12345678901234567890123456789012123456789012345678901
3 shot of choosing the right answer, which happens to be choice (A). Even 2
2345678901234567890123456789012123456789012345678901
12345678901234567890123456789012123456789012345678901
2
12345678901234567890123456789012123456789012345678901
1 if you guessed wrong that time, you gave yourself a fighting chance by 2
2
12345678901234567890123456789012123456789012345678901
understanding the GRE CAT well enough to know which answer choices 2
12345678901234567890123456789012123456789012345678901
2345678901234567890123456789012123456789012345678901
2
12345678901234567890123456789012123456789012345678901
were traps.
2
12345678901234567890123456789012123456789012345678901
12345678901234567890123456789012123456789012345678901 2
12345678901234567890123456789012123456789012345678901 2
2
1
2
12345678901234567890123456789012123456789012345678901
2
12345678901234567890123456789012123456789012345678901
2
12345678901234567890123456789012123456789012345678901
The
math
regarding
this
problem
is
covered
on
pages
100101,
the
2
12345678901234567890123456789012123456789012345678901
2
12345678901234567890123456789012123456789012345678901
section
on
probabilities.
2345678901234567890123456789012123456789012345678901
12345678901234567890123456789012123456789012345678901 2
12345678901234567890123456789012123456789012345678901 2
12345678901234567890123456789012123456789012345678901 2
12345678901234567890123456789012123456789012345678901 2
2
1
2
12345678901234567890123456789012123456789012345678901
12345678901234567890123456789012123456789012345678901 2
2
12345678901234567890123456789012123456789012345678901
12345678901234567890123456789012123456789012345678901 2
2
12345678901234567890123456789012123456789012345678901
12345678901234567890123456789012123456789012345678901 2
12345678901234567890123456789012123456789012345678901 2
12345678901234567890123456789012123456789012345678901 2
12345678901234567890123456789012123456789012345678901 2
123456789012345678901234567890121234567890123456789012

Chapter 1: Everything You Need to Know about the GRE CAT

Note

123456789012345678901234567890121234567890123456789012
12345678901234567890123456789012123456789012345678901 2
2
12345678901234567890123456789012123456789012345678901
2
12345678901234567890123456789012123456789012345678901
The
Test
2
12345678901234567890123456789012123456789012345678901
2
12345678901234567890123456789012123456789012345678901
Use
the
following
checklist
to
prepare
for
the
day
you
take
the
actual
GRE:
12345678901234567890123456789012123456789012345678901 2
2
12345678901234567890123456789012123456789012345678901
12345678901234567890123456789012123456789012345678901 2
2
12345678901234567890123456789012123456789012345678901
2
12345678901234567890123456789012123456789012345678901
2
12345678901234567890123456789012123456789012345678901
Items
to
Bring
to
the
GRE:
2
12345678901234567890123456789012123456789012345678901
2345678901234567890123456789012123456789012345678901
2
1
1. Good old picture I.D., preferably something legitimate like a 2
12345678901234567890123456789012123456789012345678901
12345678901234567890123456789012123456789012345678901
drivers license and not a badly laminated Honorary Member of 2
2
12345678901234567890123456789012123456789012345678901
2
12345678901234567890123456789012123456789012345678901
the
Justice
League
card.
2345678901234567890123456789012123456789012345678901
2
1
2
12345678901234567890123456789012123456789012345678901
2
12345678901234567890123456789012123456789012345678901
2.
Fruit
snack
or
something
light,
yet
filling,
like
nuts.
You
dont
2
12345678901234567890123456789012123456789012345678901
2
12345678901234567890123456789012123456789012345678901
want
to
eat
something
so
heavy
you
find
yourself
dying
for
a
nap.
2345678901234567890123456789012123456789012345678901
2
1
2
12345678901234567890123456789012123456789012345678901
12345678901234567890123456789012123456789012345678901
3. A bottle of water. Some would rather drown out the entire 2
2
12345678901234567890123456789012123456789012345678901
12345678901234567890123456789012123456789012345678901
experience, but this practice is frowned upon for a multitude of 2
2345678901234567890123456789012123456789012345678901
12345678901234567890123456789012123456789012345678901 22
12345678901234567890123456789012123456789012345678901
reasons.
2
12345678901234567890123456789012123456789012345678901
2
12345678901234567890123456789012123456789012345678901
4.
Overshirt
of
some
kind.
When
you
check
out
the
site
initially,
12345678901234567890123456789012123456789012345678901 22
12345678901234567890123456789012123456789012345678901
you should get an idea of what the ambient temperature is like. 2
1
12345678901234567890123456789012123456789012345678901
Dress accordingly. The testing room is often rather small, just a 2
2
12345678901234567890123456789012123456789012345678901
2
12345678901234567890123456789012123456789012345678901
cubicle
farm
of
610
spaces.
Depending
on
how
many
people
2345678901234567890123456789012123456789012345678901
12345678901234567890123456789012123456789012345678901 22
12345678901234567890123456789012123456789012345678901
are taking tests next to you, the room can be either cold or 2
12345678901234567890123456789012123456789012345678901
12345678901234567890123456789012123456789012345678901
warm, so the best advice is to start with light clothing with the 2
2
12345678901234567890123456789012123456789012345678901
2
12345678901234567890123456789012123456789012345678901
option of bundling up.
2
12345678901234567890123456789012123456789012345678901
12345678901234567890123456789012123456789012345678901 22
12345678901234567890123456789012123456789012345678901
5. A positive attitude. If thats asking too much, just dont bring a
2
1
2345678901234567890123456789012123456789012345678901
12345678901234567890123456789012123456789012345678901
negative one. Shoot for feeling relaxed and confident, since this 2
2
12345678901234567890123456789012123456789012345678901
1
disposition will help you stay sharp and focused on the task 2
12345678901234567890123456789012123456789012345678901 2
2
12345678901234567890123456789012123456789012345678901
at hand.
2
12345678901234567890123456789012123456789012345678901
2
12345678901234567890123456789012123456789012345678901
12345678901234567890123456789012123456789012345678901 2
2
12345678901234567890123456789012123456789012345678901
2
12345678901234567890123456789012123456789012345678901
Many test centers have a set of lockers just inside the testing room for you
2345678901234567890123456789012123456789012345678901
12345678901234567890123456789012123456789012345678901 22
12345678901234567890123456789012123456789012345678901
2
1 to keep valuables and whatever else you want. This is another thing you
2
12345678901234567890123456789012123456789012345678901
should look for when you are scouting out a GRE location.
12345678901234567890123456789012123456789012345678901 2
2345678901234567890123456789012123456789012345678901
12345678901234567890123456789012123456789012345678901 22
12345678901234567890123456789012123456789012345678901 2
12345678901234567890123456789012123456789012345678901 2
12345678901234567890123456789012123456789012345678901
1 The tests sign-up process is a mixture of registering for a seminar and 2
12345678901234567890123456789012123456789012345678901
getting arrested. You have to write out a mini-essay claiming youll be a 2
2
12345678901234567890123456789012123456789012345678901
12345678901234567890123456789012123456789012345678901
good person and not sell your GRE secrets to a cheesy newspaper 2
2
12345678901234567890123456789012123456789012345678901
12345678901234567890123456789012123456789012345678901
tabloidor anyone else for that matter. Then you sign in again, have 2
2345678901234567890123456789012123456789012345678901
2
12345678901234567890123456789012123456789012345678901
your identification checked, and get a mug shot taken . . . just in case. In 2
12345678901234567890123456789012123456789012345678901
2
12345678901234567890123456789012123456789012345678901
many locations, you are also filmed throughout the testing procedure. 2
12345678901234567890123456789012123456789012345678901
1 These actions are all part of the underlying theme of, Dont try 2
2
12345678901234567890123456789012123456789012345678901
2
12345678901234567890123456789012123456789012345678901
anything funny, bub.
2
12345678901234567890123456789012123456789012345678901
12345678901234567890123456789012123456789012345678901 2
12345678901234567890123456789012123456789012345678901
You have the option of wearing noise-blocking headphones while taking 2
2
12345678901234567890123456789012123456789012345678901
2
12345678901234567890123456789012123456789012345678901
the
GRE,
which
we
dont
recommended
unless
you
studied
for
the
GRE
12345678901234567890123456789012123456789012345678901 22
12345678901234567890123456789012123456789012345678901
123456789012345678901234567890121234567890123456789012 15

http://www.xtremepapers.net

Part I: Understanding the GRE

www.petersons.com

http://www.xtremepapers.net

Alert!

16

123456789012345678901234567890121234567890123456789012
12345678901234567890123456789012123456789012345678901 2
2
12345678901234567890123456789012123456789012345678901
wearing noise-blocking headphones. A little soundthe whir of the 2
12345678901234567890123456789012123456789012345678901
2
12345678901234567890123456789012123456789012345678901
computers fans, the hum of air-conditioningshould be the type of 2
12345678901234567890123456789012123456789012345678901
2
12345678901234567890123456789012123456789012345678901
background noise youre used to, and total silence can be a little freaky. 2
12345678901234567890123456789012123456789012345678901
2345678901234567890123456789012123456789012345678901
1 Unless theres someone trying to cough up a lung next to you in the room, 2
2
12345678901234567890123456789012123456789012345678901
2
12345678901234567890123456789012123456789012345678901
skip
the
phones.
2
12345678901234567890123456789012123456789012345678901
2
12345678901234567890123456789012123456789012345678901
2345678901234567890123456789012123456789012345678901
1 At long last, you get in front of the computer, ready to get your GRE 2
2
12345678901234567890123456789012123456789012345678901
2
12345678901234567890123456789012123456789012345678901
groove
on.
Unfortunately,
the
meaty
portions
of
the
GRE,
i.e.,
the
sections
2
12345678901234567890123456789012123456789012345678901
2
12345678901234567890123456789012123456789012345678901
that
count,
are
still
a
long
way
off.
First
you
have
to
fill
out
a
basic
form
2345678901234567890123456789012123456789012345678901
2
1
2
12345678901234567890123456789012123456789012345678901
for
what
seems
like
the
umpteenth
time,
then
you
get
a
market
research
2
12345678901234567890123456789012123456789012345678901
2
12345678901234567890123456789012123456789012345678901
section
where
the
computer
asks
you
a
lot
of
basic
questions
like,
Why
2
12345678901234567890123456789012123456789012345678901
2345678901234567890123456789012123456789012345678901
1 are you taking this test? and Have you taken other standardized tests 2
2
12345678901234567890123456789012123456789012345678901
12345678901234567890123456789012123456789012345678901
before? Is the computer trying to hit on you, or is it just being nosy? Its 2
2
12345678901234567890123456789012123456789012345678901
12345678901234567890123456789012123456789012345678901
up to you to decide, but the fact is, youre stuck in that chair and you have 2
2345678901234567890123456789012123456789012345678901
12345678901234567890123456789012123456789012345678901 2
2
12345678901234567890123456789012123456789012345678901
to answer all the questions before you can start taking the exam.
2
12345678901234567890123456789012123456789012345678901
2
12345678901234567890123456789012123456789012345678901
After
the
market
research
section,
you
get
another
section
asking
you
12345678901234567890123456789012123456789012345678901 2
2
12345678901234567890123456789012123456789012345678901
1 where you want the scores sent. You should have an idea of where you 2
12345678901234567890123456789012123456789012345678901
want to send your scores. If not, this section can take a while. Even if you 2
2
12345678901234567890123456789012123456789012345678901
2
12345678901234567890123456789012123456789012345678901
do
know
where
you
want
scores
sent,
scrolling
around
takes
awhile,
so
be
2345678901234567890123456789012123456789012345678901
12345678901234567890123456789012123456789012345678901 2
2
12345678901234567890123456789012123456789012345678901
prepared to spend about 510 minutes futzing around in this section.
2
12345678901234567890123456789012123456789012345678901
12345678901234567890123456789012123456789012345678901 2
12345678901234567890123456789012123456789012345678901
Once you finish leaping through these hoops, the GRE begins . . . almost. 2
2
12345678901234567890123456789012123456789012345678901
2
12345678901234567890123456789012123456789012345678901
You
now
get
a
chance
to
go
through
the
tutorial
showing
how
to
navigate
12345678901234567890123456789012123456789012345678901 2
2
12345678901234567890123456789012123456789012345678901
1 the GRE CAT. Hopefully, you will already be very familiar with these 2
2345678901234567890123456789012123456789012345678901
12345678901234567890123456789012123456789012345678901
commands, so you can just skip this section. Go through one of the 2
2
12345678901234567890123456789012123456789012345678901
1 tutorial lessons if you feel like it to reassure yourself, but this should not be 2
12345678901234567890123456789012123456789012345678901 2
12345678901234567890123456789012123456789012345678901
necessary. Ideally, you can skip this entirely and get to whats important, 2
2
12345678901234567890123456789012123456789012345678901
the test itself.
2
12345678901234567890123456789012123456789012345678901
12345678901234567890123456789012123456789012345678901 2
2
12345678901234567890123456789012123456789012345678901
2
12345678901234567890123456789012123456789012345678901
2345678901234567890123456789012123456789012345678901
12345678901234567890123456789012123456789012345678901
The GRE is like an awards show. Most people who watch awards shows 2
2
12345678901234567890123456789012123456789012345678901
1 are primarily interested in who the big winners will be. To find this out, 2
2
12345678901234567890123456789012123456789012345678901
though, they have to endure endless music performances, skits, sentimen- 2
12345678901234567890123456789012123456789012345678901
2345678901234567890123456789012123456789012345678901
2
12345678901234567890123456789012123456789012345678901
tal retrospectives, and all sorts of other fodder before the big payoff. For 2
12345678901234567890123456789012123456789012345678901
2
12345678901234567890123456789012123456789012345678901
the GRE, these distractions come in the form of market research 2
12345678901234567890123456789012123456789012345678901
1 questions, tutorials, and basic questions about who you are, and so on. 2
2
12345678901234567890123456789012123456789012345678901
12345678901234567890123456789012123456789012345678901
All you really want to do is take the sections that count toward your 2
2
12345678901234567890123456789012123456789012345678901
2
12345678901234567890123456789012123456789012345678901
score,
but
to
do
so
you
have
to
endure
the
GRE
equivalent
of
a
dance
12345678901234567890123456789012123456789012345678901 2
2345678901234567890123456789012123456789012345678901
2
12345678901234567890123456789012123456789012345678901
medley.
2
12345678901234567890123456789012123456789012345678901
12345678901234567890123456789012123456789012345678901 2
12345678901234567890123456789012123456789012345678901 2
1 The test begins! It starts with the Analytical Writing portion, and you have 2
2
12345678901234567890123456789012123456789012345678901
2
12345678901234567890123456789012123456789012345678901
75
minutes
to
respond
to
2
essay
questions.
For
many
test-takers,
the
wait
2
12345678901234567890123456789012123456789012345678901
2
12345678901234567890123456789012123456789012345678901
is
over
and
its
time
to
get
cracking.
However,
some
people
apply
to
2
12345678901234567890123456789012123456789012345678901
2345678901234567890123456789012123456789012345678901
1 programs that do not care about the Analytical score. If this is the case 2
12345678901234567890123456789012123456789012345678901 2
12345678901234567890123456789012123456789012345678901 2
12345678901234567890123456789012123456789012345678901 2
123456789012345678901234567890121234567890123456789012

Chapter 1: Everything You Need to Know about the GRE CAT

123456789012345678901234567890121234567890123456789012
12345678901234567890123456789012123456789012345678901 2
2
12345678901234567890123456789012123456789012345678901
for youand be sure this is the case before following the next piece of 2
12345678901234567890123456789012123456789012345678901
2
12345678901234567890123456789012123456789012345678901
advicethen you can be more relaxed in your approach to the essays. You 2
12345678901234567890123456789012123456789012345678901
2
12345678901234567890123456789012123456789012345678901
have to answer them, since a zero might raise eyebrows, but if the schools 2
12345678901234567890123456789012123456789012345678901
2345678901234567890123456789012123456789012345678901
1 arent going to give the Analytical score any weight, why should you? Save 2
2
12345678901234567890123456789012123456789012345678901
2
12345678901234567890123456789012123456789012345678901
your
brainpower
for
the
sections
of
the
test
that
do
matter.
2
12345678901234567890123456789012123456789012345678901
2
12345678901234567890123456789012123456789012345678901
2345678901234567890123456789012123456789012345678901
1 Once youve finished clacking out your essays, the computer adaptive part 2
2
12345678901234567890123456789012123456789012345678901
2
12345678901234567890123456789012123456789012345678901
of
the
GRE
kicks
in.
You
will
see
a
Quantitative
section,
a
Verbal
section,
2
12345678901234567890123456789012123456789012345678901
2
12345678901234567890123456789012123456789012345678901
and
an
Experimental
section.
ETS,
the
company
that
administers
the
GRE,
2345678901234567890123456789012123456789012345678901
2
1
2
12345678901234567890123456789012123456789012345678901
uses
your
answers
on
the
Experimental
section
to
fashion
future
GRE
2
12345678901234567890123456789012123456789012345678901
2
12345678901234567890123456789012123456789012345678901
CATs.
In
this
way,
ETS
seeks
to
guarantee
that
future
GRE
CATs
are
2
12345678901234567890123456789012123456789012345678901
2345678901234567890123456789012123456789012345678901
1 similar to current GRE CATs in terms of question similarity. This is all well 2
2
12345678901234567890123456789012123456789012345678901
12345678901234567890123456789012123456789012345678901
and good in terms of fairness, but what it boils down to is that you have to 2
2
12345678901234567890123456789012123456789012345678901
12345678901234567890123456789012123456789012345678901
take an entire test section that doesnt count toward your score. For 2
2345678901234567890123456789012123456789012345678901
12345678901234567890123456789012123456789012345678901 22
12345678901234567890123456789012123456789012345678901
obvious reasons, you wont know which section is the Experimental
2
12345678901234567890123456789012123456789012345678901
section ahead of time.
2
12345678901234567890123456789012123456789012345678901
12345678901234567890123456789012123456789012345678901 22
12345678901234567890123456789012123456789012345678901
1 However, you can sometimes get an idea which section is the 2
12345678901234567890123456789012123456789012345678901
experimental one. It all depends on how adept you get at understanding 2
2
12345678901234567890123456789012123456789012345678901
2
12345678901234567890123456789012123456789012345678901
the
difference
between
easy,
medium,
and
hard
problems.
Lets
say
2345678901234567890123456789012123456789012345678901
12345678901234567890123456789012123456789012345678901 22
12345678901234567890123456789012123456789012345678901
youve taken one Quantitative section and one Verbal Section. The third 2
12345678901234567890123456789012123456789012345678901
12345678901234567890123456789012123456789012345678901
section is another Quantitative section, so you know that either this 2
2
12345678901234567890123456789012123456789012345678901
12345678901234567890123456789012123456789012345678901
section or the previous Quant one is the experimental. You start off the 2
2
12345678901234567890123456789012123456789012345678901
2
12345678901234567890123456789012123456789012345678901
third
section
with
four
questions
that
you
know
you
got
correct.
Youre
12345678901234567890123456789012123456789012345678901 22
1 certain you nailed them. Yet the questions never seem to change in
2
12345678901234567890123456789012123456789012345678901
2
12345678901234567890123456789012123456789012345678901
difficulty
level,
not
like
they
did
on
the
previous
Quant
section.
There
12345678901234567890123456789012123456789012345678901 2
2
12345678901234567890123456789012123456789012345678901
are two explanations for this:
2
12345678901234567890123456789012123456789012345678901
2345678901234567890123456789012123456789012345678901
2
12345678901234567890123456789012123456789012345678901
1. You botched those questions and just dont realize it.
2
12345678901234567890123456789012123456789012345678901
2
1
2
12345678901234567890123456789012123456789012345678901
2.
You
are
working
on
the
experimental
section.
2
12345678901234567890123456789012123456789012345678901
2345678901234567890123456789012123456789012345678901
12345678901234567890123456789012123456789012345678901 22
12345678901234567890123456789012123456789012345678901
1 There is no way to be 100 percent certain which section is Experimental, 2
12345678901234567890123456789012123456789012345678901
so the smart move is to treat all sections as if they count. If you guess 2
2
12345678901234567890123456789012123456789012345678901
2345678901234567890123456789012123456789012345678901
12345678901234567890123456789012123456789012345678901
wrong about a section, your score goes down the drain, and you would 2
2
12345678901234567890123456789012123456789012345678901
2
12345678901234567890123456789012123456789012345678901
have
achieved
nothing.
Still,
it
can
be
reassuring
to
have
a
feeling
about
12345678901234567890123456789012123456789012345678901 22
1 which section is Experimental. If you get this feeling, then you should
2
12345678901234567890123456789012123456789012345678901
2
12345678901234567890123456789012123456789012345678901
realize
that
you
are
pretty
GRE
savvy,
and
that
the
intricacies
of
the
test
2
12345678901234567890123456789012123456789012345678901
2
12345678901234567890123456789012123456789012345678901
are not so mysterious after all.
12345678901234567890123456789012123456789012345678901 2
2345678901234567890123456789012123456789012345678901
12345678901234567890123456789012123456789012345678901 22
12345678901234567890123456789012123456789012345678901 2
1
2
. .112345678901234567890123456789012123456789012345678901
. And Then Some
2345678901234567890123456789012123456789012345678901
2
2345678901234567890123456789012123456789012345678901
12345678901234567890123456789012123456789012345678901 22
1 You should have an idea from this chapter that the GRE CAT is a kind of
2
12345678901234567890123456789012123456789012345678901
12345678901234567890123456789012123456789012345678901
odd test. It is unlike most of the major tests in another way, and that is 2
2
12345678901234567890123456789012123456789012345678901
overall disposition of the proctors. On most big tests (standardized exams 2
12345678901234567890123456789012123456789012345678901
2
12345678901234567890123456789012123456789012345678901
or final exams in the college class), the proctors giving tests have a serious, 2
12345678901234567890123456789012123456789012345678901
12345678901234567890123456789012123456789012345678901 2 17
123456789012345678901234567890121234567890123456789012

http://www.xtremepapers.net

Part I: Understanding the GRE

18

123456789012345678901234567890121234567890123456789012
12345678901234567890123456789012123456789012345678901 2
2
12345678901234567890123456789012123456789012345678901
formal air about them. Students are hushed, people avoid eye contact, 2
12345678901234567890123456789012123456789012345678901
2
12345678901234567890123456789012123456789012345678901
everyone is a little nervous. Its sort of an Im a 9-year-old sitting with my 2
12345678901234567890123456789012123456789012345678901
2
12345678901234567890123456789012123456789012345678901
entire adult family at the Thanksgiving dinner table kind of vibe, 2
12345678901234567890123456789012123456789012345678901
2345678901234567890123456789012123456789012345678901
1 nervousness and seriousness and formality all rolled into one. The proctors 2
2
12345678901234567890123456789012123456789012345678901
2
12345678901234567890123456789012123456789012345678901
are
the
same
way,
acting
officious
and
ever-diligent
in
order
to
make
sure
2
12345678901234567890123456789012123456789012345678901
2
12345678901234567890123456789012123456789012345678901
that
nothing
goes
wrong.
2345678901234567890123456789012123456789012345678901
2
1
2
12345678901234567890123456789012123456789012345678901
2
12345678901234567890123456789012123456789012345678901
The
GRE
CAT
is,
as
you
might
expect,
different.
Here
the
test
is
given
all
2
12345678901234567890123456789012123456789012345678901
2
12345678901234567890123456789012123456789012345678901
the
time,
every
day
and
sometimes
even
at
night
during
most
of
the
week.
2345678901234567890123456789012123456789012345678901
2
1
2
12345678901234567890123456789012123456789012345678901
Theres
nothing
exceptional
about
a
single
GRE
administration.
It
means
a
2
12345678901234567890123456789012123456789012345678901
2
12345678901234567890123456789012123456789012345678901
lot
to
you,
but
the
people
at
the
test
centers
see
people
like
you
every
day
of
2
12345678901234567890123456789012123456789012345678901
2345678901234567890123456789012123456789012345678901
2
1 the week. For them its just a days work.
2
12345678901234567890123456789012123456789012345678901
2
12345678901234567890123456789012123456789012345678901
This does not mean to imply that the proctors are lazy, uncaring, or 2
12345678901234567890123456789012123456789012345678901
2
12345678901234567890123456789012123456789012345678901
anything bad. Nothing of the sort is true. Its just that they arent going to 2
12345678901234567890123456789012123456789012345678901
2
12345678901234567890123456789012123456789012345678901
have that same Thanksgiving dinner table attitude, since they are just 2
12345678901234567890123456789012123456789012345678901
2
12345678901234567890123456789012123456789012345678901
doing a routine day of work. They will do it well and be courteous and 2
12345678901234567890123456789012123456789012345678901
2345678901234567890123456789012123456789012345678901
2
12345678901234567890123456789012123456789012345678901
1 helpful, but they wont have the sense of Wow, this is an important 2
2
12345678901234567890123456789012123456789012345678901
moment that you will.
2
12345678901234567890123456789012123456789012345678901
2
12345678901234567890123456789012123456789012345678901
2345678901234567890123456789012123456789012345678901
12345678901234567890123456789012123456789012345678901
They also wont be at the front of the classroom eagerly scanning the room 2
2
12345678901234567890123456789012123456789012345678901
2
12345678901234567890123456789012123456789012345678901
to
see
if
anyones
cheating
or
if
anyone
needs
help.
The
former
is
not
12345678901234567890123456789012123456789012345678901 2
12345678901234567890123456789012123456789012345678901
needed because you cant look at another persons GRE since it wont have 2
2
12345678901234567890123456789012123456789012345678901
2
12345678901234567890123456789012123456789012345678901
the
same
questions.
If
you
tried
to
bring
something
in
to
cheat
with
it
12345678901234567890123456789012123456789012345678901 2
2
12345678901234567890123456789012123456789012345678901
1 would be spotted on camera. If you need help you can raise your hand, but 2
2345678901234567890123456789012123456789012345678901
12345678901234567890123456789012123456789012345678901
theres no guarantee that it will be spotted immediately. Theres no person 2
2
12345678901234567890123456789012123456789012345678901
1 watching the testing room with eagle eyes since that mindset is just not 2
12345678901234567890123456789012123456789012345678901 2
12345678901234567890123456789012123456789012345678901
prevalent. The overall point is that you should not expect the proctor to 2
2
12345678901234567890123456789012123456789012345678901
come rapidly if you need something during the middle of the section. 2
12345678901234567890123456789012123456789012345678901
2
12345678901234567890123456789012123456789012345678901
Therefore, make sure you have all the scratch paper you need during a 2
12345678901234567890123456789012123456789012345678901
2
12345678901234567890123456789012123456789012345678901
break and that you have enough pencils as well. Theres no way to 2
12345678901234567890123456789012123456789012345678901
2
12345678901234567890123456789012123456789012345678901
guarantee someone will come speedily to your aid while the test is in 2
12345678901234567890123456789012123456789012345678901
2
12345678901234567890123456789012123456789012345678901
progress.
2
12345678901234567890123456789012123456789012345678901
2345678901234567890123456789012123456789012345678901
12345678901234567890123456789012123456789012345678901 2
12345678901234567890123456789012123456789012345678901
There! You now have a host of strategies to help you comprehend the 2
2
12345678901234567890123456789012123456789012345678901
2
12345678901234567890123456789012123456789012345678901
computer
adaptive
aspect
of
the
GRE.
For
some
additional
test-taking
2
1
2345678901234567890123456789012123456789012345678901
12345678901234567890123456789012123456789012345678901
strategies on how to handle each section of the GREQuantitative, 2
2
12345678901234567890123456789012123456789012345678901
2
12345678901234567890123456789012123456789012345678901
Verbal,
and
Analyticalall
you
have
to
do
is
turn
the
page.
12345678901234567890123456789012123456789012345678901 2
2
1
2
12345678901234567890123456789012123456789012345678901
2
12345678901234567890123456789012123456789012345678901
2
12345678901234567890123456789012123456789012345678901
2
12345678901234567890123456789012123456789012345678901
12345678901234567890123456789012123456789012345678901 2
2345678901234567890123456789012123456789012345678901
12345678901234567890123456789012123456789012345678901 2
2
1
2
12345678901234567890123456789012123456789012345678901
12345678901234567890123456789012123456789012345678901 2
2
12345678901234567890123456789012123456789012345678901
12345678901234567890123456789012123456789012345678901 2
12345678901234567890123456789012123456789012345678901 2
12345678901234567890123456789012123456789012345678901 2
12345678901234567890123456789012123456789012345678901 2
123456789012345678901234567890121234567890123456789012

www.petersons.com

http://www.xtremepapers.net

Chapter

2
Section Strategies
123456789012345678901234567890121234567890123456789012
2
12345678901234567890123456789012123456789012345678901
2
12345678901234567890123456789012123456789012345678901
In
the
last
chapter,
we
familiarized
you
with
the
computerized
aspect
of
the
2
12345678901234567890123456789012123456789012345678901
2345678901234567890123456789012123456789012345678901
2
12345678901234567890123456789012123456789012345678901
GRE. In this chapter, well acquaint you with the three GRE sections. Our 2
12345678901234567890123456789012123456789012345678901
12345678901234567890123456789012123456789012345678901
goal is to eliminate any element of surprise you might have while taking 2
2
12345678901234567890123456789012123456789012345678901
2
12345678901234567890123456789012123456789012345678901
the
test,
since
unexpected
problem-types
lead
to
confusion,
uncertainty,
12345678901234567890123456789012123456789012345678901 22
1 and an overall negativity that can hinder your exam performance. Well
2
12345678901234567890123456789012123456789012345678901
2
12345678901234567890123456789012123456789012345678901
tell
you
about
the
makeup
of
the
GRE
and
how
it
is
administered,
and
2
12345678901234567890123456789012123456789012345678901
2345678901234567890123456789012123456789012345678901
12345678901234567890123456789012123456789012345678901
your anxiety will diminish as the test becomes less of a mystery. While you 2
2
12345678901234567890123456789012123456789012345678901
2
12345678901234567890123456789012123456789012345678901
might
still
run
across
some
individual
problems
that
throw
you
for
a
loop,
12345678901234567890123456789012123456789012345678901 22
12345678901234567890123456789012123456789012345678901
well make sure you wont run into any question types that leave you 2
12345678901234567890123456789012123456789012345678901
2
12345678901234567890123456789012123456789012345678901
befuddled.
2
12345678901234567890123456789012123456789012345678901
12345678901234567890123456789012123456789012345678901 22
1
2
12345678901234567890123456789012123456789012345678901
2345678901234567890123456789012123456789012345678901
2
1
Quantitative
12345678901234567890123456789012123456789012345678901 2
12345678901234567890123456789012123456789012345678901 2
12345678901234567890123456789012123456789012345678901
The GRE section devoted to math goes by the mouthy word Quantita- 2
2
12345678901234567890123456789012123456789012345678901
12345678901234567890123456789012123456789012345678901
tive. Half of the problems in this section are regular multiple-choice 2
12345678901234567890123456789012123456789012345678901 2
12345678901234567890123456789012123456789012345678901
problems, but the other half are presented in a format known as 2
2
12345678901234567890123456789012123456789012345678901
2345678901234567890123456789012123456789012345678901
2
Quantitative Comparison (QC for short).
12345678901234567890123456789012123456789012345678901
12345678901234567890123456789012123456789012345678901 22
1
12345678901234567890123456789012123456789012345678901 2
12345678901234567890123456789012123456789012345678901 2
2
12345678901234567890123456789012123456789012345678901
QC Problems
2
12345678901234567890123456789012123456789012345678901
2
12345678901234567890123456789012123456789012345678901
The
directions
for
Quantitative
Comparisons
look
something
like
this:
2
12345678901234567890123456789012123456789012345678901
12345678901234567890123456789012123456789012345678901 2
2345678901234567890123456789012123456789012345678901
12345678901234567890123456789012123456789012345678901 22
12345678901234567890123456789012123456789012345678901
Directions: Column A and Column B will have one quantity each.
2
12345678901234567890123456789012123456789012345678901
2
12345678901234567890123456789012123456789012345678901
Your
goal
is
to
compare
the
quantities
in
each
column
and
choose
2
1
2345678901234567890123456789012123456789012345678901
12345678901234567890123456789012123456789012345678901 22
12345678901234567890123456789012123456789012345678901
A if the quantity in Column A is greater
2
12345678901234567890123456789012123456789012345678901
2
12345678901234567890123456789012123456789012345678901
B
if
the
quantity
in
Column
B
is
greater
2
1
2345678901234567890123456789012123456789012345678901
2
12345678901234567890123456789012123456789012345678901
C if the two quantities are equal
2
1
2
12345678901234567890123456789012123456789012345678901
D
if
the
relationship
between
the
two
quantities
cannot
be
de12345678901234567890123456789012123456789012345678901 2
2
12345678901234567890123456789012123456789012345678901
termined from the information given
2
12345678901234567890123456789012123456789012345678901
12345678901234567890123456789012123456789012345678901 2
12345678901234567890123456789012123456789012345678901 2
12345678901234567890123456789012123456789012345678901 2
123456789012345678901234567890121234567890123456789012
19

http://www.xtremepapers.net

Part I: Understanding the GRE

www.petersons.com

http://www.xtremepapers.net

Note

20

123456789012345678901234567890121234567890123456789012
12345678901234567890123456789012123456789012345678901 2
2
12345678901234567890123456789012123456789012345678901
Heres a wildly simple example of a QC problem.
2
12345678901234567890123456789012123456789012345678901
2
12345678901234567890123456789012123456789012345678901
2
12345678901234567890123456789012123456789012345678901
Column A
Column B
2
12345678901234567890123456789012123456789012345678901
2
12345678901234567890123456789012123456789012345678901
2345678901234567890123456789012123456789012345678901
2
1
h52
2
12345678901234567890123456789012123456789012345678901
2
12345678901234567890123456789012123456789012345678901
2
12345678901234567890123456789012123456789012345678901
h
1
2
12345678901234567890123456789012123456789012345678901
2345678901234567890123456789012123456789012345678901
2
1
2
12345678901234567890123456789012123456789012345678901
12345678901234567890123456789012123456789012345678901
You have two columns, A and B, and there is a quantity underneath 2
2
12345678901234567890123456789012123456789012345678901
each one. This quantity might be a variable (like Column A), a number 2
12345678901234567890123456789012123456789012345678901
2345678901234567890123456789012123456789012345678901
1 (like Column B), or some mixture of the two with a little extra thrown in 2
2
12345678901234567890123456789012123456789012345678901
2
12345678901234567890123456789012123456789012345678901
for good measure. On occasion, floating above this columnar stew is some 2
12345678901234567890123456789012123456789012345678901
2
12345678901234567890123456789012123456789012345678901
additional information that applies to the quantity in both columns, 2
12345678901234567890123456789012123456789012345678901
2
12345678901234567890123456789012123456789012345678901
like h 5 2.
2
12345678901234567890123456789012123456789012345678901
2
12345678901234567890123456789012123456789012345678901
2
12345678901234567890123456789012123456789012345678901
Like
the
example
above,
the
items
in
the
two
columns
often
cant
be
2345678901234567890123456789012123456789012345678901
12345678901234567890123456789012123456789012345678901 2
12345678901234567890123456789012123456789012345678901
compared to one another, because they are in different formats. In our 2
2
12345678901234567890123456789012123456789012345678901
2
12345678901234567890123456789012123456789012345678901
example,
one
column
has
a
variable
while
the
other
has
a
number.
You
12345678901234567890123456789012123456789012345678901 2
2
12345678901234567890123456789012123456789012345678901
1 would need to convert the variable into a number in order to compare 2
12345678901234567890123456789012123456789012345678901
them. Since the centered information states h 5 2, you know that the 2
2
12345678901234567890123456789012123456789012345678901
12345678901234567890123456789012123456789012345678901
quantity, or value, of the letter h in Column A is 2. Under Column B, the 2
2345678901234567890123456789012123456789012345678901
12345678901234567890123456789012123456789012345678901 2
12345678901234567890123456789012123456789012345678901
value is 1. There are no other possible interpretations or solutions using 2
2
12345678901234567890123456789012123456789012345678901
12345678901234567890123456789012123456789012345678901
the centered information. We know that 1 is less than 2, so the correct 2
12345678901234567890123456789012123456789012345678901 2
2
12345678901234567890123456789012123456789012345678901
answer must be (A).
2
12345678901234567890123456789012123456789012345678901
12345678901234567890123456789012123456789012345678901 2
12345678901234567890123456789012123456789012345678901 2
2
1
12345678901234567890123456789012123456789012345678901
Converting the information in both columns into easily comparable 2
2
12345678901234567890123456789012123456789012345678901
12345678901234567890123456789012123456789012345678901
forms is a good strategy for many QC problems, especially if you find that 2
12345678901234567890123456789012123456789012345678901 2
2
12345678901234567890123456789012123456789012345678901
you are stuck.
2
12345678901234567890123456789012123456789012345678901
2
12345678901234567890123456789012123456789012345678901
12345678901234567890123456789012123456789012345678901 2
2
12345678901234567890123456789012123456789012345678901
12345678901234567890123456789012123456789012345678901
Note the qualifying phrase There are no other possibile interpretations or 2
2345678901234567890123456789012123456789012345678901
12345678901234567890123456789012123456789012345678901 2
2
12345678901234567890123456789012123456789012345678901
1 solutions in the previous sentence. Exploring alternate possible answers 2
12345678901234567890123456789012123456789012345678901
is critical on QC problems, because there are often hidden outcomes that 2
2
12345678901234567890123456789012123456789012345678901
2345678901234567890123456789012123456789012345678901
12345678901234567890123456789012123456789012345678901
can affect the answer on a problem. Heres a QC problem that looks 2
2
12345678901234567890123456789012123456789012345678901
similar to the previous one.
2
12345678901234567890123456789012123456789012345678901
12345678901234567890123456789012123456789012345678901 2
2
1
Column B
Column A
2
12345678901234567890123456789012123456789012345678901
2
12345678901234567890123456789012123456789012345678901
2
2
12345678901234567890123456789012123456789012345678901
h
5
4
2
12345678901234567890123456789012123456789012345678901
12345678901234567890123456789012123456789012345678901 2
2345678901234567890123456789012123456789012345678901
2
12345678901234567890123456789012123456789012345678901
h
1
2
12345678901234567890123456789012123456789012345678901
12345678901234567890123456789012123456789012345678901 2
12345678901234567890123456789012123456789012345678901 2
1 Again, you want to convert Column A into a number so you can compare 2
2
12345678901234567890123456789012123456789012345678901
it to the number in Column B. The obvious answer is that h 5 2, since 2
12345678901234567890123456789012123456789012345678901
2
12345678901234567890123456789012123456789012345678901
(2)(2) 5 4. This would mean (A) is the answer . . . if there are no other 2
12345678901234567890123456789012123456789012345678901
2
12345678901234567890123456789012123456789012345678901
possibilities, that is. In this case, you should see that h could also equal 22, 2
12345678901234567890123456789012123456789012345678901
12345678901234567890123456789012123456789012345678901 2
12345678901234567890123456789012123456789012345678901 2
12345678901234567890123456789012123456789012345678901 2
123456789012345678901234567890121234567890123456789012

Chapter 2: Section Strategies

123456789012345678901234567890121234567890123456789012
12345678901234567890123456789012123456789012345678901 2
2
12345678901234567890123456789012123456789012345678901
since (22)( 22) 5 4. If you compare 22 with Column Bs 1, Column Bs 2
12345678901234567890123456789012123456789012345678901
2
12345678901234567890123456789012123456789012345678901
quantity is larger, so (B) is the answer in this scenario.
2
12345678901234567890123456789012123456789012345678901
12345678901234567890123456789012123456789012345678901 2
12345678901234567890123456789012123456789012345678901
One possible value for h gives you the answer (A), while another gives you 2
2
12345678901234567890123456789012123456789012345678901
2
12345678901234567890123456789012123456789012345678901
an
answer
of
(B).
Whenever
you
have
a
conflict
like
this,
the
correct
2
12345678901234567890123456789012123456789012345678901
2
12345678901234567890123456789012123456789012345678901
response
is
choice
(D),
the
answer
cannot
be
determined
by
the
2
12345678901234567890123456789012123456789012345678901
2345678901234567890123456789012123456789012345678901
1 information given. A more precise statement for choice (D) would be that 2
2
12345678901234567890123456789012123456789012345678901
2
12345678901234567890123456789012123456789012345678901
more
than
one
answer
can
be
determined,
and
theres
no
way
to
tell
2
12345678901234567890123456789012123456789012345678901
2
12345678901234567890123456789012123456789012345678901
which
answer
is
correct.
Its
not
that
you
cant
determine
an
answer,
its
2345678901234567890123456789012123456789012345678901
2
1
2
12345678901234567890123456789012123456789012345678901
just
that
you
dont
know
which
answer
is
correct.
2
12345678901234567890123456789012123456789012345678901
2
12345678901234567890123456789012123456789012345678901
12345678901234567890123456789012123456789012345678901
To properly explore all possibilities on QC problems involving variables, 2
2345678901234567890123456789012123456789012345678901
2
1
12345678901234567890123456789012123456789012345678901
dont just use easy numbers like 2, 3, and 4 when giving values to 2
2
12345678901234567890123456789012123456789012345678901
variables. Be sure to insert some weird numbers like those on the 2
12345678901234567890123456789012123456789012345678901
2
12345678901234567890123456789012123456789012345678901
following list:
2
12345678901234567890123456789012123456789012345678901
2
12345678901234567890123456789012123456789012345678901
2
12345678901234567890123456789012123456789012345678901
2
12345678901234567890123456789012123456789012345678901
2
12345678901234567890123456789012123456789012345678901
Weird
Numbers
2345678901234567890123456789012123456789012345678901
12345678901234567890123456789012123456789012345678901 22
1
2
12345678901234567890123456789012123456789012345678901
The Number
Why Its Weird
2
12345678901234567890123456789012123456789012345678901
2
12345678901234567890123456789012123456789012345678901
0
Any
number
multiplied
by
0
equals
zero.
2345678901234567890123456789012123456789012345678901
12345678901234567890123456789012123456789012345678901 22
12345678901234567890123456789012123456789012345678901
Also, any number to the zero power equals
2
12345678901234567890123456789012123456789012345678901
0
2
12345678901234567890123456789012123456789012345678901
1,
or
x
5
1.
12345678901234567890123456789012123456789012345678901 22
12345678901234567890123456789012123456789012345678901
1
Any number multiplied by 1 does not
2
12345678901234567890123456789012123456789012345678901
2
12345678901234567890123456789012123456789012345678901
change.
2
12345678901234567890123456789012123456789012345678901
2
1
2345678901234567890123456789012123456789012345678901
2
12345678901234567890123456789012123456789012345678901
Fractions between
Normally, when you multiply two numbers
2
12345678901234567890123456789012123456789012345678901
2
1
0
and
1
together,
the
end
result
is
larger.
When
you
12345678901234567890123456789012123456789012345678901 2
2
12345678901234567890123456789012123456789012345678901
multiply two fractions less than one to2
12345678901234567890123456789012123456789012345678901
2
12345678901234567890123456789012123456789012345678901
gether, the result is less than the original
12345678901234567890123456789012123456789012345678901 2
2
12345678901234567890123456789012123456789012345678901
two numbers.
2
12345678901234567890123456789012123456789012345678901
2345678901234567890123456789012123456789012345678901
2
12345678901234567890123456789012123456789012345678901
Negative numbers
Two negatives makes a positive when
2
12345678901234567890123456789012123456789012345678901
2
1
multiplying
or
dividing,
and
22
is
greater
12345678901234567890123456789012123456789012345678901 2
2
12345678901234567890123456789012123456789012345678901
than 24 even though our minds like to
2
12345678901234567890123456789012123456789012345678901
2
12345678901234567890123456789012123456789012345678901
think
that
4
is
bigger
than
2.
2
12345678901234567890123456789012123456789012345678901
2
12345678901234567890123456789012123456789012345678901
12345678901234567890123456789012123456789012345678901 2
2345678901234567890123456789012123456789012345678901
2
12345678901234567890123456789012123456789012345678901
On our second sample question, realizing the existence of 22 (the weird
2
12345678901234567890123456789012123456789012345678901
2
12345678901234567890123456789012123456789012345678901
negative number) led us to the correct answer of (D).
12345678901234567890123456789012123456789012345678901 22
1 Many incorrect QC answers are chosen in the name of speed, as people
2
12345678901234567890123456789012123456789012345678901
2
12345678901234567890123456789012123456789012345678901
look at a problem and see the answer that must be the correct one. If the 2
12345678901234567890123456789012123456789012345678901
12345678901234567890123456789012123456789012345678901
answer is so obvious, why not pick it quickly and zip on to the next 2
2
12345678901234567890123456789012123456789012345678901
2345678901234567890123456789012123456789012345678901
2
12345678901234567890123456789012123456789012345678901
1 question? The response to this hypothetical question is that answers that 2
12345678901234567890123456789012123456789012345678901
look correct are often traps designed to catch those who do not actually 2
2
12345678901234567890123456789012123456789012345678901
2
12345678901234567890123456789012123456789012345678901
work
the
problem.
Heres
a
classic
QC
trap
on
a
medium-difficulty
2345678901234567890123456789012123456789012345678901
2
1
2
12345678901234567890123456789012123456789012345678901
problem.
12345678901234567890123456789012123456789012345678901 22
12345678901234567890123456789012123456789012345678901
123456789012345678901234567890121234567890123456789012 21

http://www.xtremepapers.net

Part I: Understanding the GRE

22

123456789012345678901234567890121234567890123456789012
12345678901234567890123456789012123456789012345678901 2
2
12345678901234567890123456789012123456789012345678901
2
12345678901234567890123456789012123456789012345678901
Column A
Column B
2
12345678901234567890123456789012123456789012345678901
2
12345678901234567890123456789012123456789012345678901
4(g 1 i) 5 76
2
12345678901234567890123456789012123456789012345678901
2
12345678901234567890123456789012123456789012345678901
3i
5
57
2345678901234567890123456789012123456789012345678901
2
1
2
12345678901234567890123456789012123456789012345678901
g
g
2
12345678901234567890123456789012123456789012345678901
8
6
2
12345678901234567890123456789012123456789012345678901
2
12345678901234567890123456789012123456789012345678901
2345678901234567890123456789012123456789012345678901
2
1
12345678901234567890123456789012123456789012345678901
If g is a regular, Joe Schmo kind of number, the answer seems pretty 2
2
12345678901234567890123456789012123456789012345678901
12345678901234567890123456789012123456789012345678901
obvious. Column B is going to be greater since 8 is bigger than 6 and both 2
2
12345678901234567890123456789012123456789012345678901
12345678901234567890123456789012123456789012345678901
numbers are being taken to the same exponential power, g. An experienced 2
2
12345678901234567890123456789012123456789012345678901
12345678901234567890123456789012123456789012345678901
GRE takerwhich you will be after reading this bookshould have 2
2
12345678901234567890123456789012123456789012345678901
internal alarms going off at the sight of this problem. It just looks too 2
12345678901234567890123456789012123456789012345678901
2345678901234567890123456789012123456789012345678901
1 simple, and there has to be something more to this problem in order to 2
2
12345678901234567890123456789012123456789012345678901
2
12345678901234567890123456789012123456789012345678901
justify the fact that its a medium problem.
2
12345678901234567890123456789012123456789012345678901
2
12345678901234567890123456789012123456789012345678901
2345678901234567890123456789012123456789012345678901
12345678901234567890123456789012123456789012345678901
Take the time to solve for both g and i, and youll discover the trap. From 2
2
12345678901234567890123456789012123456789012345678901
2
12345678901234567890123456789012123456789012345678901
the
second
equation,
you
will
discover
that
i
5
19.
Placing
this
value
into
12345678901234567890123456789012123456789012345678901 2
2
12345678901234567890123456789012123456789012345678901
the first equation, you will find:
2
12345678901234567890123456789012123456789012345678901
2
1
2
12345678901234567890123456789012123456789012345678901
4(g
1
i)
5
76
2
12345678901234567890123456789012123456789012345678901
2
12345678901234567890123456789012123456789012345678901
4g
1
4i
5
76
2345678901234567890123456789012123456789012345678901
12345678901234567890123456789012123456789012345678901 2
12345678901234567890123456789012123456789012345678901 2
2
12345678901234567890123456789012123456789012345678901
4g 1 (4)(19) 5 76
2
12345678901234567890123456789012123456789012345678901
2
12345678901234567890123456789012123456789012345678901
4g
1
76
5
76
12345678901234567890123456789012123456789012345678901 2
2
12345678901234567890123456789012123456789012345678901
4g 1 76 2 76 5 76 2 76
2
12345678901234567890123456789012123456789012345678901
12345678901234567890123456789012123456789012345678901 2
2
1
4g 5 0
2
12345678901234567890123456789012123456789012345678901
2
12345678901234567890123456789012123456789012345678901
g
5
0
12345678901234567890123456789012123456789012345678901 2
2
12345678901234567890123456789012123456789012345678901
12345678901234567890123456789012123456789012345678901
If g equals zero, then Column As 6g 5 60 5 1 and Column Bs 8g 5 80 5 2
2345678901234567890123456789012123456789012345678901
12345678901234567890123456789012123456789012345678901 2
2
12345678901234567890123456789012123456789012345678901
1 1. Both columns are equal, and theres no other possible value for g, so the 2
2
12345678901234567890123456789012123456789012345678901
correct answer must be (C).
2
12345678901234567890123456789012123456789012345678901
2345678901234567890123456789012123456789012345678901
2
12345678901234567890123456789012123456789012345678901
You will see a lot of QC problems throughout this book, so use the 2
12345678901234567890123456789012123456789012345678901
1 following checklist as a way of remembering the key strategies to use when 2
12345678901234567890123456789012123456789012345678901 2
2
12345678901234567890123456789012123456789012345678901
handling these problems.
2
12345678901234567890123456789012123456789012345678901
2
12345678901234567890123456789012123456789012345678901
2
12345678901234567890123456789012123456789012345678901
2
12345678901234567890123456789012123456789012345678901
12345678901234567890123456789012123456789012345678901 2
2345678901234567890123456789012123456789012345678901
2
12345678901234567890123456789012123456789012345678901
QC Approach Check List
2
12345678901234567890123456789012123456789012345678901
2
12345678901234567890123456789012123456789012345678901
1.
For
the
answer
choices,
the
word
always
is
critical.
You
can
only
pick
12345678901234567890123456789012123456789012345678901 2
1
(A) if the quantity in Column A is always greater, and you can only 2
2
12345678901234567890123456789012123456789012345678901
12345678901234567890123456789012123456789012345678901
pick (B) if Column Bs quantity is always greater. Choice (C) is the 2
2
12345678901234567890123456789012123456789012345678901
12345678901234567890123456789012123456789012345678901
answer if the two columns are always equal, no matter what weird 2
12345678901234567890123456789012123456789012345678901 2
12345678901234567890123456789012123456789012345678901
numbers are punched in. Whenever you find a QC problem that has 2
2
12345678901234567890123456789012123456789012345678901
more than one possible outcome, (A), (B) and/or (C), you have (D) as 2
12345678901234567890123456789012123456789012345678901
2
12345678901234567890123456789012123456789012345678901
your answer.
2
12345678901234567890123456789012123456789012345678901
2345678901234567890123456789012123456789012345678901
2
1
12345678901234567890123456789012123456789012345678901 2
12345678901234567890123456789012123456789012345678901 2
12345678901234567890123456789012123456789012345678901 2
123456789012345678901234567890121234567890123456789012

www.petersons.com

http://www.xtremepapers.net

Chapter 2: Section Strategies

123456789012345678901234567890121234567890123456789012
12345678901234567890123456789012123456789012345678901 2
2
12345678901234567890123456789012123456789012345678901
2. The goal is to compare the two problems, so you should take 2
12345678901234567890123456789012123456789012345678901
2
12345678901234567890123456789012123456789012345678901
information and try to rearrange it in a way that makes comparison 2
12345678901234567890123456789012123456789012345678901
2
12345678901234567890123456789012123456789012345678901
easier. If you have a problem with a lot of variables, you should 2
12345678901234567890123456789012123456789012345678901
2345678901234567890123456789012123456789012345678901
2
1
realize the goal will be to get comparable variables into both 2
12345678901234567890123456789012123456789012345678901
12345678901234567890123456789012123456789012345678901
columns. If one column only has numbers in it, the key to that 2
2
12345678901234567890123456789012123456789012345678901
2
12345678901234567890123456789012123456789012345678901
problem
is
finding
a
way
to
convert
the
other
column
into
a
number
2345678901234567890123456789012123456789012345678901
2
1
2
12345678901234567890123456789012123456789012345678901
than
can
be
compared.
2
12345678901234567890123456789012123456789012345678901
2
12345678901234567890123456789012123456789012345678901
2
12345678901234567890123456789012123456789012345678901
3.
When
variables
are
involved,
dont
be
content
to
plug
in
one
value
for
2345678901234567890123456789012123456789012345678901
2
1
2
12345678901234567890123456789012123456789012345678901
the
variable
and
then
use
that
as
the
result.
Use
a
weird
number
to
2
12345678901234567890123456789012123456789012345678901
2
12345678901234567890123456789012123456789012345678901
challenge
your
initial
finding.
If
you
find
a
weird
number
that
leads
to
2
12345678901234567890123456789012123456789012345678901
2345678901234567890123456789012123456789012345678901
1
a different result, you know the correct choice on that problem is (D). 2
2
12345678901234567890123456789012123456789012345678901
2
12345678901234567890123456789012123456789012345678901
(See the table on page 21 of this chapter)
2
12345678901234567890123456789012123456789012345678901
2
12345678901234567890123456789012123456789012345678901
4.
Many
QC
problems
are
designed
to
look
like
there
is
one
obvious
2345678901234567890123456789012123456789012345678901
12345678901234567890123456789012123456789012345678901 22
12345678901234567890123456789012123456789012345678901
answer. Never jump to this answer unless you have done all of the 2
12345678901234567890123456789012123456789012345678901
2
12345678901234567890123456789012123456789012345678901
actual math that the problem demands. The obvious answer is often 2
12345678901234567890123456789012123456789012345678901
2
12345678901234567890123456789012123456789012345678901
an incorrect choice set there to trap hurried test-takers.
2
1
2
12345678901234567890123456789012123456789012345678901
2
12345678901234567890123456789012123456789012345678901
2
12345678901234567890123456789012123456789012345678901
2345678901234567890123456789012123456789012345678901
12345678901234567890123456789012123456789012345678901 22
12345678901234567890123456789012123456789012345678901
How Much Does Your Brain Weigh?
2
12345678901234567890123456789012123456789012345678901
2
12345678901234567890123456789012123456789012345678901
There
are
several
ways
you
can
answer
this
question.
You
could
take
your
12345678901234567890123456789012123456789012345678901 22
12345678901234567890123456789012123456789012345678901
brain out and place it on a scale, although this is not recommended. You 2
12345678901234567890123456789012123456789012345678901
12345678901234567890123456789012123456789012345678901
could find out what the average brain weighs by looking it up in a 2
2
12345678901234567890123456789012123456789012345678901
1 reference book or surfing the Net. You could place both your hands 2
2345678901234567890123456789012123456789012345678901
12345678901234567890123456789012123456789012345678901 22
12345678901234567890123456789012123456789012345678901
2
1 around your head, lean over, and use your arms as a scale.
2
12345678901234567890123456789012123456789012345678901
Or you could just guess. You wont be completely accurate, but you should 2
12345678901234567890123456789012123456789012345678901
2345678901234567890123456789012123456789012345678901
2
12345678901234567890123456789012123456789012345678901
be pretty close. Look at the following choices, and then take a guess:
2
12345678901234567890123456789012123456789012345678901
2
1
2
12345678901234567890123456789012123456789012345678901
2
12345678901234567890123456789012123456789012345678901
Which
of
the
following
is
the
most
likely
weight
of
your
brain?
2345678901234567890123456789012123456789012345678901
12345678901234567890123456789012123456789012345678901 22
12345678901234567890123456789012123456789012345678901 2
1
A. 1 pound
2
12345678901234567890123456789012123456789012345678901
2
12345678901234567890123456789012123456789012345678901
B. 3 pounds
2345678901234567890123456789012123456789012345678901
12345678901234567890123456789012123456789012345678901 22
12345678901234567890123456789012123456789012345678901
C. 6 pounds
12345678901234567890123456789012123456789012345678901 22
12345678901234567890123456789012123456789012345678901
D. 10 pounds
2
1
E. 15 pounds
2345678901234567890123456789012123456789012345678901
2
12345678901234567890123456789012123456789012345678901
12345678901234567890123456789012123456789012345678901 22
12345678901234567890123456789012123456789012345678901
The correct answer is B. Estimating an answer can sometimes lead you 2
12345678901234567890123456789012123456789012345678901
1 directly to the right answer, but this does not happen very often. More 2
2
12345678901234567890123456789012123456789012345678901
2
12345678901234567890123456789012123456789012345678901
important
than
determining
the
correct
answer,
guessing
can
often
help
2
12345678901234567890123456789012123456789012345678901
2
12345678901234567890123456789012123456789012345678901
you
eliminate
wrong
answers,
which
increases
your
odds
of
getting
a
12345678901234567890123456789012123456789012345678901 2
2345678901234567890123456789012123456789012345678901
2
12345678901234567890123456789012123456789012345678901
1 problem right. If your brain weighed 10 or 15 pounds, you would need 2
12345678901234567890123456789012123456789012345678901
metal girders in your neck to help support its weight. So, (D) and (E) can 2
2
12345678901234567890123456789012123456789012345678901
2
12345678901234567890123456789012123456789012345678901
be
eliminated,
leaving
you
three
choices.
Choice
(A)
is
a
little
light,
and
2345678901234567890123456789012123456789012345678901
2
1
2
12345678901234567890123456789012123456789012345678901
although
this
answer
provides
a
fertile
ground
for
a
cutting
remark,
its
not
12345678901234567890123456789012123456789012345678901 22
12345678901234567890123456789012123456789012345678901
123456789012345678901234567890121234567890123456789012 23

http://www.xtremepapers.net

Part I: Understanding the GRE

www.petersons.com

http://www.xtremepapers.net

Note

24

123456789012345678901234567890121234567890123456789012
12345678901234567890123456789012123456789012345678901 2
2
12345678901234567890123456789012123456789012345678901
correct. This leaves (B) and (C), and if you thought 6 pounds was a bit 2
12345678901234567890123456789012123456789012345678901
2
12345678901234567890123456789012123456789012345678901
portly for a brain, you were right. The answer is (B).
2
12345678901234567890123456789012123456789012345678901
12345678901234567890123456789012123456789012345678901 2
12345678901234567890123456789012123456789012345678901
Some problems are hard because the math required to answer them is 2
2
12345678901234567890123456789012123456789012345678901
2
12345678901234567890123456789012123456789012345678901
hard.
If
you
can
use
your
eyeballs
and
your
3-pound
brain
to
estimate
2
12345678901234567890123456789012123456789012345678901
2
12345678901234567890123456789012123456789012345678901
what
the
answer
should
be,
you
will
increase
your
chances
of
getting
a
2
12345678901234567890123456789012123456789012345678901
2345678901234567890123456789012123456789012345678901
1 problem correct. It might not lead you directly to the right answer, but it 2
2
12345678901234567890123456789012123456789012345678901
2
12345678901234567890123456789012123456789012345678901
can
steer
you
clear
of
answer
choices
that
are
clearly
wrong.
2
12345678901234567890123456789012123456789012345678901
2
12345678901234567890123456789012123456789012345678901
2345678901234567890123456789012123456789012345678901
1 Geometry problems involving angles, lines, figures, or measurements are 2
2
12345678901234567890123456789012123456789012345678901
12345678901234567890123456789012123456789012345678901
the most susceptible to estimating, although the fact that the images are 2
2
12345678901234567890123456789012123456789012345678901
12345678901234567890123456789012123456789012345678901
displayed on a computer screen makes it a little problematic. Whenever 2
2345678901234567890123456789012123456789012345678901
2
1
12345678901234567890123456789012123456789012345678901
you are given a figure on the screen, redraw it on paper in front of you as 2
2
12345678901234567890123456789012123456789012345678901
2
accurately as possible, and then work with that figure. For example:
12345678901234567890123456789012123456789012345678901
2
12345678901234567890123456789012123456789012345678901
2345678901234567890123456789012123456789012345678901
12345678901234567890123456789012123456789012345678901 2
2
12345678901234567890123456789012123456789012345678901
The shortest side of a triangle is n centimeters long. Another side is
2
12345678901234567890123456789012123456789012345678901
2
12345678901234567890123456789012123456789012345678901
three
times
this
length,
while
the
third
side
is
22
centimeters
greater
12345678901234567890123456789012123456789012345678901 2
12345678901234567890123456789012123456789012345678901
than n. If this is the case, which of the following is the only possible 2
2
1
2
12345678901234567890123456789012123456789012345678901
value
of
n?
2
12345678901234567890123456789012123456789012345678901
2
12345678901234567890123456789012123456789012345678901
2345678901234567890123456789012123456789012345678901
2
12345678901234567890123456789012123456789012345678901
A. 2 centimeters
2
12345678901234567890123456789012123456789012345678901
2
12345678901234567890123456789012123456789012345678901
B. 4 centimeters
12345678901234567890123456789012123456789012345678901 2
2
12345678901234567890123456789012123456789012345678901
C. 5 centimeters
2
12345678901234567890123456789012123456789012345678901
2
12345678901234567890123456789012123456789012345678901
D. 6 centimeters
2
12345678901234567890123456789012123456789012345678901
E. 8 centimeters
2
12345678901234567890123456789012123456789012345678901
2
1
2345678901234567890123456789012123456789012345678901
12345678901234567890123456789012123456789012345678901
Theres an equation you can use to solve this problem, but you can also 2
2
12345678901234567890123456789012123456789012345678901
1 attack it with just a number 2 pencil and some scratch paper. Start with 2
12345678901234567890123456789012123456789012345678901 2
12345678901234567890123456789012123456789012345678901
choice (A), 2 centimeters. The second side would be (2)(3) 5 6 cm and the 2
2
12345678901234567890123456789012123456789012345678901
2
12345678901234567890123456789012123456789012345678901
third
side
is
2
1
22
5
24
cm.
Now,
try
to
accurately
draw
a
triangle
with
12345678901234567890123456789012123456789012345678901 2
12345678901234567890123456789012123456789012345678901
sides of 2, 6, and 24. With one side 24 cm, how are the sides of 2 cm and 2
2
12345678901234567890123456789012123456789012345678901
2345678901234567890123456789012123456789012345678901
2
12345678901234567890123456789012123456789012345678901
6 cm actually going to connect?
2
12345678901234567890123456789012123456789012345678901
2
1
12345678901234567890123456789012123456789012345678901 2
12345678901234567890123456789012123456789012345678901 2
2
12345678901234567890123456789012123456789012345678901
2
12345678901234567890123456789012123456789012345678901
2
12345678901234567890123456789012123456789012345678901
2
12345678901234567890123456789012123456789012345678901
2
12345678901234567890123456789012123456789012345678901
Since
(A)
fails
spectacularly,
you
might
want
to
head
to
the
larger
values
2345678901234567890123456789012123456789012345678901
12345678901234567890123456789012123456789012345678901 2
12345678901234567890123456789012123456789012345678901
like choice (E). If n 5 8, then the other two sides are 24 cm and 30 cm. If 2
2
12345678901234567890123456789012123456789012345678901
12345678901234567890123456789012123456789012345678901
you freehand this, it seems to work. At this point, knowing the math 2
2
1
2
12345678901234567890123456789012123456789012345678901
would come in handy, but if you dont know it, guess (E) and move on.
2
12345678901234567890123456789012123456789012345678901
2
12345678901234567890123456789012123456789012345678901
2
12345678901234567890123456789012123456789012345678901
12345678901234567890123456789012123456789012345678901 2
2345678901234567890123456789012123456789012345678901
2
12345678901234567890123456789012123456789012345678901
1 In a triangle, the length of every side must be less than the sum of the other 2
12345678901234567890123456789012123456789012345678901
two sides. Choice (E) fulfills this, since even the length of the longest side, 2
2
12345678901234567890123456789012123456789012345678901
2
12345678901234567890123456789012123456789012345678901
30, is less than the sum of 8 and 24.
2345678901234567890123456789012123456789012345678901
2
1
12345678901234567890123456789012123456789012345678901 2
12345678901234567890123456789012123456789012345678901 2
12345678901234567890123456789012123456789012345678901 2
123456789012345678901234567890121234567890123456789012

Chapter 2: Section Strategies

123456789012345678901234567890121234567890123456789012
12345678901234567890123456789012123456789012345678901 2
2
12345678901234567890123456789012123456789012345678901
Techniques, strategy, practice, and factual knowledge can be combined to 2
12345678901234567890123456789012123456789012345678901
2
12345678901234567890123456789012123456789012345678901
give you a good score, but it doesnt guarantee a perfect score. Inevitably, 2
12345678901234567890123456789012123456789012345678901
2
12345678901234567890123456789012123456789012345678901
there will be one or two questions that seem to defy explanation, and these 2
12345678901234567890123456789012123456789012345678901
2345678901234567890123456789012123456789012345678901
1 often appear as time is winding down on the section. If you cant take an 2
2
12345678901234567890123456789012123456789012345678901
2
12345678901234567890123456789012123456789012345678901
educated
guess,
just
take
a
plain
guess
and
move
on
to
the
next
problem.
2
12345678901234567890123456789012123456789012345678901
2
12345678901234567890123456789012123456789012345678901
On
the
computer,
you
must
place
an
answer
down
on
every
problem,
even
2345678901234567890123456789012123456789012345678901
2
1
2
12345678901234567890123456789012123456789012345678901
if
it
that
means
youre
simply
filling
in
(E)
on
the
last
couple
of
questions.
2
12345678901234567890123456789012123456789012345678901
2
12345678901234567890123456789012123456789012345678901
Keep
in
mind
that
these
questions
are
not
worth
as
much
as
the
earliest
2
12345678901234567890123456789012123456789012345678901
2345678901234567890123456789012123456789012345678901
1 questions. On a 28-question test, you can randomly fill in the last 6 2
2
12345678901234567890123456789012123456789012345678901
2
12345678901234567890123456789012123456789012345678901
questions
and
still
get
a
score
in
the
high
700s.
The
point
is
that
guessing
2
12345678901234567890123456789012123456789012345678901
2
12345678901234567890123456789012123456789012345678901
late
in
the
test
wont
hurt
you
as
much
as
you
might
think.
If
you
have
to
2345678901234567890123456789012123456789012345678901
2
1
2
12345678901234567890123456789012123456789012345678901
guess
early
in
the
math
section,
try
to
use
estimating
and
common
sense
to
2
12345678901234567890123456789012123456789012345678901
2
12345678901234567890123456789012123456789012345678901
eliminate
some
answers,
and
then
just
take
your
best
shot.
2
12345678901234567890123456789012123456789012345678901
2345678901234567890123456789012123456789012345678901
12345678901234567890123456789012123456789012345678901 22
12345678901234567890123456789012123456789012345678901 2
12345678901234567890123456789012123456789012345678901 2
12345678901234567890123456789012123456789012345678901 2
Verbal
12345678901234567890123456789012123456789012345678901 2
12345678901234567890123456789012123456789012345678901 2
1 The Verbal portion of the GRE will be eerily familiar to anyone who took
2
12345678901234567890123456789012123456789012345678901
12345678901234567890123456789012123456789012345678901
the SAT when applying for college. All four question types have made an 2
2
12345678901234567890123456789012123456789012345678901
12345678901234567890123456789012123456789012345678901
appearance on the SAT at one time or another, and theyve even starred in 2
2
12345678901234567890123456789012123456789012345678901
other standardized tests as well.
2
12345678901234567890123456789012123456789012345678901
2
12345678901234567890123456789012123456789012345678901
2
12345678901234567890123456789012123456789012345678901
2345678901234567890123456789012123456789012345678901
12345678901234567890123456789012123456789012345678901 22
12345678901234567890123456789012123456789012345678901
Question Types on the GRE Verbal
2
12345678901234567890123456789012123456789012345678901
12345678901234567890123456789012123456789012345678901 22
1
Question Type
Description
2
12345678901234567890123456789012123456789012345678901
2
12345678901234567890123456789012123456789012345678901
1.
Antonyms
You
are
given
a
single
word
and
then
must
12345678901234567890123456789012123456789012345678901 2
2
12345678901234567890123456789012123456789012345678901
find its opposite in the answer choices.
2
12345678901234567890123456789012123456789012345678901
2345678901234567890123456789012123456789012345678901
12345678901234567890123456789012123456789012345678901 22
12345678901234567890123456789012123456789012345678901
2. Analogies
Youll see two words separated by a colon
2
1
at the beginning of each problem. These
2
12345678901234567890123456789012123456789012345678901
2
12345678901234567890123456789012123456789012345678901
are
called
the
stem
words,
and
the
stem
2345678901234567890123456789012123456789012345678901
12345678901234567890123456789012123456789012345678901 22
12345678901234567890123456789012123456789012345678901
words have some sort of relationship with
2
1
2
12345678901234567890123456789012123456789012345678901
each
other.
Your
job
is
to
find
the
paired
12345678901234567890123456789012123456789012345678901 2
2345678901234567890123456789012123456789012345678901
2
12345678901234567890123456789012123456789012345678901
words in the answer choices that have the
2
12345678901234567890123456789012123456789012345678901
2
12345678901234567890123456789012123456789012345678901
same,
analogous
relationship
as
the
stem
12345678901234567890123456789012123456789012345678901 22
1
words.
2
12345678901234567890123456789012123456789012345678901
2
12345678901234567890123456789012123456789012345678901
2
12345678901234567890123456789012123456789012345678901
3. Sentence
A good old fill-in-the-blank exercise where
2
12345678901234567890123456789012123456789012345678901
2
12345678901234567890123456789012123456789012345678901
Completions
you must add the correct missing word or
2345678901234567890123456789012123456789012345678901
2
12345678901234567890123456789012123456789012345678901
words to an initial sentence.
2
12345678901234567890123456789012123456789012345678901
12345678901234567890123456789012123456789012345678901 22
12345678901234567890123456789012123456789012345678901
4. Reading
Read some text and then answer questions
2
1
2
12345678901234567890123456789012123456789012345678901
Comprehension
about it. The staple of standardized verbal
2
12345678901234567890123456789012123456789012345678901
2
12345678901234567890123456789012123456789012345678901
(RC)
exams
everywhere.
12345678901234567890123456789012123456789012345678901 2
2
12345678901234567890123456789012123456789012345678901
12345678901234567890123456789012123456789012345678901 2
12345678901234567890123456789012123456789012345678901 2
12345678901234567890123456789012123456789012345678901 2
12345678901234567890123456789012123456789012345678901 2 25
123456789012345678901234567890121234567890123456789012

http://www.xtremepapers.net

Part I: Understanding the GRE

www.petersons.com

http://www.xtremepapers.net

X-Ref

26

123456789012345678901234567890121234567890123456789012
12345678901234567890123456789012123456789012345678901 2
2
12345678901234567890123456789012123456789012345678901
There are 30 problems on the Verbal section and four different question 2
12345678901234567890123456789012123456789012345678901
2
12345678901234567890123456789012123456789012345678901
types, so you might think that there will about 78 of each question type 2
12345678901234567890123456789012123456789012345678901
2
12345678901234567890123456789012123456789012345678901
on the test. It rarely works out that way. Although every CAT is different, 2
12345678901234567890123456789012123456789012345678901
2345678901234567890123456789012123456789012345678901
1 for the most part, Sentence Completion problems get short shrift on the 2
2
12345678901234567890123456789012123456789012345678901
2
12345678901234567890123456789012123456789012345678901
GRE
CAT,
and
this
is
especially
true
if
you
are
answering
a
majority
of
the
2
12345678901234567890123456789012123456789012345678901
2
12345678901234567890123456789012123456789012345678901
questions
correctly.
If
youre
doing
well,
youll
find
yourself
bombarded
by
2345678901234567890123456789012123456789012345678901
2
1
2
12345678901234567890123456789012123456789012345678901
antonyms
and
analogies.
These
two
question
types
are
very
vocabulary2
12345678901234567890123456789012123456789012345678901
2
12345678901234567890123456789012123456789012345678901
centric,
since
knowledge
of
the
words
used
is
a
large
factor
in
getting
the
2
12345678901234567890123456789012123456789012345678901
2345678901234567890123456789012123456789012345678901
1 problems right. The better you do on the GRE CAT Verbal, the more the 2
2
12345678901234567890123456789012123456789012345678901
2
12345678901234567890123456789012123456789012345678901
section
resembles
a
vocabulary
quiz
by
the
end.
2
12345678901234567890123456789012123456789012345678901
2
12345678901234567890123456789012123456789012345678901
2345678901234567890123456789012123456789012345678901
2
1
2
12345678901234567890123456789012123456789012345678901
2
12345678901234567890123456789012123456789012345678901
2
12345678901234567890123456789012123456789012345678901
2
12345678901234567890123456789012123456789012345678901
In-depth
discussion
and
suggestions
on
how
to
tackle
each
of
the
four
2345678901234567890123456789012123456789012345678901
12345678901234567890123456789012123456789012345678901 2
2
12345678901234567890123456789012123456789012345678901
Verbal question types is covered starting on page 25.
2
12345678901234567890123456789012123456789012345678901
12345678901234567890123456789012123456789012345678901 2
12345678901234567890123456789012123456789012345678901 2
12345678901234567890123456789012123456789012345678901 2
2
1
2
12345678901234567890123456789012123456789012345678901
12345678901234567890123456789012123456789012345678901
The Verbal section also starts out like a vocabulary quiz, since the first 36 2
2
12345678901234567890123456789012123456789012345678901
12345678901234567890123456789012123456789012345678901
questions are usually antonyms and some analogies. This means that on 2
2
12345678901234567890123456789012123456789012345678901
the initial part of the Verbal test (when questions count the most), a lot 2
12345678901234567890123456789012123456789012345678901
2
12345678901234567890123456789012123456789012345678901
depends on whether you know the particular word given in a question. 2
12345678901234567890123456789012123456789012345678901
2345678901234567890123456789012123456789012345678901
2
12345678901234567890123456789012123456789012345678901
This makes the beginning of the Verbal section a little arbitrary. People 2
12345678901234567890123456789012123456789012345678901
2
12345678901234567890123456789012123456789012345678901
with large vocabularies might get an unfamiliar word, while someone with 2
12345678901234567890123456789012123456789012345678901
1 a less-developed vocabulary will get a tough word whose meaning they 2
2
12345678901234567890123456789012123456789012345678901
12345678901234567890123456789012123456789012345678901
just happen to know. All in all, the start of the section is a bit of a crap 2
2
12345678901234567890123456789012123456789012345678901
2
12345678901234567890123456789012123456789012345678901
shoot.
Getting
the
right
word
or
words
can
help
your
score,
especially
if
2
12345678901234567890123456789012123456789012345678901
2345678901234567890123456789012123456789012345678901
2
12345678901234567890123456789012123456789012345678901
you dont consider yourself someone with a large vocabulary.
2
12345678901234567890123456789012123456789012345678901
2
1
2
12345678901234567890123456789012123456789012345678901
The
Verbal
section
might
seem
less
fair
than
the
Math,
but
theres
no
real
2
12345678901234567890123456789012123456789012345678901
2345678901234567890123456789012123456789012345678901
12345678901234567890123456789012123456789012345678901
difference. If youre strong in geometry, and your first two math questions 2
2
12345678901234567890123456789012123456789012345678901
1 concern geometry, then youre in luck. If geometry is your worst subject, 2
12345678901234567890123456789012123456789012345678901 2
12345678901234567890123456789012123456789012345678901
then youre out of luck. The adaptive format, with early questions 2
2
12345678901234567890123456789012123456789012345678901
2
12345678901234567890123456789012123456789012345678901
counting for more, is the culprit (or helper) in both cases.
2
12345678901234567890123456789012123456789012345678901
2
12345678901234567890123456789012123456789012345678901
In
the
end,
just
knowing
how
the
Verbal
section
will
begin
gives
you
a
12345678901234567890123456789012123456789012345678901 2
2345678901234567890123456789012123456789012345678901
2
12345678901234567890123456789012123456789012345678901
mental advantage over someone who just shows up and starts answering 2
12345678901234567890123456789012123456789012345678901
2
12345678901234567890123456789012123456789012345678901
questions blindly.
2
12345678901234567890123456789012123456789012345678901
2
1
2345678901234567890123456789012123456789012345678901
12345678901234567890123456789012123456789012345678901
A reading passage will show up sooner or later, with 24 questions 2
2
12345678901234567890123456789012123456789012345678901
2
12345678901234567890123456789012123456789012345678901
attached
to
it.
If
you
do
a
lot
of
Web
site
reading,
then
you
wont
have
12345678901234567890123456789012123456789012345678901 2
1 much problem with the computer format. Theres a scroll bar to help you 2
2
12345678901234567890123456789012123456789012345678901
2
12345678901234567890123456789012123456789012345678901
move
up
and
down
the
text,
but
the
actual
text
box
is
ridiculously
small.
It
2
12345678901234567890123456789012123456789012345678901
2
12345678901234567890123456789012123456789012345678901
only
holds
two
or
three
sentences
at
a
time,
so
no
matter
how
short
the
2
12345678901234567890123456789012123456789012345678901
2345678901234567890123456789012123456789012345678901
1 reading passage is, theres always going to be some scrolling to do. To give 2
12345678901234567890123456789012123456789012345678901 2
12345678901234567890123456789012123456789012345678901 2
12345678901234567890123456789012123456789012345678901 2
123456789012345678901234567890121234567890123456789012

Tip

Chapter 2: Section Strategies

123456789012345678901234567890121234567890123456789012
12345678901234567890123456789012123456789012345678901 2
2
12345678901234567890123456789012123456789012345678901
yourself a preview of what it will be like, open up some text on a computer, 2
12345678901234567890123456789012123456789012345678901
2
12345678901234567890123456789012123456789012345678901
and place it in a box about 3 inches by 4 inches. Give yourself some big 2
12345678901234567890123456789012123456789012345678901
2
12345678901234567890123456789012123456789012345678901
letters, and allow for a nice amount of space between each line.
2
12345678901234567890123456789012123456789012345678901
2345678901234567890123456789012123456789012345678901
2
1
2
12345678901234567890123456789012123456789012345678901
2
12345678901234567890123456789012123456789012345678901
On RC problems, the passage takes up the upper left portion of the
2
12345678901234567890123456789012123456789012345678901
2
12345678901234567890123456789012123456789012345678901
screen,
while
questions
appear
on
the
right
side.
The
bottom
of
the
screen
2345678901234567890123456789012123456789012345678901
2
1
2
12345678901234567890123456789012123456789012345678901
contains a lot of basic instructions like Time on/off and Wanna quit,
2
12345678901234567890123456789012123456789012345678901
2
12345678901234567890123456789012123456789012345678901
fraidy
cat?
stuff
that
you
will
rarely,
if
ever,
use
during
the
exam.
2
12345678901234567890123456789012123456789012345678901
2345678901234567890123456789012123456789012345678901
2
1
2
12345678901234567890123456789012123456789012345678901
2
12345678901234567890123456789012123456789012345678901
If
the
above
instructions
are
too
tech-savvy
for
you,
dont
sweat
it.
The
2
12345678901234567890123456789012123456789012345678901
2
12345678901234567890123456789012123456789012345678901
point
is
that
you
will
have
to
move
up
and
down
to
read
some
text,
so
be
2345678901234567890123456789012123456789012345678901
2
1
2
12345678901234567890123456789012123456789012345678901
prepared
to
do
just
that.
2
12345678901234567890123456789012123456789012345678901
2
12345678901234567890123456789012123456789012345678901
2
12345678901234567890123456789012123456789012345678901
The
last
problem
type,
Sentence
Completion,
translates
very
well
to
the
2345678901234567890123456789012123456789012345678901
12345678901234567890123456789012123456789012345678901 22
12345678901234567890123456789012123456789012345678901
computer. Of the four Verbal question types, these questions are probably 2
12345678901234567890123456789012123456789012345678901
12345678901234567890123456789012123456789012345678901
the easiest to prepare for and improve on, which might be why they show 2
2
12345678901234567890123456789012123456789012345678901
2
12345678901234567890123456789012123456789012345678901
1 up the least. Even so, when they do pop up on your screen, lick your chops 2
2
12345678901234567890123456789012123456789012345678901
and dive right in.
2
12345678901234567890123456789012123456789012345678901
2
12345678901234567890123456789012123456789012345678901
2345678901234567890123456789012123456789012345678901
12345678901234567890123456789012123456789012345678901 22
12345678901234567890123456789012123456789012345678901 2
12345678901234567890123456789012123456789012345678901
. . . And Youre Done
2
12345678901234567890123456789012123456789012345678901
2
12345678901234567890123456789012123456789012345678901
For
most
people,
time
is
not
as
big
an
issue
on
the
Verbal
section
as
it
is
on
12345678901234567890123456789012123456789012345678901 22
12345678901234567890123456789012123456789012345678901
the Math section. Many hard Math problems are complex, multi-step 2
12345678901234567890123456789012123456789012345678901
2
12345678901234567890123456789012123456789012345678901
1 questions that require a lot of computations in order to unravel the right 2
2345678901234567890123456789012123456789012345678901
12345678901234567890123456789012123456789012345678901
answer. This takes time. In contrast, hard questions on the Verbal section 2
2
12345678901234567890123456789012123456789012345678901
1 consist mainly of hard vocabulary words. You still need time to go through 2
12345678901234567890123456789012123456789012345678901 2
12345678901234567890123456789012123456789012345678901
the answer choices and use some strategies, but this doesnt take as long as 2
2
12345678901234567890123456789012123456789012345678901
2
12345678901234567890123456789012123456789012345678901
2
7
12345678901234567890123456789012123456789012345678901 2
2
12345678901234567890123456789012123456789012345678901
9 15
2
12345678901234567890123456789012123456789012345678901
is.
figuring
out
what
the
value
of
2345678901234567890123456789012123456789012345678901
2
12345678901234567890123456789012123456789012345678901
8 14
2
12345678901234567890123456789012123456789012345678901
2
1
25 33
2
12345678901234567890123456789012123456789012345678901
12345678901234567890123456789012123456789012345678901 2
2345678901234567890123456789012123456789012345678901
12345678901234567890123456789012123456789012345678901
Theres no need to rush through any questions on the Verbal section, since 2
2
12345678901234567890123456789012123456789012345678901
2
12345678901234567890123456789012123456789012345678901
lack
of
time
is
rarely
a
problem.
On
average,
you
have
a
minute
per
12345678901234567890123456789012123456789012345678901 22
1 question, but some questions will take less time. For example, if theres an
2
12345678901234567890123456789012123456789012345678901
12345678901234567890123456789012123456789012345678901
antonym question with a word that you know, finding the right answer 2
2
12345678901234567890123456789012123456789012345678901
12345678901234567890123456789012123456789012345678901
can take under 15 seconds. Dont spend less than15 seconds, because you 2
12345678901234567890123456789012123456789012345678901 2
12345678901234567890123456789012123456789012345678901
want to make sure not to make a careless error by speeding. That gives you 2
2
12345678901234567890123456789012123456789012345678901
a lot of extra time to spend on another problem, such as a RC question. 2
12345678901234567890123456789012123456789012345678901
2
12345678901234567890123456789012123456789012345678901
These typically take more time since you want to check your answer with 2
12345678901234567890123456789012123456789012345678901
2345678901234567890123456789012123456789012345678901
2
12345678901234567890123456789012123456789012345678901
2
1 the information in the passage.
2
12345678901234567890123456789012123456789012345678901
2
12345678901234567890123456789012123456789012345678901
If
you
are
running
out
of
time
on
a
practice
Verbal
section,
then
you
are
2
12345678901234567890123456789012123456789012345678901
2345678901234567890123456789012123456789012345678901
1 probably staring at some questions and dredging your brain to come up 2
12345678901234567890123456789012123456789012345678901 2
12345678901234567890123456789012123456789012345678901 2
12345678901234567890123456789012123456789012345678901 2 27
123456789012345678901234567890121234567890123456789012

S DS D
S DS D

http://www.xtremepapers.net

Part I: Understanding the GRE

www.petersons.com

http://www.xtremepapers.net

Alert!

123456789012345678901234567890121234567890123456789012
12345678901234567890123456789012123456789012345678901 2
2
12345678901234567890123456789012123456789012345678901
with a definition of a word. This is not a good use of your time. The 2
12345678901234567890123456789012123456789012345678901
2
12345678901234567890123456789012123456789012345678901
vocabulary on the GRE can be tough, so there will be some words whose 2
12345678901234567890123456789012123456789012345678901
2
12345678901234567890123456789012123456789012345678901
meaning you dont know. Accept this, use any elimination strategies you 2
12345678901234567890123456789012123456789012345678901
2345678901234567890123456789012123456789012345678901
2
1 can on those problems, pick an answer, and move on.
2
12345678901234567890123456789012123456789012345678901
2
12345678901234567890123456789012123456789012345678901
2
12345678901234567890123456789012123456789012345678901
2
12345678901234567890123456789012123456789012345678901
2345678901234567890123456789012123456789012345678901
2
1
2345678901234567890123456789012123456789012345678901
2
1
Analytical
Writing
2
12345678901234567890123456789012123456789012345678901
2
12345678901234567890123456789012123456789012345678901
The
first
essay
prompt
you
see
poses
an
issue,
and
then
gives
you
45
2
12345678901234567890123456789012123456789012345678901
2345678901234567890123456789012123456789012345678901
1 minutes to analyze and discuss that issue. This is your standard, Theres 2
2
12345678901234567890123456789012123456789012345678901
2
12345678901234567890123456789012123456789012345678901
no
right
or
wrong
side,
all
that
matters
is
how
well
you
present
your
side
of
2
12345678901234567890123456789012123456789012345678901
2
12345678901234567890123456789012123456789012345678901
the
case
kind
of
essay.
You
are
often
given
two
topics
to
choose
from,
so
2345678901234567890123456789012123456789012345678901
2
1
2
12345678901234567890123456789012123456789012345678901
pick
the
topic
you
feel
comfortableum,
talking
about.
2
12345678901234567890123456789012123456789012345678901
2
12345678901234567890123456789012123456789012345678901
12345678901234567890123456789012123456789012345678901
For the second essay, analysis of an argument, you only have 30 minutes to 2
2345678901234567890123456789012123456789012345678901
12345678901234567890123456789012123456789012345678901 2
12345678901234567890123456789012123456789012345678901
write. This time, you are first given some text, in which someone makes an 2
2
12345678901234567890123456789012123456789012345678901
12345678901234567890123456789012123456789012345678901
argument about something. This argument is lame in some way or another. 2
12345678901234567890123456789012123456789012345678901 2
2
12345678901234567890123456789012123456789012345678901
1 Your goal here is to evaluate the argument, i.e., explain ways in which the 2
2
arguments lame and ways in which it works.
12345678901234567890123456789012123456789012345678901
2
12345678901234567890123456789012123456789012345678901
2
12345678901234567890123456789012123456789012345678901
Combined,
the
two
essays
take
over
an
hour
to
start
at
the
beginning
of
2345678901234567890123456789012123456789012345678901
12345678901234567890123456789012123456789012345678901 2
12345678901234567890123456789012123456789012345678901
every GRE CAT. Thats a lot of time spent typing, and the key word here is 2
2
12345678901234567890123456789012123456789012345678901
2
12345678901234567890123456789012123456789012345678901
typing.
How
fast
do
you
type?
Answering
this
question
will
determine
12345678901234567890123456789012123456789012345678901 2
12345678901234567890123456789012123456789012345678901
your approach to these essays. Many English aficionados will talk 2
2
12345678901234567890123456789012123456789012345678901
2
12345678901234567890123456789012123456789012345678901
gushingly
about
the
standard
five-paragraph
essay
approach
that
virtually
12345678901234567890123456789012123456789012345678901 2
1 guarantees you a top score of 6 on the Analytical portion. Yet, this 2
2
12345678901234567890123456789012123456789012345678901
2
12345678901234567890123456789012123456789012345678901
approach
doesnt
work
if
youre
such
a
slow
typist
that
you
cant
type
five
12345678901234567890123456789012123456789012345678901 2
2
12345678901234567890123456789012123456789012345678901
paragraphs in the time allotted.
2
12345678901234567890123456789012123456789012345678901
2345678901234567890123456789012123456789012345678901
12345678901234567890123456789012123456789012345678901 2
12345678901234567890123456789012123456789012345678901 2
2
1
2
12345678901234567890123456789012123456789012345678901
As
stated
in
the
previous
chapter,
if
youre
applying
for
a
graduate
2
12345678901234567890123456789012123456789012345678901
2345678901234567890123456789012123456789012345678901
12345678901234567890123456789012123456789012345678901
program in astrophysics, your Analytical score might not be of highest 2
2
12345678901234567890123456789012123456789012345678901
1 importance. Do your best, but spend your time studying the section that 2
12345678901234567890123456789012123456789012345678901 2
2
12345678901234567890123456789012123456789012345678901
matters most, which is Math.
2
12345678901234567890123456789012123456789012345678901
2
12345678901234567890123456789012123456789012345678901
2
12345678901234567890123456789012123456789012345678901
2
12345678901234567890123456789012123456789012345678901
Before anything else, find out how fast you can type. Sit down in front of a 2
12345678901234567890123456789012123456789012345678901
2345678901234567890123456789012123456789012345678901
2
12345678901234567890123456789012123456789012345678901
computer with some word processing program. Start timing, and then type 2
12345678901234567890123456789012123456789012345678901
2
12345678901234567890123456789012123456789012345678901
something like, The quick brown fox jumps over the lazy dog over and 2
12345678901234567890123456789012123456789012345678901
1 over again until one minute is up. If you make a mistake while typing, stop 2
2
12345678901234567890123456789012123456789012345678901
12345678901234567890123456789012123456789012345678901
and correct your error while the clock is running. Now count up the 2
2
12345678901234567890123456789012123456789012345678901
2
12345678901234567890123456789012123456789012345678901
number
of
words
you
typed
correctly
in
one
minute.
Take
this
number
and
12345678901234567890123456789012123456789012345678901 2
2345678901234567890123456789012123456789012345678901
2
12345678901234567890123456789012123456789012345678901
2
1 use it to answer the following question:
2
12345678901234567890123456789012123456789012345678901
12345678901234567890123456789012123456789012345678901 2
2
12345678901234567890123456789012123456789012345678901
12345678901234567890123456789012123456789012345678901 2
12345678901234567890123456789012123456789012345678901 2
12345678901234567890123456789012123456789012345678901 2
12345678901234567890123456789012123456789012345678901 2
28
123456789012345678901234567890121234567890123456789012

Chapter 2: Section Strategies

123456789012345678901234567890121234567890123456789012
12345678901234567890123456789012123456789012345678901 2
2
12345678901234567890123456789012123456789012345678901
2
12345678901234567890123456789012123456789012345678901
Question How long should my essays be?
2
12345678901234567890123456789012123456789012345678901
2
12345678901234567890123456789012123456789012345678901
2
12345678901234567890123456789012123456789012345678901
Step
1
Take
the
amount
of
time
given
for
an
essay.
For
essay
1,
2
12345678901234567890123456789012123456789012345678901
2345678901234567890123456789012123456789012345678901
2
1
this means 45 minutes.
2
12345678901234567890123456789012123456789012345678901
2
12345678901234567890123456789012123456789012345678901
Step 2
Subtract 5 minutes. This time will be used to decide
2
12345678901234567890123456789012123456789012345678901
2
12345678901234567890123456789012123456789012345678901
what
you
want
to
write
in
your
essay.
You
shouldnt
2345678901234567890123456789012123456789012345678901
2
1
2
12345678901234567890123456789012123456789012345678901
just blaze off at the start; it helps to plan out your essay
2
12345678901234567890123456789012123456789012345678901
2
12345678901234567890123456789012123456789012345678901
a
bit.
2
12345678901234567890123456789012123456789012345678901
2345678901234567890123456789012123456789012345678901
2
1
2
12345678901234567890123456789012123456789012345678901
Step
3
40
minutes
remain.
Take
the
number
of
words
you
type
2
12345678901234567890123456789012123456789012345678901
2
12345678901234567890123456789012123456789012345678901
per minute and multiply by 40. If you can type 20
2
12345678901234567890123456789012123456789012345678901
2345678901234567890123456789012123456789012345678901
2
1
words per minute, then (20)(40) 5 800.
2
12345678901234567890123456789012123456789012345678901
2
12345678901234567890123456789012123456789012345678901
Step
4
This
is
the
number
of
words
you
could
type
if
you
knew
2
12345678901234567890123456789012123456789012345678901
2
12345678901234567890123456789012123456789012345678901
exactly
what
you
were
going
to
say
and
typed
nonstop.
2345678901234567890123456789012123456789012345678901
12345678901234567890123456789012123456789012345678901 22
12345678901234567890123456789012123456789012345678901
Since there will be many points in your essay where you
2
12345678901234567890123456789012123456789012345678901
2
12345678901234567890123456789012123456789012345678901
pause
for
a
moment
to
come
up
with
the
right
phrase
or
12345678901234567890123456789012123456789012345678901 22
12345678901234567890123456789012123456789012345678901
sentence, 800 is not really achievable. Halve this
2
1
2
12345678901234567890123456789012123456789012345678901
number
for
a
more
realistic
assessment,
giving
you
400
2
12345678901234567890123456789012123456789012345678901
2
12345678901234567890123456789012123456789012345678901
words.
2345678901234567890123456789012123456789012345678901
12345678901234567890123456789012123456789012345678901 22
12345678901234567890123456789012123456789012345678901
Step 5
If you assume a standard paragraph is around 100
2
12345678901234567890123456789012123456789012345678901
2
12345678901234567890123456789012123456789012345678901
words,
you
can
see
that
your
essay
will
not
make
the
12345678901234567890123456789012123456789012345678901 22
12345678901234567890123456789012123456789012345678901
five-paragraph format due to technical reasons (i.e.
2
12345678901234567890123456789012123456789012345678901
2
12345678901234567890123456789012123456789012345678901
your
slow
typing.)
You
only
have
time
for
four
12345678901234567890123456789012123456789012345678901 22
1
paragraphs. You should therefore state your case clearly
2
12345678901234567890123456789012123456789012345678901
2
12345678901234567890123456789012123456789012345678901
in
Paragraph
1,
back
it
up
with
claims
in
the
next
two
12345678901234567890123456789012123456789012345678901 2
2
12345678901234567890123456789012123456789012345678901
paragraphs, and then wrap things up in Paragraph 4 as
2
12345678901234567890123456789012123456789012345678901
2345678901234567890123456789012123456789012345678901
2
12345678901234567890123456789012123456789012345678901
time expires on you and your nondexterous fingers.
2
12345678901234567890123456789012123456789012345678901
2
1
2
12345678901234567890123456789012123456789012345678901
12345678901234567890123456789012123456789012345678901
Unless you practice typing or type a lot, you might be appalled at how 2
2345678901234567890123456789012123456789012345678901
2
12345678901234567890123456789012123456789012345678901
slow you are. Even with fast fingers, theres no way to compose a flawless, 2
12345678901234567890123456789012123456789012345678901
1 five-paragraph essay on a previously unknown topic in 45 minutes. 2
12345678901234567890123456789012123456789012345678901 2
2
12345678901234567890123456789012123456789012345678901
Flawless essays require time to map out, draft, and then rewrite. Maybe a 2
12345678901234567890123456789012123456789012345678901
2
12345678901234567890123456789012123456789012345678901
professional essay writerif there are any still around these dayscould 2
12345678901234567890123456789012123456789012345678901
12345678901234567890123456789012123456789012345678901
do it, but folks taking the GRE cant, and it isnt expected of them. What is 2
2
12345678901234567890123456789012123456789012345678901
2345678901234567890123456789012123456789012345678901
12345678901234567890123456789012123456789012345678901
expected is that the thoughts you do punch into the computer are good, 2
2
12345678901234567890123456789012123456789012345678901
2
12345678901234567890123456789012123456789012345678901
and
that
the
overall
structure
of
your
essay
is
sound.
12345678901234567890123456789012123456789012345678901 22
1
12345678901234567890123456789012123456789012345678901
On both essays, dont sacrifice quality for quantity. Two body (middle) 2
2
12345678901234567890123456789012123456789012345678901
2
12345678901234567890123456789012123456789012345678901
paragraphs
with
13
compelling
sentences
that
back
up
your
initial
claim
2
12345678901234567890123456789012123456789012345678901
2
12345678901234567890123456789012123456789012345678901
are
all
that
is
needed
to
get
a
decent
score
on
the
Analytical
section.
Some
2345678901234567890123456789012123456789012345678901
12345678901234567890123456789012123456789012345678901 22
1 people have even peppered their essays with jokesrelevant to the
2
12345678901234567890123456789012123456789012345678901
12345678901234567890123456789012123456789012345678901
discussion, of courseand received a 6, so you dont have to kill yourself 2
2
12345678901234567890123456789012123456789012345678901
12345678901234567890123456789012123456789012345678901
to place words on the screen. Just make sure that the words you do put 2
2
12345678901234567890123456789012123456789012345678901
2
12345678901234567890123456789012123456789012345678901
down help support your position in the essay.
12345678901234567890123456789012123456789012345678901 2 29
123456789012345678901234567890121234567890123456789012

http://www.xtremepapers.net

Part I: Understanding the GRE

30

123456789012345678901234567890121234567890123456789012
12345678901234567890123456789012123456789012345678901 2
2
12345678901234567890123456789012123456789012345678901
If you want to increase your typing skills, get a dictionary, a computer, and 2
12345678901234567890123456789012123456789012345678901
2
12345678901234567890123456789012123456789012345678901
get ready to kill two birds with one stone. Purchase a book that lists 2
12345678901234567890123456789012123456789012345678901
2
12345678901234567890123456789012123456789012345678901
college-level vocabulary words, and then spend time typing in each word 2
12345678901234567890123456789012123456789012345678901
2345678901234567890123456789012123456789012345678901
1 and its definition into a word processing document. By doing this, you will 2
2
12345678901234567890123456789012123456789012345678901
2
12345678901234567890123456789012123456789012345678901
improve
your
typing
skills
and
create
a
vocabulary
study
guide
at
the
2
12345678901234567890123456789012123456789012345678901
2
12345678901234567890123456789012123456789012345678901
same
time.
2345678901234567890123456789012123456789012345678901
2
1
2
12345678901234567890123456789012123456789012345678901
2
12345678901234567890123456789012123456789012345678901
2
12345678901234567890123456789012123456789012345678901
2
12345678901234567890123456789012123456789012345678901
2345678901234567890123456789012123456789012345678901
2
1
2
12345678901234567890123456789012123456789012345678901
2
12345678901234567890123456789012123456789012345678901
2
12345678901234567890123456789012123456789012345678901
2
12345678901234567890123456789012123456789012345678901
2345678901234567890123456789012123456789012345678901
2
1
2
12345678901234567890123456789012123456789012345678901
2
12345678901234567890123456789012123456789012345678901
2
12345678901234567890123456789012123456789012345678901
2
12345678901234567890123456789012123456789012345678901
2345678901234567890123456789012123456789012345678901
12345678901234567890123456789012123456789012345678901 2
12345678901234567890123456789012123456789012345678901 2
12345678901234567890123456789012123456789012345678901 2
12345678901234567890123456789012123456789012345678901 2
12345678901234567890123456789012123456789012345678901 2
12345678901234567890123456789012123456789012345678901 2
2
1
2
12345678901234567890123456789012123456789012345678901
2
12345678901234567890123456789012123456789012345678901
2
12345678901234567890123456789012123456789012345678901
2345678901234567890123456789012123456789012345678901
12345678901234567890123456789012123456789012345678901 2
12345678901234567890123456789012123456789012345678901 2
12345678901234567890123456789012123456789012345678901 2
12345678901234567890123456789012123456789012345678901 2
12345678901234567890123456789012123456789012345678901 2
12345678901234567890123456789012123456789012345678901 2
12345678901234567890123456789012123456789012345678901 2
12345678901234567890123456789012123456789012345678901 2
12345678901234567890123456789012123456789012345678901 2
2
1
2
12345678901234567890123456789012123456789012345678901
2
12345678901234567890123456789012123456789012345678901
12345678901234567890123456789012123456789012345678901 2
12345678901234567890123456789012123456789012345678901 2
2
12345678901234567890123456789012123456789012345678901
2
12345678901234567890123456789012123456789012345678901
2
12345678901234567890123456789012123456789012345678901
12345678901234567890123456789012123456789012345678901 2
2
12345678901234567890123456789012123456789012345678901
2
12345678901234567890123456789012123456789012345678901
2345678901234567890123456789012123456789012345678901
12345678901234567890123456789012123456789012345678901 2
12345678901234567890123456789012123456789012345678901 2
2
1
12345678901234567890123456789012123456789012345678901 2
12345678901234567890123456789012123456789012345678901 2
2
12345678901234567890123456789012123456789012345678901
2
12345678901234567890123456789012123456789012345678901
2
12345678901234567890123456789012123456789012345678901
2
12345678901234567890123456789012123456789012345678901
12345678901234567890123456789012123456789012345678901 2
2345678901234567890123456789012123456789012345678901
12345678901234567890123456789012123456789012345678901 2
12345678901234567890123456789012123456789012345678901 2
12345678901234567890123456789012123456789012345678901 2
12345678901234567890123456789012123456789012345678901 2
2
1
2
12345678901234567890123456789012123456789012345678901
2
12345678901234567890123456789012123456789012345678901
2
12345678901234567890123456789012123456789012345678901
2
12345678901234567890123456789012123456789012345678901
12345678901234567890123456789012123456789012345678901 2
2345678901234567890123456789012123456789012345678901
12345678901234567890123456789012123456789012345678901 2
2
1
2
12345678901234567890123456789012123456789012345678901
12345678901234567890123456789012123456789012345678901 2
2
12345678901234567890123456789012123456789012345678901
12345678901234567890123456789012123456789012345678901 2
12345678901234567890123456789012123456789012345678901 2
12345678901234567890123456789012123456789012345678901 2
12345678901234567890123456789012123456789012345678901 2
123456789012345678901234567890121234567890123456789012

www.petersons.com

http://www.xtremepapers.net

PART

II
Diagnosing Strengths
and Weaknesses
Diagnostic:
Dipping Your Big Toe in the GRE Waters

32

PART II
http://www.xtremepapers.net

Chapter

3
Diagnostic: Dipping Your Big Toe
in the GRE Waters

Alert!

1234567890123456789012345678901212345678901234567890123456789012123456
3456789012345678901234567890121234567890123456789012345678901212345 6
12
3456789012345678901234567890121234567890123456789012345678901212345
6
12
Now that you have some section-specific techniques, its time to put them to use. Trying a 6
3456789012345678901234567890121234567890123456789012345678901212345
12
2
3456789012345678901234567890121234567890123456789012345678901212345
6
123456789012345678901234567890121234567890123456789012345678901212345
sample set of math and verbal questions has the following benefits:
6
123456789012345678901234567890121234567890123456789012345678901212345
123456789012345678901234567890121234567890123456789012345678901212345 66
123456789012345678901234567890121234567890123456789012345678901212345
1. It gives you an idea of how you might do if you were to take the real GRE this very 6
123456789012345678901234567890121234567890123456789012345678901212345
6
123456789012345678901234567890121234567890123456789012345678901212345
moment.
6
1
3456789012345678901234567890121234567890123456789012345678901212345 6
12
3456789012345678901234567890121234567890123456789012345678901212345
12
2. It lets you see what areas of study youre strongest in, and what areas need 6
3456789012345678901234567890121234567890123456789012345678901212345
6
12
2
3456789012345678901234567890121234567890123456789012345678901212345
123456789012345678901234567890121234567890123456789012345678901212345
improvement. This knowledge can be used to fine-tune your study plans for the GRE. 6
6
123456789012345678901234567890121234567890123456789012345678901212345
6
123456789012345678901234567890121234567890123456789012345678901212345
3. No matter how you do on the diagnostic, it doesnt count.
6
123456789012345678901234567890121234567890123456789012345678901212345
123456789012345678901234567890121234567890123456789012345678901212345 66
123456789012345678901234567890121234567890123456789012345678901212345
The third point is perhaps the most important. If you answer every sample question correctly, 6
123456789012345678901234567890121234567890123456789012345678901212345
123456789012345678901234567890121234567890123456789012345678901212345
then bellow out a proud Huzzah! You cant send that score to universities, but you can feel 6
6
123456789012345678901234567890121234567890123456789012345678901212345
6
1
good
about
the
fact
that
youve
demonstrated
the
ability
to
do
well
on
GRE-type
questions.
On
2
3456789012345678901234567890121234567890123456789012345678901212345
123456789012345678901234567890121234567890123456789012345678901212345 66
123456789012345678901234567890121234567890123456789012345678901212345
the other hand, if you miss a lot of diagnostic questions, dont sweat it.
6
1
123456789012345678901234567890121234567890123456789012345678901212345 6
3456789012345678901234567890121234567890123456789012345678901212345 6
12
3456789012345678901234567890121234567890123456789012345678901212345 6
12
3456789012345678901234567890121234567890123456789012345678901212345
6
12
If you waited until the last moment and are taking this diagnostic the night before
6
123456789012345678901234567890121234567890123456789012345678901212345
3456789012345678901234567890121234567890123456789012345678901212345
6
12
the
real
GRE
CAT,
you
can
sweat
a
little.
Hopefully,
you
are
taking
this
3456789012345678901234567890121234567890123456789012345678901212345 6
12
2
3456789012345678901234567890121234567890123456789012345678901212345
6
123456789012345678901234567890121234567890123456789012345678901212345
diagnostic at least a month in advance of the real test, giving you ample time to go
6
123456789012345678901234567890121234567890123456789012345678901212345
6
1
through
the
rest
of
the
book
at
a
measured
pace.
123456789012345678901234567890121234567890123456789012345678901212345 6
123456789012345678901234567890121234567890123456789012345678901212345 6
3456789012345678901234567890121234567890123456789012345678901212345 6
12
3456789012345678901234567890121234567890123456789012345678901212345 6
12
3456789012345678901234567890121234567890123456789012345678901212345
12
The whole point of the diagnostic is to determine what subjects youre good at and what 6
3456789012345678901234567890121234567890123456789012345678901212345
6
12
6
123456789012345678901234567890121234567890123456789012345678901212345
subjects
you
need
help
in.
You
can
then
use
this
information
to
devise
a
study
plan
tailored
to
2
3456789012345678901234567890121234567890123456789012345678901212345
123456789012345678901234567890121234567890123456789012345678901212345 66
123456789012345678901234567890121234567890123456789012345678901212345
your strengths and weaknesses.
6
123456789012345678901234567890121234567890123456789012345678901212345
123456789012345678901234567890121234567890123456789012345678901212345 66
1
3456789012345678901234567890121234567890123456789012345678901212345 6
12
3456789012345678901234567890121234567890123456789012345678901212345 6
12
3456789012345678901234567890121234567890123456789012345678901212345
6
12
Should You Take It to the Next Level?
3456789012345678901234567890121234567890123456789012345678901212345
6
12
6
123456789012345678901234567890121234567890123456789012345678901212345
As
stated
earlier,
the
adaptive
nature
of
the
GRE
makes
it
likely
that
someone
shooting
for
a
2
3456789012345678901234567890121234567890123456789012345678901212345
123456789012345678901234567890121234567890123456789012345678901212345 66
1
good score will only see medium and hard questions. Since you probably didnt purchase this
3456789012345678901234567890121234567890123456789012345678901212345
6
12
123456789012345678901234567890121234567890123456789012345678901212345
book to get a lousy score, it makes sense to concentrate on medium and hard GRE questions. 6
3456789012345678901234567890121234567890123456789012345678901212345 6
12
123456789012345678901234567890121234567890123456789012345678901212345
The bulk of this book does just that. The first part of each chapter in the subject review section 6
6
123456789012345678901234567890121234567890123456789012345678901212345
123456789012345678901234567890121234567890123456789012345678901212345
focuses on how medium questions will appear on the math and verbal sections, and it provides 6
123456789012345678901234567890121234567890123456789012345678901212345 6
1234567890123456789012345678901212345678901234567890123456789012123456
32

http://www.xtremepapers.net

DIAGNOSTIC
1234567890123456789012345678901212345678901234567890123456789012123456
123456789012345678901234567890121234567890123456789012345678901212345 6
3456789012345678901234567890121234567890123456789012345678901212345 6
12
you with the facts and strategies needed to tackle these problems. The general instruction for 6
123456789012345678901234567890121234567890123456789012345678901212345
3456789012345678901234567890121234567890123456789012345678901212345
6
12
the Analytical section goes over the basic structure of the essay, giving you the knowledge 6
3456789012345678901234567890121234567890123456789012345678901212345
12
6
123456789012345678901234567890121234567890123456789012345678901212345
needed to gain a respectable score on the written portion. For most students, starting with the 6
3456789012345678901234567890121234567890123456789012345678901212345
12
2
3456789012345678901234567890121234567890123456789012345678901212345
6
1
medium-level material is the wisest course of action.
3456789012345678901234567890121234567890123456789012345678901212345
6
12
3456789012345678901234567890121234567890123456789012345678901212345 6
12
3456789012345678901234567890121234567890123456789012345678901212345
12
However, suppose you feel very confident about your math abilities. A high score on the math 6
3456789012345678901234567890121234567890123456789012345678901212345
6
12
2
3456789012345678901234567890121234567890123456789012345678901212345
1
portion of the diagnostic proves that this confidence is well-founded. If thats the case, you may 6
3456789012345678901234567890121234567890123456789012345678901212345
6
12
3456789012345678901234567890121234567890123456789012345678901212345
6
12
choose
to
skip
over
the
general
study
material
on
math
and
go
straight
to
the
Take
It
to
the
Next
3456789012345678901234567890121234567890123456789012345678901212345 6
12
3456789012345678901234567890121234567890123456789012345678901212345
12
Level material. This material focuses on more advanced topicsinvolved probabilities and 6
6
123456789012345678901234567890121234567890123456789012345678901212345
3456789012345678901234567890121234567890123456789012345678901212345
6
12
number
sets,
for
exampleand
it
shows
how
these
topics
appear
on
hard
problems.
The
Next
3456789012345678901234567890121234567890123456789012345678901212345 6
12
3456789012345678901234567890121234567890123456789012345678901212345
12
Level Verbal and Analytical sections cover advanced topics and problems in a similar fashion. 6
3456789012345678901234567890121234567890123456789012345678901212345
6
12
2
3456789012345678901234567890121234567890123456789012345678901212345
6
1
3456789012345678901234567890121234567890123456789012345678901212345
12
So whats the best study plan? Not to get too mystical or anything, but the answer to this 6
3456789012345678901234567890121234567890123456789012345678901212345 6
12
3456789012345678901234567890121234567890123456789012345678901212345
question can only be found inside you. By working through the entire book, you give yourself a 6
12
3456789012345678901234567890121234567890123456789012345678901212345
6
12
look at everything this book has to offer. Yet this approach takes time, and time is a scarce 6
2
3456789012345678901234567890121234567890123456789012345678901212345
123456789012345678901234567890121234567890123456789012345678901212345
6
123456789012345678901234567890121234567890123456789012345678901212345
commodity in many peoples lives. Therefore, you might wish to Take It to the Next Level in 6
123456789012345678901234567890121234567890123456789012345678901212345
6
123456789012345678901234567890121234567890123456789012345678901212345
subjects you feel comfortable with in order to use the time you have to concentrate on your 6
123456789012345678901234567890121234567890123456789012345678901212345
6
123456789012345678901234567890121234567890123456789012345678901212345
weakest areas.
6
1
3456789012345678901234567890121234567890123456789012345678901212345 6
12
3456789012345678901234567890121234567890123456789012345678901212345
12
The requirements of your graduate program might let you tailor your approach in another way. 6
3456789012345678901234567890121234567890123456789012345678901212345
6
12
2
3456789012345678901234567890121234567890123456789012345678901212345
123456789012345678901234567890121234567890123456789012345678901212345
For example, some fine arts programs require students to take the GRE, but these programs are 6
6
123456789012345678901234567890121234567890123456789012345678901212345
6
123456789012345678901234567890121234567890123456789012345678901212345
not
concerned
with
a
students
Quantitative
(math)
score.
If
this
is
the
case
for
you,
you
could
123456789012345678901234567890121234567890123456789012345678901212345 66
123456789012345678901234567890121234567890123456789012345678901212345
decide to skip the Next Level math, since a high score in math will not help you one way or 6
123456789012345678901234567890121234567890123456789012345678901212345
123456789012345678901234567890121234567890123456789012345678901212345
another. It would still be a good idea to go over the basic study materialyou dont want to 6
6
123456789012345678901234567890121234567890123456789012345678901212345
6
123456789012345678901234567890121234567890123456789012345678901212345
completely blow the math sectionbut a high math score is just not a relevant concern.
6
1
2
3456789012345678901234567890121234567890123456789012345678901212345
123456789012345678901234567890121234567890123456789012345678901212345 66
Before making any decision, take the diagnostic and let the results guide you. There are 28
123456789012345678901234567890121234567890123456789012345678901212345
6
1
Quantitative questions, roughly divided so that the first half contains medium questions while 6
123456789012345678901234567890121234567890123456789012345678901212345
3456789012345678901234567890121234567890123456789012345678901212345
6
12
the second half contains hard questions. If you ace 201 Quantitative problems in allotted time, 6
3456789012345678901234567890121234567890123456789012345678901212345
12
3456789012345678901234567890121234567890123456789012345678901212345
6
12
then you are a good candidate for skipping the general material if you want. If you miss 6
123456789012345678901234567890121234567890123456789012345678901212345
3456789012345678901234567890121234567890123456789012345678901212345
6
12
questions randomly throughout the section, it would be a good idea to look over both the 6
3456789012345678901234567890121234567890123456789012345678901212345
12
2
3456789012345678901234567890121234567890123456789012345678901212345
123456789012345678901234567890121234567890123456789012345678901212345
general and Next Level materials, since theres no clear pattern as to why you got some 6
6
123456789012345678901234567890121234567890123456789012345678901212345
6
1
questions
right
while
missing
others.
It
could
mean
you
were
weak
in
a
specific
subjectlike
123456789012345678901234567890121234567890123456789012345678901212345 6
123456789012345678901234567890121234567890123456789012345678901212345
determining mean/median/mode, for instancein which case a comprehensive review of a wide 6
3456789012345678901234567890121234567890123456789012345678901212345
6
12
3456789012345678901234567890121234567890123456789012345678901212345
6
12
range
of
topics
would
be
beneficial.
If
you
miss
a
bunch
of
medium
questions
but
get
all
the
3456789012345678901234567890121234567890123456789012345678901212345 6
12
3456789012345678901234567890121234567890123456789012345678901212345
12
harder problems right, chances are your problem lies in pacing. You rushed through the 6
6
123456789012345678901234567890121234567890123456789012345678901212345
2
3456789012345678901234567890121234567890123456789012345678901212345
123456789012345678901234567890121234567890123456789012345678901212345
medium questions, making careless errors, in order to spend more time on the harder problems. 6
6
123456789012345678901234567890121234567890123456789012345678901212345
6
123456789012345678901234567890121234567890123456789012345678901212345
If
this
is
the
case,
slow
down,
Sparky!
123456789012345678901234567890121234567890123456789012345678901212345 66
1
3456789012345678901234567890121234567890123456789012345678901212345 6
12
3456789012345678901234567890121234567890123456789012345678901212345 6
12
3456789012345678901234567890121234567890123456789012345678901212345 6
12
3456789012345678901234567890121234567890123456789012345678901212345 6
12
123456789012345678901234567890121234567890123456789012345678901212345 6
2
6
123456789012345678901234567890121234567890123456789012345678901212345
1 3456789012345678901234567890121234567890123456789012345678901212345 6
3456789012345678901234567890121234567890123456789012345678901212345 6
12
123456789012345678901234567890121234567890123456789012345678901212345 6
3456789012345678901234567890121234567890123456789012345678901212345 6
12
123456789012345678901234567890121234567890123456789012345678901212345 6
123456789012345678901234567890121234567890123456789012345678901212345 6
123456789012345678901234567890121234567890123456789012345678901212345 6
123456789012345678901234567890121234567890123456789012345678901212345 6 33
1234567890123456789012345678901212345678901234567890123456789012123456

http://www.xtremepapers.net

Part II: Diagnosing Strengths and Weaknesses

Tip

Note

1234567890123456789012345678901212345678901234567890123456789012123456
123456789012345678901234567890121234567890123456789012345678901212345 6
3456789012345678901234567890121234567890123456789012345678901212345 6
12
123456789012345678901234567890121234567890123456789012345678901212345 6
3456789012345678901234567890121234567890123456789012345678901212345 6
12
3456789012345678901234567890121234567890123456789012345678901212345
6
12
In our diagnostic the Quantitative section starts off with 14 QC questions, and then
6
123456789012345678901234567890121234567890123456789012345678901212345
3456789012345678901234567890121234567890123456789012345678901212345
6
12
follows
with
14
regular
math
problems.
The
14
QC
problems
are
arranged
in
order
2
3456789012345678901234567890121234567890123456789012345678901212345
6
1
3456789012345678901234567890121234567890123456789012345678901212345
6
12
of
difficulty,
and
the
14
regular
questions
are
also
in
their
own
order
of
difficulty.
3456789012345678901234567890121234567890123456789012345678901212345 6
12
3456789012345678901234567890121234567890123456789012345678901212345 6
12
3456789012345678901234567890121234567890123456789012345678901212345 6
12
2
1 3456789012345678901234567890121234567890123456789012345678901212345 6
3456789012345678901234567890121234567890123456789012345678901212345
12
Things are a little different for the Verbal section. There are 30 questions, still divided into half 6
3456789012345678901234567890121234567890123456789012345678901212345
6
12
3456789012345678901234567890121234567890123456789012345678901212345
6
12
medium/half
hard,
but
there
are
four
distinct
question
types:
3456789012345678901234567890121234567890123456789012345678901212345 6
12
2
3456789012345678901234567890121234567890123456789012345678901212345
6
1
3456789012345678901234567890121234567890123456789012345678901212345
6
12
Antonyms
3456789012345678901234567890121234567890123456789012345678901212345 6
12
3456789012345678901234567890121234567890123456789012345678901212345
6
12
Analogies
3456789012345678901234567890121234567890123456789012345678901212345
6
12
2
3456789012345678901234567890121234567890123456789012345678901212345
6
1
Sentence Completions
3456789012345678901234567890121234567890123456789012345678901212345
6
12
3456789012345678901234567890121234567890123456789012345678901212345
6
12

Reading
Comprehension
3456789012345678901234567890121234567890123456789012345678901212345 6
12
3456789012345678901234567890121234567890123456789012345678901212345 6
12
3456789012345678901234567890121234567890123456789012345678901212345
12
Questions within each question type are arranged in order of difficulty, so the first antonym you 6
3456789012345678901234567890121234567890123456789012345678901212345
6
12
3456789012345678901234567890121234567890123456789012345678901212345
12
see is (theoretically) the easiest, while the last is the hardest. After that section, you proceed to 6
3456789012345678901234567890121234567890123456789012345678901212345 6
12
3456789012345678901234567890121234567890123456789012345678901212345
12
analogies, with all analogies arranged in their own order of difficulty. For the Reading 6
2
6
123456789012345678901234567890121234567890123456789012345678901212345
6
1 3456789012345678901234567890121234567890123456789012345678901212345
Comprehension section, the first passage is easier than the second one.
3456789012345678901234567890121234567890123456789012345678901212345 6
12
3456789012345678901234567890121234567890123456789012345678901212345
6
12
You can fine-tune your approach depending on how you do within each question type. Suppose 6
3456789012345678901234567890121234567890123456789012345678901212345
12
2
3456789012345678901234567890121234567890123456789012345678901212345
123456789012345678901234567890121234567890123456789012345678901212345
you answer every analogy correctly but miss half of the Reading Comprehension problems. If 6
6
123456789012345678901234567890121234567890123456789012345678901212345
6
123456789012345678901234567890121234567890123456789012345678901212345
this
occurs,
a
voice
inside
your
head
should
start
hollering,
Focus
on
Reading
Comprehension
123456789012345678901234567890121234567890123456789012345678901212345 66
123456789012345678901234567890121234567890123456789012345678901212345
in both levels, and dont worry about analogies as much.
6
123456789012345678901234567890121234567890123456789012345678901212345
123456789012345678901234567890121234567890123456789012345678901212345 66
123456789012345678901234567890121234567890123456789012345678901212345
This book has been designed to give you a variety of study plan options. You want to take 6
123456789012345678901234567890121234567890123456789012345678901212345
6
1
charge of the GRE, and a good way to start doing this is to take charge of your study plan.
3456789012345678901234567890121234567890123456789012345678901212345
6
12
3456789012345678901234567890121234567890123456789012345678901212345 6
12
123456789012345678901234567890121234567890123456789012345678901212345 6
123456789012345678901234567890121234567890123456789012345678901212345 6
3456789012345678901234567890121234567890123456789012345678901212345
6
12
Optimal Diagnostic-Taking Conditions
3456789012345678901234567890121234567890123456789012345678901212345
6
12
3456789012345678901234567890121234567890123456789012345678901212345
12
As much as you might like to, you should not take the diagnostic while watching television, 6
123456789012345678901234567890121234567890123456789012345678901212345 6
3456789012345678901234567890121234567890123456789012345678901212345
12
answering questions only during commercial breaks. Seal yourself away in some solitary 6
3456789012345678901234567890121234567890123456789012345678901212345
6
12
2
3456789012345678901234567890121234567890123456789012345678901212345
location with a heap of scrap paper and a handful of number 2 pencils as your companions. 6
123456789012345678901234567890121234567890123456789012345678901212345
6
123456789012345678901234567890121234567890123456789012345678901212345
Give yourself 45 minutes for the Quantitative section and 30 minutes for the Verbal section. 6
1
6
123456789012345678901234567890121234567890123456789012345678901212345
Take both sections consecutively with a 1-minute break in between. Dont rush, but try to 6
123456789012345678901234567890121234567890123456789012345678901212345
2
3456789012345678901234567890121234567890123456789012345678901212345
6
123456789012345678901234567890121234567890123456789012345678901212345
answer as many of the questions as you can.
6
123456789012345678901234567890121234567890123456789012345678901212345
123456789012345678901234567890121234567890123456789012345678901212345 66
123456789012345678901234567890121234567890123456789012345678901212345 6
1
3456789012345678901234567890121234567890123456789012345678901212345
6
12
You can write your answers in the book, but make sure to do all work on scratch
3456789012345678901234567890121234567890123456789012345678901212345
6
12
3456789012345678901234567890121234567890123456789012345678901212345
6
12
paper
since
that
is
what
you
have
to
do
for
the
GRE
CAT.
Afterward,
check
to
see
3456789012345678901234567890121234567890123456789012345678901212345 6
12
6
123456789012345678901234567890121234567890123456789012345678901212345
how much scratch paper you used on each section. This will help give you an idea
3456789012345678901234567890121234567890123456789012345678901212345
6
12
3456789012345678901234567890121234567890123456789012345678901212345
6
12
of how much youll need on the real GRE.
3456789012345678901234567890121234567890123456789012345678901212345 6
12
3456789012345678901234567890121234567890123456789012345678901212345 6
12
123456789012345678901234567890121234567890123456789012345678901212345 6
2
123456789012345678901234567890121234567890123456789012345678901212345
Once finished, give yourself a pat of the back. You will have just finished a GRE-like test. It 6
6
1 3456789012345678901234567890121234567890123456789012345678901212345
3456789012345678901234567890121234567890123456789012345678901212345
6
12
wasnt a computer adaptive test, but it was a GRE-like test nevertheless.
123456789012345678901234567890121234567890123456789012345678901212345 6
3456789012345678901234567890121234567890123456789012345678901212345
6
12
Good luck!
6
123456789012345678901234567890121234567890123456789012345678901212345
123456789012345678901234567890121234567890123456789012345678901212345 6
123456789012345678901234567890121234567890123456789012345678901212345 6
123456789012345678901234567890121234567890123456789012345678901212345 6
1234567890123456789012345678901212345678901234567890123456789012123456

34

www.petersons.com

http://www.xtremepapers.net

DIAGNOSTIC
1234567890123456789012345678901212345678901234567890123456789012123456
123456789012345678901234567890121234567890123456789012345678901212345 6
3456789012345678901234567890121234567890123456789012345678901212345 6
12
6
123456789012345678901234567890121234567890123456789012345678901212345
The Test
3456789012345678901234567890121234567890123456789012345678901212345
6
12
3456789012345678901234567890121234567890123456789012345678901212345 6
12
6
123456789012345678901234567890121234567890123456789012345678901212345
Quantitative
3456789012345678901234567890121234567890123456789012345678901212345
6
12
2
3456789012345678901234567890121234567890123456789012345678901212345
6
1
3456789012345678901234567890121234567890123456789012345678901212345
6
12
General Information
3456789012345678901234567890121234567890123456789012345678901212345 6
12
3456789012345678901234567890121234567890123456789012345678901212345
6
12
1. The test has 28 questions. You have 45 minutes to complete the section.
3456789012345678901234567890121234567890123456789012345678901212345
6
12
2
3456789012345678901234567890121234567890123456789012345678901212345
6
1
3456789012345678901234567890121234567890123456789012345678901212345
6
12
2. All numbers used are real numbers.
3456789012345678901234567890121234567890123456789012345678901212345 6
12
3456789012345678901234567890121234567890123456789012345678901212345
6
12
3. All angle measurements can be assumed to be positive unless otherwise noted.
3456789012345678901234567890121234567890123456789012345678901212345
6
12
2
3456789012345678901234567890121234567890123456789012345678901212345
6
1
3456789012345678901234567890121234567890123456789012345678901212345
6
12
4.
All
figures
lie
in
the
same
plane
unless
otherwise
noted.
3456789012345678901234567890121234567890123456789012345678901212345 6
12
3456789012345678901234567890121234567890123456789012345678901212345 6
12
3456789012345678901234567890121234567890123456789012345678901212345
12
5. Drawings that accompany questions are intended to provide information useful in 6
2
6
1 3456789012345678901234567890121234567890123456789012345678901212345
3456789012345678901234567890121234567890123456789012345678901212345
6
12
answering
the
question.
The
figures
are
drawn
closely
to
scale
unless
otherwise
noted.
3456789012345678901234567890121234567890123456789012345678901212345 6
12
3456789012345678901234567890121234567890123456789012345678901212345 6
12
3456789012345678901234567890121234567890123456789012345678901212345 6
12
2
6
123456789012345678901234567890121234567890123456789012345678901212345
Directions: For questions 114, Column A and Column B will have one quantity each.
6
123456789012345678901234567890121234567890123456789012345678901212345
6
123456789012345678901234567890121234567890123456789012345678901212345
Your
goal
is
to
compare
the
quantities
in
each
column
and
choose
6
123456789012345678901234567890121234567890123456789012345678901212345
6
123456789012345678901234567890121234567890123456789012345678901212345
3456789012345678901234567890121234567890123456789012345678901212345
6
123456789012345678901234567890121234567890123456789012345678901212345
A if the quantity in Column A is greater
6
1
3456789012345678901234567890121234567890123456789012345678901212345
6
12
B if the quantity in Column B is greater
3456789012345678901234567890121234567890123456789012345678901212345 6
12
3456789012345678901234567890121234567890123456789012345678901212345
6
12
C if the two quantities are equal
3456789012345678901234567890121234567890123456789012345678901212345
6
12
D if the relationship between the two quantities cannot be determined from the
3456789012345678901234567890121234567890123456789012345678901212345
6
12
3456789012345678901234567890121234567890123456789012345678901212345
6
12
information in the problem
3456789012345678901234567890121234567890123456789012345678901212345
6
12
3456789012345678901234567890121234567890123456789012345678901212345 6
12
2
3456789012345678901234567890121234567890123456789012345678901212345
6
123456789012345678901234567890121234567890123456789012345678901212345
On some questions, information about one or both quantities will be given. This
6
123456789012345678901234567890121234567890123456789012345678901212345
6
123456789012345678901234567890121234567890123456789012345678901212345
information
will
be
centered
above
the
two
columns.
123456789012345678901234567890121234567890123456789012345678901212345 66
1
3456789012345678901234567890121234567890123456789012345678901212345 6
12
3456789012345678901234567890121234567890123456789012345678901212345 6
12
123456789012345678901234567890121234567890123456789012345678901212345 6
6
123456789012345678901234567890121234567890123456789012345678901212345
Column A
Column A
Column B
Column B
3456789012345678901234567890121234567890123456789012345678901212345
6
12
2
3456789012345678901234567890121234567890123456789012345678901212345
123456789012345678901234567890121234567890123456789012345678901212345 66
123456789012345678901234567890121234567890123456789012345678901212345
692
4
6
1 1.
3456789012345678901234567890121234567890123456789012345678901212345
6
12
1,781
15
3456789012345678901234567890121234567890123456789012345678901212345 6
12
2
3456789012345678901234567890121234567890123456789012345678901212345
123456789012345678901234567890121234567890123456789012345678901212345 66
123456789012345678901234567890121234567890123456789012345678901212345
3.
6
1
123456789012345678901234567890121234567890123456789012345678901212345 6
123456789012345678901234567890121234567890123456789012345678901212345 6
3456789012345678901234567890121234567890123456789012345678901212345 6
12
3456789012345678901234567890121234567890123456789012345678901212345 6
12
3456789012345678901234567890121234567890123456789012345678901212345 6
12
3456789012345678901234567890121234567890123456789012345678901212345 6
12
6
123456789012345678901234567890121234567890123456789012345678901212345
a2
(c2b)
2
6
123456789012345678901234567890121234567890123456789012345678901212345
6
123456789012345678901234567890121234567890123456789012345678901212345
6
123456789012345678901234567890121234567890123456789012345678901212345
2.
40
1
9
7
1
63
=
=
6
123456789012345678901234567890121234567890123456789012345678901212345
1 3456789012345678901234567890121234567890123456789012345678901212345 6
2
3456789012345678901234567890121234567890123456789012345678901212345
123456789012345678901234567890121234567890123456789012345678901212345 66
123456789012345678901234567890121234567890123456789012345678901212345 6
123456789012345678901234567890121234567890123456789012345678901212345 6
123456789012345678901234567890121234567890123456789012345678901212345 6
1
3456789012345678901234567890121234567890123456789012345678901212345 6
12
123456789012345678901234567890121234567890123456789012345678901212345 6
3456789012345678901234567890121234567890123456789012345678901212345 6
12
123456789012345678901234567890121234567890123456789012345678901212345 6
3456789012345678901234567890121234567890123456789012345678901212345 6
12
123456789012345678901234567890121234567890123456789012345678901212345 6
123456789012345678901234567890121234567890123456789012345678901212345 6
123456789012345678901234567890121234567890123456789012345678901212345 6
123456789012345678901234567890121234567890123456789012345678901212345 6 35
1234567890123456789012345678901212345678901234567890123456789012123456

http://www.xtremepapers.net

Part II: Diagnosing Strengths and Weaknesses

36

1234567890123456789012345678901212345678901234567890123456789012123456
123456789012345678901234567890121234567890123456789012345678901212345 6
3456789012345678901234567890121234567890123456789012345678901212345 6
12
6
123456789012345678901234567890121234567890123456789012345678901212345
Column A
Column B
Column A
Column B
3456789012345678901234567890121234567890123456789012345678901212345
6
12
3456789012345678901234567890121234567890123456789012345678901212345
6
12
2
1
2
8.
u5x y
x0y
6
123456789012345678901234567890121234567890123456789012345678901212345
50 1 2~2 1 3! #
@
2 2
@4~2 1 3! #
3456789012345678901234567890121234567890123456789012345678901212345
6
12
4.
2
3456789012345678901234567890121234567890123456789012345678901212345
6
1
10
x
3456789012345678901234567890121234567890123456789012345678901212345
6
12
3
2
4 2
xu
3456789012345678901234567890121234567890123456789012345678901212345
6
12
u 5x y
3456789012345678901234567890121234567890123456789012345678901212345
6
12
y
3456789012345678901234567890121234567890123456789012345678901212345 6
12
2
3456789012345678901234567890121234567890123456789012345678901212345
6
1
3456789012345678901234567890121234567890123456789012345678901212345 6
12
3456789012345678901234567890121234567890123456789012345678901212345 6
12
3456789012345678901234567890121234567890123456789012345678901212345 6
12
3456789012345678901234567890121234567890123456789012345678901212345 6
12
2
1 3456789012345678901234567890121234567890123456789012345678901212345 6
3456789012345678901234567890121234567890123456789012345678901212345 6
12
3456789012345678901234567890121234567890123456789012345678901212345 6
12
3456789012345678901234567890121234567890123456789012345678901212345
6
12
5.
A circle has a radius q.
3456789012345678901234567890121234567890123456789012345678901212345
6
12
2
3456789012345678901234567890121234567890123456789012345678901212345
6
1
3456789012345678901234567890121234567890123456789012345678901212345
6
12
Twice the area of
The circumference
9.
x is a positive integer.
3456789012345678901234567890121234567890123456789012345678901212345 6
12
3456789012345678901234567890121234567890123456789012345678901212345
6
12
the circle divided
of the circle
(x2 2 3x 1 2)
x2 2 4
3456789012345678901234567890121234567890123456789012345678901212345
6
12
by
the
radius
2
3456789012345678901234567890121234567890123456789012345678901212345
123456789012345678901234567890121234567890123456789012345678901212345 66
123456789012345678901234567890121234567890123456789012345678901212345 6
123456789012345678901234567890121234567890123456789012345678901212345 6
123456789012345678901234567890121234567890123456789012345678901212345 6
123456789012345678901234567890121234567890123456789012345678901212345 6
123456789012345678901234567890121234567890123456789012345678901212345 6
1
3456789012345678901234567890121234567890123456789012345678901212345 6
12
3456789012345678901234567890121234567890123456789012345678901212345 6
12
3456789012345678901234567890121234567890123456789012345678901212345 6
12
2
6
123456789012345678901234567890121234567890123456789012345678901212345
6
123456789012345678901234567890121234567890123456789012345678901212345
6
123456789012345678901234567890121234567890123456789012345678901212345
6
123456789012345678901234567890121234567890123456789012345678901212345
6.
The area of a
The area of a
6
123456789012345678901234567890121234567890123456789012345678901212345
3456789012345678901234567890121234567890123456789012345678901212345
6
123456789012345678901234567890121234567890123456789012345678901212345
square with a
rectangle with a
6
123456789012345678901234567890121234567890123456789012345678901212345
perimeter of 20
perimeter 20
6
123456789012345678901234567890121234567890123456789012345678901212345
123456789012345678901234567890121234567890123456789012345678901212345 66
1
3456789012345678901234567890121234567890123456789012345678901212345 6
12
3456789012345678901234567890121234567890123456789012345678901212345 6
12
123456789012345678901234567890121234567890123456789012345678901212345 6
123456789012345678901234567890121234567890123456789012345678901212345 6
3456789012345678901234567890121234567890123456789012345678901212345 6
12
3456789012345678901234567890121234567890123456789012345678901212345 6
12
3456789012345678901234567890121234567890123456789012345678901212345 6
12
6
123456789012345678901234567890121234567890123456789012345678901212345
10.
3456789012345678901234567890121234567890123456789012345678901212345
6
12
3456789012345678901234567890121234567890123456789012345678901212345 6
12
2
3456789012345678901234567890121234567890123456789012345678901212345
6
123456789012345678901234567890121234567890123456789012345678901212345
1 1 1 2 3
1 2 1 1 2
6
123456789012345678901234567890121234567890123456789012345678901212345
7.
2
1
1
2
2
1
1
2
1
6
1
3
2
7
5
4
4
5
7
5
3
123456789012345678901234567890121234567890123456789012345678901212345 6
123456789012345678901234567890121234567890123456789012345678901212345 6
3456789012345678901234567890121234567890123456789012345678901212345 6
12
3456789012345678901234567890121234567890123456789012345678901212345 6
12
3456789012345678901234567890121234567890123456789012345678901212345 6
12
3456789012345678901234567890121234567890123456789012345678901212345 6
12
123456789012345678901234567890121234567890123456789012345678901212345 6
2
6
123456789012345678901234567890121234567890123456789012345678901212345
6
123456789012345678901234567890121234567890123456789012345678901212345
Point C is at the center of the circle.
6
123456789012345678901234567890121234567890123456789012345678901212345
6
123456789012345678901234567890121234567890123456789012345678901212345
AB
5
DE
1 3456789012345678901234567890121234567890123456789012345678901212345 6
2
3456789012345678901234567890121234567890123456789012345678901212345
123456789012345678901234567890121234567890123456789012345678901212345 66
123456789012345678901234567890121234567890123456789012345678901212345
CBD
CEA
6
123456789012345678901234567890121234567890123456789012345678901212345
123456789012345678901234567890121234567890123456789012345678901212345 66
1
3456789012345678901234567890121234567890123456789012345678901212345 6
12
123456789012345678901234567890121234567890123456789012345678901212345 6
3456789012345678901234567890121234567890123456789012345678901212345 6
12
123456789012345678901234567890121234567890123456789012345678901212345 6
3456789012345678901234567890121234567890123456789012345678901212345 6
12
123456789012345678901234567890121234567890123456789012345678901212345 6
123456789012345678901234567890121234567890123456789012345678901212345 6
123456789012345678901234567890121234567890123456789012345678901212345 6
123456789012345678901234567890121234567890123456789012345678901212345 6
1234567890123456789012345678901212345678901234567890123456789012123456

www.petersons.com

http://www.xtremepapers.net

DIAGNOSTIC
1234567890123456789012345678901212345678901234567890123456789012123456
123456789012345678901234567890121234567890123456789012345678901212345 6
3456789012345678901234567890121234567890123456789012345678901212345 6
12
6
123456789012345678901234567890121234567890123456789012345678901212345
Column A
Column B
3456789012345678901234567890121234567890123456789012345678901212345
6
12
Directions:
Select
the
best
answer
for
each
3456789012345678901234567890121234567890123456789012345678901212345
6
12
11. The sum of all even
The sum of all odd
6
123456789012345678901234567890121234567890123456789012345678901212345
of
the
following
questions.
3456789012345678901234567890121234567890123456789012345678901212345
6
12
integers
one
integers
one
2
3456789012345678901234567890121234567890123456789012345678901212345
6
1
3456789012345678901234567890121234567890123456789012345678901212345
6
12
through one
through one
3456789012345678901234567890121234567890123456789012345678901212345
6
12
15.
There
are
13
marbles
in
a
bag.
One
is
red,
3456789012345678901234567890121234567890123456789012345678901212345
6
12
hundred
hundred
3456789012345678901234567890121234567890123456789012345678901212345
6
12
4
are
blue,
and
8
are
yellow.
If
a
blue
2
3456789012345678901234567890121234567890123456789012345678901212345
6
1
3456789012345678901234567890121234567890123456789012345678901212345
6
12
marble
is
drawn
from
the
bag,
what
is
the
3456789012345678901234567890121234567890123456789012345678901212345 6
12
3456789012345678901234567890121234567890123456789012345678901212345
6
12
probability that a red or yellow marble
3456789012345678901234567890121234567890123456789012345678901212345
6
12
2
3456789012345678901234567890121234567890123456789012345678901212345
6
1
will be drawn next?
3456789012345678901234567890121234567890123456789012345678901212345
6
12
3456789012345678901234567890121234567890123456789012345678901212345 6
12
3456789012345678901234567890121234567890123456789012345678901212345
6
12
1
3456789012345678901234567890121234567890123456789012345678901212345
6
12
A.
2
3456789012345678901234567890121234567890123456789012345678901212345
6
1
4
3456789012345678901234567890121234567890123456789012345678901212345
6
12
12.
The cube of an
The square of an odd
3456789012345678901234567890121234567890123456789012345678901212345
6
12
2
3456789012345678901234567890121234567890123456789012345678901212345
6
12
B.
even
prime
number
prime
number
3456789012345678901234567890121234567890123456789012345678901212345
6
12
3
2
3456789012345678901234567890121234567890123456789012345678901212345
123456789012345678901234567890121234567890123456789012345678901212345 66
123456789012345678901234567890121234567890123456789012345678901212345
8
6
123456789012345678901234567890121234567890123456789012345678901212345
6
123456789012345678901234567890121234567890123456789012345678901212345
C.
6
13
123456789012345678901234567890121234567890123456789012345678901212345
123456789012345678901234567890121234567890123456789012345678901212345 66
1
3
3456789012345678901234567890121234567890123456789012345678901212345
6
12
D.
3456789012345678901234567890121234567890123456789012345678901212345
6
12
4
3456789012345678901234567890121234567890123456789012345678901212345 6
12
2
3456789012345678901234567890121234567890123456789012345678901212345
6
123456789012345678901234567890121234567890123456789012345678901212345
9
6
123456789012345678901234567890121234567890123456789012345678901212345
13.
x,0
E.
6
123456789012345678901234567890121234567890123456789012345678901212345
13
6
123456789012345678901234567890121234567890123456789012345678901212345
2
2
6
123456789012345678901234567890121234567890123456789012345678901212345
2
12x
2
4
x
x
6
123456789012345678901234567890121234567890123456789012345678901212345
16.
Bananas
at
Maxs
Grocer
went
on
sale
for
6
123456789012345678901234567890121234567890123456789012345678901212345
2x
2
3
2
x
1
1
6
123456789012345678901234567890121234567890123456789012345678901212345
50 percent off. They were then marked
6
123456789012345678901234567890121234567890123456789012345678901212345
6
1
down
again
by
30
percent.
If
x
is
the
2
3456789012345678901234567890121234567890123456789012345678901212345
123456789012345678901234567890121234567890123456789012345678901212345 66
123456789012345678901234567890121234567890123456789012345678901212345
original price of bananas per pound, how 6
1
6
123456789012345678901234567890121234567890123456789012345678901212345
much does it cost to buy 2.5 pounds of
3456789012345678901234567890121234567890123456789012345678901212345
6
12
2
3456789012345678901234567890121234567890123456789012345678901212345
6
123456789012345678901234567890121234567890123456789012345678901212345
bananas?
6
123456789012345678901234567890121234567890123456789012345678901212345
6
1
3456789012345678901234567890121234567890123456789012345678901212345
6
12
A.
0.15x
14.
x
5
(x)(x
1
1)
3456789012345678901234567890121234567890123456789012345678901212345 6
12
2
3456789012345678901234567890121234567890123456789012345678901212345
6
123456789012345678901234567890121234567890123456789012345678901212345
B. 0.375x
6
123456789012345678901234567890121234567890123456789012345678901212345
(3)
(3)(3)
6
1
C.
0.42x
123456789012345678901234567890121234567890123456789012345678901212345 6
6
123456789012345678901234567890121234567890123456789012345678901212345
D. 0.50x
3456789012345678901234567890121234567890123456789012345678901212345
6
12
3456789012345678901234567890121234567890123456789012345678901212345
6
12
E. 0.875x
3456789012345678901234567890121234567890123456789012345678901212345 6
12
3456789012345678901234567890121234567890123456789012345678901212345 6
12
123456789012345678901234567890121234567890123456789012345678901212345 6
2
6
123456789012345678901234567890121234567890123456789012345678901212345
6
123456789012345678901234567890121234567890123456789012345678901212345
6
123456789012345678901234567890121234567890123456789012345678901212345
6
123456789012345678901234567890121234567890123456789012345678901212345
1 3456789012345678901234567890121234567890123456789012345678901212345 6
2
3456789012345678901234567890121234567890123456789012345678901212345
123456789012345678901234567890121234567890123456789012345678901212345 66
123456789012345678901234567890121234567890123456789012345678901212345 6
123456789012345678901234567890121234567890123456789012345678901212345 6
123456789012345678901234567890121234567890123456789012345678901212345 6
1
3456789012345678901234567890121234567890123456789012345678901212345 6
12
123456789012345678901234567890121234567890123456789012345678901212345 6
3456789012345678901234567890121234567890123456789012345678901212345 6
12
123456789012345678901234567890121234567890123456789012345678901212345 6
3456789012345678901234567890121234567890123456789012345678901212345 6
12
123456789012345678901234567890121234567890123456789012345678901212345 6
123456789012345678901234567890121234567890123456789012345678901212345 6
123456789012345678901234567890121234567890123456789012345678901212345 6
123456789012345678901234567890121234567890123456789012345678901212345 6 37
1234567890123456789012345678901212345678901234567890123456789012123456

http://www.xtremepapers.net

Part II: Diagnosing Strengths and Weaknesses

38

1234567890123456789012345678901212345678901234567890123456789012123456
123456789012345678901234567890121234567890123456789012345678901212345 6
3456789012345678901234567890121234567890123456789012345678901212345 6
12
17.
Questions 2021 refer to the following graphs. 6
123456789012345678901234567890121234567890123456789012345678901212345
3456789012345678901234567890121234567890123456789012345678901212345 6
12
3456789012345678901234567890121234567890123456789012345678901212345 6
12
123456789012345678901234567890121234567890123456789012345678901212345 6
3456789012345678901234567890121234567890123456789012345678901212345 6
12
123456789012345678901234567890121234567890123456789012345678901212345 6
3456789012345678901234567890121234567890123456789012345678901212345 6
12
3456789012345678901234567890121234567890123456789012345678901212345 6
12
3456789012345678901234567890121234567890123456789012345678901212345 6
12
3456789012345678901234567890121234567890123456789012345678901212345 6
12
2
1 3456789012345678901234567890121234567890123456789012345678901212345 6
3456789012345678901234567890121234567890123456789012345678901212345 6
12
3456789012345678901234567890121234567890123456789012345678901212345 6
12
3456789012345678901234567890121234567890123456789012345678901212345
6
12
If BE bisects AC, what is the length of
3456789012345678901234567890121234567890123456789012345678901212345
6
12
2
3456789012345678901234567890121234567890123456789012345678901212345
6
1
EC?
3456789012345678901234567890121234567890123456789012345678901212345
6
12
3456789012345678901234567890121234567890123456789012345678901212345 6
12
3456789012345678901234567890121234567890123456789012345678901212345 6
12
A. 2=3
3456789012345678901234567890121234567890123456789012345678901212345
6
12
2
6
1 3456789012345678901234567890121234567890123456789012345678901212345
3456789012345678901234567890121234567890123456789012345678901212345
6
12
B.
4
3456789012345678901234567890121234567890123456789012345678901212345 6
12
3456789012345678901234567890121234567890123456789012345678901212345
6
12
C. 4=3
3456789012345678901234567890121234567890123456789012345678901212345
6
12
2
3456789012345678901234567890121234567890123456789012345678901212345
123456789012345678901234567890121234567890123456789012345678901212345 66
123456789012345678901234567890121234567890123456789012345678901212345
D. 8
6
123456789012345678901234567890121234567890123456789012345678901212345
E. 8=3
6
123456789012345678901234567890121234567890123456789012345678901212345
123456789012345678901234567890121234567890123456789012345678901212345 66
123456789012345678901234567890121234567890123456789012345678901212345 6
1 18. If x is an odd integer, in which one of the
3456789012345678901234567890121234567890123456789012345678901212345 6
12
3456789012345678901234567890121234567890123456789012345678901212345
6
12
following expressions must y be even?
3456789012345678901234567890121234567890123456789012345678901212345
6
12
2
3456789012345678901234567890121234567890123456789012345678901212345
123456789012345678901234567890121234567890123456789012345678901212345 66
123456789012345678901234567890121234567890123456789012345678901212345
A. x 1 y 5 12
6
123456789012345678901234567890121234567890123456789012345678901212345
20. Which of the following has the greatest
6
123456789012345678901234567890121234567890123456789012345678901212345
B. x2 1 y 5 16
6
123456789012345678901234567890121234567890123456789012345678901212345
value?
6
123456789012345678901234567890121234567890123456789012345678901212345
C. xy 5 25
6
123456789012345678901234567890121234567890123456789012345678901212345
2
6
123456789012345678901234567890121234567890123456789012345678901212345
A. Segment of Manchester population
D. xy 5 49
6
123456789012345678901234567890121234567890123456789012345678901212345
025 in 1920
6
1
E. xy 5 98
2
3456789012345678901234567890121234567890123456789012345678901212345
6
123456789012345678901234567890121234567890123456789012345678901212345
B. Byson population in 1900
6
123456789012345678901234567890121234567890123456789012345678901212345
6
1 19. Export Auto charges $30 for an oil change
C. Byson population in 1915
6
123456789012345678901234567890121234567890123456789012345678901212345
3456789012345678901234567890121234567890123456789012345678901212345
6
12
and
$40
for
a
brake
job.
On
Tuesday,
they
D.
Segment
of
Manchester
population
2
3456789012345678901234567890121234567890123456789012345678901212345
123456789012345678901234567890121234567890123456789012345678901212345 66
123456789012345678901234567890121234567890123456789012345678901212345
charged a total of $320 for all oil changes
2550 in 1920
6
1
3456789012345678901234567890121234567890123456789012345678901212345
6
12
and
brake
checks.
They
performed
a
fourth
E.
Segment
of
Manchester
population
3456789012345678901234567890121234567890123456789012345678901212345 6
12
2
3456789012345678901234567890121234567890123456789012345678901212345
6
123456789012345678901234567890121234567890123456789012345678901212345
as many brake checks as oil changes. How
50100 in 1920
6
123456789012345678901234567890121234567890123456789012345678901212345
6
1
many
oil
changes
did
they
perform
on
123456789012345678901234567890121234567890123456789012345678901212345 6
123456789012345678901234567890121234567890123456789012345678901212345
21. Which town in which decade had the 6
Tuesday?
3456789012345678901234567890121234567890123456789012345678901212345
6
12
3456789012345678901234567890121234567890123456789012345678901212345
12
greatest percentage increase in population? 6
3456789012345678901234567890121234567890123456789012345678901212345
6
12
A. 2
3456789012345678901234567890121234567890123456789012345678901212345
6
12
A. Manchester 19001910
6
123456789012345678901234567890121234567890123456789012345678901212345
B. 4
2
3456789012345678901234567890121234567890123456789012345678901212345
6
123456789012345678901234567890121234567890123456789012345678901212345
B. Ledbetter 19201930
C. 6
6
123456789012345678901234567890121234567890123456789012345678901212345
6
123456789012345678901234567890121234567890123456789012345678901212345
C. Ledbetter 19301940
D. 8
6
123456789012345678901234567890121234567890123456789012345678901212345
6
1
D.
Byson
19201930
E.
10
2
3456789012345678901234567890121234567890123456789012345678901212345
123456789012345678901234567890121234567890123456789012345678901212345 66
123456789012345678901234567890121234567890123456789012345678901212345
E. Manchester 19301940
6
123456789012345678901234567890121234567890123456789012345678901212345
123456789012345678901234567890121234567890123456789012345678901212345 66
1
3456789012345678901234567890121234567890123456789012345678901212345 6
12
123456789012345678901234567890121234567890123456789012345678901212345 6
3456789012345678901234567890121234567890123456789012345678901212345 6
12
123456789012345678901234567890121234567890123456789012345678901212345 6
3456789012345678901234567890121234567890123456789012345678901212345 6
12
123456789012345678901234567890121234567890123456789012345678901212345 6
123456789012345678901234567890121234567890123456789012345678901212345 6
123456789012345678901234567890121234567890123456789012345678901212345 6
123456789012345678901234567890121234567890123456789012345678901212345 6
1234567890123456789012345678901212345678901234567890123456789012123456

www.petersons.com

http://www.xtremepapers.net

DIAGNOSTIC
1234567890123456789012345678901212345678901234567890123456789012123456
6
123456789012345678901234567890121234567890123456789012345678901212345
@~6a!a 2 ~4a 1 7a!#
3456789012345678901234567890121234567890123456789012345678901212345
6
12
2
7a
1
J
5
22.
25.
If
x
is
an
odd
integer,
what
is
the
value
of
half
2
3456789012345678901234567890121234567890123456789012345678901212345
6
1
a
3456789012345678901234567890121234567890123456789012345678901212345
6
12
the sum of x and the next three odd integers? 6
3456789012345678901234567890121234567890123456789012345678901212345
12
2
6
123456789012345678901234567890121234567890123456789012345678901212345
6a 2 18a
3456789012345678901234567890121234567890123456789012345678901212345
6
12
1
J
A.
A.
2(x
1
3)
2
3456789012345678901234567890121234567890123456789012345678901212345
6
1
a
3456789012345678901234567890121234567890123456789012345678901212345
6
12
B.
3(x
1
3)
2
3456789012345678901234567890121234567890123456789012345678901212345
6
12
6a
2
4a
3456789012345678901234567890121234567890123456789012345678901212345
6
12
C.
4(x
1
3)
1
J
B.
3456789012345678901234567890121234567890123456789012345678901212345
6
12
a
2
3456789012345678901234567890121234567890123456789012345678901212345
6
1
D. 5(x 1 3)
3456789012345678901234567890121234567890123456789012345678901212345
6
12
3456789012345678901234567890121234567890123456789012345678901212345
6
12
C. 7 2 a 1 J
E.
6(x
1
3)
3456789012345678901234567890121234567890123456789012345678901212345 6
12
3456789012345678901234567890121234567890123456789012345678901212345
6
12
D. 211 2 a 1 J
6
123456789012345678901234567890121234567890123456789012345678901212345
3456789012345678901234567890121234567890123456789012345678901212345
6
12
E. 13a 2 11 1 J
Questions 2628 refer to the following graphs. 6
3456789012345678901234567890121234567890123456789012345678901212345
12
3456789012345678901234567890121234567890123456789012345678901212345 6
12
3456789012345678901234567890121234567890123456789012345678901212345 6
12
2
6
1 3456789012345678901234567890121234567890123456789012345678901212345
23.
3456789012345678901234567890121234567890123456789012345678901212345
6
12
3456789012345678901234567890121234567890123456789012345678901212345 6
12
3456789012345678901234567890121234567890123456789012345678901212345 6
12
3456789012345678901234567890121234567890123456789012345678901212345 6
12
2
6
123456789012345678901234567890121234567890123456789012345678901212345
6
123456789012345678901234567890121234567890123456789012345678901212345
6
123456789012345678901234567890121234567890123456789012345678901212345
6
123456789012345678901234567890121234567890123456789012345678901212345
6
123456789012345678901234567890121234567890123456789012345678901212345
3456789012345678901234567890121234567890123456789012345678901212345
123456789012345678901234567890121234567890123456789012345678901212345 66
1
3456789012345678901234567890121234567890123456789012345678901212345 6
12
3456789012345678901234567890121234567890123456789012345678901212345
6
12
B is the midpoint of AC, and E is the
3456789012345678901234567890121234567890123456789012345678901212345
6
12
2
3456789012345678901234567890121234567890123456789012345678901212345
6
123456789012345678901234567890121234567890123456789012345678901212345
midpoint of FD. If the area of rectangle
6
123456789012345678901234567890121234567890123456789012345678901212345
6
123456789012345678901234567890121234567890123456789012345678901212345
BCDE is 12, what is the perimeter of
123456789012345678901234567890121234567890123456789012345678901212345 66
rectangle ACDF?
123456789012345678901234567890121234567890123456789012345678901212345
123456789012345678901234567890121234567890123456789012345678901212345 66
123456789012345678901234567890121234567890123456789012345678901212345
A. 18
6
123456789012345678901234567890121234567890123456789012345678901212345
6
123456789012345678901234567890121234567890123456789012345678901212345
B.
20
6
1
2
3456789012345678901234567890121234567890123456789012345678901212345
6
123456789012345678901234567890121234567890123456789012345678901212345
C. 22
6
123456789012345678901234567890121234567890123456789012345678901212345
6
1
D.
24
123456789012345678901234567890121234567890123456789012345678901212345 6
3456789012345678901234567890121234567890123456789012345678901212345
6
12
E. 26
3456789012345678901234567890121234567890123456789012345678901212345
6
12
3456789012345678901234567890121234567890123456789012345678901212345 6
12
123456789012345678901234567890121234567890123456789012345678901212345 6
3456789012345678901234567890121234567890123456789012345678901212345
6
12
24.
3456789012345678901234567890121234567890123456789012345678901212345
6
12
2
3456789012345678901234567890121234567890123456789012345678901212345
123456789012345678901234567890121234567890123456789012345678901212345 66
123456789012345678901234567890121234567890123456789012345678901212345 6
1
123456789012345678901234567890121234567890123456789012345678901212345 6
123456789012345678901234567890121234567890123456789012345678901212345 6
3456789012345678901234567890121234567890123456789012345678901212345 6
12
3456789012345678901234567890121234567890123456789012345678901212345 6
12
3456789012345678901234567890121234567890123456789012345678901212345
6
12
What is the perimeter of DABC?
3456789012345678901234567890121234567890123456789012345678901212345
6
12
123456789012345678901234567890121234567890123456789012345678901212345 6
2
3456789012345678901234567890121234567890123456789012345678901212345
6
123456789012345678901234567890121234567890123456789012345678901212345
A. 2 1 =2
6
123456789012345678901234567890121234567890123456789012345678901212345
123456789012345678901234567890121234567890123456789012345678901212345 66
123456789012345678901234567890121234567890123456789012345678901212345
B. 2 1 2=3
6
1
3456789012345678901234567890121234567890123456789012345678901212345
6
12
15 20
25
30
35
40
45
3456789012345678901234567890121234567890123456789012345678901212345
6
12
4
1
2
2
C.
=
3456789012345678901234567890121234567890123456789012345678901212345 6
12
3456789012345678901234567890121234567890123456789012345678901212345 6
12
6
123456789012345678901234567890121234567890123456789012345678901212345
D. 4 1 2=3
2
6
123456789012345678901234567890121234567890123456789012345678901212345
6
1 3456789012345678901234567890121234567890123456789012345678901212345
E. 4 1 4=3
3456789012345678901234567890121234567890123456789012345678901212345
6
12
123456789012345678901234567890121234567890123456789012345678901212345 6
3456789012345678901234567890121234567890123456789012345678901212345 6
12
123456789012345678901234567890121234567890123456789012345678901212345 6
123456789012345678901234567890121234567890123456789012345678901212345 6
123456789012345678901234567890121234567890123456789012345678901212345 6
123456789012345678901234567890121234567890123456789012345678901212345 6 39
1234567890123456789012345678901212345678901234567890123456789012123456

http://www.xtremepapers.net

Part II: Diagnosing Strengths and Weaknesses

40

1234567890123456789012345678901212345678901234567890123456789012123456
123456789012345678901234567890121234567890123456789012345678901212345 6
3456789012345678901234567890121234567890123456789012345678901212345 6
12
26. Whose annual savings/annual spending
28. Over what 5-year span did percentage of
6
123456789012345678901234567890121234567890123456789012345678901212345
3456789012345678901234567890121234567890123456789012345678901212345
6
12
ratio was the smallest?
spending increase the most?
3456789012345678901234567890121234567890123456789012345678901212345
6
12
123456789012345678901234567890121234567890123456789012345678901212345 6
3456789012345678901234567890121234567890123456789012345678901212345
6
12
A. 15
A. 1520
6
123456789012345678901234567890121234567890123456789012345678901212345
3456789012345678901234567890121234567890123456789012345678901212345
6
12
B.
20
B.
2530
3456789012345678901234567890121234567890123456789012345678901212345 6
12
3456789012345678901234567890121234567890123456789012345678901212345
6
12
C. 30
C. 3035
3456789012345678901234567890121234567890123456789012345678901212345
6
12
2
3456789012345678901234567890121234567890123456789012345678901212345
6
1
D. 35
D. 3540
3456789012345678901234567890121234567890123456789012345678901212345
6
12
3456789012345678901234567890121234567890123456789012345678901212345
6
12
E.
45
E.
4045
3456789012345678901234567890121234567890123456789012345678901212345 6
12
3456789012345678901234567890121234567890123456789012345678901212345 6
12
123456789012345678901234567890121234567890123456789012345678901212345 6
3456789012345678901234567890121234567890123456789012345678901212345
6
12
27. What category of sales had the greatest
3456789012345678901234567890121234567890123456789012345678901212345
6
12
3456789012345678901234567890121234567890123456789012345678901212345
6
amount of money spent on it if you only
12
3456789012345678901234567890121234567890123456789012345678901212345
6
12
consider
15-,
30-,
and
45-year-olds?
2
3456789012345678901234567890121234567890123456789012345678901212345
6
1
3456789012345678901234567890121234567890123456789012345678901212345 6
12
3456789012345678901234567890121234567890123456789012345678901212345
6
12
A. Home Entertainment
3456789012345678901234567890121234567890123456789012345678901212345
6
12
3456789012345678901234567890121234567890123456789012345678901212345
6
12
B.
Clothes
2
3456789012345678901234567890121234567890123456789012345678901212345
123456789012345678901234567890121234567890123456789012345678901212345 66
123456789012345678901234567890121234567890123456789012345678901212345
C. Lawn Care
6
123456789012345678901234567890121234567890123456789012345678901212345
6
123456789012345678901234567890121234567890123456789012345678901212345
D.
Sporting
Goods
123456789012345678901234567890121234567890123456789012345678901212345 66
123456789012345678901234567890121234567890123456789012345678901212345
E. Apparel
6
1
3456789012345678901234567890121234567890123456789012345678901212345 6
12
3456789012345678901234567890121234567890123456789012345678901212345 6
12
3456789012345678901234567890121234567890123456789012345678901212345 6
12
2
6
123456789012345678901234567890121234567890123456789012345678901212345
Verbal
6
123456789012345678901234567890121234567890123456789012345678901212345
6
123456789012345678901234567890121234567890123456789012345678901212345
General
Information
6
123456789012345678901234567890121234567890123456789012345678901212345
6
123456789012345678901234567890121234567890123456789012345678901212345
1.
The
test
has
30
questions.
You
have
30
minutes
to
complete
the
section.
3456789012345678901234567890121234567890123456789012345678901212345
123456789012345678901234567890121234567890123456789012345678901212345 66
123456789012345678901234567890121234567890123456789012345678901212345 6
123456789012345678901234567890121234567890123456789012345678901212345 6
123456789012345678901234567890121234567890123456789012345678901212345 6
1
Directions: Each question contains a word printed in capital letters, followed by five
3456789012345678901234567890121234567890123456789012345678901212345
6
12
3456789012345678901234567890121234567890123456789012345678901212345
6
answer choices. Choose the answer choice that contains the word or phrase most nearly
12
6
123456789012345678901234567890121234567890123456789012345678901212345
OPPOSITE in meaning to the word in capital letters.
6
123456789012345678901234567890121234567890123456789012345678901212345
3456789012345678901234567890121234567890123456789012345678901212345 6
12
2
3456789012345678901234567890121234567890123456789012345678901212345
123456789012345678901234567890121234567890123456789012345678901212345 66
123456789012345678901234567890121234567890123456789012345678901212345 6
1
3456789012345678901234567890121234567890123456789012345678901212345
6
12
1. PRISTINE :
3. LUCID :
3456789012345678901234567890121234567890123456789012345678901212345
6
12
2
3456789012345678901234567890121234567890123456789012345678901212345
123456789012345678901234567890121234567890123456789012345678901212345 66
123456789012345678901234567890121234567890123456789012345678901212345
A. dull
A. translucent
6
1
6
123456789012345678901234567890121234567890123456789012345678901212345
B.
hopeless
B.
myopic
123456789012345678901234567890121234567890123456789012345678901212345 6
2
3456789012345678901234567890121234567890123456789012345678901212345
6
123456789012345678901234567890121234567890123456789012345678901212345
C. congested
C. putrid
6
123456789012345678901234567890121234567890123456789012345678901212345
6
123456789012345678901234567890121234567890123456789012345678901212345
D.
majestic
D.
luminescent
123456789012345678901234567890121234567890123456789012345678901212345 66
1
E. polluted
E. opaque
3456789012345678901234567890121234567890123456789012345678901212345
6
12
3456789012345678901234567890121234567890123456789012345678901212345 6
12
3456789012345678901234567890121234567890123456789012345678901212345 6
12
3456789012345678901234567890121234567890123456789012345678901212345
6
12
2. IMPREGNABLE :
4. MALLEABLE :
6
123456789012345678901234567890121234567890123456789012345678901212345
2
3456789012345678901234567890121234567890123456789012345678901212345
6
123456789012345678901234567890121234567890123456789012345678901212345
A. penetrable
A. brittle
6
123456789012345678901234567890121234567890123456789012345678901212345
6
123456789012345678901234567890121234567890123456789012345678901212345
B.
tangible
B. ductile
6
123456789012345678901234567890121234567890123456789012345678901212345
6
1
C.
intractable
C.
pliant
2
3456789012345678901234567890121234567890123456789012345678901212345
123456789012345678901234567890121234567890123456789012345678901212345 66
1
D. tolerable
D. flaccid
3456789012345678901234567890121234567890123456789012345678901212345
6
12
6
123456789012345678901234567890121234567890123456789012345678901212345
E.
reliable
E.
verdant
3456789012345678901234567890121234567890123456789012345678901212345 6
12
2
3456789012345678901234567890121234567890123456789012345678901212345
6
1
123456789012345678901234567890121234567890123456789012345678901212345 6
123456789012345678901234567890121234567890123456789012345678901212345 6
123456789012345678901234567890121234567890123456789012345678901212345 6
1234567890123456789012345678901212345678901234567890123456789012123456

www.petersons.com

http://www.xtremepapers.net

DIAGNOSTIC
1234567890123456789012345678901212345678901234567890123456789012123456
123456789012345678901234567890121234567890123456789012345678901212345 6
3456789012345678901234567890121234567890123456789012345678901212345 6
12
5. CASTIGATE :
8. TRUCULENT :
6
123456789012345678901234567890121234567890123456789012345678901212345
3456789012345678901234567890121234567890123456789012345678901212345 6
12
3456789012345678901234567890121234567890123456789012345678901212345
6
12
A. criticize
A. pugnacious
6
123456789012345678901234567890121234567890123456789012345678901212345
3456789012345678901234567890121234567890123456789012345678901212345
6
12
B.
laud
B.
boorish
2
3456789012345678901234567890121234567890123456789012345678901212345
6
1
3456789012345678901234567890121234567890123456789012345678901212345
6
12
C.
ignore
C.
gentle
3456789012345678901234567890121234567890123456789012345678901212345 6
12
3456789012345678901234567890121234567890123456789012345678901212345
6
12
D. mollify
D. beneficent
3456789012345678901234567890121234567890123456789012345678901212345
6
12
2
3456789012345678901234567890121234567890123456789012345678901212345
6
1
E. obfuscate
E. altruistic
3456789012345678901234567890121234567890123456789012345678901212345
6
12
3456789012345678901234567890121234567890123456789012345678901212345 6
12
3456789012345678901234567890121234567890123456789012345678901212345 6
12
3456789012345678901234567890121234567890123456789012345678901212345
6
12
6. APPROBATION :
9. SALUBRIOUS :
2
6
1 3456789012345678901234567890121234567890123456789012345678901212345
3456789012345678901234567890121234567890123456789012345678901212345
6
12
A. disapproval
A. salutary
3456789012345678901234567890121234567890123456789012345678901212345
6
12
3456789012345678901234567890121234567890123456789012345678901212345
6
12
B.
fanfare
B.
deleterious
3456789012345678901234567890121234567890123456789012345678901212345 6
12
2
3456789012345678901234567890121234567890123456789012345678901212345
6
1
C. calamity
C. beneficial
3456789012345678901234567890121234567890123456789012345678901212345
6
12
3456789012345678901234567890121234567890123456789012345678901212345
6
12
D.
enigma
D.
acrid
3456789012345678901234567890121234567890123456789012345678901212345 6
12
3456789012345678901234567890121234567890123456789012345678901212345
6
12
E. rejoinder
E. ignominious
3456789012345678901234567890121234567890123456789012345678901212345
6
12
3456789012345678901234567890121234567890123456789012345678901212345 6
12
3456789012345678901234567890121234567890123456789012345678901212345 6
12
3456789012345678901234567890121234567890123456789012345678901212345
6
12
7. MENDACITY :
3456789012345678901234567890121234567890123456789012345678901212345
6
12
2
3456789012345678901234567890121234567890123456789012345678901212345
123456789012345678901234567890121234567890123456789012345678901212345 66
1
A. evil
3456789012345678901234567890121234567890123456789012345678901212345
6
12
B. wisdom
3456789012345678901234567890121234567890123456789012345678901212345
6
12
3456789012345678901234567890121234567890123456789012345678901212345
6
12
C.
perfidy
2
3456789012345678901234567890121234567890123456789012345678901212345
123456789012345678901234567890121234567890123456789012345678901212345 66
123456789012345678901234567890121234567890123456789012345678901212345
D. honesty
6
123456789012345678901234567890121234567890123456789012345678901212345
6
123456789012345678901234567890121234567890123456789012345678901212345
E.
irregularity
123456789012345678901234567890121234567890123456789012345678901212345 66
123456789012345678901234567890121234567890123456789012345678901212345 6
123456789012345678901234567890121234567890123456789012345678901212345 6
123456789012345678901234567890121234567890123456789012345678901212345 6
123456789012345678901234567890121234567890123456789012345678901212345 6
1 Directions: In each question, there is an initial pair of words or phrases that are related in
3456789012345678901234567890121234567890123456789012345678901212345 6
12
3456789012345678901234567890121234567890123456789012345678901212345
6
12
some manner. Select the answer choice containing the lettered pair that best expresses a
6
123456789012345678901234567890121234567890123456789012345678901212345
6
123456789012345678901234567890121234567890123456789012345678901212345
relationship
similar
to
the
initial
pair.
3456789012345678901234567890121234567890123456789012345678901212345 6
12
2
3456789012345678901234567890121234567890123456789012345678901212345
123456789012345678901234567890121234567890123456789012345678901212345 66
123456789012345678901234567890121234567890123456789012345678901212345 6
1
3456789012345678901234567890121234567890123456789012345678901212345 6
12
3456789012345678901234567890121234567890123456789012345678901212345
6
12
10. POTTER : CLAY ::
12. HOMOSAPIEN : MAMMAL ::
2
6
123456789012345678901234567890121234567890123456789012345678901212345
6
123456789012345678901234567890121234567890123456789012345678901212345
6
1 3456789012345678901234567890121234567890123456789012345678901212345
A. bull : rider
A. lung : organ
123456789012345678901234567890121234567890123456789012345678901212345 6
6
123456789012345678901234567890121234567890123456789012345678901212345
B. conductor : instrument
B. beach : ocean
3456789012345678901234567890121234567890123456789012345678901212345
6
12
3456789012345678901234567890121234567890123456789012345678901212345
6
12
C. poet : words
C. canine : feline
3456789012345678901234567890121234567890123456789012345678901212345
6
12
D. doctor : patient
D. mystery : novel
3456789012345678901234567890121234567890123456789012345678901212345
6
12
6
123456789012345678901234567890121234567890123456789012345678901212345
E.
lady
:
gentleman
E.
bovine
:
husbandry
2
3456789012345678901234567890121234567890123456789012345678901212345
123456789012345678901234567890121234567890123456789012345678901212345 66
123456789012345678901234567890121234567890123456789012345678901212345 6
123456789012345678901234567890121234567890123456789012345678901212345 6
123456789012345678901234567890121234567890123456789012345678901212345
13. IMMUTABLE : CHANGE ::
6
1 11. MINISCULE : SMALL ::
3456789012345678901234567890121234567890123456789012345678901212345 6
12
3456789012345678901234567890121234567890123456789012345678901212345
6
12
A. large : gigantic
A. political : mastermind
3456789012345678901234567890121234567890123456789012345678901212345
6
12
3456789012345678901234567890121234567890123456789012345678901212345
6
12
B. contingent : necessity
B. exuberant : happy
123456789012345678901234567890121234567890123456789012345678901212345 6
2
3456789012345678901234567890121234567890123456789012345678901212345
6
123456789012345678901234567890121234567890123456789012345678901212345
C. ecstatic : joy
C. downtrodden : unjust
6
1
3456789012345678901234567890121234567890123456789012345678901212345
6
12
D. imminent : proximity
D. humorless : hapless
123456789012345678901234567890121234567890123456789012345678901212345 6
3456789012345678901234567890121234567890123456789012345678901212345
6
12
E. humble : hubris
E. tense : relaxed
6
123456789012345678901234567890121234567890123456789012345678901212345
123456789012345678901234567890121234567890123456789012345678901212345 6
123456789012345678901234567890121234567890123456789012345678901212345 6
123456789012345678901234567890121234567890123456789012345678901212345 6 41
1234567890123456789012345678901212345678901234567890123456789012123456

http://www.xtremepapers.net

Part II: Diagnosing Strengths and Weaknesses

42

1234567890123456789012345678901212345678901234567890123456789012123456
123456789012345678901234567890121234567890123456789012345678901212345 6
3456789012345678901234567890121234567890123456789012345678901212345 6
12
14. VITUPERATE : BERATE ::
16. LACONIC : VERBOSE ::
6
123456789012345678901234567890121234567890123456789012345678901212345
3456789012345678901234567890121234567890123456789012345678901212345 6
12
3456789012345678901234567890121234567890123456789012345678901212345
6
12
A. destroy : rebuild
A. terse : taciturn
6
123456789012345678901234567890121234567890123456789012345678901212345
3456789012345678901234567890121234567890123456789012345678901212345
6
12
B.
vacillate
:
waver
B.
delicious
:
delectable
2
3456789012345678901234567890121234567890123456789012345678901212345
6
1
3456789012345678901234567890121234567890123456789012345678901212345
6
12
C.
synthesize
:
raze
C.
passive
:
lethargic
3456789012345678901234567890121234567890123456789012345678901212345 6
12
3456789012345678901234567890121234567890123456789012345678901212345
6
12
D. confuse : clarify
D. guileless : deceitful
3456789012345678901234567890121234567890123456789012345678901212345
6
12
2
3456789012345678901234567890121234567890123456789012345678901212345
6
1
E. pacify : enrage
E. fastidious : studious
3456789012345678901234567890121234567890123456789012345678901212345
6
12
3456789012345678901234567890121234567890123456789012345678901212345 6
12
3456789012345678901234567890121234567890123456789012345678901212345 6
12
3456789012345678901234567890121234567890123456789012345678901212345
6
12
15. BUCOLIC : PASTORAL ::
17. MALEVOLENT : BONHOMIE ::
2
6
1 3456789012345678901234567890121234567890123456789012345678901212345
3456789012345678901234567890121234567890123456789012345678901212345
6
12
A. frigid : lukewarm
A. hateful : disgust
3456789012345678901234567890121234567890123456789012345678901212345
6
12
3456789012345678901234567890121234567890123456789012345678901212345
6
12
B.
basic
:
advanced
B.
calm
:
anxiety
3456789012345678901234567890121234567890123456789012345678901212345 6
12
2
3456789012345678901234567890121234567890123456789012345678901212345
6
1
C. irascible : temperamental
C. dangerous : peril
3456789012345678901234567890121234567890123456789012345678901212345
6
12
3456789012345678901234567890121234567890123456789012345678901212345
6
12
D.
haughty
:
selfish
D.
hungry
:
want
3456789012345678901234567890121234567890123456789012345678901212345 6
12
3456789012345678901234567890121234567890123456789012345678901212345
6
12
E. pugnacious : optimistic
E. facetious : sarcasm
3456789012345678901234567890121234567890123456789012345678901212345
6
12
3456789012345678901234567890121234567890123456789012345678901212345 6
12
3456789012345678901234567890121234567890123456789012345678901212345 6
12
3456789012345678901234567890121234567890123456789012345678901212345 6
12
3456789012345678901234567890121234567890123456789012345678901212345 6
12
2
6
123456789012345678901234567890121234567890123456789012345678901212345
Directions: Each sentence below contains one or two blanks, with each blank
6
1 3456789012345678901234567890121234567890123456789012345678901212345
3456789012345678901234567890121234567890123456789012345678901212345
6
12
corresponding
to
an
omitted
word.
Find
the
answer
choice
that
has
the
word
or
set
or
3456789012345678901234567890121234567890123456789012345678901212345 6
12
3456789012345678901234567890121234567890123456789012345678901212345
6
12
words that best fits the meaning of the sentence as a whole.
3456789012345678901234567890121234567890123456789012345678901212345
6
12
3456789012345678901234567890121234567890123456789012345678901212345 6
12
3456789012345678901234567890121234567890123456789012345678901212345 6
12
3456789012345678901234567890121234567890123456789012345678901212345 6
12
3456789012345678901234567890121234567890123456789012345678901212345 6
12
2
6
123456789012345678901234567890121234567890123456789012345678901212345
18. The mayors unpopularity was dramati20. Normally the press secretary is fairly
6
123456789012345678901234567890121234567890123456789012345678901212345
6
123456789012345678901234567890121234567890123456789012345678901212345
cally demonstrated by his __________
__________, but in todays briefing her
6
123456789012345678901234567890121234567890123456789012345678901212345
6
1 3456789012345678901234567890121234567890123456789012345678901212345
performance in the citywide straw poll.
face was quite ___________ and her
2
3456789012345678901234567890121234567890123456789012345678901212345
123456789012345678901234567890121234567890123456789012345678901212345 66
123456789012345678901234567890121234567890123456789012345678901212345
responses lively.
A. sanguine
6
1
6
123456789012345678901234567890121234567890123456789012345678901212345
B.
dismal
A.
peppy.
.morose
3456789012345678901234567890121234567890123456789012345678901212345 6
12
2
3456789012345678901234567890121234567890123456789012345678901212345
6
123456789012345678901234567890121234567890123456789012345678901212345
C. fiery
B. subjugated. .active
6
123456789012345678901234567890121234567890123456789012345678901212345
6
1
D.
impoverished
C.
noxious.
.expressive
3456789012345678901234567890121234567890123456789012345678901212345 6
12
3456789012345678901234567890121234567890123456789012345678901212345
6
12
E. discouraging
D. concise. .perturbed
3456789012345678901234567890121234567890123456789012345678901212345
6
12
3456789012345678901234567890121234567890123456789012345678901212345
6
12
E.
subdued.
.animated
123456789012345678901234567890121234567890123456789012345678901212345 6
6
123456789012345678901234567890121234567890123456789012345678901212345
19. Country X will never stabilize its economy
6
123456789012345678901234567890121234567890123456789012345678901212345
2
3456789012345678901234567890121234567890123456789012345678901212345
123456789012345678901234567890121234567890123456789012345678901212345
if it does not ___________ its disastrous
21. A slight ___________ of the soils settling 6
6
123456789012345678901234567890121234567890123456789012345678901212345
6
123456789012345678901234567890121234567890123456789012345678901212345
inflationary spending practices.
rate caused the buildings foundation to
123456789012345678901234567890121234567890123456789012345678901212345 66
1
___________, which led to the building
A. exacerbate
3456789012345678901234567890121234567890123456789012345678901212345
6
12
3456789012345678901234567890121234567890123456789012345678901212345
6
being deemed unusable.
12
B. curtail
3456789012345678901234567890121234567890123456789012345678901212345
6
12
3456789012345678901234567890121234567890123456789012345678901212345
6
12
C.
strengthen
A.
estimation.
.settle
123456789012345678901234567890121234567890123456789012345678901212345 6
2
3456789012345678901234567890121234567890123456789012345678901212345
6
123456789012345678901234567890121234567890123456789012345678901212345
D. maximize
B. deviation. .bulge
6
123456789012345678901234567890121234567890123456789012345678901212345
6
123456789012345678901234567890121234567890123456789012345678901212345
E.
weaken
C.
miscalculation.
.shift
123456789012345678901234567890121234567890123456789012345678901212345 66
1
D. permutation. .stabilize
3456789012345678901234567890121234567890123456789012345678901212345
6
12
6
123456789012345678901234567890121234567890123456789012345678901212345
E.
escalation.
.liquefy
3456789012345678901234567890121234567890123456789012345678901212345 6
12
123456789012345678901234567890121234567890123456789012345678901212345 6
3456789012345678901234567890121234567890123456789012345678901212345 6
12
123456789012345678901234567890121234567890123456789012345678901212345 6
123456789012345678901234567890121234567890123456789012345678901212345 6
123456789012345678901234567890121234567890123456789012345678901212345 6
123456789012345678901234567890121234567890123456789012345678901212345 6
1234567890123456789012345678901212345678901234567890123456789012123456

www.petersons.com

http://www.xtremepapers.net

DIAGNOSTIC
1234567890123456789012345678901212345678901234567890123456789012123456
123456789012345678901234567890121234567890123456789012345678901212345 6
3456789012345678901234567890121234567890123456789012345678901212345 6
12
22. Prison reform in Victorian England,
23. For three centuries, classical mechanics
6
123456789012345678901234567890121234567890123456789012345678901212345
3456789012345678901234567890121234567890123456789012345678901212345
6
12
though ___________, was not far-reaching
was the __________ paradigm in physics
3456789012345678901234567890121234567890123456789012345678901212345
6
12
6
123456789012345678901234567890121234567890123456789012345678901212345
enough
to
eliminate
all
the
___________
until
developments
in
quantum
mechanics
3456789012345678901234567890121234567890123456789012345678901212345 6
12
2
3456789012345678901234567890121234567890123456789012345678901212345
6
1
vestiges of harsher times.
began to call __________ assumptions of
3456789012345678901234567890121234567890123456789012345678901212345
6
12
3456789012345678901234567890121234567890123456789012345678901212345
6
12
classical
mechanics
into
question.
3456789012345678901234567890121234567890123456789012345678901212345
6
12
A.
appreciable.
.miscellaneous
3456789012345678901234567890121234567890123456789012345678901212345 6
12
2
3456789012345678901234567890121234567890123456789012345678901212345
6
1
B. significant. .cruel
A. strongest. .tangential
3456789012345678901234567890121234567890123456789012345678901212345
6
12
3456789012345678901234567890121234567890123456789012345678901212345
6
12
C.
dogged.
.basic
B.
supreme.
.superficial
3456789012345678901234567890121234567890123456789012345678901212345 6
12
3456789012345678901234567890121234567890123456789012345678901212345
6
12
D. laudable. .benign
C. reigning. .fundamental
6
123456789012345678901234567890121234567890123456789012345678901212345
3456789012345678901234567890121234567890123456789012345678901212345
6
12
E.
wrongheaded.
.base
D.
secondary.
.basic
3456789012345678901234567890121234567890123456789012345678901212345 6
12
3456789012345678901234567890121234567890123456789012345678901212345
6
12
E. unchallenged. .facile
3456789012345678901234567890121234567890123456789012345678901212345
6
12
2
3456789012345678901234567890121234567890123456789012345678901212345
6
1
3456789012345678901234567890121234567890123456789012345678901212345 6
12
3456789012345678901234567890121234567890123456789012345678901212345 6
12
3456789012345678901234567890121234567890123456789012345678901212345
6
12
Directions: Questions follow each passage. After reading the passage, determine the best
3456789012345678901234567890121234567890123456789012345678901212345
6
12
2
3456789012345678901234567890121234567890123456789012345678901212345
6
123456789012345678901234567890121234567890123456789012345678901212345
answer for each question based on the passages content.
6
123456789012345678901234567890121234567890123456789012345678901212345
123456789012345678901234567890121234567890123456789012345678901212345 66
123456789012345678901234567890121234567890123456789012345678901212345 6
123456789012345678901234567890121234567890123456789012345678901212345 6
123456789012345678901234567890121234567890123456789012345678901212345 6
1 Passage 1
imperial family. Even when the Yamato
3456789012345678901234567890121234567890123456789012345678901212345
6
12
3456789012345678901234567890121234567890123456789012345678901212345
6
12
clan
had
nominally
laid
claim
to
the
title
3456789012345678901234567890121234567890123456789012345678901212345
6
12
Line
Though
the
founding
myth
of
the
2
3456789012345678901234567890121234567890123456789012345678901212345
6
123456789012345678901234567890121234567890123456789012345678901212345
of the imperial throne, their power was
6
123456789012345678901234567890121234567890123456789012345678901212345
Yamato
clan
sets
the
beginning
of
their
6
123456789012345678901234567890121234567890123456789012345678901212345
still
not
as
absolute
as
their
titular
6
123456789012345678901234567890121234567890123456789012345678901212345
dynasty
at
the
dawn
of
time,
the
Yamato
6
123456789012345678901234567890121234567890123456789012345678901212345
(30)
pretensions
suggested.
The
aristocratic
6
123456789012345678901234567890121234567890123456789012345678901212345
clan did not unify Japan under their
6
123456789012345678901234567890121234567890123456789012345678901212345
clan
chieftains
still
maintained
significant
6
123456789012345678901234567890121234567890123456789012345678901212345
(5)
monarchical
rule
until
the
sixth
century
6
123456789012345678901234567890121234567890123456789012345678901212345
power
bases
in
the
regions
historically
6
1
B.C.E. It is difficult to designate a specific
2
3456789012345678901234567890121234567890123456789012345678901212345
6
123456789012345678901234567890121234567890123456789012345678901212345
linked with their clans.
6
123456789012345678901234567890121234567890123456789012345678901212345
date
for
the
beginning
of
the
Yamato
6
1
This
scenario
of
power
diffusion
6
123456789012345678901234567890121234567890123456789012345678901212345
monarchy
because
the
process
of
political
3456789012345678901234567890121234567890123456789012345678901212345
6
12
(35)
under
the
political
rubric
of
monarchy
is
2
3456789012345678901234567890121234567890123456789012345678901212345
6
123456789012345678901234567890121234567890123456789012345678901212345
evolution in ancient Japan was not
6
123456789012345678901234567890121234567890123456789012345678901212345
not
peculiar
to
ancient
Japan.
In
fact,
it
is
6
1 (10) marked by clearly defined stages. Rather,
3456789012345678901234567890121234567890123456789012345678901212345
6
12
a
common
theme
in
the
narrative
of
3456789012345678901234567890121234567890123456789012345678901212345
6
12
the political evolution from aristocratic
2
3456789012345678901234567890121234567890123456789012345678901212345
123456789012345678901234567890121234567890123456789012345678901212345
kingdom formation in Medieval Europe. 6
6
123456789012345678901234567890121234567890123456789012345678901212345
rule centered on a number of clan
6
1
Many monarchs, with the notable
families; the monarchical rule dominated
6
123456789012345678901234567890121234567890123456789012345678901212345
6
123456789012345678901234567890121234567890123456789012345678901212345
(40)
exception of Charlemagne, had great
by
one
family
was
a
gradual
process.
2
3456789012345678901234567890121234567890123456789012345678901212345
123456789012345678901234567890121234567890123456789012345678901212345 66
123456789012345678901234567890121234567890123456789012345678901212345
difficulty consolidating their kingdoms
(15)
The slow pace of the transformation
6
123456789012345678901234567890121234567890123456789012345678901212345
resources under their authority given the 6
123456789012345678901234567890121234567890123456789012345678901212345
was
mirrored
by
the
means
employed
to
6
1
prominent and powerful positions of
3456789012345678901234567890121234567890123456789012345678901212345
6
12
achieve the result. The Yamato clan did
3456789012345678901234567890121234567890123456789012345678901212345
6
12
dukes, lords, and wealthy merchants that 6
3456789012345678901234567890121234567890123456789012345678901212345
12
not
militarily
subjugate
the
competing
3456789012345678901234567890121234567890123456789012345678901212345
6
12
(45)
populated their lands.
6
123456789012345678901234567890121234567890123456789012345678901212345
field
of
clans,
but
instead
gained
ascen2
3456789012345678901234567890121234567890123456789012345678901212345
123456789012345678901234567890121234567890123456789012345678901212345 66
123456789012345678901234567890121234567890123456789012345678901212345
(20)
dancy through political, economic, and
6
123456789012345678901234567890121234567890123456789012345678901212345
6
123456789012345678901234567890121234567890123456789012345678901212345
diplomatic
means.
Since
such
means
6
1
2
3456789012345678901234567890121234567890123456789012345678901212345
6
123456789012345678901234567890121234567890123456789012345678901212345
generally preclude decisive shifts in
6
1
3456789012345678901234567890121234567890123456789012345678901212345
6
12
power,
the
Yamato
clan
had
to
patiently
123456789012345678901234567890121234567890123456789012345678901212345 6
3456789012345678901234567890121234567890123456789012345678901212345
6
12
leverage their advantages until they were
6
123456789012345678901234567890121234567890123456789012345678901212345
6
123456789012345678901234567890121234567890123456789012345678901212345
(25)
able
to
demand
recognition
as
the
123456789012345678901234567890121234567890123456789012345678901212345 66
123456789012345678901234567890121234567890123456789012345678901212345
1234567890123456789012345678901212345678901234567890123456789012123456 43

http://www.xtremepapers.net

Part II: Diagnosing Strengths and Weaknesses

44

1234567890123456789012345678901212345678901234567890123456789012123456
123456789012345678901234567890121234567890123456789012345678901212345 6
3456789012345678901234567890121234567890123456789012345678901212345 6
12
24. In line 34, the word diffusion could most
6
Passage 2
123456789012345678901234567890121234567890123456789012345678901212345
3456789012345678901234567890121234567890123456789012345678901212345
6
12
reasonably be replaced by which of the
3456789012345678901234567890121234567890123456789012345678901212345
6
12
Line
Grasping
the
interrelationship
between
6
123456789012345678901234567890121234567890123456789012345678901212345
following?
3456789012345678901234567890121234567890123456789012345678901212345
6
12
structure
and
function
is
essential
to
2
3456789012345678901234567890121234567890123456789012345678901212345
6
1
3456789012345678901234567890121234567890123456789012345678901212345
6
12
A.
Assimilation
gaining
a
basic
understanding
of
molecu3456789012345678901234567890121234567890123456789012345678901212345 6
12
3456789012345678901234567890121234567890123456789012345678901212345
6
12
B. Sharing
lar biology. All biological entities are
3456789012345678901234567890121234567890123456789012345678901212345
6
12
2
3456789012345678901234567890121234567890123456789012345678901212345
6
1
(5)
C. Hegemony
composed of molecules, and every
3456789012345678901234567890121234567890123456789012345678901212345
6
12
3456789012345678901234567890121234567890123456789012345678901212345
6
12
D.
Mongering
molecule
has
a
specific
shape.
A
hydro3456789012345678901234567890121234567890123456789012345678901212345 6
12
3456789012345678901234567890121234567890123456789012345678901212345
6
12
E. Imbalances
gen molecule, for instance, has a simple
6
123456789012345678901234567890121234567890123456789012345678901212345
3456789012345678901234567890121234567890123456789012345678901212345
6
12
linear
shape,
while
DNA,
the
molecule
3456789012345678901234567890121234567890123456789012345678901212345 6
12
3456789012345678901234567890121234567890123456789012345678901212345
6
25. What is the main point of the third
12
that stores most of a cells genetic
3456789012345678901234567890121234567890123456789012345678901212345
6
12
paragraph?
2
3456789012345678901234567890121234567890123456789012345678901212345
1
(10)
information, has a complex double helix 6
3456789012345678901234567890121234567890123456789012345678901212345
6
12
3456789012345678901234567890121234567890123456789012345678901212345
6
12
shape. The shape of a molecule, comA.
Ancient
Japans
political
evolution
3456789012345678901234567890121234567890123456789012345678901212345 6
12
3456789012345678901234567890121234567890123456789012345678901212345
6
12
bined with the configuration of its
was dissimilar to Medieval European
3456789012345678901234567890121234567890123456789012345678901212345
6
12
3456789012345678901234567890121234567890123456789012345678901212345
6
12
valence
electrons
(the
electrons
available
political
evolution.
3456789012345678901234567890121234567890123456789012345678901212345 6
12
for bonding with other molecules),
3456789012345678901234567890121234567890123456789012345678901212345
6
12
B. Charlemagne was a powerful mon3456789012345678901234567890121234567890123456789012345678901212345
6
12
(15)
determine
the
structure
of
a
molecule.
2
3456789012345678901234567890121234567890123456789012345678901212345
6
123456789012345678901234567890121234567890123456789012345678901212345
arch comparable to the great emper6
1
Since this structure determines the
3456789012345678901234567890121234567890123456789012345678901212345
6
12
ors
of
ancient
Japan.
3456789012345678901234567890121234567890123456789012345678901212345
6
12
biological
functionality
of
the
molecule,
3456789012345678901234567890121234567890123456789012345678901212345
6
12
C.
Both
Medieval
European
monarchs
and
2
3456789012345678901234567890121234567890123456789012345678901212345
6
123456789012345678901234567890121234567890123456789012345678901212345
it
is
of
paramount
importance.
6
123456789012345678901234567890121234567890123456789012345678901212345
Japanese
emperors
had
difficulties
in
6
123456789012345678901234567890121234567890123456789012345678901212345
The
chemistry
of
the
brain
illustrates
6
123456789012345678901234567890121234567890123456789012345678901212345
unifying the lands under their power.
6
123456789012345678901234567890121234567890123456789012345678901212345
(20)
the
interaction
between
molecular
6
123456789012345678901234567890121234567890123456789012345678901212345
D. Dukes, lords, wealthy merchants, and
6
123456789012345678901234567890121234567890123456789012345678901212345
structure
and
biological
function.
One
6
123456789012345678901234567890121234567890123456789012345678901212345
clan chieftains were politically obtuse
6
123456789012345678901234567890121234567890123456789012345678901212345
nerve
cell
in
the
brain
communicates
with
6
1
throughout the world.
2
3456789012345678901234567890121234567890123456789012345678901212345
6
123456789012345678901234567890121234567890123456789012345678901212345
another nerve cell by the emission and
6
E. Clan chieftains of ancient Japan
123456789012345678901234567890121234567890123456789012345678901212345
6
1
uptake
of
molecular
signals.
A
transmitresembled wealthy merchants in
6
123456789012345678901234567890121234567890123456789012345678901212345
3456789012345678901234567890121234567890123456789012345678901212345
6
12
(25)
ting
nerve
cell
initiates
the
communicaMedieval
Europe
with
respect
to
the
2
3456789012345678901234567890121234567890123456789012345678901212345
123456789012345678901234567890121234567890123456789012345678901212345 66
123456789012345678901234567890121234567890123456789012345678901212345
tion with a second nerve cell known as a 6
political role each played in their
1
3456789012345678901234567890121234567890123456789012345678901212345
6
12
receiving cell. The transmitting cell
respective eras and locales.
3456789012345678901234567890121234567890123456789012345678901212345
6
12
2
3456789012345678901234567890121234567890123456789012345678901212345
6
123456789012345678901234567890121234567890123456789012345678901212345
releases signal molecules that are
6
123456789012345678901234567890121234567890123456789012345678901212345
6
1 26. What is the authors attitude toward the
structured specifically to bond with
123456789012345678901234567890121234567890123456789012345678901212345 6
6
123456789012345678901234567890121234567890123456789012345678901212345
(30)
receptor molecules located at specific
notion of absolute monarchical rule in
3456789012345678901234567890121234567890123456789012345678901212345
6
12
sites
on
the
receiving
cell.
The
signal
and
3456789012345678901234567890121234567890123456789012345678901212345
6
12
ancient
Japan
and
Medieval
Europe?
3456789012345678901234567890121234567890123456789012345678901212345
6
12
receptor molecules have complementary 6
3456789012345678901234567890121234567890123456789012345678901212345
12
6
123456789012345678901234567890121234567890123456789012345678901212345
A. Harsh criticism of the weak monarchs
molecular structures which allow for
3456789012345678901234567890121234567890123456789012345678901212345
6
12
B. Measured skepticism about the
3456789012345678901234567890121234567890123456789012345678901212345
6
12
their bonding ability. You can say the
3456789012345678901234567890121234567890123456789012345678901212345
6
12
absoluteness of the monarchs rule
3456789012345678901234567890121234567890123456789012345678901212345
6
12
(35)
signal
molecule
functions
like
a
key
while
6
123456789012345678901234567890121234567890123456789012345678901212345
C.
Strenuous
objection
to
the
monarchs
2
3456789012345678901234567890121234567890123456789012345678901212345
123456789012345678901234567890121234567890123456789012345678901212345
the receptor molecule is the lock. In this 6
6
123456789012345678901234567890121234567890123456789012345678901212345
political practices
6
123456789012345678901234567890121234567890123456789012345678901212345
analogy,
the
molecular
structure
can
be
6
123456789012345678901234567890121234567890123456789012345678901212345
D.
Moderate
praise
of
the
monarchs
6
1
thought
of
as
the
nodes
on
the
key
and
2
3456789012345678901234567890121234567890123456789012345678901212345
6
123456789012345678901234567890121234567890123456789012345678901212345
restraint
6
1
within
the
lock
that
complement
each
3456789012345678901234567890121234567890123456789012345678901212345
6
12
E.
Fulsome
praise
of
the
monarchs
6
123456789012345678901234567890121234567890123456789012345678901212345
(40)
other
and
allow
for
the
key
to
turn
the
3456789012345678901234567890121234567890123456789012345678901212345
6
12
policies
2
3456789012345678901234567890121234567890123456789012345678901212345
6
1
lock. The bonding of the signal and
6
123456789012345678901234567890121234567890123456789012345678901212345
123456789012345678901234567890121234567890123456789012345678901212345 66
123456789012345678901234567890121234567890123456789012345678901212345
1234567890123456789012345678901212345678901234567890123456789012123456

www.petersons.com

http://www.xtremepapers.net

DIAGNOSTIC
1234567890123456789012345678901212345678901234567890123456789012123456
123456789012345678901234567890121234567890123456789012345678901212345 6
3456789012345678901234567890121234567890123456789012345678901212345 6
12
receptor molecules sets in motion a series
6
29. What is the function of the second
123456789012345678901234567890121234567890123456789012345678901212345
3456789012345678901234567890121234567890123456789012345678901212345
6
12
of chemical reactions within the receiving
paragraph in relation to the passage as a
3456789012345678901234567890121234567890123456789012345678901212345
6
12
6
123456789012345678901234567890121234567890123456789012345678901212345
cell.
Thus,
through
the
biological
whole?
3456789012345678901234567890121234567890123456789012345678901212345 6
12
2
3456789012345678901234567890121234567890123456789012345678901212345
6
1 (45) mechanism of complementary molecular
3456789012345678901234567890121234567890123456789012345678901212345
6
12
A.
It
relates
an
anecdote
that
demon3456789012345678901234567890121234567890123456789012345678901212345
6
12
structure,
one
brain
cell
is
able
to
3456789012345678901234567890121234567890123456789012345678901212345
6
12
strates
the
principle
described
in
the
3456789012345678901234567890121234567890123456789012345678901212345
6
12
communicate
and
affect
another
2
3456789012345678901234567890121234567890123456789012345678901212345
6
1
first paragraph.
3456789012345678901234567890121234567890123456789012345678901212345
6
12
brain
cell.
3456789012345678901234567890121234567890123456789012345678901212345
6
12
B.
It
gives
further
explanation
to
the
3456789012345678901234567890121234567890123456789012345678901212345 6
12
3456789012345678901234567890121234567890123456789012345678901212345
6
12
theoretical hypothesis made in the
6
123456789012345678901234567890121234567890123456789012345678901212345
3456789012345678901234567890121234567890123456789012345678901212345
6
12
27. The primary purpose of this passage is to
first
paragraph.
3456789012345678901234567890121234567890123456789012345678901212345 6
12
3456789012345678901234567890121234567890123456789012345678901212345
12
C. It provides an example that illustrates 6
A. demonstrate the importance of
3456789012345678901234567890121234567890123456789012345678901212345
6
12
2
3456789012345678901234567890121234567890123456789012345678901212345
6
1
the relationship discussed in the first
biological functionality regarding
3456789012345678901234567890121234567890123456789012345678901212345
6
12
3456789012345678901234567890121234567890123456789012345678901212345
6
12
paragraph.
brain
functioning
capacities.
3456789012345678901234567890121234567890123456789012345678901212345 6
12
3456789012345678901234567890121234567890123456789012345678901212345
12
D. It presents a second viewpoint on the 6
B. criticize the contemporary theory that
3456789012345678901234567890121234567890123456789012345678901212345
6
12
3456789012345678901234567890121234567890123456789012345678901212345
6
12
issues raised in the first paragraph.
molecular
structure
and
biological
3456789012345678901234567890121234567890123456789012345678901212345 6
12
3456789012345678901234567890121234567890123456789012345678901212345
E. It debunks a common misconception 6
12
functionality are interrelated.
3456789012345678901234567890121234567890123456789012345678901212345
6
12
about the principle discussed in the
2
3456789012345678901234567890121234567890123456789012345678901212345
6
123456789012345678901234567890121234567890123456789012345678901212345
C. present competing views on the nature
6
1
first paragraph.
3456789012345678901234567890121234567890123456789012345678901212345
6
12
of
the
interrelationship
between
brain
3456789012345678901234567890121234567890123456789012345678901212345 6
12
3456789012345678901234567890121234567890123456789012345678901212345
6
12
chemistry and molecular structure.
3456789012345678901234567890121234567890123456789012345678901212345
6
12
30. Based on the passage, which of the
3456789012345678901234567890121234567890123456789012345678901212345
6
12
D.
explain
the
contemporary
theory
of
3456789012345678901234567890121234567890123456789012345678901212345
6
12
following
statements
would
the
author
3456789012345678901234567890121234567890123456789012345678901212345
6
12
the
interrelationship
between
molecu3456789012345678901234567890121234567890123456789012345678901212345
6
12
most
likely
agree
with?
2
3456789012345678901234567890121234567890123456789012345678901212345
6
123456789012345678901234567890121234567890123456789012345678901212345
lar structure and biological function6
123456789012345678901234567890121234567890123456789012345678901212345
6
123456789012345678901234567890121234567890123456789012345678901212345
ality.
A. The study of molecular functionality
123456789012345678901234567890121234567890123456789012345678901212345 66
1
E. argue that molecular structure has
is an important field in molecular
3456789012345678901234567890121234567890123456789012345678901212345
6
12
3456789012345678901234567890121234567890123456789012345678901212345
6
greater explanatory force than does
12
chemistry.
6
123456789012345678901234567890121234567890123456789012345678901212345
biological functionality.
B. The advancement of understanding in 6
123456789012345678901234567890121234567890123456789012345678901212345
3456789012345678901234567890121234567890123456789012345678901212345
6
12
molecular biology is contingent upon 6
3456789012345678901234567890121234567890123456789012345678901212345
12
3456789012345678901234567890121234567890123456789012345678901212345
6
12
28. Why does the author mention a hydrogen
further research in how molecules are 6
123456789012345678901234567890121234567890123456789012345678901212345
3456789012345678901234567890121234567890123456789012345678901212345
6
12
molecule and DNA in lines 611 in the
structured.
3456789012345678901234567890121234567890123456789012345678901212345
6
12
2
3456789012345678901234567890121234567890123456789012345678901212345
123456789012345678901234567890121234567890123456789012345678901212345
first paragraph?
C. Brain chemistry will not ultimately be 6
6
123456789012345678901234567890121234567890123456789012345678901212345
6
1
understood
until
neurotransmitters
6
123456789012345678901234567890121234567890123456789012345678901212345
A. To contrast a simply shaped molecule
6
123456789012345678901234567890121234567890123456789012345678901212345
are
fully
conceptualized.
2
3456789012345678901234567890121234567890123456789012345678901212345
6
123456789012345678901234567890121234567890123456789012345678901212345
with a complexly shaped molecule
6
123456789012345678901234567890121234567890123456789012345678901212345
D.
Biological
functionality
is
not
6
123456789012345678901234567890121234567890123456789012345678901212345
B. To demonstrate that DNA is similarly
6
123456789012345678901234567890121234567890123456789012345678901212345
dependent
on
molecular
shape.
6
1
shaped to hydrogen
2
3456789012345678901234567890121234567890123456789012345678901212345
6
123456789012345678901234567890121234567890123456789012345678901212345
E. DNA is the most complex molecule
C. To illustrate that biological function6
123456789012345678901234567890121234567890123456789012345678901212345
6
123456789012345678901234567890121234567890123456789012345678901212345
known
to
biology.
ality is not directly related to molecu6
123456789012345678901234567890121234567890123456789012345678901212345
6
1
lar
shape
2
3456789012345678901234567890121234567890123456789012345678901212345
123456789012345678901234567890121234567890123456789012345678901212345 66
123456789012345678901234567890121234567890123456789012345678901212345
D. To give examples of the molecular
6
123456789012345678901234567890121234567890123456789012345678901212345
6
123456789012345678901234567890121234567890123456789012345678901212345
structure
of
two
different
molecules
6
1
2
3456789012345678901234567890121234567890123456789012345678901212345
6
123456789012345678901234567890121234567890123456789012345678901212345
E. To undermine the assertion that
6
1
3456789012345678901234567890121234567890123456789012345678901212345
6
12
hydrogen
is
more
simply
structured
123456789012345678901234567890121234567890123456789012345678901212345 6
3456789012345678901234567890121234567890123456789012345678901212345
6
12
than DNA
6
123456789012345678901234567890121234567890123456789012345678901212345
123456789012345678901234567890121234567890123456789012345678901212345 6
123456789012345678901234567890121234567890123456789012345678901212345 6
123456789012345678901234567890121234567890123456789012345678901212345 6 45
1234567890123456789012345678901212345678901234567890123456789012123456

http://www.xtremepapers.net

Part II: Diagnosing Strengths and Weaknesses

46

1234567890123456789012345678901212345678901234567890123456789012123456
123456789012345678901234567890121234567890123456789012345678901212345 6
3456789012345678901234567890121234567890123456789012345678901212345 6
12
123456789012345678901234567890121234567890123456789012345678901212345 6
3456789012345678901234567890121234567890123456789012345678901212345
6
12
Answer Key
3456789012345678901234567890121234567890123456789012345678901212345
6
12
6
123456789012345678901234567890121234567890123456789012345678901212345
Quantitative
Verbal
3456789012345678901234567890121234567890123456789012345678901212345 6
12
2
3456789012345678901234567890121234567890123456789012345678901212345
6
1
3456789012345678901234567890121234567890123456789012345678901212345 6
12
3456789012345678901234567890121234567890123456789012345678901212345
6
12
1. B
15. D
1. E
16. D
3456789012345678901234567890121234567890123456789012345678901212345
6
12
3456789012345678901234567890121234567890123456789012345678901212345
6
12
2. A
16. E
2. A
17. B
2
3456789012345678901234567890121234567890123456789012345678901212345
6
1
3. A
17. B
3. E
18. B
3456789012345678901234567890121234567890123456789012345678901212345
6
12
3456789012345678901234567890121234567890123456789012345678901212345
6
12
4. C
18. E
4. A
19. B
3456789012345678901234567890121234567890123456789012345678901212345
6
12
3456789012345678901234567890121234567890123456789012345678901212345
6
12
5.
C
19.
D
5.
B
20.
E
2
3456789012345678901234567890121234567890123456789012345678901212345
6
1
3456789012345678901234567890121234567890123456789012345678901212345
6
12
6. D
20. E
6. A
21. C
3456789012345678901234567890121234567890123456789012345678901212345
6
12
3456789012345678901234567890121234567890123456789012345678901212345
6
12
7.
B
21.
D
7.
D
22.
B
3456789012345678901234567890121234567890123456789012345678901212345 6
12
2
3456789012345678901234567890121234567890123456789012345678901212345
6
1
8. D
22. D
8. C
23. C
3456789012345678901234567890121234567890123456789012345678901212345
6
12
3456789012345678901234567890121234567890123456789012345678901212345
6
12
9.
D
23.
C
9.
D
24.
B
3456789012345678901234567890121234567890123456789012345678901212345 6
12
3456789012345678901234567890121234567890123456789012345678901212345
6
12
10. C
24. C
10. C
25. C
3456789012345678901234567890121234567890123456789012345678901212345
6
12
3456789012345678901234567890121234567890123456789012345678901212345
6
12
11.
A
25.
A
11.
B
26.
B
3456789012345678901234567890121234567890123456789012345678901212345 6
12
3456789012345678901234567890121234567890123456789012345678901212345
6
12
12. B
26. E
12. A
27. D
3456789012345678901234567890121234567890123456789012345678901212345
6
12
2
3456789012345678901234567890121234567890123456789012345678901212345
6
123456789012345678901234567890121234567890123456789012345678901212345
13. D
27. A
13. E
28. A
6
1
3456789012345678901234567890121234567890123456789012345678901212345
6
12
14.
A
28.
B
14.
B
29.
C
3456789012345678901234567890121234567890123456789012345678901212345 6
12
3456789012345678901234567890121234567890123456789012345678901212345
6
12
15. C
30. B
3456789012345678901234567890121234567890123456789012345678901212345
6
12
3456789012345678901234567890121234567890123456789012345678901212345 6
12
3456789012345678901234567890121234567890123456789012345678901212345 6
12
3456789012345678901234567890121234567890123456789012345678901212345 6
12
3456789012345678901234567890121234567890123456789012345678901212345 6
12
2
6
123456789012345678901234567890121234567890123456789012345678901212345
6
123456789012345678901234567890121234567890123456789012345678901212345
6
123456789012345678901234567890121234567890123456789012345678901212345
6
123456789012345678901234567890121234567890123456789012345678901212345
1 3456789012345678901234567890121234567890123456789012345678901212345 6
2
3456789012345678901234567890121234567890123456789012345678901212345
123456789012345678901234567890121234567890123456789012345678901212345 66
123456789012345678901234567890121234567890123456789012345678901212345 6
1
123456789012345678901234567890121234567890123456789012345678901212345 6
3456789012345678901234567890121234567890123456789012345678901212345 6
12
3456789012345678901234567890121234567890123456789012345678901212345 6
12
3456789012345678901234567890121234567890123456789012345678901212345 6
12
123456789012345678901234567890121234567890123456789012345678901212345 6
3456789012345678901234567890121234567890123456789012345678901212345 6
12
3456789012345678901234567890121234567890123456789012345678901212345 6
12
2
6
123456789012345678901234567890121234567890123456789012345678901212345
6
123456789012345678901234567890121234567890123456789012345678901212345
1 3456789012345678901234567890121234567890123456789012345678901212345 6
123456789012345678901234567890121234567890123456789012345678901212345 6
123456789012345678901234567890121234567890123456789012345678901212345 6
3456789012345678901234567890121234567890123456789012345678901212345 6
12
3456789012345678901234567890121234567890123456789012345678901212345 6
12
3456789012345678901234567890121234567890123456789012345678901212345 6
12
3456789012345678901234567890121234567890123456789012345678901212345 6
12
123456789012345678901234567890121234567890123456789012345678901212345 6
2
6
123456789012345678901234567890121234567890123456789012345678901212345
6
123456789012345678901234567890121234567890123456789012345678901212345
6
123456789012345678901234567890121234567890123456789012345678901212345
6
123456789012345678901234567890121234567890123456789012345678901212345
1 3456789012345678901234567890121234567890123456789012345678901212345 6
2
3456789012345678901234567890121234567890123456789012345678901212345
123456789012345678901234567890121234567890123456789012345678901212345 66
123456789012345678901234567890121234567890123456789012345678901212345 6
123456789012345678901234567890121234567890123456789012345678901212345 6
123456789012345678901234567890121234567890123456789012345678901212345 6
1
3456789012345678901234567890121234567890123456789012345678901212345 6
12
123456789012345678901234567890121234567890123456789012345678901212345 6
3456789012345678901234567890121234567890123456789012345678901212345 6
12
123456789012345678901234567890121234567890123456789012345678901212345 6
3456789012345678901234567890121234567890123456789012345678901212345 6
12
123456789012345678901234567890121234567890123456789012345678901212345 6
123456789012345678901234567890121234567890123456789012345678901212345 6
123456789012345678901234567890121234567890123456789012345678901212345 6
123456789012345678901234567890121234567890123456789012345678901212345 6
1234567890123456789012345678901212345678901234567890123456789012123456

www.petersons.com

http://www.xtremepapers.net

PART

III
Section Review
Quantitative Review
Verbal Review

48

134

Analytical Writing Review

227

PART III
http://www.xtremepapers.net

Chapter

4
Quantitative Review
123456789012345678901234567890121234567890123456789012
2
12345678901234567890123456789012123456789012345678901
2
12345678901234567890123456789012123456789012345678901
2
12345678901234567890123456789012123456789012345678901
All
the
Math
You
Left
Behind
2345678901234567890123456789012123456789012345678901
12345678901234567890123456789012123456789012345678901 2
12345678901234567890123456789012123456789012345678901
For those who have not seen a triangle since high school geometry, its 2
2
12345678901234567890123456789012123456789012345678901
2
12345678901234567890123456789012123456789012345678901
scary
to
think
that
your
graduate
school
aspirations
partially
rest
on
your
12345678901234567890123456789012123456789012345678901 2
2
12345678901234567890123456789012123456789012345678901
1 ability to work with the math concepts of your youth. Those of you who 2
12345678901234567890123456789012123456789012345678901
studied math extensively in college arent off the hook either, since the 2
2
12345678901234567890123456789012123456789012345678901
12345678901234567890123456789012123456789012345678901
concepts tested on the GRE are not the ones studied by math majors or 2
2345678901234567890123456789012123456789012345678901
12345678901234567890123456789012123456789012345678901 2
12345678901234567890123456789012123456789012345678901
engineering students. Brainiacs who are used to working with orthogonal 2
2
12345678901234567890123456789012123456789012345678901
2
12345678901234567890123456789012123456789012345678901
matrices might not remember dividing with remainders.
12345678901234567890123456789012123456789012345678901 2
2
12345678901234567890123456789012123456789012345678901
Thats the half-empty news. The half-full news is that the GRE math 2
12345678901234567890123456789012123456789012345678901
2
12345678901234567890123456789012123456789012345678901
section tests a limited number of math concepts, so the math content of the 2
12345678901234567890123456789012123456789012345678901
1 GRE can be mastered. With a little bitokay, for some of you, a lotof 2
2
12345678901234567890123456789012123456789012345678901
12345678901234567890123456789012123456789012345678901
studying, you can be confident that you will not be tested on an unfamiliar 2
2
12345678901234567890123456789012123456789012345678901
2
12345678901234567890123456789012123456789012345678901
math
concept.
The
reason
for
this
comes
partly
from
the
fact
that
the
2
12345678901234567890123456789012123456789012345678901
2345678901234567890123456789012123456789012345678901
12345678901234567890123456789012123456789012345678901
material covered in the GRE math section is not very difficult. Instead, the 2
2
12345678901234567890123456789012123456789012345678901
1 difficulty comes when the test demands you apply this basic set of GRE 2
2
12345678901234567890123456789012123456789012345678901
2
12345678901234567890123456789012123456789012345678901
math
knowledge
in
creative
ways
to
solve
problems.
2345678901234567890123456789012123456789012345678901
12345678901234567890123456789012123456789012345678901 2
12345678901234567890123456789012123456789012345678901 2
1 Mastering the basic math knowledge of the GRE becomes the first goal, so 2
12345678901234567890123456789012123456789012345678901 2
12345678901234567890123456789012123456789012345678901
this is what well focus on for now (without such mastery, the math will 2
2
12345678901234567890123456789012123456789012345678901
12345678901234567890123456789012123456789012345678901
indeed be difficult). Once this is done, the second step is learning to 2
2
12345678901234567890123456789012123456789012345678901
12345678901234567890123456789012123456789012345678901
recognize how and when to apply the basic math knowledge on the GRE. 2
2
12345678901234567890123456789012123456789012345678901
This will take more time than the first part, but you will have more than 2
12345678901234567890123456789012123456789012345678901
2
12345678901234567890123456789012123456789012345678901
enough questions to practice on before you have to exercise this skill on 2
12345678901234567890123456789012123456789012345678901
2
12345678901234567890123456789012123456789012345678901
test day.
2
12345678901234567890123456789012123456789012345678901
2345678901234567890123456789012123456789012345678901
12345678901234567890123456789012123456789012345678901 2
12345678901234567890123456789012123456789012345678901
On test day, you can expect to see a lot of math jargon in the Quantitative 2
2
12345678901234567890123456789012123456789012345678901
2
12345678901234567890123456789012123456789012345678901
section.
But
dont
fear;
you
dont
have
to
be
intimidated
by
math
speak.
In
2
1
2345678901234567890123456789012123456789012345678901
2
12345678901234567890123456789012123456789012345678901
1 this section, we will define all the terms that you need to know for the test, 2
12345678901234567890123456789012123456789012345678901
and we will work with them so that you can get comfortable with phrases 2
2
12345678901234567890123456789012123456789012345678901
2
12345678901234567890123456789012123456789012345678901
like
positive
even
integers
divisible
by
4.
As
you
learn
(or
refresh
your
2345678901234567890123456789012123456789012345678901
2
1
2
12345678901234567890123456789012123456789012345678901
memory
about)
this
math
jargon,
remember
that
learning
a
math
term
is
12345678901234567890123456789012123456789012345678901 2
12345678901234567890123456789012123456789012345678901 2
123456789012345678901234567890121234567890123456789012
48

http://www.xtremepapers.net

Chapter 4: Quantitative Review

Alert!

123456789012345678901234567890121234567890123456789012
12345678901234567890123456789012123456789012345678901 2
2
12345678901234567890123456789012123456789012345678901
just like learning a new word. Dont let the jargon-ness (jargonosity?) of it 2
12345678901234567890123456789012123456789012345678901
2
12345678901234567890123456789012123456789012345678901
throw you. A math term simply refers to a specific math notion, just like 2
12345678901234567890123456789012123456789012345678901
2
12345678901234567890123456789012123456789012345678901
the word chair refers to a specific idea we have of furniture you sit on. 2
12345678901234567890123456789012123456789012345678901
2345678901234567890123456789012123456789012345678901
1 When a math term is introduced, read the definition, consider the 2
2
12345678901234567890123456789012123456789012345678901
2
12345678901234567890123456789012123456789012345678901
example,
and
think
of
it
as
your
new
word
for
the
day.
2
12345678901234567890123456789012123456789012345678901
2
12345678901234567890123456789012123456789012345678901
2345678901234567890123456789012123456789012345678901
1 So, dont let the GRE math section bully you, or your feelings, around. It 2
2
12345678901234567890123456789012123456789012345678901
2
12345678901234567890123456789012123456789012345678901
can
be
conquered
with
a
little
effort.
Roll
up
your
sleeves,
get
to
work,
and
2
12345678901234567890123456789012123456789012345678901
2
12345678901234567890123456789012123456789012345678901
comfort
yourself
with
the
thought
that
once
you
finish
the
math
section
on
2345678901234567890123456789012123456789012345678901
2
1
2
12345678901234567890123456789012123456789012345678901
the
GRE,
you
will
probably
never
have
to
be
concerned
about
the
area
of
a
2
12345678901234567890123456789012123456789012345678901
2
12345678901234567890123456789012123456789012345678901
parallelogram
again
(unless
you
decide
to
go
to
business
school
and
need
2
12345678901234567890123456789012123456789012345678901
2345678901234567890123456789012123456789012345678901
2
1 to take the GMAT, that is).
2
12345678901234567890123456789012123456789012345678901
2
12345678901234567890123456789012123456789012345678901
2
12345678901234567890123456789012123456789012345678901
2
12345678901234567890123456789012123456789012345678901
2345678901234567890123456789012123456789012345678901
2
12345678901234567890123456789012123456789012345678901
Arithmetic,
or Fun with Numbers
2
12345678901234567890123456789012123456789012345678901
12345678901234567890123456789012123456789012345678901 22
12345678901234567890123456789012123456789012345678901
Most people hear the word arithmetic and think add-subtract-multiply2
12345678901234567890123456789012123456789012345678901
2
12345678901234567890123456789012123456789012345678901
1 divide, but arithmetic actually covers many basic math concepts and 2
terms. GRE math loves to pile one math topic on top of another, so theres 2
12345678901234567890123456789012123456789012345678901
2
12345678901234567890123456789012123456789012345678901
a lot of arithmetic in the Quant section. Take your time and dont just skim 2
12345678901234567890123456789012123456789012345678901
2345678901234567890123456789012123456789012345678901
2
12345678901234567890123456789012123456789012345678901
over this portion to get to a juicier topic like Geometry, since Arithmetic 2
12345678901234567890123456789012123456789012345678901
2
12345678901234567890123456789012123456789012345678901
covers the fundamental terms that will be prevalent on almost every 2
12345678901234567890123456789012123456789012345678901
2
12345678901234567890123456789012123456789012345678901
question, in one form or another.
2
12345678901234567890123456789012123456789012345678901
12345678901234567890123456789012123456789012345678901 22
12345678901234567890123456789012123456789012345678901 2
12345678901234567890123456789012123456789012345678901 2
1
2
12345678901234567890123456789012123456789012345678901
Integers, or Pretty Much Everything Thats Not
2
12345678901234567890123456789012123456789012345678901
2
12345678901234567890123456789012123456789012345678901
a
Fraction
12345678901234567890123456789012123456789012345678901 2
12345678901234567890123456789012123456789012345678901
Integers are the numbers that you count with (1, 2, 3 and so on). They can 2
2
12345678901234567890123456789012123456789012345678901
2
12345678901234567890123456789012123456789012345678901
also
be
negative
(21,
22,
23
.
.
.).
Zero
is
also
an
integer.
The
numbers
12345678901234567890123456789012123456789012345678901 2
12345678901234567890123456789012123456789012345678901
23, 22, 21, 0, 1, 2, and 3 are all integers. Pretty much any whole number 2
2
12345678901234567890123456789012123456789012345678901
2345678901234567890123456789012123456789012345678901
12345678901234567890123456789012123456789012345678901
you can think ofany number that is not a fraction or a decimalis an 2
2
12345678901234567890123456789012123456789012345678901
1 integer, and if you put a negative in front of that number, it also is an 2
12345678901234567890123456789012123456789012345678901 2
2
12345678901234567890123456789012123456789012345678901
integer.
2
12345678901234567890123456789012123456789012345678901
2
12345678901234567890123456789012123456789012345678901
2
12345678901234567890123456789012123456789012345678901
2
12345678901234567890123456789012123456789012345678901
In
the
real
world,
the
terms
integer
and
number
are
fairly
synonymous,
12345678901234567890123456789012123456789012345678901 2
2345678901234567890123456789012123456789012345678901
2
12345678901234567890123456789012123456789012345678901
since when any real person says pick a number, they dont expect you
2
12345678901234567890123456789012123456789012345678901
13
2
12345678901234567890123456789012123456789012345678901
2
12345678901234567890123456789012123456789012345678901
to pick a fraction like . However, integer is a more precise term, and
2
1
8
2345678901234567890123456789012123456789012345678901
2
12345678901234567890123456789012123456789012345678901
since its also something people are unfamiliar with, it makes many
2
12345678901234567890123456789012123456789012345678901
2
12345678901234567890123456789012123456789012345678901
appearances
on
the
GRE.
12345678901234567890123456789012123456789012345678901 22
1
2
12345678901234567890123456789012123456789012345678901
2
12345678901234567890123456789012123456789012345678901
Besides
zero,
all
integers
are
positive
or
negative,
and
they
are
also
even
or
2
12345678901234567890123456789012123456789012345678901
2
12345678901234567890123456789012123456789012345678901
odd.
Positive
integers
are
greater
than
zero
(like
2,
15,
or
309),
and
2
12345678901234567890123456789012123456789012345678901
2345678901234567890123456789012123456789012345678901
2
1
12345678901234567890123456789012123456789012345678901 2
12345678901234567890123456789012123456789012345678901 2
12345678901234567890123456789012123456789012345678901 2 49
123456789012345678901234567890121234567890123456789012

http://www.xtremepapers.net

Part III: Section Review

50

123456789012345678901234567890121234567890123456789012
12345678901234567890123456789012123456789012345678901 2
2
12345678901234567890123456789012123456789012345678901
negative integers are less than zero (21, 223, 290). Think of the integers 2
12345678901234567890123456789012123456789012345678901
2
12345678901234567890123456789012123456789012345678901
as being along a line with zero at the center.
2
12345678901234567890123456789012123456789012345678901
12345678901234567890123456789012123456789012345678901 2
2
12345678901234567890123456789012123456789012345678901
12345678901234567890123456789012123456789012345678901 2
2
12345678901234567890123456789012123456789012345678901
2
12345678901234567890123456789012123456789012345678901
2
12345678901234567890123456789012123456789012345678901
2
12345678901234567890123456789012123456789012345678901
This
figure
above
is
called
a
number
line,
and
seeing
it
may
provoke
a
2345678901234567890123456789012123456789012345678901
2
1
12345678901234567890123456789012123456789012345678901
jarring flashback to the eighth grade. As you go from left to right on a 2
2
12345678901234567890123456789012123456789012345678901
12345678901234567890123456789012123456789012345678901
number line, the value of the number increases. Notice that this means that 2
2
12345678901234567890123456789012123456789012345678901
2
12345678901234567890123456789012123456789012345678901
25 is greater than 210.
2
12345678901234567890123456789012123456789012345678901
2
12345678901234567890123456789012123456789012345678901
The mathematical definitions for even and odd values use integers, stating 2
12345678901234567890123456789012123456789012345678901
2
12345678901234567890123456789012123456789012345678901
that an integer is even if you can divide it by 2 and have no remainder 2
12345678901234567890123456789012123456789012345678901
12345678901234567890123456789012123456789012345678901
(nothing left over). For example, 24 and 10 are even since two evenly 2
2
12345678901234567890123456789012123456789012345678901
2
12345678901234567890123456789012123456789012345678901
divides
into
both.
11
and
227,
on
the
other
hand,
are
odd
since
2
does
not
2
12345678901234567890123456789012123456789012345678901
2345678901234567890123456789012123456789012345678901
2
12345678901234567890123456789012123456789012345678901
evenly divide into them.
2
12345678901234567890123456789012123456789012345678901
12345678901234567890123456789012123456789012345678901 2
2
12345678901234567890123456789012123456789012345678901
123456789012345678901234
4123456789012345678901234
4 2
12345678901234567890123456789012123456789012345678901
12345678901234567890123
12345678901234567890123
even
1
even
5
even
e.g.,
12
1
8
5
20
4
4
2
12345678901234567890123456789012123456789012345678901
12345678901234567890123
12345678901234567890123412345678901234567890123
123456789012345678901234 2
1
2
12345678901234567890123456789012123456789012345678901
even 1 odd 5 odd
e.g., 6 1 13 5 19
2
12345678901234567890123456789012123456789012345678901
2
12345678901234567890123456789012123456789012345678901
odd
1
odd
5
even
e.g.,
7
1
9
5
16
2345678901234567890123456789012123456789012345678901
12345678901234567890123456789012123456789012345678901 2
12345678901234567890123456789012123456789012345678901 2
2
12345678901234567890123456789012123456789012345678901
even 3 even 5 even
e.g., 4 3 6 5 24
2
12345678901234567890123456789012123456789012345678901
12345678901234567890123456789012123456789012345678901 2
2
12345678901234567890123456789012123456789012345678901
even 3 odd 5 even
e.g., 3 3 8 5 24
2
12345678901234567890123456789012123456789012345678901
12345678901234567890123456789012123456789012345678901 2
2
12345678901234567890123456789012123456789012345678901
odd 3 odd 5 odd
e.g., 1 3 9 5 9
2
1
2345678901234567890123456789012123456789012345678901
12345678901234567890123456789012123456789012345678901 2
2
12345678901234567890123456789012123456789012345678901
(Remember, if you forget one of these, like what an even times an
2
1
odd is, just pick an even and an odd number and multiply them.
2
12345678901234567890123456789012123456789012345678901
2
12345678901234567890123456789012123456789012345678901
Their
product
will
remind
you.)
2345678901234567890123456789012123456789012345678901
12345678901234567890123456789012123456789012345678901 2
12345678901234567890123456789012123456789012345678901 2
2
1
2
12345678901234567890123456789012123456789012345678901
You
might
not
remember
how
basic
math
operations
work
with
that
pesky
2
12345678901234567890123456789012123456789012345678901
2345678901234567890123456789012123456789012345678901
2
12345678901234567890123456789012123456789012345678901
negative sign, so see the quick refresher on the following page.
2
12345678901234567890123456789012123456789012345678901
2
1
12345678901234567890123456789012123456789012345678901 2
12345678901234567890123456789012123456789012345678901 2
2
12345678901234567890123456789012123456789012345678901
2
12345678901234567890123456789012123456789012345678901
2
12345678901234567890123456789012123456789012345678901
2
12345678901234567890123456789012123456789012345678901
12345678901234567890123456789012123456789012345678901 2
2345678901234567890123456789012123456789012345678901
12345678901234567890123456789012123456789012345678901 2
12345678901234567890123456789012123456789012345678901 2
12345678901234567890123456789012123456789012345678901 2
12345678901234567890123456789012123456789012345678901 2
2
1
2
12345678901234567890123456789012123456789012345678901
2
12345678901234567890123456789012123456789012345678901
2
12345678901234567890123456789012123456789012345678901
2
12345678901234567890123456789012123456789012345678901
12345678901234567890123456789012123456789012345678901 2
2345678901234567890123456789012123456789012345678901
12345678901234567890123456789012123456789012345678901 2
2
1
2
12345678901234567890123456789012123456789012345678901
12345678901234567890123456789012123456789012345678901 2
2
12345678901234567890123456789012123456789012345678901
12345678901234567890123456789012123456789012345678901 2
12345678901234567890123456789012123456789012345678901 2
12345678901234567890123456789012123456789012345678901 2
12345678901234567890123456789012123456789012345678901 2
123456789012345678901234567890121234567890123456789012

www.petersons.com

http://www.xtremepapers.net

Chapter 4: Quantitative Review

123456789012345678901234567890121234567890123456789012
12345678901234567890123456789012123456789012345678901 2
2
12345678901234567890123456789012123456789012345678901
2
12345678901234567890123456789012123456789012345678901
Working with Negative Numbers
2
12345678901234567890123456789012123456789012345678901
2
12345678901234567890123456789012123456789012345678901
12345678901234567890123456
1234567890123456789012345
6123456789012345678901234
5 2
12345678901234567890123456789012123456789012345678901
1234567890123456789012345
(negative) 1 (negative)
e.g., (24) 1 (25) 5 29
12345678901234567890123456789012123456789012345678901
1234567890123456789012345 6
123456789012345678901234 5 2
6
12345678901234567890123456789012123456789012345678901
1234567890123456789012345
123456789012345678901234 5 2
5 negative
12345678901234567890123456
1234567890123456789012345 2
12345678901234567890123456789012123456789012345678901
2
12345678901234567890123456789012123456789012345678901
(positive) 1 (negative)
e.g., (10) 1 (212) 5 10 2 12
2
12345678901234567890123456789012123456789012345678901
2
12345678901234567890123456789012123456789012345678901
5
(positive)
2
(positive)
5
22
2345678901234567890123456789012123456789012345678901
2
1
2
12345678901234567890123456789012123456789012345678901
or
2
12345678901234567890123456789012123456789012345678901
2
12345678901234567890123456789012123456789012345678901
(26)
1
(8)
2
12345678901234567890123456789012123456789012345678901
2345678901234567890123456789012123456789012345678901
2
1
5826
2
12345678901234567890123456789012123456789012345678901
2
12345678901234567890123456789012123456789012345678901
5
2
2
12345678901234567890123456789012123456789012345678901
2
12345678901234567890123456789012123456789012345678901
2345678901234567890123456789012123456789012345678901
2
1 (positive) 2 (negative)
e.g., 12 2 (23) 5 12 1 3 5 15
2
12345678901234567890123456789012123456789012345678901
2
12345678901234567890123456789012123456789012345678901
5 positive 1 positive
2
12345678901234567890123456789012123456789012345678901
2
12345678901234567890123456789012123456789012345678901
(positive)
3
(negative)
5
negative
e.g.,
26
3
3
5
218
2345678901234567890123456789012123456789012345678901
12345678901234567890123456789012123456789012345678901 22
12345678901234567890123456789012123456789012345678901 2
12345678901234567890123456789012123456789012345678901
(negative) 3 (negative) 5 positive e.g., (27) 3 (24) 5 28
2
12345678901234567890123456789012123456789012345678901
12345678901234567890123456789012123456789012345678901 22
12345678901234567890123456789012123456789012345678901
e.g., 0 3 (210) 5 0
2
1 Zero is not negative, but it is
2
12345678901234567890123456789012123456789012345678901
good
to
remember
that
2
12345678901234567890123456789012123456789012345678901
2
12345678901234567890123456789012123456789012345678901
0
3
(anything)
5
0
2345678901234567890123456789012123456789012345678901
12345678901234567890123456789012123456789012345678901 22
12345678901234567890123456789012123456789012345678901 2
12345678901234567890123456789012123456789012345678901 2
12345678901234567890123456789012123456789012345678901
How many integers are there from 25 to 7 inclusive?
2
12345678901234567890123456789012123456789012345678901
12345678901234567890123456789012123456789012345678901 22
12345678901234567890123456789012123456789012345678901
A. 14
2
12345678901234567890123456789012123456789012345678901
2
12345678901234567890123456789012123456789012345678901
B.
13
2
1
2345678901234567890123456789012123456789012345678901
2
12345678901234567890123456789012123456789012345678901
C. 12
2
12345678901234567890123456789012123456789012345678901
2
1
D.
2
12345678901234567890123456789012123456789012345678901 2
2
12345678901234567890123456789012123456789012345678901
E. 212
2
12345678901234567890123456789012123456789012345678901
2
12345678901234567890123456789012123456789012345678901
12345678901234567890123456789012123456789012345678901
Heres a simple math question with two well-traveled math terms, integer 2
2
12345678901234567890123456789012123456789012345678901
12345678901234567890123456789012123456789012345678901
and inclusive. You know the first term, so if you figure out what inclusive 2
2345678901234567890123456789012123456789012345678901
2
12345678901234567890123456789012123456789012345678901
means, you should be able to solve this problem. Inclusive means you 2
12345678901234567890123456789012123456789012345678901
1 include 25 and 7 when you are counting the number of integers. How 2
12345678901234567890123456789012123456789012345678901 2
2
12345678901234567890123456789012123456789012345678901
many negative numbers are there from 25 to 7? Five. How many positive 2
12345678901234567890123456789012123456789012345678901
2
12345678901234567890123456789012123456789012345678901
numbers are there from 25 to 7? Seven. Test-takers in a hurry may add 2
12345678901234567890123456789012123456789012345678901
12345678901234567890123456789012123456789012345678901
these numbers, 5 and 7, and then pick choice (C). Yet this would be too 2
2
12345678901234567890123456789012123456789012345678901
2345678901234567890123456789012123456789012345678901
12345678901234567890123456789012123456789012345678901
hasty, for if you recall, zero is also an integer. That makes the correct 2
2
12345678901234567890123456789012123456789012345678901
2
12345678901234567890123456789012123456789012345678901
answer
for
this
problem
13,
choice
(B).
12345678901234567890123456789012123456789012345678901 22
1
12345678901234567890123456789012123456789012345678901
This problem typifies the GRE Quant section. The question contains two 2
2
12345678901234567890123456789012123456789012345678901
2
12345678901234567890123456789012123456789012345678901
basic
math
terms
and
in
many
ways
looks
deceptively
simple.
You
can
2
12345678901234567890123456789012123456789012345678901
2
12345678901234567890123456789012123456789012345678901
come
up
with
a
quick
answer,
12,
thats
just
begging
for
you
to
choose
it.
2345678901234567890123456789012123456789012345678901
12345678901234567890123456789012123456789012345678901 22
1 However, if you take time and use a strong understanding of the basic
2
12345678901234567890123456789012123456789012345678901
2
12345678901234567890123456789012123456789012345678901
terms integer and inclusive, you will not forget the presence of zero.
2
12345678901234567890123456789012123456789012345678901
2345678901234567890123456789012123456789012345678901
2
1
12345678901234567890123456789012123456789012345678901 2
12345678901234567890123456789012123456789012345678901 2
12345678901234567890123456789012123456789012345678901 2 51
123456789012345678901234567890121234567890123456789012

http://www.xtremepapers.net

Part III: Section Review

www.petersons.com

http://www.xtremepapers.net

Note

52

123456789012345678901234567890121234567890123456789012
12345678901234567890123456789012123456789012345678901 2
2
12345678901234567890123456789012123456789012345678901
The terms divisible and product are often used along with integers. 2
12345678901234567890123456789012123456789012345678901
2
12345678901234567890123456789012123456789012345678901
Divisible is the math term for when an integer can be divided by another 2
12345678901234567890123456789012123456789012345678901
2
12345678901234567890123456789012123456789012345678901
without there being a remainder. For instance, 12 is divisible by 3 since 2
12345678901234567890123456789012123456789012345678901
2345678901234567890123456789012123456789012345678901
1 12 4 3 5 4, and there is no remainder. In this instance, 3 also has a name, 2
2
12345678901234567890123456789012123456789012345678901
2
12345678901234567890123456789012123456789012345678901
divisor,
since
it
evenly
divides
into
12.
When
you
multiply
two
numbers
2
12345678901234567890123456789012123456789012345678901
2
12345678901234567890123456789012123456789012345678901
together,
the
answer
is
the
product.
So
12
is
the
product
of
3
times
4.
In
2345678901234567890123456789012123456789012345678901
2
1
2
12345678901234567890123456789012123456789012345678901
this
case,
3
and
4
are
called
factors
of
12
because
their
product
is
12.
2
12345678901234567890123456789012123456789012345678901
2
12345678901234567890123456789012123456789012345678901
2
12345678901234567890123456789012123456789012345678901
The
factors
of
12
are
the
integers
that
multiply
together
to
give
12:
1
3
12,
2345678901234567890123456789012123456789012345678901
2
1
2
12345678901234567890123456789012123456789012345678901
2
3
6,
3
3
4.
Notice
that
the
factors
of
12
and
the
divisors
of
12
are
the
2
12345678901234567890123456789012123456789012345678901
2
12345678901234567890123456789012123456789012345678901
same.
They
just
involve
using
a
different
math
operation.
Think
of
it
as
2
12345678901234567890123456789012123456789012345678901
2345678901234567890123456789012123456789012345678901
1 going up a set of stairs or heading down a set of stairs. Either way, its the 2
2
12345678901234567890123456789012123456789012345678901
2
12345678901234567890123456789012123456789012345678901
same set of stairs.
2
12345678901234567890123456789012123456789012345678901
2
12345678901234567890123456789012123456789012345678901
2345678901234567890123456789012123456789012345678901
12345678901234567890123456789012123456789012345678901 2
12345678901234567890123456789012123456789012345678901
There is another related term, multiple. The multiple of an integer is 2
2
12345678901234567890123456789012123456789012345678901
12345678901234567890123456789012123456789012345678901
that integer times counting numbers like 1, 2, 3, and up. So the 2
12345678901234567890123456789012123456789012345678901 2
2
12345678901234567890123456789012123456789012345678901
1 multiples of 3 are 3(3 3 1), 6(3 3 2), 9(3 3 3), 12(3 3 4), 15(3 3 5), 2
12345678901234567890123456789012123456789012345678901
18(3 3 6) . . . and so on. The integer 12(3 3 4) is a multiple of 3, and 2
2
12345678901234567890123456789012123456789012345678901
2
12345678901234567890123456789012123456789012345678901
its also a multiple of 4.
2345678901234567890123456789012123456789012345678901
12345678901234567890123456789012123456789012345678901 2
12345678901234567890123456789012123456789012345678901 2
12345678901234567890123456789012123456789012345678901 2
Keeping all these terms in your head can be a bit tricky. Its like having 2
12345678901234567890123456789012123456789012345678901
2
12345678901234567890123456789012123456789012345678901
identical triplet cousins whose names are Tad, Todd, and Ted. Dont lose 2
12345678901234567890123456789012123456789012345678901
2
12345678901234567890123456789012123456789012345678901
heart in this flurry of terms. None of them individually is difficult to grasp, 2
12345678901234567890123456789012123456789012345678901
2
12345678901234567890123456789012123456789012345678901
1 but they can be a little confusing if they are all heaped together at the same 2
2345678901234567890123456789012123456789012345678901
2
12345678901234567890123456789012123456789012345678901
time like in the following problem.
2
12345678901234567890123456789012123456789012345678901
2
1
2
12345678901234567890123456789012123456789012345678901
Column
B
Column
A
2
12345678901234567890123456789012123456789012345678901
2345678901234567890123456789012123456789012345678901
12345678901234567890123456789012123456789012345678901 2
2
12345678901234567890123456789012123456789012345678901
The greatest
The first even
2
1
2
12345678901234567890123456789012123456789012345678901
common
factor
of
integer
greater
than
2
12345678901234567890123456789012123456789012345678901
2345678901234567890123456789012123456789012345678901
2
12345678901234567890123456789012123456789012345678901
18 and 36
434
2
12345678901234567890123456789012123456789012345678901
2
1
2
12345678901234567890123456789012123456789012345678901
The greatest common factor, or GCF, is the largest number that is a factor 2
12345678901234567890123456789012123456789012345678901
2345678901234567890123456789012123456789012345678901
2
12345678901234567890123456789012123456789012345678901
of both 18 and 36. You might notice right off the bat that 36 5 2 3 18, 2
12345678901234567890123456789012123456789012345678901
2
12345678901234567890123456789012123456789012345678901
and so 18 must be the GCF. If you dont catch this, dont sweat it. Just start 2
12345678901234567890123456789012123456789012345678901
2
1 writing down the factors of 18 and 36.
2
12345678901234567890123456789012123456789012345678901
2
12345678901234567890123456789012123456789012345678901
2
12345678901234567890123456789012123456789012345678901
Factors
of
18:
1,
2,
3,
6,
9,
18
2
12345678901234567890123456789012123456789012345678901
12345678901234567890123456789012123456789012345678901 2
2345678901234567890123456789012123456789012345678901
2
12345678901234567890123456789012123456789012345678901
Factors of 36: 1, 2, 3, 4, 6, 9, 12, 18, 36
2
12345678901234567890123456789012123456789012345678901
12345678901234567890123456789012123456789012345678901 2
2
12345678901234567890123456789012123456789012345678901
1 18 is the largest number they share, so the integer value of Column A is 18. 2
2345678901234567890123456789012123456789012345678901
Looking
at
Column
B,
you
just
have
to
unpack
the
terms.
The
first
integer
12345678901234567890123456789012123456789012345678901 2
1 greater than 16 (4 3 4) is 17, and the first even integer greater than 16 is 2
2
12345678901234567890123456789012123456789012345678901
2
12345678901234567890123456789012123456789012345678901
18.
Column
B
also
equals
18,
so
(C)
is
the
correct
answer.
2
12345678901234567890123456789012123456789012345678901
2345678901234567890123456789012123456789012345678901
2
1
12345678901234567890123456789012123456789012345678901 2
12345678901234567890123456789012123456789012345678901 2
12345678901234567890123456789012123456789012345678901 2
123456789012345678901234567890121234567890123456789012

Tip

Chapter 4: Quantitative Review

123456789012345678901234567890121234567890123456789012
12345678901234567890123456789012123456789012345678901 2
2
12345678901234567890123456789012123456789012345678901
12345678901234567890123456789012123456789012345678901 2
2
12345678901234567890123456789012123456789012345678901
2
12345678901234567890123456789012123456789012345678901
Notice
that
both
Columns
A
and
B
describe
specific
numbers
that
will
12345678901234567890123456789012123456789012345678901 2
2
12345678901234567890123456789012123456789012345678901
have a definite relationship. Therefore, (D) cannot be the correct answer.
2
12345678901234567890123456789012123456789012345678901
2
12345678901234567890123456789012123456789012345678901
2
12345678901234567890123456789012123456789012345678901
2
12345678901234567890123456789012123456789012345678901
2
12345678901234567890123456789012123456789012345678901
One
final
integer-related
term
to
cover
concerns
prime
numbers.
Prime
2345678901234567890123456789012123456789012345678901
2
1
2
12345678901234567890123456789012123456789012345678901
numbers
are
divisible
by
only
two
integers,
themselves
and
one.
Another
2
12345678901234567890123456789012123456789012345678901
12345678901234567890123456789012123456789012345678901
way of saying this is to state a prime number only has two multiples: Itself 2
2
12345678901234567890123456789012123456789012345678901
12345678901234567890123456789012123456789012345678901
and 1. 11 is a prime number since the only integers that evenly divide into 2
2
12345678901234567890123456789012123456789012345678901
12345678901234567890123456789012123456789012345678901
11 are 11 and 1. Notice that 1 is not a prime number since it has one 2
2
12345678901234567890123456789012123456789012345678901
2
12345678901234567890123456789012123456789012345678901
multiple (1 3 1) and not two.
2345678901234567890123456789012123456789012345678901
2
1
2
12345678901234567890123456789012123456789012345678901
It is a good idea to be familiar with the smaller prime numbers because you 2
12345678901234567890123456789012123456789012345678901
12345678901234567890123456789012123456789012345678901
might need them at your fingertips for a problem. Heres the first ten: 2, 3, 2
2
12345678901234567890123456789012123456789012345678901
2345678901234567890123456789012123456789012345678901
12345678901234567890123456789012123456789012345678901
5, 7, 11, 13, 17, 19, 23, and 29. If you want any more, youll have to figure 2
2
12345678901234567890123456789012123456789012345678901
2
12345678901234567890123456789012123456789012345678901
them
out
yourself
or
pay
extra.
12345678901234567890123456789012123456789012345678901 22
12345678901234567890123456789012123456789012345678901 2
12345678901234567890123456789012123456789012345678901 2
1
If n equals the number of prime numbers greater than 10 and less
2
12345678901234567890123456789012123456789012345678901
2
12345678901234567890123456789012123456789012345678901
than 20, then 3n 5
2
12345678901234567890123456789012123456789012345678901
2345678901234567890123456789012123456789012345678901
2
12345678901234567890123456789012123456789012345678901
A. 3
2
12345678901234567890123456789012123456789012345678901
2
12345678901234567890123456789012123456789012345678901
B. 6
2
12345678901234567890123456789012123456789012345678901
2
12345678901234567890123456789012123456789012345678901
C.
9
12345678901234567890123456789012123456789012345678901 22
12345678901234567890123456789012123456789012345678901
D. 12
2
12345678901234567890123456789012123456789012345678901
2
12345678901234567890123456789012123456789012345678901
E.
15
2
1
2345678901234567890123456789012123456789012345678901
12345678901234567890123456789012123456789012345678901 22
12345678901234567890123456789012123456789012345678901
1 Before you can find 3n, you must figure out what n is. Looking at the list of 2
12345678901234567890123456789012123456789012345678901
prime numbers in the paragraph above, you can see that there are four 2
2
12345678901234567890123456789012123456789012345678901
2345678901234567890123456789012123456789012345678901
12345678901234567890123456789012123456789012345678901
prime numbers between 10 and 20: 11, 13, 17, 19. If n 5 4, then 3n will be 2
2
12345678901234567890123456789012123456789012345678901
2
1 (3)(4) 5 12. Therefore, (D) is the correct answer.
2
12345678901234567890123456789012123456789012345678901
2
12345678901234567890123456789012123456789012345678901
2345678901234567890123456789012123456789012345678901
12345678901234567890123456789012123456789012345678901 22
12345678901234567890123456789012123456789012345678901 2
1Fractions, or Pretty Much Everything Thats Not
12345678901234567890123456789012123456789012345678901 2
2
12345678901234567890123456789012123456789012345678901
an Integer
2
12345678901234567890123456789012123456789012345678901
2
12345678901234567890123456789012123456789012345678901
a
2
12345678901234567890123456789012123456789012345678901
12345678901234567890123456789012123456789012345678901
A fraction is a number made up of two integers and it looks like this: . 2
12345678901234567890123456789012123456789012345678901
b 2
12345678901234567890123456789012123456789012345678901
The number on top, a, is called the numerator. The number on the bottom, 2
2
12345678901234567890123456789012123456789012345678901
2
12345678901234567890123456789012123456789012345678901
b,
is
called
the
denominator.
(If
you
can
come
up
with
a
clever
way
to
help
2
12345678901234567890123456789012123456789012345678901
2
12345678901234567890123456789012123456789012345678901
yourself
remember
that
the
numerator
is
on
top
while
the
denominator
is
2345678901234567890123456789012123456789012345678901
12345678901234567890123456789012123456789012345678901 22
12345678901234567890123456789012123456789012345678901
on bottom, you should do so.) A fraction means you are dividing the top 2
12345678901234567890123456789012123456789012345678901
2
12345678901234567890123456789012123456789012345678901
1 by the bottom, so a is the same thing as a 4 b. Since a 5 a 4 b, b cannot 2
2345678901234567890123456789012123456789012345678901
2
12345678901234567890123456789012123456789012345678901
b
2
1 equal zero becauseb mathematicians dont think it makes
any sense to 2
12345678901234567890123456789012123456789012345678901
2
12345678901234567890123456789012123456789012345678901
divide by 0. I dont have the exact reasons for this, but in general, 2
12345678901234567890123456789012123456789012345678901
2345678901234567890123456789012123456789012345678901
2
1
12345678901234567890123456789012123456789012345678901 2
12345678901234567890123456789012123456789012345678901 2
12345678901234567890123456789012123456789012345678901 2 53
123456789012345678901234567890121234567890123456789012

http://www.xtremepapers.net

Part III: Section Review

123456789012345678901234567890121234567890123456789012
12345678901234567890123456789012123456789012345678901 2
2
12345678901234567890123456789012123456789012345678901
mathematicians are a serious lot and I dont think they would say this just 2
12345678901234567890123456789012123456789012345678901
2
12345678901234567890123456789012123456789012345678901
to pull our leg.
2
12345678901234567890123456789012123456789012345678901
12345678901234567890123456789012123456789012345678901 2
12345678901234567890123456789012123456789012345678901
The GRE math section is going to expect you to do things like add and 2
2
12345678901234567890123456789012123456789012345678901
2
12345678901234567890123456789012123456789012345678901
divide
fractions,
so
here
is
a
quick
review
of
how
to
perform
these
math
2
12345678901234567890123456789012123456789012345678901
2
12345678901234567890123456789012123456789012345678901
operations
with
fractions.
2
12345678901234567890123456789012123456789012345678901
2345678901234567890123456789012123456789012345678901
2
1
2
12345678901234567890123456789012123456789012345678901
2
12345678901234567890123456789012123456789012345678901
2
12345678901234567890123456789012123456789012345678901
1. Addition
2
12345678901234567890123456789012123456789012345678901
You
can
only
add
fractions
that
have
the
same
denominator.
If
you
are
2345678901234567890123456789012123456789012345678901
2
1
2
12345678901234567890123456789012123456789012345678901
3 2
2
12345678901234567890123456789012123456789012345678901
asked what 1 equals, you have to make the bottoms look the same. In 2
12345678901234567890123456789012123456789012345678901
4 3
2
12345678901234567890123456789012123456789012345678901
12345678901234567890123456789012123456789012345678901
math speak, this is called finding a common denominator. The common 2
2
12345678901234567890123456789012123456789012345678901
2
12345678901234567890123456789012123456789012345678901
denominator
will
be
a
number
that
is
a
multiple
of
both
4
and
3.
12
is
the
2
12345678901234567890123456789012123456789012345678901
2
12345678901234567890123456789012123456789012345678901
smallest
common
multiple
(in
math
speak,
the
least
common
multiple
or
2345678901234567890123456789012123456789012345678901
12345678901234567890123456789012123456789012345678901 2
12345678901234567890123456789012123456789012345678901
LCM). To get 12 as your denominator for both fractions, you multiply the 2
2
12345678901234567890123456789012123456789012345678901
12345678901234567890123456789012123456789012345678901
first denominator by 3 to get 12 and the second denominator by 4 2
12345678901234567890123456789012123456789012345678901 2
2
12345678901234567890123456789012123456789012345678901
2
1 to get 12.
2
12345678901234567890123456789012123456789012345678901
However, you cant just multiply only the denominator by whatever 2
12345678901234567890123456789012123456789012345678901
2
12345678901234567890123456789012123456789012345678901
number you please. You must also multiply the numerator by that same 2
12345678901234567890123456789012123456789012345678901
2
12345678901234567890123456789012123456789012345678901
4
3
2
12345678901234567890123456789012123456789012345678901
number. Multiply the first fraction by and the second one by .
2
12345678901234567890123456789012123456789012345678901
3
4
2
12345678901234567890123456789012123456789012345678901
2345678901234567890123456789012123456789012345678901
12345678901234567890123456789012123456789012345678901 2
2
12345678901234567890123456789012123456789012345678901
This gives you:
2
12345678901234567890123456789012123456789012345678901
2
12345678901234567890123456789012123456789012345678901
3
3
9
2
1
3
5
2345678901234567890123456789012123456789012345678901
2
12345678901234567890123456789012123456789012345678901
4 3 12
2
12345678901234567890123456789012123456789012345678901
2
1
2
12345678901234567890123456789012123456789012345678901
and
2
12345678901234567890123456789012123456789012345678901
2345678901234567890123456789012123456789012345678901
2
12345678901234567890123456789012123456789012345678901
2 4
8
2
12345678901234567890123456789012123456789012345678901
3 5
2
1
3 4 12
2
12345678901234567890123456789012123456789012345678901
2
12345678901234567890123456789012123456789012345678901
2345678901234567890123456789012123456789012345678901
2
12345678901234567890123456789012123456789012345678901
3 3
2 4
9
8
2
12345678901234567890123456789012123456789012345678901
1
5
1
2
1
4
3
3
4
12
12
12345678901234567890123456789012123456789012345678901 2
12345678901234567890123456789012123456789012345678901 2
2
12345678901234567890123456789012123456789012345678901
Now that the denominators are the same you can add the numerators,
2
12345678901234567890123456789012123456789012345678901
2
12345678901234567890123456789012123456789012345678901
2
12345678901234567890123456789012123456789012345678901
9
8
8
1
9
17
2
12345678901234567890123456789012123456789012345678901
1
5
5
2345678901234567890123456789012123456789012345678901
2
12345678901234567890123456789012123456789012345678901
12 12
12
12
2
12345678901234567890123456789012123456789012345678901
12345678901234567890123456789012123456789012345678901 2
12345678901234567890123456789012123456789012345678901 2
2
1 2. Subtraction
2
12345678901234567890123456789012123456789012345678901
2
12345678901234567890123456789012123456789012345678901
Like
addition,
you
have
to
have
the
same
denominator
in
order
to
subtract
2
12345678901234567890123456789012123456789012345678901
2
12345678901234567890123456789012123456789012345678901
fractions.
Once
you
have
that,
just
subtract
the
two
numbers
on
top
(the
12345678901234567890123456789012123456789012345678901 2
2345678901234567890123456789012123456789012345678901
2
12345678901234567890123456789012123456789012345678901
2
1 numerators). So,
2
12345678901234567890123456789012123456789012345678901
2
12345678901234567890123456789012123456789012345678901
9
8
928
1
3 2
2
12345678901234567890123456789012123456789012345678901
2
5
2
5
5
2345678901234567890123456789012123456789012345678901
2
1
4 3 12 12
12
12
2
12345678901234567890123456789012123456789012345678901
12345678901234567890123456789012123456789012345678901 2
12345678901234567890123456789012123456789012345678901 2
123456789012345678901234567890121234567890123456789012

SD SD

54

www.petersons.com

http://www.xtremepapers.net

Chapter 4: Quantitative Review

123456789012345678901234567890121234567890123456789012
12345678901234567890123456789012123456789012345678901 2
2
12345678901234567890123456789012123456789012345678901
3. Multiplication
2
12345678901234567890123456789012123456789012345678901
2
12345678901234567890123456789012123456789012345678901
This is a lot easier. Multiply the numerators together and multiply the 2
12345678901234567890123456789012123456789012345678901
2
12345678901234567890123456789012123456789012345678901
denominators together. For example,
2
12345678901234567890123456789012123456789012345678901
2345678901234567890123456789012123456789012345678901
2
1
2
12345678901234567890123456789012123456789012345678901
210
3
230
2
12345678901234567890123456789012123456789012345678901
3
5
2
12345678901234567890123456789012123456789012345678901
4
5
20
2
12345678901234567890123456789012123456789012345678901
2345678901234567890123456789012123456789012345678901
2
1
12345678901234567890123456789012123456789012345678901
Fractions in answer choices on the GRE will usually be in their simplest 2
2
12345678901234567890123456789012123456789012345678901
230
2
12345678901234567890123456789012123456789012345678901
12345678901234567890123456789012123456789012345678901
form, and so
would not be an answer on the GRE. It would be 2
2345678901234567890123456789012123456789012345678901
2
1
20
2
12345678901234567890123456789012123456789012345678901
simplified,
2
12345678901234567890123456789012123456789012345678901
2
12345678901234567890123456789012123456789012345678901
2
12345678901234567890123456789012123456789012345678901
230
23
3
10
23
10
23
3
1
2345678901234567890123456789012123456789012345678901
2
1
5
5
3
5
3 1 5 2 or 21
2
12345678901234567890123456789012123456789012345678901
20
2
3
10
2
10
2
2
2
2
12345678901234567890123456789012123456789012345678901
2
12345678901234567890123456789012123456789012345678901
2
12345678901234567890123456789012123456789012345678901
2345678901234567890123456789012123456789012345678901
2
12345678901234567890123456789012123456789012345678901
4. Division
2
12345678901234567890123456789012123456789012345678901
2
12345678901234567890123456789012123456789012345678901
2
7
2
12345678901234567890123456789012123456789012345678901
Dividing
two
fractions
can
look
like
this,
4
,
or
it
can
look
like
this,
2
12345678901234567890123456789012123456789012345678901
5
8
12345678901234567890123456789012123456789012345678901 22
1
2
2
12345678901234567890123456789012123456789012345678901
2
12345678901234567890123456789012123456789012345678901
5
2
12345678901234567890123456789012123456789012345678901
2345678901234567890123456789012123456789012345678901
2
12345678901234567890123456789012123456789012345678901
7
2
12345678901234567890123456789012123456789012345678901
12345678901234567890123456789012123456789012345678901 22
12345678901234567890123456789012123456789012345678901
8
2
12345678901234567890123456789012123456789012345678901
12345678901234567890123456789012123456789012345678901
These two expressions are the same. You solve them with a two-step 2
2
12345678901234567890123456789012123456789012345678901
2
12345678901234567890123456789012123456789012345678901
7
8
2
12345678901234567890123456789012123456789012345678901
process.
First,
flip
the
fraction
you
are
dividing
by,
so
that

.
In
math
2
1
8
7
2345678901234567890123456789012123456789012345678901
2
12345678901234567890123456789012123456789012345678901
speak, this flipping is called finding the reciprocal. Now multiply the 2
12345678901234567890123456789012123456789012345678901
2
1 flipped fraction and the other fraction. So,
12345678901234567890123456789012123456789012345678901 2
2
12345678901234567890123456789012123456789012345678901
2
12345678901234567890123456789012123456789012345678901
2
2
12345678901234567890123456789012123456789012345678901
2
12345678901234567890123456789012123456789012345678901
5
2
8
16
2
12345678901234567890123456789012123456789012345678901
5
3
5
2
12345678901234567890123456789012123456789012345678901
5
7
35
7
2345678901234567890123456789012123456789012345678901
12345678901234567890123456789012123456789012345678901 22
12345678901234567890123456789012123456789012345678901
8
2
1
12345678901234567890123456789012123456789012345678901 2
12345678901234567890123456789012123456789012345678901
A catchy phrase to remember the dividing procedure is, When dividing, 2
2
12345678901234567890123456789012123456789012345678901
2
12345678901234567890123456789012123456789012345678901
dont
ask
why;
just
flip
it
over
and
multiply.
If
you
forget
it,
any
2
12345678901234567890123456789012123456789012345678901
2
12345678901234567890123456789012123456789012345678901
14-year-old kid can remind you of this phrase.
12345678901234567890123456789012123456789012345678901 2
2345678901234567890123456789012123456789012345678901
2
12345678901234567890123456789012123456789012345678901
Fractions also come in hybrid form. Theyre called mixed fractions, and 2
12345678901234567890123456789012123456789012345678901
2
12345678901234567890123456789012123456789012345678901
2
2
2
12345678901234567890123456789012123456789012345678901
1 they look like 37, which means 3 1 7. In order to do any arithmetic 2
2345678901234567890123456789012123456789012345678901
12345678901234567890123456789012123456789012345678901 22
12345678901234567890123456789012123456789012345678901
operation like the ones discussed above, mixed fractions should be 2
12345678901234567890123456789012123456789012345678901
2
12345678901234567890123456789012123456789012345678901
1 converted into pure fractions. You might be able to multiply the mixed 2
2345678901234567890123456789012123456789012345678901
2
12345678901234567890123456789012123456789012345678901
2
1 fractions 32 with 43, but it would be pretty messy.
2
12345678901234567890123456789012123456789012345678901
7
5
12345678901234567890123456789012123456789012345678901 2
2
12345678901234567890123456789012123456789012345678901
12345678901234567890123456789012123456789012345678901 2
12345678901234567890123456789012123456789012345678901 2
12345678901234567890123456789012123456789012345678901 2
12345678901234567890123456789012123456789012345678901 2 55
123456789012345678901234567890121234567890123456789012

http://www.xtremepapers.net

Part III: Section Review

www.petersons.com

http://www.xtremepapers.net

Note

56

123456789012345678901234567890121234567890123456789012
12345678901234567890123456789012123456789012345678901 2
2
12345678901234567890123456789012123456789012345678901
To convert a mixed fraction into a pure fraction, first multiply the integer 2
12345678901234567890123456789012123456789012345678901
2
12345678901234567890123456789012123456789012345678901
out front by the denominator, then add that product to the numerator.
2
12345678901234567890123456789012123456789012345678901
12345678901234567890123456789012123456789012345678901 2
2
12345678901234567890123456789012123456789012345678901
2
12345678901234567890123456789012123456789012345678901
2
2
12345678901234567890123456789012123456789012345678901
3
2
12345678901234567890123456789012123456789012345678901
7
2
12345678901234567890123456789012123456789012345678901
2
12345678901234567890123456789012123456789012345678901
2345678901234567890123456789012123456789012345678901
2
1
3 3 7 5 21
(multiplying the integer 3 with the denominator 7)
2
12345678901234567890123456789012123456789012345678901
2
12345678901234567890123456789012123456789012345678901
21
1
2
5
23
(adding
the
product
to
the
numerator
2)
2
12345678901234567890123456789012123456789012345678901
2
12345678901234567890123456789012123456789012345678901
23
2345678901234567890123456789012123456789012345678901
2
1
5
2
12345678901234567890123456789012123456789012345678901
7
2
12345678901234567890123456789012123456789012345678901
2
12345678901234567890123456789012123456789012345678901
2
12345678901234567890123456789012123456789012345678901
Voila,
you
have
it.
2345678901234567890123456789012123456789012345678901
2
1
2
12345678901234567890123456789012123456789012345678901
2
12345678901234567890123456789012123456789012345678901
2
12345678901234567890123456789012123456789012345678901
2
12345678901234567890123456789012123456789012345678901
All
integers
can
be
expressed
as
a
fraction.
Place
the
integer
in
the
2345678901234567890123456789012123456789012345678901
12345678901234567890123456789012123456789012345678901 2
12345678901234567890123456789012123456789012345678901
numerator and a 1 in the denominator. For instance, 7 in fractional form 2
2
12345678901234567890123456789012123456789012345678901
2
12345678901234567890123456789012123456789012345678901
7
7
2
12345678901234567890123456789012123456789012345678901
is
.
If
you
think
about
it,
5
7
4
1
5
7
should
make
sense
to
you.
2
12345678901234567890123456789012123456789012345678901
1
1
2
1
2
12345678901234567890123456789012123456789012345678901
2
12345678901234567890123456789012123456789012345678901
3x
3
2
4
2
12345678901234567890123456789012123456789012345678901
If
4
5
4
then
3
5
2345678901234567890123456789012123456789012345678901
2
12345678901234567890123456789012123456789012345678901
2
8
3 4x
2
12345678901234567890123456789012123456789012345678901
12345678901234567890123456789012123456789012345678901 2
2
12345678901234567890123456789012123456789012345678901
2
2
12345678901234567890123456789012123456789012345678901
A.
2
12345678901234567890123456789012123456789012345678901
5
2
12345678901234567890123456789012123456789012345678901
2
12345678901234567890123456789012123456789012345678901
2
2
12345678901234567890123456789012123456789012345678901
B.
2
1
3
2345678901234567890123456789012123456789012345678901
12345678901234567890123456789012123456789012345678901 2
12345678901234567890123456789012123456789012345678901 2
2
1
C. 1
2
12345678901234567890123456789012123456789012345678901
2
12345678901234567890123456789012123456789012345678901
8
2345678901234567890123456789012123456789012345678901
2
12345678901234567890123456789012123456789012345678901
D.
2
12345678901234567890123456789012123456789012345678901
3
2
1
2
12345678901234567890123456789012123456789012345678901
2
12345678901234567890123456789012123456789012345678901
E.
4
2345678901234567890123456789012123456789012345678901
12345678901234567890123456789012123456789012345678901 2
12345678901234567890123456789012123456789012345678901 2
1 Heres a basic GRE question that seeks to overwhelm you with a bunch of 2
2
12345678901234567890123456789012123456789012345678901
fractions and arithmetic flitting about. You cant solve the second equation 2
12345678901234567890123456789012123456789012345678901
2345678901234567890123456789012123456789012345678901
2
12345678901234567890123456789012123456789012345678901
until you learn the value of x, which can be found by solving the first 2
12345678901234567890123456789012123456789012345678901
2
12345678901234567890123456789012123456789012345678901
equation:
2
12345678901234567890123456789012123456789012345678901
2
1
2345678901234567890123456789012123456789012345678901
12345678901234567890123456789012123456789012345678901 2
2
12345678901234567890123456789012123456789012345678901
3x 3
2
12345678901234567890123456789012123456789012345678901
4 54
2
12345678901234567890123456789012123456789012345678901
2
8
2
1
2345678901234567890123456789012123456789012345678901
2
12345678901234567890123456789012123456789012345678901
3x 3 3x 8 24x
2
12345678901234567890123456789012123456789012345678901
4
5
3
5
5
4x
(This
solves
the
left
side
of
the
2
12345678901234567890123456789012123456789012345678901
2
8
2
3
6
2
12345678901234567890123456789012123456789012345678901
equation.)
2
1
2345678901234567890123456789012123456789012345678901
2
12345678901234567890123456789012123456789012345678901
4x 5 4
2
1
2
12345678901234567890123456789012123456789012345678901
2
x51
12345678901234567890123456789012123456789012345678901
2
12345678901234567890123456789012123456789012345678901
2345678901234567890123456789012123456789012345678901
2
1
12345678901234567890123456789012123456789012345678901 2
12345678901234567890123456789012123456789012345678901 2
12345678901234567890123456789012123456789012345678901 2
123456789012345678901234567890121234567890123456789012

Chapter 4: Quantitative Review

123456789012345678901234567890121234567890123456789012
12345678901234567890123456789012123456789012345678901 2
2
12345678901234567890123456789012123456789012345678901
Plugging x 5 1 into the second equation will get you:
2
12345678901234567890123456789012123456789012345678901
2
12345678901234567890123456789012123456789012345678901
2
12345678901234567890123456789012123456789012345678901
2
12345678901234567890123456789012123456789012345678901
2
4
2
12345678901234567890123456789012123456789012345678901
3
5
2345678901234567890123456789012123456789012345678901
2
1
3 4x
2
12345678901234567890123456789012123456789012345678901
2
12345678901234567890123456789012123456789012345678901
or
2
12345678901234567890123456789012123456789012345678901
2
12345678901234567890123456789012123456789012345678901
2345678901234567890123456789012123456789012345678901
2
1
2
8
4
2
2
12345678901234567890123456789012123456789012345678901
5
3
5
2
12345678901234567890123456789012123456789012345678901
3
4
1
12
3
~
!
2
12345678901234567890123456789012123456789012345678901
2
12345678901234567890123456789012123456789012345678901
2345678901234567890123456789012123456789012345678901
2
1
4
12345678901234567890123456789012123456789012345678901
You might have noticed that 5 1, so multiplying out the second equation 2
2
12345678901234567890123456789012123456789012345678901
4
2
12345678901234567890123456789012123456789012345678901
wasnt
necessary.
Either
way,
you
end
up
with
(B),
the
correct
answer.
2
12345678901234567890123456789012123456789012345678901
2345678901234567890123456789012123456789012345678901
2
1
2
12345678901234567890123456789012123456789012345678901
2
12345678901234567890123456789012123456789012345678901
2
12345678901234567890123456789012123456789012345678901
2
12345678901234567890123456789012123456789012345678901
Decimals
(Fractions
in
Disguise!)
2345678901234567890123456789012123456789012345678901
12345678901234567890123456789012123456789012345678901 22
12345678901234567890123456789012123456789012345678901
Decimal form is another way to write a fraction. Decimals can easily be 2
12345678901234567890123456789012123456789012345678901
12345678901234567890123456789012123456789012345678901
converted into fractions, since every decimal has a denominator that is a 2
2
12345678901234567890123456789012123456789012345678901
2
12345678901234567890123456789012123456789012345678901
1 multiple of 10. The decimal 0.7 is the fraction 7 , while the decimal 0.765 2
2
12345678901234567890123456789012123456789012345678901
10
2
12345678901234567890123456789012123456789012345678901
765
2
12345678901234567890123456789012123456789012345678901
.
You
take
the
number
and
make
it
the
numerator,
then
you
look
is
2345678901234567890123456789012123456789012345678901
2
12345678901234567890123456789012123456789012345678901
1,000
2
12345678901234567890123456789012123456789012345678901
2
12345678901234567890123456789012123456789012345678901
and
count
how
many
spaces
there
are
to
the
right
of
the
decimal
point.
12345678901234567890123456789012123456789012345678901 22
12345678901234567890123456789012123456789012345678901
This number equals the number of zeroes you need to place after a 1 in the 2
12345678901234567890123456789012123456789012345678901
12345678901234567890123456789012123456789012345678901
denominator. 0.7 has only one space to the right, so you put only one zero 2
2
12345678901234567890123456789012123456789012345678901
12345678901234567890123456789012123456789012345678901
after the 1, giving you 10 in the denominator. The decimal 0.765 has three 2
2
1
2
12345678901234567890123456789012123456789012345678901
spaces, so you put three zeroes, giving you 1,000 in the denominator.
2
12345678901234567890123456789012123456789012345678901
2
12345678901234567890123456789012123456789012345678901
All the numbers to the right of the decimal point have place values that end 2
12345678901234567890123456789012123456789012345678901
2
12345678901234567890123456789012123456789012345678901
in the suffix -th. The decimal 0.7 is expressed seven tenths, while 0.25 2
12345678901234567890123456789012123456789012345678901
2
12345678901234567890123456789012123456789012345678901
is twenty-five hundredths. Place values on either side of the decimal 2
12345678901234567890123456789012123456789012345678901
2
12345678901234567890123456789012123456789012345678901
point might crop up, so its good to have a quick review:
2
12345678901234567890123456789012123456789012345678901
2345678901234567890123456789012123456789012345678901
12345678901234567890123456789012123456789012345678901 22
12345678901234567890123456789012123456789012345678901
2
1 Take the number 934.167:
12345678901234567890123456789012123456789012345678901 2
2
12345678901234567890123456789012123456789012345678901
12345678901234567890123456789011234567
2
12345678901234567890123456789012123456789012345678901
123456789012345678901234567890
9 is in the hundreds place, so the 9 means1
900
11234567
2
12345678901234567890123456789012123456789012345678901
123456789012345678901234567890
1234567
12345678901234567890123456789011234567
2
12345678901234567890123456789012123456789012345678901
2
12345678901234567890123456789012123456789012345678901
3 is in the tens place, so the 3 means 30
12345678901234567890123456789012123456789012345678901 2
2345678901234567890123456789012123456789012345678901
2
12345678901234567890123456789012123456789012345678901
4 is in the units place, so the 4 means 4
2
12345678901234567890123456789012123456789012345678901
12345678901234567890123456789012123456789012345678901 22
12345678901234567890123456789012123456789012345678901
2
1
1 is in the tenths place, so the 1 means 1
2
12345678901234567890123456789012123456789012345678901
10
2
12345678901234567890123456789012123456789012345678901
2
12345678901234567890123456789012123456789012345678901
2
12345678901234567890123456789012123456789012345678901
6
6
is
in
the
hundredths
place,
so
the
6
means
12345678901234567890123456789012123456789012345678901 2
2345678901234567890123456789012123456789012345678901
2
12345678901234567890123456789012123456789012345678901
100
2
1
2
12345678901234567890123456789012123456789012345678901
2
12345678901234567890123456789012123456789012345678901
7
7
is
in
the
thousands
place,
so
the
7
means
2
12345678901234567890123456789012123456789012345678901
2345678901234567890123456789012123456789012345678901
2
1
1,000
2
12345678901234567890123456789012123456789012345678901
12345678901234567890123456789012123456789012345678901 22
12345678901234567890123456789012123456789012345678901
123456789012345678901234567890121234567890123456789012 57

http://www.xtremepapers.net

Part III: Section Review

www.petersons.com

http://www.xtremepapers.net

Alert!

58

123456789012345678901234567890121234567890123456789012
12345678901234567890123456789012123456789012345678901 2
2
12345678901234567890123456789012123456789012345678901
12345678901234567890123456789012123456789012345678901 2
2
12345678901234567890123456789012123456789012345678901
2
12345678901234567890123456789012123456789012345678901
A
tricky
part
about
place
values
and
decimals
is
that
there
is
no
unit-ths
12345678901234567890123456789012123456789012345678901 2
12345678901234567890123456789012123456789012345678901
value. In this example, the 4 is four units and the number to the left of 2
2
12345678901234567890123456789012123456789012345678901
12345678901234567890123456789012123456789012345678901
it is the tens. However, the number immediately to the right of 4over 2
2
12345678901234567890123456789012123456789012345678901
2
12345678901234567890123456789012123456789012345678901
the decimal point divideis the tenths, not unit-ths.
2
12345678901234567890123456789012123456789012345678901
2345678901234567890123456789012123456789012345678901
2
1
2
12345678901234567890123456789012123456789012345678901
2
12345678901234567890123456789012123456789012345678901
When you add and subtract with decimals you need to make sure you line 2
12345678901234567890123456789012123456789012345678901
2
12345678901234567890123456789012123456789012345678901
up the decimal points. For instance, 345.4 minus 202.45 is:
2
12345678901234567890123456789012123456789012345678901
2
12345678901234567890123456789012123456789012345678901
2
12345678901234567890123456789012123456789012345678901
345.4
2
12345678901234567890123456789012123456789012345678901
2
12345678901234567890123456789012123456789012345678901
2345678901234567890123456789012123456789012345678901
2
1
2 202.45
2
12345678901234567890123456789012123456789012345678901
2
12345678901234567890123456789012123456789012345678901
When multiplying decimals, do the multiplication as if the decimal points 2
12345678901234567890123456789012123456789012345678901
2
12345678901234567890123456789012123456789012345678901
werent there. Just ignore them for starters. Once you have the product, 2
12345678901234567890123456789012123456789012345678901
2
12345678901234567890123456789012123456789012345678901
count how many decimal places you have behind the decimal in both 2
12345678901234567890123456789012123456789012345678901
2
12345678901234567890123456789012123456789012345678901
factors, and then put the decimal that many places into the product. If that 2
12345678901234567890123456789012123456789012345678901
2345678901234567890123456789012123456789012345678901
2
12345678901234567890123456789012123456789012345678901
1 sounds confusing, in application its not. For instance, 3.5 3 0.5 175 2
12345678901234567890123456789012123456789012345678901
(multiplying forgetting the decimals). Or, there are two places behind the 2
2
12345678901234567890123456789012123456789012345678901
2
12345678901234567890123456789012123456789012345678901
decimal
in
the
two
factors,
one
for
each
factor.
Putting
two
numbers
2345678901234567890123456789012123456789012345678901
12345678901234567890123456789012123456789012345678901 2
12345678901234567890123456789012123456789012345678901
behind the decimal in the product gives you 1.75. If you change the initial 2
2
12345678901234567890123456789012123456789012345678901
2
12345678901234567890123456789012123456789012345678901
numbers
to
0.35
and
0.005,
you
still
multiply
and
get
175
to
start.
But
12345678901234567890123456789012123456789012345678901 2
12345678901234567890123456789012123456789012345678901
now there are five spaces to the left of the decimal point (two in 0.35 and 2
2
12345678901234567890123456789012123456789012345678901
2
12345678901234567890123456789012123456789012345678901
three
in
0.005),
so
your
final
answer
will
be
0.00175
for
that
problem.
12345678901234567890123456789012123456789012345678901 2
2
1
12345678901234567890123456789012123456789012345678901
To divide a decimal by a decimal, first move the decimal in the divisor until 2
2
12345678901234567890123456789012123456789012345678901
12345678901234567890123456789012123456789012345678901
it is a whole number, and then move the decimal point as many spaces in 2
12345678901234567890123456789012123456789012345678901 2
12345678901234567890123456789012123456789012345678901
the number being divided. The expression 2.3 4 0.5 would look like this 2
2
12345678901234567890123456789012123456789012345678901
2
12345678901234567890123456789012123456789012345678901
23.0
in
long
division
form.
You
would
have
to
move
the
decimal
one
5
=
12345678901234567890123456789012123456789012345678901 2
12345678901234567890123456789012123456789012345678901
space to the left to transform 2.3 into the integer 23, so you must move 2
2
12345678901234567890123456789012123456789012345678901
2345678901234567890123456789012123456789012345678901
12345678901234567890123456789012123456789012345678901
the divisor one space to the left as well, making 0.5 into 5. Now do the 2
2
12345678901234567890123456789012123456789012345678901
1 division, and put the decimal in the quotient right where it lies in the 2
12345678901234567890123456789012123456789012345678901 2
2
12345678901234567890123456789012123456789012345678901
4.6
2
12345678901234567890123456789012123456789012345678901
2
12345678901234567890123456789012123456789012345678901
number being divided, 5q23.0.
2
12345678901234567890123456789012123456789012345678901
2
12345678901234567890123456789012123456789012345678901
12345678901234567890123456789012123456789012345678901 2
2345678901234567890123456789012123456789012345678901
12345678901234567890123456789012123456789012345678901 2
12345678901234567890123456789012123456789012345678901 2
12345678901234567890123456789012123456789012345678901 2
12345678901234567890123456789012123456789012345678901 2
2
1
2
12345678901234567890123456789012123456789012345678901
2
12345678901234567890123456789012123456789012345678901
2
12345678901234567890123456789012123456789012345678901
2
12345678901234567890123456789012123456789012345678901
12345678901234567890123456789012123456789012345678901 2
2345678901234567890123456789012123456789012345678901
12345678901234567890123456789012123456789012345678901 2
2
1
2
12345678901234567890123456789012123456789012345678901
12345678901234567890123456789012123456789012345678901 2
2
12345678901234567890123456789012123456789012345678901
12345678901234567890123456789012123456789012345678901 2
12345678901234567890123456789012123456789012345678901 2
12345678901234567890123456789012123456789012345678901 2
12345678901234567890123456789012123456789012345678901 2
123456789012345678901234567890121234567890123456789012

Chapter 4: Quantitative Review

123456789012345678901234567890121234567890123456789012
12345678901234567890123456789012123456789012345678901 2
2
12345678901234567890123456789012123456789012345678901
a
2
12345678901234567890123456789012123456789012345678901
12345678901234567890123456789012123456789012345678901
If you need to convert a fraction into a decimal, remember that means 2
2
12345678901234567890123456789012123456789012345678901
b
2
12345678901234567890123456789012123456789012345678901
a
4
b.
So
if
you
are
given
a
fraction
and
you
need
to
know
its
value
in
2
12345678901234567890123456789012123456789012345678901
2345678901234567890123456789012123456789012345678901
1 decimal form, divide the top by the bottom, and you have it. For example, 2
2
12345678901234567890123456789012123456789012345678901
2
12345678901234567890123456789012123456789012345678901
2
2
12345678901234567890123456789012123456789012345678901
is
in
decimal
form,
divide
2
by
7,
if
you
need
to
know
what
2
12345678901234567890123456789012123456789012345678901
7
2345678901234567890123456789012123456789012345678901
2
1
2
12345678901234567890123456789012123456789012345678901
2
12345678901234567890123456789012123456789012345678901
0.28
2
12345678901234567890123456789012123456789012345678901
2
12345678901234567890123456789012123456789012345678901
7
2.00
q
2345678901234567890123456789012123456789012345678901
2
1
2
12345678901234567890123456789012123456789012345678901
14
2
12345678901234567890123456789012123456789012345678901
2
12345678901234567890123456789012123456789012345678901
2
12345678901234567890123456789012123456789012345678901
60
2345678901234567890123456789012123456789012345678901
2
1
2
12345678901234567890123456789012123456789012345678901
56
2
12345678901234567890123456789012123456789012345678901
2
12345678901234567890123456789012123456789012345678901
4
2
12345678901234567890123456789012123456789012345678901
2345678901234567890123456789012123456789012345678901
12345678901234567890123456789012123456789012345678901 22
12345678901234567890123456789012123456789012345678901
2
2
12345678901234567890123456789012123456789012345678901
' 0.28
2
12345678901234567890123456789012123456789012345678901
7
12345678901234567890123456789012123456789012345678901 22
12345678901234567890123456789012123456789012345678901 2
1
Column B
Column A
2
12345678901234567890123456789012123456789012345678901
2
12345678901234567890123456789012123456789012345678901
2
d, c, e are distinct prime numbers and
12345678901234567890123456789012123456789012345678901
2345678901234567890123456789012123456789012345678901
2
12345678901234567890123456789012123456789012345678901
d 1 e 5 c. They are also part of the six
2
12345678901234567890123456789012123456789012345678901
2
12345678901234567890123456789012123456789012345678901
digit number 54de.3c.
2
12345678901234567890123456789012123456789012345678901
12345678901234567890123456789012123456789012345678901 22
12345678901234567890123456789012123456789012345678901
The value in the
The value in the
2
12345678901234567890123456789012123456789012345678901
2
12345678901234567890123456789012123456789012345678901
tens
place
in
the
tenths
place
in
the
12345678901234567890123456789012123456789012345678901 22
1
6-digit number
6-digit number
2
12345678901234567890123456789012123456789012345678901
2
12345678901234567890123456789012123456789012345678901
12345678901234567890123456789012123456789012345678901 2
12345678901234567890123456789012123456789012345678901
The test is trying to confuse you with math speak. Dont let it. Distinct 2
2
12345678901234567890123456789012123456789012345678901
2345678901234567890123456789012123456789012345678901
12345678901234567890123456789012123456789012345678901
prime numbers are prime numbers that are different from each other, so 2
2
12345678901234567890123456789012123456789012345678901
1 d e c. You can also infer that d, c, e must be 2, 3, 5, or 7 since those are 2
2
12345678901234567890123456789012123456789012345678901
2
all the single digit prime numbers.
12345678901234567890123456789012123456789012345678901
2345678901234567890123456789012123456789012345678901
12345678901234567890123456789012123456789012345678901 22
12345678901234567890123456789012123456789012345678901
1 Lets look at Column A and try to pin it down. The value in the tens place 2
12345678901234567890123456789012123456789012345678901
is d. With what you know, d 1 e 5 c could be 2 1 3 5 5 or 2 1 5 5 7, but 2
2
12345678901234567890123456789012123456789012345678901
2345678901234567890123456789012123456789012345678901
12345678901234567890123456789012123456789012345678901
theres no way to tell which equation is valid. Therefore, d could be 2, 3, 2
2
12345678901234567890123456789012123456789012345678901
2
12345678901234567890123456789012123456789012345678901
or
5.
That
narrows
it
down
as
much
as
possible,
so
its
time
to
turn
to
12345678901234567890123456789012123456789012345678901 22
1 Column B. The tenths place is the one just to the right of the decimal,
2
12345678901234567890123456789012123456789012345678901
2
12345678901234567890123456789012123456789012345678901
which
is
3.
2
12345678901234567890123456789012123456789012345678901
2
12345678901234567890123456789012123456789012345678901
12345678901234567890123456789012123456789012345678901
This gives us three possibilities. Depending on ds value, it could be less 2
2345678901234567890123456789012123456789012345678901
12345678901234567890123456789012123456789012345678901 22
12345678901234567890123456789012123456789012345678901
than, equal to, or greater than 3. This makes Column A less than, equal to,
2
12345678901234567890123456789012123456789012345678901
12345678901234567890123456789012123456789012345678901
or greater than Column B, depending on what value you use for d. 2
2
1
2
12345678901234567890123456789012123456789012345678901
Therefore, choice (D) is the correct answer.
2
12345678901234567890123456789012123456789012345678901
2
12345678901234567890123456789012123456789012345678901
12345678901234567890123456789012123456789012345678901 2
2
12345678901234567890123456789012123456789012345678901
12345678901234567890123456789012123456789012345678901 2
12345678901234567890123456789012123456789012345678901 2
12345678901234567890123456789012123456789012345678901 2
12345678901234567890123456789012123456789012345678901 2 59
123456789012345678901234567890121234567890123456789012

http://www.xtremepapers.net

Part III: Section Review

60

123456789012345678901234567890121234567890123456789012
12345678901234567890123456789012123456789012345678901 2
2
12345678901234567890123456789012123456789012345678901
2
12345678901234567890123456789012123456789012345678901
Exponents(Those Tiny Raised Numbers) and Roots
2
12345678901234567890123456789012123456789012345678901
2
12345678901234567890123456789012123456789012345678901
Exponents
can
be
a
little
weird,
and
thats
putting
it
nicely.
A
basic
2
12345678901234567890123456789012123456789012345678901
2
2
12345678901234567890123456789012123456789012345678901
exponent
looks
like
4
,
with
the
integer
4
called
the
base
and
the
number
2
2345678901234567890123456789012123456789012345678901
2
1
2
12345678901234567890123456789012123456789012345678901
called the power, or exponent. The expression 4 is math shorthand for 2
2
12345678901234567890123456789012123456789012345678901
12345678901234567890123456789012123456789012345678901
4 3 4, which might make you think people have to be in a real hurry to use 2
2
12345678901234567890123456789012123456789012345678901
12345678901234567890123456789012123456789012345678901
that shorthand. Exponents come in handy when you deal with numbers 2
2
12345678901234567890123456789012123456789012345678901
2
12345678901234567890123456789012123456789012345678901
like 4350, since writing out 350 fours is a bit time-consuming.
2
12345678901234567890123456789012123456789012345678901
2
12345678901234567890123456789012123456789012345678901
While
you
might
be
using
some
arithmeticadding,
subtracting,
fractions,
2345678901234567890123456789012123456789012345678901
2
1
12345678901234567890123456789012123456789012345678901
and decimalsin your everyday life, youre usually not walking around 2
2
12345678901234567890123456789012123456789012345678901
2
12345678901234567890123456789012123456789012345678901
th
power.
Here
is
a
condensed
version
of
how
raising
values
to
the
n
2
12345678901234567890123456789012123456789012345678901
2345678901234567890123456789012123456789012345678901
1 exponents work. Look it over, play with them (plug in numbers and such), 2
2
12345678901234567890123456789012123456789012345678901
2
12345678901234567890123456789012123456789012345678901
and
get
comfortable
with
them.
2
12345678901234567890123456789012123456789012345678901
2
12345678901234567890123456789012123456789012345678901
2345678901234567890123456789012123456789012345678901
12345678901234567890123456789012123456789012345678901 2
12345678901234567890123456789012123456789012345678901 2
2
12345678901234567890123456789012123456789012345678901
Exponent Refresher
2
12345678901234567890123456789012123456789012345678901
0
2
12345678901234567890123456789012123456789012345678901
x
5
1
2
12345678901234567890123456789012123456789012345678901
1
2
1
x
5
x
2
12345678901234567890123456789012123456789012345678901
2
12345678901234567890123456789012123456789012345678901
2
2
12345678901234567890123456789012123456789012345678901
x 5x3x
2345678901234567890123456789012123456789012345678901
2
12345678901234567890123456789012123456789012345678901
2
12345678901234567890123456789012123456789012345678901
x3 5 x 3 x 3 x
12345678901234567890123456789012123456789012345678901 2
2
12345678901234567890123456789012123456789012345678901
You get the picture. But just for good measure, heres one more:
2
12345678901234567890123456789012123456789012345678901
2
12345678901234567890123456789012123456789012345678901
3
4
2
12345678901234567890123456789012123456789012345678901
xx
5
x
12345678901234567890123456789012123456789012345678901 2
12345678901234567890123456789012123456789012345678901 2
2
1 Exponents can also be negative.
2
12345678901234567890123456789012123456789012345678901
2
12345678901234567890123456789012123456789012345678901
1
2
12345678901234567890123456789012123456789012345678901
21
5
x
2
12345678901234567890123456789012123456789012345678901
x
2
12345678901234567890123456789012123456789012345678901
2345678901234567890123456789012123456789012345678901
12345678901234567890123456789012123456789012345678901 2
2
12345678901234567890123456789012123456789012345678901
1
1
22
2
1
x
5
5
2
2
12345678901234567890123456789012123456789012345678901
x
3
x
x
2
12345678901234567890123456789012123456789012345678901
2345678901234567890123456789012123456789012345678901
12345678901234567890123456789012123456789012345678901 2
2
12345678901234567890123456789012123456789012345678901
1 You can add and subtract numbers with exponents if they have the same 2
2
12345678901234567890123456789012123456789012345678901
base and are raised to the same power.
2
12345678901234567890123456789012123456789012345678901
2345678901234567890123456789012123456789012345678901
2
12345678901234567890123456789012123456789012345678901
2
2
2
2
12345678901234567890123456789012123456789012345678901
1
2x
5
5x
3x
12345678901234567890123456789012123456789012345678901 2
2
12345678901234567890123456789012123456789012345678901
2x2 2 x2 5 x2
2
1
2345678901234567890123456789012123456789012345678901
2
12345678901234567890123456789012123456789012345678901
3
2
3
2
12345678901234567890123456789012123456789012345678901
but
3x
1
3x

6x
since
they
are
not
raised
to
the
same
power.
12345678901234567890123456789012123456789012345678901 2
12345678901234567890123456789012123456789012345678901 2
2
1 You can also multiply and divide numbers if they have the same base.
2
12345678901234567890123456789012123456789012345678901
2
12345678901234567890123456789012123456789012345678901
a b
a1b
2
12345678901234567890123456789012123456789012345678901
x x 5x
2
12345678901234567890123456789012123456789012345678901
2
12345678901234567890123456789012123456789012345678901
2
3
5
2
3
For
instance,
x
3
x
5
x
.
This
makes
sense
if
you
see
that
x
3
x
5
2345678901234567890123456789012123456789012345678901
12345678901234567890123456789012123456789012345678901 2
2
1 (x 3 x)(x 3 x 3 x) 5 x5.
2
12345678901234567890123456789012123456789012345678901
2
12345678901234567890123456789012123456789012345678901
a
2
12345678901234567890123456789012123456789012345678901
x
a
2
b
2345678901234567890123456789012123456789012345678901
2
1
5x
2
12345678901234567890123456789012123456789012345678901
b
x
2
12345678901234567890123456789012123456789012345678901
12345678901234567890123456789012123456789012345678901 2
123456789012345678901234567890121234567890123456789012

www.petersons.com

http://www.xtremepapers.net

Chapter 4: Quantitative Review

123456789012345678901234567890121234567890123456789012
12345678901234567890123456789012123456789012345678901 2
2
12345678901234567890123456789012123456789012345678901
For example,
2
12345678901234567890123456789012123456789012345678901
2
12345678901234567890123456789012123456789012345678901
2
12345678901234567890123456789012123456789012345678901
3
2
12345678901234567890123456789012123456789012345678901
x
3
2
2
2
12345678901234567890123456789012123456789012345678901
5
x
5
x
2345678901234567890123456789012123456789012345678901
2
1
x2
2
12345678901234567890123456789012123456789012345678901
2
12345678901234567890123456789012123456789012345678901
2
12345678901234567890123456789012123456789012345678901
This
also
makes
sense
if
you
see
that
2
12345678901234567890123456789012123456789012345678901
2345678901234567890123456789012123456789012345678901
2
1
2
12345678901234567890123456789012123456789012345678901
3
x
xxx
2
12345678901234567890123456789012123456789012345678901
5
5x
2
12345678901234567890123456789012123456789012345678901
2
xx
2
12345678901234567890123456789012123456789012345678901
x
2345678901234567890123456789012123456789012345678901
2
1
2
12345678901234567890123456789012123456789012345678901
You can also raise a number with an exponent to a power. In gambling 2
12345678901234567890123456789012123456789012345678901
2
12345678901234567890123456789012123456789012345678901
terms, this would be like saying, Ill see your exponent, and raise you 2
12345678901234567890123456789012123456789012345678901
2345678901234567890123456789012123456789012345678901
2
1 another.
2
12345678901234567890123456789012123456789012345678901
2
12345678901234567890123456789012123456789012345678901
a b
ab
2
12345678901234567890123456789012123456789012345678901
)
5
x
(x
2
12345678901234567890123456789012123456789012345678901
2345678901234567890123456789012123456789012345678901
12345678901234567890123456789012123456789012345678901 22
12345678901234567890123456789012123456789012345678901
So,
2
12345678901234567890123456789012123456789012345678901
2
12345678901234567890123456789012123456789012345678901
1
2
3
6
23
3
29
2
12345678901234567890123456789012123456789012345678901
(x
)
5
x
or
(x
)
5
x
5
9
2
12345678901234567890123456789012123456789012345678901
x
2
1
2
12345678901234567890123456789012123456789012345678901
Column A
Column B
2
12345678901234567890123456789012123456789012345678901
2
12345678901234567890123456789012123456789012345678901
2345678901234567890123456789012123456789012345678901
2
12345678901234567890123456789012123456789012345678901
a and b are positive distinct integers.
2
12345678901234567890123456789012123456789012345678901
2
12345678901234567890123456789012123456789012345678901
22
2
2
2
2
12345678901234567890123456789012123456789012345678901
2a
4b
10
b
~
!
~
!
~
!
2
12345678901234567890123456789012123456789012345678901
2
2
12345678901234567890123456789012123456789012345678901
a2b23
5~2a! a2a2
2
12345678901234567890123456789012123456789012345678901
12345678901234567890123456789012123456789012345678901 22
12345678901234567890123456789012123456789012345678901
1 Here is another problem filled with complex-looking math things, 2
12345678901234567890123456789012123456789012345678901
standard issue for the GRE math problem visual. At first, dont worry 2
2
12345678901234567890123456789012123456789012345678901
12345678901234567890123456789012123456789012345678901
about the positive distinct integer bit. Just simplify the expressions step 2
12345678901234567890123456789012123456789012345678901 2
2
12345678901234567890123456789012123456789012345678901
by step.
2
12345678901234567890123456789012123456789012345678901
2
12345678901234567890123456789012123456789012345678901
Column B
Column A
2
12345678901234567890123456789012123456789012345678901
2
12345678901234567890123456789012123456789012345678901
2
12345678901234567890123456789012123456789012345678901
22
2
2
2
2345678901234567890123456789012123456789012345678901
2
12345678901234567890123456789012123456789012345678901
10(b )
~2a ! ~4b!
2
12345678901234567890123456789012123456789012345678901
2 23
2 2 2
2
1
a
b
5(2a)
a
a
12345678901234567890123456789012123456789012345678901 2
12345678901234567890123456789012123456789012345678901 2
2
12345678901234567890123456789012123456789012345678901
4b~b3!
10b4
2
12345678901234567890123456789012123456789012345678901
2
12345678901234567890123456789012123456789012345678901
5 2
5
2
2
2
4
2
12345678901234567890123456789012123456789012345678901
a
2a
5
4a
a
~
!
~
!
12345678901234567890123456789012123456789012345678901 2
2345678901234567890123456789012123456789012345678901
12345678901234567890123456789012123456789012345678901 22
12345678901234567890123456789012123456789012345678901
4b4
10b4
2
12345678901234567890123456789012123456789012345678901
5
5
2
12345678901234567890123456789012123456789012345678901
6
6
2
1
4a
20a
2345678901234567890123456789012123456789012345678901
12345678901234567890123456789012123456789012345678901 22
12345678901234567890123456789012123456789012345678901
2
12345678901234567890123456789012123456789012345678901
b4
b4
2
12345678901234567890123456789012123456789012345678901
5
5
2
1
6
6
a
2a
2345678901234567890123456789012123456789012345678901
12345678901234567890123456789012123456789012345678901 22
1
2
12345678901234567890123456789012123456789012345678901
12345678901234567890123456789012123456789012345678901 2
12345678901234567890123456789012123456789012345678901
Now that youve done the work, you can go back to the positive distinct 2
2
12345678901234567890123456789012123456789012345678901
2
12345678901234567890123456789012123456789012345678901
integers
phrase.
Since
the
two
variables
are
positive
integers,
the
12345678901234567890123456789012123456789012345678901 22
12345678901234567890123456789012123456789012345678901
123456789012345678901234567890121234567890123456789012 61

http://www.xtremepapers.net

Part III: Section Review

www.petersons.com

http://www.xtremepapers.net

Tip

62

123456789012345678901234567890121234567890123456789012
12345678901234567890123456789012123456789012345678901 2
2
12345678901234567890123456789012123456789012345678901
denominator in Column A is smaller than the denominator in Column B, 2
12345678901234567890123456789012123456789012345678901
2
12345678901234567890123456789012123456789012345678901
while the numerators are equal. Try substituting some small positive 2
12345678901234567890123456789012123456789012345678901
2
12345678901234567890123456789012123456789012345678901
integers into the equation if you like. If the denominator in Column B is 2
12345678901234567890123456789012123456789012345678901
2345678901234567890123456789012123456789012345678901
2
1
1 1
2
12345678901234567890123456789012123456789012345678901
greater, this means that Column A is greater in value, since . . Using 2
12345678901234567890123456789012123456789012345678901
2 8
2
12345678901234567890123456789012123456789012345678901
2
12345678901234567890123456789012123456789012345678901
these
simpler
values,
you
can
see
how
the
numerators
are
equal
and
the
2345678901234567890123456789012123456789012345678901
2
1
2
12345678901234567890123456789012123456789012345678901
fraction
with
the
smaller
denominator
is
the
largest
value.
2
12345678901234567890123456789012123456789012345678901
2
12345678901234567890123456789012123456789012345678901
2
12345678901234567890123456789012123456789012345678901
2345678901234567890123456789012123456789012345678901
2
1
2
12345678901234567890123456789012123456789012345678901
If
the
variables
werent
distinct
they
could
both
equal
1,
and
the
two
2
12345678901234567890123456789012123456789012345678901
2
12345678901234567890123456789012123456789012345678901
fractions
would
be
equal.
2
12345678901234567890123456789012123456789012345678901
2345678901234567890123456789012123456789012345678901
2
1
2
12345678901234567890123456789012123456789012345678901
2
12345678901234567890123456789012123456789012345678901
4
2
12345678901234567890123456789012123456789012345678901
2
2
12345678901234567890123456789012123456789012345678901
equal?
What
does
2345678901234567890123456789012123456789012345678901
2
12345678901234567890123456789012123456789012345678901
4243
2
12345678901234567890123456789012123456789012345678901
2
12345678901234567890123456789012123456789012345678901
26
2
12345678901234567890123456789012123456789012345678901
A. 2
2
12345678901234567890123456789012123456789012345678901
23
2
12345678901234567890123456789012123456789012345678901
B. 2
2
1
21
2
12345678901234567890123456789012123456789012345678901
C. 2
2
12345678901234567890123456789012123456789012345678901
2
2
12345678901234567890123456789012123456789012345678901
D. 4
2345678901234567890123456789012123456789012345678901
2
12345678901234567890123456789012123456789012345678901
E. 25
2
12345678901234567890123456789012123456789012345678901
12345678901234567890123456789012123456789012345678901 2
2
12345678901234567890123456789012123456789012345678901
One way to do this is to raise each number to its power, multiply where 2
12345678901234567890123456789012123456789012345678901
12345678901234567890123456789012123456789012345678901
needed, and then simplify. You might be able to work this method on this 2
2
12345678901234567890123456789012123456789012345678901
2
12345678901234567890123456789012123456789012345678901
problem,
but
if
the
numbers
are
raised
to
higher
powers,
this
method
12345678901234567890123456789012123456789012345678901 2
1 becomes very difficult very quickly. The other way is to try to make the 2
2
12345678901234567890123456789012123456789012345678901
2
12345678901234567890123456789012123456789012345678901
numbers
in
the
numerator
and
the
denominator
have
the
same
base
so
that
12345678901234567890123456789012123456789012345678901 2
2
12345678901234567890123456789012123456789012345678901
you can simplify directly. Since 4 5 (2)(2) then
2
12345678901234567890123456789012123456789012345678901
2345678901234567890123456789012123456789012345678901
2
12345678901234567890123456789012123456789012345678901
2
2
2 2
4
2
12345678901234567890123456789012123456789012345678901
4 5 [(2)(2)] 5 2 2 5 2 ,
2
1
2
12345678901234567890123456789012123456789012345678901
2
12345678901234567890123456789012123456789012345678901
and
2345678901234567890123456789012123456789012345678901
12345678901234567890123456789012123456789012345678901 2
2
12345678901234567890123456789012123456789012345678901
43 5 [(2)(2)]3 5 2323 5 26
2
1
12345678901234567890123456789012123456789012345678901 2
2
12345678901234567890123456789012123456789012345678901
So we can rewrite the fraction like this,
2
12345678901234567890123456789012123456789012345678901
2
12345678901234567890123456789012123456789012345678901
4
4
4
2
12345678901234567890123456789012123456789012345678901
2
2
2
2
12345678901234567890123456789012123456789012345678901
5
5
4 6
10
2
12345678901234567890123456789012123456789012345678901
2 3
2
2
2
4
4
2345678901234567890123456789012123456789012345678901
12345678901234567890123456789012123456789012345678901 2
12345678901234567890123456789012123456789012345678901 2
2
12345678901234567890123456789012123456789012345678901
This can be simplified straightforwardly,
2
12345678901234567890123456789012123456789012345678901
2
1
4
2345678901234567890123456789012123456789012345678901
2
12345678901234567890123456789012123456789012345678901
2
4 210
26
2
12345678901234567890123456789012123456789012345678901
5
2
2
5
2
10
2
12345678901234567890123456789012123456789012345678901
2
12345678901234567890123456789012123456789012345678901 2
1 You could also have transformed the numerator into a base of 4. Either 2
2
12345678901234567890123456789012123456789012345678901
2
12345678901234567890123456789012123456789012345678901
method would lead to the right answer, although since a majority of the 2
12345678901234567890123456789012123456789012345678901
2
12345678901234567890123456789012123456789012345678901
answer choices have a base of 2, transforming everything into that base 2
12345678901234567890123456789012123456789012345678901
2345678901234567890123456789012123456789012345678901
2
1
12345678901234567890123456789012123456789012345678901 2
12345678901234567890123456789012123456789012345678901 2
12345678901234567890123456789012123456789012345678901 2
123456789012345678901234567890121234567890123456789012

Chapter 4: Quantitative Review

123456789012345678901234567890121234567890123456789012
12345678901234567890123456789012123456789012345678901 2
2
12345678901234567890123456789012123456789012345678901
increases your chances of simply coming up with the right answer without 2
12345678901234567890123456789012123456789012345678901
2
12345678901234567890123456789012123456789012345678901
having to do one last conversion.
2
12345678901234567890123456789012123456789012345678901
12345678901234567890123456789012123456789012345678901 2
12345678901234567890123456789012123456789012345678901
The annoying relative of the exponent is the square root. It looks like this 2
2
12345678901234567890123456789012123456789012345678901
2
12345678901234567890123456789012123456789012345678901
12345678901234567890123456789012123456789012345678901
, and its the opposite of squaring a number. When you square 4, you 2
=
2
12345678901234567890123456789012123456789012345678901
multiply it by itself to get 16, but when you take the square root of 4, you 2
12345678901234567890123456789012123456789012345678901
2345678901234567890123456789012123456789012345678901
2
1
2
12345678901234567890123456789012123456789012345678901
get
the
number
that
times
itself
gives
4(2
3
2)
5
4,
so
4
52.
Square
=
2
12345678901234567890123456789012123456789012345678901
1
2
12345678901234567890123456789012123456789012345678901
2
2
roots can also be written as exponents, =x 5 x .
12345678901234567890123456789012123456789012345678901
2345678901234567890123456789012123456789012345678901
2
1
2
12345678901234567890123456789012123456789012345678901
Even more annoying than the square root is the cube root. Just look at it, 2
12345678901234567890123456789012123456789012345678901
2
12345678901234567890123456789012123456789012345678901
3
12345678901234567890123456789012123456789012345678901
. Something like this practically screams convoluted math process 2
=
2345678901234567890123456789012123456789012345678901
2
1
12345678901234567890123456789012123456789012345678901
up ahead. When you cube something, you multiply it by itself twice (so 2
2
12345678901234567890123456789012123456789012345678901
the cube of 3 is 33 5 3 3 3 3 3 5 27). The cube root is the opposite, 2
12345678901234567890123456789012123456789012345678901
2
12345678901234567890123456789012123456789012345678901
3
3
12345678901234567890123456789012123456789012345678901
27 5 3 or =8 5 2 because 2 3 2 3 2 5 8. Cube roots are written as 2
=
2
12345678901234567890123456789012123456789012345678901
1
3
2
12345678901234567890123456789012123456789012345678901
2
12345678901234567890123456789012123456789012345678901
exponents like this, =x 5 x3.
2
12345678901234567890123456789012123456789012345678901
2345678901234567890123456789012123456789012345678901
2
12345678901234567890123456789012123456789012345678901
2
25
2
1
2
2
12345678901234567890123456789012123456789012345678901
3!~x12!#
~
!~
@
2x
x
2
12345678901234567890123456789012123456789012345678901
If
x
5
3
then
3
2
12345678901234567890123456789012123456789012345678901
2345678901234567890123456789012123456789012345678901
2
12345678901234567890123456789012123456789012345678901
4~x4!
2
12345678901234567890123456789012123456789012345678901
2
12345678901234567890123456789012123456789012345678901
A.
1
12345678901234567890123456789012123456789012345678901 22
12345678901234567890123456789012123456789012345678901
B. =3
2
12345678901234567890123456789012123456789012345678901
2
12345678901234567890123456789012123456789012345678901
C. 3
2
12345678901234567890123456789012123456789012345678901
2
12345678901234567890123456789012123456789012345678901
D.
27
2
1
2345678901234567890123456789012123456789012345678901
2
12345678901234567890123456789012123456789012345678901
E. 81
2
12345678901234567890123456789012123456789012345678901
2
1
2
12345678901234567890123456789012123456789012345678901
There
is
good
news
and
bad
news
about
this
problem.
The
good
news
is
2
12345678901234567890123456789012123456789012345678901
2345678901234567890123456789012123456789012345678901
12345678901234567890123456789012123456789012345678901
that with a problem like this, there are no hidden tricks. The bad news is 2
2
12345678901234567890123456789012123456789012345678901
1 that you have to work this problem out step by step, but you know 2
2
12345678901234567890123456789012123456789012345678901
2
12345678901234567890123456789012123456789012345678901
everything
you
need
to
know
to
do
that.
To
the
solving,
then!
2345678901234567890123456789012123456789012345678901
12345678901234567890123456789012123456789012345678901 22
12345678901234567890123456789012123456789012345678901
2
25
2
2
1
2 25
3
2
12345678901234567890123456789012123456789012345678901
3!~x12!
~
!
~
2x
x
2
2
2
12345678901234567890123456789012123456789012345678901
3x12x 4
5
x
x
3
2345678901234567890123456789012123456789012345678901
2
12345678901234567890123456789012123456789012345678901
4
2
12345678901234567890123456789012123456789012345678901
!
~
4 x
12345678901234567890123456789012123456789012345678901 22
12345678901234567890123456789012123456789012345678901
1 The four in the denominator is cancelled out by the 2 that gets squared. 2
2
12345678901234567890123456789012123456789012345678901
Now we need to find a common denominator for the exponents in order to 2
12345678901234567890123456789012123456789012345678901
2
12345678901234567890123456789012123456789012345678901
add and subtract these fractions. This process was discussed earlier on 2
12345678901234567890123456789012123456789012345678901
12345678901234567890123456789012123456789012345678901
page 62, and as you can see, straightforward concepts have a way of 2
2
12345678901234567890123456789012123456789012345678901
2
12345678901234567890123456789012123456789012345678901
showing
up
in
odd
places
on
a
GRE
math
question.
12
is
an
easy
choice.
2
12345678901234567890123456789012123456789012345678901
2
12345678901234567890123456789012123456789012345678901
2
12345678901234567890123456789012123456789012345678901
2 25
24 1 8 1 25 2 9
48
3
2
2
4
2345678901234567890123456789012123456789012345678901
2
12345678901234567890123456789012123456789012345678901
12
5 x12 5 x
x x3x12x 4 5 x
2
1
2
12345678901234567890123456789012123456789012345678901
2
12345678901234567890123456789012123456789012345678901
If
x
5
3
then
the
answer
is
81,
or
(E).
2
12345678901234567890123456789012123456789012345678901
2345678901234567890123456789012123456789012345678901
2
1
12345678901234567890123456789012123456789012345678901 2
12345678901234567890123456789012123456789012345678901 2
12345678901234567890123456789012123456789012345678901 2 63
123456789012345678901234567890121234567890123456789012

http://www.xtremepapers.net

Part III: Section Review

S D

64

www.petersons.com

http://www.xtremepapers.net

Note

123456789012345678901234567890121234567890123456789012
12345678901234567890123456789012123456789012345678901 2
2
12345678901234567890123456789012123456789012345678901
12345678901234567890123456789012123456789012345678901 2
2
12345678901234567890123456789012123456789012345678901
Common Exponents to Know
2
12345678901234567890123456789012123456789012345678901
2
12345678901234567890123456789012123456789012345678901
3
4
5
2
12345678901234567890123456789012123456789012345678901
2
5
8,
2
5
16,
2
5
32
2345678901234567890123456789012123456789012345678901
2
1
2
12345678901234567890123456789012123456789012345678901
3
4
3 5 27, 3 5 81
2
12345678901234567890123456789012123456789012345678901
2
12345678901234567890123456789012123456789012345678901
3
2
12345678901234567890123456789012123456789012345678901
4 5 64
2345678901234567890123456789012123456789012345678901
2
1
3
2
12345678901234567890123456789012123456789012345678901
5
5
125
2
12345678901234567890123456789012123456789012345678901
2
12345678901234567890123456789012123456789012345678901
2
12345678901234567890123456789012123456789012345678901
2345678901234567890123456789012123456789012345678901
1 One more tip when dealing with exponents: Always remember that the 2
2
12345678901234567890123456789012123456789012345678901
2
2
12345678901234567890123456789012123456789012345678901
5
36,
its
base
can
be
a
negative
number.
If
a
problem
states,
x
2
12345678901234567890123456789012123456789012345678901
2
12345678901234567890123456789012123456789012345678901
common
to
assume
that
x
5
6.
This
is
a
tried-and-true
test
trap.
Without
2345678901234567890123456789012123456789012345678901
2
1
12345678901234567890123456789012123456789012345678901
any more information given in the problem, all you know is that x could be 2
2
12345678901234567890123456789012123456789012345678901
12345678901234567890123456789012123456789012345678901
6 or 26. Any number raised to an even power will yield a positive number, 2
2
12345678901234567890123456789012123456789012345678901
12345678901234567890123456789012123456789012345678901
but the base could be positive or negative. When the exponent is odd, the 2
2
12345678901234567890123456789012123456789012345678901
2
12345678901234567890123456789012123456789012345678901
base is the same sign as what it equals.
2
12345678901234567890123456789012123456789012345678901
2
12345678901234567890123456789012123456789012345678901
5
5
232
then
x
5
22.
For
instance,
if
x
2345678901234567890123456789012123456789012345678901
12345678901234567890123456789012123456789012345678901 2
2
1
2
12345678901234567890123456789012123456789012345678901
2
12345678901234567890123456789012123456789012345678901
2
12345678901234567890123456789012123456789012345678901
2345678901234567890123456789012123456789012345678901
2
12345678901234567890123456789012123456789012345678901
Percents Are about 4.6% of This Entire Chapter
2
12345678901234567890123456789012123456789012345678901
2
12345678901234567890123456789012123456789012345678901
Percents
are
not
an
alien
concept,
but
the
GRE
expects
a
greater
12345678901234567890123456789012123456789012345678901 2
12345678901234567890123456789012123456789012345678901
understanding of percents than what you need to navigate the local 2
2
12345678901234567890123456789012123456789012345678901
2
12345678901234567890123456789012123456789012345678901
grocery
store.
It
isnt
a
huge
amount
of
additional
information
that
you
12345678901234567890123456789012123456789012345678901 2
2
12345678901234567890123456789012123456789012345678901
1 need to understand, but a few things probably need to be ironed out in 2
2345678901234567890123456789012123456789012345678901
2
12345678901234567890123456789012123456789012345678901
your mind about percents. Lets get to ironing.
2
12345678901234567890123456789012123456789012345678901
2
1 Percent just means per hundred or divided by one hundred.
12345678901234567890123456789012123456789012345678901 2
2
12345678901234567890123456789012123456789012345678901
2
12345678901234567890123456789012123456789012345678901
27
2
12345678901234567890123456789012123456789012345678901
So 27% 5
, or
2
12345678901234567890123456789012123456789012345678901
100
2
12345678901234567890123456789012123456789012345678901
2
12345678901234567890123456789012123456789012345678901
2345678901234567890123456789012123456789012345678901
2
12345678901234567890123456789012123456789012345678901
0.6
6
2
12345678901234567890123456789012123456789012345678901
0.6% 5
5
2
1
100 1,000
12345678901234567890123456789012123456789012345678901 2
12345678901234567890123456789012123456789012345678901
If bananas are 40% off at the store, and their normal price is $3/pound, 2
2
12345678901234567890123456789012123456789012345678901
2
12345678901234567890123456789012123456789012345678901
then
their
sale
price
is
2
12345678901234567890123456789012123456789012345678901
2
12345678901234567890123456789012123456789012345678901
2
12345678901234567890123456789012123456789012345678901
(100%
2
40%)($3
pound)
5
(60%)
($3
pound)
2345678901234567890123456789012123456789012345678901
12345678901234567890123456789012123456789012345678901 2
2
12345678901234567890123456789012123456789012345678901
60
2
12345678901234567890123456789012123456789012345678901
$3 pound!
5
2
12345678901234567890123456789012123456789012345678901
~
2
100
1
2345678901234567890123456789012123456789012345678901
12345678901234567890123456789012123456789012345678901 2
2
12345678901234567890123456789012123456789012345678901
5 (0.6)($3 pound)
2
12345678901234567890123456789012123456789012345678901
2
12345678901234567890123456789012123456789012345678901
5
$1.80
pound
2
1
2345678901234567890123456789012123456789012345678901
12345678901234567890123456789012123456789012345678901 2
2
1
2
12345678901234567890123456789012123456789012345678901
12345678901234567890123456789012123456789012345678901 2
2
12345678901234567890123456789012123456789012345678901
12345678901234567890123456789012123456789012345678901 2
12345678901234567890123456789012123456789012345678901 2
12345678901234567890123456789012123456789012345678901 2
12345678901234567890123456789012123456789012345678901 2
123456789012345678901234567890121234567890123456789012

Chapter 4: Quantitative Review

123456789012345678901234567890121234567890123456789012
12345678901234567890123456789012123456789012345678901 2
2
12345678901234567890123456789012123456789012345678901
Notice that we multiplied the price times 60% because the sales price was 2
12345678901234567890123456789012123456789012345678901
2
12345678901234567890123456789012123456789012345678901
40% off. You could also have found out how much the bananas were 2
12345678901234567890123456789012123456789012345678901
2
12345678901234567890123456789012123456789012345678901
discounted due to the sale, and then subtracted that number from the 2
12345678901234567890123456789012123456789012345678901
2345678901234567890123456789012123456789012345678901
2
1 original price. This would give you:
2
12345678901234567890123456789012123456789012345678901
2
12345678901234567890123456789012123456789012345678901
2
12345678901234567890123456789012123456789012345678901
40
2
12345678901234567890123456789012123456789012345678901
2345678901234567890123456789012123456789012345678901
2
1
(40 sale)($3 pound) 5
~$3 pound!
2
12345678901234567890123456789012123456789012345678901
100
2
12345678901234567890123456789012123456789012345678901
2
12345678901234567890123456789012123456789012345678901
5
(0.4)($3
pound)
2
12345678901234567890123456789012123456789012345678901
2345678901234567890123456789012123456789012345678901
2
1
5 $1.20 discount
2
12345678901234567890123456789012123456789012345678901
2
12345678901234567890123456789012123456789012345678901
2
12345678901234567890123456789012123456789012345678901
Subtract
$1.20,
the
discount,
from
the
original
price
of
$3/pound
and
you
2
12345678901234567890123456789012123456789012345678901
2345678901234567890123456789012123456789012345678901
2
1 get $1.80/pound, the same answer as before.
2
12345678901234567890123456789012123456789012345678901
2
12345678901234567890123456789012123456789012345678901
12345678901234567890123456789012123456789012345678901
A straightforward use of percent is when you are given two numbers and 2
2
12345678901234567890123456789012123456789012345678901
12345678901234567890123456789012123456789012345678901
asked to figure out a percentage value. A simple question of this kind 2
2
12345678901234567890123456789012123456789012345678901
might ask, What percentage of 33 is 3? To answer this problem, it helps 2
12345678901234567890123456789012123456789012345678901
2
12345678901234567890123456789012123456789012345678901
to travel back in time to the fifth grade and dredge up the skill known as 2
12345678901234567890123456789012123456789012345678901
2345678901234567890123456789012123456789012345678901
2
12345678901234567890123456789012123456789012345678901
1 cross-multiplying. Given two fractions equal to each other, you can 2
2
12345678901234567890123456789012123456789012345678901
multiply the numerator of the first fraction with the denominator of the 2
12345678901234567890123456789012123456789012345678901
12345678901234567890123456789012123456789012345678901
second fraction. Then you can multiply the numerator of the second 2
2
12345678901234567890123456789012123456789012345678901
12345678901234567890123456789012123456789012345678901
fraction with the denominator of the first fraction, and set both products 2
2
12345678901234567890123456789012123456789012345678901
2
12345678901234567890123456789012123456789012345678901
equal
to
each
other.
Like
many
basic
math
concepts,
the
explanation
is
2
12345678901234567890123456789012123456789012345678901
2345678901234567890123456789012123456789012345678901
2
12345678901234567890123456789012123456789012345678901
tougher than the actual cross-multiplying.
2
12345678901234567890123456789012123456789012345678901
12345678901234567890123456789012123456789012345678901 22
12345678901234567890123456789012123456789012345678901
1 One way to set up this problem is 3 5 x , because you want to know 2
2
12345678901234567890123456789012123456789012345678901
33 100
2
12345678901234567890123456789012123456789012345678901
what percentage the original fraction equals. Solving for x by cross- 2
12345678901234567890123456789012123456789012345678901
2
12345678901234567890123456789012123456789012345678901
multiplying yields
2
12345678901234567890123456789012123456789012345678901
2345678901234567890123456789012123456789012345678901
12345678901234567890123456789012123456789012345678901 22
12345678901234567890123456789012123456789012345678901
x
3
2
1
2
12345678901234567890123456789012123456789012345678901
5

2
12345678901234567890123456789012123456789012345678901
33 100
2345678901234567890123456789012123456789012345678901
12345678901234567890123456789012123456789012345678901 22
12345678901234567890123456789012123456789012345678901
~33!~x! 5 ~3!~100! (the second step is the
2
1
2
12345678901234567890123456789012123456789012345678901
cross-multiplying
one)
33x
5
300

12345678901234567890123456789012123456789012345678901 2
2345678901234567890123456789012123456789012345678901
12345678901234567890123456789012123456789012345678901 22
12345678901234567890123456789012123456789012345678901
300
2
12345678901234567890123456789012123456789012345678901
x5
'9
2
12345678901234567890123456789012123456789012345678901
33
2
1
2345678901234567890123456789012123456789012345678901
2
12345678901234567890123456789012123456789012345678901
With no calculator, 9 is a good estimate. Therefore, 3 is about 9% of 33. 2
12345678901234567890123456789012123456789012345678901
12345678901234567890123456789012123456789012345678901 22
12345678901234567890123456789012123456789012345678901 2
1
2
12345678901234567890123456789012123456789012345678901
2
12345678901234567890123456789012123456789012345678901
3
1
2
12345678901234567890123456789012123456789012345678901
The
shortcut
on
this
problem
was
to
see
that
5
,
and
using
2
12345678901234567890123456789012123456789012345678901
33
11
2
12345678901234567890123456789012123456789012345678901
1
2345678901234567890123456789012123456789012345678901
12345678901234567890123456789012123456789012345678901 22
1 fraction-decimal conversions, 5 0.09 5 9%.
2
12345678901234567890123456789012123456789012345678901
11
2
12345678901234567890123456789012123456789012345678901
2
12345678901234567890123456789012123456789012345678901
2345678901234567890123456789012123456789012345678901
2
1
12345678901234567890123456789012123456789012345678901 2
12345678901234567890123456789012123456789012345678901 2
12345678901234567890123456789012123456789012345678901 2 65
123456789012345678901234567890121234567890123456789012

X-Ref

S D

http://www.xtremepapers.net

Part III: Section Review

www.petersons.com

http://www.xtremepapers.net

Note

66

123456789012345678901234567890121234567890123456789012
12345678901234567890123456789012123456789012345678901 2
2
12345678901234567890123456789012123456789012345678901
Those are the user-friendly percent questions. Percent increase and 2
12345678901234567890123456789012123456789012345678901
2
12345678901234567890123456789012123456789012345678901
decrease problems are a tad bit trickier. With these problems, the critical 2
12345678901234567890123456789012123456789012345678901
2
12345678901234567890123456789012123456789012345678901
point to remember is that any increase or decrease is relative to the original 2
12345678901234567890123456789012123456789012345678901
2345678901234567890123456789012123456789012345678901
1 value (what is increasing or decreasing). You need to use the original value 2
2
12345678901234567890123456789012123456789012345678901
2
12345678901234567890123456789012123456789012345678901
if
you
are
trying
to
find
the
percent
change.
Suppose
a
problem
tells
you
2
12345678901234567890123456789012123456789012345678901
2
12345678901234567890123456789012123456789012345678901
that
a
certain
stock
started
the
year
priced
at
$40/share
and
ended
the
year
2345678901234567890123456789012123456789012345678901
2
1
2
12345678901234567890123456789012123456789012345678901
at
$56/share.
What
was
the
percentage
change?
2
12345678901234567890123456789012123456789012345678901
2
12345678901234567890123456789012123456789012345678901
2
12345678901234567890123456789012123456789012345678901
This
percentage
change
is
relative
to
the
original
value
of
$40/share.
To
2345678901234567890123456789012123456789012345678901
2
1
2
12345678901234567890123456789012123456789012345678901
find
the
percentage
change,
you
find
how
much
the
stock
price
changed
2
12345678901234567890123456789012123456789012345678901
2
12345678901234567890123456789012123456789012345678901
4
$16
share
2
12345678901234567890123456789012123456789012345678901
5
5
0.4
($16/share)
and
divide
it
by
the
original
value,
2345678901234567890123456789012123456789012345678901
2
1
$40 share
10
2
12345678901234567890123456789012123456789012345678901
5 40%. Since the change was an increase, the stock price increased 40%. 2
12345678901234567890123456789012123456789012345678901
2
12345678901234567890123456789012123456789012345678901
The same method would be used to find a percentage decrease.
2
12345678901234567890123456789012123456789012345678901
2345678901234567890123456789012123456789012345678901
12345678901234567890123456789012123456789012345678901 2
12345678901234567890123456789012123456789012345678901
You might see a percentage change problem like this but be given the 2
2
12345678901234567890123456789012123456789012345678901
2
12345678901234567890123456789012123456789012345678901
percentage
change
and
the
final
value
of
the
item
and
asked
to
find
the
12345678901234567890123456789012123456789012345678901 2
2
12345678901234567890123456789012123456789012345678901
1 original value. In that case, you are doing the same problem you just did in 2
12345678901234567890123456789012123456789012345678901
reverse. Here is the formula you can use to solve either kind of problem, 2
2
12345678901234567890123456789012123456789012345678901
2
12345678901234567890123456789012123456789012345678901
|x
2
y|
2345678901234567890123456789012123456789012345678901
12345678901234567890123456789012123456789012345678901
5 z, where x is the original value, y is the second value, and z is the 2
2
12345678901234567890123456789012123456789012345678901
x
2
12345678901234567890123456789012123456789012345678901
percentage change. (If the lines around the numerator are like alien 2
12345678901234567890123456789012123456789012345678901
2
12345678901234567890123456789012123456789012345678901
scribblings to you, look forward to the section on absolute value for an 2
12345678901234567890123456789012123456789012345678901
2
12345678901234567890123456789012123456789012345678901
explanation of them.)
2
12345678901234567890123456789012123456789012345678901
12345678901234567890123456789012123456789012345678901 2
2
1
2
12345678901234567890123456789012123456789012345678901
The price of a house increased 15%, and then decreased 20%.
2
12345678901234567890123456789012123456789012345678901
2
12345678901234567890123456789012123456789012345678901
Compared
to
the
original
price,
the
final
price
was
12345678901234567890123456789012123456789012345678901 2
2
12345678901234567890123456789012123456789012345678901
2
12345678901234567890123456789012123456789012345678901
A. an 8% decrease.
2
12345678901234567890123456789012123456789012345678901
2
12345678901234567890123456789012123456789012345678901
B. a 5% decrease.
2
12345678901234567890123456789012123456789012345678901
C. the same.
2
12345678901234567890123456789012123456789012345678901
2345678901234567890123456789012123456789012345678901
2
12345678901234567890123456789012123456789012345678901
D. a 3% increase.
2
12345678901234567890123456789012123456789012345678901
2
1
E. a 5% increase.
2
12345678901234567890123456789012123456789012345678901
12345678901234567890123456789012123456789012345678901 2
2345678901234567890123456789012123456789012345678901
12345678901234567890123456789012123456789012345678901 2
2
12345678901234567890123456789012123456789012345678901
Choices (B) and (E) are the big traps here. It seems so inviting to just add 2
12345678901234567890123456789012123456789012345678901
2
12345678901234567890123456789012123456789012345678901
1 or subtract percentages together, but this is almost always wrong. This is 2
2345678901234567890123456789012123456789012345678901
2
12345678901234567890123456789012123456789012345678901
because the two percents are often reflections of different numerical 2
12345678901234567890123456789012123456789012345678901
12345678901234567890123456789012123456789012345678901
values. In other words, you could add 50% of 100 and 25% of 100 2
2
12345678901234567890123456789012123456789012345678901
1 together only because they have the same base value, 100. But you cant 2
2345678901234567890123456789012123456789012345678901
12345678901234567890123456789012123456789012345678901 2
12345678901234567890123456789012123456789012345678901
just add up 50% of 2 and 25% of 100 because these are percentages of 2
2
12345678901234567890123456789012123456789012345678901
2
12345678901234567890123456789012123456789012345678901
different
numbers,
2
and
100.
2
1
2345678901234567890123456789012123456789012345678901
12345678901234567890123456789012123456789012345678901 2
2
1
2
12345678901234567890123456789012123456789012345678901
Theres
a
simple
trick
that
makes
this
kind
of
problem
a
lot
easier.
If
youre
12345678901234567890123456789012123456789012345678901 2
12345678901234567890123456789012123456789012345678901
not told what the original price of the house is, pick a value and work the 2
2
12345678901234567890123456789012123456789012345678901
2
12345678901234567890123456789012123456789012345678901
problem
with
it.
You
want
to
make
things
easy,
so
dont
pick
a
tough
12345678901234567890123456789012123456789012345678901 2
12345678901234567890123456789012123456789012345678901 2
123456789012345678901234567890121234567890123456789012

Tip

Chapter 4: Quantitative Review

123456789012345678901234567890121234567890123456789012
12345678901234567890123456789012123456789012345678901 2
2
12345678901234567890123456789012123456789012345678901
number like 17.53. Using the value of 100 is a much better choice, as this 2
12345678901234567890123456789012123456789012345678901
2
12345678901234567890123456789012123456789012345678901
makes the first percent change easy to figure. A 15% increase on 100 is 15, 2
12345678901234567890123456789012123456789012345678901
2
12345678901234567890123456789012123456789012345678901
and so after the first increase the price of the house is 115.
2
12345678901234567890123456789012123456789012345678901
2345678901234567890123456789012123456789012345678901
2
1
2
12345678901234567890123456789012123456789012345678901
You
now
have
to
calculate
a
20%
decrease
on
a
house
valued
at
115.
If
2
12345678901234567890123456789012123456789012345678901
2
12345678901234567890123456789012123456789012345678901
you
recall
those
fraction-decimal
conversions
on
page
57,
20%
5
0.20
2
12345678901234567890123456789012123456789012345678901
2345678901234567890123456789012123456789012345678901
2
1
1
2
12345678901234567890123456789012123456789012345678901
.
Therefore,
to
find
the
value
of
the
20%
decrease,
multiply
115
by
5
2
12345678901234567890123456789012123456789012345678901
5
2
12345678901234567890123456789012123456789012345678901
2
12345678901234567890123456789012123456789012345678901
1 115 115
2345678901234567890123456789012123456789012345678901
1 one fifth, 3
5
5 23. The price of the house decreased 23 from 2
2
12345678901234567890123456789012123456789012345678901
5
1
5
2
12345678901234567890123456789012123456789012345678901
115,
which
means
the
final
value
of
the
house
is
92(115
2
23).
What
is
the
2
12345678901234567890123456789012123456789012345678901
2
12345678901234567890123456789012123456789012345678901
percentage
change
from
100
to
92?
Its
an
8%
decrease,
choice
(A).
2345678901234567890123456789012123456789012345678901
2
1
2
12345678901234567890123456789012123456789012345678901
2
12345678901234567890123456789012123456789012345678901
2
12345678901234567890123456789012123456789012345678901
2
12345678901234567890123456789012123456789012345678901
2345678901234567890123456789012123456789012345678901
2
12345678901234567890123456789012123456789012345678901
Ratios, Where You Can Compare Apples to
2
12345678901234567890123456789012123456789012345678901
2
12345678901234567890123456789012123456789012345678901
Oranges
12345678901234567890123456789012123456789012345678901 22
12345678901234567890123456789012123456789012345678901
Ratio problems on standardized tests are like affable guests on a talk 2
12345678901234567890123456789012123456789012345678901
1 show: They make a lot of appearances. Ratios can be tested in lots of 2
2
12345678901234567890123456789012123456789012345678901
12345678901234567890123456789012123456789012345678901
different kinds of problems, which is one reason for their common 2
2
12345678901234567890123456789012123456789012345678901
12345678901234567890123456789012123456789012345678901
occurrence. You might see ratios in a word problem or a probability 2
2
12345678901234567890123456789012123456789012345678901
2
12345678901234567890123456789012123456789012345678901
problem, or maybe in just a straightforward ratio problem.
2
12345678901234567890123456789012123456789012345678901
2
12345678901234567890123456789012123456789012345678901
Ratios
are
a
fancy
way
of
writing
fractions.
There
are
a
couple
of
different
2345678901234567890123456789012123456789012345678901
12345678901234567890123456789012123456789012345678901 22
12345678901234567890123456789012123456789012345678901
forms of ratios that you might see on a test, but they are all equivalent to 2
12345678901234567890123456789012123456789012345678901
2
12345678901234567890123456789012123456789012345678901
1 each other and they are also all equivalent to the fractional form of the 2
2345678901234567890123456789012123456789012345678901
12345678901234567890123456789012123456789012345678901
ratio. In a word problem, a ratio would look like this: 9 bananas were 2
2
12345678901234567890123456789012123456789012345678901
1 sold for every 4 apples that were sold. The ratio of bananas to apples sold 2
12345678901234567890123456789012123456789012345678901 2
12345678901234567890123456789012123456789012345678901
is 9 to 4, or 9:4. These three expressions are equivalent, and they are 2
2
12345678901234567890123456789012123456789012345678901
2
12345678901234567890123456789012123456789012345678901
9
2
12345678901234567890123456789012123456789012345678901
(the
first
number
in
the
ratio
is
the
numerator
equivalent
to
the
fraction
2
12345678901234567890123456789012123456789012345678901
4
2
12345678901234567890123456789012123456789012345678901
2345678901234567890123456789012123456789012345678901
2
12345678901234567890123456789012123456789012345678901
and the second the denominator).
2
12345678901234567890123456789012123456789012345678901
2
1
12345678901234567890123456789012123456789012345678901 2
2
12345678901234567890123456789012123456789012345678901
From a precise mathematical standpoint, ratios and fractions are not
2
12345678901234567890123456789012123456789012345678901
2
12345678901234567890123456789012123456789012345678901
completely interchangeable. However, the precise distinction between
2
12345678901234567890123456789012123456789012345678901
2
12345678901234567890123456789012123456789012345678901
ratios and fractions is not relevant for the GRE, so dont sweat it.
12345678901234567890123456789012123456789012345678901 2
2345678901234567890123456789012123456789012345678901
12345678901234567890123456789012123456789012345678901 22
12345678901234567890123456789012123456789012345678901 2
12345678901234567890123456789012123456789012345678901
Like fractions, ratios can be reduced to lowest terms. If the ratio had been
2
12345678901234567890123456789012123456789012345678901
1 9 bananas sold for every 3 apples, the ratio would be 9:3 or 3:1. These 2
2345678901234567890123456789012123456789012345678901
2
12345678901234567890123456789012123456789012345678901
3
9
2
12345678901234567890123456789012123456789012345678901
ratios are the same, just as is the same thing as .
2
12345678901234567890123456789012123456789012345678901
2
3
1
12345678901234567890123456789012123456789012345678901
2
1
2345678901234567890123456789012123456789012345678901
2
12345678901234567890123456789012123456789012345678901
1 Since you can compare fractions and say that one is greater than another, 2
12345678901234567890123456789012123456789012345678901
you can do the same thing with ratios. For instance, which ratio is greater, 2
2
12345678901234567890123456789012123456789012345678901
2
12345678901234567890123456789012123456789012345678901
4
2345678901234567890123456789012123456789012345678901
1 4:3 or 5:4? Convert both to fractions and compare. Is greater or smaller 2
2
12345678901234567890123456789012123456789012345678901
3
2
12345678901234567890123456789012123456789012345678901
12345678901234567890123456789012123456789012345678901 2 67
123456789012345678901234567890121234567890123456789012

http://www.xtremepapers.net

Part III: Section Review

123456789012345678901234567890121234567890123456789012
12345678901234567890123456789012123456789012345678901 2
2
12345678901234567890123456789012123456789012345678901
5
2
12345678901234567890123456789012123456789012345678901
12345678901234567890123456789012123456789012345678901
than ? It is greater because a third is greater than a fourth (bigger 2
2
12345678901234567890123456789012123456789012345678901
4
2
12345678901234567890123456789012123456789012345678901
denominator
means
smaller
value,
just
like
earlier),
and
four
thirds
is
a
2
12345678901234567890123456789012123456789012345678901
2345678901234567890123456789012123456789012345678901
2
1 third greater than one, while five fourths is a fourth greater than one.
2
12345678901234567890123456789012123456789012345678901
2
12345678901234567890123456789012123456789012345678901
2
12345678901234567890123456789012123456789012345678901
Column
B
Column
A
2
12345678901234567890123456789012123456789012345678901
2345678901234567890123456789012123456789012345678901
2
1
2
12345678901234567890123456789012123456789012345678901
Before 4 girls entered the class, there were
2
12345678901234567890123456789012123456789012345678901
2
12345678901234567890123456789012123456789012345678901
18 boys and 16 girls in the fourth grade
2
12345678901234567890123456789012123456789012345678901
2345678901234567890123456789012123456789012345678901
2
1
class at Santiago Elementary School.
2
12345678901234567890123456789012123456789012345678901
2
12345678901234567890123456789012123456789012345678901
The original ratio of
The final ratio of
2
12345678901234567890123456789012123456789012345678901
2
12345678901234567890123456789012123456789012345678901
boys
to
girls
in
girls
to
boys
in
2345678901234567890123456789012123456789012345678901
2
1
2
12345678901234567890123456789012123456789012345678901
the class
the class
2
12345678901234567890123456789012123456789012345678901
2
12345678901234567890123456789012123456789012345678901
2
12345678901234567890123456789012123456789012345678901
2345678901234567890123456789012123456789012345678901
12345678901234567890123456789012123456789012345678901
First, you know that you can determine the two ratios, which means you 2
2
12345678901234567890123456789012123456789012345678901
2
12345678901234567890123456789012123456789012345678901
can
compare
them.
Since
there
will
be
a
definite
relationship
between
12345678901234567890123456789012123456789012345678901 2
12345678901234567890123456789012123456789012345678901
Column A and Column B, choice (D) is out. The original ratio of boys to 2
2
12345678901234567890123456789012123456789012345678901
2
1
20 10
18 9
12345678901234567890123456789012123456789012345678901
5 . The final ratio of girls to boys is 20:18 or
5 . 2
girls is 18:16 or
2
12345678901234567890123456789012123456789012345678901
16 8
18
9
2
12345678901234567890123456789012123456789012345678901
Column
A
is
one
eighth
greater
than
one,
and
Column
B
is
one
ninth
2345678901234567890123456789012123456789012345678901
12345678901234567890123456789012123456789012345678901 2
2
12345678901234567890123456789012123456789012345678901
greater than one, so (A) is greater.
2
12345678901234567890123456789012123456789012345678901
12345678901234567890123456789012123456789012345678901 2
12345678901234567890123456789012123456789012345678901 2
12345678901234567890123456789012123456789012345678901 2
2
12345678901234567890123456789012123456789012345678901
Absolute Value (not a vodka-related concept)
2
12345678901234567890123456789012123456789012345678901
2
12345678901234567890123456789012123456789012345678901
Do
you
remember
the
number
line
on
page
50
of
this
chapter?
The
concept
2
1
2345678901234567890123456789012123456789012345678901
12345678901234567890123456789012123456789012345678901
of absolute value is related to the number line. The absolute value of a 2
2
12345678901234567890123456789012123456789012345678901
1 number is how many numbers (how far) it is away from zero on the 2
12345678901234567890123456789012123456789012345678901 2
12345678901234567890123456789012123456789012345678901
number line. 214 is 14 numbers away from zero on the number line, so the 2
2
12345678901234567890123456789012123456789012345678901
12345678901234567890123456789012123456789012345678901
absolute value of 214 is 14. Of course, the powers-that-be came up with a 2
12345678901234567890123456789012123456789012345678901 2
12345678901234567890123456789012123456789012345678901
cute little way to denote absolute value. For our example, its like this: 2
2
12345678901234567890123456789012123456789012345678901
2345678901234567890123456789012123456789012345678901
2
12345678901234567890123456789012123456789012345678901
|214| 5 14.
2
12345678901234567890123456789012123456789012345678901
2
1
12345678901234567890123456789012123456789012345678901 2
12345678901234567890123456789012123456789012345678901 2
Algebra
2
12345678901234567890123456789012123456789012345678901
2
12345678901234567890123456789012123456789012345678901
12345678901234567890123456789012123456789012345678901
The word algebra usually conjures up memories of numbers and letters 2
2
12345678901234567890123456789012123456789012345678901
2
12345678901234567890123456789012123456789012345678901
3
2345678901234567890123456789012123456789012345678901
12345678901234567890123456789012123456789012345678901
mashed together in strings like 9y 1 3x 5 2 . In simplest terms, algebra 2
2
12345678901234567890123456789012123456789012345678901
4
2
12345678901234567890123456789012123456789012345678901
uses
placeholders,
called
variables,
as
substitutes
for
a
known
value.
Using
12345678901234567890123456789012123456789012345678901 2
1 variables in this manner often allows someone to set up an equation and 2
2
12345678901234567890123456789012123456789012345678901
2
12345678901234567890123456789012123456789012345678901
then
solve
this
equation
to
find
the
actual
value
of
the
variable.
2
12345678901234567890123456789012123456789012345678901
2
12345678901234567890123456789012123456789012345678901
2
12345678901234567890123456789012123456789012345678901
Setting
up
equations
with
variables
and
manipulating
these
equations
are
2345678901234567890123456789012123456789012345678901
12345678901234567890123456789012123456789012345678901 2
1 standard fare for all standardized math tests, and the GRE is no exception. 2
2
12345678901234567890123456789012123456789012345678901
12345678901234567890123456789012123456789012345678901
Problems filled with variables will be easily recognizable algebra prob- 2
2
12345678901234567890123456789012123456789012345678901
12345678901234567890123456789012123456789012345678901
lems, but these are not the only ones you will encounter. On others, the key 2
2
12345678901234567890123456789012123456789012345678901
12345678901234567890123456789012123456789012345678901
will be to create the correct equation on your own, although this might not 2
12345678901234567890123456789012123456789012345678901 2
68
123456789012345678901234567890121234567890123456789012

www.petersons.com

http://www.xtremepapers.net

Alert!

Chapter 4: Quantitative Review

123456789012345678901234567890121234567890123456789012
12345678901234567890123456789012123456789012345678901 2
2
12345678901234567890123456789012123456789012345678901
be the most obvious step required. The end point is that you should get 2
12345678901234567890123456789012123456789012345678901
2
12345678901234567890123456789012123456789012345678901
comfortable with the idea of using variables and making equations, since 2
12345678901234567890123456789012123456789012345678901
2
12345678901234567890123456789012123456789012345678901
this is yet another tool you can use to crack open and solve GRE 2
12345678901234567890123456789012123456789012345678901
2345678901234567890123456789012123456789012345678901
2
1 Quantitative questions.
2
12345678901234567890123456789012123456789012345678901
2
12345678901234567890123456789012123456789012345678901
2
12345678901234567890123456789012123456789012345678901
2
12345678901234567890123456789012123456789012345678901
2345678901234567890123456789012123456789012345678901
2
1 Many algebra problems have a geometry component, and others are filled
2
12345678901234567890123456789012123456789012345678901
2
12345678901234567890123456789012123456789012345678901
with
the
sort
of
math
jargon
you
just
finished
studying.
As
you
work
2
12345678901234567890123456789012123456789012345678901
2
12345678901234567890123456789012123456789012345678901
through
the
rest
of
this
chapter,
you
will
see
how
a
firm
foundation
of
2345678901234567890123456789012123456789012345678901
2
1
2
12345678901234567890123456789012123456789012345678901
arithmetic
aids
you
on
all
types
of
problems.
2
12345678901234567890123456789012123456789012345678901
2
12345678901234567890123456789012123456789012345678901
2
12345678901234567890123456789012123456789012345678901
2345678901234567890123456789012123456789012345678901
2
1
2
12345678901234567890123456789012123456789012345678901
Equations
2
12345678901234567890123456789012123456789012345678901
2
12345678901234567890123456789012123456789012345678901
Equations
are
like
sentences
written
in
math.
Unlike
sentences
written
in
2
12345678901234567890123456789012123456789012345678901
2345678901234567890123456789012123456789012345678901
12345678901234567890123456789012123456789012345678901
English, sentences written in math always follow the same form and rules: 2
2
12345678901234567890123456789012123456789012345678901
12345678901234567890123456789012123456789012345678901 22
12345678901234567890123456789012123456789012345678901
1. There is stuff on the right side of the equation, and it equals the 2
12345678901234567890123456789012123456789012345678901
2
12345678901234567890123456789012123456789012345678901
stuff on the left side of the equation.
2
1
2
12345678901234567890123456789012123456789012345678901
12345678901234567890123456789012123456789012345678901
2. If you do something to the right side, you have to do the exact 2
2
12345678901234567890123456789012123456789012345678901
12345678901234567890123456789012123456789012345678901
same thing to the left side, and vice versa. (That is how you can 2
2
12345678901234567890123456789012123456789012345678901
12345678901234567890123456789012123456789012345678901
keep the equal sign between them. If you do the same thing to 2
2
12345678901234567890123456789012123456789012345678901
each side, they remain equal.)
2
12345678901234567890123456789012123456789012345678901
2345678901234567890123456789012123456789012345678901
12345678901234567890123456789012123456789012345678901 22
12345678901234567890123456789012123456789012345678901
3. It doesnt matter what you do to one side, so long as you follow 2
12345678901234567890123456789012123456789012345678901
2
12345678901234567890123456789012123456789012345678901
Rule #2.
2
1
2345678901234567890123456789012123456789012345678901
12345678901234567890123456789012123456789012345678901 22
12345678901234567890123456789012123456789012345678901
1 Burn Rules #2 and #3 into your head, since youll often be given an 2
12345678901234567890123456789012123456789012345678901
equation to start with and asked to do something to it. Anythings fair 2
2
12345678901234567890123456789012123456789012345678901
2345678901234567890123456789012123456789012345678901
12345678901234567890123456789012123456789012345678901
gamemultiplying, dividing, raising to a power, cubing the root, adding 2
2
12345678901234567890123456789012123456789012345678901
2
1 43,000,000,000so long as its done to both sides.
2
12345678901234567890123456789012123456789012345678901
2
12345678901234567890123456789012123456789012345678901
2345678901234567890123456789012123456789012345678901
2
12345678901234567890123456789012123456789012345678901
If 3x 2 12 5 33, then what does x equal?
2
12345678901234567890123456789012123456789012345678901
1 This is a simple equation. To find out what x equals, you need to get x all 2
12345678901234567890123456789012123456789012345678901 2
12345678901234567890123456789012123456789012345678901
by itself on one side of the equation. The variable x will then equal 2
2
12345678901234567890123456789012123456789012345678901
2
12345678901234567890123456789012123456789012345678901
everything on the other side of the equation.
2
12345678901234567890123456789012123456789012345678901
2
12345678901234567890123456789012123456789012345678901
2
12345678901234567890123456789012123456789012345678901
3x
2
12
5
33
2345678901234567890123456789012123456789012345678901
12345678901234567890123456789012123456789012345678901 22
12345678901234567890123456789012123456789012345678901
3x 2 12 1 12 5 33 1 12 By adding twelve to both sides, we can
2
12345678901234567890123456789012123456789012345678901
2
12345678901234567890123456789012123456789012345678901
get
rid
of
the
212
on
the
left
side.
2
1
2345678901234567890123456789012123456789012345678901
12345678901234567890123456789012123456789012345678901 22
12345678901234567890123456789012123456789012345678901
3x 5 45
2
12345678901234567890123456789012123456789012345678901
2
12345678901234567890123456789012123456789012345678901
Dividing
both
sides
by
three
will
get
rid
3x
4
3
5
45
4
3
2
1
2345678901234567890123456789012123456789012345678901
2
12345678901234567890123456789012123456789012345678901
of the 3 in front of the x.
x 5 15
2
1
2
12345678901234567890123456789012123456789012345678901
2
12345678901234567890123456789012123456789012345678901
There
you
have
it.
2
12345678901234567890123456789012123456789012345678901
2345678901234567890123456789012123456789012345678901
2
1
12345678901234567890123456789012123456789012345678901 2
12345678901234567890123456789012123456789012345678901 2
12345678901234567890123456789012123456789012345678901 2 69
123456789012345678901234567890121234567890123456789012

http://www.xtremepapers.net

Part III: Section Review

70

123456789012345678901234567890121234567890123456789012
12345678901234567890123456789012123456789012345678901 2
2
12345678901234567890123456789012123456789012345678901
Notice that in the example there was only one equation and one variable. 2
12345678901234567890123456789012123456789012345678901
2
12345678901234567890123456789012123456789012345678901
The next step up is a problem where you are given two equations and two 2
12345678901234567890123456789012123456789012345678901
2
12345678901234567890123456789012123456789012345678901
unknowns. Its all still good, because if you have two equations you can 2
12345678901234567890123456789012123456789012345678901
2345678901234567890123456789012123456789012345678901
1 solve for two unknowns. It just takes a little bit longer. (The general math 2
2
12345678901234567890123456789012123456789012345678901
2
12345678901234567890123456789012123456789012345678901
rule
is
that
you
can
solve
for
as
many
unknowns
as
you
have
distinct
2
12345678901234567890123456789012123456789012345678901
2
12345678901234567890123456789012123456789012345678901
equations.)
2345678901234567890123456789012123456789012345678901
2
1
2
12345678901234567890123456789012123456789012345678901
2
12345678901234567890123456789012123456789012345678901
2
12345678901234567890123456789012123456789012345678901
If
4x
1
3y
5
13
and
x
1
2y
5
2,
then
what
is
the
value
of
27xy?
2
12345678901234567890123456789012123456789012345678901
2345678901234567890123456789012123456789012345678901
2
1
2
12345678901234567890123456789012123456789012345678901
A. 224
2
12345678901234567890123456789012123456789012345678901
2
B. 24
12345678901234567890123456789012123456789012345678901
2
12345678901234567890123456789012123456789012345678901
C.
21
2345678901234567890123456789012123456789012345678901
2
1
2
12345678901234567890123456789012123456789012345678901
D. 4
2
12345678901234567890123456789012123456789012345678901
2
12345678901234567890123456789012123456789012345678901
E.
28
2
12345678901234567890123456789012123456789012345678901
2345678901234567890123456789012123456789012345678901
12345678901234567890123456789012123456789012345678901 2
12345678901234567890123456789012123456789012345678901
There are two different ways to try to solve this problem. The first way is 2
2
12345678901234567890123456789012123456789012345678901
2
12345678901234567890123456789012123456789012345678901
the
solve
and
plug.
You
solve
for
one
variable
in
one
equation
and
then
12345678901234567890123456789012123456789012345678901 2
2
12345678901234567890123456789012123456789012345678901
2
1 plug it into the other equation. It looks like this:
2
12345678901234567890123456789012123456789012345678901
2
12345678901234567890123456789012123456789012345678901
(Plug
this
in
for
x
in
the
other
x
1
2y
5
2

x
5
2
2
2y
2
12345678901234567890123456789012123456789012345678901
2345678901234567890123456789012123456789012345678901
2
12345678901234567890123456789012123456789012345678901
equation.)
2
12345678901234567890123456789012123456789012345678901
2
12345678901234567890123456789012123456789012345678901
(Now we have one equation with
2
12345678901234567890123456789012123456789012345678901
4~2 2 2y! 1 3y 5 13
2
12345678901234567890123456789012123456789012345678901
one
unknown,
so
we
can
solve
12345678901234567890123456789012123456789012345678901 2
2
12345678901234567890123456789012123456789012345678901
for y.)
2
12345678901234567890123456789012123456789012345678901
12345678901234567890123456789012123456789012345678901 2
2
1
8 2 8y 1 3y 5 13
2
12345678901234567890123456789012123456789012345678901
2
12345678901234567890123456789012123456789012345678901
25y 5 5
2
12345678901234567890123456789012123456789012345678901
2
12345678901234567890123456789012123456789012345678901
(We
can
take
this
information
and
2
12345678901234567890123456789012123456789012345678901
y 5 21
2345678901234567890123456789012123456789012345678901
2
12345678901234567890123456789012123456789012345678901
plug
it
back
into
our
equation
2
12345678901234567890123456789012123456789012345678901
x
5
2
2
2y
2
1
with
x.)
2
12345678901234567890123456789012123456789012345678901
5
2
2
2(21)
2
12345678901234567890123456789012123456789012345678901
2345678901234567890123456789012123456789012345678901
2
12345678901234567890123456789012123456789012345678901
54
2
12345678901234567890123456789012123456789012345678901
2
1
2
12345678901234567890123456789012123456789012345678901
Once
you
have
the
values
for
both
variables,
you
can
determine
what
12345678901234567890123456789012123456789012345678901 2
2345678901234567890123456789012123456789012345678901
2
12345678901234567890123456789012123456789012345678901
27xy equals.
2
12345678901234567890123456789012123456789012345678901
12345678901234567890123456789012123456789012345678901 2
2
12345678901234567890123456789012123456789012345678901
7xy 5
2
1
2345678901234567890123456789012123456789012345678901
12345678901234567890123456789012123456789012345678901 2
2
12345678901234567890123456789012123456789012345678901
7~4!~21! 5 228
12345678901234567890123456789012123456789012345678901 2
12345678901234567890123456789012123456789012345678901 2
2
1
2
12345678901234567890123456789012123456789012345678901
2
12345678901234567890123456789012123456789012345678901
2
12345678901234567890123456789012123456789012345678901
2
12345678901234567890123456789012123456789012345678901
12345678901234567890123456789012123456789012345678901 2
2345678901234567890123456789012123456789012345678901
12345678901234567890123456789012123456789012345678901 2
2
1
2
12345678901234567890123456789012123456789012345678901
12345678901234567890123456789012123456789012345678901 2
2
12345678901234567890123456789012123456789012345678901
12345678901234567890123456789012123456789012345678901 2
12345678901234567890123456789012123456789012345678901 2
12345678901234567890123456789012123456789012345678901 2
12345678901234567890123456789012123456789012345678901 2
123456789012345678901234567890121234567890123456789012

www.petersons.com

http://www.xtremepapers.net

Chapter 4: Quantitative Review

123456789012345678901234567890121234567890123456789012
12345678901234567890123456789012123456789012345678901 2
2
12345678901234567890123456789012123456789012345678901
That was the first way. The second way is a little more inventive and can be 2
12345678901234567890123456789012123456789012345678901
2
12345678901234567890123456789012123456789012345678901
quicker. To do it, you want to make the coefficients of one of the variables 2
12345678901234567890123456789012123456789012345678901
2
12345678901234567890123456789012123456789012345678901
the same in both equations. Then you can subtract one equation from the 2
12345678901234567890123456789012123456789012345678901
2345678901234567890123456789012123456789012345678901
1 other and so eliminate one of the variables. In our example, it is easiest to 2
2
12345678901234567890123456789012123456789012345678901
2
12345678901234567890123456789012123456789012345678901
make
both
of
the
xs
have
a
coefficient
of
4.
You
can
get
this
by
2
12345678901234567890123456789012123456789012345678901
2
12345678901234567890123456789012123456789012345678901
multiplying
the
second
equation
by
4:
2345678901234567890123456789012123456789012345678901
2
1
2
12345678901234567890123456789012123456789012345678901
2
12345678901234567890123456789012123456789012345678901
x 1 2y 5 2
2
12345678901234567890123456789012123456789012345678901
2
12345678901234567890123456789012123456789012345678901
2345678901234567890123456789012123456789012345678901
2
1
4~x 1 2y! 5 4~2! Rule #2 at work!
2
12345678901234567890123456789012123456789012345678901
2
12345678901234567890123456789012123456789012345678901
4x
1
2y
5
8
2
12345678901234567890123456789012123456789012345678901
2
12345678901234567890123456789012123456789012345678901
2345678901234567890123456789012123456789012345678901
2
1 Line the two equations up and subtract one from the other:
2
12345678901234567890123456789012123456789012345678901
2
12345678901234567890123456789012123456789012345678901
2
12345678901234567890123456789012123456789012345678901
4x
1
3y
5
13
2
12345678901234567890123456789012123456789012345678901
2345678901234567890123456789012123456789012345678901
2
12345678901234567890123456789012123456789012345678901
2 4x 1 8y 5 8
2
12345678901234567890123456789012123456789012345678901
2
12345678901234567890123456789012123456789012345678901
25y 5 5
2
12345678901234567890123456789012123456789012345678901
12345678901234567890123456789012123456789012345678901 22
12345678901234567890123456789012123456789012345678901
y 5 21
2
1
2
12345678901234567890123456789012123456789012345678901
Plugging this back in, you will find that x 5 4. As you would hope, this is 2
12345678901234567890123456789012123456789012345678901
2
12345678901234567890123456789012123456789012345678901
the same result that we got doing the problem the first way.
2
12345678901234567890123456789012123456789012345678901
2
12345678901234567890123456789012123456789012345678901
2
12345678901234567890123456789012123456789012345678901
2
12345678901234567890123456789012123456789012345678901
2
12345678901234567890123456789012123456789012345678901
2345678901234567890123456789012123456789012345678901
2
12345678901234567890123456789012123456789012345678901
More Complicated Equations
2
12345678901234567890123456789012123456789012345678901
2
12345678901234567890123456789012123456789012345678901
Equations
on
the
GRE
dont
get
much
more
complicated
than
the
previous
12345678901234567890123456789012123456789012345678901 22
1 ones. There are, however, quadratic equations occasionally lurking around
2
12345678901234567890123456789012123456789012345678901
2
12345678901234567890123456789012123456789012345678901
the
test.
These
sound
intimidating,
but
if
you
got
through
high
school
12345678901234567890123456789012123456789012345678901 2
12345678901234567890123456789012123456789012345678901
algebra, you have already mastered these imps once. Youll just need a 2
2
12345678901234567890123456789012123456789012345678901
2345678901234567890123456789012123456789012345678901
2
12345678901234567890123456789012123456789012345678901
little refresher so you can master them again.
2
12345678901234567890123456789012123456789012345678901
2
1
12345678901234567890123456789012123456789012345678901
A quadratic equation means that one variable will be raised to a power like 2
2
12345678901234567890123456789012123456789012345678901
this, x2. In order to solve these buggers, you often have to factor.
2
12345678901234567890123456789012123456789012345678901
2
12345678901234567890123456789012123456789012345678901
12345678901234567890123456789012123456789012345678901 2
12345678901234567890123456789012123456789012345678901 2
12345678901234567890123456789012123456789012345678901 2
2
12345678901234567890123456789012123456789012345678901
Factor Refresher
2
12345678901234567890123456789012123456789012345678901
2
12345678901234567890123456789012123456789012345678901
2
12345678901234567890123456789012123456789012345678901
ab
1
ac
5
a
b
1
c
~
!
12345678901234567890123456789012123456789012345678901 2
2345678901234567890123456789012123456789012345678901
2
12345678901234567890123456789012123456789012345678901
~a 1 b!~c 1 d!5 ac 1 ad 1 bc 1 bd
2
12345678901234567890123456789012123456789012345678901
2
12345678901234567890123456789012123456789012345678901
2
2
2
2
12345678901234567890123456789012123456789012345678901
a 1 2ab 1 b 5 ~a 1 b!
2
1
2
2
2
2345678901234567890123456789012123456789012345678901
2
12345678901234567890123456789012123456789012345678901
a 2 2ab 1 b 5 ~a 2 b!
2
12345678901234567890123456789012123456789012345678901
2
2
2
12345678901234567890123456789012123456789012345678901
a
2
b
5
a
2
b
a
1
b
~
!~
!
12345678901234567890123456789012123456789012345678901 22
1
2
12345678901234567890123456789012123456789012345678901
12345678901234567890123456789012123456789012345678901 2
2
12345678901234567890123456789012123456789012345678901
12345678901234567890123456789012123456789012345678901 2
2
12345678901234567890123456789012123456789012345678901
12345678901234567890123456789012123456789012345678901 2
12345678901234567890123456789012123456789012345678901 2
12345678901234567890123456789012123456789012345678901 2
12345678901234567890123456789012123456789012345678901 2 71
123456789012345678901234567890121234567890123456789012

http://www.xtremepapers.net

Part III: Section Review

72

123456789012345678901234567890121234567890123456789012
12345678901234567890123456789012123456789012345678901 2
2
12345678901234567890123456789012123456789012345678901
Think back, way back, to Algebra I. You might recall the memory aid 2
12345678901234567890123456789012123456789012345678901
2
12345678901234567890123456789012123456789012345678901
called FOIL. FOIL is used when you are multiplying two groups of two 2
12345678901234567890123456789012123456789012345678901
2
12345678901234567890123456789012123456789012345678901
variables that look like this: (a 1 b)(c 1 d). The expression stands for, 2
12345678901234567890123456789012123456789012345678901
2345678901234567890123456789012123456789012345678901
2
1 First, Outside, Inside, Last.
2
12345678901234567890123456789012123456789012345678901
2
12345678901234567890123456789012123456789012345678901
2
12345678901234567890123456789012123456789012345678901
You
multiply
the
First
terms
(a
3
c),
2
12345678901234567890123456789012123456789012345678901
2345678901234567890123456789012123456789012345678901
2
1
2
12345678901234567890123456789012123456789012345678901
the
Outside
terms
(a
3
d),
2
12345678901234567890123456789012123456789012345678901
2
12345678901234567890123456789012123456789012345678901
2
12345678901234567890123456789012123456789012345678901
the Inside terms (b 3 c),
2345678901234567890123456789012123456789012345678901
2
1
2
12345678901234567890123456789012123456789012345678901
the Last terms (b 3 d),
2
12345678901234567890123456789012123456789012345678901
2
12345678901234567890123456789012123456789012345678901
2
12345678901234567890123456789012123456789012345678901
and
then
you
add
them
all
together.
2345678901234567890123456789012123456789012345678901
2
1
2
12345678901234567890123456789012123456789012345678901
2
12345678901234567890123456789012123456789012345678901
If
you
are
asked
to
expandthats
the
term
often
usedthe
expression,
2
12345678901234567890123456789012123456789012345678901
2
12345678901234567890123456789012123456789012345678901
just
follow
FOIL:
2345678901234567890123456789012123456789012345678901
12345678901234567890123456789012123456789012345678901 2
12345678901234567890123456789012123456789012345678901 2
2
12345678901234567890123456789012123456789012345678901
~2x 1 3!~x 2 4! 5
2
12345678901234567890123456789012123456789012345678901
2
12345678901234567890123456789012123456789012345678901
2
2x
2
8x
1
3x2
12
5
12345678901234567890123456789012123456789012345678901 2
2
1
2
12345678901234567890123456789012123456789012345678901
2x2 2 5x 2 12 5
2
12345678901234567890123456789012123456789012345678901
2
12345678901234567890123456789012123456789012345678901
Factoring
is
the
reverse
of
FOIL.
Factoring
is
when
you
make
the
math
2345678901234567890123456789012123456789012345678901
12345678901234567890123456789012123456789012345678901 2
2
12345678901234567890123456789012123456789012345678901
expression have parentheses instead of the other way around, so you take 2
12345678901234567890123456789012123456789012345678901
12345678901234567890123456789012123456789012345678901
something like 2x2 2 5x 2 12 and must factor it into two parentheses like 2
2
12345678901234567890123456789012123456789012345678901
2
12345678901234567890123456789012123456789012345678901
the (2x 1 3)(x 2 4) you started with above.
2
12345678901234567890123456789012123456789012345678901
12345678901234567890123456789012123456789012345678901 2
2
12345678901234567890123456789012123456789012345678901
1 Factoring is 2often not as straightforward as doing FOIL. Lets use the 2
2345678901234567890123456789012123456789012345678901
12345678901234567890123456789012123456789012345678901
expression x 2 2x 2 15 as an example of what needs to be factored. First, 2
2
12345678901234567890123456789012123456789012345678901
1 if there is no coefficient in front of the squared termx2 in our 2
12345678901234567890123456789012123456789012345678901 2
12345678901234567890123456789012123456789012345678901
examplethen you can put an x in the first place of each parentheses, 2
2
12345678901234567890123456789012123456789012345678901
12345678901234567890123456789012123456789012345678901
(x
)(x
). Next, notice that the 15 has a minus in front of it. This 2
12345678901234567890123456789012123456789012345678901 2
12345678901234567890123456789012123456789012345678901
could only be the case if one of the last terms in the parentheses is negative 2
2
12345678901234567890123456789012123456789012345678901
2345678901234567890123456789012123456789012345678901
2
12345678901234567890123456789012123456789012345678901
and the other last term is positive, which means (x 1 )(x 2 ).
2
12345678901234567890123456789012123456789012345678901
1 The product of the last two terms is 15. What factors give fifteen? 1 3 15 2
12345678901234567890123456789012123456789012345678901 2
2
12345678901234567890123456789012123456789012345678901
and 3 3 5 answers this problem. One of these factored pairs is the one 2
12345678901234567890123456789012123456789012345678901
12345678901234567890123456789012123456789012345678901
needed to complete the parentheses, but figuring out which one is correct 2
2
12345678901234567890123456789012123456789012345678901
2
12345678901234567890123456789012123456789012345678901
can
take
some
trial
and
error.
The
difference
between
the
factors
must
be
2
12345678901234567890123456789012123456789012345678901 2
2345678901234567890123456789012123456789012345678901
12345678901234567890123456789012123456789012345678901
since that is the coefficient of the x-term, so 3 3 5 has to be what goes in 2
2
12345678901234567890123456789012123456789012345678901
2
12345678901234567890123456789012123456789012345678901
the
blanks
of
our
parentheses.
But
which
goes
where?
Since
the
x-term
is
12345678901234567890123456789012123456789012345678901 2
1 negative (22), the larger number, 5, should go in front of the minus sign. 2
2
12345678901234567890123456789012123456789012345678901
2
12345678901234567890123456789012123456789012345678901
This
gives
us
(x
1
3)(x
2
5).
2
12345678901234567890123456789012123456789012345678901
2
12345678901234567890123456789012123456789012345678901
12345678901234567890123456789012123456789012345678901 2
2345678901234567890123456789012123456789012345678901
12345678901234567890123456789012123456789012345678901 2
2
1
2
12345678901234567890123456789012123456789012345678901
12345678901234567890123456789012123456789012345678901 2
2
12345678901234567890123456789012123456789012345678901
12345678901234567890123456789012123456789012345678901 2
12345678901234567890123456789012123456789012345678901 2
12345678901234567890123456789012123456789012345678901 2
12345678901234567890123456789012123456789012345678901 2
123456789012345678901234567890121234567890123456789012

www.petersons.com

http://www.xtremepapers.net

Note

Chapter 4: Quantitative Review

123456789012345678901234567890121234567890123456789012
12345678901234567890123456789012123456789012345678901 2
2
12345678901234567890123456789012123456789012345678901
12345678901234567890123456789012123456789012345678901 2
2
12345678901234567890123456789012123456789012345678901
You can always check to see if you have factored correctly by FOILing to
2
12345678901234567890123456789012123456789012345678901
2
12345678901234567890123456789012123456789012345678901
see
if
you
get
what
you
started
with:
2
12345678901234567890123456789012123456789012345678901
2345678901234567890123456789012123456789012345678901
2
1
2
12345678901234567890123456789012123456789012345678901
2
2
x 1 3!~x 2 5! 5 x 2 5x 1 3x 2 15 5 x 2 2x 2 15
~
2
12345678901234567890123456789012123456789012345678901
2
12345678901234567890123456789012123456789012345678901
2
12345678901234567890123456789012123456789012345678901
2345678901234567890123456789012123456789012345678901
2
1
2
12345678901234567890123456789012123456789012345678901
You
want
to
be
comfortable
using
FOIL
and
factoring,
since
many
2
12345678901234567890123456789012123456789012345678901
2
12345678901234567890123456789012123456789012345678901
problems
will
require
these
math
operations.
If
this
one
example
of
2
12345678901234567890123456789012123456789012345678901
2345678901234567890123456789012123456789012345678901
1 factoring didnt get all those old factoring juices flowing, then it is a good 2
2
12345678901234567890123456789012123456789012345678901
2
12345678901234567890123456789012123456789012345678901
idea
to
play
around
with
factoring
to
get
more
comfortable
with
it.
Make
2
12345678901234567890123456789012123456789012345678901
2
12345678901234567890123456789012123456789012345678901
up
a
few
expressions
like
(x
2
5)(x
2
2)
or
(x
1
8)(x
1
2),
expand
them,
2345678901234567890123456789012123456789012345678901
2
1
12345678901234567890123456789012123456789012345678901
and then factor them. With practice, factoring can become as straightfor- 2
2
12345678901234567890123456789012123456789012345678901
2
12345678901234567890123456789012123456789012345678901
ward as expanding.
2
12345678901234567890123456789012123456789012345678901
2345678901234567890123456789012123456789012345678901
12345678901234567890123456789012123456789012345678901 22
Column B
Column A
12345678901234567890123456789012123456789012345678901
12345678901234567890123456789012123456789012345678901 22
12345678901234567890123456789012123456789012345678901
c is a positive integer.
2
12345678901234567890123456789012123456789012345678901
2
12345678901234567890123456789012123456789012345678901
2
2
2
1
c
2
4
2c
2
c
2
6
2
12345678901234567890123456789012123456789012345678901
2
12345678901234567890123456789012123456789012345678901
2
12345678901234567890123456789012123456789012345678901
2345678901234567890123456789012123456789012345678901
12345678901234567890123456789012123456789012345678901
When you see a problem like this, you should immediately suspect that you 2
2
12345678901234567890123456789012123456789012345678901
12345678901234567890123456789012123456789012345678901
can factor these two expressions and then compare them directly. The GRE 2
12345678901234567890123456789012123456789012345678901 22
12345678901234567890123456789012123456789012345678901
isnt going to give a problem where the only way to solve it is just plug in
2
12345678901234567890123456789012123456789012345678901
numbers randomly; thats just not how its designed. Column A is one of the 2
12345678901234567890123456789012123456789012345678901
2
12345678901234567890123456789012123456789012345678901
classic factors from the chart, although you may have to look hard to see 2
12345678901234567890123456789012123456789012345678901
1 a2 2 b2 5 (a 2 b)(a 1 b). Its there, though, and c2 2 4 5 (c 1 2)(c 2 2). 2
2
12345678901234567890123456789012123456789012345678901
2
12345678901234567890123456789012123456789012345678901
2
12345678901234567890123456789012123456789012345678901
Keep
these
results
in
mind
when
factoring
Column
B.
You
should
be
on
the
12345678901234567890123456789012123456789012345678901 2
12345678901234567890123456789012123456789012345678901
look out for a (c 1 2) or a (c 2 2) since that would make it easier to directly 2
2
12345678901234567890123456789012123456789012345678901
2
12345678901234567890123456789012123456789012345678901
compare
Column
A
and
B.
In
factoring
Column
B,
the
first
terms
can
only
12345678901234567890123456789012123456789012345678901 2
12345678901234567890123456789012123456789012345678901
be: (2c
)(c
). If you dont see that, play with the numbers and it 2
2
12345678901234567890123456789012123456789012345678901
2345678901234567890123456789012123456789012345678901
12345678901234567890123456789012123456789012345678901
should become clear. Since there is a negative sign in front of the 6, there 2
2
12345678901234567890123456789012123456789012345678901
1 must be one plus and one minus sign in the parentheses. The factor pairs of 2
12345678901234567890123456789012123456789012345678901 2
12345678901234567890123456789012123456789012345678901
6 are 1 3 6 and 2 3 3. Since the middle c-term has a coefficient of one, 2 2
2
12345678901234567890123456789012123456789012345678901
2
12345678901234567890123456789012123456789012345678901
and
3
are
probably
our
factors,
but
with
the
2
in
front
of
the
first
c,
we
2
12345678901234567890123456789012123456789012345678901
2
12345678901234567890123456789012123456789012345678901
cant be sure.
12345678901234567890123456789012123456789012345678901 2
2345678901234567890123456789012123456789012345678901
2
12345678901234567890123456789012123456789012345678901
Play with the numbers to see if 2 and 3 can make it work. After some 2
12345678901234567890123456789012123456789012345678901
2
12345678901234567890123456789012123456789012345678901
unavoidable trial and error, you should hit upon (2c 1 3)(c 2 2). Since 2
12345678901234567890123456789012123456789012345678901
1 both columns have a (c 2 2) term in them, you can disregard that term in 2
2
12345678901234567890123456789012123456789012345678901
2
12345678901234567890123456789012123456789012345678901
each column and focus on the other terms. You are now left comparing (c 2
12345678901234567890123456789012123456789012345678901
12345678901234567890123456789012123456789012345678901
1 2) and (2c 1 3). Since c is a positive integer, it seems likely that (2c 1 3) 2
2
12345678901234567890123456789012123456789012345678901
2345678901234567890123456789012123456789012345678901
2
12345678901234567890123456789012123456789012345678901
1 is greater, but plug in a few numbers to confirm this if you arent sure. 2
12345678901234567890123456789012123456789012345678901
Make sure to plug in c 5 1, since 1 is the lowest positive integer and 2
2
12345678901234567890123456789012123456789012345678901
2
12345678901234567890123456789012123456789012345678901
therefore
the
most
likely
candidate
for
making
(A)
greater
or
equal
to
(B).
2345678901234567890123456789012123456789012345678901
2
1
2
12345678901234567890123456789012123456789012345678901
It
turns
out
that
(B)
is
always
greater
than
(A),
so
the
correct
answer
is
(B).
12345678901234567890123456789012123456789012345678901 22
12345678901234567890123456789012123456789012345678901
123456789012345678901234567890121234567890123456789012 73

http://www.xtremepapers.net

Part III: Section Review

www.petersons.com

http://www.xtremepapers.net

Note

74

123456789012345678901234567890121234567890123456789012
12345678901234567890123456789012123456789012345678901 2
2
12345678901234567890123456789012123456789012345678901
When factoring quadratic equations, the first thing to be done is to get all 2
12345678901234567890123456789012123456789012345678901
2
12345678901234567890123456789012123456789012345678901
the numbers and variables on one side and have the other side of the 2
12345678901234567890123456789012123456789012345678901
2
12345678901234567890123456789012123456789012345678901
equation equal zero. Suppose you were given the equation x2 1 x 2 6 5 6, 2
12345678901234567890123456789012123456789012345678901
2345678901234567890123456789012123456789012345678901
2
1 and asked to determine the value(s) of x. The first step is:
2
12345678901234567890123456789012123456789012345678901
2
12345678901234567890123456789012123456789012345678901
2
12345678901234567890123456789012123456789012345678901
2
x 1x2656
2
12345678901234567890123456789012123456789012345678901
2345678901234567890123456789012123456789012345678901
2
1
2
2
12345678901234567890123456789012123456789012345678901
x
1
x
2
6
2
6
5
6
2
6

2
12345678901234567890123456789012123456789012345678901
2
2
12345678901234567890123456789012123456789012345678901
x
1
x
2
12
5
0
2
12345678901234567890123456789012123456789012345678901
2345678901234567890123456789012123456789012345678901
2
1
2
12345678901234567890123456789012123456789012345678901
2
12345678901234567890123456789012123456789012345678901
2
12345678901234567890123456789012123456789012345678901
2
12345678901234567890123456789012123456789012345678901
There
is
a
good
reason
why
you
do
this,
but
you
dont
need
to
know
it
2345678901234567890123456789012123456789012345678901
2
1
2
12345678901234567890123456789012123456789012345678901
because
GRE
math
tests
the
hows
and
not
the
whys
of
algebra.
2
12345678901234567890123456789012123456789012345678901
2
12345678901234567890123456789012123456789012345678901
2
12345678901234567890123456789012123456789012345678901
2345678901234567890123456789012123456789012345678901
12345678901234567890123456789012123456789012345678901 2
12345678901234567890123456789012123456789012345678901 2
12345678901234567890123456789012123456789012345678901
Now its time to factor, and with some work you will get: x2 1 x 2 12 5 2
2
12345678901234567890123456789012123456789012345678901
2
12345678901234567890123456789012123456789012345678901
(x
1
4)(x
2
3).
To
determine
the
value(s)
of
x,
look
at
the
two
sets
of
12345678901234567890123456789012123456789012345678901 2
1 parentheses and remember that any number multiplied by zero is zero. You 2
2
12345678901234567890123456789012123456789012345678901
2
12345678901234567890123456789012123456789012345678901
can
deduce
that
either
(x
1
4)
5
0
or
(x
2
3)
5
0
equals
zero,
although
at
2
12345678901234567890123456789012123456789012345678901
2345678901234567890123456789012123456789012345678901
12345678901234567890123456789012123456789012345678901
this point you dont know which. When you are dealing with quadratic 2
2
12345678901234567890123456789012123456789012345678901
2
12345678901234567890123456789012123456789012345678901
equations
(equations
that
have
squared
terms
in
them),
there
will
often
be
12345678901234567890123456789012123456789012345678901 2
12345678901234567890123456789012123456789012345678901
two answers. In this example, x equals 24 or 3. Keep the two answers are 2
2
12345678901234567890123456789012123456789012345678901
2
12345678901234567890123456789012123456789012345678901
correct
point
in
mind
when
you
see
quadratic
equations,
since
thats
the
12345678901234567890123456789012123456789012345678901 2
2
12345678901234567890123456789012123456789012345678901
2
1 kind of funky math knowledge that the test-makers love to test.
2345678901234567890123456789012123456789012345678901
12345678901234567890123456789012123456789012345678901 2
2
12345678901234567890123456789012123456789012345678901
2
3x 1 11x 1 6
2
1
5 0, then x 5
If 2
2
12345678901234567890123456789012123456789012345678901
x
1
6x
1
9
2
12345678901234567890123456789012123456789012345678901
2345678901234567890123456789012123456789012345678901
12345678901234567890123456789012123456789012345678901 2
2
12345678901234567890123456789012123456789012345678901
2
2
1
2
A.
2
12345678901234567890123456789012123456789012345678901
3
2
12345678901234567890123456789012123456789012345678901
2345678901234567890123456789012123456789012345678901
2
12345678901234567890123456789012123456789012345678901
B. 21
2
12345678901234567890123456789012123456789012345678901
2
1
2
12345678901234567890123456789012123456789012345678901
3
2
12345678901234567890123456789012123456789012345678901
C.
2345678901234567890123456789012123456789012345678901
2
12345678901234567890123456789012123456789012345678901
5
2
12345678901234567890123456789012123456789012345678901
2
12345678901234567890123456789012123456789012345678901
D. 2
2
12345678901234567890123456789012123456789012345678901
2
1
2345678901234567890123456789012123456789012345678901
2
12345678901234567890123456789012123456789012345678901
E. 4
2
12345678901234567890123456789012123456789012345678901
2
12345678901234567890123456789012123456789012345678901
As always, dont be intimidated by how complicated that algebra 2
12345678901234567890123456789012123456789012345678901
1 expression looks. If you see something like this, you should know that the 2
2
12345678901234567890123456789012123456789012345678901
2
12345678901234567890123456789012123456789012345678901
winning strategy on this type of problem is to find a way to simplify it. The 2
12345678901234567890123456789012123456789012345678901
2
12345678901234567890123456789012123456789012345678901
denominator looks less intricate, so lets start there:
2
12345678901234567890123456789012123456789012345678901
2345678901234567890123456789012123456789012345678901
2
12345678901234567890123456789012123456789012345678901
2
2
1
1
6x
1
9
5
(x
1
3)(x
1
3)
x
2
12345678901234567890123456789012123456789012345678901
12345678901234567890123456789012123456789012345678901 2
2
12345678901234567890123456789012123456789012345678901
12345678901234567890123456789012123456789012345678901 2
12345678901234567890123456789012123456789012345678901 2
12345678901234567890123456789012123456789012345678901 2
12345678901234567890123456789012123456789012345678901 2
123456789012345678901234567890121234567890123456789012

Chapter 4: Quantitative Review

123456789012345678901234567890121234567890123456789012
12345678901234567890123456789012123456789012345678901 2
2
12345678901234567890123456789012123456789012345678901
It is a good bet that in factoring the numerator, you will find a (x 1 3) 2
12345678901234567890123456789012123456789012345678901
2
12345678901234567890123456789012123456789012345678901
term. Start with that premise and see where it leads you:
2
12345678901234567890123456789012123456789012345678901
12345678901234567890123456789012123456789012345678901 2
2
12345678901234567890123456789012123456789012345678901
3x2 1 11x 1 6 5 (x 1 3)(3x 1 2)
2
12345678901234567890123456789012123456789012345678901
2
12345678901234567890123456789012123456789012345678901
2
12345678901234567890123456789012123456789012345678901
There
it
is.
Its
not
a
coincidence,
either.
Putting
it
all
together,
you
have:
2
12345678901234567890123456789012123456789012345678901
2
12345678901234567890123456789012123456789012345678901
2
2345678901234567890123456789012123456789012345678901
2
1
3x 1 11x 1 6 ~x 1 3!~3x 1 2! ~3x 1 2!
2
12345678901234567890123456789012123456789012345678901
5
50
5
2
12345678901234567890123456789012123456789012345678901
2
x
1
3
x
1
3
x
1
3
x
1
6x
1
9
~
!~
! ~
!
2
12345678901234567890123456789012123456789012345678901
2
12345678901234567890123456789012123456789012345678901
2345678901234567890123456789012123456789012345678901
1 Remember that a fraction equals zero if the numerator equals zero, and 2
2
12345678901234567890123456789012123456789012345678901
2
12345678901234567890123456789012123456789012345678901
2
2
12345678901234567890123456789012123456789012345678901
(3x
1
2)
5
0
if
x
5
2
,
which
is
the
correct
answer,
choice
(A).
2
12345678901234567890123456789012123456789012345678901
3
2345678901234567890123456789012123456789012345678901
2
1
2
12345678901234567890123456789012123456789012345678901
2
12345678901234567890123456789012123456789012345678901
2
12345678901234567890123456789012123456789012345678901
2
12345678901234567890123456789012123456789012345678901
Not
Equations
But
Inequalities
2345678901234567890123456789012123456789012345678901
12345678901234567890123456789012123456789012345678901 22
12345678901234567890123456789012123456789012345678901
Inequalities are like equations except that there is not an equal sign between 2
12345678901234567890123456789012123456789012345678901
2
12345678901234567890123456789012123456789012345678901
the left and right side. Instead you might find one of the following:
2
12345678901234567890123456789012123456789012345678901
12345678901234567890123456789012123456789012345678901 22
1
Various Inequality Signs
2
12345678901234567890123456789012123456789012345678901
2
12345678901234567890123456789012123456789012345678901

not
equal
to
2
12345678901234567890123456789012123456789012345678901
2345678901234567890123456789012123456789012345678901
2
12345678901234567890123456789012123456789012345678901
, less than
2
12345678901234567890123456789012123456789012345678901
12345678901234567890123456789012123456789012345678901 22
12345678901234567890123456789012123456789012345678901
less than or equal to
2
12345678901234567890123456789012123456789012345678901
2
12345678901234567890123456789012123456789012345678901
. greater than
2
12345678901234567890123456789012123456789012345678901
2
12345678901234567890123456789012123456789012345678901
greater than or equal to
2
12345678901234567890123456789012123456789012345678901
2
1
2345678901234567890123456789012123456789012345678901
12345678901234567890123456789012123456789012345678901
If 4x 1 2 , 12, then 4x 1 2 is less than 12. If you needed to solve for x in 2
2
12345678901234567890123456789012123456789012345678901
1 this inequality, the rules are the same as for equations. Whatever you do to 2
12345678901234567890123456789012123456789012345678901 2
2
12345678901234567890123456789012123456789012345678901
one side, you have to do to the other. For example:
2
12345678901234567890123456789012123456789012345678901
2
12345678901234567890123456789012123456789012345678901
2
12345678901234567890123456789012123456789012345678901
4x
1
2
,
12

2
12345678901234567890123456789012123456789012345678901
2
12345678901234567890123456789012123456789012345678901
4x
1
2
2
2
,
12
2
2

2345678901234567890123456789012123456789012345678901
12345678901234567890123456789012123456789012345678901 22
12345678901234567890123456789012123456789012345678901 2
1
4x , 10
2
12345678901234567890123456789012123456789012345678901
2
12345678901234567890123456789012123456789012345678901
4x
4
4
,
10
4
4

2345678901234567890123456789012123456789012345678901
12345678901234567890123456789012123456789012345678901 22
12345678901234567890123456789012123456789012345678901
5
2
12345678901234567890123456789012123456789012345678901
x,
2
12345678901234567890123456789012123456789012345678901
2
2
1
2345678901234567890123456789012123456789012345678901
12345678901234567890123456789012123456789012345678901 22
12345678901234567890123456789012123456789012345678901
There is one caveat to working with inequalities. If you multiply or divide 2
12345678901234567890123456789012123456789012345678901
2
12345678901234567890123456789012123456789012345678901
1 by a negative number, the direction of the inequality sign switches. Freaky, 2
2345678901234567890123456789012123456789012345678901
12345678901234567890123456789012123456789012345678901
eh? Suppose you need to solve the inequality 4 1 2g . 8 2 4g. First, collect 2
2
12345678901234567890123456789012123456789012345678901
2
12345678901234567890123456789012123456789012345678901
all
the
gs
on
one
side:
12345678901234567890123456789012123456789012345678901 22
1
2
12345678901234567890123456789012123456789012345678901
4 1 2g . 8 2 4g
2
12345678901234567890123456789012123456789012345678901
2
12345678901234567890123456789012123456789012345678901
4 2 8 1 2g 2 2 g . 8 2 8 2 4 g 2 2 g
2
12345678901234567890123456789012123456789012345678901
2
12345678901234567890123456789012123456789012345678901
2345678901234567890123456789012123456789012345678901
2
1
2 4 . 2 6g
2
12345678901234567890123456789012123456789012345678901
12345678901234567890123456789012123456789012345678901 22
12345678901234567890123456789012123456789012345678901
123456789012345678901234567890121234567890123456789012 75

http://www.xtremepapers.net

Part III: Section Review

76

123456789012345678901234567890121234567890123456789012
12345678901234567890123456789012123456789012345678901 2
2
12345678901234567890123456789012123456789012345678901
Divide each side by 26 and remember to switch the inequality. You get 2
12345678901234567890123456789012123456789012345678901
2
12345678901234567890123456789012123456789012345678901
2
2
12345678901234567890123456789012123456789012345678901
, g.
2
12345678901234567890123456789012123456789012345678901
3
2
12345678901234567890123456789012123456789012345678901
2345678901234567890123456789012123456789012345678901
2
1
2
12345678901234567890123456789012123456789012345678901
Column
B
Column
A
2
12345678901234567890123456789012123456789012345678901
2
12345678901234567890123456789012123456789012345678901
2
12345678901234567890123456789012123456789012345678901
10 . 2 2x 1 3 . 4
2345678901234567890123456789012123456789012345678901
2
1
2
12345678901234567890123456789012123456789012345678901
x
23
2
12345678901234567890123456789012123456789012345678901
2
12345678901234567890123456789012123456789012345678901
2
12345678901234567890123456789012123456789012345678901
2345678901234567890123456789012123456789012345678901
1 If 10 . 22x 1 3 . 4 looks a little overwhelming, just think of it as two 2
2
12345678901234567890123456789012123456789012345678901
2
12345678901234567890123456789012123456789012345678901
separate
inequalities
(10
.
22x
13
and
22x
1
3
.
4)
and
solve
them
2
12345678901234567890123456789012123456789012345678901
2
12345678901234567890123456789012123456789012345678901
separately.
2345678901234567890123456789012123456789012345678901
2
1
2
12345678901234567890123456789012123456789012345678901
2
12345678901234567890123456789012123456789012345678901
22x
1
3
.
4

2
12345678901234567890123456789012123456789012345678901
2
12345678901234567890123456789012123456789012345678901
22x
1
3
2
3
.
4
2
3

2345678901234567890123456789012123456789012345678901
12345678901234567890123456789012123456789012345678901 2
12345678901234567890123456789012123456789012345678901 2
2
12345678901234567890123456789012123456789012345678901
22x . 1
2
12345678901234567890123456789012123456789012345678901
2
12345678901234567890123456789012123456789012345678901
22x
4
22
.
1
4
22

12345678901234567890123456789012123456789012345678901 2
2
1
1
2
12345678901234567890123456789012123456789012345678901
x,2
2
12345678901234567890123456789012123456789012345678901
2
2
12345678901234567890123456789012123456789012345678901
2345678901234567890123456789012123456789012345678901
2
12345678901234567890123456789012123456789012345678901
and
2
12345678901234567890123456789012123456789012345678901
12345678901234567890123456789012123456789012345678901 2
2
12345678901234567890123456789012123456789012345678901
10 . 22x 1 3
2
12345678901234567890123456789012123456789012345678901
12345678901234567890123456789012123456789012345678901 2
2
12345678901234567890123456789012123456789012345678901
10 2 3 . 22x 1 3 2 3
2
12345678901234567890123456789012123456789012345678901
2
12345678901234567890123456789012123456789012345678901
7
.
22x

2
1
2345678901234567890123456789012123456789012345678901
2
12345678901234567890123456789012123456789012345678901
7 4 22 . 22x 4 22
2
12345678901234567890123456789012123456789012345678901
2
1
2
12345678901234567890123456789012123456789012345678901
7
2
12345678901234567890123456789012123456789012345678901
2 ,x
2345678901234567890123456789012123456789012345678901
2
12345678901234567890123456789012123456789012345678901
2
2
12345678901234567890123456789012123456789012345678901
1 Column B is an actual number, 23. As for Column A, all you know is that 2
2
12345678901234567890123456789012123456789012345678901
2
12345678901234567890123456789012123456789012345678901
x
is
between
negative
seven
halves
(23.5)
and
negative
one
and
one
half.
2345678901234567890123456789012123456789012345678901
12345678901234567890123456789012123456789012345678901 2
2
12345678901234567890123456789012123456789012345678901
1 This means that x could be smaller than, equal to, or greater than 23, 2
2
12345678901234567890123456789012123456789012345678901
depending on where it is within its range. The correct answer is (D).
2
12345678901234567890123456789012123456789012345678901
2345678901234567890123456789012123456789012345678901
12345678901234567890123456789012123456789012345678901 2
12345678901234567890123456789012123456789012345678901 2
12345678901234567890123456789012123456789012345678901 2
12345678901234567890123456789012123456789012345678901 2
2
1A Statistics Excursus (Look It Up!)
2
12345678901234567890123456789012123456789012345678901
2
12345678901234567890123456789012123456789012345678901
The
next
three
statistical
concepts
are
kind
of
like
the
Marx
Brothers.
2
12345678901234567890123456789012123456789012345678901
2
12345678901234567890123456789012123456789012345678901
There
are
three
main
ones,
and
then
another
thats
sort
of
the
Zeppo
of
the
12345678901234567890123456789012123456789012345678901 2
2345678901234567890123456789012123456789012345678901
12345678901234567890123456789012123456789012345678901
bunch. Some combination of mean, median, and mode will make an 2
2
12345678901234567890123456789012123456789012345678901
2
12345678901234567890123456789012123456789012345678901
appearance
on
your
GRE
math
section.
Test-makers
like
to
make
12345678901234567890123456789012123456789012345678901 2
1 questions using these statistical concepts because they test your ability to 2
2
12345678901234567890123456789012123456789012345678901
2
12345678901234567890123456789012123456789012345678901
apply a concept to a math scenario.
2
12345678901234567890123456789012123456789012345678901
12345678901234567890123456789012123456789012345678901 2
2
12345678901234567890123456789012123456789012345678901
12345678901234567890123456789012123456789012345678901 2
12345678901234567890123456789012123456789012345678901 2
12345678901234567890123456789012123456789012345678901 2
12345678901234567890123456789012123456789012345678901 2
123456789012345678901234567890121234567890123456789012

www.petersons.com

http://www.xtremepapers.net

Chapter 4: Quantitative Review

123456789012345678901234567890121234567890123456789012
12345678901234567890123456789012123456789012345678901 2
2
12345678901234567890123456789012123456789012345678901
2
12345678901234567890123456789012123456789012345678901
Statistics Refresher
2
12345678901234567890123456789012123456789012345678901
2
12345678901234567890123456789012123456789012345678901
2
12345678901234567890123456789012123456789012345678901
arithmetic
mean,
mean,
or
sum
of
numbers
divided
by
the
2
12345678901234567890123456789012123456789012345678901
2345678901234567890123456789012123456789012345678901
2
1
average
number of items in set
2
12345678901234567890123456789012123456789012345678901
2
12345678901234567890123456789012123456789012345678901
median
middle number when set is
2
12345678901234567890123456789012123456789012345678901
2
12345678901234567890123456789012123456789012345678901
lined
up
in
numerical
order
2345678901234567890123456789012123456789012345678901
2
1
2
12345678901234567890123456789012123456789012345678901
2
12345678901234567890123456789012123456789012345678901
mode
number
that
appears
most
often
2
12345678901234567890123456789012123456789012345678901
2
12345678901234567890123456789012123456789012345678901
in
a
set
2345678901234567890123456789012123456789012345678901
2
1
2
12345678901234567890123456789012123456789012345678901
range
difference between the greatest
2
12345678901234567890123456789012123456789012345678901
2
12345678901234567890123456789012123456789012345678901
and
smallest
values
in
set
2
12345678901234567890123456789012123456789012345678901
2345678901234567890123456789012123456789012345678901
2
1
2
12345678901234567890123456789012123456789012345678901
2
12345678901234567890123456789012123456789012345678901
To
compare
all
the
terms,
consider
the
number
set,
{14,
12,
8,
15,
5,
8}.
To
2
12345678901234567890123456789012123456789012345678901
2
12345678901234567890123456789012123456789012345678901
determine
the
mean,
you
add
up
the
numbers
in
the
set
(14
1
12
1
8
1
15
2345678901234567890123456789012123456789012345678901
12345678901234567890123456789012123456789012345678901 22
12345678901234567890123456789012123456789012345678901
1 5 1 8 5 62), then divide the sum by number of items in set. There are six 2
12345678901234567890123456789012123456789012345678901
2
12345678901234567890123456789012123456789012345678901
62
1
2
12345678901234567890123456789012123456789012345678901
items,
or
numbers,
total,
so
5
10
.
This
is
the
average
mean
of
the
set.
2
12345678901234567890123456789012123456789012345678901
6
3
2
1
2
12345678901234567890123456789012123456789012345678901
When
people
use
the
word
average
in
everyday
speech,
they
are
referring
2
12345678901234567890123456789012123456789012345678901
2
12345678901234567890123456789012123456789012345678901
to
mean.
But
there
are
two
other
ways
to
find
what
can
be
considered
an
2345678901234567890123456789012123456789012345678901
12345678901234567890123456789012123456789012345678901 22
12345678901234567890123456789012123456789012345678901
average of a group and these are median and mode. To find the median, 2
12345678901234567890123456789012123456789012345678901
12345678901234567890123456789012123456789012345678901
you must first line up the numbers in numerical order, {5, 8, 8, 12, 14, 15}. 2
2
12345678901234567890123456789012123456789012345678901
12345678901234567890123456789012123456789012345678901
You then pick the one in the middle, and thats the median. This works 2
2
12345678901234567890123456789012123456789012345678901
2
12345678901234567890123456789012123456789012345678901
quickly
if
there
is
an
odd
number
of
items
in
the
set,
but
it
takes
a
teensy
bit
12345678901234567890123456789012123456789012345678901 22
1 of work if there is an even number of items in the set. If thats the case, the
2
12345678901234567890123456789012123456789012345678901
2
12345678901234567890123456789012123456789012345678901
there
is
not
one
single
number
in
the
middle.
Instead,
there
are
two
in
the
12345678901234567890123456789012123456789012345678901 2
12345678901234567890123456789012123456789012345678901
middle, 8 and 12. To find the median of this set, you must find the mean of 2
2
12345678901234567890123456789012123456789012345678901
2345678901234567890123456789012123456789012345678901
2
12345678901234567890123456789012123456789012345678901
8 1 12 20
12345678901234567890123456789012123456789012345678901
5
5 10. The median of this group of 2
the two middle numbers.
2
1
2
2
2
12345678901234567890123456789012123456789012345678901
numbers
is
10,
even
though
10
is
not
one
of
the
numbers
listed
in
the
set.
2
12345678901234567890123456789012123456789012345678901
2345678901234567890123456789012123456789012345678901
12345678901234567890123456789012123456789012345678901 22
12345678901234567890123456789012123456789012345678901
1 The mode of a set is the number that appears most often. It is 8 for the 2
12345678901234567890123456789012123456789012345678901
number set in question, since this is the only number that occurs more than 2
2
12345678901234567890123456789012123456789012345678901
2345678901234567890123456789012123456789012345678901
12345678901234567890123456789012123456789012345678901
once. To find the range (the Zeppo of this group), you subtract the smallest 2
2
12345678901234567890123456789012123456789012345678901
2
12345678901234567890123456789012123456789012345678901
number
from
the
greatest
number,
15
2
5
5
10.
12345678901234567890123456789012123456789012345678901 22
1
12345678901234567890123456789012123456789012345678901
The test, though, wont ask you questions as easy as, What is the 2
2
12345678901234567890123456789012123456789012345678901
average? It will expect you to be able to find these different values and 2
12345678901234567890123456789012123456789012345678901
2
12345678901234567890123456789012123456789012345678901
then work a problem from there. For example, with the number set above, 2
12345678901234567890123456789012123456789012345678901
2345678901234567890123456789012123456789012345678901
2
12345678901234567890123456789012123456789012345678901
you might be asked a QC question like:
2
12345678901234567890123456789012123456789012345678901
12345678901234567890123456789012123456789012345678901 22
12345678901234567890123456789012123456789012345678901
Column B
Column A
2
1
2
12345678901234567890123456789012123456789012345678901
2
12345678901234567890123456789012123456789012345678901
{14,
12,
8,
15,
5,
8}
2
12345678901234567890123456789012123456789012345678901
12345678901234567890123456789012123456789012345678901 2
2
12345678901234567890123456789012123456789012345678901
The median of
The range of the set
2
12345678901234567890123456789012123456789012345678901
2
12345678901234567890123456789012123456789012345678901
the set
12345678901234567890123456789012123456789012345678901 22
12345678901234567890123456789012123456789012345678901
123456789012345678901234567890121234567890123456789012 77

http://www.xtremepapers.net

Part III: Section Review

78

123456789012345678901234567890121234567890123456789012
12345678901234567890123456789012123456789012345678901 2
2
12345678901234567890123456789012123456789012345678901
You have already found the value of 10 for both these concepts, so the 2
12345678901234567890123456789012123456789012345678901
2
12345678901234567890123456789012123456789012345678901
answer to this question would be (C). This problem is easy for anyone 2
12345678901234567890123456789012123456789012345678901
2
12345678901234567890123456789012123456789012345678901
knowledgeable with the terms, but to someone who doesnt know what is 2
12345678901234567890123456789012123456789012345678901
2345678901234567890123456789012123456789012345678901
2
1 being asked, it would be time for a one-in-four guess.
2
12345678901234567890123456789012123456789012345678901
2
12345678901234567890123456789012123456789012345678901
2
12345678901234567890123456789012123456789012345678901
2
12345678901234567890123456789012123456789012345678901
Gina
has
received
the
following
four
scores
on
tests:
87,
92,
90,
and
2345678901234567890123456789012123456789012345678901
2
1
2
12345678901234567890123456789012123456789012345678901
85.
If
after
a
fifth
test
Ginas
test
average
was
91,
what
was
her
2
12345678901234567890123456789012123456789012345678901
2
12345678901234567890123456789012123456789012345678901
score
on
the
fifth
test?
2
12345678901234567890123456789012123456789012345678901
2345678901234567890123456789012123456789012345678901
2
1
2
12345678901234567890123456789012123456789012345678901
A. 85
2
12345678901234567890123456789012123456789012345678901
2
B. 88
12345678901234567890123456789012123456789012345678901
2
12345678901234567890123456789012123456789012345678901
C.
95
2345678901234567890123456789012123456789012345678901
2
1
2
12345678901234567890123456789012123456789012345678901
D. 98
2
12345678901234567890123456789012123456789012345678901
2
12345678901234567890123456789012123456789012345678901
E.
101
2
12345678901234567890123456789012123456789012345678901
2345678901234567890123456789012123456789012345678901
12345678901234567890123456789012123456789012345678901 2
12345678901234567890123456789012123456789012345678901
There are two ways to attack this problem. The first is to set up an 2
2
12345678901234567890123456789012123456789012345678901
2
12345678901234567890123456789012123456789012345678901
equation
using
the
average
formula
and
place
a
variable
for
the
unknown
12345678901234567890123456789012123456789012345678901 2
2
12345678901234567890123456789012123456789012345678901
1 fifth test score, 87 1 92 1 90 1 85 1 x 5 91. You then solve for x. The 2
2
12345678901234567890123456789012123456789012345678901
5
2
12345678901234567890123456789012123456789012345678901
12345678901234567890123456789012123456789012345678901
second way of attack is to take the likely answer choices and use them as 2
2345678901234567890123456789012123456789012345678901
2
12345678901234567890123456789012123456789012345678901
the fifth test score and see if the set averages to 91. With a little mental 2
12345678901234567890123456789012123456789012345678901
2
12345678901234567890123456789012123456789012345678901
work, you can guess that neither 85 nor 88 is the correct answer since the 2
12345678901234567890123456789012123456789012345678901
2
12345678901234567890123456789012123456789012345678901
fifth test will need to be above 91 to bring the average up to that. 2
12345678901234567890123456789012123456789012345678901
2
12345678901234567890123456789012123456789012345678901
Proceeding with this approach, you should start with 98, the middle of the 2
12345678901234567890123456789012123456789012345678901
2
12345678901234567890123456789012123456789012345678901
1 three remaining numbers. This way, if 98 is not the answer, you should be 2
2345678901234567890123456789012123456789012345678901
12345678901234567890123456789012123456789012345678901
able to figure out what the right answer is. Placing 98 as the fifth score, 2
2
12345678901234567890123456789012123456789012345678901
1 you get 87 1 92 1 90 1 85 1 98 5 452. However, since 91 3 5 5 455, 2
12345678901234567890123456789012123456789012345678901 2
12345678901234567890123456789012123456789012345678901
you know this answer choice is 3 points too low. You need a higher score, 2
2
12345678901234567890123456789012123456789012345678901
2
12345678901234567890123456789012123456789012345678901
one
preferably
higher
by
three
points.
Looking
at
your
answer
choices,
12345678901234567890123456789012123456789012345678901 2
12345678901234567890123456789012123456789012345678901
there is only one possible choice thats higher, choice (E). Its the correct 2
2
12345678901234567890123456789012123456789012345678901
2345678901234567890123456789012123456789012345678901
2
12345678901234567890123456789012123456789012345678901
answer.
2
12345678901234567890123456789012123456789012345678901
2
1
12345678901234567890123456789012123456789012345678901
This approach might seem unconventional or non-mathy, but thats good. 2
12345678901234567890123456789012123456789012345678901 2
12345678901234567890123456789012123456789012345678901
You dont want to limit yourself to a single method for all problems. The 2
2
12345678901234567890123456789012123456789012345678901
12345678901234567890123456789012123456789012345678901
more creative you get in approaching problems, the more flexibility you 2
2
12345678901234567890123456789012123456789012345678901
give yourself to find the right answer.
2
12345678901234567890123456789012123456789012345678901
2345678901234567890123456789012123456789012345678901
12345678901234567890123456789012123456789012345678901 2
12345678901234567890123456789012123456789012345678901 2
12345678901234567890123456789012123456789012345678901 2
12345678901234567890123456789012123456789012345678901 2
2
1Word Problems
2
12345678901234567890123456789012123456789012345678901
2
12345678901234567890123456789012123456789012345678901
A
train
leaves
Pittsburgh
at
12
p.m.
(EST)
traveling
due
east
at
a
speed
of
2
12345678901234567890123456789012123456789012345678901
2
12345678901234567890123456789012123456789012345678901
45
mph.
A
second
train
leaves
San
Francisco
at
2
p.m.
(PST)
traveling
at
50
12345678901234567890123456789012123456789012345678901 2
2345678901234567890123456789012123456789012345678901
2
12345678901234567890123456789012123456789012345678901
2
1 mph heading due north. When will the two trains cross paths?
2
12345678901234567890123456789012123456789012345678901
12345678901234567890123456789012123456789012345678901 2
2
12345678901234567890123456789012123456789012345678901
12345678901234567890123456789012123456789012345678901 2
12345678901234567890123456789012123456789012345678901 2
12345678901234567890123456789012123456789012345678901 2
12345678901234567890123456789012123456789012345678901 2
123456789012345678901234567890121234567890123456789012

www.petersons.com

http://www.xtremepapers.net

Alert!

Chapter 4: Quantitative Review

123456789012345678901234567890121234567890123456789012
12345678901234567890123456789012123456789012345678901 2
2
12345678901234567890123456789012123456789012345678901
12345678901234567890123456789012123456789012345678901 2
2
12345678901234567890123456789012123456789012345678901
2
12345678901234567890123456789012123456789012345678901
2
12345678901234567890123456789012123456789012345678901
The trains will never meet. The Pittsburgh locomotive will soon run into
2
12345678901234567890123456789012123456789012345678901
2345678901234567890123456789012123456789012345678901
2
1 the Atlantic Ocean, while the San Fran train will take a while before
2
12345678901234567890123456789012123456789012345678901
2
12345678901234567890123456789012123456789012345678901
sliding off into the Pacific.
2
12345678901234567890123456789012123456789012345678901
2
12345678901234567890123456789012123456789012345678901
2345678901234567890123456789012123456789012345678901
2
1
2
12345678901234567890123456789012123456789012345678901
2
12345678901234567890123456789012123456789012345678901
Admittedly, you probably wont see that word problem on test day, but it 2
12345678901234567890123456789012123456789012345678901
2
12345678901234567890123456789012123456789012345678901
does illustrate that word problems are questions that contain a set of math 2
12345678901234567890123456789012123456789012345678901
12345678901234567890123456789012123456789012345678901
conditions in written form. Many word problems on the GRE involve 2
2
12345678901234567890123456789012123456789012345678901
2
12345678901234567890123456789012123456789012345678901
converting
a
statement
in
English
into
an
algebra
equation,
and
then
2
12345678901234567890123456789012123456789012345678901
2345678901234567890123456789012123456789012345678901
1 solving the equation for the desired variable. Suppose you are told that 2
2
12345678901234567890123456789012123456789012345678901
2
12345678901234567890123456789012123456789012345678901
two
doughnuts
and
three
coffees
cost
$2.70
at
the
local
bakery.
At
the
2
12345678901234567890123456789012123456789012345678901
2
12345678901234567890123456789012123456789012345678901
same
bakery,
2
doughnuts
and
a
coffee
cost
$1.50.
You
can
translate
the
2345678901234567890123456789012123456789012345678901
12345678901234567890123456789012123456789012345678901 22
12345678901234567890123456789012123456789012345678901
first sentence into algebra like this:
2
12345678901234567890123456789012123456789012345678901
12345678901234567890123456789012123456789012345678901 22
12345678901234567890123456789012123456789012345678901
2 doughnuts 1 3 coffees 5 $2.70
2
12345678901234567890123456789012123456789012345678901
2
1
2d 1 3c 5 $2.70
2
12345678901234567890123456789012123456789012345678901
2
12345678901234567890123456789012123456789012345678901
2
12345678901234567890123456789012123456789012345678901
With
this
one
equation,
you
dont
have
enough
information
to
determine
2345678901234567890123456789012123456789012345678901
12345678901234567890123456789012123456789012345678901 22
12345678901234567890123456789012123456789012345678901
how much a coffee or a doughnut costs. But if you translate the second 2
12345678901234567890123456789012123456789012345678901
12345678901234567890123456789012123456789012345678901
statement into an equation, then you will have two unknowns and two 2
2
12345678901234567890123456789012123456789012345678901
12345678901234567890123456789012123456789012345678901
equations. This will give you the same set-up as seen on page 70 earlier, 2
2
12345678901234567890123456789012123456789012345678901
2
12345678901234567890123456789012123456789012345678901
when
you
solve
for
two
variables
using
two
equations.
12345678901234567890123456789012123456789012345678901 22
1
12345678901234567890123456789012123456789012345678901
The second equation is 2d 1 c 5 $1.50. If you subtract the second 2
2
12345678901234567890123456789012123456789012345678901
12345678901234567890123456789012123456789012345678901
equation from the first, the 2d terms cancels out and you are left with 2
12345678901234567890123456789012123456789012345678901 2
12345678901234567890123456789012123456789012345678901
2c 5 $1.20 c 5 $0.60. Plugging in the price for coffee into the second 2
2
12345678901234567890123456789012123456789012345678901
equation, you can solve for d, which is 0.45. Ta da! You used algebra to 2
12345678901234567890123456789012123456789012345678901
2
12345678901234567890123456789012123456789012345678901
decipher a word problem and then manipulated two equations to find the 2
12345678901234567890123456789012123456789012345678901
2
12345678901234567890123456789012123456789012345678901
values of two unknown variables. Youll be an expert GRE mathematician 2
12345678901234567890123456789012123456789012345678901
2
12345678901234567890123456789012123456789012345678901
yet!
2
12345678901234567890123456789012123456789012345678901
12345678901234567890123456789012123456789012345678901 2
12345678901234567890123456789012123456789012345678901
The key to translating English into algebra is to do it piece by piece. 2
2
12345678901234567890123456789012123456789012345678901
2
12345678901234567890123456789012123456789012345678901
Suppose
a
question
states,
The
difference
between
Tom
and
Johns
age
is
2
12345678901234567890123456789012123456789012345678901
2
12345678901234567890123456789012123456789012345678901
13,
and
Tom
is
older.
The
term
difference
means
you
are
subtracting,
and
12345678901234567890123456789012123456789012345678901 2
2345678901234567890123456789012123456789012345678901
12345678901234567890123456789012123456789012345678901
the word is means equals, so our translation would be T 2 J 5 13 (it is 2
2
12345678901234567890123456789012123456789012345678901
2
12345678901234567890123456789012123456789012345678901
T
minus
J
because
T
is
older).
12345678901234567890123456789012123456789012345678901 22
1
12345678901234567890123456789012123456789012345678901
There are a number of key words and phrases often used when translating 2
2
12345678901234567890123456789012123456789012345678901
2
12345678901234567890123456789012123456789012345678901
English to algebra, as you can see from the chart on the next page.
2
12345678901234567890123456789012123456789012345678901
12345678901234567890123456789012123456789012345678901 2
2345678901234567890123456789012123456789012345678901
12345678901234567890123456789012123456789012345678901 22
1
2
12345678901234567890123456789012123456789012345678901
12345678901234567890123456789012123456789012345678901 2
2
12345678901234567890123456789012123456789012345678901
12345678901234567890123456789012123456789012345678901 2
12345678901234567890123456789012123456789012345678901 2
12345678901234567890123456789012123456789012345678901 2
12345678901234567890123456789012123456789012345678901 2 79
123456789012345678901234567890121234567890123456789012

http://www.xtremepapers.net

Part III: Section Review

80

123456789012345678901234567890121234567890123456789012
12345678901234567890123456789012123456789012345678901 2
2
12345678901234567890123456789012123456789012345678901
2
12345678901234567890123456789012123456789012345678901
Translation Key
2
12345678901234567890123456789012123456789012345678901
2
12345678901234567890123456789012123456789012345678901
2
12345678901234567890123456789012123456789012345678901
If
You
See
This
Do
This
2
12345678901234567890123456789012123456789012345678901
2345678901234567890123456789012123456789012345678901
2
1
sum, plus, added to, combined with, greater
Add
2
12345678901234567890123456789012123456789012345678901
2
12345678901234567890123456789012123456789012345678901
than, increased by
2
12345678901234567890123456789012123456789012345678901
2
12345678901234567890123456789012123456789012345678901
2345678901234567890123456789012123456789012345678901
2
1
difference between, minus, subtracted from,
Subtract
2
12345678901234567890123456789012123456789012345678901
2
12345678901234567890123456789012123456789012345678901
less
than,
reduced
by
2
12345678901234567890123456789012123456789012345678901
2
12345678901234567890123456789012123456789012345678901
product,
times,
multiplied
by,
twice,
doubled,
Multiply
2345678901234567890123456789012123456789012345678901
2
1
2
12345678901234567890123456789012123456789012345678901
half
2
12345678901234567890123456789012123456789012345678901
2
12345678901234567890123456789012123456789012345678901
2
12345678901234567890123456789012123456789012345678901
quotient,
divided
by,
per,
ratio
of
blank
to
Divide
2345678901234567890123456789012123456789012345678901
2
1
2
12345678901234567890123456789012123456789012345678901
blank, out of
2
12345678901234567890123456789012123456789012345678901
2
12345678901234567890123456789012123456789012345678901
equals,
will
be,
was,
is
the
same
as,
costs,
Equals
2
12345678901234567890123456789012123456789012345678901
2345678901234567890123456789012123456789012345678901
2
12345678901234567890123456789012123456789012345678901
adds up to, results in
2
12345678901234567890123456789012123456789012345678901
12345678901234567890123456789012123456789012345678901 2
12345678901234567890123456789012123456789012345678901 2
12345678901234567890123456789012123456789012345678901 2
2
12345678901234567890123456789012123456789012345678901
A mixture of 14 ounces of water and oil is 50% water (by weight).
2
1
How many ounces of oil must be added to the mixture to produce a 2
12345678901234567890123456789012123456789012345678901
2
12345678901234567890123456789012123456789012345678901
new mixture that is only 25% water?
2
12345678901234567890123456789012123456789012345678901
2345678901234567890123456789012123456789012345678901
12345678901234567890123456789012123456789012345678901 2
2
12345678901234567890123456789012123456789012345678901
A. 10
2
12345678901234567890123456789012123456789012345678901
2
12345678901234567890123456789012123456789012345678901
B.
14
12345678901234567890123456789012123456789012345678901 2
2
12345678901234567890123456789012123456789012345678901
C. 16
2
12345678901234567890123456789012123456789012345678901
2
12345678901234567890123456789012123456789012345678901
D.
20
12345678901234567890123456789012123456789012345678901 2
2
1
E. 28
2
12345678901234567890123456789012123456789012345678901
2
12345678901234567890123456789012123456789012345678901
2
12345678901234567890123456789012123456789012345678901
This
is
a
little
tricky,
so
we
have
a
little
unpacking
to
do.
In
the
first
12345678901234567890123456789012123456789012345678901 2
12345678901234567890123456789012123456789012345678901
mixture, 50% of the 14 ounces is water, which we could represent by 2
2
12345678901234567890123456789012123456789012345678901
2
12345678901234567890123456789012123456789012345678901
14~0.5!
12345678901234567890123456789012123456789012345678901
5 0.5. The numerator is the number of ounces of water, and the 2
2
12345678901234567890123456789012123456789012345678901
14
2
12345678901234567890123456789012123456789012345678901
denominator
is
the
total
number
of
ounces.
That
might
seem
obvious,
but
2345678901234567890123456789012123456789012345678901
12345678901234567890123456789012123456789012345678901 2
2
12345678901234567890123456789012123456789012345678901
1 it is helpful for being able to write down the second mixture, which is 2
12345678901234567890123456789012123456789012345678901
going to have ounces of oil added to it (added to the denominator in this 2
2
12345678901234567890123456789012123456789012345678901
2345678901234567890123456789012123456789012345678901
12345678901234567890123456789012123456789012345678901
case) until the percentage equals 0.25. This can be written down as 2
2
12345678901234567890123456789012123456789012345678901
2
12345678901234567890123456789012123456789012345678901
0.5
14
~
!
2
12345678901234567890123456789012123456789012345678901
5
0.25.
All
that
remains
is
to
solve
for
x,
which
after
some
2
1 14 1 x
2345678901234567890123456789012123456789012345678901
12345678901234567890123456789012123456789012345678901 2
12345678901234567890123456789012123456789012345678901
working will get you x 5 14, choice (B). The alternate way to do this 2
2
12345678901234567890123456789012123456789012345678901
problem is to start with the answer choices and work backward. Always 2
12345678901234567890123456789012123456789012345678901
1 start with the middle value so that if it does not yield the right answer, you 2
2
12345678901234567890123456789012123456789012345678901
2
12345678901234567890123456789012123456789012345678901
will know if you need to go higher or lower (and so eliminate the answer 2
12345678901234567890123456789012123456789012345678901
2
12345678901234567890123456789012123456789012345678901
choices in the opposite direction). If you add 16 ounces of oil, then you 2
12345678901234567890123456789012123456789012345678901
2345678901234567890123456789012123456789012345678901
2
12345678901234567890123456789012123456789012345678901
1 will have a total of 30 ounces with 7 ounces of water. That is a little under 2
12345678901234567890123456789012123456789012345678901
25% water, so 16 is a little too much to add. 14 then would be a good 2
2
12345678901234567890123456789012123456789012345678901
2
12345678901234567890123456789012123456789012345678901
guess
if
you
are
crunched
for
time.
2345678901234567890123456789012123456789012345678901
2
1
12345678901234567890123456789012123456789012345678901 2
12345678901234567890123456789012123456789012345678901 2
12345678901234567890123456789012123456789012345678901 2
123456789012345678901234567890121234567890123456789012

www.petersons.com

http://www.xtremepapers.net

Chapter 4: Quantitative Review

Tip

123456789012345678901234567890121234567890123456789012
12345678901234567890123456789012123456789012345678901 2
2
12345678901234567890123456789012123456789012345678901
12345678901234567890123456789012123456789012345678901 2
Geometry
2
12345678901234567890123456789012123456789012345678901
2
12345678901234567890123456789012123456789012345678901
Fortunately,
there
will
be
no
proofs
and
no
mention
of
Euclid
in
the
GRE,
12345678901234567890123456789012123456789012345678901 2
12345678901234567890123456789012123456789012345678901
so you wont have to revisit those painful experiences from high school. 2
2
12345678901234567890123456789012123456789012345678901
2
12345678901234567890123456789012123456789012345678901
There
is
a
limited
set
of
basic
geometry
that
you
will
be
expected
to
2
12345678901234567890123456789012123456789012345678901
2
12345678901234567890123456789012123456789012345678901
understand
and
apply
to
solve
problems,
so
the
first
order
of
business
is
2
12345678901234567890123456789012123456789012345678901
2345678901234567890123456789012123456789012345678901
2
1 the review of this material.
2
12345678901234567890123456789012123456789012345678901
2
12345678901234567890123456789012123456789012345678901
2
12345678901234567890123456789012123456789012345678901
2
12345678901234567890123456789012123456789012345678901
2345678901234567890123456789012123456789012345678901
2
1Lines and Angles
2
12345678901234567890123456789012123456789012345678901
2
12345678901234567890123456789012123456789012345678901
It doesnt get much more basic than the line. Most of the geometric figures 2
12345678901234567890123456789012123456789012345678901
2
12345678901234567890123456789012123456789012345678901
that you see on test day will be composed of lines or segments of lines, with 2
12345678901234567890123456789012123456789012345678901
2
12345678901234567890123456789012123456789012345678901
circles being the major exception.
2
12345678901234567890123456789012123456789012345678901
2
12345678901234567890123456789012123456789012345678901
2
12345678901234567890123456789012123456789012345678901
2345678901234567890123456789012123456789012345678901
12345678901234567890123456789012123456789012345678901 22
12345678901234567890123456789012123456789012345678901 2
12345678901234567890123456789012123456789012345678901 2
12345678901234567890123456789012123456789012345678901
The test will use the notation AB sometimes to refer to the line segment 2
12345678901234567890123456789012123456789012345678901
2
12345678901234567890123456789012123456789012345678901
1 AB. This notation will also refer to the length of the line segment AB. You 2
2
12345678901234567890123456789012123456789012345678901
will be able to tell from the context which is intended.
2
12345678901234567890123456789012123456789012345678901
2
12345678901234567890123456789012123456789012345678901
2345678901234567890123456789012123456789012345678901
2
12345678901234567890123456789012123456789012345678901
The angle is one step up in complexity. It will include at least two lines.
2
12345678901234567890123456789012123456789012345678901
12345678901234567890123456789012123456789012345678901 22
12345678901234567890123456789012123456789012345678901 2
12345678901234567890123456789012123456789012345678901 2
12345678901234567890123456789012123456789012345678901 2
12345678901234567890123456789012123456789012345678901 2
12345678901234567890123456789012123456789012345678901 2
12345678901234567890123456789012123456789012345678901 2
1
2
12345678901234567890123456789012123456789012345678901
2
12345678901234567890123456789012123456789012345678901
12345678901234567890123456789012123456789012345678901 2
12345678901234567890123456789012123456789012345678901 2
2
12345678901234567890123456789012123456789012345678901
When two lines intersect as in the above figure, the angles across from each 2
12345678901234567890123456789012123456789012345678901
2
12345678901234567890123456789012123456789012345678901
other are called vertical angles, and they are equal (so a 5 c and b 5 d). 2
12345678901234567890123456789012123456789012345678901
2
12345678901234567890123456789012123456789012345678901
The sum of the four angles is 360 degrees, while the sum of any of the 2
12345678901234567890123456789012123456789012345678901
2345678901234567890123456789012123456789012345678901
12345678901234567890123456789012123456789012345678901
angles adjacent to each other is 180 degrees ( a 1 b or d 1 c, for 2
2
12345678901234567890123456789012123456789012345678901
1 example). The adjacent angles sum to 180 degrees because two adjacent 2
12345678901234567890123456789012123456789012345678901 2
12345678901234567890123456789012123456789012345678901
angles in our example define the angle measure of a straight line, and the 2
2
12345678901234567890123456789012123456789012345678901
2
12345678901234567890123456789012123456789012345678901
measure
of
a
straight
line
is
180
degrees.
If
a
5
90,
then
you
could
deduce
2
12345678901234567890123456789012123456789012345678901
2
12345678901234567890123456789012123456789012345678901
that
all
four
of
the
angles
are
90
degrees
because
if
a
5
90
then
c
5
90,
12345678901234567890123456789012123456789012345678901 2
2345678901234567890123456789012123456789012345678901
12345678901234567890123456789012123456789012345678901
and if c 5 90 then d 5 90 because d and c must sum to 180 degrees, and 2
2
12345678901234567890123456789012123456789012345678901
2
12345678901234567890123456789012123456789012345678901
so
on
around
the
diagram.
Ninety-degree
angles
also
go
by
the
special
12345678901234567890123456789012123456789012345678901 22
1 name of right angles. They have a special symbol as well, the .
2
12345678901234567890123456789012123456789012345678901
2
12345678901234567890123456789012123456789012345678901
2
12345678901234567890123456789012123456789012345678901
2
12345678901234567890123456789012123456789012345678901
2
12345678901234567890123456789012123456789012345678901
Angles
that
add
up
to
180
degrees
are
called
supplementary
angles.
2345678901234567890123456789012123456789012345678901
12345678901234567890123456789012123456789012345678901 22
1 Angles that add up to 90 degrees are called complementary angles.
2
12345678901234567890123456789012123456789012345678901
12345678901234567890123456789012123456789012345678901 2
2
12345678901234567890123456789012123456789012345678901
12345678901234567890123456789012123456789012345678901 2
12345678901234567890123456789012123456789012345678901 2
12345678901234567890123456789012123456789012345678901 2
12345678901234567890123456789012123456789012345678901 2 81
123456789012345678901234567890121234567890123456789012

http://www.xtremepapers.net

Part III: Section Review

82

123456789012345678901234567890121234567890123456789012
12345678901234567890123456789012123456789012345678901 2
2
12345678901234567890123456789012123456789012345678901
12345678901234567890123456789012123456789012345678901 2
2
12345678901234567890123456789012123456789012345678901
2
12345678901234567890123456789012123456789012345678901
12345678901234567890123456789012123456789012345678901 2
2
12345678901234567890123456789012123456789012345678901
12345678901234567890123456789012123456789012345678901 2
2
12345678901234567890123456789012123456789012345678901
2
12345678901234567890123456789012123456789012345678901
12345678901234567890123456789012123456789012345678901
If two lines intersect and the angles formed by their intersection are right 2
2
12345678901234567890123456789012123456789012345678901
12345678901234567890123456789012123456789012345678901
angles, the lines are said to be perpendicular. In our example, if the angles 2
2
12345678901234567890123456789012123456789012345678901
2
12345678901234567890123456789012123456789012345678901
are right angles then l1 is perpendicular to l2, which is denoted as l1 l2.
2
12345678901234567890123456789012123456789012345678901
2
12345678901234567890123456789012123456789012345678901
Two
lines
can
have
one
other
kind
of
special
relationship,
which
is
known
2345678901234567890123456789012123456789012345678901
2
1
12345678901234567890123456789012123456789012345678901
as parallel and denoted by two close-together straight lines such as l1 i l2. 2
2
12345678901234567890123456789012123456789012345678901
2
12345678901234567890123456789012123456789012345678901
Like
train
tracks,
parallel
lines
never
intersect.
Two
parallel
lines
are
not
2
12345678901234567890123456789012123456789012345678901
2345678901234567890123456789012123456789012345678901
1 that interesting, but if you cross two parallel lines with a third line, 2
2
12345678901234567890123456789012123456789012345678901
2
12345678901234567890123456789012123456789012345678901
hoo
boy!
2
12345678901234567890123456789012123456789012345678901
2
12345678901234567890123456789012123456789012345678901
2345678901234567890123456789012123456789012345678901
12345678901234567890123456789012123456789012345678901 2
12345678901234567890123456789012123456789012345678901 2
12345678901234567890123456789012123456789012345678901 2
12345678901234567890123456789012123456789012345678901 2
12345678901234567890123456789012123456789012345678901 2
12345678901234567890123456789012123456789012345678901 2
2
1
2
12345678901234567890123456789012123456789012345678901
2
12345678901234567890123456789012123456789012345678901
2
12345678901234567890123456789012123456789012345678901
2345678901234567890123456789012123456789012345678901
12345678901234567890123456789012123456789012345678901 2
12345678901234567890123456789012123456789012345678901 2
12345678901234567890123456789012123456789012345678901 2
12345678901234567890123456789012123456789012345678901
Since they are vertical angles, 1 equals 4, 2 equals 3, 5 equals 8, and 6 2
2
12345678901234567890123456789012123456789012345678901
12345678901234567890123456789012123456789012345678901
equals 7. Also, angles 2 and 4, 1 and 3, 5 and 6, 5 and 7, and a host of 2
2
12345678901234567890123456789012123456789012345678901
2
12345678901234567890123456789012123456789012345678901
others
are
all
supplementary
angles.
Thats
just
the
start.
Alternate
interior
12345678901234567890123456789012123456789012345678901 2
1 angles are also equal. To unpack that term, consider that interior refers to 2
2
12345678901234567890123456789012123456789012345678901
12345678901234567890123456789012123456789012345678901
an angle on the inside (3, 4, 5, 6) of the parallel lines. Alternate means the 2
12345678901234567890123456789012123456789012345678901 2
12345678901234567890123456789012123456789012345678901
angles that are on different sides of the non-parallel line (3 and 6, 4 and 5). 2
2
12345678901234567890123456789012123456789012345678901
2345678901234567890123456789012123456789012345678901
12345678901234567890123456789012123456789012345678901
So 3 5 6 and 4 5 5. Notice this means we can deduce a lot more about the 2
2
12345678901234567890123456789012123456789012345678901
2
1 relationships of the different angles (1 5 8, 2 5 7).
2
12345678901234567890123456789012123456789012345678901
2
12345678901234567890123456789012123456789012345678901
Column
B
Column
A
2345678901234567890123456789012123456789012345678901
12345678901234567890123456789012123456789012345678901 2
12345678901234567890123456789012123456789012345678901 2
2
1
12345678901234567890123456789012123456789012345678901 2
12345678901234567890123456789012123456789012345678901 2
2
12345678901234567890123456789012123456789012345678901
2
12345678901234567890123456789012123456789012345678901
2
12345678901234567890123456789012123456789012345678901
2
12345678901234567890123456789012123456789012345678901
12345678901234567890123456789012123456789012345678901 2
2345678901234567890123456789012123456789012345678901
12345678901234567890123456789012123456789012345678901 2
12345678901234567890123456789012123456789012345678901 2
12345678901234567890123456789012123456789012345678901 2
2
l1 i l2
12345678901234567890123456789012123456789012345678901
2
1
2345678901234567890123456789012123456789012345678901
2
12345678901234567890123456789012123456789012345678901
a
b
2
12345678901234567890123456789012123456789012345678901
12345678901234567890123456789012123456789012345678901 2
12345678901234567890123456789012123456789012345678901 2
1 This looks a little different than the figure we were just working with, but 2
2
12345678901234567890123456789012123456789012345678901
12345678901234567890123456789012123456789012345678901
if you rotate it ninety degrees, you can see that it is basically the same. 2
2
12345678901234567890123456789012123456789012345678901
12345678901234567890123456789012123456789012345678901
b 5 90 because it and the right angle must sum to 180 (supplementary 2
2
12345678901234567890123456789012123456789012345678901
angles). At this point there are two ways to get the answer. You could 2
12345678901234567890123456789012123456789012345678901
2
12345678901234567890123456789012123456789012345678901
reason, Two lines that are parallel will intersect a third line at the same 2
12345678901234567890123456789012123456789012345678901
12345678901234567890123456789012123456789012345678901 2
123456789012345678901234567890121234567890123456789012

www.petersons.com

http://www.xtremepapers.net

Chapter 4: Quantitative Review

123456789012345678901234567890121234567890123456789012
12345678901234567890123456789012123456789012345678901 2
2
12345678901234567890123456789012123456789012345678901
angle, which would mean that a is a right angle. If you didnt see that, 2
12345678901234567890123456789012123456789012345678901
2
12345678901234567890123456789012123456789012345678901
dont worry, since there is another way to get the answer. b is equal to the 2
12345678901234567890123456789012123456789012345678901
2
12345678901234567890123456789012123456789012345678901
angle across from it (vertical angles), and so b is also equal to the angle 2
12345678901234567890123456789012123456789012345678901
2345678901234567890123456789012123456789012345678901
1 across from a (alternate interior angles). So a and b are equal since they are 2
2
12345678901234567890123456789012123456789012345678901
2
12345678901234567890123456789012123456789012345678901
equal
to
the
same
angle.
The
correct
answer
is
(C).
2
12345678901234567890123456789012123456789012345678901
2
12345678901234567890123456789012123456789012345678901
2345678901234567890123456789012123456789012345678901
2
1
2
12345678901234567890123456789012123456789012345678901
2
12345678901234567890123456789012123456789012345678901
2
12345678901234567890123456789012123456789012345678901
Triangles
2
12345678901234567890123456789012123456789012345678901
2345678901234567890123456789012123456789012345678901
1 Triangles are tri-angled geometric figures that have three angles and three 2
2
12345678901234567890123456789012123456789012345678901
2
12345678901234567890123456789012123456789012345678901
sides.
There
are
many
different
kinds
of
triangles
that
all
have
different
2
12345678901234567890123456789012123456789012345678901
2
12345678901234567890123456789012123456789012345678901
names,
but
we
dont
need
to
concern
ourselves
with
those
here.
The
GRE
2345678901234567890123456789012123456789012345678901
2
1
12345678901234567890123456789012123456789012345678901
will test some general principles about triangles and will also focus on a 2
2
12345678901234567890123456789012123456789012345678901
12345678901234567890123456789012123456789012345678901
few special kinds of triangles. The wisest course of action is to review these 2
2
12345678901234567890123456789012123456789012345678901
12345678901234567890123456789012123456789012345678901
general principles and then get really familiar with the triangles that are 2
2
12345678901234567890123456789012123456789012345678901
2
12345678901234567890123456789012123456789012345678901
the GREs favorites.
2
12345678901234567890123456789012123456789012345678901
2
12345678901234567890123456789012123456789012345678901
A
rule
that
holds
true
for
all
triangles
is
that
the
three
angles
sum
to
180
2345678901234567890123456789012123456789012345678901
12345678901234567890123456789012123456789012345678901 22
1 degrees. This means that if you know two angles of a triangle, then you can
2
12345678901234567890123456789012123456789012345678901
2
12345678901234567890123456789012123456789012345678901
find
the
third.
So
if
a
triangle
has
two
angles
that
are
equal
and
third
angle
2
12345678901234567890123456789012123456789012345678901
2345678901234567890123456789012123456789012345678901
2
12345678901234567890123456789012123456789012345678901
is 50 degrees, what is the measure of the other two angles?
2
12345678901234567890123456789012123456789012345678901
12345678901234567890123456789012123456789012345678901 22
12345678901234567890123456789012123456789012345678901
To answer this, lets go back in this chapter and employ some algebra. 2
12345678901234567890123456789012123456789012345678901
2
12345678901234567890123456789012123456789012345678901
Calling the two equal angles x, you get
2
12345678901234567890123456789012123456789012345678901
12345678901234567890123456789012123456789012345678901 22
12345678901234567890123456789012123456789012345678901
2x 1 50 5 180
2
1
2
12345678901234567890123456789012123456789012345678901
2x 1 50 2 50 5 180 2 50
2
12345678901234567890123456789012123456789012345678901
12345678901234567890123456789012123456789012345678901 2
2
12345678901234567890123456789012123456789012345678901
2x 5 130
2
12345678901234567890123456789012123456789012345678901
2345678901234567890123456789012123456789012345678901
2
12345678901234567890123456789012123456789012345678901
2x 4 2 5 130 4 2
2
12345678901234567890123456789012123456789012345678901
2
1
x
5
65
2
12345678901234567890123456789012123456789012345678901
2
12345678901234567890123456789012123456789012345678901
2345678901234567890123456789012123456789012345678901
12345678901234567890123456789012123456789012345678901
Another general rule for trianglesreferenced earlier in the bookis that 2
2
12345678901234567890123456789012123456789012345678901
1 any two sides of a triangle must be longer than the third side. Otherwise, 2
12345678901234567890123456789012123456789012345678901 2
12345678901234567890123456789012123456789012345678901
the sides would not meet. You might need this information on a QC where 2
2
12345678901234567890123456789012123456789012345678901
12345678901234567890123456789012123456789012345678901
one column is the length of the longest side of the triangle and the other 2
2
12345678901234567890123456789012123456789012345678901
12345678901234567890123456789012123456789012345678901
column is the maximum length of the other shorter sides combined. In that 2
12345678901234567890123456789012123456789012345678901 2
12345678901234567890123456789012123456789012345678901
case, the other two sides must be longer for the figure to actually be a 2
2
12345678901234567890123456789012123456789012345678901
2
12345678901234567890123456789012123456789012345678901
triangle.
2
12345678901234567890123456789012123456789012345678901
2
12345678901234567890123456789012123456789012345678901
Another
principle
that
holds
for
all
triangles
is
the
formula
for
the
area
of
2345678901234567890123456789012123456789012345678901
12345678901234567890123456789012123456789012345678901 22
12345678901234567890123456789012123456789012345678901
1
2
12345678901234567890123456789012123456789012345678901
a triangle, A 5 ~base!~height!. Using this formula would be simple if you 2
12345678901234567890123456789012123456789012345678901
2
2
1
12345678901234567890123456789012123456789012345678901
knew what the base and height of the triangle are, but most problems 2
2
12345678901234567890123456789012123456789012345678901
12345678901234567890123456789012123456789012345678901
arent going to tell you that directly. To find them, you must know that the 2
12345678901234567890123456789012123456789012345678901 2
12345678901234567890123456789012123456789012345678901
height is the measure of any line segment that begins at one of the vertices 2
2
12345678901234567890123456789012123456789012345678901
12345678901234567890123456789012123456789012345678901
and ends at the opposite side, forming a right angle with that side, and that 2
12345678901234567890123456789012123456789012345678901 2
12345678901234567890123456789012123456789012345678901 2 83
123456789012345678901234567890121234567890123456789012

http://www.xtremepapers.net

Part III: Section Review

84

123456789012345678901234567890121234567890123456789012
12345678901234567890123456789012123456789012345678901 2
2
12345678901234567890123456789012123456789012345678901
the base is the length of the line that the height forms the right angle with. 2
12345678901234567890123456789012123456789012345678901
2
12345678901234567890123456789012123456789012345678901
Look at this figure if that explanation didnt clear everything up for you.
2
12345678901234567890123456789012123456789012345678901
12345678901234567890123456789012123456789012345678901 2
2
12345678901234567890123456789012123456789012345678901
12345678901234567890123456789012123456789012345678901 2
2
12345678901234567890123456789012123456789012345678901
2
12345678901234567890123456789012123456789012345678901
2
12345678901234567890123456789012123456789012345678901
2
12345678901234567890123456789012123456789012345678901
2345678901234567890123456789012123456789012345678901
2
1
2
12345678901234567890123456789012123456789012345678901
2
12345678901234567890123456789012123456789012345678901
2
12345678901234567890123456789012123456789012345678901
2
12345678901234567890123456789012123456789012345678901
2345678901234567890123456789012123456789012345678901
2
1
2
12345678901234567890123456789012123456789012345678901
One
thing
to
keep
in
mind
is
that
the
base
and
the
height
of
a
triangle
are
2
12345678901234567890123456789012123456789012345678901
2
12345678901234567890123456789012123456789012345678901
not
fixed
values.
In
the
figure
above,
we
could
have
drawn
a
line
from
C
2
12345678901234567890123456789012123456789012345678901
2345678901234567890123456789012123456789012345678901
1 and perpendicular with AB. In that case, the height would be the 2
2
12345678901234567890123456789012123456789012345678901
12345678901234567890123456789012123456789012345678901
perpendicular line going from C to AB, and the base would be AB. To 2
2
12345678901234567890123456789012123456789012345678901
12345678901234567890123456789012123456789012345678901
decide what to make the height and what to make the base, follow this rule 2
2345678901234567890123456789012123456789012345678901
12345678901234567890123456789012123456789012345678901 2
12345678901234567890123456789012123456789012345678901
of thumb: find the base and height wherever it is easiest. Generally it is 2
2
12345678901234567890123456789012123456789012345678901
12345678901234567890123456789012123456789012345678901
harder to find the heightunless you can draw the height so that it is one 2
2
12345678901234567890123456789012123456789012345678901
of the triangle sidesso approach an area problem looking for the easiest 2
12345678901234567890123456789012123456789012345678901
2
1 way to find the height.
2
12345678901234567890123456789012123456789012345678901
2
12345678901234567890123456789012123456789012345678901
2
12345678901234567890123456789012123456789012345678901
2345678901234567890123456789012123456789012345678901
12345678901234567890123456789012123456789012345678901 2
12345678901234567890123456789012123456789012345678901 2
12345678901234567890123456789012123456789012345678901 2
12345678901234567890123456789012123456789012345678901 2
12345678901234567890123456789012123456789012345678901 2
12345678901234567890123456789012123456789012345678901 2
12345678901234567890123456789012123456789012345678901 2
2
12345678901234567890123456789012123456789012345678901
Sometimes you have to get really creative in finding the height of a 2
12345678901234567890123456789012123456789012345678901
1 triangle. That is what the figure above shows. The height doesnt have to 2
2
12345678901234567890123456789012123456789012345678901
2
12345678901234567890123456789012123456789012345678901
be a line inside the triangle. The height is the length of the line from one 2
12345678901234567890123456789012123456789012345678901
2
12345678901234567890123456789012123456789012345678901
angle to its opposite side if that line is perpendicular to the opposite side. 2
12345678901234567890123456789012123456789012345678901
2345678901234567890123456789012123456789012345678901
12345678901234567890123456789012123456789012345678901 2
12345678901234567890123456789012123456789012345678901 2
2
1
2
12345678901234567890123456789012123456789012345678901
2
12345678901234567890123456789012123456789012345678901
2345678901234567890123456789012123456789012345678901
12345678901234567890123456789012123456789012345678901 2
12345678901234567890123456789012123456789012345678901 2
2
1
12345678901234567890123456789012123456789012345678901 2
12345678901234567890123456789012123456789012345678901 2
2
12345678901234567890123456789012123456789012345678901
2
12345678901234567890123456789012123456789012345678901
2
12345678901234567890123456789012123456789012345678901
What
is
the
area
of
the
above
figure?
2
12345678901234567890123456789012123456789012345678901
12345678901234567890123456789012123456789012345678901 2
2345678901234567890123456789012123456789012345678901
2
12345678901234567890123456789012123456789012345678901
A. 3
2
12345678901234567890123456789012123456789012345678901
B. 6
2
12345678901234567890123456789012123456789012345678901
2
12345678901234567890123456789012123456789012345678901
C. 8
2
1
2345678901234567890123456789012123456789012345678901
2
12345678901234567890123456789012123456789012345678901
D. 9
2
12345678901234567890123456789012123456789012345678901
2
12345678901234567890123456789012123456789012345678901
E. 12
2
12345678901234567890123456789012123456789012345678901
2
1
2345678901234567890123456789012123456789012345678901
12345678901234567890123456789012123456789012345678901 2
2
1
2
12345678901234567890123456789012123456789012345678901
12345678901234567890123456789012123456789012345678901 2
2
12345678901234567890123456789012123456789012345678901
12345678901234567890123456789012123456789012345678901 2
12345678901234567890123456789012123456789012345678901 2
12345678901234567890123456789012123456789012345678901 2
12345678901234567890123456789012123456789012345678901 2
123456789012345678901234567890121234567890123456789012

www.petersons.com

http://www.xtremepapers.net

Chapter 4: Quantitative Review

123456789012345678901234567890121234567890123456789012
12345678901234567890123456789012123456789012345678901 2
2
12345678901234567890123456789012123456789012345678901
With the triangle-ometry that we have covered thus far, there is only one 2
12345678901234567890123456789012123456789012345678901
2
12345678901234567890123456789012123456789012345678901
way to pick the height and base so that you can solve this problem.
2
12345678901234567890123456789012123456789012345678901
12345678901234567890123456789012123456789012345678901 2
2
12345678901234567890123456789012123456789012345678901
12345678901234567890123456789012123456789012345678901 2
2
12345678901234567890123456789012123456789012345678901
2
12345678901234567890123456789012123456789012345678901
2
12345678901234567890123456789012123456789012345678901
2
12345678901234567890123456789012123456789012345678901
2345678901234567890123456789012123456789012345678901
2
1
2
12345678901234567890123456789012123456789012345678901
2
12345678901234567890123456789012123456789012345678901
2
12345678901234567890123456789012123456789012345678901
2
12345678901234567890123456789012123456789012345678901
2345678901234567890123456789012123456789012345678901
1 You can pick one of the sides to be the height since it forms a right angle 2
2
12345678901234567890123456789012123456789012345678901
1
2
12345678901234567890123456789012123456789012345678901
12345678901234567890123456789012123456789012345678901
with the side it intersects. So A 5 ~3!~4! 5 6. Therefore, the correct 2
2
12345678901234567890123456789012123456789012345678901
2
2
12345678901234567890123456789012123456789012345678901
answer is (B).
2
12345678901234567890123456789012123456789012345678901
2
12345678901234567890123456789012123456789012345678901
2
12345678901234567890123456789012123456789012345678901
2
12345678901234567890123456789012123456789012345678901
2345678901234567890123456789012123456789012345678901
12345678901234567890123456789012123456789012345678901 22
12345678901234567890123456789012123456789012345678901
Special Triangles
12345678901234567890123456789012123456789012345678901 22
12345678901234567890123456789012123456789012345678901
The important thing about special triangles is that the GRE likes to use
2
12345678901234567890123456789012123456789012345678901
2
12345678901234567890123456789012123456789012345678901
1 them. When one pops up on the test, you need to be able to identify it and 2
2
12345678901234567890123456789012123456789012345678901
then use its special properties to solve the problem.
2
12345678901234567890123456789012123456789012345678901
2
12345678901234567890123456789012123456789012345678901
The
first
kind
of
special
triangle
to
discuss
is
usually
pretty
easy
to
spot.
A
2345678901234567890123456789012123456789012345678901
12345678901234567890123456789012123456789012345678901 22
12345678901234567890123456789012123456789012345678901
right triangle is a triangle that has one 90-degree angle in it. Visually, this 2
12345678901234567890123456789012123456789012345678901
2
12345678901234567890123456789012123456789012345678901
angle might be denoted by one of those box symbols in it. Recalling that all 2
12345678901234567890123456789012123456789012345678901
12345678901234567890123456789012123456789012345678901
three angles in a triangle always equal 180, you know that in a right 2
2
12345678901234567890123456789012123456789012345678901
2
12345678901234567890123456789012123456789012345678901
triangle,
the
other
two
angles
always
sum
to
90.
12345678901234567890123456789012123456789012345678901 22
1
12345678901234567890123456789012123456789012345678901
The Pythagorean theorem is a special equation that only applies to right 2
2
12345678901234567890123456789012123456789012345678901
2
2
2
2
12345678901234567890123456789012123456789012345678901
1
b
5
c
,
where
c
is
the
triangles.
Call
it
the
Big
P,
and
its
equation
is
a
12345678901234567890123456789012123456789012345678901 2
12345678901234567890123456789012123456789012345678901
length of the side opposite the right angle (called the hypotenuse by the 2
2
12345678901234567890123456789012123456789012345678901
12345678901234567890123456789012123456789012345678901
math establishment) and a and b are the lengths of the other two sides. 2
12345678901234567890123456789012123456789012345678901 2
12345678901234567890123456789012123456789012345678901
With this equation, if you know two sides of a right triangle, you can 2
2
12345678901234567890123456789012123456789012345678901
2345678901234567890123456789012123456789012345678901
12345678901234567890123456789012123456789012345678901
always find the third. That is a basic move the test will expect you to be 2
2
12345678901234567890123456789012123456789012345678901
1 able to make. A problem wont be that easy, but that will be one necessary 2
2
12345678901234567890123456789012123456789012345678901
step for solving a problem.
2
12345678901234567890123456789012123456789012345678901
2345678901234567890123456789012123456789012345678901
12345678901234567890123456789012123456789012345678901 22
12345678901234567890123456789012123456789012345678901
Column B
Column A
2
12345678901234567890123456789012123456789012345678901
12345678901234567890123456789012123456789012345678901 22
1
2
12345678901234567890123456789012123456789012345678901
2
12345678901234567890123456789012123456789012345678901
2
12345678901234567890123456789012123456789012345678901
2
12345678901234567890123456789012123456789012345678901
12345678901234567890123456789012123456789012345678901 2
2345678901234567890123456789012123456789012345678901
12345678901234567890123456789012123456789012345678901 22
12345678901234567890123456789012123456789012345678901 2
12345678901234567890123456789012123456789012345678901
Area of DABC
Area of DDEF
2
12345678901234567890123456789012123456789012345678901
2
1
2345678901234567890123456789012123456789012345678901
12345678901234567890123456789012123456789012345678901 22
1 To answer this question, you need to be able to determine the base and the
2
12345678901234567890123456789012123456789012345678901
height of each of the triangles. Can you do that? You can with the right 2
12345678901234567890123456789012123456789012345678901
2
12345678901234567890123456789012123456789012345678901
triangle, but you cant with DDEF. Choice (D) is the correct answer.
2
12345678901234567890123456789012123456789012345678901
12345678901234567890123456789012123456789012345678901 2
12345678901234567890123456789012123456789012345678901 2
12345678901234567890123456789012123456789012345678901 2 85
123456789012345678901234567890121234567890123456789012

http://www.xtremepapers.net

Part III: Section Review

86

123456789012345678901234567890121234567890123456789012
12345678901234567890123456789012123456789012345678901 2
2
12345678901234567890123456789012123456789012345678901
A second kind of special triangle is the isosceles triangle. It has a funny 2
12345678901234567890123456789012123456789012345678901
2
12345678901234567890123456789012123456789012345678901
name but a simple meaning: Two of the sides of the triangle are equal. A 2
12345678901234567890123456789012123456789012345678901
2
12345678901234567890123456789012123456789012345678901
problem will either tell you that a triangle is isosceles and expect you to 2
12345678901234567890123456789012123456789012345678901
2345678901234567890123456789012123456789012345678901
2
1 know what that means, or you might see a figure like this:
2
12345678901234567890123456789012123456789012345678901
2
12345678901234567890123456789012123456789012345678901
2
12345678901234567890123456789012123456789012345678901
2
12345678901234567890123456789012123456789012345678901
2345678901234567890123456789012123456789012345678901
2
1
2
12345678901234567890123456789012123456789012345678901
2
12345678901234567890123456789012123456789012345678901
2
12345678901234567890123456789012123456789012345678901
2
12345678901234567890123456789012123456789012345678901
2345678901234567890123456789012123456789012345678901
2
1
2
12345678901234567890123456789012123456789012345678901
2
12345678901234567890123456789012123456789012345678901
2
12345678901234567890123456789012123456789012345678901
The
lines
through
two
of
the
sides
of
the
triangle
mean
that
those
two
sides
2
12345678901234567890123456789012123456789012345678901
2345678901234567890123456789012123456789012345678901
2
1 are equal.
2
12345678901234567890123456789012123456789012345678901
2
12345678901234567890123456789012123456789012345678901
2
12345678901234567890123456789012123456789012345678901
Now
lets
think
about
triangles
for
a
second.
If
one
angle
of
a
triangle
is
2
12345678901234567890123456789012123456789012345678901
2345678901234567890123456789012123456789012345678901
12345678901234567890123456789012123456789012345678901
largesay 120 degreeswill the side opposite that angle be large or small 2
2
12345678901234567890123456789012123456789012345678901
2
12345678901234567890123456789012123456789012345678901
in
comparison
to
the
other
angles?
Draw
out
a
triangle
with
this
angle.
The
12345678901234567890123456789012123456789012345678901 2
12345678901234567890123456789012123456789012345678901
side opposite the big angle will be big in comparison to the other sides. The 2
2
12345678901234567890123456789012123456789012345678901
1 converse is also true. The side opposite the small angle will be small in 2
2
12345678901234567890123456789012123456789012345678901
2
12345678901234567890123456789012123456789012345678901
comparison to the other sides.
2
12345678901234567890123456789012123456789012345678901
2345678901234567890123456789012123456789012345678901
2
12345678901234567890123456789012123456789012345678901
With these illustrations, you can see that the size of an angle in a triangle is 2
12345678901234567890123456789012123456789012345678901
2
12345678901234567890123456789012123456789012345678901
related to the length of the side opposite that angle. Again, the converse is 2
12345678901234567890123456789012123456789012345678901
2
12345678901234567890123456789012123456789012345678901
true; the length of a side of a triangle is related to the size of the angle 2
12345678901234567890123456789012123456789012345678901
12345678901234567890123456789012123456789012345678901
opposite it. Now think about an isosceles triangle, where two sides are the 2
2
12345678901234567890123456789012123456789012345678901
2
12345678901234567890123456789012123456789012345678901
same
length.
Since
the
sides
are
equal,
the
angles
opposite
those
sides
are
2
1
2345678901234567890123456789012123456789012345678901
2
12345678901234567890123456789012123456789012345678901
also equal. In the isosceles triangle above, A 5 B.
2
12345678901234567890123456789012123456789012345678901
2
1
2
12345678901234567890123456789012123456789012345678901
A
wrinkle
on
the
isosceles
triangle
is
the
right
isosceles
triangle.
Its
2
12345678901234567890123456789012123456789012345678901
2345678901234567890123456789012123456789012345678901
12345678901234567890123456789012123456789012345678901
nothing more than a right triangle with two sides that are equal. You know 2
2
12345678901234567890123456789012123456789012345678901
1 one angle equals 90 degrees, and that the other two sum to 90 degrees. 2
2
12345678901234567890123456789012123456789012345678901
12345678901234567890123456789012123456789012345678901
Since you also know the other two angles are equal, it can be stated 2
2345678901234567890123456789012123456789012345678901
12345678901234567890123456789012123456789012345678901 2
2
12345678901234567890123456789012123456789012345678901
2
1 algebraically that
12345678901234567890123456789012123456789012345678901 2
2
12345678901234567890123456789012123456789012345678901
x 1 x 5 90
2
12345678901234567890123456789012123456789012345678901
2
12345678901234567890123456789012123456789012345678901
2x
5
90
Here,
x
is
the
angle
for
the
two
equal
sides.
2
12345678901234567890123456789012123456789012345678901
2
12345678901234567890123456789012123456789012345678901
x
5
45
12345678901234567890123456789012123456789012345678901 2
2345678901234567890123456789012123456789012345678901
12345678901234567890123456789012123456789012345678901 2
12345678901234567890123456789012123456789012345678901
The other two angles are 45 degrees. If the right triangle is a special 2
2
12345678901234567890123456789012123456789012345678901
2
12345678901234567890123456789012123456789012345678901
triangle,
then
the
45-45-90
triangle
is
a
super
special
triangle
because
it
2
1
2345678901234567890123456789012123456789012345678901
2
12345678901234567890123456789012123456789012345678901
is a special kind of right triangle.
2
12345678901234567890123456789012123456789012345678901
12345678901234567890123456789012123456789012345678901 2
12345678901234567890123456789012123456789012345678901 2
2
1
2
12345678901234567890123456789012123456789012345678901
12345678901234567890123456789012123456789012345678901 2
2
12345678901234567890123456789012123456789012345678901
12345678901234567890123456789012123456789012345678901 2
2
12345678901234567890123456789012123456789012345678901
12345678901234567890123456789012123456789012345678901 2
12345678901234567890123456789012123456789012345678901 2
12345678901234567890123456789012123456789012345678901 2
12345678901234567890123456789012123456789012345678901 2
123456789012345678901234567890121234567890123456789012

www.petersons.com

http://www.xtremepapers.net

Note

Chapter 4: Quantitative Review

123456789012345678901234567890121234567890123456789012
12345678901234567890123456789012123456789012345678901 2
2
12345678901234567890123456789012123456789012345678901
As you can see, the relationships between the sides and angles of a 2
12345678901234567890123456789012123456789012345678901
2
12345678901234567890123456789012123456789012345678901
45-45-90 triangle are all nailed down. The test-makers like this feature of 2
12345678901234567890123456789012123456789012345678901
2
12345678901234567890123456789012123456789012345678901
the triangle because they figure that if they give you enough information to 2
12345678901234567890123456789012123456789012345678901
2345678901234567890123456789012123456789012345678901
1 identify a triangle as a 45-45-90 triangle, you should be able to determine 2
2
12345678901234567890123456789012123456789012345678901
2
12345678901234567890123456789012123456789012345678901
most
everything
about
that
triangle.
2
12345678901234567890123456789012123456789012345678901
2
12345678901234567890123456789012123456789012345678901
2345678901234567890123456789012123456789012345678901
2
1
2
12345678901234567890123456789012123456789012345678901
2
12345678901234567890123456789012123456789012345678901
2
12345678901234567890123456789012123456789012345678901
If you want to know how the side ratios are determined, just use the
2
12345678901234567890123456789012123456789012345678901
2345678901234567890123456789012123456789012345678901
2
1 Pythagorean theorem on the 45-45-90.
2
12345678901234567890123456789012123456789012345678901
2
12345678901234567890123456789012123456789012345678901
2
12345678901234567890123456789012123456789012345678901
2
12345678901234567890123456789012123456789012345678901
2345678901234567890123456789012123456789012345678901
2
1
2
12345678901234567890123456789012123456789012345678901
2
12345678901234567890123456789012123456789012345678901
2
12345678901234567890123456789012123456789012345678901
2
12345678901234567890123456789012123456789012345678901
2345678901234567890123456789012123456789012345678901
12345678901234567890123456789012123456789012345678901 22
12345678901234567890123456789012123456789012345678901 2
12345678901234567890123456789012123456789012345678901 2
12345678901234567890123456789012123456789012345678901 2
12345678901234567890123456789012123456789012345678901 2
12345678901234567890123456789012123456789012345678901 2
1
If a 5 c, then x 5
2
12345678901234567890123456789012123456789012345678901
2
12345678901234567890123456789012123456789012345678901
A.
2
2
12345678901234567890123456789012123456789012345678901
2345678901234567890123456789012123456789012345678901
2
12345678901234567890123456789012123456789012345678901
B. =3
2
12345678901234567890123456789012123456789012345678901
12345678901234567890123456789012123456789012345678901 22
12345678901234567890123456789012123456789012345678901
C. =5
2
12345678901234567890123456789012123456789012345678901
2
12345678901234567890123456789012123456789012345678901
D. =6
2
12345678901234567890123456789012123456789012345678901
2
12345678901234567890123456789012123456789012345678901
E. 4
2
12345678901234567890123456789012123456789012345678901
2
1
2345678901234567890123456789012123456789012345678901
12345678901234567890123456789012123456789012345678901
This is a busy little diagram, but youll get some of those. If your test radar 2
2
12345678901234567890123456789012123456789012345678901
1 is working, you should suspect that there is a special triangle in here 2
12345678901234567890123456789012123456789012345678901 2
12345678901234567890123456789012123456789012345678901
somewhere. Notice that b must be 90 for the angles in the top triangle to 2
2
12345678901234567890123456789012123456789012345678901
2
12345678901234567890123456789012123456789012345678901
sum
to
180
degrees,
and
since
a
5
c,
they
both
must
be
45
because
a
1
b
1
12345678901234567890123456789012123456789012345678901 2
12345678901234567890123456789012123456789012345678901
c 5 180 (it is a straight line). This means that the triangle on the right is a 2
2
12345678901234567890123456789012123456789012345678901
2345678901234567890123456789012123456789012345678901
12345678901234567890123456789012123456789012345678901
45-45-90 triangle, and since we know the length of one side we can find 2
2
12345678901234567890123456789012123456789012345678901
2
1 the length of all sides, including x, which is:
12345678901234567890123456789012123456789012345678901 2
12345678901234567890123456789012123456789012345678901 2
2
12345678901234567890123456789012123456789012345678901
x 5 (length of one side of the 45-45-90 triangle)(=2)
2
12345678901234567890123456789012123456789012345678901
2
12345678901234567890123456789012123456789012345678901
2
12345678901234567890123456789012123456789012345678901
3
2
5
6
or
choice
(D)
x
5
=
=
=
12345678901234567890123456789012123456789012345678901 2
2345678901234567890123456789012123456789012345678901
12345678901234567890123456789012123456789012345678901 22
12345678901234567890123456789012123456789012345678901 2
12345678901234567890123456789012123456789012345678901 2
12345678901234567890123456789012123456789012345678901 2
1
2
12345678901234567890123456789012123456789012345678901
2
12345678901234567890123456789012123456789012345678901
2
12345678901234567890123456789012123456789012345678901
2
12345678901234567890123456789012123456789012345678901
12345678901234567890123456789012123456789012345678901 2
2345678901234567890123456789012123456789012345678901
12345678901234567890123456789012123456789012345678901 22
1
2
12345678901234567890123456789012123456789012345678901
12345678901234567890123456789012123456789012345678901 2
2
12345678901234567890123456789012123456789012345678901
12345678901234567890123456789012123456789012345678901 2
12345678901234567890123456789012123456789012345678901 2
12345678901234567890123456789012123456789012345678901 2
12345678901234567890123456789012123456789012345678901 2 87
123456789012345678901234567890121234567890123456789012

http://www.xtremepapers.net

Part III: Section Review

88

123456789012345678901234567890121234567890123456789012
12345678901234567890123456789012123456789012345678901 2
2
12345678901234567890123456789012123456789012345678901
The next kind of super special triangle is the 30-60-90 triangle. There is 2
12345678901234567890123456789012123456789012345678901
2
12345678901234567890123456789012123456789012345678901
not a special name for it, but they appear on the test with great frequency 2
12345678901234567890123456789012123456789012345678901
2
12345678901234567890123456789012123456789012345678901
for the same reason that the 45-45-90 triangle does. You are expected to 2
12345678901234567890123456789012123456789012345678901
2345678901234567890123456789012123456789012345678901
1 identify the 30-60-90 triangle and then use its properties to solve 2
2
12345678901234567890123456789012123456789012345678901
2
12345678901234567890123456789012123456789012345678901
problems.
2
12345678901234567890123456789012123456789012345678901
2
12345678901234567890123456789012123456789012345678901
2345678901234567890123456789012123456789012345678901
2
1
2
12345678901234567890123456789012123456789012345678901
2
12345678901234567890123456789012123456789012345678901
2
12345678901234567890123456789012123456789012345678901
2
12345678901234567890123456789012123456789012345678901
2345678901234567890123456789012123456789012345678901
2
1
2
12345678901234567890123456789012123456789012345678901
2
12345678901234567890123456789012123456789012345678901
2
12345678901234567890123456789012123456789012345678901
2
12345678901234567890123456789012123456789012345678901
2345678901234567890123456789012123456789012345678901
2
1
2
12345678901234567890123456789012123456789012345678901
12345678901234567890123456789012123456789012345678901
Call the smallest side length x. This will be the side opposite the 30-degree 2
2
12345678901234567890123456789012123456789012345678901
2
12345678901234567890123456789012123456789012345678901
angle.
The
side
opposite
the
60-degree
angle
is
x
3,
and
the
length
of
the
=
2345678901234567890123456789012123456789012345678901
12345678901234567890123456789012123456789012345678901 2
2
12345678901234567890123456789012123456789012345678901
hypotenuse is always 2x.
2
12345678901234567890123456789012123456789012345678901
12345678901234567890123456789012123456789012345678901 2
12345678901234567890123456789012123456789012345678901
The equilateral triangle, the triangle whose sides are equal, is related to the 2
2
12345678901234567890123456789012123456789012345678901
1 30-60-90 triangle. An equilateral triangle is like two 30-60-90 triangles 2
2
12345678901234567890123456789012123456789012345678901
2
12345678901234567890123456789012123456789012345678901
back-to-back.
2
12345678901234567890123456789012123456789012345678901
2345678901234567890123456789012123456789012345678901
12345678901234567890123456789012123456789012345678901 2
12345678901234567890123456789012123456789012345678901 2
12345678901234567890123456789012123456789012345678901 2
12345678901234567890123456789012123456789012345678901 2
12345678901234567890123456789012123456789012345678901 2
12345678901234567890123456789012123456789012345678901 2
12345678901234567890123456789012123456789012345678901 2
12345678901234567890123456789012123456789012345678901 2
12345678901234567890123456789012123456789012345678901 2
2
1
12345678901234567890123456789012123456789012345678901
Since the sides of an equilateral triangle are all equal, what must the 2
2
12345678901234567890123456789012123456789012345678901
2
12345678901234567890123456789012123456789012345678901
relationship
of
the
angles
be?
Equal.
This
means
that
3
sides
5
180.
12345678901234567890123456789012123456789012345678901 2
2
12345678901234567890123456789012123456789012345678901
2
12345678901234567890123456789012123456789012345678901
So,
2
12345678901234567890123456789012123456789012345678901
12345678901234567890123456789012123456789012345678901 2
2
12345678901234567890123456789012123456789012345678901
3s 5 180
2
12345678901234567890123456789012123456789012345678901
2345678901234567890123456789012123456789012345678901
2
12345678901234567890123456789012123456789012345678901
s 5 60
2
12345678901234567890123456789012123456789012345678901
2
1
2
12345678901234567890123456789012123456789012345678901
For
every
equilateral
triangle,
each
angle
is
60
degrees.
The
lengths
of
all
3
12345678901234567890123456789012123456789012345678901 2
2345678901234567890123456789012123456789012345678901
2
12345678901234567890123456789012123456789012345678901
sides in an equilateral triangle are of equal lengths.
2
12345678901234567890123456789012123456789012345678901
12345678901234567890123456789012123456789012345678901 2
2
12345678901234567890123456789012123456789012345678901
1 There are two other special right triangles that appear regularly on the 2
2345678901234567890123456789012123456789012345678901
2
12345678901234567890123456789012123456789012345678901
GRE. Both are defined by the relationships of the sides.
2
12345678901234567890123456789012123456789012345678901
12345678901234567890123456789012123456789012345678901 2
12345678901234567890123456789012123456789012345678901 2
2
1
2
12345678901234567890123456789012123456789012345678901
2
12345678901234567890123456789012123456789012345678901
2
12345678901234567890123456789012123456789012345678901
2
12345678901234567890123456789012123456789012345678901
12345678901234567890123456789012123456789012345678901 2
2345678901234567890123456789012123456789012345678901
12345678901234567890123456789012123456789012345678901 2
2
1
12345678901234567890123456789012123456789012345678901
If you recognize one of these triangles in a problem, and you know two 2
12345678901234567890123456789012123456789012345678901 2
2
12345678901234567890123456789012123456789012345678901
sides of the triangle, you can simply read off the length of the third side.
2
12345678901234567890123456789012123456789012345678901
12345678901234567890123456789012123456789012345678901 2
12345678901234567890123456789012123456789012345678901 2
12345678901234567890123456789012123456789012345678901 2
123456789012345678901234567890121234567890123456789012

www.petersons.com

http://www.xtremepapers.net

Chapter 4: Quantitative Review

123456789012345678901234567890121234567890123456789012
12345678901234567890123456789012123456789012345678901 2
2
12345678901234567890123456789012123456789012345678901
12345678901234567890123456789012123456789012345678901 2
2
12345678901234567890123456789012123456789012345678901
2
12345678901234567890123456789012123456789012345678901
12345678901234567890123456789012123456789012345678901 2
2
12345678901234567890123456789012123456789012345678901
12345678901234567890123456789012123456789012345678901 2
2
12345678901234567890123456789012123456789012345678901
2
12345678901234567890123456789012123456789012345678901
2
12345678901234567890123456789012123456789012345678901
2
12345678901234567890123456789012123456789012345678901
2345678901234567890123456789012123456789012345678901
2
1
2
12345678901234567890123456789012123456789012345678901
2
12345678901234567890123456789012123456789012345678901
2
12345678901234567890123456789012123456789012345678901
If y 5 3, then x 5
2
12345678901234567890123456789012123456789012345678901
2345678901234567890123456789012123456789012345678901
2
1
A. 21
2
12345678901234567890123456789012123456789012345678901
2
12345678901234567890123456789012123456789012345678901
B.
0
2
12345678901234567890123456789012123456789012345678901
2
12345678901234567890123456789012123456789012345678901
C.
1
2345678901234567890123456789012123456789012345678901
2
1
2
12345678901234567890123456789012123456789012345678901
D.
2
2
12345678901234567890123456789012123456789012345678901
2
12345678901234567890123456789012123456789012345678901
E.
3
2
12345678901234567890123456789012123456789012345678901
2345678901234567890123456789012123456789012345678901
12345678901234567890123456789012123456789012345678901 22
12345678901234567890123456789012123456789012345678901
When you see the 30-degree angle, you should immediately start looking 2
12345678901234567890123456789012123456789012345678901
12345678901234567890123456789012123456789012345678901
to see if you have 30-60-90 triangle. The top triangle is a right triangle 2
2
12345678901234567890123456789012123456789012345678901
2
12345678901234567890123456789012123456789012345678901
1 since the bottom right angle is vertical with the right angle. This means the 2
12345678901234567890123456789012123456789012345678901
top triangle is a 30-60-90 triangle. So 2(x 1 y) 5 6, which means that if y 2
2
12345678901234567890123456789012123456789012345678901
2
12345678901234567890123456789012123456789012345678901
5 3 then x 5 0.
2345678901234567890123456789012123456789012345678901
12345678901234567890123456789012123456789012345678901 22
12345678901234567890123456789012123456789012345678901 2
12345678901234567890123456789012123456789012345678901 2
12345678901234567890123456789012123456789012345678901 2
12345678901234567890123456789012123456789012345678901
Triangles: Similar and Congruent
2
12345678901234567890123456789012123456789012345678901
12345678901234567890123456789012123456789012345678901
When you say that a Cheeto looks similar to an orange caterpillar, you 2
2
12345678901234567890123456789012123456789012345678901
2
12345678901234567890123456789012123456789012345678901
probably
arent
using
an
exact
definition
of
similar.
When
mathematicians
2
1
2345678901234567890123456789012123456789012345678901
12345678901234567890123456789012123456789012345678901
use the term similar, they are being much more precise. To a mathemati- 2
2
12345678901234567890123456789012123456789012345678901
1 cian, two triangles are similar if all corresponding angles are equal and all 2
12345678901234567890123456789012123456789012345678901 2
12345678901234567890123456789012123456789012345678901
corresponding sides are proportional. Similar triangles look the same, but 2
2
12345678901234567890123456789012123456789012345678901
2
12345678901234567890123456789012123456789012345678901
one
can
be
bigger
than
the
other.
12345678901234567890123456789012123456789012345678901 2
2
12345678901234567890123456789012123456789012345678901
2
12345678901234567890123456789012123456789012345678901
2345678901234567890123456789012123456789012345678901
12345678901234567890123456789012123456789012345678901 22
12345678901234567890123456789012123456789012345678901 2
1
12345678901234567890123456789012123456789012345678901 2
12345678901234567890123456789012123456789012345678901 2
2
12345678901234567890123456789012123456789012345678901
2
12345678901234567890123456789012123456789012345678901
2
12345678901234567890123456789012123456789012345678901
2
12345678901234567890123456789012123456789012345678901
12345678901234567890123456789012123456789012345678901
Triangle M has sides that are three times as large as triangle N, but all 2
2345678901234567890123456789012123456789012345678901
12345678901234567890123456789012123456789012345678901 22
12345678901234567890123456789012123456789012345678901
angles are equal. It doesnt matter that triangle N is flipped around in
2
12345678901234567890123456789012123456789012345678901
12345678901234567890123456789012123456789012345678901
relation to M. Since the angles are equal and the sides proportional, the 2
2
1 two triangles are congruent.
2
12345678901234567890123456789012123456789012345678901
2
12345678901234567890123456789012123456789012345678901
2
12345678901234567890123456789012123456789012345678901
If
you
were
to
increase
the
size
of
triangle
Ns
sides
by
a
factor
of
3,
then
2
12345678901234567890123456789012123456789012345678901
2
12345678901234567890123456789012123456789012345678901
you
would
have
two
triangles
with
equal
sides
and
angles.
This
is
the
2345678901234567890123456789012123456789012345678901
12345678901234567890123456789012123456789012345678901 22
1 definition of congruent triangles.
2
12345678901234567890123456789012123456789012345678901
12345678901234567890123456789012123456789012345678901 2
2
12345678901234567890123456789012123456789012345678901
12345678901234567890123456789012123456789012345678901 2
12345678901234567890123456789012123456789012345678901 2
12345678901234567890123456789012123456789012345678901 2
12345678901234567890123456789012123456789012345678901 2 89
123456789012345678901234567890121234567890123456789012

http://www.xtremepapers.net

Part III: Section Review

www.petersons.com

http://www.xtremepapers.net

Note

90

123456789012345678901234567890121234567890123456789012
12345678901234567890123456789012123456789012345678901 2
2
12345678901234567890123456789012123456789012345678901
2
12345678901234567890123456789012123456789012345678901
Other Polygons
2
12345678901234567890123456789012123456789012345678901
2
12345678901234567890123456789012123456789012345678901
Polygon
is
the
fancy
geometric
word
for
shapes
that
have
three
or
more
12345678901234567890123456789012123456789012345678901 2
12345678901234567890123456789012123456789012345678901
sides. Lots of shapes are polygons like triangles, squares, pentagons, and 2
2
12345678901234567890123456789012123456789012345678901
2
12345678901234567890123456789012123456789012345678901
octagons, to name just a few.
2
12345678901234567890123456789012123456789012345678901
2
12345678901234567890123456789012123456789012345678901
The one general rule to remember about polygons comes in the form of a 2
12345678901234567890123456789012123456789012345678901
2345678901234567890123456789012123456789012345678901
1 formula. The interior angles of an n-sided polygon sum to (n 2 2)(180). 2
2
12345678901234567890123456789012123456789012345678901
2
12345678901234567890123456789012123456789012345678901
You can try this formula out on a triangle and get (3 2 2)(180) 5 180. This 2
12345678901234567890123456789012123456789012345678901
2
12345678901234567890123456789012123456789012345678901
restates the old all three angles of a triangle sum to 180 rule that should 2
12345678901234567890123456789012123456789012345678901
2
12345678901234567890123456789012123456789012345678901
be hammered about halfway into your skull by now. You dont need the 2
12345678901234567890123456789012123456789012345678901
12345678901234567890123456789012123456789012345678901
(n 2 2)(180) formula to find out the sum of the interior angles of a 2
2
12345678901234567890123456789012123456789012345678901
2345678901234567890123456789012123456789012345678901
1 triangle, but you might need it if you need to find the sum of the interior 2
2
12345678901234567890123456789012123456789012345678901
2
12345678901234567890123456789012123456789012345678901
angles
of
a
hexagon.
2
12345678901234567890123456789012123456789012345678901
2
12345678901234567890123456789012123456789012345678901
2345678901234567890123456789012123456789012345678901
12345678901234567890123456789012123456789012345678901
The rectangle is a polygon. It has four sides, so the sum of its interior 2
2
12345678901234567890123456789012123456789012345678901
2
12345678901234567890123456789012123456789012345678901
angles
must
be
(4
2
2)(180)
5
360.
There
are
two
important
properties
of
12345678901234567890123456789012123456789012345678901 2
2
12345678901234567890123456789012123456789012345678901
rectangle:
2
12345678901234567890123456789012123456789012345678901
2
1
2
1. All interior angles are 90 degrees.
12345678901234567890123456789012123456789012345678901
2
12345678901234567890123456789012123456789012345678901
2
12345678901234567890123456789012123456789012345678901
2.
Opposite
sides
are
parallel
and
equal
in
length.
2345678901234567890123456789012123456789012345678901
12345678901234567890123456789012123456789012345678901 2
12345678901234567890123456789012123456789012345678901 2
12345678901234567890123456789012123456789012345678901 2
12345678901234567890123456789012123456789012345678901 2
12345678901234567890123456789012123456789012345678901 2
12345678901234567890123456789012123456789012345678901 2
12345678901234567890123456789012123456789012345678901 2
12345678901234567890123456789012123456789012345678901 2
12345678901234567890123456789012123456789012345678901 2
2
1
2
12345678901234567890123456789012123456789012345678901
2
12345678901234567890123456789012123456789012345678901
12345678901234567890123456789012123456789012345678901 2
12345678901234567890123456789012123456789012345678901 2
2
12345678901234567890123456789012123456789012345678901
2
12345678901234567890123456789012123456789012345678901
2
12345678901234567890123456789012123456789012345678901
12345678901234567890123456789012123456789012345678901 2
12345678901234567890123456789012123456789012345678901
Note that only opposite sides of a rectangle need to be equal in length. If all 2
2
12345678901234567890123456789012123456789012345678901
2345678901234567890123456789012123456789012345678901
2
sides of a rectangle are equal, you have the figure known as a square.
12345678901234567890123456789012123456789012345678901
12345678901234567890123456789012123456789012345678901 2
2
1
12345678901234567890123456789012123456789012345678901 2
12345678901234567890123456789012123456789012345678901 2
12345678901234567890123456789012123456789012345678901
The perimeter of a rectangle can be found by the formula P 5 2(l 1 w) 2
2
12345678901234567890123456789012123456789012345678901
12345678901234567890123456789012123456789012345678901
and its area is A 5 l 3 w. For a square, the equations are pretty basic: 2
2
12345678901234567890123456789012123456789012345678901
2
12345678901234567890123456789012123456789012345678901
P 5 4s and A 5 s2.
2345678901234567890123456789012123456789012345678901
12345678901234567890123456789012123456789012345678901 2
12345678901234567890123456789012123456789012345678901 2
12345678901234567890123456789012123456789012345678901 2
12345678901234567890123456789012123456789012345678901 2
2
1
2
12345678901234567890123456789012123456789012345678901
2
12345678901234567890123456789012123456789012345678901
2
12345678901234567890123456789012123456789012345678901
2
12345678901234567890123456789012123456789012345678901
12345678901234567890123456789012123456789012345678901 2
2345678901234567890123456789012123456789012345678901
12345678901234567890123456789012123456789012345678901 2
2
1
2
12345678901234567890123456789012123456789012345678901
12345678901234567890123456789012123456789012345678901 2
2
12345678901234567890123456789012123456789012345678901
12345678901234567890123456789012123456789012345678901 2
12345678901234567890123456789012123456789012345678901 2
12345678901234567890123456789012123456789012345678901 2
12345678901234567890123456789012123456789012345678901 2
123456789012345678901234567890121234567890123456789012

Chapter 4: Quantitative Review

123456789012345678901234567890121234567890123456789012
12345678901234567890123456789012123456789012345678901 2
2
12345678901234567890123456789012123456789012345678901
2
12345678901234567890123456789012123456789012345678901
Column A
Column B
2
12345678901234567890123456789012123456789012345678901
2
12345678901234567890123456789012123456789012345678901
12345678901234567890123456789012123456789012345678901 2
2
12345678901234567890123456789012123456789012345678901
12345678901234567890123456789012123456789012345678901 2
2
12345678901234567890123456789012123456789012345678901
2
12345678901234567890123456789012123456789012345678901
2
12345678901234567890123456789012123456789012345678901
2
12345678901234567890123456789012123456789012345678901
2345678901234567890123456789012123456789012345678901
2
1
2
12345678901234567890123456789012123456789012345678901
2
12345678901234567890123456789012123456789012345678901
In
the
figure
above,
ACDF
is
a
rectangle,
2
12345678901234567890123456789012123456789012345678901
2
12345678901234567890123456789012123456789012345678901
and
ABEF
and
BCDE
are
squares.
2345678901234567890123456789012123456789012345678901
2
1
2
12345678901234567890123456789012123456789012345678901
2
12345678901234567890123456789012123456789012345678901
32
Area
of
ACDF
2
12345678901234567890123456789012123456789012345678901
2
12345678901234567890123456789012123456789012345678901
2345678901234567890123456789012123456789012345678901
1 You have to find the area of ACDF to compare the two columns, which 2
2
12345678901234567890123456789012123456789012345678901
2
12345678901234567890123456789012123456789012345678901
means you need to find the length and the width of the rectangle. The 2
12345678901234567890123456789012123456789012345678901
2
12345678901234567890123456789012123456789012345678901
diagonal in the square bisects two right angles, creating your new friend the 2
12345678901234567890123456789012123456789012345678901
2
12345678901234567890123456789012123456789012345678901
12345678901234567890123456789012123456789012345678901
45-45-90 triangle. This makes =32 the hypotenuse. From this, you can 2
2
12345678901234567890123456789012123456789012345678901
2
12345678901234567890123456789012123456789012345678901
32
=
2345678901234567890123456789012123456789012345678901
12345678901234567890123456789012123456789012345678901
5 =16 5 4. Line segment EF 2
2
1 read off the side length of the square,
2
12345678901234567890123456789012123456789012345678901
2
=
2
12345678901234567890123456789012123456789012345678901
5
4,
as
does
ED,
AF,
AB,
BC,
and
BE.
This
makes
the
length
of
the
rect2
12345678901234567890123456789012123456789012345678901
2345678901234567890123456789012123456789012345678901
12345678901234567890123456789012123456789012345678901
angle 8 (4 1 4), while the width the rectangle is 4. Placing these values into 2
2
12345678901234567890123456789012123456789012345678901
2
12345678901234567890123456789012123456789012345678901
the
formula
for
the
area
of
a
rectangle
gives
you
4
3
8
5
32.
Choice
(C)
is
12345678901234567890123456789012123456789012345678901 22
12345678901234567890123456789012123456789012345678901
the correct answer.
2
12345678901234567890123456789012123456789012345678901
12345678901234567890123456789012123456789012345678901 22
12345678901234567890123456789012123456789012345678901
The parallelogram and the trapezoid are two other 4-sided polygons to
2
12345678901234567890123456789012123456789012345678901
1 know. A parallelogram, like a rectangle, has opposite sides that are parallel 2
2345678901234567890123456789012123456789012345678901
12345678901234567890123456789012123456789012345678901 22
12345678901234567890123456789012123456789012345678901
equal. Unlike a rectangle, the interior angles of parallelograms are not
2
1 and
right angles. The opposite angles of a parallelogram are equal and they 2
12345678901234567890123456789012123456789012345678901
2
12345678901234567890123456789012123456789012345678901
sum to 360, which makes sense if you remember that n-sided polygon 2
12345678901234567890123456789012123456789012345678901
2
12345678901234567890123456789012123456789012345678901
formula.
2
12345678901234567890123456789012123456789012345678901
2
12345678901234567890123456789012123456789012345678901
2
12345678901234567890123456789012123456789012345678901
2345678901234567890123456789012123456789012345678901
12345678901234567890123456789012123456789012345678901 22
12345678901234567890123456789012123456789012345678901 2
1
12345678901234567890123456789012123456789012345678901 2
12345678901234567890123456789012123456789012345678901 2
2
12345678901234567890123456789012123456789012345678901
2
12345678901234567890123456789012123456789012345678901
2
12345678901234567890123456789012123456789012345678901
2
12345678901234567890123456789012123456789012345678901
12345678901234567890123456789012123456789012345678901 2
2345678901234567890123456789012123456789012345678901
12345678901234567890123456789012123456789012345678901 22
12345678901234567890123456789012123456789012345678901
The area of a parallelogram is the base times the height. These values are
2
12345678901234567890123456789012123456789012345678901
12345678901234567890123456789012123456789012345678901
determined similarly to the way you determine the base and height of a 2
2
1
12345678901234567890123456789012123456789012345678901
triangle. Pick the base, then draw a line perpendicular to it that intersects 2
2
12345678901234567890123456789012123456789012345678901
with the side across from it. That line is the height. The one difference is 2
12345678901234567890123456789012123456789012345678901
2
12345678901234567890123456789012123456789012345678901
that the height of a parallelogram doesnt have to intersect with an angle. 2
12345678901234567890123456789012123456789012345678901
2345678901234567890123456789012123456789012345678901
12345678901234567890123456789012123456789012345678901 22
1 As you can see, trapezoids do not have equal opposite sides or angles. In
2
12345678901234567890123456789012123456789012345678901
2
12345678901234567890123456789012123456789012345678901
fact,
to
be
a
trapezoid,
all
you
need
is
4
sides,
where
2
of
those
sides
are
2
12345678901234567890123456789012123456789012345678901
2345678901234567890123456789012123456789012345678901
2
1 parallel. In the trapezoid shown, the top and bottom sides are parallel.
12345678901234567890123456789012123456789012345678901 2
12345678901234567890123456789012123456789012345678901 2
12345678901234567890123456789012123456789012345678901 2 91
123456789012345678901234567890121234567890123456789012

http://www.xtremepapers.net

Part III: Section Review

92

123456789012345678901234567890121234567890123456789012
12345678901234567890123456789012123456789012345678901 2
2
12345678901234567890123456789012123456789012345678901
12345678901234567890123456789012123456789012345678901 2
2
12345678901234567890123456789012123456789012345678901
2
12345678901234567890123456789012123456789012345678901
12345678901234567890123456789012123456789012345678901 2
2
12345678901234567890123456789012123456789012345678901
12345678901234567890123456789012123456789012345678901 2
2
12345678901234567890123456789012123456789012345678901
2
12345678901234567890123456789012123456789012345678901
What
is
the
area
of
the
above
figure?
2
12345678901234567890123456789012123456789012345678901
2
12345678901234567890123456789012123456789012345678901
2345678901234567890123456789012123456789012345678901
2
1
A. 20
2
12345678901234567890123456789012123456789012345678901
2
12345678901234567890123456789012123456789012345678901
B.
28
2
12345678901234567890123456789012123456789012345678901
2
12345678901234567890123456789012123456789012345678901
C.
32
2345678901234567890123456789012123456789012345678901
2
1
2
12345678901234567890123456789012123456789012345678901
D. 34
2
12345678901234567890123456789012123456789012345678901
2
12345678901234567890123456789012123456789012345678901
E. 40
2
12345678901234567890123456789012123456789012345678901
2345678901234567890123456789012123456789012345678901
1 If you have no idea how to do this problem, you can at least eliminate 2
2
12345678901234567890123456789012123456789012345678901
2
12345678901234567890123456789012123456789012345678901
choices (A) and (B), since you know that this trapezoid will be bigger than 2
12345678901234567890123456789012123456789012345678901
2
12345678901234567890123456789012123456789012345678901
a rectangle with length 7 and width 4. Thats because you can see how this 2
12345678901234567890123456789012123456789012345678901
12345678901234567890123456789012123456789012345678901
trapezoid is like a rectangle with length 7 and width 4 with a triangle 2
2
12345678901234567890123456789012123456789012345678901
2
12345678901234567890123456789012123456789012345678901
attached
to
it.
Viewing
it
this
way
is
the
key
to
the
problem.
To
find
the
2
12345678901234567890123456789012123456789012345678901
2345678901234567890123456789012123456789012345678901
2
12345678901234567890123456789012123456789012345678901
1 area of a trapezoid, first draw a dotted line to turn the trapezoid into a 2
2
12345678901234567890123456789012123456789012345678901
rectangle and a triangle.
2
12345678901234567890123456789012123456789012345678901
2
12345678901234567890123456789012123456789012345678901
2345678901234567890123456789012123456789012345678901
12345678901234567890123456789012123456789012345678901 2
12345678901234567890123456789012123456789012345678901 2
12345678901234567890123456789012123456789012345678901 2
12345678901234567890123456789012123456789012345678901 2
12345678901234567890123456789012123456789012345678901 2
12345678901234567890123456789012123456789012345678901 2
12345678901234567890123456789012123456789012345678901 2
12345678901234567890123456789012123456789012345678901 2
12345678901234567890123456789012123456789012345678901 2
1 You can read off the third side of the triangle if you recognize that it is a 2
2
12345678901234567890123456789012123456789012345678901
12345678901234567890123456789012123456789012345678901
3-4-5 triangle. The area of the rectangle is 28, and the area of the triangle 2
2
12345678901234567890123456789012123456789012345678901
2
12345678901234567890123456789012123456789012345678901
1
2
12345678901234567890123456789012123456789012345678901
3
4
5
6.
So
the
area
of
the
trapezoid
is
34,
choice
(D).
You
found
is
~
!~
!
2345678901234567890123456789012123456789012345678901
2
12345678901234567890123456789012123456789012345678901
2
2
12345678901234567890123456789012123456789012345678901
1 the answer, and you also saw a way in which special triangles can be 2
2
12345678901234567890123456789012123456789012345678901
2
12345678901234567890123456789012123456789012345678901
cropped and provide the key to a problem.
2345678901234567890123456789012123456789012345678901
12345678901234567890123456789012123456789012345678901 2
12345678901234567890123456789012123456789012345678901 2
2
1
12345678901234567890123456789012123456789012345678901 2
12345678901234567890123456789012123456789012345678901 2
2
12345678901234567890123456789012123456789012345678901
2
12345678901234567890123456789012123456789012345678901
2
12345678901234567890123456789012123456789012345678901
2
12345678901234567890123456789012123456789012345678901
12345678901234567890123456789012123456789012345678901 2
2345678901234567890123456789012123456789012345678901
12345678901234567890123456789012123456789012345678901 2
12345678901234567890123456789012123456789012345678901 2
12345678901234567890123456789012123456789012345678901 2
12345678901234567890123456789012123456789012345678901 2
2
1
2
12345678901234567890123456789012123456789012345678901
2
12345678901234567890123456789012123456789012345678901
2
12345678901234567890123456789012123456789012345678901
2
12345678901234567890123456789012123456789012345678901
12345678901234567890123456789012123456789012345678901 2
2345678901234567890123456789012123456789012345678901
12345678901234567890123456789012123456789012345678901 2
2
1
2
12345678901234567890123456789012123456789012345678901
12345678901234567890123456789012123456789012345678901 2
2
12345678901234567890123456789012123456789012345678901
12345678901234567890123456789012123456789012345678901 2
12345678901234567890123456789012123456789012345678901 2
12345678901234567890123456789012123456789012345678901 2
12345678901234567890123456789012123456789012345678901 2
123456789012345678901234567890121234567890123456789012

www.petersons.com

http://www.xtremepapers.net

Chapter 4: Quantitative Review

123456789012345678901234567890121234567890123456789012
12345678901234567890123456789012123456789012345678901 2
2
12345678901234567890123456789012123456789012345678901
2
12345678901234567890123456789012123456789012345678901
Circles
2
12345678901234567890123456789012123456789012345678901
2
12345678901234567890123456789012123456789012345678901
For
many,
circles
evoke
ideas
like
elegance
and
eternity.
Sadly,
elegance
is
12345678901234567890123456789012123456789012345678901 2
12345678901234567890123456789012123456789012345678901
not one of standards commonly tested on the GRE. Instead, the more 2
2
12345678901234567890123456789012123456789012345678901
12345678901234567890123456789012123456789012345678901
mundane properties of the circle are focused on, so any musings about 2
2
12345678901234567890123456789012123456789012345678901
2
12345678901234567890123456789012123456789012345678901
eternity must take a back seat to the definition of circumference.
2
12345678901234567890123456789012123456789012345678901
2345678901234567890123456789012123456789012345678901
2
1
2
12345678901234567890123456789012123456789012345678901
2
12345678901234567890123456789012123456789012345678901
2
12345678901234567890123456789012123456789012345678901
2
12345678901234567890123456789012123456789012345678901
2345678901234567890123456789012123456789012345678901
2
1
2
12345678901234567890123456789012123456789012345678901
2
12345678901234567890123456789012123456789012345678901
2
12345678901234567890123456789012123456789012345678901
2
12345678901234567890123456789012123456789012345678901
2345678901234567890123456789012123456789012345678901
2
1
2
12345678901234567890123456789012123456789012345678901
2
12345678901234567890123456789012123456789012345678901
2
12345678901234567890123456789012123456789012345678901
2
12345678901234567890123456789012123456789012345678901
2345678901234567890123456789012123456789012345678901
12345678901234567890123456789012123456789012345678901 22
12345678901234567890123456789012123456789012345678901 2
12345678901234567890123456789012123456789012345678901 2
12345678901234567890123456789012123456789012345678901 2
12345678901234567890123456789012123456789012345678901
O is called the center of the circle, and all the points along the edge of the 2
12345678901234567890123456789012123456789012345678901
1 circle are the same distance from O. That distance, r, is called the radius. 2
2
12345678901234567890123456789012123456789012345678901
2
12345678901234567890123456789012123456789012345678901
The
radius
is
any
line
that
goes
from
the
center
to
the
edge
of
the
circle.
If
2
12345678901234567890123456789012123456789012345678901
2345678901234567890123456789012123456789012345678901
12345678901234567890123456789012123456789012345678901
a line starts at the edge of the circle, runs through O, and continues on to 2
2
12345678901234567890123456789012123456789012345678901
2
12345678901234567890123456789012123456789012345678901
the
opposite
edge,
it
is
called
the
diameter.
By
definition,
every
diameter
is
12345678901234567890123456789012123456789012345678901 22
12345678901234567890123456789012123456789012345678901
made up of two radii, so diameter 5 2(radius). Any line whose endpoints
2
12345678901234567890123456789012123456789012345678901
12345678901234567890123456789012123456789012345678901
are on the edge of the circle is called a chord, like MN. The diameter is one 2
12345678901234567890123456789012123456789012345678901 22
12345678901234567890123456789012123456789012345678901
2
1 particular chord.
2
12345678901234567890123456789012123456789012345678901
If you were to walk along the edge of a circle once, the distance you would 2
12345678901234567890123456789012123456789012345678901
2
12345678901234567890123456789012123456789012345678901
have traveled is called the circumference. The circumference is like the 2
12345678901234567890123456789012123456789012345678901
2
12345678901234567890123456789012123456789012345678901
perimeter of a polygon except that you have to be a little more 2
12345678901234567890123456789012123456789012345678901
12345678901234567890123456789012123456789012345678901
sophisticated in calculating it because the circumference isnt composed of 2
2
12345678901234567890123456789012123456789012345678901
2
12345678901234567890123456789012123456789012345678901
straight
lines.
This
is
the
beauty
of
the
concept
of
pi,
often
shown
by
the
2
12345678901234567890123456789012123456789012345678901
2345678901234567890123456789012123456789012345678901
12345678901234567890123456789012123456789012345678901
symbol p. Mathematicians way back when came up with this concept, and 2
2
12345678901234567890123456789012123456789012345678901
2
1 boy, does it help.
12345678901234567890123456789012123456789012345678901 2
12345678901234567890123456789012123456789012345678901 2
12345678901234567890123456789012123456789012345678901
To dispel the mystery around it, p is simply defined as the circumference of 2
2
12345678901234567890123456789012123456789012345678901
2
12345678901234567890123456789012123456789012345678901
a
circle
divided
by
its
diameter:
2
12345678901234567890123456789012123456789012345678901
12345678901234567890123456789012123456789012345678901 2
2345678901234567890123456789012123456789012345678901
2
12345678901234567890123456789012123456789012345678901
circumference
2
12345678901234567890123456789012123456789012345678901
p
5
2
12345678901234567890123456789012123456789012345678901
diameter
12345678901234567890123456789012123456789012345678901 22
1
12345678901234567890123456789012123456789012345678901
The neat thing about p is that its value remains constant regardless of the 2
2
12345678901234567890123456789012123456789012345678901
12345678901234567890123456789012123456789012345678901
size of the circle. The ancient mathematicians figured this value to be 2
2
12345678901234567890123456789012123456789012345678901
12345678901234567890123456789012123456789012345678901
3.14159 . . . and a bunch of other numbers, since pi goes on for a long, 2
2345678901234567890123456789012123456789012345678901
12345678901234567890123456789012123456789012345678901 22
1 long time. For our purposes, just remember that it is a little bigger than 3,
2
12345678901234567890123456789012123456789012345678901
2
about 3.14.
12345678901234567890123456789012123456789012345678901
2
12345678901234567890123456789012123456789012345678901
2345678901234567890123456789012123456789012345678901
2
1
12345678901234567890123456789012123456789012345678901 2
12345678901234567890123456789012123456789012345678901 2
12345678901234567890123456789012123456789012345678901 2 93
123456789012345678901234567890121234567890123456789012

http://www.xtremepapers.net

Part III: Section Review

S D

94

www.petersons.com

http://www.xtremepapers.net

Note

123456789012345678901234567890121234567890123456789012
12345678901234567890123456789012123456789012345678901 2
2
12345678901234567890123456789012123456789012345678901
From the definition of p, you can determine the formula for circumfer- 2
12345678901234567890123456789012123456789012345678901
2
12345678901234567890123456789012123456789012345678901
ence:
2
12345678901234567890123456789012123456789012345678901
12345678901234567890123456789012123456789012345678901 2
2
12345678901234567890123456789012123456789012345678901
2
12345678901234567890123456789012123456789012345678901
circumference
2
12345678901234567890123456789012123456789012345678901
p5

2
12345678901234567890123456789012123456789012345678901
diameter
2
12345678901234567890123456789012123456789012345678901
2
12345678901234567890123456789012123456789012345678901
c
2345678901234567890123456789012123456789012345678901
2
1
p5
2
12345678901234567890123456789012123456789012345678901
d
2
12345678901234567890123456789012123456789012345678901
2
12345678901234567890123456789012123456789012345678901
c
2
12345678901234567890123456789012123456789012345678901
p
3
d
5
3
d

2345678901234567890123456789012123456789012345678901
2
1
d
2
12345678901234567890123456789012123456789012345678901
2
12345678901234567890123456789012123456789012345678901
2
12345678901234567890123456789012123456789012345678901
pd
5
c
2
12345678901234567890123456789012123456789012345678901
2345678901234567890123456789012123456789012345678901
2
1
p 3 diameter 5 circumference
2
12345678901234567890123456789012123456789012345678901
2
12345678901234567890123456789012123456789012345678901
2
12345678901234567890123456789012123456789012345678901
or
c
5
pd
for
short.
Remember
that
since
the
diameter
is
twice
the
radius,
2
12345678901234567890123456789012123456789012345678901
2345678901234567890123456789012123456789012345678901
2
12345678901234567890123456789012123456789012345678901
you can also state:
2
12345678901234567890123456789012123456789012345678901
12345678901234567890123456789012123456789012345678901 2
2
12345678901234567890123456789012123456789012345678901
c 5 pd 5 2pr
2
12345678901234567890123456789012123456789012345678901
2
12345678901234567890123456789012123456789012345678901
1 Lets go back to that mythical walk around the circle. If you only walked 2
12345678901234567890123456789012123456789012345678901
partway around the circle, the distance that you walked would be called an 2
2
12345678901234567890123456789012123456789012345678901
2
12345678901234567890123456789012123456789012345678901
arc
of
the
circle.
Imagine
you
walked
from
A
through
B
to
C
in
the
figure
2345678901234567890123456789012123456789012345678901
12345678901234567890123456789012123456789012345678901 2
2
12345678901234567890123456789012123456789012345678901
below:
2
12345678901234567890123456789012123456789012345678901
12345678901234567890123456789012123456789012345678901 2
12345678901234567890123456789012123456789012345678901 2
12345678901234567890123456789012123456789012345678901 2
12345678901234567890123456789012123456789012345678901 2
12345678901234567890123456789012123456789012345678901 2
12345678901234567890123456789012123456789012345678901 2
2
1
2
12345678901234567890123456789012123456789012345678901
2
12345678901234567890123456789012123456789012345678901
12345678901234567890123456789012123456789012345678901 2
12345678901234567890123456789012123456789012345678901 2
2
12345678901234567890123456789012123456789012345678901
2
12345678901234567890123456789012123456789012345678901
2
12345678901234567890123456789012123456789012345678901
12345678901234567890123456789012123456789012345678901 2
2
12345678901234567890123456789012123456789012345678901
2
12345678901234567890123456789012123456789012345678901
2345678901234567890123456789012123456789012345678901
12345678901234567890123456789012123456789012345678901 2
12345678901234567890123456789012123456789012345678901 2
2
1
12345678901234567890123456789012123456789012345678901 2
12345678901234567890123456789012123456789012345678901 2
2
12345678901234567890123456789012123456789012345678901
12345678901234567890123456789012123456789012345678901
This arc would be referred to as arc ABC, since it starts at A, goes through 2
2
12345678901234567890123456789012123456789012345678901
2
12345678901234567890123456789012123456789012345678901
B, and ends at C.
12345678901234567890123456789012123456789012345678901 2
2345678901234567890123456789012123456789012345678901
12345678901234567890123456789012123456789012345678901 2
12345678901234567890123456789012123456789012345678901 2
12345678901234567890123456789012123456789012345678901 2
12345678901234567890123456789012123456789012345678901 2
1 The distance you walked is part of the circumference, but what part is it? 2
2
12345678901234567890123456789012123456789012345678901
12345678901234567890123456789012123456789012345678901
To answer this, you must recall that there are 360 degrees in a circle. (If 2
2
12345678901234567890123456789012123456789012345678901
12345678901234567890123456789012123456789012345678901
you think this is a foolish number, blame the Babylonians.) The angle of 2
12345678901234567890123456789012123456789012345678901 2
12345678901234567890123456789012123456789012345678901
the arc is 60 degrees, and since there are 360 degrees in a circle, the arc 2
2
12345678901234567890123456789012123456789012345678901
60
1
2
12345678901234567890123456789012123456789012345678901
2
12345678901234567890123456789012123456789012345678901
5 .
angle is one sixth of the degrees in the circle
2
12345678901234567890123456789012123456789012345678901
360
6
2345678901234567890123456789012123456789012345678901
2
1
12345678901234567890123456789012123456789012345678901 2
12345678901234567890123456789012123456789012345678901 2
12345678901234567890123456789012123456789012345678901 2
123456789012345678901234567890121234567890123456789012

Chapter 4: Quantitative Review

123456789012345678901234567890121234567890123456789012
12345678901234567890123456789012123456789012345678901 2
2
12345678901234567890123456789012123456789012345678901
The arc length will have the same relationship with the circumference as 2
12345678901234567890123456789012123456789012345678901
2
12345678901234567890123456789012123456789012345678901
the arc angle does with the total number of degrees in a circle. Therefore, 2
12345678901234567890123456789012123456789012345678901
2
12345678901234567890123456789012123456789012345678901
1
2
12345678901234567890123456789012123456789012345678901
arcABC
5
c.
All
this
can
be
encapsulated
in
a
formula,
the
arc
length
5
2345678901234567890123456789012123456789012345678901
2
1
6
2
12345678901234567890123456789012123456789012345678901
2
12345678901234567890123456789012123456789012345678901
n
2
12345678901234567890123456789012123456789012345678901
3
2pr,
where
n
is
the
measure
of
the
arc
angle.
2
12345678901234567890123456789012123456789012345678901
360
2345678901234567890123456789012123456789012345678901
2
1
2
12345678901234567890123456789012123456789012345678901
2
The area of a circle is pr . This means that if you know the radius (or the 2
12345678901234567890123456789012123456789012345678901
2
12345678901234567890123456789012123456789012345678901
diameter) of a circle, you can find the area and the circumference of that 2
12345678901234567890123456789012123456789012345678901
2345678901234567890123456789012123456789012345678901
1 circle. A common test question is to give enough information to find the 2
2
12345678901234567890123456789012123456789012345678901
2
12345678901234567890123456789012123456789012345678901
radius
of
a
circle
and
ask
for
the
area
or
circumference
of
a
circle.
2
12345678901234567890123456789012123456789012345678901
2
12345678901234567890123456789012123456789012345678901
2345678901234567890123456789012123456789012345678901
2
1
Column A
Column B
2
12345678901234567890123456789012123456789012345678901
2
12345678901234567890123456789012123456789012345678901
2
12345678901234567890123456789012123456789012345678901
The
area
of
a
circle
is
py.
2
12345678901234567890123456789012123456789012345678901
2345678901234567890123456789012123456789012345678901
2
12345678901234567890123456789012123456789012345678901
3
2
12345678901234567890123456789012123456789012345678901
The diameter of the
2
12345678901234567890123456789012123456789012345678901
y2
2
12345678901234567890123456789012123456789012345678901
circle
2
12345678901234567890123456789012123456789012345678901
y
12345678901234567890123456789012123456789012345678901 22
1
2
12345678901234567890123456789012123456789012345678901
2
12345678901234567890123456789012123456789012345678901
2
12345678901234567890123456789012123456789012345678901
If
this
question
is
a
mystery
to
you,
you
can
at
least
reason
that
Column
A
2345678901234567890123456789012123456789012345678901
12345678901234567890123456789012123456789012345678901 22
12345678901234567890123456789012123456789012345678901
will have a y in it. This means that the two columns will probably have a 2
12345678901234567890123456789012123456789012345678901
2
12345678901234567890123456789012123456789012345678901
definite relationship, so (D) is out. The trick is to make Column A look like 2
12345678901234567890123456789012123456789012345678901
12345678901234567890123456789012123456789012345678901
something you can compare to Column B, which you can do if you figure 2
2
12345678901234567890123456789012123456789012345678901
2
12345678901234567890123456789012123456789012345678901
out
what
the
diameter
of
the
circle
is:
12345678901234567890123456789012123456789012345678901 22
1
2
12345678901234567890123456789012123456789012345678901
Area of a circle 5 pr2
2
12345678901234567890123456789012123456789012345678901
2
12345678901234567890123456789012123456789012345678901
2
2
12345678901234567890123456789012123456789012345678901
py
5
pr

2
12345678901234567890123456789012123456789012345678901
2345678901234567890123456789012123456789012345678901
2
12345678901234567890123456789012123456789012345678901
py 4 p 5 pr2 4 p
2
12345678901234567890123456789012123456789012345678901
2
1
2
2
12345678901234567890123456789012123456789012345678901
y5r
2
12345678901234567890123456789012123456789012345678901
2345678901234567890123456789012123456789012345678901
2
12345678901234567890123456789012123456789012345678901
y5r
=
2
12345678901234567890123456789012123456789012345678901
2
1
12345678901234567890123456789012123456789012345678901 2
12345678901234567890123456789012123456789012345678901
If thats the radius, then the diameter is twice that, or d 5 2=y. Compare 2
2
12345678901234567890123456789012123456789012345678901
this with Column B after you sort it all out:
2
12345678901234567890123456789012123456789012345678901
2
12345678901234567890123456789012123456789012345678901
2
12345678901234567890123456789012123456789012345678901
3
2
12345678901234567890123456789012123456789012345678901
3
1
2
2
y
2345678901234567890123456789012123456789012345678901
2
12345678901234567890123456789012123456789012345678901
2
5 y2 2 2 5 y2 5 =y
12345678901234567890123456789012123456789012345678901
2
12345678901234567890123456789012123456789012345678901
y
12345678901234567890123456789012123456789012345678901 22
1 (A) is greater.
2
12345678901234567890123456789012123456789012345678901
2
12345678901234567890123456789012123456789012345678901
2
12345678901234567890123456789012123456789012345678901
2
12345678901234567890123456789012123456789012345678901
12345678901234567890123456789012123456789012345678901 2
2345678901234567890123456789012123456789012345678901
12345678901234567890123456789012123456789012345678901 22
1
2
12345678901234567890123456789012123456789012345678901
12345678901234567890123456789012123456789012345678901 2
2
12345678901234567890123456789012123456789012345678901
12345678901234567890123456789012123456789012345678901 2
12345678901234567890123456789012123456789012345678901 2
12345678901234567890123456789012123456789012345678901 2
12345678901234567890123456789012123456789012345678901 2 95
123456789012345678901234567890121234567890123456789012

http://www.xtremepapers.net

Part III: Section Review

www.petersons.com

http://www.xtremepapers.net

Alert!

96

123456789012345678901234567890121234567890123456789012
12345678901234567890123456789012123456789012345678901 2
2
12345678901234567890123456789012123456789012345678901
12345678901234567890123456789012123456789012345678901 2
2
12345678901234567890123456789012123456789012345678901
2
12345678901234567890123456789012123456789012345678901
12345678901234567890123456789012123456789012345678901 2
12345678901234567890123456789012123456789012345678901
If you didnt know that the y had to be positive since it is a distance, you 2
2
12345678901234567890123456789012123456789012345678901
2
12345678901234567890123456789012123456789012345678901
would not know which column was greater.
2
12345678901234567890123456789012123456789012345678901
2
12345678901234567890123456789012123456789012345678901
2
12345678901234567890123456789012123456789012345678901
2345678901234567890123456789012123456789012345678901
2
1
2
12345678901234567890123456789012123456789012345678901
2
12345678901234567890123456789012123456789012345678901
A tangent to a circle is a line that has one point in common with a circle. 2
12345678901234567890123456789012123456789012345678901
2
12345678901234567890123456789012123456789012345678901
The figure below is an illustration of a circle and one tangent line. Notice 2
12345678901234567890123456789012123456789012345678901
12345678901234567890123456789012123456789012345678901
that the tangent line forms a right angle with the radius that shares a point 2
2
12345678901234567890123456789012123456789012345678901
2
12345678901234567890123456789012123456789012345678901
with
it.
Tuck
that
fact
away
in
your
head
because
you
might
need
it
later.
2
12345678901234567890123456789012123456789012345678901
2345678901234567890123456789012123456789012345678901
2
1
2
12345678901234567890123456789012123456789012345678901
2
12345678901234567890123456789012123456789012345678901
2
12345678901234567890123456789012123456789012345678901
2
12345678901234567890123456789012123456789012345678901
2345678901234567890123456789012123456789012345678901
12345678901234567890123456789012123456789012345678901 2
12345678901234567890123456789012123456789012345678901 2
12345678901234567890123456789012123456789012345678901 2
12345678901234567890123456789012123456789012345678901
Another word that might come up with circles or other shapes is inscribed. 2
2
12345678901234567890123456789012123456789012345678901
2
12345678901234567890123456789012123456789012345678901
1 A polygon is inscribed (in-scribed, drawn in) in a circle if each of its 2
12345678901234567890123456789012123456789012345678901
vertices lies on the circle. A circle can also be inscribed in a polygon if each 2
2
12345678901234567890123456789012123456789012345678901
12345678901234567890123456789012123456789012345678901
of the polygons sides is tangent to the circle. Circumscribed is related to 2
2345678901234567890123456789012123456789012345678901
12345678901234567890123456789012123456789012345678901 2
2
12345678901234567890123456789012123456789012345678901
inscribed in an antonym-like way.
2
12345678901234567890123456789012123456789012345678901
12345678901234567890123456789012123456789012345678901 2
12345678901234567890123456789012123456789012345678901 2
12345678901234567890123456789012123456789012345678901 2
12345678901234567890123456789012123456789012345678901 2
12345678901234567890123456789012123456789012345678901 2
12345678901234567890123456789012123456789012345678901 2
2
1
2
12345678901234567890123456789012123456789012345678901
2
12345678901234567890123456789012123456789012345678901
12345678901234567890123456789012123456789012345678901 2
12345678901234567890123456789012123456789012345678901 2
2
12345678901234567890123456789012123456789012345678901
12345678901234567890123456789012123456789012345678901
Notice that when figures are inscribed and circumscribed, they share 2
2
12345678901234567890123456789012123456789012345678901
2
12345678901234567890123456789012123456789012345678901
certain
features.
With
the
circle
inscribed
above
in
the
square,
the
diameter
2
12345678901234567890123456789012123456789012345678901
2
12345678901234567890123456789012123456789012345678901
of
the
circle
is
also
the
length
of
one
side
of
the
square.
These
sort
of
2345678901234567890123456789012123456789012345678901
12345678901234567890123456789012123456789012345678901 2
2
12345678901234567890123456789012123456789012345678901
1 overlapping features are what the GRE focuses on when there are multiple 2
12345678901234567890123456789012123456789012345678901
figures involved in a problem. Be aware of what features figures share 2
2
12345678901234567890123456789012123456789012345678901
2345678901234567890123456789012123456789012345678901
2
12345678901234567890123456789012123456789012345678901
because that often is the clue to the answer.
2
12345678901234567890123456789012123456789012345678901
2
12345678901234567890123456789012123456789012345678901
Column B
Column A
2
12345678901234567890123456789012123456789012345678901
2
1
2345678901234567890123456789012123456789012345678901
12345678901234567890123456789012123456789012345678901 2
12345678901234567890123456789012123456789012345678901 2
12345678901234567890123456789012123456789012345678901 2
12345678901234567890123456789012123456789012345678901 2
2
1
2
12345678901234567890123456789012123456789012345678901
2
12345678901234567890123456789012123456789012345678901
2
12345678901234567890123456789012123456789012345678901
The area of the above circle is 16p.
2
12345678901234567890123456789012123456789012345678901
12345678901234567890123456789012123456789012345678901 2
2345678901234567890123456789012123456789012345678901
2
12345678901234567890123456789012123456789012345678901
The area of the
6l
2
1
2
12345678901234567890123456789012123456789012345678901
rectangle
12345678901234567890123456789012123456789012345678901 2
2
12345678901234567890123456789012123456789012345678901
12345678901234567890123456789012123456789012345678901 2
12345678901234567890123456789012123456789012345678901 2
12345678901234567890123456789012123456789012345678901 2
12345678901234567890123456789012123456789012345678901 2
123456789012345678901234567890121234567890123456789012

Chapter 4: Quantitative Review

123456789012345678901234567890121234567890123456789012
12345678901234567890123456789012123456789012345678901 2
2
12345678901234567890123456789012123456789012345678901
Look at the features the two figures share. Notice that the diameter of the 2
12345678901234567890123456789012123456789012345678901
2
12345678901234567890123456789012123456789012345678901
circle is the width of the rectangle. If you can find the diameter, you can 2
12345678901234567890123456789012123456789012345678901
2
12345678901234567890123456789012123456789012345678901
compare the two columns:
2
12345678901234567890123456789012123456789012345678901
2345678901234567890123456789012123456789012345678901
2
1
2
2
12345678901234567890123456789012123456789012345678901
A
5
pr
2
12345678901234567890123456789012123456789012345678901
2
12345678901234567890123456789012123456789012345678901
2
16p
5
pr
2
12345678901234567890123456789012123456789012345678901
2345678901234567890123456789012123456789012345678901
2
1
2
2
12345678901234567890123456789012123456789012345678901
16 5 r
2
12345678901234567890123456789012123456789012345678901
2
12345678901234567890123456789012123456789012345678901
r
5
4
2
12345678901234567890123456789012123456789012345678901
2345678901234567890123456789012123456789012345678901
2
1
12345678901234567890123456789012123456789012345678901
If the radius is 4, then the diameter is twice that, so d 5 8. The area of the 2
2
12345678901234567890123456789012123456789012345678901
2
12345678901234567890123456789012123456789012345678901
rectangle is 8l, which means that (A) is greater.
2
12345678901234567890123456789012123456789012345678901
2345678901234567890123456789012123456789012345678901
1 One last point: Concentric circles have the same center but do not have the 2
2
12345678901234567890123456789012123456789012345678901
2
12345678901234567890123456789012123456789012345678901
same radius.
2
12345678901234567890123456789012123456789012345678901
2
12345678901234567890123456789012123456789012345678901
2345678901234567890123456789012123456789012345678901
12345678901234567890123456789012123456789012345678901 22
12345678901234567890123456789012123456789012345678901 2
12345678901234567890123456789012123456789012345678901 2
12345678901234567890123456789012123456789012345678901 2
12345678901234567890123456789012123456789012345678901 2
12345678901234567890123456789012123456789012345678901 2
1
2
12345678901234567890123456789012123456789012345678901
2
12345678901234567890123456789012123456789012345678901
2
12345678901234567890123456789012123456789012345678901
2345678901234567890123456789012123456789012345678901
12345678901234567890123456789012123456789012345678901 22
12345678901234567890123456789012123456789012345678901 2
12345678901234567890123456789012123456789012345678901 2
12345678901234567890123456789012123456789012345678901 2
12345678901234567890123456789012123456789012345678901 2
12345678901234567890123456789012123456789012345678901 2
12345678901234567890123456789012123456789012345678901 2
12345678901234567890123456789012123456789012345678901 2
12345678901234567890123456789012123456789012345678901
2
1Coordinate Geometry
12345678901234567890123456789012123456789012345678901
The above phrase refers to geometry done on a coordinate plane. When 2
2
12345678901234567890123456789012123456789012345678901
2
12345678901234567890123456789012123456789012345678901
you
see
the
following
visual,
youll
either
remember
coordinate
geometry
12345678901234567890123456789012123456789012345678901 2
2
12345678901234567890123456789012123456789012345678901
or playing the game Battleship! as a child. B . . . 4 . . . Miss!
2
12345678901234567890123456789012123456789012345678901
2
12345678901234567890123456789012123456789012345678901
12345678901234567890123456789012123456789012345678901 2
2
12345678901234567890123456789012123456789012345678901
2
12345678901234567890123456789012123456789012345678901
2345678901234567890123456789012123456789012345678901
12345678901234567890123456789012123456789012345678901 22
12345678901234567890123456789012123456789012345678901 2
1
12345678901234567890123456789012123456789012345678901 2
12345678901234567890123456789012123456789012345678901 2
2
12345678901234567890123456789012123456789012345678901
2
12345678901234567890123456789012123456789012345678901
2
12345678901234567890123456789012123456789012345678901
2
12345678901234567890123456789012123456789012345678901
12345678901234567890123456789012123456789012345678901 2
2345678901234567890123456789012123456789012345678901
12345678901234567890123456789012123456789012345678901 22
12345678901234567890123456789012123456789012345678901 2
12345678901234567890123456789012123456789012345678901 2
12345678901234567890123456789012123456789012345678901 2
1
2
12345678901234567890123456789012123456789012345678901
2
12345678901234567890123456789012123456789012345678901
2
12345678901234567890123456789012123456789012345678901
2
12345678901234567890123456789012123456789012345678901
12345678901234567890123456789012123456789012345678901 2
2345678901234567890123456789012123456789012345678901
12345678901234567890123456789012123456789012345678901 22
1 A coordinate plane is just two number lines perpendicular to each other.
2
12345678901234567890123456789012123456789012345678901
2
12345678901234567890123456789012123456789012345678901
The
one
going
up
and
down
is
called
the
y-axis,
and
the
one
going
left
and
2
12345678901234567890123456789012123456789012345678901
2345678901234567890123456789012123456789012345678901
1 right is called the x-axis. Any point on the plane gets two coordinates, an 2
12345678901234567890123456789012123456789012345678901 2
12345678901234567890123456789012123456789012345678901 2
12345678901234567890123456789012123456789012345678901 2 97
123456789012345678901234567890121234567890123456789012

http://www.xtremepapers.net

Part III: Section Review

98

123456789012345678901234567890121234567890123456789012
12345678901234567890123456789012123456789012345678901 2
2
12345678901234567890123456789012123456789012345678901
x one and a y one, and they tell you where the point goes on the plane. The 2
12345678901234567890123456789012123456789012345678901
2
12345678901234567890123456789012123456789012345678901
notation for this looks like this (x,y), with the x-coordinate always first.
2
12345678901234567890123456789012123456789012345678901
12345678901234567890123456789012123456789012345678901 2
12345678901234567890123456789012123456789012345678901
The number in the x spot tells you where along the x-axis the point goes, 2
2
12345678901234567890123456789012123456789012345678901
2
12345678901234567890123456789012123456789012345678901
and
the
number
in
the
y
spot
tells
you
where
along
the
y-axis
the
point
2
12345678901234567890123456789012123456789012345678901
2
12345678901234567890123456789012123456789012345678901
goes.
The
points
(24,2)
and
(0,3)
would
be
here:
2
12345678901234567890123456789012123456789012345678901
2345678901234567890123456789012123456789012345678901
2
1
2
12345678901234567890123456789012123456789012345678901
2
12345678901234567890123456789012123456789012345678901
2
12345678901234567890123456789012123456789012345678901
2
12345678901234567890123456789012123456789012345678901
2345678901234567890123456789012123456789012345678901
2
1
2
12345678901234567890123456789012123456789012345678901
2
12345678901234567890123456789012123456789012345678901
2
12345678901234567890123456789012123456789012345678901
2
12345678901234567890123456789012123456789012345678901
2345678901234567890123456789012123456789012345678901
2
1
2
12345678901234567890123456789012123456789012345678901
2
12345678901234567890123456789012123456789012345678901
2
12345678901234567890123456789012123456789012345678901
2
12345678901234567890123456789012123456789012345678901
2345678901234567890123456789012123456789012345678901
12345678901234567890123456789012123456789012345678901 2
12345678901234567890123456789012123456789012345678901 2
12345678901234567890123456789012123456789012345678901 2
12345678901234567890123456789012123456789012345678901 2
12345678901234567890123456789012123456789012345678901 2
2
12345678901234567890123456789012123456789012345678901
1 Say that a line was drawn between those two points. Would the slope be 2
12345678901234567890123456789012123456789012345678901
steep or flat? Looking at it, the slope would rise if you were going from left 2
2
12345678901234567890123456789012123456789012345678901
2
12345678901234567890123456789012123456789012345678901
to
right,
and
that
is
the
direction
that
geometrists
are
always
going
on
2345678901234567890123456789012123456789012345678901
12345678901234567890123456789012123456789012345678901 2
12345678901234567890123456789012123456789012345678901
coordinate planes. Yet you can say more than the slope is going up. You 2
2
12345678901234567890123456789012123456789012345678901
2
12345678901234567890123456789012123456789012345678901
can
give
it
a
value
since
there
is
a
mathematical
definition
of
slope.
It
is
the
12345678901234567890123456789012123456789012345678901 2
12345678901234567890123456789012123456789012345678901
change in the y values over the change in the x values, also called the rise 2
2
12345678901234567890123456789012123456789012345678901
2
12345678901234567890123456789012123456789012345678901
over
the
run.
12345678901234567890123456789012123456789012345678901 2
2
1
2
12345678901234567890123456789012123456789012345678901
In this case, the slope is
2
12345678901234567890123456789012123456789012345678901
12345678901234567890123456789012123456789012345678901 2
2
12345678901234567890123456789012123456789012345678901
322
1
Dy y1 2 y2
2
12345678901234567890123456789012123456789012345678901
5
5
5
2345678901234567890123456789012123456789012345678901
2
12345678901234567890123456789012123456789012345678901
Dx x1 2 x2 0 2 ~24! 4
2
12345678901234567890123456789012123456789012345678901
2
1
12345678901234567890123456789012123456789012345678901
Always subtract the coordinate values of the point on the right from the 2
2
12345678901234567890123456789012123456789012345678901
12345678901234567890123456789012123456789012345678901
coordinate values from the point on the left. The slope is positive 2
2
12345678901234567890123456789012123456789012345678901
12345678901234567890123456789012123456789012345678901
one fourth, which means that the line goes up one in the y-direction for 2
2
12345678901234567890123456789012123456789012345678901
every four it goes over in the x-direction (again, if you are traveling left to 2
12345678901234567890123456789012123456789012345678901
2345678901234567890123456789012123456789012345678901
2
12345678901234567890123456789012123456789012345678901
right). A negative slope means that the line goes down from left to right 2
12345678901234567890123456789012123456789012345678901
2
12345678901234567890123456789012123456789012345678901
instead of up.
2
12345678901234567890123456789012123456789012345678901
2
1
2345678901234567890123456789012123456789012345678901
12345678901234567890123456789012123456789012345678901
The geometry part of coordinate geometry comes in when geometrical 2
2
12345678901234567890123456789012123456789012345678901
2
12345678901234567890123456789012123456789012345678901
figures
are
put
on
the
coordinate
plane.
In
one
way,
this
makes
the
12345678901234567890123456789012123456789012345678901 2
1 geometry easier because you have the coordinate plane to help do the 2
2
12345678901234567890123456789012123456789012345678901
2
12345678901234567890123456789012123456789012345678901
geometry.
In
another
way,
though,
the
test
will
up
the
complexity
of
a
2
12345678901234567890123456789012123456789012345678901
2
12345678901234567890123456789012123456789012345678901
problem
by
expecting
you
to
make
inferences
about
the
geometrical
12345678901234567890123456789012123456789012345678901 2
2345678901234567890123456789012123456789012345678901
2
12345678901234567890123456789012123456789012345678901
2
1 figures based on your knowledge of coordinate geometry.
2
12345678901234567890123456789012123456789012345678901
12345678901234567890123456789012123456789012345678901 2
2
12345678901234567890123456789012123456789012345678901
12345678901234567890123456789012123456789012345678901 2
12345678901234567890123456789012123456789012345678901 2
12345678901234567890123456789012123456789012345678901 2
12345678901234567890123456789012123456789012345678901 2
123456789012345678901234567890121234567890123456789012

www.petersons.com

http://www.xtremepapers.net

Chapter 4: Quantitative Review

123456789012345678901234567890121234567890123456789012
12345678901234567890123456789012123456789012345678901 2
2
12345678901234567890123456789012123456789012345678901
2
12345678901234567890123456789012123456789012345678901
Column A
Column B
2
12345678901234567890123456789012123456789012345678901
2
12345678901234567890123456789012123456789012345678901
A figure is defined by these points (1,1),
2
12345678901234567890123456789012123456789012345678901
2
12345678901234567890123456789012123456789012345678901
(3,1),
and
(1,22)
on
a
coordinate
plane.
2345678901234567890123456789012123456789012345678901
2
1
2
12345678901234567890123456789012123456789012345678901
2
12345678901234567890123456789012123456789012345678901
The
area
of
the
The
perimeter
of
2
12345678901234567890123456789012123456789012345678901
2
12345678901234567890123456789012123456789012345678901
figure
the
figure
2345678901234567890123456789012123456789012345678901
2
1
2
12345678901234567890123456789012123456789012345678901
2
12345678901234567890123456789012123456789012345678901
12345678901234567890123456789012123456789012345678901
If this question mystifies you, or you are short on time, realize that the 2
2
12345678901234567890123456789012123456789012345678901
12345678901234567890123456789012123456789012345678901
perimeter of a figure and the area of a figure will have a definite 2
2
12345678901234567890123456789012123456789012345678901
12345678901234567890123456789012123456789012345678901
relationship, and so the answer is probably not (D). That makes guessing a 2
2
12345678901234567890123456789012123456789012345678901
one in three chance. If you have the time to do the problem, you should 2
12345678901234567890123456789012123456789012345678901
2345678901234567890123456789012123456789012345678901
1 suspect that this figure is a triangle since it is defined by three points. 2
2
12345678901234567890123456789012123456789012345678901
2
12345678901234567890123456789012123456789012345678901
Regardless of what you suspect, though, the way to attack this problem is 2
12345678901234567890123456789012123456789012345678901
2
12345678901234567890123456789012123456789012345678901
to quickly plot the points on a coordinate plane
2
12345678901234567890123456789012123456789012345678901
2
12345678901234567890123456789012123456789012345678901
2
12345678901234567890123456789012123456789012345678901
2
12345678901234567890123456789012123456789012345678901
2
12345678901234567890123456789012123456789012345678901
2345678901234567890123456789012123456789012345678901
12345678901234567890123456789012123456789012345678901 22
1
2
12345678901234567890123456789012123456789012345678901
2
12345678901234567890123456789012123456789012345678901
2
12345678901234567890123456789012123456789012345678901
2345678901234567890123456789012123456789012345678901
12345678901234567890123456789012123456789012345678901 22
12345678901234567890123456789012123456789012345678901 2
12345678901234567890123456789012123456789012345678901 2
12345678901234567890123456789012123456789012345678901 2
12345678901234567890123456789012123456789012345678901 2
12345678901234567890123456789012123456789012345678901 2
12345678901234567890123456789012123456789012345678901 2
12345678901234567890123456789012123456789012345678901 2
12345678901234567890123456789012123456789012345678901 2
1 It is a right triangle, and you can use this fact to find both the area and the
2
12345678901234567890123456789012123456789012345678901
2
12345678901234567890123456789012123456789012345678901
perimeter. Lets do the area first. The distance between (1,1) and (1,22) is 2
12345678901234567890123456789012123456789012345678901
2
12345678901234567890123456789012123456789012345678901
three units, so the height is three. The distance between (1,1) and (3,1) is 2
12345678901234567890123456789012123456789012345678901
2345678901234567890123456789012123456789012345678901
2
12345678901234567890123456789012123456789012345678901
two units, making it the base.
2
12345678901234567890123456789012123456789012345678901
2
1
2
12345678901234567890123456789012123456789012345678901
1
2
12345678901234567890123456789012123456789012345678901
bh
A
5
2345678901234567890123456789012123456789012345678901
2
12345678901234567890123456789012123456789012345678901
2
2
12345678901234567890123456789012123456789012345678901
2
1
2
12345678901234567890123456789012123456789012345678901
1
2
12345678901234567890123456789012123456789012345678901
2
3
5
3
A
5
~
!~
!
2345678901234567890123456789012123456789012345678901
2
12345678901234567890123456789012123456789012345678901
2
2
12345678901234567890123456789012123456789012345678901
12345678901234567890123456789012123456789012345678901 22
12345678901234567890123456789012123456789012345678901
2
1 The perimeter is the sum of the sides. You have
2
12345678901234567890123456789012123456789012345678901
2
12345678901234567890123456789012123456789012345678901
Perimeter
5
2
1
3
1
hypotenuse
2
12345678901234567890123456789012123456789012345678901
1
2
12345678901234567890123456789012123456789012345678901
2
3
2
12345678901234567890123456789012123456789012345678901
2
2
1
3
Perimeter
5
5
1
~
!
2345678901234567890123456789012123456789012345678901
12345678901234567890123456789012123456789012345678901 22
12345678901234567890123456789012123456789012345678901 2
12345678901234567890123456789012123456789012345678901
The Pythagorean theorem was used for the hypotenuse. At this point, you
2
12345678901234567890123456789012123456789012345678901
1 dont have to finish the problem since this sum is clearly greater than 3. 2
2345678901234567890123456789012123456789012345678901
2
12345678901234567890123456789012123456789012345678901
2
1 Therefore, choice (B) is the correct answer.
2
12345678901234567890123456789012123456789012345678901
12345678901234567890123456789012123456789012345678901 2
2
12345678901234567890123456789012123456789012345678901
12345678901234567890123456789012123456789012345678901 2
12345678901234567890123456789012123456789012345678901 2
12345678901234567890123456789012123456789012345678901 2
12345678901234567890123456789012123456789012345678901 2 99
123456789012345678901234567890121234567890123456789012

http://www.xtremepapers.net

Part III: Section Review

123456789012345678901234567890121234567890123456789012
12345678901234567890123456789012123456789012345678901 2
2
12345678901234567890123456789012123456789012345678901
2
12345678901234567890123456789012123456789012345678901
Data
Analysis
2
12345678901234567890123456789012123456789012345678901
2
12345678901234567890123456789012123456789012345678901
If
you
read
USA
Today
on
a
regular
basis,
youll
have
experience
with
data
12345678901234567890123456789012123456789012345678901 2
12345678901234567890123456789012123456789012345678901
analysis. The data that you are expected to analyze usually comes in the 2
2
12345678901234567890123456789012123456789012345678901
2
12345678901234567890123456789012123456789012345678901
form
of
chart,
graph,
or
table,
three
things
that
national
newspapers
love
2
12345678901234567890123456789012123456789012345678901
2
12345678901234567890123456789012123456789012345678901
to
print.
Its
visual
information
that
needs
to
be
understood
and
then
used
2
12345678901234567890123456789012123456789012345678901
2345678901234567890123456789012123456789012345678901
2
1 in some manner.
2
12345678901234567890123456789012123456789012345678901
2
12345678901234567890123456789012123456789012345678901
12345678901234567890123456789012123456789012345678901
Before jumping into all the different graphs, there is one other topic often 2
2
12345678901234567890123456789012123456789012345678901
12345678901234567890123456789012123456789012345678901
placed under the Data Analysis umbrella. What are the odds you can guess 2
2
12345678901234567890123456789012123456789012345678901
2
12345678901234567890123456789012123456789012345678901
what it is?
2
12345678901234567890123456789012123456789012345678901
2
12345678901234567890123456789012123456789012345678901
2345678901234567890123456789012123456789012345678901
2
1
2
12345678901234567890123456789012123456789012345678901
2
12345678901234567890123456789012123456789012345678901
Probability
2
12345678901234567890123456789012123456789012345678901
2
12345678901234567890123456789012123456789012345678901
If
you
flip
a
coin,
what
is
the
chance
that
it
will
land
on
heads?
Pretty
much
2345678901234567890123456789012123456789012345678901
12345678901234567890123456789012123456789012345678901 2
12345678901234567890123456789012123456789012345678901
everyone knows the answer is one out of two or fifty-fifty. This is 2
2
12345678901234567890123456789012123456789012345678901
2
12345678901234567890123456789012123456789012345678901
because
there
are
two
possible
outcomes
that
are
equally
probable
12345678901234567890123456789012123456789012345678901 2
2
12345678901234567890123456789012123456789012345678901
1 (landing heads, landing tails), so the chance the coin will land on heads is 2
2
12345678901234567890123456789012123456789012345678901
one out of two.
2
12345678901234567890123456789012123456789012345678901
2
12345678901234567890123456789012123456789012345678901
2345678901234567890123456789012123456789012345678901
12345678901234567890123456789012123456789012345678901
Thats kid stuff, but you just covered the basics of probability theory. In 2
2
12345678901234567890123456789012123456789012345678901
12345678901234567890123456789012123456789012345678901
any probability scenario, the probability of A occurring is the number of 2
12345678901234567890123456789012123456789012345678901 2
12345678901234567890123456789012123456789012345678901
ways A can occur divided by the total number of possible outcomes. In our 2
2
12345678901234567890123456789012123456789012345678901
12345678901234567890123456789012123456789012345678901
case, A is the coin landing on heads, and that can only happen one way. 2
2
12345678901234567890123456789012123456789012345678901
1
2
12345678901234567890123456789012123456789012345678901
2
1 There are two possible outcomes, heads or tails, so the probability is .
2345678901234567890123456789012123456789012345678901
2
12345678901234567890123456789012123456789012345678901
2
12345678901234567890123456789012123456789012345678901 2
2
1
number of desired outcomes
2
12345678901234567890123456789012123456789012345678901
probability
5
2
12345678901234567890123456789012123456789012345678901
total
number
of
possible
outcomes
2345678901234567890123456789012123456789012345678901
12345678901234567890123456789012123456789012345678901 2
12345678901234567890123456789012123456789012345678901 2
1 Things get a little more interesting when you start talking about multiple 2
2
12345678901234567890123456789012123456789012345678901
2
12345678901234567890123456789012123456789012345678901
event
probability,
but
thats
nothing
you
cant
handle
if
you
followed
2345678901234567890123456789012123456789012345678901
12345678901234567890123456789012123456789012345678901 2
2
12345678901234567890123456789012123456789012345678901
1 things thus far. Say you flip the coin again. What are the chances that it will 2
2
12345678901234567890123456789012123456789012345678901
1
2
12345678901234567890123456789012123456789012345678901
land
on
heads
this
time?
Its
still
.
If
you
thought
otherwise,
remember
2345678901234567890123456789012123456789012345678901
2
12345678901234567890123456789012123456789012345678901
2
2
12345678901234567890123456789012123456789012345678901
that each coin toss is independent and unaffected by previous coin tosses. 2
12345678901234567890123456789012123456789012345678901
12345678901234567890123456789012123456789012345678901 2
1 Moving up one more notch, what is the probability of tossing the coin twice 2
2
12345678901234567890123456789012123456789012345678901
12345678901234567890123456789012123456789012345678901
and it landing on heads twice? It is the probability of the first event times 2
2
12345678901234567890123456789012123456789012345678901
2
12345678901234567890123456789012123456789012345678901
1
1
1
2
12345678901234567890123456789012123456789012345678901
5
the
probability
of
the
second
event.
Therefore,
probability
5
.
2345678901234567890123456789012123456789012345678901
2
12345678901234567890123456789012123456789012345678901
2 2
4
2
12345678901234567890123456789012123456789012345678901
12345678901234567890123456789012123456789012345678901
Thats the basics of multiple-event probability. The probability of multiple 2
12345678901234567890123456789012123456789012345678901 2
2
1 events occurring is the product of the individual probabilities involved.
2
12345678901234567890123456789012123456789012345678901
12345678901234567890123456789012123456789012345678901 2
2
12345678901234567890123456789012123456789012345678901
12345678901234567890123456789012123456789012345678901 2
2
12345678901234567890123456789012123456789012345678901
12345678901234567890123456789012123456789012345678901 2
12345678901234567890123456789012123456789012345678901 2
12345678901234567890123456789012123456789012345678901 2
2
100 12345678901234567890123456789012123456789012345678901
123456789012345678901234567890121234567890123456789012

S DS D

www.petersons.com

http://www.xtremepapers.net

Chapter 4: Quantitative Review

123456789012345678901234567890121234567890123456789012
12345678901234567890123456789012123456789012345678901 2
2
12345678901234567890123456789012123456789012345678901
There are 5 boys and 7 girls in the fifth grade class at Saddleback
2
12345678901234567890123456789012123456789012345678901
2
12345678901234567890123456789012123456789012345678901
Elementary. What is the probability that if 2 students are chosen at
2
12345678901234567890123456789012123456789012345678901
2
12345678901234567890123456789012123456789012345678901
random
from
the
class,
that
both
students
will
be
girls?
2
12345678901234567890123456789012123456789012345678901
2345678901234567890123456789012123456789012345678901
2
1
2
12345678901234567890123456789012123456789012345678901
7
2
12345678901234567890123456789012123456789012345678901
A.
2
12345678901234567890123456789012123456789012345678901
24
2
12345678901234567890123456789012123456789012345678901
2345678901234567890123456789012123456789012345678901
2
1
7
2
12345678901234567890123456789012123456789012345678901
2
12345678901234567890123456789012123456789012345678901
B.
2
12345678901234567890123456789012123456789012345678901
22
2
12345678901234567890123456789012123456789012345678901
2345678901234567890123456789012123456789012345678901
2
1
1
2
12345678901234567890123456789012123456789012345678901
C.
2
12345678901234567890123456789012123456789012345678901
2
2
12345678901234567890123456789012123456789012345678901
2
12345678901234567890123456789012123456789012345678901
7
2345678901234567890123456789012123456789012345678901
2
1
D.
2
12345678901234567890123456789012123456789012345678901
12
2
12345678901234567890123456789012123456789012345678901
2
12345678901234567890123456789012123456789012345678901
2
12345678901234567890123456789012123456789012345678901
11
2345678901234567890123456789012123456789012345678901
2
12345678901234567890123456789012123456789012345678901
E.
2
12345678901234567890123456789012123456789012345678901
12
12345678901234567890123456789012123456789012345678901 22
12345678901234567890123456789012123456789012345678901
This probability problem involves two events, and you first need to 2
12345678901234567890123456789012123456789012345678901
2
12345678901234567890123456789012123456789012345678901
1 determine the probability of each event. A girl getting picked first has a 2
2
12345678901234567890123456789012123456789012345678901
7
2
12345678901234567890123456789012123456789012345678901
since
there
are
7
girls
and
a
total
of
12
students
in
the
probability
of
2
12345678901234567890123456789012123456789012345678901
12
2345678901234567890123456789012123456789012345678901
12345678901234567890123456789012123456789012345678901 22
12345678901234567890123456789012123456789012345678901
6
12345678901234567890123456789012123456789012345678901
class. The probability of a girl getting picked the second time is . Why? 2
2
12345678901234567890123456789012123456789012345678901
11
2
12345678901234567890123456789012123456789012345678901
Because
one
girl
has
been
picked
already,
leaving
only
6
girls
left
in
the
12345678901234567890123456789012123456789012345678901 22
12345678901234567890123456789012123456789012345678901
class of 11 students. Multiplying the two probabilities together gives you: 2
12345678901234567890123456789012123456789012345678901
12345678901234567890123456789012123456789012345678901 22
1
6
7
2
12345678901234567890123456789012123456789012345678901
3
5
2
12345678901234567890123456789012123456789012345678901
12 11
2
12345678901234567890123456789012123456789012345678901
12345678901234567890123456789012123456789012345678901 2
2
12345678901234567890123456789012123456789012345678901
~7 3 6! 5
2
12345678901234567890123456789012123456789012345678901
2
12345678901234567890123456789012123456789012345678901
2
12345678901234567890123456789012123456789012345678901
~
12 3 11!
2
12345678901234567890123456789012123456789012345678901
2
12345678901234567890123456789012123456789012345678901
7
2345678901234567890123456789012123456789012345678901
2
12345678901234567890123456789012123456789012345678901
5
2
12345678901234567890123456789012123456789012345678901
22
2
1
12345678901234567890123456789012123456789012345678901 2
2
12345678901234567890123456789012123456789012345678901
Thats choice (B).
2
12345678901234567890123456789012123456789012345678901
2
12345678901234567890123456789012123456789012345678901
2
12345678901234567890123456789012123456789012345678901
2
12345678901234567890123456789012123456789012345678901
12345678901234567890123456789012123456789012345678901 2
2345678901234567890123456789012123456789012345678901
12345678901234567890123456789012123456789012345678901 22
12345678901234567890123456789012123456789012345678901 2
12345678901234567890123456789012123456789012345678901 2
12345678901234567890123456789012123456789012345678901 2
1
2
12345678901234567890123456789012123456789012345678901
2
12345678901234567890123456789012123456789012345678901
2
12345678901234567890123456789012123456789012345678901
2
12345678901234567890123456789012123456789012345678901
12345678901234567890123456789012123456789012345678901 2
2345678901234567890123456789012123456789012345678901
12345678901234567890123456789012123456789012345678901 22
1
2
12345678901234567890123456789012123456789012345678901
12345678901234567890123456789012123456789012345678901 2
2
12345678901234567890123456789012123456789012345678901
12345678901234567890123456789012123456789012345678901 2
12345678901234567890123456789012123456789012345678901 2
12345678901234567890123456789012123456789012345678901 2
12345678901234567890123456789012123456789012345678901 2 101
123456789012345678901234567890121234567890123456789012

http://www.xtremepapers.net

Part III: Section Review

www.petersons.com

http://www.xtremepapers.net

Note

123456789012345678901234567890121234567890123456789012
12345678901234567890123456789012123456789012345678901 2
2
12345678901234567890123456789012123456789012345678901
2
12345678901234567890123456789012123456789012345678901
Workin the Data
2
12345678901234567890123456789012123456789012345678901
2
12345678901234567890123456789012123456789012345678901
Data
or
information
can
come
to
you
in
lots
of
different
forms.
You
are
12345678901234567890123456789012123456789012345678901 2
12345678901234567890123456789012123456789012345678901
getting text information right now, for instance, as you read this sentence. 2
2
12345678901234567890123456789012123456789012345678901
12345678901234567890123456789012123456789012345678901
Earlier you might have looked at the table of contents of this book to get 2
2
12345678901234567890123456789012123456789012345678901
12345678901234567890123456789012123456789012345678901
some information about where to start reading. The GRE likes to test your 2
2
12345678901234567890123456789012123456789012345678901
12345678901234567890123456789012123456789012345678901
ability to take in information and answer questions about that informa- 2
2
12345678901234567890123456789012123456789012345678901
12345678901234567890123456789012123456789012345678901
tion. Specifically, the GRE math section will present information or data in 2
2
12345678901234567890123456789012123456789012345678901
the form of a line graph, a circle graph, a bar graph, a table, or some 2
12345678901234567890123456789012123456789012345678901
2345678901234567890123456789012123456789012345678901
1 combination of all of these, and then expect you to answer questions about 2
2
12345678901234567890123456789012123456789012345678901
2
12345678901234567890123456789012123456789012345678901
the information presented. You might think this sounds too easy, and in a 2
12345678901234567890123456789012123456789012345678901
2
12345678901234567890123456789012123456789012345678901
sense it is. All the answers will be right in front of you in a graph or table. 2
12345678901234567890123456789012123456789012345678901
12345678901234567890123456789012123456789012345678901
The hard part is that the GRE is pretty creative in coming up with 2
2
12345678901234567890123456789012123456789012345678901
2
12345678901234567890123456789012123456789012345678901
complicated
tables
and
charts
and
graphs,
which
makes
understanding
2
12345678901234567890123456789012123456789012345678901
2345678901234567890123456789012123456789012345678901
2
12345678901234567890123456789012123456789012345678901
and using them a challenge.
2
12345678901234567890123456789012123456789012345678901
12345678901234567890123456789012123456789012345678901 2
12345678901234567890123456789012123456789012345678901 2
2
12345678901234567890123456789012123456789012345678901
Usually you will see one or two sets of graphs and tables on the GRE math 2
12345678901234567890123456789012123456789012345678901
1 section. Each set will most likely have multiple parts, like a circle graph 2
2
12345678901234567890123456789012123456789012345678901
2
12345678901234567890123456789012123456789012345678901
combined
with
a
table
or
a
diagram
coupled
with
a
bar
graph.
Each
set
2
12345678901234567890123456789012123456789012345678901
2345678901234567890123456789012123456789012345678901
2
12345678901234567890123456789012123456789012345678901
will have 24 questions that refer to it.
2
12345678901234567890123456789012123456789012345678901
12345678901234567890123456789012123456789012345678901 2
12345678901234567890123456789012123456789012345678901 2
2
12345678901234567890123456789012123456789012345678901
The first thing to do when you come upon a set of graphs and charts is read 2
12345678901234567890123456789012123456789012345678901
12345678901234567890123456789012123456789012345678901
the title. It sounds simplistic, but do it. The simplest steps can make the 2
2
12345678901234567890123456789012123456789012345678901
2
12345678901234567890123456789012123456789012345678901
great
difference,
so:
2
1
2345678901234567890123456789012123456789012345678901
12345678901234567890123456789012123456789012345678901 2
2
12345678901234567890123456789012123456789012345678901
1. Read the title.
2
1
12345678901234567890123456789012123456789012345678901 2
2
12345678901234567890123456789012123456789012345678901
2. Read whatever key there is.
2
12345678901234567890123456789012123456789012345678901
2
12345678901234567890123456789012123456789012345678901
3. Read down any graph axis to see what information is being 2
12345678901234567890123456789012123456789012345678901
2
12345678901234567890123456789012123456789012345678901
presented.
2
12345678901234567890123456789012123456789012345678901
2345678901234567890123456789012123456789012345678901
12345678901234567890123456789012123456789012345678901 2
12345678901234567890123456789012123456789012345678901
4. If there are two graphs or charts, look to see how they relate to 2
2
1
2
12345678901234567890123456789012123456789012345678901
each
other.
12345678901234567890123456789012123456789012345678901 2
2345678901234567890123456789012123456789012345678901
12345678901234567890123456789012123456789012345678901 2
12345678901234567890123456789012123456789012345678901
Basically, spend 20 to 30 seconds getting familiar with the layout of 2
2
12345678901234567890123456789012123456789012345678901
2
12345678901234567890123456789012123456789012345678901
everything,
then
proceed
to
the
first
question.
This
will
not
be
a
waste
of
2
1
2345678901234567890123456789012123456789012345678901
12345678901234567890123456789012123456789012345678901
time, as you will be able to answer each of the following questions more 2
2
12345678901234567890123456789012123456789012345678901
12345678901234567890123456789012123456789012345678901
quickly. These kinds of questions try to trap you with an overload of 2
12345678901234567890123456789012123456789012345678901 2
1 information you have to wade through to get to the answer. If you dont 2
2
12345678901234567890123456789012123456789012345678901
12345678901234567890123456789012123456789012345678901
have a good sense of what you are wading through, you will probably get 2
2
12345678901234567890123456789012123456789012345678901
12345678901234567890123456789012123456789012345678901
bogged down and frustrated. That is why getting a sense of the 2
12345678901234567890123456789012123456789012345678901 2
2
information being presented is task one.
12345678901234567890123456789012123456789012345678901
12345678901234567890123456789012123456789012345678901 2
2
12345678901234567890123456789012123456789012345678901
12345678901234567890123456789012123456789012345678901 2
2
12345678901234567890123456789012123456789012345678901
12345678901234567890123456789012123456789012345678901 2
12345678901234567890123456789012123456789012345678901 2
12345678901234567890123456789012123456789012345678901 2
2
102 12345678901234567890123456789012123456789012345678901
123456789012345678901234567890121234567890123456789012

Tip

Chapter 4: Quantitative Review

123456789012345678901234567890121234567890123456789012
12345678901234567890123456789012123456789012345678901 2
2
12345678901234567890123456789012123456789012345678901
12345678901234567890123456789012123456789012345678901 2
2
12345678901234567890123456789012123456789012345678901
2
12345678901234567890123456789012123456789012345678901
12345678901234567890123456789012123456789012345678901 2
2
12345678901234567890123456789012123456789012345678901
12345678901234567890123456789012123456789012345678901 2
2
12345678901234567890123456789012123456789012345678901
2
12345678901234567890123456789012123456789012345678901
2
12345678901234567890123456789012123456789012345678901
2
12345678901234567890123456789012123456789012345678901
2345678901234567890123456789012123456789012345678901
2
1
2
12345678901234567890123456789012123456789012345678901
2
12345678901234567890123456789012123456789012345678901
2
12345678901234567890123456789012123456789012345678901
2
12345678901234567890123456789012123456789012345678901
2345678901234567890123456789012123456789012345678901
2
1
2
12345678901234567890123456789012123456789012345678901
2
12345678901234567890123456789012123456789012345678901
2
12345678901234567890123456789012123456789012345678901
2
12345678901234567890123456789012123456789012345678901
2345678901234567890123456789012123456789012345678901
2
1
2
12345678901234567890123456789012123456789012345678901
2
12345678901234567890123456789012123456789012345678901
2
12345678901234567890123456789012123456789012345678901
2
12345678901234567890123456789012123456789012345678901
2345678901234567890123456789012123456789012345678901
2
12345678901234567890123456789012123456789012345678901
For which year was the ratio of exports to imports the greatest?
2
12345678901234567890123456789012123456789012345678901
2
12345678901234567890123456789012123456789012345678901
A. 1978
2
12345678901234567890123456789012123456789012345678901
2
12345678901234567890123456789012123456789012345678901
B.
1982
12345678901234567890123456789012123456789012345678901 22
1
C. 1984
2
12345678901234567890123456789012123456789012345678901
2
12345678901234567890123456789012123456789012345678901
D.
1987
2
12345678901234567890123456789012123456789012345678901
2345678901234567890123456789012123456789012345678901
2
12345678901234567890123456789012123456789012345678901
E. 1980
2
12345678901234567890123456789012123456789012345678901
12345678901234567890123456789012123456789012345678901 22
12345678901234567890123456789012123456789012345678901
First, determine which line is for exports and which is for imports. Once 2
12345678901234567890123456789012123456789012345678901
12345678901234567890123456789012123456789012345678901
you have that straight, you have to pull out the trusty concept of ratios. 2
2
12345678901234567890123456789012123456789012345678901
2
12345678901234567890123456789012123456789012345678901
Remember
that
ratios
can
be
expressed
as
fractions,
and
here
the
exports
12345678901234567890123456789012123456789012345678901 22
1 will be the numerator and the imports the denominator. To get a big ratio,
2
12345678901234567890123456789012123456789012345678901
12345678901234567890123456789012123456789012345678901
you are looking for years when the exports were high and the imports were 2
12345678901234567890123456789012123456789012345678901 2
12345678901234567890123456789012123456789012345678901
low. Eyeballing the graph, which years are candidates? In 1984 and 1987 2
2
12345678901234567890123456789012123456789012345678901
2345678901234567890123456789012123456789012345678901
12345678901234567890123456789012123456789012345678901
the export-import lines are farthest apart, so these are the best candidates, 2
2
12345678901234567890123456789012123456789012345678901
1 and they are both answer choices. In 1984 the ratio was approximately 2
2
12345678901234567890123456789012123456789012345678901
15.5
14
2
12345678901234567890123456789012123456789012345678901
.
3,
and
in
1987
the
ratio
was
approximately
,
3.
Choice
(C),
2345678901234567890123456789012123456789012345678901
2
12345678901234567890123456789012123456789012345678901
4
5.5
2
12345678901234567890123456789012123456789012345678901
2
1 1984, is the correct answer.
12345678901234567890123456789012123456789012345678901 2
12345678901234567890123456789012123456789012345678901 2
2
12345678901234567890123456789012123456789012345678901
2
12345678901234567890123456789012123456789012345678901
2
12345678901234567890123456789012123456789012345678901
This
question
shows
how
basic
concepts
like
ratios
are
used
in
other
areas
2
12345678901234567890123456789012123456789012345678901
2
12345678901234567890123456789012123456789012345678901
to
make
questions
more
difficult.
2345678901234567890123456789012123456789012345678901
12345678901234567890123456789012123456789012345678901 22
12345678901234567890123456789012123456789012345678901 2
12345678901234567890123456789012123456789012345678901 2
12345678901234567890123456789012123456789012345678901 2
1
What was the greatest consecutive two-year total of exports?
2
12345678901234567890123456789012123456789012345678901
2
12345678901234567890123456789012123456789012345678901
2
12345678901234567890123456789012123456789012345678901
A.
14
billion
2
12345678901234567890123456789012123456789012345678901
2
12345678901234567890123456789012123456789012345678901
B. 18 billion
2345678901234567890123456789012123456789012345678901
12345678901234567890123456789012123456789012345678901 22
1
C. 24 billion
2
12345678901234567890123456789012123456789012345678901
2
12345678901234567890123456789012123456789012345678901
D. 26 billion
2
12345678901234567890123456789012123456789012345678901
2345678901234567890123456789012123456789012345678901
2
1
E. 28 billion
12345678901234567890123456789012123456789012345678901 2
12345678901234567890123456789012123456789012345678901 2
12345678901234567890123456789012123456789012345678901 2 103
123456789012345678901234567890121234567890123456789012

http://www.xtremepapers.net

Part III: Section Review

123456789012345678901234567890121234567890123456789012
12345678901234567890123456789012123456789012345678901 2
2
12345678901234567890123456789012123456789012345678901
Youre looking for exports, so youre looking at the dotted line. The 2
12345678901234567890123456789012123456789012345678901
2
12345678901234567890123456789012123456789012345678901
greatest consecutive two-year total will be the two consecutive years 2
12345678901234567890123456789012123456789012345678901
2
12345678901234567890123456789012123456789012345678901
whose sum is the greatest. Eyeballing the chart, the sets of years that are 2
12345678901234567890123456789012123456789012345678901
2345678901234567890123456789012123456789012345678901
1 the best candidates will be the ones where the export line spikes upward. 2
2
12345678901234567890123456789012123456789012345678901
2
12345678901234567890123456789012123456789012345678901
You
have
two
good
candidates:
8384
and
8788.
Summing
each,
you
will
2
12345678901234567890123456789012123456789012345678901
2
12345678901234567890123456789012123456789012345678901
find
8384
is
a
little
more
than
26
billion,
and
8788
is
about
76
billion.
2345678901234567890123456789012123456789012345678901
2
1
2
12345678901234567890123456789012123456789012345678901
Therefore,
choice
(D)
is
the
correct
answer.
2
12345678901234567890123456789012123456789012345678901
2
12345678901234567890123456789012123456789012345678901
2
12345678901234567890123456789012123456789012345678901
The
previous
example
involved
one
line
graph.
Things
can
get
a
little
more
2345678901234567890123456789012123456789012345678901
2
1
2
12345678901234567890123456789012123456789012345678901
complicated
when
there
are
multiple
graphs
involved.
Again,
take
a
few
2
12345678901234567890123456789012123456789012345678901
2
12345678901234567890123456789012123456789012345678901
seconds
to
get
familiar
with
the
graphs
involved,
and
especially
take
note
2
12345678901234567890123456789012123456789012345678901
2345678901234567890123456789012123456789012345678901
2
1 of how the graphs are related.
2
12345678901234567890123456789012123456789012345678901
2
12345678901234567890123456789012123456789012345678901
2
12345678901234567890123456789012123456789012345678901
Student
Enrollment
Part-Time
or
2
12345678901234567890123456789012123456789012345678901
Full-Time
Employed
at
College
T:
19761980
2345678901234567890123456789012123456789012345678901
12345678901234567890123456789012123456789012345678901 2
12345678901234567890123456789012123456789012345678901 2
12345678901234567890123456789012123456789012345678901 2
2
12345678901234567890123456789012123456789012345678901
Breakdown of Student
2
12345678901234567890123456789012123456789012345678901
Majors
at
College
T
in
1979
12345678901234567890123456789012123456789012345678901 2
2
1
2
12345678901234567890123456789012123456789012345678901
2
12345678901234567890123456789012123456789012345678901
2
12345678901234567890123456789012123456789012345678901
2345678901234567890123456789012123456789012345678901
12345678901234567890123456789012123456789012345678901 2
12345678901234567890123456789012123456789012345678901 2
12345678901234567890123456789012123456789012345678901 2
12345678901234567890123456789012123456789012345678901 2
12345678901234567890123456789012123456789012345678901 2
12345678901234567890123456789012123456789012345678901 2
12345678901234567890123456789012123456789012345678901 2
12345678901234567890123456789012123456789012345678901 2
12345678901234567890123456789012123456789012345678901 2
2
1
2
12345678901234567890123456789012123456789012345678901
2
12345678901234567890123456789012123456789012345678901
12345678901234567890123456789012123456789012345678901 2
12345678901234567890123456789012123456789012345678901 2
2
12345678901234567890123456789012123456789012345678901
2
12345678901234567890123456789012123456789012345678901
2
12345678901234567890123456789012123456789012345678901
In what years did the percentage of part-time employed students at
12345678901234567890123456789012123456789012345678901 2
2
12345678901234567890123456789012123456789012345678901
college T increase most dramatically?
2
12345678901234567890123456789012123456789012345678901
2345678901234567890123456789012123456789012345678901
2
12345678901234567890123456789012123456789012345678901
A. 19761977
2
12345678901234567890123456789012123456789012345678901
2
1
B. 19771978
2
12345678901234567890123456789012123456789012345678901
2
12345678901234567890123456789012123456789012345678901
C.
19781979
2345678901234567890123456789012123456789012345678901
12345678901234567890123456789012123456789012345678901 2
2
12345678901234567890123456789012123456789012345678901
D. 19791980
2
12345678901234567890123456789012123456789012345678901
2
12345678901234567890123456789012123456789012345678901
E.
19771979
2
1
2345678901234567890123456789012123456789012345678901
12345678901234567890123456789012123456789012345678901 2
12345678901234567890123456789012123456789012345678901
It might take a moment to determine whether the bar graph or the pie 2
2
12345678901234567890123456789012123456789012345678901
2
12345678901234567890123456789012123456789012345678901
chart
will
provide
the
answer.
Since
part-time
students
are
listed
on
the
bar
2
1
2345678901234567890123456789012123456789012345678901
12345678901234567890123456789012123456789012345678901
graph, thats where you should look. Specifically, you are looking for two 2
2
12345678901234567890123456789012123456789012345678901
2
12345678901234567890123456789012123456789012345678901
years
where
the
striped
lines
had
the
greatest
percentage
increase.
The
12345678901234567890123456789012123456789012345678901 2
1 actual numbers wont be as importantindeed, they will be wrong 2
2
12345678901234567890123456789012123456789012345678901
2
12345678901234567890123456789012123456789012345678901
answersas the percentages you derive from them.
2
12345678901234567890123456789012123456789012345678901
2
12345678901234567890123456789012123456789012345678901
1978 to 1979 is out because it is a decrease. If time was running out, you 2
12345678901234567890123456789012123456789012345678901
2345678901234567890123456789012123456789012345678901
1 could eliminate this choice and take a guess. Since the question concerns 2
12345678901234567890123456789012123456789012345678901 2
12345678901234567890123456789012123456789012345678901 2
2
104 12345678901234567890123456789012123456789012345678901
123456789012345678901234567890121234567890123456789012

www.petersons.com

http://www.xtremepapers.net

Note

Chapter 4: Quantitative Review

123456789012345678901234567890121234567890123456789012
12345678901234567890123456789012123456789012345678901 2
2
12345678901234567890123456789012123456789012345678901
percentage change, take a close look at 1976 to 1977 because 1976 is 2
12345678901234567890123456789012123456789012345678901
2
12345678901234567890123456789012123456789012345678901
small in comparison to the other years. 100 to 150 is a 50% increase. How 2
12345678901234567890123456789012123456789012345678901
2
12345678901234567890123456789012123456789012345678901
about the other years? 1977 to 1978 have the raw numbers 150 to 250, 2
12345678901234567890123456789012123456789012345678901
2345678901234567890123456789012123456789012345678901
1 which is a 66% increase (100 is 66% or two thirds of 150). 1979 to 1980 2
2
12345678901234567890123456789012123456789012345678901
2
12345678901234567890123456789012123456789012345678901
starts
at
200
and
goes
to
350,
which
is
a
75%
increase
(150
is
75%
of
2
12345678901234567890123456789012123456789012345678901
2
12345678901234567890123456789012123456789012345678901
200).
This
is
the
biggest
increase,
so
(D)
is
the
correct
answer.
2345678901234567890123456789012123456789012345678901
2
1
2
12345678901234567890123456789012123456789012345678901
2
12345678901234567890123456789012123456789012345678901
2
12345678901234567890123456789012123456789012345678901
Percentage
increases
are
greater
when
the
starting
numbers
are
very
small,
2
12345678901234567890123456789012123456789012345678901
2345678901234567890123456789012123456789012345678901
2
1 which is why a company like General Electric (GE) might improve its
2
12345678901234567890123456789012123456789012345678901
2
12345678901234567890123456789012123456789012345678901
profits
by
30
billion
dollars
and
only
have
a
1%
increase.
The
fact
is
that
2
12345678901234567890123456789012123456789012345678901
2
12345678901234567890123456789012123456789012345678901
GE
was
already
making
so
much
cash,
the
additional
amount
is
a
small
2345678901234567890123456789012123456789012345678901
2
1
2
12345678901234567890123456789012123456789012345678901
percentage
increase.
In
contrast,
if
a
kid
sets
up
a
lemonade
stand
and
2
12345678901234567890123456789012123456789012345678901
2
12345678901234567890123456789012123456789012345678901
makes
$1
on
Saturday
and
$3
on
Sunday,
his/her
profits
increased
by
2
12345678901234567890123456789012123456789012345678901
2345678901234567890123456789012123456789012345678901
2
12345678901234567890123456789012123456789012345678901
300% from one day to the next.
2
12345678901234567890123456789012123456789012345678901
12345678901234567890123456789012123456789012345678901 22
12345678901234567890123456789012123456789012345678901 2
12345678901234567890123456789012123456789012345678901
Which of the following represents the greatest number of students?
2
12345678901234567890123456789012123456789012345678901
2
1
2
12345678901234567890123456789012123456789012345678901
A. Part-time employed in 1976
2
12345678901234567890123456789012123456789012345678901
2
12345678901234567890123456789012123456789012345678901
B. Full-time employed in 1976
2345678901234567890123456789012123456789012345678901
12345678901234567890123456789012123456789012345678901 22
12345678901234567890123456789012123456789012345678901
C. Humanities majors in 1979
12345678901234567890123456789012123456789012345678901 22
D. Engineering majors in 1979
12345678901234567890123456789012123456789012345678901
2
12345678901234567890123456789012123456789012345678901
E. Full-time employed in 1978
2
12345678901234567890123456789012123456789012345678901
12345678901234567890123456789012123456789012345678901 22
12345678901234567890123456789012123456789012345678901
This question involves the bar graph and the pie chart. Look down the left 2
12345678901234567890123456789012123456789012345678901
1 side of the bar graph to get a sense of the numbers that the graph is 2
2
12345678901234567890123456789012123456789012345678901
2
12345678901234567890123456789012123456789012345678901
displaying.
Then,
try
to
get
the
same
sense
with
the
pie
chart.
12345678901234567890123456789012123456789012345678901 2
12345678901234567890123456789012123456789012345678901 2
12345678901234567890123456789012123456789012345678901
Now attack the answer choices. You can eliminate (A) because all the 2
2
12345678901234567890123456789012123456789012345678901
2
12345678901234567890123456789012123456789012345678901
full-time
numbers
are
greater
than
all
the
part-time
numbers.
Reading
(B)
12345678901234567890123456789012123456789012345678901 2
12345678901234567890123456789012123456789012345678901
off the bar graph gets you 300. Looking at choices (C) and (D), you can see 2
2
12345678901234567890123456789012123456789012345678901
2345678901234567890123456789012123456789012345678901
12345678901234567890123456789012123456789012345678901
that the percentage of Humanities (23%) is less than the percentage of 2
2
12345678901234567890123456789012123456789012345678901
1 Engineering students, at 24%. (D) has to be greater than (C), so you dont 2
12345678901234567890123456789012123456789012345678901 2
12345678901234567890123456789012123456789012345678901
even need to figure out the exact number for it. To calculate the number of 2
2
12345678901234567890123456789012123456789012345678901
Engineering majors in 1979, youll need to establish the total number of 2
12345678901234567890123456789012123456789012345678901
2
12345678901234567890123456789012123456789012345678901
students in that year. That number is the total height of the bars in the bar 2
12345678901234567890123456789012123456789012345678901
2
12345678901234567890123456789012123456789012345678901
graph for 1979: 700 students. For Engineering majors, you must 2
12345678901234567890123456789012123456789012345678901
2
12345678901234567890123456789012123456789012345678901
determine what is 24% of 700. It works out to a little less than 280. 2
12345678901234567890123456789012123456789012345678901
2
12345678901234567890123456789012123456789012345678901
Reading (E) off the bar graph, it is 750. Choice (B) is the answer.
2
12345678901234567890123456789012123456789012345678901
2345678901234567890123456789012123456789012345678901
12345678901234567890123456789012123456789012345678901 22
12345678901234567890123456789012123456789012345678901 2
12345678901234567890123456789012123456789012345678901 2
12345678901234567890123456789012123456789012345678901 2
1
2
12345678901234567890123456789012123456789012345678901
12345678901234567890123456789012123456789012345678901 2
2
12345678901234567890123456789012123456789012345678901
12345678901234567890123456789012123456789012345678901 2
2
12345678901234567890123456789012123456789012345678901
12345678901234567890123456789012123456789012345678901 2
12345678901234567890123456789012123456789012345678901 2
12345678901234567890123456789012123456789012345678901 2
12345678901234567890123456789012123456789012345678901 2 105
123456789012345678901234567890121234567890123456789012

http://www.xtremepapers.net

Take It to
the Next Level
123456789012345678901234567890121234567890123456789012
2
12345678901234567890123456789012123456789012345678901
2
12345678901234567890123456789012123456789012345678901
2345678901234567890123456789012123456789012345678901
2
1
Math:
The Multi-Step Complicated Stuff
2
12345678901234567890123456789012123456789012345678901
2
12345678901234567890123456789012123456789012345678901
The
harder
questions
on
the
GRE
do
not
require
you
to
understand
more
2
12345678901234567890123456789012123456789012345678901
2
12345678901234567890123456789012123456789012345678901
sophisticated
math.
The
math
concepts
required
to
answer
a
500-level
2345678901234567890123456789012123456789012345678901
12345678901234567890123456789012123456789012345678901 2
12345678901234567890123456789012123456789012345678901
problem and a 750-level problem are, for the most part, the same. The 2
2
12345678901234567890123456789012123456789012345678901
2
12345678901234567890123456789012123456789012345678901
difference
is
that
the
750-level
problem
will
challenge
you
to
be
more
12345678901234567890123456789012123456789012345678901 2
2
12345678901234567890123456789012123456789012345678901
1 clever, creative, and inventive in your approach. Showing you how to 2
2
12345678901234567890123456789012123456789012345678901
out-clever the test will be a major focus of this chapter.
2
12345678901234567890123456789012123456789012345678901
2
12345678901234567890123456789012123456789012345678901
2345678901234567890123456789012123456789012345678901
12345678901234567890123456789012123456789012345678901
Before that starts, there are a few math concepts you are probably only 2
2
12345678901234567890123456789012123456789012345678901
going to see on harder questions. These harder concepts are really just 2
12345678901234567890123456789012123456789012345678901
2
12345678901234567890123456789012123456789012345678901
extensions of what weve already covered in this chapter, so its not as if 2
12345678901234567890123456789012123456789012345678901
2
12345678901234567890123456789012123456789012345678901
the easier questions are algebra and the harder ones are calculus. Basically, 2
12345678901234567890123456789012123456789012345678901
2
12345678901234567890123456789012123456789012345678901
the harder questions contain the basics revved up to make problems more 2
12345678901234567890123456789012123456789012345678901
2
1 interesting and difficult.
2
12345678901234567890123456789012123456789012345678901
2
12345678901234567890123456789012123456789012345678901
2
12345678901234567890123456789012123456789012345678901
If
a
math
concept
comes
up
that
you
are
not
familiar
with,
or
you
cant
12345678901234567890123456789012123456789012345678901 2
12345678901234567890123456789012123456789012345678901
remember exactly what it is, look back to the beginning of the chapter to 2
2
12345678901234567890123456789012123456789012345678901
2
12345678901234567890123456789012123456789012345678901
see
if
it
is
covered
there.
If
youve
skipped
the
beginning
of
the
chapter,
its
12345678901234567890123456789012123456789012345678901 2
12345678901234567890123456789012123456789012345678901
not a bad idea to leaf through it anyway and make sure that all the math 2
2
12345678901234567890123456789012123456789012345678901
2345678901234567890123456789012123456789012345678901
12345678901234567890123456789012123456789012345678901
content is fresh in your mind. Once thats done, you can Take It to the 2
2
12345678901234567890123456789012123456789012345678901
2
1 Next Level and start honing your clever skills.
12345678901234567890123456789012123456789012345678901 2
12345678901234567890123456789012123456789012345678901 2
2
12345678901234567890123456789012123456789012345678901
2
12345678901234567890123456789012123456789012345678901
2345678901234567890123456789012123456789012345678901
2
1
Advanced
Arithmetic
2
12345678901234567890123456789012123456789012345678901
12345678901234567890123456789012123456789012345678901 2
2345678901234567890123456789012123456789012345678901
12345678901234567890123456789012123456789012345678901
Hard arithmetic problems will use jargon to make problems more 2
2
12345678901234567890123456789012123456789012345678901
12345678901234567890123456789012123456789012345678901
complex. Problems will include multiple phrases like the sum of the first 2
12345678901234567890123456789012123456789012345678901 2
1 three prime numbers or the product of two consecutive integers. The 2
2
12345678901234567890123456789012123456789012345678901
way to decode such math speak is to attack it piece by piece and phrase by 2
12345678901234567890123456789012123456789012345678901
2
12345678901234567890123456789012123456789012345678901
phrase. Unless you are a math wunderkind, trying to conceptualize whole 2
12345678901234567890123456789012123456789012345678901
2
12345678901234567890123456789012123456789012345678901
sentences of math speak at one time is impossible. At the least, it is 2
12345678901234567890123456789012123456789012345678901
2
12345678901234567890123456789012123456789012345678901
foolhardy.
2
12345678901234567890123456789012123456789012345678901
12345678901234567890123456789012123456789012345678901 2
12345678901234567890123456789012123456789012345678901
Lets decode these two phrases. The first three prime numbers are 2, 3, and 2
2
12345678901234567890123456789012123456789012345678901
2
12345678901234567890123456789012123456789012345678901
5,
and
their
sum
is
10.
In
the
second
phrase,
since
we
dont
know
what
the
12345678901234567890123456789012123456789012345678901 2
12345678901234567890123456789012123456789012345678901 2
123456789012345678901234567890121234567890123456789012
106

http://www.xtremepapers.net

123456789012345678901234567890121234567890123456789012
12345678901234567890123456789012123456789012345678901 2
2
12345678901234567890123456789012123456789012345678901
first integer is, lets call it x. The product of x and the next integer is 2
12345678901234567890123456789012123456789012345678901
2
12345678901234567890123456789012123456789012345678901
(x)(x 1 1). None of that was too difficult, but suppose a problem reads, If 2
12345678901234567890123456789012123456789012345678901
2
12345678901234567890123456789012123456789012345678901
the sum of the first three prime numbers plus 2 equals the product of two 2
12345678901234567890123456789012123456789012345678901
2345678901234567890123456789012123456789012345678901
2
1 consecutive integers, what is the value of the first integer?
2
12345678901234567890123456789012123456789012345678901
2
12345678901234567890123456789012123456789012345678901
2
12345678901234567890123456789012123456789012345678901
2
12345678901234567890123456789012123456789012345678901
2345678901234567890123456789012123456789012345678901
2
1
2
12345678901234567890123456789012123456789012345678901
Using algebra in the form of the variable x to help describe an arithmetic
2
12345678901234567890123456789012123456789012345678901
2
12345678901234567890123456789012123456789012345678901
expression
is
fairly
common
practice.
Dont
hesitate
to
draw
information
2
12345678901234567890123456789012123456789012345678901
2345678901234567890123456789012123456789012345678901
2
1 from any field of math if you believe it will help you solve a problem.
2
12345678901234567890123456789012123456789012345678901
2
12345678901234567890123456789012123456789012345678901
2
12345678901234567890123456789012123456789012345678901
2
12345678901234567890123456789012123456789012345678901
2345678901234567890123456789012123456789012345678901
2
1
2
12345678901234567890123456789012123456789012345678901
If
you
decode
this
jargon
piece
by
piece,
it
is
manageable.
The
sum
of
the
2
12345678901234567890123456789012123456789012345678901
2
12345678901234567890123456789012123456789012345678901
first
three
prime
numbers
plus
2
is
12,
and
you
already
know
what
the
2
12345678901234567890123456789012123456789012345678901
2345678901234567890123456789012123456789012345678901
12345678901234567890123456789012123456789012345678901
product of two consecutive integers is. Therefore, you can set up the 2
2
12345678901234567890123456789012123456789012345678901
2
12345678901234567890123456789012123456789012345678901
equation:
12345678901234567890123456789012123456789012345678901 22
12345678901234567890123456789012123456789012345678901 2
12345678901234567890123456789012123456789012345678901
~x!~x 1 1! 5 12
2
1
2
2
12345678901234567890123456789012123456789012345678901
x
1
x
5
12

2
12345678901234567890123456789012123456789012345678901
2
12345678901234567890123456789012123456789012345678901
2
x
1
x
2
12
5
12
2
12

2345678901234567890123456789012123456789012345678901
12345678901234567890123456789012123456789012345678901 22
12345678901234567890123456789012123456789012345678901
2
12345678901234567890123456789012123456789012345678901
x2 1 x 2 12 5 0
2
12345678901234567890123456789012123456789012345678901
2
12345678901234567890123456789012123456789012345678901
x
1
4
x
2
3
5
0

~
!~
!
12345678901234567890123456789012123456789012345678901 22
12345678901234567890123456789012123456789012345678901
x 5 24,3
2
12345678901234567890123456789012123456789012345678901
12345678901234567890123456789012123456789012345678901 22
1 Theres a bit of everything in the above problem, and thats not unusual
2
12345678901234567890123456789012123456789012345678901
2
12345678901234567890123456789012123456789012345678901
for
hard
problems.
You
had
to
use
arithmetic
to
decipher
the
problem
and
12345678901234567890123456789012123456789012345678901 2
12345678901234567890123456789012123456789012345678901
algebra to set up an equation. Then you had to do some factoring to split 2
2
12345678901234567890123456789012123456789012345678901
2345678901234567890123456789012123456789012345678901
12345678901234567890123456789012123456789012345678901
the equation x2 1 x 2 12 5 0 in order to find the values for x. If you 2
2
12345678901234567890123456789012123456789012345678901
2
1 handle each step, though, the answer falls into your lap.
2
12345678901234567890123456789012123456789012345678901
2
12345678901234567890123456789012123456789012345678901
2345678901234567890123456789012123456789012345678901
2
12345678901234567890123456789012123456789012345678901
Column A
Column B
2
12345678901234567890123456789012123456789012345678901
2
1
2
12345678901234567890123456789012123456789012345678901
a c e
2
12345678901234567890123456789012123456789012345678901
1
1
.
.
.
2345678901234567890123456789012123456789012345678901
2
12345678901234567890123456789012123456789012345678901
b d f
2
12345678901234567890123456789012123456789012345678901
2
12345678901234567890123456789012123456789012345678901
is
a
sum
in
which
all
the
variables
are
12345678901234567890123456789012123456789012345678901 22
1
positive consecutive integers greater than 1
2
12345678901234567890123456789012123456789012345678901
2
12345678901234567890123456789012123456789012345678901
that increase in alphabetical order.
2
12345678901234567890123456789012123456789012345678901
2
12345678901234567890123456789012123456789012345678901
The third term in
2
12345678901234567890123456789012123456789012345678901
a
2
2
2345678901234567890123456789012123456789012345678901
2
12345678901234567890123456789012123456789012345678901
the sum
2
12345678901234567890123456789012123456789012345678901
a
2
1
12345678901234567890123456789012123456789012345678901 22
12345678901234567890123456789012123456789012345678901 2
1
2
12345678901234567890123456789012123456789012345678901
12345678901234567890123456789012123456789012345678901
First, you must decode. Positive consecutive integers are things like 4, 5, 2
2
12345678901234567890123456789012123456789012345678901
and 6, and these increase in alphabetical order, which means that b 5 a 1 1 2
12345678901234567890123456789012123456789012345678901
2
12345678901234567890123456789012123456789012345678901
12345678901234567890123456789012123456789012345678901 2
12345678901234567890123456789012123456789012345678901 2
12345678901234567890123456789012123456789012345678901 2
12345678901234567890123456789012123456789012345678901 2 107
123456789012345678901234567890121234567890123456789012

Take It to the Next Level

Alert!

Chapter 4: Quantitative Review

http://www.xtremepapers.net

Part III: Section Review

123456789012345678901234567890121234567890123456789012
12345678901234567890123456789012123456789012345678901 2
2
12345678901234567890123456789012123456789012345678901
and c 5 b 1 1. Notice that since b 5 a 1 1, then c 5 (a 1 1) 1 1 5 a 1 2. 2
12345678901234567890123456789012123456789012345678901
2
12345678901234567890123456789012123456789012345678901
This means that the sum can be rewritten
2
12345678901234567890123456789012123456789012345678901
12345678901234567890123456789012123456789012345678901 2
2
12345678901234567890123456789012123456789012345678901
a
a12 a14
2
12345678901234567890123456789012123456789012345678901
1
1
....
2
12345678901234567890123456789012123456789012345678901
a11 a13 a15
2
12345678901234567890123456789012123456789012345678901
2
12345678901234567890123456789012123456789012345678901
2
12345678901234567890123456789012123456789012345678901
a
1
4
2345678901234567890123456789012123456789012345678901
2
1 This makes the third term
. Thats Column A, so far.
2
12345678901234567890123456789012123456789012345678901
a15
2
12345678901234567890123456789012123456789012345678901
2
12345678901234567890123456789012123456789012345678901
2
12345678901234567890123456789012123456789012345678901
Compare
this
to
the
fraction
in
Column
B.
The
smallest
value
that
a
can
2345678901234567890123456789012123456789012345678901
2
1
2
12345678901234567890123456789012123456789012345678901
have
is
2,
so
it
will
be
interesting
to
start
with
this
number
and
see
what
2
12345678901234567890123456789012123456789012345678901
2
12345678901234567890123456789012123456789012345678901
6
2
12345678901234567890123456789012123456789012345678901
,
while
Column
B
is
you
get.
Column
A
becomes
2345678901234567890123456789012123456789012345678901
2
1
7
2
12345678901234567890123456789012123456789012345678901
2
12345678901234567890123456789012123456789012345678901
a
2
2
2
2
2
0
2
12345678901234567890123456789012123456789012345678901
5
5
5
0.
2
12345678901234567890123456789012123456789012345678901
a
2
1
2
2
1
1
2345678901234567890123456789012123456789012345678901
12345678901234567890123456789012123456789012345678901 2
12345678901234567890123456789012123456789012345678901 2
2
12345678901234567890123456789012123456789012345678901
Column A is greater is this instance, but to play it safe, it helps to try 2
12345678901234567890123456789012123456789012345678901
2
12345678901234567890123456789012123456789012345678901
another number. A small number was used the first time, so a large number 2
12345678901234567890123456789012123456789012345678901
2
1
14
2
12345678901234567890123456789012123456789012345678901
in
can
be
plugged
in
this
time
for
variety.
Making
a
5
10,
you
get
2
12345678901234567890123456789012123456789012345678901
15
2
12345678901234567890123456789012123456789012345678901
2345678901234567890123456789012123456789012345678901
2
12345678901234567890123456789012123456789012345678901
8
2
12345678901234567890123456789012123456789012345678901
in
Column
B.
Column
A
is
again
greater,
and
placing
an
Column
A
and
2
12345678901234567890123456789012123456789012345678901
9
2
12345678901234567890123456789012123456789012345678901
even
larger
number
will
probably
not
make
a
difference.
The
correct
12345678901234567890123456789012123456789012345678901 2
2
12345678901234567890123456789012123456789012345678901
answer is (A).
2
12345678901234567890123456789012123456789012345678901
12345678901234567890123456789012123456789012345678901 2
2
12345678901234567890123456789012123456789012345678901
1 Ratios are another arithmetic topic that can be made interesting. Suppose 2
2345678901234567890123456789012123456789012345678901
12345678901234567890123456789012123456789012345678901
you are told the ratio of a to b is 4:7 and the ratio of b to c is 8:7. You are 2
2
12345678901234567890123456789012123456789012345678901
2
1 then asked, What is the ratio of a to c?
12345678901234567890123456789012123456789012345678901 2
2
12345678901234567890123456789012123456789012345678901
12345678901234567890123456789012123456789012345678901
The easiest way to approach such a problem is to make the common 2
2
12345678901234567890123456789012123456789012345678901
12345678901234567890123456789012123456789012345678901
element in both ratios, b, the same so that you can directly compare a and 2
2
12345678901234567890123456789012123456789012345678901
12345678901234567890123456789012123456789012345678901
c. To do that, find a common multiple of 7 and 8, preferably the least 2
2345678901234567890123456789012123456789012345678901
12345678901234567890123456789012123456789012345678901 2
2
12345678901234567890123456789012123456789012345678901
2
1 common multiple. 56 works, since its 7 3 8.
2
12345678901234567890123456789012123456789012345678901
First
ratio:
a
to
b
5
4:7.
Multiplied
by
8,
this
becomes
12345678901234567890123456789012123456789012345678901 2
2345678901234567890123456789012123456789012345678901
12345678901234567890123456789012123456789012345678901 2
2
12345678901234567890123456789012123456789012345678901
a to b 5 32:56
2
12345678901234567890123456789012123456789012345678901
12345678901234567890123456789012123456789012345678901 2
2
1 Second ratio: b to c 5 8:7. Multiplied by 7, this becomes
2
12345678901234567890123456789012123456789012345678901
2
12345678901234567890123456789012123456789012345678901
2
12345678901234567890123456789012123456789012345678901
b to c 5 56:49
2
12345678901234567890123456789012123456789012345678901
2
12345678901234567890123456789012123456789012345678901
For
both
ratios,
b
is
now
56.
This
allows
you
write
to
both
ratios
as
a
2345678901234567890123456789012123456789012345678901
12345678901234567890123456789012123456789012345678901 2
2
12345678901234567890123456789012123456789012345678901
single long ratio:
2
12345678901234567890123456789012123456789012345678901
12345678901234567890123456789012123456789012345678901 2
2
1
a to b to c 5 32:56:49
2
12345678901234567890123456789012123456789012345678901
12345678901234567890123456789012123456789012345678901 2
12345678901234567890123456789012123456789012345678901
You can now see that a : c 5 32 : 49. If these numbers in the ratio had any 2
2
12345678901234567890123456789012123456789012345678901
2
12345678901234567890123456789012123456789012345678901
common
multiples,
then
the
ratio
should
be
simplified.
2345678901234567890123456789012123456789012345678901
2
1
12345678901234567890123456789012123456789012345678901 2
12345678901234567890123456789012123456789012345678901 2
2
108 12345678901234567890123456789012123456789012345678901
123456789012345678901234567890121234567890123456789012

www.petersons.com

http://www.xtremepapers.net

Chapter 4: Quantitative Review

Take It to the Next Level

X-Ref

123456789012345678901234567890121234567890123456789012
12345678901234567890123456789012123456789012345678901 2
2
12345678901234567890123456789012123456789012345678901
12345678901234567890123456789012123456789012345678901 2
2
12345678901234567890123456789012123456789012345678901
2
12345678901234567890123456789012123456789012345678901
2
12345678901234567890123456789012123456789012345678901
Finding
a
common
element
for
these
ratios
is
similar
to
the
practice
of
2
12345678901234567890123456789012123456789012345678901
2345678901234567890123456789012123456789012345678901
2
1 finding a common denominator before adding and subtracting fractions.
2
12345678901234567890123456789012123456789012345678901
2
12345678901234567890123456789012123456789012345678901
See page 54 of this chapter.
2
12345678901234567890123456789012123456789012345678901
2
12345678901234567890123456789012123456789012345678901
2345678901234567890123456789012123456789012345678901
2
1
2
12345678901234567890123456789012123456789012345678901
2
12345678901234567890123456789012123456789012345678901
2
If m and n are integers and (m 1 n) is even, then which of the
2
12345678901234567890123456789012123456789012345678901
2
12345678901234567890123456789012123456789012345678901
following
must
be
true?
2345678901234567890123456789012123456789012345678901
2
1
2
12345678901234567890123456789012123456789012345678901
2
12345678901234567890123456789012123456789012345678901
I.
Both
integers
are
even.
2
12345678901234567890123456789012123456789012345678901
2
12345678901234567890123456789012123456789012345678901
II.
Both
integers
are
odd.
2345678901234567890123456789012123456789012345678901
2
1
2
12345678901234567890123456789012123456789012345678901
III.
One
integer
is
even
and
one
integer
is
odd.
2
12345678901234567890123456789012123456789012345678901
2
12345678901234567890123456789012123456789012345678901
2
12345678901234567890123456789012123456789012345678901
A.
I
only
2345678901234567890123456789012123456789012345678901
12345678901234567890123456789012123456789012345678901 22
12345678901234567890123456789012123456789012345678901
B. II only
2
12345678901234567890123456789012123456789012345678901
2
12345678901234567890123456789012123456789012345678901
C. III only
12345678901234567890123456789012123456789012345678901 22
12345678901234567890123456789012123456789012345678901
D. I and II only
2
1
2
12345678901234567890123456789012123456789012345678901
E. I and III only
2
12345678901234567890123456789012123456789012345678901
2
12345678901234567890123456789012123456789012345678901
To
attack
this
one,
you
have
to
work
through
each
option
to
see
if
it
must
2345678901234567890123456789012123456789012345678901
12345678901234567890123456789012123456789012345678901 22
12345678901234567890123456789012123456789012345678901
be true. Using FOIL, you know that if (m 1 n)2 is even, then m2 1 mn 1 n2 2
12345678901234567890123456789012123456789012345678901
2
12345678901234567890123456789012123456789012345678901
is also even.
2
12345678901234567890123456789012123456789012345678901
2
12345678901234567890123456789012123456789012345678901
2
2
2
12345678901234567890123456789012123456789012345678901
I.
must
be
true
because
m
1
2mn
1
n

even
1
even
1
even
5
even.
12345678901234567890123456789012123456789012345678901 22
12345678901234567890123456789012123456789012345678901
That eliminates choices (B) and (C). If youre pressed for time, you 2
1
2
12345678901234567890123456789012123456789012345678901
could guess now with one in three odds of guessing right.
2
12345678901234567890123456789012123456789012345678901
2
12345678901234567890123456789012123456789012345678901
2
2
12345678901234567890123456789012123456789012345678901
II. can be written as: m 1 2mn 1 n odd 1 even 1 odd 5 even 1 2
2
12345678901234567890123456789012123456789012345678901
2
12345678901234567890123456789012123456789012345678901
even 5 even. This does work, so (B) is out.
2
12345678901234567890123456789012123456789012345678901
2
12345678901234567890123456789012123456789012345678901
2
2
III. can be written as: (m 1 2mn 1 n even 1 even 1 odd 5 even 1 2
12345678901234567890123456789012123456789012345678901
2
12345678901234567890123456789012123456789012345678901
odd 5 odd. This doesnt work, so (D) is the correct answer.
2
12345678901234567890123456789012123456789012345678901
2
12345678901234567890123456789012123456789012345678901
2
12345678901234567890123456789012123456789012345678901
This
approach
uses
basic
facts
about
how
even
and
odd
numbers
12345678901234567890123456789012123456789012345678901 2
12345678901234567890123456789012123456789012345678901
interact when multiplied and added together. If this approach is too 2
2
12345678901234567890123456789012123456789012345678901
2
12345678901234567890123456789012123456789012345678901
abstract
for
you,
just
pick
two
numbers
for
m
and
n
according
to
each
2
12345678901234567890123456789012123456789012345678901
2
12345678901234567890123456789012123456789012345678901
scenario
and
work
the
problem
through
using
those
numbers.
If
you
12345678901234567890123456789012123456789012345678901 2
2345678901234567890123456789012123456789012345678901
12345678901234567890123456789012123456789012345678901
dont make a careless math error, you should get the same answer, (D). 2
2
12345678901234567890123456789012123456789012345678901
12345678901234567890123456789012123456789012345678901 22
12345678901234567890123456789012123456789012345678901 2
1
2
12345678901234567890123456789012123456789012345678901
2
Harder
Algebra
12345678901234567890123456789012123456789012345678901
2
12345678901234567890123456789012123456789012345678901
2
12345678901234567890123456789012123456789012345678901
There
are
three
primary
ways
the
GRE
will
increase
the
difficulty
on
12345678901234567890123456789012123456789012345678901 2
2345678901234567890123456789012123456789012345678901
2
12345678901234567890123456789012123456789012345678901
1 algebra problems. The first relies on shock value. On this track, you are 2
12345678901234567890123456789012123456789012345678901
presented with one or more monstrous-looking algebraic expressions that 2
2
12345678901234567890123456789012123456789012345678901
12345678901234567890123456789012123456789012345678901
are supposed to shock you into some form of problem-solving paralysis. If 2
2345678901234567890123456789012123456789012345678901
2
1
12345678901234567890123456789012123456789012345678901
you overcome this initial shock, keep in mind that almost every ugly 2
12345678901234567890123456789012123456789012345678901 2
12345678901234567890123456789012123456789012345678901 2 109
123456789012345678901234567890121234567890123456789012

http://www.xtremepapers.net

Part III: Section Review

123456789012345678901234567890121234567890123456789012
12345678901234567890123456789012123456789012345678901 2
2
12345678901234567890123456789012123456789012345678901
algebraic expression will have a way to simplify it quickly. These problems 2
12345678901234567890123456789012123456789012345678901
2
12345678901234567890123456789012123456789012345678901
are designed to shock, but they almost always can be simplified quickly.
2
12345678901234567890123456789012123456789012345678901
2
12345678901234567890123456789012123456789012345678901
2
2
12345678901234567890123456789012123456789012345678901
1
14x
1
6
4x
2345678901234567890123456789012123456789012345678901
1 If you see an expression like
, you should immediately 2
2
12345678901234567890123456789012123456789012345678901
4x 1 2
2
12345678901234567890123456789012123456789012345678901
2
12345678901234567890123456789012123456789012345678901
think,
There
must
be
a
quick
way
to
simplify
it.
With
algebra
in
2
12345678901234567890123456789012123456789012345678901
2345678901234567890123456789012123456789012345678901
1 fractions, always be on the lookout for canceling terms that are in both the 2
2
12345678901234567890123456789012123456789012345678901
12345678901234567890123456789012123456789012345678901
numerator and the denominator. Since the denominator is simpler, start 2
2
12345678901234567890123456789012123456789012345678901
12345678901234567890123456789012123456789012345678901
there: 4x 1 2 5 2(2x 1 1). Now look for a (2x 1 1) in the numerator. Seek 2
2345678901234567890123456789012123456789012345678901
2
1
2
12345678901234567890123456789012123456789012345678901
and ye shall find:
2
12345678901234567890123456789012123456789012345678901
2
12345678901234567890123456789012123456789012345678901
2
2
2
12345678901234567890123456789012123456789012345678901
4x 1 14x 1 6 5 2~2x 1 7x 1 3!
2345678901234567890123456789012123456789012345678901
2
1
2
12345678901234567890123456789012123456789012345678901
5
2
2x
1
1
x
1
3
~
!~
!
2
12345678901234567890123456789012123456789012345678901
2
12345678901234567890123456789012123456789012345678901
2
12345678901234567890123456789012123456789012345678901
Therefore,
2345678901234567890123456789012123456789012345678901
12345678901234567890123456789012123456789012345678901 2
2
12345678901234567890123456789012123456789012345678901
2
2
12345678901234567890123456789012123456789012345678901
1
14x
1
6
4x
2(2x
1
1)(x
1
3)
2
12345678901234567890123456789012123456789012345678901
5
5
x
1
3
2
12345678901234567890123456789012123456789012345678901
4x
1
2
2(2x
1
1)
12345678901234567890123456789012123456789012345678901 2
2
1
2
12345678901234567890123456789012123456789012345678901
That whole mess boils down to a simple x 1 3.
2
12345678901234567890123456789012123456789012345678901
2
12345678901234567890123456789012123456789012345678901
For
an
idea
about
where
to
start
simplifying,
think
of
these
guidelines:
2345678901234567890123456789012123456789012345678901
12345678901234567890123456789012123456789012345678901 2
12345678901234567890123456789012123456789012345678901 2
2
12345678901234567890123456789012123456789012345678901
1. Look at the numerator and denominator and start with which- 2
12345678901234567890123456789012123456789012345678901
2
12345678901234567890123456789012123456789012345678901
ever one looks simpler.
2
12345678901234567890123456789012123456789012345678901
12345678901234567890123456789012123456789012345678901 2
12345678901234567890123456789012123456789012345678901
2. Once you factor the simpler term, look for those factors in the 2
2
12345678901234567890123456789012123456789012345678901
2
1
harder
term.
They
should
be
there
in
some
form
or
another.
2345678901234567890123456789012123456789012345678901
12345678901234567890123456789012123456789012345678901 2
12345678901234567890123456789012123456789012345678901 2
1
3. Cancel out any similar terms you can, and enjoy your new 2
2
12345678901234567890123456789012123456789012345678901
2
12345678901234567890123456789012123456789012345678901
simplified equation.
2345678901234567890123456789012123456789012345678901
12345678901234567890123456789012123456789012345678901 2
2
12345678901234567890123456789012123456789012345678901
1 Expecting cleverness is the second way that the GRE will try to make 2
2
12345678901234567890123456789012123456789012345678901
algebra difficult. A problem will expect you to make a clever algebra 2
12345678901234567890123456789012123456789012345678901
2345678901234567890123456789012123456789012345678901
2
12345678901234567890123456789012123456789012345678901
maneuver to finish the problem. You are especially liable to see this on 2
12345678901234567890123456789012123456789012345678901
2
1 QCs. The following problem illustrates this idea.
12345678901234567890123456789012123456789012345678901 2
12345678901234567890123456789012123456789012345678901 2
2
12345678901234567890123456789012123456789012345678901
Column B
Column A
2
12345678901234567890123456789012123456789012345678901
2
12345678901234567890123456789012123456789012345678901
2
12345678901234567890123456789012123456789012345678901
a
and
b
are
positive
constants
12345678901234567890123456789012123456789012345678901 2
2345678901234567890123456789012123456789012345678901
2
12345678901234567890123456789012123456789012345678901
and 0 , x , y.
2
12345678901234567890123456789012123456789012345678901
2
12345678901234567890123456789012123456789012345678901
2
2
12345678901234567890123456789012123456789012345678901
(x1y)
x
2
1
2345678901234567890123456789012123456789012345678901
2
12345678901234567890123456789012123456789012345678901
a
ab
2
12345678901234567890123456789012123456789012345678901
2
12345678901234567890123456789012123456789012345678901
b
12345678901234567890123456789012123456789012345678901 2
2
1
y
2
12345678901234567890123456789012123456789012345678901
12345678901234567890123456789012123456789012345678901 2
2
12345678901234567890123456789012123456789012345678901
12345678901234567890123456789012123456789012345678901 2
2
12345678901234567890123456789012123456789012345678901
12345678901234567890123456789012123456789012345678901 2
12345678901234567890123456789012123456789012345678901 2
12345678901234567890123456789012123456789012345678901 2
2
110 12345678901234567890123456789012123456789012345678901
123456789012345678901234567890121234567890123456789012

www.petersons.com

http://www.xtremepapers.net

Chapter 4: Quantitative Review

123456789012345678901234567890121234567890123456789012
12345678901234567890123456789012123456789012345678901 2
2
12345678901234567890123456789012123456789012345678901
First simplify Column A. To do this, you have to divide each fraction 2
12345678901234567890123456789012123456789012345678901
2
12345678901234567890123456789012123456789012345678901
before multiplying.
2
12345678901234567890123456789012123456789012345678901
12345678901234567890123456789012123456789012345678901 2
2
12345678901234567890123456789012123456789012345678901
2
12345678901234567890123456789012123456789012345678901
x y
2
12345678901234567890123456789012123456789012345678901
2
12345678901234567890123456789012123456789012345678901
a b
x b
y y
2
12345678901234567890123456789012123456789012345678901
3
3
5
4
4
2
12345678901234567890123456789012123456789012345678901
a
y
b
b
b
y
2345678901234567890123456789012123456789012345678901
2
1
2
12345678901234567890123456789012123456789012345678901
y
b
2
12345678901234567890123456789012123456789012345678901
2
12345678901234567890123456789012123456789012345678901
x
y
y
b
2
12345678901234567890123456789012123456789012345678901
5
3
3
3
2345678901234567890123456789012123456789012345678901
2
1
a b
b y
2
12345678901234567890123456789012123456789012345678901
2
12345678901234567890123456789012123456789012345678901
2
12345678901234567890123456789012123456789012345678901
xy
2
12345678901234567890123456789012123456789012345678901
5
2345678901234567890123456789012123456789012345678901
2
1
ab
2
12345678901234567890123456789012123456789012345678901
2
12345678901234567890123456789012123456789012345678901
12345678901234567890123456789012123456789012345678901
As you can see, the second term cancels itself out, leaving only the single 2
2
12345678901234567890123456789012123456789012345678901
xy
2
12345678901234567890123456789012123456789012345678901
2
12345678901234567890123456789012123456789012345678901
fraction . Column B can be rewritten as:
2
12345678901234567890123456789012123456789012345678901
ab
2
12345678901234567890123456789012123456789012345678901
2
12345678901234567890123456789012123456789012345678901
2
2
2
x
1
y
1
2xy
1
y
x
~
!
2345678901234567890123456789012123456789012345678901
2
12345678901234567890123456789012123456789012345678901
5
2
1
ab
ab
2
12345678901234567890123456789012123456789012345678901
2
12345678901234567890123456789012123456789012345678901
2
12345678901234567890123456789012123456789012345678901
You
can
disregard
the
denominators
since
they
are
the
same
in
each
2345678901234567890123456789012123456789012345678901
12345678901234567890123456789012123456789012345678901 22
12345678901234567890123456789012123456789012345678901
column. Comparing the numerators, you can see a 2xy in Column B 2
12345678901234567890123456789012123456789012345678901
12345678901234567890123456789012123456789012345678901
(among other values) compared to a single xy in Column A. Since the 2
2
12345678901234567890123456789012123456789012345678901
12345678901234567890123456789012123456789012345678901
statement says that both integers are positive, it is clear that (B) is greater. 2
2
12345678901234567890123456789012123456789012345678901
12345678901234567890123456789012123456789012345678901 22
12345678901234567890123456789012123456789012345678901
1 It didnt take earthshaking genius to solve the last problem. Instead, all 2
12345678901234567890123456789012123456789012345678901
you needed was confidence that the two columns could be readily 2
2
12345678901234567890123456789012123456789012345678901
12345678901234567890123456789012123456789012345678901
compared and the ability to jump in and start performing mathematical 2
12345678901234567890123456789012123456789012345678901 2
2
12345678901234567890123456789012123456789012345678901
operations to help compare the two.
2
12345678901234567890123456789012123456789012345678901
2
12345678901234567890123456789012123456789012345678901
Embedding algebra in a coordinate plane is the third way that the GRE 2
12345678901234567890123456789012123456789012345678901
12345678901234567890123456789012123456789012345678901
will make algebra more challenging. You might encounter a polygon 2
2
12345678901234567890123456789012123456789012345678901
2345678901234567890123456789012123456789012345678901
12345678901234567890123456789012123456789012345678901
whose side lengths are given in terms of algebraic expressions, as opposed 2
2
12345678901234567890123456789012123456789012345678901
1 to simply numbers or constants. The key to solving such problems is to not 2
12345678901234567890123456789012123456789012345678901 2
12345678901234567890123456789012123456789012345678901
be unnerved by the combination of geometry and algebra. Normally you 2
2
12345678901234567890123456789012123456789012345678901
2
12345678901234567890123456789012123456789012345678901
will
need
to
start
with
the
geometry
and
see
what
you
can
infer
from
it,
and
2
12345678901234567890123456789012123456789012345678901
2
12345678901234567890123456789012123456789012345678901
then
examine
the
algebra.
12345678901234567890123456789012123456789012345678901 2
2345678901234567890123456789012123456789012345678901
12345678901234567890123456789012123456789012345678901 22
12345678901234567890123456789012123456789012345678901 2
12345678901234567890123456789012123456789012345678901 2
12345678901234567890123456789012123456789012345678901 2
1
2
12345678901234567890123456789012123456789012345678901
2
12345678901234567890123456789012123456789012345678901
2
12345678901234567890123456789012123456789012345678901
2
12345678901234567890123456789012123456789012345678901
12345678901234567890123456789012123456789012345678901 2
2345678901234567890123456789012123456789012345678901
12345678901234567890123456789012123456789012345678901 22
1
2
12345678901234567890123456789012123456789012345678901
12345678901234567890123456789012123456789012345678901 2
2
12345678901234567890123456789012123456789012345678901
12345678901234567890123456789012123456789012345678901 2
12345678901234567890123456789012123456789012345678901 2
12345678901234567890123456789012123456789012345678901 2
12345678901234567890123456789012123456789012345678901 2 111
123456789012345678901234567890121234567890123456789012

S D S D

Take It to the Next Level

S D S D

http://www.xtremepapers.net

Part III: Section Review

www.petersons.com

http://www.xtremepapers.net

Note

123456789012345678901234567890121234567890123456789012
12345678901234567890123456789012123456789012345678901 2
2
12345678901234567890123456789012123456789012345678901
If you are given this figure and then asked to solve for x, start with the 2
12345678901234567890123456789012123456789012345678901
2
12345678901234567890123456789012123456789012345678901
geometry. You have a right triangle, since the angle at the origin (0, 0) is 90 2
12345678901234567890123456789012123456789012345678901
2
12345678901234567890123456789012123456789012345678901
degrees. The side along the y-axis is length 3, and the side along the x-axis 2
12345678901234567890123456789012123456789012345678901
2345678901234567890123456789012123456789012345678901
1 is length 4. This means you have an old friend, the 3:4:5 right triangle, 2
2
12345678901234567890123456789012123456789012345678901
2
12345678901234567890123456789012123456789012345678901
guest
starring
in
a
coordinate
grid
problem.
2
12345678901234567890123456789012123456789012345678901
2
12345678901234567890123456789012123456789012345678901
2345678901234567890123456789012123456789012345678901
1 Now turn to the algebra. The diagram contains an expression, 3x 1 y, for 2
2
12345678901234567890123456789012123456789012345678901
2
12345678901234567890123456789012123456789012345678901
the
hypotenuse,
but
you
know
that
the
length
is
5
since
its
a
3:
4:
5
right
2
12345678901234567890123456789012123456789012345678901
2
12345678901234567890123456789012123456789012345678901
triangle.
This
gives
you
the
equation
3x
1
y
5
5.
From
the
side
of
length
3,
2345678901234567890123456789012123456789012345678901
2
1
2
12345678901234567890123456789012123456789012345678901
you
can
get
x
1
y
5
3.
Once
you
have
two
equations
(x
1
y
5
3
and
2
12345678901234567890123456789012123456789012345678901
2
12345678901234567890123456789012123456789012345678901
3x
1
y
5
5)
and
two
variables,
you
can
subtract
the
former
equation
from
2
12345678901234567890123456789012123456789012345678901
2345678901234567890123456789012123456789012345678901
2
1 the latter and eliminate y. 2x 5 2 x 5 1
2
12345678901234567890123456789012123456789012345678901
2
12345678901234567890123456789012123456789012345678901
No one single step in this problem was difficult, but the problem was 2
12345678901234567890123456789012123456789012345678901
2
12345678901234567890123456789012123456789012345678901
involved since it was a messy pile of heaped-together geometry and algebra 2
12345678901234567890123456789012123456789012345678901
2
12345678901234567890123456789012123456789012345678901
concepts. If you deal with them separately, and take each in its turn, the 2
12345678901234567890123456789012123456789012345678901
2
12345678901234567890123456789012123456789012345678901
problem becomes quite manageable.
2
12345678901234567890123456789012123456789012345678901
2345678901234567890123456789012123456789012345678901
12345678901234567890123456789012123456789012345678901 2
1 A symbol that might come up in an algebra problem is the !. In English 2
2
12345678901234567890123456789012123456789012345678901
2
12345678901234567890123456789012123456789012345678901
grammar,
this
is
that
excited
bastion
of
punctuation,
the
exclamation
2
12345678901234567890123456789012123456789012345678901
2345678901234567890123456789012123456789012345678901
12345678901234567890123456789012123456789012345678901
point. In math, however, the ! is the symbol for factorials. With a 2
2
12345678901234567890123456789012123456789012345678901
2
12345678901234567890123456789012123456789012345678901
factorial,
you
take
the
initial
number
(say
5!)
and
multiply
it
by
all
integers
12345678901234567890123456789012123456789012345678901 2
2
12345678901234567890123456789012123456789012345678901
between 5 and 1.
2
12345678901234567890123456789012123456789012345678901
12345678901234567890123456789012123456789012345678901 2
2
12345678901234567890123456789012123456789012345678901
Sample factorials:
2
12345678901234567890123456789012123456789012345678901
2
1
5!
5
5
3
4
3
3
3
2
3
1
5
120
2345678901234567890123456789012123456789012345678901
12345678901234567890123456789012123456789012345678901 2
2
12345678901234567890123456789012123456789012345678901
3! 5 3 3 2 3 1 5 6
2
1
2
12345678901234567890123456789012123456789012345678901
or
2
12345678901234567890123456789012123456789012345678901
2345678901234567890123456789012123456789012345678901
12345678901234567890123456789012123456789012345678901
x! 5 x 3 (x 2 1) 3 (x 2 2) 3 . . . (It goes on until the last term is 1.) 2
2
12345678901234567890123456789012123456789012345678901
2
1
2
12345678901234567890123456789012123456789012345678901
2
12345678901234567890123456789012123456789012345678901
2345678901234567890123456789012123456789012345678901
12345678901234567890123456789012123456789012345678901 2
12345678901234567890123456789012123456789012345678901 2
2
1 You can only take the factorial of an integer greater than 1.
12345678901234567890123456789012123456789012345678901 2
12345678901234567890123456789012123456789012345678901 2
2
12345678901234567890123456789012123456789012345678901
2
12345678901234567890123456789012123456789012345678901
2
12345678901234567890123456789012123456789012345678901
2
12345678901234567890123456789012123456789012345678901
2
12345678901234567890123456789012123456789012345678901
x!
2345678901234567890123456789012123456789012345678901
2
12345678901234567890123456789012123456789012345678901
If x is two greater than y, how can you simplify ?
2
12345678901234567890123456789012123456789012345678901
y!
2
12345678901234567890123456789012123456789012345678901
2
12345678901234567890123456789012123456789012345678901
3
2
A.
y
1
3y
1
2y
2
1
2345678901234567890123456789012123456789012345678901
12345678901234567890123456789012123456789012345678901 2
2
12345678901234567890123456789012123456789012345678901
2
y
2
12345678901234567890123456789012123456789012345678901
B.
2
12345678901234567890123456789012123456789012345678901
2
2
1
x
2345678901234567890123456789012123456789012345678901
2
12345678901234567890123456789012123456789012345678901
2
2
2
1
C.
y
1
3xy
1
2xy
2
12345678901234567890123456789012123456789012345678901
2
12345678901234567890123456789012123456789012345678901
D.
(y
1
2)(y
1
1)
2
12345678901234567890123456789012123456789012345678901
2345678901234567890123456789012123456789012345678901
2
1
2
12345678901234567890123456789012123456789012345678901
E. (y 2 2)(y 2 1)
12345678901234567890123456789012123456789012345678901 2
2
112 12345678901234567890123456789012123456789012345678901
123456789012345678901234567890121234567890123456789012

Chapter 4: Quantitative Review

Take It to the Next Level

123456789012345678901234567890121234567890123456789012
12345678901234567890123456789012123456789012345678901 2
2
12345678901234567890123456789012123456789012345678901
Since x 5 y 1 2, then
2
12345678901234567890123456789012123456789012345678901
2
12345678901234567890123456789012123456789012345678901
2
12345678901234567890123456789012123456789012345678901
2
12345678901234567890123456789012123456789012345678901
x! ~y 1 2!! ~y 1 2!~y 1 1!~y!!
2
12345678901234567890123456789012123456789012345678901
5
5
5 ~y 1 2!~y 1 1!.
2345678901234567890123456789012123456789012345678901
2
1
y!
y!
y!
2
12345678901234567890123456789012123456789012345678901
2
12345678901234567890123456789012123456789012345678901
Factorials problems are pretty simple once you get over the freak-out 2
12345678901234567890123456789012123456789012345678901
2
12345678901234567890123456789012123456789012345678901
element of seeing an exclamation point.
2
12345678901234567890123456789012123456789012345678901
2
12345678901234567890123456789012123456789012345678901
2
12345678901234567890123456789012123456789012345678901
2
12345678901234567890123456789012123456789012345678901
2
12345678901234567890123456789012123456789012345678901
2345678901234567890123456789012123456789012345678901
2
1Word Problems
2
12345678901234567890123456789012123456789012345678901
2
12345678901234567890123456789012123456789012345678901
Word
problems
are
common
two-tier
questions
that
prompt
you
to
first
2
12345678901234567890123456789012123456789012345678901
2
12345678901234567890123456789012123456789012345678901
translate
English
into
algebra
and
then
solve
the
algebra.
The
simplest
way
2345678901234567890123456789012123456789012345678901
2
1
2
12345678901234567890123456789012123456789012345678901
to
make
a
word
problem
harder
is
to
make
the
translation
from
English
to
2
12345678901234567890123456789012123456789012345678901
2
12345678901234567890123456789012123456789012345678901
algebra
more
difficult.
The
actual
algebra
that
you
will
have
to
do
on
a
2
12345678901234567890123456789012123456789012345678901
2345678901234567890123456789012123456789012345678901
12345678901234567890123456789012123456789012345678901
500-level word problem and 700-level word problem is going to be similar, 2
2
12345678901234567890123456789012123456789012345678901
2
12345678901234567890123456789012123456789012345678901
but
getting
to
the
algebra
becomes
more
of
a
challenge.
12345678901234567890123456789012123456789012345678901 22
12345678901234567890123456789012123456789012345678901 2
12345678901234567890123456789012123456789012345678901
2
1 Heres a classic word problem set-up:
2
12345678901234567890123456789012123456789012345678901
How many liters of a solution that is 15 percent water by volume have 2
12345678901234567890123456789012123456789012345678901
12345678901234567890123456789012123456789012345678901
to be added to 3 liters of a solution that is 30 percent water by volume 2
2
12345678901234567890123456789012123456789012345678901
2
12345678901234567890123456789012123456789012345678901
to create a solution that is 20 percent water by volume?
2
12345678901234567890123456789012123456789012345678901
2
12345678901234567890123456789012123456789012345678901
2
12345678901234567890123456789012123456789012345678901
All
that
was
a
quite
a
tangle.
To
untangle
it,
there
is
a
basic
form
to
follow
2345678901234567890123456789012123456789012345678901
12345678901234567890123456789012123456789012345678901 22
12345678901234567890123456789012123456789012345678901
for mixture problems. Its called the balance method since you are 2
12345678901234567890123456789012123456789012345678901
2
12345678901234567890123456789012123456789012345678901
1 balancing the left and right side of the equation. On the left side, put the 2
2345678901234567890123456789012123456789012345678901
2
12345678901234567890123456789012123456789012345678901
original solution:
2
12345678901234567890123456789012123456789012345678901
2
1
2
12345678901234567890123456789012123456789012345678901
Number of original liters (original% 2 final%) 5 3(30 2 20)
2
12345678901234567890123456789012123456789012345678901
2345678901234567890123456789012123456789012345678901
2
12345678901234567890123456789012123456789012345678901
And on the right side put the new solution,
2
12345678901234567890123456789012123456789012345678901
2
1
2
12345678901234567890123456789012123456789012345678901
n(final%
2
added%)
5
n(20
2
15)
2
12345678901234567890123456789012123456789012345678901
2345678901234567890123456789012123456789012345678901
12345678901234567890123456789012123456789012345678901 22
12345678901234567890123456789012123456789012345678901
1 where n is the number of liters of the 15% solution. Now just solve for n: 2
12345678901234567890123456789012123456789012345678901 2
2
12345678901234567890123456789012123456789012345678901
3~30 2 20! 5 n~20 2 15!
2
12345678901234567890123456789012123456789012345678901
2
12345678901234567890123456789012123456789012345678901
90 2 60 5 n~5!
2
12345678901234567890123456789012123456789012345678901
2
12345678901234567890123456789012123456789012345678901
2
12345678901234567890123456789012123456789012345678901
30 5 5n
2345678901234567890123456789012123456789012345678901
12345678901234567890123456789012123456789012345678901 22
12345678901234567890123456789012123456789012345678901
30 4 5 5 5n 4 5
2
12345678901234567890123456789012123456789012345678901
2
12345678901234567890123456789012123456789012345678901
6
5
n
2
1
2345678901234567890123456789012123456789012345678901
12345678901234567890123456789012123456789012345678901 22
12345678901234567890123456789012123456789012345678901
Take your time to muck through the English of a word problem and you 2
12345678901234567890123456789012123456789012345678901
2
12345678901234567890123456789012123456789012345678901
2
1 will end up with a perfectly solvable algebraic equation.
2345678901234567890123456789012123456789012345678901
12345678901234567890123456789012123456789012345678901 22
1
2
12345678901234567890123456789012123456789012345678901
12345678901234567890123456789012123456789012345678901 2
2
12345678901234567890123456789012123456789012345678901
12345678901234567890123456789012123456789012345678901 2
12345678901234567890123456789012123456789012345678901 2
12345678901234567890123456789012123456789012345678901 2
12345678901234567890123456789012123456789012345678901 2 113
123456789012345678901234567890121234567890123456789012

http://www.xtremepapers.net

Part III: Section Review

123456789012345678901234567890121234567890123456789012
12345678901234567890123456789012123456789012345678901 2
2
12345678901234567890123456789012123456789012345678901
2
12345678901234567890123456789012123456789012345678901
Advanced Statistics
2
12345678901234567890123456789012123456789012345678901
2
12345678901234567890123456789012123456789012345678901
There
are
some
interesting
ways
to
manipulate
statistical
notions
like
12345678901234567890123456789012123456789012345678901 2
12345678901234567890123456789012123456789012345678901
average or median. Harder questions on the GRE math section will expect 2
2
12345678901234567890123456789012123456789012345678901
12345678901234567890123456789012123456789012345678901
you to know the basic set of statistic notionscovered on pages 7678 in 2
2
12345678901234567890123456789012123456789012345678901
2
12345678901234567890123456789012123456789012345678901
this chapterand how to use them creatively to problem solve.
2
12345678901234567890123456789012123456789012345678901
2345678901234567890123456789012123456789012345678901
1 For example, you commonly employ the concept of mean to find the 2
2
12345678901234567890123456789012123456789012345678901
2
12345678901234567890123456789012123456789012345678901
average of a set of numbers, but suppose you were asked for the sum of 2
12345678901234567890123456789012123456789012345678901
2
12345678901234567890123456789012123456789012345678901
the set of numbers instead? If 18 is the average of a set of numbers that 2
12345678901234567890123456789012123456789012345678901
2
12345678901234567890123456789012123456789012345678901
has 10 members, then can you find the sum of the set of numbers? To do 2
12345678901234567890123456789012123456789012345678901
12345678901234567890123456789012123456789012345678901
this, you have to multiply the mean average by the number of terms, 2
2
12345678901234567890123456789012123456789012345678901
2345678901234567890123456789012123456789012345678901
2
1 18 3 10 5 180.
2
12345678901234567890123456789012123456789012345678901
2
12345678901234567890123456789012123456789012345678901
2
12345678901234567890123456789012123456789012345678901
Thats
simple
enough,
but
it
gets
trickier.
Suppose
you
are
asked,
What
is
2
12345678901234567890123456789012123456789012345678901
2345678901234567890123456789012123456789012345678901
12345678901234567890123456789012123456789012345678901
the sum of the integers 20 to 50 inclusive? Actually adding all the terms 2
2
12345678901234567890123456789012123456789012345678901
2
12345678901234567890123456789012123456789012345678901
up
cant
be
the
best
way
to
solve
this
problem
because
that
would
take
all
12345678901234567890123456789012123456789012345678901 2
12345678901234567890123456789012123456789012345678901
day. You could do it, but there has to be a simpler way. The shortcut 2
2
12345678901234567890123456789012123456789012345678901
1 method here is that the sum of consecutive numbers is equal to the average 2
2
12345678901234567890123456789012123456789012345678901
2
12345678901234567890123456789012123456789012345678901
of the numbers times the number of terms:
2
12345678901234567890123456789012123456789012345678901
2345678901234567890123456789012123456789012345678901
2
12345678901234567890123456789012123456789012345678901
sum 5 (average) 3 (number of terms)
2
12345678901234567890123456789012123456789012345678901
12345678901234567890123456789012123456789012345678901 2
2
12345678901234567890123456789012123456789012345678901
In our problem, the average is 35 (halfway between 20 and 50), so
2
12345678901234567890123456789012123456789012345678901
12345678901234567890123456789012123456789012345678901 2
2
12345678901234567890123456789012123456789012345678901
sum 5 (35) 3 (31) 5 1,085
2
12345678901234567890123456789012123456789012345678901
12345678901234567890123456789012123456789012345678901 2
1 The notion of weighted averages can make finding the average of a 2
2
12345678901234567890123456789012123456789012345678901
12345678901234567890123456789012123456789012345678901
number set a little more interesting. A weighted average is when certain 2
12345678901234567890123456789012123456789012345678901 2
12345678901234567890123456789012123456789012345678901
terms in a set are counted more (weighted more) than other terms in 2
2
12345678901234567890123456789012123456789012345678901
2345678901234567890123456789012123456789012345678901
2
12345678901234567890123456789012123456789012345678901
the set.
2
12345678901234567890123456789012123456789012345678901
2
1
2
12345678901234567890123456789012123456789012345678901
In
a
certain
class,
the
average
boys
score
is
50
and
the
average
girls
2
12345678901234567890123456789012123456789012345678901
2345678901234567890123456789012123456789012345678901
2
12345678901234567890123456789012123456789012345678901
score is 47. If there are twice as many girls in the class as boys,
2
12345678901234567890123456789012123456789012345678901
2
1
what
is
the
class
average?
12345678901234567890123456789012123456789012345678901 2
12345678901234567890123456789012123456789012345678901 2
2
12345678901234567890123456789012123456789012345678901
A. 49.5
2
12345678901234567890123456789012123456789012345678901
2
12345678901234567890123456789012123456789012345678901
B. 49
2
12345678901234567890123456789012123456789012345678901
2
12345678901234567890123456789012123456789012345678901
C. 48.5
2345678901234567890123456789012123456789012345678901
12345678901234567890123456789012123456789012345678901 2
2
12345678901234567890123456789012123456789012345678901
D. 48
12345678901234567890123456789012123456789012345678901 2
2
12345678901234567890123456789012123456789012345678901
E. 47.5
2
1
2345678901234567890123456789012123456789012345678901
2
12345678901234567890123456789012123456789012345678901
There are not equal amounts of girls and boys, so you have to weight their 2
12345678901234567890123456789012123456789012345678901
12345678901234567890123456789012123456789012345678901
averages to get to the overall average. Since there are twice as many girls as 2
2
12345678901234567890123456789012123456789012345678901
1 boys, there are twice as many terms that go into the 47 average. The 2
2345678901234567890123456789012123456789012345678901
12345678901234567890123456789012123456789012345678901 2
2
1
2
12345678901234567890123456789012123456789012345678901
12345678901234567890123456789012123456789012345678901 2
2
12345678901234567890123456789012123456789012345678901
12345678901234567890123456789012123456789012345678901 2
12345678901234567890123456789012123456789012345678901 2
12345678901234567890123456789012123456789012345678901 2
2
114 12345678901234567890123456789012123456789012345678901
123456789012345678901234567890121234567890123456789012

www.petersons.com

http://www.xtremepapers.net

Chapter 4: Quantitative Review

Take It to the Next Level

123456789012345678901234567890121234567890123456789012
12345678901234567890123456789012123456789012345678901 2
2
12345678901234567890123456789012123456789012345678901
simplest way is to assume a three-person class where there is one boy and 2
12345678901234567890123456789012123456789012345678901
2
12345678901234567890123456789012123456789012345678901
two girls. The boy scored a 47 and the girls scored 50s, so
2
12345678901234567890123456789012123456789012345678901
12345678901234567890123456789012123456789012345678901 2
2
12345678901234567890123456789012123456789012345678901
2
12345678901234567890123456789012123456789012345678901
1 3 50 1 2 3 47 144
2
12345678901234567890123456789012123456789012345678901
average 5
5
5 48
2
12345678901234567890123456789012123456789012345678901
3
3
2
12345678901234567890123456789012123456789012345678901
2
12345678901234567890123456789012123456789012345678901
The
denominator
is
3
because
there
are
three
terms
in
this
number
set
(50,
2345678901234567890123456789012123456789012345678901
2
1
2
12345678901234567890123456789012123456789012345678901
47, and 47) to reflect the mythical three people in the classroom.
2
12345678901234567890123456789012123456789012345678901
2
12345678901234567890123456789012123456789012345678901
2
12345678901234567890123456789012123456789012345678901
Another
variation
on
the
average
is
when
you
start
with
a
number
set,
add
2345678901234567890123456789012123456789012345678901
2
1
2
12345678901234567890123456789012123456789012345678901
or
subtract
a
term,
and
then
must
find
the
new
average.
2
12345678901234567890123456789012123456789012345678901
2
12345678901234567890123456789012123456789012345678901
2
12345678901234567890123456789012123456789012345678901
Column
B
Column
A
2345678901234567890123456789012123456789012345678901
2
1
2
12345678901234567890123456789012123456789012345678901
2
12345678901234567890123456789012123456789012345678901
The mean average of 4 numbers is 26.
2
12345678901234567890123456789012123456789012345678901
2
12345678901234567890123456789012123456789012345678901
One number is taken from the set,
2345678901234567890123456789012123456789012345678901
12345678901234567890123456789012123456789012345678901 22
12345678901234567890123456789012123456789012345678901
and the new mean is 23.
12345678901234567890123456789012123456789012345678901 22
12345678901234567890123456789012123456789012345678901
The number taken
The largest number
2
12345678901234567890123456789012123456789012345678901
2
12345678901234567890123456789012123456789012345678901
from the set
left in the set
2
1
2
12345678901234567890123456789012123456789012345678901
2
12345678901234567890123456789012123456789012345678901
Start
with
Column
A
unless
you
see
something
that
makes
you
feel
2
12345678901234567890123456789012123456789012345678901
2345678901234567890123456789012123456789012345678901
12345678901234567890123456789012123456789012345678901
Column B is simple. For Column A, the number taken must be the 2
2
12345678901234567890123456789012123456789012345678901
2
12345678901234567890123456789012123456789012345678901
difference
between
the
sums
of
the
two
sets:
12345678901234567890123456789012123456789012345678901 22
12345678901234567890123456789012123456789012345678901 2
12345678901234567890123456789012123456789012345678901
(original sum) 2 (new sum)
2
12345678901234567890123456789012123456789012345678901
2
12345678901234567890123456789012123456789012345678901
(26 3 4) 2 (23 3 3) 5 2 15
2
12345678901234567890123456789012123456789012345678901
2
1
2345678901234567890123456789012123456789012345678901
12345678901234567890123456789012123456789012345678901
For Column A, you know that 215 was taken from the number set. Think 2
2
12345678901234567890123456789012123456789012345678901
1 about how this number is much lower than the average after it left and 2
12345678901234567890123456789012123456789012345678901 2
12345678901234567890123456789012123456789012345678901
before it left. 215 was obviously one of the lower numbers in the set, and 2
2
12345678901234567890123456789012123456789012345678901
2
12345678901234567890123456789012123456789012345678901
the
higher
numbers
brought
the
average
up
to
26
and
then
up
to
23.
It
12345678901234567890123456789012123456789012345678901 2
12345678901234567890123456789012123456789012345678901
makes sense that Column B definitely contains a number larger than 215, 2
2
12345678901234567890123456789012123456789012345678901
2345678901234567890123456789012123456789012345678901
2
12345678901234567890123456789012123456789012345678901
so choice (B) is the correct answer for this average problem.
2
12345678901234567890123456789012123456789012345678901
2
1
12345678901234567890123456789012123456789012345678901
Compound interest problems involve a loaded math concept. Simple 2
12345678901234567890123456789012123456789012345678901 2
12345678901234567890123456789012123456789012345678901
interest is when the interest applies only to the principal over a given 2
2
12345678901234567890123456789012123456789012345678901
12345678901234567890123456789012123456789012345678901
period. If a $10,000 account has a 5 percent simple interest rate 2
2
12345678901234567890123456789012123456789012345678901
compounded annually, then after six months the interest will be:
2
12345678901234567890123456789012123456789012345678901
2345678901234567890123456789012123456789012345678901
12345678901234567890123456789012123456789012345678901 22
12345678901234567890123456789012123456789012345678901
principal 3 rate 3 time 5 10,000 3 0.05 3 0.5
2
12345678901234567890123456789012123456789012345678901
2
12345678901234567890123456789012123456789012345678901
5
5,000
3
0.05
2
1
2345678901234567890123456789012123456789012345678901
2
12345678901234567890123456789012123456789012345678901
5 250
2
12345678901234567890123456789012123456789012345678901
12345678901234567890123456789012123456789012345678901 22
12345678901234567890123456789012123456789012345678901
1 The first number is the cash, the second value is the percent interest 2
12345678901234567890123456789012123456789012345678901
expressed in decimal form as 0.05, and the 0.5 at the end shows that you 2
2
12345678901234567890123456789012123456789012345678901
2
12345678901234567890123456789012123456789012345678901
are
halfway
(six
months)
through
the
annual
interest
period.
12345678901234567890123456789012123456789012345678901 2
2
12345678901234567890123456789012123456789012345678901
12345678901234567890123456789012123456789012345678901
Things get a little more complicated with compounding interest. Under 2
2
12345678901234567890123456789012123456789012345678901
this format, the interest rate is applied to the principal and to the 2
12345678901234567890123456789012123456789012345678901
12345678901234567890123456789012123456789012345678901 2 115
123456789012345678901234567890121234567890123456789012

http://www.xtremepapers.net

Part III: Section Review

www.petersons.com

http://www.xtremepapers.net

Alert!

123456789012345678901234567890121234567890123456789012
12345678901234567890123456789012123456789012345678901 2
2
12345678901234567890123456789012123456789012345678901
accumulated interest. There is a complicated formula for compound 2
12345678901234567890123456789012123456789012345678901
2
12345678901234567890123456789012123456789012345678901
interest, but you can do without it. If you come across a compound interest 2
12345678901234567890123456789012123456789012345678901
2
12345678901234567890123456789012123456789012345678901
problem, simply calculate the simple interest for the scenario, and the 2
12345678901234567890123456789012123456789012345678901
2345678901234567890123456789012123456789012345678901
1 compound interest will be a little more than this. There is typically a small 2
2
12345678901234567890123456789012123456789012345678901
2
12345678901234567890123456789012123456789012345678901
difference
between
the
two
because
the
interest
accrued
on
the
interest
will
2
12345678901234567890123456789012123456789012345678901
2
12345678901234567890123456789012123456789012345678901
be
small.
2345678901234567890123456789012123456789012345678901
2
1
2
12345678901234567890123456789012123456789012345678901
2
12345678901234567890123456789012123456789012345678901
2
12345678901234567890123456789012123456789012345678901
2
12345678901234567890123456789012123456789012345678901
Supposedly,
many
savings
accounts
at
banks
work
on
the
concept
of
2345678901234567890123456789012123456789012345678901
2
1
2
12345678901234567890123456789012123456789012345678901
compound
interest,
but
like
many
Americans,
the
author
has
never
2
12345678901234567890123456789012123456789012345678901
2
12345678901234567890123456789012123456789012345678901
experienced
this
fact
firsthand.
The
author
is,
however,
well
aware
of
the
2
12345678901234567890123456789012123456789012345678901
2345678901234567890123456789012123456789012345678901
2
1 interest formulas used by credit card companies.
2
12345678901234567890123456789012123456789012345678901
2
12345678901234567890123456789012123456789012345678901
2
12345678901234567890123456789012123456789012345678901
2
12345678901234567890123456789012123456789012345678901
2345678901234567890123456789012123456789012345678901
12345678901234567890123456789012123456789012345678901 2
Geometry
12345678901234567890123456789012123456789012345678901 2
2
12345678901234567890123456789012123456789012345678901
To double the geometric fun (and the difficulty), harder geometry 2
12345678901234567890123456789012123456789012345678901
2
12345678901234567890123456789012123456789012345678901
problems will include multiple geometric shapes. A square might be 2
12345678901234567890123456789012123456789012345678901
1 inscribed in a circle. A triangle might be located inside a trapezoid. 2
2
12345678901234567890123456789012123456789012345678901
2
12345678901234567890123456789012123456789012345678901
Regardless
of
the
particulars,
the
idea
behind
increasing
the
difficulty
is
the
2
12345678901234567890123456789012123456789012345678901
2345678901234567890123456789012123456789012345678901
12345678901234567890123456789012123456789012345678901
same. Things look cluttered when there are multiple figures, and that 2
2
12345678901234567890123456789012123456789012345678901
2
12345678901234567890123456789012123456789012345678901
clutter
can
distract
you
from
problem-solving
tasks.
You
combat
the
12345678901234567890123456789012123456789012345678901 2
2
12345678901234567890123456789012123456789012345678901
clutter by:
2
12345678901234567890123456789012123456789012345678901
12345678901234567890123456789012123456789012345678901 2
2
12345678901234567890123456789012123456789012345678901
1. First determining exactly what you are trying to find
2
12345678901234567890123456789012123456789012345678901
2
1
2345678901234567890123456789012123456789012345678901
2
12345678901234567890123456789012123456789012345678901
2. Considering each of the figures separately
2
12345678901234567890123456789012123456789012345678901
2
1
3. Trying to find the relationships between the figures that will help 2
12345678901234567890123456789012123456789012345678901
2
12345678901234567890123456789012123456789012345678901
solve the problem
2
12345678901234567890123456789012123456789012345678901
2
12345678901234567890123456789012123456789012345678901
2
12345678901234567890123456789012123456789012345678901
All
three
steps
are
important,
but
the
third
step
is
the
one
that
holds
the
2
12345678901234567890123456789012123456789012345678901
2
12345678901234567890123456789012123456789012345678901
key
to
most
of
these
questions.
To
try
out
this
methodology,
consider
a
2345678901234567890123456789012123456789012345678901
12345678901234567890123456789012123456789012345678901 2
2
12345678901234567890123456789012123456789012345678901
2
1 square inscribed in a circle like the following:
12345678901234567890123456789012123456789012345678901 2
12345678901234567890123456789012123456789012345678901 2
2
12345678901234567890123456789012123456789012345678901
2
12345678901234567890123456789012123456789012345678901
2
12345678901234567890123456789012123456789012345678901
2
12345678901234567890123456789012123456789012345678901
12345678901234567890123456789012123456789012345678901 2
2345678901234567890123456789012123456789012345678901
12345678901234567890123456789012123456789012345678901 2
12345678901234567890123456789012123456789012345678901 2
12345678901234567890123456789012123456789012345678901 2
12345678901234567890123456789012123456789012345678901 2
2
1
12345678901234567890123456789012123456789012345678901
Your goal is to find the radius of the circle, but all you have been given is 2
2
12345678901234567890123456789012123456789012345678901
12345678901234567890123456789012123456789012345678901
the area of the square. First, locate the radius of the circle. If you think 2
2
12345678901234567890123456789012123456789012345678901
12345678901234567890123456789012123456789012345678901
about it, you can see that the diagonal of the square is also the diameter of 2
2345678901234567890123456789012123456789012345678901
12345678901234567890123456789012123456789012345678901 2
1 the circle, and the radius is always half a circles diameter. So half of the 2
2
12345678901234567890123456789012123456789012345678901
12345678901234567890123456789012123456789012345678901
diagonal of the square will equal the radius. Second, you can use the area 2
2
12345678901234567890123456789012123456789012345678901
formula of a square to figure out the length of the sides. If area is 9, then 2
12345678901234567890123456789012123456789012345678901
2
12345678901234567890123456789012123456789012345678901
the sides are all length 3.
2
12345678901234567890123456789012123456789012345678901
2
116 12345678901234567890123456789012123456789012345678901
123456789012345678901234567890121234567890123456789012

Chapter 4: Quantitative Review

Take It to the Next Level

123456789012345678901234567890121234567890123456789012
12345678901234567890123456789012123456789012345678901 2
2
12345678901234567890123456789012123456789012345678901
Now imagine the diagonal. If you draw it out, you would see yourself 2
12345678901234567890123456789012123456789012345678901
2
12345678901234567890123456789012123456789012345678901
making two 45-45-90 right triangles out of the square. Finding the 2
12345678901234567890123456789012123456789012345678901
2
12345678901234567890123456789012123456789012345678901
hypotenuse of one these triangles is equal to finding the squares diagonal, 2
12345678901234567890123456789012123456789012345678901
2345678901234567890123456789012123456789012345678901
1 and that is equal to the circles diameter. Follow all the steps (try it on your 2
2
12345678901234567890123456789012123456789012345678901
2
12345678901234567890123456789012123456789012345678901
2
12345678901234567890123456789012123456789012345678901
3=2
2
12345678901234567890123456789012123456789012345678901
.
own),
and
you
should
end
up
with
a
radius
of
2345678901234567890123456789012123456789012345678901
2
1
2
2
12345678901234567890123456789012123456789012345678901
2
12345678901234567890123456789012123456789012345678901
To
help
you
recognize
the
relationship
that
two
or
more
figures
share,
ask
2
12345678901234567890123456789012123456789012345678901
2
12345678901234567890123456789012123456789012345678901
yourself
questions
like,
Do
they
share
a
side,
or
an
angle?
or
What
is
2345678901234567890123456789012123456789012345678901
2
1
2
12345678901234567890123456789012123456789012345678901
the
relationship
between
their
areas?
Once
you
have
picked
out
the
2
12345678901234567890123456789012123456789012345678901
2
12345678901234567890123456789012123456789012345678901
relationship
that
links
the
two
figures
together,
the
actual
geometry
is
often
2
12345678901234567890123456789012123456789012345678901
2345678901234567890123456789012123456789012345678901
2
1 no harder than what is covered in the first section.
2
12345678901234567890123456789012123456789012345678901
2
12345678901234567890123456789012123456789012345678901
2
12345678901234567890123456789012123456789012345678901
Column
B
Column
A
2
12345678901234567890123456789012123456789012345678901
2345678901234567890123456789012123456789012345678901
12345678901234567890123456789012123456789012345678901 22
12345678901234567890123456789012123456789012345678901 2
12345678901234567890123456789012123456789012345678901 2
12345678901234567890123456789012123456789012345678901 2
12345678901234567890123456789012123456789012345678901 2
12345678901234567890123456789012123456789012345678901 2
1
2
12345678901234567890123456789012123456789012345678901
2
12345678901234567890123456789012123456789012345678901
2
12345678901234567890123456789012123456789012345678901
2345678901234567890123456789012123456789012345678901
12345678901234567890123456789012123456789012345678901 22
12345678901234567890123456789012123456789012345678901 2
12345678901234567890123456789012123456789012345678901 2
12345678901234567890123456789012123456789012345678901 2
12345678901234567890123456789012123456789012345678901 2
12345678901234567890123456789012123456789012345678901 2
12345678901234567890123456789012123456789012345678901 2
12345678901234567890123456789012123456789012345678901
The difference of the areas of trapezoid
2
12345678901234567890123456789012123456789012345678901
2
1
ABDH and trapezoid FECJ is 15.
2345678901234567890123456789012123456789012345678901
12345678901234567890123456789012123456789012345678901 22
12345678901234567890123456789012123456789012345678901
The area of the
The area of the
2
1
2
12345678901234567890123456789012123456789012345678901
dotted
region
striped
region
2
12345678901234567890123456789012123456789012345678901
2345678901234567890123456789012123456789012345678901
12345678901234567890123456789012123456789012345678901 22
12345678901234567890123456789012123456789012345678901
1 Locate the two trapezoids. How are they geometrically related? Both share 2
12345678901234567890123456789012123456789012345678901
the striped region and they have the same height, 5. Now you can try to 2
2
12345678901234567890123456789012123456789012345678901
2345678901234567890123456789012123456789012345678901
12345678901234567890123456789012123456789012345678901
compare the striped region and the dotted region. Can you determine the 2
2
12345678901234567890123456789012123456789012345678901
1 area of the dotted region? Yes, since both are rectangles with length and 2
12345678901234567890123456789012123456789012345678901 2
12345678901234567890123456789012123456789012345678901
width readily obvious. However, you cannot determine the area of the 2
2
12345678901234567890123456789012123456789012345678901
12345678901234567890123456789012123456789012345678901
striped region, since theres no way to figure out its width. The difference 2
2
12345678901234567890123456789012123456789012345678901
12345678901234567890123456789012123456789012345678901
in areas of the trapezoids does not help us, for when the areas of the 2
2
12345678901234567890123456789012123456789012345678901
trapezoid are subtracted, the area of the striped region in each just cancels 2
12345678901234567890123456789012123456789012345678901
2
12345678901234567890123456789012123456789012345678901
out. Those triangles on both ends of the figure hold an unknown amount 2
12345678901234567890123456789012123456789012345678901
2
12345678901234567890123456789012123456789012345678901
of area for both trapezoids, making any attempt to subtract one 2
12345678901234567890123456789012123456789012345678901
2345678901234567890123456789012123456789012345678901
2
12345678901234567890123456789012123456789012345678901
trapezoids area from the other impossible. Therefore, (D) is the correct 2
12345678901234567890123456789012123456789012345678901
2
12345678901234567890123456789012123456789012345678901
answer.
2
12345678901234567890123456789012123456789012345678901
2
1
2345678901234567890123456789012123456789012345678901
12345678901234567890123456789012123456789012345678901 22
1
2
12345678901234567890123456789012123456789012345678901
12345678901234567890123456789012123456789012345678901 2
2
12345678901234567890123456789012123456789012345678901
12345678901234567890123456789012123456789012345678901 2
12345678901234567890123456789012123456789012345678901 2
12345678901234567890123456789012123456789012345678901 2
12345678901234567890123456789012123456789012345678901 2 117
123456789012345678901234567890121234567890123456789012

http://www.xtremepapers.net

Part III: Section Review

123456789012345678901234567890121234567890123456789012
12345678901234567890123456789012123456789012345678901 2
2
12345678901234567890123456789012123456789012345678901
2
12345678901234567890123456789012123456789012345678901
Coordinate Geometry
2
12345678901234567890123456789012123456789012345678901
2
12345678901234567890123456789012123456789012345678901
Reach
back
into
your
past
and
dust
off
the
equation
y
5
mx
1
b.
This
12345678901234567890123456789012123456789012345678901 2
12345678901234567890123456789012123456789012345678901
formula can be used to describe a straight line in a coordinate plane. The y 2
2
12345678901234567890123456789012123456789012345678901
12345678901234567890123456789012123456789012345678901
and the x variables stand the value of y and x, respectively, of any point on 2
2
12345678901234567890123456789012123456789012345678901
12345678901234567890123456789012123456789012345678901
a particular line. m is the slope of the line and b is the y-intercept (where 2
2
12345678901234567890123456789012123456789012345678901
12345678901234567890123456789012123456789012345678901
the line crosses the y-axis). The beauty of this equation is that if you are 2
2
12345678901234567890123456789012123456789012345678901
12345678901234567890123456789012123456789012345678901
given some of the partssuch as the slope and one point on the linethere 2
2
12345678901234567890123456789012123456789012345678901
is a way to figure out all the other parts of the equation.
2
12345678901234567890123456789012123456789012345678901
2345678901234567890123456789012123456789012345678901
2
1
2
12345678901234567890123456789012123456789012345678901
To be able to use this formula and just read off the slope or the y-intercept, 2
12345678901234567890123456789012123456789012345678901
12345678901234567890123456789012123456789012345678901
the equation has to be in the y 5 mx 1 b form, with y by itself on one side, 2
2
12345678901234567890123456789012123456789012345678901
2345678901234567890123456789012123456789012345678901
1 and the x on the other side summed with b. If you are given a linear 2
2
12345678901234567890123456789012123456789012345678901
2
12345678901234567890123456789012123456789012345678901
equation
of
a
line
in
the
form
2y
1
5x
5
b,
you
would
have
to
do
some
2
12345678901234567890123456789012123456789012345678901
2
12345678901234567890123456789012123456789012345678901
manipulating:
2345678901234567890123456789012123456789012345678901
12345678901234567890123456789012123456789012345678901 2
12345678901234567890123456789012123456789012345678901 2
2
12345678901234567890123456789012123456789012345678901
2y 1 5x 5 23
2
12345678901234567890123456789012123456789012345678901
2
12345678901234567890123456789012123456789012345678901
2y
1
5x
2
5x
5
23
2
5x

12345678901234567890123456789012123456789012345678901 2
2
1
2
12345678901234567890123456789012123456789012345678901
2y
5
25x
2
3

2
12345678901234567890123456789012123456789012345678901
2
12345678901234567890123456789012123456789012345678901
2y
4
2
5
25x
2
3
4
2

~
!
~
!
2345678901234567890123456789012123456789012345678901
12345678901234567890123456789012123456789012345678901 2
2
12345678901234567890123456789012123456789012345678901
5
3
2
12345678901234567890123456789012123456789012345678901
y52 x2
2
12345678901234567890123456789012123456789012345678901
2
2
2
12345678901234567890123456789012123456789012345678901
12345678901234567890123456789012123456789012345678901 2
2
12345678901234567890123456789012123456789012345678901
5
2
12345678901234567890123456789012123456789012345678901
The
slope
is
negative
five
halves,
2
,
and
the
y-intercept
is
negative
2
12345678901234567890123456789012123456789012345678901
2
2
1
2345678901234567890123456789012123456789012345678901
2
12345678901234567890123456789012123456789012345678901
3
2
12345678901234567890123456789012123456789012345678901
2
.
three
halves,
2
1
2
12345678901234567890123456789012123456789012345678901 2
12345678901234567890123456789012123456789012345678901
The slope equation works for a linebut nothing else. You wont see 2
2
12345678901234567890123456789012123456789012345678901
2
12345678901234567890123456789012123456789012345678901
many
things
graphed
besides
lines
on
the
GRE,
but
you
might
see
a
2
12345678901234567890123456789012123456789012345678901
2
2
12345678901234567890123456789012123456789012345678901
quadratic
equation
(an
equation
that
has
a
squared
term
like
y
5
x
).
To
2
12345678901234567890123456789012123456789012345678901
2345678901234567890123456789012123456789012345678901
12345678901234567890123456789012123456789012345678901
make things more confusing, the y may be exchanged for an f(x) so it looks 2
2
12345678901234567890123456789012123456789012345678901
1 like this: f(x) 5 x2. Then the whole thing is renamed a function. It is 2
12345678901234567890123456789012123456789012345678901 2
12345678901234567890123456789012123456789012345678901
simplest to just think of the f(x) as a more complicated way to write y. As 2
2
12345678901234567890123456789012123456789012345678901
2
12345678901234567890123456789012123456789012345678901
you
might
imagine,
the
graph
of
this
quadratic
is
not
as
simple
as
the
graph
2
12345678901234567890123456789012123456789012345678901
2
12345678901234567890123456789012123456789012345678901
of
a
line,
but
it
has
a
basic
form
that
simplifies
it.
12345678901234567890123456789012123456789012345678901 2
2345678901234567890123456789012123456789012345678901
12345678901234567890123456789012123456789012345678901 2
12345678901234567890123456789012123456789012345678901 2
12345678901234567890123456789012123456789012345678901 2
12345678901234567890123456789012123456789012345678901 2
2
1
2
12345678901234567890123456789012123456789012345678901
2
12345678901234567890123456789012123456789012345678901
2
12345678901234567890123456789012123456789012345678901
2
12345678901234567890123456789012123456789012345678901
12345678901234567890123456789012123456789012345678901 2
2345678901234567890123456789012123456789012345678901
12345678901234567890123456789012123456789012345678901 2
2
1
2
12345678901234567890123456789012123456789012345678901
12345678901234567890123456789012123456789012345678901 2
2
12345678901234567890123456789012123456789012345678901
12345678901234567890123456789012123456789012345678901 2
12345678901234567890123456789012123456789012345678901 2
12345678901234567890123456789012123456789012345678901 2
2
118 12345678901234567890123456789012123456789012345678901
123456789012345678901234567890121234567890123456789012

S D

S D

www.petersons.com

http://www.xtremepapers.net

Chapter 4: Quantitative Review

Note

Take It to the Next Level

123456789012345678901234567890121234567890123456789012
12345678901234567890123456789012123456789012345678901 2
2
12345678901234567890123456789012123456789012345678901
12345678901234567890123456789012123456789012345678901 2
2
12345678901234567890123456789012123456789012345678901
2
12345678901234567890123456789012123456789012345678901
12345678901234567890123456789012123456789012345678901 2
2
12345678901234567890123456789012123456789012345678901
12345678901234567890123456789012123456789012345678901 2
2
12345678901234567890123456789012123456789012345678901
2
12345678901234567890123456789012123456789012345678901
2
12345678901234567890123456789012123456789012345678901
2
12345678901234567890123456789012123456789012345678901
2345678901234567890123456789012123456789012345678901
2
1
2
12345678901234567890123456789012123456789012345678901
2
12345678901234567890123456789012123456789012345678901
2
12345678901234567890123456789012123456789012345678901
2
12345678901234567890123456789012123456789012345678901
2345678901234567890123456789012123456789012345678901
2
1
2
12345678901234567890123456789012123456789012345678901
2
12345678901234567890123456789012123456789012345678901
2
12345678901234567890123456789012123456789012345678901
2
12345678901234567890123456789012123456789012345678901
2345678901234567890123456789012123456789012345678901
2
1
2
12345678901234567890123456789012123456789012345678901
2
12345678901234567890123456789012123456789012345678901
2
12345678901234567890123456789012123456789012345678901
2
12345678901234567890123456789012123456789012345678901
2345678901234567890123456789012123456789012345678901
12345678901234567890123456789012123456789012345678901 22
12345678901234567890123456789012123456789012345678901 2
12345678901234567890123456789012123456789012345678901 2
12345678901234567890123456789012123456789012345678901 2
12345678901234567890123456789012123456789012345678901 2
12345678901234567890123456789012123456789012345678901 2
1
2
12345678901234567890123456789012123456789012345678901
2
12345678901234567890123456789012123456789012345678901
Each
of
these
graphs
shares
a
basic
shape
called
a
parabola.
Examine
the
2
12345678901234567890123456789012123456789012345678901
2345678901234567890123456789012123456789012345678901
12345678901234567890123456789012123456789012345678901
different graphs and you should see the logic of how they are graphed. For 2
2
12345678901234567890123456789012123456789012345678901
2
2
12345678901234567890123456789012123456789012345678901
1
1
creates
an
upsideexample,
a
negative
squared
term
like
f(x)
5
2x
12345678901234567890123456789012123456789012345678901 22
12345678901234567890123456789012123456789012345678901
down hump. If there is an integer after the squared value, then this number
2
12345678901234567890123456789012123456789012345678901
12345678901234567890123456789012123456789012345678901
is the value of y where the crest of the hump occurs. You could see this with 2
2
12345678901234567890123456789012123456789012345678901
12345678901234567890123456789012123456789012345678901
f(x) 5 2x2 1 1 with a crest at 1 and f(x) 5 x2 2 2 with a crest at y 5 22. 2
2
1
2
12345678901234567890123456789012123456789012345678901
The last coordinate geometry concept to cover is the distance formula. It 2
12345678901234567890123456789012123456789012345678901
2
12345678901234567890123456789012123456789012345678901
allows you to find the distance between two points on the coordinate plane 2
12345678901234567890123456789012123456789012345678901
2
12345678901234567890123456789012123456789012345678901
using the following formula:
2
12345678901234567890123456789012123456789012345678901
2
12345678901234567890123456789012123456789012345678901
2
12345678901234567890123456789012123456789012345678901
2
2
2
12345678901234567890123456789012123456789012345678901
x
2
x
1
y
2
y
d
5
=
~
!
~
!
1
2
1
2
2
12345678901234567890123456789012123456789012345678901
2345678901234567890123456789012123456789012345678901
12345678901234567890123456789012123456789012345678901 22
12345678901234567890123456789012123456789012345678901
If you know two points, just plug in the x values and the y values into the
2
1
2
12345678901234567890123456789012123456789012345678901
equation and then solve for the distance. Its that simple.
12345678901234567890123456789012123456789012345678901 2
2345678901234567890123456789012123456789012345678901
12345678901234567890123456789012123456789012345678901 22
12345678901234567890123456789012123456789012345678901 2
12345678901234567890123456789012123456789012345678901 2
12345678901234567890123456789012123456789012345678901 2
1 If the distance formula reminds you of the Pythagorean theorem, youre
2
12345678901234567890123456789012123456789012345678901
2
12345678901234567890123456789012123456789012345678901
quite observant, because the distance formula is based on it.
2
12345678901234567890123456789012123456789012345678901
2
12345678901234567890123456789012123456789012345678901
12345678901234567890123456789012123456789012345678901 2
2345678901234567890123456789012123456789012345678901
12345678901234567890123456789012123456789012345678901 22
12345678901234567890123456789012123456789012345678901 2
12345678901234567890123456789012123456789012345678901 2
12345678901234567890123456789012123456789012345678901 2
1
2
12345678901234567890123456789012123456789012345678901
12345678901234567890123456789012123456789012345678901 2
2
12345678901234567890123456789012123456789012345678901
12345678901234567890123456789012123456789012345678901 2
2
12345678901234567890123456789012123456789012345678901
12345678901234567890123456789012123456789012345678901 2
12345678901234567890123456789012123456789012345678901 2
12345678901234567890123456789012123456789012345678901 2
12345678901234567890123456789012123456789012345678901 2 119
123456789012345678901234567890121234567890123456789012

http://www.xtremepapers.net

Part III: Section Review

123456789012345678901234567890121234567890123456789012
12345678901234567890123456789012123456789012345678901 2
2
12345678901234567890123456789012123456789012345678901
If A(2, 24) and B(3, 6) are two points on a rectangular coordinate
2
12345678901234567890123456789012123456789012345678901
2
12345678901234567890123456789012123456789012345678901
system, what is the distance between A and B?
2
12345678901234567890123456789012123456789012345678901
12345678901234567890123456789012123456789012345678901 2
2
12345678901234567890123456789012123456789012345678901
A. 1
2
12345678901234567890123456789012123456789012345678901
2
12345678901234567890123456789012123456789012345678901
B.
10
2
12345678901234567890123456789012123456789012345678901
2
12345678901234567890123456789012123456789012345678901
C.
101
2
12345678901234567890123456789012123456789012345678901
=
2345678901234567890123456789012123456789012345678901
2
1
D. 11
2
12345678901234567890123456789012123456789012345678901
2
12345678901234567890123456789012123456789012345678901
E. 15
2
12345678901234567890123456789012123456789012345678901
2
12345678901234567890123456789012123456789012345678901
2345678901234567890123456789012123456789012345678901
2
1 Hauling out the distance formula:
2
12345678901234567890123456789012123456789012345678901
2
12345678901234567890123456789012123456789012345678901
2
12345678901234567890123456789012123456789012345678901
2
2
2
2
2
12345678901234567890123456789012123456789012345678901
d 5 =~x1 2 x2! 1 ~y1 2 y2! 5 =~2 2 3! 1 ~24 2 6!
2345678901234567890123456789012123456789012345678901
2
1
2
12345678901234567890123456789012123456789012345678901
5 =1 1 100
2
12345678901234567890123456789012123456789012345678901
2
12345678901234567890123456789012123456789012345678901
2
12345678901234567890123456789012123456789012345678901
5
101
=
2345678901234567890123456789012123456789012345678901
12345678901234567890123456789012123456789012345678901 2
2
12345678901234567890123456789012123456789012345678901
This is choice (C).
2
12345678901234567890123456789012123456789012345678901
12345678901234567890123456789012123456789012345678901 2
12345678901234567890123456789012123456789012345678901
That might look like a big number, but it actually is just a little bit bigger 2
2
12345678901234567890123456789012123456789012345678901
1 than 10. If you visualize where the two points are, this answer makes 2
2
12345678901234567890123456789012123456789012345678901
2
12345678901234567890123456789012123456789012345678901
sense.
2
12345678901234567890123456789012123456789012345678901
2345678901234567890123456789012123456789012345678901
12345678901234567890123456789012123456789012345678901 2
12345678901234567890123456789012123456789012345678901 2
12345678901234567890123456789012123456789012345678901 2
2
12345678901234567890123456789012123456789012345678901
3-D Figures
2
12345678901234567890123456789012123456789012345678901
2
12345678901234567890123456789012123456789012345678901
Another way to up the geometry ante is by upping the number of 2
12345678901234567890123456789012123456789012345678901
2
12345678901234567890123456789012123456789012345678901
dimensions from two to three. The 3-D figures that you will come across 2
12345678901234567890123456789012123456789012345678901
1 on the GRE are all built upon the figures that were covered in the first 2
2
12345678901234567890123456789012123456789012345678901
12345678901234567890123456789012123456789012345678901
section (rectangles, squares, circles, triangles). The 2-D figures are given 2
2
12345678901234567890123456789012123456789012345678901
2
12345678901234567890123456789012123456789012345678901
height,
so
instead
of
dealing
just
with
a
square
you
now
get
a
cube,
a
3-D
2
12345678901234567890123456789012123456789012345678901
2345678901234567890123456789012123456789012345678901
2
12345678901234567890123456789012123456789012345678901
figure composed of six squares.
2
12345678901234567890123456789012123456789012345678901
2
1
2
12345678901234567890123456789012123456789012345678901
If
you
need
a
refresher
on
the
2-D
geometry,
buzz
through
the
geometry
2
12345678901234567890123456789012123456789012345678901
2345678901234567890123456789012123456789012345678901
12345678901234567890123456789012123456789012345678901
section earlier in this chapter. If not, go ahead and dive into the 3-D 2
2
12345678901234567890123456789012123456789012345678901
2
1 geometry.
12345678901234567890123456789012123456789012345678901 2
12345678901234567890123456789012123456789012345678901 2
2
12345678901234567890123456789012123456789012345678901
2
12345678901234567890123456789012123456789012345678901
2
12345678901234567890123456789012123456789012345678901
2
12345678901234567890123456789012123456789012345678901
12345678901234567890123456789012123456789012345678901 2
2345678901234567890123456789012123456789012345678901
12345678901234567890123456789012123456789012345678901 2
12345678901234567890123456789012123456789012345678901 2
12345678901234567890123456789012123456789012345678901 2
12345678901234567890123456789012123456789012345678901 2
2
1
2
12345678901234567890123456789012123456789012345678901
2
12345678901234567890123456789012123456789012345678901
2
12345678901234567890123456789012123456789012345678901
2
12345678901234567890123456789012123456789012345678901
12345678901234567890123456789012123456789012345678901 2
2345678901234567890123456789012123456789012345678901
12345678901234567890123456789012123456789012345678901 2
2
1
2
12345678901234567890123456789012123456789012345678901
12345678901234567890123456789012123456789012345678901 2
2
12345678901234567890123456789012123456789012345678901
12345678901234567890123456789012123456789012345678901 2
12345678901234567890123456789012123456789012345678901 2
12345678901234567890123456789012123456789012345678901 2
2
120 12345678901234567890123456789012123456789012345678901
123456789012345678901234567890121234567890123456789012

www.petersons.com

http://www.xtremepapers.net

Chapter 4: Quantitative Review

Take It to the Next Level

123456789012345678901234567890121234567890123456789012
12345678901234567890123456789012123456789012345678901 2
2
12345678901234567890123456789012123456789012345678901
12345678901234567890123456789012123456789012345678901 2
2
12345678901234567890123456789012123456789012345678901
2
12345678901234567890123456789012123456789012345678901
12345678901234567890123456789012123456789012345678901 2
2
12345678901234567890123456789012123456789012345678901
12345678901234567890123456789012123456789012345678901 2
2
12345678901234567890123456789012123456789012345678901
2
12345678901234567890123456789012123456789012345678901
2
12345678901234567890123456789012123456789012345678901
2
12345678901234567890123456789012123456789012345678901
2345678901234567890123456789012123456789012345678901
2
1
2
12345678901234567890123456789012123456789012345678901
2
12345678901234567890123456789012123456789012345678901
2
12345678901234567890123456789012123456789012345678901
2
12345678901234567890123456789012123456789012345678901
2345678901234567890123456789012123456789012345678901
2
1
2
12345678901234567890123456789012123456789012345678901
2
12345678901234567890123456789012123456789012345678901
2
12345678901234567890123456789012123456789012345678901
2
12345678901234567890123456789012123456789012345678901
2345678901234567890123456789012123456789012345678901
2
1
2
12345678901234567890123456789012123456789012345678901
2
12345678901234567890123456789012123456789012345678901
The
above
figure
is
a
rectangular
solid.
What
is
the
shortest
distance
2
12345678901234567890123456789012123456789012345678901
2
12345678901234567890123456789012123456789012345678901
from
A
to
F?
2345678901234567890123456789012123456789012345678901
12345678901234567890123456789012123456789012345678901 22
12345678901234567890123456789012123456789012345678901 2
12345678901234567890123456789012123456789012345678901
A. =18
2
12345678901234567890123456789012123456789012345678901
12345678901234567890123456789012123456789012345678901 22
12345678901234567890123456789012123456789012345678901
B. =20
2
1
2
12345678901234567890123456789012123456789012345678901
C. =28
2
12345678901234567890123456789012123456789012345678901
2
12345678901234567890123456789012123456789012345678901
2345678901234567890123456789012123456789012345678901
2
12345678901234567890123456789012123456789012345678901
D. =34
2
12345678901234567890123456789012123456789012345678901
12345678901234567890123456789012123456789012345678901 22
12345678901234567890123456789012123456789012345678901
E. 6
2
12345678901234567890123456789012123456789012345678901
12345678901234567890123456789012123456789012345678901 22
12345678901234567890123456789012123456789012345678901
The shortest distance between two points is a line, so the question is asking
12345678901234567890123456789012123456789012345678901 22
12345678901234567890123456789012123456789012345678901
to find the distance of the line AF. If you want a hint about how to find
2
1 you
it,
think
triangles.
2345678901234567890123456789012123456789012345678901
2
12345678901234567890123456789012123456789012345678901
12345678901234567890123456789012123456789012345678901 22
1 Notice that AF is the hypotenuse of DACF, which is a right triangle. If you
12345678901234567890123456789012123456789012345678901 2
12345678901234567890123456789012123456789012345678901
find AC, you can use it along with CF in the Pythagorean theorem to 2
2
12345678901234567890123456789012123456789012345678901
2
12345678901234567890123456789012123456789012345678901
find
AF.
12345678901234567890123456789012123456789012345678901 2
2
12345678901234567890123456789012123456789012345678901
2
12345678901234567890123456789012123456789012345678901
2345678901234567890123456789012123456789012345678901
12345678901234567890123456789012123456789012345678901 22
12345678901234567890123456789012123456789012345678901 2
1
12345678901234567890123456789012123456789012345678901 2
12345678901234567890123456789012123456789012345678901 2
2
12345678901234567890123456789012123456789012345678901
2
12345678901234567890123456789012123456789012345678901
2
12345678901234567890123456789012123456789012345678901
2
12345678901234567890123456789012123456789012345678901
12345678901234567890123456789012123456789012345678901 2
2345678901234567890123456789012123456789012345678901
12345678901234567890123456789012123456789012345678901 22
12345678901234567890123456789012123456789012345678901 2
12345678901234567890123456789012123456789012345678901 2
12345678901234567890123456789012123456789012345678901 2
1
2
12345678901234567890123456789012123456789012345678901
2
12345678901234567890123456789012123456789012345678901
2
12345678901234567890123456789012123456789012345678901
2
12345678901234567890123456789012123456789012345678901
12345678901234567890123456789012123456789012345678901 2
2345678901234567890123456789012123456789012345678901
12345678901234567890123456789012123456789012345678901 22
1
Look at the top surface of the figure. You should see a square with sides of 2
12345678901234567890123456789012123456789012345678901
2
12345678901234567890123456789012123456789012345678901
length 3, and the diagonal is the hypotenuse of a 45-45-90 right triangle, 2
12345678901234567890123456789012123456789012345678901
2345678901234567890123456789012123456789012345678901
2
1
12345678901234567890123456789012123456789012345678901 2
12345678901234567890123456789012123456789012345678901 2
12345678901234567890123456789012123456789012345678901 2 121
123456789012345678901234567890121234567890123456789012

http://www.xtremepapers.net

Part III: Section Review

123456789012345678901234567890121234567890123456789012
12345678901234567890123456789012123456789012345678901 2
2
12345678901234567890123456789012123456789012345678901
12345678901234567890123456789012123456789012345678901
just like it was earlier in this chapter. This makes AC 5 3=2, since it is 2
2
12345678901234567890123456789012123456789012345678901
2
12345678901234567890123456789012123456789012345678901
the
hypotenuse
of
a
45-45-90
triangle
with
smaller
sides
of
length
3.
12345678901234567890123456789012123456789012345678901 2
2
12345678901234567890123456789012123456789012345678901
2
12345678901234567890123456789012123456789012345678901
Put that value into the next Pythagorean theorem as follows:
2
12345678901234567890123456789012123456789012345678901
2
12345678901234567890123456789012123456789012345678901
2
2
2
2
12345678901234567890123456789012123456789012345678901
AF
5
AC
1
CF
~
!
~
!
~
!
2
12345678901234567890123456789012123456789012345678901
2
2
2345678901234567890123456789012123456789012345678901
2
1
~
AF! 5 ~3=2! 1 16 5 18 1 16
2
12345678901234567890123456789012123456789012345678901
2
12345678901234567890123456789012123456789012345678901
2
2
12345678901234567890123456789012123456789012345678901
AF
5
34
~
!
2
12345678901234567890123456789012123456789012345678901
2345678901234567890123456789012123456789012345678901
2
1
2
12345678901234567890123456789012123456789012345678901
AF 5 =34
2
12345678901234567890123456789012123456789012345678901
2
12345678901234567890123456789012123456789012345678901
2
12345678901234567890123456789012123456789012345678901
2345678901234567890123456789012123456789012345678901
2
1
2
12345678901234567890123456789012123456789012345678901
2
12345678901234567890123456789012123456789012345678901
2
12345678901234567890123456789012123456789012345678901
2
12345678901234567890123456789012123456789012345678901
2345678901234567890123456789012123456789012345678901
12345678901234567890123456789012123456789012345678901 2
12345678901234567890123456789012123456789012345678901 2
12345678901234567890123456789012123456789012345678901 2
12345678901234567890123456789012123456789012345678901 2
12345678901234567890123456789012123456789012345678901 2
12345678901234567890123456789012123456789012345678901 2
2
1
2
12345678901234567890123456789012123456789012345678901
2
12345678901234567890123456789012123456789012345678901
2
12345678901234567890123456789012123456789012345678901
2345678901234567890123456789012123456789012345678901
12345678901234567890123456789012123456789012345678901 2
12345678901234567890123456789012123456789012345678901 2
12345678901234567890123456789012123456789012345678901 2
12345678901234567890123456789012123456789012345678901 2
12345678901234567890123456789012123456789012345678901 2
12345678901234567890123456789012123456789012345678901 2
12345678901234567890123456789012123456789012345678901 2
12345678901234567890123456789012123456789012345678901 2
12345678901234567890123456789012123456789012345678901 2
2
1
2
12345678901234567890123456789012123456789012345678901
2
12345678901234567890123456789012123456789012345678901
12345678901234567890123456789012123456789012345678901 2
12345678901234567890123456789012123456789012345678901 2
2
12345678901234567890123456789012123456789012345678901
2
12345678901234567890123456789012123456789012345678901
2
12345678901234567890123456789012123456789012345678901
12345678901234567890123456789012123456789012345678901 2
2
12345678901234567890123456789012123456789012345678901
2
12345678901234567890123456789012123456789012345678901
2345678901234567890123456789012123456789012345678901
2
12345678901234567890123456789012123456789012345678901
If a sphere has a radius of r, what is the greatest distance along the
2
12345678901234567890123456789012123456789012345678901
2
1
surface of the sphere that two points could be from each other?
12345678901234567890123456789012123456789012345678901 2
2
12345678901234567890123456789012123456789012345678901
A. 2r
2
12345678901234567890123456789012123456789012345678901
2
12345678901234567890123456789012123456789012345678901
B. pr
2
12345678901234567890123456789012123456789012345678901
2
12345678901234567890123456789012123456789012345678901
C.
2pr
12345678901234567890123456789012123456789012345678901 2
2345678901234567890123456789012123456789012345678901
12345678901234567890123456789012123456789012345678901 2
2
12345678901234567890123456789012123456789012345678901
4
2
12345678901234567890123456789012123456789012345678901
pr
D.
2
12345678901234567890123456789012123456789012345678901
3
2
1
2345678901234567890123456789012123456789012345678901
2
12345678901234567890123456789012123456789012345678901
4 2
2
12345678901234567890123456789012123456789012345678901
2
12345678901234567890123456789012123456789012345678901
pr
E.
2
12345678901234567890123456789012123456789012345678901
3
2
1
2345678901234567890123456789012123456789012345678901
2
12345678901234567890123456789012123456789012345678901
1 Sometimes coming up with a real-life situation will help you understand an 2
12345678901234567890123456789012123456789012345678901
abstract geometrical one. For our purposes, think of the earth as a sphere. 2
2
12345678901234567890123456789012123456789012345678901
2
12345678901234567890123456789012123456789012345678901
In
real
terms,
the
questions
is
asking,
If
you
are
at
the
North
Pole,
what
2345678901234567890123456789012123456789012345678901
2
1
12345678901234567890123456789012123456789012345678901 2
12345678901234567890123456789012123456789012345678901 2
2
122 12345678901234567890123456789012123456789012345678901
123456789012345678901234567890121234567890123456789012

www.petersons.com

http://www.xtremepapers.net

Chapter 4: Quantitative Review

Take It to the Next Level

Alert!

123456789012345678901234567890121234567890123456789012
12345678901234567890123456789012123456789012345678901 2
2
12345678901234567890123456789012123456789012345678901
point on the earth is farthest from you if you had to travel along the 2
12345678901234567890123456789012123456789012345678901
2
12345678901234567890123456789012123456789012345678901
earths surface?
2
12345678901234567890123456789012123456789012345678901
12345678901234567890123456789012123456789012345678901 2
2
12345678901234567890123456789012123456789012345678901
12345678901234567890123456789012123456789012345678901 2
2
12345678901234567890123456789012123456789012345678901
2
12345678901234567890123456789012123456789012345678901
2
12345678901234567890123456789012123456789012345678901
Technically,
the
earth
is
an
oblate
spheroid,
not
a
sphere.
Thankfully,
2
12345678901234567890123456789012123456789012345678901
2345678901234567890123456789012123456789012345678901
2
1 oblate spheroids are NOT on the GRE.
2
12345678901234567890123456789012123456789012345678901
2
12345678901234567890123456789012123456789012345678901
2
12345678901234567890123456789012123456789012345678901
2
12345678901234567890123456789012123456789012345678901
2345678901234567890123456789012123456789012345678901
2
1
2
12345678901234567890123456789012123456789012345678901
2
12345678901234567890123456789012123456789012345678901
Viewed
in
this
way,
the
answer,
the
South
Pole,
should
be
obvious.
The
2
12345678901234567890123456789012123456789012345678901
2
12345678901234567890123456789012123456789012345678901
same
is
true
for
the
abstract
sphere
question.
To
determine
that
distance,
2345678901234567890123456789012123456789012345678901
2
1
2
12345678901234567890123456789012123456789012345678901
imagine
drawing
a
circle
on
the
surface
of
the
sphere
that
runs
through
its
2
12345678901234567890123456789012123456789012345678901
2
12345678901234567890123456789012123456789012345678901
North
and
South
Pole.
You
can
see
then
that
those
two
points
are
a
half
2
12345678901234567890123456789012123456789012345678901
2345678901234567890123456789012123456789012345678901
12345678901234567890123456789012123456789012345678901
circumference of that circle from each other. The radius of that circle is r, 2
2
12345678901234567890123456789012123456789012345678901
12345678901234567890123456789012123456789012345678901
the same as the radius of the sphere. This makes the circumference 2pr, 2
12345678901234567890123456789012123456789012345678901 22
12345678901234567890123456789012123456789012345678901
and half of that is pr. Thats choice (B).
2
12345678901234567890123456789012123456789012345678901
2
1
2
12345678901234567890123456789012123456789012345678901
2
12345678901234567890123456789012123456789012345678901
2
12345678901234567890123456789012123456789012345678901
Data
Analysis
2345678901234567890123456789012123456789012345678901
12345678901234567890123456789012123456789012345678901 22
12345678901234567890123456789012123456789012345678901 2
12345678901234567890123456789012123456789012345678901
Measures of Central Location and Dispersion (or
2
12345678901234567890123456789012123456789012345678901
2
12345678901234567890123456789012123456789012345678901
How
Close
and
Far
Apart
Stuff
Is)
12345678901234567890123456789012123456789012345678901 22
12345678901234567890123456789012123456789012345678901 2
12345678901234567890123456789012123456789012345678901
Mean, median, and mode were covered earlier in this chapter. These are
2
12345678901234567890123456789012123456789012345678901
1 notions from statistics that give different ways to measure the central 2
2345678901234567890123456789012123456789012345678901
2
12345678901234567890123456789012123456789012345678901
location of a number set. Basically, they are different ways to say where the 2
12345678901234567890123456789012123456789012345678901
2
1 middle of a bunch of numbers is.
12345678901234567890123456789012123456789012345678901 2
2
12345678901234567890123456789012123456789012345678901
12345678901234567890123456789012123456789012345678901
The dispersion of a number set is the opposite of the middleness or 2
2
12345678901234567890123456789012123456789012345678901
2
12345678901234567890123456789012123456789012345678901
clusteredness
of
a
number
set.
The
range
is
one
measure
of
the
2
12345678901234567890123456789012123456789012345678901
2
12345678901234567890123456789012123456789012345678901
dispersion
since
it
tells
you
how
far
apart
the
extremes
are.
Standard
2345678901234567890123456789012123456789012345678901
12345678901234567890123456789012123456789012345678901 22
12345678901234567890123456789012123456789012345678901
1 deviation is another measure of dispersion. It measures on average how 2
12345678901234567890123456789012123456789012345678901
much numbers of a set differ from the middle of that set (basically how 2
2
12345678901234567890123456789012123456789012345678901
2345678901234567890123456789012123456789012345678901
12345678901234567890123456789012123456789012345678901
spread out the numbers are). There is an involved way to calculate the 2
2
12345678901234567890123456789012123456789012345678901
12345678901234567890123456789012123456789012345678901
standard deviation, but you dont need to bother with that. Standard 2
12345678901234567890123456789012123456789012345678901 22
1 deviation, though, might appear on a question, and so you will need to
2
12345678901234567890123456789012123456789012345678901
2
12345678901234567890123456789012123456789012345678901
grasp the concept. To that end, consider the following problem.
2
12345678901234567890123456789012123456789012345678901
2
12345678901234567890123456789012123456789012345678901
2
Column B
Column A
12345678901234567890123456789012123456789012345678901
2345678901234567890123456789012123456789012345678901
12345678901234567890123456789012123456789012345678901 22
12345678901234567890123456789012123456789012345678901
Standard Deviation
Standard Deviation
2
12345678901234567890123456789012123456789012345678901
2
12345678901234567890123456789012123456789012345678901
of
this
number
set
of
this
number
set
2
1
2345678901234567890123456789012123456789012345678901
2
12345678901234567890123456789012123456789012345678901
[5, 10, 2, 4]
[1, 14, 3, 22]
2
1
2
12345678901234567890123456789012123456789012345678901
2
12345678901234567890123456789012123456789012345678901
You
dont
need
to
actually
calculate
the
two
standard
deviations
to
answer
2
12345678901234567890123456789012123456789012345678901
2345678901234567890123456789012123456789012345678901
1 this question. Estimate the mean of the number set in Column A. Its 2
12345678901234567890123456789012123456789012345678901 2
12345678901234567890123456789012123456789012345678901 2
12345678901234567890123456789012123456789012345678901 2 123
123456789012345678901234567890121234567890123456789012

http://www.xtremepapers.net

Part III: Section Review

123456789012345678901234567890121234567890123456789012
12345678901234567890123456789012123456789012345678901 2
2
12345678901234567890123456789012123456789012345678901
probably around 4 or 5, and the numbers in the set deviate from this by, at 2
12345678901234567890123456789012123456789012345678901
2
12345678901234567890123456789012123456789012345678901
most, 6. So, the standard deviation will be something less than 6. Now 2
12345678901234567890123456789012123456789012345678901
2
12345678901234567890123456789012123456789012345678901
estimate the mean for Column B. Its probably around 11, and the number 2
12345678901234567890123456789012123456789012345678901
2345678901234567890123456789012123456789012345678901
1 set deviates from this by 10 or more. The standard deviation is going to be 2
2
12345678901234567890123456789012123456789012345678901
2
12345678901234567890123456789012123456789012345678901
greater
in
Column
B.
The
answer
will
be
Column
B,
in
part
because
the
2
12345678901234567890123456789012123456789012345678901
2
12345678901234567890123456789012123456789012345678901
numbers
in
Column
B
are
more
spread
out
than
the
numbers
in
Column
A.
2345678901234567890123456789012123456789012345678901
2
1
2
12345678901234567890123456789012123456789012345678901
2
12345678901234567890123456789012123456789012345678901
Standard
deviation
is
not
the
only
way
to
make
these
statistics
concepts
2
12345678901234567890123456789012123456789012345678901
2
12345678901234567890123456789012123456789012345678901
interesting.
The
test-writers
can
craft
difficult
problems
with
median,
2345678901234567890123456789012123456789012345678901
2
1
2
12345678901234567890123456789012123456789012345678901
mode,
range,
and
average.
2
12345678901234567890123456789012123456789012345678901
2
12345678901234567890123456789012123456789012345678901
2
12345678901234567890123456789012123456789012345678901
Column B
Column A
2345678901234567890123456789012123456789012345678901
2
1
2
12345678901234567890123456789012123456789012345678901
A number set is composed of five non2
12345678901234567890123456789012123456789012345678901
2
12345678901234567890123456789012123456789012345678901
negative integers, and its average is 20.
2
12345678901234567890123456789012123456789012345678901
2345678901234567890123456789012123456789012345678901
12345678901234567890123456789012123456789012345678901 2
2
12345678901234567890123456789012123456789012345678901
The sum of the
The greatest
2
12345678901234567890123456789012123456789012345678901
2
12345678901234567890123456789012123456789012345678901
possible
range
of
number
set
12345678901234567890123456789012123456789012345678901 2
2
12345678901234567890123456789012123456789012345678901
the number set
2
1
2
12345678901234567890123456789012123456789012345678901
2
12345678901234567890123456789012123456789012345678901
12345678901234567890123456789012123456789012345678901
Start with Column B. The sum of the number set is the average times the 2
2345678901234567890123456789012123456789012345678901
12345678901234567890123456789012123456789012345678901 2
2
12345678901234567890123456789012123456789012345678901
number of elements:
2
12345678901234567890123456789012123456789012345678901
2
12345678901234567890123456789012123456789012345678901
sum
2
12345678901234567890123456789012123456789012345678901
5
average

sum
5
5
3
20
5
100
2
12345678901234567890123456789012123456789012345678901
5
2
12345678901234567890123456789012123456789012345678901
12345678901234567890123456789012123456789012345678901 2
2
12345678901234567890123456789012123456789012345678901
2
1 Column B is 100.
2345678901234567890123456789012123456789012345678901
12345678901234567890123456789012123456789012345678901 2
12345678901234567890123456789012123456789012345678901
As for Column A, the greatest possible range will maximize the highest 2
2
1
12345678901234567890123456789012123456789012345678901
element and minimize the lowest element. The set is composed of 2
2
12345678901234567890123456789012123456789012345678901
non-negative integers, so the smallest element possible is 0. There is no 2
12345678901234567890123456789012123456789012345678901
2
12345678901234567890123456789012123456789012345678901
mention that the elements are distinct, so four of the elements could be 2
12345678901234567890123456789012123456789012345678901
2
12345678901234567890123456789012123456789012345678901
zero. In that case, the fifth element would have to be 100 for the sum to be 2
12345678901234567890123456789012123456789012345678901
2345678901234567890123456789012123456789012345678901
2
12345678901234567890123456789012123456789012345678901
100. 100 is actually the greatest that a number in the set could be, since the 2
12345678901234567890123456789012123456789012345678901
1 sum cannot exceed 100 and there are no negative numbers to add. So the 2
12345678901234567890123456789012123456789012345678901 2
12345678901234567890123456789012123456789012345678901
greatest possible range is 100 2 0 5 100. Column A and B are equal, 2
2
12345678901234567890123456789012123456789012345678901
2
12345678901234567890123456789012123456789012345678901
making
(C)
the
correct
answer.
2
12345678901234567890123456789012123456789012345678901
2
12345678901234567890123456789012123456789012345678901
12345678901234567890123456789012123456789012345678901 2
2345678901234567890123456789012123456789012345678901
12345678901234567890123456789012123456789012345678901 2
12345678901234567890123456789012123456789012345678901 2
2
12345678901234567890123456789012123456789012345678901
Counting Stuff
2
12345678901234567890123456789012123456789012345678901
1 There are a few different kinds of word problems on the GRE that involve 2
2345678901234567890123456789012123456789012345678901
12345678901234567890123456789012123456789012345678901 2
12345678901234567890123456789012123456789012345678901
counting possible outcomes or permutations of a set of items. Each has a 2
2
12345678901234567890123456789012123456789012345678901
2
12345678901234567890123456789012123456789012345678901
different kind of way to solve them, so well review them one by one.
2
1
2345678901234567890123456789012123456789012345678901
12345678901234567890123456789012123456789012345678901 2
2
1
2
12345678901234567890123456789012123456789012345678901
Different
Ways
to
Count
Stuff
12345678901234567890123456789012123456789012345678901 2
12345678901234567890123456789012123456789012345678901
1. If one task has n possible outcomes and a second one has m possible 2
2
12345678901234567890123456789012123456789012345678901
2
12345678901234567890123456789012123456789012345678901
outcomes,
then
the
joint
occurrence
of
the
two
tasks
has
n
3
m
possible
12345678901234567890123456789012123456789012345678901 2
2
124 12345678901234567890123456789012123456789012345678901
123456789012345678901234567890121234567890123456789012

www.petersons.com

http://www.xtremepapers.net

Chapter 4: Quantitative Review

Take It to the Next Level

123456789012345678901234567890121234567890123456789012
12345678901234567890123456789012123456789012345678901 2
2
12345678901234567890123456789012123456789012345678901
outcomes. That postulate was a bit vague, so consider an example. If 2
12345678901234567890123456789012123456789012345678901
2
12345678901234567890123456789012123456789012345678901
between Town A and Town B there are four roads, and between Town B 2
12345678901234567890123456789012123456789012345678901
2
12345678901234567890123456789012123456789012345678901
and Town C there are five roads, how many different routes are there from 2
12345678901234567890123456789012123456789012345678901
2345678901234567890123456789012123456789012345678901
1 Town A to Town C that travel through Town B? One way to attack this 2
2
12345678901234567890123456789012123456789012345678901
2
12345678901234567890123456789012123456789012345678901
problem
is
to
sketch
out
the
scenario
and
start
counting
up
the
2
12345678901234567890123456789012123456789012345678901
2
12345678901234567890123456789012123456789012345678901
possibilities.
The
other
way
is
to
use
our
nifty
little
formula,
n
3
m
5
2345678901234567890123456789012123456789012345678901
2
1
2
12345678901234567890123456789012123456789012345678901
4
3
5
5
20.
2
12345678901234567890123456789012123456789012345678901
2
12345678901234567890123456789012123456789012345678901
2
12345678901234567890123456789012123456789012345678901
2.
Permutation
problems
prompt
you
to
determine
how
many
different
2345678901234567890123456789012123456789012345678901
2
1
2
12345678901234567890123456789012123456789012345678901
permutations,
or
outcomes,
are
possible.
A
permutation
word
problem
2
12345678901234567890123456789012123456789012345678901
2
12345678901234567890123456789012123456789012345678901
might
ask,
If
there
are
six
car
spots
at
the
Destin
Apartments
and
six
2
12345678901234567890123456789012123456789012345678901
2345678901234567890123456789012123456789012345678901
1 different cars owned by the renters, how many different arrangements of 2
2
12345678901234567890123456789012123456789012345678901
12345678901234567890123456789012123456789012345678901
cars are possible in the parking lot? You solve this problem by 2
2
12345678901234567890123456789012123456789012345678901
12345678901234567890123456789012123456789012345678901
multiplying the number of possibilities for the first spot (6 cars can park 2
2345678901234567890123456789012123456789012345678901
12345678901234567890123456789012123456789012345678901 22
12345678901234567890123456789012123456789012345678901
there) times the number of remaining possibilities for the second spot (5
2
12345678901234567890123456789012123456789012345678901
cars can if the first spot is filled) times the number of remaining 2
12345678901234567890123456789012123456789012345678901
2
12345678901234567890123456789012123456789012345678901
possibilities for the third spot (4) and so on. In other words, 6 3 5 3 4 3 2
12345678901234567890123456789012123456789012345678901
1 3 3 2 3 1. This is the same thing as 6!, our good friend the factorial. In 2
2
12345678901234567890123456789012123456789012345678901
2
12345678901234567890123456789012123456789012345678901
general,
when
you
come
across
a
permutation
problem
like
this
one,
you
2
12345678901234567890123456789012123456789012345678901
2345678901234567890123456789012123456789012345678901
12345678901234567890123456789012123456789012345678901
can use this factorial shortcut rather than going through all the logic. The 2
2
12345678901234567890123456789012123456789012345678901
2
12345678901234567890123456789012123456789012345678901
basic
formula
is
n!;
the
number
of
ways
that
n
different
objects
can
be
12345678901234567890123456789012123456789012345678901 22
12345678901234567890123456789012123456789012345678901
ordered is n!.
2
12345678901234567890123456789012123456789012345678901
12345678901234567890123456789012123456789012345678901 22
12345678901234567890123456789012123456789012345678901
3. The last kind of counting problem involves determining the number of 2
12345678901234567890123456789012123456789012345678901
1 different ways that sets of items can be selected from a larger set of items. 2
2
12345678901234567890123456789012123456789012345678901
2
12345678901234567890123456789012123456789012345678901
A
good
example
of
this
is
selecting
committees
of
three
people
from
a
12345678901234567890123456789012123456789012345678901 2
12345678901234567890123456789012123456789012345678901
group of five people. How many different ways can this be done? You 2
2
12345678901234567890123456789012123456789012345678901
2345678901234567890123456789012123456789012345678901
12345678901234567890123456789012123456789012345678901
could try to work out the possibilities with a systematic use of logic, or you 2
2
12345678901234567890123456789012123456789012345678901
1 could use the handy-dandy formula that has all the logic packed into it. 2
2
12345678901234567890123456789012123456789012345678901
2
n!
12345678901234567890123456789012123456789012345678901
2345678901234567890123456789012123456789012345678901
12345678901234567890123456789012123456789012345678901
, where n is the size of the larger set, and r is the 2
The formula is
2
12345678901234567890123456789012123456789012345678901
n
2
r
!r!
~
!
2
1 size of the groups
being
selected.
In
our
example,
the
formula
would
be
12345678901234567890123456789012123456789012345678901 2
2
12345678901234567890123456789012123456789012345678901
applied like this:
2
12345678901234567890123456789012123456789012345678901
2
12345678901234567890123456789012123456789012345678901
2
12345678901234567890123456789012123456789012345678901
5
3
4
3
3!
20
5!
2
12345678901234567890123456789012123456789012345678901
5
5
5
10
2
12345678901234567890123456789012123456789012345678901
5
2
3
!3!
2!3!
2!
~
!
2345678901234567890123456789012123456789012345678901
12345678901234567890123456789012123456789012345678901 22
12345678901234567890123456789012123456789012345678901 2
12345678901234567890123456789012123456789012345678901 2
12345678901234567890123456789012123456789012345678901 2
1
2
12345678901234567890123456789012123456789012345678901
Probability
2
12345678901234567890123456789012123456789012345678901
2
12345678901234567890123456789012123456789012345678901
The
basics
of
single
and
multi-event
probability
were
covered
in
the
first
2
12345678901234567890123456789012123456789012345678901
2
12345678901234567890123456789012123456789012345678901
section.
Yet
there
are
some
multi-event
probability
scenarios
that
go
2345678901234567890123456789012123456789012345678901
12345678901234567890123456789012123456789012345678901 22
1 beyond the basic. If two events are independent, then the probability of
2
12345678901234567890123456789012123456789012345678901
12345678901234567890123456789012123456789012345678901
both events occurring is the product of the two individual probabilities. 2
2
12345678901234567890123456789012123456789012345678901
12345678901234567890123456789012123456789012345678901
Now suppose you know the probabilities of event A and event B 2
2
12345678901234567890123456789012123456789012345678901
12345678901234567890123456789012123456789012345678901 22
12345678901234567890123456789012123456789012345678901 125
123456789012345678901234567890121234567890123456789012

http://www.xtremepapers.net

Part III: Section Review

123456789012345678901234567890121234567890123456789012
12345678901234567890123456789012123456789012345678901 2
2
12345678901234567890123456789012123456789012345678901
individually and want to find the probabilities of either A or B occurring 2
12345678901234567890123456789012123456789012345678901
2
12345678901234567890123456789012123456789012345678901
(as opposed to the probability of A and B occurring).
2
12345678901234567890123456789012123456789012345678901
12345678901234567890123456789012123456789012345678901 2
2
12345678901234567890123456789012123456789012345678901
A fundamental rule of probability is:
2
12345678901234567890123456789012123456789012345678901
2
12345678901234567890123456789012123456789012345678901
2
12345678901234567890123456789012123456789012345678901
P(A
or
B)
5
P(A)
1
P(B)
2
P(A)P(B)
2
12345678901234567890123456789012123456789012345678901
2
12345678901234567890123456789012123456789012345678901
2345678901234567890123456789012123456789012345678901
1 Suppose there is a one-in-two chance of event A occurring and a 2
2
12345678901234567890123456789012123456789012345678901
12345678901234567890123456789012123456789012345678901
two-in-five chance for event B. Expressed in decimal terms, this makes the 2
2
12345678901234567890123456789012123456789012345678901
2
12345678901234567890123456789012123456789012345678901
1
2
2345678901234567890123456789012123456789012345678901
1 probability of A 5 0.5
and probability of B 5 0.4
. Putting these 2
2
12345678901234567890123456789012123456789012345678901
2
5
2
12345678901234567890123456789012123456789012345678901
values
into
the
formula
gets
you:
2
12345678901234567890123456789012123456789012345678901
2
12345678901234567890123456789012123456789012345678901
2345678901234567890123456789012123456789012345678901
2
1
P ~A or B! 5 P~A! 1 P~B! 2 P~A!P~B!
2
12345678901234567890123456789012123456789012345678901
2
12345678901234567890123456789012123456789012345678901
2
12345678901234567890123456789012123456789012345678901
P ~A or B! 5 0.5 1 0.4 2 ~0.5!~0.4!
2
12345678901234567890123456789012123456789012345678901
2345678901234567890123456789012123456789012345678901
2
12345678901234567890123456789012123456789012345678901
P ~A or B! 5 0.9 2 0.2 5 0.7
2
12345678901234567890123456789012123456789012345678901
12345678901234567890123456789012123456789012345678901 2
12345678901234567890123456789012123456789012345678901
0.7 translates to a seven-in-ten chance, showing that the probability of 2
2
12345678901234567890123456789012123456789012345678901
2
12345678901234567890123456789012123456789012345678901
1 A or B occurring is larger than the probability of either single event 2
2
12345678901234567890123456789012123456789012345678901
occurring.
2
12345678901234567890123456789012123456789012345678901
2
12345678901234567890123456789012123456789012345678901
2345678901234567890123456789012123456789012345678901
2
12345678901234567890123456789012123456789012345678901
A bag of marbles contains 12 red marbles, 8 blue marbles, and 5
2
12345678901234567890123456789012123456789012345678901
2
12345678901234567890123456789012123456789012345678901
green
marbles.
What
is
the
probability
that
a
green
and
then
a
blue
12345678901234567890123456789012123456789012345678901 2
2
12345678901234567890123456789012123456789012345678901
marble will be chosen in two attempts?
2
12345678901234567890123456789012123456789012345678901
12345678901234567890123456789012123456789012345678901 2
2
12345678901234567890123456789012123456789012345678901
1
2
12345678901234567890123456789012123456789012345678901
A.
2
1
15
2345678901234567890123456789012123456789012345678901
12345678901234567890123456789012123456789012345678901 2
2
12345678901234567890123456789012123456789012345678901
1
2
1
B.
2
12345678901234567890123456789012123456789012345678901
10
2
12345678901234567890123456789012123456789012345678901
2345678901234567890123456789012123456789012345678901
12345678901234567890123456789012123456789012345678901 2
2
12345678901234567890123456789012123456789012345678901
1
2
1
C.
2
12345678901234567890123456789012123456789012345678901
5
2
12345678901234567890123456789012123456789012345678901
2345678901234567890123456789012123456789012345678901
2
12345678901234567890123456789012123456789012345678901
12
2
12345678901234567890123456789012123456789012345678901
D.
2
1
25
12345678901234567890123456789012123456789012345678901 2
2
12345678901234567890123456789012123456789012345678901
13
2
12345678901234567890123456789012123456789012345678901
2
12345678901234567890123456789012123456789012345678901
E.
25
2
12345678901234567890123456789012123456789012345678901
2
12345678901234567890123456789012123456789012345678901
2
12345678901234567890123456789012123456789012345678901
Determine
the
probability
of
each
event
individually.
The
probability
of
2345678901234567890123456789012123456789012345678901
12345678901234567890123456789012123456789012345678901 2
2
12345678901234567890123456789012123456789012345678901
the first event is:
2
12345678901234567890123456789012123456789012345678901
12345678901234567890123456789012123456789012345678901 2
2
1
5
5
1
2
12345678901234567890123456789012123456789012345678901
5
5
2
12345678901234567890123456789012123456789012345678901
12
1
8
1
5
25
5
2
12345678901234567890123456789012123456789012345678901
2
12345678901234567890123456789012123456789012345678901
2
12345678901234567890123456789012123456789012345678901
The
numerator,
5,
is
the
number
of
green
marbles
divided
by
the
total
2345678901234567890123456789012123456789012345678901
12345678901234567890123456789012123456789012345678901 2
1 number of marbles. For the second probability, you must remember to 2
2
12345678901234567890123456789012123456789012345678901
12345678901234567890123456789012123456789012345678901
take the first transaction into account. This means there are only 24 total 2
2
12345678901234567890123456789012123456789012345678901
2
12345678901234567890123456789012123456789012345678901
marbles and 4 green marbles.
2
12345678901234567890123456789012123456789012345678901
12345678901234567890123456789012123456789012345678901 2
2
126 12345678901234567890123456789012123456789012345678901
123456789012345678901234567890121234567890123456789012

SD

SD

www.petersons.com

http://www.xtremepapers.net

123456789012345678901234567890121234567890123456789012
12345678901234567890123456789012123456789012345678901 2
2
12345678901234567890123456789012123456789012345678901
This makes the blue marble equation:
2
12345678901234567890123456789012123456789012345678901
2
12345678901234567890123456789012123456789012345678901
2
12345678901234567890123456789012123456789012345678901
2
12345678901234567890123456789012123456789012345678901
8
8
1
2
12345678901234567890123456789012123456789012345678901
5
5
2345678901234567890123456789012123456789012345678901
2
1
12 1 8 1 4 24 3
2
12345678901234567890123456789012123456789012345678901
2
12345678901234567890123456789012123456789012345678901
The probability of both occurring is the product of the individual 2
12345678901234567890123456789012123456789012345678901
2
12345678901234567890123456789012123456789012345678901
probabilities:
2
12345678901234567890123456789012123456789012345678901
2
12345678901234567890123456789012123456789012345678901
2
12345678901234567890123456789012123456789012345678901
2
12345678901234567890123456789012123456789012345678901
1 1
1
2
12345678901234567890123456789012123456789012345678901
3 5
2345678901234567890123456789012123456789012345678901
2
1
3 5 15
2
12345678901234567890123456789012123456789012345678901
2
12345678901234567890123456789012123456789012345678901
The correct answer is (A).
2
12345678901234567890123456789012123456789012345678901
2
12345678901234567890123456789012123456789012345678901
2345678901234567890123456789012123456789012345678901
2
1
2
12345678901234567890123456789012123456789012345678901
2
12345678901234567890123456789012123456789012345678901
Given the same scenario, what would the probability of picking a green
2
12345678901234567890123456789012123456789012345678901
2
12345678901234567890123456789012123456789012345678901
marble
first
or
a
blue
marble
second?
2345678901234567890123456789012123456789012345678901
12345678901234567890123456789012123456789012345678901 22
12345678901234567890123456789012123456789012345678901 2
12345678901234567890123456789012123456789012345678901
P ~A or B! 5 P~A! 1 P~B! 2 P~A!P~B!
2
12345678901234567890123456789012123456789012345678901
12345678901234567890123456789012123456789012345678901 22
12345678901234567890123456789012123456789012345678901
1 1
1
2
1
5 1 2
2
12345678901234567890123456789012123456789012345678901
5 3 15
2
12345678901234567890123456789012123456789012345678901
2
12345678901234567890123456789012123456789012345678901
8
1
2345678901234567890123456789012123456789012345678901
2
12345678901234567890123456789012123456789012345678901
5
2
2
12345678901234567890123456789012123456789012345678901
15
15
12345678901234567890123456789012123456789012345678901 22
12345678901234567890123456789012123456789012345678901
7
2
12345678901234567890123456789012123456789012345678901
2
12345678901234567890123456789012123456789012345678901
5
2
12345678901234567890123456789012123456789012345678901
15
12345678901234567890123456789012123456789012345678901 22
12345678901234567890123456789012123456789012345678901
2
1 .
2
12345678901234567890123456789012123456789012345678901
2
12345678901234567890123456789012123456789012345678901
2
12345678901234567890123456789012123456789012345678901
Column A
Column B
12345678901234567890123456789012123456789012345678901 2
2
12345678901234567890123456789012123456789012345678901
There are 24 students in a class. Half are
2
12345678901234567890123456789012123456789012345678901
2
12345678901234567890123456789012123456789012345678901
male and half are female. At random,
2
12345678901234567890123456789012123456789012345678901
2
12345678901234567890123456789012123456789012345678901
3
students
are
chosen
successively
2
12345678901234567890123456789012123456789012345678901
2345678901234567890123456789012123456789012345678901
2
12345678901234567890123456789012123456789012345678901
from the class.
2
12345678901234567890123456789012123456789012345678901
2
1
2
12345678901234567890123456789012123456789012345678901
The
probability
of
a
The
probability
of
a
12345678901234567890123456789012123456789012345678901 2
2345678901234567890123456789012123456789012345678901
2
12345678901234567890123456789012123456789012345678901
boy, then a girl,
girl, then a boy,
2
12345678901234567890123456789012123456789012345678901
2
12345678901234567890123456789012123456789012345678901
then
a
girl,
then
a
girl
12345678901234567890123456789012123456789012345678901 22
1
being chosen
being chosen
2
12345678901234567890123456789012123456789012345678901
2
12345678901234567890123456789012123456789012345678901
These are both and situations, not or situations. The probability of either 2
12345678901234567890123456789012123456789012345678901
2
12345678901234567890123456789012123456789012345678901
first event is 0.5 since the class is evenly split. The second event in both 2
12345678901234567890123456789012123456789012345678901
2345678901234567890123456789012123456789012345678901
2
12345678901234567890123456789012123456789012345678901
columns then switches genders, so again the probability would be the 2
12345678901234567890123456789012123456789012345678901
2
12345678901234567890123456789012123456789012345678901
12
2
12345678901234567890123456789012123456789012345678901
.
For
the
third
event,
in
both
columns
a
girl
and
a
boy
have
same,
2
1
23
2345678901234567890123456789012123456789012345678901
12345678901234567890123456789012123456789012345678901 22
1 already been chosen, so the probability of picking a male or female is again
2
12345678901234567890123456789012123456789012345678901
2
12345678901234567890123456789012123456789012345678901
the
same.
So
the
third
events
also
have
the
same
probability.
The
products
2
12345678901234567890123456789012123456789012345678901
2345678901234567890123456789012123456789012345678901
2
1 of equal factors are equal, so (C) is the correct answer.
12345678901234567890123456789012123456789012345678901 2
12345678901234567890123456789012123456789012345678901 2
12345678901234567890123456789012123456789012345678901 2 127
123456789012345678901234567890121234567890123456789012

Take It to the Next Level

Note

Chapter 4: Quantitative Review

http://www.xtremepapers.net

Part III: Section Review

123456789012345678901234567890121234567890123456789012
12345678901234567890123456789012123456789012345678901 2
2
12345678901234567890123456789012123456789012345678901
2
12345678901234567890123456789012123456789012345678901
Get Graphical with the Data
2
12345678901234567890123456789012123456789012345678901
2
12345678901234567890123456789012123456789012345678901
You
will
see
one
or
two
sets
of
graphs
and
charts
on
the
GRE
math
section.
12345678901234567890123456789012123456789012345678901 2
12345678901234567890123456789012123456789012345678901
Each set will have 2 to 4 questions, so understanding and interpreting the 2
2
12345678901234567890123456789012123456789012345678901
12345678901234567890123456789012123456789012345678901
graph sets are important. As was said in the first section, there is good 2
2
12345678901234567890123456789012123456789012345678901
12345678901234567890123456789012123456789012345678901
news and bad news for this. The good news is that the answers for the 2
2
12345678901234567890123456789012123456789012345678901
12345678901234567890123456789012123456789012345678901
graph set questions are always right in front of you. The bad news is that 2
2
12345678901234567890123456789012123456789012345678901
12345678901234567890123456789012123456789012345678901
the GRE can come up with some pretty complex graph sets that make 2
2
12345678901234567890123456789012123456789012345678901
finding the answers a challenge. The way to combat this complexifying 2
12345678901234567890123456789012123456789012345678901
2345678901234567890123456789012123456789012345678901
1 is to spend 10 to 20 seconds familiarizing yourself with a graph set when it 2
2
12345678901234567890123456789012123456789012345678901
2
12345678901234567890123456789012123456789012345678901
comes up. Scroll all the way down so that you see all the pertinent 2
12345678901234567890123456789012123456789012345678901
2
12345678901234567890123456789012123456789012345678901
information. Look at any key. Read the titles of everything. Inspect graph 2
12345678901234567890123456789012123456789012345678901
12345678901234567890123456789012123456789012345678901
axes so that you know with what units the information is being presented. 2
2
12345678901234567890123456789012123456789012345678901
2
12345678901234567890123456789012123456789012345678901
Basically,
get
comfortable
with
the
graph
sets.
This
way,
when
you
start
on
2
12345678901234567890123456789012123456789012345678901
2345678901234567890123456789012123456789012345678901
12345678901234567890123456789012123456789012345678901
the questions you will be ready to hone in on whatever information a 2
2
12345678901234567890123456789012123456789012345678901
2
12345678901234567890123456789012123456789012345678901
question
asks
for.
12345678901234567890123456789012123456789012345678901 2
12345678901234567890123456789012123456789012345678901 2
2
12345678901234567890123456789012123456789012345678901
1 There are two ways that the GRE increases the difficulty of these kinds of 2
12345678901234567890123456789012123456789012345678901
problems. The first is to give you wackier or more unconventional graph 2
2
12345678901234567890123456789012123456789012345678901
12345678901234567890123456789012123456789012345678901
sets. Instead of getting just a line graph and a bar graph, you might get a 2
2345678901234567890123456789012123456789012345678901
12345678901234567890123456789012123456789012345678901 2
12345678901234567890123456789012123456789012345678901
line graph that has a right and left axis with different figures graphed, and 2
2
12345678901234567890123456789012123456789012345678901
12345678901234567890123456789012123456789012345678901
also a vertical pie chart. The second way the test-makers increase the 2
12345678901234567890123456789012123456789012345678901 2
12345678901234567890123456789012123456789012345678901
difficulty is by increasing the number of steps needed to find the answer. 2
2
12345678901234567890123456789012123456789012345678901
Instead of just having to read the answer off a bar graph, you might have 2
12345678901234567890123456789012123456789012345678901
2
12345678901234567890123456789012123456789012345678901
1 to take information from both graphs and then do a little math. Although 2
2345678901234567890123456789012123456789012345678901
2
12345678901234567890123456789012123456789012345678901
these multi-stage problems take time, graph set questions will not involve 2
12345678901234567890123456789012123456789012345678901
1 more than two or three steps. The GRE tests your ability to quickly and 2
12345678901234567890123456789012123456789012345678901 2
12345678901234567890123456789012123456789012345678901
correctly interpret graph sets, not your ability to do sophisticated analysis 2
2
12345678901234567890123456789012123456789012345678901
2
12345678901234567890123456789012123456789012345678901
of the information presented.
2
12345678901234567890123456789012123456789012345678901
2
12345678901234567890123456789012123456789012345678901
2
12345678901234567890123456789012123456789012345678901
2345678901234567890123456789012123456789012345678901
12345678901234567890123456789012123456789012345678901 2
12345678901234567890123456789012123456789012345678901 2
2
1
12345678901234567890123456789012123456789012345678901 2
12345678901234567890123456789012123456789012345678901 2
2
12345678901234567890123456789012123456789012345678901
2
12345678901234567890123456789012123456789012345678901
2
12345678901234567890123456789012123456789012345678901
2
12345678901234567890123456789012123456789012345678901
12345678901234567890123456789012123456789012345678901 2
2345678901234567890123456789012123456789012345678901
12345678901234567890123456789012123456789012345678901 2
12345678901234567890123456789012123456789012345678901 2
12345678901234567890123456789012123456789012345678901 2
12345678901234567890123456789012123456789012345678901 2
2
1
2
12345678901234567890123456789012123456789012345678901
2
12345678901234567890123456789012123456789012345678901
2
12345678901234567890123456789012123456789012345678901
2
12345678901234567890123456789012123456789012345678901
12345678901234567890123456789012123456789012345678901 2
2345678901234567890123456789012123456789012345678901
12345678901234567890123456789012123456789012345678901 2
2
1
2
12345678901234567890123456789012123456789012345678901
12345678901234567890123456789012123456789012345678901 2
2
12345678901234567890123456789012123456789012345678901
12345678901234567890123456789012123456789012345678901 2
12345678901234567890123456789012123456789012345678901 2
12345678901234567890123456789012123456789012345678901 2
2
128 12345678901234567890123456789012123456789012345678901
123456789012345678901234567890121234567890123456789012

www.petersons.com

http://www.xtremepapers.net

Chapter 4: Quantitative Review

Take It to the Next Level

123456789012345678901234567890121234567890123456789012
12345678901234567890123456789012123456789012345678901 2
2
12345678901234567890123456789012123456789012345678901
12345678901234567890123456789012123456789012345678901 2
2
12345678901234567890123456789012123456789012345678901
2
12345678901234567890123456789012123456789012345678901
2
12345678901234567890123456789012123456789012345678901
165
2
12345678901234567890123456789012123456789012345678901
2345678901234567890123456789012123456789012345678901
2
1
2
12345678901234567890123456789012123456789012345678901
2
12345678901234567890123456789012123456789012345678901
2
12345678901234567890123456789012123456789012345678901
2
12345678901234567890123456789012123456789012345678901
2345678901234567890123456789012123456789012345678901
2
1
2
12345678901234567890123456789012123456789012345678901
2
12345678901234567890123456789012123456789012345678901
2
12345678901234567890123456789012123456789012345678901
2
12345678901234567890123456789012123456789012345678901
2345678901234567890123456789012123456789012345678901
2
1
2
12345678901234567890123456789012123456789012345678901
2
12345678901234567890123456789012123456789012345678901
2
12345678901234567890123456789012123456789012345678901
2
12345678901234567890123456789012123456789012345678901
2345678901234567890123456789012123456789012345678901
2
1
2
12345678901234567890123456789012123456789012345678901
2
12345678901234567890123456789012123456789012345678901
2
12345678901234567890123456789012123456789012345678901
2
12345678901234567890123456789012123456789012345678901
2345678901234567890123456789012123456789012345678901
12345678901234567890123456789012123456789012345678901 22
12345678901234567890123456789012123456789012345678901 2
12345678901234567890123456789012123456789012345678901 2
12345678901234567890123456789012123456789012345678901 2
12345678901234567890123456789012123456789012345678901 2
12345678901234567890123456789012123456789012345678901 2
1
2
12345678901234567890123456789012123456789012345678901
2
12345678901234567890123456789012123456789012345678901
2
12345678901234567890123456789012123456789012345678901
2345678901234567890123456789012123456789012345678901
12345678901234567890123456789012123456789012345678901 22
12345678901234567890123456789012123456789012345678901 2
12345678901234567890123456789012123456789012345678901 2
12345678901234567890123456789012123456789012345678901 2
12345678901234567890123456789012123456789012345678901 2
12345678901234567890123456789012123456789012345678901
What percentage of the European Travelers surveyed did NOT
2
12345678901234567890123456789012123456789012345678901
2
12345678901234567890123456789012123456789012345678901
travel
to
Africa?
12345678901234567890123456789012123456789012345678901 22
1
2
12345678901234567890123456789012123456789012345678901
A. 34%
2
12345678901234567890123456789012123456789012345678901
2
12345678901234567890123456789012123456789012345678901
B. 43%
12345678901234567890123456789012123456789012345678901 2
2
12345678901234567890123456789012123456789012345678901
C. 57%
2
12345678901234567890123456789012123456789012345678901
D. 63%
2
12345678901234567890123456789012123456789012345678901
2
12345678901234567890123456789012123456789012345678901
E. 70%
2
12345678901234567890123456789012123456789012345678901
2
12345678901234567890123456789012123456789012345678901
2345678901234567890123456789012123456789012345678901
12345678901234567890123456789012123456789012345678901
This question only involves the Venn diagram, so focus on it. You will find 2
2
12345678901234567890123456789012123456789012345678901
1 that the total number of European travelers is 250 (165 African travelers 2
12345678901234567890123456789012123456789012345678901 2
12345678901234567890123456789012123456789012345678901
1 155 Asian travelers). You will note that by looking closely at the Venn 2
2
12345678901234567890123456789012123456789012345678901
2
12345678901234567890123456789012123456789012345678901
diagram
that
there
is
a
shaded
region
shared
by
both
Africa
and
Asia
2
12345678901234567890123456789012123456789012345678901
2
12345678901234567890123456789012123456789012345678901
(represented
by
70).
12345678901234567890123456789012123456789012345678901 2
2345678901234567890123456789012123456789012345678901
12345678901234567890123456789012123456789012345678901 22
12345678901234567890123456789012123456789012345678901
Youll calculate the travelers represented in the non-shaded portion of the 2
12345678901234567890123456789012123456789012345678901
2
12345678901234567890123456789012123456789012345678901
1 Venn diagram by subtracting 70 from both sides. So, for Africa 165 2 70 2
2345678901234567890123456789012123456789012345678901
2
12345678901234567890123456789012123456789012345678901
5 95 travelers. And Asia, 155 2 70 5 85 travelers.
2
12345678901234567890123456789012123456789012345678901
2
12345678901234567890123456789012123456789012345678901
Remember our total amount of European travelers is 250. Therefore, for 2
12345678901234567890123456789012123456789012345678901
2
1
85
95
2
12345678901234567890123456789012123456789012345678901
(or approximately 38 percent) and for Asia
(or approxi- 2
Africa
12345678901234567890123456789012123456789012345678901
250
250
2
12345678901234567890123456789012123456789012345678901
2
12345678901234567890123456789012123456789012345678901
mately
34
percent).
The
question
only
asks
for
the
percentage
that
did
2
12345678901234567890123456789012123456789012345678901
2345678901234567890123456789012123456789012345678901
2
1 NOT travel to Africa, so the correct answer is (A), 34 percent.
12345678901234567890123456789012123456789012345678901 2
12345678901234567890123456789012123456789012345678901 2
12345678901234567890123456789012123456789012345678901 2 129
123456789012345678901234567890121234567890123456789012

http://www.xtremepapers.net

Part III: Section Review

123456789012345678901234567890121234567890123456789012
12345678901234567890123456789012123456789012345678901 2
2
12345678901234567890123456789012123456789012345678901
How many of the European travelers surveyed are over 50 and have 2
12345678901234567890123456789012123456789012345678901
2
12345678901234567890123456789012123456789012345678901
traveled to Asia?
2
12345678901234567890123456789012123456789012345678901
12345678901234567890123456789012123456789012345678901 2
2
12345678901234567890123456789012123456789012345678901
A. 28
2
12345678901234567890123456789012123456789012345678901
2
12345678901234567890123456789012123456789012345678901
B.
38
2
12345678901234567890123456789012123456789012345678901
2
12345678901234567890123456789012123456789012345678901
C.
42
2
12345678901234567890123456789012123456789012345678901
2345678901234567890123456789012123456789012345678901
2
1
D. 50
2
12345678901234567890123456789012123456789012345678901
2
12345678901234567890123456789012123456789012345678901
E.
54
2
12345678901234567890123456789012123456789012345678901
2
12345678901234567890123456789012123456789012345678901
2345678901234567890123456789012123456789012345678901
1 This question involves both figures. The Venn diagram indicates there are 2
2
12345678901234567890123456789012123456789012345678901
12345678901234567890123456789012123456789012345678901
155 travelers to Asia, so the percentages in the bar graph are percentages 2
2
12345678901234567890123456789012123456789012345678901
12345678901234567890123456789012123456789012345678901
of 155. Find the 51100 category and read the value of the lightly-shaded 2
2345678901234567890123456789012123456789012345678901
2
1
12345678901234567890123456789012123456789012345678901
column. It is about midway between 25% and 30%, so estimate it at 2
2
12345678901234567890123456789012123456789012345678901
2
27.5%.
12345678901234567890123456789012123456789012345678901
2
12345678901234567890123456789012123456789012345678901
2345678901234567890123456789012123456789012345678901
2
12345678901234567890123456789012123456789012345678901
Ten percent of 155 is 15.5, and so 20% is 31. Five percent of 155 is 7.75. 2
12345678901234567890123456789012123456789012345678901
12345678901234567890123456789012123456789012345678901
If we add the 20% number to the 5% number, we should get (31 1 7.75 5 2
2
12345678901234567890123456789012123456789012345678901
2
12345678901234567890123456789012123456789012345678901
38.75).
If
25%
of
155
is
38.75,
then
27.5%
must
be
a
little
higher
than
12345678901234567890123456789012123456789012345678901 2
2
1 this. Choice (C) is in the right neighborhood.
2
12345678901234567890123456789012123456789012345678901
2
12345678901234567890123456789012123456789012345678901
2
12345678901234567890123456789012123456789012345678901
2345678901234567890123456789012123456789012345678901
12345678901234567890123456789012123456789012345678901 2
12345678901234567890123456789012123456789012345678901 2
12345678901234567890123456789012123456789012345678901 2
12345678901234567890123456789012123456789012345678901 2
12345678901234567890123456789012123456789012345678901 2
12345678901234567890123456789012123456789012345678901 2
12345678901234567890123456789012123456789012345678901 2
12345678901234567890123456789012123456789012345678901 2
12345678901234567890123456789012123456789012345678901 2
2
1
2
12345678901234567890123456789012123456789012345678901
2
12345678901234567890123456789012123456789012345678901
12345678901234567890123456789012123456789012345678901 2
12345678901234567890123456789012123456789012345678901 2
2
12345678901234567890123456789012123456789012345678901
2
12345678901234567890123456789012123456789012345678901
2
12345678901234567890123456789012123456789012345678901
12345678901234567890123456789012123456789012345678901 2
2
12345678901234567890123456789012123456789012345678901
2
12345678901234567890123456789012123456789012345678901
2345678901234567890123456789012123456789012345678901
12345678901234567890123456789012123456789012345678901 2
12345678901234567890123456789012123456789012345678901 2
2
1
12345678901234567890123456789012123456789012345678901 2
12345678901234567890123456789012123456789012345678901 2
2
12345678901234567890123456789012123456789012345678901
2
12345678901234567890123456789012123456789012345678901
2
12345678901234567890123456789012123456789012345678901
2
12345678901234567890123456789012123456789012345678901
12345678901234567890123456789012123456789012345678901 2
2345678901234567890123456789012123456789012345678901
12345678901234567890123456789012123456789012345678901 2
12345678901234567890123456789012123456789012345678901 2
12345678901234567890123456789012123456789012345678901 2
12345678901234567890123456789012123456789012345678901 2
2
1
2
12345678901234567890123456789012123456789012345678901
2
12345678901234567890123456789012123456789012345678901
2
12345678901234567890123456789012123456789012345678901
2
12345678901234567890123456789012123456789012345678901
12345678901234567890123456789012123456789012345678901 2
2345678901234567890123456789012123456789012345678901
12345678901234567890123456789012123456789012345678901 2
2
1
2
12345678901234567890123456789012123456789012345678901
12345678901234567890123456789012123456789012345678901 2
2
12345678901234567890123456789012123456789012345678901
12345678901234567890123456789012123456789012345678901 2
12345678901234567890123456789012123456789012345678901 2
12345678901234567890123456789012123456789012345678901 2
2
130 12345678901234567890123456789012123456789012345678901
123456789012345678901234567890121234567890123456789012

www.petersons.com

http://www.xtremepapers.net

Chapter 4: Quantitative Review

Take It to the Next Level

123456789012345678901234567890121234567890123456789012
12345678901234567890123456789012123456789012345678901 2
2
12345678901234567890123456789012123456789012345678901
You might also see a less conventional graph set than the previous one. The 2
12345678901234567890123456789012123456789012345678901
2
12345678901234567890123456789012123456789012345678901
test likes to see if you can think on your feet by presenting information in 2
12345678901234567890123456789012123456789012345678901
2
12345678901234567890123456789012123456789012345678901
a format that you might not be expecting. Dont be surprised if you see 2
12345678901234567890123456789012123456789012345678901
2345678901234567890123456789012123456789012345678901
2
1 something wacky like the following on test day.
2
12345678901234567890123456789012123456789012345678901
2
12345678901234567890123456789012123456789012345678901
2
12345678901234567890123456789012123456789012345678901
2
12345678901234567890123456789012123456789012345678901
2345678901234567890123456789012123456789012345678901
2
1
2
12345678901234567890123456789012123456789012345678901
2
12345678901234567890123456789012123456789012345678901
2
12345678901234567890123456789012123456789012345678901
2
12345678901234567890123456789012123456789012345678901
2345678901234567890123456789012123456789012345678901
2
1
2
12345678901234567890123456789012123456789012345678901
2
12345678901234567890123456789012123456789012345678901
2
12345678901234567890123456789012123456789012345678901
2
12345678901234567890123456789012123456789012345678901
2345678901234567890123456789012123456789012345678901
2
1
2
12345678901234567890123456789012123456789012345678901
2
12345678901234567890123456789012123456789012345678901
2
12345678901234567890123456789012123456789012345678901
2
12345678901234567890123456789012123456789012345678901
2345678901234567890123456789012123456789012345678901
12345678901234567890123456789012123456789012345678901 22
12345678901234567890123456789012123456789012345678901 2
12345678901234567890123456789012123456789012345678901 2
12345678901234567890123456789012123456789012345678901 2
12345678901234567890123456789012123456789012345678901 2
12345678901234567890123456789012123456789012345678901 2
1
2
12345678901234567890123456789012123456789012345678901
2
12345678901234567890123456789012123456789012345678901
2
12345678901234567890123456789012123456789012345678901
2345678901234567890123456789012123456789012345678901
12345678901234567890123456789012123456789012345678901 22
12345678901234567890123456789012123456789012345678901 2
12345678901234567890123456789012123456789012345678901 2
12345678901234567890123456789012123456789012345678901 2
12345678901234567890123456789012123456789012345678901 2
12345678901234567890123456789012123456789012345678901 2
12345678901234567890123456789012123456789012345678901 2
12345678901234567890123456789012123456789012345678901 2
12345678901234567890123456789012123456789012345678901 2
1
2
12345678901234567890123456789012123456789012345678901
How many miles is the section of Rt. 2 that runs through the city
2
12345678901234567890123456789012123456789012345678901
2
12345678901234567890123456789012123456789012345678901
limits of Douglass?
12345678901234567890123456789012123456789012345678901 2
2
12345678901234567890123456789012123456789012345678901
A. 2
2
12345678901234567890123456789012123456789012345678901
2
12345678901234567890123456789012123456789012345678901
B. =6
2
12345678901234567890123456789012123456789012345678901
2
12345678901234567890123456789012123456789012345678901
2
12345678901234567890123456789012123456789012345678901
C. =8
2345678901234567890123456789012123456789012345678901
12345678901234567890123456789012123456789012345678901 22
12345678901234567890123456789012123456789012345678901
D. 3
2
1
2
12345678901234567890123456789012123456789012345678901
E.
12
=
2
12345678901234567890123456789012123456789012345678901
2345678901234567890123456789012123456789012345678901
12345678901234567890123456789012123456789012345678901 22
12345678901234567890123456789012123456789012345678901
Rt. 2 in the city limits works out to be the hypotenuse of two different right 2
12345678901234567890123456789012123456789012345678901
2
12345678901234567890123456789012123456789012345678901
1 triangles. You can determine the length of the section of Rt. 2 with either 2
2345678901234567890123456789012123456789012345678901
12345678901234567890123456789012123456789012345678901
triangle, so pick one and go with it. Both of the other sides of the triangles 2
2
12345678901234567890123456789012123456789012345678901
2
12345678901234567890123456789012123456789012345678901
are
2
miles,
so
both
triangles
are
45-45-90
triangles.
The
length
of
the
12345678901234567890123456789012123456789012345678901 22
1 hypotenuse can simply be read off, 2=2, which is equal to choice (C).
2
12345678901234567890123456789012123456789012345678901
2
12345678901234567890123456789012123456789012345678901
2
12345678901234567890123456789012123456789012345678901
2
12345678901234567890123456789012123456789012345678901
12345678901234567890123456789012123456789012345678901 2
2345678901234567890123456789012123456789012345678901
12345678901234567890123456789012123456789012345678901 22
1
2
12345678901234567890123456789012123456789012345678901
12345678901234567890123456789012123456789012345678901 2
2
12345678901234567890123456789012123456789012345678901
12345678901234567890123456789012123456789012345678901 2
12345678901234567890123456789012123456789012345678901 2
12345678901234567890123456789012123456789012345678901 2
12345678901234567890123456789012123456789012345678901 2 131
123456789012345678901234567890121234567890123456789012

http://www.xtremepapers.net

Part III: Section Review

www.petersons.com

http://www.xtremepapers.net

Tip

123456789012345678901234567890121234567890123456789012
12345678901234567890123456789012123456789012345678901 2
2
12345678901234567890123456789012123456789012345678901
What approximate percentage of Douglass County is within the city 2
12345678901234567890123456789012123456789012345678901
2
12345678901234567890123456789012123456789012345678901
limits of Douglass?
2
12345678901234567890123456789012123456789012345678901
12345678901234567890123456789012123456789012345678901 2
2
12345678901234567890123456789012123456789012345678901
A. 20%
2
12345678901234567890123456789012123456789012345678901
2
12345678901234567890123456789012123456789012345678901
B.
33%
2
12345678901234567890123456789012123456789012345678901
2
12345678901234567890123456789012123456789012345678901
C.
42%
2
12345678901234567890123456789012123456789012345678901
2345678901234567890123456789012123456789012345678901
2
1
D. 50%
2
12345678901234567890123456789012123456789012345678901
2
12345678901234567890123456789012123456789012345678901
E.
66%
2
12345678901234567890123456789012123456789012345678901
2
12345678901234567890123456789012123456789012345678901
2345678901234567890123456789012123456789012345678901
1 The areas of the county and the city have to be determined to find this 2
2
12345678901234567890123456789012123456789012345678901
12345678901234567890123456789012123456789012345678901
percentage. This is greatly simplified by the fact that both areas are 2
2
12345678901234567890123456789012123456789012345678901
12345678901234567890123456789012123456789012345678901
rectangles, which you can infer from the note that all the lines are parallel 2
2345678901234567890123456789012123456789012345678901
2
1
12345678901234567890123456789012123456789012345678901
and at right angles in the diagram. Examining the right length of the 2
2
12345678901234567890123456789012123456789012345678901
diagram, the length of the diagram is 6 (4 miles 12 miles). The width is 3 2
12345678901234567890123456789012123456789012345678901
2
12345678901234567890123456789012123456789012345678901
(read off from the bottom width as 0.5 12 1 0.5).
2
12345678901234567890123456789012123456789012345678901
2
12345678901234567890123456789012123456789012345678901
2
12345678901234567890123456789012123456789012345678901
This
makes
the
area
of
the
county
3
3
6
5
18.
The
length
of
the
city
is
3
2
12345678901234567890123456789012123456789012345678901
2
12345678901234567890123456789012123456789012345678901
and
the
width
is
2,
so
the
citys
area
is
6.
The
ratio
of
city
area
to
county
2345678901234567890123456789012123456789012345678901
12345678901234567890123456789012123456789012345678901 2
2
1
1
6
2
12345678901234567890123456789012123456789012345678901
5
'
0.33.
The
correct
answer
is
(B).
area
then
is
2
12345678901234567890123456789012123456789012345678901
18
3
2
12345678901234567890123456789012123456789012345678901
2345678901234567890123456789012123456789012345678901
12345678901234567890123456789012123456789012345678901 2
12345678901234567890123456789012123456789012345678901 2
12345678901234567890123456789012123456789012345678901
You can drop the units since all the units are the same and will cancel each 2
2
12345678901234567890123456789012123456789012345678901
2
12345678901234567890123456789012123456789012345678901
other
out.
12345678901234567890123456789012123456789012345678901 2
12345678901234567890123456789012123456789012345678901 2
12345678901234567890123456789012123456789012345678901 2
12345678901234567890123456789012123456789012345678901 2
2
1
If you are 2 miles due southeast of the intersection of Rt. 7 and
2
12345678901234567890123456789012123456789012345678901
2
12345678901234567890123456789012123456789012345678901
Rt.
4,
where
are
you?
12345678901234567890123456789012123456789012345678901 2
12345678901234567890123456789012123456789012345678901 2
2
12345678901234567890123456789012123456789012345678901
A. In the city
2
12345678901234567890123456789012123456789012345678901
2
12345678901234567890123456789012123456789012345678901
B.
Outside
the
city
but
in
the
county
12345678901234567890123456789012123456789012345678901 2
2
12345678901234567890123456789012123456789012345678901
C. Outside the county
2
12345678901234567890123456789012123456789012345678901
2345678901234567890123456789012123456789012345678901
2
12345678901234567890123456789012123456789012345678901
D. Along Rt. 10
2
12345678901234567890123456789012123456789012345678901
2
1
E. Along Rt. 2
12345678901234567890123456789012123456789012345678901 2
2
12345678901234567890123456789012123456789012345678901
First find the intersection of Rt. 7 and Rt. 4. It is on the left side. According 2
12345678901234567890123456789012123456789012345678901
2
12345678901234567890123456789012123456789012345678901
to the compass rose, Rt. 4 runs north-south and Rt. 7 runs east-west. 2
12345678901234567890123456789012123456789012345678901
2
12345678901234567890123456789012123456789012345678901
Southeast then would be halfway between heading east on Rt. 7 and south 2
12345678901234567890123456789012123456789012345678901
2345678901234567890123456789012123456789012345678901
12345678901234567890123456789012123456789012345678901
on Rt. 4. If there were a line defining this direction, it would form a 2
2
12345678901234567890123456789012123456789012345678901
2
12345678901234567890123456789012123456789012345678901
45-degree
angle
with
Rt.
7
and
with
Rt.
4.
If
that
line
is
long
enough,
it
will
12345678901234567890123456789012123456789012345678901 2
1 form two 45-45-90 triangles like you saw that Rt. 2 did two questions 2
2
12345678901234567890123456789012123456789012345678901
2
12345678901234567890123456789012123456789012345678901
back.
However,
in
that
instance
you
found
that
the
length
of
Rt.
2
in
the
2
12345678901234567890123456789012123456789012345678901
2
12345678901234567890123456789012123456789012345678901
2
miles,
which
is
greater
than
2
miles.
So
2
miles
southeast
of
city
was
2
=
2
12345678901234567890123456789012123456789012345678901
2345678901234567890123456789012123456789012345678901
2
12345678901234567890123456789012123456789012345678901
1 the intersection is not outside of the box defined by the city limits south of 2
2
12345678901234567890123456789012123456789012345678901
Rt. 7. So you would be inside the city, choice (A).
2
12345678901234567890123456789012123456789012345678901
2
12345678901234567890123456789012123456789012345678901
2345678901234567890123456789012123456789012345678901
2
1
12345678901234567890123456789012123456789012345678901 2
12345678901234567890123456789012123456789012345678901 2
2
132 12345678901234567890123456789012123456789012345678901
123456789012345678901234567890121234567890123456789012

Chapter 4: Quantitative Review

Take It to the Next Level

123456789012345678901234567890121234567890123456789012
12345678901234567890123456789012123456789012345678901 2
12345678901234567890123456789012123456789012345678901 22
A 112345678901234567890123456789012123456789012345678901
Final Word
2345678901234567890123456789012123456789012345678901
2
2
12345678901234567890123456789012123456789012345678901
The
Next
Level
section
of
this
chapter
covers
many
of
the
favorite
ways
to
12345678901234567890123456789012123456789012345678901 2
12345678901234567890123456789012123456789012345678901
make a GRE problem hard, but not all of them. Invariably, if you are doing 2
2
12345678901234567890123456789012123456789012345678901
2
12345678901234567890123456789012123456789012345678901
really
well
on
the
GRE
Math
section,
you
will
encounter
something
that
at
2
12345678901234567890123456789012123456789012345678901
2
12345678901234567890123456789012123456789012345678901
first
glance
looks
impossibly
complex.
Ask
yourself,
How
did
the
2
12345678901234567890123456789012123456789012345678901
2345678901234567890123456789012123456789012345678901
1 test-makers make this problem hard? Is it a multi-step problem involving 2
2
12345678901234567890123456789012123456789012345678901
2
12345678901234567890123456789012123456789012345678901
a
host
of
equations?
If
so,
the
goal
is
often
to
find
terms
that
cancel
each
2
12345678901234567890123456789012123456789012345678901
2
12345678901234567890123456789012123456789012345678901
other
out.
Does
the
question
have
a
bunch
of
interconnected
figures?
If
so,
2345678901234567890123456789012123456789012345678901
2
1
12345678901234567890123456789012123456789012345678901
the key is usually to figure out facts about individual figures and then use 2
2
12345678901234567890123456789012123456789012345678901
12345678901234567890123456789012123456789012345678901
this knowledge to gain information about other figures. With an 2
2
12345678901234567890123456789012123456789012345678901
12345678901234567890123456789012123456789012345678901
aggressive attitude and a willingness to get creative and keep working, 2
2
12345678901234567890123456789012123456789012345678901
2
12345678901234567890123456789012123456789012345678901
many of the hardest GRE problems will be yours for the taking.
2
12345678901234567890123456789012123456789012345678901
2
12345678901234567890123456789012123456789012345678901
2345678901234567890123456789012123456789012345678901
12345678901234567890123456789012123456789012345678901 22
12345678901234567890123456789012123456789012345678901 2
12345678901234567890123456789012123456789012345678901 2
12345678901234567890123456789012123456789012345678901 2
12345678901234567890123456789012123456789012345678901 2
12345678901234567890123456789012123456789012345678901 2
1
2
12345678901234567890123456789012123456789012345678901
2
12345678901234567890123456789012123456789012345678901
2
12345678901234567890123456789012123456789012345678901
2345678901234567890123456789012123456789012345678901
12345678901234567890123456789012123456789012345678901 22
12345678901234567890123456789012123456789012345678901 2
12345678901234567890123456789012123456789012345678901 2
12345678901234567890123456789012123456789012345678901 2
12345678901234567890123456789012123456789012345678901 2
12345678901234567890123456789012123456789012345678901 2
12345678901234567890123456789012123456789012345678901 2
12345678901234567890123456789012123456789012345678901 2
12345678901234567890123456789012123456789012345678901 2
1
2
12345678901234567890123456789012123456789012345678901
2
12345678901234567890123456789012123456789012345678901
12345678901234567890123456789012123456789012345678901 2
12345678901234567890123456789012123456789012345678901 2
2
12345678901234567890123456789012123456789012345678901
2
12345678901234567890123456789012123456789012345678901
2
12345678901234567890123456789012123456789012345678901
12345678901234567890123456789012123456789012345678901 2
2
12345678901234567890123456789012123456789012345678901
2
12345678901234567890123456789012123456789012345678901
2345678901234567890123456789012123456789012345678901
12345678901234567890123456789012123456789012345678901 22
12345678901234567890123456789012123456789012345678901 2
1
12345678901234567890123456789012123456789012345678901 2
12345678901234567890123456789012123456789012345678901 2
2
12345678901234567890123456789012123456789012345678901
2
12345678901234567890123456789012123456789012345678901
2
12345678901234567890123456789012123456789012345678901
2
12345678901234567890123456789012123456789012345678901
12345678901234567890123456789012123456789012345678901 2
2345678901234567890123456789012123456789012345678901
12345678901234567890123456789012123456789012345678901 22
12345678901234567890123456789012123456789012345678901 2
12345678901234567890123456789012123456789012345678901 2
12345678901234567890123456789012123456789012345678901 2
1
2
12345678901234567890123456789012123456789012345678901
2
12345678901234567890123456789012123456789012345678901
2
12345678901234567890123456789012123456789012345678901
2
12345678901234567890123456789012123456789012345678901
12345678901234567890123456789012123456789012345678901 2
2345678901234567890123456789012123456789012345678901
12345678901234567890123456789012123456789012345678901 22
1
2
12345678901234567890123456789012123456789012345678901
12345678901234567890123456789012123456789012345678901 2
2
12345678901234567890123456789012123456789012345678901
12345678901234567890123456789012123456789012345678901 2
12345678901234567890123456789012123456789012345678901 2
12345678901234567890123456789012123456789012345678901 2
12345678901234567890123456789012123456789012345678901 2 133
123456789012345678901234567890121234567890123456789012

http://www.xtremepapers.net

Chapter

5
Verbal Review
123456789012345678901234567890121234567890123456789012
2
12345678901234567890123456789012123456789012345678901
2
12345678901234567890123456789012123456789012345678901
2
12345678901234567890123456789012123456789012345678901
The
Perfect
Charm
for
Those
Shooting
to
2345678901234567890123456789012123456789012345678901
12345678901234567890123456789012123456789012345678901 2
2
1
Score
above a 500
2
12345678901234567890123456789012123456789012345678901
2
12345678901234567890123456789012123456789012345678901
When
people
talk
about
standardized
tests,
they
often
fall
into
one
of
two
2
12345678901234567890123456789012123456789012345678901
2345678901234567890123456789012123456789012345678901
2
12345678901234567890123456789012123456789012345678901
1 camps. The first group believes that the tests are a reasonably accurate 2
12345678901234567890123456789012123456789012345678901
indication of a persons intelligence. It stands to reason, then, that 2
2
12345678901234567890123456789012123456789012345678901
2
12345678901234567890123456789012123456789012345678901
someone
who
scores
well
on
a
standardized
test
must
be
smart.
The
2345678901234567890123456789012123456789012345678901
12345678901234567890123456789012123456789012345678901 2
12345678901234567890123456789012123456789012345678901
second camp does not hold standardized tests in such a high esteem light. 2
2
12345678901234567890123456789012123456789012345678901
2
12345678901234567890123456789012123456789012345678901
To
people
in
the
second
camp,
the
only
thing
a
standardized
test
measures
12345678901234567890123456789012123456789012345678901 2
12345678901234567890123456789012123456789012345678901
is how well someone does on that particular standardized test. To them, 2
2
12345678901234567890123456789012123456789012345678901
2
12345678901234567890123456789012123456789012345678901
standardized
tests
are
too
arbitrary
and
narrowly
focused
to
be
any
kind
of
12345678901234567890123456789012123456789012345678901 2
2
1 accurate indicator of intelligence.
2
12345678901234567890123456789012123456789012345678901
2
12345678901234567890123456789012123456789012345678901
12345678901234567890123456789012123456789012345678901
By the time you are finished reading through this section (and this book), 2
12345678901234567890123456789012123456789012345678901 2
12345678901234567890123456789012123456789012345678901
many of you in the first camp will have migrated to the second camp. This 2
2
12345678901234567890123456789012123456789012345678901
is because many of you will no longer think of the GRE Verbal section as a 2
12345678901234567890123456789012123456789012345678901
2
12345678901234567890123456789012123456789012345678901
comprehensive, multiple-choice exam that uses a variety of question 2
12345678901234567890123456789012123456789012345678901
2
12345678901234567890123456789012123456789012345678901
formats to accurately gauge a persons level of English mastery. In place of 2
12345678901234567890123456789012123456789012345678901
2
12345678901234567890123456789012123456789012345678901
this idea, many of you will view the test in a much different way.
2
12345678901234567890123456789012123456789012345678901
12345678901234567890123456789012123456789012345678901 2
12345678901234567890123456789012123456789012345678901 2
2
12345678901234567890123456789012123456789012345678901
2
12345678901234567890123456789012123456789012345678901
2
12345678901234567890123456789012123456789012345678901
The
GRE
Verbal
Section
Is
a
Vocabulary
Quiz
2
12345678901234567890123456789012123456789012345678901
2
12345678901234567890123456789012123456789012345678901
Gone
Wild
2345678901234567890123456789012123456789012345678901
12345678901234567890123456789012123456789012345678901 2
12345678901234567890123456789012123456789012345678901 2
12345678901234567890123456789012123456789012345678901
For the Verbal section, you get 30 minutes to answer 30 questions that 2
2
12345678901234567890123456789012123456789012345678901
2
1 come in four different types:
2345678901234567890123456789012123456789012345678901
12345678901234567890123456789012123456789012345678901 2
2
12345678901234567890123456789012123456789012345678901
1. Antonyms
2
12345678901234567890123456789012123456789012345678901
12345678901234567890123456789012123456789012345678901 2
2
1
2. Analogies
2
12345678901234567890123456789012123456789012345678901
12345678901234567890123456789012123456789012345678901 2
2
12345678901234567890123456789012123456789012345678901
3. Sentence Completions (just SC if youre in a hurry)
2
12345678901234567890123456789012123456789012345678901
2
12345678901234567890123456789012123456789012345678901
2345678901234567890123456789012123456789012345678901
2
1
4. Reading Comprehension (RC for those on the go)
2
12345678901234567890123456789012123456789012345678901
12345678901234567890123456789012123456789012345678901 2
12345678901234567890123456789012123456789012345678901 2
123456789012345678901234567890121234567890123456789012
134

http://www.xtremepapers.net

Alert!

Chapter 5: Verbal Review

123456789012345678901234567890121234567890123456789012
12345678901234567890123456789012123456789012345678901 2
2
12345678901234567890123456789012123456789012345678901
A chart describing these four question typesas well as a discussion about 2
12345678901234567890123456789012123456789012345678901
2
12345678901234567890123456789012123456789012345678901
the basic approach to the GRE Verbal testcan be found starting on page 2
12345678901234567890123456789012123456789012345678901
2
12345678901234567890123456789012123456789012345678901
25. Be sure to read this section if you havent already done so. The bulk of 2
12345678901234567890123456789012123456789012345678901
2345678901234567890123456789012123456789012345678901
1 this chapter will discuss each question type in detail, but you need to fit 2
2
12345678901234567890123456789012123456789012345678901
2
12345678901234567890123456789012123456789012345678901
these
individual
ideas
into
the
overall
Verbal
strategy
covered
in
the
earlier
2
12345678901234567890123456789012123456789012345678901
2
12345678901234567890123456789012123456789012345678901
section.
2345678901234567890123456789012123456789012345678901
2
1
2
12345678901234567890123456789012123456789012345678901
2
12345678901234567890123456789012123456789012345678901
Like
the
Quantitative
section,
the
GRE
Verbal
section
is
scored
on
a
scale
2
12345678901234567890123456789012123456789012345678901
2
12345678901234567890123456789012123456789012345678901
from
200
to
800,
with
800
a
perfect
score.
Theoretically,
a
500
would
be
2345678901234567890123456789012123456789012345678901
2
1
2
12345678901234567890123456789012123456789012345678901
an
average
score,
and
this
chapter
is
going
to
emphasize
strategies
to
help
2
12345678901234567890123456789012123456789012345678901
2
12345678901234567890123456789012123456789012345678901
you
score
above
a
500
on
the
GRE
Verbal
section.
A
Verbal
score
in
the
2
12345678901234567890123456789012123456789012345678901
2345678901234567890123456789012123456789012345678901
1 high 500s to mid 600s can be considered a solid score, although if you are 2
2
12345678901234567890123456789012123456789012345678901
12345678901234567890123456789012123456789012345678901
applying for an English program you will most likely want to stay above 2
2
12345678901234567890123456789012123456789012345678901
12345678901234567890123456789012123456789012345678901
600. The fact that the scores are scaled makes for some strange 2
2345678901234567890123456789012123456789012345678901
12345678901234567890123456789012123456789012345678901 22
12345678901234567890123456789012123456789012345678901
discrepancies between the percentile scores and scaled scores. For
2
12345678901234567890123456789012123456789012345678901
example, a 700 Verbal might translate to the 96th percentile, meaning that 2
12345678901234567890123456789012123456789012345678901
2
12345678901234567890123456789012123456789012345678901
this score was better than 95 out of 100 test-takers. However, a higher 2
12345678901234567890123456789012123456789012345678901
1 Quantitative scaled score might translate to a lower percentile score 2
2
12345678901234567890123456789012123456789012345678901
2
12345678901234567890123456789012123456789012345678901
because
of
the
results
of
all
the
other
people
taking
the
GRE
CAT.
On
the
2
12345678901234567890123456789012123456789012345678901
2345678901234567890123456789012123456789012345678901
12345678901234567890123456789012123456789012345678901
same test where a 700 Verbal equals the 96th percentile, a 760 2
2
12345678901234567890123456789012123456789012345678901
2
12345678901234567890123456789012123456789012345678901
rd
percentile.
Even
Quantitative
scaled
score
might
only
rank
in
the
83
12345678901234567890123456789012123456789012345678901 22
12345678901234567890123456789012123456789012345678901
though the scaled score is higher, the percentile ranking is lower. If this 2
12345678901234567890123456789012123456789012345678901
12345678901234567890123456789012123456789012345678901
seems a little odd, thats because it is. The end result is that it is usually 2
2
12345678901234567890123456789012123456789012345678901
2
12345678901234567890123456789012123456789012345678901
tougher
to
get
a
high
Verbal
scaled
score
than
it
is
to
get
a
high
2
1
2345678901234567890123456789012123456789012345678901
12345678901234567890123456789012123456789012345678901
Quantitative scaled score because the pool of GRE test-takers is skewed 2
2
12345678901234567890123456789012123456789012345678901
2
1 with people who score better in Verbal than Math.
12345678901234567890123456789012123456789012345678901 2
2
12345678901234567890123456789012123456789012345678901
2
12345678901234567890123456789012123456789012345678901
2
12345678901234567890123456789012123456789012345678901
You
receive
your
scaled
scores
immediately
after
completing
the
entire
12345678901234567890123456789012123456789012345678901 2
2
12345678901234567890123456789012123456789012345678901
test. There are also percentile scores (099%) for both the Verbal and
2
12345678901234567890123456789012123456789012345678901
2345678901234567890123456789012123456789012345678901
2
12345678901234567890123456789012123456789012345678901
Quantitative sections. These percentile scores can be found in the same
2
12345678901234567890123456789012123456789012345678901
2
1 letter that gives you your Analytical score. The percentile score can be a
12345678901234567890123456789012123456789012345678901 2
2
12345678901234567890123456789012123456789012345678901
little strange, since it varies over time depending on all the people taking
2
12345678901234567890123456789012123456789012345678901
2
12345678901234567890123456789012123456789012345678901
the test. Most graduate programs are more interested in the scaled scores,
2
12345678901234567890123456789012123456789012345678901
2
12345678901234567890123456789012123456789012345678901
so dont lose too much sleep trying to figure out the vagaries of percentile
12345678901234567890123456789012123456789012345678901 2
2345678901234567890123456789012123456789012345678901
2
12345678901234567890123456789012123456789012345678901
scores.
2
12345678901234567890123456789012123456789012345678901
12345678901234567890123456789012123456789012345678901 22
12345678901234567890123456789012123456789012345678901 2
1 Although there are four different question types, the first three all have a
2
12345678901234567890123456789012123456789012345678901
heavy vocabulary component. The first question type, Antonyms, is 2
12345678901234567890123456789012123456789012345678901
2
12345678901234567890123456789012123456789012345678901
almost exclusively a vocabulary quiz, since you often need to know the 2
12345678901234567890123456789012123456789012345678901
2
12345678901234567890123456789012123456789012345678901
definition of a word in order to pick its opposite. There are some guessing 2
12345678901234567890123456789012123456789012345678901
2
12345678901234567890123456789012123456789012345678901
techniques that enable you to work on an antonym problem even if you 2
12345678901234567890123456789012123456789012345678901
12345678901234567890123456789012123456789012345678901
dont know the definition of the word, but for the most part many 2
2
12345678901234567890123456789012123456789012345678901
2345678901234567890123456789012123456789012345678901
2
1 antonym problems boil down to, Do you know the word or not?
12345678901234567890123456789012123456789012345678901 2
12345678901234567890123456789012123456789012345678901 2
12345678901234567890123456789012123456789012345678901 2 135
123456789012345678901234567890121234567890123456789012

http://www.xtremepapers.net

Part III: Section Review

123456789012345678901234567890121234567890123456789012
12345678901234567890123456789012123456789012345678901 2
2
12345678901234567890123456789012123456789012345678901
Both analogies and sentence completions have formats that are more 2
12345678901234567890123456789012123456789012345678901
2
12345678901234567890123456789012123456789012345678901
complicated than antonyms, and both formats can be approached with a 2
12345678901234567890123456789012123456789012345678901
2
12345678901234567890123456789012123456789012345678901
host of methods. However, the presence of difficult vocabulary words in 2
12345678901234567890123456789012123456789012345678901
2345678901234567890123456789012123456789012345678901
1 the answer choices can make things difficult. For instance, you might read 2
2
12345678901234567890123456789012123456789012345678901
2
12345678901234567890123456789012123456789012345678901
a
SC
problem
and
realize
the
word
in
the
blank
needs
to
mean
something
2
12345678901234567890123456789012123456789012345678901
2
12345678901234567890123456789012123456789012345678901
akin
to
small,
but
then
look
down
and
see
that
your
answer
choices
are:
2345678901234567890123456789012123456789012345678901
2
1
2
12345678901234567890123456789012123456789012345678901
2
12345678901234567890123456789012123456789012345678901
A.
weighty
2
12345678901234567890123456789012123456789012345678901
2
12345678901234567890123456789012123456789012345678901
B.
pulchritudinous
2345678901234567890123456789012123456789012345678901
2
1
2
12345678901234567890123456789012123456789012345678901
C.
pusillanimous
2
12345678901234567890123456789012123456789012345678901
2
12345678901234567890123456789012123456789012345678901
D.
voluminous
2
12345678901234567890123456789012123456789012345678901
2345678901234567890123456789012123456789012345678901
2
1
E. angry
2
12345678901234567890123456789012123456789012345678901
2
12345678901234567890123456789012123456789012345678901
Choices (A) and (E) are well-known words, but (B), (C), and (D) are not as 2
12345678901234567890123456789012123456789012345678901
2
12345678901234567890123456789012123456789012345678901
common. For this question, you could eliminate (A) and (E) since you 2
12345678901234567890123456789012123456789012345678901
2
12345678901234567890123456789012123456789012345678901
know that neither of these words means small, but you would then be 2
12345678901234567890123456789012123456789012345678901
2
12345678901234567890123456789012123456789012345678901
left with a choice of (B), (C), and (D). Many of you might be able to look at 2
12345678901234567890123456789012123456789012345678901
2345678901234567890123456789012123456789012345678901
2
12345678901234567890123456789012123456789012345678901
1 voluminous and recognize the root volume in the word, making this choice 2
12345678901234567890123456789012123456789012345678901
unlikely to be the answer. This would leave you with two choices to guess 2
2
12345678901234567890123456789012123456789012345678901
2
12345678901234567890123456789012123456789012345678901
from,
illustrating
how
good
test
strategies
can
take
you
a
long
way,
but
2345678901234567890123456789012123456789012345678901
12345678901234567890123456789012123456789012345678901 2
12345678901234567890123456789012123456789012345678901
sometimes not all the way to finding the answer outright. For the last push, 2
2
12345678901234567890123456789012123456789012345678901
2
12345678901234567890123456789012123456789012345678901
you
would
have
to
know
that
pulchritudinous
is
an
ugly-sounding
word
12345678901234567890123456789012123456789012345678901 2
12345678901234567890123456789012123456789012345678901
that surprisingly means beautiful, while pusillanimous is a very large 2
2
12345678901234567890123456789012123456789012345678901
2
12345678901234567890123456789012123456789012345678901
word
that
means
small.
Hows
that
for
opposites?
12345678901234567890123456789012123456789012345678901 2
2
1
12345678901234567890123456789012123456789012345678901
Three fourths of the GRE Verbal question types have a vocabulary 2
2
12345678901234567890123456789012123456789012345678901
12345678901234567890123456789012123456789012345678901
component to them. Combine this with the fact that the first 36 questions 2
12345678901234567890123456789012123456789012345678901 2
12345678901234567890123456789012123456789012345678901
you face will be both antonyms and analogies, and that the first questions 2
2
12345678901234567890123456789012123456789012345678901
on a computer adaptive test section count more than the questions at the 2
12345678901234567890123456789012123456789012345678901
2
12345678901234567890123456789012123456789012345678901
end. The end result is a GRE Verbal exam that is a lot like a souped-up 2
12345678901234567890123456789012123456789012345678901
2
12345678901234567890123456789012123456789012345678901
vocabulary test, or a vocabulary quiz gone wild.
2
12345678901234567890123456789012123456789012345678901
2
12345678901234567890123456789012123456789012345678901
2
12345678901234567890123456789012123456789012345678901
If
you
wanted
to
practice
your
Analytical
skills,
you
can
write
an
essay
12345678901234567890123456789012123456789012345678901 2
12345678901234567890123456789012123456789012345678901
showing why the GRE Verbal section is not like a vocabulary quiz on 2
2
12345678901234567890123456789012123456789012345678901
2
12345678901234567890123456789012123456789012345678901
steroids.
You
could
go
through
and
find
the
flaws
in
the
authors
2
12345678901234567890123456789012123456789012345678901
2
12345678901234567890123456789012123456789012345678901
argument,
which
admittedly
is
not
airtight.
This
might
help
you
improve
12345678901234567890123456789012123456789012345678901 2
2345678901234567890123456789012123456789012345678901
12345678901234567890123456789012123456789012345678901
your Analytical score, but it wont do much for the GRE Verbal score. For 2
2
12345678901234567890123456789012123456789012345678901
2
12345678901234567890123456789012123456789012345678901
help
in
this
area,
the
first
thing
you
should
do
is
improve
your
vocabulary,
12345678901234567890123456789012123456789012345678901 2
2
1 because it plays a huge part on the GRE Verbal.
2
12345678901234567890123456789012123456789012345678901
2
12345678901234567890123456789012123456789012345678901
2
12345678901234567890123456789012123456789012345678901
2
12345678901234567890123456789012123456789012345678901
12345678901234567890123456789012123456789012345678901 2
2345678901234567890123456789012123456789012345678901
12345678901234567890123456789012123456789012345678901 2
2
1
2
12345678901234567890123456789012123456789012345678901
12345678901234567890123456789012123456789012345678901 2
2
12345678901234567890123456789012123456789012345678901
12345678901234567890123456789012123456789012345678901 2
12345678901234567890123456789012123456789012345678901 2
12345678901234567890123456789012123456789012345678901 2
2
136 12345678901234567890123456789012123456789012345678901
123456789012345678901234567890121234567890123456789012

www.petersons.com

http://www.xtremepapers.net

Chapter 5: Verbal Review

123456789012345678901234567890121234567890123456789012
12345678901234567890123456789012123456789012345678901 2
2
12345678901234567890123456789012123456789012345678901
2
12345678901234567890123456789012123456789012345678901
Upgrading Your Personal Lexicon
2
12345678901234567890123456789012123456789012345678901
2
12345678901234567890123456789012123456789012345678901
In
the
ideal
scenario,
you
are
reading
this
book
when
you
are
5
years
old.
12345678901234567890123456789012123456789012345678901 2
12345678901234567890123456789012123456789012345678901
It might be the first book you ever read. If this is the case, then the GRE 2
2
12345678901234567890123456789012123456789012345678901
12345678901234567890123456789012123456789012345678901
CAT is almost two decades away, and you have an ample amount of time 2
2
12345678901234567890123456789012123456789012345678901
12345678901234567890123456789012123456789012345678901
to learn a ton of tough vocabulary words. Put the lollipop down and crack 2
2
12345678901234567890123456789012123456789012345678901
2
12345678901234567890123456789012123456789012345678901
open that dictionary, youngster!
2
12345678901234567890123456789012123456789012345678901
2
12345678901234567890123456789012123456789012345678901
Sadly, most of you reading this book do not have as much time as the wee 2
12345678901234567890123456789012123456789012345678901
2
12345678901234567890123456789012123456789012345678901
child described above. Thats okay, too! It means you have an actual life. 2
12345678901234567890123456789012123456789012345678901
2
12345678901234567890123456789012123456789012345678901
The simple fact is that the more time you have before the test, the more 2
12345678901234567890123456789012123456789012345678901
2
12345678901234567890123456789012123456789012345678901
vocabulary words you can learn.
2
12345678901234567890123456789012123456789012345678901
2345678901234567890123456789012123456789012345678901
2
1
2
12345678901234567890123456789012123456789012345678901
2
12345678901234567890123456789012123456789012345678901
2
12345678901234567890123456789012123456789012345678901
Tips on Learning New Vocabulary
2
12345678901234567890123456789012123456789012345678901
2345678901234567890123456789012123456789012345678901
12345678901234567890123456789012123456789012345678901
1. Strive for Quality over Quantity. Sort of learning the definition of 2
2
12345678901234567890123456789012123456789012345678901
12345678901234567890123456789012123456789012345678901
1,000 words is not as good as actually learning 100 words. Building 2
2
12345678901234567890123456789012123456789012345678901
vocabulary for a single test is a bit problematic to begin with, since theres 2
12345678901234567890123456789012123456789012345678901
2
12345678901234567890123456789012123456789012345678901
1 no way you can cover all the possible words that might appear. Therefore, 2
2
12345678901234567890123456789012123456789012345678901
dont blow a gasket trying to guarantee youll recognize every word you 2
12345678901234567890123456789012123456789012345678901
2
12345678901234567890123456789012123456789012345678901
see, since it just isnt going to happen. Instead, make sure the new words 2
12345678901234567890123456789012123456789012345678901
12345678901234567890123456789012123456789012345678901
you do learn are remembered. That way, if one of these words shows up as 2
2
12345678901234567890123456789012123456789012345678901
2
12345678901234567890123456789012123456789012345678901
an
antonym,
you
have
a
great
chance
to
answer
that
question
correctly.
2
12345678901234567890123456789012123456789012345678901
2345678901234567890123456789012123456789012345678901
12345678901234567890123456789012123456789012345678901 22
12345678901234567890123456789012123456789012345678901
People learn at different rates. Learning about 30 new words a week is a 2
12345678901234567890123456789012123456789012345678901
2
12345678901234567890123456789012123456789012345678901
1 nice goal. If you start six weeks before the test, that gives you almost 200 2
2345678901234567890123456789012123456789012345678901
12345678901234567890123456789012123456789012345678901
new words. Feel free to adjust the 30 words/week figure up or down 2
2
12345678901234567890123456789012123456789012345678901
1 depending on your own ability and available time. Just make sure that you 2
12345678901234567890123456789012123456789012345678901 2
12345678901234567890123456789012123456789012345678901
actually learnand rememberthe definitions of the words you are 2
2
12345678901234567890123456789012123456789012345678901
2
12345678901234567890123456789012123456789012345678901
studying.
12345678901234567890123456789012123456789012345678901 2
2
12345678901234567890123456789012123456789012345678901
2. Dont Let an Unknown Word Cross Your Path Again. The best way to 2
12345678901234567890123456789012123456789012345678901
2345678901234567890123456789012123456789012345678901
2
12345678901234567890123456789012123456789012345678901
learn new words is when a word appears in the context of other words. 2
12345678901234567890123456789012123456789012345678901
1 Suppose you happen to be reading a magazine, book, or newspaper when 2
12345678901234567890123456789012123456789012345678901 2
12345678901234567890123456789012123456789012345678901
you come across the word charivari. Looking at the context of the word 2
2
12345678901234567890123456789012123456789012345678901
2
12345678901234567890123456789012123456789012345678901
(how
the
word
is
used
in
the
sentence),
you
can
sort
of
figure
out
its
2
12345678901234567890123456789012123456789012345678901
2
12345678901234567890123456789012123456789012345678901
meaning.
This
allows
you
to
keep
on
reading,
even
if
you
dont
know
12345678901234567890123456789012123456789012345678901 2
2345678901234567890123456789012123456789012345678901
2
12345678901234567890123456789012123456789012345678901
exactly what charivari means.
2
12345678901234567890123456789012123456789012345678901
12345678901234567890123456789012123456789012345678901 22
12345678901234567890123456789012123456789012345678901
1 Instead of settling for this quasi-comprehension, write the unknown word 2
12345678901234567890123456789012123456789012345678901
down and then look it up later. Or look it up right then, if you have a 2
2
12345678901234567890123456789012123456789012345678901
12345678901234567890123456789012123456789012345678901
dictionary nearby. The goal is to add that word to your mental stable of 2
2
12345678901234567890123456789012123456789012345678901
12345678901234567890123456789012123456789012345678901
words, and its much easier to do this with words that you actually 2
2345678901234567890123456789012123456789012345678901
12345678901234567890123456789012123456789012345678901 22
1 encounter in the course of a day. Encountering the word in context helps
2
12345678901234567890123456789012123456789012345678901
2
12345678901234567890123456789012123456789012345678901
anchor the word in your mind.
2
12345678901234567890123456789012123456789012345678901
2345678901234567890123456789012123456789012345678901
2
1
12345678901234567890123456789012123456789012345678901 2
12345678901234567890123456789012123456789012345678901 2
12345678901234567890123456789012123456789012345678901 2 137
123456789012345678901234567890121234567890123456789012

http://www.xtremepapers.net

Part III: Section Review

www.petersons.com

http://www.xtremepapers.net

Note

123456789012345678901234567890121234567890123456789012
12345678901234567890123456789012123456789012345678901 2
2
12345678901234567890123456789012123456789012345678901
12345678901234567890123456789012123456789012345678901 2
12345678901234567890123456789012123456789012345678901
You can keep a small notebook with you to write unknown words down, 2
2
12345678901234567890123456789012123456789012345678901
2
12345678901234567890123456789012123456789012345678901
or
if
youre
techno-savvy
you
can
store
the
words
in
a
PDA
or
some
2
12345678901234567890123456789012123456789012345678901
2345678901234567890123456789012123456789012345678901
1 similar device. For optimal benefit, write down the unknown word and 2
2
12345678901234567890123456789012123456789012345678901
12345678901234567890123456789012123456789012345678901
the sentence in which it appears. When you look up the definition, you 2
2
12345678901234567890123456789012123456789012345678901
12345678901234567890123456789012123456789012345678901
can then add the definition to the word and the sample sentence you 2
2345678901234567890123456789012123456789012345678901
2
1
12345678901234567890123456789012123456789012345678901
already have. It almost goes without saying that creating a list this way 2
2
12345678901234567890123456789012123456789012345678901
2
12345678901234567890123456789012123456789012345678901
gives you a vocabulary review sheet at the same time.
2
12345678901234567890123456789012123456789012345678901
2345678901234567890123456789012123456789012345678901
2
1
2
12345678901234567890123456789012123456789012345678901
12345678901234567890123456789012123456789012345678901
Writing down and learning the meaning of unknown words will help 2
2
12345678901234567890123456789012123456789012345678901
improve your vocabulary, and it will have benefits beyond the GRE CAT 2
12345678901234567890123456789012123456789012345678901
2345678901234567890123456789012123456789012345678901
1 Verbal. You might be able to use some of these tough words on the 2
2
12345678901234567890123456789012123456789012345678901
2
12345678901234567890123456789012123456789012345678901
Analytical essay, and in general youll just have more words at your 2
12345678901234567890123456789012123456789012345678901
2
12345678901234567890123456789012123456789012345678901
disposal on a day-by-day basis. You will find that a large vocabulary can 2
12345678901234567890123456789012123456789012345678901
2
12345678901234567890123456789012123456789012345678901
be very helpful in social, educational, and business settings.
2
12345678901234567890123456789012123456789012345678901
2
12345678901234567890123456789012123456789012345678901
2
12345678901234567890123456789012123456789012345678901
3.
Do
Some
Test-Specific
Vocabulary
Reading.
If
context
learning
is
the
2345678901234567890123456789012123456789012345678901
12345678901234567890123456789012123456789012345678901 2
1 best way to pump up your GRE vocabulary, the second-best way is to buy 2
2
12345678901234567890123456789012123456789012345678901
2
12345678901234567890123456789012123456789012345678901
a
Boost
Your
GRE
Vocabulary
book.
There
are
also
a
slew
of
Web
sites
2
12345678901234567890123456789012123456789012345678901
2345678901234567890123456789012123456789012345678901
2
12345678901234567890123456789012123456789012345678901
covering GRE vocabulary.
2
12345678901234567890123456789012123456789012345678901
12345678901234567890123456789012123456789012345678901 2
12345678901234567890123456789012123456789012345678901
Regardless of what source you use, a GRE-specific vocabulary list can help 2
2
12345678901234567890123456789012123456789012345678901
12345678901234567890123456789012123456789012345678901
focus your studies to cover words that have a good likelihood of appearing 2
2
12345678901234567890123456789012123456789012345678901
on the GRE. However, this advantage is offset by two opposing factors. 2
12345678901234567890123456789012123456789012345678901
2
12345678901234567890123456789012123456789012345678901
1 The first factor is that learning words from a book does not guarantee that 2
2345678901234567890123456789012123456789012345678901
2
12345678901234567890123456789012123456789012345678901
youll remember the word in the future. Typically, the high density of new 2
12345678901234567890123456789012123456789012345678901
1 words you encounter and the artificial method in which you experience 2
12345678901234567890123456789012123456789012345678901 2
12345678901234567890123456789012123456789012345678901
them translates to a lower retention level. Instead of anchoring the word in 2
2
12345678901234567890123456789012123456789012345678901
12345678901234567890123456789012123456789012345678901
your mind, you might see it on the test and say, Oh yeah, I remember that 2
2
12345678901234567890123456789012123456789012345678901
2
12345678901234567890123456789012123456789012345678901
was
one
of
the
words
I
studied.
This
recollection
might
lead
you
to
recall
2
12345678901234567890123456789012123456789012345678901
2345678901234567890123456789012123456789012345678901
12345678901234567890123456789012123456789012345678901
the words definition, but often all you have is a vague idea of what the 2
2
12345678901234567890123456789012123456789012345678901
2
1 word means. Still, this is better than nothing.
12345678901234567890123456789012123456789012345678901 2
12345678901234567890123456789012123456789012345678901 2
12345678901234567890123456789012123456789012345678901
The second problem is that even if you do memorize 200 GRE-specific 2
2
12345678901234567890123456789012123456789012345678901
12345678901234567890123456789012123456789012345678901
words, the chances you will encounter a bunch of them on the actual test is 2
2
12345678901234567890123456789012123456789012345678901
12345678901234567890123456789012123456789012345678901
still rather small. Picture the Gulf of Mexico in your mind. Even if you 2
2345678901234567890123456789012123456789012345678901
12345678901234567890123456789012123456789012345678901 2
12345678901234567890123456789012123456789012345678901
know every fish off the coast of Florida, does this mean you know every 2
2
12345678901234567890123456789012123456789012345678901
12345678901234567890123456789012123456789012345678901
fish in the entire gulf? The answer is no, because the volume of water in the 2
1 gulf is simply too big. The same holds true for the amount of vocabulary 2
2
12345678901234567890123456789012123456789012345678901
2
12345678901234567890123456789012123456789012345678901
that might appear on the GRE. The number of possible words is too large 2
12345678901234567890123456789012123456789012345678901
2
12345678901234567890123456789012123456789012345678901
for any single book, or volume of books, to contain.
2
12345678901234567890123456789012123456789012345678901
2345678901234567890123456789012123456789012345678901
12345678901234567890123456789012123456789012345678901 2
1 This doesnt mean you must gnash your teeth and scream to the stars 2
2
12345678901234567890123456789012123456789012345678901
2
12345678901234567890123456789012123456789012345678901
about
the
injustice
of
the
GRE,
although
if
that
makes
you
feel
better,
go
2
12345678901234567890123456789012123456789012345678901
2345678901234567890123456789012123456789012345678901
1 right ahead. Instead, simply realize that you will not know the exact 2
12345678901234567890123456789012123456789012345678901 2
12345678901234567890123456789012123456789012345678901 2
2
138 12345678901234567890123456789012123456789012345678901
123456789012345678901234567890121234567890123456789012

Chapter 5: Verbal Review

Tip

123456789012345678901234567890121234567890123456789012
12345678901234567890123456789012123456789012345678901 2
2
12345678901234567890123456789012123456789012345678901
definition of every word you encounter. Thats fine. Everybody else taking 2
12345678901234567890123456789012123456789012345678901
2
12345678901234567890123456789012123456789012345678901
the GRE is in the same boat as you, to one extent or another. Studying 2
12345678901234567890123456789012123456789012345678901
2
12345678901234567890123456789012123456789012345678901
from a GRE-specific list might place 13 problems right in your lap. Since 2
12345678901234567890123456789012123456789012345678901
2345678901234567890123456789012123456789012345678901
1 the test is only 30 questions long, every question helps. If some recently 2
2
12345678901234567890123456789012123456789012345678901
2
12345678901234567890123456789012123456789012345678901
learned
vocabulary
helps
you
on
the
first
5
questions,
then
it
will
have
2
12345678901234567890123456789012123456789012345678901
2
12345678901234567890123456789012123456789012345678901
been
worth
it.
2345678901234567890123456789012123456789012345678901
2
1
2
12345678901234567890123456789012123456789012345678901
2
12345678901234567890123456789012123456789012345678901
The
rest
of
the
chapter
will
cover
various
question-specific
techniques
you
2
12345678901234567890123456789012123456789012345678901
2
12345678901234567890123456789012123456789012345678901
can
employ.
Vocabulary
was
covered
first
since
some
of
these
techniques
2345678901234567890123456789012123456789012345678901
2
1
2
12345678901234567890123456789012123456789012345678901
hinge
upon
whether
or
not
you
know
the
definition
of
the
words
used.
2
12345678901234567890123456789012123456789012345678901
2
12345678901234567890123456789012123456789012345678901
Compiling
new
vocabulary
words
and
learning
questions
strategies
go
2
12345678901234567890123456789012123456789012345678901
2345678901234567890123456789012123456789012345678901
2
1 hand in hand, so lets get started.
2
12345678901234567890123456789012123456789012345678901
2
12345678901234567890123456789012123456789012345678901
2
12345678901234567890123456789012123456789012345678901
2
12345678901234567890123456789012123456789012345678901
2345678901234567890123456789012123456789012345678901
12345678901234567890123456789012123456789012345678901 22
Antonyms
12345678901234567890123456789012123456789012345678901 2
12345678901234567890123456789012123456789012345678901 2
12345678901234567890123456789012123456789012345678901
In test-taking lingo, a GRE antonym question consists of a stem word
2
12345678901234567890123456789012123456789012345678901
2
12345678901234567890123456789012123456789012345678901
2
1 followed by 5 answer choices. For example:
2
12345678901234567890123456789012123456789012345678901
2
12345678901234567890123456789012123456789012345678901
CIRCUITOUS
:
2
12345678901234567890123456789012123456789012345678901
2345678901234567890123456789012123456789012345678901
12345678901234567890123456789012123456789012345678901 22
12345678901234567890123456789012123456789012345678901
A. direct
2
12345678901234567890123456789012123456789012345678901
2
12345678901234567890123456789012123456789012345678901
B.
electrical
12345678901234567890123456789012123456789012345678901 22
12345678901234567890123456789012123456789012345678901
C. wooden
2
12345678901234567890123456789012123456789012345678901
2
12345678901234567890123456789012123456789012345678901
D.
oblique
12345678901234567890123456789012123456789012345678901 22
1
E. long-winded
2
12345678901234567890123456789012123456789012345678901
2
12345678901234567890123456789012123456789012345678901
The stem word is circuitous, and one of the five answer choices below this 2
12345678901234567890123456789012123456789012345678901
2
12345678901234567890123456789012123456789012345678901
stem word has a meaning that is most nearly opposite in meaning to 2
12345678901234567890123456789012123456789012345678901
2345678901234567890123456789012123456789012345678901
2
12345678901234567890123456789012123456789012345678901
circuitous. Dont look yet at the answer choices, though. The best way to 2
12345678901234567890123456789012123456789012345678901
1 approach every antonym problem is to focus on the stem word first and 2
2
12345678901234567890123456789012123456789012345678901
2
12345678901234567890123456789012123456789012345678901
answer
this
question
as
honestly
as
possible:
Do
I
know
the
meaning
of
2345678901234567890123456789012123456789012345678901
12345678901234567890123456789012123456789012345678901 22
12345678901234567890123456789012123456789012345678901
2
1 this word?
12345678901234567890123456789012123456789012345678901 2
12345678901234567890123456789012123456789012345678901 2
2
12345678901234567890123456789012123456789012345678901
Although these questions are commonly referred to as antonyms, not
2
12345678901234567890123456789012123456789012345678901
2
12345678901234567890123456789012123456789012345678901
every
stem
word/correct
answer
is
an
antonym
pair
in
the
purest
sense.
2
12345678901234567890123456789012123456789012345678901
2
12345678901234567890123456789012123456789012345678901
This
is
why
the
phrase
most
nearly
opposite
(or
something
similar)
will
2345678901234567890123456789012123456789012345678901
12345678901234567890123456789012123456789012345678901 22
12345678901234567890123456789012123456789012345678901
be used to describe these questions. The phrase is vague enough to give
2
12345678901234567890123456789012123456789012345678901
2
12345678901234567890123456789012123456789012345678901
the
test-writers
wiggle
room
if
they
want
to
use
two
words
that
arent
2
1
2345678901234567890123456789012123456789012345678901
2
12345678901234567890123456789012123456789012345678901
exactly antonyms, but do have meanings that are fairly opposite.
2
12345678901234567890123456789012123456789012345678901
12345678901234567890123456789012123456789012345678901 22
12345678901234567890123456789012123456789012345678901 2
1 This question triggers your approach to every antonym problem. Looking
2
12345678901234567890123456789012123456789012345678901
2
12345678901234567890123456789012123456789012345678901
at
the
answer
choices
before
answering
it
might
influence
your
thinking,
as
2
12345678901234567890123456789012123456789012345678901
2
12345678901234567890123456789012123456789012345678901
you
make
an
incorrect
connection
with
the
stem
word
and
an
answer
2
12345678901234567890123456789012123456789012345678901
2345678901234567890123456789012123456789012345678901
1 choice that is incorrect. The reason for this common mistake has to do 2
12345678901234567890123456789012123456789012345678901 2
12345678901234567890123456789012123456789012345678901 2
12345678901234567890123456789012123456789012345678901 2 139
123456789012345678901234567890121234567890123456789012

http://www.xtremepapers.net

Part III: Section Review

123456789012345678901234567890121234567890123456789012
12345678901234567890123456789012123456789012345678901 2
2
12345678901234567890123456789012123456789012345678901
with the way wrong answer choices are created. In general, wrong answer 2
12345678901234567890123456789012123456789012345678901
2
12345678901234567890123456789012123456789012345678901
choices often take the form of words that can commonly be associated 2
12345678901234567890123456789012123456789012345678901
2
12345678901234567890123456789012123456789012345678901
with the stem word. Call this the Salt-and-Pepper Trap. The stem word 2
12345678901234567890123456789012123456789012345678901
2345678901234567890123456789012123456789012345678901
1 is salt, and directly below it is the answer choice pepper. Your mind 2
2
12345678901234567890123456789012123456789012345678901
2
12345678901234567890123456789012123456789012345678901
commonly
associates
these
two
items
together,
in
kind
of
an
opposite
way.
2
12345678901234567890123456789012123456789012345678901
2
12345678901234567890123456789012123456789012345678901
So
like
the
old
tongue-twister
you
pick
pepper,
even
though
the
two
words
2345678901234567890123456789012123456789012345678901
2
1
2
12345678901234567890123456789012123456789012345678901
are
not
antonyms.
They
arent
really
related
in
any
way
except
for
the
fact
2
12345678901234567890123456789012123456789012345678901
2
12345678901234567890123456789012123456789012345678901
that
they
are
both
common
spices.
Focusing
on
the
stem
word
definition
2
12345678901234567890123456789012123456789012345678901
2345678901234567890123456789012123456789012345678901
2
1 first prevents you from falling into the Salt-and-Pepper Trap. Achoo!
2
12345678901234567890123456789012123456789012345678901
2
12345678901234567890123456789012123456789012345678901
2
12345678901234567890123456789012123456789012345678901
Answering
the
question,
Do
I
know
the
meaning
of
the
stem
word?
2
12345678901234567890123456789012123456789012345678901
2345678901234567890123456789012123456789012345678901
1 produces three outcomes, Yes, No, and Maybe. Each of these 2
2
12345678901234567890123456789012123456789012345678901
2
12345678901234567890123456789012123456789012345678901
answers leads to a different technique.
2
12345678901234567890123456789012123456789012345678901
2
12345678901234567890123456789012123456789012345678901
2345678901234567890123456789012123456789012345678901
12345678901234567890123456789012123456789012345678901 2
12345678901234567890123456789012123456789012345678901 2
12345678901234567890123456789012123456789012345678901
Outcome #1: Yes, I Know the Meaning of the Word 2
2
12345678901234567890123456789012123456789012345678901
2
12345678901234567890123456789012123456789012345678901
As
you
might
expect,
this
is
the
answer
you
want
the
most.
If
you
do
know
12345678901234567890123456789012123456789012345678901 2
1 the meaning of circuitous, define it mentally in a sentence like, Circuitous 2
2
12345678901234567890123456789012123456789012345678901
2
12345678901234567890123456789012123456789012345678901
means
a
long,
round-about
course
or
path.
Now,
anticipate
what
the
2
12345678901234567890123456789012123456789012345678901
2345678901234567890123456789012123456789012345678901
12345678901234567890123456789012123456789012345678901
antonym of circuitous will be. By anticipating the antonym, you approach 2
2
12345678901234567890123456789012123456789012345678901
2
12345678901234567890123456789012123456789012345678901
the
answer
choices
with
a
focus
that
will
help
you
separate
the
wrong
12345678901234567890123456789012123456789012345678901 2
12345678901234567890123456789012123456789012345678901
answers from the better choices. People who dont approach the answer 2
2
12345678901234567890123456789012123456789012345678901
2
12345678901234567890123456789012123456789012345678901
choices
with
such
a
clear
focus
often
shop
the
answer
choices,
looking
at
12345678901234567890123456789012123456789012345678901 2
2
12345678901234567890123456789012123456789012345678901
1 each one and trying them on to see if they fit as the stem words antonyms. 2
2345678901234567890123456789012123456789012345678901
12345678901234567890123456789012123456789012345678901
Since these wrong answers are designed to be tempting, this approach can 2
2
12345678901234567890123456789012123456789012345678901
2
1 easily lead to a poor choice.
12345678901234567890123456789012123456789012345678901 2
2
12345678901234567890123456789012123456789012345678901
12345678901234567890123456789012123456789012345678901
By anticipating the antonym, you give yourself a clear idea of what you are 2
2
12345678901234567890123456789012123456789012345678901
looking for. You dont have to spend minutes crafting a perfect response, 2
12345678901234567890123456789012123456789012345678901
2
12345678901234567890123456789012123456789012345678901
just compose a mental sentence along the lines of, the antonym of 2
12345678901234567890123456789012123456789012345678901
2345678901234567890123456789012123456789012345678901
2
12345678901234567890123456789012123456789012345678901
circuitous will be a word like straight or blunt.
2
12345678901234567890123456789012123456789012345678901
2
1
2
12345678901234567890123456789012123456789012345678901
A.
direct
12345678901234567890123456789012123456789012345678901 2
2345678901234567890123456789012123456789012345678901
2
12345678901234567890123456789012123456789012345678901
B. electrical
2
12345678901234567890123456789012123456789012345678901
2
12345678901234567890123456789012123456789012345678901
C.
wooden
12345678901234567890123456789012123456789012345678901 2
2
1
D. oblique
2
12345678901234567890123456789012123456789012345678901
2
12345678901234567890123456789012123456789012345678901
E.
long-winded
2
12345678901234567890123456789012123456789012345678901
2
12345678901234567890123456789012123456789012345678901
12345678901234567890123456789012123456789012345678901
Using this mental sentence as a guide, choice (A) stands out as the correct 2
2345678901234567890123456789012123456789012345678901
12345678901234567890123456789012123456789012345678901 2
12345678901234567890123456789012123456789012345678901
answer. You might not know exactly what oblique means, so you dont 2
2
12345678901234567890123456789012123456789012345678901
12345678901234567890123456789012123456789012345678901
have to cross out that answer if you dont want to. All other choices are 2
1 clearly wrong, although if you went shopping in the answer choices, you 2
2
12345678901234567890123456789012123456789012345678901
2
12345678901234567890123456789012123456789012345678901
might have liked electrical since the word circuitous has the word circuit 2
12345678901234567890123456789012123456789012345678901
2
12345678901234567890123456789012123456789012345678901
in it.
2
12345678901234567890123456789012123456789012345678901
2345678901234567890123456789012123456789012345678901
2
1
12345678901234567890123456789012123456789012345678901 2
12345678901234567890123456789012123456789012345678901 2
2
140 12345678901234567890123456789012123456789012345678901
123456789012345678901234567890121234567890123456789012

www.petersons.com

http://www.xtremepapers.net

Chapter 5: Verbal Review

123456789012345678901234567890121234567890123456789012
12345678901234567890123456789012123456789012345678901 2
2
12345678901234567890123456789012123456789012345678901
If you know the meaning of the stem word, chances are good you would 2
12345678901234567890123456789012123456789012345678901
2
12345678901234567890123456789012123456789012345678901
get the answer correct even without anticipating the correct choice. Even 2
12345678901234567890123456789012123456789012345678901
2
12345678901234567890123456789012123456789012345678901
so, anticipating the correct answer virtually eliminates the chance of a 2
12345678901234567890123456789012123456789012345678901
2345678901234567890123456789012123456789012345678901
1 wrong answer. Its a sound test-taking strategy, and one you should always 2
2
12345678901234567890123456789012123456789012345678901
2
12345678901234567890123456789012123456789012345678901
employ.
2
12345678901234567890123456789012123456789012345678901
2
12345678901234567890123456789012123456789012345678901
2345678901234567890123456789012123456789012345678901
2
1
2
12345678901234567890123456789012123456789012345678901
2
12345678901234567890123456789012123456789012345678901
Outcome
#2:
I
Sort
of
Know
the
Meaning
of
That
2
12345678901234567890123456789012123456789012345678901
2
12345678901234567890123456789012123456789012345678901
Word
.
.
.
Maybe
2345678901234567890123456789012123456789012345678901
2
1
2
12345678901234567890123456789012123456789012345678901
There
are
many
words
that
fall
into
this
category.
They
are
like
the
2
12345678901234567890123456789012123456789012345678901
2
12345678901234567890123456789012123456789012345678901
neighbors
you
occasionally
see
down
the
street
or
the
people
who
work
at
2
12345678901234567890123456789012123456789012345678901
2345678901234567890123456789012123456789012345678901
1 your local grocery store. You recognize them when you see them, but if 2
2
12345678901234567890123456789012123456789012345678901
2
12345678901234567890123456789012123456789012345678901
pressed,
you
have
to
admit
that
you
dont
know
that
much
about
them.
2
12345678901234567890123456789012123456789012345678901
2
12345678901234567890123456789012123456789012345678901
2345678901234567890123456789012123456789012345678901
12345678901234567890123456789012123456789012345678901
People encounter these passing acquaintance words every day. If you are 2
2
12345678901234567890123456789012123456789012345678901
12345678901234567890123456789012123456789012345678901
following the suggestions listed earlier in this chapter, you are writing 2
12345678901234567890123456789012123456789012345678901 22
12345678901234567890123456789012123456789012345678901
down these words and then looking up their definitions later. This allows
2
12345678901234567890123456789012123456789012345678901
1 you to change your answer from Maybe I know this word to Yes, I 2
2
12345678901234567890123456789012123456789012345678901
know this word, something that greatly helps your GRE Verbal chances. 2
12345678901234567890123456789012123456789012345678901
2
12345678901234567890123456789012123456789012345678901
Still, there will always be some words that you recognize but can never 2
12345678901234567890123456789012123456789012345678901
2
12345678901234567890123456789012123456789012345678901
quite define.
2
12345678901234567890123456789012123456789012345678901
2
12345678901234567890123456789012123456789012345678901
2
12345678901234567890123456789012123456789012345678901
2345678901234567890123456789012123456789012345678901
2
12345678901234567890123456789012123456789012345678901
BRAVADO :
2
12345678901234567890123456789012123456789012345678901
12345678901234567890123456789012123456789012345678901 22
12345678901234567890123456789012123456789012345678901
A. callowness
2
1
2345678901234567890123456789012123456789012345678901
2
12345678901234567890123456789012123456789012345678901
B. greed
2
12345678901234567890123456789012123456789012345678901
2
1
C. swagger
12345678901234567890123456789012123456789012345678901 2
2
12345678901234567890123456789012123456789012345678901
D. timorousness
2
12345678901234567890123456789012123456789012345678901
E. hapless
2
12345678901234567890123456789012123456789012345678901
12345678901234567890123456789012123456789012345678901 2
2
12345678901234567890123456789012123456789012345678901
Assume that you dont quite know the definition of the stem word 2
12345678901234567890123456789012123456789012345678901
2345678901234567890123456789012123456789012345678901
12345678901234567890123456789012123456789012345678901
bravado. However, you have seen the word in some novels, and you think 2
2
12345678901234567890123456789012123456789012345678901
1 that brav- in bravado is the same as the brav- in brave or bravery. This 2
12345678901234567890123456789012123456789012345678901 2
12345678901234567890123456789012123456789012345678901
would mean that bravado means something similar to brave or courageous 2
2
12345678901234567890123456789012123456789012345678901
2
12345678901234567890123456789012123456789012345678901
.
.
.
if
the
bravis
the
same
for
both
words.
2
12345678901234567890123456789012123456789012345678901
2
12345678901234567890123456789012123456789012345678901
2
12345678901234567890123456789012123456789012345678901
The
above
train
of
thought
is
commendable,
and
it
shows
someone
2345678901234567890123456789012123456789012345678901
12345678901234567890123456789012123456789012345678901 22
12345678901234567890123456789012123456789012345678901
working to decipher the meaning of a word with an educated guess. This is 2
12345678901234567890123456789012123456789012345678901
2
12345678901234567890123456789012123456789012345678901
1 a good strategy. To determine whether or not the guess is correct, you need 2
2345678901234567890123456789012123456789012345678901
12345678901234567890123456789012123456789012345678901
to develop your awareness of how the computer adaptive GRE works. For 2
2
12345678901234567890123456789012123456789012345678901
12345678901234567890123456789012123456789012345678901
example, if bravado is the first question in the GRE Verbal section, which 2
12345678901234567890123456789012123456789012345678901 22
1 would imply that it is a question of medium difficulty. This question
2
12345678901234567890123456789012123456789012345678901
wont be simple, but a little worklike figuring out the definition by 2
12345678901234567890123456789012123456789012345678901
2
12345678901234567890123456789012123456789012345678901
comparing the root bravshould place you on the right course. In this 2
12345678901234567890123456789012123456789012345678901
2
12345678901234567890123456789012123456789012345678901
scenario, bravado probably does share some similarity with the more 2
12345678901234567890123456789012123456789012345678901
2
12345678901234567890123456789012123456789012345678901
common word brave.
2
12345678901234567890123456789012123456789012345678901
12345678901234567890123456789012123456789012345678901 2 141
123456789012345678901234567890121234567890123456789012

http://www.xtremepapers.net

Part III: Section Review

www.petersons.com

http://www.xtremepapers.net

Note

123456789012345678901234567890121234567890123456789012
12345678901234567890123456789012123456789012345678901 2
2
12345678901234567890123456789012123456789012345678901
However, if bravado is the fifth question, and youre very confident youve 2
12345678901234567890123456789012123456789012345678901
2
12345678901234567890123456789012123456789012345678901
answered the previous four questions correctly, then the scenario is 2
12345678901234567890123456789012123456789012345678901
2
12345678901234567890123456789012123456789012345678901
different. You should realize that your correct answers have caused the test 2
12345678901234567890123456789012123456789012345678901
2345678901234567890123456789012123456789012345678901
1 to adapt and start feeding you harder problems, so the root brav- in 2
2
12345678901234567890123456789012123456789012345678901
2
12345678901234567890123456789012123456789012345678901
bravado
is
a
decoy
put
there
to
catch
unwary
students.
Theres
no
way
a
2
12345678901234567890123456789012123456789012345678901
2
12345678901234567890123456789012123456789012345678901
hard
GRE
Verbal
question
would
be
so
easy
to
decipher.
2345678901234567890123456789012123456789012345678901
2
1
2
12345678901234567890123456789012123456789012345678901
2
12345678901234567890123456789012123456789012345678901
If
the
previous
paragraph
makes
sense
to
you,
then
you
are
well
on
your
2
12345678901234567890123456789012123456789012345678901
2
12345678901234567890123456789012123456789012345678901
way
to
becoming
a
great
test-taker.
2345678901234567890123456789012123456789012345678901
2
1
2
12345678901234567890123456789012123456789012345678901
12345678901234567890123456789012123456789012345678901
For this example, suppose bravado is the first question you encounter on 2
2
12345678901234567890123456789012123456789012345678901
12345678901234567890123456789012123456789012345678901
the GRE Verbal, which means the brav- root theory is valid. Since you sort 2
2345678901234567890123456789012123456789012345678901
2
1
12345678901234567890123456789012123456789012345678901
of know the definition, you can sort of anticipate an answer. The opposite 2
2
12345678901234567890123456789012123456789012345678901
of bravery would be cowardice or something similar, so the opposite of 2
12345678901234567890123456789012123456789012345678901
2
12345678901234567890123456789012123456789012345678901
bravado should be something like cowardice as well. Glancing at the 2
12345678901234567890123456789012123456789012345678901
2
12345678901234567890123456789012123456789012345678901
answer choices, you have:
2
12345678901234567890123456789012123456789012345678901
2
12345678901234567890123456789012123456789012345678901
2
12345678901234567890123456789012123456789012345678901
A.
callowness
2345678901234567890123456789012123456789012345678901
12345678901234567890123456789012123456789012345678901 2
2
1
B. greed
2
12345678901234567890123456789012123456789012345678901
2
12345678901234567890123456789012123456789012345678901
C.
swagger
2
12345678901234567890123456789012123456789012345678901
2345678901234567890123456789012123456789012345678901
2
12345678901234567890123456789012123456789012345678901
D. timorousness
2
12345678901234567890123456789012123456789012345678901
2
12345678901234567890123456789012123456789012345678901
E.
hapless
12345678901234567890123456789012123456789012345678901 2
12345678901234567890123456789012123456789012345678901 2
12345678901234567890123456789012123456789012345678901 2
12345678901234567890123456789012123456789012345678901
Some people like to think in terms of positive/negative words. Painting 2
2
12345678901234567890123456789012123456789012345678901
2
12345678901234567890123456789012123456789012345678901
with
broad
strokes,
positive
words
mean
things
like
larger/happier/better/
2
1
2345678901234567890123456789012123456789012345678901
12345678901234567890123456789012123456789012345678901
good, while negative words mean the opposite. Bravado has the root 2
2
12345678901234567890123456789012123456789012345678901
1 brav- in it, and bravery would be considered positive. You can could then 2
12345678901234567890123456789012123456789012345678901 2
12345678901234567890123456789012123456789012345678901
go into the answer choices and look for negative words. This technique 2
2
12345678901234567890123456789012123456789012345678901
12345678901234567890123456789012123456789012345678901
does not always work, though, as many answer choices are negative 2
12345678901234567890123456789012123456789012345678901 2
12345678901234567890123456789012123456789012345678901
words but with different meanings. Nevertheless, thinking in terms of 2
2
12345678901234567890123456789012123456789012345678901
2345678901234567890123456789012123456789012345678901
12345678901234567890123456789012123456789012345678901
positive/negative can help for some words in which the only thing you can 2
2
12345678901234567890123456789012123456789012345678901
1 recall about them is whether or not they mean something good or bad. 2
12345678901234567890123456789012123456789012345678901 2
12345678901234567890123456789012123456789012345678901 2
2
12345678901234567890123456789012123456789012345678901
These choices illustrate the influence that vocabulary has on the GRE 2
12345678901234567890123456789012123456789012345678901
2
12345678901234567890123456789012123456789012345678901
Verbal, since some of the answer choice words are as hard as or harder 2
12345678901234567890123456789012123456789012345678901
2
12345678901234567890123456789012123456789012345678901
than the stem word itself. Still, choices (B), (C), and (E) have nothing to do 2
12345678901234567890123456789012123456789012345678901
2
12345678901234567890123456789012123456789012345678901
with cowardice, so you can cross them out. This leaves you with a 50/50 2
12345678901234567890123456789012123456789012345678901
2
12345678901234567890123456789012123456789012345678901
chance, not bad considering you arent certain of the stem word definition. 2
12345678901234567890123456789012123456789012345678901
2345678901234567890123456789012123456789012345678901
2
12345678901234567890123456789012123456789012345678901
Therefore, the correct answer is (D).
2
12345678901234567890123456789012123456789012345678901
12345678901234567890123456789012123456789012345678901 2
12345678901234567890123456789012123456789012345678901 2
2
1
2
12345678901234567890123456789012123456789012345678901
12345678901234567890123456789012123456789012345678901 2
2
12345678901234567890123456789012123456789012345678901
12345678901234567890123456789012123456789012345678901 2
2
12345678901234567890123456789012123456789012345678901
12345678901234567890123456789012123456789012345678901 2
12345678901234567890123456789012123456789012345678901 2
12345678901234567890123456789012123456789012345678901 2
2
142 12345678901234567890123456789012123456789012345678901
123456789012345678901234567890121234567890123456789012

Chapter 5: Verbal Review

123456789012345678901234567890121234567890123456789012
12345678901234567890123456789012123456789012345678901 2
2
12345678901234567890123456789012123456789012345678901
2
12345678901234567890123456789012123456789012345678901
Outcome #3: No, I Have No Idea What That
2
12345678901234567890123456789012123456789012345678901
2
12345678901234567890123456789012123456789012345678901
Word
Means
12345678901234567890123456789012123456789012345678901 2
12345678901234567890123456789012123456789012345678901
For most people, the question is not if you will encounter a word you dont 2
2
12345678901234567890123456789012123456789012345678901
2
12345678901234567890123456789012123456789012345678901
know,
but
when
you
will
run
across
that
word.
Dont
beat
yourself
up
2
12345678901234567890123456789012123456789012345678901
2
12345678901234567890123456789012123456789012345678901
about
this,
as
the
words
used
on
the
GRE
Verbal
section
reflect
a
pretty
2
12345678901234567890123456789012123456789012345678901
2345678901234567890123456789012123456789012345678901
2
1 advanced vocabulary pool. For example:
2
12345678901234567890123456789012123456789012345678901
2
12345678901234567890123456789012123456789012345678901
2
12345678901234567890123456789012123456789012345678901
WINSOME
:
2
12345678901234567890123456789012123456789012345678901
2345678901234567890123456789012123456789012345678901
2
1
2
12345678901234567890123456789012123456789012345678901
A.
unpleasant
2
12345678901234567890123456789012123456789012345678901
2
12345678901234567890123456789012123456789012345678901
B.
defeated
2
12345678901234567890123456789012123456789012345678901
2345678901234567890123456789012123456789012345678901
2
1
C. garrulous
2
12345678901234567890123456789012123456789012345678901
2
12345678901234567890123456789012123456789012345678901
D.
whiny
2
12345678901234567890123456789012123456789012345678901
2
12345678901234567890123456789012123456789012345678901
E.
spurious
2345678901234567890123456789012123456789012345678901
12345678901234567890123456789012123456789012345678901 22
12345678901234567890123456789012123456789012345678901 2
12345678901234567890123456789012123456789012345678901
If you dont know what the word winsome means, and if your mind can
2
12345678901234567890123456789012123456789012345678901
12345678901234567890123456789012123456789012345678901
not even summon up a passing memory of seeing the word in print, then 2
12345678901234567890123456789012123456789012345678901 22
1 theres no way to anticipate the answer. Instead, you need to attack the
2
12345678901234567890123456789012123456789012345678901
answer choices, scrutinize them and try to eliminate unlikely answers, and 2
12345678901234567890123456789012123456789012345678901
2
12345678901234567890123456789012123456789012345678901
then take a guess.
2
12345678901234567890123456789012123456789012345678901
2
12345678901234567890123456789012123456789012345678901
2
12345678901234567890123456789012123456789012345678901
There
are
two
primary
ways
to
attack
answer
choices.
The
first
one
has
2
12345678901234567890123456789012123456789012345678901
2
12345678901234567890123456789012123456789012345678901
been
touched
on
in
the
previous
section,
where
you
took
your
knowledge
2345678901234567890123456789012123456789012345678901
12345678901234567890123456789012123456789012345678901 22
12345678901234567890123456789012123456789012345678901
of the adaptive test format and used it to your advantage. Suppose the 2
12345678901234567890123456789012123456789012345678901
2
12345678901234567890123456789012123456789012345678901
1 word winsome appears after you have answered several questions 2
2345678901234567890123456789012123456789012345678901
12345678901234567890123456789012123456789012345678901
correctly. Personally, you think its a tough word (which is true), and that 2
2
12345678901234567890123456789012123456789012345678901
1 you are in a hard portion of the GRE Verbal (a good assumption if youve 2
12345678901234567890123456789012123456789012345678901 2
12345678901234567890123456789012123456789012345678901
been answering questions correctly). This means theres no easy shortcut 2
2
12345678901234567890123456789012123456789012345678901
2
12345678901234567890123456789012123456789012345678901
on
this
problem.
In
fact,
anything
that
looks
like
a
shortcut
is
probably
too
12345678901234567890123456789012123456789012345678901 2
2
12345678901234567890123456789012123456789012345678901
good to be true, and can be eliminated.
2
12345678901234567890123456789012123456789012345678901
2345678901234567890123456789012123456789012345678901
2
12345678901234567890123456789012123456789012345678901
The root win- appears in the word winsome, which might lead some to 2
12345678901234567890123456789012123456789012345678901
1 think the word has something to do with victory or winning. Choice (B) 2
12345678901234567890123456789012123456789012345678901 2
2
12345678901234567890123456789012123456789012345678901
then practically screams, I must be the answer! Knowing where you are 2
12345678901234567890123456789012123456789012345678901
2
12345678901234567890123456789012123456789012345678901
on the test, and how tough a word winsome is, you can therefore eliminate 2
12345678901234567890123456789012123456789012345678901
12345678901234567890123456789012123456789012345678901
choice (B). You know its there to catch unwary test-takers looking for an 2
2
12345678901234567890123456789012123456789012345678901
2345678901234567890123456789012123456789012345678901
2
12345678901234567890123456789012123456789012345678901
easy out when there isnt one.
2
12345678901234567890123456789012123456789012345678901
12345678901234567890123456789012123456789012345678901 22
12345678901234567890123456789012123456789012345678901
1 One answer choice is out, but four remain. The second elimination 2
12345678901234567890123456789012123456789012345678901
technique can now be employed. Call it the Sandwich Technique. Try to 2
2
12345678901234567890123456789012123456789012345678901
2
12345678901234567890123456789012123456789012345678901
answer
the
question,
What
is
the
opposite
of
sandwich?
Is
it
a
pizza?
2
12345678901234567890123456789012123456789012345678901
2
12345678901234567890123456789012123456789012345678901
Soup?
A
salad?
The
point
is
there
is
no
opposite
word
for
sandwich.
Many
2345678901234567890123456789012123456789012345678901
12345678901234567890123456789012123456789012345678901 22
1 words have no opposite, or you have to get very creative to come up with
2
12345678901234567890123456789012123456789012345678901
12345678901234567890123456789012123456789012345678901
an opposite. Words without true opposites are very poor candidates to be 2
2
12345678901234567890123456789012123456789012345678901
12345678901234567890123456789012123456789012345678901
the correct answer on an Antonym problem, so you can cross them out on 2
2
12345678901234567890123456789012123456789012345678901
2
12345678901234567890123456789012123456789012345678901
hard problems.
12345678901234567890123456789012123456789012345678901 2 143
123456789012345678901234567890121234567890123456789012

http://www.xtremepapers.net

Part III: Section Review

123456789012345678901234567890121234567890123456789012
12345678901234567890123456789012123456789012345678901 2
2
12345678901234567890123456789012123456789012345678901
Looking over the four remaining choices, choice (A) has a very clear 2
12345678901234567890123456789012123456789012345678901
2
12345678901234567890123456789012123456789012345678901
opposite. You might not know what garrulous or spurious means, so you 2
12345678901234567890123456789012123456789012345678901
2
12345678901234567890123456789012123456789012345678901
cant eliminate them if you dont know what they mean. It turns out both 2
12345678901234567890123456789012123456789012345678901
2345678901234567890123456789012123456789012345678901
1 have valid opposites. Look at (D), though, whiny. What is the opposite of 2
2
12345678901234567890123456789012123456789012345678901
2
12345678901234567890123456789012123456789012345678901
whiny?
Unwhiny?
Polite?
Not
annoying?
You
can
make
a
case
that
there
is
2
12345678901234567890123456789012123456789012345678901
2
12345678901234567890123456789012123456789012345678901
a
word
that
means
the
opposite
of
whiny
.
.
.
but
you
have
to
make
a
case
2345678901234567890123456789012123456789012345678901
2
1
2
12345678901234567890123456789012123456789012345678901
in
order
to
do
so.
This
makes
it
unlikely
to
be
the
right
answer,
so
you
can
2
12345678901234567890123456789012123456789012345678901
2
12345678901234567890123456789012123456789012345678901
eliminate
choice
(D).
2
12345678901234567890123456789012123456789012345678901
2345678901234567890123456789012123456789012345678901
2
1
2
12345678901234567890123456789012123456789012345678901
You
are
left
with
three
choices.
Take
a
guess,
and
keep
in
mind
that
2
12345678901234567890123456789012123456789012345678901
2
12345678901234567890123456789012123456789012345678901
one-in-three
odds
are
not
too
bad
when
you
are
dealing
with
a
completely
2
12345678901234567890123456789012123456789012345678901
2345678901234567890123456789012123456789012345678901
1 unfamiliar word. The correct answer choice happens to be the one that 2
2
12345678901234567890123456789012123456789012345678901
12345678901234567890123456789012123456789012345678901
could easily be an antonym. A winsome person is charming and pleasant, 2
2
12345678901234567890123456789012123456789012345678901
2
12345678901234567890123456789012123456789012345678901
the opposite of unpleasant, choice (A).
2345678901234567890123456789012123456789012345678901
12345678901234567890123456789012123456789012345678901 2
2
12345678901234567890123456789012123456789012345678901
There are several methods for dealing with antonym problems, but 2
12345678901234567890123456789012123456789012345678901
2
12345678901234567890123456789012123456789012345678901
vocabulary plays a large part in the overall strategy. The same can also be 2
12345678901234567890123456789012123456789012345678901
2
12345678901234567890123456789012123456789012345678901
1 said of another vocabulary-centric question type, the analogy questions. 2
2
12345678901234567890123456789012123456789012345678901
2
12345678901234567890123456789012123456789012345678901
2
12345678901234567890123456789012123456789012345678901
2345678901234567890123456789012123456789012345678901
12345678901234567890123456789012123456789012345678901 2
1
ANALOGY
is to GRE as FEATHER is to OSTRICH 22
12345678901234567890123456789012123456789012345678901
12345678901234567890123456789012123456789012345678901
Analogies are similar in form to antonyms, and they also have a heavy 2
2
12345678901234567890123456789012123456789012345678901
2345678901234567890123456789012123456789012345678901
12345678901234567890123456789012123456789012345678901
vocabulary component. However, where there is only one stem word for 2
2
12345678901234567890123456789012123456789012345678901
2
12345678901234567890123456789012123456789012345678901
an
antonym,
an
analogy
question
starts
off
with
two
stem
words
that
bear
12345678901234567890123456789012123456789012345678901 2
1 some type of relationship to one another. As you will see, understanding 2
2
12345678901234567890123456789012123456789012345678901
2
12345678901234567890123456789012123456789012345678901
the
relationship
between
the
two
stem
words
is
the
critical
skill
to
master
12345678901234567890123456789012123456789012345678901 2
2
12345678901234567890123456789012123456789012345678901
when handling analogy problems.
2
12345678901234567890123456789012123456789012345678901
2345678901234567890123456789012123456789012345678901
12345678901234567890123456789012123456789012345678901 2
2
12345678901234567890123456789012123456789012345678901
FEATHER : OSTRICH ::
2
1
2
12345678901234567890123456789012123456789012345678901
2
12345678901234567890123456789012123456789012345678901
A.
beak
:
raven
2345678901234567890123456789012123456789012345678901
12345678901234567890123456789012123456789012345678901 2
2
12345678901234567890123456789012123456789012345678901
B. scale : lizard
2
1
2
12345678901234567890123456789012123456789012345678901
C.
foot
:
chicken
12345678901234567890123456789012123456789012345678901 2
2345678901234567890123456789012123456789012345678901
2
12345678901234567890123456789012123456789012345678901
D. desert : cactus
2
12345678901234567890123456789012123456789012345678901
2
12345678901234567890123456789012123456789012345678901
E.
tail
:
mammal
12345678901234567890123456789012123456789012345678901 2
2
1
12345678901234567890123456789012123456789012345678901
Theres a colon between the two stem words, but there is also some kind of 2
2
12345678901234567890123456789012123456789012345678901
12345678901234567890123456789012123456789012345678901
link between the two words. One of the answer choicesthe correct 2
2
12345678901234567890123456789012123456789012345678901
12345678901234567890123456789012123456789012345678901
onewill share this same link, or analogous relationship. Hence the term, 2
2345678901234567890123456789012123456789012345678901
12345678901234567890123456789012123456789012345678901 2
2
12345678901234567890123456789012123456789012345678901
analogies.
2
12345678901234567890123456789012123456789012345678901
12345678901234567890123456789012123456789012345678901 2
2
1
2
12345678901234567890123456789012123456789012345678901
12345678901234567890123456789012123456789012345678901 2
2
12345678901234567890123456789012123456789012345678901
12345678901234567890123456789012123456789012345678901 2
2
12345678901234567890123456789012123456789012345678901
12345678901234567890123456789012123456789012345678901 2
12345678901234567890123456789012123456789012345678901 2
12345678901234567890123456789012123456789012345678901 2
2
144 12345678901234567890123456789012123456789012345678901
123456789012345678901234567890121234567890123456789012

www.petersons.com

http://www.xtremepapers.net

Tip

Chapter 5: Verbal Review

123456789012345678901234567890121234567890123456789012
12345678901234567890123456789012123456789012345678901 2
2
12345678901234567890123456789012123456789012345678901
To get a clearer picture of what is needed on an analogy problem, it helps 2
12345678901234567890123456789012123456789012345678901
2
12345678901234567890123456789012123456789012345678901
to get rid of the colon and format the question in the following manner:
2
12345678901234567890123456789012123456789012345678901
12345678901234567890123456789012123456789012345678901 2
2
12345678901234567890123456789012123456789012345678901
2
12345678901234567890123456789012123456789012345678901
FEATHER _____________________________ OSTRICH ::
2
12345678901234567890123456789012123456789012345678901
2
12345678901234567890123456789012123456789012345678901
2
12345678901234567890123456789012123456789012345678901
A.
beak
______________________________
raven
2
12345678901234567890123456789012123456789012345678901
2345678901234567890123456789012123456789012345678901
2
1
B. scale ______________________________ lizard
2
12345678901234567890123456789012123456789012345678901
2
12345678901234567890123456789012123456789012345678901
C. foot _______________________________ chicken
2
12345678901234567890123456789012123456789012345678901
2
12345678901234567890123456789012123456789012345678901
D. desert ____________________________
__ cactus
2345678901234567890123456789012123456789012345678901
2
1
2
12345678901234567890123456789012123456789012345678901
E. tail ________________________________ mammal
2
12345678901234567890123456789012123456789012345678901
2
12345678901234567890123456789012123456789012345678901
If
the
question
were
set
up
this
way,
it
would
be
easy
to
see
that
what
you
2
12345678901234567890123456789012123456789012345678901
2345678901234567890123456789012123456789012345678901
1 need to do is write some text that shows the link between the two stem 2
2
12345678901234567890123456789012123456789012345678901
2
12345678901234567890123456789012123456789012345678901
words.
You
can
have
something
like
2
12345678901234567890123456789012123456789012345678901
2
12345678901234567890123456789012123456789012345678901
2345678901234567890123456789012123456789012345678901
2
12345678901234567890123456789012123456789012345678901
a FEATHER is part of an OSTRICH ::
2
12345678901234567890123456789012123456789012345678901
12345678901234567890123456789012123456789012345678901 22
12345678901234567890123456789012123456789012345678901
Creating a link between the two stem words is the first step of every 2
12345678901234567890123456789012123456789012345678901
2
12345678901234567890123456789012123456789012345678901
1 analogy problem. If you can create a good link, then you can take that link 2
12345678901234567890123456789012123456789012345678901
down into the answer choices and see which answer choices share that 2
2
12345678901234567890123456789012123456789012345678901
2
12345678901234567890123456789012123456789012345678901
link.
2345678901234567890123456789012123456789012345678901
12345678901234567890123456789012123456789012345678901 22
12345678901234567890123456789012123456789012345678901 2
12345678901234567890123456789012123456789012345678901 2
12345678901234567890123456789012123456789012345678901
Granted, you cant create a link between the stem words if you dont
2
12345678901234567890123456789012123456789012345678901
2
12345678901234567890123456789012123456789012345678901
know what one or both of the words means. Strategies to deal with these
2
12345678901234567890123456789012123456789012345678901
2
12345678901234567890123456789012123456789012345678901
circumstances are covered later in the chapter.
12345678901234567890123456789012123456789012345678901 22
1
2
12345678901234567890123456789012123456789012345678901
2
12345678901234567890123456789012123456789012345678901
A. beak is part of a raven
12345678901234567890123456789012123456789012345678901 2
2
12345678901234567890123456789012123456789012345678901
B. scale is part of a lizard
2
12345678901234567890123456789012123456789012345678901
C.
foot
is
part
of
a
chicken
2345678901234567890123456789012123456789012345678901
12345678901234567890123456789012123456789012345678901 22
12345678901234567890123456789012123456789012345678901
D. desert is part of a cactus
2
1
2
12345678901234567890123456789012123456789012345678901
E.
tail
is
part
of
a
mammal
2
12345678901234567890123456789012123456789012345678901
2345678901234567890123456789012123456789012345678901
12345678901234567890123456789012123456789012345678901 22
12345678901234567890123456789012123456789012345678901
1 A desert is NOT part of a cactus, so choice (D) is not going to be the right 2
12345678901234567890123456789012123456789012345678901
answer on this problem since it does not contain the same link as the two 2
2
12345678901234567890123456789012123456789012345678901
2345678901234567890123456789012123456789012345678901
12345678901234567890123456789012123456789012345678901
stem words (desert : cactus do share a link, its just not the right one for 2
2
12345678901234567890123456789012123456789012345678901
2
12345678901234567890123456789012123456789012345678901
this
problem).
Choices
(A),
(B),
(C),
and
(E)
all
seem
to
share
the
same
link.
12345678901234567890123456789012123456789012345678901 22
1 This does not mean that all four choices are correct. Instead, we must
2
12345678901234567890123456789012123456789012345678901
2
12345678901234567890123456789012123456789012345678901
refine
our
initial
link
and
be
a
little
more
precise
in
describing
how
the
2
12345678901234567890123456789012123456789012345678901
2
12345678901234567890123456789012123456789012345678901
words
feather
and
ostrich
are
connected.
A
little
bit
of
thinking
here
will
12345678901234567890123456789012123456789012345678901 2
2345678901234567890123456789012123456789012345678901
2
12345678901234567890123456789012123456789012345678901
make the problem easy to solve.
2
12345678901234567890123456789012123456789012345678901
2
12345678901234567890123456789012123456789012345678901
Some people might say that feathers are used by birds to fly. This is 2
12345678901234567890123456789012123456789012345678901
1 somewhat true, although it could be argued that the wings are what enable 2
2
12345678901234567890123456789012123456789012345678901
2
12345678901234567890123456789012123456789012345678901
a bird to fly. In any case, the ostrich is a flightless bird, so creating a link 2
12345678901234567890123456789012123456789012345678901
2
12345678901234567890123456789012123456789012345678901
that states, A feather is part of an ostrich that is used to fly would be 2
12345678901234567890123456789012123456789012345678901
2345678901234567890123456789012123456789012345678901
2
1 incorrect.
12345678901234567890123456789012123456789012345678901 2
12345678901234567890123456789012123456789012345678901 2
12345678901234567890123456789012123456789012345678901 2 145
123456789012345678901234567890121234567890123456789012

http://www.xtremepapers.net

Part III: Section Review

123456789012345678901234567890121234567890123456789012
12345678901234567890123456789012123456789012345678901 2
2
12345678901234567890123456789012123456789012345678901
What do the feathers do for an ostrich? Without over-thinking it, feathers 2
12345678901234567890123456789012123456789012345678901
2
12345678901234567890123456789012123456789012345678901
cover the outside of the bird and protect it in the same way that skin 2
12345678901234567890123456789012123456789012345678901
2
12345678901234567890123456789012123456789012345678901
protects us. You can then modify our initial linking sentence to read 2
12345678901234567890123456789012123456789012345678901
2345678901234567890123456789012123456789012345678901
2
1 something like:
2
12345678901234567890123456789012123456789012345678901
2
12345678901234567890123456789012123456789012345678901
2
12345678901234567890123456789012123456789012345678901
a
FEATHER
is
part
of
the
outer
covering
of
an
OSTRICH
2
12345678901234567890123456789012123456789012345678901
2345678901234567890123456789012123456789012345678901
2
1
12345678901234567890123456789012123456789012345678901
a FEATHER is the part of an OSTRICH found along the outside 2
2
12345678901234567890123456789012123456789012345678901
2
12345678901234567890123456789012123456789012345678901
of its body
2
12345678901234567890123456789012123456789012345678901
2345678901234567890123456789012123456789012345678901
1
a FEATHER is the part of an OSTRICH that covers it like skin 2
2
12345678901234567890123456789012123456789012345678901
2
12345678901234567890123456789012123456789012345678901
2
12345678901234567890123456789012123456789012345678901
All
three
of
these
sentences
give
a
more
precise
definition
of
the
connection
2
12345678901234567890123456789012123456789012345678901
2345678901234567890123456789012123456789012345678901
1 between feather and ostrich. Use any of them above and see how they fit in 2
2
12345678901234567890123456789012123456789012345678901
2
12345678901234567890123456789012123456789012345678901
with the remaining answer choices.
2
12345678901234567890123456789012123456789012345678901
2
12345678901234567890123456789012123456789012345678901
is
part
of
the
outer
covering
of
a
raven
A.
beak
2345678901234567890123456789012123456789012345678901
12345678901234567890123456789012123456789012345678901 2
2
12345678901234567890123456789012123456789012345678901
B. scale is part of the outer covering of a lizard
2
12345678901234567890123456789012123456789012345678901
2
12345678901234567890123456789012123456789012345678901
C.
foot
is
part
of
the
outer
covering
of
a
chicken
12345678901234567890123456789012123456789012345678901 2
2
12345678901234567890123456789012123456789012345678901
E. tail is part of the outer covering of a mammal
2
1
2
12345678901234567890123456789012123456789012345678901
You could try to make the case that choices (A), (C), and (E) are still valid, 2
12345678901234567890123456789012123456789012345678901
2
12345678901234567890123456789012123456789012345678901
but it should be clear that choice (B) is the best answer for this analogy 2
12345678901234567890123456789012123456789012345678901
2
12345678901234567890123456789012123456789012345678901
problem. Scales cover the outside of lizards, and feathers cover the exterior 2
12345678901234567890123456789012123456789012345678901
2
12345678901234567890123456789012123456789012345678901
of ostriches.
2
12345678901234567890123456789012123456789012345678901
2345678901234567890123456789012123456789012345678901
12345678901234567890123456789012123456789012345678901 2
12345678901234567890123456789012123456789012345678901
If you know both the stem words for an analogy question, then you should 2
2
12345678901234567890123456789012123456789012345678901
2
12345678901234567890123456789012123456789012345678901
focus
on
creating
a
link
that
describes
the
connection
between
the
two
2
1
2345678901234567890123456789012123456789012345678901
12345678901234567890123456789012123456789012345678901
words. Once this is done, you simply take that link and check it against all 2
2
12345678901234567890123456789012123456789012345678901
1 the answer choices to find the one that shares the same relationship. That 2
12345678901234567890123456789012123456789012345678901 2
2
12345678901234567890123456789012123456789012345678901
is your correct answer.
2
12345678901234567890123456789012123456789012345678901
2
12345678901234567890123456789012123456789012345678901
12345678901234567890123456789012123456789012345678901
Creating good links can take some practice. Quite often, you might start 2
2
12345678901234567890123456789012123456789012345678901
12345678901234567890123456789012123456789012345678901
out with a general link and then have to modify it to be more specific. The 2
2345678901234567890123456789012123456789012345678901
2
12345678901234567890123456789012123456789012345678901
chart below discusses some of the more common types of Analogy links 2
12345678901234567890123456789012123456789012345678901
2
1 that will be encountered on the GRE.
12345678901234567890123456789012123456789012345678901 2
12345678901234567890123456789012123456789012345678901 2
2
12345678901234567890123456789012123456789012345678901
2
12345678901234567890123456789012123456789012345678901
Common
Types
of
Analogies
2
12345678901234567890123456789012123456789012345678901
2
12345678901234567890123456789012123456789012345678901
1.
Synonyms:
Synonyms
are
fairly
simple
analogies,
since
the
two
words
12345678901234567890123456789012123456789012345678901 2
2345678901234567890123456789012123456789012345678901
12345678901234567890123456789012123456789012345678901
mean the same thing. An example would be PREVARICATE : LIE, 2
2
12345678901234567890123456789012123456789012345678901
2
12345678901234567890123456789012123456789012345678901
since
prevaricate
is
just
a
fancy
word
meaning
to
lie.
Synonym
12345678901234567890123456789012123456789012345678901 2
1
analogies like to combine one tough word with a simple one, leaving 2
2
12345678901234567890123456789012123456789012345678901
2
12345678901234567890123456789012123456789012345678901
you
to
wonder
what
the
relationship
between
the
words
are.
The
2
12345678901234567890123456789012123456789012345678901
2
12345678901234567890123456789012123456789012345678901
words
could
be
synonyms,
or
.
.
.
12345678901234567890123456789012123456789012345678901 2
2345678901234567890123456789012123456789012345678901
2
12345678901234567890123456789012123456789012345678901
1 2. Antonyms: These analogies are exactly what you would expect them 2
12345678901234567890123456789012123456789012345678901
to be. PREVARICATION: TRUTH is an example of an antonym 2
2
12345678901234567890123456789012123456789012345678901
2
12345678901234567890123456789012123456789012345678901
analogy.
Many
synonym
and
antonym
analogies
use
adjectives
as
the
2345678901234567890123456789012123456789012345678901
2
1
2
12345678901234567890123456789012123456789012345678901
stem
words.
12345678901234567890123456789012123456789012345678901 2
2
146 12345678901234567890123456789012123456789012345678901
123456789012345678901234567890121234567890123456789012

www.petersons.com

http://www.xtremepapers.net

Note

Chapter 5: Verbal Review

123456789012345678901234567890121234567890123456789012
12345678901234567890123456789012123456789012345678901 2
2
12345678901234567890123456789012123456789012345678901
3. Degree: Degree analogies combine two things that are similar in 2
12345678901234567890123456789012123456789012345678901
2
12345678901234567890123456789012123456789012345678901
nature but different in scale. BREEZE : GALE is a good example 2
12345678901234567890123456789012123456789012345678901
2
12345678901234567890123456789012123456789012345678901
since they are both winds, although a gale is much stronger.
2
12345678901234567890123456789012123456789012345678901
2345678901234567890123456789012123456789012345678901
2
1
2
12345678901234567890123456789012123456789012345678901
4.
Set/Subset:
Fans
of
categories
will
like
these
analogies.
Subset
2
12345678901234567890123456789012123456789012345678901
2
12345678901234567890123456789012123456789012345678901
analogies
often
employ
linking
phrases
such
as
kind
of
or
part
2
12345678901234567890123456789012123456789012345678901
2345678901234567890123456789012123456789012345678901
1
of. ELM : TREE is a simple example, since an elm is a kind of tree. 2
2
12345678901234567890123456789012123456789012345678901
2
12345678901234567890123456789012123456789012345678901
FEATHER
:
OSTRICH
could
also
be
considered
one
of
these
2
12345678901234567890123456789012123456789012345678901
2
12345678901234567890123456789012123456789012345678901
analogies,
but
if
you
disagree,
thats
fine,
too.
These
categories
2345678901234567890123456789012123456789012345678901
2
1
2
12345678901234567890123456789012123456789012345678901
should
be
used
if
they
help
you
come
up
with
good
links,
but
it
2
12345678901234567890123456789012123456789012345678901
2
12345678901234567890123456789012123456789012345678901
doesnt
really
matter
whether
or
not
each
category
is
distinct
from
all
2
12345678901234567890123456789012123456789012345678901
2345678901234567890123456789012123456789012345678901
2
1
the others.
2
12345678901234567890123456789012123456789012345678901
2
12345678901234567890123456789012123456789012345678901
5. Function: Certain things have been made with a specific purpose or 2
12345678901234567890123456789012123456789012345678901
2
12345678901234567890123456789012123456789012345678901
function. For instance, what do gloves do? GLOVES : HANDS is a 2
12345678901234567890123456789012123456789012345678901
2
12345678901234567890123456789012123456789012345678901
good example of a function relationship, since gloves are meant to be 2
12345678901234567890123456789012123456789012345678901
2
12345678901234567890123456789012123456789012345678901
worn on the hands.
2
12345678901234567890123456789012123456789012345678901
2345678901234567890123456789012123456789012345678901
12345678901234567890123456789012123456789012345678901 22
1
2
12345678901234567890123456789012123456789012345678901
You might also think that GLOVES : WARM is a good link, but this is not
2
12345678901234567890123456789012123456789012345678901
2
12345678901234567890123456789012123456789012345678901
as
strong.
Gloves
can
be
used
to
keep
your
hands
warm,
but
gloves
can
2345678901234567890123456789012123456789012345678901
12345678901234567890123456789012123456789012345678901 22
12345678901234567890123456789012123456789012345678901
also be worn for other purposes. A person wearing gloves while washing
2
12345678901234567890123456789012123456789012345678901
2
12345678901234567890123456789012123456789012345678901
dishes
is
wearing
gloves
for
protection,
not
warmth.
The
two
stem
words
12345678901234567890123456789012123456789012345678901 22
12345678901234567890123456789012123456789012345678901
will always have a strong link, but the words in the answer choices do not
2
12345678901234567890123456789012123456789012345678901
2
12345678901234567890123456789012123456789012345678901
always
contain
strong
links.
If
you
saw
GLOVES
:
WARM
as
an
answer
12345678901234567890123456789012123456789012345678901 22
1 choice, you could eliminate it as a likely answer choice because the link
2
12345678901234567890123456789012123456789012345678901
2
12345678901234567890123456789012123456789012345678901
between
the
two
words
is
not
very
strong.
12345678901234567890123456789012123456789012345678901 2
12345678901234567890123456789012123456789012345678901 2
2
12345678901234567890123456789012123456789012345678901
12345678901234567890123456789012123456789012345678901
6. Definitional: This is a big catch-all category. One stem word often 2
2
12345678901234567890123456789012123456789012345678901
2
12345678901234567890123456789012123456789012345678901
plays
a
big
part
in
defining
the
other
stem
word.
CANDLE
:
WAX
is
2
12345678901234567890123456789012123456789012345678901
2
12345678901234567890123456789012123456789012345678901
an
example,
since
it
is
hard
to
describe
a
candle
without
using
the
2345678901234567890123456789012123456789012345678901
12345678901234567890123456789012123456789012345678901 22
12345678901234567890123456789012123456789012345678901
term wax. While there are some candles that might not be made out 2
1
12345678901234567890123456789012123456789012345678901
of wax, the overwhelming majority are, so a link like a candle is 2
2
12345678901234567890123456789012123456789012345678901
2345678901234567890123456789012123456789012345678901
12345678901234567890123456789012123456789012345678901
made out of wax might be all you need to answer a problem 2
2
12345678901234567890123456789012123456789012345678901
12345678901234567890123456789012123456789012345678901
correctly. There is the possibility you would have to get more specific, 2
12345678901234567890123456789012123456789012345678901 22
1
so you could modify this first definition and change it to something
2
12345678901234567890123456789012123456789012345678901
more precise such as, A candle is made out of wax, which is then 2
12345678901234567890123456789012123456789012345678901
2
12345678901234567890123456789012123456789012345678901
burned away.
2
12345678901234567890123456789012123456789012345678901
12345678901234567890123456789012123456789012345678901 2
2345678901234567890123456789012123456789012345678901
12345678901234567890123456789012123456789012345678901
This list could be given more categories, but the overall goal is to show you 2
2
12345678901234567890123456789012123456789012345678901
2
12345678901234567890123456789012123456789012345678901
some
common
routes
you
can
take
to
create
a
link
between
the
stem
12345678901234567890123456789012123456789012345678901 22
1 words. Look at the two analogies questions below, and come up with
2
12345678901234567890123456789012123456789012345678901
2
12345678901234567890123456789012123456789012345678901
strong
links
that
describe
a
connection
between
the
two
words.
See
if
your
2
12345678901234567890123456789012123456789012345678901
2
12345678901234567890123456789012123456789012345678901
links
are
good
enough
to
enable
you
to
answer
both
questions
correctly.
2
12345678901234567890123456789012123456789012345678901
2345678901234567890123456789012123456789012345678901
2
1
12345678901234567890123456789012123456789012345678901 2
12345678901234567890123456789012123456789012345678901 2
12345678901234567890123456789012123456789012345678901 2 147
123456789012345678901234567890121234567890123456789012

http://www.xtremepapers.net

Part III: Section Review

123456789012345678901234567890121234567890123456789012
12345678901234567890123456789012123456789012345678901 2
2
12345678901234567890123456789012123456789012345678901
SHEPHERD : FLOCK ::
2
12345678901234567890123456789012123456789012345678901
2
12345678901234567890123456789012123456789012345678901
2
12345678901234567890123456789012123456789012345678901
A.
apothecary
:
drugs
12345678901234567890123456789012123456789012345678901 2
2
12345678901234567890123456789012123456789012345678901
B. butcher : carcass
2
12345678901234567890123456789012123456789012345678901
2
12345678901234567890123456789012123456789012345678901
C.
physician
:
patients
2
12345678901234567890123456789012123456789012345678901
2
12345678901234567890123456789012123456789012345678901
D.
farmer
:
corn
2
12345678901234567890123456789012123456789012345678901
2345678901234567890123456789012123456789012345678901
2
1
E. businessperson : employer
2
12345678901234567890123456789012123456789012345678901
2
12345678901234567890123456789012123456789012345678901
2
12345678901234567890123456789012123456789012345678901
2
12345678901234567890123456789012123456789012345678901
BANEFUL : BENEFIT ::
2345678901234567890123456789012123456789012345678901
2
1
2
12345678901234567890123456789012123456789012345678901
A. helpful : cure
2
12345678901234567890123456789012123456789012345678901
2
12345678901234567890123456789012123456789012345678901
B.
tragic
:
end
2
12345678901234567890123456789012123456789012345678901
2345678901234567890123456789012123456789012345678901
2
1
C. toxic : poison
2
12345678901234567890123456789012123456789012345678901
2
12345678901234567890123456789012123456789012345678901
D.
flawless
:
aberration
2
12345678901234567890123456789012123456789012345678901
2
12345678901234567890123456789012123456789012345678901
E.
disguised
:
concealment
2345678901234567890123456789012123456789012345678901
12345678901234567890123456789012123456789012345678901 2
12345678901234567890123456789012123456789012345678901 2
12345678901234567890123456789012123456789012345678901
You can describe the first two stem words in either functional or 2
2
12345678901234567890123456789012123456789012345678901
2
12345678901234567890123456789012123456789012345678901
definitional
terms:
12345678901234567890123456789012123456789012345678901 2
2
1
2
12345678901234567890123456789012123456789012345678901
Functional route:
2
12345678901234567890123456789012123456789012345678901
2
12345678901234567890123456789012123456789012345678901
he
function
of
a
SHEPHERD
is
to
take
care
of
his/her
FLOCK.
2345678901234567890123456789012123456789012345678901
12345678901234567890123456789012123456789012345678901 2
12345678901234567890123456789012123456789012345678901 2
2
12345678901234567890123456789012123456789012345678901
Definitional route:
2
12345678901234567890123456789012123456789012345678901
12345678901234567890123456789012123456789012345678901 2
2
12345678901234567890123456789012123456789012345678901
A SHEPHERD is someone who takes care of a FLOCK.
2
12345678901234567890123456789012123456789012345678901
12345678901234567890123456789012123456789012345678901 2
2
12345678901234567890123456789012123456789012345678901
1 Both links do a good job of describing the connection between a shepherd 2
2345678901234567890123456789012123456789012345678901
12345678901234567890123456789012123456789012345678901
and a flock. Taking either link and heading into the answer choices, you 2
2
12345678901234567890123456789012123456789012345678901
1 should see that choice (C) works better than any other. A PHYSICIAN is 2
12345678901234567890123456789012123456789012345678901 2
12345678901234567890123456789012123456789012345678901
someone who takes care of his/her PATIENTS. Some people will balk at 2
2
12345678901234567890123456789012123456789012345678901
this choice, mainly because they do not want to be compared with sheep. 2
12345678901234567890123456789012123456789012345678901
2
12345678901234567890123456789012123456789012345678901
Yet the whole point of analogous relationships is to show a shared 2
12345678901234567890123456789012123456789012345678901
2
12345678901234567890123456789012123456789012345678901
similarity between two seemingly dissimilar groups.
2
12345678901234567890123456789012123456789012345678901
2
12345678901234567890123456789012123456789012345678901
2
12345678901234567890123456789012123456789012345678901
For
the
second
analogy
problem,
the
words
baneful
and
benefit
have
12345678901234567890123456789012123456789012345678901 2
12345678901234567890123456789012123456789012345678901
opposite connotations. Something that is BANEFUL has no BENEFIT, so 2
2
12345678901234567890123456789012123456789012345678901
2
12345678901234567890123456789012123456789012345678901
you
can
use
the
link
has
no
to
check
all
the
answer
choices.
Choices
(C)
2
12345678901234567890123456789012123456789012345678901
2
12345678901234567890123456789012123456789012345678901
and
(E)
are
mostly
synonyms,
so
while
there
is
a
strong
link
between
the
12345678901234567890123456789012123456789012345678901 2
2345678901234567890123456789012123456789012345678901
12345678901234567890123456789012123456789012345678901
words in these answer choices, they do not have the link you are looking 2
2
12345678901234567890123456789012123456789012345678901
2
12345678901234567890123456789012123456789012345678901
for.
Something
FLAWLESS
has
no
ABERRATION,
making
(D)
the
correct
12345678901234567890123456789012123456789012345678901 2
2
1 answer.
2
12345678901234567890123456789012123456789012345678901
2
12345678901234567890123456789012123456789012345678901
12345678901234567890123456789012123456789012345678901
The words used in the second problem were deliberately a lot tougher than 2
2
12345678901234567890123456789012123456789012345678901
the words in the first problem. If you didnt know what baneful meant, it 2
12345678901234567890123456789012123456789012345678901
2345678901234567890123456789012123456789012345678901
2
12345678901234567890123456789012123456789012345678901
1 would be hard to make a link between the two stem words. There will be 2
2
12345678901234567890123456789012123456789012345678901
some tough stem words on the GRE, so there are some strategies you can 2
12345678901234567890123456789012123456789012345678901
2
12345678901234567890123456789012123456789012345678901
use to approach these problems.
2
12345678901234567890123456789012123456789012345678901
12345678901234567890123456789012123456789012345678901 2
12345678901234567890123456789012123456789012345678901 2
2
148 12345678901234567890123456789012123456789012345678901
123456789012345678901234567890121234567890123456789012

www.petersons.com

http://www.xtremepapers.net

Chapter 5: Verbal Review

123456789012345678901234567890121234567890123456789012
12345678901234567890123456789012123456789012345678901 2
2
12345678901234567890123456789012123456789012345678901
2
12345678901234567890123456789012123456789012345678901
What to Do When the Stem Words Are Just
2
12345678901234567890123456789012123456789012345678901
2
12345678901234567890123456789012123456789012345678901
Too
Freaky
12345678901234567890123456789012123456789012345678901 2
12345678901234567890123456789012123456789012345678901
There will always be a strong link between the two stem words. This is not 2
2
12345678901234567890123456789012123456789012345678901
2
12345678901234567890123456789012123456789012345678901
the
case
with
the
answer
choices.
Some
of
these
will
have
weak
links,
or
2
12345678901234567890123456789012123456789012345678901
2
12345678901234567890123456789012123456789012345678901
even
no
links
at
all.
Since
the
correct
answer
must
have
the
same,
strong
2
12345678901234567890123456789012123456789012345678901
2345678901234567890123456789012123456789012345678901
1 link as the two stem words, any answer choices with weak links can be 2
2
12345678901234567890123456789012123456789012345678901
12345678901234567890123456789012123456789012345678901
eliminated. Theres no way they could be the right answer, because the 2
2
12345678901234567890123456789012123456789012345678901
2
12345678901234567890123456789012123456789012345678901
right answer must have a strong link.
2345678901234567890123456789012123456789012345678901
2
1
2
12345678901234567890123456789012123456789012345678901
This is the best elimination technique you have, so be prepared to use it on 2
12345678901234567890123456789012123456789012345678901
2
12345678901234567890123456789012123456789012345678901
lots of problems. Lets look back at the very first analogy problem in this 2
12345678901234567890123456789012123456789012345678901
2345678901234567890123456789012123456789012345678901
2
1 section.
2
12345678901234567890123456789012123456789012345678901
2
12345678901234567890123456789012123456789012345678901
2
12345678901234567890123456789012123456789012345678901
2
12345678901234567890123456789012123456789012345678901
FEATHER
:
OSTRICH
::
2345678901234567890123456789012123456789012345678901
12345678901234567890123456789012123456789012345678901 22
12345678901234567890123456789012123456789012345678901 2
12345678901234567890123456789012123456789012345678901
A. beak : raven
2
12345678901234567890123456789012123456789012345678901
2
12345678901234567890123456789012123456789012345678901
B. scale : lizard
12345678901234567890123456789012123456789012345678901 22
1
C. foot : chicken
2
12345678901234567890123456789012123456789012345678901
D. desert : cactus
2
12345678901234567890123456789012123456789012345678901
2
12345678901234567890123456789012123456789012345678901
E.
tail
:
mammal
2345678901234567890123456789012123456789012345678901
12345678901234567890123456789012123456789012345678901 22
12345678901234567890123456789012123456789012345678901 2
12345678901234567890123456789012123456789012345678901
Assume you have no idea what the word feather means. Go down to your 2
12345678901234567890123456789012123456789012345678901
2
12345678901234567890123456789012123456789012345678901
answer choices and try to make links between the two words.
2
12345678901234567890123456789012123456789012345678901
12345678901234567890123456789012123456789012345678901 22
12345678901234567890123456789012123456789012345678901
A. beak : ravena BEAK is like the mouth of a RAVEN
2
12345678901234567890123456789012123456789012345678901
2
1
B.
scale
:
lizarda
LIZARD
is
covered
in
SCALES
(Its
OK
to
2345678901234567890123456789012123456789012345678901
12345678901234567890123456789012123456789012345678901 22
12345678901234567890123456789012123456789012345678901
reverse direction if you need to, and you can add a plural if
2
1
2
12345678901234567890123456789012123456789012345678901
you like)
2
12345678901234567890123456789012123456789012345678901
2345678901234567890123456789012123456789012345678901
2
12345678901234567890123456789012123456789012345678901
C. foot : chickena CHICKEN kinda has a FOOT
2
12345678901234567890123456789012123456789012345678901
2
1
D. desert : cactusa DESERT is where you commonly find
2
12345678901234567890123456789012123456789012345678901
2
12345678901234567890123456789012123456789012345678901
CACTUS
2345678901234567890123456789012123456789012345678901
2
12345678901234567890123456789012123456789012345678901
E. tail : mammala TAIL can often be found on a MAMMAL,
2
12345678901234567890123456789012123456789012345678901
2
1
but not always
2
12345678901234567890123456789012123456789012345678901
12345678901234567890123456789012123456789012345678901 2
2345678901234567890123456789012123456789012345678901
12345678901234567890123456789012123456789012345678901
Choice (C) is not really a strong link, and (E) is pretty shaky, too. 2
2
12345678901234567890123456789012123456789012345678901
2
12345678901234567890123456789012123456789012345678901
Mammals
can
have
tails,
but
not
every
one
does.
Both
(C)
and
(E)
are
12345678901234567890123456789012123456789012345678901 22
1 weak choices that can be crossed out as possible answers. Choices (A) and
2
12345678901234567890123456789012123456789012345678901
2
12345678901234567890123456789012123456789012345678901
(D)
have
fine
links.
But
if
(D)
is
the
right
answer,
then
the
stem
words
2
12345678901234567890123456789012123456789012345678901
2
12345678901234567890123456789012123456789012345678901
would
have
to
share
the
same
link,
meaning
that
a
FEATHER
is
where
12345678901234567890123456789012123456789012345678901 2
2345678901234567890123456789012123456789012345678901
12345678901234567890123456789012123456789012345678901
you commonly find an OSTRICH. You would pick this if you thought 2
2
12345678901234567890123456789012123456789012345678901
2
12345678901234567890123456789012123456789012345678901
there
was
a
specific
word
out
there
meaning,
the
place
where
ostriches
12345678901234567890123456789012123456789012345678901 22
1 are commonly found. The existence of such a word is unlikely, so this
2
12345678901234567890123456789012123456789012345678901
12345678901234567890123456789012123456789012345678901
takes (D) out of the picture. You are left with (A) or (C), two solid choices. 2
2
12345678901234567890123456789012123456789012345678901
2
12345678901234567890123456789012123456789012345678901
Lets revisit choice (C) once more before bidding a fond farewell to this 2
12345678901234567890123456789012123456789012345678901
2345678901234567890123456789012123456789012345678901
1 problem. Since foot and chicken do not share any real connection, why is 2
12345678901234567890123456789012123456789012345678901 2
12345678901234567890123456789012123456789012345678901 2
12345678901234567890123456789012123456789012345678901 2 149
123456789012345678901234567890121234567890123456789012

http://www.xtremepapers.net

Part III: Section Review

www.petersons.com

http://www.xtremepapers.net

Note

123456789012345678901234567890121234567890123456789012
12345678901234567890123456789012123456789012345678901 2
2
12345678901234567890123456789012123456789012345678901
this choice even here? The reason lies in the fact that most people like to 2
12345678901234567890123456789012123456789012345678901
2
12345678901234567890123456789012123456789012345678901
play the Connect the Words game when tackling tough analogies 2
12345678901234567890123456789012123456789012345678901
2
12345678901234567890123456789012123456789012345678901
problems. This means they might connect the second stem word with the 2
12345678901234567890123456789012123456789012345678901
2345678901234567890123456789012123456789012345678901
1 second word in an answer choice, or maybe theyll take the first stem word 2
2
12345678901234567890123456789012123456789012345678901
2
12345678901234567890123456789012123456789012345678901
and
make
a
connection
with
the
second
word
of
a
different
answer
choice.
2
12345678901234567890123456789012123456789012345678901
2
12345678901234567890123456789012123456789012345678901
Theyll
make
links
however
they
can,
with
no
regard
for
the
strict
rules
2345678901234567890123456789012123456789012345678901
2
1
2
12345678901234567890123456789012123456789012345678901
regarding
analogous
relationships.
In
terms
of
this
question,
this
means
2
12345678901234567890123456789012123456789012345678901
2
12345678901234567890123456789012123456789012345678901
people
will
link
ostrich,
a
bird,
with
chicken
or
raven,
two
other
birds.
2
12345678901234567890123456789012123456789012345678901
2345678901234567890123456789012123456789012345678901
1 Who cares what the first words are? Chickens and ostriches are both birds, 2
2
12345678901234567890123456789012123456789012345678901
2
12345678901234567890123456789012123456789012345678901
arent
they?
2
12345678901234567890123456789012123456789012345678901
2
12345678901234567890123456789012123456789012345678901
2345678901234567890123456789012123456789012345678901
1 None of these Connect the Words links are valid. They are traps 2
2
12345678901234567890123456789012123456789012345678901
12345678901234567890123456789012123456789012345678901
designed to lead you astray. It doesnt matter that chicken and ostriches are 2
2
12345678901234567890123456789012123456789012345678901
12345678901234567890123456789012123456789012345678901
both birds. You can only make links between the two paired words, and 2
2345678901234567890123456789012123456789012345678901
12345678901234567890123456789012123456789012345678901 2
2
12345678901234567890123456789012123456789012345678901
then compare links to one another.
2
12345678901234567890123456789012123456789012345678901
2
12345678901234567890123456789012123456789012345678901
The
temptation
to
play
Connect
the
Words
gets
stronger
when
you
have
12345678901234567890123456789012123456789012345678901 2
2
12345678901234567890123456789012123456789012345678901
2
1 some tough words that you might not know.
2
12345678901234567890123456789012123456789012345678901
2
12345678901234567890123456789012123456789012345678901
2
12345678901234567890123456789012123456789012345678901
Looking
back
at
the
shepherd
and
flock
problem,
you
can
see
two
answer
2345678901234567890123456789012123456789012345678901
12345678901234567890123456789012123456789012345678901 2
2
12345678901234567890123456789012123456789012345678901
choices designed to catch people making incorrect connections. The 2
12345678901234567890123456789012123456789012345678901
12345678901234567890123456789012123456789012345678901
words farmer and corn do not really have a strong link, since farmers can 2
2
12345678901234567890123456789012123456789012345678901
12345678901234567890123456789012123456789012345678901
grow many other things besides corn. Its there to catch people combining 2
2
12345678901234567890123456789012123456789012345678901
2
12345678901234567890123456789012123456789012345678901
shepherd
and
farmer,
two
pastoral
occupations.
Some
people
might
12345678901234567890123456789012123456789012345678901 2
1 equate shepherds with tending herd animals of some sort, and these 2
2
12345678901234567890123456789012123456789012345678901
2
12345678901234567890123456789012123456789012345678901
animals
often
end
up
with
a
butcher,
the
first
word
in
choice
(B).
12345678901234567890123456789012123456789012345678901 2
12345678901234567890123456789012123456789012345678901 2
2
12345678901234567890123456789012123456789012345678901
2
12345678901234567890123456789012123456789012345678901
BANEFUL : BENEFIT ::
2
12345678901234567890123456789012123456789012345678901
12345678901234567890123456789012123456789012345678901 2
2
12345678901234567890123456789012123456789012345678901
A. helpful : cure
2
12345678901234567890123456789012123456789012345678901
2345678901234567890123456789012123456789012345678901
2
12345678901234567890123456789012123456789012345678901
B. tragic : end
2
12345678901234567890123456789012123456789012345678901
2
1
C. toxic : poison
12345678901234567890123456789012123456789012345678901 2
2
12345678901234567890123456789012123456789012345678901
D. flawless : aberration
2
12345678901234567890123456789012123456789012345678901
2
E. camouflaged : concealment
12345678901234567890123456789012123456789012345678901
2
12345678901234567890123456789012123456789012345678901
2
12345678901234567890123456789012123456789012345678901
If
you
dont
know
what
baneful
means,
attack
this
question
by
heading
12345678901234567890123456789012123456789012345678901 2
2345678901234567890123456789012123456789012345678901
12345678901234567890123456789012123456789012345678901
into the answer choices to test the links there. Choice (C) contains two 2
2
12345678901234567890123456789012123456789012345678901
2
12345678901234567890123456789012123456789012345678901
synonyms,
so
it
remains
a
likely
pick
if
you
think
baneful
and
benefit
are
12345678901234567890123456789012123456789012345678901 2
1 synonymous. When you try to make a link for choices (A) and (B), you will 2
2
12345678901234567890123456789012123456789012345678901
2
12345678901234567890123456789012123456789012345678901
find
that
the
words
are
not
really
related.
Choice
(A)
is
probably
there
for
2
12345678901234567890123456789012123456789012345678901
2
12345678901234567890123456789012123456789012345678901
people
connecting
cure
and
benefit,
two
positive
words.
You
should
realize
12345678901234567890123456789012123456789012345678901 2
2345678901234567890123456789012123456789012345678901
2
12345678901234567890123456789012123456789012345678901
1 by now that choice (A) is wrong for two reasons: It has a weak link, and 2
2
12345678901234567890123456789012123456789012345678901
its a trap for folks playing Connect the Words.
2
12345678901234567890123456789012123456789012345678901
2
12345678901234567890123456789012123456789012345678901
2345678901234567890123456789012123456789012345678901
2
1
12345678901234567890123456789012123456789012345678901 2
12345678901234567890123456789012123456789012345678901 2
2
150 12345678901234567890123456789012123456789012345678901
123456789012345678901234567890121234567890123456789012

Chapter 5: Verbal Review

123456789012345678901234567890121234567890123456789012
12345678901234567890123456789012123456789012345678901 2
2
12345678901234567890123456789012123456789012345678901
Choices (C), (D), and (E) all contain good links, so they should all remain. 2
12345678901234567890123456789012123456789012345678901
2
12345678901234567890123456789012123456789012345678901
A one-in-three chance may not be ideal, but it shows once again that you 2
12345678901234567890123456789012123456789012345678901
2
12345678901234567890123456789012123456789012345678901
can use techniques to give yourself a fighting chance even on problems 2
12345678901234567890123456789012123456789012345678901
2345678901234567890123456789012123456789012345678901
2
1 where you dont know the meaning of some of the words.
2
12345678901234567890123456789012123456789012345678901
2
12345678901234567890123456789012123456789012345678901
2
12345678901234567890123456789012123456789012345678901
2
12345678901234567890123456789012123456789012345678901
2345678901234567890123456789012123456789012345678901
2
1
2345678901234567890123456789012123456789012345678901
2
1
This
__________
of
the
Book
Deals
with
2
12345678901234567890123456789012123456789012345678901
2
12345678901234567890123456789012123456789012345678901
Sentence
__________
2
12345678901234567890123456789012123456789012345678901
2345678901234567890123456789012123456789012345678901
2
1
12345678901234567890123456789012123456789012345678901
If you could figure out that heading, you are well on your way to 2
2
12345678901234567890123456789012123456789012345678901
succeeding on Sentence Completions (SC) problems. Like the heading, SC 2
12345678901234567890123456789012123456789012345678901
2
12345678901234567890123456789012123456789012345678901
problems feature a sentenceoften a convoluted one filled with multiple 2
12345678901234567890123456789012123456789012345678901
2
12345678901234567890123456789012123456789012345678901
types of punctuation (commas, semicolons, and so forth), kind of like the 2
12345678901234567890123456789012123456789012345678901
2
12345678901234567890123456789012123456789012345678901
sentence you are currently readingthat has one or more words missing 2
12345678901234567890123456789012123456789012345678901
2345678901234567890123456789012123456789012345678901
2
12345678901234567890123456789012123456789012345678901
from it. A sample SC problem would look like:
2
12345678901234567890123456789012123456789012345678901
12345678901234567890123456789012123456789012345678901 22
12345678901234567890123456789012123456789012345678901 2
12345678901234567890123456789012123456789012345678901
SC problems feature a sentenceoften a convoluted one filled with
2
12345678901234567890123456789012123456789012345678901
2
1
multiple
types
of
__________
(commas,
semicolons,
and
so
forth),
2
12345678901234567890123456789012123456789012345678901
2
12345678901234567890123456789012123456789012345678901
kind
of
like
the
sentence
you
are
currently
readingthat
has
one
or
2
12345678901234567890123456789012123456789012345678901
2345678901234567890123456789012123456789012345678901
2
12345678901234567890123456789012123456789012345678901
more words missing from it.
2
12345678901234567890123456789012123456789012345678901
2
12345678901234567890123456789012123456789012345678901
A. pauses
2
12345678901234567890123456789012123456789012345678901
2
12345678901234567890123456789012123456789012345678901
B.
punctuation
12345678901234567890123456789012123456789012345678901 22
12345678901234567890123456789012123456789012345678901
C. grammar
2
12345678901234567890123456789012123456789012345678901
2
12345678901234567890123456789012123456789012345678901
D.
issues
2
1
2345678901234567890123456789012123456789012345678901
2
12345678901234567890123456789012123456789012345678901
E. subjects
2
12345678901234567890123456789012123456789012345678901
2
1
2
12345678901234567890123456789012123456789012345678901
Using
context
is
the
critical
element
with
SC
problems.
You
are
not
2
12345678901234567890123456789012123456789012345678901
2345678901234567890123456789012123456789012345678901
12345678901234567890123456789012123456789012345678901
required to conjecture randomly to figure out what the missing word 2
2
12345678901234567890123456789012123456789012345678901
1 should be. Instead, every sentence will contain hints and clues in the 2
2
12345678901234567890123456789012123456789012345678901
2
12345678901234567890123456789012123456789012345678901
context
that
should
steer
you
toward
the
right
word
to
place
in
the
missing
2345678901234567890123456789012123456789012345678901
12345678901234567890123456789012123456789012345678901 22
12345678901234567890123456789012123456789012345678901
1 blank. In the sample sentence above, two words immediately following the 2
12345678901234567890123456789012123456789012345678901
blank are commas and semicolons, both forms of punctuation. These are 2
2
12345678901234567890123456789012123456789012345678901
2345678901234567890123456789012123456789012345678901
12345678901234567890123456789012123456789012345678901
the main clues that should lead you to the correct answer of choice (B), 2
2
12345678901234567890123456789012123456789012345678901
12345678901234567890123456789012123456789012345678901
punctuation. Of course, having the entire sentence directly above the 2
12345678901234567890123456789012123456789012345678901 22
1 problem also helps.
2
12345678901234567890123456789012123456789012345678901
2
12345678901234567890123456789012123456789012345678901
2
12345678901234567890123456789012123456789012345678901
2
12345678901234567890123456789012123456789012345678901
12345678901234567890123456789012123456789012345678901 2
2345678901234567890123456789012123456789012345678901
12345678901234567890123456789012123456789012345678901 22
12345678901234567890123456789012123456789012345678901 2
12345678901234567890123456789012123456789012345678901 2
12345678901234567890123456789012123456789012345678901 2
1
2
12345678901234567890123456789012123456789012345678901
12345678901234567890123456789012123456789012345678901 2
2
12345678901234567890123456789012123456789012345678901
12345678901234567890123456789012123456789012345678901 2
2
12345678901234567890123456789012123456789012345678901
12345678901234567890123456789012123456789012345678901 2
12345678901234567890123456789012123456789012345678901 2
12345678901234567890123456789012123456789012345678901 2
12345678901234567890123456789012123456789012345678901 2 151
123456789012345678901234567890121234567890123456789012

http://www.xtremepapers.net

Part III: Section Review

http://www.xtremepapers.net

Tip

www.petersons.com

X-Ref

123456789012345678901234567890121234567890123456789012
12345678901234567890123456789012123456789012345678901 2
2
12345678901234567890123456789012123456789012345678901
12345678901234567890123456789012123456789012345678901 2
12345678901234567890123456789012123456789012345678901
The idea of context should be familiar to everyone enhancing their 2
2
12345678901234567890123456789012123456789012345678901
2
12345678901234567890123456789012123456789012345678901
vocabulary
by
grabbing
unfamiliar
words
in
the
context
of
everyday
2
12345678901234567890123456789012123456789012345678901
2345678901234567890123456789012123456789012345678901
1 reading. (This topic was discussed on page 137 in this section.) When you 2
2
12345678901234567890123456789012123456789012345678901
12345678901234567890123456789012123456789012345678901
encounter an unfamiliar word in context, you can often determine its 2
2
12345678901234567890123456789012123456789012345678901
12345678901234567890123456789012123456789012345678901
meaning from the way it is used in the sentence it is in. You figure out the 2
2345678901234567890123456789012123456789012345678901
2
1
12345678901234567890123456789012123456789012345678901
meaning of the word through its context, and this practice is very similar 2
2
12345678901234567890123456789012123456789012345678901
12345678901234567890123456789012123456789012345678901
to the strategy used to determine what word should appear in the blank 2
2
12345678901234567890123456789012123456789012345678901
space of the Sentence Completion problem.
2
12345678901234567890123456789012123456789012345678901
2
12345678901234567890123456789012123456789012345678901
2
12345678901234567890123456789012123456789012345678901
2
12345678901234567890123456789012123456789012345678901
Just like antonym problems, anticipating the answer will help you on SC 2
12345678901234567890123456789012123456789012345678901
2345678901234567890123456789012123456789012345678901
1 problems. People who read the original sentence and then go shopping in 2
2
12345678901234567890123456789012123456789012345678901
2
12345678901234567890123456789012123456789012345678901
the answer choices increase their possibility of picking a wrong answer. 2
12345678901234567890123456789012123456789012345678901
2
12345678901234567890123456789012123456789012345678901
The incorrect answers are created to catch shoppers, so dont follow this 2
12345678901234567890123456789012123456789012345678901
12345678901234567890123456789012123456789012345678901
pattern of behavior. Instead, read the original sentence slowly, and try to 2
2
12345678901234567890123456789012123456789012345678901
2
12345678901234567890123456789012123456789012345678901
decide
what
word
would
fit
best
in
each
blank.
2
12345678901234567890123456789012123456789012345678901
2345678901234567890123456789012123456789012345678901
12345678901234567890123456789012123456789012345678901 2
1 Figuring out a word can be difficult, but remember that every sentence will 2
2
12345678901234567890123456789012123456789012345678901
2
12345678901234567890123456789012123456789012345678901
contain
context
clues
to
help
you
determine
the
right
answer.
If
the
missing
2
12345678901234567890123456789012123456789012345678901
2345678901234567890123456789012123456789012345678901
12345678901234567890123456789012123456789012345678901
word doesnt come to you right away, scan the sentence and look for words 2
2
12345678901234567890123456789012123456789012345678901
2
12345678901234567890123456789012123456789012345678901
or
phrases
that
provide
a
clue.
There
will
always
be
some.
If
you
come
up
12345678901234567890123456789012123456789012345678901 2
12345678901234567890123456789012123456789012345678901
with a word but are a bit uncertain it is the correct one, finding the context 2
2
12345678901234567890123456789012123456789012345678901
2
12345678901234567890123456789012123456789012345678901
clues can help you decide whether or not you made the right choice.
12345678901234567890123456789012123456789012345678901 2
12345678901234567890123456789012123456789012345678901 2
2
1
Determined to bring back an egg of the Emperor penguin, Apsley
2
12345678901234567890123456789012123456789012345678901
2
12345678901234567890123456789012123456789012345678901
and his companions struggled through ferocious blizzards,
2
12345678901234567890123456789012123456789012345678901
2
12345678901234567890123456789012123456789012345678901
__________
low
temperatures,
and
across
dangerously
unknown
2
12345678901234567890123456789012123456789012345678901
2345678901234567890123456789012123456789012345678901
2
12345678901234567890123456789012123456789012345678901
Antarctic __________ in order to complete their mission.
2
12345678901234567890123456789012123456789012345678901
2
1
2
12345678901234567890123456789012123456789012345678901
A.
appallingly.
.terrain
2
12345678901234567890123456789012123456789012345678901
2345678901234567890123456789012123456789012345678901
2
12345678901234567890123456789012123456789012345678901
B. terrifyingly. .errors
2
12345678901234567890123456789012123456789012345678901
2
1
C. cryptically. .ice
12345678901234567890123456789012123456789012345678901 2
2
12345678901234567890123456789012123456789012345678901
D. bitingly. .forces
2
12345678901234567890123456789012123456789012345678901
2
12345678901234567890123456789012123456789012345678901
E. stingingly. .circumstances
2
12345678901234567890123456789012123456789012345678901
2
12345678901234567890123456789012123456789012345678901
12345678901234567890123456789012123456789012345678901 2
2345678901234567890123456789012123456789012345678901
12345678901234567890123456789012123456789012345678901
The sample sentence also contains directional words. These are words 2
2
12345678901234567890123456789012123456789012345678901
12345678901234567890123456789012123456789012345678901
that change or continue the flow of a sentence. A conjunction like and 2
12345678901234567890123456789012123456789012345678901 2
1 continues the flow of a sentence, while words like but, although, or 2
2
12345678901234567890123456789012123456789012345678901
12345678901234567890123456789012123456789012345678901
however change the flow of sentence. Since changing the flow makes the 2
2
12345678901234567890123456789012123456789012345678901
meaning of a sentence harder to determine, you can expect to see at least 2
12345678901234567890123456789012123456789012345678901
2
12345678901234567890123456789012123456789012345678901
some changing directional words on your SC problems.
2
12345678901234567890123456789012123456789012345678901
12345678901234567890123456789012123456789012345678901 2
2
12345678901234567890123456789012123456789012345678901
2
12345678901234567890123456789012123456789012345678901
This
two-blank
SC
problem
has
a
host
of
context
clues.
We
can
break
2
12345678901234567890123456789012123456789012345678901
2345678901234567890123456789012123456789012345678901
2
1 down this entire sentence and discuss the relevance of all the words.
12345678901234567890123456789012123456789012345678901 2
12345678901234567890123456789012123456789012345678901 2
2
152 12345678901234567890123456789012123456789012345678901
123456789012345678901234567890121234567890123456789012

Chapter 5: Verbal Review

123456789012345678901234567890121234567890123456789012
12345678901234567890123456789012123456789012345678901 2
2
12345678901234567890123456789012123456789012345678901
2
12345678901234567890123456789012123456789012345678901
Word or Phrase
Significance
2
12345678901234567890123456789012123456789012345678901
2
12345678901234567890123456789012123456789012345678901
2
12345678901234567890123456789012123456789012345678901
Determined
to
On
many
SC
problems,
the
introductory
phrase
2
12345678901234567890123456789012123456789012345678901
2345678901234567890123456789012123456789012345678901
2
1
bring back an
is employed to set up a change in the flow of the
2
12345678901234567890123456789012123456789012345678901
2
12345678901234567890123456789012123456789012345678901
egg
of
the
Emsentence.
These
introductory
phrases
start
with
2
12345678901234567890123456789012123456789012345678901
2
12345678901234567890123456789012123456789012345678901
peror
penguin
words
like
although,
while,
and
something
2345678901234567890123456789012123456789012345678901
2
1
2
12345678901234567890123456789012123456789012345678901
related.
However,
this
phrase
does
not
do
this.
2
12345678901234567890123456789012123456789012345678901
2
12345678901234567890123456789012123456789012345678901
The key words in this phrase might be Deter2
12345678901234567890123456789012123456789012345678901
2345678901234567890123456789012123456789012345678901
2
1
minedshowing resolutionand Emperor pen2
12345678901234567890123456789012123456789012345678901
2
12345678901234567890123456789012123456789012345678901
guin, which would lead to start thinking about
2
12345678901234567890123456789012123456789012345678901
2
12345678901234567890123456789012123456789012345678901
cold weather, since penguins are associated
2345678901234567890123456789012123456789012345678901
2
1
with cold weather.
2
12345678901234567890123456789012123456789012345678901
2
12345678901234567890123456789012123456789012345678901
2
12345678901234567890123456789012123456789012345678901
Apsley
and
his
Not
much
here
except
for
the
fact
that
Apsleys
2
12345678901234567890123456789012123456789012345678901
2345678901234567890123456789012123456789012345678901
2
12345678901234567890123456789012123456789012345678901
companions
a decidely British name? In terms of answering
2
12345678901234567890123456789012123456789012345678901
2
12345678901234567890123456789012123456789012345678901
a
SC
question,
the
phrase
and
his
companions
12345678901234567890123456789012123456789012345678901 22
12345678901234567890123456789012123456789012345678901
is completely worthless. Whether or not Apsley
2
12345678901234567890123456789012123456789012345678901
2
1
acted alone or with an entire troupe of clowns
2
12345678901234567890123456789012123456789012345678901
2
12345678901234567890123456789012123456789012345678901
does not help you figure out the missing words.
2
12345678901234567890123456789012123456789012345678901
2345678901234567890123456789012123456789012345678901
2
12345678901234567890123456789012123456789012345678901
Think of these worthless phrases as sentence
2
12345678901234567890123456789012123456789012345678901
2
12345678901234567890123456789012123456789012345678901
completion chaff.
12345678901234567890123456789012123456789012345678901 22
12345678901234567890123456789012123456789012345678901
struggled
Heres a big clue that ties in well with the
2
12345678901234567890123456789012123456789012345678901
2
12345678901234567890123456789012123456789012345678901
through
feropenguin
clue
from
earlier.
The
words
penguin
12345678901234567890123456789012123456789012345678901 22
12345678901234567890123456789012123456789012345678901
cious blizzards
and blizzards should evoke images of very cold
2
1
2
12345678901234567890123456789012123456789012345678901
weather.
The words struggled and ferocious
2
12345678901234567890123456789012123456789012345678901
2
12345678901234567890123456789012123456789012345678901
reinforce
the
idea
that
the
place
Apsley
is
at
is
12345678901234567890123456789012123456789012345678901 2
2
12345678901234567890123456789012123456789012345678901
very harsh and cold.
2
12345678901234567890123456789012123456789012345678901
2
12345678901234567890123456789012123456789012345678901
__________ low Given the way this sentence is going, you
2
12345678901234567890123456789012123456789012345678901
2
12345678901234567890123456789012123456789012345678901
temperatures
should
conjecture
that
the
missing
word
should
2
12345678901234567890123456789012123456789012345678901
2345678901234567890123456789012123456789012345678901
2
12345678901234567890123456789012123456789012345678901
be something like extremely or terribly.
2
12345678901234567890123456789012123456789012345678901
2
1
The
phrase
pleasantly
low
temperatures
12345678901234567890123456789012123456789012345678901 2
2
12345678901234567890123456789012123456789012345678901
simply would not jive with everything youve
2
12345678901234567890123456789012123456789012345678901
2
12345678901234567890123456789012123456789012345678901
read
up
to
now.
2
12345678901234567890123456789012123456789012345678901
2
12345678901234567890123456789012123456789012345678901
and across
Apsley hasnt gone to poach penguin eggs from
2
12345678901234567890123456789012123456789012345678901
2345678901234567890123456789012123456789012345678901
2
12345678901234567890123456789012123456789012345678901
dangerously una zoo. Hes obviously fighting to get eggs in the
2
12345678901234567890123456789012123456789012345678901
2
12345678901234567890123456789012123456789012345678901
known Antarctic Antarctic. Picturing Antarctica in your mind,
2
12345678901234567890123456789012123456789012345678901
2
1
__________
you
might
think
the
ice
would
fit
well
into
the
2345678901234567890123456789012123456789012345678901
12345678901234567890123456789012123456789012345678901 22
12345678901234567890123456789012123456789012345678901
sentence. It would, but so would land or
2
12345678901234567890123456789012123456789012345678901
2
12345678901234567890123456789012123456789012345678901
ground.
Keep
both
options
in
mind.
2
1
2345678901234567890123456789012123456789012345678901
12345678901234567890123456789012123456789012345678901 22
1
in order to
SC filler, but three cheers for completing the
2
12345678901234567890123456789012123456789012345678901
2
12345678901234567890123456789012123456789012345678901
complete their
mission.
2
12345678901234567890123456789012123456789012345678901
2345678901234567890123456789012123456789012345678901
2
mission
1
12345678901234567890123456789012123456789012345678901 2
12345678901234567890123456789012123456789012345678901 2
12345678901234567890123456789012123456789012345678901 2 153
123456789012345678901234567890121234567890123456789012

http://www.xtremepapers.net

Part III: Section Review

123456789012345678901234567890121234567890123456789012
12345678901234567890123456789012123456789012345678901 2
2
12345678901234567890123456789012123456789012345678901
Now that you have potential words for each blank, look at the answer 2
12345678901234567890123456789012123456789012345678901
2
12345678901234567890123456789012123456789012345678901
choices. Start with only one word, and eliminate any answer choices that 2
12345678901234567890123456789012123456789012345678901
2
12345678901234567890123456789012123456789012345678901
do not fit the words you came up with. Start with the first blank and cross 2
12345678901234567890123456789012123456789012345678901
2345678901234567890123456789012123456789012345678901
2
1 out any words that do not mean terribly or extremely.
2
12345678901234567890123456789012123456789012345678901
2
12345678901234567890123456789012123456789012345678901
2
12345678901234567890123456789012123456789012345678901
A.
appallingly.
.terrain
2
12345678901234567890123456789012123456789012345678901
2345678901234567890123456789012123456789012345678901
2
1
B. terrifyingly. .errors
2
12345678901234567890123456789012123456789012345678901
2
12345678901234567890123456789012123456789012345678901
cryptically.
.ice
C
2
12345678901234567890123456789012123456789012345678901
2
12345678901234567890123456789012123456789012345678901
D.
bitingly.
.forces
2345678901234567890123456789012123456789012345678901
2
1
2
12345678901234567890123456789012123456789012345678901
E.
stingingly.
.circumstances
2
12345678901234567890123456789012123456789012345678901
2
12345678901234567890123456789012123456789012345678901
12345678901234567890123456789012123456789012345678901
Choice (C) can be eliminated, since cryptically low temperatures does 2
2345678901234567890123456789012123456789012345678901
2
1
12345678901234567890123456789012123456789012345678901
not make sense, and it does not mean the same thing as what you are 2
2
12345678901234567890123456789012123456789012345678901
looking for. Note that the second word in choice (C), ice, is a very good 2
12345678901234567890123456789012123456789012345678901
2
12345678901234567890123456789012123456789012345678901
word to use for that blank. People shopping might pick choice (C) because 2
12345678901234567890123456789012123456789012345678901
2
12345678901234567890123456789012123456789012345678901
the second word fits so well, even though the first word is a poor choice. If 2
12345678901234567890123456789012123456789012345678901
2
12345678901234567890123456789012123456789012345678901
you work one blank at a time and eliminate improbable answer choices, 2
12345678901234567890123456789012123456789012345678901
2345678901234567890123456789012123456789012345678901
2
12345678901234567890123456789012123456789012345678901
2
1 you spare yourself this mistake.
2
12345678901234567890123456789012123456789012345678901
2
12345678901234567890123456789012123456789012345678901
For
the
second
blank,
ice
works
nicely
but
its
already
gone.
The
word
2
12345678901234567890123456789012123456789012345678901
2345678901234567890123456789012123456789012345678901
12345678901234567890123456789012123456789012345678901
errors does not work very well, so (B) can be crossed out. This leaves (A), 2
2
12345678901234567890123456789012123456789012345678901
2
12345678901234567890123456789012123456789012345678901
(D),
and
(E).
The
word
terrain
is
synonymous
with
ground,
making
it
the
12345678901234567890123456789012123456789012345678901 2
12345678901234567890123456789012123456789012345678901
best choice from the ones left. (A) is your answer, then. You didnt start out 2
2
12345678901234567890123456789012123456789012345678901
2
12345678901234567890123456789012123456789012345678901
thinking
of
the
words
appallingly
and
terrain,
but
you
understood
the
12345678901234567890123456789012123456789012345678901 2
2
12345678901234567890123456789012123456789012345678901
1 context clues well enough to choose words that led you to these correct 2
2345678901234567890123456789012123456789012345678901
2
12345678901234567890123456789012123456789012345678901
answers.
2
12345678901234567890123456789012123456789012345678901
1 The approach outlined above should be used on all SC questions, one 2
12345678901234567890123456789012123456789012345678901 2
2
12345678901234567890123456789012123456789012345678901
blank or two. As always, vocabulary plays a factor, since it is difficult to 2
12345678901234567890123456789012123456789012345678901
2
12345678901234567890123456789012123456789012345678901
eliminate a word like cryptically if you are uncertain of what it means. 2
12345678901234567890123456789012123456789012345678901
2
12345678901234567890123456789012123456789012345678901
Even though vocabulary works as a limiting factor on SC problems, the 2
12345678901234567890123456789012123456789012345678901
2345678901234567890123456789012123456789012345678901
12345678901234567890123456789012123456789012345678901
fact remains that sentence completion questions are very techniquable. 2
2
12345678901234567890123456789012123456789012345678901
1 If you can understand the sentence, you have a shot at getting the answer 2
12345678901234567890123456789012123456789012345678901 2
2
12345678901234567890123456789012123456789012345678901
right.
2
12345678901234567890123456789012123456789012345678901
2
12345678901234567890123456789012123456789012345678901
2
12345678901234567890123456789012123456789012345678901
This
is
the
best
thing
about
SC
problems.
The
worst
thing
is
that
there
are
2
12345678901234567890123456789012123456789012345678901
2
12345678901234567890123456789012123456789012345678901
usually
fewer
SC
problems
than
any
other
question
type.
You
might
see
as
2345678901234567890123456789012123456789012345678901
12345678901234567890123456789012123456789012345678901 2
12345678901234567890123456789012123456789012345678901
few as five, and these arrive near the back end of the test where their 2
2
12345678901234567890123456789012123456789012345678901
12345678901234567890123456789012123456789012345678901
impact on scoring is lessened. Isnt it strange how the question type thats 2
2
1
12345678901234567890123456789012123456789012345678901
most solvable appears the least, while the question type thats little more 2
2
12345678901234567890123456789012123456789012345678901
2
12345678901234567890123456789012123456789012345678901
than a do-you-know-the-word-or-not? appears the most? Hmm. . . .
2
12345678901234567890123456789012123456789012345678901
12345678901234567890123456789012123456789012345678901 2
2345678901234567890123456789012123456789012345678901
12345678901234567890123456789012123456789012345678901 2
2
1
2
12345678901234567890123456789012123456789012345678901
12345678901234567890123456789012123456789012345678901 2
2
12345678901234567890123456789012123456789012345678901
12345678901234567890123456789012123456789012345678901 2
12345678901234567890123456789012123456789012345678901 2
12345678901234567890123456789012123456789012345678901 2
2
154 12345678901234567890123456789012123456789012345678901
123456789012345678901234567890121234567890123456789012

www.petersons.com

http://www.xtremepapers.net

Chapter 5: Verbal Review

Tip

123456789012345678901234567890121234567890123456789012
12345678901234567890123456789012123456789012345678901 2
2
12345678901234567890123456789012123456789012345678901
2
12345678901234567890123456789012123456789012345678901
Reading
Comprehension, the Kevin Bacon of
2
12345678901234567890123456789012123456789012345678901
2
12345678901234567890123456789012123456789012345678901
Standardized
Tests
(Its
in
Every
Test)
12345678901234567890123456789012123456789012345678901 2
2
12345678901234567890123456789012123456789012345678901
Odds are high that you have already taken scads of tests containing 2
12345678901234567890123456789012123456789012345678901
2
12345678901234567890123456789012123456789012345678901
Reading Comprehension (RC) problems. The format, then, should hold 2
12345678901234567890123456789012123456789012345678901
2
12345678901234567890123456789012123456789012345678901
no surprise: You get a pile of words to read and must answer questions 2
12345678901234567890123456789012123456789012345678901
2345678901234567890123456789012123456789012345678901
1 about these words to show that you understood said pile. RC on the GRE 2
2
12345678901234567890123456789012123456789012345678901
2
12345678901234567890123456789012123456789012345678901
Verbal
section
is
no
different,
although
the
twenty-first
century
techno2
12345678901234567890123456789012123456789012345678901
2
12345678901234567890123456789012123456789012345678901
version
of
RC
means
that
you
must
do
your
reading
on
a
computer
screen,
2345678901234567890123456789012123456789012345678901
2
1
2
12345678901234567890123456789012123456789012345678901
scrolling
up
and
down
to
read
the
words
that
appear
in
an
annoyingly
2
12345678901234567890123456789012123456789012345678901
2
12345678901234567890123456789012123456789012345678901
small
text
box.
2
12345678901234567890123456789012123456789012345678901
2345678901234567890123456789012123456789012345678901
2
1
2
12345678901234567890123456789012123456789012345678901
Since
the
format
is
so
familiar,
there
wont
be
any
surprises
when
your
first
2
12345678901234567890123456789012123456789012345678901
2
12345678901234567890123456789012123456789012345678901
RC
questions
pop
up
on
the
screen.
These
questions
do
not
focus
on
2
12345678901234567890123456789012123456789012345678901
2345678901234567890123456789012123456789012345678901
12345678901234567890123456789012123456789012345678901
vocabulary as much as the other three question types, providing a bit of 2
2
12345678901234567890123456789012123456789012345678901
12345678901234567890123456789012123456789012345678901
relief in that arena. These two aspects work to your advantage, but not 2
12345678901234567890123456789012123456789012345678901 22
12345678901234567890123456789012123456789012345678901
everything about RC is rosy. The main drawback is that you have to spend
2
12345678901234567890123456789012123456789012345678901
1 an inordinate amount of timecompared to the other GRE problem 2
2
12345678901234567890123456789012123456789012345678901
typesto answer a small amount of questions. This can lead to problems 2
12345678901234567890123456789012123456789012345678901
2
12345678901234567890123456789012123456789012345678901
if you are a very slow reader, as you might find yourself taking 46 minutes 2
12345678901234567890123456789012123456789012345678901
2
12345678901234567890123456789012123456789012345678901
of a 30-minute test just to answer two measly questions. The ratio of what 2
12345678901234567890123456789012123456789012345678901
2
12345678901234567890123456789012123456789012345678901
you put in to answer RC questions to what you get out of it is really lousy. 2
12345678901234567890123456789012123456789012345678901
2345678901234567890123456789012123456789012345678901
12345678901234567890123456789012123456789012345678901 22
12345678901234567890123456789012123456789012345678901
You should always take whatever time is needed to get a question right, 2
12345678901234567890123456789012123456789012345678901
2
12345678901234567890123456789012123456789012345678901
1 even if this means having to guess on some questions at the very end. Many 2
2345678901234567890123456789012123456789012345678901
12345678901234567890123456789012123456789012345678901
people tend to rush when taking a standardized test, and this is something 2
2
12345678901234567890123456789012123456789012345678901
1 you never want to do as it leads to careless errors. Careless errors are 2
12345678901234567890123456789012123456789012345678901 2
12345678901234567890123456789012123456789012345678901
compounded by the adaptive nature of the GRE, so people flying through 2
2
12345678901234567890123456789012123456789012345678901
2
12345678901234567890123456789012123456789012345678901
RC
problems
find
themselves
at
an
even
greater
disadvantage.
To
12345678901234567890123456789012123456789012345678901 2
12345678901234567890123456789012123456789012345678901
overcome any pacing problems before they appear, simply understand that 2
2
12345678901234567890123456789012123456789012345678901
2345678901234567890123456789012123456789012345678901
12345678901234567890123456789012123456789012345678901
you will have to take more time on RC problems, which means you should 2
2
12345678901234567890123456789012123456789012345678901
1 take less time on some other problems. For example, if you definitely 2
12345678901234567890123456789012123456789012345678901 2
12345678901234567890123456789012123456789012345678901
know the answer to an antonym problem, it should only take you about 2
2
12345678901234567890123456789012123456789012345678901
15 seconds to mark the correct answer.
2
12345678901234567890123456789012123456789012345678901
2
12345678901234567890123456789012123456789012345678901
2
12345678901234567890123456789012123456789012345678901
12345678901234567890123456789012123456789012345678901 2
2345678901234567890123456789012123456789012345678901
2
12345678901234567890123456789012123456789012345678901
A typical RC passage runs about 200 words in length and has 23
2
12345678901234567890123456789012123456789012345678901
questions associated with it. Longer passagesthose over and around
2
12345678901234567890123456789012123456789012345678901
2
12345678901234567890123456789012123456789012345678901
2
1 300 wordsmight have as many as 4 questions attached. On a normal
2345678901234567890123456789012123456789012345678901
2
12345678901234567890123456789012123456789012345678901
GRE, RC questions are shy, appearing for the first time only after you
2
12345678901234567890123456789012123456789012345678901
2
12345678901234567890123456789012123456789012345678901
have answered about 810 other questions.
2
12345678901234567890123456789012123456789012345678901
2
1
2345678901234567890123456789012123456789012345678901
12345678901234567890123456789012123456789012345678901 22
1 Along with their different format, RC problems require a different
2
12345678901234567890123456789012123456789012345678901
2
12345678901234567890123456789012123456789012345678901
approach.
The
overall
goal
is
to
make
sure
you
answer
the
questions
2
12345678901234567890123456789012123456789012345678901
2345678901234567890123456789012123456789012345678901
1 correctly in the shortest amount of time possible. To help you do this, read 2
12345678901234567890123456789012123456789012345678901 2
12345678901234567890123456789012123456789012345678901 2
12345678901234567890123456789012123456789012345678901 2 155
123456789012345678901234567890121234567890123456789012

http://www.xtremepapers.net

Part III: Section Review

123456789012345678901234567890121234567890123456789012
12345678901234567890123456789012123456789012345678901 2
2
12345678901234567890123456789012123456789012345678901
the question before you start reading the passage. Keep the question in the 2
12345678901234567890123456789012123456789012345678901
2
12345678901234567890123456789012123456789012345678901
back of your mind when tackling the passage. If you can answer the 2
12345678901234567890123456789012123456789012345678901
2
12345678901234567890123456789012123456789012345678901
question accurately before reading the entire passage, by all means do so. 2
12345678901234567890123456789012123456789012345678901
2345678901234567890123456789012123456789012345678901
1 The whole goal is to answer questions, not while away time reading a 2
2
12345678901234567890123456789012123456789012345678901
2
12345678901234567890123456789012123456789012345678901
bland
passage.
2
12345678901234567890123456789012123456789012345678901
2
12345678901234567890123456789012123456789012345678901
2345678901234567890123456789012123456789012345678901
1 The content of the passages is not going to electrify you. The passages are 2
2
12345678901234567890123456789012123456789012345678901
2
12345678901234567890123456789012123456789012345678901
non-fiction
and
tackle
a
diverse
range
of
scientific
and
non-scientific
2
12345678901234567890123456789012123456789012345678901
2
12345678901234567890123456789012123456789012345678901
topics.
It
should
be
noted
that
all
the
information
presented
has
to
be
2345678901234567890123456789012123456789012345678901
2
1
2
12345678901234567890123456789012123456789012345678901
accurate,
and
that
the
answers
to
the
questions
must
also
be
factually
2
12345678901234567890123456789012123456789012345678901
2
12345678901234567890123456789012123456789012345678901
accurate.
Therefore,
if
you
get
a
passage
about
the
War
of
the
Austrian
2
12345678901234567890123456789012123456789012345678901
2345678901234567890123456789012123456789012345678901
1 Succession and you just happen to be a big fan of the Hapsburg dynasty, 2
2
12345678901234567890123456789012123456789012345678901
12345678901234567890123456789012123456789012345678901
then youre in luck! On a more general note, you can use common sense to 2
2
12345678901234567890123456789012123456789012345678901
2
12345678901234567890123456789012123456789012345678901
eliminate some answer choices that are unlikely to be real or true.
2345678901234567890123456789012123456789012345678901
12345678901234567890123456789012123456789012345678901 2
2
12345678901234567890123456789012123456789012345678901
When you start reading the passages, try to understand the main point of 2
12345678901234567890123456789012123456789012345678901
2
12345678901234567890123456789012123456789012345678901
the passage first. This is often found hiding out in the first paragraph. Your 2
12345678901234567890123456789012123456789012345678901
2
12345678901234567890123456789012123456789012345678901
1 overall goal is to understand the main points of the passage and each of its 2
12345678901234567890123456789012123456789012345678901
paragraphs, but you dont need to waste time absorbing all the 2
2
12345678901234567890123456789012123456789012345678901
2
12345678901234567890123456789012123456789012345678901
information
into
your
long-term
memory.
Have
an
idea
of
where
all
the
2345678901234567890123456789012123456789012345678901
12345678901234567890123456789012123456789012345678901 2
12345678901234567890123456789012123456789012345678901
facts are, since you want to be able to go to the right part of the passage to 2
2
12345678901234567890123456789012123456789012345678901
2
12345678901234567890123456789012123456789012345678901
answer
any
questions
posed.
At
the
same
time,
you
dont
want
to
waste
12345678901234567890123456789012123456789012345678901 2
12345678901234567890123456789012123456789012345678901
any time reading information thats not essential to answering the 2
2
12345678901234567890123456789012123456789012345678901
2
12345678901234567890123456789012123456789012345678901
questions,
since
answering
questions
is
the
only
reason
youre
here.
12345678901234567890123456789012123456789012345678901 2
1 Finding the right balance between passage reading and question answering 2
2
12345678901234567890123456789012123456789012345678901
2
12345678901234567890123456789012123456789012345678901
takes
some
practice,
but
thats
why
there
are
three
pencil-and-paper
tests
12345678901234567890123456789012123456789012345678901 2
2
12345678901234567890123456789012123456789012345678901
in this book as well as others on the CD.
2
12345678901234567890123456789012123456789012345678901
2345678901234567890123456789012123456789012345678901
2
12345678901234567890123456789012123456789012345678901
Heres a short sample paragraph:
2
12345678901234567890123456789012123456789012345678901
2
1
2
12345678901234567890123456789012123456789012345678901
2
12345678901234567890123456789012123456789012345678901
Passage
1
2345678901234567890123456789012123456789012345678901
12345678901234567890123456789012123456789012345678901 2
12345678901234567890123456789012123456789012345678901 2
2
1
Line Viewing the underwater world through a land-based perspec2
12345678901234567890123456789012123456789012345678901
2
12345678901234567890123456789012123456789012345678901
tive,
it
is
easy
to
consider
a
colony
of
corala
broad
term
used
2345678901234567890123456789012123456789012345678901
12345678901234567890123456789012123456789012345678901 2
2
12345678901234567890123456789012123456789012345678901
to describe the marine organisms, like those of the genus
2
12345678901234567890123456789012123456789012345678901
2
12345678901234567890123456789012123456789012345678901
Corallium, which inhabit many tropical watersas something
2
1
2345678901234567890123456789012123456789012345678901
12345678901234567890123456789012123456789012345678901
(5)
similar to a bed of flowers. Both corals and flowers have many 2
2
12345678901234567890123456789012123456789012345678901
2
12345678901234567890123456789012123456789012345678901
species that display brilliant markings and have intricate
12345678901234567890123456789012123456789012345678901 2
shapes pleasing to the human eye. Yet the relationship between 2
1
2
12345678901234567890123456789012123456789012345678901
flowers and corals is predominantly an aesthetic one, as the
2
12345678901234567890123456789012123456789012345678901
2
12345678901234567890123456789012123456789012345678901
basic composition of a coral polyp, an animal, differs mark2
12345678901234567890123456789012123456789012345678901
2
12345678901234567890123456789012123456789012345678901
(10)
edly
from
that
of
a
self-photosynthesizing
plant
like
a
rose
or
2345678901234567890123456789012123456789012345678901
12345678901234567890123456789012123456789012345678901 2
2
1
zinnia.
2
12345678901234567890123456789012123456789012345678901
2
12345678901234567890123456789012123456789012345678901
It
could
even
be
argued
that
lichen,
a
fungus,
bears
more
in
2
12345678901234567890123456789012123456789012345678901
2345678901234567890123456789012123456789012345678901
2
1
common with corals than flowers, since both lichen and corals
2
12345678901234567890123456789012123456789012345678901
12345678901234567890123456789012123456789012345678901 2
2
156 12345678901234567890123456789012123456789012345678901
123456789012345678901234567890121234567890123456789012

www.petersons.com

http://www.xtremepapers.net

Note

Chapter 5: Verbal Review

123456789012345678901234567890121234567890123456789012
12345678901234567890123456789012123456789012345678901 2
2
12345678901234567890123456789012123456789012345678901
are typified by beneficial relationships with photosynthesizing
2
12345678901234567890123456789012123456789012345678901
2
12345678901234567890123456789012123456789012345678901
(15)
algae. While there are some species of coral that can exist
2
12345678901234567890123456789012123456789012345678901
2
12345678901234567890123456789012123456789012345678901
solely
by
capturing
food,
most
corals
receive
energy
by
hosting
2
12345678901234567890123456789012123456789012345678901
2345678901234567890123456789012123456789012345678901
2
1
millions of single-celled organisms commonly referred to as
2
12345678901234567890123456789012123456789012345678901
2
12345678901234567890123456789012123456789012345678901
zooxanthellae.
The
zooxanthellae
are
provided
safety
from
2
12345678901234567890123456789012123456789012345678901
2
12345678901234567890123456789012123456789012345678901
predation
as
well
as
certain
by-products
of
the
corals
2345678901234567890123456789012123456789012345678901
2
1
2
12345678901234567890123456789012123456789012345678901
(20)
metabolism,
like
ammonia,
which
zooxanthellae
require
to
2
12345678901234567890123456789012123456789012345678901
2
12345678901234567890123456789012123456789012345678901
grow
and
reproduce.
In
return,
a
portion
of
the
energy
2
12345678901234567890123456789012123456789012345678901
2345678901234567890123456789012123456789012345678901
2
1
produced by the zooxanthellae through photosynthesis is used
2
12345678901234567890123456789012123456789012345678901
2
12345678901234567890123456789012123456789012345678901
by
the
coral
to
further
its
own
needs
for
sustenance,
growth,
2
12345678901234567890123456789012123456789012345678901
2
12345678901234567890123456789012123456789012345678901
and
reproduction.
2345678901234567890123456789012123456789012345678901
2
1
2
12345678901234567890123456789012123456789012345678901
12345678901234567890123456789012123456789012345678901
The text is wordy, the sentences are long, the style is bookish, and the topic 2
2
12345678901234567890123456789012123456789012345678901
12345678901234567890123456789012123456789012345678901
somewhat dull. In short, its your typical RC passage on a standardized 2
2345678901234567890123456789012123456789012345678901
12345678901234567890123456789012123456789012345678901 22
12345678901234567890123456789012123456789012345678901
test. Instead of trying to gain some deep insight from the text, just get an
2
12345678901234567890123456789012123456789012345678901
idea of what each paragraph is about and then dive into the questions.
2
12345678901234567890123456789012123456789012345678901
12345678901234567890123456789012123456789012345678901 22
12345678901234567890123456789012123456789012345678901 2
1
The author of the passage uses the visual comparison of coral to a
2
12345678901234567890123456789012123456789012345678901
2
12345678901234567890123456789012123456789012345678901
bed
of
flowers
in
order
to
illustrate
2
12345678901234567890123456789012123456789012345678901
2345678901234567890123456789012123456789012345678901
12345678901234567890123456789012123456789012345678901 22
12345678901234567890123456789012123456789012345678901
A. how human perceptions of other habitats are influenced by
2
12345678901234567890123456789012123456789012345678901
2
12345678901234567890123456789012123456789012345678901
their own environment.
12345678901234567890123456789012123456789012345678901 22
12345678901234567890123456789012123456789012345678901
B. the zoological similarities that exist between corals and
2
12345678901234567890123456789012123456789012345678901
2
12345678901234567890123456789012123456789012345678901
flowers.
12345678901234567890123456789012123456789012345678901 22
1
C. why a better understanding of corals is needed in order to
2
12345678901234567890123456789012123456789012345678901
2
preserve endangered forms.
12345678901234567890123456789012123456789012345678901
2
12345678901234567890123456789012123456789012345678901
D. the multiple ways that the perspective organisms are regarded.
2
12345678901234567890123456789012123456789012345678901
2
12345678901234567890123456789012123456789012345678901
E.
the
ability
of
underwater
life
to
mimic
that
of
the
land-based
2345678901234567890123456789012123456789012345678901
12345678901234567890123456789012123456789012345678901 22
12345678901234567890123456789012123456789012345678901
world.
2
1
2
12345678901234567890123456789012123456789012345678901
2
12345678901234567890123456789012123456789012345678901
2345678901234567890123456789012123456789012345678901
12345678901234567890123456789012123456789012345678901 22
12345678901234567890123456789012123456789012345678901
If you were really cagy, you would have zipped down and read this
2
1 question before tackling the text of the passage. It doesnt matter if you
12345678901234567890123456789012123456789012345678901 2
2
12345678901234567890123456789012123456789012345678901
didnt do this for this question, but make sure to get in the habit of
2
12345678901234567890123456789012123456789012345678901
2
12345678901234567890123456789012123456789012345678901
reviewing the questions before tackling the text.
2
12345678901234567890123456789012123456789012345678901
2
12345678901234567890123456789012123456789012345678901
12345678901234567890123456789012123456789012345678901 2
2345678901234567890123456789012123456789012345678901
2
12345678901234567890123456789012123456789012345678901
This question covers a bit of text in the first paragraph, so dont try to 2
12345678901234567890123456789012123456789012345678901
2
12345678901234567890123456789012123456789012345678901
answer the question without looking back at the first paragraph. Theres a 2
12345678901234567890123456789012123456789012345678901
1 tendency for students who feel rushed to try to answer a problem from 2
2
12345678901234567890123456789012123456789012345678901
2
12345678901234567890123456789012123456789012345678901
memory, as if great amounts of time are saved by not referring back to the 2
12345678901234567890123456789012123456789012345678901
12345678901234567890123456789012123456789012345678901
passage. Some seconds might be shaved off, its true, but these seconds 2
2
12345678901234567890123456789012123456789012345678901
2345678901234567890123456789012123456789012345678901
2
12345678901234567890123456789012123456789012345678901
1 wont help much if you get the problem wrong. Go back and look over the 2
12345678901234567890123456789012123456789012345678901
first paragraph to give yourself the best shot possible to find the correct 2
2
12345678901234567890123456789012123456789012345678901
2
12345678901234567890123456789012123456789012345678901
answer.
2345678901234567890123456789012123456789012345678901
2
1
12345678901234567890123456789012123456789012345678901 2
12345678901234567890123456789012123456789012345678901 2
12345678901234567890123456789012123456789012345678901 2 157
123456789012345678901234567890121234567890123456789012

http://www.xtremepapers.net

Part III: Section Review

123456789012345678901234567890121234567890123456789012
12345678901234567890123456789012123456789012345678901 2
2
12345678901234567890123456789012123456789012345678901
Instead of jumping right to the correct answer, lets review why some of the 2
12345678901234567890123456789012123456789012345678901
2
12345678901234567890123456789012123456789012345678901
other choices are not the right one. Choice (C) sounds good, and people 2
12345678901234567890123456789012123456789012345678901
2
12345678901234567890123456789012123456789012345678901
who are very passionate about the environment might agree with the 2
12345678901234567890123456789012123456789012345678901
2345678901234567890123456789012123456789012345678901
1 statement whole-heartedly. Yet the author of the passage never once 2
2
12345678901234567890123456789012123456789012345678901
2
12345678901234567890123456789012123456789012345678901
mentions
the
environmental
angle,
so
the
answer
choice
is
just
there
to
trip
2
12345678901234567890123456789012123456789012345678901
2
12345678901234567890123456789012123456789012345678901
up
GRE-taking
members
of
Greenpeace.
Think
of
this
as
the
True,
but
So
2345678901234567890123456789012123456789012345678901
2
1
2
12345678901234567890123456789012123456789012345678901
What?
wrong
answer
choice.
There
will
be
answer
choices
that
sound
2
12345678901234567890123456789012123456789012345678901
2
12345678901234567890123456789012123456789012345678901
really
good
but
have
nothing
to
do
with
the
actual
text
you
are
working
2
12345678901234567890123456789012123456789012345678901
2345678901234567890123456789012123456789012345678901
2
1 with. Cross these out.
2
12345678901234567890123456789012123456789012345678901
2
12345678901234567890123456789012123456789012345678901
2
12345678901234567890123456789012123456789012345678901
Choices
(B)
and
(E)
both
use
words
and
phrases
that
can
be
found
in
the
2
12345678901234567890123456789012123456789012345678901
2345678901234567890123456789012123456789012345678901
1 passage itself. Choice (B) has the words corals, flower, similarities, and the 2
2
12345678901234567890123456789012123456789012345678901
12345678901234567890123456789012123456789012345678901
prefix zoo-, all of which appear in the passage. Choice (E) has underwater 2
2
12345678901234567890123456789012123456789012345678901
12345678901234567890123456789012123456789012345678901
and land-based set up in a nice, contrasting way. Both of these choices 2
2345678901234567890123456789012123456789012345678901
12345678901234567890123456789012123456789012345678901 2
12345678901234567890123456789012123456789012345678901
offer looks over substance, since the actual meaning of each sentence 2
2
12345678901234567890123456789012123456789012345678901
makes them incorrect answer choices. They are like a bracelet made out of 2
12345678901234567890123456789012123456789012345678901
2
12345678901234567890123456789012123456789012345678901
pearls and whipped cream. Even though the pearls look nice, the bracelet 2
12345678901234567890123456789012123456789012345678901
1 itself cannot be worn. Choice (B) actually states that there are deeper 2
2
12345678901234567890123456789012123456789012345678901
2
12345678901234567890123456789012123456789012345678901
similarities
between
flowers
and
corals,
when
the
whole
paragraph
is
2
12345678901234567890123456789012123456789012345678901
2345678901234567890123456789012123456789012345678901
12345678901234567890123456789012123456789012345678901
about how these similarities are superficial. The verb mimic in choice (E) 2
2
12345678901234567890123456789012123456789012345678901
2
12345678901234567890123456789012123456789012345678901
makes
this
statement
totally
incorrect,
since
there
is
nothing
in
the
passage
12345678901234567890123456789012123456789012345678901 2
2
12345678901234567890123456789012123456789012345678901
to back up this claim.
2
12345678901234567890123456789012123456789012345678901
12345678901234567890123456789012123456789012345678901 2
12345678901234567890123456789012123456789012345678901
This leaves only two choices, (A) and (D). Quite often, the correct answer 2
2
12345678901234567890123456789012123456789012345678901
1 for an RC question merely paraphrases, or restates, text from the passage 2
2345678901234567890123456789012123456789012345678901
12345678901234567890123456789012123456789012345678901 2
2
12345678901234567890123456789012123456789012345678901
1 using different words. The passage starts by showing how a land-based 2
12345678901234567890123456789012123456789012345678901
perspective can lead to the wrong conclusion about corals. In other words, 2
2
12345678901234567890123456789012123456789012345678901
2345678901234567890123456789012123456789012345678901
12345678901234567890123456789012123456789012345678901
peoples perceptions of other habitats (the ocean) are influenced by their 2
2
12345678901234567890123456789012123456789012345678901
1 own environment, the land. Thats choice (A), and thats your paraphrased 2
2
12345678901234567890123456789012123456789012345678901
2
12345678901234567890123456789012123456789012345678901
correct answer.
2345678901234567890123456789012123456789012345678901
12345678901234567890123456789012123456789012345678901 2
12345678901234567890123456789012123456789012345678901 2
2
1
From the second paragraph, it can be inferred that
2
12345678901234567890123456789012123456789012345678901
12345678901234567890123456789012123456789012345678901 2
2345678901234567890123456789012123456789012345678901
2
12345678901234567890123456789012123456789012345678901
A. lichen use zooxanthellae algae for use in photosynthesis.
2
12345678901234567890123456789012123456789012345678901
2
12345678901234567890123456789012123456789012345678901
B.
lichen
provides
some
form
of
photosynthesizing
algae
with
the
12345678901234567890123456789012123456789012345678901 2
2
1
same basic nutrients that corals provide zooxanthellae.
2
12345678901234567890123456789012123456789012345678901
2
12345678901234567890123456789012123456789012345678901
C.
zooxanthellae
serve
a
function
with
corals
that
is
served
by
2
12345678901234567890123456789012123456789012345678901
2
12345678901234567890123456789012123456789012345678901
some photosynthesizing organism in lichen.
12345678901234567890123456789012123456789012345678901 2
2345678901234567890123456789012123456789012345678901
12345678901234567890123456789012123456789012345678901
D. corals are not as dependent on their photosynthesizing partners 2
2
12345678901234567890123456789012123456789012345678901
2
12345678901234567890123456789012123456789012345678901
as lichen is.
12345678901234567890123456789012123456789012345678901 2
2
E. corals and lichen are both able to capture food for themselves
1
2
12345678901234567890123456789012123456789012345678901
if necessary.
2
12345678901234567890123456789012123456789012345678901
2
12345678901234567890123456789012123456789012345678901
12345678901234567890123456789012123456789012345678901 2
2
12345678901234567890123456789012123456789012345678901
12345678901234567890123456789012123456789012345678901 2
12345678901234567890123456789012123456789012345678901 2
12345678901234567890123456789012123456789012345678901 2
2
158 12345678901234567890123456789012123456789012345678901
123456789012345678901234567890121234567890123456789012

www.petersons.com

http://www.xtremepapers.net

Tip

Chapter 5: Verbal Review

123456789012345678901234567890121234567890123456789012
12345678901234567890123456789012123456789012345678901 2
2
12345678901234567890123456789012123456789012345678901
12345678901234567890123456789012123456789012345678901 2
2
12345678901234567890123456789012123456789012345678901
Most RC problems are not going to ask you to find information
2
12345678901234567890123456789012123456789012345678901
2
12345678901234567890123456789012123456789012345678901
specifically
stated
in
the
passage.
Instead,
you
will
be
asked
to
infer
or
2
12345678901234567890123456789012123456789012345678901
2345678901234567890123456789012123456789012345678901
2
1 conclude an answer, based on what was written. This means that while
2
12345678901234567890123456789012123456789012345678901
2
12345678901234567890123456789012123456789012345678901
the correct answer is not stated directly, the clues needed to find the right
2
12345678901234567890123456789012123456789012345678901
2
12345678901234567890123456789012123456789012345678901
answer are there. So, think like you did on the SC problems and search the
2345678901234567890123456789012123456789012345678901
2
1
2
12345678901234567890123456789012123456789012345678901
passage text for the clues that lead you to the right answer. The clues will
2
12345678901234567890123456789012123456789012345678901
2
12345678901234567890123456789012123456789012345678901
always be there; you need only find them and they will lead you to the
2
12345678901234567890123456789012123456789012345678901
correct
answer.
2345678901234567890123456789012123456789012345678901
2
1
2
12345678901234567890123456789012123456789012345678901
2
12345678901234567890123456789012123456789012345678901
2
12345678901234567890123456789012123456789012345678901
If this question seems confusing, its because it is. Many RC problems and 2
12345678901234567890123456789012123456789012345678901
2345678901234567890123456789012123456789012345678901
1 answer choices appear to have been penned by Theodore Dreiser. (For 2
2
12345678901234567890123456789012123456789012345678901
2
12345678901234567890123456789012123456789012345678901
those non-English majors, this joke means that they are very laboriously 2
12345678901234567890123456789012123456789012345678901
2
12345678901234567890123456789012123456789012345678901
written.) The whole point of these choices is to get you unnerved to the 2
12345678901234567890123456789012123456789012345678901
12345678901234567890123456789012123456789012345678901
extent that you hurry and make a poor decision. If you understand that 2
2
12345678901234567890123456789012123456789012345678901
2
12345678901234567890123456789012123456789012345678901
this
is
the
trap,
you
can
avoid
it.
2
12345678901234567890123456789012123456789012345678901
2345678901234567890123456789012123456789012345678901
12345678901234567890123456789012123456789012345678901 22
1 First, you know where to look for the right answer, since the question
2
12345678901234567890123456789012123456789012345678901
2
12345678901234567890123456789012123456789012345678901
states,
in
the
second
passage.
Accept
any
free
information
thats
given.
2
12345678901234567890123456789012123456789012345678901
2345678901234567890123456789012123456789012345678901
12345678901234567890123456789012123456789012345678901
The answer choices continue to make various claims about the analogous 2
2
12345678901234567890123456789012123456789012345678901
2
12345678901234567890123456789012123456789012345678901
remember
this
word?relationship
between
corals
and
zooxanthellae
and
12345678901234567890123456789012123456789012345678901 22
12345678901234567890123456789012123456789012345678901
lichen with some photosynthesizing organism. In other words,
2
12345678901234567890123456789012123456789012345678901
2
12345678901234567890123456789012123456789012345678901
CORAL : ZOOXANTHELLAE :: LICHEN : Some unnamed 2
12345678901234567890123456789012123456789012345678901
2
12345678901234567890123456789012123456789012345678901
photosynthesizing organism
2
1
2345678901234567890123456789012123456789012345678901
12345678901234567890123456789012123456789012345678901 22
12345678901234567890123456789012123456789012345678901
2
1 You can speak GRE, too!
12345678901234567890123456789012123456789012345678901 2
12345678901234567890123456789012123456789012345678901
Choices (B), (D), and (E) dont work because you dont know enough 2
2
12345678901234567890123456789012123456789012345678901
2
12345678901234567890123456789012123456789012345678901
about
lichen.
Theres
nothing
in
the
passage
that
says
how
dependent
12345678901234567890123456789012123456789012345678901 2
12345678901234567890123456789012123456789012345678901
lichen is, choice (D), or whether lichen can capture its own food, choice 2
2
12345678901234567890123456789012123456789012345678901
2345678901234567890123456789012123456789012345678901
12345678901234567890123456789012123456789012345678901
(E). These answers wouldnt be inferences, they would be unfounded 2
2
12345678901234567890123456789012123456789012345678901
1 guesses. Furthermore, you dont know if lichen uses zooxanthellae 2
12345678901234567890123456789012123456789012345678901 2
12345678901234567890123456789012123456789012345678901
algaean unlikely choice (A) since lichen is not found underwateror if 2
2
12345678901234567890123456789012123456789012345678901
12345678901234567890123456789012123456789012345678901
lichen provides its guest algae with basic nutrients, choice (B). All you 2
2
12345678901234567890123456789012123456789012345678901
12345678901234567890123456789012123456789012345678901
know is that there is an analogy between lichen and its photosynthesizing 2
2
12345678901234567890123456789012123456789012345678901
organism and corals with zooxanthellae. The correct answer is (C).
2
12345678901234567890123456789012123456789012345678901
2
12345678901234567890123456789012123456789012345678901
2
12345678901234567890123456789012123456789012345678901
One
last
note:
People
with
saltwater
aquariums
might
have
known
about
2
12345678901234567890123456789012123456789012345678901
2
12345678901234567890123456789012123456789012345678901
corals
and
their
little
sun-loving
buddies,
showing
how
outside
knowledge
2345678901234567890123456789012123456789012345678901
12345678901234567890123456789012123456789012345678901 22
12345678901234567890123456789012123456789012345678901
can come in handy depending on the subject you get.
2
12345678901234567890123456789012123456789012345678901
12345678901234567890123456789012123456789012345678901 22
1
2
12345678901234567890123456789012123456789012345678901
12345678901234567890123456789012123456789012345678901 2
2
12345678901234567890123456789012123456789012345678901
12345678901234567890123456789012123456789012345678901 2
2
12345678901234567890123456789012123456789012345678901
12345678901234567890123456789012123456789012345678901 2
12345678901234567890123456789012123456789012345678901 2
12345678901234567890123456789012123456789012345678901 2
12345678901234567890123456789012123456789012345678901 2 159
123456789012345678901234567890121234567890123456789012

http://www.xtremepapers.net

Part III: Section Review

123456789012345678901234567890121234567890123456789012
12345678901234567890123456789012123456789012345678901 2
2
12345678901234567890123456789012123456789012345678901
2
12345678901234567890123456789012123456789012345678901
One
Last Note about Vocabulary
2
12345678901234567890123456789012123456789012345678901
2
12345678901234567890123456789012123456789012345678901
And
that
note
is,
study
more
vocabulary!
All
the
techniques
discussed
12345678901234567890123456789012123456789012345678901 2
12345678901234567890123456789012123456789012345678901
above are useful, but if you dont know the words, their effectiveness will 2
2
12345678901234567890123456789012123456789012345678901
2
12345678901234567890123456789012123456789012345678901
be
greatly
reduced.
The
other
advantage
to
studying
vocabulary
is
that
you
2
12345678901234567890123456789012123456789012345678901
2
12345678901234567890123456789012123456789012345678901
can
use
these
advanced
words
to
make
your
Analytical
essays
sound
more
2
12345678901234567890123456789012123456789012345678901
2345678901234567890123456789012123456789012345678901
1 urbane and educated. For more advice on how to improve your essays, 2
2
12345678901234567890123456789012123456789012345678901
2
12345678901234567890123456789012123456789012345678901
turn
to
Chapter
6
and
start
reading.
2
12345678901234567890123456789012123456789012345678901
2
12345678901234567890123456789012123456789012345678901
2345678901234567890123456789012123456789012345678901
2
1
2
12345678901234567890123456789012123456789012345678901
2
12345678901234567890123456789012123456789012345678901
2
12345678901234567890123456789012123456789012345678901
2
12345678901234567890123456789012123456789012345678901
2345678901234567890123456789012123456789012345678901
2
1
2
12345678901234567890123456789012123456789012345678901
2
12345678901234567890123456789012123456789012345678901
2
12345678901234567890123456789012123456789012345678901
2
12345678901234567890123456789012123456789012345678901
2345678901234567890123456789012123456789012345678901
12345678901234567890123456789012123456789012345678901 2
12345678901234567890123456789012123456789012345678901 2
12345678901234567890123456789012123456789012345678901 2
12345678901234567890123456789012123456789012345678901 2
12345678901234567890123456789012123456789012345678901 2
12345678901234567890123456789012123456789012345678901 2
2
1
2
12345678901234567890123456789012123456789012345678901
2
12345678901234567890123456789012123456789012345678901
2
12345678901234567890123456789012123456789012345678901
2345678901234567890123456789012123456789012345678901
12345678901234567890123456789012123456789012345678901 2
12345678901234567890123456789012123456789012345678901 2
12345678901234567890123456789012123456789012345678901 2
12345678901234567890123456789012123456789012345678901 2
12345678901234567890123456789012123456789012345678901 2
12345678901234567890123456789012123456789012345678901 2
12345678901234567890123456789012123456789012345678901 2
12345678901234567890123456789012123456789012345678901 2
12345678901234567890123456789012123456789012345678901 2
2
1
2
12345678901234567890123456789012123456789012345678901
2
12345678901234567890123456789012123456789012345678901
12345678901234567890123456789012123456789012345678901 2
12345678901234567890123456789012123456789012345678901 2
2
12345678901234567890123456789012123456789012345678901
2
12345678901234567890123456789012123456789012345678901
2
12345678901234567890123456789012123456789012345678901
12345678901234567890123456789012123456789012345678901 2
2
12345678901234567890123456789012123456789012345678901
2
12345678901234567890123456789012123456789012345678901
2345678901234567890123456789012123456789012345678901
12345678901234567890123456789012123456789012345678901 2
12345678901234567890123456789012123456789012345678901 2
2
1
12345678901234567890123456789012123456789012345678901 2
12345678901234567890123456789012123456789012345678901 2
2
12345678901234567890123456789012123456789012345678901
2
12345678901234567890123456789012123456789012345678901
2
12345678901234567890123456789012123456789012345678901
2
12345678901234567890123456789012123456789012345678901
12345678901234567890123456789012123456789012345678901 2
2345678901234567890123456789012123456789012345678901
12345678901234567890123456789012123456789012345678901 2
12345678901234567890123456789012123456789012345678901 2
12345678901234567890123456789012123456789012345678901 2
12345678901234567890123456789012123456789012345678901 2
2
1
2
12345678901234567890123456789012123456789012345678901
2
12345678901234567890123456789012123456789012345678901
2
12345678901234567890123456789012123456789012345678901
2
12345678901234567890123456789012123456789012345678901
12345678901234567890123456789012123456789012345678901 2
2345678901234567890123456789012123456789012345678901
12345678901234567890123456789012123456789012345678901 2
2
1
2
12345678901234567890123456789012123456789012345678901
12345678901234567890123456789012123456789012345678901 2
2
12345678901234567890123456789012123456789012345678901
12345678901234567890123456789012123456789012345678901 2
12345678901234567890123456789012123456789012345678901 2
12345678901234567890123456789012123456789012345678901 2
2
160 12345678901234567890123456789012123456789012345678901
123456789012345678901234567890121234567890123456789012

www.petersons.com

http://www.xtremepapers.net

Take It
to the Next Level

Alert!

123456789012345678901234567890121234567890123456789012
2
12345678901234567890123456789012123456789012345678901
2
12345678901234567890123456789012123456789012345678901
2345678901234567890123456789012123456789012345678901
2
1
Here
There Be Wyverns
2
12345678901234567890123456789012123456789012345678901
2
12345678901234567890123456789012123456789012345678901
(and
other
Difficult
Words)
2
12345678901234567890123456789012123456789012345678901
2
12345678901234567890123456789012123456789012345678901
Graduate
programs
use
GRE
scores
in
a
bewildering
variety
of
ways,
but
2345678901234567890123456789012123456789012345678901
12345678901234567890123456789012123456789012345678901 22
12345678901234567890123456789012123456789012345678901
in the end it all basically comes down to making piles. Graduate programs 2
12345678901234567890123456789012123456789012345678901
2
12345678901234567890123456789012123456789012345678901
start out with a huge pile of applicants, and they need to do things to make 2
12345678901234567890123456789012123456789012345678901
2
12345678901234567890123456789012123456789012345678901
1 this big pile smaller. The big pile gets whittled down one way or another, 2
12345678901234567890123456789012123456789012345678901
and when it gets down to a manageable size, then people start to get 2
2
12345678901234567890123456789012123456789012345678901
2
12345678901234567890123456789012123456789012345678901
seriously
considered.
2345678901234567890123456789012123456789012345678901
12345678901234567890123456789012123456789012345678901 22
12345678901234567890123456789012123456789012345678901 2
12345678901234567890123456789012123456789012345678901
Your GRE score is one easy, easy way to whittle down a pile. Viewed this 2
12345678901234567890123456789012123456789012345678901
12345678901234567890123456789012123456789012345678901
way, you can see that your goal is to have a GRE score good enough to 2
2
12345678901234567890123456789012123456789012345678901
2
12345678901234567890123456789012123456789012345678901
keep
from
getting
placed
on
the
wrong
pile.
Some
programs
employ
the
12345678901234567890123456789012123456789012345678901 22
12345678901234567890123456789012123456789012345678901
1 combined Quantitative and Verbal scores, which is good for you because it 2
12345678901234567890123456789012123456789012345678901
is easier to gain points on the math side than it is on the verbal side. If you 2
2
12345678901234567890123456789012123456789012345678901
12345678901234567890123456789012123456789012345678901
wanted to boost your score by about 100 points, you could shoot for a 2
12345678901234567890123456789012123456789012345678901 2
2
12345678901234567890123456789012123456789012345678901
70-point improvement in math and a 30-point increase in verbal.
2
12345678901234567890123456789012123456789012345678901
2
12345678901234567890123456789012123456789012345678901
12345678901234567890123456789012123456789012345678901 2
2
12345678901234567890123456789012123456789012345678901
A higher number of humanities programs require the GRE, so the major
2
12345678901234567890123456789012123456789012345678901
2345678901234567890123456789012123456789012345678901
2
12345678901234567890123456789012123456789012345678901
proportion of people taking the test are better at the verbal portion than
2
12345678901234567890123456789012123456789012345678901
2
1 the math portion. This verbal-centric skew makes it harder to get a high
2
12345678901234567890123456789012123456789012345678901
verbal score, since relatively speaking most people are doing better on
2
12345678901234567890123456789012123456789012345678901
2345678901234567890123456789012123456789012345678901
2
12345678901234567890123456789012123456789012345678901
that section than on the math section.
2
12345678901234567890123456789012123456789012345678901
12345678901234567890123456789012123456789012345678901 22
12345678901234567890123456789012123456789012345678901 2
1 Other graduate programslike those with the word English in
2
12345678901234567890123456789012123456789012345678901
2
12345678901234567890123456789012123456789012345678901
themare often not concerned about your Quantitative score. These 2
12345678901234567890123456789012123456789012345678901
2
12345678901234567890123456789012123456789012345678901
programs are going to whittle down their piles based on Verbal and 2
12345678901234567890123456789012123456789012345678901
2345678901234567890123456789012123456789012345678901
12345678901234567890123456789012123456789012345678901
perhaps Analytical scores. If this is the case for you, then gunning for a 2
2
12345678901234567890123456789012123456789012345678901
2
12345678901234567890123456789012123456789012345678901
high
Verbal
score
is
the
right
plan.
However,
if
the
program
is
more
12345678901234567890123456789012123456789012345678901 22
1 concerned with overall score, keep in mind that it will be easier to gain
2
12345678901234567890123456789012123456789012345678901
2
12345678901234567890123456789012123456789012345678901
extra
points
in
the
math
portion.
2
12345678901234567890123456789012123456789012345678901
12345678901234567890123456789012123456789012345678901 2
2
12345678901234567890123456789012123456789012345678901
12345678901234567890123456789012123456789012345678901 2
12345678901234567890123456789012123456789012345678901 2
12345678901234567890123456789012123456789012345678901 2
12345678901234567890123456789012123456789012345678901 2
123456789012345678901234567890121234567890123456789012
161

http://www.xtremepapers.net

Part III: Section Review

www.petersons.com

http://www.xtremepapers.net

Note

123456789012345678901234567890121234567890123456789012
12345678901234567890123456789012123456789012345678901 2
2
12345678901234567890123456789012123456789012345678901
Getting a high score in the GRE Verbal sectionhigh 600s to low 700sis 2
12345678901234567890123456789012123456789012345678901
2
12345678901234567890123456789012123456789012345678901
not very simple, since it often boils down to one word. To be more precise, 2
12345678901234567890123456789012123456789012345678901
2
12345678901234567890123456789012123456789012345678901
it comes down to one word that means all words: vocabulary.
2
12345678901234567890123456789012123456789012345678901
2345678901234567890123456789012123456789012345678901
2
1
2
12345678901234567890123456789012123456789012345678901
2
12345678901234567890123456789012123456789012345678901
2
12345678901234567890123456789012123456789012345678901
2
A 112345678901234567890123456789012123456789012345678901
Simple Formula to Make Medium Verbal
2345678901234567890123456789012123456789012345678901
2
2345678901234567890123456789012123456789012345678901
2
1
Problems
Harder: Use Tougher Words
2
12345678901234567890123456789012123456789012345678901
2
12345678901234567890123456789012123456789012345678901
12345678901234567890123456789012123456789012345678901
There are no different question types to separate medium problems from 2
2345678901234567890123456789012123456789012345678901
2
1
12345678901234567890123456789012123456789012345678901
hard ones. Antonyms, analogies, and sentence completions will all have 2
2
12345678901234567890123456789012123456789012345678901
the same format, but instead of the antonym BULKY youll have the word 2
12345678901234567890123456789012123456789012345678901
2
12345678901234567890123456789012123456789012345678901
CORPULENT. Both words mean the same thing, but corpulent is a more 2
12345678901234567890123456789012123456789012345678901
2
12345678901234567890123456789012123456789012345678901
difficult word. Analogies will still be formed using the common 2
12345678901234567890123456789012123456789012345678901
2
12345678901234567890123456789012123456789012345678901
relationships described on page 144, but once again the words used will be 2
12345678901234567890123456789012123456789012345678901
2345678901234567890123456789012123456789012345678901
2
12345678901234567890123456789012123456789012345678901
tougher and not as commonly known. Ditto for sentence completions, as 2
12345678901234567890123456789012123456789012345678901
12345678901234567890123456789012123456789012345678901
answer choices will come from the same stockpile of advanced vocabulary. 2
2
12345678901234567890123456789012123456789012345678901
2
12345678901234567890123456789012123456789012345678901
The
emphasis
on
tougher
words
will
be
slightest
for
reading
comprehen12345678901234567890123456789012123456789012345678901 2
1 sion, but even there passages may feature a large amount of technical and 2
2
12345678901234567890123456789012123456789012345678901
2
12345678901234567890123456789012123456789012345678901
scientific
jargon
within
the
text.
It
will
be
like
the
coral/zooxanthellae
2
12345678901234567890123456789012123456789012345678901
2345678901234567890123456789012123456789012345678901
12345678901234567890123456789012123456789012345678901
passage on pages 156157, but with even more terms such as photosyn- 2
2
12345678901234567890123456789012123456789012345678901
2
12345678901234567890123456789012123456789012345678901
thesizing
organism
bandied
about.
12345678901234567890123456789012123456789012345678901 2
12345678901234567890123456789012123456789012345678901 2
12345678901234567890123456789012123456789012345678901
The adaptive format, and the manner in which the GRE Verbal starts, only 2
2
12345678901234567890123456789012123456789012345678901
12345678901234567890123456789012123456789012345678901
exacerbates the problem. The first 36 problems youll face will be either 2
12345678901234567890123456789012123456789012345678901 2
1 antonyms or analogies, and if youre shooting for a high score its going to 2
2
12345678901234567890123456789012123456789012345678901
be important to get most of these questions right. Since these questions are 2
12345678901234567890123456789012123456789012345678901
2
12345678901234567890123456789012123456789012345678901
little more than vocabulary problems, knowing the words used on these 2
12345678901234567890123456789012123456789012345678901
2
12345678901234567890123456789012123456789012345678901
problems is going to take a combination of knowledge and luck. The 2
12345678901234567890123456789012123456789012345678901
2
12345678901234567890123456789012123456789012345678901
knowledge will come in the form of a good lexicon, and the luck will come 2
12345678901234567890123456789012123456789012345678901
2
12345678901234567890123456789012123456789012345678901
since not everybody knows the definition of every word.
2
12345678901234567890123456789012123456789012345678901
2345678901234567890123456789012123456789012345678901
12345678901234567890123456789012123456789012345678901 2
12345678901234567890123456789012123456789012345678901 2
2
1
12345678901234567890123456789012123456789012345678901 2
2
12345678901234567890123456789012123456789012345678901
Notice the term exacerbates is used instead of increases. To help your 2
12345678901234567890123456789012123456789012345678901
2
12345678901234567890123456789012123456789012345678901
score, it makes sense to jump on the GRE Tough Words bandwagon.
2
12345678901234567890123456789012123456789012345678901
2
12345678901234567890123456789012123456789012345678901
12345678901234567890123456789012123456789012345678901 2
2345678901234567890123456789012123456789012345678901
12345678901234567890123456789012123456789012345678901 2
12345678901234567890123456789012123456789012345678901 2
2
12345678901234567890123456789012123456789012345678901
Since vocabulary is the key to getting a high Verbal score, the bulk of this 2
12345678901234567890123456789012123456789012345678901
1 chapter will focus on expanding the list of tough words you know. Before 2
2
12345678901234567890123456789012123456789012345678901
2
12345678901234567890123456789012123456789012345678901
this occurs, though, a brief note about the various question types will be 2
12345678901234567890123456789012123456789012345678901
2
12345678901234567890123456789012123456789012345678901
covered.
2
12345678901234567890123456789012123456789012345678901
2345678901234567890123456789012123456789012345678901
12345678901234567890123456789012123456789012345678901 2
2
1
2
12345678901234567890123456789012123456789012345678901
12345678901234567890123456789012123456789012345678901 2
2
12345678901234567890123456789012123456789012345678901
12345678901234567890123456789012123456789012345678901 2
12345678901234567890123456789012123456789012345678901 2
12345678901234567890123456789012123456789012345678901 2
2
162 12345678901234567890123456789012123456789012345678901
123456789012345678901234567890121234567890123456789012

Chapter 5: Verbal Review

Take It to the Next Level

123456789012345678901234567890121234567890123456789012
12345678901234567890123456789012123456789012345678901 2
2
12345678901234567890123456789012123456789012345678901
2
12345678901234567890123456789012123456789012345678901
Similar Strategies, Harder Problems
2
12345678901234567890123456789012123456789012345678901
2
12345678901234567890123456789012123456789012345678901
The
question-specific
verbal
strategiescovered
earlier
in
this
chapter
12345678901234567890123456789012123456789012345678901 2
12345678901234567890123456789012123456789012345678901
are the same for antonyms, analogies, and sentence completions. You 2
2
12345678901234567890123456789012123456789012345678901
12345678901234567890123456789012123456789012345678901
always want to make a link between the stem words on an analogy, 2
2
12345678901234567890123456789012123456789012345678901
12345678901234567890123456789012123456789012345678901
regardless of when the question shows up and how difficult the words are. 2
2
12345678901234567890123456789012123456789012345678901
12345678901234567890123456789012123456789012345678901
Similarly, the approach to antonyms is the same: Focus on the stem word, 2
2
12345678901234567890123456789012123456789012345678901
12345678901234567890123456789012123456789012345678901
decide whether or not you know the meaning of this word, and then 2
2
12345678901234567890123456789012123456789012345678901
proceed accordingly.
2
12345678901234567890123456789012123456789012345678901
2345678901234567890123456789012123456789012345678901
2
1
2
12345678901234567890123456789012123456789012345678901
Difficult SC problems will typically have two blanks instead of just one, 2
12345678901234567890123456789012123456789012345678901
12345678901234567890123456789012123456789012345678901
making you search for clues for both missing words. When the test decides 2
2
12345678901234567890123456789012123456789012345678901
2345678901234567890123456789012123456789012345678901
1 to get really clever, you get a situation where you cant figure out either 2
2
12345678901234567890123456789012123456789012345678901
2
12345678901234567890123456789012123456789012345678901
blank
because
they
are
linked.
The
sample
sentence
below
shows
this
in
2
12345678901234567890123456789012123456789012345678901
2
12345678901234567890123456789012123456789012345678901
action:
2345678901234567890123456789012123456789012345678901
12345678901234567890123456789012123456789012345678901 22
12345678901234567890123456789012123456789012345678901 2
12345678901234567890123456789012123456789012345678901
The guests were __________ at the __________ state of the basement. 2
12345678901234567890123456789012123456789012345678901
12345678901234567890123456789012123456789012345678901 22
12345678901234567890123456789012123456789012345678901
1 The problem here isnt that you cant place any words. Rather, many 2
different combinations of words work in the two places, since the clue to 2
12345678901234567890123456789012123456789012345678901
2
12345678901234567890123456789012123456789012345678901
the first blank (a missing verb) is the second blank (an adjective describing 2
12345678901234567890123456789012123456789012345678901
2345678901234567890123456789012123456789012345678901
2
12345678901234567890123456789012123456789012345678901
the basement).
2
12345678901234567890123456789012123456789012345678901
12345678901234567890123456789012123456789012345678901 22
12345678901234567890123456789012123456789012345678901
The guests could be:
2
12345678901234567890123456789012123456789012345678901
12345678901234567890123456789012123456789012345678901 22
12345678901234567890123456789012123456789012345678901
delighted at the magnificent state of the basement.
2
12345678901234567890123456789012123456789012345678901
12345678901234567890123456789012123456789012345678901 22
1
disgusted at the slovenly state of the basement.
2
12345678901234567890123456789012123456789012345678901
2
12345678901234567890123456789012123456789012345678901
bemused at the whimsical state of the basement.
2
12345678901234567890123456789012123456789012345678901
12345678901234567890123456789012123456789012345678901 2
12345678901234567890123456789012123456789012345678901
Its uncertain how you get a basement to be whimsical, but if you could, 2
2
12345678901234567890123456789012123456789012345678901
2
12345678901234567890123456789012123456789012345678901
then the last choice works just as well as the others.
2
12345678901234567890123456789012123456789012345678901
2
12345678901234567890123456789012123456789012345678901
2
12345678901234567890123456789012123456789012345678901
If
you
come
across
one
of
these
two-blank
SC
questions,
congratulate
2345678901234567890123456789012123456789012345678901
12345678901234567890123456789012123456789012345678901 22
12345678901234567890123456789012123456789012345678901
1 yourself for doing well on the test so far, since you wouldnt get one of 2
12345678901234567890123456789012123456789012345678901
these puppies if you were answering questions poorly. After you are done 2
2
12345678901234567890123456789012123456789012345678901
2345678901234567890123456789012123456789012345678901
12345678901234567890123456789012123456789012345678901
patting your back, determine what the link between the two missing words 2
2
12345678901234567890123456789012123456789012345678901
12345678901234567890123456789012123456789012345678901
is. This link is usually just similar (like in the example above) or 2
12345678901234567890123456789012123456789012345678901 22
1 opposite. An opposite SC would resemble:
2
12345678901234567890123456789012123456789012345678901
2
12345678901234567890123456789012123456789012345678901
The guests were not __________ at the __________ state of the 2
12345678901234567890123456789012123456789012345678901
2
12345678901234567890123456789012123456789012345678901
basement.
2
12345678901234567890123456789012123456789012345678901
2345678901234567890123456789012123456789012345678901
12345678901234567890123456789012123456789012345678901 22
12345678901234567890123456789012123456789012345678901
The not flips the connection between the two words from similar to 2
12345678901234567890123456789012123456789012345678901
2
12345678901234567890123456789012123456789012345678901
2
1 opposite.
2345678901234567890123456789012123456789012345678901
12345678901234567890123456789012123456789012345678901 22
1
2
12345678901234567890123456789012123456789012345678901
12345678901234567890123456789012123456789012345678901 2
2
12345678901234567890123456789012123456789012345678901
12345678901234567890123456789012123456789012345678901 2
12345678901234567890123456789012123456789012345678901 2
12345678901234567890123456789012123456789012345678901 2
12345678901234567890123456789012123456789012345678901 2 163
123456789012345678901234567890121234567890123456789012

http://www.xtremepapers.net

Part III: Section Review

123456789012345678901234567890121234567890123456789012
12345678901234567890123456789012123456789012345678901 2
2
12345678901234567890123456789012123456789012345678901
The guests were __________ at the __________ state of the
2
12345678901234567890123456789012123456789012345678901
2
12345678901234567890123456789012123456789012345678901
basement.
2
12345678901234567890123456789012123456789012345678901
12345678901234567890123456789012123456789012345678901 2
2
12345678901234567890123456789012123456789012345678901
A. appalled. .tidy
2
12345678901234567890123456789012123456789012345678901
2
12345678901234567890123456789012123456789012345678901
B.
appreciative.
.flooded
2
12345678901234567890123456789012123456789012345678901
2
12345678901234567890123456789012123456789012345678901
C.
befuddled.
.paltry
2
12345678901234567890123456789012123456789012345678901
2345678901234567890123456789012123456789012345678901
2
1
D. enthralled. .singular
2
12345678901234567890123456789012123456789012345678901
2
12345678901234567890123456789012123456789012345678901
E.
ambivalent.
.mildewed
2
12345678901234567890123456789012123456789012345678901
2
12345678901234567890123456789012123456789012345678901
2345678901234567890123456789012123456789012345678901
1 Once you figure out the link, its time to hit the answer choices and start 2
2
12345678901234567890123456789012123456789012345678901
12345678901234567890123456789012123456789012345678901
sifting through the vocabulary. Choice (A) has opposites, so thats a wrong 2
2
12345678901234567890123456789012123456789012345678901
12345678901234567890123456789012123456789012345678901
link. Choices (B) and (E) dont work very well, as the second term does not 2
2345678901234567890123456789012123456789012345678901
2
1
12345678901234567890123456789012123456789012345678901
do a very good job of justifying the first one. Also, flooded and mildewed 2
2
12345678901234567890123456789012123456789012345678901
are two things that happen to real basements, so these answer choices are 2
12345678901234567890123456789012123456789012345678901
2
12345678901234567890123456789012123456789012345678901
probably there to distract you. Paring things down to two good choices, 2
12345678901234567890123456789012123456789012345678901
2
12345678901234567890123456789012123456789012345678901
(C) and (D), there is no reason to think people would be confused 2
12345678901234567890123456789012123456789012345678901
2
12345678901234567890123456789012123456789012345678901
(befuddled) by a spare (paltry) basement. Choice (D) is the best answer, as 2
12345678901234567890123456789012123456789012345678901
2345678901234567890123456789012123456789012345678901
2
12345678901234567890123456789012123456789012345678901
1 something singular (wondrous) would cause people to be enthralled 2
2
12345678901234567890123456789012123456789012345678901
(captivated).
2
12345678901234567890123456789012123456789012345678901
2
12345678901234567890123456789012123456789012345678901
2345678901234567890123456789012123456789012345678901
12345678901234567890123456789012123456789012345678901
After you survive the first eight or so questions on the GRE Verbal, youll 2
2
12345678901234567890123456789012123456789012345678901
2
12345678901234567890123456789012123456789012345678901
get
your
first
shot
at
the
Reading
Comp
passage.
If
you
are
scoring
very
12345678901234567890123456789012123456789012345678901 2
12345678901234567890123456789012123456789012345678901
well, this passage should be a tough RC passage, which means a densely 2
2
12345678901234567890123456789012123456789012345678901
2
12345678901234567890123456789012123456789012345678901
worded
passage
on
some
arcane
topic.
One
way
to
handle
this
type
of
12345678901234567890123456789012123456789012345678901 2
2
12345678901234567890123456789012123456789012345678901
1 dense passage is to jot down notes on a sheet of scratch paper. Note-taking 2
2345678901234567890123456789012123456789012345678901
12345678901234567890123456789012123456789012345678901
can help you understand the passage better, although it will slow you 2
2
12345678901234567890123456789012123456789012345678901
1 down a bit. However, imagine that you are in a classroom and listening to 2
12345678901234567890123456789012123456789012345678901 2
12345678901234567890123456789012123456789012345678901
a professor drone on about a boring subject that you know will be on the 2
2
12345678901234567890123456789012123456789012345678901
mid-term. Will you understand the topic better if you take notes, or if you 2
12345678901234567890123456789012123456789012345678901
2
12345678901234567890123456789012123456789012345678901
dont? Again, this takes additional time, but if you are gunning for a high 2
12345678901234567890123456789012123456789012345678901
2
12345678901234567890123456789012123456789012345678901
score, you should have the time if your wide knowledge of vocabulary 2
12345678901234567890123456789012123456789012345678901
2
12345678901234567890123456789012123456789012345678901
helped you sail through the antonym problems.
2
12345678901234567890123456789012123456789012345678901
12345678901234567890123456789012123456789012345678901 2
12345678901234567890123456789012123456789012345678901
Jotting down a quick note about each paragraph will help you understand 2
2
12345678901234567890123456789012123456789012345678901
2
12345678901234567890123456789012123456789012345678901
the
content
of
each
one.
This
will
come
in
handy
when
you
tackle
the
RC
2
12345678901234567890123456789012123456789012345678901
2
12345678901234567890123456789012123456789012345678901
problems.
It
is
highly
unlikely
that
any
of
these
problems
will
be
a
question
12345678901234567890123456789012123456789012345678901 2
2345678901234567890123456789012123456789012345678901
12345678901234567890123456789012123456789012345678901
as straightforward as, What would be a good title for this essay? 2
2
12345678901234567890123456789012123456789012345678901
2
12345678901234567890123456789012123456789012345678901
Instead,
expect
to
see
convoluted
problems
like:
12345678901234567890123456789012123456789012345678901 2
2
1
12345678901234567890123456789012123456789012345678901
If Dr. Andersen changes her mind about the social behavior of 2
2
12345678901234567890123456789012123456789012345678901
12345678901234567890123456789012123456789012345678901
nudibranchs, then which of the following statements would she be most 2
2
12345678901234567890123456789012123456789012345678901
2
likely to agree with?
12345678901234567890123456789012123456789012345678901
2345678901234567890123456789012123456789012345678901
12345678901234567890123456789012123456789012345678901 2
1 Faced with this twisted piece of writing, you would first have to go into the 2
2
12345678901234567890123456789012123456789012345678901
2
12345678901234567890123456789012123456789012345678901
passage
to
the
section
where
Dr.
Andersen
discusses
nudibranchs
and/or
2
12345678901234567890123456789012123456789012345678901
2345678901234567890123456789012123456789012345678901
1 social behavior. Then you would have to infer what she thinks about it, 2
12345678901234567890123456789012123456789012345678901 2
12345678901234567890123456789012123456789012345678901 2
2
164 12345678901234567890123456789012123456789012345678901
123456789012345678901234567890121234567890123456789012

www.petersons.com

http://www.xtremepapers.net

Chapter 5: Verbal Review

Take It to the Next Level

Tip

123456789012345678901234567890121234567890123456789012
12345678901234567890123456789012123456789012345678901 2
2
12345678901234567890123456789012123456789012345678901
figure out what the opposite of that is, and then look through the answer 2
12345678901234567890123456789012123456789012345678901
2
12345678901234567890123456789012123456789012345678901
choices to find the paraphrased answer that matches this opposite 2
12345678901234567890123456789012123456789012345678901
2
12345678901234567890123456789012123456789012345678901
inference.
2
12345678901234567890123456789012123456789012345678901
2345678901234567890123456789012123456789012345678901
2
1
2
12345678901234567890123456789012123456789012345678901
2
12345678901234567890123456789012123456789012345678901
You would have to infer what Dr. Andersen thinks because it is very
2
12345678901234567890123456789012123456789012345678901
2
12345678901234567890123456789012123456789012345678901
doubtful
anything
will
be
given
to
you
on
a
tough
RC
question.
Nothing
2345678901234567890123456789012123456789012345678901
2
1
2
12345678901234567890123456789012123456789012345678901
will just be waiting in the passage; you will have to take the clues in the
2
12345678901234567890123456789012123456789012345678901
2
12345678901234567890123456789012123456789012345678901
passage
and
use
them
to
deduce
the
information
you
need.
2
12345678901234567890123456789012123456789012345678901
2345678901234567890123456789012123456789012345678901
2
1
2
12345678901234567890123456789012123456789012345678901
2
12345678901234567890123456789012123456789012345678901
The
additional
work
required
to
answer
a
hard
RC
problem
makes
these
2
12345678901234567890123456789012123456789012345678901
2
12345678901234567890123456789012123456789012345678901
questions
more
involved
and
time-consuming.
Yet
they
are
still
very
2345678901234567890123456789012123456789012345678901
2
1
2
12345678901234567890123456789012123456789012345678901
solvable
if
you
take
the
time
to
look
back
into
the
passage
and
infer
the
2
12345678901234567890123456789012123456789012345678901
2
12345678901234567890123456789012123456789012345678901
answer
that
you
need.
2
12345678901234567890123456789012123456789012345678901
2345678901234567890123456789012123456789012345678901
12345678901234567890123456789012123456789012345678901 22
12345678901234567890123456789012123456789012345678901 2
12345678901234567890123456789012123456789012345678901 2
12345678901234567890123456789012123456789012345678901 2
12345678901234567890123456789012123456789012345678901
Free
Words! Free Words!
2
12345678901234567890123456789012123456789012345678901
1 To acquire a comprehensive list of all the words that might appear on the 2
2
12345678901234567890123456789012123456789012345678901
2
12345678901234567890123456789012123456789012345678901
GRE,
simply
go
down
to
your
local
bookstore
and
look
for
a
dictionary
of
2
12345678901234567890123456789012123456789012345678901
2345678901234567890123456789012123456789012345678901
12345678901234567890123456789012123456789012345678901
at least 1,500 pages. For a slightly smaller list of advanced words, try the 2
2
12345678901234567890123456789012123456789012345678901
2
12345678901234567890123456789012123456789012345678901
set
below.
Theres
no
way
to
tell
which
batch
of
words
you
will
get
on
your
12345678901234567890123456789012123456789012345678901 22
12345678901234567890123456789012123456789012345678901
particular GRE, but any increase in your stockpile of tough words 2
12345678901234567890123456789012123456789012345678901
2
12345678901234567890123456789012123456789012345678901
increases your odds of succeeding on the test.
2
12345678901234567890123456789012123456789012345678901
2
12345678901234567890123456789012123456789012345678901
1 To really get the most out of this list, write out each word and then provide 2
2345678901234567890123456789012123456789012345678901
2
12345678901234567890123456789012123456789012345678901
a definition and sample sentence. You can put this list of words in a 2
12345678901234567890123456789012123456789012345678901
1 notebook, handheld PDA, or plain old flash cards. Use whatever media 2
12345678901234567890123456789012123456789012345678901 2
2
12345678901234567890123456789012123456789012345678901
you are most comfortable with, but make sure to compile the list in a way 2
12345678901234567890123456789012123456789012345678901
2
12345678901234567890123456789012123456789012345678901
that makes it easy for you to review the contents.
2
12345678901234567890123456789012123456789012345678901
2
12345678901234567890123456789012123456789012345678901
2
12345678901234567890123456789012123456789012345678901
If
you
are
willing
to
take
the
time,
get
a
thesaurus
and
include
a
list
of
2345678901234567890123456789012123456789012345678901
12345678901234567890123456789012123456789012345678901 22
12345678901234567890123456789012123456789012345678901
1 possible antonyms for each applicable word. Some words wont have 2
12345678901234567890123456789012123456789012345678901
antonyms, but many of them will, and studying the antonyms of difficult 2
2
12345678901234567890123456789012123456789012345678901
2345678901234567890123456789012123456789012345678901
12345678901234567890123456789012123456789012345678901
words has obvious, obvious advantages. Some of those antonyms will be 2
2
12345678901234567890123456789012123456789012345678901
2
12345678901234567890123456789012123456789012345678901
hard
words
themselves,
giving
you
an
easy
way
to
increase
your
12345678901234567890123456789012123456789012345678901 22
1 vocabulary even further.
2
12345678901234567890123456789012123456789012345678901
2
12345678901234567890123456789012123456789012345678901
2
12345678901234567890123456789012123456789012345678901
2
12345678901234567890123456789012123456789012345678901
12345678901234567890123456789012123456789012345678901 2
2345678901234567890123456789012123456789012345678901
12345678901234567890123456789012123456789012345678901 22
12345678901234567890123456789012123456789012345678901 2
12345678901234567890123456789012123456789012345678901 2
12345678901234567890123456789012123456789012345678901 2
1
2
12345678901234567890123456789012123456789012345678901
12345678901234567890123456789012123456789012345678901 2
2
12345678901234567890123456789012123456789012345678901
12345678901234567890123456789012123456789012345678901 2
2
12345678901234567890123456789012123456789012345678901
12345678901234567890123456789012123456789012345678901 2
12345678901234567890123456789012123456789012345678901 2
12345678901234567890123456789012123456789012345678901 2
12345678901234567890123456789012123456789012345678901 2 165
123456789012345678901234567890121234567890123456789012

http://www.xtremepapers.net

Part III: Section Review

123456789012345678901234567890121234567890123456789012
12345678901234567890123456789012123456789012345678901 2
2
12345678901234567890123456789012123456789012345678901
2
12345678901234567890123456789012123456789012345678901
The
Word List
2
12345678901234567890123456789012123456789012345678901
2
12345678901234567890123456789012123456789012345678901
12345678901234567890123456789012123456789012345678901 2
2
12345678901234567890123456789012123456789012345678901
2
12345678901234567890123456789012123456789012345678901
Word
Origin Latin brevis 5 short. Also found in English brevity.
2
12345678901234567890123456789012123456789012345678901
2
12345678901234567890123456789012123456789012345678901
abbreviate
(verb)
To
make
briefer,
to
shorten.
Because
time
2
12345678901234567890123456789012123456789012345678901
2
12345678901234567890123456789012123456789012345678901
was
running
out,
the
speaker
was
forced
to
abbreviate
2345678901234567890123456789012123456789012345678901
2
1
2
12345678901234567890123456789012123456789012345678901
his
remarks.
abbreviation
(noun).
2
12345678901234567890123456789012123456789012345678901
2
12345678901234567890123456789012123456789012345678901
2
12345678901234567890123456789012123456789012345678901
aberration
(noun)
A
deviation
from
what
is
normal
or
2345678901234567890123456789012123456789012345678901
2
1
2
12345678901234567890123456789012123456789012345678901
natural,
an
abnormality.
Jacks
extravagant
lunch
at
2
12345678901234567890123456789012123456789012345678901
2
12345678901234567890123456789012123456789012345678901
Lutece
was
an
aberration
from
his
usual
meal,
a
peanut
2
12345678901234567890123456789012123456789012345678901
2345678901234567890123456789012123456789012345678901
2
1
butter sandwich and a diet soda. aberrant (adjective).
2
12345678901234567890123456789012123456789012345678901
2
12345678901234567890123456789012123456789012345678901
abeyance (noun) A temporary lapse in activity; suspension. 2
12345678901234567890123456789012123456789012345678901
2
12345678901234567890123456789012123456789012345678901
In the aftermath of the bombing, all normal activities 2
12345678901234567890123456789012123456789012345678901
2
12345678901234567890123456789012123456789012345678901
were held in abeyance.
2
12345678901234567890123456789012123456789012345678901
2
12345678901234567890123456789012123456789012345678901
2
12345678901234567890123456789012123456789012345678901
abjure
(verb)
To
renounce
or
reject;
to
officially
disclaim.
2345678901234567890123456789012123456789012345678901
12345678901234567890123456789012123456789012345678901 2
1
While being tried by the inquisition in 1633, Galileo 2
2
12345678901234567890123456789012123456789012345678901
2
12345678901234567890123456789012123456789012345678901
abjured
all
his
writings
holding
that
Earth
and
other
2
12345678901234567890123456789012123456789012345678901
2345678901234567890123456789012123456789012345678901
2
12345678901234567890123456789012123456789012345678901
planets revolved around the sun.
2
12345678901234567890123456789012123456789012345678901
12345678901234567890123456789012123456789012345678901 2
12345678901234567890123456789012123456789012345678901 2
2
12345678901234567890123456789012123456789012345678901
2
12345678901234567890123456789012123456789012345678901
Word
Origin Latin abradare 5 to scrape. Also found in English abrasive.
2
12345678901234567890123456789012123456789012345678901
2
12345678901234567890123456789012123456789012345678901
abrade (verb) To irritate by rubbing; to wear down in spirit. 2
12345678901234567890123456789012123456789012345678901
1
Olgas conditioning facial abraded Sabrinas skin so 2
2
12345678901234567890123456789012123456789012345678901
12345678901234567890123456789012123456789012345678901
severely that she vowed never to let anyones hands 2
2
12345678901234567890123456789012123456789012345678901
2
12345678901234567890123456789012123456789012345678901
touch
her
face
again.
abrasion
(noun).
2
12345678901234567890123456789012123456789012345678901
2345678901234567890123456789012123456789012345678901
12345678901234567890123456789012123456789012345678901 2
12345678901234567890123456789012123456789012345678901
abridge (verb) To shorten, to reduce. The Bill of Rights is 2
2
1
2
12345678901234567890123456789012123456789012345678901
designed
to
prevent
Congress
from
abridging
the
rights
2
12345678901234567890123456789012123456789012345678901
2345678901234567890123456789012123456789012345678901
2
12345678901234567890123456789012123456789012345678901
of Americans. abridgment (noun).
2
12345678901234567890123456789012123456789012345678901
2
1
12345678901234567890123456789012123456789012345678901
abrogate (verb) To nullify, to abolish. During World War II, 2
12345678901234567890123456789012123456789012345678901 2
the United States abrogated the rights of Japanese 2
12345678901234567890123456789012123456789012345678901
2
12345678901234567890123456789012123456789012345678901
Americans by detaining them in internment camps. 2
12345678901234567890123456789012123456789012345678901
2
12345678901234567890123456789012123456789012345678901
abrogation (noun).
2
12345678901234567890123456789012123456789012345678901
2345678901234567890123456789012123456789012345678901
12345678901234567890123456789012123456789012345678901 2
12345678901234567890123456789012123456789012345678901
abscond (verb) To make a secret departure, to elope. 2
2
12345678901234567890123456789012123456789012345678901
2
12345678901234567890123456789012123456789012345678901
Theresa
will
never
forgive
her
daughter,
Elena,
for
2
1
2345678901234567890123456789012123456789012345678901
12345678901234567890123456789012123456789012345678901
absconding to Miami with Philip when they were 2
2
12345678901234567890123456789012123456789012345678901
2
12345678901234567890123456789012123456789012345678901
only
17.
12345678901234567890123456789012123456789012345678901 2
2
1
12345678901234567890123456789012123456789012345678901
accretion (noun) A gradual build-up or enlargement. My 2
2
12345678901234567890123456789012123456789012345678901
12345678901234567890123456789012123456789012345678901
mothers house is a mess due to her steady accretion of 2
12345678901234567890123456789012123456789012345678901 2
2
12345678901234567890123456789012123456789012345678901
bric-a-brac and her inability to throw anything away.
2
12345678901234567890123456789012123456789012345678901
12345678901234567890123456789012123456789012345678901 2
12345678901234567890123456789012123456789012345678901 2
2
166 12345678901234567890123456789012123456789012345678901
123456789012345678901234567890121234567890123456789012

www.petersons.com

http://www.xtremepapers.net

Chapter 5: Verbal Review

Take It to the Next Level

123456789012345678901234567890121234567890123456789012
12345678901234567890123456789012123456789012345678901 2
2
12345678901234567890123456789012123456789012345678901
adjunct (noun) Something added to another thing, but not a 2
12345678901234567890123456789012123456789012345678901
2
12345678901234567890123456789012123456789012345678901
part of it; an associate or assistant. While Felix and Fritz 2
12345678901234567890123456789012123456789012345678901
2
12345678901234567890123456789012123456789012345678901
were adjuncts to Professor Himmelman during his 2
12345678901234567890123456789012123456789012345678901
2345678901234567890123456789012123456789012345678901
2
1
experiments in electrodynamics, they did not receive 2
12345678901234567890123456789012123456789012345678901
2
12345678901234567890123456789012123456789012345678901
credit when the results were published.
2
12345678901234567890123456789012123456789012345678901
2
12345678901234567890123456789012123456789012345678901
2345678901234567890123456789012123456789012345678901
1
adroit (adjective) Skillful, adept. The writer Laurie Colwin 2
2
12345678901234567890123456789012123456789012345678901
2
12345678901234567890123456789012123456789012345678901
was
particularly
adroit
at
concocting
love
stories
involv2
12345678901234567890123456789012123456789012345678901
2
12345678901234567890123456789012123456789012345678901
ing
admirable
and
quirky
female
heroines
and
men
who
2345678901234567890123456789012123456789012345678901
2
1
2
12345678901234567890123456789012123456789012345678901
deserve
them.
2
12345678901234567890123456789012123456789012345678901
2
12345678901234567890123456789012123456789012345678901
12345678901234567890123456789012123456789012345678901
adulterate (verb) To corrupt, to make impure. Unlike the 2
2345678901234567890123456789012123456789012345678901
2
1
12345678901234567890123456789012123456789012345678901
chickens from the large poultry companies, Murrays 2
2
12345678901234567890123456789012123456789012345678901
free-roaming chickens have not been adulterated with 2
12345678901234567890123456789012123456789012345678901
2
12345678901234567890123456789012123456789012345678901
hormones and other additives.
2
12345678901234567890123456789012123456789012345678901
2
12345678901234567890123456789012123456789012345678901
2
12345678901234567890123456789012123456789012345678901
2
12345678901234567890123456789012123456789012345678901
2
12345678901234567890123456789012123456789012345678901
Latin vertere 5 to turn. Also found in English adversary, adverse,
2345678901234567890123456789012123456789012345678901
2
12345678901234567890123456789012123456789012345678901
Word
Origin reverse, vertical, and vertigo.
2
1
2
12345678901234567890123456789012123456789012345678901
2
12345678901234567890123456789012123456789012345678901
adversary
(noun)
An
enemy
or
opponent.
When
the
former
2
12345678901234567890123456789012123456789012345678901
2345678901234567890123456789012123456789012345678901
12345678901234567890123456789012123456789012345678901
Soviet Union became an American ally, the United States 2
2
12345678901234567890123456789012123456789012345678901
2
12345678901234567890123456789012123456789012345678901
lost
its
last
major
international
adversary.
adverse
12345678901234567890123456789012123456789012345678901 22
12345678901234567890123456789012123456789012345678901
(adjective).
2
12345678901234567890123456789012123456789012345678901
12345678901234567890123456789012123456789012345678901 22
12345678901234567890123456789012123456789012345678901
aesthete (noun) Someone devoted to beauty and to beautiful
2
12345678901234567890123456789012123456789012345678901
1
things. A renowned aesthete, Oscar Wilde was the 2
2345678901234567890123456789012123456789012345678901
12345678901234567890123456789012123456789012345678901 22
center of a group that glorified beauty and adopted the
12345678901234567890123456789012123456789012345678901
2
1
slogan art for arts sake. aesthetic (adjective).
2
12345678901234567890123456789012123456789012345678901
2
12345678901234567890123456789012123456789012345678901
2345678901234567890123456789012123456789012345678901
12345678901234567890123456789012123456789012345678901
affability (noun) The quality of being easy to talk to and 2
2
12345678901234567890123456789012123456789012345678901
2
1
gracious.
Affability
is
a
much-desired
trait
in
any
2
12345678901234567890123456789012123456789012345678901
2
12345678901234567890123456789012123456789012345678901
profession
that
involves
dealing
with
many
people
on
a
2345678901234567890123456789012123456789012345678901
12345678901234567890123456789012123456789012345678901 22
12345678901234567890123456789012123456789012345678901
daily basis. affable (adjective).
2
1
12345678901234567890123456789012123456789012345678901 2
12345678901234567890123456789012123456789012345678901
affected (adjective) False, artificial. At one time, Japanese 2
2
12345678901234567890123456789012123456789012345678901
2
12345678901234567890123456789012123456789012345678901
women
were
taught
to
speak
in
an
affected
high-pitched
2
12345678901234567890123456789012123456789012345678901
12345678901234567890123456789012123456789012345678901
voice, which was thought girlishly attractive. affect 2
12345678901234567890123456789012123456789012345678901 2
2
12345678901234567890123456789012123456789012345678901
(verb), affectation (noun).
2
12345678901234567890123456789012123456789012345678901
2
12345678901234567890123456789012123456789012345678901
affinity (noun) A feeling of shared attraction, kinship; a 2
12345678901234567890123456789012123456789012345678901
2
12345678901234567890123456789012123456789012345678901
similarity. When they first fell in love, Andrew and 2
12345678901234567890123456789012123456789012345678901
2
12345678901234567890123456789012123456789012345678901
Tanya marveled over their affinity for bluegrass music, 2
12345678901234567890123456789012123456789012345678901
12345678901234567890123456789012123456789012345678901
obscure French poetry, and beer taken with a squirt of 2
2
12345678901234567890123456789012123456789012345678901
2345678901234567890123456789012123456789012345678901
12345678901234567890123456789012123456789012345678901
lemon juice. People often say there is a striking affinity 2
2
1
2
12345678901234567890123456789012123456789012345678901
between
dogs
and
their
owners
(but
please
dont
tell
12345678901234567890123456789012123456789012345678901 2
12345678901234567890123456789012123456789012345678901
Clara that she and her bassett hound are starting to 2
2
12345678901234567890123456789012123456789012345678901
2
12345678901234567890123456789012123456789012345678901
resemble
each
other).
12345678901234567890123456789012123456789012345678901 22
12345678901234567890123456789012123456789012345678901 167
123456789012345678901234567890121234567890123456789012

http://www.xtremepapers.net

Part III: Section Review

123456789012345678901234567890121234567890123456789012
12345678901234567890123456789012123456789012345678901 2
2
12345678901234567890123456789012123456789012345678901
aggrandize (verb) To make bigger or greater; to inflate. 2
12345678901234567890123456789012123456789012345678901
2
12345678901234567890123456789012123456789012345678901
When he was mayor of New York City, Ed Koch was 2
12345678901234567890123456789012123456789012345678901
2
12345678901234567890123456789012123456789012345678901
renowned for aggrandizing his accomplishments and 2
12345678901234567890123456789012123456789012345678901
2345678901234567890123456789012123456789012345678901
2
1
strolling through city events shouting, Howm I 2
12345678901234567890123456789012123456789012345678901
2
12345678901234567890123456789012123456789012345678901
doing? aggrandizement (noun).
2
12345678901234567890123456789012123456789012345678901
2
12345678901234567890123456789012123456789012345678901
2345678901234567890123456789012123456789012345678901
1
agitation (noun) A disturbance; a disturbing feeling of 2
2
12345678901234567890123456789012123456789012345678901
2
12345678901234567890123456789012123456789012345678901
upheaval
and
excitement.
After
the
CEO
announced
the
2
12345678901234567890123456789012123456789012345678901
2
12345678901234567890123456789012123456789012345678901
coming
layoffs,
the
employees
agitation
was
evident
as
2345678901234567890123456789012123456789012345678901
2
1
2
12345678901234567890123456789012123456789012345678901
they
remained
in
the
auditorium
talking
excitedly
2
12345678901234567890123456789012123456789012345678901
2
12345678901234567890123456789012123456789012345678901
among
themselves.
agitated
(adjective),
agitate
(verb).
2
12345678901234567890123456789012123456789012345678901
2345678901234567890123456789012123456789012345678901
2
1
12345678901234567890123456789012123456789012345678901
alias (noun) An assumed name. Determined not to reveal 2
2
12345678901234567890123456789012123456789012345678901
his upper-class roots, Harold Steerforth Hetherington 2
12345678901234567890123456789012123456789012345678901
2
12345678901234567890123456789012123456789012345678901
III went under the alias of Hound Dog when playing 2
12345678901234567890123456789012123456789012345678901
2
12345678901234567890123456789012123456789012345678901
trumpet in his blues band.
2
12345678901234567890123456789012123456789012345678901
2
12345678901234567890123456789012123456789012345678901
2
12345678901234567890123456789012123456789012345678901
allegiance
(noun)
Loyalty
or
devotion
shown
to
ones
2345678901234567890123456789012123456789012345678901
12345678901234567890123456789012123456789012345678901 2
1
government or to a person, group, or cause. At the 2
2
12345678901234567890123456789012123456789012345678901
2
12345678901234567890123456789012123456789012345678901
moving
naturalization
ceremony,
forty-three
new
Ameri2
12345678901234567890123456789012123456789012345678901
2345678901234567890123456789012123456789012345678901
12345678901234567890123456789012123456789012345678901
cans from twenty-five lands swore allegiance to the 2
2
12345678901234567890123456789012123456789012345678901
2
12345678901234567890123456789012123456789012345678901
United
States.
12345678901234567890123456789012123456789012345678901 2
12345678901234567890123456789012123456789012345678901 2
12345678901234567890123456789012123456789012345678901
allocate (verb) To apportion for a specific purpose; to 2
2
12345678901234567890123456789012123456789012345678901
12345678901234567890123456789012123456789012345678901
distribute. The president talked about the importance of 2
12345678901234567890123456789012123456789012345678901 2
1
education and health care in his State of the Union 2
2
12345678901234567890123456789012123456789012345678901
address, but, in the end, the administration did not 2
12345678901234567890123456789012123456789012345678901
2
12345678901234567890123456789012123456789012345678901
allocate enough resources for these pressing concerns. 2
12345678901234567890123456789012123456789012345678901
2
12345678901234567890123456789012123456789012345678901
allocation (noun).
2
12345678901234567890123456789012123456789012345678901
2
12345678901234567890123456789012123456789012345678901
2
12345678901234567890123456789012123456789012345678901
amalgamate
(verb)
To
blend
thoroughly.
The
tendency
of
2
12345678901234567890123456789012123456789012345678901
2
12345678901234567890123456789012123456789012345678901
grains
to
sort
when
they
should
mix
makes
it
difficult
for
2345678901234567890123456789012123456789012345678901
12345678901234567890123456789012123456789012345678901 2
12345678901234567890123456789012123456789012345678901
manufacturers to create powders that are amalgamated. 2
2
1
2
12345678901234567890123456789012123456789012345678901
amalgamation
(noun).
12345678901234567890123456789012123456789012345678901 2
2345678901234567890123456789012123456789012345678901
12345678901234567890123456789012123456789012345678901 2
12345678901234567890123456789012123456789012345678901
ameliorate (verb) To make something better or more 2
2
12345678901234567890123456789012123456789012345678901
12345678901234567890123456789012123456789012345678901
tolerable. The living conditions of the tenants were 2
2
1
12345678901234567890123456789012123456789012345678901
certainly ameliorated when the landlord finally installed 2
2
12345678901234567890123456789012123456789012345678901
12345678901234567890123456789012123456789012345678901
washing machines and dryers in the basement. ameliora- 2
2
12345678901234567890123456789012123456789012345678901
2
12345678901234567890123456789012123456789012345678901
tion (noun).
2345678901234567890123456789012123456789012345678901
12345678901234567890123456789012123456789012345678901 2
2
12345678901234567890123456789012123456789012345678901
amortize (verb) To pay off or reduce a debt gradually 2
12345678901234567890123456789012123456789012345678901
12345678901234567890123456789012123456789012345678901
through periodic payments. If you dont need to take a 2
2
1
2345678901234567890123456789012123456789012345678901
12345678901234567890123456789012123456789012345678901
lump sum tax deduction, its best to amortize large 2
2
1
2
12345678901234567890123456789012123456789012345678901
business
expenditures
by
spreading
the
cost
out
over
12345678901234567890123456789012123456789012345678901 2
2
12345678901234567890123456789012123456789012345678901
several years.
2
12345678901234567890123456789012123456789012345678901
12345678901234567890123456789012123456789012345678901 2
12345678901234567890123456789012123456789012345678901 2
2
168 12345678901234567890123456789012123456789012345678901
123456789012345678901234567890121234567890123456789012

www.petersons.com

http://www.xtremepapers.net

Chapter 5: Verbal Review

Take It to the Next Level

123456789012345678901234567890121234567890123456789012
12345678901234567890123456789012123456789012345678901 2
2
12345678901234567890123456789012123456789012345678901
amplify (verb) To enlarge, expand, or increase. Uncertain as 2
12345678901234567890123456789012123456789012345678901
2
12345678901234567890123456789012123456789012345678901
to whether they understood, the students asked the 2
12345678901234567890123456789012123456789012345678901
2
12345678901234567890123456789012123456789012345678901
teacher to amplify his explanation. amplification (noun). 2
12345678901234567890123456789012123456789012345678901
2345678901234567890123456789012123456789012345678901
2
1
2
12345678901234567890123456789012123456789012345678901
2
12345678901234567890123456789012123456789012345678901
2
12345678901234567890123456789012123456789012345678901
Greek chronos 5 time. Also found in English chronic, chronicle,
2
12345678901234567890123456789012123456789012345678901
Word
Origin chronograph, chronology, and synchronize.
2345678901234567890123456789012123456789012345678901
2
1
2
12345678901234567890123456789012123456789012345678901
2
12345678901234567890123456789012123456789012345678901
anachronistic
(adjective)
Out
of
the
proper
time.
The
2
12345678901234567890123456789012123456789012345678901
2
12345678901234567890123456789012123456789012345678901
reference,
in
Shakespeares
Julius
Caesar,
to
the
clock
2345678901234567890123456789012123456789012345678901
2
1
2
12345678901234567890123456789012123456789012345678901
striking
twelve
is
anachronistic,
since
there
were
no
2
12345678901234567890123456789012123456789012345678901
2
12345678901234567890123456789012123456789012345678901
striking
timepieces
in
ancient
Rome.
anachronism
(noun).
2
12345678901234567890123456789012123456789012345678901
2345678901234567890123456789012123456789012345678901
2
1
12345678901234567890123456789012123456789012345678901
anarchy (noun) Absence of law or order. For several months 2
2
12345678901234567890123456789012123456789012345678901
after the Nazi government was destroyed, there was no 2
12345678901234567890123456789012123456789012345678901
2
12345678901234567890123456789012123456789012345678901
effective government in parts of Germany, and anarchy 2
12345678901234567890123456789012123456789012345678901
2
12345678901234567890123456789012123456789012345678901
ruled. anarchic (adjective).
2
12345678901234567890123456789012123456789012345678901
2
12345678901234567890123456789012123456789012345678901
2
12345678901234567890123456789012123456789012345678901
animosity
(noun)
Hostility,
resentment.
During
the
last
2345678901234567890123456789012123456789012345678901
12345678901234567890123456789012123456789012345678901 22
1
debate, the candidates could no longer disguise their 2
12345678901234567890123456789012123456789012345678901
2
12345678901234567890123456789012123456789012345678901
animosity and began to trade accusations and insults.
2
12345678901234567890123456789012123456789012345678901
2345678901234567890123456789012123456789012345678901
12345678901234567890123456789012123456789012345678901 22
12345678901234567890123456789012123456789012345678901
anomaly (noun) Something different or irregular. The tiny 2
12345678901234567890123456789012123456789012345678901
12345678901234567890123456789012123456789012345678901
planet Pluto, orbiting next to the giants Jupiter, Saturn, 2
2
12345678901234567890123456789012123456789012345678901
12345678901234567890123456789012123456789012345678901
and Neptune, has long appeared to be an anomaly. 2
2
12345678901234567890123456789012123456789012345678901
2
12345678901234567890123456789012123456789012345678901
anomalous (adjective).
12345678901234567890123456789012123456789012345678901 22
1
antagonism (noun) Hostility, conflict, opposition. As more 2
12345678901234567890123456789012123456789012345678901
2
12345678901234567890123456789012123456789012345678901
and more reporters investigated the Watergate scandal, 2
12345678901234567890123456789012123456789012345678901
2
12345678901234567890123456789012123456789012345678901
antagonism between the Nixon administration and the 2
12345678901234567890123456789012123456789012345678901
2345678901234567890123456789012123456789012345678901
12345678901234567890123456789012123456789012345678901
press increased. antagonistic (adjective), antagonize 2
2
12345678901234567890123456789012123456789012345678901
2
1
(verb).
2
12345678901234567890123456789012123456789012345678901
2
12345678901234567890123456789012123456789012345678901
2345678901234567890123456789012123456789012345678901
12345678901234567890123456789012123456789012345678901 22
12345678901234567890123456789012123456789012345678901
Greek pathos 5 suffering. Also found in English apathy, empathy,
2
1
Word
Origin pathetic, pathos, and sympathy.
2
12345678901234567890123456789012123456789012345678901
12345678901234567890123456789012123456789012345678901 2
2345678901234567890123456789012123456789012345678901
12345678901234567890123456789012123456789012345678901 22
12345678901234567890123456789012123456789012345678901
antipathy (noun) A long-held feeling of dislike or aversion. 2
12345678901234567890123456789012123456789012345678901
12345678901234567890123456789012123456789012345678901
When asked why he didnt call for help immediately 2
2
1
2345678901234567890123456789012123456789012345678901
12345678901234567890123456789012123456789012345678901
after his wife fell into a coma, the defendant emphasized 2
2
12345678901234567890123456789012123456789012345678901
2
12345678901234567890123456789012123456789012345678901
his wifes utter antipathy to doctors.
12345678901234567890123456789012123456789012345678901 22
1
apprehension (noun) A feeling of fear or foreboding; an 2
12345678901234567890123456789012123456789012345678901
2
12345678901234567890123456789012123456789012345678901
arrest. The peculiar feeling of apprehension that Harold 2
12345678901234567890123456789012123456789012345678901
12345678901234567890123456789012123456789012345678901
Pinter creates in his plays derives as much from the long 2
2
12345678901234567890123456789012123456789012345678901
2345678901234567890123456789012123456789012345678901
12345678901234567890123456789012123456789012345678901
silences between speeches as from the speeches them- 2
2
1
2
12345678901234567890123456789012123456789012345678901
selves.
The
policewomans
dramatic
apprehension
of
the
12345678901234567890123456789012123456789012345678901 2
12345678901234567890123456789012123456789012345678901
gunman took place in full view of the midtown lunch 2
2
12345678901234567890123456789012123456789012345678901
2
12345678901234567890123456789012123456789012345678901
crowd.
apprehend
(verb).
12345678901234567890123456789012123456789012345678901 22
12345678901234567890123456789012123456789012345678901 169
123456789012345678901234567890121234567890123456789012

http://www.xtremepapers.net

Part III: Section Review

123456789012345678901234567890121234567890123456789012
12345678901234567890123456789012123456789012345678901 2
2
12345678901234567890123456789012123456789012345678901
arabesque (noun) Intricate decorative patterns involving 2
12345678901234567890123456789012123456789012345678901
2
12345678901234567890123456789012123456789012345678901
intertwining lines and sometimes incorporating flowers, 2
12345678901234567890123456789012123456789012345678901
2
12345678901234567890123456789012123456789012345678901
animals, and fruits. Borders of gold and fanciful 2
12345678901234567890123456789012123456789012345678901
2345678901234567890123456789012123456789012345678901
2
1
arabesques surround the Arabic script on every page of 2
12345678901234567890123456789012123456789012345678901
2
12345678901234567890123456789012123456789012345678901
this ancient edition of the Koran.
2
12345678901234567890123456789012123456789012345678901
2
12345678901234567890123456789012123456789012345678901
2345678901234567890123456789012123456789012345678901
2
1
2
12345678901234567890123456789012123456789012345678901
2345678901234567890123456789012123456789012345678901
2
1
Latin arbiter 5 judge. Also found in English arbiter, arbitrage, and
Word
Origin arbitrate.
2
12345678901234567890123456789012123456789012345678901
2
12345678901234567890123456789012123456789012345678901
2345678901234567890123456789012123456789012345678901
2
1
2
12345678901234567890123456789012123456789012345678901
arbitrary
(adjective)
Based
on
random
or
merely
personal
2
12345678901234567890123456789012123456789012345678901
2
12345678901234567890123456789012123456789012345678901
preference.
Both
computers
cost
the
same
and
had
the
2
12345678901234567890123456789012123456789012345678901
2345678901234567890123456789012123456789012345678901
1
same features, so in the end I made an arbitrary decision 2
2
12345678901234567890123456789012123456789012345678901
2
12345678901234567890123456789012123456789012345678901
about which one to buy.
2
12345678901234567890123456789012123456789012345678901
2
12345678901234567890123456789012123456789012345678901
archaic
(adjective)
Old-fashioned,
obsolete.
Those
who
2345678901234567890123456789012123456789012345678901
12345678901234567890123456789012123456789012345678901 2
2
12345678901234567890123456789012123456789012345678901
believe in open marriage often declare that they will 2
12345678901234567890123456789012123456789012345678901
2
12345678901234567890123456789012123456789012345678901
not be bound by archaic laws and religious rituals, but 2
12345678901234567890123456789012123456789012345678901
12345678901234567890123456789012123456789012345678901
state instead that love alone should bring two people 2
2
1
2
12345678901234567890123456789012123456789012345678901
together.
archaism
(noun).
2
12345678901234567890123456789012123456789012345678901
2
12345678901234567890123456789012123456789012345678901
2345678901234567890123456789012123456789012345678901
12345678901234567890123456789012123456789012345678901
ardor (noun) A strong feeling of passion, energy, or zeal. 2
2
12345678901234567890123456789012123456789012345678901
2
12345678901234567890123456789012123456789012345678901
The
young
revolutionary
proclaimed
his
convictions
12345678901234567890123456789012123456789012345678901 2
12345678901234567890123456789012123456789012345678901
with an ardor that excited the crowd. ardent (adjective). 2
2
12345678901234567890123456789012123456789012345678901
12345678901234567890123456789012123456789012345678901 2
12345678901234567890123456789012123456789012345678901
arid (adjective) Very dry; boring and meaningless. The arid 2
2
12345678901234567890123456789012123456789012345678901
1
climate of Arizona makes farming difficult. Some find 2
2345678901234567890123456789012123456789012345678901
12345678901234567890123456789012123456789012345678901 2
the law a fascinating topic, but for me it is an arid 2
12345678901234567890123456789012123456789012345678901
2
1
discipline. aridity (noun).
2
12345678901234567890123456789012123456789012345678901
2
12345678901234567890123456789012123456789012345678901
2345678901234567890123456789012123456789012345678901
12345678901234567890123456789012123456789012345678901 2
12345678901234567890123456789012123456789012345678901 2
2
1
Latin articulus I 5 joint, division. Also found in English arthritis,
2
12345678901234567890123456789012123456789012345678901
Word
Origin article, and inarticulate.
2
12345678901234567890123456789012123456789012345678901
2345678901234567890123456789012123456789012345678901
12345678901234567890123456789012123456789012345678901 2
12345678901234567890123456789012123456789012345678901
articulate (adjective) To express oneself clearly and effec- 2
2
1
2
12345678901234567890123456789012123456789012345678901
tively.
Compared
to
George
Bush,
with
his
stammering
12345678901234567890123456789012123456789012345678901 2
2345678901234567890123456789012123456789012345678901
12345678901234567890123456789012123456789012345678901
and his frequently incomplete sentences, Bill Clinton 2
2
12345678901234567890123456789012123456789012345678901
2
12345678901234567890123456789012123456789012345678901
was
considered
a
highly
articulate
president.
12345678901234567890123456789012123456789012345678901 2
2
1
12345678901234567890123456789012123456789012345678901
asperity (noun) Harshness, severity. Total silence at the 2
2
12345678901234567890123456789012123456789012345678901
12345678901234567890123456789012123456789012345678901
dinner table, baths in icy water, prayers five times a 2
2
12345678901234567890123456789012123456789012345678901
daythese practices all contributed to the asperity of 2
12345678901234567890123456789012123456789012345678901
2345678901234567890123456789012123456789012345678901
2
12345678901234567890123456789012123456789012345678901
life in the monastery.
2
12345678901234567890123456789012123456789012345678901
12345678901234567890123456789012123456789012345678901 2
12345678901234567890123456789012123456789012345678901
assail (verb) To attack with blows or words. When the 2
2
1
2345678901234567890123456789012123456789012345678901
12345678901234567890123456789012123456789012345678901
presidents cabinet members rose to justify the case for 2
2
1
2
12345678901234567890123456789012123456789012345678901
military
intervention
in
Iraq,
they
were
assailed
by
many
12345678901234567890123456789012123456789012345678901 2
12345678901234567890123456789012123456789012345678901
audience members who were critical of U.S. policy. 2
2
12345678901234567890123456789012123456789012345678901
2
12345678901234567890123456789012123456789012345678901
assailant
(noun).
12345678901234567890123456789012123456789012345678901 2
2
170 12345678901234567890123456789012123456789012345678901
123456789012345678901234567890121234567890123456789012

www.petersons.com

http://www.xtremepapers.net

Chapter 5: Verbal Review

Take It to the Next Level

123456789012345678901234567890121234567890123456789012
12345678901234567890123456789012123456789012345678901 2
2
12345678901234567890123456789012123456789012345678901
assay (verb) To analyze for particular components; to 2
12345678901234567890123456789012123456789012345678901
2
12345678901234567890123456789012123456789012345678901
determine weight, quality etc. The jeweler assayed the 2
12345678901234567890123456789012123456789012345678901
2
12345678901234567890123456789012123456789012345678901
stone pendant Gwyneth inherited from her mother and 2
12345678901234567890123456789012123456789012345678901
2345678901234567890123456789012123456789012345678901
2
1
found it to contain a topaz of high quality.
2
12345678901234567890123456789012123456789012345678901
2
12345678901234567890123456789012123456789012345678901
2
12345678901234567890123456789012123456789012345678901
assimilate
(verb)
To
absorb
into
a
system
or
culture.
New
2
12345678901234567890123456789012123456789012345678901
2345678901234567890123456789012123456789012345678901
1
York City has assimilated one group of immigrants after 2
2
12345678901234567890123456789012123456789012345678901
2
12345678901234567890123456789012123456789012345678901
another,
from
the
Jewish,
German,
and
Irish
immigrants
2
12345678901234567890123456789012123456789012345678901
2
12345678901234567890123456789012123456789012345678901
who
arrived
at
the
turn
of
the
last
century
to
the
waves
2345678901234567890123456789012123456789012345678901
2
1
2
12345678901234567890123456789012123456789012345678901
of
Mexican
and
Latin
American
immigrants
who
2
12345678901234567890123456789012123456789012345678901
2
12345678901234567890123456789012123456789012345678901
arrived
in
the
1980s.
assimilated
(adjective).
2
12345678901234567890123456789012123456789012345678901
2345678901234567890123456789012123456789012345678901
2
1
12345678901234567890123456789012123456789012345678901
assuage (verb) To ease, to pacify. Knowing that the pilots 2
2
12345678901234567890123456789012123456789012345678901
record was perfect did little to assuage Linnets fear of 2
12345678901234567890123456789012123456789012345678901
2
12345678901234567890123456789012123456789012345678901
flying in the two-seater airplane.
2
12345678901234567890123456789012123456789012345678901
2
12345678901234567890123456789012123456789012345678901
2
12345678901234567890123456789012123456789012345678901
audacious
(adjective)
Bold,
daring,
adventurous.
Her
plan
2
12345678901234567890123456789012123456789012345678901
2
12345678901234567890123456789012123456789012345678901
to
cross
the
Atlantic
single-handed
in
a
twelve-foot
2345678901234567890123456789012123456789012345678901
12345678901234567890123456789012123456789012345678901 22
1
sailboat was an audacious, if not reckless, one. audacity 2
12345678901234567890123456789012123456789012345678901
2
12345678901234567890123456789012123456789012345678901
(noun).
2
12345678901234567890123456789012123456789012345678901
2345678901234567890123456789012123456789012345678901
12345678901234567890123456789012123456789012345678901 22
12345678901234567890123456789012123456789012345678901
authoritarian (adjective) Favoring or demanding blind obe- 2
12345678901234567890123456789012123456789012345678901
12345678901234567890123456789012123456789012345678901
dience to leaders. Despite most Americans strong belief 2
2
12345678901234567890123456789012123456789012345678901
12345678901234567890123456789012123456789012345678901
in democracy, the American government has sometimes 2
2
12345678901234567890123456789012123456789012345678901
12345678901234567890123456789012123456789012345678901
supported authoritarian regimes in other countries. 2
12345678901234567890123456789012123456789012345678901 22
1
authoritarianism (noun).
2
12345678901234567890123456789012123456789012345678901
2
12345678901234567890123456789012123456789012345678901
authoritative (adjective) Official, conclusive. For over five 2
12345678901234567890123456789012123456789012345678901
2
12345678901234567890123456789012123456789012345678901
decades, American parents regarded Doctor Benjamin 2
12345678901234567890123456789012123456789012345678901
2345678901234567890123456789012123456789012345678901
12345678901234567890123456789012123456789012345678901
Spock as the most authoritative voice on baby and child 2
2
12345678901234567890123456789012123456789012345678901
2
1
care.
authority
(noun),
authorize
(verb).
2
12345678901234567890123456789012123456789012345678901
2
12345678901234567890123456789012123456789012345678901
2345678901234567890123456789012123456789012345678901
12345678901234567890123456789012123456789012345678901
avenge (verb) To exact a punishment for or on behalf of 2
2
12345678901234567890123456789012123456789012345678901
2
1
someone.
In
Shakespeares
tragedy
Hamlet,
the
ghost
of
12345678901234567890123456789012123456789012345678901 2
12345678901234567890123456789012123456789012345678901
the dead king of Denmark visits his son, Prince Hamlet, 2
2
12345678901234567890123456789012123456789012345678901
2
12345678901234567890123456789012123456789012345678901
and
urges
him
to
avenge
his
murder.
2
12345678901234567890123456789012123456789012345678901
2
12345678901234567890123456789012123456789012345678901
12345678901234567890123456789012123456789012345678901
aver (verb) To claim to be true; to avouch. The fact that the 2
2345678901234567890123456789012123456789012345678901
2
12345678901234567890123456789012123456789012345678901
key witness averred the defendants innocence is what 2
12345678901234567890123456789012123456789012345678901
2
12345678901234567890123456789012123456789012345678901
ultimately swayed the jury to deliver a not guilty 2
12345678901234567890123456789012123456789012345678901
2
1
verdict.
2
12345678901234567890123456789012123456789012345678901
2
12345678901234567890123456789012123456789012345678901
2
12345678901234567890123456789012123456789012345678901
avow
(verb)
To
declare
boldly.
Immediately
after
Cyrus
2
12345678901234567890123456789012123456789012345678901
2
12345678901234567890123456789012123456789012345678901
avowed
his
atheism
at
our
church
fund-raiser,
there
was
2345678901234567890123456789012123456789012345678901
12345678901234567890123456789012123456789012345678901 22
1
a long, uncomfortable silence. avowal (noun), avowed 2
12345678901234567890123456789012123456789012345678901
2
12345678901234567890123456789012123456789012345678901
(adjective).
2
12345678901234567890123456789012123456789012345678901
2345678901234567890123456789012123456789012345678901
2
1
12345678901234567890123456789012123456789012345678901 2
12345678901234567890123456789012123456789012345678901 2
12345678901234567890123456789012123456789012345678901 2 171
123456789012345678901234567890121234567890123456789012

http://www.xtremepapers.net

Part III: Section Review

123456789012345678901234567890121234567890123456789012
12345678901234567890123456789012123456789012345678901 2
2
12345678901234567890123456789012123456789012345678901
barren (adjective) Desolate; infertile. The subarctic tundra 2
12345678901234567890123456789012123456789012345678901
2
12345678901234567890123456789012123456789012345678901
is a barren wasteland inhabited only by lichens and 2
12345678901234567890123456789012123456789012345678901
2
12345678901234567890123456789012123456789012345678901
mosses. Women who try to conceive in their 40s are 2
12345678901234567890123456789012123456789012345678901
2345678901234567890123456789012123456789012345678901
2
1
often barren and must turn to artificial means of 2
12345678901234567890123456789012123456789012345678901
2
12345678901234567890123456789012123456789012345678901
producing a child.
2
12345678901234567890123456789012123456789012345678901
2
12345678901234567890123456789012123456789012345678901
2345678901234567890123456789012123456789012345678901
1
belligerent (adjective) Quarrelsome, combative. Mrs. Juni- 2
2
12345678901234567890123456789012123456789012345678901
2
12345678901234567890123456789012123456789012345678901
per
was
so
belligerent
toward
the
clerks
at
the
local
2
12345678901234567890123456789012123456789012345678901
2
12345678901234567890123456789012123456789012345678901
stores
that
they
cringed
when
they
saw
her
coming.
2345678901234567890123456789012123456789012345678901
2
1
2
12345678901234567890123456789012123456789012345678901
12345678901234567890123456789012123456789012345678901
belligerent (noun) An opposing army, a party waging war. 2
2
12345678901234567890123456789012123456789012345678901
12345678901234567890123456789012123456789012345678901
The Union and Confederate forces were the belligerents 2
2345678901234567890123456789012123456789012345678901
2
1
2
12345678901234567890123456789012123456789012345678901
in the American Civil War.
2
12345678901234567890123456789012123456789012345678901
2
12345678901234567890123456789012123456789012345678901
2
12345678901234567890123456789012123456789012345678901
2345678901234567890123456789012123456789012345678901
2
12345678901234567890123456789012123456789012345678901
Latin bene 5 well. Also found in English benediction, benefactor,
2
1
Word
Origin
beneficent, beneficial, benefit, and benign.
2
12345678901234567890123456789012123456789012345678901
2
12345678901234567890123456789012123456789012345678901
2
12345678901234567890123456789012123456789012345678901
benevolent
(adjective)
Wishing
or
doing
good.
In
old
age,
2345678901234567890123456789012123456789012345678901
12345678901234567890123456789012123456789012345678901 2
1
Carnegie used his wealth for benevolent purposes, 2
2
12345678901234567890123456789012123456789012345678901
2
12345678901234567890123456789012123456789012345678901
donating
large
sums
to
found
libraries
and
schools
2
12345678901234567890123456789012123456789012345678901
2345678901234567890123456789012123456789012345678901
2
12345678901234567890123456789012123456789012345678901
around the country. benevolence (noun).
2
12345678901234567890123456789012123456789012345678901
12345678901234567890123456789012123456789012345678901 2
12345678901234567890123456789012123456789012345678901
berate (verb) To scold or criticize harshly. The judge angrily 2
2
12345678901234567890123456789012123456789012345678901
12345678901234567890123456789012123456789012345678901
berated the two lawyers for their childish and unprofes- 2
2
12345678901234567890123456789012123456789012345678901
2
12345678901234567890123456789012123456789012345678901
sional behavior.
12345678901234567890123456789012123456789012345678901 2
2
1
boggle (verb) To overwhelm with amazement. The ability of 2
12345678901234567890123456789012123456789012345678901
2
12345678901234567890123456789012123456789012345678901
physicists to isolate the most infinitesimal particles of 2
12345678901234567890123456789012123456789012345678901
2
12345678901234567890123456789012123456789012345678901
matter truly boggles the mind.
2
12345678901234567890123456789012123456789012345678901
2345678901234567890123456789012123456789012345678901
12345678901234567890123456789012123456789012345678901 2
12345678901234567890123456789012123456789012345678901
bogus (adjective) Phony, a sham. Senior citizens are often 2
2
1
2
12345678901234567890123456789012123456789012345678901
the
target
of
telemarketing
scams
pushing
bogus
invest2
12345678901234567890123456789012123456789012345678901
2345678901234567890123456789012123456789012345678901
2
12345678901234567890123456789012123456789012345678901
ment opportunities.
2
12345678901234567890123456789012123456789012345678901
2
1
12345678901234567890123456789012123456789012345678901
bombastic (adjective) Inflated or pompous in style. Old- 2
12345678901234567890123456789012123456789012345678901 2
12345678901234567890123456789012123456789012345678901
fashioned bombastic political speeches dont work on 2
2
12345678901234567890123456789012123456789012345678901
12345678901234567890123456789012123456789012345678901
television, which demands a more intimate, personal 2
2
12345678901234567890123456789012123456789012345678901
2
12345678901234567890123456789012123456789012345678901
style of communication. bombast (noun).
2345678901234567890123456789012123456789012345678901
12345678901234567890123456789012123456789012345678901 2
2
12345678901234567890123456789012123456789012345678901
boor (noun) Crude, insensitive and overbearing. Harold 2
12345678901234567890123456789012123456789012345678901
12345678901234567890123456789012123456789012345678901
was well-known to be a boor; at parties he horrified 2
2
1
2345678901234567890123456789012123456789012345678901
12345678901234567890123456789012123456789012345678901
people with stories of his past sexual exploits and old, 2
2
12345678901234567890123456789012123456789012345678901
2
12345678901234567890123456789012123456789012345678901
off-color
jokes.
boorish
(adjective).
12345678901234567890123456789012123456789012345678901 2
2
1
12345678901234567890123456789012123456789012345678901
brazenly (adverb) Acting with disrespectful boldness. Some 2
2
12345678901234567890123456789012123456789012345678901
2
12345678901234567890123456789012123456789012345678901
say
that
the
former
White
House
intern
brazenly
threw
12345678901234567890123456789012123456789012345678901 2
12345678901234567890123456789012123456789012345678901
herself at the president, but the American public will 2
2
12345678901234567890123456789012123456789012345678901
2
12345678901234567890123456789012123456789012345678901
probably never know the full truth. brazen (adjective).
12345678901234567890123456789012123456789012345678901 2
2
172 12345678901234567890123456789012123456789012345678901
123456789012345678901234567890121234567890123456789012

www.petersons.com

http://www.xtremepapers.net

Chapter 5: Verbal Review

Take It to the Next Level

123456789012345678901234567890121234567890123456789012
12345678901234567890123456789012123456789012345678901 2
2
12345678901234567890123456789012123456789012345678901
broach (verb) To bring up an issue for discussion, to 2
12345678901234567890123456789012123456789012345678901
2
12345678901234567890123456789012123456789012345678901
propose. Knowing my fathers strictness about adhering 2
12345678901234567890123456789012123456789012345678901
2
12345678901234567890123456789012123456789012345678901
to a budget, I just cant seem to broach the subject of my 2
12345678901234567890123456789012123456789012345678901
2345678901234567890123456789012123456789012345678901
2
1
massive credit-card debt.
2
12345678901234567890123456789012123456789012345678901
2
12345678901234567890123456789012123456789012345678901
2
12345678901234567890123456789012123456789012345678901
burgeon
(verb)
To
bloom,
literally
or
figuratively.
Due
to
2
12345678901234567890123456789012123456789012345678901
2345678901234567890123456789012123456789012345678901
1
the extremely mild winter, the forsythia burgeoned as 2
2
12345678901234567890123456789012123456789012345678901
2
12345678901234567890123456789012123456789012345678901
early
as
March.
The
story
of
two
prison
inmates
in
2
12345678901234567890123456789012123456789012345678901
2
12345678901234567890123456789012123456789012345678901
Manuel
Puigs
play
The
Kiss
of
The
Spiderwoman
is
2345678901234567890123456789012123456789012345678901
2
1
2
12345678901234567890123456789012123456789012345678901
testimony
that
tenderness
can
burgeon
in
the
most
2
12345678901234567890123456789012123456789012345678901
2
12345678901234567890123456789012123456789012345678901
unlikely
places.
2
12345678901234567890123456789012123456789012345678901
2345678901234567890123456789012123456789012345678901
2
1
12345678901234567890123456789012123456789012345678901
burnish (verb) To shine by polishing, literally or figura- 2
2
12345678901234567890123456789012123456789012345678901
tively. After stripping seven layers of old paint off the 2
12345678901234567890123456789012123456789012345678901
2
12345678901234567890123456789012123456789012345678901
antique door, the carpenter stained the wood and 2
12345678901234567890123456789012123456789012345678901
2
12345678901234567890123456789012123456789012345678901
burnished it to a rich hue. When Bill Gates, the 2
12345678901234567890123456789012123456789012345678901
2
12345678901234567890123456789012123456789012345678901
wealthiest man in the country, decided to endorse the 2
12345678901234567890123456789012123456789012345678901
2345678901234567890123456789012123456789012345678901
2
12345678901234567890123456789012123456789012345678901
Big Bertha line of Golf Clubs, many suggested that he 2
1
2
12345678901234567890123456789012123456789012345678901
was trying to burnish his image as a regular guy.
2
12345678901234567890123456789012123456789012345678901
2
12345678901234567890123456789012123456789012345678901
2345678901234567890123456789012123456789012345678901
12345678901234567890123456789012123456789012345678901
buttress (noun) Something that supports or strengthens. 2
2
12345678901234567890123456789012123456789012345678901
2
12345678901234567890123456789012123456789012345678901
The
endorsement
of
the
American
Medical
Association
12345678901234567890123456789012123456789012345678901 22
12345678901234567890123456789012123456789012345678901
is a powerful buttress for the claims made on behalf of 2
12345678901234567890123456789012123456789012345678901
2
12345678901234567890123456789012123456789012345678901
this new medicine. buttress (verb).
2
12345678901234567890123456789012123456789012345678901
12345678901234567890123456789012123456789012345678901 22
1
cacophony (noun) Discordant sounds; dissonance. In the
2
12345678901234567890123456789012123456789012345678901
minutes before classes start, the high schools halls are 2
12345678901234567890123456789012123456789012345678901
2
12345678901234567890123456789012123456789012345678901
filled with a cacophony of shrieks, shouts, banging 2
12345678901234567890123456789012123456789012345678901
2
12345678901234567890123456789012123456789012345678901
locker doors, and pounding feet. cacophonous (adjec- 2
12345678901234567890123456789012123456789012345678901
2
12345678901234567890123456789012123456789012345678901
tive).
2
12345678901234567890123456789012123456789012345678901
2
12345678901234567890123456789012123456789012345678901
2
12345678901234567890123456789012123456789012345678901
cadge
(verb)
To
beg
for,
to
sponge.
Few
in
our
crowd
want
2345678901234567890123456789012123456789012345678901
12345678901234567890123456789012123456789012345678901 22
12345678901234567890123456789012123456789012345678901
to go out on the town with Piper, since he routinely 2
1
2
12345678901234567890123456789012123456789012345678901
cadges cigarettes, subway tokens, and drinks.
2
12345678901234567890123456789012123456789012345678901
2345678901234567890123456789012123456789012345678901
12345678901234567890123456789012123456789012345678901 22
12345678901234567890123456789012123456789012345678901
calibrate (verb) To determine or mark graduations (of a 2
12345678901234567890123456789012123456789012345678901
12345678901234567890123456789012123456789012345678901
measuring instrument); to adjust or finely tune. We tried 2
2
1
2345678901234567890123456789012123456789012345678901
12345678901234567890123456789012123456789012345678901
to calibrate the heating to Rufuss liking, but he still 2
2
12345678901234567890123456789012123456789012345678901
12345678901234567890123456789012123456789012345678901
ended up shivering in our living room. calibration 2
12345678901234567890123456789012123456789012345678901 22
1
(noun).
2
12345678901234567890123456789012123456789012345678901
2
12345678901234567890123456789012123456789012345678901
castigate (verb) To chastise; to punish severely. The editor 2
12345678901234567890123456789012123456789012345678901
12345678901234567890123456789012123456789012345678901
castigated Bob for repeatedly failing to meet his 2
2
12345678901234567890123456789012123456789012345678901
2345678901234567890123456789012123456789012345678901
2
12345678901234567890123456789012123456789012345678901
deadlines. castigation (noun).
2
1
2
12345678901234567890123456789012123456789012345678901
12345678901234567890123456789012123456789012345678901 2
2
12345678901234567890123456789012123456789012345678901
12345678901234567890123456789012123456789012345678901 2
12345678901234567890123456789012123456789012345678901 2
12345678901234567890123456789012123456789012345678901 2
12345678901234567890123456789012123456789012345678901 2 173
123456789012345678901234567890121234567890123456789012

http://www.xtremepapers.net

Part III: Section Review

123456789012345678901234567890121234567890123456789012
12345678901234567890123456789012123456789012345678901 2
2
12345678901234567890123456789012123456789012345678901
catalytic (adjective) Bringing about, causing, or producing 2
12345678901234567890123456789012123456789012345678901
2
12345678901234567890123456789012123456789012345678901
some result. The conditions for revolution existed in 2
12345678901234567890123456789012123456789012345678901
2
12345678901234567890123456789012123456789012345678901
America by 1765; the disputes about taxation that arose 2
12345678901234567890123456789012123456789012345678901
2345678901234567890123456789012123456789012345678901
2
1
during the following decade were the catalytic events 2
12345678901234567890123456789012123456789012345678901
2
12345678901234567890123456789012123456789012345678901
that sparked the rebellion. catalyze (verb).
2
12345678901234567890123456789012123456789012345678901
2
12345678901234567890123456789012123456789012345678901
2345678901234567890123456789012123456789012345678901
2
1
2
12345678901234567890123456789012123456789012345678901
2
12345678901234567890123456789012123456789012345678901
Greek kaustikos 5 burning. Also found in English holocaust.
Word
Origin
2
12345678901234567890123456789012123456789012345678901
2
12345678901234567890123456789012123456789012345678901
2345678901234567890123456789012123456789012345678901
1
caustic (adjective) Burning, corrosive. No pretensions were 2
2
12345678901234567890123456789012123456789012345678901
12345678901234567890123456789012123456789012345678901
safe when the famous satirist H. L. Mencken unleashed 2
2
12345678901234567890123456789012123456789012345678901
his caustic wit.
2
12345678901234567890123456789012123456789012345678901
2345678901234567890123456789012123456789012345678901
2
1
12345678901234567890123456789012123456789012345678901
chaos (noun) Disorder, confusion, chance. The first few 2
2
12345678901234567890123456789012123456789012345678901
2
12345678901234567890123456789012123456789012345678901
moments
after
the
explosion
were
pure
chaos:
No
one
2
12345678901234567890123456789012123456789012345678901
2345678901234567890123456789012123456789012345678901
12345678901234567890123456789012123456789012345678901
was sure what had happened, and the area was filled 2
2
12345678901234567890123456789012123456789012345678901
2
12345678901234567890123456789012123456789012345678901
with
people
running
and
yelling.
chaotic
(adjective).
12345678901234567890123456789012123456789012345678901 2
12345678901234567890123456789012123456789012345678901 2
12345678901234567890123456789012123456789012345678901
charisma (noun) Dynamic charm or appeal. Eva Peron was 2
2
1
2
12345678901234567890123456789012123456789012345678901
such
a
fiery
orator
and
had
so
much
charisma
that
she
2
12345678901234567890123456789012123456789012345678901
12345678901234567890123456789012123456789012345678901
commanded an enormous political following. charis- 2
2345678901234567890123456789012123456789012345678901
12345678901234567890123456789012123456789012345678901 2
2
12345678901234567890123456789012123456789012345678901
matic (adjective).
2
12345678901234567890123456789012123456789012345678901
2
12345678901234567890123456789012123456789012345678901
chary (adjective) Slow to accept, cautious. Yuan was chary 2
12345678901234567890123456789012123456789012345678901
2
12345678901234567890123456789012123456789012345678901
about going out with Xinhua, since she had been badly 2
12345678901234567890123456789012123456789012345678901
2
12345678901234567890123456789012123456789012345678901
hurt in her previous relationship.
2
12345678901234567890123456789012123456789012345678901
2
1
2345678901234567890123456789012123456789012345678901
12345678901234567890123456789012123456789012345678901
chronology (noun) An arrangement of events by order of 2
2
12345678901234567890123456789012123456789012345678901
2
1
occurrence,
a
list
of
dates;
the
science
of
time.
If
you
ask
12345678901234567890123456789012123456789012345678901 2
12345678901234567890123456789012123456789012345678901
Susan about her 2-year-old son, she will give you a 2
2
12345678901234567890123456789012123456789012345678901
2
12345678901234567890123456789012123456789012345678901
chronology
of
his
accomplishments
and
childhood
12345678901234567890123456789012123456789012345678901 2
12345678901234567890123456789012123456789012345678901
illnesses, from the day he was born to the present. The 2
2
12345678901234567890123456789012123456789012345678901
2345678901234567890123456789012123456789012345678901
12345678901234567890123456789012123456789012345678901
village of Copan was where Mayan astronomical 2
2
12345678901234567890123456789012123456789012345678901
1
learning, as applied to chronology, achieved its most 2
12345678901234567890123456789012123456789012345678901 2
12345678901234567890123456789012123456789012345678901
accurate expression in the famous Mayan calendar. 2
2
12345678901234567890123456789012123456789012345678901
2
12345678901234567890123456789012123456789012345678901
chronological (adjective).
2
12345678901234567890123456789012123456789012345678901
2
12345678901234567890123456789012123456789012345678901
churlish
(adjective)
Coarse
and
ill-mannered.
Few
journal12345678901234567890123456789012123456789012345678901 2
2345678901234567890123456789012123456789012345678901
2
12345678901234567890123456789012123456789012345678901
ists were eager to interview the aging film star, since he 2
12345678901234567890123456789012123456789012345678901
2
12345678901234567890123456789012123456789012345678901
was reputed to be a churlish, uncooperative subject. 2
12345678901234567890123456789012123456789012345678901
2
1
churl (noun).
2
12345678901234567890123456789012123456789012345678901
2
12345678901234567890123456789012123456789012345678901
2
12345678901234567890123456789012123456789012345678901
2
12345678901234567890123456789012123456789012345678901
12345678901234567890123456789012123456789012345678901 2
2345678901234567890123456789012123456789012345678901
12345678901234567890123456789012123456789012345678901 2
2
1
2
12345678901234567890123456789012123456789012345678901
12345678901234567890123456789012123456789012345678901 2
2
12345678901234567890123456789012123456789012345678901
12345678901234567890123456789012123456789012345678901 2
12345678901234567890123456789012123456789012345678901 2
12345678901234567890123456789012123456789012345678901 2
2
174 12345678901234567890123456789012123456789012345678901
123456789012345678901234567890121234567890123456789012

www.petersons.com

http://www.xtremepapers.net

Chapter 5: Verbal Review

Take It to the Next Level

123456789012345678901234567890121234567890123456789012
12345678901234567890123456789012123456789012345678901 2
2
12345678901234567890123456789012123456789012345678901
2
12345678901234567890123456789012123456789012345678901
Latin circus 5 circle. Also found in English circumference,
2345678901234567890123456789012123456789012345678901
2
1
Word
Origin
2
12345678901234567890123456789012123456789012345678901
circumnavigate, circumscribe, and circumvent.
12345678901234567890123456789012123456789012345678901 2
2
12345678901234567890123456789012123456789012345678901
circumspect (adjective) Prudent, cautious. After he had 2
12345678901234567890123456789012123456789012345678901
2
12345678901234567890123456789012123456789012345678901
been acquitted of the sexual harassment charge, the 2
12345678901234567890123456789012123456789012345678901
2
12345678901234567890123456789012123456789012345678901
sergeant realized he would have to be more circumspect 2
12345678901234567890123456789012123456789012345678901
2345678901234567890123456789012123456789012345678901
2
1
in his dealings with the female cadets. circumspection 2
12345678901234567890123456789012123456789012345678901
2
12345678901234567890123456789012123456789012345678901
(noun).
2
12345678901234567890123456789012123456789012345678901
2
12345678901234567890123456789012123456789012345678901
2345678901234567890123456789012123456789012345678901
1
cleave (verb) NOTE: A tricky verb that can mean either to 2
2
12345678901234567890123456789012123456789012345678901
2
12345678901234567890123456789012123456789012345678901
stick
closely
together
or
to
split
apart.
(Pay
attention
to
2
12345678901234567890123456789012123456789012345678901
2
12345678901234567890123456789012123456789012345678901
context.)
The
more
abusive
his
father
became,
the
more
2345678901234567890123456789012123456789012345678901
2
1
2
12345678901234567890123456789012123456789012345678901
Timothy
cleaved
to
his
mother
and
refused
to
let
her
out
2
12345678901234567890123456789012123456789012345678901
2
12345678901234567890123456789012123456789012345678901
of
his
sight.
Sometimes
a
few
words
carelessly
spoken
2
12345678901234567890123456789012123456789012345678901
2345678901234567890123456789012123456789012345678901
12345678901234567890123456789012123456789012345678901
are enough to cleave a married couple and leave the 2
2
12345678901234567890123456789012123456789012345678901
2
12345678901234567890123456789012123456789012345678901
relationship in shambles. cleavage (noun).
12345678901234567890123456789012123456789012345678901 22
12345678901234567890123456789012123456789012345678901
coagulant (noun) Any material that causes another to 2
12345678901234567890123456789012123456789012345678901
2
1
thicken or clot. Hemophilia is characterized by excessive 2
12345678901234567890123456789012123456789012345678901
2
12345678901234567890123456789012123456789012345678901
bleeding from even the slightest cut and is caused by a 2
12345678901234567890123456789012123456789012345678901
2345678901234567890123456789012123456789012345678901
12345678901234567890123456789012123456789012345678901
lack of one of the coagulants necessary for blood 2
2
12345678901234567890123456789012123456789012345678901
2
12345678901234567890123456789012123456789012345678901
clotting.
coagulate
(verb).
12345678901234567890123456789012123456789012345678901 22
12345678901234567890123456789012123456789012345678901 2
12345678901234567890123456789012123456789012345678901
coalesce (verb) To fuse, to unite. The music we know as jazz 2
12345678901234567890123456789012123456789012345678901
12345678901234567890123456789012123456789012345678901
coalesced from diverse elements from many musical 2
2
12345678901234567890123456789012123456789012345678901
2
1
cultures,
including
those
of
West
Africa,
America,
and
2345678901234567890123456789012123456789012345678901
12345678901234567890123456789012123456789012345678901 22
12345678901234567890123456789012123456789012345678901
Europe. coalescence (noun).
2
1
12345678901234567890123456789012123456789012345678901 2
12345678901234567890123456789012123456789012345678901
coerce (verb) To force someone either to do something or to 2
2
12345678901234567890123456789012123456789012345678901
refrain from doing something. The Miranda ruling 2
12345678901234567890123456789012123456789012345678901
2
12345678901234567890123456789012123456789012345678901
prevents police from coercing a confession by forcing 2
12345678901234567890123456789012123456789012345678901
2
12345678901234567890123456789012123456789012345678901
them to read criminals their rights. coercion (noun).
2
12345678901234567890123456789012123456789012345678901
2
12345678901234567890123456789012123456789012345678901
2
12345678901234567890123456789012123456789012345678901
cogent
(adjective)
Forceful
and
convincing.
The
committee
12345678901234567890123456789012123456789012345678901 2
12345678901234567890123456789012123456789012345678901
members were won over to the project by the cogent 2
2
12345678901234567890123456789012123456789012345678901
2
12345678901234567890123456789012123456789012345678901
arguments
of
the
chairman.
cogency
(noun).
2
12345678901234567890123456789012123456789012345678901
2
12345678901234567890123456789012123456789012345678901
12345678901234567890123456789012123456789012345678901 2
2345678901234567890123456789012123456789012345678901
2
12345678901234567890123456789012123456789012345678901
Latin mensura 5 to measure. Also found in English measure,
2
1
Word
Origin immeasurable, immense, and mensuration.
2
12345678901234567890123456789012123456789012345678901
2
12345678901234567890123456789012123456789012345678901
12345678901234567890123456789012123456789012345678901 2
2345678901234567890123456789012123456789012345678901
12345678901234567890123456789012123456789012345678901
commensurate (adjective) Aligned with, proportional. Many 2
2
12345678901234567890123456789012123456789012345678901
12345678901234567890123456789012123456789012345678901
Ph.D.s in the humanities do not feel their paltry salaries 2
2
12345678901234567890123456789012123456789012345678901
are commensurate with their abilities, their experience, 2
1
2345678901234567890123456789012123456789012345678901
2
12345678901234567890123456789012123456789012345678901
or the heavy workload they are asked to bear.
2
1
2
12345678901234567890123456789012123456789012345678901
12345678901234567890123456789012123456789012345678901 2
2
12345678901234567890123456789012123456789012345678901
12345678901234567890123456789012123456789012345678901 2
12345678901234567890123456789012123456789012345678901 2
12345678901234567890123456789012123456789012345678901 2
12345678901234567890123456789012123456789012345678901 2 175
123456789012345678901234567890121234567890123456789012

http://www.xtremepapers.net

Part III: Section Review

123456789012345678901234567890121234567890123456789012
12345678901234567890123456789012123456789012345678901 2
2
12345678901234567890123456789012123456789012345678901
commingle (verb) To blend, to mix. Just as he had when he 2
12345678901234567890123456789012123456789012345678901
2
12345678901234567890123456789012123456789012345678901
was only five years old, Elmer did not allow any of the 2
12345678901234567890123456789012123456789012345678901
2
12345678901234567890123456789012123456789012345678901
foods on his plate to commingle: The beans must not 2
12345678901234567890123456789012123456789012345678901
2345678901234567890123456789012123456789012345678901
2
1
merge with the rice nor the chicken rub shoulders with 2
12345678901234567890123456789012123456789012345678901
2
12345678901234567890123456789012123456789012345678901
the broccoli!
2
12345678901234567890123456789012123456789012345678901
2
12345678901234567890123456789012123456789012345678901
2345678901234567890123456789012123456789012345678901
1
complaisant (adjective) Tending to bow to others wishes; 2
2
12345678901234567890123456789012123456789012345678901
2
12345678901234567890123456789012123456789012345678901
amiable.
Of
the
two
Dashwood
sisters,
Elinor
was
the
2
12345678901234567890123456789012123456789012345678901
2
12345678901234567890123456789012123456789012345678901
more
complaisant,
often
putting
the
strictures
of
society
2345678901234567890123456789012123456789012345678901
2
1
12345678901234567890123456789012123456789012345678901
and family above her own desires. complaisance (noun). 2
2
12345678901234567890123456789012123456789012345678901
2
12345678901234567890123456789012123456789012345678901
compound
(verb)
To
intensify,
to
exacerbate.
When
you
2
12345678901234567890123456789012123456789012345678901
2345678901234567890123456789012123456789012345678901
2
1
make a faux pas, my father advised me, dont compound 2
12345678901234567890123456789012123456789012345678901
12345678901234567890123456789012123456789012345678901
the problem by apologizing profusely; just say youre 2
2
12345678901234567890123456789012123456789012345678901
2
12345678901234567890123456789012123456789012345678901
sorry
and
get
on
with
life!
2345678901234567890123456789012123456789012345678901
12345678901234567890123456789012123456789012345678901 2
12345678901234567890123456789012123456789012345678901 2
12345678901234567890123456789012123456789012345678901
conceivable (adjective) Possible, imaginable. Its possible to 2
2
12345678901234567890123456789012123456789012345678901
12345678901234567890123456789012123456789012345678901
find people with every conceivable interest by surfing the 2
12345678901234567890123456789012123456789012345678901 2
1
World Wide Webfrom fans of minor film stars to 2
2
12345678901234567890123456789012123456789012345678901
12345678901234567890123456789012123456789012345678901
those who study the mating habits of crustaceans. 2
2
12345678901234567890123456789012123456789012345678901
2
12345678901234567890123456789012123456789012345678901
conception (noun).
2
12345678901234567890123456789012123456789012345678901
2
12345678901234567890123456789012123456789012345678901
concur
(verb)
To
agree,
to
approve.
We
concur
that
a
2
12345678901234567890123456789012123456789012345678901
2
12345678901234567890123456789012123456789012345678901
toddler
functions
best
on
a
fairly
reliable
schedule;
2345678901234567890123456789012123456789012345678901
12345678901234567890123456789012123456789012345678901 2
12345678901234567890123456789012123456789012345678901
however, my husband tends to be a bit more rigid than I 2
2
12345678901234567890123456789012123456789012345678901
2
12345678901234567890123456789012123456789012345678901
am.
concurrence
(noun).
2
1
2345678901234567890123456789012123456789012345678901
12345678901234567890123456789012123456789012345678901 2
12345678901234567890123456789012123456789012345678901
condensation (noun) A reduction to a denser form (from steam 2
2
1
12345678901234567890123456789012123456789012345678901
to water); an abridgment of a literary work. The conden- 2
2
12345678901234567890123456789012123456789012345678901
12345678901234567890123456789012123456789012345678901
sation of humidity on the cars windshield made it difficult 2
2
12345678901234567890123456789012123456789012345678901
for me to see the road. It seems as though every beach 2
12345678901234567890123456789012123456789012345678901
2
12345678901234567890123456789012123456789012345678901
house Ive ever rented features a shelf full of Readers Di- 2
12345678901234567890123456789012123456789012345678901
2345678901234567890123456789012123456789012345678901
2
12345678901234567890123456789012123456789012345678901
gest condensations of b-grade novels. condense (verb).
2
12345678901234567890123456789012123456789012345678901
2
1
2
12345678901234567890123456789012123456789012345678901
condescending
(adjective)
Having
an
attitude
of
superiority
12345678901234567890123456789012123456789012345678901 2
2345678901234567890123456789012123456789012345678901
12345678901234567890123456789012123456789012345678901
toward another; patronizing. What a cute little car! 2
2
12345678901234567890123456789012123456789012345678901
2
12345678901234567890123456789012123456789012345678901
she
remarked
in
a
condescending
fashion.
I
suppose
its
12345678901234567890123456789012123456789012345678901 2
1
the nicest one someone like you could afford! conde- 2
2
12345678901234567890123456789012123456789012345678901
2
12345678901234567890123456789012123456789012345678901
scension
(noun).
2
12345678901234567890123456789012123456789012345678901
2
12345678901234567890123456789012123456789012345678901
condone
(verb)
To
overlook,
to
permit
to
happen.
Schools
12345678901234567890123456789012123456789012345678901 2
2345678901234567890123456789012123456789012345678901
2
12345678901234567890123456789012123456789012345678901
with Zero Tolerance policies do not condone alcohol, 2
12345678901234567890123456789012123456789012345678901
2
12345678901234567890123456789012123456789012345678901
drugs, vandalism, or violence on school grounds.
2
12345678901234567890123456789012123456789012345678901
2
1
2345678901234567890123456789012123456789012345678901
12345678901234567890123456789012123456789012345678901
congruent (adjective) Coinciding; harmonious. Fortunately, 2
2
1
2
12345678901234567890123456789012123456789012345678901
the
two
employees
who
had
been
asked
to
organize
the
12345678901234567890123456789012123456789012345678901 2
12345678901234567890123456789012123456789012345678901
department had congruent views on the budget. congru- 2
2
12345678901234567890123456789012123456789012345678901
2
12345678901234567890123456789012123456789012345678901
ence
(noun).
12345678901234567890123456789012123456789012345678901 2
2
176 12345678901234567890123456789012123456789012345678901
123456789012345678901234567890121234567890123456789012

www.petersons.com

http://www.xtremepapers.net

Chapter 5: Verbal Review

Take It to the Next Level

123456789012345678901234567890121234567890123456789012
12345678901234567890123456789012123456789012345678901 2
2
12345678901234567890123456789012123456789012345678901
2
12345678901234567890123456789012123456789012345678901
Latin jungere 5 to join. Also found in English injunction, junction,
2345678901234567890123456789012123456789012345678901
2
1
Word
Origin
2
12345678901234567890123456789012123456789012345678901
and juncture.
12345678901234567890123456789012123456789012345678901 2
2
12345678901234567890123456789012123456789012345678901
conjunction (noun) The occurrence of two or more events 2
12345678901234567890123456789012123456789012345678901
2
12345678901234567890123456789012123456789012345678901
together in time or space; in astronomy, the point at 2
12345678901234567890123456789012123456789012345678901
2
12345678901234567890123456789012123456789012345678901
which two celestial bodies have the least separation. 2
12345678901234567890123456789012123456789012345678901
2345678901234567890123456789012123456789012345678901
2
1
Low inflation, occurring in conjunction with low 2
12345678901234567890123456789012123456789012345678901
12345678901234567890123456789012123456789012345678901
unemployment and relatively low interest rates, has 2
2
12345678901234567890123456789012123456789012345678901
2
12345678901234567890123456789012123456789012345678901
enabled
the
United
States
to
enjoy
a
long
period
of
2345678901234567890123456789012123456789012345678901
2
1
2
12345678901234567890123456789012123456789012345678901
sustained
economic
growth.
The
moon
is
in
conjunction
2
12345678901234567890123456789012123456789012345678901
2
12345678901234567890123456789012123456789012345678901
with
the
sun
when
it
is
new;
if
the
conjunction
is
perfect,
2
12345678901234567890123456789012123456789012345678901
2345678901234567890123456789012123456789012345678901
2
1
an eclipse of the sun will occur. conjoin (verb).
2
12345678901234567890123456789012123456789012345678901
2
12345678901234567890123456789012123456789012345678901
2
12345678901234567890123456789012123456789012345678901
consolation
(noun)
Relief
or
comfort
in
sorrow
or
suffering.
2
12345678901234567890123456789012123456789012345678901
2345678901234567890123456789012123456789012345678901
12345678901234567890123456789012123456789012345678901
Although we miss our dog very much, it is a consolation 2
2
12345678901234567890123456789012123456789012345678901
12345678901234567890123456789012123456789012345678901
to know that she died quickly, without much suffering. 2
12345678901234567890123456789012123456789012345678901 22
12345678901234567890123456789012123456789012345678901
console (verb).
2
12345678901234567890123456789012123456789012345678901
2
1
consternation (noun) Shock, amazement, dismay. When a 2
12345678901234567890123456789012123456789012345678901
2
12345678901234567890123456789012123456789012345678901
voice in the back of the church shouted out, I know 2
12345678901234567890123456789012123456789012345678901
2345678901234567890123456789012123456789012345678901
12345678901234567890123456789012123456789012345678901
why they should not be married! the entire gathering 2
2
12345678901234567890123456789012123456789012345678901
2
12345678901234567890123456789012123456789012345678901
was
thrown
into
consternation.
12345678901234567890123456789012123456789012345678901 22
12345678901234567890123456789012123456789012345678901 2
12345678901234567890123456789012123456789012345678901
convergence (noun) The act of coming together in unity or 2
12345678901234567890123456789012123456789012345678901
12345678901234567890123456789012123456789012345678901
similarity. A remarkable example of evolutionary conver- 2
2
12345678901234567890123456789012123456789012345678901
2
1
gence
can
be
seen
in
the
shark
and
the
dolphin,
two
sea
2345678901234567890123456789012123456789012345678901
12345678901234567890123456789012123456789012345678901 22
12345678901234567890123456789012123456789012345678901
creatures that developed from different origins to 2
1
12345678901234567890123456789012123456789012345678901
become very similar in form and appearance. converge 2
2
12345678901234567890123456789012123456789012345678901
2345678901234567890123456789012123456789012345678901
2
12345678901234567890123456789012123456789012345678901
(verb).
2
12345678901234567890123456789012123456789012345678901
2
1
2
12345678901234567890123456789012123456789012345678901
2
12345678901234567890123456789012123456789012345678901
Latin
vivere
5
to
live.
Also
found
in
English
revive,
vital,
vivid,
and
2345678901234567890123456789012123456789012345678901
2
12345678901234567890123456789012123456789012345678901
Word
Origin vivisection.
2
12345678901234567890123456789012123456789012345678901
2
1
2
12345678901234567890123456789012123456789012345678901
conviviality
(noun)
Fond
of
good
company
and
eating
and
12345678901234567890123456789012123456789012345678901 2
2345678901234567890123456789012123456789012345678901
12345678901234567890123456789012123456789012345678901
drinking. The conviviality of my fellow employees 2
2
12345678901234567890123456789012123456789012345678901
2
12345678901234567890123456789012123456789012345678901
seemed
to
turn
every
staff
meeting
into
a
party,
complete
12345678901234567890123456789012123456789012345678901 22
1
with snacks, drinks, and lots of hearty laughter. conviv- 2
12345678901234567890123456789012123456789012345678901
2
12345678901234567890123456789012123456789012345678901
ial (adjective).
2
12345678901234567890123456789012123456789012345678901
2
12345678901234567890123456789012123456789012345678901
12345678901234567890123456789012123456789012345678901 2
2345678901234567890123456789012123456789012345678901
2
12345678901234567890123456789012123456789012345678901
Latin volvere 5 to roll. Also found in English devolve, involve,
2
1
Word
Origin revolution, revolve, and voluble.
2
12345678901234567890123456789012123456789012345678901
2
12345678901234567890123456789012123456789012345678901
12345678901234567890123456789012123456789012345678901 2
2345678901234567890123456789012123456789012345678901
12345678901234567890123456789012123456789012345678901
convoluted (adjective) Twisting, complicated, intricate. In- 2
2
1
12345678901234567890123456789012123456789012345678901
come tax law has become so convoluted that its easy for 2
2
12345678901234567890123456789012123456789012345678901
people to violate it completely by accident. convolute 2
12345678901234567890123456789012123456789012345678901
2345678901234567890123456789012123456789012345678901
2
1
(verb), convolution (noun).
2
12345678901234567890123456789012123456789012345678901
12345678901234567890123456789012123456789012345678901 22
12345678901234567890123456789012123456789012345678901 177
123456789012345678901234567890121234567890123456789012

http://www.xtremepapers.net

Part III: Section Review

123456789012345678901234567890121234567890123456789012
12345678901234567890123456789012123456789012345678901 2
2
12345678901234567890123456789012123456789012345678901
cordon (verb) To form a protective or restrictive barrier. 2
12345678901234567890123456789012123456789012345678901
2
12345678901234567890123456789012123456789012345678901
Well before the Academy Awards ceremony began, the 2
12345678901234567890123456789012123456789012345678901
2
12345678901234567890123456789012123456789012345678901
police cordoned off the hordes of fans who were 2
12345678901234567890123456789012123456789012345678901
2345678901234567890123456789012123456789012345678901
2
1
desperate to ogle the arriving stars. cordon (noun).
2
12345678901234567890123456789012123456789012345678901
2
12345678901234567890123456789012123456789012345678901
2
12345678901234567890123456789012123456789012345678901
corral
(verb)
To
enclose,
to
collect,
to
gather.
Tyrone
2
12345678901234567890123456789012123456789012345678901
2345678901234567890123456789012123456789012345678901
1
couldnt enjoy the wedding at all, since he spent most of 2
2
12345678901234567890123456789012123456789012345678901
2
12345678901234567890123456789012123456789012345678901
his
time
corralling
his
two
children
into
the
reception
2
12345678901234567890123456789012123456789012345678901
2
12345678901234567890123456789012123456789012345678901
room
and
preventing
them
from
running
amok
through
2345678901234567890123456789012123456789012345678901
2
1
2
12345678901234567890123456789012123456789012345678901
the
Potters
mansion.
corral
(noun).
2
12345678901234567890123456789012123456789012345678901
2
12345678901234567890123456789012123456789012345678901
12345678901234567890123456789012123456789012345678901
corroborating (adjective) Supporting with evidence; confirm- 2
2345678901234567890123456789012123456789012345678901
2
1
12345678901234567890123456789012123456789012345678901
ing. A passerby who had witnessed the crime gave 2
2
12345678901234567890123456789012123456789012345678901
corroborating testimony about the presence of the 2
12345678901234567890123456789012123456789012345678901
2
12345678901234567890123456789012123456789012345678901
accused person. corroborate (verb), corroboration (noun). 2
12345678901234567890123456789012123456789012345678901
2
12345678901234567890123456789012123456789012345678901
2
12345678901234567890123456789012123456789012345678901
corrosive
(adjective)
Eating
away,
gnawing,
or
destroying.
2
12345678901234567890123456789012123456789012345678901
2
12345678901234567890123456789012123456789012345678901
Years
of
poverty
and
hard
work
had
a
corrosive
effect
on
2345678901234567890123456789012123456789012345678901
12345678901234567890123456789012123456789012345678901 2
1
her strength and beauty. corrode (verb), corrosion 2
2
12345678901234567890123456789012123456789012345678901
2
12345678901234567890123456789012123456789012345678901
(noun).
2
12345678901234567890123456789012123456789012345678901
2345678901234567890123456789012123456789012345678901
12345678901234567890123456789012123456789012345678901 2
12345678901234567890123456789012123456789012345678901
cosmopolitanism (noun) International sophistication; world- 2
2
12345678901234567890123456789012123456789012345678901
12345678901234567890123456789012123456789012345678901
liness. Budapest is known for its cosmopolitanism, 2
12345678901234567890123456789012123456789012345678901 2
12345678901234567890123456789012123456789012345678901
perhaps because it was the first Eastern European city to 2
2
12345678901234567890123456789012123456789012345678901
12345678901234567890123456789012123456789012345678901
be more open to capitalism and influences from the 2
12345678901234567890123456789012123456789012345678901 2
2
1
West. cosmopolitan (adjective).
2
12345678901234567890123456789012123456789012345678901
2
12345678901234567890123456789012123456789012345678901
covert (adjective) Secret, clandestine. The CIA has often 2
12345678901234567890123456789012123456789012345678901
2
12345678901234567890123456789012123456789012345678901
been criticized for its covert operations in the domestic 2
12345678901234567890123456789012123456789012345678901
2345678901234567890123456789012123456789012345678901
12345678901234567890123456789012123456789012345678901
policies of foreign countries, such as the failed Bay of 2
2
12345678901234567890123456789012123456789012345678901
2
1
Pigs
operation
in
Cuba.
2
12345678901234567890123456789012123456789012345678901
2
12345678901234567890123456789012123456789012345678901
2345678901234567890123456789012123456789012345678901
12345678901234567890123456789012123456789012345678901
covetous (adjective) Envious, particularly of anothers pos- 2
2
12345678901234567890123456789012123456789012345678901
2
1
sessions.
Benita
would
never
admit
to
being
covetous
of
12345678901234567890123456789012123456789012345678901 2
12345678901234567890123456789012123456789012345678901
my new sable jacket, but I found it odd that she couldnt 2
2
12345678901234567890123456789012123456789012345678901
2
12345678901234567890123456789012123456789012345678901
refrain
from
trying
it
on
each
time
we
met.
covet
(verb).
2
12345678901234567890123456789012123456789012345678901
2
12345678901234567890123456789012123456789012345678901
12345678901234567890123456789012123456789012345678901
craven (adjective) Cowardly. Local Gay and Lesbian activ- 2
2345678901234567890123456789012123456789012345678901
2
12345678901234567890123456789012123456789012345678901
ists were outraged by the craven behavior of a policeman 2
12345678901234567890123456789012123456789012345678901
2
12345678901234567890123456789012123456789012345678901
who refused to come to the aid of an HIV-positive 2
12345678901234567890123456789012123456789012345678901
2
1
accident victim.
2
12345678901234567890123456789012123456789012345678901
2
12345678901234567890123456789012123456789012345678901
2
12345678901234567890123456789012123456789012345678901
credulous
(adjective)
Ready
to
believe;
gullible.
Elaine
was
2
12345678901234567890123456789012123456789012345678901
2
12345678901234567890123456789012123456789012345678901
not
very
credulous
of
the
explanation
Serge
gave
for
his
2345678901234567890123456789012123456789012345678901
12345678901234567890123456789012123456789012345678901 2
2
1
acquisition of the Matisse lithograph. credulity (noun).
2
12345678901234567890123456789012123456789012345678901
12345678901234567890123456789012123456789012345678901 2
2
12345678901234567890123456789012123456789012345678901
12345678901234567890123456789012123456789012345678901 2
12345678901234567890123456789012123456789012345678901 2
12345678901234567890123456789012123456789012345678901 2
2
178 12345678901234567890123456789012123456789012345678901
123456789012345678901234567890121234567890123456789012

www.petersons.com

http://www.xtremepapers.net

Chapter 5: Verbal Review

Take It to the Next Level

123456789012345678901234567890121234567890123456789012
12345678901234567890123456789012123456789012345678901 2
2
12345678901234567890123456789012123456789012345678901
cryptic (adjective) Puzzling, ambiguous. I was puzzled by 2
12345678901234567890123456789012123456789012345678901
2
12345678901234567890123456789012123456789012345678901
the cryptic message left on my answering machine about 2
12345678901234567890123456789012123456789012345678901
2
12345678901234567890123456789012123456789012345678901
a shipment of pomegranates from an anonymous 2
12345678901234567890123456789012123456789012345678901
2345678901234567890123456789012123456789012345678901
2
1
donor.
2
12345678901234567890123456789012123456789012345678901
2
12345678901234567890123456789012123456789012345678901
2
12345678901234567890123456789012123456789012345678901
culmination
(noun)
The
climax.
The
Los
Angeles
riots,
in
2
12345678901234567890123456789012123456789012345678901
2345678901234567890123456789012123456789012345678901
1
the aftermath of the Rodney King verdict, were the 2
2
12345678901234567890123456789012123456789012345678901
2
12345678901234567890123456789012123456789012345678901
culmination
of
long-standing
racial
tensions
between
the
2
12345678901234567890123456789012123456789012345678901
2
12345678901234567890123456789012123456789012345678901
residents
of
South
Central
LA
and
the
police.
culminate
2345678901234567890123456789012123456789012345678901
2
1
2
12345678901234567890123456789012123456789012345678901
(verb).
2
12345678901234567890123456789012123456789012345678901
2
12345678901234567890123456789012123456789012345678901
12345678901234567890123456789012123456789012345678901
culpable (adjective) Deserving blame, guilty. Although he 2
2345678901234567890123456789012123456789012345678901
2
1
12345678901234567890123456789012123456789012345678901
committed the crime, because he was mentally ill he 2
2
12345678901234567890123456789012123456789012345678901
should not be considered culpable for his actions. 2
12345678901234567890123456789012123456789012345678901
2
12345678901234567890123456789012123456789012345678901
culpability (noun).
2
12345678901234567890123456789012123456789012345678901
2
12345678901234567890123456789012123456789012345678901
2
12345678901234567890123456789012123456789012345678901
curmudgeon
(noun)
A
crusty,
ill-tempered
person.
Todd
2
12345678901234567890123456789012123456789012345678901
2
12345678901234567890123456789012123456789012345678901
hated
to
drive
with
his
Uncle
Jasper,
a
notorious
2345678901234567890123456789012123456789012345678901
12345678901234567890123456789012123456789012345678901 22
1
curmudgeon, who complained non-stop about the 2
12345678901234567890123456789012123456789012345678901
12345678901234567890123456789012123456789012345678901
air-conditioning and Todds driving. curmudgeonly (ad- 2
2
12345678901234567890123456789012123456789012345678901
2345678901234567890123456789012123456789012345678901
2
12345678901234567890123456789012123456789012345678901
jective).
2
12345678901234567890123456789012123456789012345678901
12345678901234567890123456789012123456789012345678901 22
12345678901234567890123456789012123456789012345678901
cursory (adjective) Hasty and superficial. Detective Martinez 2
12345678901234567890123456789012123456789012345678901
12345678901234567890123456789012123456789012345678901
was rebuked by his superior officer for drawing conclu- 2
2
12345678901234567890123456789012123456789012345678901
12345678901234567890123456789012123456789012345678901
sions about the murder after only a cursory examination 2
12345678901234567890123456789012123456789012345678901 22
1
of the crime scene.
2
12345678901234567890123456789012123456789012345678901
2
12345678901234567890123456789012123456789012345678901
debilitating (adjective) Weakening; sapping the strength of. 2
12345678901234567890123456789012123456789012345678901
2
12345678901234567890123456789012123456789012345678901
One cant help but marvel at the courage Steven 2
12345678901234567890123456789012123456789012345678901
2345678901234567890123456789012123456789012345678901
12345678901234567890123456789012123456789012345678901
Hawking displays in the face of such a debilitating 2
2
12345678901234567890123456789012123456789012345678901
2
1
disease
as
ALS.
debilitate
(verb).
2
12345678901234567890123456789012123456789012345678901
2
12345678901234567890123456789012123456789012345678901
2345678901234567890123456789012123456789012345678901
12345678901234567890123456789012123456789012345678901 22
12345678901234567890123456789012123456789012345678901 2
1
Word
Origin Latin celer 5 swift. Also found in English accelerate and celerity.
2
12345678901234567890123456789012123456789012345678901
2
12345678901234567890123456789012123456789012345678901
decelerate
(verb)
To
slow
down.
Randall
didnt
decelerate
2345678901234567890123456789012123456789012345678901
12345678901234567890123456789012123456789012345678901 22
12345678901234567890123456789012123456789012345678901
enough on the winding roads, and he ended up smashing 2
12345678901234567890123456789012123456789012345678901
2
12345678901234567890123456789012123456789012345678901
his new sports utility vehicle into a guard rail. decelera- 2
1
2345678901234567890123456789012123456789012345678901
2
12345678901234567890123456789012123456789012345678901
tion (noun).
2
12345678901234567890123456789012123456789012345678901
12345678901234567890123456789012123456789012345678901 22
12345678901234567890123456789012123456789012345678901
decimation (noun) Almost complete destruction. Michael 2
1
12345678901234567890123456789012123456789012345678901
Moores documentary Roger and Me chronicles the 2
2
12345678901234567890123456789012123456789012345678901
2
12345678901234567890123456789012123456789012345678901
decimation
of
the
economy
of
Flint,
Michigan,
after
the
2
12345678901234567890123456789012123456789012345678901
2
12345678901234567890123456789012123456789012345678901
closing
of
a
General
Motors
factory.
decimate
(verb).
2345678901234567890123456789012123456789012345678901
12345678901234567890123456789012123456789012345678901 22
1
12345678901234567890123456789012123456789012345678901
decry (verb) To criticize or condemn. Cigarette ads aimed at 2
2
12345678901234567890123456789012123456789012345678901
12345678901234567890123456789012123456789012345678901
youngsters have led many to decry the unfair marketing 2
2345678901234567890123456789012123456789012345678901
2
1
2
12345678901234567890123456789012123456789012345678901
tactics of the tobacco industry.
12345678901234567890123456789012123456789012345678901 22
12345678901234567890123456789012123456789012345678901 179
123456789012345678901234567890121234567890123456789012

http://www.xtremepapers.net

Part III: Section Review

123456789012345678901234567890121234567890123456789012
12345678901234567890123456789012123456789012345678901 2
2
12345678901234567890123456789012123456789012345678901
defamation (noun) Act of harming someone by libel or 2
12345678901234567890123456789012123456789012345678901
2
12345678901234567890123456789012123456789012345678901
slander. When the article in The National Enquirer 2
12345678901234567890123456789012123456789012345678901
2
12345678901234567890123456789012123456789012345678901
implied that she was somehow responsible for her 2
12345678901234567890123456789012123456789012345678901
2345678901234567890123456789012123456789012345678901
2
1
husbands untimely death, Renata instructed her lawyer 2
12345678901234567890123456789012123456789012345678901
12345678901234567890123456789012123456789012345678901
to sue the paper for defamation of character. defame 2
2
12345678901234567890123456789012123456789012345678901
2
12345678901234567890123456789012123456789012345678901
(verb).
2345678901234567890123456789012123456789012345678901
2
1
2
12345678901234567890123456789012123456789012345678901
2
12345678901234567890123456789012123456789012345678901
defer
(verb)
To
graciously
submit
to
anothers
will;
to
2
12345678901234567890123456789012123456789012345678901
2
12345678901234567890123456789012123456789012345678901
delegate.
In
all
matters
relating
to
the
childrens
religious
2345678901234567890123456789012123456789012345678901
2
1
2
12345678901234567890123456789012123456789012345678901
education,
Joy
deferred
to
her
husband,
since
he
clearly
2
12345678901234567890123456789012123456789012345678901
2
12345678901234567890123456789012123456789012345678901
cared
more
about
giving
them
a
solid
grounding
in
2
12345678901234567890123456789012123456789012345678901
2345678901234567890123456789012123456789012345678901
2
1
Judaism. deference (noun).
2
12345678901234567890123456789012123456789012345678901
2
12345678901234567890123456789012123456789012345678901
deliberate (verb) To think about an issue before reaching a 2
12345678901234567890123456789012123456789012345678901
2
12345678901234567890123456789012123456789012345678901
decision. The legal pundits covering the O.J. Simpson 2
12345678901234567890123456789012123456789012345678901
2
12345678901234567890123456789012123456789012345678901
trial were shocked by the short time the jury took to 2
12345678901234567890123456789012123456789012345678901
2
12345678901234567890123456789012123456789012345678901
deliberate after a trial that lasted months. deliberation 2
12345678901234567890123456789012123456789012345678901
2345678901234567890123456789012123456789012345678901
2
12345678901234567890123456789012123456789012345678901
(noun).
2
1
2
12345678901234567890123456789012123456789012345678901
2
12345678901234567890123456789012123456789012345678901
demagogue
(noun)
A
leader
who
plays
dishonestly
on
the
2
12345678901234567890123456789012123456789012345678901
2345678901234567890123456789012123456789012345678901
12345678901234567890123456789012123456789012345678901
prejudices and emotions of his followers. Senator Joseph 2
2
12345678901234567890123456789012123456789012345678901
2
12345678901234567890123456789012123456789012345678901
McCarthy
was
a
demagogue
who
used
the
paranoia
and
12345678901234567890123456789012123456789012345678901 2
12345678901234567890123456789012123456789012345678901
biases of the anti-communist 1950s as a way of seizing 2
2
12345678901234567890123456789012123456789012345678901
2
12345678901234567890123456789012123456789012345678901
fame
and
considerable
power
in
Washington.
demagogu12345678901234567890123456789012123456789012345678901 2
2
12345678901234567890123456789012123456789012345678901
ery (noun).
2
1
2345678901234567890123456789012123456789012345678901
12345678901234567890123456789012123456789012345678901 2
12345678901234567890123456789012123456789012345678901 2
2
1
Greek demos 5 people. Also found in English democracy,
2
12345678901234567890123456789012123456789012345678901
Word
Origin
2
12345678901234567890123456789012123456789012345678901
demographic, and endemic.
2345678901234567890123456789012123456789012345678901
12345678901234567890123456789012123456789012345678901 2
2
12345678901234567890123456789012123456789012345678901
demographic (adjective) Relating to the statistical study of 2
1
2
12345678901234567890123456789012123456789012345678901
population. Three demographic groups have been the 2
12345678901234567890123456789012123456789012345678901
2345678901234567890123456789012123456789012345678901
12345678901234567890123456789012123456789012345678901
intense focus of marketing strategy: baby boomers, born 2
2
12345678901234567890123456789012123456789012345678901
2
1
between
1946
and
1964;
baby
busters,
or
the
youth
12345678901234567890123456789012123456789012345678901 2
12345678901234567890123456789012123456789012345678901
market, born between 1965 and 1976; and a group 2
2
12345678901234567890123456789012123456789012345678901
2
12345678901234567890123456789012123456789012345678901
referred
to
as
tweens,
those
born
between
1977
and
2
12345678901234567890123456789012123456789012345678901
2
12345678901234567890123456789012123456789012345678901
1983.
demography
(noun),
demographics
(noun).
12345678901234567890123456789012123456789012345678901 2
2345678901234567890123456789012123456789012345678901
12345678901234567890123456789012123456789012345678901 2
12345678901234567890123456789012123456789012345678901
demonstratively (adverb) Openly displaying feeling. The 2
2
12345678901234567890123456789012123456789012345678901
12345678901234567890123456789012123456789012345678901
young congressman demonstratively campaigned for 2
2
1
12345678901234567890123456789012123456789012345678901
reelection, kissing every baby and hugging every senior 2
2
12345678901234567890123456789012123456789012345678901
12345678901234567890123456789012123456789012345678901
citizen at the Saugerties Chrysanthemum festival. demon- 2
2
12345678901234567890123456789012123456789012345678901
strative (adjective).
2
12345678901234567890123456789012123456789012345678901
2345678901234567890123456789012123456789012345678901
12345678901234567890123456789012123456789012345678901 2
2
1
2
12345678901234567890123456789012123456789012345678901
12345678901234567890123456789012123456789012345678901 2
2
12345678901234567890123456789012123456789012345678901
12345678901234567890123456789012123456789012345678901 2
12345678901234567890123456789012123456789012345678901 2
12345678901234567890123456789012123456789012345678901 2
2
180 12345678901234567890123456789012123456789012345678901
123456789012345678901234567890121234567890123456789012

www.petersons.com

http://www.xtremepapers.net

Chapter 5: Verbal Review

Take It to the Next Level

123456789012345678901234567890121234567890123456789012
12345678901234567890123456789012123456789012345678901 2
2
12345678901234567890123456789012123456789012345678901
derisive (adjective) Expressing ridicule or scorn. Many 2
12345678901234567890123456789012123456789012345678901
2
12345678901234567890123456789012123456789012345678901
womens groups were derisive of Avons choice of a male 2
12345678901234567890123456789012123456789012345678901
2
12345678901234567890123456789012123456789012345678901
CEO, since the company derives its $5.1 billion in sales 2
12345678901234567890123456789012123456789012345678901
2345678901234567890123456789012123456789012345678901
2
1
from an army of female salespeople. derision (noun).
2
12345678901234567890123456789012123456789012345678901
2
12345678901234567890123456789012123456789012345678901
2
12345678901234567890123456789012123456789012345678901
derivative
(adjective)
Imitating
or
borrowed
from
a
particu2
12345678901234567890123456789012123456789012345678901
2345678901234567890123456789012123456789012345678901
1
lar source. When a person first writes poetry, her poems 2
2
12345678901234567890123456789012123456789012345678901
2
12345678901234567890123456789012123456789012345678901
are
apt
to
be
derivative
of
whatever
poetry
she
most
2
12345678901234567890123456789012123456789012345678901
2
12345678901234567890123456789012123456789012345678901
enjoys
reading.
derivation
(noun),
derive
(verb).
2345678901234567890123456789012123456789012345678901
2
1
2
12345678901234567890123456789012123456789012345678901
12345678901234567890123456789012123456789012345678901
desiccate (verb) To dry out, to wither; to drain of vitality. 2
2
12345678901234567890123456789012123456789012345678901
12345678901234567890123456789012123456789012345678901
The long drought thoroughly desiccated our garden; 2
2345678901234567890123456789012123456789012345678901
2
1
12345678901234567890123456789012123456789012345678901
what was once a glorious Eden was now a scorched and 2
2
12345678901234567890123456789012123456789012345678901
hellish wasteland. A recent spate of books has debunked 2
12345678901234567890123456789012123456789012345678901
2
12345678901234567890123456789012123456789012345678901
the myth that menopause desiccates women and af- 2
12345678901234567890123456789012123456789012345678901
2
12345678901234567890123456789012123456789012345678901
firmed, instead, that women often reach heights of 2
12345678901234567890123456789012123456789012345678901
2
12345678901234567890123456789012123456789012345678901
creativity in their later years. desiccant (noun), desicca- 2
12345678901234567890123456789012123456789012345678901
2345678901234567890123456789012123456789012345678901
2
12345678901234567890123456789012123456789012345678901
tion (noun).
2
1
2
12345678901234567890123456789012123456789012345678901
2
12345678901234567890123456789012123456789012345678901
despotic
(adjective)
Oppressive
and
tyrannical.
During
the
2
12345678901234567890123456789012123456789012345678901
2345678901234567890123456789012123456789012345678901
12345678901234567890123456789012123456789012345678901
despotic reign of Idi Amin in the 1970s, an estimated 2
2
12345678901234567890123456789012123456789012345678901
2
12345678901234567890123456789012123456789012345678901
200,000
Ugandans
were
killed.
despot
(noun).
12345678901234567890123456789012123456789012345678901 22
12345678901234567890123456789012123456789012345678901 2
12345678901234567890123456789012123456789012345678901
desultory (adjective) Disconnected, aimless. Tinas few des2
12345678901234567890123456789012123456789012345678901
12345678901234567890123456789012123456789012345678901
ultory stabs at conversation fell flat as Guy just sat there, 2
12345678901234567890123456789012123456789012345678901 22
1
stony-faced; it was a disastrous first date.
2
12345678901234567890123456789012123456789012345678901
2
12345678901234567890123456789012123456789012345678901
deviate (verb) To depart from a standard or norm. Having 2
12345678901234567890123456789012123456789012345678901
2
12345678901234567890123456789012123456789012345678901
agreed upon a spending budget for the company, we 2
12345678901234567890123456789012123456789012345678901
2345678901234567890123456789012123456789012345678901
12345678901234567890123456789012123456789012345678901
mustnt deviate from it; if we do, we may run out of 2
2
12345678901234567890123456789012123456789012345678901
2
1
money
before
the
year
ends.
deviation
(noun).
2
12345678901234567890123456789012123456789012345678901
2
12345678901234567890123456789012123456789012345678901
2345678901234567890123456789012123456789012345678901
12345678901234567890123456789012123456789012345678901
diatribe (noun) Abusive or bitter speech or writing. While 2
2
12345678901234567890123456789012123456789012345678901
2
1
angry
conservatives
dismissed
Susan
Faludis
Backlash
12345678901234567890123456789012123456789012345678901 2
12345678901234567890123456789012123456789012345678901
as a feminist diatribe, it is actually a meticulously 2
2
12345678901234567890123456789012123456789012345678901
2
12345678901234567890123456789012123456789012345678901
researched
book.
2
12345678901234567890123456789012123456789012345678901
2
12345678901234567890123456789012123456789012345678901
12345678901234567890123456789012123456789012345678901
diffident (adjective) Hesitant, reserved, shy. Someone with a 2
2345678901234567890123456789012123456789012345678901
2
12345678901234567890123456789012123456789012345678901
diffident personality is most likely to succeed in a career 2
12345678901234567890123456789012123456789012345678901
2
12345678901234567890123456789012123456789012345678901
that involves little public contact. diffidence (noun).
2
12345678901234567890123456789012123456789012345678901
2
1
2345678901234567890123456789012123456789012345678901
12345678901234567890123456789012123456789012345678901
digress (verb) To wander from the main path or the main 2
2
12345678901234567890123456789012123456789012345678901
2
12345678901234567890123456789012123456789012345678901
topic.
My
high
school
biology
teacher
loved
to
digress
12345678901234567890123456789012123456789012345678901 22
1
from science into personal anecdotes about his college 2
12345678901234567890123456789012123456789012345678901
2
12345678901234567890123456789012123456789012345678901
adventures. digression (noun), digressive (adjective).
2
12345678901234567890123456789012123456789012345678901
12345678901234567890123456789012123456789012345678901 2
2
12345678901234567890123456789012123456789012345678901
12345678901234567890123456789012123456789012345678901 2
12345678901234567890123456789012123456789012345678901 2
12345678901234567890123456789012123456789012345678901 2
12345678901234567890123456789012123456789012345678901 2 181
123456789012345678901234567890121234567890123456789012

http://www.xtremepapers.net

Part III: Section Review

123456789012345678901234567890121234567890123456789012
12345678901234567890123456789012123456789012345678901 2
2
12345678901234567890123456789012123456789012345678901
dirge (noun) Song or hymn of grief. When Princess Diana 2
12345678901234567890123456789012123456789012345678901
2
12345678901234567890123456789012123456789012345678901
was killed in a car crash, Elton John resurrected his hit 2
12345678901234567890123456789012123456789012345678901
2
12345678901234567890123456789012123456789012345678901
song Candle in the Wind, rewrote it as Good-bye 2
12345678901234567890123456789012123456789012345678901
2345678901234567890123456789012123456789012345678901
2
1
Englands Rose, and created one of the most widely 2
12345678901234567890123456789012123456789012345678901
2
12345678901234567890123456789012123456789012345678901
heard funeral dirges of all time.
2
12345678901234567890123456789012123456789012345678901
2
12345678901234567890123456789012123456789012345678901
2345678901234567890123456789012123456789012345678901
1
disabuse (verb) To correct a fallacy, to clarify. I hated to 2
2
12345678901234567890123456789012123456789012345678901
2
12345678901234567890123456789012123456789012345678901
disabuse
Filbert,
who
is
a
passionate
collector
of
musical
2
12345678901234567890123456789012123456789012345678901
2
12345678901234567890123456789012123456789012345678901
trivia,
but
I
had
to
tell
him
that
the
Monkees
hadnt
2345678901234567890123456789012123456789012345678901
2
1
2
12345678901234567890123456789012123456789012345678901
played
their
own
instruments
on
almost
all
of
their
2
12345678901234567890123456789012123456789012345678901
2
12345678901234567890123456789012123456789012345678901
albums.
2
12345678901234567890123456789012123456789012345678901
2345678901234567890123456789012123456789012345678901
2
1
12345678901234567890123456789012123456789012345678901
disburse (verb) To pay out or distribute (funds or property). 2
2
12345678901234567890123456789012123456789012345678901
Jaime was flabbergasted when his fathers will disbursed 2
12345678901234567890123456789012123456789012345678901
2
12345678901234567890123456789012123456789012345678901
all of the old mans financial assets to Raymundo and 2
12345678901234567890123456789012123456789012345678901
2
12345678901234567890123456789012123456789012345678901
left him with only a few sticks of furniture. disbursement 2
12345678901234567890123456789012123456789012345678901
2
12345678901234567890123456789012123456789012345678901
(noun).
2
12345678901234567890123456789012123456789012345678901
2345678901234567890123456789012123456789012345678901
12345678901234567890123456789012123456789012345678901 2
1
discern (verb) To detect, notice, or observe. With difficulty, 2
2
12345678901234567890123456789012123456789012345678901
2
12345678901234567890123456789012123456789012345678901
I
could
discern
the
shape
of
a
whale
off
the
starboard
2
12345678901234567890123456789012123456789012345678901
2345678901234567890123456789012123456789012345678901
12345678901234567890123456789012123456789012345678901
bow, but it was too far away to determine its size or 2
2
12345678901234567890123456789012123456789012345678901
2
12345678901234567890123456789012123456789012345678901
species.
discernment
(noun).
12345678901234567890123456789012123456789012345678901 2
12345678901234567890123456789012123456789012345678901 2
12345678901234567890123456789012123456789012345678901
discordant (adjective) Characterized by conflict. Stories and 2
2
12345678901234567890123456789012123456789012345678901
12345678901234567890123456789012123456789012345678901
films about discordant relationships that resolve them- 2
12345678901234567890123456789012123456789012345678901 2
1
selves happily are always more interesting than stories 2
2
12345678901234567890123456789012123456789012345678901
about content couples who simply stay content. discor- 2
12345678901234567890123456789012123456789012345678901
2
12345678901234567890123456789012123456789012345678901
dance (noun).
2
12345678901234567890123456789012123456789012345678901
2
12345678901234567890123456789012123456789012345678901
2345678901234567890123456789012123456789012345678901
12345678901234567890123456789012123456789012345678901
discourse (noun) Formal and orderly exchange of ideas, a 2
2
12345678901234567890123456789012123456789012345678901
2
1
discussion.
In
the
late
twentieth
century,
cloning
and
2
12345678901234567890123456789012123456789012345678901
2
12345678901234567890123456789012123456789012345678901
other
feats
of
genetic
engineering
became
popular
topics
2345678901234567890123456789012123456789012345678901
12345678901234567890123456789012123456789012345678901 2
2
12345678901234567890123456789012123456789012345678901
of public discourse. discursive (adjective).
2
1
12345678901234567890123456789012123456789012345678901 2
12345678901234567890123456789012123456789012345678901 2
2
12345678901234567890123456789012123456789012345678901
Latin credere 5 to believe. Also found in English credential,
2345678901234567890123456789012123456789012345678901
2
1
Word
Origin credible, credit, credo, credulous, and incredible.
2
12345678901234567890123456789012123456789012345678901
2
12345678901234567890123456789012123456789012345678901
12345678901234567890123456789012123456789012345678901 2
2345678901234567890123456789012123456789012345678901
12345678901234567890123456789012123456789012345678901
discredit (verb) To cause disbelief in the accuracy of some 2
2
12345678901234567890123456789012123456789012345678901
12345678901234567890123456789012123456789012345678901
statement or the reliability of a person. Although many 2
12345678901234567890123456789012123456789012345678901 2
people still believe in UFOs, among scientists the reports 2
1
2
12345678901234567890123456789012123456789012345678901
of alien encounters have been thoroughly discredited. 2
12345678901234567890123456789012123456789012345678901
2
12345678901234567890123456789012123456789012345678901
2
12345678901234567890123456789012123456789012345678901
discreet
(adjective)
Showing
good
judgment
in
speech
and
12345678901234567890123456789012123456789012345678901 2
2345678901234567890123456789012123456789012345678901
12345678901234567890123456789012123456789012345678901
behavior. Be discreet when discussing confidential busi- 2
2
1
2
12345678901234567890123456789012123456789012345678901
ness
mattersdont
talk
among
strangers
on
the
eleva12345678901234567890123456789012123456789012345678901 2
2
12345678901234567890123456789012123456789012345678901
tor, for example. discretion (noun).
2
12345678901234567890123456789012123456789012345678901
12345678901234567890123456789012123456789012345678901 2
12345678901234567890123456789012123456789012345678901 2
2
182 12345678901234567890123456789012123456789012345678901
123456789012345678901234567890121234567890123456789012

www.petersons.com

http://www.xtremepapers.net

Chapter 5: Verbal Review

Take It to the Next Level

123456789012345678901234567890121234567890123456789012
12345678901234567890123456789012123456789012345678901 2
2
12345678901234567890123456789012123456789012345678901
discrete (adjective) Separate, unconnected. Canadians get 2
12345678901234567890123456789012123456789012345678901
2
12345678901234567890123456789012123456789012345678901
peeved when people cant seem to distinguish between 2
12345678901234567890123456789012123456789012345678901
2
12345678901234567890123456789012123456789012345678901
Canada and the United States, forgetting that Canada 2
12345678901234567890123456789012123456789012345678901
2345678901234567890123456789012123456789012345678901
2
1
has its own discrete heritage and culture.
2
12345678901234567890123456789012123456789012345678901
2
12345678901234567890123456789012123456789012345678901
2
12345678901234567890123456789012123456789012345678901
disparity
(noun)
Difference
in
quality
or
kind.
There
is
often
2
12345678901234567890123456789012123456789012345678901
2345678901234567890123456789012123456789012345678901
1
a disparity between the kind of serious, high-quality 2
2
12345678901234567890123456789012123456789012345678901
2
12345678901234567890123456789012123456789012345678901
television
people
say
they
want
and
the
low-brow
2
12345678901234567890123456789012123456789012345678901
2
12345678901234567890123456789012123456789012345678901
programs
they
actually
watch.
disparate
(adjective).
2345678901234567890123456789012123456789012345678901
2
1
2
12345678901234567890123456789012123456789012345678901
2
12345678901234567890123456789012123456789012345678901
2
12345678901234567890123456789012123456789012345678901
Latin
simulare
5
to
resemble.
Also
found
in
English
semblance,
2
12345678901234567890123456789012123456789012345678901
Word
Origin
2345678901234567890123456789012123456789012345678901
2
1
similarity, simulacrum, simultaneous, and verisimilitude.
2
12345678901234567890123456789012123456789012345678901
2
12345678901234567890123456789012123456789012345678901
dissemble (verb) To pretend, to simulate. When the police 2
12345678901234567890123456789012123456789012345678901
2
12345678901234567890123456789012123456789012345678901
asked whether Nancy knew anything about the crime, 2
12345678901234567890123456789012123456789012345678901
2
12345678901234567890123456789012123456789012345678901
she dissembled innocence.
2
12345678901234567890123456789012123456789012345678901
2
12345678901234567890123456789012123456789012345678901
2
12345678901234567890123456789012123456789012345678901
dissipate
(verb)
To
spread
out
or
scatter.
The
windows
and
2345678901234567890123456789012123456789012345678901
12345678901234567890123456789012123456789012345678901 22
1
doors were opened, allowing the smoke that had filled 2
12345678901234567890123456789012123456789012345678901
2
12345678901234567890123456789012123456789012345678901
the room to dissipate. dissipation (noun).
2
12345678901234567890123456789012123456789012345678901
2345678901234567890123456789012123456789012345678901
12345678901234567890123456789012123456789012345678901 22
12345678901234567890123456789012123456789012345678901
dissonance (noun) Lack of music harmony; lack of agree- 2
12345678901234567890123456789012123456789012345678901
12345678901234567890123456789012123456789012345678901
ment between ideas. Most modern music is character- 2
2
12345678901234567890123456789012123456789012345678901
12345678901234567890123456789012123456789012345678901
ized by dissonance, which many listeners find hard to 2
2
12345678901234567890123456789012123456789012345678901
12345678901234567890123456789012123456789012345678901
enjoy. There is a noticeable dissonance between two 2
12345678901234567890123456789012123456789012345678901 22
1
common beliefs of most conservatives: their faith in
2
12345678901234567890123456789012123456789012345678901
unfettered free markets and their preference for tradi- 2
12345678901234567890123456789012123456789012345678901
2
12345678901234567890123456789012123456789012345678901
tional social values. dissonant (adjective).
2
12345678901234567890123456789012123456789012345678901
2
12345678901234567890123456789012123456789012345678901
2345678901234567890123456789012123456789012345678901
12345678901234567890123456789012123456789012345678901
distillation (noun) Something distilled, an essence or ex- 2
2
12345678901234567890123456789012123456789012345678901
2
1
tract.
In
chemistry,
a
process
that
drives
gas
or
vapor
2
12345678901234567890123456789012123456789012345678901
2
12345678901234567890123456789012123456789012345678901
from
liquids
or
solids.
Sharon
Oldss
poems
are
power2345678901234567890123456789012123456789012345678901
12345678901234567890123456789012123456789012345678901 22
12345678901234567890123456789012123456789012345678901
ful distillations of motherhood and other primal experi- 2
1
12345678901234567890123456789012123456789012345678901
ences. In Mrs. Hornmeisters chemistry class, our first 2
2
12345678901234567890123456789012123456789012345678901
2345678901234567890123456789012123456789012345678901
12345678901234567890123456789012123456789012345678901
experiment was to create a distillation of carbon gas 2
2
12345678901234567890123456789012123456789012345678901
2
12345678901234567890123456789012123456789012345678901
from
wood.
distill
(verb).
12345678901234567890123456789012123456789012345678901 22
1
12345678901234567890123456789012123456789012345678901
diverge (verb) To move in different directions. Frosts poem 2
2
12345678901234567890123456789012123456789012345678901
12345678901234567890123456789012123456789012345678901
The Road Not Taken tells of the choice he made when 2
2
12345678901234567890123456789012123456789012345678901
12345678901234567890123456789012123456789012345678901
Two roads diverged in a yellow wood. divergence 2
2345678901234567890123456789012123456789012345678901
2
12345678901234567890123456789012123456789012345678901
(noun), divergent (adjective).
2
12345678901234567890123456789012123456789012345678901
12345678901234567890123456789012123456789012345678901 22
12345678901234567890123456789012123456789012345678901
diversify (verb) To balance by adding variety. Any financial 2
1
12345678901234567890123456789012123456789012345678901
manager will recommend that you diversify your stock 2
2
12345678901234567890123456789012123456789012345678901
2
12345678901234567890123456789012123456789012345678901
portfolio
by
holding
some
less-volatile
blue-chip
stocks
12345678901234567890123456789012123456789012345678901 2
12345678901234567890123456789012123456789012345678901
along with more growth-oriented technology issues. 2
2
12345678901234567890123456789012123456789012345678901
2
12345678901234567890123456789012123456789012345678901
diversification
(noun),
diversified
(adjective).
12345678901234567890123456789012123456789012345678901 22
12345678901234567890123456789012123456789012345678901 183
123456789012345678901234567890121234567890123456789012

http://www.xtremepapers.net

Part III: Section Review

123456789012345678901234567890121234567890123456789012
12345678901234567890123456789012123456789012345678901 2
2
12345678901234567890123456789012123456789012345678901
divest (verb) To rid (oneself) or be freed of property, 2
12345678901234567890123456789012123456789012345678901
2
12345678901234567890123456789012123456789012345678901
authority, or title. In order to turn around its ailing 2
12345678901234567890123456789012123456789012345678901
2
12345678901234567890123456789012123456789012345678901
company and concentrate on imaging, Eastman Kodak 2
12345678901234567890123456789012123456789012345678901
2345678901234567890123456789012123456789012345678901
2
1
divested itself of peripheral businesses in the areas of 2
12345678901234567890123456789012123456789012345678901
12345678901234567890123456789012123456789012345678901
household products, clinical diagnostics, and pharma- 2
2
12345678901234567890123456789012123456789012345678901
2
12345678901234567890123456789012123456789012345678901
ceuticals.
divestiture
(noun).
2345678901234567890123456789012123456789012345678901
2
1
2
12345678901234567890123456789012123456789012345678901
2
12345678901234567890123456789012123456789012345678901
divulge
(verb)
To
reveal.
The
people
who
count
the
votes
for
2
12345678901234567890123456789012123456789012345678901
2
12345678901234567890123456789012123456789012345678901
the
Oscar
awards
are
under
strict
orders
not
to
divulge
2345678901234567890123456789012123456789012345678901
2
1
2
12345678901234567890123456789012123456789012345678901
the
names
of
the
winners.
2
12345678901234567890123456789012123456789012345678901
2
12345678901234567890123456789012123456789012345678901
12345678901234567890123456789012123456789012345678901
dogmatic (adjective) Holding firmly to a particular set of 2
2345678901234567890123456789012123456789012345678901
2
1
12345678901234567890123456789012123456789012345678901
beliefs with little or no basis. Believers in Marxist 2
2
12345678901234567890123456789012123456789012345678901
doctrine tend to be dogmatic, ignoring evidence that 2
12345678901234567890123456789012123456789012345678901
2
12345678901234567890123456789012123456789012345678901
contradicts their beliefs or explaining it away. dogma 2
12345678901234567890123456789012123456789012345678901
2
12345678901234567890123456789012123456789012345678901
(noun), dogmatism (noun).
2
12345678901234567890123456789012123456789012345678901
2
12345678901234567890123456789012123456789012345678901
2
12345678901234567890123456789012123456789012345678901
dolt
(noun)
A
stupid
or
foolish
person.
Due
to
his
frequent
2345678901234567890123456789012123456789012345678901
12345678901234567890123456789012123456789012345678901 2
1
verbal blunders, politician Dan Quayle was widely 2
2
12345678901234567890123456789012123456789012345678901
2
12345678901234567890123456789012123456789012345678901
considered
to
be
a
dolt.
2
12345678901234567890123456789012123456789012345678901
2345678901234567890123456789012123456789012345678901
12345678901234567890123456789012123456789012345678901 2
12345678901234567890123456789012123456789012345678901 2
2
12345678901234567890123456789012123456789012345678901
2
12345678901234567890123456789012123456789012345678901
Word
Origin Latin dormire 5 to sleep. Also found in English dormitory.
12345678901234567890123456789012123456789012345678901 2
2
12345678901234567890123456789012123456789012345678901
dormant (adjective) Temporarily inactive, as if asleep. An 2
12345678901234567890123456789012123456789012345678901
2
12345678901234567890123456789012123456789012345678901
eruption of Mt. Rainier, a dormant volcano in Washing- 2
12345678901234567890123456789012123456789012345678901
1
ton state, would cause massive, life-threatening mud 2
2
12345678901234567890123456789012123456789012345678901
12345678901234567890123456789012123456789012345678901
slides in the surrounding area. Bill preferred to think 2
2
12345678901234567890123456789012123456789012345678901
2
12345678901234567890123456789012123456789012345678901
that
his
sex
drive
was
dormant
rather
than
extinct.
2
12345678901234567890123456789012123456789012345678901
2345678901234567890123456789012123456789012345678901
2
12345678901234567890123456789012123456789012345678901
dormancy (noun).
2
12345678901234567890123456789012123456789012345678901
2
1
2
12345678901234567890123456789012123456789012345678901
dross
(noun)
Something
that
is
trivial
or
inferior;
an
2
12345678901234567890123456789012123456789012345678901
2345678901234567890123456789012123456789012345678901
12345678901234567890123456789012123456789012345678901
impurity. As a reader for the Paris Review, Julia spent 2
2
12345678901234567890123456789012123456789012345678901
1
most of her time sifting through piles of manuscripts to 2
12345678901234567890123456789012123456789012345678901 2
2
12345678901234567890123456789012123456789012345678901
separate the extraordinary poems from the dross.
2
12345678901234567890123456789012123456789012345678901
2
12345678901234567890123456789012123456789012345678901
dubious (adjective) Doubtful, uncertain. Despite the chair- 2
12345678901234567890123456789012123456789012345678901
2
12345678901234567890123456789012123456789012345678901
mans attempts to convince the committee members that 2
12345678901234567890123456789012123456789012345678901
2345678901234567890123456789012123456789012345678901
2
12345678901234567890123456789012123456789012345678901
his plan would succeed, most of them remained dubious. 2
12345678901234567890123456789012123456789012345678901
2
12345678901234567890123456789012123456789012345678901
dubiety (noun).
2
12345678901234567890123456789012123456789012345678901
2
1
2345678901234567890123456789012123456789012345678901
12345678901234567890123456789012123456789012345678901
dupe (noun) Someone who is easily cheated. My cousin 2
2
12345678901234567890123456789012123456789012345678901
2
12345678901234567890123456789012123456789012345678901
Ravi
is
such
a
dupe;
he
actually
gets
excited
when
he
12345678901234567890123456789012123456789012345678901 2
1
receives those envelopes saying Ravi Murtugudde, you 2
2
12345678901234567890123456789012123456789012345678901
2
12345678901234567890123456789012123456789012345678901
may
have
won
a
million
dollars,
and
he
even
goes
so
far
2
12345678901234567890123456789012123456789012345678901
2
12345678901234567890123456789012123456789012345678901
as
to
try
claiming
his
prize.
2
12345678901234567890123456789012123456789012345678901
2345678901234567890123456789012123456789012345678901
2
1
12345678901234567890123456789012123456789012345678901 2
12345678901234567890123456789012123456789012345678901 2
2
184 12345678901234567890123456789012123456789012345678901
123456789012345678901234567890121234567890123456789012

www.petersons.com

http://www.xtremepapers.net

Chapter 5: Verbal Review

Take It to the Next Level

123456789012345678901234567890121234567890123456789012
12345678901234567890123456789012123456789012345678901 2
2
12345678901234567890123456789012123456789012345678901
eccentricity (noun) Odd or whimsical behavior. Rock star 2
12345678901234567890123456789012123456789012345678901
2
12345678901234567890123456789012123456789012345678901
Michael Jackson is now better known for his offstage 2
12345678901234567890123456789012123456789012345678901
2
12345678901234567890123456789012123456789012345678901
eccentricitiessuch as sleeping in an oxygen tank, 2
12345678901234567890123456789012123456789012345678901
2345678901234567890123456789012123456789012345678901
2
1
wearing a surgical mask, and building his own theme 2
12345678901234567890123456789012123456789012345678901
12345678901234567890123456789012123456789012345678901
parkthan for his onstage performances. eccentric 2
2
12345678901234567890123456789012123456789012345678901
2
12345678901234567890123456789012123456789012345678901
(adjective).
2345678901234567890123456789012123456789012345678901
2
1
2
12345678901234567890123456789012123456789012345678901
2
12345678901234567890123456789012123456789012345678901
edifying
(adjective)
Instructive,
enlightening.
Ariel
would
2
12345678901234567890123456789012123456789012345678901
2
12345678901234567890123456789012123456789012345678901
never
admit
it
to
her
high-brow
friends,
but
she
found
2345678901234567890123456789012123456789012345678901
2
1
2
12345678901234567890123456789012123456789012345678901
the
latest
self-help
best-seller
edifying
and
actually
2
12345678901234567890123456789012123456789012345678901
2
12345678901234567890123456789012123456789012345678901
helpful.
edification
(noun),
edify
(verb).
2
12345678901234567890123456789012123456789012345678901
2345678901234567890123456789012123456789012345678901
2
1
2
12345678901234567890123456789012123456789012345678901
2
12345678901234567890123456789012123456789012345678901
Latin
facere
5
to
do.
Also
found
in
English
facility,
factor,
facsimile,
2
12345678901234567890123456789012123456789012345678901
Word
Origin
2
12345678901234567890123456789012123456789012345678901
and faculty.
2345678901234567890123456789012123456789012345678901
12345678901234567890123456789012123456789012345678901 22
12345678901234567890123456789012123456789012345678901
efficacy (noun) The power to produce the desired effect. 2
12345678901234567890123456789012123456789012345678901
2
12345678901234567890123456789012123456789012345678901
While teams have been enormously popular in the 2
12345678901234567890123456789012123456789012345678901
12345678901234567890123456789012123456789012345678901
workplace, there are some who now question their 2
2
1
2
12345678901234567890123456789012123456789012345678901
efficacy
and
say
that
one
head
is
better
than
ten.
2
12345678901234567890123456789012123456789012345678901
2
12345678901234567890123456789012123456789012345678901
efficacious
(noun).
2345678901234567890123456789012123456789012345678901
12345678901234567890123456789012123456789012345678901 22
12345678901234567890123456789012123456789012345678901 2
12345678901234567890123456789012123456789012345678901
effrontery (noun) Shameless boldness. The sports world 2
12345678901234567890123456789012123456789012345678901
12345678901234567890123456789012123456789012345678901
was shocked when a pro basketball player had the 2
2
12345678901234567890123456789012123456789012345678901
2
12345678901234567890123456789012123456789012345678901
effrontery
to
choke
the
head
coach
of
his
team
during
a
12345678901234567890123456789012123456789012345678901 22
12345678901234567890123456789012123456789012345678901
practice session.
2
1
2345678901234567890123456789012123456789012345678901
12345678901234567890123456789012123456789012345678901 22
elaborate (verb) To expand upon something; develop. One
12345678901234567890123456789012123456789012345678901
2
1
characteristic of the best essayists is their ability to 2
12345678901234567890123456789012123456789012345678901
2
12345678901234567890123456789012123456789012345678901
elaborate ideas through examples, lists, similes, small 2
12345678901234567890123456789012123456789012345678901
2
12345678901234567890123456789012123456789012345678901
variations, and even exaggerations. elaborate (adjec- 2
12345678901234567890123456789012123456789012345678901
2
12345678901234567890123456789012123456789012345678901
tive), elaboration (noun).
2
12345678901234567890123456789012123456789012345678901
2345678901234567890123456789012123456789012345678901
12345678901234567890123456789012123456789012345678901 22
12345678901234567890123456789012123456789012345678901
elegy (noun) A song or poem expressing sorrow. Thomas 2
1
12345678901234567890123456789012123456789012345678901
Grays Elegy Written in a Country Churchyard, one 2
2
12345678901234567890123456789012123456789012345678901
2345678901234567890123456789012123456789012345678901
12345678901234567890123456789012123456789012345678901
of the most famous elegies in Western literature, mourns 2
2
12345678901234567890123456789012123456789012345678901
2
12345678901234567890123456789012123456789012345678901
the
unsung,
inglorious
lives
of
the
souls
buried
in
an
12345678901234567890123456789012123456789012345678901 22
1
obscure, rustic graveyard. elegaic (adjective).
2
12345678901234567890123456789012123456789012345678901
2
12345678901234567890123456789012123456789012345678901
12345678901234567890123456789012123456789012345678901
embellish (verb) To enhance or exaggerate; to decorate. The 2
2
12345678901234567890123456789012123456789012345678901
long-married couple told their stories in tandem, with 2
12345678901234567890123456789012123456789012345678901
2345678901234567890123456789012123456789012345678901
2
12345678901234567890123456789012123456789012345678901
the husband outlining the plot and the wife embellishing 2
12345678901234567890123456789012123456789012345678901
2
12345678901234567890123456789012123456789012345678901
it with colorful details. embellished (adjective).
2
12345678901234567890123456789012123456789012345678901
2
1
2345678901234567890123456789012123456789012345678901
12345678901234567890123456789012123456789012345678901 22
1
2
12345678901234567890123456789012123456789012345678901
12345678901234567890123456789012123456789012345678901 2
2
12345678901234567890123456789012123456789012345678901
12345678901234567890123456789012123456789012345678901 2
12345678901234567890123456789012123456789012345678901 2
12345678901234567890123456789012123456789012345678901 2
12345678901234567890123456789012123456789012345678901 2 185
123456789012345678901234567890121234567890123456789012

http://www.xtremepapers.net

Part III: Section Review

123456789012345678901234567890121234567890123456789012
12345678901234567890123456789012123456789012345678901 2
2
12345678901234567890123456789012123456789012345678901
embezzle (verb) To steal money or property that has been 2
12345678901234567890123456789012123456789012345678901
2
12345678901234567890123456789012123456789012345678901
entrusted to your care. The church treasurer was found 2
12345678901234567890123456789012123456789012345678901
2
12345678901234567890123456789012123456789012345678901
to have embezzled thousands of dollars by writing 2
12345678901234567890123456789012123456789012345678901
2345678901234567890123456789012123456789012345678901
2
1
phony checks on the church bank account. embezzle- 2
12345678901234567890123456789012123456789012345678901
2
12345678901234567890123456789012123456789012345678901
ment (noun).
2
12345678901234567890123456789012123456789012345678901
2
12345678901234567890123456789012123456789012345678901
2345678901234567890123456789012123456789012345678901
1
emollient (noun) Something that softens or soothes. She 2
2
12345678901234567890123456789012123456789012345678901
2
12345678901234567890123456789012123456789012345678901
used
a
hand
cream
as
an
emollient
on
her
dry,
2
12345678901234567890123456789012123456789012345678901
2
12345678901234567890123456789012123456789012345678901
work-roughened
hands.
emollient
(adjective).
2345678901234567890123456789012123456789012345678901
2
1
2
12345678901234567890123456789012123456789012345678901
12345678901234567890123456789012123456789012345678901
empirical (adjective) Based on experience or personal obser- 2
2
12345678901234567890123456789012123456789012345678901
12345678901234567890123456789012123456789012345678901
vation. Although many people believe in ESP, scientists 2
2345678901234567890123456789012123456789012345678901
2
1
12345678901234567890123456789012123456789012345678901
have found no empirical evidence of its existence. 2
2
12345678901234567890123456789012123456789012345678901
2
empiricism (noun).
12345678901234567890123456789012123456789012345678901
2
12345678901234567890123456789012123456789012345678901
2345678901234567890123456789012123456789012345678901
2
12345678901234567890123456789012123456789012345678901
emulate (verb) To imitate or copy. The British band Oasis is 2
12345678901234567890123456789012123456789012345678901
12345678901234567890123456789012123456789012345678901
quite open about their desire to emulate their idols, the 2
2
12345678901234567890123456789012123456789012345678901
2
12345678901234567890123456789012123456789012345678901
Beatles.
emulation
(noun).
12345678901234567890123456789012123456789012345678901 2
2
1
2
12345678901234567890123456789012123456789012345678901
encomium
(noun)
A
formal
expression
of
praise.
For
many
2
12345678901234567890123456789012123456789012345678901
2
12345678901234567890123456789012123456789012345678901
filmmakers,
winning
the
Palm
dOr
at
the
Cannes
Film
2345678901234567890123456789012123456789012345678901
12345678901234567890123456789012123456789012345678901 2
2
12345678901234567890123456789012123456789012345678901
Festival is considered the highest encomium.
2
12345678901234567890123456789012123456789012345678901
12345678901234567890123456789012123456789012345678901 2
12345678901234567890123456789012123456789012345678901
enervate (verb) To reduce the energy or strength of someone 2
2
12345678901234567890123456789012123456789012345678901
12345678901234567890123456789012123456789012345678901
or something. The stress of the operation left her feeling 2
2
12345678901234567890123456789012123456789012345678901
enervated for about two weeks. enervation (noun).
2
12345678901234567890123456789012123456789012345678901
2
1
2345678901234567890123456789012123456789012345678901
12345678901234567890123456789012123456789012345678901
engender (verb) To produce, to cause. Countless disagree- 2
2
12345678901234567890123456789012123456789012345678901
2
1
ments
over
the
proper
use
of
national
forests
and
12345678901234567890123456789012123456789012345678901 2
12345678901234567890123456789012123456789012345678901
parklands have engendered feelings of hostility between 2
2
12345678901234567890123456789012123456789012345678901
2
12345678901234567890123456789012123456789012345678901
ranchers
and
environmentalists.
12345678901234567890123456789012123456789012345678901 2
2
12345678901234567890123456789012123456789012345678901
2
12345678901234567890123456789012123456789012345678901
enhance
(verb)
To
improve
in
value
or
quality.
New
kitchen
2345678901234567890123456789012123456789012345678901
12345678901234567890123456789012123456789012345678901 2
12345678901234567890123456789012123456789012345678901
appliances will enhance your house and increase the 2
2
1
12345678901234567890123456789012123456789012345678901
amount of money youll make when you sell it. 2
12345678901234567890123456789012123456789012345678901 2
2
12345678901234567890123456789012123456789012345678901
enhancement (noun).
2
12345678901234567890123456789012123456789012345678901
2
12345678901234567890123456789012123456789012345678901
enigmatic (adjective) Puzzling, mysterious. Alain Resnais 2
12345678901234567890123456789012123456789012345678901
2
12345678901234567890123456789012123456789012345678901
enigmatic film Last Year at Marienbad sets up a puzzle 2
12345678901234567890123456789012123456789012345678901
2
12345678901234567890123456789012123456789012345678901
that is never resolved: A man meets a woman at a hotel 2
12345678901234567890123456789012123456789012345678901
12345678901234567890123456789012123456789012345678901
and believes he once had an affair with her-or did he? 2
2
12345678901234567890123456789012123456789012345678901
2345678901234567890123456789012123456789012345678901
2
12345678901234567890123456789012123456789012345678901
enigma (noun).
2
12345678901234567890123456789012123456789012345678901
12345678901234567890123456789012123456789012345678901 2
12345678901234567890123456789012123456789012345678901
enmity (noun) Hatred, hostility, ill will. Long-standing 2
2
1
2345678901234567890123456789012123456789012345678901
12345678901234567890123456789012123456789012345678901
enmity, like that between the Protestants and Catholics 2
2
1
2
12345678901234567890123456789012123456789012345678901
in
Northern
Ireland,
is
difficult
to
overcome.
12345678901234567890123456789012123456789012345678901 2
2
12345678901234567890123456789012123456789012345678901
12345678901234567890123456789012123456789012345678901 2
12345678901234567890123456789012123456789012345678901 2
12345678901234567890123456789012123456789012345678901 2
2
186 12345678901234567890123456789012123456789012345678901
123456789012345678901234567890121234567890123456789012

www.petersons.com

http://www.xtremepapers.net

Chapter 5: Verbal Review

Take It to the Next Level

123456789012345678901234567890121234567890123456789012
12345678901234567890123456789012123456789012345678901 2
2
12345678901234567890123456789012123456789012345678901
ensure (verb) To make certain; to guarantee. In order to 2
12345678901234567890123456789012123456789012345678901
2
12345678901234567890123456789012123456789012345678901
ensure a sufficient crop of programmers and engineers 2
12345678901234567890123456789012123456789012345678901
2
12345678901234567890123456789012123456789012345678901
for the future, the United States needs to raise the quality 2
12345678901234567890123456789012123456789012345678901
2345678901234567890123456789012123456789012345678901
2
1
of its math and science schooling.
2
12345678901234567890123456789012123456789012345678901
2
12345678901234567890123456789012123456789012345678901
2
12345678901234567890123456789012123456789012345678901
2
12345678901234567890123456789012123456789012345678901
2345678901234567890123456789012123456789012345678901
2
1
Latin aequus 5 equal. Also found in English equality, equanimity,
2
12345678901234567890123456789012123456789012345678901
Word
Origin and equation.
2
12345678901234567890123456789012123456789012345678901
2
12345678901234567890123456789012123456789012345678901
2
12345678901234567890123456789012123456789012345678901
equable
(adjective)
Steady,
uniform.
While
many
people
2345678901234567890123456789012123456789012345678901
2
1
2
12345678901234567890123456789012123456789012345678901
cant
see
how
Helena
could
possibly
be
attracted
to
2
12345678901234567890123456789012123456789012345678901
2
12345678901234567890123456789012123456789012345678901
Boring
Bruno,
his
equable
nature
is
the
perfect
2
12345678901234567890123456789012123456789012345678901
2345678901234567890123456789012123456789012345678901
2
1
complement to her volatile personality.
2
12345678901234567890123456789012123456789012345678901
2
12345678901234567890123456789012123456789012345678901
equivocate (verb) To use misleading or intentionally confus- 2
12345678901234567890123456789012123456789012345678901
2
12345678901234567890123456789012123456789012345678901
ing language. When Pedro pressed Renee for an answer 2
12345678901234567890123456789012123456789012345678901
2
12345678901234567890123456789012123456789012345678901
to his marriage proposal, she equivocated by saying, 2
12345678901234567890123456789012123456789012345678901
2
12345678901234567890123456789012123456789012345678901
Ive just got to know when your Mercedes will be out 2
12345678901234567890123456789012123456789012345678901
2345678901234567890123456789012123456789012345678901
12345678901234567890123456789012123456789012345678901
of the shop! equivocal (adjective), equivocation (noun). 2
2
1
2
12345678901234567890123456789012123456789012345678901
2
12345678901234567890123456789012123456789012345678901
epicure
(noun)
Someone
who
appreciates
fine
wine
and
fine
2
12345678901234567890123456789012123456789012345678901
2345678901234567890123456789012123456789012345678901
12345678901234567890123456789012123456789012345678901
food, a gourmand. M.F.K. Fisher, a famous epicure, 2
2
12345678901234567890123456789012123456789012345678901
2
12345678901234567890123456789012123456789012345678901
begins
her
book
The
Gastronomical
Me
by
saying,
12345678901234567890123456789012123456789012345678901 22
12345678901234567890123456789012123456789012345678901
There is a communion of more than bodies when bread 2
12345678901234567890123456789012123456789012345678901
2
12345678901234567890123456789012123456789012345678901
is broken and wine is drunk. epicurean (adjective).
2
12345678901234567890123456789012123456789012345678901
12345678901234567890123456789012123456789012345678901 22
1
epithet (noun) Term or words used to characterize a person
2
12345678901234567890123456789012123456789012345678901
or thing, often in a disparaging way. In her recorded 2
12345678901234567890123456789012123456789012345678901
2
12345678901234567890123456789012123456789012345678901
phone conversations with Linda Tripp, Monica Lewinsky 2
12345678901234567890123456789012123456789012345678901
2
12345678901234567890123456789012123456789012345678901
is said to have referred to President Clinton by a number 2
12345678901234567890123456789012123456789012345678901
2
12345678901234567890123456789012123456789012345678901
of epithets, including The Creep and The Big He. 2
12345678901234567890123456789012123456789012345678901
2
12345678901234567890123456789012123456789012345678901
epithetical (adjective).
2
12345678901234567890123456789012123456789012345678901
2345678901234567890123456789012123456789012345678901
12345678901234567890123456789012123456789012345678901 22
12345678901234567890123456789012123456789012345678901 2
1
2
12345678901234567890123456789012123456789012345678901
Word
Origin Latin radix 5 root. Also found in English radical.
2
12345678901234567890123456789012123456789012345678901
2345678901234567890123456789012123456789012345678901
12345678901234567890123456789012123456789012345678901 22
12345678901234567890123456789012123456789012345678901
eradicate (verb) To destroy completely. American society
12345678901234567890123456789012123456789012345678901 22
12345678901234567890123456789012123456789012345678901
has failed to eradicate racism, although some of its worst
2
1
effects have been reduced. eradication (noun).
2345678901234567890123456789012123456789012345678901
2
12345678901234567890123456789012123456789012345678901
12345678901234567890123456789012123456789012345678901 22
12345678901234567890123456789012123456789012345678901
erudition (noun) Extensive knowledge, usually acquired 2
12345678901234567890123456789012123456789012345678901
1
from books. When Dorothea first saw Mr. Casaubons 2
2
12345678901234567890123456789012123456789012345678901
2
12345678901234567890123456789012123456789012345678901
voluminous
library
she
was
awed,
but
after
their
2
12345678901234567890123456789012123456789012345678901
2
12345678901234567890123456789012123456789012345678901
marriage
she
quickly
realized
that
erudition
is
no
12345678901234567890123456789012123456789012345678901 2
2345678901234567890123456789012123456789012345678901
2
12345678901234567890123456789012123456789012345678901
substitute for originality. erudite (adjective).
2
1
2
12345678901234567890123456789012123456789012345678901
12345678901234567890123456789012123456789012345678901 2
2
12345678901234567890123456789012123456789012345678901
12345678901234567890123456789012123456789012345678901 2
12345678901234567890123456789012123456789012345678901 2
12345678901234567890123456789012123456789012345678901 2
12345678901234567890123456789012123456789012345678901 2 187
123456789012345678901234567890121234567890123456789012

http://www.xtremepapers.net

Part III: Section Review

123456789012345678901234567890121234567890123456789012
12345678901234567890123456789012123456789012345678901 2
2
12345678901234567890123456789012123456789012345678901
esoterica (noun) Items of interest to a select group. The fish 2
12345678901234567890123456789012123456789012345678901
2
12345678901234567890123456789012123456789012345678901
symposium at St. Antonys College in Oxford explored 2
12345678901234567890123456789012123456789012345678901
2
12345678901234567890123456789012123456789012345678901
all manner of esoterica relating to fish, as is evidenced in 2
12345678901234567890123456789012123456789012345678901
2345678901234567890123456789012123456789012345678901
2
1
presentations such as The Buoyant Slippery Lipids of 2
12345678901234567890123456789012123456789012345678901
12345678901234567890123456789012123456789012345678901
the Escolar and Orange Roughy or Food on Board 2
2
12345678901234567890123456789012123456789012345678901
2
12345678901234567890123456789012123456789012345678901
Whale
Shipsfrom
the
Inedible
to
the
Incredible.
2345678901234567890123456789012123456789012345678901
2
1
2
12345678901234567890123456789012123456789012345678901
esoteric
(adjective).
2
12345678901234567890123456789012123456789012345678901
2
12345678901234567890123456789012123456789012345678901
2
12345678901234567890123456789012123456789012345678901
espouse
(verb)
To
take
up
as
a
cause;
to
adopt.
No
politician
2345678901234567890123456789012123456789012345678901
2
1
2
12345678901234567890123456789012123456789012345678901
in
America
today
will
openly
espouse
racism,
although
2
12345678901234567890123456789012123456789012345678901
2
12345678901234567890123456789012123456789012345678901
some
behave
and
speak
in
racially
prejudiced
ways.
2
12345678901234567890123456789012123456789012345678901
2345678901234567890123456789012123456789012345678901
2
1
12345678901234567890123456789012123456789012345678901
estimable (adjective) Worthy of esteem and admiration. 2
2
12345678901234567890123456789012123456789012345678901
After a tragic fire raged through Malden Mills, the 2
12345678901234567890123456789012123456789012345678901
2
12345678901234567890123456789012123456789012345678901
estimable mill owner, Aaron Feuerstein, restarted opera- 2
12345678901234567890123456789012123456789012345678901
2
12345678901234567890123456789012123456789012345678901
tions and rebuilt the company within just one month. 2
12345678901234567890123456789012123456789012345678901
2
12345678901234567890123456789012123456789012345678901
esteem (noun).
2
12345678901234567890123456789012123456789012345678901
2345678901234567890123456789012123456789012345678901
12345678901234567890123456789012123456789012345678901 2
1
eulogy (noun) A formal tribute usually delivered at a 2
2
12345678901234567890123456789012123456789012345678901
2
12345678901234567890123456789012123456789012345678901
funeral.
Most
people
in
Britain
applauded
Lord
Earl
2
12345678901234567890123456789012123456789012345678901
2345678901234567890123456789012123456789012345678901
12345678901234567890123456789012123456789012345678901
Spencers eulogy for Princess Diana, not only as a warm 2
2
12345678901234567890123456789012123456789012345678901
2
12345678901234567890123456789012123456789012345678901
tribute
to
his
sister
Diana
but
also
as
a
biting
indictment
12345678901234567890123456789012123456789012345678901 2
2
12345678901234567890123456789012123456789012345678901
of the Royal Family. eulogize (verb).
2
12345678901234567890123456789012123456789012345678901
12345678901234567890123456789012123456789012345678901 2
12345678901234567890123456789012123456789012345678901
euphemism (noun) An agreeable expression that is substi- 2
2
12345678901234567890123456789012123456789012345678901
1
tuted for an offensive one. Some of the more creative 2
2345678901234567890123456789012123456789012345678901
12345678901234567890123456789012123456789012345678901 2
euphemisms for layoffs in current use are: release of 2
12345678901234567890123456789012123456789012345678901
2
1
resources, involuntary severance, strengthening 2
12345678901234567890123456789012123456789012345678901
2
12345678901234567890123456789012123456789012345678901
global effectiveness, and career transition program. 2
12345678901234567890123456789012123456789012345678901
2
12345678901234567890123456789012123456789012345678901
euphemistic (adjective).
2
12345678901234567890123456789012123456789012345678901
2
12345678901234567890123456789012123456789012345678901
2
12345678901234567890123456789012123456789012345678901
2345678901234567890123456789012123456789012345678901
12345678901234567890123456789012123456789012345678901 2
2
12345678901234567890123456789012123456789012345678901
acer 5 sharp. Also found in English acerbity, acrid, and
2
1 Origin Latin
Word
acrimonious.
12345678901234567890123456789012123456789012345678901 2
12345678901234567890123456789012123456789012345678901 2
12345678901234567890123456789012123456789012345678901
exacerbate (verb) To make worse or more severe. The roads 2
2
12345678901234567890123456789012123456789012345678901
2
12345678901234567890123456789012123456789012345678901
in
our
town
already
have
too
much
traffic;
building
a
2
12345678901234567890123456789012123456789012345678901
2
12345678901234567890123456789012123456789012345678901
new
shopping
mall
will
exacerbate
the
problem.
2345678901234567890123456789012123456789012345678901
12345678901234567890123456789012123456789012345678901 2
12345678901234567890123456789012123456789012345678901 2
12345678901234567890123456789012123456789012345678901
excoriation (noun) The act of condemning someone with 2
2
12345678901234567890123456789012123456789012345678901
harsh words. In the small office we shared, it was 2
1
2345678901234567890123456789012123456789012345678901
2
12345678901234567890123456789012123456789012345678901
painful to hear my bosss constant excoriation of his 2
12345678901234567890123456789012123456789012345678901
2
12345678901234567890123456789012123456789012345678901
assistant for the smallest faultsa misdirected letter, an 2
12345678901234567890123456789012123456789012345678901
2
1
unclear phone message, or even a tepid cup of coffee. 2
12345678901234567890123456789012123456789012345678901
2
12345678901234567890123456789012123456789012345678901
excoriate (verb).
2
12345678901234567890123456789012123456789012345678901
12345678901234567890123456789012123456789012345678901 2
2
12345678901234567890123456789012123456789012345678901
12345678901234567890123456789012123456789012345678901 2
12345678901234567890123456789012123456789012345678901 2
12345678901234567890123456789012123456789012345678901 2
2
188 12345678901234567890123456789012123456789012345678901
123456789012345678901234567890121234567890123456789012

www.petersons.com

http://www.xtremepapers.net

Chapter 5: Verbal Review

Take It to the Next Level

123456789012345678901234567890121234567890123456789012
12345678901234567890123456789012123456789012345678901 2
2
12345678901234567890123456789012123456789012345678901
exculpate (verb) To free from blame or guilt. When 2
12345678901234567890123456789012123456789012345678901
2
12345678901234567890123456789012123456789012345678901
someone else confessed to the crime, the previous 2
12345678901234567890123456789012123456789012345678901
2
12345678901234567890123456789012123456789012345678901
suspect was exculpated. exculpation (noun), exculpa- 2
12345678901234567890123456789012123456789012345678901
2345678901234567890123456789012123456789012345678901
2
1
tory (adjective).
2
12345678901234567890123456789012123456789012345678901
2
12345678901234567890123456789012123456789012345678901
2
12345678901234567890123456789012123456789012345678901
executor
(noun)
The
person
appointed
to
execute
some2
12345678901234567890123456789012123456789012345678901
2345678901234567890123456789012123456789012345678901
1
ones will. As the executor of his Aunt Idas will, Phil 2
2
12345678901234567890123456789012123456789012345678901
2
12345678901234567890123456789012123456789012345678901
must
deal
with
squabbling
relatives,
conniving
lawyers,
2
12345678901234567890123456789012123456789012345678901
2
12345678901234567890123456789012123456789012345678901
and
the
ruinous
state
of
Idas
house.
2345678901234567890123456789012123456789012345678901
2
1
2
12345678901234567890123456789012123456789012345678901
12345678901234567890123456789012123456789012345678901
exigent (adjective) Urgent, requiring immediate attention. A 2
2
12345678901234567890123456789012123456789012345678901
12345678901234567890123456789012123456789012345678901
two-year-old is likely to behave as if her every demand is 2
2345678901234567890123456789012123456789012345678901
2
1
12345678901234567890123456789012123456789012345678901
exigent, even if it involves simply retrieving a beloved 2
2
12345678901234567890123456789012123456789012345678901
2
teddy bear from under the couch. exigency (noun).
12345678901234567890123456789012123456789012345678901
2
12345678901234567890123456789012123456789012345678901
2345678901234567890123456789012123456789012345678901
2
12345678901234567890123456789012123456789012345678901
expedient (adjective) Providing an immediate advantage or 2
12345678901234567890123456789012123456789012345678901
12345678901234567890123456789012123456789012345678901
serving ones immediate self-interest. When the passen- 2
2
12345678901234567890123456789012123456789012345678901
2
12345678901234567890123456789012123456789012345678901
ger
next
to
her
was
strafed
by
a
bullet,
Sharon
chose
the
12345678901234567890123456789012123456789012345678901 22
1
most expedient means to stop the bleeding; she whipped 2
12345678901234567890123456789012123456789012345678901
12345678901234567890123456789012123456789012345678901
off her pantyhose and made an impromptu, but effec- 2
2
12345678901234567890123456789012123456789012345678901
2345678901234567890123456789012123456789012345678901
2
12345678901234567890123456789012123456789012345678901
tive, tourniquet. expediency (noun).
2
12345678901234567890123456789012123456789012345678901
12345678901234567890123456789012123456789012345678901 22
12345678901234567890123456789012123456789012345678901
extant (adjective) Currently in existence. Of the seven 2
12345678901234567890123456789012123456789012345678901
12345678901234567890123456789012123456789012345678901
ancient Wonders of the World, only the pyramids of 2
2
12345678901234567890123456789012123456789012345678901
2
12345678901234567890123456789012123456789012345678901
Egypt are still extant.
12345678901234567890123456789012123456789012345678901 22
1
2
12345678901234567890123456789012123456789012345678901
2
12345678901234567890123456789012123456789012345678901
Latin
tenere
5
to
hold.
Also
found
in
English
retain,
tenable,
2
12345678901234567890123456789012123456789012345678901
Word
Origin
tenant, tenet, and tenure.
2
12345678901234567890123456789012123456789012345678901
2
12345678901234567890123456789012123456789012345678901
2345678901234567890123456789012123456789012345678901
12345678901234567890123456789012123456789012345678901
extenuate (verb) To make less serious. Karens guilt is 2
2
12345678901234567890123456789012123456789012345678901
2
1
extenuated
by
the
fact
that
she
was
only
12
when
she
2
12345678901234567890123456789012123456789012345678901
2
12345678901234567890123456789012123456789012345678901
committed
the
theft.
extenuating
(adjective),
extenua2345678901234567890123456789012123456789012345678901
12345678901234567890123456789012123456789012345678901 22
12345678901234567890123456789012123456789012345678901
tion (noun).
2
1
12345678901234567890123456789012123456789012345678901 2
12345678901234567890123456789012123456789012345678901
extol (verb) To greatly praise. At the party convention, one 2
2
12345678901234567890123456789012123456789012345678901
2
12345678901234567890123456789012123456789012345678901
speaker
after
another
took
to
the
podium
to
extol
the
2
12345678901234567890123456789012123456789012345678901
2
12345678901234567890123456789012123456789012345678901
virtues of their candidate for the presidency.
12345678901234567890123456789012123456789012345678901 2
2345678901234567890123456789012123456789012345678901
2
12345678901234567890123456789012123456789012345678901
extraneous (adjective) Irrelevant, nonessential. One review 2
12345678901234567890123456789012123456789012345678901
2
12345678901234567890123456789012123456789012345678901
of the new Chekhov biography said the author had 2
12345678901234567890123456789012123456789012345678901
2
1
bogged down the book with far too many extraneous 2
12345678901234567890123456789012123456789012345678901
2
12345678901234567890123456789012123456789012345678901
details, such as the dates of Chekhovs bouts of diarrhea. 2
12345678901234567890123456789012123456789012345678901
2
12345678901234567890123456789012123456789012345678901
12345678901234567890123456789012123456789012345678901 2
2345678901234567890123456789012123456789012345678901
12345678901234567890123456789012123456789012345678901 22
1
2
12345678901234567890123456789012123456789012345678901
12345678901234567890123456789012123456789012345678901 2
2
12345678901234567890123456789012123456789012345678901
12345678901234567890123456789012123456789012345678901 2
12345678901234567890123456789012123456789012345678901 2
12345678901234567890123456789012123456789012345678901 2
12345678901234567890123456789012123456789012345678901 2 189
123456789012345678901234567890121234567890123456789012

http://www.xtremepapers.net

Part III: Section Review

123456789012345678901234567890121234567890123456789012
12345678901234567890123456789012123456789012345678901 2
2
12345678901234567890123456789012123456789012345678901
extrapolate (verb) To deduce from something known, to 2
12345678901234567890123456789012123456789012345678901
2
12345678901234567890123456789012123456789012345678901
infer. Meteorologists were able to use old weather 2
12345678901234567890123456789012123456789012345678901
2
12345678901234567890123456789012123456789012345678901
records to extrapolate backward and compile lists of El 2
12345678901234567890123456789012123456789012345678901
2345678901234567890123456789012123456789012345678901
2
1
Nio years and their effects over the last century. 2
12345678901234567890123456789012123456789012345678901
2
12345678901234567890123456789012123456789012345678901
extrapolation (noun).
2
12345678901234567890123456789012123456789012345678901
2
12345678901234567890123456789012123456789012345678901
2345678901234567890123456789012123456789012345678901
1
extricate (verb) To free from a difficult or complicated 2
2
12345678901234567890123456789012123456789012345678901
2
12345678901234567890123456789012123456789012345678901
situation.
Much
of
the
humor
in
the
TV
show
I
Love
2
12345678901234567890123456789012123456789012345678901
2
12345678901234567890123456789012123456789012345678901
Lucy
comes
in
watching
Lucy
try
to
extricate
herself
2345678901234567890123456789012123456789012345678901
2
1
2
12345678901234567890123456789012123456789012345678901
from
the
problems
she
creates
by
fibbing
or
trickery.
2
12345678901234567890123456789012123456789012345678901
2
12345678901234567890123456789012123456789012345678901
extricable
(adjective).
2
12345678901234567890123456789012123456789012345678901
2345678901234567890123456789012123456789012345678901
2
1
12345678901234567890123456789012123456789012345678901
facetious (adjective) Humorous in a mocking way; not 2
2
12345678901234567890123456789012123456789012345678901
serious. French composer Erik Satie often concealed his 2
12345678901234567890123456789012123456789012345678901
2
12345678901234567890123456789012123456789012345678901
serious artistic intent by giving his works facetious titles 2
12345678901234567890123456789012123456789012345678901
2
12345678901234567890123456789012123456789012345678901
such as Three Pieces in the Shape of a Pear.
2
12345678901234567890123456789012123456789012345678901
2
12345678901234567890123456789012123456789012345678901
2
12345678901234567890123456789012123456789012345678901
facilitate
(verb)
To
make
easier
or
to
moderate.
When
the
2345678901234567890123456789012123456789012345678901
12345678901234567890123456789012123456789012345678901 2
1
issue of racism reared its ugly head, the company 2
2
12345678901234567890123456789012123456789012345678901
2
12345678901234567890123456789012123456789012345678901
brought
in
a
consultant
to
facilitate
a
discussion
of
2
12345678901234567890123456789012123456789012345678901
2345678901234567890123456789012123456789012345678901
12345678901234567890123456789012123456789012345678901
diversity in the workplace. facile (adjective), facility 2
2
12345678901234567890123456789012123456789012345678901
2
12345678901234567890123456789012123456789012345678901
(noun).
12345678901234567890123456789012123456789012345678901 2
12345678901234567890123456789012123456789012345678901 2
12345678901234567890123456789012123456789012345678901
fallacy (noun) An error in fact or logic. Its a fallacy to think 2
2
12345678901234567890123456789012123456789012345678901
12345678901234567890123456789012123456789012345678901
that natural means healthful; after all, the deadly 2
12345678901234567890123456789012123456789012345678901 2
1
poison arsenic is completely natural. fallacious (adjec- 2
2
12345678901234567890123456789012123456789012345678901
2
tive).
12345678901234567890123456789012123456789012345678901
12345678901234567890123456789012123456789012345678901 2
2
12345678901234567890123456789012123456789012345678901
fatuous (adjective) Inanely foolish; silly. Once backstage, 2
12345678901234567890123456789012123456789012345678901
2345678901234567890123456789012123456789012345678901
12345678901234567890123456789012123456789012345678901
Elizabeth showered the opera singer with fatuous praise 2
2
12345678901234567890123456789012123456789012345678901
2
1
and
embarrassing
confessions,
which
he
clearly
had
no
2
12345678901234567890123456789012123456789012345678901
2
12345678901234567890123456789012123456789012345678901
interest
in
hearing.
2345678901234567890123456789012123456789012345678901
12345678901234567890123456789012123456789012345678901 2
12345678901234567890123456789012123456789012345678901 2
1
fawn (verb) To flatter in a particularly subservient manner. 2
2
12345678901234567890123456789012123456789012345678901
2
12345678901234567890123456789012123456789012345678901
Mildly
disgusted,
Pedro
stood
alone
at
the
bar
and
2345678901234567890123456789012123456789012345678901
12345678901234567890123456789012123456789012345678901 2
12345678901234567890123456789012123456789012345678901
watched Renee fawn over the heir to the Fabco Surgical 2
2
12345678901234567890123456789012123456789012345678901
2
12345678901234567890123456789012123456789012345678901
Appliances fortune.
2
1
2345678901234567890123456789012123456789012345678901
2
12345678901234567890123456789012123456789012345678901
feckless (adjective) Weak and ineffective; irresponsible. Our 2
12345678901234567890123456789012123456789012345678901
2
12345678901234567890123456789012123456789012345678901
co-op board president is a feckless fellow who has let 2
12345678901234567890123456789012123456789012345678901
2
1
much-needed repairs go unattended while our mainte- 2
12345678901234567890123456789012123456789012345678901
2
12345678901234567890123456789012123456789012345678901
nance fees continue to rise.
2
12345678901234567890123456789012123456789012345678901
2
12345678901234567890123456789012123456789012345678901
2
12345678901234567890123456789012123456789012345678901
feint
(noun)
A
bluff;
a
mock
blow.
It
didnt
take
us
long
to
2345678901234567890123456789012123456789012345678901
12345678901234567890123456789012123456789012345678901 2
1
realize that Gabys tears and stomachaches were all a 2
2
12345678901234567890123456789012123456789012345678901
2
12345678901234567890123456789012123456789012345678901
feint,
since
they
appeared
so
regularly
at
her
bedtime.
2
12345678901234567890123456789012123456789012345678901
2345678901234567890123456789012123456789012345678901
2
1
12345678901234567890123456789012123456789012345678901 2
12345678901234567890123456789012123456789012345678901 2
2
190 12345678901234567890123456789012123456789012345678901
123456789012345678901234567890121234567890123456789012

www.petersons.com

http://www.xtremepapers.net

Chapter 5: Verbal Review

Take It to the Next Level

123456789012345678901234567890121234567890123456789012
12345678901234567890123456789012123456789012345678901 2
2
12345678901234567890123456789012123456789012345678901
ferret (verb) To bring to light by an extensive search. With 2
12345678901234567890123456789012123456789012345678901
2
12345678901234567890123456789012123456789012345678901
his repeated probing and questions, Fritz was able to 2
12345678901234567890123456789012123456789012345678901
2
12345678901234567890123456789012123456789012345678901
ferret out the location of Myrnas safe deposit box.
2
12345678901234567890123456789012123456789012345678901
2345678901234567890123456789012123456789012345678901
2
1
2
12345678901234567890123456789012123456789012345678901
finesse
(noun)
Skillful
maneuvering;
delicate
workmanship.
2
12345678901234567890123456789012123456789012345678901
2
12345678901234567890123456789012123456789012345678901
With
her
usual
finesse,
Charmaine
gently
persuaded
the
2
12345678901234567890123456789012123456789012345678901
2345678901234567890123456789012123456789012345678901
1
Duncans not to install a motorized Santa and sleigh on 2
2
12345678901234567890123456789012123456789012345678901
2
12345678901234567890123456789012123456789012345678901
their
front
lawn.
2
12345678901234567890123456789012123456789012345678901
2
12345678901234567890123456789012123456789012345678901
2345678901234567890123456789012123456789012345678901
1
florid (adjective) Flowery, fancy; reddish. The grand ball- 2
2
12345678901234567890123456789012123456789012345678901
12345678901234567890123456789012123456789012345678901
room was decorated in a florid style. Years of heavy 2
2
12345678901234567890123456789012123456789012345678901
2
12345678901234567890123456789012123456789012345678901
drinking had given him a florid complexion.
2345678901234567890123456789012123456789012345678901
2
1
2
12345678901234567890123456789012123456789012345678901
2
12345678901234567890123456789012123456789012345678901
2
12345678901234567890123456789012123456789012345678901
Latin
fluere
5
to
flow.
Also
found
in
English
affluent,
effluvia,
fluid,
2
12345678901234567890123456789012123456789012345678901
Word
Origin
and
influx.
2345678901234567890123456789012123456789012345678901
12345678901234567890123456789012123456789012345678901 22
12345678901234567890123456789012123456789012345678901 2
12345678901234567890123456789012123456789012345678901
flourish (noun) An extraneous embellishment; a dramatic 2
12345678901234567890123456789012123456789012345678901
12345678901234567890123456789012123456789012345678901
gesture. The napkin rings made out of intertwined ferns 2
2
12345678901234567890123456789012123456789012345678901
2
1
and
flowers
were
just
the
kind
of
flourish
one
would
2
12345678901234567890123456789012123456789012345678901
2
12345678901234567890123456789012123456789012345678901
expect
from
Carol,
a
slavish
follower
of
Martha
Stewart.
2
12345678901234567890123456789012123456789012345678901
2345678901234567890123456789012123456789012345678901
12345678901234567890123456789012123456789012345678901 22
12345678901234567890123456789012123456789012345678901
fluctuation (noun) A shifting back and forth. Investment 2
12345678901234567890123456789012123456789012345678901
12345678901234567890123456789012123456789012345678901
analysts predict fluctuations in the Dow Jones Industrial 2
2
12345678901234567890123456789012123456789012345678901
12345678901234567890123456789012123456789012345678901
Average due to the instability of the value of the dollar. 2
2
12345678901234567890123456789012123456789012345678901
2
12345678901234567890123456789012123456789012345678901
fluctuate (verb).
12345678901234567890123456789012123456789012345678901 22
1
foil (verb) To thwart or frustrate. I was certain that Jerrys 2
12345678901234567890123456789012123456789012345678901
2
12345678901234567890123456789012123456789012345678901
tendency to insert himself into everyones conversations 2
12345678901234567890123456789012123456789012345678901
2
12345678901234567890123456789012123456789012345678901
would foil my chances to have a private word with 2
12345678901234567890123456789012123456789012345678901
2345678901234567890123456789012123456789012345678901
2
12345678901234567890123456789012123456789012345678901
Helen.
2
12345678901234567890123456789012123456789012345678901
2
1
2
12345678901234567890123456789012123456789012345678901
foment
(verb)
To
rouse
or
incite.
The
petty
tyrannies
and
2
12345678901234567890123456789012123456789012345678901
2345678901234567890123456789012123456789012345678901
12345678901234567890123456789012123456789012345678901
indignities inflicted on the workers by upper manage- 2
2
12345678901234567890123456789012123456789012345678901
2
1
ment
helped
foment
the
walkout
at
the
meat-processing
12345678901234567890123456789012123456789012345678901 2
2
12345678901234567890123456789012123456789012345678901
plant.
2
12345678901234567890123456789012123456789012345678901
2
12345678901234567890123456789012123456789012345678901
12345678901234567890123456789012123456789012345678901
forestall (verb) To hinder or prevent by taking action in 2
2
12345678901234567890123456789012123456789012345678901
12345678901234567890123456789012123456789012345678901
advance. The pilots calm, levelheaded demeanor during 2
2345678901234567890123456789012123456789012345678901
2
12345678901234567890123456789012123456789012345678901
the turbulence forestalled any hysteria among the 2
12345678901234567890123456789012123456789012345678901
2
12345678901234567890123456789012123456789012345678901
passengers of Flight 268.
2
12345678901234567890123456789012123456789012345678901
2
1
2345678901234567890123456789012123456789012345678901
12345678901234567890123456789012123456789012345678901
fortuitous (adjective) Lucky, fortunate. Although the mayor 2
2
12345678901234567890123456789012123456789012345678901
2
12345678901234567890123456789012123456789012345678901
claimed
credit
for
the
falling
crime
rate,
it
was
really
12345678901234567890123456789012123456789012345678901 22
1
caused by a series of fortuitous accidents.
2
12345678901234567890123456789012123456789012345678901
12345678901234567890123456789012123456789012345678901 2
2
12345678901234567890123456789012123456789012345678901
12345678901234567890123456789012123456789012345678901 2
2
12345678901234567890123456789012123456789012345678901
12345678901234567890123456789012123456789012345678901 2
12345678901234567890123456789012123456789012345678901 2
12345678901234567890123456789012123456789012345678901 2
12345678901234567890123456789012123456789012345678901 2 191
123456789012345678901234567890121234567890123456789012

http://www.xtremepapers.net

Part III: Section Review

123456789012345678901234567890121234567890123456789012
12345678901234567890123456789012123456789012345678901 2
2
12345678901234567890123456789012123456789012345678901
foster (verb) To nurture or encourage. The white-water 2
12345678901234567890123456789012123456789012345678901
2
12345678901234567890123456789012123456789012345678901
rafting trip was supposed to foster creative problem 2
12345678901234567890123456789012123456789012345678901
2
12345678901234567890123456789012123456789012345678901
solving and teamwork between the account executives 2
12345678901234567890123456789012123456789012345678901
2345678901234567890123456789012123456789012345678901
2
1
and the creative staff at Apex Advertising Agency.
2
12345678901234567890123456789012123456789012345678901
2
12345678901234567890123456789012123456789012345678901
2
12345678901234567890123456789012123456789012345678901
fracas
(noun)
A
noisy
fight;
a
brawl.
As
Bill
approached
the
2
12345678901234567890123456789012123456789012345678901
2345678901234567890123456789012123456789012345678901
1
stadium ticket window, he was alarmed to see the fracas 2
2
12345678901234567890123456789012123456789012345678901
2
12345678901234567890123456789012123456789012345678901
that
had
broken
out
between
a
group
of
Giants
fans
and
2
12345678901234567890123456789012123456789012345678901
2
12345678901234567890123456789012123456789012345678901
a
man
wearing
a
Cowboys
jersey
and
helmet.
2345678901234567890123456789012123456789012345678901
2
1
2
12345678901234567890123456789012123456789012345678901
12345678901234567890123456789012123456789012345678901
functionary (noun) Someone holding office in a political 2
2
12345678901234567890123456789012123456789012345678901
12345678901234567890123456789012123456789012345678901
party or government. The man shaking hands with the 2
2345678901234567890123456789012123456789012345678901
2
1
12345678901234567890123456789012123456789012345678901
governor was a low-ranking Democratic Party function- 2
2
12345678901234567890123456789012123456789012345678901
2
ary who had worked to garner the Hispanic vote.
12345678901234567890123456789012123456789012345678901
2
12345678901234567890123456789012123456789012345678901
2345678901234567890123456789012123456789012345678901
2
12345678901234567890123456789012123456789012345678901
gainsay (verb) To contradict or oppose; deny, dispute. Dot 2
12345678901234567890123456789012123456789012345678901
12345678901234567890123456789012123456789012345678901
would gainsay her married sisters efforts to introduce 2
2
12345678901234567890123456789012123456789012345678901
2
12345678901234567890123456789012123456789012345678901
her
to
eligible
men
by
refusing
to
either
leave
her
ailing
12345678901234567890123456789012123456789012345678901 2
2
1
canary or give up her thrice-weekly bingo nights.
2
12345678901234567890123456789012123456789012345678901
2
12345678901234567890123456789012123456789012345678901
2
12345678901234567890123456789012123456789012345678901
garble
(verb)
To
distort
or
slur.
No
matter
how
much
money
2345678901234567890123456789012123456789012345678901
12345678901234567890123456789012123456789012345678901 2
12345678901234567890123456789012123456789012345678901
the Metropolitan Transit Authority spends on improv- 2
2
12345678901234567890123456789012123456789012345678901
12345678901234567890123456789012123456789012345678901
ing the subway trains, the public address system in 2
12345678901234567890123456789012123456789012345678901 2
12345678901234567890123456789012123456789012345678901
almost every station seems to garble each announce- 2
2
12345678901234567890123456789012123456789012345678901
2
12345678901234567890123456789012123456789012345678901
ment. garbled (adjective).
12345678901234567890123456789012123456789012345678901 2
2
1
garrulous (adjective) Annoyingly talkative. Claude pre- 2
12345678901234567890123456789012123456789012345678901
2
12345678901234567890123456789012123456789012345678901
tended to be asleep so he could avoid his garrulous 2
12345678901234567890123456789012123456789012345678901
2
12345678901234567890123456789012123456789012345678901
seatmate, a self-proclaimed expert on bonsai cultivation. 2
12345678901234567890123456789012123456789012345678901
2345678901234567890123456789012123456789012345678901
12345678901234567890123456789012123456789012345678901 2
12345678901234567890123456789012123456789012345678901 2
2
1
2
12345678901234567890123456789012123456789012345678901
Latin genus 5 type or kind; birth. Also found in English congenital,
2
Word
Origin genetic, genital, genre, genuine, and genus.
12345678901234567890123456789012123456789012345678901
2345678901234567890123456789012123456789012345678901
12345678901234567890123456789012123456789012345678901 2
12345678901234567890123456789012123456789012345678901 2
1
generic (adjective) General; having no brand name. Connie 2
2
12345678901234567890123456789012123456789012345678901
2
12345678901234567890123456789012123456789012345678901
tried
to
reduce
her
grocery
bills
by
religiously
clipping
2345678901234567890123456789012123456789012345678901
12345678901234567890123456789012123456789012345678901 2
2
12345678901234567890123456789012123456789012345678901
coupons and buying generic brands of most products.
2
12345678901234567890123456789012123456789012345678901
12345678901234567890123456789012123456789012345678901 2
1
gist (noun) The main point, the essence. Although they felt 2
2
12345678901234567890123456789012123456789012345678901
sympathy for the victims family, the jurors were won 2
12345678901234567890123456789012123456789012345678901
2
12345678901234567890123456789012123456789012345678901
over by the gist of the defenses argument: There was 2
12345678901234567890123456789012123456789012345678901
2
12345678901234567890123456789012123456789012345678901
insufficient evidence to convict.
2
12345678901234567890123456789012123456789012345678901
2
12345678901234567890123456789012123456789012345678901
2
12345678901234567890123456789012123456789012345678901
2
12345678901234567890123456789012123456789012345678901
12345678901234567890123456789012123456789012345678901 2
2345678901234567890123456789012123456789012345678901
12345678901234567890123456789012123456789012345678901 2
2
1
2
12345678901234567890123456789012123456789012345678901
12345678901234567890123456789012123456789012345678901 2
2
12345678901234567890123456789012123456789012345678901
12345678901234567890123456789012123456789012345678901 2
12345678901234567890123456789012123456789012345678901 2
12345678901234567890123456789012123456789012345678901 2
2
192 12345678901234567890123456789012123456789012345678901
123456789012345678901234567890121234567890123456789012

www.petersons.com

http://www.xtremepapers.net

Chapter 5: Verbal Review

Take It to the Next Level

123456789012345678901234567890121234567890123456789012
12345678901234567890123456789012123456789012345678901 2
2
12345678901234567890123456789012123456789012345678901
gouge (verb) To cut out, to scoop out with ones thumbs or 2
12345678901234567890123456789012123456789012345678901
2
12345678901234567890123456789012123456789012345678901
a sharp instrument; to overcharge, to cheat. Instead of 2
12345678901234567890123456789012123456789012345678901
2
12345678901234567890123456789012123456789012345678901
picking the lock with a credit card, the clumsy thieves 2
12345678901234567890123456789012123456789012345678901
2345678901234567890123456789012123456789012345678901
2
1
gouged a hole in my door. The consumer watchdog 2
12345678901234567890123456789012123456789012345678901
12345678901234567890123456789012123456789012345678901
group accused the clothing stores of gouging customers 2
2
12345678901234567890123456789012123456789012345678901
2
12345678901234567890123456789012123456789012345678901
with
high
prices.
2345678901234567890123456789012123456789012345678901
2
1
2
12345678901234567890123456789012123456789012345678901
2
12345678901234567890123456789012123456789012345678901
guile
(noun)
Deceit,
duplicity.
In
Margaret
Mitchells
Gone
2
12345678901234567890123456789012123456789012345678901
2
12345678901234567890123456789012123456789012345678901
With
the
Wind,
Scarlett
OHara
uses
her
guile
to
2345678901234567890123456789012123456789012345678901
2
1
2
12345678901234567890123456789012123456789012345678901
manipulate
two
men
and
then
is
matched
for
wits
by
a
2
12345678901234567890123456789012123456789012345678901
2
12345678901234567890123456789012123456789012345678901
third,
Rhett
Butler.
guileful
(adjective).
2
12345678901234567890123456789012123456789012345678901
2345678901234567890123456789012123456789012345678901
2
1
12345678901234567890123456789012123456789012345678901
gullible (adjective) Easily fooled. Terry was so gullible she 2
2
12345678901234567890123456789012123456789012345678901
actually believed Roberts stories of his connections to 2
12345678901234567890123456789012123456789012345678901
2
12345678901234567890123456789012123456789012345678901
the Czar and Czarina. gullibility (noun).
2
12345678901234567890123456789012123456789012345678901
2
12345678901234567890123456789012123456789012345678901
2
12345678901234567890123456789012123456789012345678901
hackneyed
(adjective)
Without
originality,
trite.
When
some2
12345678901234567890123456789012123456789012345678901
2
12345678901234567890123456789012123456789012345678901
one
invented
the
phrase,
No
pain,
no
gain,
it
was
2345678901234567890123456789012123456789012345678901
12345678901234567890123456789012123456789012345678901 22
1
clever and witty, but now it is so commonly heard that it 2
12345678901234567890123456789012123456789012345678901
2
12345678901234567890123456789012123456789012345678901
seems hackneyed.
2
12345678901234567890123456789012123456789012345678901
2345678901234567890123456789012123456789012345678901
12345678901234567890123456789012123456789012345678901 22
12345678901234567890123456789012123456789012345678901
harrow (verb) To cultivate with a harrow; to torment or 2
12345678901234567890123456789012123456789012345678901
12345678901234567890123456789012123456789012345678901
vex. During grade school, my sister was harrowed 2
2
12345678901234567890123456789012123456789012345678901
12345678901234567890123456789012123456789012345678901
mercilessly for being overweight. harrowing (adjective) 2
2
12345678901234567890123456789012123456789012345678901
12345678901234567890123456789012123456789012345678901
nerve-wracking, traumatic. Jon Krakauers harrowing 2
12345678901234567890123456789012123456789012345678901 22
1
book Into Thin Air chronicles the tragic consequences of
2
12345678901234567890123456789012123456789012345678901
2
leading groups of untrained climbers up Mt. Everest.
12345678901234567890123456789012123456789012345678901
12345678901234567890123456789012123456789012345678901 2
2
12345678901234567890123456789012123456789012345678901
haughty (adjective) Overly proud. The fashion model strode 2
12345678901234567890123456789012123456789012345678901
2345678901234567890123456789012123456789012345678901
12345678901234567890123456789012123456789012345678901
down the runway, her hips thrust forward and a haughty 2
2
12345678901234567890123456789012123456789012345678901
2
1
expression,
something
like
a
sneer,
on
her
face.
haughti2
12345678901234567890123456789012123456789012345678901
2
12345678901234567890123456789012123456789012345678901
ness
(noun).
2345678901234567890123456789012123456789012345678901
12345678901234567890123456789012123456789012345678901 22
12345678901234567890123456789012123456789012345678901 2
1
hierarchy (noun) A ranking of people, things, or ideas from 2
12345678901234567890123456789012123456789012345678901
12345678901234567890123456789012123456789012345678901
highest to lowest. A cabinet secretary ranks just below 2
2
12345678901234567890123456789012123456789012345678901
2
12345678901234567890123456789012123456789012345678901
the
president
and
vice
president
in
the
hierarchy
of
the
2
12345678901234567890123456789012123456789012345678901
12345678901234567890123456789012123456789012345678901
governments executive branch. hierarchical (adjective). 2
12345678901234567890123456789012123456789012345678901 2
2
12345678901234567890123456789012123456789012345678901
2
12345678901234567890123456789012123456789012345678901
2
12345678901234567890123456789012123456789012345678901
Greek
homos
5
same.
Also
found
in
English
homologous,
2
12345678901234567890123456789012123456789012345678901
Word
Origin
2
12345678901234567890123456789012123456789012345678901
homonym, and homosexual.
2345678901234567890123456789012123456789012345678901
12345678901234567890123456789012123456789012345678901 22
12345678901234567890123456789012123456789012345678901
homogeneous (adjective) Uniform, made entirely of one 2
12345678901234567890123456789012123456789012345678901
12345678901234567890123456789012123456789012345678901
thing. Its hard to think of a more homogenous group 2
2
1
2345678901234567890123456789012123456789012345678901
12345678901234567890123456789012123456789012345678901
than those eerie children in Village of the Damned, who 2
2
1
2
12345678901234567890123456789012123456789012345678901
all
had
perfect
features,
white-blond
hair,
and
silver,
12345678901234567890123456789012123456789012345678901 2
2
12345678901234567890123456789012123456789012345678901
penetrating eyes.
2
12345678901234567890123456789012123456789012345678901
12345678901234567890123456789012123456789012345678901 2
12345678901234567890123456789012123456789012345678901 2
12345678901234567890123456789012123456789012345678901 2 193
123456789012345678901234567890121234567890123456789012

http://www.xtremepapers.net

Part III: Section Review

123456789012345678901234567890121234567890123456789012
12345678901234567890123456789012123456789012345678901 2
2
12345678901234567890123456789012123456789012345678901
hoodwink (verb) To deceive by trickery or false appear- 2
12345678901234567890123456789012123456789012345678901
2
12345678901234567890123456789012123456789012345678901
ances; to dupe. That was my cousin Ravi calling to say 2
12345678901234567890123456789012123456789012345678901
2
12345678901234567890123456789012123456789012345678901
that hes been hoodwinked again, this time by some 2
12345678901234567890123456789012123456789012345678901
2345678901234567890123456789012123456789012345678901
2
1
outfit offering time shares in a desolate tract of land in 2
12345678901234567890123456789012123456789012345678901
2
12345678901234567890123456789012123456789012345678901
central Florida.
2
12345678901234567890123456789012123456789012345678901
2
12345678901234567890123456789012123456789012345678901
2345678901234567890123456789012123456789012345678901
1
hone (verb) To improve and make more acute or affective. 2
2
12345678901234567890123456789012123456789012345678901
2
12345678901234567890123456789012123456789012345678901
While
she
was
a
receptionist,
Norma
honed
her
skills
as
2
12345678901234567890123456789012123456789012345678901
2
12345678901234567890123456789012123456789012345678901
a
stand-up
comic
by
trying
out
jokes
on
the
tense
crowd
2345678901234567890123456789012123456789012345678901
2
1
2
12345678901234567890123456789012123456789012345678901
in
the
waiting
room.
2
12345678901234567890123456789012123456789012345678901
2
12345678901234567890123456789012123456789012345678901
12345678901234567890123456789012123456789012345678901
iconoclast (noun) Someone who attacks traditional beliefs 2
2345678901234567890123456789012123456789012345678901
2
1
12345678901234567890123456789012123456789012345678901
or institutions. Comedian Dennis Miller relishes his 2
2
12345678901234567890123456789012123456789012345678901
reputation as an iconoclast, though people in power 2
12345678901234567890123456789012123456789012345678901
2
12345678901234567890123456789012123456789012345678901
often resent his satirical jabs. iconoclasm (noun), icono- 2
12345678901234567890123456789012123456789012345678901
2
12345678901234567890123456789012123456789012345678901
clastic (adjective).
2
12345678901234567890123456789012123456789012345678901
2
12345678901234567890123456789012123456789012345678901
2
12345678901234567890123456789012123456789012345678901
idolatry
(noun)
The
worship
of
a
person,
thing,
or
institu2345678901234567890123456789012123456789012345678901
12345678901234567890123456789012123456789012345678901 2
1
tion as a god. In communist China, admiration for Mao 2
2
12345678901234567890123456789012123456789012345678901
2
12345678901234567890123456789012123456789012345678901
resembled
idolatry;
his
picture
was
displayed
every2
12345678901234567890123456789012123456789012345678901
2345678901234567890123456789012123456789012345678901
12345678901234567890123456789012123456789012345678901
where, and millions of Chinese memorized his sayings 2
2
12345678901234567890123456789012123456789012345678901
2
12345678901234567890123456789012123456789012345678901
and
repeated
them
endlessly.
idolatrous
(adjective).
12345678901234567890123456789012123456789012345678901 2
12345678901234567890123456789012123456789012345678901 2
12345678901234567890123456789012123456789012345678901
idyll (noun) A rustic, romantic interlude; poetry or prose 2
2
12345678901234567890123456789012123456789012345678901
12345678901234567890123456789012123456789012345678901
that celebrates simple pastoral life. Her picnic with Max 2
12345678901234567890123456789012123456789012345678901 2
1
at Fahnstock Lake was not the serene idyll she had 2
2
12345678901234567890123456789012123456789012345678901
envisioned; instead, they were surrounded by hundreds 2
12345678901234567890123456789012123456789012345678901
2
12345678901234567890123456789012123456789012345678901
of other picnickers blaring music from their boom boxes 2
12345678901234567890123456789012123456789012345678901
2
12345678901234567890123456789012123456789012345678901
and cracking open soda cans. idyllic (adjective).
2
12345678901234567890123456789012123456789012345678901
2
12345678901234567890123456789012123456789012345678901
2
12345678901234567890123456789012123456789012345678901
illicit
(adjective)
Illegal,
wrongful.
When
Janet
caught
her
2
12345678901234567890123456789012123456789012345678901
2
12345678901234567890123456789012123456789012345678901
13-year-old
son
and
his
friend
downloading
illicit
2345678901234567890123456789012123456789012345678901
12345678901234567890123456789012123456789012345678901 2
12345678901234567890123456789012123456789012345678901
pornographic photos from the World Wide Web, she 2
2
1
2
12345678901234567890123456789012123456789012345678901
promptly
pulled
the
plug
on
his
computer.
12345678901234567890123456789012123456789012345678901 2
2345678901234567890123456789012123456789012345678901
12345678901234567890123456789012123456789012345678901 2
12345678901234567890123456789012123456789012345678901
illuminate (verb) To brighten with light; to enlighten or 2
2
12345678901234567890123456789012123456789012345678901
12345678901234567890123456789012123456789012345678901
elucidate; to decorate (a manuscript). The frosted-glass 2
2
1
12345678901234567890123456789012123456789012345678901
sconces in the dressing rooms at Le Cirque not only 2
2
12345678901234567890123456789012123456789012345678901
12345678901234567890123456789012123456789012345678901
illuminate the rooms but make everyone look like a 2
2
12345678901234567890123456789012123456789012345678901
12345678901234567890123456789012123456789012345678901
movie star. Alice Munro is a writer who can illuminate 2
2345678901234567890123456789012123456789012345678901
2
12345678901234567890123456789012123456789012345678901
an entire character with a few deft sentences.
2
12345678901234567890123456789012123456789012345678901
12345678901234567890123456789012123456789012345678901 2
12345678901234567890123456789012123456789012345678901
immaterial (adjective) Of no consequence, unimportant. 2
2
1
2345678901234567890123456789012123456789012345678901
12345678901234567890123456789012123456789012345678901
The fact that your travel agent is your best friends son 2
2
1
2
12345678901234567890123456789012123456789012345678901
should
be
immaterial,
I
told
Rosa;
If
he
keeps
putting
12345678901234567890123456789012123456789012345678901 2
12345678901234567890123456789012123456789012345678901
you on hold and acting nasty, just take your business 2
2
12345678901234567890123456789012123456789012345678901
2
12345678901234567890123456789012123456789012345678901
elsewhere.
12345678901234567890123456789012123456789012345678901 2
2
194 12345678901234567890123456789012123456789012345678901
123456789012345678901234567890121234567890123456789012

www.petersons.com

http://www.xtremepapers.net

Chapter 5: Verbal Review

Take It to the Next Level

123456789012345678901234567890121234567890123456789012
12345678901234567890123456789012123456789012345678901 2
2
12345678901234567890123456789012123456789012345678901
immaculate (adjective) Totally unblemished, spotlessly clean. 2
12345678901234567890123456789012123456789012345678901
2
12345678901234567890123456789012123456789012345678901
The cream-colored upholstery in my new Porsche was 2
12345678901234567890123456789012123456789012345678901
2
12345678901234567890123456789012123456789012345678901
immaculatethat is, until a raccoon came in through 2
12345678901234567890123456789012123456789012345678901
2345678901234567890123456789012123456789012345678901
2
1
the window and tracked mud across the seats.
2
12345678901234567890123456789012123456789012345678901
2
12345678901234567890123456789012123456789012345678901
2
12345678901234567890123456789012123456789012345678901
2
12345678901234567890123456789012123456789012345678901
2345678901234567890123456789012123456789012345678901
2
1
Latin mutare 5 to change. Also found in English immutable,
2
12345678901234567890123456789012123456789012345678901
Word
Origin mutant, and mutation.
2
12345678901234567890123456789012123456789012345678901
2
12345678901234567890123456789012123456789012345678901
2
12345678901234567890123456789012123456789012345678901
immutable
(adjective)
Incapable
of
change.
Does
there
ever
2345678901234567890123456789012123456789012345678901
2
1
2
12345678901234567890123456789012123456789012345678901
come
an
age
when
we
realize
that
our
parents
person2
12345678901234567890123456789012123456789012345678901
2
12345678901234567890123456789012123456789012345678901
alities
are
immutable,
when
we
can
relax
and
stop
trying
2
12345678901234567890123456789012123456789012345678901
2345678901234567890123456789012123456789012345678901
2
1
to make them change?
2
12345678901234567890123456789012123456789012345678901
2
12345678901234567890123456789012123456789012345678901
impartial (adjective) Fair, equal, unbiased. If a judge is not 2
12345678901234567890123456789012123456789012345678901
2
12345678901234567890123456789012123456789012345678901
impartial, then all of her rulings are questionable. 2
12345678901234567890123456789012123456789012345678901
2
12345678901234567890123456789012123456789012345678901
impartiality (noun).
2
12345678901234567890123456789012123456789012345678901
2
12345678901234567890123456789012123456789012345678901
2
12345678901234567890123456789012123456789012345678901
impassivity
(noun)
Apathy,
unresponsiveness.
Dot
truly
2345678901234567890123456789012123456789012345678901
12345678901234567890123456789012123456789012345678901 22
1
thinks that Mr. Right will magically show up on her 2
12345678901234567890123456789012123456789012345678901
12345678901234567890123456789012123456789012345678901
door step, and her utter impassivity regarding her social 2
2
12345678901234567890123456789012123456789012345678901
2345678901234567890123456789012123456789012345678901
2
12345678901234567890123456789012123456789012345678901
life makes me want to shake her! impassive (adjective).
2
12345678901234567890123456789012123456789012345678901
12345678901234567890123456789012123456789012345678901 22
12345678901234567890123456789012123456789012345678901
imperceptible (adjective) Impossible to perceive, inaudible, 2
12345678901234567890123456789012123456789012345678901
12345678901234567890123456789012123456789012345678901
or incomprehensible. The sound of footsteps was almost 2
2
12345678901234567890123456789012123456789012345678901
12345678901234567890123456789012123456789012345678901
imperceptible, but Donalds paranoia had reached such 2
12345678901234567890123456789012123456789012345678901 22
1
a pitch that he immediately assumed he was being
2
12345678901234567890123456789012123456789012345678901
2
followed.
12345678901234567890123456789012123456789012345678901
12345678901234567890123456789012123456789012345678901 2
2
12345678901234567890123456789012123456789012345678901
imperturbable (adjective) Marked by extreme calm, impas- 2
12345678901234567890123456789012123456789012345678901
2345678901234567890123456789012123456789012345678901
12345678901234567890123456789012123456789012345678901
sivity, and steadiness. The proper English butler in 2
2
12345678901234567890123456789012123456789012345678901
2
1
Kazuo
Ishiguros
novel
The
Remains
of
the
Day
appears
2
12345678901234567890123456789012123456789012345678901
2
12345678901234567890123456789012123456789012345678901
completely
imperturbable
even
when
his
father
dies
or
2345678901234567890123456789012123456789012345678901
12345678901234567890123456789012123456789012345678901 22
12345678901234567890123456789012123456789012345678901
when his own heart is breaking.
2
1
12345678901234567890123456789012123456789012345678901 2
12345678901234567890123456789012123456789012345678901
impetuous (adjective) Acting hastily or impulsively. Bens 2
2
12345678901234567890123456789012123456789012345678901
2
12345678901234567890123456789012123456789012345678901
resignation
was
an
impetuous
act;
he
did
it
without
2
12345678901234567890123456789012123456789012345678901
2
12345678901234567890123456789012123456789012345678901
thinking, and he soon regretted it. impetuosity (noun).
12345678901234567890123456789012123456789012345678901 2
2345678901234567890123456789012123456789012345678901
12345678901234567890123456789012123456789012345678901 22
12345678901234567890123456789012123456789012345678901 2
12345678901234567890123456789012123456789012345678901
Latin placare 5 to please. Also found in English complacent,
2
12345678901234567890123456789012123456789012345678901
Word
Origin placate, and placid.
2
1
2345678901234567890123456789012123456789012345678901
12345678901234567890123456789012123456789012345678901 22
12345678901234567890123456789012123456789012345678901
implacable (adjective) Unbending, resolute. The state of 2
12345678901234567890123456789012123456789012345678901
12345678901234567890123456789012123456789012345678901
Israel is implacable in its policy of never negotiating 2
2
1
2345678901234567890123456789012123456789012345678901
2
12345678901234567890123456789012123456789012345678901
with criminals.
2
1
2
12345678901234567890123456789012123456789012345678901
12345678901234567890123456789012123456789012345678901 2
2
12345678901234567890123456789012123456789012345678901
12345678901234567890123456789012123456789012345678901 2
12345678901234567890123456789012123456789012345678901 2
12345678901234567890123456789012123456789012345678901 2
12345678901234567890123456789012123456789012345678901 2 195
123456789012345678901234567890121234567890123456789012

http://www.xtremepapers.net

Part III: Section Review

123456789012345678901234567890121234567890123456789012
12345678901234567890123456789012123456789012345678901 2
2
12345678901234567890123456789012123456789012345678901
implosion (noun) To collapse inward from outside pressure. 2
12345678901234567890123456789012123456789012345678901
2
12345678901234567890123456789012123456789012345678901
While it is difficult to know what is going on in North 2
12345678901234567890123456789012123456789012345678901
2
12345678901234567890123456789012123456789012345678901
Korea, no one can rule out a violent implosion of the 2
12345678901234567890123456789012123456789012345678901
2345678901234567890123456789012123456789012345678901
2
1
North Korean regime and a subsequent flood of refugees 2
12345678901234567890123456789012123456789012345678901
2
12345678901234567890123456789012123456789012345678901
across its borders. implode (verb).
2
12345678901234567890123456789012123456789012345678901
2
12345678901234567890123456789012123456789012345678901
2345678901234567890123456789012123456789012345678901
1
incessant (adjective) Unceasing. The incessant blaring of the 2
2
12345678901234567890123456789012123456789012345678901
2
12345678901234567890123456789012123456789012345678901
neighbors
car
alarm
made
it
impossible
for
me
to
2
12345678901234567890123456789012123456789012345678901
2
12345678901234567890123456789012123456789012345678901
concentrate
on
my
upcoming
Bar
exam.
2345678901234567890123456789012123456789012345678901
2
1
2
12345678901234567890123456789012123456789012345678901
12345678901234567890123456789012123456789012345678901
inchoate (adjective) Only partly formed or formulated. At 2
2
12345678901234567890123456789012123456789012345678901
12345678901234567890123456789012123456789012345678901
editorial meetings, Nancy had a habit of presenting her 2
2345678901234567890123456789012123456789012345678901
2
1
12345678901234567890123456789012123456789012345678901
inchoate book ideas before she had a chance to fully 2
2
12345678901234567890123456789012123456789012345678901
2
determine their feasibility.
12345678901234567890123456789012123456789012345678901
2
12345678901234567890123456789012123456789012345678901
2345678901234567890123456789012123456789012345678901
12345678901234567890123456789012123456789012345678901 2
12345678901234567890123456789012123456789012345678901 2
2
12345678901234567890123456789012123456789012345678901
Latin caedere 5 to cut. Also found in English concise, decide,
2
12345678901234567890123456789012123456789012345678901
Word
Origin excise, incision, and precise.
12345678901234567890123456789012123456789012345678901 2
12345678901234567890123456789012123456789012345678901 2
1
incise (verb) To carve into, to engrave. My wife felt 2
2
12345678901234567890123456789012123456789012345678901
2
12345678901234567890123456789012123456789012345678901
nostalgic
about
the
old
elm
tree
since
we
had
incised
our
2
12345678901234567890123456789012123456789012345678901
2345678901234567890123456789012123456789012345678901
2
12345678901234567890123456789012123456789012345678901
initials in it when we were both in high school.
2
12345678901234567890123456789012123456789012345678901
12345678901234567890123456789012123456789012345678901 2
12345678901234567890123456789012123456789012345678901
incisive (adjective) Admirably direct and decisive. Ted 2
2
12345678901234567890123456789012123456789012345678901
12345678901234567890123456789012123456789012345678901
Koppels incisive questions have made many politicians 2
2
12345678901234567890123456789012123456789012345678901
2
12345678901234567890123456789012123456789012345678901
squirm and stammer.
12345678901234567890123456789012123456789012345678901 2
2
1
incongruous (adjective) Unlikely. Art makes incongruous 2
12345678901234567890123456789012123456789012345678901
2
12345678901234567890123456789012123456789012345678901
alliances, as when punk rockers, Tibetan folk musicians, 2
12345678901234567890123456789012123456789012345678901
2
12345678901234567890123456789012123456789012345678901
gospel singers, and beat poets shared the stage at the 2
12345678901234567890123456789012123456789012345678901
2345678901234567890123456789012123456789012345678901
2
12345678901234567890123456789012123456789012345678901
Tibet House benefit concert. incongruity (noun).
2
12345678901234567890123456789012123456789012345678901
2
1
2
12345678901234567890123456789012123456789012345678901
incorrigible
(adjective)
Impossible
to
manage
or
reform.
2
12345678901234567890123456789012123456789012345678901
2345678901234567890123456789012123456789012345678901
12345678901234567890123456789012123456789012345678901
Lou is an incorrigible trickster, constantly playing 2
2
12345678901234567890123456789012123456789012345678901
2
1
practical
jokes
no
matter
how
much
his
friends
com12345678901234567890123456789012123456789012345678901 2
2
12345678901234567890123456789012123456789012345678901
plain.
2
12345678901234567890123456789012123456789012345678901
2
12345678901234567890123456789012123456789012345678901
12345678901234567890123456789012123456789012345678901
incursion (noun) A hostile entrance into a territory; a foray 2
2
12345678901234567890123456789012123456789012345678901
12345678901234567890123456789012123456789012345678901
into an activity or venture. It is a little-known fact that 2
2345678901234567890123456789012123456789012345678901
2
12345678901234567890123456789012123456789012345678901
the Central Intelligence Agency organized military 2
12345678901234567890123456789012123456789012345678901
2
12345678901234567890123456789012123456789012345678901
incursions into China during the 1950s. The ComicCon 2
12345678901234567890123456789012123456789012345678901
2
1
was Barbaras first incursion into the world of comic 2
12345678901234567890123456789012123456789012345678901
2
12345678901234567890123456789012123456789012345678901
strip artists.
2
12345678901234567890123456789012123456789012345678901
2
12345678901234567890123456789012123456789012345678901
2
12345678901234567890123456789012123456789012345678901
indefatigable
(adjective)
Tireless.
Eleanor
Roosevelts
inde2345678901234567890123456789012123456789012345678901
12345678901234567890123456789012123456789012345678901 2
1
fatigable dedication to the cause of human welfare won 2
2
12345678901234567890123456789012123456789012345678901
2
12345678901234567890123456789012123456789012345678901
her
affection
and
honor
throughout
the
world.
indefati2
12345678901234567890123456789012123456789012345678901
2345678901234567890123456789012123456789012345678901
2
1
gability (noun).
2
12345678901234567890123456789012123456789012345678901
12345678901234567890123456789012123456789012345678901 2
2
196 12345678901234567890123456789012123456789012345678901
123456789012345678901234567890121234567890123456789012

www.petersons.com

http://www.xtremepapers.net

Chapter 5: Verbal Review

Take It to the Next Level

123456789012345678901234567890121234567890123456789012
12345678901234567890123456789012123456789012345678901 2
2
12345678901234567890123456789012123456789012345678901
indelicate (adjective) Blunt, undisguised. No sooner had we 2
12345678901234567890123456789012123456789012345678901
2
12345678901234567890123456789012123456789012345678901
sat down to eat than Mark made an indelicate remark 2
12345678901234567890123456789012123456789012345678901
2
12345678901234567890123456789012123456789012345678901
about my high salary.
2
12345678901234567890123456789012123456789012345678901
2345678901234567890123456789012123456789012345678901
2
1
2
12345678901234567890123456789012123456789012345678901
inevitable
(adjective)
Unable
to
be
avoided.
Once
the
2
12345678901234567890123456789012123456789012345678901
2
12345678901234567890123456789012123456789012345678901
Japanese
attacked
Pearl
Harbor,
U.S.
involvement
in
2
12345678901234567890123456789012123456789012345678901
2345678901234567890123456789012123456789012345678901
2
1
World War II was inevitable. inevitability (noun).
2
12345678901234567890123456789012123456789012345678901
2
12345678901234567890123456789012123456789012345678901
2
12345678901234567890123456789012123456789012345678901
infer
(verb)
To
conclude,
to
deduce.
Can
I
infer
from
your
2
12345678901234567890123456789012123456789012345678901
2345678901234567890123456789012123456789012345678901
1
hostile tone of voice that you are still angry about 2
2
12345678901234567890123456789012123456789012345678901
2
12345678901234567890123456789012123456789012345678901
yesterdays incident? inference (noun).
2
12345678901234567890123456789012123456789012345678901
2
12345678901234567890123456789012123456789012345678901
inimical
(adjective)
Unfriendly,
hostile;
adverse
or
difficult.
2345678901234567890123456789012123456789012345678901
2
1
2
12345678901234567890123456789012123456789012345678901
Relations between Greece and Turkey have been inimi- 2
12345678901234567890123456789012123456789012345678901
2
12345678901234567890123456789012123456789012345678901
cal for centuries.
2
12345678901234567890123456789012123456789012345678901
2345678901234567890123456789012123456789012345678901
12345678901234567890123456789012123456789012345678901 22
12345678901234567890123456789012123456789012345678901
inimitable (adjective) Incapable of being imitated, match- 2
12345678901234567890123456789012123456789012345678901
12345678901234567890123456789012123456789012345678901
less. John F. Kennedys administration dazzled the 2
2
12345678901234567890123456789012123456789012345678901
12345678901234567890123456789012123456789012345678901
public, partly because of the inimitable style and 2
2
1
2
12345678901234567890123456789012123456789012345678901
elegance
of
his
wife,
Jacqueline.
2
12345678901234567890123456789012123456789012345678901
2
12345678901234567890123456789012123456789012345678901
2345678901234567890123456789012123456789012345678901
12345678901234567890123456789012123456789012345678901
inopportune (adjective) Awkward, untimely. When Gus 2
2
12345678901234567890123456789012123456789012345678901
12345678901234567890123456789012123456789012345678901
heard raised voices and the crash of breaking china 2
12345678901234567890123456789012123456789012345678901 22
12345678901234567890123456789012123456789012345678901
behind the kitchen door, he realized that hed picked an
2
12345678901234567890123456789012123456789012345678901
2
12345678901234567890123456789012123456789012345678901
inopportune moment to visit the Fairlights.
12345678901234567890123456789012123456789012345678901 22
12345678901234567890123456789012123456789012345678901
inscrutability (noun) Quality of being extremely difficult to 2
1
12345678901234567890123456789012123456789012345678901
interpret or understand, mysteriousness. I am still 2
2
12345678901234567890123456789012123456789012345678901
2
12345678901234567890123456789012123456789012345678901
puzzling
over
the
inscrutability
of
the
package
I
received
12345678901234567890123456789012123456789012345678901 2
12345678901234567890123456789012123456789012345678901
yesterday, which contained twenty pomegranates and a 2
2
12345678901234567890123456789012123456789012345678901
2
12345678901234567890123456789012123456789012345678901
note
that
said
simply
Yours.
inscrutable
(adjective).
12345678901234567890123456789012123456789012345678901 2
2
12345678901234567890123456789012123456789012345678901
2
12345678901234567890123456789012123456789012345678901
insensible
(adjective)
Unaware,
incognizant;
unconscious,
2345678901234567890123456789012123456789012345678901
12345678901234567890123456789012123456789012345678901 22
12345678901234567890123456789012123456789012345678901
out cold. Its a good thing that Marty was insensible to 2
1
12345678901234567890123456789012123456789012345678901
the titters and laughter that greeted his arrival in the 2
2
12345678901234567890123456789012123456789012345678901
2345678901234567890123456789012123456789012345678901
12345678901234567890123456789012123456789012345678901
ballroom. In the latest episode of police brutality, an 2
2
12345678901234567890123456789012123456789012345678901
12345678901234567890123456789012123456789012345678901
innocent young man was beaten insensible after two 2
12345678901234567890123456789012123456789012345678901 22
1
cops stormed his apartment.
2
12345678901234567890123456789012123456789012345678901
2
12345678901234567890123456789012123456789012345678901
insipid (adjective) Flavorless, uninteresting. Most TV shows 2
12345678901234567890123456789012123456789012345678901
12345678901234567890123456789012123456789012345678901
are so insipid that you can watch them while reading or 2
2
12345678901234567890123456789012123456789012345678901
2345678901234567890123456789012123456789012345678901
2
12345678901234567890123456789012123456789012345678901
chatting without missing a thing. insipidity (noun).
2
12345678901234567890123456789012123456789012345678901
12345678901234567890123456789012123456789012345678901 22
12345678901234567890123456789012123456789012345678901 2
1
2
12345678901234567890123456789012123456789012345678901
12345678901234567890123456789012123456789012345678901 2
2
12345678901234567890123456789012123456789012345678901
12345678901234567890123456789012123456789012345678901 2
2
12345678901234567890123456789012123456789012345678901
12345678901234567890123456789012123456789012345678901 2
12345678901234567890123456789012123456789012345678901 2
12345678901234567890123456789012123456789012345678901 2
12345678901234567890123456789012123456789012345678901 2 197
123456789012345678901234567890121234567890123456789012

http://www.xtremepapers.net

Part III: Section Review

123456789012345678901234567890121234567890123456789012
12345678901234567890123456789012123456789012345678901 2
2
12345678901234567890123456789012123456789012345678901
insinuate (verb) Hint or intimate; to creep in. During an 2
12345678901234567890123456789012123456789012345678901
2
12345678901234567890123456789012123456789012345678901
extremely unusual broadcast, the anchor man insinu- 2
12345678901234567890123456789012123456789012345678901
2
12345678901234567890123456789012123456789012345678901
ated that the Washington bureau chief was having a 2
12345678901234567890123456789012123456789012345678901
2345678901234567890123456789012123456789012345678901
2
1
nervous breakdown. Marla managed to insinuate herself 2
12345678901234567890123456789012123456789012345678901
12345678901234567890123456789012123456789012345678901
into the Duchess of Yorks conversation during the 2
2
12345678901234567890123456789012123456789012345678901
2
12345678901234567890123456789012123456789012345678901
Weight
Watchers
promotion
event.
insinuation
(noun).
2345678901234567890123456789012123456789012345678901
2
1
2
12345678901234567890123456789012123456789012345678901
2
12345678901234567890123456789012123456789012345678901
insolence
(noun)
An
attitude
or
behavior
that
is
bold
and
2
12345678901234567890123456789012123456789012345678901
2
12345678901234567890123456789012123456789012345678901
disrespectful.
Some
feel
that
news
reporters
who
shout
2345678901234567890123456789012123456789012345678901
2
1
2
12345678901234567890123456789012123456789012345678901
accusatory
questions
at
the
president
are
behaving
with
2
12345678901234567890123456789012123456789012345678901
2
12345678901234567890123456789012123456789012345678901
insolence
toward
his
high
office.
insolent
(adjective).
2
12345678901234567890123456789012123456789012345678901
2345678901234567890123456789012123456789012345678901
2
1
12345678901234567890123456789012123456789012345678901
insoluble (adjective) Unable to be solved, irresolvable; 2
2
12345678901234567890123456789012123456789012345678901
indissoluble. Fermats last theorem remained insoluble 2
12345678901234567890123456789012123456789012345678901
2
12345678901234567890123456789012123456789012345678901
for over 300 years until a young mathematician from 2
12345678901234567890123456789012123456789012345678901
2
12345678901234567890123456789012123456789012345678901
Princeton solved it in 1995. If you are a gum chewer, 2
12345678901234567890123456789012123456789012345678901
2
12345678901234567890123456789012123456789012345678901
you probably wouldnt like to know that insoluble 2
12345678901234567890123456789012123456789012345678901
2345678901234567890123456789012123456789012345678901
2
12345678901234567890123456789012123456789012345678901
plastics are a common ingredient of most popular gums. 2
1
2
12345678901234567890123456789012123456789012345678901
2
12345678901234567890123456789012123456789012345678901
insular
(adjective)
Narrow
or
isolated
in
attitude
or
view2
12345678901234567890123456789012123456789012345678901
2345678901234567890123456789012123456789012345678901
12345678901234567890123456789012123456789012345678901
point. New Yorkers are famous for their insular atti- 2
2
12345678901234567890123456789012123456789012345678901
2
12345678901234567890123456789012123456789012345678901
tudes;
they
seem
to
think
that
nothing
important
has
12345678901234567890123456789012123456789012345678901 2
2
12345678901234567890123456789012123456789012345678901
ever happened outside of their city. insularity (noun).
2
12345678901234567890123456789012123456789012345678901
12345678901234567890123456789012123456789012345678901 2
12345678901234567890123456789012123456789012345678901
intercede (verb) To step in, to moderate; to mediate or 2
2
12345678901234567890123456789012123456789012345678901
1
negotiate on behalf of someone else. After their rejection 2
2345678901234567890123456789012123456789012345678901
12345678901234567890123456789012123456789012345678901 2
by the co-op board, Kevin and Sol asked Rachel, another 2
12345678901234567890123456789012123456789012345678901
2
1
tenant, to intercede for them at the next board meeting. 2
12345678901234567890123456789012123456789012345678901
2
12345678901234567890123456789012123456789012345678901
intercession (noun).
2
12345678901234567890123456789012123456789012345678901
2
12345678901234567890123456789012123456789012345678901
2
12345678901234567890123456789012123456789012345678901
interim
(noun)
A
break
or
interlude.
In
the
interim
between
2
12345678901234567890123456789012123456789012345678901
2
12345678901234567890123456789012123456789012345678901
figure-skating
programs,
the
exhausted
skaters
retreat
to
2345678901234567890123456789012123456789012345678901
12345678901234567890123456789012123456789012345678901 2
2
12345678901234567890123456789012123456789012345678901
the kiss and cry room to wait for their scores.
2
1
12345678901234567890123456789012123456789012345678901 2
12345678901234567890123456789012123456789012345678901
interpolate (verb) To interject. The directors decision to 2
2
12345678901234567890123456789012123456789012345678901
2
12345678901234567890123456789012123456789012345678901
interpolate
topical
political
jokes
into
his
production
of
2
12345678901234567890123456789012123456789012345678901
12345678901234567890123456789012123456789012345678901
Shakespeares Twelfth Night was not viewed kindly by 2
12345678901234567890123456789012123456789012345678901 2
2
12345678901234567890123456789012123456789012345678901
the critics. interpolation (noun).
2
12345678901234567890123456789012123456789012345678901
2
12345678901234567890123456789012123456789012345678901
intransigent (adjective) Unwilling to compromise. Despite 2
12345678901234567890123456789012123456789012345678901
2
12345678901234567890123456789012123456789012345678901
the mediators attempts to suggest a fair solution to the 2
12345678901234567890123456789012123456789012345678901
2
12345678901234567890123456789012123456789012345678901
disagreement, the two parties were intransigent, forcing 2
12345678901234567890123456789012123456789012345678901
2
12345678901234567890123456789012123456789012345678901
a showdown. intransigence (noun).
2
12345678901234567890123456789012123456789012345678901
2345678901234567890123456789012123456789012345678901
12345678901234567890123456789012123456789012345678901 2
2
1
2
12345678901234567890123456789012123456789012345678901
12345678901234567890123456789012123456789012345678901 2
2
12345678901234567890123456789012123456789012345678901
12345678901234567890123456789012123456789012345678901 2
12345678901234567890123456789012123456789012345678901 2
12345678901234567890123456789012123456789012345678901 2
2
198 12345678901234567890123456789012123456789012345678901
123456789012345678901234567890121234567890123456789012

www.petersons.com

http://www.xtremepapers.net

Chapter 5: Verbal Review

Take It to the Next Level

123456789012345678901234567890121234567890123456789012
12345678901234567890123456789012123456789012345678901 2
2
12345678901234567890123456789012123456789012345678901
intrinsically (adverb) Essentially, inherently. There is noth- 2
12345678901234567890123456789012123456789012345678901
2
12345678901234567890123456789012123456789012345678901
ing intrinsically difficult about upgrading a computers 2
12345678901234567890123456789012123456789012345678901
2
12345678901234567890123456789012123456789012345678901
microprocessor, yet Al was afraid to even open up the 2
12345678901234567890123456789012123456789012345678901
2345678901234567890123456789012123456789012345678901
2
1
hard drive. intrinsic (adjective).
2
12345678901234567890123456789012123456789012345678901
2
12345678901234567890123456789012123456789012345678901
2
12345678901234567890123456789012123456789012345678901
2
12345678901234567890123456789012123456789012345678901
2345678901234567890123456789012123456789012345678901
2
1
Word
Origin Latin unda 5 wave. Also found in English undulate.
2
12345678901234567890123456789012123456789012345678901
2
12345678901234567890123456789012123456789012345678901
12345678901234567890123456789012123456789012345678901
inundate (verb) To overwhelm; to flood. When America 2
2
12345678901234567890123456789012123456789012345678901
12345678901234567890123456789012123456789012345678901
Online first announced its flat-rate pricing, the company 2
2
12345678901234567890123456789012123456789012345678901
12345678901234567890123456789012123456789012345678901
was inundated with new customers, and thus began the 2
2
12345678901234567890123456789012123456789012345678901
annoying delays in service. inundation (noun).
2
12345678901234567890123456789012123456789012345678901
2345678901234567890123456789012123456789012345678901
2
1
12345678901234567890123456789012123456789012345678901
invective (noun) Insulting, abusive language. I remained 2
2
12345678901234567890123456789012123456789012345678901
2
12345678901234567890123456789012123456789012345678901
unscathed
by
his
blistering
invective
because
in
my
heart
2
12345678901234567890123456789012123456789012345678901
2345678901234567890123456789012123456789012345678901
2
12345678901234567890123456789012123456789012345678901
I knew I had done the right thing.
2
12345678901234567890123456789012123456789012345678901
12345678901234567890123456789012123456789012345678901 22
12345678901234567890123456789012123456789012345678901
invigorate (verb) To give energy to, to stimulate. As her car 2
12345678901234567890123456789012123456789012345678901
12345678901234567890123456789012123456789012345678901
climbed the mountain road, Lucinda felt herself invigo- 2
2
1
2
12345678901234567890123456789012123456789012345678901
rated
by
the
clear
air
and
the
cool
breezes.
invigoration
2
12345678901234567890123456789012123456789012345678901
2
12345678901234567890123456789012123456789012345678901
(noun).
2345678901234567890123456789012123456789012345678901
12345678901234567890123456789012123456789012345678901 22
12345678901234567890123456789012123456789012345678901
irascible (adjective) Easily provoked into anger, hot-headed. 2
12345678901234567890123456789012123456789012345678901
2
12345678901234567890123456789012123456789012345678901
Soup chef Al Yeganah, the model for Seinfelds Soup 2
12345678901234567890123456789012123456789012345678901
2
12345678901234567890123456789012123456789012345678901
Nazi, is an irascible man who flies into a temper if his 2
12345678901234567890123456789012123456789012345678901
2
12345678901234567890123456789012123456789012345678901
customers dont follow his rigid procedure for purchas- 2
12345678901234567890123456789012123456789012345678901
2
1
ing soup. irascibility (noun).
2
12345678901234567890123456789012123456789012345678901
2
12345678901234567890123456789012123456789012345678901
2
12345678901234567890123456789012123456789012345678901
jeopardize
(verb)
To
put
in
danger.
Terrorist
attacks
on
12345678901234567890123456789012123456789012345678901 2
12345678901234567890123456789012123456789012345678901
civilians jeopardize the fragile peace in the Middle East. 2
2
12345678901234567890123456789012123456789012345678901
2
12345678901234567890123456789012123456789012345678901
jeopardy
(noun).
12345678901234567890123456789012123456789012345678901 2
2
12345678901234567890123456789012123456789012345678901
12345678901234567890123456789012123456789012345678901
jocular (adjective) Humorous, amusing. Listening to the 2
2345678901234567890123456789012123456789012345678901
12345678901234567890123456789012123456789012345678901 22
12345678901234567890123456789012123456789012345678901
CEO launch into yet another uproarious anecdote, Ted
2
1
12345678901234567890123456789012123456789012345678901
was frankly surprised by the jocular nature of the 2
12345678901234567890123456789012123456789012345678901 2
2
emergency board meeting. jocularity (noun).
12345678901234567890123456789012123456789012345678901
2
12345678901234567890123456789012123456789012345678901
2
12345678901234567890123456789012123456789012345678901
labyrinthine
(adjective)
Extremely
intricate
or
involved;
2
12345678901234567890123456789012123456789012345678901
2
12345678901234567890123456789012123456789012345678901
circuitous.
Was
I
the
only
one
who
couldnt
follow
the
2345678901234567890123456789012123456789012345678901
12345678901234567890123456789012123456789012345678901 22
12345678901234567890123456789012123456789012345678901
labyrinthine plot of the movie L.A. Confidential? I was 2
12345678901234567890123456789012123456789012345678901
12345678901234567890123456789012123456789012345678901
so confused I had to watch it twice to see who did it. 2
2
1
2345678901234567890123456789012123456789012345678901
12345678901234567890123456789012123456789012345678901 22
12345678901234567890123456789012123456789012345678901
laconic (adjective) Concise to the point of terseness; taci- 2
12345678901234567890123456789012123456789012345678901
12345678901234567890123456789012123456789012345678901
turn. Tall, handsome, and laconic, the actor Gary 2
2
1
2345678901234567890123456789012123456789012345678901
12345678901234567890123456789012123456789012345678901
Cooper came to personify the strong, silent American, a 2
2
1
2
12345678901234567890123456789012123456789012345678901
man of action and few words.
12345678901234567890123456789012123456789012345678901 2
2
12345678901234567890123456789012123456789012345678901
12345678901234567890123456789012123456789012345678901 2
12345678901234567890123456789012123456789012345678901 2
12345678901234567890123456789012123456789012345678901 2
12345678901234567890123456789012123456789012345678901 2 199
123456789012345678901234567890121234567890123456789012

http://www.xtremepapers.net

Part III: Section Review

123456789012345678901234567890121234567890123456789012
12345678901234567890123456789012123456789012345678901 2
2
12345678901234567890123456789012123456789012345678901
lambaste (verb) To give someone a dressing-down; to attack 2
12345678901234567890123456789012123456789012345678901
2
12345678901234567890123456789012123456789012345678901
someone verbally; to whip. Once inside the locker room, 2
12345678901234567890123456789012123456789012345678901
2
12345678901234567890123456789012123456789012345678901
the coach thoroughly lambasted the team for their 2
12345678901234567890123456789012123456789012345678901
2345678901234567890123456789012123456789012345678901
2
1
incompetent performance on the football field.
2
12345678901234567890123456789012123456789012345678901
2
12345678901234567890123456789012123456789012345678901
2
12345678901234567890123456789012123456789012345678901
2
12345678901234567890123456789012123456789012345678901
2345678901234567890123456789012123456789012345678901
2
1
Latin laus 5 praise. Also found in English applaud, laud, laudatory,
2
12345678901234567890123456789012123456789012345678901
Word
Origin and plaudit.
2
12345678901234567890123456789012123456789012345678901
2
12345678901234567890123456789012123456789012345678901
2
12345678901234567890123456789012123456789012345678901
laudable
(adjective)
Commendable,
praiseworthy.
The
2345678901234567890123456789012123456789012345678901
2
1
2
12345678901234567890123456789012123456789012345678901
Hunts
Point
nonprofit
organization
has
embarked
on
a
2
12345678901234567890123456789012123456789012345678901
2
12345678901234567890123456789012123456789012345678901
series
of
laudable
ventures
pairing
businesses
and
2
12345678901234567890123456789012123456789012345678901
2345678901234567890123456789012123456789012345678901
2
1
disadvantaged youth.
2
12345678901234567890123456789012123456789012345678901
2
12345678901234567890123456789012123456789012345678901
lethargic (adjective) Lacking energy; sluggish. Visitors to 2
12345678901234567890123456789012123456789012345678901
2
12345678901234567890123456789012123456789012345678901
the zoo are surprised that the lions appear so lethargic, 2
12345678901234567890123456789012123456789012345678901
2
12345678901234567890123456789012123456789012345678901
but, in the wild, lions sleep up to 18 hours a day. 2
12345678901234567890123456789012123456789012345678901
2
12345678901234567890123456789012123456789012345678901
lethargy (noun).
2
12345678901234567890123456789012123456789012345678901
2345678901234567890123456789012123456789012345678901
12345678901234567890123456789012123456789012345678901 2
1
levy (verb) To demand payment or collection of a tax or fee. 2
2
12345678901234567890123456789012123456789012345678901
2
12345678901234567890123456789012123456789012345678901
The
environmental
activists
pushed
Congress
to
levy
2
12345678901234567890123456789012123456789012345678901
2345678901234567890123456789012123456789012345678901
12345678901234567890123456789012123456789012345678901
higher taxes on gasoline, but the automakers lobbyists 2
2
12345678901234567890123456789012123456789012345678901
2
12345678901234567890123456789012123456789012345678901
quashed
their
plans.
12345678901234567890123456789012123456789012345678901 2
12345678901234567890123456789012123456789012345678901 2
12345678901234567890123456789012123456789012345678901
lien (noun) A claim against a property for the satisfaction of 2
2
12345678901234567890123456789012123456789012345678901
12345678901234567890123456789012123456789012345678901
a debt. Nat was in such financial straits when he died 2
12345678901234567890123456789012123456789012345678901 2
1
that his Fishkill property had several liens against it and 2
2
12345678901234567890123456789012123456789012345678901
2
all of his furniture was being repossessed.
12345678901234567890123456789012123456789012345678901
12345678901234567890123456789012123456789012345678901 2
2
12345678901234567890123456789012123456789012345678901
limn (verb) To outline in distinct detail; to delineate. Like 2
12345678901234567890123456789012123456789012345678901
2345678901234567890123456789012123456789012345678901
12345678901234567890123456789012123456789012345678901
many of her novels, Edith Whartons The Age of 2
2
12345678901234567890123456789012123456789012345678901
2
1
Innocence
expertly
limns
the
tyranny
of
New
Yorks
2
12345678901234567890123456789012123456789012345678901
2
12345678901234567890123456789012123456789012345678901
upper-class
society
in
the
1800s.
2345678901234567890123456789012123456789012345678901
12345678901234567890123456789012123456789012345678901 2
12345678901234567890123456789012123456789012345678901 2
1
loquacity (noun) Talkativeness, wordiness. While some 2
2
12345678901234567890123456789012123456789012345678901
2
12345678901234567890123456789012123456789012345678901
people
deride
his
loquacity
and
his
tendency
to
use
2345678901234567890123456789012123456789012345678901
12345678901234567890123456789012123456789012345678901 2
12345678901234567890123456789012123456789012345678901
outrageous rhymes, no one can doubt that Jesse Jackson 2
2
12345678901234567890123456789012123456789012345678901
2
12345678901234567890123456789012123456789012345678901
is a powerful orator. loquacious (adjective).
2
1
2345678901234567890123456789012123456789012345678901
12345678901234567890123456789012123456789012345678901 2
12345678901234567890123456789012123456789012345678901 2
2
12345678901234567890123456789012123456789012345678901
Latin lux 5 light. Also found in English elucidate, pellucid, and
2
12345678901234567890123456789012123456789012345678901
Word
Origin
2
1
translucent.
2345678901234567890123456789012123456789012345678901
12345678901234567890123456789012123456789012345678901 2
2
12345678901234567890123456789012123456789012345678901
lucid (adjective) Clear and understandable. Hawkings A 2
12345678901234567890123456789012123456789012345678901
12345678901234567890123456789012123456789012345678901
Short History of the Universe is a lucid explanation of a 2
2
1
2345678901234567890123456789012123456789012345678901
12345678901234567890123456789012123456789012345678901
difficult topic, modern scientific theories of the origin of 2
2
1
2
12345678901234567890123456789012123456789012345678901
the
universe.
lucidity
(noun).
12345678901234567890123456789012123456789012345678901 2
2
12345678901234567890123456789012123456789012345678901
12345678901234567890123456789012123456789012345678901 2
12345678901234567890123456789012123456789012345678901 2
12345678901234567890123456789012123456789012345678901 2
2
200 12345678901234567890123456789012123456789012345678901
123456789012345678901234567890121234567890123456789012

www.petersons.com

http://www.xtremepapers.net

Chapter 5: Verbal Review

Take It to the Next Level

123456789012345678901234567890121234567890123456789012
12345678901234567890123456789012123456789012345678901 2
2
12345678901234567890123456789012123456789012345678901
magnanimous (adjective) Noble, generous. When media 2
12345678901234567890123456789012123456789012345678901
2
12345678901234567890123456789012123456789012345678901
titan Ted Turner pledged a gift of $1 billion to the 2
12345678901234567890123456789012123456789012345678901
2
12345678901234567890123456789012123456789012345678901
United Nations, he challenged other wealthy people to 2
12345678901234567890123456789012123456789012345678901
2345678901234567890123456789012123456789012345678901
2
1
be equally magnanimous. magnanimity (noun).
2
12345678901234567890123456789012123456789012345678901
2
12345678901234567890123456789012123456789012345678901
2
12345678901234567890123456789012123456789012345678901
maladroit
(adjective)
Inept,
awkward.
It
was
painful
to
2
12345678901234567890123456789012123456789012345678901
2345678901234567890123456789012123456789012345678901
1
watch the young congressmans maladroit delivery of 2
2
12345678901234567890123456789012123456789012345678901
2
12345678901234567890123456789012123456789012345678901
the
nominating
speech.
2
12345678901234567890123456789012123456789012345678901
2
12345678901234567890123456789012123456789012345678901
malinger
(verb)
To
pretend
illness
to
avoid
work.
During
2345678901234567890123456789012123456789012345678901
2
1
2
12345678901234567890123456789012123456789012345678901
the labor dispute, hundreds of employees malingered, 2
12345678901234567890123456789012123456789012345678901
2
12345678901234567890123456789012123456789012345678901
forcing the company to slow production and costing it 2
12345678901234567890123456789012123456789012345678901
2345678901234567890123456789012123456789012345678901
2
1
millions in profits.
2
12345678901234567890123456789012123456789012345678901
2
12345678901234567890123456789012123456789012345678901
2
12345678901234567890123456789012123456789012345678901
malleable
(adjective)
Able
to
be
changed,
shaped,
or
formed
2
12345678901234567890123456789012123456789012345678901
2345678901234567890123456789012123456789012345678901
12345678901234567890123456789012123456789012345678901
by outside pressures. Gold is a very useful metal because 2
2
12345678901234567890123456789012123456789012345678901
2
12345678901234567890123456789012123456789012345678901
it
is
so
malleable.
A
childs
personality
is
malleable,
and
12345678901234567890123456789012123456789012345678901 22
12345678901234567890123456789012123456789012345678901
is often deeply influenced by things her parents say and
2
12345678901234567890123456789012123456789012345678901
2
1
do. malleability (noun).
2
12345678901234567890123456789012123456789012345678901
2
12345678901234567890123456789012123456789012345678901
2
12345678901234567890123456789012123456789012345678901
2345678901234567890123456789012123456789012345678901
2
12345678901234567890123456789012123456789012345678901
Latin mandare 5 entrust, order. Also found in English command,
2
1
Word
Origin
demand, and remand.
2
12345678901234567890123456789012123456789012345678901
2
12345678901234567890123456789012123456789012345678901
2
12345678901234567890123456789012123456789012345678901
mandate
(noun)
Order,
command.
The
new
policy
on
gays
2345678901234567890123456789012123456789012345678901
12345678901234567890123456789012123456789012345678901 22
12345678901234567890123456789012123456789012345678901
in the military went into effect as soon as the president 2
12345678901234567890123456789012123456789012345678901
12345678901234567890123456789012123456789012345678901
issued his mandate about it. mandate (verb), mandatory 2
2
1
2345678901234567890123456789012123456789012345678901
2
12345678901234567890123456789012123456789012345678901
(adjective).
2
12345678901234567890123456789012123456789012345678901
2
1
12345678901234567890123456789012123456789012345678901
marginal (adjective) At the outer edge or fringe; of minimal 2
2
12345678901234567890123456789012123456789012345678901
12345678901234567890123456789012123456789012345678901
quality or acceptability. In spite of the trend toward 2
2
12345678901234567890123456789012123456789012345678901
12345678901234567890123456789012123456789012345678901
greater paternal involvement in child-rearing, most 2
2
12345678901234567890123456789012123456789012345678901
fathers still have a marginal role in their childrens lives. 2
12345678901234567890123456789012123456789012345678901
2345678901234567890123456789012123456789012345678901
2
12345678901234567890123456789012123456789012345678901
Jerrys GRE scores were so marginal that he didnt get 2
12345678901234567890123456789012123456789012345678901
2
1
accepted into the graduate schools of his choice.
2
12345678901234567890123456789012123456789012345678901
12345678901234567890123456789012123456789012345678901 2
2345678901234567890123456789012123456789012345678901
12345678901234567890123456789012123456789012345678901
marginalize (verb) To push toward the fringes; to make less 2
2
12345678901234567890123456789012123456789012345678901
2
12345678901234567890123456789012123456789012345678901
consequential.
Hannah
argued
that
the
designation
of
a
12345678901234567890123456789012123456789012345678901 22
1
certain month as Black History Month or Gay and 2
12345678901234567890123456789012123456789012345678901
12345678901234567890123456789012123456789012345678901
Lesbian Book Month actually does a disservice to 2
2
12345678901234567890123456789012123456789012345678901
2
12345678901234567890123456789012123456789012345678901
minorities by marginalizing them.
12345678901234567890123456789012123456789012345678901 2
2345678901234567890123456789012123456789012345678901
2
12345678901234567890123456789012123456789012345678901
martial (adjective) Of, relating to, or suited to military life. 2
12345678901234567890123456789012123456789012345678901
12345678901234567890123456789012123456789012345678901
My old teacher, Miss Woody, had such a martial 2
2
12345678901234567890123456789012123456789012345678901
2
1
demeanor
that
youd
think
she
was
running
a
boot
camp
2345678901234567890123456789012123456789012345678901
12345678901234567890123456789012123456789012345678901 22
1
instead of teaching fifth grade. The military seized 2
12345678901234567890123456789012123456789012345678901
12345678901234567890123456789012123456789012345678901
control of Myanmar in 1988, and this embattled country 2
2
12345678901234567890123456789012123456789012345678901
2345678901234567890123456789012123456789012345678901
2
1
has been ruled by martial law since then.
2
12345678901234567890123456789012123456789012345678901
12345678901234567890123456789012123456789012345678901 22
12345678901234567890123456789012123456789012345678901 201
123456789012345678901234567890121234567890123456789012

http://www.xtremepapers.net

Part III: Section Review

123456789012345678901234567890121234567890123456789012
12345678901234567890123456789012123456789012345678901 2
2
12345678901234567890123456789012123456789012345678901
2
12345678901234567890123456789012123456789012345678901
Latin medius 5 middle. Also found in English intermediate, media,
2345678901234567890123456789012123456789012345678901
2
1
Word
Origin
2
12345678901234567890123456789012123456789012345678901
and medium.
12345678901234567890123456789012123456789012345678901 2
2
12345678901234567890123456789012123456789012345678901
mediate (verb) To reconcile differences between two parties. 2
12345678901234567890123456789012123456789012345678901
2
12345678901234567890123456789012123456789012345678901
During the baseball strike, both the players and the club 2
12345678901234567890123456789012123456789012345678901
2
12345678901234567890123456789012123456789012345678901
owners expressed willingness to have the president 2
12345678901234567890123456789012123456789012345678901
2345678901234567890123456789012123456789012345678901
2
1
mediate the dispute. mediation (noun).
2
12345678901234567890123456789012123456789012345678901
2
12345678901234567890123456789012123456789012345678901
2
12345678901234567890123456789012123456789012345678901
mercenary
(adjective)
Doing
something
only
for
pay
or
for
2
12345678901234567890123456789012123456789012345678901
2345678901234567890123456789012123456789012345678901
1
personal advantage. People have criticized the U.S. 2
2
12345678901234567890123456789012123456789012345678901
2
12345678901234567890123456789012123456789012345678901
motives
in
the
Persian
Gulf
War
as
mercenary,
pointing
2
12345678901234567890123456789012123456789012345678901
2
12345678901234567890123456789012123456789012345678901
out
that
the
U.S.
would
not
have
come
to
Kuwaits
2345678901234567890123456789012123456789012345678901
2
1
2
12345678901234567890123456789012123456789012345678901
defense
had
it
grown
carrots
rather
than
produced
oil.
2
12345678901234567890123456789012123456789012345678901
2
12345678901234567890123456789012123456789012345678901
mercenary
(noun).
2
12345678901234567890123456789012123456789012345678901
2345678901234567890123456789012123456789012345678901
12345678901234567890123456789012123456789012345678901 2
12345678901234567890123456789012123456789012345678901
mercurial (adjective) Changing quickly and unpredictably. 2
2
12345678901234567890123456789012123456789012345678901
The mercurial personality of Robin Williams, with his 2
12345678901234567890123456789012123456789012345678901
2
12345678901234567890123456789012123456789012345678901
many voices and styles, made him a natural choice to 2
12345678901234567890123456789012123456789012345678901
2
1
play the part of the ever-changing genie in Aladdin.
2
12345678901234567890123456789012123456789012345678901
2
12345678901234567890123456789012123456789012345678901
2
12345678901234567890123456789012123456789012345678901
metamorphose
(verb)
To
undergo
a
striking
transforma2345678901234567890123456789012123456789012345678901
12345678901234567890123456789012123456789012345678901 2
12345678901234567890123456789012123456789012345678901
tion. In just a century, book publishers have metamor- 2
2
12345678901234567890123456789012123456789012345678901
2
12345678901234567890123456789012123456789012345678901
phosed
from
independent,
exclusively
literary
businesses
12345678901234567890123456789012123456789012345678901 2
12345678901234567890123456789012123456789012345678901
to minor divisions in multimedia entertainment conglom- 2
2
12345678901234567890123456789012123456789012345678901
2
12345678901234567890123456789012123456789012345678901
erates.
metamorphosis
(noun).
12345678901234567890123456789012123456789012345678901 2
2
1
12345678901234567890123456789012123456789012345678901
meticulous (adjective) Very careful with details. Watch 2
2
12345678901234567890123456789012123456789012345678901
12345678901234567890123456789012123456789012345678901
repair calls for a craftsperson who is patient and 2
12345678901234567890123456789012123456789012345678901 2
2
12345678901234567890123456789012123456789012345678901
meticulous.
2
12345678901234567890123456789012123456789012345678901
2
12345678901234567890123456789012123456789012345678901
mettle (noun) Strength of spirit; stamina. Lindas mettle 2
12345678901234567890123456789012123456789012345678901
2
12345678901234567890123456789012123456789012345678901
was severely tested while she served as the only female 2
12345678901234567890123456789012123456789012345678901
2345678901234567890123456789012123456789012345678901
12345678901234567890123456789012123456789012345678901
attorney at Smith, Futterweitt, Houghton, and Dobbs. 2
2
12345678901234567890123456789012123456789012345678901
2
1
mettlesome
(adjective).
12345678901234567890123456789012123456789012345678901 2
12345678901234567890123456789012123456789012345678901 2
12345678901234567890123456789012123456789012345678901
mimicry (noun) Imitation, aping. The continued popularity 2
2
12345678901234567890123456789012123456789012345678901
2
12345678901234567890123456789012123456789012345678901
of
Elvis
Presley
has
given
rise
to
a
class
of
entertainers
2
12345678901234567890123456789012123456789012345678901
2
12345678901234567890123456789012123456789012345678901
who
make
a
living
through
mimicry
of
The
King.
2345678901234567890123456789012123456789012345678901
12345678901234567890123456789012123456789012345678901 2
2
12345678901234567890123456789012123456789012345678901
mimic (noun and verb).
2
12345678901234567890123456789012123456789012345678901
12345678901234567890123456789012123456789012345678901 2
1
minatory (adjective) Menacing, threatening. As soon as she 2
2
12345678901234567890123456789012123456789012345678901
met Mrs. Danforth, the head housemaid at Manderlay, 2
12345678901234567890123456789012123456789012345678901
2
12345678901234567890123456789012123456789012345678901
the young bride was cowed by her minatory manner and 2
12345678901234567890123456789012123456789012345678901
2
12345678901234567890123456789012123456789012345678901
quickly retreated to the morning room.
2
12345678901234567890123456789012123456789012345678901
12345678901234567890123456789012123456789012345678901 2
2
12345678901234567890123456789012123456789012345678901
12345678901234567890123456789012123456789012345678901 2
2
12345678901234567890123456789012123456789012345678901
12345678901234567890123456789012123456789012345678901 2
12345678901234567890123456789012123456789012345678901 2
12345678901234567890123456789012123456789012345678901 2
2
202 12345678901234567890123456789012123456789012345678901
123456789012345678901234567890121234567890123456789012

www.petersons.com

http://www.xtremepapers.net

Chapter 5: Verbal Review

Take It to the Next Level

123456789012345678901234567890121234567890123456789012
12345678901234567890123456789012123456789012345678901 2
2
12345678901234567890123456789012123456789012345678901
mince (verb) To chop into small pieces; to speak with 2
12345678901234567890123456789012123456789012345678901
2
12345678901234567890123456789012123456789012345678901
decorum and restraint. Malaysias prime minister, 2
12345678901234567890123456789012123456789012345678901
2
12345678901234567890123456789012123456789012345678901
Mahathir Mohamad, is not a man known to mince 2
12345678901234567890123456789012123456789012345678901
2345678901234567890123456789012123456789012345678901
2
1
words; he has accused satellite TV of poisoning Asia and 2
12345678901234567890123456789012123456789012345678901
12345678901234567890123456789012123456789012345678901
has denounced the Australian press as congenital 2
2
12345678901234567890123456789012123456789012345678901
2
12345678901234567890123456789012123456789012345678901
liars.
2345678901234567890123456789012123456789012345678901
2
1
2
12345678901234567890123456789012123456789012345678901
2
12345678901234567890123456789012123456789012345678901
2
12345678901234567890123456789012123456789012345678901
2
12345678901234567890123456789012123456789012345678901
Greek anthropos 5 human. Also found in English anthropology,
Word
Origin
2345678901234567890123456789012123456789012345678901
2
1
anthropoid, anthropomorphic, and philanthropy.
2
12345678901234567890123456789012123456789012345678901
2
12345678901234567890123456789012123456789012345678901
2
12345678901234567890123456789012123456789012345678901
misanthrope
(noun)
Someone
who
hates
or
distrusts
all
2
12345678901234567890123456789012123456789012345678901
2345678901234567890123456789012123456789012345678901
1
people. In the beloved Christmas classic, Its a Wonder- 2
2
12345678901234567890123456789012123456789012345678901
12345678901234567890123456789012123456789012345678901
ful Life, Lionel Barrymore plays Potter, the wealthy 2
2
12345678901234567890123456789012123456789012345678901
12345678901234567890123456789012123456789012345678901
misanthrope who is determined to make life miserable 2
2345678901234567890123456789012123456789012345678901
12345678901234567890123456789012123456789012345678901 22
12345678901234567890123456789012123456789012345678901
for everyone, and particularly for the young, idealistic
2
12345678901234567890123456789012123456789012345678901
George Bailey. misanthropic (adjective), misanthropy 2
12345678901234567890123456789012123456789012345678901
2
12345678901234567890123456789012123456789012345678901
(noun).
2
12345678901234567890123456789012123456789012345678901
2
1
2
12345678901234567890123456789012123456789012345678901
miscreant
(adjective)
Unbelieving,
heretical;
evil,
villainous.
2
12345678901234567890123456789012123456789012345678901
2
12345678901234567890123456789012123456789012345678901
After
a
one-year
run
playing
Iago
in
Othello,
and
then
2345678901234567890123456789012123456789012345678901
12345678901234567890123456789012123456789012345678901 22
12345678901234567890123456789012123456789012345678901
two years playing Bill Sikes in Oliver, Sean was tired of 2
12345678901234567890123456789012123456789012345678901
2
12345678901234567890123456789012123456789012345678901
being typecast in miscreant roles. miscreant (noun).
2
12345678901234567890123456789012123456789012345678901
12345678901234567890123456789012123456789012345678901 22
12345678901234567890123456789012123456789012345678901
mitigate (verb) To make less severe; to relieve. Theres no 2
12345678901234567890123456789012123456789012345678901
12345678901234567890123456789012123456789012345678901
doubt that Wallace committed the assault, but the verbal 2
2
1
2345678901234567890123456789012123456789012345678901
12345678901234567890123456789012123456789012345678901
abuse Wallace had received helps to explain his behavior 2
2
12345678901234567890123456789012123456789012345678901
2
1
and somewhat mitigates his guilt. mitigation (noun).
12345678901234567890123456789012123456789012345678901 2
2
12345678901234567890123456789012123456789012345678901
monopoly (noun) A condition in which there is only one 2
12345678901234567890123456789012123456789012345678901
2
12345678901234567890123456789012123456789012345678901
seller of a certain commodity. Wary of Microsofts 2
12345678901234567890123456789012123456789012345678901
2
12345678901234567890123456789012123456789012345678901
seeming monopoly of the computer operating-system 2
12345678901234567890123456789012123456789012345678901
2345678901234567890123456789012123456789012345678901
12345678901234567890123456789012123456789012345678901
business, rivals are asking for government intervention. 2
2
12345678901234567890123456789012123456789012345678901
2
1
monopolistic
(adjective).
12345678901234567890123456789012123456789012345678901 2
12345678901234567890123456789012123456789012345678901 2
12345678901234567890123456789012123456789012345678901
monotonous (adjective) Tediously uniform, unchanging. 2
2
12345678901234567890123456789012123456789012345678901
2
12345678901234567890123456789012123456789012345678901
Brian
Enos
Music
for
Airports
is
characterized
by
2
12345678901234567890123456789012123456789012345678901
2
12345678901234567890123456789012123456789012345678901
minimal
melodies,
subtle
textures,
and
variable
repeti2345678901234567890123456789012123456789012345678901
12345678901234567890123456789012123456789012345678901 22
12345678901234567890123456789012123456789012345678901
tion, which I find rather bland and monotonous. 2
12345678901234567890123456789012123456789012345678901
2
12345678901234567890123456789012123456789012345678901
monotony (noun).
2
1
2345678901234567890123456789012123456789012345678901
2
12345678901234567890123456789012123456789012345678901
morose (adjective) Gloomy, sullen. After Chucks girlfriend 2
12345678901234567890123456789012123456789012345678901
2
12345678901234567890123456789012123456789012345678901
dumped him, he lay around the house for a couple of 2
12345678901234567890123456789012123456789012345678901
2
1
days, refusing to come to the phone and feeling morose. 2
12345678901234567890123456789012123456789012345678901
12345678901234567890123456789012123456789012345678901 2
2
12345678901234567890123456789012123456789012345678901
12345678901234567890123456789012123456789012345678901 2
2
12345678901234567890123456789012123456789012345678901
12345678901234567890123456789012123456789012345678901 2
12345678901234567890123456789012123456789012345678901 2
12345678901234567890123456789012123456789012345678901 2
12345678901234567890123456789012123456789012345678901 2 203
123456789012345678901234567890121234567890123456789012

http://www.xtremepapers.net

Part III: Section Review

123456789012345678901234567890121234567890123456789012
12345678901234567890123456789012123456789012345678901 2
2
12345678901234567890123456789012123456789012345678901
mutation (noun) A significant change; in biology, a perma- 2
12345678901234567890123456789012123456789012345678901
2
12345678901234567890123456789012123456789012345678901
nent change in hereditary material. Most genetic muta- 2
12345678901234567890123456789012123456789012345678901
2
12345678901234567890123456789012123456789012345678901
tions are not beneficial, since any change in the delicate 2
12345678901234567890123456789012123456789012345678901
2345678901234567890123456789012123456789012345678901
2
1
balance of an organism tends to be disruptive. mutate 2
12345678901234567890123456789012123456789012345678901
2
12345678901234567890123456789012123456789012345678901
(verb).
2
12345678901234567890123456789012123456789012345678901
2
12345678901234567890123456789012123456789012345678901
2345678901234567890123456789012123456789012345678901
2
1
2
12345678901234567890123456789012123456789012345678901
2345678901234567890123456789012123456789012345678901
2
1
Latin frangere 5 to break. Also found in English fraction, fractious,
Word
Origin fracture, frangible, infraction, and refract.
2
12345678901234567890123456789012123456789012345678901
2
12345678901234567890123456789012123456789012345678901
2345678901234567890123456789012123456789012345678901
2
1
2
12345678901234567890123456789012123456789012345678901
nadir
(noun)
Lowest
point.
Pedro
and
Renees
marriage
2
12345678901234567890123456789012123456789012345678901
2
12345678901234567890123456789012123456789012345678901
reached
a
new
nadir
last
Christmas
Eve
when
Pedro
2
12345678901234567890123456789012123456789012345678901
2345678901234567890123456789012123456789012345678901
1
locked Renee out of the house upon her return from the 2
2
12345678901234567890123456789012123456789012345678901
2
12345678901234567890123456789012123456789012345678901
supposed business trip.
2
12345678901234567890123456789012123456789012345678901
2
12345678901234567890123456789012123456789012345678901
nascent
(adjective)
Newly
born,
just
beginning.
While
her
2345678901234567890123456789012123456789012345678901
12345678901234567890123456789012123456789012345678901 2
2
12345678901234567890123456789012123456789012345678901
artistry was still nascent, it was 15-year-old Tara 2
12345678901234567890123456789012123456789012345678901
2
12345678901234567890123456789012123456789012345678901
Lipinskis technical wizardry that enabled her to win a 2
12345678901234567890123456789012123456789012345678901
12345678901234567890123456789012123456789012345678901
gold medal in the 1998 Winter Olympics. nascence 2
2
1
2
12345678901234567890123456789012123456789012345678901
(noun).
2
12345678901234567890123456789012123456789012345678901
2
12345678901234567890123456789012123456789012345678901
2345678901234567890123456789012123456789012345678901
12345678901234567890123456789012123456789012345678901
noisome (adjective) Putrid, fetid, noxious. We were con- 2
2
12345678901234567890123456789012123456789012345678901
2
12345678901234567890123456789012123456789012345678901
vinced
that
the
noisome
odor
infiltrating
every
corner
of
12345678901234567890123456789012123456789012345678901 2
2
12345678901234567890123456789012123456789012345678901
our building was evidence of a mouldering corpse.
2
12345678901234567890123456789012123456789012345678901
12345678901234567890123456789012123456789012345678901 2
12345678901234567890123456789012123456789012345678901
notorious (adjective) Famous, especially for evil actions or 2
2
12345678901234567890123456789012123456789012345678901
1
qualities. Warner Brothers produced a series of movies 2
2345678901234567890123456789012123456789012345678901
12345678901234567890123456789012123456789012345678901 2
about notorious gangsters such as John Dillinger and Al 2
12345678901234567890123456789012123456789012345678901
2
1
Capone. notoriety (noun).
2
12345678901234567890123456789012123456789012345678901
2
12345678901234567890123456789012123456789012345678901
2345678901234567890123456789012123456789012345678901
12345678901234567890123456789012123456789012345678901 2
12345678901234567890123456789012123456789012345678901 2
2
1 Origin Latin durus 5 hard. Also found in English durable and endure.
Word
2
12345678901234567890123456789012123456789012345678901
2
12345678901234567890123456789012123456789012345678901
2345678901234567890123456789012123456789012345678901
12345678901234567890123456789012123456789012345678901
obdurate (adjective) Unwilling to change; stubborn, inflex- 2
2
12345678901234567890123456789012123456789012345678901
1
ible. Despite the many pleas he received, the governor 2
12345678901234567890123456789012123456789012345678901 2
12345678901234567890123456789012123456789012345678901
was obdurate in his refusal to grant clemency to the 2
2
12345678901234567890123456789012123456789012345678901
2
12345678901234567890123456789012123456789012345678901
convicted murderer.
2
12345678901234567890123456789012123456789012345678901
2
12345678901234567890123456789012123456789012345678901
oblivious
(adjective)
Unaware,
unconscious.
Karen
prac12345678901234567890123456789012123456789012345678901 2
2345678901234567890123456789012123456789012345678901
2
12345678901234567890123456789012123456789012345678901
ticed her oboe solo with complete concentration, oblivi- 2
12345678901234567890123456789012123456789012345678901
12345678901234567890123456789012123456789012345678901
ous to the noise and activity around her. oblivion (noun), 2
2
12345678901234567890123456789012123456789012345678901
2
1
obviousness
(noun).
2345678901234567890123456789012123456789012345678901
12345678901234567890123456789012123456789012345678901 2
12345678901234567890123456789012123456789012345678901 2
12345678901234567890123456789012123456789012345678901 2
12345678901234567890123456789012123456789012345678901 2
2
1
2
12345678901234567890123456789012123456789012345678901
12345678901234567890123456789012123456789012345678901 2
2
12345678901234567890123456789012123456789012345678901
12345678901234567890123456789012123456789012345678901 2
2
12345678901234567890123456789012123456789012345678901
12345678901234567890123456789012123456789012345678901 2
12345678901234567890123456789012123456789012345678901 2
12345678901234567890123456789012123456789012345678901 2
2
204 12345678901234567890123456789012123456789012345678901
123456789012345678901234567890121234567890123456789012

www.petersons.com

http://www.xtremepapers.net

Chapter 5: Verbal Review

Take It to the Next Level

123456789012345678901234567890121234567890123456789012
12345678901234567890123456789012123456789012345678901 2
2
12345678901234567890123456789012123456789012345678901
obscure (adjective) Little known; hard to understand. Mendel 2
12345678901234567890123456789012123456789012345678901
2
12345678901234567890123456789012123456789012345678901
was an obscure monk until decades after his death, when 2
12345678901234567890123456789012123456789012345678901
2
12345678901234567890123456789012123456789012345678901
his scientific work was finally discovered. Most people 2
12345678901234567890123456789012123456789012345678901
2345678901234567890123456789012123456789012345678901
2
1
find the writings of James Joyce obscure; hence the 2
12345678901234567890123456789012123456789012345678901
12345678901234567890123456789012123456789012345678901
popularity of books that explain the many odd refer- 2
2
12345678901234567890123456789012123456789012345678901
2
12345678901234567890123456789012123456789012345678901
ences
and
tricks
of
language
in
his
work.
obscure
(verb),
2345678901234567890123456789012123456789012345678901
2
1
2
12345678901234567890123456789012123456789012345678901
obscurity
(noun).
2
12345678901234567890123456789012123456789012345678901
2
12345678901234567890123456789012123456789012345678901
2
12345678901234567890123456789012123456789012345678901
obsolete
(adjective)
No
longer
current;
old-fashioned.
W.
H.
2345678901234567890123456789012123456789012345678901
2
1
2
12345678901234567890123456789012123456789012345678901
Auden
said
that
his
ideal
landscape
would
contain
water
2
12345678901234567890123456789012123456789012345678901
2
12345678901234567890123456789012123456789012345678901
wheels,
grain
mills,
and
other
forms
of
obsolete
machin2
12345678901234567890123456789012123456789012345678901
2345678901234567890123456789012123456789012345678901
2
1
ery. obsolescence (noun).
2
12345678901234567890123456789012123456789012345678901
2
12345678901234567890123456789012123456789012345678901
obstinate (adjective) Stubborn, unyielding. Despite years of 2
12345678901234567890123456789012123456789012345678901
2
12345678901234567890123456789012123456789012345678901
government effort, the problem of drug abuse remains 2
12345678901234567890123456789012123456789012345678901
2
12345678901234567890123456789012123456789012345678901
obstinate. obstinacy (noun).
2
12345678901234567890123456789012123456789012345678901
2
12345678901234567890123456789012123456789012345678901
2
12345678901234567890123456789012123456789012345678901
obtuse
(adjective)
Dull-witted,
insensitive;
incomprehen2345678901234567890123456789012123456789012345678901
12345678901234567890123456789012123456789012345678901 22
1
sible, unclear, or imprecise. Amy was so obtuse she 2
12345678901234567890123456789012123456789012345678901
12345678901234567890123456789012123456789012345678901
didnt realize that Alexi had proposed marriage to her. 2
2
12345678901234567890123456789012123456789012345678901
2345678901234567890123456789012123456789012345678901
12345678901234567890123456789012123456789012345678901
French psychoanalyst Jacques Lacans collection of 2
2
12345678901234567890123456789012123456789012345678901
2
12345678901234567890123456789012123456789012345678901
papers,
Ecrits,
is
notoriously
obtuse,
yet
it
has
still
been
12345678901234567890123456789012123456789012345678901 22
12345678901234567890123456789012123456789012345678901
highly influential in linguistics, film theory, and literary 2
12345678901234567890123456789012123456789012345678901
2
12345678901234567890123456789012123456789012345678901
criticism.
2
12345678901234567890123456789012123456789012345678901
12345678901234567890123456789012123456789012345678901 22
1
obviate (verb) Preclude, make unnecessary. Truman Capotes
2
12345678901234567890123456789012123456789012345678901
meticulous accuracy and total recall obviated the need 2
12345678901234567890123456789012123456789012345678901
2
12345678901234567890123456789012123456789012345678901
for note-taking when he wrote his account of a 1959 2
12345678901234567890123456789012123456789012345678901
2
12345678901234567890123456789012123456789012345678901
murder, In Cold Blood.
2
12345678901234567890123456789012123456789012345678901
2
12345678901234567890123456789012123456789012345678901
2
12345678901234567890123456789012123456789012345678901
odium
(noun)
Intense
feeling
of
hatred,
abhorrence.
When
2
12345678901234567890123456789012123456789012345678901
2
12345678901234567890123456789012123456789012345678901
the
neighbors
learned
that
a
convicted
sex
offender
was
2345678901234567890123456789012123456789012345678901
12345678901234567890123456789012123456789012345678901 22
12345678901234567890123456789012123456789012345678901
now living in their midst, they could not restrain their 2
1
12345678901234567890123456789012123456789012345678901
odium and began harassing the man whenever he left his 2
2
12345678901234567890123456789012123456789012345678901
2345678901234567890123456789012123456789012345678901
2
12345678901234567890123456789012123456789012345678901
house. odious (adjective).
2
12345678901234567890123456789012123456789012345678901
12345678901234567890123456789012123456789012345678901 22
12345678901234567890123456789012123456789012345678901
opprobrium (noun) Dishonor, disapproval. Switzerland re2
1
2345678901234567890123456789012123456789012345678901
12345678901234567890123456789012123456789012345678901
cently came under public opprobrium when it was 2
2
12345678901234567890123456789012123456789012345678901
12345678901234567890123456789012123456789012345678901
revealed that Swiss bankers had hoarded the gold the 2
12345678901234567890123456789012123456789012345678901 22
1
Nazis had confiscated from their victims. opprobrious
2
12345678901234567890123456789012123456789012345678901
(adjective).
2
12345678901234567890123456789012123456789012345678901
2
12345678901234567890123456789012123456789012345678901
2
12345678901234567890123456789012123456789012345678901
orthodox
(adjective)
In
religion,
conforming
to
a
certain
12345678901234567890123456789012123456789012345678901 2
2345678901234567890123456789012123456789012345678901
12345678901234567890123456789012123456789012345678901
doctrine; conventional. George Eliots relationship with 2
2
1
2
12345678901234567890123456789012123456789012345678901
George
Lewes,
a
married
journalist,
offended
the
12345678901234567890123456789012123456789012345678901 2
12345678901234567890123456789012123456789012345678901
sensibilities of her more orthodox peers. orthodoxy 2
2
12345678901234567890123456789012123456789012345678901
2
12345678901234567890123456789012123456789012345678901
(noun).
12345678901234567890123456789012123456789012345678901 22
12345678901234567890123456789012123456789012345678901 205
123456789012345678901234567890121234567890123456789012

http://www.xtremepapers.net

Part III: Section Review

123456789012345678901234567890121234567890123456789012
12345678901234567890123456789012123456789012345678901 2
2
12345678901234567890123456789012123456789012345678901
ossified (adjective) In biology, to turn into bone; to become 2
12345678901234567890123456789012123456789012345678901
2
12345678901234567890123456789012123456789012345678901
rigidly conventional and opposed to change. His harsh 2
12345678901234567890123456789012123456789012345678901
2
12345678901234567890123456789012123456789012345678901
view of co-education had ossified over the years so that 2
12345678901234567890123456789012123456789012345678901
2345678901234567890123456789012123456789012345678901
2
1
he was now the only teacher who sought to bar girls 2
12345678901234567890123456789012123456789012345678901
2
12345678901234567890123456789012123456789012345678901
from the venerable boys school. ossification (noun).
2
12345678901234567890123456789012123456789012345678901
2
12345678901234567890123456789012123456789012345678901
2345678901234567890123456789012123456789012345678901
1
ostentatious (adjective) Overly showy, pretentious. To show 2
2
12345678901234567890123456789012123456789012345678901
2
12345678901234567890123456789012123456789012345678901
off
his
new
wealth,
the
financier
threw
an
ostentatious
2
12345678901234567890123456789012123456789012345678901
2
12345678901234567890123456789012123456789012345678901
party
featuring
a
full
orchestra,
a
famous
singer,
and
tens
2345678901234567890123456789012123456789012345678901
2
1
2
12345678901234567890123456789012123456789012345678901
of
thousands
of
dollars
worth
of
food.
ostentation
2
12345678901234567890123456789012123456789012345678901
2
12345678901234567890123456789012123456789012345678901
(noun).
2
12345678901234567890123456789012123456789012345678901
2345678901234567890123456789012123456789012345678901
2
1
12345678901234567890123456789012123456789012345678901
ostracize (verb) To exclude from a group. In Biblical times, 2
2
12345678901234567890123456789012123456789012345678901
those who suffered from the disease of leprosy were 2
12345678901234567890123456789012123456789012345678901
2
12345678901234567890123456789012123456789012345678901
ostracized and forced to live alone. ostracism (noun).
2
12345678901234567890123456789012123456789012345678901
2
12345678901234567890123456789012123456789012345678901
2
12345678901234567890123456789012123456789012345678901
paean
(adjective)
A
joyous
expression
of
praise,
gratitude,
2
12345678901234567890123456789012123456789012345678901
2
12345678901234567890123456789012123456789012345678901
or
triumph.
Choreographer
Paul
Taylors
dance
Even2345678901234567890123456789012123456789012345678901
12345678901234567890123456789012123456789012345678901 2
1
tide is a sublime paean to remembered love, with 2
2
12345678901234567890123456789012123456789012345678901
2
12345678901234567890123456789012123456789012345678901
couple
after
loving
couple
looking
back
as
they
embrace
2
12345678901234567890123456789012123456789012345678901
2345678901234567890123456789012123456789012345678901
2
12345678901234567890123456789012123456789012345678901
an unknown future.
2
12345678901234567890123456789012123456789012345678901
12345678901234567890123456789012123456789012345678901 2
12345678901234567890123456789012123456789012345678901
parody (noun) An imitation created for comic effect; a 2
2
12345678901234567890123456789012123456789012345678901
12345678901234567890123456789012123456789012345678901
caricature. While the creators of the 1970s comedy 2
2
12345678901234567890123456789012123456789012345678901
12345678901234567890123456789012123456789012345678901
series All in the Family intended Archie Bunker to be a 2
12345678901234567890123456789012123456789012345678901 2
1
parody of closed-mindedeness in Americans, large num- 2
2
12345678901234567890123456789012123456789012345678901
bers of people adopted Bunker as a working-class hero. 2
12345678901234567890123456789012123456789012345678901
12345678901234567890123456789012123456789012345678901 2
2
12345678901234567890123456789012123456789012345678901
parse (verb) To break a sentence down into grammatical 2
12345678901234567890123456789012123456789012345678901
2345678901234567890123456789012123456789012345678901
12345678901234567890123456789012123456789012345678901
components; to analyze bit by bit. In the wake of the sex 2
2
12345678901234567890123456789012123456789012345678901
2
1
scandal,
journalists
parsed
every
utterance
by
adminis2
12345678901234567890123456789012123456789012345678901
2
12345678901234567890123456789012123456789012345678901
tration
officials
regarding
the
presidents
alleged
promis2345678901234567890123456789012123456789012345678901
12345678901234567890123456789012123456789012345678901 2
12345678901234567890123456789012123456789012345678901
cuity. At $1.25 million a day, Titanic is the most 2
2
1
2
12345678901234567890123456789012123456789012345678901
expensive
movie
ever
made,
but
director
James
Cameron
12345678901234567890123456789012123456789012345678901 2
2345678901234567890123456789012123456789012345678901
12345678901234567890123456789012123456789012345678901
refused to parse the films enormous budget for inquisi- 2
2
12345678901234567890123456789012123456789012345678901
2
12345678901234567890123456789012123456789012345678901
tive
reporters.
12345678901234567890123456789012123456789012345678901 2
2
1
12345678901234567890123456789012123456789012345678901
partisan (adjective) Reflecting strong allegiance to a particu- 2
2
12345678901234567890123456789012123456789012345678901
12345678901234567890123456789012123456789012345678901
lar party or cause. The vote on the presidents budget 2
2
12345678901234567890123456789012123456789012345678901
12345678901234567890123456789012123456789012345678901
was strictly partisan: Every member of the presidents 2
2345678901234567890123456789012123456789012345678901
2
12345678901234567890123456789012123456789012345678901
party voted yes, and all others voted no. partisan (noun). 2
12345678901234567890123456789012123456789012345678901
12345678901234567890123456789012123456789012345678901 2
12345678901234567890123456789012123456789012345678901
pastoral (adjective) Simple and rustic, bucolic, rural. While 2
2
1
2345678901234567890123456789012123456789012345678901
12345678901234567890123456789012123456789012345678901
industry grew and the country expanded westward, the 2
2
1
2
12345678901234567890123456789012123456789012345678901
Hudson
River
School
of
painters
depicted
the
landscape
12345678901234567890123456789012123456789012345678901 2
12345678901234567890123456789012123456789012345678901
as a pastoral setting where humans and nature could 2
2
12345678901234567890123456789012123456789012345678901
2
12345678901234567890123456789012123456789012345678901
coexist.
12345678901234567890123456789012123456789012345678901 2
2
206 12345678901234567890123456789012123456789012345678901
123456789012345678901234567890121234567890123456789012

www.petersons.com

http://www.xtremepapers.net

Chapter 5: Verbal Review

Take It to the Next Level

123456789012345678901234567890121234567890123456789012
12345678901234567890123456789012123456789012345678901 2
2
12345678901234567890123456789012123456789012345678901
patron (noun) A special guardian or protector; a wealthy or 2
12345678901234567890123456789012123456789012345678901
2
12345678901234567890123456789012123456789012345678901
influential supporter of the arts. Dominique de Menil 2
12345678901234567890123456789012123456789012345678901
2
12345678901234567890123456789012123456789012345678901
used her considerable wealth to become a well-known 2
12345678901234567890123456789012123456789012345678901
2345678901234567890123456789012123456789012345678901
2
1
patron of the arts; she and her husband owned a 2
12345678901234567890123456789012123456789012345678901
12345678901234567890123456789012123456789012345678901
collection of more than 10,000 pieces ranging from 2
2
12345678901234567890123456789012123456789012345678901
2
12345678901234567890123456789012123456789012345678901
cubist
paintings
to
tribal
artifacts.
patronize
(verb).
2345678901234567890123456789012123456789012345678901
2
1
2
12345678901234567890123456789012123456789012345678901
2
12345678901234567890123456789012123456789012345678901
peccadillo
(noun)
A
minor
offense,
a
lapse.
What
Dr.
Sykes
2
12345678901234567890123456789012123456789012345678901
2
12345678901234567890123456789012123456789012345678901
saw
as
a
major
offensebeing
addressed
as
Marge
2345678901234567890123456789012123456789012345678901
2
1
2
12345678901234567890123456789012123456789012345678901
rather
than
DoctorTina
saw
as
a
mere
peccadillo
and
2
12345678901234567890123456789012123456789012345678901
2
12345678901234567890123456789012123456789012345678901
one
that
certainly
should
not
have
lost
her
the
job.
2
12345678901234567890123456789012123456789012345678901
2345678901234567890123456789012123456789012345678901
2
1
12345678901234567890123456789012123456789012345678901
pedantic (adjective) Academic, bookish. The men Hillary 2
2
12345678901234567890123456789012123456789012345678901
met through personal ads in the New York Review of 2
12345678901234567890123456789012123456789012345678901
2
12345678901234567890123456789012123456789012345678901
Books were invariably pasty-skinned pedantic types 2
12345678901234567890123456789012123456789012345678901
2
12345678901234567890123456789012123456789012345678901
who dropped the names of nineteenth-century writers in 2
12345678901234567890123456789012123456789012345678901
2
12345678901234567890123456789012123456789012345678901
every sentence. pedantry (noun).
2
12345678901234567890123456789012123456789012345678901
2345678901234567890123456789012123456789012345678901
12345678901234567890123456789012123456789012345678901 22
1
pedestrian (adjective) Unimaginative, ordinary. The new 2
12345678901234567890123456789012123456789012345678901
12345678901234567890123456789012123456789012345678901
Italian restaurant received a bad review due to its 2
2
12345678901234567890123456789012123456789012345678901
2345678901234567890123456789012123456789012345678901
12345678901234567890123456789012123456789012345678901
reliance on pedestrian dishes such as pasta with marinara 2
2
12345678901234567890123456789012123456789012345678901
2
12345678901234567890123456789012123456789012345678901
sauce
or
chicken
parmigiana.
12345678901234567890123456789012123456789012345678901 22
12345678901234567890123456789012123456789012345678901 2
12345678901234567890123456789012123456789012345678901 2
12345678901234567890123456789012123456789012345678901
Latin fides 5 faith. Also found in English confide, confidence,
2
12345678901234567890123456789012123456789012345678901
Word
Origin fidelity, and infidel.
2
12345678901234567890123456789012123456789012345678901
2
1
2345678901234567890123456789012123456789012345678901
12345678901234567890123456789012123456789012345678901 22
perfidious (adjective) Disloyal, treacherous. Although he
12345678901234567890123456789012123456789012345678901
2
1
was one of the most talented generals of the American 2
12345678901234567890123456789012123456789012345678901
2
12345678901234567890123456789012123456789012345678901
Revolution, Benedict Arnold is remembered today as a 2
12345678901234567890123456789012123456789012345678901
2
12345678901234567890123456789012123456789012345678901
perfidious betrayer of the patriot cause. perfidy (noun). 2
12345678901234567890123456789012123456789012345678901
2
12345678901234567890123456789012123456789012345678901
2
12345678901234567890123456789012123456789012345678901
peripatetic
(adjective)
Moving
or
traveling
from
place
to
2345678901234567890123456789012123456789012345678901
12345678901234567890123456789012123456789012345678901 22
12345678901234567890123456789012123456789012345678901
place; always on the go. In Barbara Wilsons Trouble in 2
1
12345678901234567890123456789012123456789012345678901
Transylvania, peripatetic translator Cassandra Reilly is 2
2
12345678901234567890123456789012123456789012345678901
2345678901234567890123456789012123456789012345678901
12345678901234567890123456789012123456789012345678901
on the road again, this time to China by way of 2
2
12345678901234567890123456789012123456789012345678901
2
12345678901234567890123456789012123456789012345678901
Budapest,
where
she
plans
to
catch
the
TransMongolian
12345678901234567890123456789012123456789012345678901 22
1
Express.
2
12345678901234567890123456789012123456789012345678901
2
12345678901234567890123456789012123456789012345678901
12345678901234567890123456789012123456789012345678901
permeate (verb) To spread through or penetrate. Little by 2
2
12345678901234567890123456789012123456789012345678901
little, the smell of gas from the broken pipe permeated 2
12345678901234567890123456789012123456789012345678901
2345678901234567890123456789012123456789012345678901
2
12345678901234567890123456789012123456789012345678901
the house.
2
12345678901234567890123456789012123456789012345678901
12345678901234567890123456789012123456789012345678901 22
12345678901234567890123456789012123456789012345678901
personification (noun) The embodiment of a thing or an 2
1
12345678901234567890123456789012123456789012345678901
abstract idea in human form. Many people view 2
2
12345678901234567890123456789012123456789012345678901
2
12345678901234567890123456789012123456789012345678901
Theodore
Kaczynski,
the
killer
known
as
the
Unabomber,
12345678901234567890123456789012123456789012345678901 2
2
12345678901234567890123456789012123456789012345678901
as the very personification of evil. personify (verb).
2
12345678901234567890123456789012123456789012345678901
12345678901234567890123456789012123456789012345678901 2
12345678901234567890123456789012123456789012345678901 2
12345678901234567890123456789012123456789012345678901 2 207
123456789012345678901234567890121234567890123456789012

http://www.xtremepapers.net

Part III: Section Review

123456789012345678901234567890121234567890123456789012
12345678901234567890123456789012123456789012345678901 2
2
12345678901234567890123456789012123456789012345678901
pervasive (adjective) Spreading throughout. As news of the 2
12345678901234567890123456789012123456789012345678901
2
12345678901234567890123456789012123456789012345678901
disaster reached the town, a pervasive sense of gloom 2
12345678901234567890123456789012123456789012345678901
2
12345678901234567890123456789012123456789012345678901
could be felt everywhere. pervade (verb).
2
12345678901234567890123456789012123456789012345678901
2345678901234567890123456789012123456789012345678901
2
1
2
12345678901234567890123456789012123456789012345678901
philistine
(noun)
Someone
who
is
smugly
ignorant
and
2
12345678901234567890123456789012123456789012345678901
2
12345678901234567890123456789012123456789012345678901
uncultured.
A
true
philistine,
Meg
claimed
she
didnt
2
12345678901234567890123456789012123456789012345678901
2345678901234567890123456789012123456789012345678901
1
read any book that wasnt either recommended by 2
2
12345678901234567890123456789012123456789012345678901
2
12345678901234567890123456789012123456789012345678901
Oprah
Winfrey
or
on
the
best-seller
list.
philistine
2
12345678901234567890123456789012123456789012345678901
2
12345678901234567890123456789012123456789012345678901
(adjective).
2345678901234567890123456789012123456789012345678901
2
1
2
12345678901234567890123456789012123456789012345678901
12345678901234567890123456789012123456789012345678901
pith (noun) The core, the essential part; in biology, the 2
2
12345678901234567890123456789012123456789012345678901
12345678901234567890123456789012123456789012345678901
central strand of tissue in the stems of most vascular 2
2345678901234567890123456789012123456789012345678901
2
1
12345678901234567890123456789012123456789012345678901
plants. After spending seventeen years in psychoanaly- 2
2
12345678901234567890123456789012123456789012345678901
sis, Frieda had finally come face to face with the pith of 2
12345678901234567890123456789012123456789012345678901
2
12345678901234567890123456789012123456789012345678901
her deep-seated anxiety. pithy (adjective).
2
12345678901234567890123456789012123456789012345678901
2
12345678901234567890123456789012123456789012345678901
2
12345678901234567890123456789012123456789012345678901
placate
(verb)
To
soothe
or
appease.
The
waiter
tried
to
2
12345678901234567890123456789012123456789012345678901
2
12345678901234567890123456789012123456789012345678901
placate
the
angry
customer
with
the
offer
of
a
free
2345678901234567890123456789012123456789012345678901
12345678901234567890123456789012123456789012345678901 2
2
1
dessert. placatory (adjective).
2
12345678901234567890123456789012123456789012345678901
2
12345678901234567890123456789012123456789012345678901
2
12345678901234567890123456789012123456789012345678901
placid
(adjective)
Unmarked
by
disturbance;
complacent.
2345678901234567890123456789012123456789012345678901
12345678901234567890123456789012123456789012345678901 2
12345678901234567890123456789012123456789012345678901
Dr. Kahn was convinced that the placid exterior 2
2
12345678901234567890123456789012123456789012345678901
12345678901234567890123456789012123456789012345678901
presented by Frieda in her early analysis sessions masked 2
12345678901234567890123456789012123456789012345678901 2
2
12345678901234567890123456789012123456789012345678901
a deeply disturbed psyche. placidity (noun).
2
12345678901234567890123456789012123456789012345678901
2
12345678901234567890123456789012123456789012345678901
plaintive (adjective) Expressing suffering or melancholy. In 2
12345678901234567890123456789012123456789012345678901
2
1
the beloved childrens book The Secret Garden, Mary is 2
12345678901234567890123456789012123456789012345678901
2
12345678901234567890123456789012123456789012345678901
disturbed by plaintive cries echoing in the corridors of 2
12345678901234567890123456789012123456789012345678901
2
12345678901234567890123456789012123456789012345678901
gloomy Misselthwaite Manor.
2
12345678901234567890123456789012123456789012345678901
2345678901234567890123456789012123456789012345678901
12345678901234567890123456789012123456789012345678901 2
12345678901234567890123456789012123456789012345678901
plastic (adjective) Able to be molded or reshaped. Because it 2
2
1
2
12345678901234567890123456789012123456789012345678901
is
highly
plastic,
clay
is
an
easy
material
for
beginning
2
12345678901234567890123456789012123456789012345678901
2345678901234567890123456789012123456789012345678901
2
12345678901234567890123456789012123456789012345678901
sculptors to use. plasticity (noun).
2
12345678901234567890123456789012123456789012345678901
2
1
12345678901234567890123456789012123456789012345678901
plausible (adjective) Apparently believable. The idea that a 2
12345678901234567890123456789012123456789012345678901 2
12345678901234567890123456789012123456789012345678901
widespread conspiracy to kill the president has been 2
2
12345678901234567890123456789012123456789012345678901
12345678901234567890123456789012123456789012345678901
kept secret by all the participants for over thirty years 2
2
12345678901234567890123456789012123456789012345678901
2
12345678901234567890123456789012123456789012345678901
hardly seems plausible. plausibility (noun).
2345678901234567890123456789012123456789012345678901
12345678901234567890123456789012123456789012345678901 2
2
12345678901234567890123456789012123456789012345678901
platitude (noun) A trite remark or saying; a clich. How 2
12345678901234567890123456789012123456789012345678901
12345678901234567890123456789012123456789012345678901
typical of June to send a sympathy card filled with 2
2
1
2345678901234567890123456789012123456789012345678901
12345678901234567890123456789012123456789012345678901
mindless platitudes like One day at a time, rather than 2
2
12345678901234567890123456789012123456789012345678901
2
12345678901234567890123456789012123456789012345678901
calling
the
grieving
widow
on
the
phone.
platitudinous
12345678901234567890123456789012123456789012345678901 2
2
1
(adjective).
2
12345678901234567890123456789012123456789012345678901
12345678901234567890123456789012123456789012345678901 2
12345678901234567890123456789012123456789012345678901
plummet (verb) To dive or plunge. On October 27, 1997, 2
2
12345678901234567890123456789012123456789012345678901
2
12345678901234567890123456789012123456789012345678901
the
stock
market
plummeted
by
554
points
and
left
us
all
2345678901234567890123456789012123456789012345678901
2
1
2
12345678901234567890123456789012123456789012345678901
wondering if the bull market was finally over.
12345678901234567890123456789012123456789012345678901 2
2
208 12345678901234567890123456789012123456789012345678901
123456789012345678901234567890121234567890123456789012

www.petersons.com

http://www.xtremepapers.net

Chapter 5: Verbal Review

Take It to the Next Level

123456789012345678901234567890121234567890123456789012
12345678901234567890123456789012123456789012345678901 2
2
12345678901234567890123456789012123456789012345678901
polarize (adjective) To separate into opposing groups or 2
12345678901234567890123456789012123456789012345678901
2
12345678901234567890123456789012123456789012345678901
forces. For years, the abortion debate polarized the 2
12345678901234567890123456789012123456789012345678901
2
12345678901234567890123456789012123456789012345678901
American people, with many people voicing views at 2
12345678901234567890123456789012123456789012345678901
2345678901234567890123456789012123456789012345678901
2
1
either extreme and few people trying to find a middle 2
12345678901234567890123456789012123456789012345678901
2
12345678901234567890123456789012123456789012345678901
ground. polarization (noun).
2
12345678901234567890123456789012123456789012345678901
2
12345678901234567890123456789012123456789012345678901
2345678901234567890123456789012123456789012345678901
1
ponderous (adjective) Unwieldy and bulky; oppressively 2
2
12345678901234567890123456789012123456789012345678901
2
12345678901234567890123456789012123456789012345678901
dull.
Unfortunately,
the
film
director
weighed
the
movie
2
12345678901234567890123456789012123456789012345678901
2
12345678901234567890123456789012123456789012345678901
down
with
a
ponderous
voice-over
narrated
by
the
2345678901234567890123456789012123456789012345678901
2
1
2
12345678901234567890123456789012123456789012345678901
protagonist
as
an
old
man.
2
12345678901234567890123456789012123456789012345678901
2
12345678901234567890123456789012123456789012345678901
12345678901234567890123456789012123456789012345678901
poseur (noun) Someone who pretends to be what he isnt. 2
2345678901234567890123456789012123456789012345678901
2
1
12345678901234567890123456789012123456789012345678901
Gerald had pretensions for literary stardom with his 2
2
12345678901234567890123456789012123456789012345678901
book proposal on an obscure World War II battle, yet 2
12345678901234567890123456789012123456789012345678901
2
12345678901234567890123456789012123456789012345678901
most agents soon realized that the book would never be 2
12345678901234567890123456789012123456789012345678901
2
12345678901234567890123456789012123456789012345678901
written and categorized him as a poseur.
2
12345678901234567890123456789012123456789012345678901
2
12345678901234567890123456789012123456789012345678901
2
12345678901234567890123456789012123456789012345678901
positivism
(noun)
A
philosophy
that
denies
speculation
and
2345678901234567890123456789012123456789012345678901
12345678901234567890123456789012123456789012345678901 22
1
assumes that the only knowledge is scientific knowledge. 2
12345678901234567890123456789012123456789012345678901
12345678901234567890123456789012123456789012345678901
David Hume carried his positivism to an extreme when 2
2
12345678901234567890123456789012123456789012345678901
2345678901234567890123456789012123456789012345678901
12345678901234567890123456789012123456789012345678901
he argued that our expectation that the sun will rise 2
2
12345678901234567890123456789012123456789012345678901
2
12345678901234567890123456789012123456789012345678901
tomorrow
has
no
basis
in
reason
and
is
purely
a
matter
12345678901234567890123456789012123456789012345678901 22
12345678901234567890123456789012123456789012345678901
of belief. positivistic (adjective).
2
12345678901234567890123456789012123456789012345678901
12345678901234567890123456789012123456789012345678901 22
12345678901234567890123456789012123456789012345678901
pragmatism (noun) A belief in approaching problems
2
12345678901234567890123456789012123456789012345678901
1
through practical rather than theoretical means. 2
2345678901234567890123456789012123456789012345678901
12345678901234567890123456789012123456789012345678901 22
Roosevelts attitude toward the economic troubles of the
12345678901234567890123456789012123456789012345678901
2
1
Depression was based on pragmatism: Try some- 2
12345678901234567890123456789012123456789012345678901
2
12345678901234567890123456789012123456789012345678901
thing, he said; If it doesnt work, try something else. 2
12345678901234567890123456789012123456789012345678901
2
12345678901234567890123456789012123456789012345678901
pragmatic (adjective).
2
12345678901234567890123456789012123456789012345678901
2
12345678901234567890123456789012123456789012345678901
2
12345678901234567890123456789012123456789012345678901
precedent
(noun)
An
earlier
occurrence
that
serves
as
an
2345678901234567890123456789012123456789012345678901
12345678901234567890123456789012123456789012345678901 22
12345678901234567890123456789012123456789012345678901
example for a decision. In a legal system that reveres 2
1
12345678901234567890123456789012123456789012345678901
precedent, even defining the nature of a completely new 2
2
12345678901234567890123456789012123456789012345678901
2345678901234567890123456789012123456789012345678901
2
12345678901234567890123456789012123456789012345678901
type of dispute can seem impossible. precede (verb).
2
12345678901234567890123456789012123456789012345678901
12345678901234567890123456789012123456789012345678901 22
12345678901234567890123456789012123456789012345678901
precept (noun) A general principle or law. One of the
2
1
2345678901234567890123456789012123456789012345678901
12345678901234567890123456789012123456789012345678901
central precepts of Tai Chi Chuan is the necessity of 2
2
12345678901234567890123456789012123456789012345678901
12345678901234567890123456789012123456789012345678901
allowing ki (cosmic energy) to flow through ones body 2
12345678901234567890123456789012123456789012345678901 22
1
in slow, graceful movements.
2
12345678901234567890123456789012123456789012345678901
2
12345678901234567890123456789012123456789012345678901
precipitate (verb) To spur or activate. In the summer of 2
12345678901234567890123456789012123456789012345678901
12345678901234567890123456789012123456789012345678901
1997, the selling off of the Thai baht precipitated a 2
2
12345678901234567890123456789012123456789012345678901
2345678901234567890123456789012123456789012345678901
2
12345678901234567890123456789012123456789012345678901
currency crisis that spread throughout Asia.
2
1
2
12345678901234567890123456789012123456789012345678901
12345678901234567890123456789012123456789012345678901 2
2
12345678901234567890123456789012123456789012345678901
12345678901234567890123456789012123456789012345678901 2
12345678901234567890123456789012123456789012345678901 2
12345678901234567890123456789012123456789012345678901 2
12345678901234567890123456789012123456789012345678901 2 209
123456789012345678901234567890121234567890123456789012

http://www.xtremepapers.net

Part III: Section Review

123456789012345678901234567890121234567890123456789012
12345678901234567890123456789012123456789012345678901 2
2
12345678901234567890123456789012123456789012345678901
2
12345678901234567890123456789012123456789012345678901
Latin claudere 5 to close. Also found in English conclude, include,
2345678901234567890123456789012123456789012345678901
2
1
Word
Origin
2
12345678901234567890123456789012123456789012345678901
recluse, and seclude.
12345678901234567890123456789012123456789012345678901 2
2
12345678901234567890123456789012123456789012345678901
preclude (verb) To prevent, to hinder. Unfortunately, Jas- 2
12345678901234567890123456789012123456789012345678901
2
12345678901234567890123456789012123456789012345678901
mines appointment at the New Age Expo precluded her 2
12345678901234567890123456789012123456789012345678901
2
12345678901234567890123456789012123456789012345678901
attendance at our weekend Workshop for Shamans and 2
12345678901234567890123456789012123456789012345678901
2345678901234567890123456789012123456789012345678901
2
1
Psychics. preclusive (adjective), preclusion (noun).
2
12345678901234567890123456789012123456789012345678901
2
12345678901234567890123456789012123456789012345678901
2
12345678901234567890123456789012123456789012345678901
precursor
(noun)
A
forerunner,
a
predecessor.
The
Kodak
2
12345678901234567890123456789012123456789012345678901
2345678901234567890123456789012123456789012345678901
1
Brownie camera, a small boxy camera made of jute 2
2
12345678901234567890123456789012123456789012345678901
2
12345678901234567890123456789012123456789012345678901
board
and
wood,
was
the
precursor
to
todays
sleek
2
12345678901234567890123456789012123456789012345678901
2
12345678901234567890123456789012123456789012345678901
35mm
cameras.
precursory
(adjective).
2345678901234567890123456789012123456789012345678901
2
1
2
12345678901234567890123456789012123456789012345678901
12345678901234567890123456789012123456789012345678901
preponderance (noun) A superiority in weight, size, or 2
2
12345678901234567890123456789012123456789012345678901
12345678901234567890123456789012123456789012345678901
quantity; a majority. In Seattle, there is a great prepon- 2
2345678901234567890123456789012123456789012345678901
12345678901234567890123456789012123456789012345678901 2
12345678901234567890123456789012123456789012345678901
derance of seasonal affective disorder, or SAD, a malady 2
2
12345678901234567890123456789012123456789012345678901
brought on by light starvation during the dark North- 2
12345678901234567890123456789012123456789012345678901
2
12345678901234567890123456789012123456789012345678901
west winter. preponderate (verb).
2
12345678901234567890123456789012123456789012345678901
2
1
2
12345678901234567890123456789012123456789012345678901
presage
(verb)
To
foretell,
to
anticipate.
According
to
2
12345678901234567890123456789012123456789012345678901
2
12345678901234567890123456789012123456789012345678901
folklore,
a
red
sky
at
dawn
presages
a
day
of
stormy
2345678901234567890123456789012123456789012345678901
12345678901234567890123456789012123456789012345678901 2
2
12345678901234567890123456789012123456789012345678901
weather.
2
12345678901234567890123456789012123456789012345678901
12345678901234567890123456789012123456789012345678901 2
12345678901234567890123456789012123456789012345678901
prescience (noun) Foreknowledge or foresight. When she 2
2
12345678901234567890123456789012123456789012345678901
2
12345678901234567890123456789012123456789012345678901
saw
the
characteristic
eerie
yellowish-black
light
in
the
12345678901234567890123456789012123456789012345678901 2
12345678901234567890123456789012123456789012345678901
sky, Dorothy had the prescience to seek shelter in the 2
2
1
2345678901234567890123456789012123456789012345678901
2
12345678901234567890123456789012123456789012345678901
storm cellar. prescient (adjective).
2
12345678901234567890123456789012123456789012345678901
2
1
presumptuous (adjective) Going beyond the limits of cour- 2
12345678901234567890123456789012123456789012345678901
2
12345678901234567890123456789012123456789012345678901
tesy or appropriateness. The senator winced when the 2
12345678901234567890123456789012123456789012345678901
2
12345678901234567890123456789012123456789012345678901
presumptuous young staffer addressed him as Ted. 2
12345678901234567890123456789012123456789012345678901
2
12345678901234567890123456789012123456789012345678901
presume (verb), presumption (noun).
2
12345678901234567890123456789012123456789012345678901
2345678901234567890123456789012123456789012345678901
12345678901234567890123456789012123456789012345678901 2
12345678901234567890123456789012123456789012345678901
prevaricate (verb) To lie, to equivocate. When it became 2
2
1
2
12345678901234567890123456789012123456789012345678901
clear
to
the
FBI
that
the
mobster
had
threatened
the
12345678901234567890123456789012123456789012345678901 2
2345678901234567890123456789012123456789012345678901
12345678901234567890123456789012123456789012345678901
12-year-old witness, they could well understand why he 2
2
12345678901234567890123456789012123456789012345678901
2
12345678901234567890123456789012123456789012345678901
had
prevaricated
during
the
hearing.
12345678901234567890123456789012123456789012345678901 2
2
1
2
12345678901234567890123456789012123456789012345678901
2
12345678901234567890123456789012123456789012345678901
Latin primus 5 first. Also found in English primate, primitive,
2
12345678901234567890123456789012123456789012345678901
Word
Origin
2
12345678901234567890123456789012123456789012345678901
primogeniture, and primodial.
12345678901234567890123456789012123456789012345678901 2
2345678901234567890123456789012123456789012345678901
2
12345678901234567890123456789012123456789012345678901
primacy (noun) State of being the utmost in importance; 2
12345678901234567890123456789012123456789012345678901
2
12345678901234567890123456789012123456789012345678901
preeminence. The anthropologist Ruth Benedict was an 2
12345678901234567890123456789012123456789012345678901
2
1
inspiration to Margaret Mead for her emphasis on the 2
12345678901234567890123456789012123456789012345678901
2
12345678901234567890123456789012123456789012345678901
primacy of culture in the formation of an individuals 2
12345678901234567890123456789012123456789012345678901
2
12345678901234567890123456789012123456789012345678901
personality. primal (adjective).
2
12345678901234567890123456789012123456789012345678901
2345678901234567890123456789012123456789012345678901
2
1
12345678901234567890123456789012123456789012345678901 2
12345678901234567890123456789012123456789012345678901 2
2
210 12345678901234567890123456789012123456789012345678901
123456789012345678901234567890121234567890123456789012

www.petersons.com

http://www.xtremepapers.net

Chapter 5: Verbal Review

Take It to the Next Level

123456789012345678901234567890121234567890123456789012
12345678901234567890123456789012123456789012345678901 2
2
12345678901234567890123456789012123456789012345678901
pristine (adjective) Pure, undefiled. As climbers who have 2
12345678901234567890123456789012123456789012345678901
2
12345678901234567890123456789012123456789012345678901
scaled Mt. Everest can attest, the trails to the summit are 2
12345678901234567890123456789012123456789012345678901
2
12345678901234567890123456789012123456789012345678901
hardly in pristine condition and are actually strewn with 2
12345678901234567890123456789012123456789012345678901
2345678901234567890123456789012123456789012345678901
2
1
trash.
2
12345678901234567890123456789012123456789012345678901
2
12345678901234567890123456789012123456789012345678901
2
12345678901234567890123456789012123456789012345678901
probity
(noun)
Goodness,
integrity.
The
vicious
editorial
2
12345678901234567890123456789012123456789012345678901
2345678901234567890123456789012123456789012345678901
1
attacked the moral probity of the senatorial candidate, 2
2
12345678901234567890123456789012123456789012345678901
2
12345678901234567890123456789012123456789012345678901
saying
he
had
profited
handsomely
from
his
pet
project,
2
12345678901234567890123456789012123456789012345678901
2
12345678901234567890123456789012123456789012345678901
the
senior-citizen
housing
project.
2345678901234567890123456789012123456789012345678901
2
1
2
12345678901234567890123456789012123456789012345678901
12345678901234567890123456789012123456789012345678901
procure (verb) To obtain by using particular care and effort. 2
2
12345678901234567890123456789012123456789012345678901
12345678901234567890123456789012123456789012345678901
Through partnerships with a large number of specialty 2
2345678901234567890123456789012123456789012345678901
2
1
12345678901234567890123456789012123456789012345678901
wholesalers, W.W. Grainger is able to procure a startling 2
2
12345678901234567890123456789012123456789012345678901
array of products for its customers, from bear repellent 2
12345678901234567890123456789012123456789012345678901
2
12345678901234567890123456789012123456789012345678901
for Alaska pipeline workers to fork-lift trucks and toilet 2
12345678901234567890123456789012123456789012345678901
2
12345678901234567890123456789012123456789012345678901
paper. procurement (noun).
2
12345678901234567890123456789012123456789012345678901
2
12345678901234567890123456789012123456789012345678901
2
12345678901234567890123456789012123456789012345678901
prodigality
(noun)
The
condition
of
being
wastefully
extrava2345678901234567890123456789012123456789012345678901
12345678901234567890123456789012123456789012345678901 22
1
gant. Richard was ashamed of the prodigality of his 2
12345678901234567890123456789012123456789012345678901
12345678901234567890123456789012123456789012345678901
brides parents when he realized that the cost of the 2
2
12345678901234567890123456789012123456789012345678901
2345678901234567890123456789012123456789012345678901
12345678901234567890123456789012123456789012345678901
wedding reception alone was more than his father 2
2
12345678901234567890123456789012123456789012345678901
2
12345678901234567890123456789012123456789012345678901
earned
in
one
year.
prodigal
(adjective).
12345678901234567890123456789012123456789012345678901 22
12345678901234567890123456789012123456789012345678901 2
12345678901234567890123456789012123456789012345678901
proliferate (verb) To increase or multiply. Over the past
2
12345678901234567890123456789012123456789012345678901
12345678901234567890123456789012123456789012345678901
fifteen years, high-tech companies have proliferated in 2
12345678901234567890123456789012123456789012345678901 22
1
northern California, Massachusetts, and other regions.
2
12345678901234567890123456789012123456789012345678901
2
proliferation (noun).
12345678901234567890123456789012123456789012345678901
12345678901234567890123456789012123456789012345678901 2
2
12345678901234567890123456789012123456789012345678901
prolixity (noun) A diffuseness; a rambling and verbose 2
12345678901234567890123456789012123456789012345678901
2345678901234567890123456789012123456789012345678901
12345678901234567890123456789012123456789012345678901
quality. The prolixity of Sarahs dissertation on Otto- 2
2
12345678901234567890123456789012123456789012345678901
2
1
man
history
defied
even
her
advisers
attempts
to
read
it.
2
12345678901234567890123456789012123456789012345678901
2
12345678901234567890123456789012123456789012345678901
prolix
(adjective).
2345678901234567890123456789012123456789012345678901
12345678901234567890123456789012123456789012345678901 22
12345678901234567890123456789012123456789012345678901 2
1
prophetic (adjective) Auspicious, predictive of whats to 2
12345678901234567890123456789012123456789012345678901
12345678901234567890123456789012123456789012345678901
come. We often look at every event leading up to a new 2
2
12345678901234567890123456789012123456789012345678901
2
12345678901234567890123456789012123456789012345678901
love
affair
as
propheticthe
flat
tire
that
caused
us
to
be
2
12345678901234567890123456789012123456789012345678901
12345678901234567890123456789012123456789012345678901
late for work, the chance meeting in the elevator, the 2
12345678901234567890123456789012123456789012345678901 2
12345678901234567890123456789012123456789012345678901
horoscope that argued a new beginning. prophecy 2
2
12345678901234567890123456789012123456789012345678901
2
12345678901234567890123456789012123456789012345678901
(noun), prophesy (verb).
2
12345678901234567890123456789012123456789012345678901
2
12345678901234567890123456789012123456789012345678901
propagate
(verb)
To
cause
to
grow;
to
foster.
John
Smithsons
2345678901234567890123456789012123456789012345678901
12345678901234567890123456789012123456789012345678901 22
12345678901234567890123456789012123456789012345678901
will left his fortune for the founding of an institution to 2
12345678901234567890123456789012123456789012345678901
12345678901234567890123456789012123456789012345678901
propagate knowledge, leaving open whether that meant 2
2
1
2345678901234567890123456789012123456789012345678901
12345678901234567890123456789012123456789012345678901
a university, a library, or a museum. propagation (noun). 2
2
1
2
12345678901234567890123456789012123456789012345678901
12345678901234567890123456789012123456789012345678901 2
2
12345678901234567890123456789012123456789012345678901
12345678901234567890123456789012123456789012345678901 2
12345678901234567890123456789012123456789012345678901 2
12345678901234567890123456789012123456789012345678901 2
12345678901234567890123456789012123456789012345678901 2 211
123456789012345678901234567890121234567890123456789012

http://www.xtremepapers.net

Part III: Section Review

123456789012345678901234567890121234567890123456789012
12345678901234567890123456789012123456789012345678901 2
2
12345678901234567890123456789012123456789012345678901
propitiating (adjective) Conciliatory, mollifying or appeas- 2
12345678901234567890123456789012123456789012345678901
2
12345678901234567890123456789012123456789012345678901
ing. Managements offer of a five-percent raise was 2
12345678901234567890123456789012123456789012345678901
2
12345678901234567890123456789012123456789012345678901
meant as a propitiating gesture, yet the striking workers 2
12345678901234567890123456789012123456789012345678901
2345678901234567890123456789012123456789012345678901
2
1
were unimpressed. propitiate (verb).
2
12345678901234567890123456789012123456789012345678901
2
12345678901234567890123456789012123456789012345678901
2
12345678901234567890123456789012123456789012345678901
propriety
(noun)
Appropriateness.
Some
people
expressed
2
12345678901234567890123456789012123456789012345678901
2345678901234567890123456789012123456789012345678901
1
doubts about the propriety of Clintons discussing his 2
2
12345678901234567890123456789012123456789012345678901
2
12345678901234567890123456789012123456789012345678901
underwear
on
MTV.
2
12345678901234567890123456789012123456789012345678901
2
12345678901234567890123456789012123456789012345678901
2345678901234567890123456789012123456789012345678901
1
proximity (noun) Closeness, nearness. Neighborhood resi- 2
2
12345678901234567890123456789012123456789012345678901
12345678901234567890123456789012123456789012345678901
dents were angry over the proximity of the proposed 2
2
12345678901234567890123456789012123456789012345678901
12345678901234567890123456789012123456789012345678901
sewage plant to the local elementary school. proximate 2
2345678901234567890123456789012123456789012345678901
2
1
2
12345678901234567890123456789012123456789012345678901
(adjective).
2
12345678901234567890123456789012123456789012345678901
2
12345678901234567890123456789012123456789012345678901
pundit
(noun)
Someone
who
offers
opinions
in
an
authori2
12345678901234567890123456789012123456789012345678901
2345678901234567890123456789012123456789012345678901
2
12345678901234567890123456789012123456789012345678901
tative style. The Sunday morning talk shows are filled 2
12345678901234567890123456789012123456789012345678901
12345678901234567890123456789012123456789012345678901
with pundits, each with his or her own theory about this 2
2
12345678901234567890123456789012123456789012345678901
2
12345678901234567890123456789012123456789012345678901
weeks
political
news.
12345678901234567890123456789012123456789012345678901 2
2
1
2
12345678901234567890123456789012123456789012345678901
2
12345678901234567890123456789012123456789012345678901
Latin pungere = to jab, to prick. Also found in English pugilist,
2
12345678901234567890123456789012123456789012345678901
Word
Origin
2345678901234567890123456789012123456789012345678901
2
12345678901234567890123456789012123456789012345678901
punctuate, and puncture.
2
12345678901234567890123456789012123456789012345678901
12345678901234567890123456789012123456789012345678901 2
12345678901234567890123456789012123456789012345678901
pungency (noun) Marked by having a sharp, biting quality. 2
2
12345678901234567890123456789012123456789012345678901
12345678901234567890123456789012123456789012345678901
Unfortunately, the pungency of the fresh cilantro over- 2
2
12345678901234567890123456789012123456789012345678901
12345678901234567890123456789012123456789012345678901
whelmed the delicate flavor of the poached flounder. 2
12345678901234567890123456789012123456789012345678901 2
2
1
pungent (adjective).
2
12345678901234567890123456789012123456789012345678901
2
12345678901234567890123456789012123456789012345678901
purify (verb) To make pure, clean, or perfect. The new 2
12345678901234567890123456789012123456789012345678901
2
12345678901234567890123456789012123456789012345678901
water-treatment plant is supposed to purify the drinking 2
12345678901234567890123456789012123456789012345678901
2345678901234567890123456789012123456789012345678901
12345678901234567890123456789012123456789012345678901
water provided to everyone in the nearby towns. 2
2
12345678901234567890123456789012123456789012345678901
2
1
purification
(noun).
2
12345678901234567890123456789012123456789012345678901
2
12345678901234567890123456789012123456789012345678901
2345678901234567890123456789012123456789012345678901
12345678901234567890123456789012123456789012345678901
quiescent (adjective) In a state of rest or inactivity; latent. 2
2
12345678901234567890123456789012123456789012345678901
2
1
Pollys
ulcer
has
been
quiescent
ever
since
her
mother-in12345678901234567890123456789012123456789012345678901 2
12345678901234567890123456789012123456789012345678901
law moved out of the condo, which was well over a year 2
2
12345678901234567890123456789012123456789012345678901
2
12345678901234567890123456789012123456789012345678901
ago.
quiescence
(noun).
2
12345678901234567890123456789012123456789012345678901
2
12345678901234567890123456789012123456789012345678901
12345678901234567890123456789012123456789012345678901
quixotic (adjective) Foolishly romantic, idealistic to an 2
2345678901234567890123456789012123456789012345678901
2
12345678901234567890123456789012123456789012345678901
impractical degree. In the novel Shoeless Joe, Ray 2
12345678901234567890123456789012123456789012345678901
2
12345678901234567890123456789012123456789012345678901
Kinsella carries out a quixotic plan to build a baseball 2
12345678901234567890123456789012123456789012345678901
2
1
field in the hopes that past baseball greats will come to 2
12345678901234567890123456789012123456789012345678901
2
12345678901234567890123456789012123456789012345678901
play there.
2
12345678901234567890123456789012123456789012345678901
2
12345678901234567890123456789012123456789012345678901
12345678901234567890123456789012123456789012345678901 2
2345678901234567890123456789012123456789012345678901
12345678901234567890123456789012123456789012345678901 2
2
1
2
12345678901234567890123456789012123456789012345678901
12345678901234567890123456789012123456789012345678901 2
2
12345678901234567890123456789012123456789012345678901
12345678901234567890123456789012123456789012345678901 2
12345678901234567890123456789012123456789012345678901 2
12345678901234567890123456789012123456789012345678901 2
2
212 12345678901234567890123456789012123456789012345678901
123456789012345678901234567890121234567890123456789012

www.petersons.com

http://www.xtremepapers.net

Chapter 5: Verbal Review

Take It to the Next Level

123456789012345678901234567890121234567890123456789012
12345678901234567890123456789012123456789012345678901 2
2
12345678901234567890123456789012123456789012345678901
quotidian (adjective) Occurring every day; commonplace 2
12345678901234567890123456789012123456789012345678901
2
12345678901234567890123456789012123456789012345678901
and ordinary. Most of the time, we long to escape from 2
12345678901234567890123456789012123456789012345678901
2
12345678901234567890123456789012123456789012345678901
quotidian concerns, but in the midst of a crisis we want 2
12345678901234567890123456789012123456789012345678901
2345678901234567890123456789012123456789012345678901
2
1
nothing more than to be plagued by such simple 2
12345678901234567890123456789012123456789012345678901
2
12345678901234567890123456789012123456789012345678901
problems as a leaky faucet or a whining child.
2
12345678901234567890123456789012123456789012345678901
2
12345678901234567890123456789012123456789012345678901
2345678901234567890123456789012123456789012345678901
1
raconteur (noun) An excellent storyteller. A member of the 2
2
12345678901234567890123456789012123456789012345678901
2
12345678901234567890123456789012123456789012345678901
Algonquin
Round
Table,
Robert
Benchley
was
a
natural
2
12345678901234567890123456789012123456789012345678901
2
12345678901234567890123456789012123456789012345678901
raconteur
with
a
seemingly
endless
ability
to
turn
daily
2345678901234567890123456789012123456789012345678901
2
1
2
12345678901234567890123456789012123456789012345678901
life
and
its
irritations
into
entertaining
commentary.
2
12345678901234567890123456789012123456789012345678901
2
12345678901234567890123456789012123456789012345678901
12345678901234567890123456789012123456789012345678901
rancorous (adjective) Marked by deeply embedded bitter- 2
2345678901234567890123456789012123456789012345678901
2
1
12345678901234567890123456789012123456789012345678901
ness or animosity. While Ralph and Kishu have been 2
2
12345678901234567890123456789012123456789012345678901
separated for three years, their relationship is so 2
12345678901234567890123456789012123456789012345678901
2
12345678901234567890123456789012123456789012345678901
rancorous that they had to hire a professional mediator 2
12345678901234567890123456789012123456789012345678901
2
12345678901234567890123456789012123456789012345678901
just to discuss divorce arrangements. rancor (noun).
2
12345678901234567890123456789012123456789012345678901
2
12345678901234567890123456789012123456789012345678901
2
12345678901234567890123456789012123456789012345678901
rapacious
(adjective)
Excessively
grasping
or
greedy.
Some
2345678901234567890123456789012123456789012345678901
12345678901234567890123456789012123456789012345678901 22
1
see global currency speculators like George Soros as 2
12345678901234567890123456789012123456789012345678901
12345678901234567890123456789012123456789012345678901
rapacious parasites who destroy economies and then 2
2
12345678901234567890123456789012123456789012345678901
2345678901234567890123456789012123456789012345678901
2
12345678901234567890123456789012123456789012345678901
line their pockets with the profits. rapacity (noun).
2
12345678901234567890123456789012123456789012345678901
12345678901234567890123456789012123456789012345678901 22
12345678901234567890123456789012123456789012345678901
rarefied (adjective) Of interest or relating to a small, refined 2
12345678901234567890123456789012123456789012345678901
12345678901234567890123456789012123456789012345678901
circle; less dense, thinner. Those whose names dot the 2
2
12345678901234567890123456789012123456789012345678901
12345678901234567890123456789012123456789012345678901
society pages live in a rarefied world where its entirely 2
12345678901234567890123456789012123456789012345678901 22
1
normal to dine on caviar for breakfast or order a $2,000
2
12345678901234567890123456789012123456789012345678901
bottle of wine at Le Cirque. When she reached the 2
12345678901234567890123456789012123456789012345678901
2
12345678901234567890123456789012123456789012345678901
summit of Mt. McKinley, Deborah could hardly breathe 2
12345678901234567890123456789012123456789012345678901
2
12345678901234567890123456789012123456789012345678901
in the rarefied air.
2
12345678901234567890123456789012123456789012345678901
2
12345678901234567890123456789012123456789012345678901
2
12345678901234567890123456789012123456789012345678901
raucous
(adjective)
Boisterous,
unruly
and
wild.
Sounds
of
2
12345678901234567890123456789012123456789012345678901
2
12345678901234567890123456789012123456789012345678901
shouts
and
raucous
laughter
drifted
out
of
the
hotel
2345678901234567890123456789012123456789012345678901
12345678901234567890123456789012123456789012345678901 22
12345678901234567890123456789012123456789012345678901
room where Felipes bachelor party was being held.
2
1
12345678901234567890123456789012123456789012345678901 2
12345678901234567890123456789012123456789012345678901
reactionary (adjective) Ultra conservative. Every day, over 2
2
12345678901234567890123456789012123456789012345678901
2
12345678901234567890123456789012123456789012345678901
twenty
million
listeners
tune
in
to
hear
Rush
Limbaugh
2
12345678901234567890123456789012123456789012345678901
12345678901234567890123456789012123456789012345678901
spew his reactionary opinions about feminazis and 2
12345678901234567890123456789012123456789012345678901 2
2
12345678901234567890123456789012123456789012345678901
environmental fanatics. reactionary (noun).
2
12345678901234567890123456789012123456789012345678901
2
12345678901234567890123456789012123456789012345678901
recede (verb) To draw back, to ebb, to abate. Once his 2
12345678901234567890123456789012123456789012345678901
2
12345678901234567890123456789012123456789012345678901
hairline began to recede, Hap took to wearing bizarre 2
12345678901234567890123456789012123456789012345678901
2
12345678901234567890123456789012123456789012345678901
accessories, like velvet ascots, to divert attention from it. 2
12345678901234567890123456789012123456789012345678901
2
12345678901234567890123456789012123456789012345678901
recession (noun).
2
12345678901234567890123456789012123456789012345678901
2345678901234567890123456789012123456789012345678901
12345678901234567890123456789012123456789012345678901 22
1
reclusive (adjective) Withdrawn from society. During the 2
12345678901234567890123456789012123456789012345678901
12345678901234567890123456789012123456789012345678901
last years of her life, Garbo led a reclusive existence, 2
2
12345678901234567890123456789012123456789012345678901
2345678901234567890123456789012123456789012345678901
2
1
rarely appearing in public. recluse (noun).
2
12345678901234567890123456789012123456789012345678901
12345678901234567890123456789012123456789012345678901 22
12345678901234567890123456789012123456789012345678901 213
123456789012345678901234567890121234567890123456789012

http://www.xtremepapers.net

Part III: Section Review

123456789012345678901234567890121234567890123456789012
12345678901234567890123456789012123456789012345678901 2
2
12345678901234567890123456789012123456789012345678901
reconcile (verb) To make consistent or harmonious. 2
12345678901234567890123456789012123456789012345678901
2
12345678901234567890123456789012123456789012345678901
Roosevelts greatness as a leader can be seen in his 2
12345678901234567890123456789012123456789012345678901
2
12345678901234567890123456789012123456789012345678901
ability to reconcile the differing demands and values of 2
12345678901234567890123456789012123456789012345678901
2345678901234567890123456789012123456789012345678901
2
1
the varied groups that supported him. reconciliation 2
12345678901234567890123456789012123456789012345678901
2
12345678901234567890123456789012123456789012345678901
(noun).
2
12345678901234567890123456789012123456789012345678901
2
12345678901234567890123456789012123456789012345678901
2345678901234567890123456789012123456789012345678901
1
recompense (noun) Compensation for a service rendered or 2
2
12345678901234567890123456789012123456789012345678901
2
12345678901234567890123456789012123456789012345678901
to
pay
for
damages.
The
5%
of
the
estate
that
Phil
2
12345678901234567890123456789012123456789012345678901
2
12345678901234567890123456789012123456789012345678901
received
as
executor
of
his
Aunt
Idas
will
is
small
2345678901234567890123456789012123456789012345678901
2
1
2
12345678901234567890123456789012123456789012345678901
recompense
for
the
headaches
he
endured
in
settling
her
2
12345678901234567890123456789012123456789012345678901
2
12345678901234567890123456789012123456789012345678901
affairs.
recompense
(verb).
2
12345678901234567890123456789012123456789012345678901
2345678901234567890123456789012123456789012345678901
2
1
12345678901234567890123456789012123456789012345678901
recondite (adjective) Profound, deep, abstruse. Professor 2
2
12345678901234567890123456789012123456789012345678901
Miyakis recondite knowledge of seventeenth-century 2
12345678901234567890123456789012123456789012345678901
2
12345678901234567890123456789012123456789012345678901
Flemish painters made him a prizedif barely under- 2
12345678901234567890123456789012123456789012345678901
2
12345678901234567890123456789012123456789012345678901
stoodmember of the art history department.
2
12345678901234567890123456789012123456789012345678901
2
12345678901234567890123456789012123456789012345678901
2
12345678901234567890123456789012123456789012345678901
redemptive
(adjective)
Liberating
and
reforming.
While
she
2345678901234567890123456789012123456789012345678901
12345678901234567890123456789012123456789012345678901 2
1
doesnt attend formal church services, Carrie is a firm 2
2
12345678901234567890123456789012123456789012345678901
2
12345678901234567890123456789012123456789012345678901
believer
in
the
redemptive
power
of
prayer.
redeem
2
12345678901234567890123456789012123456789012345678901
2345678901234567890123456789012123456789012345678901
2
12345678901234567890123456789012123456789012345678901
(verb), redemption (noun).
2
12345678901234567890123456789012123456789012345678901
12345678901234567890123456789012123456789012345678901 2
12345678901234567890123456789012123456789012345678901 2
2
12345678901234567890123456789012123456789012345678901
Latin frangere = to break. Also found in English fraction, fractions,
2
12345678901234567890123456789012123456789012345678901
Word
Origin fracture, frangible, infraction, and refract.
2
12345678901234567890123456789012123456789012345678901
12345678901234567890123456789012123456789012345678901 2
12345678901234567890123456789012123456789012345678901 2
1
refractory (adjective) Stubbornly resisting control or author- 2
2
12345678901234567890123456789012123456789012345678901
ity. Like a refractory child, Jill stomped out of the car, 2
12345678901234567890123456789012123456789012345678901
2
12345678901234567890123456789012123456789012345678901
slammed the door, and said she would walk home, even 2
12345678901234567890123456789012123456789012345678901
2
12345678901234567890123456789012123456789012345678901
though her house was 10 miles away.
2
12345678901234567890123456789012123456789012345678901
2
12345678901234567890123456789012123456789012345678901
2
12345678901234567890123456789012123456789012345678901
relevance
(noun)
Connection
to
the
matter
at
hand;
perti2
12345678901234567890123456789012123456789012345678901
2
12345678901234567890123456789012123456789012345678901
nence.
Testimony
in
a
criminal
trial
may
only
be
2345678901234567890123456789012123456789012345678901
12345678901234567890123456789012123456789012345678901 2
12345678901234567890123456789012123456789012345678901
admitted to the extent that it has clear relevance to the 2
2
1
2
12345678901234567890123456789012123456789012345678901
question
of
guilt
or
innocence.
relevant
(adjective).
12345678901234567890123456789012123456789012345678901 2
2345678901234567890123456789012123456789012345678901
12345678901234567890123456789012123456789012345678901 2
12345678901234567890123456789012123456789012345678901
reparation (noun) The act of making amends; payment of 2
2
12345678901234567890123456789012123456789012345678901
12345678901234567890123456789012123456789012345678901
damages by a defeated nation to the victors. The Treaty 2
2
1
12345678901234567890123456789012123456789012345678901
of Versailles, signed in 1919, formally asserted Germa- 2
2
12345678901234567890123456789012123456789012345678901
12345678901234567890123456789012123456789012345678901
nys war guilt and ordered it to pay reparations to the 2
2
12345678901234567890123456789012123456789012345678901
2
allies.
12345678901234567890123456789012123456789012345678901
2345678901234567890123456789012123456789012345678901
12345678901234567890123456789012123456789012345678901 2
2
12345678901234567890123456789012123456789012345678901
reproof (noun) A reprimand, a reproach or castigation. Joe 2
12345678901234567890123456789012123456789012345678901
12345678901234567890123456789012123456789012345678901
thought being grounded for one month was a harsh 2
2
1
2345678901234567890123456789012123456789012345678901
12345678901234567890123456789012123456789012345678901
reproof for coming home late only once. reprove (verb). 2
2
1
2
12345678901234567890123456789012123456789012345678901
12345678901234567890123456789012123456789012345678901 2
2
12345678901234567890123456789012123456789012345678901
12345678901234567890123456789012123456789012345678901 2
12345678901234567890123456789012123456789012345678901 2
12345678901234567890123456789012123456789012345678901 2
2
214 12345678901234567890123456789012123456789012345678901
123456789012345678901234567890121234567890123456789012

www.petersons.com

http://www.xtremepapers.net

Chapter 5: Verbal Review

Take It to the Next Level

123456789012345678901234567890121234567890123456789012
12345678901234567890123456789012123456789012345678901 2
2
12345678901234567890123456789012123456789012345678901
repugnant (adjective) Causing dislike or disgust. After the 2
12345678901234567890123456789012123456789012345678901
2
12345678901234567890123456789012123456789012345678901
news broke about Mad Cow Disease, much of the 2
12345678901234567890123456789012123456789012345678901
2
12345678901234567890123456789012123456789012345678901
beef-loving British public began to find the thought of a 2
12345678901234567890123456789012123456789012345678901
2345678901234567890123456789012123456789012345678901
2
1
Sunday roast repugnant.
2
12345678901234567890123456789012123456789012345678901
2
12345678901234567890123456789012123456789012345678901
2
12345678901234567890123456789012123456789012345678901
requiem
(noun)
A
musical
composition
or
poem
written
to
2
12345678901234567890123456789012123456789012345678901
2345678901234567890123456789012123456789012345678901
1
honor the dead. Many financial analysts think that the 2
2
12345678901234567890123456789012123456789012345678901
2
12345678901234567890123456789012123456789012345678901
ailing
typewriter
company
should
simply
say
a
requiem
2
12345678901234567890123456789012123456789012345678901
2
12345678901234567890123456789012123456789012345678901
for
itself
and
shut
down;
however,
the
CEO
has
other
plans.
2345678901234567890123456789012123456789012345678901
2
1
2
12345678901234567890123456789012123456789012345678901
12345678901234567890123456789012123456789012345678901
repudiate (verb) To reject, to renounce. After it became 2
2
12345678901234567890123456789012123456789012345678901
12345678901234567890123456789012123456789012345678901
known that Duke had been a leader of the Ku Klux 2
2345678901234567890123456789012123456789012345678901
2
1
12345678901234567890123456789012123456789012345678901
Klan, most Republican leaders repudiated him. repudia- 2
2
12345678901234567890123456789012123456789012345678901
2
tion (noun).
12345678901234567890123456789012123456789012345678901
2
12345678901234567890123456789012123456789012345678901
2345678901234567890123456789012123456789012345678901
2
12345678901234567890123456789012123456789012345678901
resilient (adjective) Able to recover from difficulty. A pro 2
12345678901234567890123456789012123456789012345678901
12345678901234567890123456789012123456789012345678901
athlete must be mentally resilient, able to lose a game 2
2
12345678901234567890123456789012123456789012345678901
2
12345678901234567890123456789012123456789012345678901
one
day
and
come
back
the
next
with
renewed
enthusi12345678901234567890123456789012123456789012345678901 22
1
asm and confidence. resilience (noun).
2
12345678901234567890123456789012123456789012345678901
2
12345678901234567890123456789012123456789012345678901
2
12345678901234567890123456789012123456789012345678901
resonant
(adjective)
Full
of
special
import
or
meaning.
I
2345678901234567890123456789012123456789012345678901
12345678901234567890123456789012123456789012345678901 22
12345678901234567890123456789012123456789012345678901
found the speakers words particularly resonant because 2
12345678901234567890123456789012123456789012345678901
12345678901234567890123456789012123456789012345678901
I, too, had served in Vietnam and felt the same mixture 2
2
12345678901234567890123456789012123456789012345678901
2
12345678901234567890123456789012123456789012345678901
of shame and pride. resonance (noun).
2
12345678901234567890123456789012123456789012345678901
2
12345678901234567890123456789012123456789012345678901
resplendent (adjective) Glowing, shining. In late December, 2
12345678901234567890123456789012123456789012345678901
2
1
midtown New York is resplendent with holiday lights 2
12345678901234567890123456789012123456789012345678901
2
12345678901234567890123456789012123456789012345678901
and decorations. resplendence (noun).
2
12345678901234567890123456789012123456789012345678901
12345678901234567890123456789012123456789012345678901 2
12345678901234567890123456789012123456789012345678901
rite (noun) Ceremony. From October to May, the Patwin 2
2
12345678901234567890123456789012123456789012345678901
2
12345678901234567890123456789012123456789012345678901
Indians
of
Californias
Sacramento
Valley
held
a
series
12345678901234567890123456789012123456789012345678901 2
12345678901234567890123456789012123456789012345678901
of rites and dances designed to bring the tribe health and 2
2
12345678901234567890123456789012123456789012345678901
2345678901234567890123456789012123456789012345678901
2
12345678901234567890123456789012123456789012345678901
prosperity.
2
12345678901234567890123456789012123456789012345678901
2
1
12345678901234567890123456789012123456789012345678901
rogue (noun) A mischievously dishonest person; a scamp. In 2
12345678901234567890123456789012123456789012345678901 2
12345678901234567890123456789012123456789012345678901
Jane Austens Pride and Prejudice, Wickham, a charm- 2
2
12345678901234567890123456789012123456789012345678901
12345678901234567890123456789012123456789012345678901
ing rogue, seduces Darcys young sister Georgiana and 2
2
12345678901234567890123456789012123456789012345678901
2
12345678901234567890123456789012123456789012345678901
later does the same thing with Kitty Bennett.
2345678901234567890123456789012123456789012345678901
12345678901234567890123456789012123456789012345678901 22
12345678901234567890123456789012123456789012345678901
ruffian (noun) A brute, roughneck or bully. In Dickens 2
12345678901234567890123456789012123456789012345678901
12345678901234567890123456789012123456789012345678901
Oliver Twist, Fagin instructs his gang of orphaned 2
2
1
2345678901234567890123456789012123456789012345678901
2
12345678901234567890123456789012123456789012345678901
ruffians on the arts of picking pockets and shoplifting.
2
12345678901234567890123456789012123456789012345678901
12345678901234567890123456789012123456789012345678901 22
12345678901234567890123456789012123456789012345678901
rumination (noun) The act of engaging in contemplation. 2
1
12345678901234567890123456789012123456789012345678901
Marcel Prousts semi-autobiographical novel cycle, Re- 2
2
12345678901234567890123456789012123456789012345678901
2
12345678901234567890123456789012123456789012345678901
membrance
of
Things
Past,
is
less
a
narrative
than
an
12345678901234567890123456789012123456789012345678901 2
12345678901234567890123456789012123456789012345678901
extended rumination on the nature of memory. ruminate 2
2
12345678901234567890123456789012123456789012345678901
2
12345678901234567890123456789012123456789012345678901
(verb).
12345678901234567890123456789012123456789012345678901 22
12345678901234567890123456789012123456789012345678901 215
123456789012345678901234567890121234567890123456789012

http://www.xtremepapers.net

Part III: Section Review

123456789012345678901234567890121234567890123456789012
12345678901234567890123456789012123456789012345678901 2
2
12345678901234567890123456789012123456789012345678901
sage (noun) A person of great wisdom, a sagacious philoso- 2
12345678901234567890123456789012123456789012345678901
2
12345678901234567890123456789012123456789012345678901
pher. It was the Chinese sage Confucius who first taught 2
12345678901234567890123456789012123456789012345678901
2
12345678901234567890123456789012123456789012345678901
what is now known the world over as The Golden 2
12345678901234567890123456789012123456789012345678901
2345678901234567890123456789012123456789012345678901
2
1
Rule. sagacious (adjective), sagacity (noun).
2
12345678901234567890123456789012123456789012345678901
2
12345678901234567890123456789012123456789012345678901
2
12345678901234567890123456789012123456789012345678901
2
12345678901234567890123456789012123456789012345678901
2345678901234567890123456789012123456789012345678901
2
1
Latin salus 5 health. Also found in English salubrious, salutation,
2
12345678901234567890123456789012123456789012345678901
Word
Origin and salute.
2
12345678901234567890123456789012123456789012345678901
2
12345678901234567890123456789012123456789012345678901
2
12345678901234567890123456789012123456789012345678901
salutary
(adjective)
Restorative,
healthful.
I
find
a
short
dip
2345678901234567890123456789012123456789012345678901
2
1
2
12345678901234567890123456789012123456789012345678901
in
an
icy
stream
to
be
extremely
salutary,
although
the
2
12345678901234567890123456789012123456789012345678901
2
12345678901234567890123456789012123456789012345678901
health
benefits
of
my
bracing
swims
are,
as
yet,
unclear.
2
12345678901234567890123456789012123456789012345678901
2345678901234567890123456789012123456789012345678901
2
1
12345678901234567890123456789012123456789012345678901
sanction (verb) Support or authorize. Even after a bomb 2
2
12345678901234567890123456789012123456789012345678901
exploded on the front porch of his home, the Reverend 2
12345678901234567890123456789012123456789012345678901
2
12345678901234567890123456789012123456789012345678901
Martin Luther King Jr. refused to sanction any violent 2
12345678901234567890123456789012123456789012345678901
2
12345678901234567890123456789012123456789012345678901
response and urged his angry followers to love their 2
12345678901234567890123456789012123456789012345678901
2
12345678901234567890123456789012123456789012345678901
enemies. sanctify (verb), sanction (noun).
2
12345678901234567890123456789012123456789012345678901
2345678901234567890123456789012123456789012345678901
12345678901234567890123456789012123456789012345678901 2
1
sap (verb) To exhaust, to deplete. The exhaustive twelve-city 2
2
12345678901234567890123456789012123456789012345678901
2
12345678901234567890123456789012123456789012345678901
reading
tour
so
sapped
the
novelists
strength
that
she
2
12345678901234567890123456789012123456789012345678901
2345678901234567890123456789012123456789012345678901
12345678901234567890123456789012123456789012345678901
told her publicist that she hoped her next book would be 2
2
12345678901234567890123456789012123456789012345678901
2
12345678901234567890123456789012123456789012345678901
a
flop!
While
Uganda
is
making
enormous
economic
12345678901234567890123456789012123456789012345678901 2
12345678901234567890123456789012123456789012345678901
strides under President Yoweri Musevini, rebel fighting 2
2
12345678901234567890123456789012123456789012345678901
2
12345678901234567890123456789012123456789012345678901
has
sapped
much
of
the
countrys
resources.
12345678901234567890123456789012123456789012345678901 2
12345678901234567890123456789012123456789012345678901 2
1
satiate (verb) To fulfill to or beyond capacity. Judging by the 2
2
12345678901234567890123456789012123456789012345678901
current crop of films featuring serial killers, rape, ritual 2
12345678901234567890123456789012123456789012345678901
2
12345678901234567890123456789012123456789012345678901
murder, gunslinging, and plain old-fashioned slugfests, 2
12345678901234567890123456789012123456789012345678901
2
12345678901234567890123456789012123456789012345678901
the public appetite for violence has not yet been satiated. 2
12345678901234567890123456789012123456789012345678901
2
12345678901234567890123456789012123456789012345678901
satiation (noun), satiety (noun).
2
12345678901234567890123456789012123456789012345678901
2
12345678901234567890123456789012123456789012345678901
2
12345678901234567890123456789012123456789012345678901
saturate
(verb)
To
drench
or
suffuse
with
liquid
or
anything
2345678901234567890123456789012123456789012345678901
12345678901234567890123456789012123456789012345678901 2
12345678901234567890123456789012123456789012345678901
that permeates or invades. The hostesss furious dabbing 2
2
1
2
12345678901234567890123456789012123456789012345678901
at
the
tablecloth
was
in
vain,
since
the
spilt
wine
had
12345678901234567890123456789012123456789012345678901 2
2345678901234567890123456789012123456789012345678901
12345678901234567890123456789012123456789012345678901
already saturated the damask cloth. saturation (noun), 2
2
12345678901234567890123456789012123456789012345678901
2
12345678901234567890123456789012123456789012345678901
saturated
(adjective).
12345678901234567890123456789012123456789012345678901 2
2
1
12345678901234567890123456789012123456789012345678901
scrutinize (verb) To study closely. The lawyer scrutinized 2
2
12345678901234567890123456789012123456789012345678901
12345678901234567890123456789012123456789012345678901
the contract, searching for any detail that could pose a 2
2
12345678901234567890123456789012123456789012345678901
2
12345678901234567890123456789012123456789012345678901
risk for her client. scrutiny (noun).
2345678901234567890123456789012123456789012345678901
12345678901234567890123456789012123456789012345678901 2
2
12345678901234567890123456789012123456789012345678901
scurvy (adjective) Shabby, low. I couldnt believe that 2
12345678901234567890123456789012123456789012345678901
12345678901234567890123456789012123456789012345678901
Farouk was so scurvy as to open up my computer files 2
2
1
2345678901234567890123456789012123456789012345678901
2
12345678901234567890123456789012123456789012345678901
and read my e-mail.
2
1
2
12345678901234567890123456789012123456789012345678901
12345678901234567890123456789012123456789012345678901 2
2
12345678901234567890123456789012123456789012345678901
12345678901234567890123456789012123456789012345678901 2
12345678901234567890123456789012123456789012345678901 2
12345678901234567890123456789012123456789012345678901 2
2
216 12345678901234567890123456789012123456789012345678901
123456789012345678901234567890121234567890123456789012

www.petersons.com

http://www.xtremepapers.net

Chapter 5: Verbal Review

Take It to the Next Level

123456789012345678901234567890121234567890123456789012
12345678901234567890123456789012123456789012345678901 2
2
12345678901234567890123456789012123456789012345678901
2
12345678901234567890123456789012123456789012345678901
Latin sequi 5 to follow. Also found in English consequence,
2345678901234567890123456789012123456789012345678901
2
1
Word
Origin
2
12345678901234567890123456789012123456789012345678901
sequel, and subsequent.
12345678901234567890123456789012123456789012345678901 2
2
12345678901234567890123456789012123456789012345678901
sequential (adjective) Arranged in an order or series. The 2
12345678901234567890123456789012123456789012345678901
2
12345678901234567890123456789012123456789012345678901
courses required for the chemistry major are sequential; 2
12345678901234567890123456789012123456789012345678901
2
12345678901234567890123456789012123456789012345678901
you must take them in the prescribed order since each 2
12345678901234567890123456789012123456789012345678901
2345678901234567890123456789012123456789012345678901
2
1
course builds on the previous ones. sequence (noun).
2
12345678901234567890123456789012123456789012345678901
2
12345678901234567890123456789012123456789012345678901
2
12345678901234567890123456789012123456789012345678901
sedulous
(adjective)
Diligent,
industrious.
Those
who
are
2
12345678901234567890123456789012123456789012345678901
2345678901234567890123456789012123456789012345678901
1
most sedulous about studying this vocabulary list are 2
2
12345678901234567890123456789012123456789012345678901
2
12345678901234567890123456789012123456789012345678901
likely
to
breeze
through
the
antonyms
sections
of
their
2
12345678901234567890123456789012123456789012345678901
2
12345678901234567890123456789012123456789012345678901
GRE
exam.
2345678901234567890123456789012123456789012345678901
2
1
2
12345678901234567890123456789012123456789012345678901
12345678901234567890123456789012123456789012345678901
sidereal (adjective) Relating to the stars or the constella- 2
2
12345678901234567890123456789012123456789012345678901
12345678901234567890123456789012123456789012345678901
tions. Jacqueline was interested in matters sidereal and 2
2345678901234567890123456789012123456789012345678901
12345678901234567890123456789012123456789012345678901 22
12345678901234567890123456789012123456789012345678901
was always begging my father to take the dusty old
2
12345678901234567890123456789012123456789012345678901
telescope out of our garage.
2
12345678901234567890123456789012123456789012345678901
12345678901234567890123456789012123456789012345678901 22
12345678901234567890123456789012123456789012345678901
signatory (noun) Someone who signs an official document 2
1
12345678901234567890123456789012123456789012345678901
or petition along with others. Alex urged me to join the 2
2
12345678901234567890123456789012123456789012345678901
2
12345678901234567890123456789012123456789012345678901
other
signatories
and
add
my
name
to
the
petition
2345678901234567890123456789012123456789012345678901
12345678901234567890123456789012123456789012345678901 22
12345678901234567890123456789012123456789012345678901
against toxic sludge in organic foods, but I simply did 2
12345678901234567890123456789012123456789012345678901
12345678901234567890123456789012123456789012345678901
not care enough about the issue. The signatories of the 2
2
12345678901234567890123456789012123456789012345678901
12345678901234567890123456789012123456789012345678901
Declaration of Independence included John Adams, 2
2
12345678901234567890123456789012123456789012345678901
2
12345678901234567890123456789012123456789012345678901
Benjamin
Franklin,
John
Hancock,
and
Thomas
12345678901234567890123456789012123456789012345678901 22
1
Jefferson.
2
12345678901234567890123456789012123456789012345678901
2
12345678901234567890123456789012123456789012345678901
12345678901234567890123456789012123456789012345678901
sinuous (noun) Winding, circuitous, serpentine. Frank 2
12345678901234567890123456789012123456789012345678901 2
12345678901234567890123456789012123456789012345678901
Gehrys sinuous design for the Guggenheim Museum in 2
2
12345678901234567890123456789012123456789012345678901
Bilbao, Spain, has led people to hail the museum as the 2
12345678901234567890123456789012123456789012345678901
2
12345678901234567890123456789012123456789012345678901
first great building of the twenty-first century. sinuosity 2
12345678901234567890123456789012123456789012345678901
2
12345678901234567890123456789012123456789012345678901
(noun).
2
12345678901234567890123456789012123456789012345678901
2
12345678901234567890123456789012123456789012345678901
2
12345678901234567890123456789012123456789012345678901
specious
(adjective)
Deceptively
plausible
or
attractive.
The
12345678901234567890123456789012123456789012345678901 2
12345678901234567890123456789012123456789012345678901
infomercial for Fat-Away offered mainly specious 2
2
12345678901234567890123456789012123456789012345678901
2
12345678901234567890123456789012123456789012345678901
arguments
for
a
product
that
is,
essentially,
a
heavy-duty
2
12345678901234567890123456789012123456789012345678901
2
12345678901234567890123456789012123456789012345678901
girdle.
12345678901234567890123456789012123456789012345678901 2
2345678901234567890123456789012123456789012345678901
12345678901234567890123456789012123456789012345678901 22
12345678901234567890123456789012123456789012345678901
spontaneous (adjective) Happening without plan or outside 2
12345678901234567890123456789012123456789012345678901
12345678901234567890123456789012123456789012345678901
cause. When the news of Kennedys assassination hit the 2
2
1
2345678901234567890123456789012123456789012345678901
12345678901234567890123456789012123456789012345678901
airwaves, people everywhere gathered in a spontaneous 2
2
12345678901234567890123456789012123456789012345678901
12345678901234567890123456789012123456789012345678901
effort to express their shock and grief. spontaneity 2
12345678901234567890123456789012123456789012345678901 22
(noun).
1
2
12345678901234567890123456789012123456789012345678901
12345678901234567890123456789012123456789012345678901 2
2
12345678901234567890123456789012123456789012345678901
12345678901234567890123456789012123456789012345678901 2
2
12345678901234567890123456789012123456789012345678901
12345678901234567890123456789012123456789012345678901 2
12345678901234567890123456789012123456789012345678901 2
12345678901234567890123456789012123456789012345678901 2
12345678901234567890123456789012123456789012345678901 2 217
123456789012345678901234567890121234567890123456789012

http://www.xtremepapers.net

Part III: Section Review

123456789012345678901234567890121234567890123456789012
12345678901234567890123456789012123456789012345678901 2
2
12345678901234567890123456789012123456789012345678901
spurious (adjective) False, fake. The so-called Piltdown 2
12345678901234567890123456789012123456789012345678901
2
12345678901234567890123456789012123456789012345678901
Man, supposed to be the fossil of a primitive human, 2
12345678901234567890123456789012123456789012345678901
2
12345678901234567890123456789012123456789012345678901
turned out to be spurious, though who created the hoax 2
12345678901234567890123456789012123456789012345678901
2345678901234567890123456789012123456789012345678901
2
1
is still uncertain.
2
12345678901234567890123456789012123456789012345678901
2
12345678901234567890123456789012123456789012345678901
2
12345678901234567890123456789012123456789012345678901
splice
(verb)
To
unite
by
interweaving
separate
strands
or
2
12345678901234567890123456789012123456789012345678901
2345678901234567890123456789012123456789012345678901
1
parts. Amateur filmmaker Duddy Kravitz shocked and 2
2
12345678901234567890123456789012123456789012345678901
2
12345678901234567890123456789012123456789012345678901
angered
his
clients
by
splicing
footage
of
tribal
rituals
2
12345678901234567890123456789012123456789012345678901
2
12345678901234567890123456789012123456789012345678901
into
his
films
of
their
weddings
and
bar
mitzvahs.
2345678901234567890123456789012123456789012345678901
2
1
2
12345678901234567890123456789012123456789012345678901
12345678901234567890123456789012123456789012345678901
squander (verb) To use up carelessly, to waste. Those who 2
2
12345678901234567890123456789012123456789012345678901
12345678901234567890123456789012123456789012345678901
had made donations to the charity were outraged to 2
2345678901234567890123456789012123456789012345678901
2
1
12345678901234567890123456789012123456789012345678901
learn that its director had squandered millions on fancy 2
2
12345678901234567890123456789012123456789012345678901
dinners, first-class travel, and an expensive apartment 2
12345678901234567890123456789012123456789012345678901
2
12345678901234567890123456789012123456789012345678901
for entertaining.
2
12345678901234567890123456789012123456789012345678901
2
12345678901234567890123456789012123456789012345678901
2
12345678901234567890123456789012123456789012345678901
stanch
(verb)
To
stop
the
flow.
When
Edison
began
to
bleed
2
12345678901234567890123456789012123456789012345678901
2
12345678901234567890123456789012123456789012345678901
profusely,
Dr.
Munger
stanched
the
blood
flow
by
2345678901234567890123456789012123456789012345678901
12345678901234567890123456789012123456789012345678901 2
2
1
applying direct pressure to the wound.
2
12345678901234567890123456789012123456789012345678901
2
12345678901234567890123456789012123456789012345678901
2
12345678901234567890123456789012123456789012345678901
stint
(verb)
To
limit,
to
restrain.
The
British
bed
and
2345678901234567890123456789012123456789012345678901
12345678901234567890123456789012123456789012345678901 2
12345678901234567890123456789012123456789012345678901
breakfast certainly did not stint on the breakfast part of 2
2
12345678901234567890123456789012123456789012345678901
12345678901234567890123456789012123456789012345678901
the equation; they provided us with fried tomatoes, fried 2
12345678901234567890123456789012123456789012345678901 2
12345678901234567890123456789012123456789012345678901
sausages, fried eggs, smoked kippers, fried bread, fried 2
2
12345678901234567890123456789012123456789012345678901
12345678901234567890123456789012123456789012345678901
mushrooms, and bowls of a cereal called Wheatabix 2
12345678901234567890123456789012123456789012345678901 2
2
1
(which tasted like cardboard). stinting (adjective).
2
12345678901234567890123456789012123456789012345678901
2
12345678901234567890123456789012123456789012345678901
stolid (adjective) Impassive, unemotional. The popular ani- 2
12345678901234567890123456789012123456789012345678901
2
12345678901234567890123456789012123456789012345678901
mated television series King of the Hill chronicles the 2
12345678901234567890123456789012123456789012345678901
2345678901234567890123456789012123456789012345678901
12345678901234567890123456789012123456789012345678901
woes of a stolid, conservative Texan confronting chang- 2
2
12345678901234567890123456789012123456789012345678901
2
1
ing
times.
stolidity
(noun).
2
12345678901234567890123456789012123456789012345678901
2
12345678901234567890123456789012123456789012345678901
2345678901234567890123456789012123456789012345678901
12345678901234567890123456789012123456789012345678901
subordination (noun) The state of being subservient or 2
2
12345678901234567890123456789012123456789012345678901
2
1
treated
as
less
valuable.
Heather
left
the
naval
academy
12345678901234567890123456789012123456789012345678901 2
12345678901234567890123456789012123456789012345678901
because she could no longer stand the subordination of 2
2
12345678901234567890123456789012123456789012345678901
2
12345678901234567890123456789012123456789012345678901
every
personal
whim
or
desire
to
the
rigorous
demands
2
12345678901234567890123456789012123456789012345678901
2
12345678901234567890123456789012123456789012345678901
of military life. subordinate (verb).
12345678901234567890123456789012123456789012345678901 2
2345678901234567890123456789012123456789012345678901
12345678901234567890123456789012123456789012345678901 2
12345678901234567890123456789012123456789012345678901 2
12345678901234567890123456789012123456789012345678901 2
12345678901234567890123456789012123456789012345678901 2
2
1
2
12345678901234567890123456789012123456789012345678901
2
12345678901234567890123456789012123456789012345678901
2
12345678901234567890123456789012123456789012345678901
2
12345678901234567890123456789012123456789012345678901
12345678901234567890123456789012123456789012345678901 2
2345678901234567890123456789012123456789012345678901
12345678901234567890123456789012123456789012345678901 2
2
1
2
12345678901234567890123456789012123456789012345678901
12345678901234567890123456789012123456789012345678901 2
2
12345678901234567890123456789012123456789012345678901
12345678901234567890123456789012123456789012345678901 2
12345678901234567890123456789012123456789012345678901 2
12345678901234567890123456789012123456789012345678901 2
2
218 12345678901234567890123456789012123456789012345678901
123456789012345678901234567890121234567890123456789012

www.petersons.com

http://www.xtremepapers.net

Chapter 5: Verbal Review

Take It to the Next Level

123456789012345678901234567890121234567890123456789012
12345678901234567890123456789012123456789012345678901 2
2
12345678901234567890123456789012123456789012345678901
2
12345678901234567890123456789012123456789012345678901
Latin poena 5 pain. Also found in English impunity, penal, penalty,
2345678901234567890123456789012123456789012345678901
2
1
Word
Origin
2
12345678901234567890123456789012123456789012345678901
and punishment.
12345678901234567890123456789012123456789012345678901 2
2
12345678901234567890123456789012123456789012345678901
subpoena (noun) An order of a court, legislation or grand 2
12345678901234567890123456789012123456789012345678901
2
12345678901234567890123456789012123456789012345678901
jury that compels a witness to be present at a trial or 2
12345678901234567890123456789012123456789012345678901
2
12345678901234567890123456789012123456789012345678901
hearing. The young mans lawyer asked the judge to 2
12345678901234567890123456789012123456789012345678901
2345678901234567890123456789012123456789012345678901
2
1
subpoena a boa constrictor into court on the grounds 2
12345678901234567890123456789012123456789012345678901
12345678901234567890123456789012123456789012345678901
that the police had used the snake as an instrument of 2
2
12345678901234567890123456789012123456789012345678901
2
12345678901234567890123456789012123456789012345678901
terror
to
coerce
his
confession.
2345678901234567890123456789012123456789012345678901
2
1
2
12345678901234567890123456789012123456789012345678901
2
12345678901234567890123456789012123456789012345678901
subside
(verb)
To
settle
or
die
down.
The
celebrated
lecturer
2
12345678901234567890123456789012123456789012345678901
2
12345678901234567890123456789012123456789012345678901
had
to
wait
ten
minutes
for
the
applause
to
subside
2345678901234567890123456789012123456789012345678901
2
1
2
12345678901234567890123456789012123456789012345678901
before
he
began
his
speech.
2
12345678901234567890123456789012123456789012345678901
2
12345678901234567890123456789012123456789012345678901
12345678901234567890123456789012123456789012345678901
subsidization (noun) The state of being financed by a grant 2
2345678901234567890123456789012123456789012345678901
12345678901234567890123456789012123456789012345678901 22
12345678901234567890123456789012123456789012345678901
from a government or other agency. Without subsidiza2
12345678901234567890123456789012123456789012345678901
tion, the nations passenger rail system would probably 2
12345678901234567890123456789012123456789012345678901
2
12345678901234567890123456789012123456789012345678901
go bankrupt. subsidize (verb).
2
12345678901234567890123456789012123456789012345678901
2
1
2
12345678901234567890123456789012123456789012345678901
substantiated
(adjective)
Verified
or
supported
by
evidence.
2
12345678901234567890123456789012123456789012345678901
2
12345678901234567890123456789012123456789012345678901
The
charge
that
Nixon
had
helped
to
cover
up
crimes
2345678901234567890123456789012123456789012345678901
12345678901234567890123456789012123456789012345678901 22
12345678901234567890123456789012123456789012345678901
was substantiated by his comments about it on a series 2
12345678901234567890123456789012123456789012345678901
12345678901234567890123456789012123456789012345678901
of audiotapes. substantiate (verb), substantiation (noun). 2
2
12345678901234567890123456789012123456789012345678901
12345678901234567890123456789012123456789012345678901 22
12345678901234567890123456789012123456789012345678901
subsume (verb) To encompass or engulf within something 2
12345678901234567890123456789012123456789012345678901
12345678901234567890123456789012123456789012345678901
larger. In Alan Dershowitzs Reversal of Fortune, he 2
2
1
2345678901234567890123456789012123456789012345678901
12345678901234567890123456789012123456789012345678901
makes it clear that his work as a lawyer subsumes his 2
2
12345678901234567890123456789012123456789012345678901
2
1
personal life.
12345678901234567890123456789012123456789012345678901 2
2
12345678901234567890123456789012123456789012345678901
subterranean (adjective) Under the surface of the earth. 2
12345678901234567890123456789012123456789012345678901
2
12345678901234567890123456789012123456789012345678901
Subterranean testing of nuclear weapons was permitted 2
12345678901234567890123456789012123456789012345678901
2
12345678901234567890123456789012123456789012345678901
under the Nuclear Test Ban Treaty of 1963.
2
12345678901234567890123456789012123456789012345678901
2345678901234567890123456789012123456789012345678901
12345678901234567890123456789012123456789012345678901 22
12345678901234567890123456789012123456789012345678901
summarily (adverb) Quickly and concisely. No sooner had I 2
1
12345678901234567890123456789012123456789012345678901
voiced my concerns about the new ad campaign than my 2
2
12345678901234567890123456789012123456789012345678901
2345678901234567890123456789012123456789012345678901
12345678901234567890123456789012123456789012345678901
boss put her hand on my elbow and summarily ushered 2
2
12345678901234567890123456789012123456789012345678901
2
12345678901234567890123456789012123456789012345678901
me
out
of
her
office.
12345678901234567890123456789012123456789012345678901 22
1
12345678901234567890123456789012123456789012345678901
superficial (adjective) On the surface only; without depth or 2
2
12345678901234567890123456789012123456789012345678901
12345678901234567890123456789012123456789012345678901
substance. Her wound was only superficial and required 2
2
12345678901234567890123456789012123456789012345678901
12345678901234567890123456789012123456789012345678901
no treatment except a light bandage. His superficial 2
2345678901234567890123456789012123456789012345678901
2
12345678901234567890123456789012123456789012345678901
attractiveness hides the fact that his personality is lifeless 2
12345678901234567890123456789012123456789012345678901
2
12345678901234567890123456789012123456789012345678901
and his mind is dull. superficiality (noun).
2
12345678901234567890123456789012123456789012345678901
2
1
2345678901234567890123456789012123456789012345678901
12345678901234567890123456789012123456789012345678901 22
1
2
12345678901234567890123456789012123456789012345678901
12345678901234567890123456789012123456789012345678901 2
2
12345678901234567890123456789012123456789012345678901
12345678901234567890123456789012123456789012345678901 2
12345678901234567890123456789012123456789012345678901 2
12345678901234567890123456789012123456789012345678901 2
12345678901234567890123456789012123456789012345678901 2 219
123456789012345678901234567890121234567890123456789012

http://www.xtremepapers.net

Part III: Section Review

123456789012345678901234567890121234567890123456789012
12345678901234567890123456789012123456789012345678901 2
2
12345678901234567890123456789012123456789012345678901
superimpose (verb) To place or lay over or above some- 2
12345678901234567890123456789012123456789012345678901
2
12345678901234567890123456789012123456789012345678901
thing. The artist stirred controversy by superimposing 2
12345678901234567890123456789012123456789012345678901
2
12345678901234567890123456789012123456789012345678901
portraits of certain contemporary politicians over im- 2
12345678901234567890123456789012123456789012345678901
2345678901234567890123456789012123456789012345678901
2
1
ages of such reviled historical figures as Hitler and 2
12345678901234567890123456789012123456789012345678901
2
12345678901234567890123456789012123456789012345678901
Stalin.
2
12345678901234567890123456789012123456789012345678901
2
12345678901234567890123456789012123456789012345678901
2345678901234567890123456789012123456789012345678901
1
supersede (verb) To displace, to substitute or supplant. Im 2
2
12345678901234567890123456789012123456789012345678901
2
12345678901234567890123456789012123456789012345678901
sorry,
the
principal
announced,
but
todays
afternoon
2
12345678901234567890123456789012123456789012345678901
2
12345678901234567890123456789012123456789012345678901
classes
will
be
superseded
by
an
assembly
on
drug
and
2345678901234567890123456789012123456789012345678901
2
1
2
12345678901234567890123456789012123456789012345678901
alcohol
abuse.
2
12345678901234567890123456789012123456789012345678901
2
12345678901234567890123456789012123456789012345678901
12345678901234567890123456789012123456789012345678901
supine (adjective) Prone. One always feels rather vulnerable 2
2345678901234567890123456789012123456789012345678901
2
1
12345678901234567890123456789012123456789012345678901
when wearing a flimsy paper gown and lying supine on a 2
2
12345678901234567890123456789012123456789012345678901
2
doctors examining table.
12345678901234567890123456789012123456789012345678901
2
12345678901234567890123456789012123456789012345678901
2345678901234567890123456789012123456789012345678901
12345678901234567890123456789012123456789012345678901
supposition (noun) Assumption, conjecture. While most 2
2
12345678901234567890123456789012123456789012345678901
2
12345678901234567890123456789012123456789012345678901
climate
researchers
believe
that
increasing
levels
of
12345678901234567890123456789012123456789012345678901 2
12345678901234567890123456789012123456789012345678901
greenhouse gases will warm the planet, skeptics claim 2
2
12345678901234567890123456789012123456789012345678901
2
1
that
this
theory
is
mere
supposition.
suppose
(verb).
2
12345678901234567890123456789012123456789012345678901
2
12345678901234567890123456789012123456789012345678901
surge (noun) A gush; a swelling or sweeping forward. When 2
12345678901234567890123456789012123456789012345678901
2345678901234567890123456789012123456789012345678901
2
12345678901234567890123456789012123456789012345678901
Mattel gave the Barbie Doll a makeover in the late 2
12345678901234567890123456789012123456789012345678901
2
12345678901234567890123456789012123456789012345678901
eighties, coming out with dolls like doctor Barbie and 2
12345678901234567890123456789012123456789012345678901
2
12345678901234567890123456789012123456789012345678901
astronaut Barbie, the company experienced a surge in 2
12345678901234567890123456789012123456789012345678901
2
12345678901234567890123456789012123456789012345678901
sales.
2
12345678901234567890123456789012123456789012345678901
12345678901234567890123456789012123456789012345678901 2
2
1
2
12345678901234567890123456789012123456789012345678901
2345678901234567890123456789012123456789012345678901
2
1
Latin tangere 5 to touch. Also found in English contact,
Word
Origin contiguous, tactile, tangent, and tangible.
2
12345678901234567890123456789012123456789012345678901
12345678901234567890123456789012123456789012345678901 2
2
12345678901234567890123456789012123456789012345678901
12345678901234567890123456789012123456789012345678901
tangential (adjective) Touching lightly; only slightly con- 2
2
12345678901234567890123456789012123456789012345678901
2
12345678901234567890123456789012123456789012345678901
nected
or
related.
Having
enrolled
in
a
class
on
2
12345678901234567890123456789012123456789012345678901
12345678901234567890123456789012123456789012345678901
African-American history, the students found the teach- 2
2345678901234567890123456789012123456789012345678901
12345678901234567890123456789012123456789012345678901 2
12345678901234567890123456789012123456789012345678901
ers stories about his travels in South America only of 2
2
1
2
12345678901234567890123456789012123456789012345678901
tangential interest. tangent (noun).
12345678901234567890123456789012123456789012345678901 2
2345678901234567890123456789012123456789012345678901
2
12345678901234567890123456789012123456789012345678901
tedium (noun) Boredom. For most people, watching even a 2
12345678901234567890123456789012123456789012345678901
2
12345678901234567890123456789012123456789012345678901
fifteen-minute broadcast of Earth as seen from space 2
12345678901234567890123456789012123456789012345678901
1
would be an exercise in sheer tedium. tedious (adjective). 2
2
12345678901234567890123456789012123456789012345678901
2
12345678901234567890123456789012123456789012345678901
2
12345678901234567890123456789012123456789012345678901
temperance
(noun)
Moderation
or
restraint
in
feelings
and
2
12345678901234567890123456789012123456789012345678901
2
12345678901234567890123456789012123456789012345678901
behavior.
Most
professional
athletes
practice
temper2345678901234567890123456789012123456789012345678901
12345678901234567890123456789012123456789012345678901 2
12345678901234567890123456789012123456789012345678901
ance in their personal habits; too much eating or 2
2
12345678901234567890123456789012123456789012345678901
2
12345678901234567890123456789012123456789012345678901
drinking
and
too
many
late
nights,
they
know,
can
harm
2
1
2345678901234567890123456789012123456789012345678901
2
12345678901234567890123456789012123456789012345678901
their performance.
2
1
2
12345678901234567890123456789012123456789012345678901
12345678901234567890123456789012123456789012345678901
temperate (adjective) Moderate, calm. The warm gulf 2
2
12345678901234567890123456789012123456789012345678901
streams are largely responsible for the temperate climate 2
12345678901234567890123456789012123456789012345678901
2
12345678901234567890123456789012123456789012345678901
of the British Isles.
2
12345678901234567890123456789012123456789012345678901
2
220 12345678901234567890123456789012123456789012345678901
123456789012345678901234567890121234567890123456789012

www.petersons.com

http://www.xtremepapers.net

Chapter 5: Verbal Review

Take It to the Next Level

123456789012345678901234567890121234567890123456789012
12345678901234567890123456789012123456789012345678901 2
2
12345678901234567890123456789012123456789012345678901
tenuous (adjective) Lacking in substance; weak, flimsy, very 2
12345678901234567890123456789012123456789012345678901
2
12345678901234567890123456789012123456789012345678901
thin. His tenuous grasp of the Spanish language was 2
12345678901234567890123456789012123456789012345678901
2
12345678901234567890123456789012123456789012345678901
evident when he addressed Seor Chavez as Seora.
2
12345678901234567890123456789012123456789012345678901
2345678901234567890123456789012123456789012345678901
2
1
2
12345678901234567890123456789012123456789012345678901
terrestrial
(adjective)
Of
the
earth.
The
movie
Close
Encoun2
12345678901234567890123456789012123456789012345678901
2
12345678901234567890123456789012123456789012345678901
ters
of
the
Third
Kind
tells
the
story
of
the
first
contact
2
12345678901234567890123456789012123456789012345678901
2345678901234567890123456789012123456789012345678901
1
between beings from outer space and terrestrial crea- 2
2
12345678901234567890123456789012123456789012345678901
2
12345678901234567890123456789012123456789012345678901
tures.
2
12345678901234567890123456789012123456789012345678901
2
12345678901234567890123456789012123456789012345678901
throwback
(noun)
A
reversion
to
an
earlier
type;
an
2345678901234567890123456789012123456789012345678901
2
1
2
12345678901234567890123456789012123456789012345678901
atavism. The new Volkswagen Beetle, with its familiar 2
12345678901234567890123456789012123456789012345678901
12345678901234567890123456789012123456789012345678901
bubble shape, looks like a throwback to the sixties, but 2
2
12345678901234567890123456789012123456789012345678901
2345678901234567890123456789012123456789012345678901
1
it is actually packed with modern high-tech equipment. 2
2
12345678901234567890123456789012123456789012345678901
2
12345678901234567890123456789012123456789012345678901
12345678901234567890123456789012123456789012345678901
tiff (noun) A small, almost inconsequential quarrel or 2
2
12345678901234567890123456789012123456789012345678901
12345678901234567890123456789012123456789012345678901
disagreement. Megan and Bruce got into a tiff when 2
2
12345678901234567890123456789012123456789012345678901
2
12345678901234567890123456789012123456789012345678901
Bruce criticized her smoking.
2
12345678901234567890123456789012123456789012345678901
2
12345678901234567890123456789012123456789012345678901
tirade
(noun)
A
long,
harshly
critical
speech.
Reformed
2345678901234567890123456789012123456789012345678901
12345678901234567890123456789012123456789012345678901 22
1
smokers, like Bruce, are prone to delivering tirades on 2
12345678901234567890123456789012123456789012345678901
2
12345678901234567890123456789012123456789012345678901
the evils of smoking.
2
12345678901234567890123456789012123456789012345678901
2345678901234567890123456789012123456789012345678901
12345678901234567890123456789012123456789012345678901 22
12345678901234567890123456789012123456789012345678901
torpor (noun) Apathy, sluggishness. Stranded in an airless 2
12345678901234567890123456789012123456789012345678901
12345678901234567890123456789012123456789012345678901
hotel room in Madras after a 27-hour train ride, I felt 2
2
12345678901234567890123456789012123456789012345678901
12345678901234567890123456789012123456789012345678901
such overwhelming torpor that I doubted I would make 2
2
12345678901234567890123456789012123456789012345678901
2
12345678901234567890123456789012123456789012345678901
it
to
Bangalore,
the
next
leg
of
my
journey.
torpid
12345678901234567890123456789012123456789012345678901 22
1
(adjective).
2
12345678901234567890123456789012123456789012345678901
2
12345678901234567890123456789012123456789012345678901
tout (verb) To praise highly, to brag publicly. A much 2
12345678901234567890123456789012123456789012345678901
2
12345678901234567890123456789012123456789012345678901
happier Eileen is now touting the benefits of Prozac, but, 2
12345678901234567890123456789012123456789012345678901
2345678901234567890123456789012123456789012345678901
2
12345678901234567890123456789012123456789012345678901
to tell you the truth, I miss her witty, self-lacerating 2
12345678901234567890123456789012123456789012345678901
2
1
commentaries.
2
12345678901234567890123456789012123456789012345678901
2
12345678901234567890123456789012123456789012345678901
2345678901234567890123456789012123456789012345678901
12345678901234567890123456789012123456789012345678901 22
12345678901234567890123456789012123456789012345678901 2
1
Latin tractare 5 to handle. Also found in English intractable,
2
12345678901234567890123456789012123456789012345678901
Word
Origin tractate, and traction.
12345678901234567890123456789012123456789012345678901 2
2345678901234567890123456789012123456789012345678901
12345678901234567890123456789012123456789012345678901 22
12345678901234567890123456789012123456789012345678901
tractable (adjective) Obedient, manageable. When he turned 2
12345678901234567890123456789012123456789012345678901
12345678901234567890123456789012123456789012345678901
3, Harrison suddenly became a tractable, well-mannered 2
2
1
2345678901234567890123456789012123456789012345678901
12345678901234567890123456789012123456789012345678901
little boy after being, quite frankly, an unruly little 2
2
12345678901234567890123456789012123456789012345678901
2
12345678901234567890123456789012123456789012345678901
monster!
12345678901234567890123456789012123456789012345678901 22
1
tranquillity (noun) Freedom from disturbance or turmoil; calm. 2
12345678901234567890123456789012123456789012345678901
2
12345678901234567890123456789012123456789012345678901
Seeking the tranquillity of country life, she moved from 2
12345678901234567890123456789012123456789012345678901
2
12345678901234567890123456789012123456789012345678901
New York City to rural Vermont. tranquil (adjective).
2
12345678901234567890123456789012123456789012345678901
2345678901234567890123456789012123456789012345678901
12345678901234567890123456789012123456789012345678901 22
1
2
12345678901234567890123456789012123456789012345678901
12345678901234567890123456789012123456789012345678901 2
2
12345678901234567890123456789012123456789012345678901
12345678901234567890123456789012123456789012345678901 2
12345678901234567890123456789012123456789012345678901 2
12345678901234567890123456789012123456789012345678901 2
12345678901234567890123456789012123456789012345678901 2 221
123456789012345678901234567890121234567890123456789012

http://www.xtremepapers.net

Part III: Section Review

123456789012345678901234567890121234567890123456789012
12345678901234567890123456789012123456789012345678901 2
2
12345678901234567890123456789012123456789012345678901
transgress (verb) To go past limits; to violate. If Iraq has 2
12345678901234567890123456789012123456789012345678901
2
12345678901234567890123456789012123456789012345678901
developed biological weapons, then it has transgressed 2
12345678901234567890123456789012123456789012345678901
2
12345678901234567890123456789012123456789012345678901
the UNs rules against manufacturing weapons of mass 2
12345678901234567890123456789012123456789012345678901
2345678901234567890123456789012123456789012345678901
2
1
destruction. transgression (noun).
2
12345678901234567890123456789012123456789012345678901
2
12345678901234567890123456789012123456789012345678901
2
12345678901234567890123456789012123456789012345678901
transmute
(verb)
To
change
in
form
or
substance.
Practitio2
12345678901234567890123456789012123456789012345678901
2345678901234567890123456789012123456789012345678901
1
ners of alchemy, a forebear of modern chemistry, tried to 2
2
12345678901234567890123456789012123456789012345678901
2
12345678901234567890123456789012123456789012345678901
discover
ways
to
transmute
metals
such
as
iron
into
2
12345678901234567890123456789012123456789012345678901
2
12345678901234567890123456789012123456789012345678901
gold.
transmutation
(noun).
2345678901234567890123456789012123456789012345678901
2
1
2
12345678901234567890123456789012123456789012345678901
12345678901234567890123456789012123456789012345678901
treacherous (adjective) Untrustworthy or disloyal; danger- 2
2
12345678901234567890123456789012123456789012345678901
12345678901234567890123456789012123456789012345678901
ous or unreliable. Nazi Germany proved to be a 2
2345678901234567890123456789012123456789012345678901
2
1
12345678901234567890123456789012123456789012345678901
treacherous ally, first signing a peace pact with the 2
2
12345678901234567890123456789012123456789012345678901
Soviet Union, then invading. Be careful crossing the rope 2
12345678901234567890123456789012123456789012345678901
2
12345678901234567890123456789012123456789012345678901
bridge; parts of the span are badly frayed and treacher- 2
12345678901234567890123456789012123456789012345678901
2
12345678901234567890123456789012123456789012345678901
ous. treachery (noun).
2
12345678901234567890123456789012123456789012345678901
2
12345678901234567890123456789012123456789012345678901
2
12345678901234567890123456789012123456789012345678901
tremor
(noun)
An
involuntary
shaking
or
trembling.
Michael
2345678901234567890123456789012123456789012345678901
12345678901234567890123456789012123456789012345678901 2
1
still manages to appear calm and at ease despite the 2
2
12345678901234567890123456789012123456789012345678901
2
12345678901234567890123456789012123456789012345678901
tremors
caused
by
Parkinsons
disease.
Brooke
felt
the
2
12345678901234567890123456789012123456789012345678901
2345678901234567890123456789012123456789012345678901
12345678901234567890123456789012123456789012345678901
first tremors of the 1989 San Francisco earthquake while 2
2
12345678901234567890123456789012123456789012345678901
2
12345678901234567890123456789012123456789012345678901
she
was
sitting
in
Candlestick
Park
watching
a
Giants
12345678901234567890123456789012123456789012345678901 2
2
12345678901234567890123456789012123456789012345678901
baseball game.
2
12345678901234567890123456789012123456789012345678901
12345678901234567890123456789012123456789012345678901 2
12345678901234567890123456789012123456789012345678901
trenchant (adjective) Caustic and incisive. Essayist H. L. 2
2
12345678901234567890123456789012123456789012345678901
1
Mencken was known for his trenchant wit and was 2
2345678901234567890123456789012123456789012345678901
12345678901234567890123456789012123456789012345678901 2
famed for mercilessly puncturing the American middle 2
12345678901234567890123456789012123456789012345678901
2
1
class (which he called the booboisie).
2
12345678901234567890123456789012123456789012345678901
2
12345678901234567890123456789012123456789012345678901
2345678901234567890123456789012123456789012345678901
12345678901234567890123456789012123456789012345678901 2
12345678901234567890123456789012123456789012345678901 2
2
1 Origin Latin trepidus 5 alarmed. Also found in English intrepid.
Word
2
12345678901234567890123456789012123456789012345678901
2
12345678901234567890123456789012123456789012345678901
2345678901234567890123456789012123456789012345678901
12345678901234567890123456789012123456789012345678901
trepidation (noun) Fear and anxiety. After the tragedy of 2
2
12345678901234567890123456789012123456789012345678901
1
TWA flight 800, many previously fearless flyers were 2
12345678901234567890123456789012123456789012345678901 2
12345678901234567890123456789012123456789012345678901
filled with trepidation whenever they stepped into an 2
2
12345678901234567890123456789012123456789012345678901
2
12345678901234567890123456789012123456789012345678901
airplane.
2
12345678901234567890123456789012123456789012345678901
2
12345678901234567890123456789012123456789012345678901
turbulent
(adjective)
Agitated
or
disturbed.
The
night
be12345678901234567890123456789012123456789012345678901 2
2345678901234567890123456789012123456789012345678901
2
12345678901234567890123456789012123456789012345678901
fore the championship match, Martina was unable to 2
12345678901234567890123456789012123456789012345678901
12345678901234567890123456789012123456789012345678901
sleep, her mind turbulent with fears and hopes. turbu- 2
2
12345678901234567890123456789012123456789012345678901
2
1
lence
(noun).
2345678901234567890123456789012123456789012345678901
12345678901234567890123456789012123456789012345678901 2
12345678901234567890123456789012123456789012345678901 2
12345678901234567890123456789012123456789012345678901
turpitude (noun) Depravity, wickedness. Radical feminists 2
2
12345678901234567890123456789012123456789012345678901
2
1
who
contrast
womens
essential
goodness
with
mens
2345678901234567890123456789012123456789012345678901
12345678901234567890123456789012123456789012345678901 2
1
moral turpitude can be likened to religious fundamental- 2
2
12345678901234567890123456789012123456789012345678901
2
12345678901234567890123456789012123456789012345678901
ists
who
make
a
clear
distinction
between
the
saved
and
2
12345678901234567890123456789012123456789012345678901
2345678901234567890123456789012123456789012345678901
2
1
the damned.
2
12345678901234567890123456789012123456789012345678901
12345678901234567890123456789012123456789012345678901 2
2
222 12345678901234567890123456789012123456789012345678901
123456789012345678901234567890121234567890123456789012

www.petersons.com

http://www.xtremepapers.net

Chapter 5: Verbal Review

Take It to the Next Level

123456789012345678901234567890121234567890123456789012
12345678901234567890123456789012123456789012345678901 2
2
12345678901234567890123456789012123456789012345678901
tyro (noun) Novice, amateur. For an absolute tyro on the 2
12345678901234567890123456789012123456789012345678901
2
12345678901234567890123456789012123456789012345678901
ski slopes, Gina was surprisingly agile at taking the 2
12345678901234567890123456789012123456789012345678901
2
12345678901234567890123456789012123456789012345678901
moguls.
2
12345678901234567890123456789012123456789012345678901
2345678901234567890123456789012123456789012345678901
2
1
2
12345678901234567890123456789012123456789012345678901
unalloyed
(adjective)
Unqualified,
pure.
Holding
his
new2
12345678901234567890123456789012123456789012345678901
2
12345678901234567890123456789012123456789012345678901
born
son
for
the
first
time,
Malik
felt
an
unalloyed
2
12345678901234567890123456789012123456789012345678901
2345678901234567890123456789012123456789012345678901
1
happiness that was unlike anything he had ever experi- 2
2
12345678901234567890123456789012123456789012345678901
2
12345678901234567890123456789012123456789012345678901
enced
in
his
45
years.
2
12345678901234567890123456789012123456789012345678901
2
12345678901234567890123456789012123456789012345678901
2345678901234567890123456789012123456789012345678901
1
undermine (verb) To excavate beneath; to subvert, to 2
2
12345678901234567890123456789012123456789012345678901
12345678901234567890123456789012123456789012345678901
weaken. Dot continued to undermine my efforts to find 2
2
12345678901234567890123456789012123456789012345678901
12345678901234567890123456789012123456789012345678901
her a date by showing up at our dinner parties in her 2
2345678901234567890123456789012123456789012345678901
2
1
2
12345678901234567890123456789012123456789012345678901
ratty old sweat suit.
2
12345678901234567890123456789012123456789012345678901
2
12345678901234567890123456789012123456789012345678901
unfeigned
(adjective)
Genuine,
sincere.
Lashawn
responded
2
12345678901234567890123456789012123456789012345678901
2345678901234567890123456789012123456789012345678901
2
12345678901234567890123456789012123456789012345678901
with such unfeigned astonishment when we all leapt out 2
12345678901234567890123456789012123456789012345678901
12345678901234567890123456789012123456789012345678901
of the kitchen that I think she had had no inkling of the 2
2
12345678901234567890123456789012123456789012345678901
2
12345678901234567890123456789012123456789012345678901
surprise
party.
12345678901234567890123456789012123456789012345678901 22
1
12345678901234567890123456789012123456789012345678901
univocal (adjective) With a single voice. While they came 2
2
12345678901234567890123456789012123456789012345678901
2
12345678901234567890123456789012123456789012345678901
from
different
backgrounds
and
classes,
the
employees
2345678901234567890123456789012123456789012345678901
12345678901234567890123456789012123456789012345678901 22
12345678901234567890123456789012123456789012345678901
were univocal in their demands that the corrupt CEO 2
12345678901234567890123456789012123456789012345678901
2
12345678901234567890123456789012123456789012345678901
resign immediately.
2
12345678901234567890123456789012123456789012345678901
12345678901234567890123456789012123456789012345678901 22
12345678901234567890123456789012123456789012345678901
unstinting (adjective) Giving with unrestrained generosity.
2
12345678901234567890123456789012123456789012345678901
Few people will be able to match the unstinting 2
12345678901234567890123456789012123456789012345678901
2
1
dedication and care that Mother Theresa lavished on the 2
12345678901234567890123456789012123456789012345678901
2
12345678901234567890123456789012123456789012345678901
poor people of Calcutta.
2
12345678901234567890123456789012123456789012345678901
12345678901234567890123456789012123456789012345678901 2
2
12345678901234567890123456789012123456789012345678901
2
12345678901234567890123456789012123456789012345678901
2345678901234567890123456789012123456789012345678901
2
1
Latin urbs 5 city. Also found in English suburb and urban.
Word
Origin
12345678901234567890123456789012123456789012345678901 2
2
12345678901234567890123456789012123456789012345678901
12345678901234567890123456789012123456789012345678901
urbanity (noun) Sophistication, suaveness and polish. Part 2
2345678901234567890123456789012123456789012345678901
12345678901234567890123456789012123456789012345678901 22
12345678901234567890123456789012123456789012345678901
of the fun in a Cary Grant movie lies in seeing whether
2
1
12345678901234567890123456789012123456789012345678901
the star can be made to lose his urbanity and elegance in 2
12345678901234567890123456789012123456789012345678901 2
the midst of chaotic or kooky situations. urbane 2
12345678901234567890123456789012123456789012345678901
2
12345678901234567890123456789012123456789012345678901
(adjective).
2
12345678901234567890123456789012123456789012345678901
2
12345678901234567890123456789012123456789012345678901
2
12345678901234567890123456789012123456789012345678901
usurious
(adjective)
Lending
money
at
an
unconscionably
2345678901234567890123456789012123456789012345678901
12345678901234567890123456789012123456789012345678901 22
12345678901234567890123456789012123456789012345678901
high interest rate. Some people feel that Shakespeares 2
12345678901234567890123456789012123456789012345678901
12345678901234567890123456789012123456789012345678901
portrayal of the Jew, Shylock, the usurious money lender 2
2
1
2345678901234567890123456789012123456789012345678901
12345678901234567890123456789012123456789012345678901
in The Merchant of Venice, has enflamed prejudice 2
2
12345678901234567890123456789012123456789012345678901
2
12345678901234567890123456789012123456789012345678901
against
the
Jews.
usury
(adjective).
12345678901234567890123456789012123456789012345678901 22
1
2
12345678901234567890123456789012123456789012345678901
12345678901234567890123456789012123456789012345678901 2
2
12345678901234567890123456789012123456789012345678901
12345678901234567890123456789012123456789012345678901 2
2
12345678901234567890123456789012123456789012345678901
12345678901234567890123456789012123456789012345678901 2
12345678901234567890123456789012123456789012345678901 2
12345678901234567890123456789012123456789012345678901 2
12345678901234567890123456789012123456789012345678901 2 223
123456789012345678901234567890121234567890123456789012

http://www.xtremepapers.net

Part III: Section Review

123456789012345678901234567890121234567890123456789012
12345678901234567890123456789012123456789012345678901 2
2
12345678901234567890123456789012123456789012345678901
2
12345678901234567890123456789012123456789012345678901
Latin validus 5 strong. Also found in English invalid, invaluable,
2345678901234567890123456789012123456789012345678901
2
1
Word
Origin
2
12345678901234567890123456789012123456789012345678901
prevail, and value.
12345678901234567890123456789012123456789012345678901 2
2
12345678901234567890123456789012123456789012345678901
validate (verb) To officially approve or confirm. The 2
12345678901234567890123456789012123456789012345678901
2
12345678901234567890123456789012123456789012345678901
election of the president is formally validated when the 2
12345678901234567890123456789012123456789012345678901
2
12345678901234567890123456789012123456789012345678901
members of the Electoral College meet to confirm the 2
12345678901234567890123456789012123456789012345678901
2345678901234567890123456789012123456789012345678901
2
1
verdict of the voters. valid (adjective), validity (noun).
2
12345678901234567890123456789012123456789012345678901
2
12345678901234567890123456789012123456789012345678901
2
12345678901234567890123456789012123456789012345678901
vapid
(adjective)
Flat,
flavorless.
Whenever
I
have
insom2
12345678901234567890123456789012123456789012345678901
2345678901234567890123456789012123456789012345678901
1
nia, I just tune the clock radio to Lite FM, and soon 2
2
12345678901234567890123456789012123456789012345678901
2
12345678901234567890123456789012123456789012345678901
those
vapid
songs
from
the
seventies
have
me
floating
2
12345678901234567890123456789012123456789012345678901
2
12345678901234567890123456789012123456789012345678901
away
to
dreamland.
vapidity
(noun).
2345678901234567890123456789012123456789012345678901
2
1
2
12345678901234567890123456789012123456789012345678901
12345678901234567890123456789012123456789012345678901
venal (adjective) Corrupt, mercenary. Sese Seko Mobuto 2
2
12345678901234567890123456789012123456789012345678901
12345678901234567890123456789012123456789012345678901
was the venal dictator of Zaire who reportedly diverted 2
2345678901234567890123456789012123456789012345678901
12345678901234567890123456789012123456789012345678901 2
12345678901234567890123456789012123456789012345678901
millions of dollars in foreign aid to his own personal 2
2
12345678901234567890123456789012123456789012345678901
fortune. venality (noun).
2
12345678901234567890123456789012123456789012345678901
12345678901234567890123456789012123456789012345678901 2
12345678901234567890123456789012123456789012345678901
veneer (noun) A superficial or deceptive covering. Beneath 2
2
1
2
12345678901234567890123456789012123456789012345678901
her
folksy
veneer,
Samantha
is
a
shrewd
and
calculating
2
12345678901234567890123456789012123456789012345678901
2
12345678901234567890123456789012123456789012345678901
businessperson
just
waiting
for
the
right
moment
to
2345678901234567890123456789012123456789012345678901
12345678901234567890123456789012123456789012345678901 2
2
12345678901234567890123456789012123456789012345678901
pounce.
2
12345678901234567890123456789012123456789012345678901
12345678901234567890123456789012123456789012345678901 2
12345678901234567890123456789012123456789012345678901
venerate (verb) To admire or honor. In Communist China, 2
2
12345678901234567890123456789012123456789012345678901
2
12345678901234567890123456789012123456789012345678901
Mao
Tse-Tung
is
venerated
as
an
almost
godlike
figure.
12345678901234567890123456789012123456789012345678901 2
2
12345678901234567890123456789012123456789012345678901
venerable (adjective), veneration (noun).
2
1
2345678901234567890123456789012123456789012345678901
12345678901234567890123456789012123456789012345678901 2
veracious (adjective) Truthful, earnest. Many people still 2
12345678901234567890123456789012123456789012345678901
2
1
feel that Anita Hill was entirely veracious in her 2
12345678901234567890123456789012123456789012345678901
2
12345678901234567890123456789012123456789012345678901
allegations of sexual harassment during the Clarence 2
12345678901234567890123456789012123456789012345678901
2
12345678901234567890123456789012123456789012345678901
Thomas confirmation hearings. veracity (noun).
2
12345678901234567890123456789012123456789012345678901
2
12345678901234567890123456789012123456789012345678901
2
12345678901234567890123456789012123456789012345678901
verify
(verb)
To
prove
to
be
true.
The
contents
of
Robert
L.
2345678901234567890123456789012123456789012345678901
12345678901234567890123456789012123456789012345678901 2
12345678901234567890123456789012123456789012345678901
Ripleys syndicated Believe it or Not cartoons could 2
2
1
2
12345678901234567890123456789012123456789012345678901
not
be
verified,
yet
the
public
still
thrilled
to
reports
of
12345678901234567890123456789012123456789012345678901 2
2345678901234567890123456789012123456789012345678901
12345678901234567890123456789012123456789012345678901
the man with two pupils in each eye, the human 2
2
12345678901234567890123456789012123456789012345678901
2
12345678901234567890123456789012123456789012345678901
unicorn,
and
other
amazing
oddities.
verification
12345678901234567890123456789012123456789012345678901 2
2
1
(noun).
2
12345678901234567890123456789012123456789012345678901
2
12345678901234567890123456789012123456789012345678901
2
12345678901234567890123456789012123456789012345678901
2
12345678901234567890123456789012123456789012345678901
Latin
verus
5
true.
Also
found
in
English
verisimilitude,
veritable,
2
12345678901234567890123456789012123456789012345678901
Word
Origin
2345678901234567890123456789012123456789012345678901
2
12345678901234567890123456789012123456789012345678901
and verity.
2
12345678901234567890123456789012123456789012345678901
2
12345678901234567890123456789012123456789012345678901
veritable (adjective) Authentic. A French antiques dealer 2
12345678901234567890123456789012123456789012345678901
2
1
recently claimed that a fifteenth-century child-sized suit 2
12345678901234567890123456789012123456789012345678901
2
12345678901234567890123456789012123456789012345678901
of armor that he purchased in 1994 is the veritable suit 2
12345678901234567890123456789012123456789012345678901
2
12345678901234567890123456789012123456789012345678901
of armor worn by heroine Joan of Arc.
2
12345678901234567890123456789012123456789012345678901
2345678901234567890123456789012123456789012345678901
2
1
12345678901234567890123456789012123456789012345678901 2
12345678901234567890123456789012123456789012345678901 2
2
224 12345678901234567890123456789012123456789012345678901
123456789012345678901234567890121234567890123456789012

www.petersons.com

http://www.xtremepapers.net

Chapter 5: Verbal Review

Take It to the Next Level

123456789012345678901234567890121234567890123456789012
12345678901234567890123456789012123456789012345678901 2
2
12345678901234567890123456789012123456789012345678901
vindictive (adjective) Spiteful. Paula embarked on a string 2
12345678901234567890123456789012123456789012345678901
2
12345678901234567890123456789012123456789012345678901
of petty, vindictive acts against her philandering boy- 2
12345678901234567890123456789012123456789012345678901
2
12345678901234567890123456789012123456789012345678901
friend, such as mixing dry cat food with his cereal and 2
12345678901234567890123456789012123456789012345678901
2345678901234567890123456789012123456789012345678901
2
1
snipping the blooms off his prize African violets.
2
12345678901234567890123456789012123456789012345678901
2
12345678901234567890123456789012123456789012345678901
2
12345678901234567890123456789012123456789012345678901
viscid
(adjective)
Sticky.
The
3M
companys
Post-It,
a
2
12345678901234567890123456789012123456789012345678901
2345678901234567890123456789012123456789012345678901
1
simple piece of paper with one viscid side, has become as 2
2
12345678901234567890123456789012123456789012345678901
2
12345678901234567890123456789012123456789012345678901
commonplaceand
as
indispensableas
the
paper
clip.
2
12345678901234567890123456789012123456789012345678901
2
12345678901234567890123456789012123456789012345678901
2345678901234567890123456789012123456789012345678901
1
viscous (adjective) Having a gelatinous or gooey quality. I 2
2
12345678901234567890123456789012123456789012345678901
12345678901234567890123456789012123456789012345678901
put too much liquid in the batter, and so my Black Forest 2
2
12345678901234567890123456789012123456789012345678901
2
12345678901234567890123456789012123456789012345678901
cake turned out to be a viscous, inedible mass.
2345678901234567890123456789012123456789012345678901
2
1
2
12345678901234567890123456789012123456789012345678901
vitiate (verb) To pollute, to impair. When they voted to ban 2
12345678901234567890123456789012123456789012345678901
2
12345678901234567890123456789012123456789012345678901
smoking from all bars in California, the public affirmed 2
12345678901234567890123456789012123456789012345678901
2345678901234567890123456789012123456789012345678901
2
12345678901234567890123456789012123456789012345678901
their belief that smoking vitiates the health of all people, 2
12345678901234567890123456789012123456789012345678901
2
12345678901234567890123456789012123456789012345678901
not just smokers.
2
12345678901234567890123456789012123456789012345678901
12345678901234567890123456789012123456789012345678901 22
12345678901234567890123456789012123456789012345678901
vituperative (adjective) Verbally abusive, insulting. Eliza- 2
1
12345678901234567890123456789012123456789012345678901
beth Taylor won an award for her harrowing portrayal 2
2
12345678901234567890123456789012123456789012345678901
2
12345678901234567890123456789012123456789012345678901
of
Martha,
the
bitter,
vituperative
wife
of
a
college
2345678901234567890123456789012123456789012345678901
12345678901234567890123456789012123456789012345678901 22
12345678901234567890123456789012123456789012345678901
professor in Edward Albees Whos Afraid of Virginia 2
12345678901234567890123456789012123456789012345678901
2
12345678901234567890123456789012123456789012345678901
Woolf? vituperate (verb).
2
12345678901234567890123456789012123456789012345678901
12345678901234567890123456789012123456789012345678901 22
12345678901234567890123456789012123456789012345678901
volatile (adjective) Quickly changing; fleeting, transitory;
2
12345678901234567890123456789012123456789012345678901
prone to violence. Public opinion is notoriously volatile; 2
12345678901234567890123456789012123456789012345678901
2
1
a politician who is very popular one month may be 2
12345678901234567890123456789012123456789012345678901
2
12345678901234567890123456789012123456789012345678901
voted out of office the next. volatility (noun).
2
12345678901234567890123456789012123456789012345678901
12345678901234567890123456789012123456789012345678901 2
12345678901234567890123456789012123456789012345678901
volubility (noun) Quality of being overly talkative, glib. As 2
2
12345678901234567890123456789012123456789012345678901
2
12345678901234567890123456789012123456789012345678901
Lorraines
anxiety
increased,
her
volubility
increased
in
12345678901234567890123456789012123456789012345678901 2
12345678901234567890123456789012123456789012345678901
direct proportion, so during her job interview the poor 2
2
12345678901234567890123456789012123456789012345678901
2345678901234567890123456789012123456789012345678901
12345678901234567890123456789012123456789012345678901
interviewer couldnt get a word in edgewise. voluble 2
2
12345678901234567890123456789012123456789012345678901
2
1
(adjective).
12345678901234567890123456789012123456789012345678901 2
12345678901234567890123456789012123456789012345678901 2
2
12345678901234567890123456789012123456789012345678901
2
12345678901234567890123456789012123456789012345678901
Latin vorare 5 to eat. Also found in English carnivorous, devour,
2
12345678901234567890123456789012123456789012345678901
Word
Origin
2
12345678901234567890123456789012123456789012345678901
and omnivorous.
12345678901234567890123456789012123456789012345678901 2
2345678901234567890123456789012123456789012345678901
2
12345678901234567890123456789012123456789012345678901
voracious (adjective) Gluttonous, ravenous. Are all your 2
12345678901234567890123456789012123456789012345678901
2
12345678901234567890123456789012123456789012345678901
appetites so voracious? Wesley asked Nina as he 2
12345678901234567890123456789012123456789012345678901
2
1
watched her finish off seven miniature sandwiches and 2
12345678901234567890123456789012123456789012345678901
2
12345678901234567890123456789012123456789012345678901
two lamb kabob skewers in a matter of minutes. 2
12345678901234567890123456789012123456789012345678901
2
12345678901234567890123456789012123456789012345678901
voracity (noun).
2
12345678901234567890123456789012123456789012345678901
2345678901234567890123456789012123456789012345678901
12345678901234567890123456789012123456789012345678901 22
1
wag (noun) Wit, joker. Tom was getting tired of his role as 2
12345678901234567890123456789012123456789012345678901
12345678901234567890123456789012123456789012345678901
the comical wag who injected life into Kathys otherwise 2
2
12345678901234567890123456789012123456789012345678901
2345678901234567890123456789012123456789012345678901
2
1
tedious parties. waggish (adjective).
2
12345678901234567890123456789012123456789012345678901
12345678901234567890123456789012123456789012345678901 22
12345678901234567890123456789012123456789012345678901 225
123456789012345678901234567890121234567890123456789012

http://www.xtremepapers.net

Part III: Section Review

123456789012345678901234567890121234567890123456789012
12345678901234567890123456789012123456789012345678901 2
2
12345678901234567890123456789012123456789012345678901
whimsical (adjective) Based on a capricious, carefree, or 2
12345678901234567890123456789012123456789012345678901
2
12345678901234567890123456789012123456789012345678901
sudden impulse or idea; fanciful, playful. Dave Barrys 2
12345678901234567890123456789012123456789012345678901
2
12345678901234567890123456789012123456789012345678901
Book of Bad Songs is filled with the kind of goofy jokes 2
12345678901234567890123456789012123456789012345678901
2345678901234567890123456789012123456789012345678901
2
1
that are typical of his whimsical sense of humor. whim 2
12345678901234567890123456789012123456789012345678901
2
12345678901234567890123456789012123456789012345678901
(noun).
2
12345678901234567890123456789012123456789012345678901
2
12345678901234567890123456789012123456789012345678901
2345678901234567890123456789012123456789012345678901
1
xenophobia (noun) Fear of foreigners or outsiders. Slobodan 2
2
12345678901234567890123456789012123456789012345678901
2
12345678901234567890123456789012123456789012345678901
Milosevics
nationalistic
talk
played
on
the
deep
xeno2
12345678901234567890123456789012123456789012345678901
2
12345678901234567890123456789012123456789012345678901
phobia
of
the
Serbs,
who
after
500
years
of
brutal
2345678901234567890123456789012123456789012345678901
2
1
2
12345678901234567890123456789012123456789012345678901
Ottoman
occupation
had
come
to
distrust
all
outsiders.
2
12345678901234567890123456789012123456789012345678901
2
12345678901234567890123456789012123456789012345678901
12345678901234567890123456789012123456789012345678901
zenith (noun) Highest point. Compiling the vocabulary list 2
2345678901234567890123456789012123456789012345678901
2
1
12345678901234567890123456789012123456789012345678901
for the Insiders Guide to the GRE was the zenith of my 2
2
12345678901234567890123456789012123456789012345678901
literary career: after this, there was nowhere to go but 2
12345678901234567890123456789012123456789012345678901
2
12345678901234567890123456789012123456789012345678901
downhill.
2
12345678901234567890123456789012123456789012345678901
2
12345678901234567890123456789012123456789012345678901
2
12345678901234567890123456789012123456789012345678901
2
12345678901234567890123456789012123456789012345678901
2
12345678901234567890123456789012123456789012345678901
2345678901234567890123456789012123456789012345678901
12345678901234567890123456789012123456789012345678901 2
2
1
2
12345678901234567890123456789012123456789012345678901
2
12345678901234567890123456789012123456789012345678901
2
12345678901234567890123456789012123456789012345678901
2345678901234567890123456789012123456789012345678901
12345678901234567890123456789012123456789012345678901 2
12345678901234567890123456789012123456789012345678901 2
12345678901234567890123456789012123456789012345678901 2
12345678901234567890123456789012123456789012345678901 2
12345678901234567890123456789012123456789012345678901 2
12345678901234567890123456789012123456789012345678901 2
12345678901234567890123456789012123456789012345678901 2
12345678901234567890123456789012123456789012345678901 2
12345678901234567890123456789012123456789012345678901 2
2
1
2
12345678901234567890123456789012123456789012345678901
2
12345678901234567890123456789012123456789012345678901
12345678901234567890123456789012123456789012345678901 2
12345678901234567890123456789012123456789012345678901 2
2
12345678901234567890123456789012123456789012345678901
2
12345678901234567890123456789012123456789012345678901
2
12345678901234567890123456789012123456789012345678901
12345678901234567890123456789012123456789012345678901 2
2
12345678901234567890123456789012123456789012345678901
2
12345678901234567890123456789012123456789012345678901
2345678901234567890123456789012123456789012345678901
12345678901234567890123456789012123456789012345678901 2
12345678901234567890123456789012123456789012345678901 2
2
1
12345678901234567890123456789012123456789012345678901 2
12345678901234567890123456789012123456789012345678901 2
2
12345678901234567890123456789012123456789012345678901
2
12345678901234567890123456789012123456789012345678901
2
12345678901234567890123456789012123456789012345678901
2
12345678901234567890123456789012123456789012345678901
12345678901234567890123456789012123456789012345678901 2
2345678901234567890123456789012123456789012345678901
12345678901234567890123456789012123456789012345678901 2
12345678901234567890123456789012123456789012345678901 2
12345678901234567890123456789012123456789012345678901 2
12345678901234567890123456789012123456789012345678901 2
2
1
2
12345678901234567890123456789012123456789012345678901
2
12345678901234567890123456789012123456789012345678901
2
12345678901234567890123456789012123456789012345678901
2
12345678901234567890123456789012123456789012345678901
12345678901234567890123456789012123456789012345678901 2
2345678901234567890123456789012123456789012345678901
12345678901234567890123456789012123456789012345678901 2
2
1
2
12345678901234567890123456789012123456789012345678901
12345678901234567890123456789012123456789012345678901 2
2
12345678901234567890123456789012123456789012345678901
12345678901234567890123456789012123456789012345678901 2
12345678901234567890123456789012123456789012345678901 2
12345678901234567890123456789012123456789012345678901 2
2
226 12345678901234567890123456789012123456789012345678901
123456789012345678901234567890121234567890123456789012

www.petersons.com

http://www.xtremepapers.net

Chapter

6
Analytical Writing Review

Note

123456789012345678901234567890121234567890123456789012
2
12345678901234567890123456789012123456789012345678901
2
12345678901234567890123456789012123456789012345678901
2
12345678901234567890123456789012123456789012345678901
Analytical
101:
Fundamental
Essay
Writing
Skills
2345678901234567890123456789012123456789012345678901
12345678901234567890123456789012123456789012345678901 22
12345678901234567890123456789012123456789012345678901
The two-essay Analytical section is the first section you encounter after 2
12345678901234567890123456789012123456789012345678901
12345678901234567890123456789012123456789012345678901
surviving the interminable form-filling-out pre-GRE shenanigans. It does 2
2
12345678901234567890123456789012123456789012345678901
2
12345678901234567890123456789012123456789012345678901
1 not have much in common with its Math and Verbal siblings. The 2
12345678901234567890123456789012123456789012345678901
Analytical section is scored differently, structured differently, and takes 2
2
12345678901234567890123456789012123456789012345678901
12345678901234567890123456789012123456789012345678901
longer to complete than both other sections combined. This last point can 2
2345678901234567890123456789012123456789012345678901
12345678901234567890123456789012123456789012345678901 22
12345678901234567890123456789012123456789012345678901
be somewhat frustrating if your Analytical score is not relevant to your
2
12345678901234567890123456789012123456789012345678901
12345678901234567890123456789012123456789012345678901
particular graduate program, since you will have to spend the first 75 2
12345678901234567890123456789012123456789012345678901 22
12345678901234567890123456789012123456789012345678901
minutes of the test mucking about and wasting valuable brainpower units
2
12345678901234567890123456789012123456789012345678901
in a section thats not important to you. If this is the case, conserve your 2
12345678901234567890123456789012123456789012345678901
2
12345678901234567890123456789012123456789012345678901
1 energy for the sections that matter, but dont blow off the section either. 2
2345678901234567890123456789012123456789012345678901
12345678901234567890123456789012123456789012345678901 22
12345678901234567890123456789012123456789012345678901 2
1
12345678901234567890123456789012123456789012345678901 2
2
12345678901234567890123456789012123456789012345678901
If you are an international student or if English is your second language,
2
12345678901234567890123456789012123456789012345678901
2
12345678901234567890123456789012123456789012345678901
your Analytical score might be scrutinized by universities to determine
2
12345678901234567890123456789012123456789012345678901
how fluent you are in English.
2
12345678901234567890123456789012123456789012345678901
2
12345678901234567890123456789012123456789012345678901
2345678901234567890123456789012123456789012345678901
12345678901234567890123456789012123456789012345678901 22
12345678901234567890123456789012123456789012345678901 2
1
12345678901234567890123456789012123456789012345678901
Immediately after you complete your GRE CAT, your Quantitative and 2
2
12345678901234567890123456789012123456789012345678901
2345678901234567890123456789012123456789012345678901
12345678901234567890123456789012123456789012345678901
Verbal scores appear on the screen. The Analytical score takes two to six 2
2
12345678901234567890123456789012123456789012345678901
weeks longer to get, since your essays must be sent off to GRE-Land, 2
12345678901234567890123456789012123456789012345678901
2
12345678901234567890123456789012123456789012345678901
1 where they are read and graded by professional readers. These readers 2
2345678901234567890123456789012123456789012345678901
2
12345678901234567890123456789012123456789012345678901
might be English graduate students, former English teachers, or educa- 2
12345678901234567890123456789012123456789012345678901
2
12345678901234567890123456789012123456789012345678901
tional professionals of one ilk or another. Whatever their background, the 2
12345678901234567890123456789012123456789012345678901
2
1 professional readers share some traits in common:
2
12345678901234567890123456789012123456789012345678901
2
12345678901234567890123456789012123456789012345678901
2
12345678901234567890123456789012123456789012345678901
1.
They
are
experts
in
the
essay-writing
format
and
are
well2
12345678901234567890123456789012123456789012345678901
2
12345678901234567890123456789012123456789012345678901
qualified
to
make
judgments
about
the
effectiveness
of
a
2345678901234567890123456789012123456789012345678901
12345678901234567890123456789012123456789012345678901 22
1
particular essay.
2
12345678901234567890123456789012123456789012345678901
12345678901234567890123456789012123456789012345678901 2
12345678901234567890123456789012123456789012345678901
2. They use a shared rubric to evaluate both essays and decide upon 2
2
12345678901234567890123456789012123456789012345678901
2
12345678901234567890123456789012123456789012345678901
a single score for each test-taker.
12345678901234567890123456789012123456789012345678901 22
12345678901234567890123456789012123456789012345678901
123456789012345678901234567890121234567890123456789012
227

http://www.xtremepapers.net

Part III: Section Review

www.petersons.com

http://www.xtremepapers.net

Tip

123456789012345678901234567890121234567890123456789012
12345678901234567890123456789012123456789012345678901 2
2
12345678901234567890123456789012123456789012345678901
3. All the ones who give out the lowest scores are members of Mrs. 2
12345678901234567890123456789012123456789012345678901
2
12345678901234567890123456789012123456789012345678901
Smiths 8th grade class at Lawrence Middle School.
2
12345678901234567890123456789012123456789012345678901
12345678901234567890123456789012123456789012345678901 2
2
12345678901234567890123456789012123456789012345678901
12345678901234567890123456789012123456789012345678901 2
12345678901234567890123456789012123456789012345678901
The ugly-sounding term rubric often rears its head when the topic of essay 2
2
12345678901234567890123456789012123456789012345678901
grading comes up. A rubric is little more than a set of rules, or standards, 2
12345678901234567890123456789012123456789012345678901
2
12345678901234567890123456789012123456789012345678901
regarding a topic like essay-writing. Since essay-grading is inherently 2
12345678901234567890123456789012123456789012345678901
2
12345678901234567890123456789012123456789012345678901
subjective, a rubric helps codify certain general points that every good 2
12345678901234567890123456789012123456789012345678901
2
12345678901234567890123456789012123456789012345678901
essay should have. Standards like Every essay should have a clear, 2
12345678901234567890123456789012123456789012345678901
2345678901234567890123456789012123456789012345678901
1 well-articulated point of view and A well-written essay will contain a 2
2
12345678901234567890123456789012123456789012345678901
2
12345678901234567890123456789012123456789012345678901
series
of
concise
supporting
details
that
back
up
the
argument
are
two
2
12345678901234567890123456789012123456789012345678901
2
12345678901234567890123456789012123456789012345678901
examples
that
usually
appear
on
an
essay-writing
rubric.
Although
a
2345678901234567890123456789012123456789012345678901
2
1
2
12345678901234567890123456789012123456789012345678901
rubric
helps
professional
readers
standardize
the
essay-grading
process,
2
12345678901234567890123456789012123456789012345678901
2
12345678901234567890123456789012123456789012345678901
there
is
still
room
for
interpretation
and
differences
of
opinion.
2
12345678901234567890123456789012123456789012345678901
2345678901234567890123456789012123456789012345678901
12345678901234567890123456789012123456789012345678901 2
12345678901234567890123456789012123456789012345678901 2
12345678901234567890123456789012123456789012345678901
The third point might not be factually accurate (its more of an outright lie), 2
2
12345678901234567890123456789012123456789012345678901
2
12345678901234567890123456789012123456789012345678901
but
the
other
two
points
are
true.
Both
essays
are
read,
evaluated,
and
given
12345678901234567890123456789012123456789012345678901 2
1 a score from 0 to 6 by two different readers. A 6 is the highest score, while a 2
2
12345678901234567890123456789012123456789012345678901
2
12345678901234567890123456789012123456789012345678901
0
means
you
didnt
make
much
sense
or
use
real
words.
You
can
get
half2
12345678901234567890123456789012123456789012345678901
2345678901234567890123456789012123456789012345678901
12345678901234567890123456789012123456789012345678901
point scores if one reader gives you a 4 while another gives you a 5; if thats 2
2
12345678901234567890123456789012123456789012345678901
12345678901234567890123456789012123456789012345678901
the case, you end up with a 4.5 on that test. If two readers disagree by a lot, 2
12345678901234567890123456789012123456789012345678901 2
2
12345678901234567890123456789012123456789012345678901
then a third reader is brought in to help resolve the disagreement.
2
12345678901234567890123456789012123456789012345678901
2
12345678901234567890123456789012123456789012345678901
A 4 or 5 are both good scores, a 3s a bit dodgy, and a 1 or 2 show you have 2
12345678901234567890123456789012123456789012345678901
2
12345678901234567890123456789012123456789012345678901
1 some deficiencies in the essay-writing arena. The goal of this section is to 2
2345678901234567890123456789012123456789012345678901
2
12345678901234567890123456789012123456789012345678901
give you the knowledge needed to score at least a 4 on the essay. A 5 or 6 is 2
12345678901234567890123456789012123456789012345678901
1 also a possibility, but the primary goal is to keep your score above a 3. 2
12345678901234567890123456789012123456789012345678901 2
2
12345678901234567890123456789012123456789012345678901
Level 2 Analytical, which begins on page 245, focuses on scoring a 6.
2
12345678901234567890123456789012123456789012345678901
2
12345678901234567890123456789012123456789012345678901
12345678901234567890123456789012123456789012345678901 2
2
12345678901234567890123456789012123456789012345678901
2
12345678901234567890123456789012123456789012345678901
The
Issue
Essay
2345678901234567890123456789012123456789012345678901
12345678901234567890123456789012123456789012345678901 2
2
12345678901234567890123456789012123456789012345678901
1 For an idea of what the issue will be, look no further than the official GRE 2
12345678901234567890123456789012123456789012345678901
Web site, www.gre.org. Every possible issue topic is listed there (the 2
2
12345678901234567890123456789012123456789012345678901
2345678901234567890123456789012123456789012345678901
12345678901234567890123456789012123456789012345678901
current link is www.gre.org/issuetop.html, but this might change if the site 2
2
12345678901234567890123456789012123456789012345678901
2
12345678901234567890123456789012123456789012345678901
gets
redesigned).
There
are
more
than
200
topics
listed,
so
writing
sample
12345678901234567890123456789012123456789012345678901 2
1 essays for each one would not be the best use of your time. Then again, it 2
2
12345678901234567890123456789012123456789012345678901
2
12345678901234567890123456789012123456789012345678901
wouldnt
hurt
to
practice
writing
a
couple
of
sample
essays
from
this
list
if
2
12345678901234567890123456789012123456789012345678901
2
12345678901234567890123456789012123456789012345678901
you
feel
your
essay-writing
skills
need
work.
12345678901234567890123456789012123456789012345678901 2
2345678901234567890123456789012123456789012345678901
12345678901234567890123456789012123456789012345678901 2
12345678901234567890123456789012123456789012345678901
Looking over the list, you will see that most topics are broad statements 2
2
12345678901234567890123456789012123456789012345678901
12345678901234567890123456789012123456789012345678901
that leave themselves open for interpretation. A statement like, A nation 2
2
1
should be ruled by its telemarketers is the kind of open-ended topic that 2
12345678901234567890123456789012123456789012345678901
2
12345678901234567890123456789012123456789012345678901
just dares you to agree or disagree with it.
2
12345678901234567890123456789012123456789012345678901
12345678901234567890123456789012123456789012345678901 2
2
12345678901234567890123456789012123456789012345678901
12345678901234567890123456789012123456789012345678901 2
12345678901234567890123456789012123456789012345678901 2
12345678901234567890123456789012123456789012345678901 2
2
228 12345678901234567890123456789012123456789012345678901
123456789012345678901234567890121234567890123456789012

Alert!

Chapter 6: Analytical Writing Review

123456789012345678901234567890121234567890123456789012
12345678901234567890123456789012123456789012345678901 2
2
12345678901234567890123456789012123456789012345678901
The first essay begins by presenting you with two issues, not just one. You 2
12345678901234567890123456789012123456789012345678901
2
12345678901234567890123456789012123456789012345678901
get to choose which issue you would like to gab about. You might get two 2
12345678901234567890123456789012123456789012345678901
2
12345678901234567890123456789012123456789012345678901
topics similar to the ones below:
2
12345678901234567890123456789012123456789012345678901
2345678901234567890123456789012123456789012345678901
2
1
2
12345678901234567890123456789012123456789012345678901
1.
The
creation
of
a
theater
company,
however
small,
increases
the
2
12345678901234567890123456789012123456789012345678901
2
12345678901234567890123456789012123456789012345678901
attractiveness
of
a
city
to
visitors.
The
theater
company
demon2
12345678901234567890123456789012123456789012345678901
2345678901234567890123456789012123456789012345678901
1
strates that a city is committed to artistic endeavors, and such a 2
2
12345678901234567890123456789012123456789012345678901
2
12345678901234567890123456789012123456789012345678901
commitment
is
usually
associated
with
good
governance.
2
12345678901234567890123456789012123456789012345678901
2
12345678901234567890123456789012123456789012345678901
2345678901234567890123456789012123456789012345678901
2
1
2. Quick decisions lead to success in todays business world.
2
12345678901234567890123456789012123456789012345678901
2
12345678901234567890123456789012123456789012345678901
2
12345678901234567890123456789012123456789012345678901
2
12345678901234567890123456789012123456789012345678901
Since
the
idea
of
being
sued
is
always
unappealing,
the
two
sample
topics
2345678901234567890123456789012123456789012345678901
2
1
2
12345678901234567890123456789012123456789012345678901
listed
here
are
NOT
from
the
official
list.
Theyre
not
even
close
to
any
of
2
12345678901234567890123456789012123456789012345678901
2
12345678901234567890123456789012123456789012345678901
the topics listed there, thank goodness. Rest assured that the methods and
2
12345678901234567890123456789012123456789012345678901
2345678901234567890123456789012123456789012345678901
2
12345678901234567890123456789012123456789012345678901
strategies discussed throughout the section will work on all the topics
2
12345678901234567890123456789012123456789012345678901
2
12345678901234567890123456789012123456789012345678901
from the official list, even though these two sample topics are
12345678901234567890123456789012123456789012345678901 22
12345678901234567890123456789012123456789012345678901
NOTN.O.T.from the official list.
2
12345678901234567890123456789012123456789012345678901
2
1
2
12345678901234567890123456789012123456789012345678901
Its nice to have a choice of two topics to write about, but dont waste too 2
12345678901234567890123456789012123456789012345678901
2
12345678901234567890123456789012123456789012345678901
much time deciding which one you want to discuss. The clock starts 2
12345678901234567890123456789012123456789012345678901
2
12345678901234567890123456789012123456789012345678901
counting down once the topics appear, so you want to decide as quickly as 2
12345678901234567890123456789012123456789012345678901
2
12345678901234567890123456789012123456789012345678901
possible. Every second you spend thinking about the topic you dont 2
12345678901234567890123456789012123456789012345678901
2345678901234567890123456789012123456789012345678901
2
12345678901234567890123456789012123456789012345678901
choose is time completely wasted. You want to maximize the amount of 2
12345678901234567890123456789012123456789012345678901
12345678901234567890123456789012123456789012345678901
time you have for the topic you do choose, so read the two choices and 2
2
12345678901234567890123456789012123456789012345678901
1 then ask yourself, Which topic can I write about more? Getting words 2
2345678901234567890123456789012123456789012345678901
12345678901234567890123456789012123456789012345678901 22
12345678901234567890123456789012123456789012345678901
1 onto the screen is key, so make a decision about which topic you feel more 2
12345678901234567890123456789012123456789012345678901
loquacious about. If youre the kind of person who reads the Wall Street 2
2
12345678901234567890123456789012123456789012345678901
2345678901234567890123456789012123456789012345678901
12345678901234567890123456789012123456789012345678901
Journal, choose the second topic. If youre someone who flips through the 2
2
12345678901234567890123456789012123456789012345678901
1 Wall Street Journal searching for comics and movie reviews, the first topic 2
2
12345678901234567890123456789012123456789012345678901
2
12345678901234567890123456789012123456789012345678901
is
probably
better
for
you.
2345678901234567890123456789012123456789012345678901
12345678901234567890123456789012123456789012345678901 22
12345678901234567890123456789012123456789012345678901 2
1 Once you pick a topic, you need to decide whether you want to agree or
12345678901234567890123456789012123456789012345678901 2
12345678901234567890123456789012123456789012345678901
disagree with the initial statement. Each topic sentence presents one side of 2
2
12345678901234567890123456789012123456789012345678901
an issue, and its up to you to present your perspective on an issue 2
12345678901234567890123456789012123456789012345678901
2
12345678901234567890123456789012123456789012345678901
(thats the official moniker for this part of the Analytical section). It 2
12345678901234567890123456789012123456789012345678901
2
12345678901234567890123456789012123456789012345678901
doesnt matter whether you choose to agree or disagree, since theres no 2
12345678901234567890123456789012123456789012345678901
2
12345678901234567890123456789012123456789012345678901
single right answer to these topics. All that matters is how well you 2
12345678901234567890123456789012123456789012345678901
2
12345678901234567890123456789012123456789012345678901
explain your side of the argument with supporting details arranged in a 2
12345678901234567890123456789012123456789012345678901
2345678901234567890123456789012123456789012345678901
2
12345678901234567890123456789012123456789012345678901
logical manner.
2
12345678901234567890123456789012123456789012345678901
12345678901234567890123456789012123456789012345678901 22
12345678901234567890123456789012123456789012345678901
1 Lets say you choose the first topic sentence, and feeling cantankerous you 2
12345678901234567890123456789012123456789012345678901
decide to disagree with the opinion that theater companies increase the 2
2
12345678901234567890123456789012123456789012345678901
2
12345678901234567890123456789012123456789012345678901
attractiveness
of
a
city.
With
about
43
minutes
remaining,
take
about
5
12345678901234567890123456789012123456789012345678901 2
12345678901234567890123456789012123456789012345678901
minutes to brainstorm some ideas that will help support this viewpoint. Jot 2
2
12345678901234567890123456789012123456789012345678901
2
12345678901234567890123456789012123456789012345678901
these
points
down
on
scratch
paper,
and
try
to
decide
where
you
want
to
12345678901234567890123456789012123456789012345678901 22
12345678901234567890123456789012123456789012345678901 229
123456789012345678901234567890121234567890123456789012

http://www.xtremepapers.net

Part III: Section Review

www.petersons.com

http://www.xtremepapers.net

X-Ref

123456789012345678901234567890121234567890123456789012
12345678901234567890123456789012123456789012345678901 2
2
12345678901234567890123456789012123456789012345678901
place them in the essay. There is a fine balance between spending time 2
12345678901234567890123456789012123456789012345678901
2
12345678901234567890123456789012123456789012345678901
formulating your essay outline and getting down to actually writing it. The 2
12345678901234567890123456789012123456789012345678901
2
12345678901234567890123456789012123456789012345678901
more planning you do, the better your essay will appear in terms of 2
12345678901234567890123456789012123456789012345678901
2345678901234567890123456789012123456789012345678901
1 organization. Countering this attribute is the fact that a well-planned essay 2
2
12345678901234567890123456789012123456789012345678901
2
12345678901234567890123456789012123456789012345678901
of
only
two
paragraphs
is
not
as
good
as
a
four-paragraph
essay
with
some
2
12345678901234567890123456789012123456789012345678901
2
12345678901234567890123456789012123456789012345678901
continuity
problems.
Five
minutes
is
a
good
amount
of
time
to
plan.
After
2345678901234567890123456789012123456789012345678901
2
1
2
12345678901234567890123456789012123456789012345678901
that,
start
clacking
those
keys.
2
12345678901234567890123456789012123456789012345678901
2
12345678901234567890123456789012123456789012345678901
2
12345678901234567890123456789012123456789012345678901
2345678901234567890123456789012123456789012345678901
1 As stated on page 28, your typing speed is a factor in the Analytical 2
2
12345678901234567890123456789012123456789012345678901
2
12345678901234567890123456789012123456789012345678901
section,
since
getting
words
into
the
computer
can
be
a
time-consuming
2
12345678901234567890123456789012123456789012345678901
2
12345678901234567890123456789012123456789012345678901
process.
Take
the
test
on
page
28
to
see
how
many
words
you
should
be
2345678901234567890123456789012123456789012345678901
2
1
2
12345678901234567890123456789012123456789012345678901
able
to
type
for
the
first
essay.
If
you
are
a
speedy
typist,
you
can
give
2
12345678901234567890123456789012123456789012345678901
2
12345678901234567890123456789012123456789012345678901
yourself
a
little
more
time
to
brainstorm
ideas
and
organize
your
2
12345678901234567890123456789012123456789012345678901
2345678901234567890123456789012123456789012345678901
12345678901234567890123456789012123456789012345678901
thoughts. If you are an average typist, shoot for a four- to five-paragraph 2
2
12345678901234567890123456789012123456789012345678901
2
12345678901234567890123456789012123456789012345678901
essay.
If
you
are
a
very
slow,
hunt-and-peck,
are-these-letters-moving12345678901234567890123456789012123456789012345678901 2
12345678901234567890123456789012123456789012345678901
around-on-me? kind of typist, youll need to improve your speed. You 2
2
12345678901234567890123456789012123456789012345678901
1 want to be able to write at least 400 words for the first essay. Start 2
2
12345678901234567890123456789012123456789012345678901
12345678901234567890123456789012123456789012345678901
keeping a journal on your computer, write letters to your friends, type out 2
2
12345678901234567890123456789012123456789012345678901
12345678901234567890123456789012123456789012345678901
a manifesto, but do something to increase your speed so that your slow 2
2
12345678901234567890123456789012123456789012345678901
2
12345678901234567890123456789012123456789012345678901
typing no longer becomes an issue that can affect your score.
2
12345678901234567890123456789012123456789012345678901
2
12345678901234567890123456789012123456789012345678901
2345678901234567890123456789012123456789012345678901
12345678901234567890123456789012123456789012345678901 2
12345678901234567890123456789012123456789012345678901
Take the time now and jot down some ideas about how theaters do not 2
2
12345678901234567890123456789012123456789012345678901
make a city more attractive to visitors. You might have some ideas similar 2
12345678901234567890123456789012123456789012345678901
1 to the ones below, but the topic is broad enough that there are many 2
2
12345678901234567890123456789012123456789012345678901
2
12345678901234567890123456789012123456789012345678901
different ways to craft an essay arguing that this is not true.
2
12345678901234567890123456789012123456789012345678901
12345678901234567890123456789012123456789012345678901 2
12345678901234567890123456789012123456789012345678901
1. The introduction of a local theater company has little or 2
2
12345678901234567890123456789012123456789012345678901
12345678901234567890123456789012123456789012345678901
no impact at all on a citys attractiveness to visitors. (Main 2
2
12345678901234567890123456789012123456789012345678901
2
12345678901234567890123456789012123456789012345678901
idea,
first
sentence)
2
12345678901234567890123456789012123456789012345678901
2345678901234567890123456789012123456789012345678901
12345678901234567890123456789012123456789012345678901 2
12345678901234567890123456789012123456789012345678901
2. Local theater is not something that most visitors think 2
2
1
2
12345678901234567890123456789012123456789012345678901
about
when
considering
a
citypollution,
crime,
economic
12345678901234567890123456789012123456789012345678901 2
2345678901234567890123456789012123456789012345678901
2
12345678901234567890123456789012123456789012345678901
vitality all bigger factors.
2
12345678901234567890123456789012123456789012345678901
12345678901234567890123456789012123456789012345678901 2
12345678901234567890123456789012123456789012345678901
3. Tourist attractions can increase a citys attractiveness, 2
2
1
2345678901234567890123456789012123456789012345678901
2
12345678901234567890123456789012123456789012345678901
BUT
2
12345678901234567890123456789012123456789012345678901
2
12345678901234567890123456789012123456789012345678901
How much of a draw is a local theater?
2
12345678901234567890123456789012123456789012345678901
2
1
2345678901234567890123456789012123456789012345678901
12345678901234567890123456789012123456789012345678901
Except for NYC Broadway, what place is known for its 2
2
12345678901234567890123456789012123456789012345678901
2
12345678901234567890123456789012123456789012345678901
plays?
12345678901234567890123456789012123456789012345678901 2
2
1
12345678901234567890123456789012123456789012345678901
One-of-a-kind natural wonders, or museums of one 2
2
12345678901234567890123456789012123456789012345678901
2
12345678901234567890123456789012123456789012345678901
type
of
another,
these
are
things
that
increase
a
citys
12345678901234567890123456789012123456789012345678901 2
2
12345678901234567890123456789012123456789012345678901
attractiveness.
2
12345678901234567890123456789012123456789012345678901
12345678901234567890123456789012123456789012345678901 2
12345678901234567890123456789012123456789012345678901 2
2
230 12345678901234567890123456789012123456789012345678901
123456789012345678901234567890121234567890123456789012

Chapter 6: Analytical Writing Review

123456789012345678901234567890121234567890123456789012
12345678901234567890123456789012123456789012345678901 2
2
12345678901234567890123456789012123456789012345678901
12345678901234567890123456789012123456789012345678901
In this light, local theater is about as important as a 2
2
12345678901234567890123456789012123456789012345678901
2
12345678901234567890123456789012123456789012345678901
local
grocer.
Everyone
likes
having
one,
but
its
not
some
12345678901234567890123456789012123456789012345678901 2
2
12345678901234567890123456789012123456789012345678901
kind of tourist draw or city enhancement.
2
12345678901234567890123456789012123456789012345678901
12345678901234567890123456789012123456789012345678901
4. Also, some lousy governments have presided over great art, 2
2
12345678901234567890123456789012123456789012345678901
2
12345678901234567890123456789012123456789012345678901
so
connection
between
governance
and
pursuit
of
artistic
2
12345678901234567890123456789012123456789012345678901
2345678901234567890123456789012123456789012345678901
2
1
excellence pretty flimsy.
2
12345678901234567890123456789012123456789012345678901
2
12345678901234567890123456789012123456789012345678901
2
12345678901234567890123456789012123456789012345678901
The
last
statement
attacks
the
link
connecting
artistic
endeavor
with
good
2
12345678901234567890123456789012123456789012345678901
2345678901234567890123456789012123456789012345678901
1 governance, but the other statements dont have anything to do with it. 2
2
12345678901234567890123456789012123456789012345678901
12345678901234567890123456789012123456789012345678901
Statement 4 might make it into the essay, but its not crucial that it does. If 2
2
12345678901234567890123456789012123456789012345678901
12345678901234567890123456789012123456789012345678901
this essay were untimed, it would help to mount a comprehensive attack 2
2345678901234567890123456789012123456789012345678901
2
1
12345678901234567890123456789012123456789012345678901
on all salient points that you want to disagree with, but hey! This is Essay 2
2
12345678901234567890123456789012123456789012345678901
Writing under Pressure, and no one expects you to be able to cover all 2
12345678901234567890123456789012123456789012345678901
2
12345678901234567890123456789012123456789012345678901
bases with the paltry amount of time available.
2
12345678901234567890123456789012123456789012345678901
2
12345678901234567890123456789012123456789012345678901
2
12345678901234567890123456789012123456789012345678901
Statement
1
provides
your
perspective
on
the
issue,
placing
you
firmly
in
2
12345678901234567890123456789012123456789012345678901
2
12345678901234567890123456789012123456789012345678901
the
Ive
listened
to
the
original
statement
and
I
disagree
camp.
Make
2345678901234567890123456789012123456789012345678901
12345678901234567890123456789012123456789012345678901 22
1 sure you very clearly state whether you are for or against the original issue,
2
12345678901234567890123456789012123456789012345678901
2
12345678901234567890123456789012123456789012345678901
and
do
so
in
the
first
one
or
two
sentences.
The
readers
are
not
going
to
2
12345678901234567890123456789012123456789012345678901
2345678901234567890123456789012123456789012345678901
12345678901234567890123456789012123456789012345678901
spend four hours on every essay. Its more accurate to say that they will 2
2
12345678901234567890123456789012123456789012345678901
2
12345678901234567890123456789012123456789012345678901
spend
less
than
10
minutes
on
both
essays
combined,
in
which
case
you
12345678901234567890123456789012123456789012345678901 22
12345678901234567890123456789012123456789012345678901
want to have your position very clearly stated right away, in plain view and 2
12345678901234567890123456789012123456789012345678901
12345678901234567890123456789012123456789012345678901
for everyone to see. Mentally, a reader can then place a check alongside the 2
2
12345678901234567890123456789012123456789012345678901
12345678901234567890123456789012123456789012345678901
rubric standard that says, Every essay should have a clear, well- 2
2
1
2
12345678901234567890123456789012123456789012345678901
articulated point of view.
2
12345678901234567890123456789012123456789012345678901
2
12345678901234567890123456789012123456789012345678901
Your first paragraph could look like something like this:
2
12345678901234567890123456789012123456789012345678901
2
12345678901234567890123456789012123456789012345678901
2345678901234567890123456789012123456789012345678901
2
12345678901234567890123456789012123456789012345678901
Although a local theater company can have many positive 2
12345678901234567890123456789012123456789012345678901
1
effects on a community, it is unlikely that such a company will 2
2
12345678901234567890123456789012123456789012345678901
2
12345678901234567890123456789012123456789012345678901
have
any
impact
on
how
visitors
view
that
city.
There
are
many
2345678901234567890123456789012123456789012345678901
12345678901234567890123456789012123456789012345678901 22
12345678901234567890123456789012123456789012345678901
different ways people consider a city not their own. While there 2
1
12345678901234567890123456789012123456789012345678901
are certainly some people who use theater as part of their 2
2
12345678901234567890123456789012123456789012345678901
2345678901234567890123456789012123456789012345678901
12345678901234567890123456789012123456789012345678901
criteria, this number is probably very small. It is much more 2
2
12345678901234567890123456789012123456789012345678901
2
12345678901234567890123456789012123456789012345678901
likely
that
most
visitors
will
use
other
factors
to
decide
12345678901234567890123456789012123456789012345678901 22
1
whether or not a city is attractive to them, and the existence 2
12345678901234567890123456789012123456789012345678901
2
12345678901234567890123456789012123456789012345678901
of a local theater will not even be an issue.
2
12345678901234567890123456789012123456789012345678901
2
12345678901234567890123456789012123456789012345678901
2
One paragraph down, three to go.
12345678901234567890123456789012123456789012345678901
2345678901234567890123456789012123456789012345678901
12345678901234567890123456789012123456789012345678901 22
12345678901234567890123456789012123456789012345678901
Having stated your position, you must now back it up. The readers will 2
12345678901234567890123456789012123456789012345678901
2
12345678901234567890123456789012123456789012345678901
1 expect your second paragraph to back up your position using specific 2
2345678901234567890123456789012123456789012345678901
2
12345678901234567890123456789012123456789012345678901
1 details, so dont disappoint them. Flesh out Statement 2, and use precise 2
12345678901234567890123456789012123456789012345678901
examples whenever possible. Cite specific examples that work in your 2
2
12345678901234567890123456789012123456789012345678901
2
12345678901234567890123456789012123456789012345678901
favor.
The
point
is
to
be
convincing
by
using
facts
to
state
your
point.
You
2345678901234567890123456789012123456789012345678901
2
1
2
12345678901234567890123456789012123456789012345678901
cant
just
say,
Cmon!
Believe
me!
I
know
what
Im
talkin
about!
12345678901234567890123456789012123456789012345678901 22
12345678901234567890123456789012123456789012345678901 231
123456789012345678901234567890121234567890123456789012

http://www.xtremepapers.net

Part III: Section Review

123456789012345678901234567890121234567890123456789012
12345678901234567890123456789012123456789012345678901 2
2
12345678901234567890123456789012123456789012345678901
Sample Second Paragraph
2
12345678901234567890123456789012123456789012345678901
2
12345678901234567890123456789012123456789012345678901
2
12345678901234567890123456789012123456789012345678901
(A)
Looking
back
over
the
past
ten
years,
many
of
the
Most
12345678901234567890123456789012123456789012345678901 2
12345678901234567890123456789012123456789012345678901
popular cities in the U.S. became popular for reasons that had 2
2
12345678901234567890123456789012123456789012345678901
2
12345678901234567890123456789012123456789012345678901
(B)
Seattle
became
known
as
a
nothing
to
do
with
theater.
2
12345678901234567890123456789012123456789012345678901
2
12345678901234567890123456789012123456789012345678901
place
where
the
trendiest
rock-and-roll
music
was
created.
2
12345678901234567890123456789012123456789012345678901
2345678901234567890123456789012123456789012345678901
2
1
(C) That was good enough for a lot of young people. (D) Silicon
2
12345678901234567890123456789012123456789012345678901
2
12345678901234567890123456789012123456789012345678901
Valley
was
the
center
of
the
Internet
explosion,
and
people
2
12345678901234567890123456789012123456789012345678901
2
12345678901234567890123456789012123456789012345678901
flocked
there
in
search
of
great
jobs,
not
great
Shakespeare.
2345678901234567890123456789012123456789012345678901
2
1
2
12345678901234567890123456789012123456789012345678901
(E)
Several
older
cities
revitalized
their
image
with
large
2
12345678901234567890123456789012123456789012345678901
2
12345678901234567890123456789012123456789012345678901
public
renovations
like
the
Harbor
project
in
Baltimore.
2
12345678901234567890123456789012123456789012345678901
2345678901234567890123456789012123456789012345678901
2
1
(F) Baltimores harbor area now contains restaurants, malls,
2
12345678901234567890123456789012123456789012345678901
2
12345678901234567890123456789012123456789012345678901
(G) No
an
aquarium,
and
various
other
tourist
attractions.
2
12345678901234567890123456789012123456789012345678901
2
12345678901234567890123456789012123456789012345678901
local theater, though, showing how it was not factor.
2345678901234567890123456789012123456789012345678901
12345678901234567890123456789012123456789012345678901 2
12345678901234567890123456789012123456789012345678901 2
12345678901234567890123456789012123456789012345678901 2
12345678901234567890123456789012123456789012345678901
Admittedly, this is not the greatest second paragraph the world has ever 2
2
12345678901234567890123456789012123456789012345678901
2
12345678901234567890123456789012123456789012345678901
1 seen. Sentence A capitalizes the word Most for no reason. Sentences B and 2
12345678901234567890123456789012123456789012345678901
C could probably be combined to form one single good sentence instead of 2
2
12345678901234567890123456789012123456789012345678901
2
12345678901234567890123456789012123456789012345678901
two
choppy
sentences.
Sentence
D
is
a
bit
vague
on
details,
with
phrases
2345678901234567890123456789012123456789012345678901
12345678901234567890123456789012123456789012345678901 2
12345678901234567890123456789012123456789012345678901
like Silicon Valley and Internet explosion tossed around with little 2
2
12345678901234567890123456789012123456789012345678901
2
12345678901234567890123456789012123456789012345678901
explanation. Sentence E has the word Harbor capitalized incorrectly.
12345678901234567890123456789012123456789012345678901 2
12345678901234567890123456789012123456789012345678901 2
12345678901234567890123456789012123456789012345678901
Sentences E, F, and G are all culled from the authors brief visit to 2
2
12345678901234567890123456789012123456789012345678901
Baltimore last year, so a bold statement like G is not known to be 2
12345678901234567890123456789012123456789012345678901
1 definitively true. They are just the authors recollections from a brief 2
2
12345678901234567890123456789012123456789012345678901
2
12345678901234567890123456789012123456789012345678901
three-day visit two years ago that were dredged up from memory to help 2
12345678901234567890123456789012123456789012345678901
2
12345678901234567890123456789012123456789012345678901
bolster a point on the standardized test. Someone could go to Baltimore or 2
12345678901234567890123456789012123456789012345678901
2345678901234567890123456789012123456789012345678901
12345678901234567890123456789012123456789012345678901
do research in a library to determine whether or not local theater was a 2
2
12345678901234567890123456789012123456789012345678901
1 factor in the Baltimore harbor renaissance. However, that someone is not 2
2
12345678901234567890123456789012123456789012345678901
2
12345678901234567890123456789012123456789012345678901
going
to
be
the
reader
of
your
essay,
since
this
individual
has
only
a
couple
2345678901234567890123456789012123456789012345678901
12345678901234567890123456789012123456789012345678901 2
2
12345678901234567890123456789012123456789012345678901
1 of minutes to read through your essays, assign a grade, and then move on 2
12345678901234567890123456789012123456789012345678901
to the next. In other words, while you cantand shouldntlie outright, 2
2
12345678901234567890123456789012123456789012345678901
2345678901234567890123456789012123456789012345678901
12345678901234567890123456789012123456789012345678901
you can use broad, plausible facts to prove your point. Looking over the 2
2
12345678901234567890123456789012123456789012345678901
2
12345678901234567890123456789012123456789012345678901
statements
made
in
the
second
paragraph,
you
can
see
that
all
of
them
are
12345678901234567890123456789012123456789012345678901 2
1 a nice blend of specificity (naming the actual city) and generalization 2
2
12345678901234567890123456789012123456789012345678901
2
12345678901234567890123456789012123456789012345678901
(people
flocked
there
in
search
of
great
jobs).
2
12345678901234567890123456789012123456789012345678901
2
12345678901234567890123456789012123456789012345678901
12345678901234567890123456789012123456789012345678901
For all its faults, this second paragraph uses a host of supporting details to 2
2345678901234567890123456789012123456789012345678901
2
12345678901234567890123456789012123456789012345678901
back up the initial perspective stated in the first paragraph. That is exactly 2
12345678901234567890123456789012123456789012345678901
2
12345678901234567890123456789012123456789012345678901
what you want a second paragraph to do in order to get a good score on 2
12345678901234567890123456789012123456789012345678901
1 this essay. If this were an untimed essay, you would be expected to correct 2
2
12345678901234567890123456789012123456789012345678901
2
12345678901234567890123456789012123456789012345678901
mistakes like improperly capitalized words, and you might also be 2
12345678901234567890123456789012123456789012345678901
12345678901234567890123456789012123456789012345678901
expected to provide actual data. For instance, an untimed essay should 2
2
12345678901234567890123456789012123456789012345678901
2345678901234567890123456789012123456789012345678901
1 provide actual census figures for the growth rate (and unemployment rate) 2
12345678901234567890123456789012123456789012345678901 2
12345678901234567890123456789012123456789012345678901 2
2
232 12345678901234567890123456789012123456789012345678901
123456789012345678901234567890121234567890123456789012

www.petersons.com

http://www.xtremepapers.net

Chapter 6: Analytical Writing Review

123456789012345678901234567890121234567890123456789012
12345678901234567890123456789012123456789012345678901 2
2
12345678901234567890123456789012123456789012345678901
in Silicon Valley between 1990 and 2000, and it should probably give a 2
12345678901234567890123456789012123456789012345678901
2
12345678901234567890123456789012123456789012345678901
more precise geographical location than just Silicon Valley.
2
12345678901234567890123456789012123456789012345678901
12345678901234567890123456789012123456789012345678901 2
12345678901234567890123456789012123456789012345678901
But this isnt an untimed essay, is it? The people reading this essay dont 2
2
12345678901234567890123456789012123456789012345678901
2
12345678901234567890123456789012123456789012345678901
expect
exact
facts.
They
are
looking
for
a
slew
of
supporting
details
that
2
12345678901234567890123456789012123456789012345678901
2
12345678901234567890123456789012123456789012345678901
back
up
the
original
position,
and
thats
just
what
the
second
paragraph
2
12345678901234567890123456789012123456789012345678901
2345678901234567890123456789012123456789012345678901
1 gives them. The paragraph covers three different areas of the country and 2
2
12345678901234567890123456789012123456789012345678901
2
12345678901234567890123456789012123456789012345678901
shows
how
local
theater
was
not
a
factor
in
making
them
attractive
to
2
12345678901234567890123456789012123456789012345678901
2
12345678901234567890123456789012123456789012345678901
visitors.
2345678901234567890123456789012123456789012345678901
2
1
2
12345678901234567890123456789012123456789012345678901
12345678901234567890123456789012123456789012345678901
With the first paragraph firmly supported by the second, its time to 2
2
12345678901234567890123456789012123456789012345678901
2
12345678901234567890123456789012123456789012345678901
change tack.
2345678901234567890123456789012123456789012345678901
2
1
2
12345678901234567890123456789012123456789012345678901
Your third paragraph should be the I understand the other side of the 2
12345678901234567890123456789012123456789012345678901
2
12345678901234567890123456789012123456789012345678901
argument, I just dont agree with it paragraph. Somewhere in your essay 2
12345678901234567890123456789012123456789012345678901
2345678901234567890123456789012123456789012345678901
2
12345678901234567890123456789012123456789012345678901
the readers want to see that you understand both sides of the issue, and the 2
12345678901234567890123456789012123456789012345678901
12345678901234567890123456789012123456789012345678901
third paragraphs as good a place as any. So throw the opposition a bone, 2
2
12345678901234567890123456789012123456789012345678901
2
12345678901234567890123456789012123456789012345678901
but
then
show
how
this
little
point
still
does
not
alter
the
fact
that
your
12345678901234567890123456789012123456789012345678901 22
1 initial position is the superior one.
2
12345678901234567890123456789012123456789012345678901
2
12345678901234567890123456789012123456789012345678901
2
12345678901234567890123456789012123456789012345678901
2345678901234567890123456789012123456789012345678901
2
12345678901234567890123456789012123456789012345678901
Sample Third Paragraph
2
12345678901234567890123456789012123456789012345678901
2
12345678901234567890123456789012123456789012345678901
(H) Of course people do enjoy going to the theater occasion2
12345678901234567890123456789012123456789012345678901
2
12345678901234567890123456789012123456789012345678901
ally,
so
having
a
local
theater
company
would
make
a
city
12345678901234567890123456789012123456789012345678901 22
12345678901234567890123456789012123456789012345678901
more attractive for some. (I) But who, exactly? (J) You would
2
12345678901234567890123456789012123456789012345678901
2
12345678901234567890123456789012123456789012345678901
think
a
local
theater
would
benefit
the
local
population
the
2
1
2345678901234567890123456789012123456789012345678901
2
12345678901234567890123456789012123456789012345678901
most, and that visitors would hardly notice it. (K) Most people
2
12345678901234567890123456789012123456789012345678901
2
1
who
come
in
from
out
of
town
want
to
go
see
large,
unique
12345678901234567890123456789012123456789012345678901 2
2
12345678901234567890123456789012123456789012345678901
tourist attractions like Graceland or an exceptional natural
2
12345678901234567890123456789012123456789012345678901
2
12345678901234567890123456789012123456789012345678901
(L)
The
local
theater
is
there
to
raise
wonder
like
Pikes
Peak.
12345678901234567890123456789012123456789012345678901 2
2
12345678901234567890123456789012123456789012345678901
the artistic consciousness of the community, not attract
2
12345678901234567890123456789012123456789012345678901
2345678901234567890123456789012123456789012345678901
12345678901234567890123456789012123456789012345678901
visitors from another city who may well have a Local theater 2
2
12345678901234567890123456789012123456789012345678901
2
1
scene
of
their
own.
12345678901234567890123456789012123456789012345678901 2
12345678901234567890123456789012123456789012345678901 2
2
12345678901234567890123456789012123456789012345678901
2
12345678901234567890123456789012123456789012345678901
Again, this paragraph is not flawless, nor does it have to be. All it must do 2
12345678901234567890123456789012123456789012345678901
12345678901234567890123456789012123456789012345678901
is show that you are a balanced person able to see both sides of an issue, 2
2
12345678901234567890123456789012123456789012345678901
2345678901234567890123456789012123456789012345678901
12345678901234567890123456789012123456789012345678901
and that you have reasons for rejecting the opposing side in favor of your 2
2
12345678901234567890123456789012123456789012345678901
2
12345678901234567890123456789012123456789012345678901
own.
Sentence
H
is
the
proverbial
bone,
and
the
rest
of
the
paragraph
does
12345678901234567890123456789012123456789012345678901 22
1 its best to explain why this viewpoint is not as strong as your own. This is
2
12345678901234567890123456789012123456789012345678901
2
12345678901234567890123456789012123456789012345678901
done
by
making
broad,
plausible
statements
with
a
dash
of
specificity
if
2
12345678901234567890123456789012123456789012345678901
2
12345678901234567890123456789012123456789012345678901
possible.
Sentence
L
works
fine
as,
Most
people
who
come
in
from
out
of
12345678901234567890123456789012123456789012345678901 2
2345678901234567890123456789012123456789012345678901
2
12345678901234567890123456789012123456789012345678901
1 town want to go see large, unique tourist attractions or exceptional 2
12345678901234567890123456789012123456789012345678901
natural wonders. It works better by including precise examples, since this 2
2
12345678901234567890123456789012123456789012345678901
2
12345678901234567890123456789012123456789012345678901
helps
reinforce
the
point
and
it
shows
the
reader
that
you
can
cite
specific
2345678901234567890123456789012123456789012345678901
2
1
12345678901234567890123456789012123456789012345678901
examples if you have to. Think of it as your way of saying, If I had time, 2
12345678901234567890123456789012123456789012345678901 2
12345678901234567890123456789012123456789012345678901 2 233
123456789012345678901234567890121234567890123456789012

http://www.xtremepapers.net

Part III: Section Review

123456789012345678901234567890121234567890123456789012
12345678901234567890123456789012123456789012345678901 2
2
12345678901234567890123456789012123456789012345678901
I would bore you with precise details, but I only have time for a few right 2
12345678901234567890123456789012123456789012345678901
2
12345678901234567890123456789012123456789012345678901
now.
2
12345678901234567890123456789012123456789012345678901
12345678901234567890123456789012123456789012345678901 2
12345678901234567890123456789012123456789012345678901
Specific examples like this help mask some other, vaguer claims. For 2
2
12345678901234567890123456789012123456789012345678901
2
12345678901234567890123456789012123456789012345678901
instance,
what
exactly
does
the
artistic
consciousness
of
the
community
2
12345678901234567890123456789012123456789012345678901
2
12345678901234567890123456789012123456789012345678901
in
Sentence
E
mean?
The
precise
meaning
is
up
for
grabs,
but
it
sure
sounds
2
12345678901234567890123456789012123456789012345678901
2345678901234567890123456789012123456789012345678901
1 good, doesnt it? Readers skimming their way through this essay wont 2
2
12345678901234567890123456789012123456789012345678901
2
12345678901234567890123456789012123456789012345678901
pause,
place
their
chin
in
their
hand,
and
deeply
reflect
upon
the
exact
2
12345678901234567890123456789012123456789012345678901
2
12345678901234567890123456789012123456789012345678901
meaning
of
that
phrase.
Instead,
theyll
just
go
uh-huh
absently
and
2345678901234567890123456789012123456789012345678901
2
1
2
12345678901234567890123456789012123456789012345678901
keep
on
blazing
through
the
text.
You
will
get
the
benefit
of
the
doubt
due
2
12345678901234567890123456789012123456789012345678901
2
12345678901234567890123456789012123456789012345678901
to
the
time
constraints
of
the
format.
2
12345678901234567890123456789012123456789012345678901
2345678901234567890123456789012123456789012345678901
2
1
12345678901234567890123456789012123456789012345678901
With three paragraphs in the bag, its time to check the clock. If you have 2
2
12345678901234567890123456789012123456789012345678901
less than 5 minutes left, you should hammer out a concluding sentence or 2
12345678901234567890123456789012123456789012345678901
2
12345678901234567890123456789012123456789012345678901
two for paragraph four and then glance over the rest of your essay. Try to 2
12345678901234567890123456789012123456789012345678901
2
12345678901234567890123456789012123456789012345678901
catch any obvious grammatical errors, and try to work on the overall flow 2
12345678901234567890123456789012123456789012345678901
2
12345678901234567890123456789012123456789012345678901
of the essay, making sure that one paragraph flows well into another. You 2
12345678901234567890123456789012123456789012345678901
2345678901234567890123456789012123456789012345678901
2
12345678901234567890123456789012123456789012345678901
1 could use this time to combine Sentences B and C in the second paragraph, 2
2
12345678901234567890123456789012123456789012345678901
for instance.
2
12345678901234567890123456789012123456789012345678901
2
12345678901234567890123456789012123456789012345678901
2345678901234567890123456789012123456789012345678901
12345678901234567890123456789012123456789012345678901
If you finish three paragraphs and have around 10 minutes left, then you 2
2
12345678901234567890123456789012123456789012345678901
2
12345678901234567890123456789012123456789012345678901
can
use
the
word-processing
format
to
your
advantage
in
the
following
12345678901234567890123456789012123456789012345678901 2
2
12345678901234567890123456789012123456789012345678901
ways:
2
12345678901234567890123456789012123456789012345678901
12345678901234567890123456789012123456789012345678901 2
12345678901234567890123456789012123456789012345678901
1. Write a 2- to 3-sentence final paragraph that restates your take on 2
2
12345678901234567890123456789012123456789012345678901
2
1
the initial proposition.
2345678901234567890123456789012123456789012345678901
12345678901234567890123456789012123456789012345678901 2
2
12345678901234567890123456789012123456789012345678901
2. Look through everything youve written and catch any blatant 2
1
2
12345678901234567890123456789012123456789012345678901
grammatical mistakes.
2
12345678901234567890123456789012123456789012345678901
2345678901234567890123456789012123456789012345678901
12345678901234567890123456789012123456789012345678901 2
12345678901234567890123456789012123456789012345678901
3. Now, try to slip in another paragraph between the third 2
2
1
2
12345678901234567890123456789012123456789012345678901
paragraph
and
the
final
one.
Your
conclusion
is
already
in
place,
2
12345678901234567890123456789012123456789012345678901
2345678901234567890123456789012123456789012345678901
12345678901234567890123456789012123456789012345678901
so you dont have to worry about ending your essay properly. 2
2
12345678901234567890123456789012123456789012345678901
2
1
You
can
use
the
remaining
time
to
create
a
fourth
paragraph
that
12345678901234567890123456789012123456789012345678901 2
12345678901234567890123456789012123456789012345678901
piles on more detail to support your belief. You might attack the 2
2
12345678901234567890123456789012123456789012345678901
2
12345678901234567890123456789012123456789012345678901
good
governance
angle
at
this
point,
which
is
something
you
2
12345678901234567890123456789012123456789012345678901
2
12345678901234567890123456789012123456789012345678901
havent really addressed in the rest of your essay.
12345678901234567890123456789012123456789012345678901 2
2345678901234567890123456789012123456789012345678901
2
12345678901234567890123456789012123456789012345678901
4. When the clock hits 1 minute to go, stop work on the fourth 2
12345678901234567890123456789012123456789012345678901
2
12345678901234567890123456789012123456789012345678901
paragraph and make sure the transition between it and your final 2
12345678901234567890123456789012123456789012345678901
2
1
paragraph is smooth. If you only had a chance to write two 2
12345678901234567890123456789012123456789012345678901
2
12345678901234567890123456789012123456789012345678901
sentences in the fourth paragraph, then just tack your two- 2
12345678901234567890123456789012123456789012345678901
12345678901234567890123456789012123456789012345678901
sentence conclusion to the end of it. However, if you were able to 2
2
12345678901234567890123456789012123456789012345678901
2345678901234567890123456789012123456789012345678901
12345678901234567890123456789012123456789012345678901
write 35 sentences in the fourth paragraph, dont combine the 2
2
1
2
12345678901234567890123456789012123456789012345678901
final
paragraph
with
the
fourth.
12345678901234567890123456789012123456789012345678901 2
2
12345678901234567890123456789012123456789012345678901
12345678901234567890123456789012123456789012345678901 2
12345678901234567890123456789012123456789012345678901 2
12345678901234567890123456789012123456789012345678901 2
2
234 12345678901234567890123456789012123456789012345678901
123456789012345678901234567890121234567890123456789012

www.petersons.com

http://www.xtremepapers.net

Chapter 6: Analytical Writing Review

123456789012345678901234567890121234567890123456789012
12345678901234567890123456789012123456789012345678901 2
2
12345678901234567890123456789012123456789012345678901
If you had to write out your essay, you wouldnt be able to go back and 2
12345678901234567890123456789012123456789012345678901
2
12345678901234567890123456789012123456789012345678901
insert an entire paragraph into the rest of the text. Yet the computer format 2
12345678901234567890123456789012123456789012345678901
2
12345678901234567890123456789012123456789012345678901
allows you to do that, so you should take advantage of it. Writing the final 2
12345678901234567890123456789012123456789012345678901
2345678901234567890123456789012123456789012345678901
1 paragraph first fulfills the rubric standard, restates original position 2
2
12345678901234567890123456789012123456789012345678901
2
12345678901234567890123456789012123456789012345678901
clearly,
a
basic
requirement
in
essay-writing.
You
can
then
use
the
2
12345678901234567890123456789012123456789012345678901
2
12345678901234567890123456789012123456789012345678901
remaining
time
to
expound
upon
your
perspective
on
the
issue.
No
one
2345678901234567890123456789012123456789012345678901
2
1
2
12345678901234567890123456789012123456789012345678901
except
you
will
know
that
you
didnt
write
the
entire
essay
out
in
perfect
2
12345678901234567890123456789012123456789012345678901
2
12345678901234567890123456789012123456789012345678901
linear
fashion.
All
they
will
see
is
a
well-organized
five-paragraph
essay.
2
12345678901234567890123456789012123456789012345678901
2345678901234567890123456789012123456789012345678901
2
1
2
12345678901234567890123456789012123456789012345678901
2
12345678901234567890123456789012123456789012345678901
Sample Last Paragraph to Write First
2
12345678901234567890123456789012123456789012345678901
2
12345678901234567890123456789012123456789012345678901
Local
theater
has
benefits,
but
these
benefits
serve
mainly
2345678901234567890123456789012123456789012345678901
2
1
2
12345678901234567890123456789012123456789012345678901
the community itself and remain unknown or of no concern to
2
12345678901234567890123456789012123456789012345678901
2
12345678901234567890123456789012123456789012345678901
outsiders.
Therefore,
the
creation
of
a
local
theater
2
12345678901234567890123456789012123456789012345678901
2345678901234567890123456789012123456789012345678901
2
12345678901234567890123456789012123456789012345678901
company is unlikely to have any impact on whether or not a
2
12345678901234567890123456789012123456789012345678901
2
12345678901234567890123456789012123456789012345678901
city
is
attractive
to
outside
visitors.
12345678901234567890123456789012123456789012345678901 22
12345678901234567890123456789012123456789012345678901 2
12345678901234567890123456789012123456789012345678901 2
1
2
12345678901234567890123456789012123456789012345678901
Then, squeeze in a fourth paragraph above that can go something like:
2
12345678901234567890123456789012123456789012345678901
2
12345678901234567890123456789012123456789012345678901
2345678901234567890123456789012123456789012345678901
2
12345678901234567890123456789012123456789012345678901
(M) Furthermore, while the connection between theater and
2
12345678901234567890123456789012123456789012345678901
2
12345678901234567890123456789012123456789012345678901
city
attractiveness
is
fairly
slim,
the
connection
between
art
12345678901234567890123456789012123456789012345678901 22
12345678901234567890123456789012123456789012345678901
and good government is even more so. (N) A city might provide
2
12345678901234567890123456789012123456789012345678901
2
12345678901234567890123456789012123456789012345678901
(O)
Although
public
funds
to
help
a
local
theater
get
started.
12345678901234567890123456789012123456789012345678901 22
12345678901234567890123456789012123456789012345678901
some would like this, others would call it bad government,
2
1
12345678901234567890123456789012123456789012345678901
since that was tax money that could be spent elsewhere, like 2
2
12345678901234567890123456789012123456789012345678901
2
12345678901234567890123456789012123456789012345678901
(P)
In
this
in
a
public
school
or
to
provide
more
police
officers.
12345678901234567890123456789012123456789012345678901 2
12345678901234567890123456789012123456789012345678901
light, the connection between theater and government would 2
2
12345678901234567890123456789012123456789012345678901
2
12345678901234567890123456789012123456789012345678901
be
considered
bad
government
since
public
funds
are
being
12345678901234567890123456789012123456789012345678901 2
2
12345678901234567890123456789012123456789012345678901
spent poorly.
2
12345678901234567890123456789012123456789012345678901
2345678901234567890123456789012123456789012345678901
12345678901234567890123456789012123456789012345678901 22
12345678901234567890123456789012123456789012345678901 2
1 This fourth paragraph has mistakes just like the others before it, but it still
12345678901234567890123456789012123456789012345678901 2
12345678901234567890123456789012123456789012345678901
achieves its purpose, which is to show your ability to approach the issue 2
2
12345678901234567890123456789012123456789012345678901
2
12345678901234567890123456789012123456789012345678901
from
more
than
one
angle.
Sentence
N
sets
up
a
what-if
hypothesis,
and
2
12345678901234567890123456789012123456789012345678901
2
12345678901234567890123456789012123456789012345678901
then
Sentence
O
shows
how
this
could
be
considered
poor
government.
12345678901234567890123456789012123456789012345678901 2
2345678901234567890123456789012123456789012345678901
12345678901234567890123456789012123456789012345678901
Sentence O is a bit unwieldy, but thats OK, since the phrase like in a 2
2
12345678901234567890123456789012123456789012345678901
2
12345678901234567890123456789012123456789012345678901
public
school
or
to
provide
more
police
officers
shows
that
you
could
be
12345678901234567890123456789012123456789012345678901 22
1 more specific . . . if only you had more time (fake sigh). Since you dont
2
12345678901234567890123456789012123456789012345678901
2
12345678901234567890123456789012123456789012345678901
have enough time, the essay has to stay as is.
2
12345678901234567890123456789012123456789012345678901
2
12345678901234567890123456789012123456789012345678901
Grouping all the sample paragraphs together gives you a good blueprint 2
12345678901234567890123456789012123456789012345678901
2345678901234567890123456789012123456789012345678901
2
12345678901234567890123456789012123456789012345678901
1 on how to achieve a solid score on the first Analytical essay. You dont 2
2
12345678901234567890123456789012123456789012345678901
have to follow this blueprint exactly, but it wouldnt hurt you if you did. 2
12345678901234567890123456789012123456789012345678901
2
12345678901234567890123456789012123456789012345678901
Whether or not you add any variations, make sure that your first essay 2
12345678901234567890123456789012123456789012345678901
2
12345678901234567890123456789012123456789012345678901
clearly contains the following points:
2
12345678901234567890123456789012123456789012345678901
12345678901234567890123456789012123456789012345678901 2 235
123456789012345678901234567890121234567890123456789012

http://www.xtremepapers.net

Part III: Section Review

123456789012345678901234567890121234567890123456789012
12345678901234567890123456789012123456789012345678901 2
2
12345678901234567890123456789012123456789012345678901
12345678901234567890123456789012123456789012345678901 2
2
12345678901234567890123456789012123456789012345678901
Key Points to Include on the First Essay
2
12345678901234567890123456789012123456789012345678901
12345678901234567890123456789012123456789012345678901 2
2
12345678901234567890123456789012123456789012345678901
2
12345678901234567890123456789012123456789012345678901
1. A Clearly Stated Position. Dont send the readers on a scavenger
2
12345678901234567890123456789012123456789012345678901
2
12345678901234567890123456789012123456789012345678901
hunt to see how you feel about the issue. Make sure your point of
2
12345678901234567890123456789012123456789012345678901
2
12345678901234567890123456789012123456789012345678901
view is clearly stated. Basically, you will either disagree or agree
2345678901234567890123456789012123456789012345678901
2
1
with the initial topic sentence. The safest place to state your point
2
12345678901234567890123456789012123456789012345678901
2
12345678901234567890123456789012123456789012345678901
of view is the first paragraph.
2
12345678901234567890123456789012123456789012345678901
2
12345678901234567890123456789012123456789012345678901
2345678901234567890123456789012123456789012345678901
2
1 2. A Cartload of Plausible Supporting Details. You cant use the
2
12345678901234567890123456789012123456789012345678901
2
12345678901234567890123456789012123456789012345678901
phrase
because
I
say
so
to
back
up
your
argument.
You
need
to
2
12345678901234567890123456789012123456789012345678901
2
12345678901234567890123456789012123456789012345678901
come
up
with
believable
reasons
that
justify
your
position.
The
2345678901234567890123456789012123456789012345678901
2
1
2
12345678901234567890123456789012123456789012345678901
more
specific
they
are,
the
better,
but
the
format
of
the
2
12345678901234567890123456789012123456789012345678901
2
12345678901234567890123456789012123456789012345678901
time-intensive
essay
will
prevent
you
from
using
hard
data.
You
2
12345678901234567890123456789012123456789012345678901
2345678901234567890123456789012123456789012345678901
2
12345678901234567890123456789012123456789012345678901
could memorize an almanac or census book if you like, but this is
2
12345678901234567890123456789012123456789012345678901
2
12345678901234567890123456789012123456789012345678901
probably
not
a
good
use
of
your
time.
Instead,
paint
with
broad
12345678901234567890123456789012123456789012345678901 2
2
12345678901234567890123456789012123456789012345678901
strokes with phrases like, many people enjoy fine seafood and
2
12345678901234567890123456789012123456789012345678901
2
1
computer skills are becoming increasingly important in the
2
12345678901234567890123456789012123456789012345678901
2
12345678901234567890123456789012123456789012345678901
modern world. Both of these statements are believable, even
2
12345678901234567890123456789012123456789012345678901
2345678901234567890123456789012123456789012345678901
2
12345678901234567890123456789012123456789012345678901
though they contain no hard facts.
2
12345678901234567890123456789012123456789012345678901
2
12345678901234567890123456789012123456789012345678901
3. Demonstrate That You Can See Other Sides Point of View, Even
2
12345678901234567890123456789012123456789012345678901
2
12345678901234567890123456789012123456789012345678901
Though
You
Disagree
with
It.
Sadly,
actual
debate
is
very
rare,
at
12345678901234567890123456789012123456789012345678901 2
2
12345678901234567890123456789012123456789012345678901
least on television. On TV, most shows put two implacable
2
12345678901234567890123456789012123456789012345678901
2
12345678901234567890123456789012123456789012345678901
enemies
on
screen
together,
and
then
watch
both
sides
use
every
2
1
2345678901234567890123456789012123456789012345678901
2
12345678901234567890123456789012123456789012345678901
sophistic trick in the book to browbeat their opponent and win
2
12345678901234567890123456789012123456789012345678901
2
1
the
argument.
It
might
make
for
entertaining
TV,
but
its
a
lousy
12345678901234567890123456789012123456789012345678901 2
2
12345678901234567890123456789012123456789012345678901
discourse. Such an extreme, unbending position is the exact
2
12345678901234567890123456789012123456789012345678901
2
12345678901234567890123456789012123456789012345678901
opposite
of
what
you
want
in
your
essay.
Take
a
sentence
or
two
12345678901234567890123456789012123456789012345678901 2
2
12345678901234567890123456789012123456789012345678901
to show how you understand the opponents point of view, and
2
12345678901234567890123456789012123456789012345678901
2345678901234567890123456789012123456789012345678901
2
12345678901234567890123456789012123456789012345678901
then use the rest of the paragraph to explain why you feel this
2
12345678901234567890123456789012123456789012345678901
2
1
position
is
not
as
strong
as
the
one
you
believe.
12345678901234567890123456789012123456789012345678901 2
12345678901234567890123456789012123456789012345678901 2
2
12345678901234567890123456789012123456789012345678901
4. Have a Clear Conclusion. Its a basic point of every essay, and the
2
12345678901234567890123456789012123456789012345678901
readers will be looking for it. If possible, try to rephrase your
2
12345678901234567890123456789012123456789012345678901
2
12345678901234567890123456789012123456789012345678901
original position. Dont use the exact same words as you did at the
2
12345678901234567890123456789012123456789012345678901
2345678901234567890123456789012123456789012345678901
2
12345678901234567890123456789012123456789012345678901
beginning of the essay.
2
12345678901234567890123456789012123456789012345678901
12345678901234567890123456789012123456789012345678901 2
12345678901234567890123456789012123456789012345678901 2
2
1
2
12345678901234567890123456789012123456789012345678901
12345678901234567890123456789012123456789012345678901
Look at the some of the sample issues at the GRE Web site and decide how 2
2
12345678901234567890123456789012123456789012345678901
12345678901234567890123456789012123456789012345678901
you would approach them using the information in the chart above. 2
2
12345678901234567890123456789012123456789012345678901
Would you agree or disagree? What facts would you use to support your 2
12345678901234567890123456789012123456789012345678901
2
12345678901234567890123456789012123456789012345678901
case? How would you show that you understood the other sides view? If 2
12345678901234567890123456789012123456789012345678901
2
12345678901234567890123456789012123456789012345678901
the Analytical score is important to you, give yourself 45 minutes and 2
12345678901234567890123456789012123456789012345678901
2345678901234567890123456789012123456789012345678901
1 practice writing some sample essays. Use a computer to type out your 2
12345678901234567890123456789012123456789012345678901 2
12345678901234567890123456789012123456789012345678901 2
2
236 12345678901234567890123456789012123456789012345678901
123456789012345678901234567890121234567890123456789012

www.petersons.com

http://www.xtremepapers.net

Chapter 6: Analytical Writing Review

Note

123456789012345678901234567890121234567890123456789012
12345678901234567890123456789012123456789012345678901 2
2
12345678901234567890123456789012123456789012345678901
response, since you want to practice your typing skills as well. Since there 2
12345678901234567890123456789012123456789012345678901
2
12345678901234567890123456789012123456789012345678901
are about 240 topics listed, it is not very likely that a topic you practice on 2
12345678901234567890123456789012123456789012345678901
2
12345678901234567890123456789012123456789012345678901
will be one of the ones you get for your actual GRE. Then again, stranger 2
12345678901234567890123456789012123456789012345678901
2345678901234567890123456789012123456789012345678901
1 things have happened, and people do get lucky. If you glance over a bunch 2
2
12345678901234567890123456789012123456789012345678901
2
12345678901234567890123456789012123456789012345678901
of
the
issue
topicssay,
about
half
of
themand
think
about
how
you
2
12345678901234567890123456789012123456789012345678901
2
12345678901234567890123456789012123456789012345678901
would
respond
to
them,
you
will
be
making
your
own
luck.
2345678901234567890123456789012123456789012345678901
2
1
2
12345678901234567890123456789012123456789012345678901
2
12345678901234567890123456789012123456789012345678901
The
last
thing
to
note
is
that
you
should
take
the
entire
45
minutes
on
the
2
12345678901234567890123456789012123456789012345678901
2
12345678901234567890123456789012123456789012345678901
essay.
When
its
over,
stretch
your
fingers
and
get
prepared
for
some
more
2345678901234567890123456789012123456789012345678901
2
1
2
12345678901234567890123456789012123456789012345678901
typing.
The
Argument
essay
is
next
on
the
menu.
2
12345678901234567890123456789012123456789012345678901
2
12345678901234567890123456789012123456789012345678901
2
12345678901234567890123456789012123456789012345678901
2345678901234567890123456789012123456789012345678901
2
1
2345678901234567890123456789012123456789012345678901 2
1
The
Argument Essay
2
12345678901234567890123456789012123456789012345678901
2
12345678901234567890123456789012123456789012345678901
12345678901234567890123456789012123456789012345678901
The second essay is 15 minutes shorter than the first, and you are only 2
2345678901234567890123456789012123456789012345678901
12345678901234567890123456789012123456789012345678901 22
12345678901234567890123456789012123456789012345678901
given one topic. A lengthy list of possible topics can again be found at the
2
12345678901234567890123456789012123456789012345678901
12345678901234567890123456789012123456789012345678901
GRE Web site, specifically at www.gre.org/argutop.html. Most topics are 2
12345678901234567890123456789012123456789012345678901 22
12345678901234567890123456789012123456789012345678901
1 around 100 words in length and are written to resemble editorials, memos, 2
2
letters, and reports from fictional people or businesses.
12345678901234567890123456789012123456789012345678901
2
12345678901234567890123456789012123456789012345678901
2
12345678901234567890123456789012123456789012345678901
Read
a
bunch
of
these
arguments
to
get
a
feel
for
what
they
are
like.
The
2345678901234567890123456789012123456789012345678901
12345678901234567890123456789012123456789012345678901 22
12345678901234567890123456789012123456789012345678901
first thing you should notice is that every argument has a point. Quite 2
12345678901234567890123456789012123456789012345678901
12345678901234567890123456789012123456789012345678901
often, this point is the first sentence, although in other instances it is 2
2
12345678901234567890123456789012123456789012345678901
12345678901234567890123456789012123456789012345678901
farther into the text. The pointwhich can also be called the topic 2
2
12345678901234567890123456789012123456789012345678901
2
12345678901234567890123456789012123456789012345678901
statement,
main
idea
of
the
argument,
or
conclusionis
a
judgment
or
12345678901234567890123456789012123456789012345678901 22
1 belief made by the author of the topic. This point can be something like,
2
12345678901234567890123456789012123456789012345678901
2
12345678901234567890123456789012123456789012345678901
Therefore,
people
should
try
to
reduce
their
intake
of
salmon
if
they
are
12345678901234567890123456789012123456789012345678901 2
12345678901234567890123456789012123456789012345678901
concerned about mercury poisoning or It is obvious that higher salaries 2
2
12345678901234567890123456789012123456789012345678901
2345678901234567890123456789012123456789012345678901
2
12345678901234567890123456789012123456789012345678901
will lead to a higher number of new products.
2
12345678901234567890123456789012123456789012345678901
2
1
2
12345678901234567890123456789012123456789012345678901
2
12345678901234567890123456789012123456789012345678901
Words
like
therefore,
clearly,
and
thus
are
big
signposts
that
scream,
2345678901234567890123456789012123456789012345678901
12345678901234567890123456789012123456789012345678901 22
12345678901234567890123456789012123456789012345678901
2
1 Main point up ahead! These words are often used after a series of
2
12345678901234567890123456789012123456789012345678901
supporting
details,
and
they
act
as
a
verbal
transition
between
the
12345678901234567890123456789012123456789012345678901 2
2345678901234567890123456789012123456789012345678901
2
12345678901234567890123456789012123456789012345678901
supporting details and the main idea supported by the details. You should
2
12345678901234567890123456789012123456789012345678901
2
12345678901234567890123456789012123456789012345678901
be familiar with them, as it is likely you used one of these exact words
12345678901234567890123456789012123456789012345678901 22
1 when restating your main point at the end of the first essay.
2
12345678901234567890123456789012123456789012345678901
2
12345678901234567890123456789012123456789012345678901
2
12345678901234567890123456789012123456789012345678901
12345678901234567890123456789012123456789012345678901
The main point of the argument is something that can be disputed. To help 2
12345678901234567890123456789012123456789012345678901 2
12345678901234567890123456789012123456789012345678901
convince you of the main idea, the author surrounds the main idea with 2
2
12345678901234567890123456789012123456789012345678901
supporting details that back up the initial claim. These supporting details 2
12345678901234567890123456789012123456789012345678901
2
12345678901234567890123456789012123456789012345678901
serve the same function here as the supporting details you created for the 2
12345678901234567890123456789012123456789012345678901
2345678901234567890123456789012123456789012345678901
2
12345678901234567890123456789012123456789012345678901
1 first essay: They provide facts that support the authors claim. Some 2
2
12345678901234567890123456789012123456789012345678901
arguments consist of one supporting idea, which is fleshed out and 2
12345678901234567890123456789012123456789012345678901
12345678901234567890123456789012123456789012345678901
described in detail for the rest of the paragraph. Other arguments make 2
2
12345678901234567890123456789012123456789012345678901
2
12345678901234567890123456789012123456789012345678901
34
different
points,
each
of
which
supports
the
main
point.
If
you
scan
12345678901234567890123456789012123456789012345678901 22
12345678901234567890123456789012123456789012345678901 237
123456789012345678901234567890121234567890123456789012

http://www.xtremepapers.net

Part III: Section Review

123456789012345678901234567890121234567890123456789012
12345678901234567890123456789012123456789012345678901 2
2
12345678901234567890123456789012123456789012345678901
through the sample passages provided online, you can find examples of 2
12345678901234567890123456789012123456789012345678901
2
12345678901234567890123456789012123456789012345678901
both types, as well as many more that fall between these two poles.
2
12345678901234567890123456789012123456789012345678901
12345678901234567890123456789012123456789012345678901 2
12345678901234567890123456789012123456789012345678901
The ability to dissect each argument into a main idea and its supporting 2
2
12345678901234567890123456789012123456789012345678901
2
12345678901234567890123456789012123456789012345678901
details
is
the
first
step
to
doing
well
on
the
second
essay.
If
you
feel
you
2
12345678901234567890123456789012123456789012345678901
2
12345678901234567890123456789012123456789012345678901
need
some
practice
at
this,
take
a
sheet
of
lined
paper
and
write
the
2
12345678901234567890123456789012123456789012345678901
2345678901234567890123456789012123456789012345678901
2
1 following:
2
12345678901234567890123456789012123456789012345678901
2
12345678901234567890123456789012123456789012345678901
2
12345678901234567890123456789012123456789012345678901
The
main
idea
of
this
argument
is:
2
12345678901234567890123456789012123456789012345678901
2345678901234567890123456789012123456789012345678901
2
1
2
12345678901234567890123456789012123456789012345678901
2
12345678901234567890123456789012123456789012345678901
2
12345678901234567890123456789012123456789012345678901
2
12345678901234567890123456789012123456789012345678901
2345678901234567890123456789012123456789012345678901
2
1
2
12345678901234567890123456789012123456789012345678901
The supporting details used to make this claim are:
2
12345678901234567890123456789012123456789012345678901
2
12345678901234567890123456789012123456789012345678901
2
12345678901234567890123456789012123456789012345678901
1.
2345678901234567890123456789012123456789012345678901
12345678901234567890123456789012123456789012345678901 2
12345678901234567890123456789012123456789012345678901 2
12345678901234567890123456789012123456789012345678901 2
12345678901234567890123456789012123456789012345678901 2
12345678901234567890123456789012123456789012345678901 2
2
12345678901234567890123456789012123456789012345678901
2.
2
1
2
12345678901234567890123456789012123456789012345678901
2
12345678901234567890123456789012123456789012345678901
2
12345678901234567890123456789012123456789012345678901
2345678901234567890123456789012123456789012345678901
2
12345678901234567890123456789012123456789012345678901
3.
2
12345678901234567890123456789012123456789012345678901
12345678901234567890123456789012123456789012345678901 2
12345678901234567890123456789012123456789012345678901 2
12345678901234567890123456789012123456789012345678901 2
12345678901234567890123456789012123456789012345678901 2
12345678901234567890123456789012123456789012345678901
Scroll through the GRE argument passages, pick one at random, and place 2
2
12345678901234567890123456789012123456789012345678901
it into the above template. Determine the main idea, and write out that 2
12345678901234567890123456789012123456789012345678901
1 sentence in the space provided. Then list the supporting details that back 2
2
12345678901234567890123456789012123456789012345678901
2
12345678901234567890123456789012123456789012345678901
up the claim. Since some details might be connected, make a note like to 2
12345678901234567890123456789012123456789012345678901
2
12345678901234567890123456789012123456789012345678901
show that two facts are linked. Repeat this exercise several times until you 2
12345678901234567890123456789012123456789012345678901
2345678901234567890123456789012123456789012345678901
12345678901234567890123456789012123456789012345678901
are able to spot a conclusion and its supporting details with little difficulty. 2
2
12345678901234567890123456789012123456789012345678901
1 It should help that you spent the first essay deciding on your own point of 2
2
12345678901234567890123456789012123456789012345678901
2
12345678901234567890123456789012123456789012345678901
view
and
then
backing
it
up
with
supporting
details.
For
the
second
essay,
2345678901234567890123456789012123456789012345678901
12345678901234567890123456789012123456789012345678901 2
2
12345678901234567890123456789012123456789012345678901
1 the main point and supporting details have been given to you. You just 2
2
12345678901234567890123456789012123456789012345678901
have to determine which is which.
2
12345678901234567890123456789012123456789012345678901
2345678901234567890123456789012123456789012345678901
12345678901234567890123456789012123456789012345678901 2
12345678901234567890123456789012123456789012345678901
When youve sliced apart a bunch of essays, look with a skeptical eye at the 2
2
12345678901234567890123456789012123456789012345678901
12345678901234567890123456789012123456789012345678901
conclusions and the supporting details used to prove them. How airtight 2
2
1
12345678901234567890123456789012123456789012345678901
are these arguments? Are any of these arguments going to be eligible for 2
2
12345678901234567890123456789012123456789012345678901
2
12345678901234567890123456789012123456789012345678901
the Nobel prize in logic?
2
12345678901234567890123456789012123456789012345678901
2
12345678901234567890123456789012123456789012345678901
The
answer
is
No.
A
thousand
times,
no!
Every
single
one
of
these
2345678901234567890123456789012123456789012345678901
12345678901234567890123456789012123456789012345678901 2
2
12345678901234567890123456789012123456789012345678901
arguments can be attacked and discredited in one way or another. To 2
12345678901234567890123456789012123456789012345678901
2
12345678901234567890123456789012123456789012345678901
1 succeed on the second essay, your primary mission is to recognize these 2
2345678901234567890123456789012123456789012345678901
2
12345678901234567890123456789012123456789012345678901
2
1 errors and write about them.
2
12345678901234567890123456789012123456789012345678901
12345678901234567890123456789012123456789012345678901 2
2
12345678901234567890123456789012123456789012345678901
12345678901234567890123456789012123456789012345678901 2
12345678901234567890123456789012123456789012345678901 2
12345678901234567890123456789012123456789012345678901 2
2
238 12345678901234567890123456789012123456789012345678901
123456789012345678901234567890121234567890123456789012

www.petersons.com

http://www.xtremepapers.net

Chapter 6: Analytical Writing Review

123456789012345678901234567890121234567890123456789012
12345678901234567890123456789012123456789012345678901 2
2
12345678901234567890123456789012123456789012345678901
2
12345678901234567890123456789012123456789012345678901
Find the Flaw
2
12345678901234567890123456789012123456789012345678901
2
12345678901234567890123456789012123456789012345678901
The
second
essay
is
officially
called
Analyze
an
Argument,
but
you
12345678901234567890123456789012123456789012345678901 2
12345678901234567890123456789012123456789012345678901
should think of it as a game called Find the Flaw. Using this name will 2
2
12345678901234567890123456789012123456789012345678901
12345678901234567890123456789012123456789012345678901
get you in the proper mindset. When you read the argument essay, you 2
2
12345678901234567890123456789012123456789012345678901
12345678901234567890123456789012123456789012345678901
want to be much more critical than you usually are. When doing regular 2
2
12345678901234567890123456789012123456789012345678901
12345678901234567890123456789012123456789012345678901
reading, people often give an author the benefit of the doubt and keep an 2
2
12345678901234567890123456789012123456789012345678901
12345678901234567890123456789012123456789012345678901
open mind that what is being written could be valid. This means you might 2
2
12345678901234567890123456789012123456789012345678901
read an editorial, mentally shrug, and think, Yes, I suppose that could be 2
12345678901234567890123456789012123456789012345678901
2345678901234567890123456789012123456789012345678901
2
1 true.
2
12345678901234567890123456789012123456789012345678901
2
12345678901234567890123456789012123456789012345678901
2
12345678901234567890123456789012123456789012345678901
You
dont
want
to
be
so
kind
on
the
second
essay.
Once
the
clock
starts
2
12345678901234567890123456789012123456789012345678901
2345678901234567890123456789012123456789012345678901
2
1 ticking, your goal is to:
2
12345678901234567890123456789012123456789012345678901
2
12345678901234567890123456789012123456789012345678901
2
12345678901234567890123456789012123456789012345678901
1.
Read
the
argument
2
12345678901234567890123456789012123456789012345678901
2345678901234567890123456789012123456789012345678901
12345678901234567890123456789012123456789012345678901 22
12345678901234567890123456789012123456789012345678901
2. Determine the main point
2
12345678901234567890123456789012123456789012345678901
2
12345678901234567890123456789012123456789012345678901
3. Determine the supporting details
2
12345678901234567890123456789012123456789012345678901
12345678901234567890123456789012123456789012345678901 22
1
4. Brainstorm alternatives that attack either the main idea or a 2
12345678901234567890123456789012123456789012345678901
2
12345678901234567890123456789012123456789012345678901
supporting detail
2
12345678901234567890123456789012123456789012345678901
2345678901234567890123456789012123456789012345678901
12345678901234567890123456789012123456789012345678901 22
12345678901234567890123456789012123456789012345678901
5. Write about these flaws in the argument
2
12345678901234567890123456789012123456789012345678901
12345678901234567890123456789012123456789012345678901 22
12345678901234567890123456789012123456789012345678901
A run-through of these steps can be shown using this sample argument
2
12345678901234567890123456789012123456789012345678901
2
12345678901234567890123456789012123456789012345678901
(which once again has NOT been plagiarized from the official site).
12345678901234567890123456789012123456789012345678901 22
12345678901234567890123456789012123456789012345678901
The following memorandum was written by the President of 2
1
2
12345678901234567890123456789012123456789012345678901
Dextrose Bank.
2
12345678901234567890123456789012123456789012345678901
12345678901234567890123456789012123456789012345678901 2
12345678901234567890123456789012123456789012345678901
The President of Sucrose Bank, our main competitor, 2
2
12345678901234567890123456789012123456789012345678901
2345678901234567890123456789012123456789012345678901
12345678901234567890123456789012123456789012345678901
recently mentioned that over half of Sucroses new 2
2
12345678901234567890123456789012123456789012345678901
2
1
customers
were
people
passing
by
who
simply
walked
in
2
12345678901234567890123456789012123456789012345678901
2
12345678901234567890123456789012123456789012345678901
the
front
door.
With
this
in
mind,
a
large
portion
of
this
2345678901234567890123456789012123456789012345678901
12345678901234567890123456789012123456789012345678901 22
12345678901234567890123456789012123456789012345678901
years marketing budget will be shifted to improving the 2
1
12345678901234567890123456789012123456789012345678901
exterior look of our bank. This will make our bank look 2
2
12345678901234567890123456789012123456789012345678901
2345678901234567890123456789012123456789012345678901
12345678901234567890123456789012123456789012345678901
more attractive to people passing by, and we should be 2
2
12345678901234567890123456789012123456789012345678901
12345678901234567890123456789012123456789012345678901
able to attract new customers the same way Sucrose 2
12345678901234567890123456789012123456789012345678901 22
1
Bank did.
2
12345678901234567890123456789012123456789012345678901
2
12345678901234567890123456789012123456789012345678901
Having read the paragraph, Step 1 is complete. Now its time to determine 2
12345678901234567890123456789012123456789012345678901
2
12345678901234567890123456789012123456789012345678901
what the main point is. Some of you might think the main point is, a large 2
12345678901234567890123456789012123456789012345678901
2345678901234567890123456789012123456789012345678901
2
12345678901234567890123456789012123456789012345678901
portion of this years marketing budget will be shifted to improving the 2
12345678901234567890123456789012123456789012345678901
12345678901234567890123456789012123456789012345678901
exterior look of our bank, since this is the big change at Dextrose Bank. 2
2
12345678901234567890123456789012123456789012345678901
1 However, keep in mind that a main point is a point of view. Its something 2
2345678901234567890123456789012123456789012345678901
12345678901234567890123456789012123456789012345678901 22
1 that a particular person believes, but its not a fact. The statement, A
2
12345678901234567890123456789012123456789012345678901
2
12345678901234567890123456789012123456789012345678901
large
portion
of
this
years
marketing
budget
.
.
.
is
a
fact.
You
cant
agree
2
12345678901234567890123456789012123456789012345678901
2345678901234567890123456789012123456789012345678901
2
1 or disagree, so its not a point of view.
12345678901234567890123456789012123456789012345678901 2
12345678901234567890123456789012123456789012345678901 2
12345678901234567890123456789012123456789012345678901 2 239
123456789012345678901234567890121234567890123456789012

http://www.xtremepapers.net

Part III: Section Review

www.petersons.com

http://www.xtremepapers.net

Tip

123456789012345678901234567890121234567890123456789012
12345678901234567890123456789012123456789012345678901 2
2
12345678901234567890123456789012123456789012345678901
12345678901234567890123456789012123456789012345678901 2
12345678901234567890123456789012123456789012345678901
Again, thinking about the first essay will help you determine the main 2
2
12345678901234567890123456789012123456789012345678901
2
12345678901234567890123456789012123456789012345678901
point
of
the
second
essay.
Recall
how
you
decided
whether
to
agree
or
2
12345678901234567890123456789012123456789012345678901
2345678901234567890123456789012123456789012345678901
1 disagree with the initial statement, since this subjective statement could be 2
2
12345678901234567890123456789012123456789012345678901
12345678901234567890123456789012123456789012345678901
interpreted either way. The main idea of the argument essay will be a 2
2
12345678901234567890123456789012123456789012345678901
12345678901234567890123456789012123456789012345678901
statement you could either agree or disagree with, although you want to 2
2345678901234567890123456789012123456789012345678901
2
1
2
12345678901234567890123456789012123456789012345678901
do more disagreeing than agreeing.
2
12345678901234567890123456789012123456789012345678901
2
12345678901234567890123456789012123456789012345678901
2
12345678901234567890123456789012123456789012345678901
2345678901234567890123456789012123456789012345678901
1 The conclusion reached by the President of Dextrose Bank is that spending 2
2
12345678901234567890123456789012123456789012345678901
12345678901234567890123456789012123456789012345678901
money on the exterior will make our bank look more attractive to people 2
2
12345678901234567890123456789012123456789012345678901
passing by, and we should be able to attract new customers the same way 2
12345678901234567890123456789012123456789012345678901
2345678901234567890123456789012123456789012345678901
1 Sucrose Bank did. This is the Presidents belief, but its not a fact like the 2
2
12345678901234567890123456789012123456789012345678901
2
12345678901234567890123456789012123456789012345678901
temperature is currently 80 degrees.
2
12345678901234567890123456789012123456789012345678901
2
12345678901234567890123456789012123456789012345678901
2345678901234567890123456789012123456789012345678901
12345678901234567890123456789012123456789012345678901
Just for starters, some people might think the bank looks less attractive, 2
2
12345678901234567890123456789012123456789012345678901
2
12345678901234567890123456789012123456789012345678901
and
others
might
like
the
way
the
bank
looks
but
not
go
in
and
start
a
new
12345678901234567890123456789012123456789012345678901 2
12345678901234567890123456789012123456789012345678901
account because of it. Both of these scenarios call the Presidents 2
2
12345678901234567890123456789012123456789012345678901
1 conclusion into question. While the Presidents point may or may not be 2
2
12345678901234567890123456789012123456789012345678901
2
12345678901234567890123456789012123456789012345678901
valid,
it
is
still
the
conclusion,
so
Step
2
is
complete.
Once
you
have
the
2
12345678901234567890123456789012123456789012345678901
2345678901234567890123456789012123456789012345678901
12345678901234567890123456789012123456789012345678901
conclusion, pretty much everything else becomes the supporting details 2
2
12345678901234567890123456789012123456789012345678901
2
12345678901234567890123456789012123456789012345678901
used
to
justify
the
claim,
so
Step
3
falls
into
place
as
well.
12345678901234567890123456789012123456789012345678901 2
12345678901234567890123456789012123456789012345678901 2
12345678901234567890123456789012123456789012345678901
Reading the argument and determining whats what should take no more 2
2
12345678901234567890123456789012123456789012345678901
than 3 minutes, leaving you with 27 minutes for the last two steps. With 2
12345678901234567890123456789012123456789012345678901
2
12345678901234567890123456789012123456789012345678901
1 any essay (even a timed one), its always a good idea to do a bit of 2
2345678901234567890123456789012123456789012345678901
2
12345678901234567890123456789012123456789012345678901
brainstorming before you start writing. This essays no exception, so spend 2
12345678901234567890123456789012123456789012345678901
1 about 35 minutes coming up with ways to show why the Presidents 2
12345678901234567890123456789012123456789012345678901 2
2
12345678901234567890123456789012123456789012345678901
conclusion might not hold true.
2
12345678901234567890123456789012123456789012345678901
2
12345678901234567890123456789012123456789012345678901
2
12345678901234567890123456789012123456789012345678901
You
wouldnt
have
to
write
out
every
alternative.
Just
take
quick
notes
to
2
12345678901234567890123456789012123456789012345678901
2
12345678901234567890123456789012123456789012345678901
remind
yourself.
Once
the
clock
reads
23
minutes
to
go,
you
should
stop
2345678901234567890123456789012123456789012345678901
12345678901234567890123456789012123456789012345678901 2
2
12345678901234567890123456789012123456789012345678901
1 brainstorming and start writing. Your first sentence can be something like, 2
12345678901234567890123456789012123456789012345678901
Although [Main Point . . . Blah Blah Blah] has some validity, it does not 2
2
12345678901234567890123456789012123456789012345678901
2345678901234567890123456789012123456789012345678901
12345678901234567890123456789012123456789012345678901
hold up to intense scrutiny. There are several ways that this conclusion can 2
2
12345678901234567890123456789012123456789012345678901
12345678901234567890123456789012123456789012345678901
be proven wrong, such as. . . . Then you put your essay-writing cap on 2
12345678901234567890123456789012123456789012345678901 2
2
1 and light the sucker up for all its flaws.
2
12345678901234567890123456789012123456789012345678901
2
12345678901234567890123456789012123456789012345678901
To do this, you need to come up with alternatives to what is assumed in the 2
12345678901234567890123456789012123456789012345678901
2
12345678901234567890123456789012123456789012345678901
text. These alternatives require a little bit of creativity on your part, but 2
12345678901234567890123456789012123456789012345678901
2345678901234567890123456789012123456789012345678901
2
12345678901234567890123456789012123456789012345678901
once you practice on some sample passages, youll get the hang of it. These 2
12345678901234567890123456789012123456789012345678901
12345678901234567890123456789012123456789012345678901
alternativesyou can call them alternative reasons, outcomes, explana- 2
2
12345678901234567890123456789012123456789012345678901
1 tions, scenarios, and possibilitiesare all designed to undercut a specific 2
2345678901234567890123456789012123456789012345678901
12345678901234567890123456789012123456789012345678901 2
1 part of the original text. Look at the previous chart that shows the 2
2
12345678901234567890123456789012123456789012345678901
2
12345678901234567890123456789012123456789012345678901
conclusion.
The
bank
president
assumes
that
the
new
look
will
make
our
2
12345678901234567890123456789012123456789012345678901
2345678901234567890123456789012123456789012345678901
1 bank look more attractive, so Alternative #1 is created by simply stating 2
12345678901234567890123456789012123456789012345678901 2
12345678901234567890123456789012123456789012345678901 2
2
240 12345678901234567890123456789012123456789012345678901
123456789012345678901234567890121234567890123456789012

Chapter 6: Analytical Writing Review

123456789012345678901234567890121234567890123456789012
12345678901234567890123456789012123456789012345678901 2
2
12345678901234567890123456789012123456789012345678901
12345678901234567890123456789012123456789012345678901 2
2
12345678901234567890123456789012123456789012345678901
Finding Flaws in the Argument
2
12345678901234567890123456789012123456789012345678901
12345678901234567890123456789012123456789012345678901 2
2
12345678901234567890123456789012123456789012345678901
2
12345678901234567890123456789012123456789012345678901
Conclusion: This will make our bank look more attractive to people
2
12345678901234567890123456789012123456789012345678901
2
12345678901234567890123456789012123456789012345678901
passing by, and we should be able to attract new customers the same
2
12345678901234567890123456789012123456789012345678901
2
12345678901234567890123456789012123456789012345678901
way Sucrose Bank did.
2345678901234567890123456789012123456789012345678901
2
1
2
12345678901234567890123456789012123456789012345678901
Alternative #1: What if people dont like the new look? This
2
12345678901234567890123456789012123456789012345678901
2
12345678901234567890123456789012123456789012345678901
would
discourage
them
from
entering,
the
exact
opposite
of
what
2
12345678901234567890123456789012123456789012345678901
2345678901234567890123456789012123456789012345678901
2
1
is wanted.
2
12345678901234567890123456789012123456789012345678901
2
12345678901234567890123456789012123456789012345678901
2
12345678901234567890123456789012123456789012345678901
Alternative
#2:
What
if
exterior
appearance
had
nothing
to
do
2
12345678901234567890123456789012123456789012345678901
2345678901234567890123456789012123456789012345678901
2
1
with Sucroses success with walk-ins? If Sucrose was located near
2
12345678901234567890123456789012123456789012345678901
2
12345678901234567890123456789012123456789012345678901
a
busy,
trendy
supermarket,
people
might
walk
in
and
start
an
2
12345678901234567890123456789012123456789012345678901
2
12345678901234567890123456789012123456789012345678901
account
there
because
it
was
convenient.
2345678901234567890123456789012123456789012345678901
12345678901234567890123456789012123456789012345678901 22
12345678901234567890123456789012123456789012345678901 2
12345678901234567890123456789012123456789012345678901
Supporting Detail: With this in mind, a large portion of this years
2
12345678901234567890123456789012123456789012345678901
marketing budget will be shifted to improving the exterior look of
2
12345678901234567890123456789012123456789012345678901
2
12345678901234567890123456789012123456789012345678901
our bank.
2
1
2
12345678901234567890123456789012123456789012345678901
2
12345678901234567890123456789012123456789012345678901
Alternative
#3:
What
if
Dextrose
has
had
success
with
radio
and
2
12345678901234567890123456789012123456789012345678901
2345678901234567890123456789012123456789012345678901
2
12345678901234567890123456789012123456789012345678901
print ads? Taking money in the budget away from these
2
12345678901234567890123456789012123456789012345678901
2
12345678901234567890123456789012123456789012345678901
successful
marketing
strategies
might
hurt
the
overall
number
of
12345678901234567890123456789012123456789012345678901 22
12345678901234567890123456789012123456789012345678901
new customers. Walk-ins might increase, but they wouldnt
2
12345678901234567890123456789012123456789012345678901
2
12345678901234567890123456789012123456789012345678901
counteract
the
number
of
customers
lost
by
abandoning
or
12345678901234567890123456789012123456789012345678901 22
12345678901234567890123456789012123456789012345678901
cutting back on the proven methods of radio and print ads.
2
1
2
12345678901234567890123456789012123456789012345678901
2
12345678901234567890123456789012123456789012345678901
Alternative #4: What if there are better ways to spend money and
12345678901234567890123456789012123456789012345678901 2
2
12345678901234567890123456789012123456789012345678901
attract walk ins? Dextrose Bank could leave the exterior as is, but
2
12345678901234567890123456789012123456789012345678901
2345678901234567890123456789012123456789012345678901
2
12345678901234567890123456789012123456789012345678901
throw a benefit concert featuring local musicians. This might
2
12345678901234567890123456789012123456789012345678901
2
generate a large crowd of people who then sign up as new
1
2
12345678901234567890123456789012123456789012345678901
customers.
2
12345678901234567890123456789012123456789012345678901
2345678901234567890123456789012123456789012345678901
12345678901234567890123456789012123456789012345678901 22
12345678901234567890123456789012123456789012345678901
Supporting Detail: The President of Sucrose Bank, our main
2
1
2
12345678901234567890123456789012123456789012345678901
competitor,
recently
mentioned
that
over
half
of
Sucroses
new
12345678901234567890123456789012123456789012345678901 2
2345678901234567890123456789012123456789012345678901
2
12345678901234567890123456789012123456789012345678901
customers were people passing by who simply walked in the front
2
12345678901234567890123456789012123456789012345678901
2
12345678901234567890123456789012123456789012345678901
door.
12345678901234567890123456789012123456789012345678901 22
1
2
12345678901234567890123456789012123456789012345678901
Alternative #5: Putting it bluntly, what if the President of Sucrose
2
12345678901234567890123456789012123456789012345678901
2
12345678901234567890123456789012123456789012345678901
Bank
is
lying?
Suppose
the
Sucrose
Bank
president
made
those
2
12345678901234567890123456789012123456789012345678901
2
12345678901234567890123456789012123456789012345678901
remarks knowing that his/her counterpart at Dextrose Bank
2345678901234567890123456789012123456789012345678901
12345678901234567890123456789012123456789012345678901 22
12345678901234567890123456789012123456789012345678901
would react strongly to them? If this is the case, then the Dextrose
2
12345678901234567890123456789012123456789012345678901
2
12345678901234567890123456789012123456789012345678901
President has been played like a violin, and large amounts of
2
1
2345678901234567890123456789012123456789012345678901
2
Dextrose Bank marketing money are being spent to counter a
12345678901234567890123456789012123456789012345678901
2
1
claim that is not even true.
2
12345678901234567890123456789012123456789012345678901
12345678901234567890123456789012123456789012345678901 2
2
12345678901234567890123456789012123456789012345678901
12345678901234567890123456789012123456789012345678901 2
12345678901234567890123456789012123456789012345678901 2
12345678901234567890123456789012123456789012345678901 2
12345678901234567890123456789012123456789012345678901 2 241
123456789012345678901234567890121234567890123456789012

http://www.xtremepapers.net

Part III: Section Review

www.petersons.com

http://www.xtremepapers.net

Alert!

123456789012345678901234567890121234567890123456789012
12345678901234567890123456789012123456789012345678901 2
2
12345678901234567890123456789012123456789012345678901
a different outcome. To flesh out Alternative #1, you could have something 2
12345678901234567890123456789012123456789012345678901
2
12345678901234567890123456789012123456789012345678901
like the following paragraph.
2
12345678901234567890123456789012123456789012345678901
12345678901234567890123456789012123456789012345678901 2
2
12345678901234567890123456789012123456789012345678901
2
12345678901234567890123456789012123456789012345678901
(A) The President of Dextrose Bank believes that the new
2
12345678901234567890123456789012123456789012345678901
2
12345678901234567890123456789012123456789012345678901
look
will
make
the
bank
more
attractive
to
passers
by,
but
2
12345678901234567890123456789012123456789012345678901
2
12345678901234567890123456789012123456789012345678901
(B)
Some
people
might
not
like
the
this
is
not
necessarily
true.
2345678901234567890123456789012123456789012345678901
2
1
2
12345678901234567890123456789012123456789012345678901
(C)
For
new
decor,
and
avoid
the
bank
for
that
reason.
2
12345678901234567890123456789012123456789012345678901
2
12345678901234567890123456789012123456789012345678901
example,
suppose
the
bank
spends
a
lot
of
money
and
plants
a
2
12345678901234567890123456789012123456789012345678901
2345678901234567890123456789012123456789012345678901
2
1
large number of flowers and bushes around the parking lot.
2
12345678901234567890123456789012123456789012345678901
2
12345678901234567890123456789012123456789012345678901
(D) However, the state is in the midst of a severe drought.
2
12345678901234567890123456789012123456789012345678901
2
12345678901234567890123456789012123456789012345678901
(E) Some people might think the bank looks nice, but others
2345678901234567890123456789012123456789012345678901
2
1
2
12345678901234567890123456789012123456789012345678901
would believe that the bank is foolishly wasting water and
2
12345678901234567890123456789012123456789012345678901
money during a drought. (F) These people would be turned
2
12345678901234567890123456789012123456789012345678901
2
12345678901234567890123456789012123456789012345678901
off
by
the
expensive
remodeling,
considering
it
wasteful
2345678901234567890123456789012123456789012345678901
12345678901234567890123456789012123456789012345678901 2
2
12345678901234567890123456789012123456789012345678901
and boorish.
2
12345678901234567890123456789012123456789012345678901
12345678901234567890123456789012123456789012345678901 2
12345678901234567890123456789012123456789012345678901 2
12345678901234567890123456789012123456789012345678901 2
1 Sentence A is the set-up sentence. It clearly states which part of the 2
2
12345678901234567890123456789012123456789012345678901
2
12345678901234567890123456789012123456789012345678901
argument
you
are
going
to
attack.
As
you
might
expect,
starting
every
2
12345678901234567890123456789012123456789012345678901
2345678901234567890123456789012123456789012345678901
12345678901234567890123456789012123456789012345678901
paragraph with a simple set-up sentence is a good game plan. Sentence B 2
2
12345678901234567890123456789012123456789012345678901
12345678901234567890123456789012123456789012345678901
states an alternative outcome, showing how this alternative leads to 2
12345678901234567890123456789012123456789012345678901 2
12345678901234567890123456789012123456789012345678901
actions opposite of what the bank president assumes will happen. 2
2
12345678901234567890123456789012123456789012345678901
12345678901234567890123456789012123456789012345678901
Sentences C and D are where you have to get a little creative. Basically, you 2
2
12345678901234567890123456789012123456789012345678901
need to think of a reason why people would not like a new exterior for a 2
12345678901234567890123456789012123456789012345678901
1 bank. The sample above goes the ecological conservationist route, 2
2
12345678901234567890123456789012123456789012345678901
2
12345678901234567890123456789012123456789012345678901
setting up a mythical drought that the cads at the bank care nothing about. 2
12345678901234567890123456789012123456789012345678901
2
12345678901234567890123456789012123456789012345678901
Sentences E and F explain why the eco-conservationists are now less likely 2
12345678901234567890123456789012123456789012345678901
2345678901234567890123456789012123456789012345678901
12345678901234567890123456789012123456789012345678901
to open an account at Dextrose Bank now that the exterior has been 2
2
12345678901234567890123456789012123456789012345678901
2
1 expensively remodeled.
2
12345678901234567890123456789012123456789012345678901
2
12345678901234567890123456789012123456789012345678901
2345678901234567890123456789012123456789012345678901
12345678901234567890123456789012123456789012345678901 2
2
12345678901234567890123456789012123456789012345678901
1 The eco-conservationist route is not the only explanation you can give. 2
12345678901234567890123456789012123456789012345678901
You could also say, Suppose the renovations to the exterior require large 2
2
12345678901234567890123456789012123456789012345678901
2345678901234567890123456789012123456789012345678901
12345678901234567890123456789012123456789012345678901
parts of the bank building (including the bank sign) to be covered in 2
2
12345678901234567890123456789012123456789012345678901
2
12345678901234567890123456789012123456789012345678901
scaffolding
and
other
construction
materials.
This
might
make
it
hard
for
12345678901234567890123456789012123456789012345678901 2
1 people new to the area to realize that the building under construction was 2
2
12345678901234567890123456789012123456789012345678901
2
12345678901234567890123456789012123456789012345678901
a
bank.
Since
newly
moved
people
are
more
likely
to
need
a
new
account,
2
12345678901234567890123456789012123456789012345678901
2
12345678901234567890123456789012123456789012345678901
the
work
on
the
exterior
would
hurt
Dextroses
ability
to
sign
these
12345678901234567890123456789012123456789012345678901 2
2345678901234567890123456789012123456789012345678901
12345678901234567890123456789012123456789012345678901
people up. This isnt perfect, but perfection isnt required for a 2
2
12345678901234567890123456789012123456789012345678901
2
12345678901234567890123456789012123456789012345678901
30-minute
essay.
You
just
need
to
show
that
you
can
break
down
an
12345678901234567890123456789012123456789012345678901 2
2
1 argument and come up with alternatives.
2
12345678901234567890123456789012123456789012345678901
12345678901234567890123456789012123456789012345678901 2
2
12345678901234567890123456789012123456789012345678901
12345678901234567890123456789012123456789012345678901
The essay does not require you to go out and actually find an 2
2
12345678901234567890123456789012123456789012345678901
12345678901234567890123456789012123456789012345678901
eco-conservationist who supports your opinion. You do not need to have a 2
2
12345678901234567890123456789012123456789012345678901
12345678901234567890123456789012123456789012345678901
sentence that shows the national membership figures for the Sierra Club. 2
2
242 12345678901234567890123456789012123456789012345678901
123456789012345678901234567890121234567890123456789012

Chapter 6: Analytical Writing Review

123456789012345678901234567890121234567890123456789012
12345678901234567890123456789012123456789012345678901 2
2
12345678901234567890123456789012123456789012345678901
All you have to do is come up with a plausible (theres that word again!) 2
12345678901234567890123456789012123456789012345678901
2
12345678901234567890123456789012123456789012345678901
alternative that shows how the initial point might be wrong. Once you 2
12345678901234567890123456789012123456789012345678901
2
12345678901234567890123456789012123456789012345678901
explain your alternative, you have done what is needed for the second 2
12345678901234567890123456789012123456789012345678901
2345678901234567890123456789012123456789012345678901
1 essay. Now you need some more of it, so its time to start another 2
2
12345678901234567890123456789012123456789012345678901
2
12345678901234567890123456789012123456789012345678901
paragraph
and
attack
something
else
from
the
original
argument.
2
12345678901234567890123456789012123456789012345678901
2
12345678901234567890123456789012123456789012345678901
2345678901234567890123456789012123456789012345678901
1 If you are wondering why the emphasis is on finding flaws in the 2
2
12345678901234567890123456789012123456789012345678901
2
12345678901234567890123456789012123456789012345678901
argument,
the
answer
is
simple.
The
GRE
readers
arent
going
to
be
kind
2
12345678901234567890123456789012123456789012345678901
2
12345678901234567890123456789012123456789012345678901
to
an
essay
that
states,
After
looking
over
the
argument,
everything
the
2345678901234567890123456789012123456789012345678901
2
1
2
12345678901234567890123456789012123456789012345678901
speaker
claims
is
true.
Anyone
who
doesnt
understand
the
argument
can
2
12345678901234567890123456789012123456789012345678901
2
12345678901234567890123456789012123456789012345678901
write
this,
so
it
doesnt
really
show
that
you
did
any
thinking.
The
readers
2
12345678901234567890123456789012123456789012345678901
2345678901234567890123456789012123456789012345678901
1 want evidence of some mental work on your part. This is why you must 2
2
12345678901234567890123456789012123456789012345678901
12345678901234567890123456789012123456789012345678901
attack the argument, pulling it apart and coming up with alternative 2
2
12345678901234567890123456789012123456789012345678901
2
12345678901234567890123456789012123456789012345678901
possibilities that subvert the main point.
2345678901234567890123456789012123456789012345678901
12345678901234567890123456789012123456789012345678901 22
12345678901234567890123456789012123456789012345678901
However, you can recycle the old I understand both sides of the issue, but 2
12345678901234567890123456789012123456789012345678901
2
12345678901234567890123456789012123456789012345678901
theyre wrong concept from the first essay. This is done below with 2
12345678901234567890123456789012123456789012345678901
2
12345678901234567890123456789012123456789012345678901
2
1 Alternative #3.
2
12345678901234567890123456789012123456789012345678901
2
12345678901234567890123456789012123456789012345678901
2
12345678901234567890123456789012123456789012345678901
The
Dextrose
Bank
President
has
decided
to
shift
a
(G)
2345678901234567890123456789012123456789012345678901
12345678901234567890123456789012123456789012345678901 22
12345678901234567890123456789012123456789012345678901
large portion of this years marketing budget . . . to improving
2
12345678901234567890123456789012123456789012345678901
12345678901234567890123456789012123456789012345678901
(H) A new exterior might indeed 2
the
exterior
look
of
our
bank.
12345678901234567890123456789012123456789012345678901 22
12345678901234567890123456789012123456789012345678901
bring in some new customers, since a bold new exterior might
2
12345678901234567890123456789012123456789012345678901
2
12345678901234567890123456789012123456789012345678901
(I) However, the shift in marketing
increase
walk-in
interest.
12345678901234567890123456789012123456789012345678901 22
1
money might not be the best for the overall number of new
2
12345678901234567890123456789012123456789012345678901
customers. (J) Suppose the shift in money means that the bank 2
12345678901234567890123456789012123456789012345678901
2
12345678901234567890123456789012123456789012345678901
can no longer do many print and radio ads? (K) If these ads
2
12345678901234567890123456789012123456789012345678901
2
12345678901234567890123456789012123456789012345678901
were
very
successful,
their
decline
might
mean
fewer
new
2345678901234567890123456789012123456789012345678901
12345678901234567890123456789012123456789012345678901 22
12345678901234567890123456789012123456789012345678901
customers come in after reading or hearing an ad. (L) This
2
1
2
12345678901234567890123456789012123456789012345678901
means
that
the
number
of
walk-in
new
customers
increases,
2
12345678901234567890123456789012123456789012345678901
2345678901234567890123456789012123456789012345678901
2
12345678901234567890123456789012123456789012345678901
but the number of radio and print new customers drops.
2
12345678901234567890123456789012123456789012345678901
2
1
(M)
Overall,
this
might
lead
to
a
decline
in
the
total
number
12345678901234567890123456789012123456789012345678901 2
2
12345678901234567890123456789012123456789012345678901
of new customers, and Dextrose Bank will be worse off.
2
12345678901234567890123456789012123456789012345678901
2
12345678901234567890123456789012123456789012345678901
2
12345678901234567890123456789012123456789012345678901
2
12345678901234567890123456789012123456789012345678901
12345678901234567890123456789012123456789012345678901
Sentence G is a set-up sentence that uses quotes to show exactly what part 2
2345678901234567890123456789012123456789012345678901
2
12345678901234567890123456789012123456789012345678901
of the argument is going to be scrutinized. Sentence H shows, Hey, dude, 2
12345678901234567890123456789012123456789012345678901
2
12345678901234567890123456789012123456789012345678901
I can totally see where youre coming from. The other side of the issue has 2
12345678901234567890123456789012123456789012345678901
1 now been given a little credit. The rest of the paragraph proceeds to show 2
2
12345678901234567890123456789012123456789012345678901
2
12345678901234567890123456789012123456789012345678901
why this point of view is flawed. Sentence I is a transition sentence, as you 2
12345678901234567890123456789012123456789012345678901
12345678901234567890123456789012123456789012345678901
go from agreeing to disagreeing. The next four sentences follow the same 2
2
12345678901234567890123456789012123456789012345678901
2345678901234567890123456789012123456789012345678901
2
12345678901234567890123456789012123456789012345678901
1 blueprint as the first paragraph. Sentences J and K create a hypothetical 2
12345678901234567890123456789012123456789012345678901
situationwhich you can totally make up so long as its somewhat believ- 2
2
12345678901234567890123456789012123456789012345678901
2
12345678901234567890123456789012123456789012345678901
ableand
Sentences
L
and
M
show
how
this
hypothetical
situation
will
2345678901234567890123456789012123456789012345678901
2
1
2
12345678901234567890123456789012123456789012345678901
lead
to
an
outcome
exactly
opposite
of
what
the
Dextrose
Bank
President
12345678901234567890123456789012123456789012345678901 22
12345678901234567890123456789012123456789012345678901 243
123456789012345678901234567890121234567890123456789012

http://www.xtremepapers.net

Part III: Section Review

123456789012345678901234567890121234567890123456789012
12345678901234567890123456789012123456789012345678901 2
2
12345678901234567890123456789012123456789012345678901
assumes will happen. Not every sentence in this second paragraph is stellar, 2
12345678901234567890123456789012123456789012345678901
2
12345678901234567890123456789012123456789012345678901
and if you had time, you might not want to use both radio and print ads. 2
12345678901234567890123456789012123456789012345678901
2
12345678901234567890123456789012123456789012345678901
Using only one would simplify the paragraph and still get the point across. 2
12345678901234567890123456789012123456789012345678901
2345678901234567890123456789012123456789012345678901
1 But you dont have time, so it doesnt matter. The second paragraph is fine 2
2
12345678901234567890123456789012123456789012345678901
2
12345678901234567890123456789012123456789012345678901
as
it
is
for
a
30-minute
Analyze
the
Argument
essay.
2
12345678901234567890123456789012123456789012345678901
2
12345678901234567890123456789012123456789012345678901
2345678901234567890123456789012123456789012345678901
1 After two paragraphs, you might not have much time left. Following the 2
2
12345678901234567890123456789012123456789012345678901
2
12345678901234567890123456789012123456789012345678901
template
of
the
first
essay,
write
a
brief
closing
and
then
use
any
remaining
2
12345678901234567890123456789012123456789012345678901
2
12345678901234567890123456789012123456789012345678901
time
to
hammer
out
more
alternatives.
Your
closing
might
be
something
2345678901234567890123456789012123456789012345678901
2
1
2
12345678901234567890123456789012123456789012345678901
like
the
following:
2
12345678901234567890123456789012123456789012345678901
2
12345678901234567890123456789012123456789012345678901
2
12345678901234567890123456789012123456789012345678901
The
Dextrose
Bank
Presidents
plan
to
gain
more
new
walk-in
2345678901234567890123456789012123456789012345678901
2
1
2
12345678901234567890123456789012123456789012345678901
customers might work. However, if it doesnt, it could be
2
12345678901234567890123456789012123456789012345678901
2
12345678901234567890123456789012123456789012345678901
because
of
one
of
the
reasons
listed
above,
since
the
2
12345678901234567890123456789012123456789012345678901
2345678901234567890123456789012123456789012345678901
2
12345678901234567890123456789012123456789012345678901
Presidents plan is by no means foolproof.
2
12345678901234567890123456789012123456789012345678901
12345678901234567890123456789012123456789012345678901 2
12345678901234567890123456789012123456789012345678901 2
12345678901234567890123456789012123456789012345678901 2
2
12345678901234567890123456789012123456789012345678901
1 The above closes out the essay nicely. The vague phrase it could be 2
12345678901234567890123456789012123456789012345678901
because of one of the reasons listed works quite well, because you 2
2
12345678901234567890123456789012123456789012345678901
12345678901234567890123456789012123456789012345678901
wouldnt be expected to list every one of the reasons in the closing. You 2
2345678901234567890123456789012123456789012345678901
12345678901234567890123456789012123456789012345678901 2
12345678901234567890123456789012123456789012345678901
can now use all the time remaining to add as many reasons as you can type. 2
2
12345678901234567890123456789012123456789012345678901
2
12345678901234567890123456789012123456789012345678901
When
the
clock
starts
flashing
3
minutes,
finish
up
the
sentence
you
are
12345678901234567890123456789012123456789012345678901 2
12345678901234567890123456789012123456789012345678901
writing and then look over the essay for any grammatical mistakes. 2
2
12345678901234567890123456789012123456789012345678901
2
12345678901234567890123456789012123456789012345678901
Correct
any
mistakes
you
can
find
until
the
clock
winds
down.
Once
the
12345678901234567890123456789012123456789012345678901 2
1 time is up, the essay will vanish, and the Analytical portion of the GRE will 2
2
12345678901234567890123456789012123456789012345678901
2
12345678901234567890123456789012123456789012345678901
be
over.
12345678901234567890123456789012123456789012345678901 2
12345678901234567890123456789012123456789012345678901 2
2
12345678901234567890123456789012123456789012345678901
2
12345678901234567890123456789012123456789012345678901
2
12345678901234567890123456789012123456789012345678901
2
12345678901234567890123456789012123456789012345678901
. . . And One More Thing
2
12345678901234567890123456789012123456789012345678901
12345678901234567890123456789012123456789012345678901
The following words were used in this chapter. They are all college-level 2
2345678901234567890123456789012123456789012345678901
12345678901234567890123456789012123456789012345678901 2
2
12345678901234567890123456789012123456789012345678901
1 vocabulary words that might appear on the Verbal portion of the GRE. If 2
12345678901234567890123456789012123456789012345678901
you didnt know the meaning of these words, add them to your vocabulary 2
2
12345678901234567890123456789012123456789012345678901
2345678901234567890123456789012123456789012345678901
lists now. The best way to improve your vocabulary is to learn new words 2
12345678901234567890123456789012123456789012345678901
2
12345678901234567890123456789012123456789012345678901
in the context of reading, so find the definitions of all the words below and 2
12345678901234567890123456789012123456789012345678901
2
12345678901234567890123456789012123456789012345678901
2
1 then look back and find them throughout the chapter:
2345678901234567890123456789012123456789012345678901
12345678901234567890123456789012123456789012345678901 2
12345678901234567890123456789012123456789012345678901 2
2
12345678901234567890123456789012123456789012345678901
cad
proverbial
2
12345678901234567890123456789012123456789012345678901
2
1
cantankerous
renaissance
2345678901234567890123456789012123456789012345678901
12345678901234567890123456789012123456789012345678901 2
2
12345678901234567890123456789012123456789012345678901
endeavor
rubric
2
12345678901234567890123456789012123456789012345678901
2
12345678901234567890123456789012123456789012345678901
interminable
scrutiny
(three
different
times!)
2
1
2345678901234567890123456789012123456789012345678901
2
12345678901234567890123456789012123456789012345678901
loquacious
sophistic
2
1
2
12345678901234567890123456789012123456789012345678901
moniker
12345678901234567890123456789012123456789012345678901 2
2
12345678901234567890123456789012123456789012345678901
12345678901234567890123456789012123456789012345678901 2
12345678901234567890123456789012123456789012345678901 2
12345678901234567890123456789012123456789012345678901 2
2
244 12345678901234567890123456789012123456789012345678901
123456789012345678901234567890121234567890123456789012

www.petersons.com

http://www.xtremepapers.net

Take It to
the Next Level
123456789012345678901234567890121234567890123456789012
2
12345678901234567890123456789012123456789012345678901
2
12345678901234567890123456789012123456789012345678901
2345678901234567890123456789012123456789012345678901
2
1
Acing
the Essays
2
12345678901234567890123456789012123456789012345678901
2
12345678901234567890123456789012123456789012345678901
The
fundamentals
of
sound
essay
writing
are
covered
earlier
in
this
2
12345678901234567890123456789012123456789012345678901
2
12345678901234567890123456789012123456789012345678901
chapter.
If
you
follow
those
basic
stepsclearly
stated
main
idea
in
first
2345678901234567890123456789012123456789012345678901
12345678901234567890123456789012123456789012345678901 22
12345678901234567890123456789012123456789012345678901
paragraph of first essay, finding the flaw in the argument for the second 2
12345678901234567890123456789012123456789012345678901
12345678901234567890123456789012123456789012345678901
essay, and so forthyou should get a decent score on the Analytical 2
2
12345678901234567890123456789012123456789012345678901
2
12345678901234567890123456789012123456789012345678901
1 portion of the GRE. However, if you are an English major or someone 2
12345678901234567890123456789012123456789012345678901
shooting to get into a highly competitive program that stresses writing 2
2
12345678901234567890123456789012123456789012345678901
12345678901234567890123456789012123456789012345678901
skills, a decent score might not be all that you want. Anything less then a 5 2
2345678901234567890123456789012123456789012345678901
12345678901234567890123456789012123456789012345678901 22
12345678901234567890123456789012123456789012345678901
might not be enough; you want a 5.5 or ideally a 6.
2
12345678901234567890123456789012123456789012345678901
2
12345678901234567890123456789012123456789012345678901
Theres no way to guarantee you a perfect score. Essays are just too 2
12345678901234567890123456789012123456789012345678901
2
12345678901234567890123456789012123456789012345678901
subjective, and one readers idea of an excellent essay might differ from 2
12345678901234567890123456789012123456789012345678901
2
12345678901234567890123456789012123456789012345678901
what another reader thinks a perfect essay should look like. But there are a 2
12345678901234567890123456789012123456789012345678901
1 host of different strategies you can use to transform a good essay into a 2
2
12345678901234567890123456789012123456789012345678901
2
12345678901234567890123456789012123456789012345678901
superior essay, and these strategies will be covered in this chapter.
2
12345678901234567890123456789012123456789012345678901
12345678901234567890123456789012123456789012345678901 2
2
12345678901234567890123456789012123456789012345678901
2
12345678901234567890123456789012123456789012345678901
2
12345678901234567890123456789012123456789012345678901
2
12345678901234567890123456789012123456789012345678901
Mixing
It
up
on
the
First
Essay
2
12345678901234567890123456789012123456789012345678901
2
12345678901234567890123456789012123456789012345678901
Since
the
first
essay
is
longer
and
requires
more
mental
work
than
the
2345678901234567890123456789012123456789012345678901
12345678901234567890123456789012123456789012345678901 22
12345678901234567890123456789012123456789012345678901
1 second, it stands to reason that this essay affects your overall Analytical 2
12345678901234567890123456789012123456789012345678901
score more than the second essay. The first essay is also more open-ended, 2
2
12345678901234567890123456789012123456789012345678901
2345678901234567890123456789012123456789012345678901
12345678901234567890123456789012123456789012345678901
which gives you a greater range of creativity on this essay than on the 2
2
12345678901234567890123456789012123456789012345678901
2
12345678901234567890123456789012123456789012345678901
second.
For
the
second
essay,
the
goal
is
to
analyze
the
argument,
and
12345678901234567890123456789012123456789012345678901 22
1 this limits your responses since you must stay focused on the argument
2
12345678901234567890123456789012123456789012345678901
12345678901234567890123456789012123456789012345678901
given. This set-up is good news for you, because it means that you can 2
2
12345678901234567890123456789012123456789012345678901
12345678901234567890123456789012123456789012345678901
demonstrate your snazzy writing skills on the essay that counts more 2
12345678901234567890123456789012123456789012345678901 2
2
12345678901234567890123456789012123456789012345678901
toward your overall score.
2
12345678901234567890123456789012123456789012345678901
2
12345678901234567890123456789012123456789012345678901
Before delving into specific techniques to improve your essay, the general 2
12345678901234567890123456789012123456789012345678901
2
12345678901234567890123456789012123456789012345678901
theme you should shoot for is to write a standard essay, but with frills. 2
12345678901234567890123456789012123456789012345678901
2
12345678901234567890123456789012123456789012345678901
Think of a perfect score of 6 as a 4 1 2. The 4 comes from having all the 2
12345678901234567890123456789012123456789012345678901
12345678901234567890123456789012123456789012345678901
basic requirements needed in an essay, such as a main idea supported by 2
2
12345678901234567890123456789012123456789012345678901
2345678901234567890123456789012123456789012345678901
1 details. Every essay needs the basics, and yours is no exception. Yet most 2
12345678901234567890123456789012123456789012345678901 2
12345678901234567890123456789012123456789012345678901 2
12345678901234567890123456789012123456789012345678901 2
123456789012345678901234567890121234567890123456789012
245

http://www.xtremepapers.net

Part III: Section Review

www.petersons.com

http://www.xtremepapers.net

Note

123456789012345678901234567890121234567890123456789012
12345678901234567890123456789012123456789012345678901 2
2
12345678901234567890123456789012123456789012345678901
students are familiar with the basic essay format, so just writing a standard 2
12345678901234567890123456789012123456789012345678901
2
12345678901234567890123456789012123456789012345678901
essay might not be enough to get you a 6. Thats where the 12 idea 2
12345678901234567890123456789012123456789012345678901
2
12345678901234567890123456789012123456789012345678901
comes in. The extra 2 points comes from distinguishing your essay in some 2
12345678901234567890123456789012123456789012345678901
2345678901234567890123456789012123456789012345678901
1 way. Your overall goal is to make your essay different in a way that shows 2
2
12345678901234567890123456789012123456789012345678901
2
12345678901234567890123456789012123456789012345678901
you
are
capable
of
writing
more
than
a
standard
essay.
2
12345678901234567890123456789012123456789012345678901
2
12345678901234567890123456789012123456789012345678901
2345678901234567890123456789012123456789012345678901
2
1
2
12345678901234567890123456789012123456789012345678901
The 4 1 2 concept seems to imply that simply writing a basic essay will 2
12345678901234567890123456789012123456789012345678901
2
12345678901234567890123456789012123456789012345678901
never get you a score higher than a 4. This is not its purpose, nor is the 2
12345678901234567890123456789012123456789012345678901
2345678901234567890123456789012123456789012345678901
1 implication true. If you totally nail a basic essaya clear main idea, great 2
2
12345678901234567890123456789012123456789012345678901
2
12345678901234567890123456789012123456789012345678901
set
of
supporting
details,
excellent
organization
on
the
paragraph
and
2
12345678901234567890123456789012123456789012345678901
2
12345678901234567890123456789012123456789012345678901
sentence
level,
and
good
overall
grammaryou
could
easily
earn
a
6
that
2345678901234567890123456789012123456789012345678901
2
1
2
12345678901234567890123456789012123456789012345678901
way.
In
the
end,
it
all
depends
on
the
reader.
The
point
behind
the
4
1
2
2
12345678901234567890123456789012123456789012345678901
2
12345678901234567890123456789012123456789012345678901
concept
is
to
emphasize
that
adding
advanced
writing
flourishes
will
help
2
12345678901234567890123456789012123456789012345678901
2345678901234567890123456789012123456789012345678901
12345678901234567890123456789012123456789012345678901
separate your essay from the standard essays. If you add these flourishes 2
2
12345678901234567890123456789012123456789012345678901
2
12345678901234567890123456789012123456789012345678901
well,
it
will
greatly
increase
your
chance
of
getting
a
5
or
6
since
you
will
12345678901234567890123456789012123456789012345678901 2
2
12345678901234567890123456789012123456789012345678901
be demonstrating your advanced writing ability.
2
12345678901234567890123456789012123456789012345678901
2
1
2
12345678901234567890123456789012123456789012345678901
2
12345678901234567890123456789012123456789012345678901
How
you
decide
to
make
your
essay
stand
out
from
the
pack
is
up
to
you.
2
12345678901234567890123456789012123456789012345678901
2345678901234567890123456789012123456789012345678901
2
12345678901234567890123456789012123456789012345678901
Here are three different methods that you can use:
2
12345678901234567890123456789012123456789012345678901
12345678901234567890123456789012123456789012345678901 2
12345678901234567890123456789012123456789012345678901 2
12345678901234567890123456789012123456789012345678901 2
2
12345678901234567890123456789012123456789012345678901
1.
Tweak the Basic Format
2
12345678901234567890123456789012123456789012345678901
2
12345678901234567890123456789012123456789012345678901
Its a simple fact that repetition leads to boredom for most people. You 2
12345678901234567890123456789012123456789012345678901
1 have to think that GRE readers get tired of seeing the same essays over and 2
2
12345678901234567890123456789012123456789012345678901
12345678901234567890123456789012123456789012345678901
over again. This doesnt mean theyll start giving out bad grades to 2
2
12345678901234567890123456789012123456789012345678901
2
12345678901234567890123456789012123456789012345678901
standard
essays.
It
means
that
they
might
perk
up
a
bit
if
your
essay
plays
2
12345678901234567890123456789012123456789012345678901
2345678901234567890123456789012123456789012345678901
2
12345678901234567890123456789012123456789012345678901
with the basic format a bit.
2
12345678901234567890123456789012123456789012345678901
2
1
2
12345678901234567890123456789012123456789012345678901
For
instance,
the
basic
essay
starts
with
the
main
idea
clearly
stated
in
the
2
12345678901234567890123456789012123456789012345678901
2345678901234567890123456789012123456789012345678901
12345678901234567890123456789012123456789012345678901
first paragraph. So tweak the format by placing your main idea in the 2
2
12345678901234567890123456789012123456789012345678901
1 second paragraph. Use the first paragraph to set the scene, vividly 2
12345678901234567890123456789012123456789012345678901 2
12345678901234567890123456789012123456789012345678901
describing an imaginary scenario that will back up your claim. Or start the 2
2
12345678901234567890123456789012123456789012345678901
12345678901234567890123456789012123456789012345678901
paragraph off with a good quote, provided the quote could then be used to 2
2
12345678901234567890123456789012123456789012345678901
12345678901234567890123456789012123456789012345678901
back up your point. Do something that captures the readers attention 2
12345678901234567890123456789012123456789012345678901 2
2
12345678901234567890123456789012123456789012345678901
while remaining relevant to the discussion at hand.
2
12345678901234567890123456789012123456789012345678901
2
12345678901234567890123456789012123456789012345678901
Page 229 earlier in this chapter listed two fictitious essay topics. The first 2
12345678901234567890123456789012123456789012345678901
12345678901234567890123456789012123456789012345678901
one has already been discussed, so we will use the second topic to take it to 2
2
12345678901234567890123456789012123456789012345678901
2
12345678901234567890123456789012123456789012345678901
the next level.
2
12345678901234567890123456789012123456789012345678901
2
12345678901234567890123456789012123456789012345678901
12345678901234567890123456789012123456789012345678901 2
2345678901234567890123456789012123456789012345678901
12345678901234567890123456789012123456789012345678901 2
2
1
2
12345678901234567890123456789012123456789012345678901
12345678901234567890123456789012123456789012345678901 2
2
12345678901234567890123456789012123456789012345678901
12345678901234567890123456789012123456789012345678901 2
12345678901234567890123456789012123456789012345678901 2
12345678901234567890123456789012123456789012345678901 2
2
246 12345678901234567890123456789012123456789012345678901
123456789012345678901234567890121234567890123456789012

Chapter 6: Analytical Writing Review

Take It to the Next Level

123456789012345678901234567890121234567890123456789012
12345678901234567890123456789012123456789012345678901 2
2
12345678901234567890123456789012123456789012345678901
Sample Topic:
2
12345678901234567890123456789012123456789012345678901
2
12345678901234567890123456789012123456789012345678901
Quick decisions lead to success in todays business world.
2
12345678901234567890123456789012123456789012345678901
12345678901234567890123456789012123456789012345678901 2
2
12345678901234567890123456789012123456789012345678901
Setting the scene with an imaginary scenario. (agrees with the topic)
2
12345678901234567890123456789012123456789012345678901
2
12345678901234567890123456789012123456789012345678901
2
12345678901234567890123456789012123456789012345678901
The
owners
of
Murpu
Labs
face
a
quandary.
Over
the
past
2
12345678901234567890123456789012123456789012345678901
2
12345678901234567890123456789012123456789012345678901
six
months,
theyve
worked
long
hours
developing
a
new
2345678901234567890123456789012123456789012345678901
2
1
2
12345678901234567890123456789012123456789012345678901
software
program
that
makes
it
easier
to
transfer
video
2
12345678901234567890123456789012123456789012345678901
2
12345678901234567890123456789012123456789012345678901
images
over
the
Internet.
This
market
is
currently
2
12345678901234567890123456789012123456789012345678901
2345678901234567890123456789012123456789012345678901
1
underserved, and large profits are available for a company 2
2
12345678901234567890123456789012123456789012345678901
2
12345678901234567890123456789012123456789012345678901
that
fulfills
customer
demands
in
this
area.
Yet
the
Murpu
2
12345678901234567890123456789012123456789012345678901
2
12345678901234567890123456789012123456789012345678901
software
still
has
some
bugs
to
be
worked
out.
With
start-up
2345678901234567890123456789012123456789012345678901
2
1
12345678901234567890123456789012123456789012345678901
money drying up, the company owners find themselves 2
2
12345678901234567890123456789012123456789012345678901
pondering the question, Do we release the flawed software 2
12345678901234567890123456789012123456789012345678901
2
12345678901234567890123456789012123456789012345678901
or not?
2
12345678901234567890123456789012123456789012345678901
2
12345678901234567890123456789012123456789012345678901
2
12345678901234567890123456789012123456789012345678901
Murpus
owners
should
stop
deliberating
and
start
selling,
2
12345678901234567890123456789012123456789012345678901
2
12345678901234567890123456789012123456789012345678901
since
speedy
decisions
are
crucial
in
todays
business
world.
2345678901234567890123456789012123456789012345678901
12345678901234567890123456789012123456789012345678901 22
1
12345678901234567890123456789012123456789012345678901
If you have knowledge of an actual business that hesitated and lost, by all 2
2
12345678901234567890123456789012123456789012345678901
2
12345678901234567890123456789012123456789012345678901
means
use
it.
Using
a
historical
anecdote
here
would
be
even
better,
since
it
2345678901234567890123456789012123456789012345678901
12345678901234567890123456789012123456789012345678901 22
12345678901234567890123456789012123456789012345678901
would give your argument the weight of some real historical events behind 2
12345678901234567890123456789012123456789012345678901
12345678901234567890123456789012123456789012345678901
it. However, a lack of precise business history will not hurt the stylistic 2
2
12345678901234567890123456789012123456789012345678901
12345678901234567890123456789012123456789012345678901
effects of the anecdotal first paragraph. You just create a fictitious 2
2
12345678901234567890123456789012123456789012345678901
12345678901234567890123456789012123456789012345678901
company and place them in the situation you want them to be stuck in. 2
12345678901234567890123456789012123456789012345678901 22
1 The point is to give the first paragraph some punch by creating a specific
2
12345678901234567890123456789012123456789012345678901
situation, one that draws the readers in and has them wondering how they 2
12345678901234567890123456789012123456789012345678901
2
12345678901234567890123456789012123456789012345678901
would act if they were running Murpu Labs.
2
12345678901234567890123456789012123456789012345678901
2
12345678901234567890123456789012123456789012345678901
2345678901234567890123456789012123456789012345678901
12345678901234567890123456789012123456789012345678901
The sentence that starts the second paragraph, Murpus owners should 2
2
12345678901234567890123456789012123456789012345678901
1 stop deliberating and start selling, since speedy decisions are crucial in 2
2
12345678901234567890123456789012123456789012345678901
2
12345678901234567890123456789012123456789012345678901
todays
business
world
ties
into
the
first
paragraph
and
shows
just
where
2345678901234567890123456789012123456789012345678901
12345678901234567890123456789012123456789012345678901 22
12345678901234567890123456789012123456789012345678901
1 you stand on the topic. Its your viewpoint, cleverly placed in the second 2
12345678901234567890123456789012123456789012345678901
paragraph, not the first. You can then expound upon this theme for the 2
2
12345678901234567890123456789012123456789012345678901
2345678901234567890123456789012123456789012345678901
2
12345678901234567890123456789012123456789012345678901
rest of the second paragraph.
2
12345678901234567890123456789012123456789012345678901
12345678901234567890123456789012123456789012345678901 22
12345678901234567890123456789012123456789012345678901
1 Since you started out with software as your business model, you can make 2
12345678901234567890123456789012123456789012345678901
that the I-see-the-other-side-of-the-coin angle for the third paragraph, too. 2
2
12345678901234567890123456789012123456789012345678901
12345678901234567890123456789012123456789012345678901
Speedy decisions are critical in the fast-paced computer 2
2
12345678901234567890123456789012123456789012345678901
2
12345678901234567890123456789012123456789012345678901
software
world,
but
its
true
that
other
business
areas
are
not
2345678901234567890123456789012123456789012345678901
12345678901234567890123456789012123456789012345678901 22
12345678901234567890123456789012123456789012345678901
so time-intensive. Banking, and institutional lending in general, 2
12345678901234567890123456789012123456789012345678901
12345678901234567890123456789012123456789012345678901
are two areas in which deliberate decision making are well 2
2
1
2345678901234567890123456789012123456789012345678901
12345678901234567890123456789012123456789012345678901
established. Yet technology has a way of speeding up the pace 2
2
1
2
12345678901234567890123456789012123456789012345678901
of
the
world,
and
even
such
ancient
sectors
as
banking
might
12345678901234567890123456789012123456789012345678901 2
12345678901234567890123456789012123456789012345678901
find themselves pushed to make decisions more rapidly than 2
2
12345678901234567890123456789012123456789012345678901
2
12345678901234567890123456789012123456789012345678901
they
would
have
in
the
past.
12345678901234567890123456789012123456789012345678901 22
12345678901234567890123456789012123456789012345678901 247
123456789012345678901234567890121234567890123456789012

http://www.xtremepapers.net

Part III: Section Review

www.petersons.com

http://www.xtremepapers.net

Tip

123456789012345678901234567890121234567890123456789012
12345678901234567890123456789012123456789012345678901 2
2
12345678901234567890123456789012123456789012345678901
12345678901234567890123456789012123456789012345678901 2
12345678901234567890123456789012123456789012345678901
In the sample essay outlined earlier, only one paragraph is devoted to 2
2
12345678901234567890123456789012123456789012345678901
2
12345678901234567890123456789012123456789012345678901
showing
the
other
sides
point
and
then
shooting
it
down.
You
can
alter
2
12345678901234567890123456789012123456789012345678901
2345678901234567890123456789012123456789012345678901
1 this standard essay format by creating every paragraph in the I see your 2
2
12345678901234567890123456789012123456789012345678901
12345678901234567890123456789012123456789012345678901
side, but mines stronger mold. Each paragraph would start off with you 2
2
12345678901234567890123456789012123456789012345678901
12345678901234567890123456789012123456789012345678901
stating your opponents viewpoint, but would then explain why your side 2
2345678901234567890123456789012123456789012345678901
2
1
12345678901234567890123456789012123456789012345678901
of the argument is better. This is a little tougher to pull off in a short 2
2
12345678901234567890123456789012123456789012345678901
2
12345678901234567890123456789012123456789012345678901
amount of time, which is why it would make your essay stand out.
2
12345678901234567890123456789012123456789012345678901
2345678901234567890123456789012123456789012345678901
2
1
2
12345678901234567890123456789012123456789012345678901
12345678901234567890123456789012123456789012345678901
The above paragraph shows how you can alter the initial outline of an 2
2
12345678901234567890123456789012123456789012345678901
essay just a bit, and still keep the rest of it the same. As always, you use 2
12345678901234567890123456789012123456789012345678901
2345678901234567890123456789012123456789012345678901
1 vague yet plausible facts that cannot be disputed. The phrase institutional 2
2
12345678901234567890123456789012123456789012345678901
2
12345678901234567890123456789012123456789012345678901
lending in general sounds very good and could mean such a huge range of 2
12345678901234567890123456789012123456789012345678901
2
12345678901234567890123456789012123456789012345678901
things its not even funny. In a time-intensive essay where readers cant 2
12345678901234567890123456789012123456789012345678901
2
12345678901234567890123456789012123456789012345678901
pause and deeply ponder, it fits wonderfully.
2
12345678901234567890123456789012123456789012345678901
2
12345678901234567890123456789012123456789012345678901
2
12345678901234567890123456789012123456789012345678901
Instead
of
starting
out
with
a
scenario,
you
could
also
begin
with
a
quote.
2345678901234567890123456789012123456789012345678901
12345678901234567890123456789012123456789012345678901 2
2
1
2
12345678901234567890123456789012123456789012345678901
Starting
things
off
with
a
quote.
(disagrees
with
the
topic)
2
12345678901234567890123456789012123456789012345678901
2
12345678901234567890123456789012123456789012345678901
2345678901234567890123456789012123456789012345678901
12345678901234567890123456789012123456789012345678901
Edward Young once wrote, Be wise with speed; a fool at forty 2
2
12345678901234567890123456789012123456789012345678901
is a fool indeed. Although the English poet and dramatist lived 2
12345678901234567890123456789012123456789012345678901
2
12345678901234567890123456789012123456789012345678901
over three hundred years ago, Youngs words are still relevant 2
12345678901234567890123456789012123456789012345678901
12345678901234567890123456789012123456789012345678901
today. They highlight the fact that rash decisions are often 2
2
12345678901234567890123456789012123456789012345678901
12345678901234567890123456789012123456789012345678901
bad decisions, and such foolishness leaves people open to 2
2
12345678901234567890123456789012123456789012345678901
2
1
embarrassment
and
other
unwanted
consequences.
2345678901234567890123456789012123456789012345678901
12345678901234567890123456789012123456789012345678901 2
12345678901234567890123456789012123456789012345678901 2
1
Quick decisions can also translate to bad business, which is 2
2
12345678901234567890123456789012123456789012345678901
2
12345678901234567890123456789012123456789012345678901
why
they
should
be
avoided
in
todays
business
world.
2345678901234567890123456789012123456789012345678901
12345678901234567890123456789012123456789012345678901 2
12345678901234567890123456789012123456789012345678901 2
1 Now, the main problem with this set-up is that you have to have the quote 2
2
12345678901234567890123456789012123456789012345678901
12345678901234567890123456789012123456789012345678901
from Edward Young floating around in your head in order to use it on the 2
2345678901234567890123456789012123456789012345678901
12345678901234567890123456789012123456789012345678901 2
2
12345678901234567890123456789012123456789012345678901
2
1 Analytical section. There are three ways to go about achieving this.
2
12345678901234567890123456789012123456789012345678901
Starting
with
a
quote
or
adding
fictitious
scenarios
are
just
two
ways
to
12345678901234567890123456789012123456789012345678901 2
2345678901234567890123456789012123456789012345678901
2
12345678901234567890123456789012123456789012345678901
tweak the basic essay format. Anything you do that demonstrates you can 2
12345678901234567890123456789012123456789012345678901
2
12345678901234567890123456789012123456789012345678901
write more than just a standard essay will improve your chances of 2
12345678901234567890123456789012123456789012345678901
1 garnering a high score on the Analytical portion of the GRE. Admittedly, 2
2
12345678901234567890123456789012123456789012345678901
2
12345678901234567890123456789012123456789012345678901
you
have
to
be
able
to
pull
off
the
altered
essay
format
successfully,
so
2
12345678901234567890123456789012123456789012345678901
2
12345678901234567890123456789012123456789012345678901
dont
add
change
just
for
changes
sake.
Adopting
a
writing
style
that
is
12345678901234567890123456789012123456789012345678901 2
2345678901234567890123456789012123456789012345678901
12345678901234567890123456789012123456789012345678901
obviously artificial will not help your cause, so never strain or stretch to 2
2
12345678901234567890123456789012123456789012345678901
2
12345678901234567890123456789012123456789012345678901
place
a
quote
just
to
give
your
essay
a
quote.
If
tweaking
the
essay
format
12345678901234567890123456789012123456789012345678901 2
1 in some way feels good to you, do it. Your instincts are better than you 2
2
12345678901234567890123456789012123456789012345678901
2
12345678901234567890123456789012123456789012345678901
might
think,
and
a
more
natural
writing
style
can
greatly
benefit
your
2
12345678901234567890123456789012123456789012345678901
2
12345678901234567890123456789012123456789012345678901
essay,
as
you
will
see
in
the
following
section.
2
12345678901234567890123456789012123456789012345678901
2345678901234567890123456789012123456789012345678901
2
1
12345678901234567890123456789012123456789012345678901 2
12345678901234567890123456789012123456789012345678901 2
2
248 12345678901234567890123456789012123456789012345678901
123456789012345678901234567890121234567890123456789012

Chapter 6: Analytical Writing Review

Take It to the Next Level

123456789012345678901234567890121234567890123456789012
12345678901234567890123456789012123456789012345678901 2
2
12345678901234567890123456789012123456789012345678901
12345678901234567890123456789012123456789012345678901 2
2
12345678901234567890123456789012123456789012345678901
Three Ways to Have Snappy Quotes Floating around in
2
12345678901234567890123456789012123456789012345678901
2
12345678901234567890123456789012123456789012345678901
Your
Head
2
12345678901234567890123456789012123456789012345678901
2345678901234567890123456789012123456789012345678901
2
1
2
12345678901234567890123456789012123456789012345678901
2
12345678901234567890123456789012123456789012345678901
1. Be the Kind of Person Who Likes to Memorize Quotes. Many
2
12345678901234567890123456789012123456789012345678901
2
12345678901234567890123456789012123456789012345678901
English majors like nothing more than to snap off a few lines
2345678901234567890123456789012123456789012345678901
2
1
of Shakespeare to prove their point. You might find this habit
2
12345678901234567890123456789012123456789012345678901
2
12345678901234567890123456789012123456789012345678901
annoying in the real world, but in GRE-Land its quite a useful
2
12345678901234567890123456789012123456789012345678901
2
12345678901234567890123456789012123456789012345678901
habit.
Even
if
youre
not
an
English
major,
many
people
have
2345678901234567890123456789012123456789012345678901
2
1
2
12345678901234567890123456789012123456789012345678901
proverbs (like Haste makes waste), Biblical quotes, and
2
12345678901234567890123456789012123456789012345678901
2
12345678901234567890123456789012123456789012345678901
snippets
of
famous
speeches
floating
around
in
their
grey
mat2
12345678901234567890123456789012123456789012345678901
2345678901234567890123456789012123456789012345678901
2
1
ter. These quotes can all be used if the situation warrants it.
2
12345678901234567890123456789012123456789012345678901
2
12345678901234567890123456789012123456789012345678901
2
12345678901234567890123456789012123456789012345678901
One
caveat
is
that
you
want
to
use
quotes
that
are
as
literary
2
12345678901234567890123456789012123456789012345678901
2345678901234567890123456789012123456789012345678901
2
12345678901234567890123456789012123456789012345678901
as possible. Shakespeare is good but John Dryden is better,
2
12345678901234567890123456789012123456789012345678901
2
12345678901234567890123456789012123456789012345678901
since
a
Dryden
quote
shows
that
you
have
knowledge
of
12345678901234567890123456789012123456789012345678901 22
12345678901234567890123456789012123456789012345678901
famous literature that goes beyond the basics. If you need help
2
12345678901234567890123456789012123456789012345678901
2
1
remembering this idea, think of the acronym FIBTS: Faulkner
2
12345678901234567890123456789012123456789012345678901
2
12345678901234567890123456789012123456789012345678901
Is Better Than Seuss.
2
12345678901234567890123456789012123456789012345678901
2345678901234567890123456789012123456789012345678901
2
12345678901234567890123456789012123456789012345678901
2. Start a List of Cool Quotes You Like. This method works best
2
12345678901234567890123456789012123456789012345678901
2
12345678901234567890123456789012123456789012345678901
if you are someone who reads literature on a regular basis. If
2
12345678901234567890123456789012123456789012345678901
2
12345678901234567890123456789012123456789012345678901
you
do,
you
should
come
across
neat
phrases
that
make
you
12345678901234567890123456789012123456789012345678901 22
12345678901234567890123456789012123456789012345678901
think to yourself, Hey! That sentence rocks! Start a quote
2
12345678901234567890123456789012123456789012345678901
2
12345678901234567890123456789012123456789012345678901
list
where
you
write
down
these
good
phrases.
The
list
will
2
1
2345678901234567890123456789012123456789012345678901
2
12345678901234567890123456789012123456789012345678901
help you remember the quote, and you can always refresh
2
12345678901234567890123456789012123456789012345678901
2
1
your
memory
by
rereading
the
list
from
time
to
time.
12345678901234567890123456789012123456789012345678901 2
2
12345678901234567890123456789012123456789012345678901
2
12345678901234567890123456789012123456789012345678901
3. Memorize a Bunch of Quotes for the GRE. This might not be
2
12345678901234567890123456789012123456789012345678901
2
12345678901234567890123456789012123456789012345678901
the
best
use
of
your
time,
but
to
achieve
it,
all
you
need
is
a
2
12345678901234567890123456789012123456789012345678901
2
12345678901234567890123456789012123456789012345678901
book of famous quotes and a computer connected to the
2345678901234567890123456789012123456789012345678901
12345678901234567890123456789012123456789012345678901 22
12345678901234567890123456789012123456789012345678901
Internet. Use the computer to scroll around and view all the
2
1
2
12345678901234567890123456789012123456789012345678901
different GRE essay topics, and use the quote book to
12345678901234567890123456789012123456789012345678901 2
2345678901234567890123456789012123456789012345678901
2
12345678901234567890123456789012123456789012345678901
rummage for quotes that you think would work well for each
2
12345678901234567890123456789012123456789012345678901
topic. This idea is part of a larger plan to prep for the essay
2
12345678901234567890123456789012123456789012345678901
2
12345678901234567890123456789012123456789012345678901
section, and it will be discussed in greater detail starting on
2
1
2345678901234567890123456789012123456789012345678901
2
12345678901234567890123456789012123456789012345678901
page 251.
2
12345678901234567890123456789012123456789012345678901
12345678901234567890123456789012123456789012345678901 22
12345678901234567890123456789012123456789012345678901 2
1
2
12345678901234567890123456789012123456789012345678901
2.
Use Your Voice
2
12345678901234567890123456789012123456789012345678901
2
12345678901234567890123456789012123456789012345678901
Writing
in
your
voice
is
the
best
way
to
help
distinguish
your
essay
from
2
12345678901234567890123456789012123456789012345678901
2
12345678901234567890123456789012123456789012345678901
others,
but
its
the
hardest
one
to
achieve.
Theres
no
way
to
really
teach
2345678901234567890123456789012123456789012345678901
12345678901234567890123456789012123456789012345678901 22
1 voice, either, since a persons voice must come from the individual.
2
12345678901234567890123456789012123456789012345678901
2
12345678901234567890123456789012123456789012345678901
However,
the
idea
of
voice
can
be
discussed
and
described,
and
you
can
2
12345678901234567890123456789012123456789012345678901
2345678901234567890123456789012123456789012345678901
2
1 take it from there and run with it as long as you can.
12345678901234567890123456789012123456789012345678901 2
12345678901234567890123456789012123456789012345678901 2
12345678901234567890123456789012123456789012345678901 2 249
123456789012345678901234567890121234567890123456789012

http://www.xtremepapers.net

Part III: Section Review

www.petersons.com

http://www.xtremepapers.net

Tip

123456789012345678901234567890121234567890123456789012
12345678901234567890123456789012123456789012345678901 2
2
12345678901234567890123456789012123456789012345678901
12345678901234567890123456789012123456789012345678901 2
12345678901234567890123456789012123456789012345678901
In writing, voice refers to an authors distinctive style of using words. 2
2
12345678901234567890123456789012123456789012345678901
2
12345678901234567890123456789012123456789012345678901
Many
established
writers
have
such
a
unique
voice
that
you
can
2
12345678901234567890123456789012123456789012345678901
2345678901234567890123456789012123456789012345678901
1 determine when a piece of writing is theirs after a couple of sentences. 2
2
12345678901234567890123456789012123456789012345678901
12345678901234567890123456789012123456789012345678901
Suppose you read the words, Round about there/In the Valley of 2
2
12345678901234567890123456789012123456789012345678901
12345678901234567890123456789012123456789012345678901
Crair/Lived an old Hoobavax/By the name of Snare and were asked to 2
2345678901234567890123456789012123456789012345678901
2
1
12345678901234567890123456789012123456789012345678901
determine the writer. You would probably guess the well-known 2
2
12345678901234567890123456789012123456789012345678901
12345678901234567890123456789012123456789012345678901
childrens author Dr. Seuss, since these words are written in the voice of 2
2
12345678901234567890123456789012123456789012345678901
Dr. Seuss.
2
12345678901234567890123456789012123456789012345678901
2
12345678901234567890123456789012123456789012345678901
2
12345678901234567890123456789012123456789012345678901
2
12345678901234567890123456789012123456789012345678901
Take a moment and think about your own writing. Look over some old 2
12345678901234567890123456789012123456789012345678901
2345678901234567890123456789012123456789012345678901
1 journal entries, term papers, poems, or short stories, if you have any laying 2
2
12345678901234567890123456789012123456789012345678901
2
12345678901234567890123456789012123456789012345678901
around. Is your writing very serious, or do you include humor whenever 2
12345678901234567890123456789012123456789012345678901
2
12345678901234567890123456789012123456789012345678901
possible? Do you prefer a short, terse journalistic style full of short 2
12345678901234567890123456789012123456789012345678901
12345678901234567890123456789012123456789012345678901
sentences in small paragraphs, or do you tend toward lengthy paragraphs 2
2
12345678901234567890123456789012123456789012345678901
2
12345678901234567890123456789012123456789012345678901
with
long,
highly
developed
sentences?
No
answer
is
better
than
another.
2
12345678901234567890123456789012123456789012345678901
2345678901234567890123456789012123456789012345678901
2
12345678901234567890123456789012123456789012345678901
1 The overarching goal is to differentiate your essay from the pack (in a 2
12345678901234567890123456789012123456789012345678901
good way), and using a style thats your own is an excellent way to do this. 2
2
12345678901234567890123456789012123456789012345678901
2
12345678901234567890123456789012123456789012345678901
2345678901234567890123456789012123456789012345678901
12345678901234567890123456789012123456789012345678901
Like everything, using your voice has some qualifiers attached to it. Not 2
2
12345678901234567890123456789012123456789012345678901
2
12345678901234567890123456789012123456789012345678901
every
type
of
voice
can
be
used
for
an
essay.
For
an
example
of
an
inap12345678901234567890123456789012123456789012345678901 2
12345678901234567890123456789012123456789012345678901
propriate voice, suppose you like to write in a style similar to the French 2
2
12345678901234567890123456789012123456789012345678901
12345678901234567890123456789012123456789012345678901
writer Celine. Your essay would be a mishmash of thoughts connected by 2
12345678901234567890123456789012123456789012345678901 2
2
12345678901234567890123456789012123456789012345678901
1 ellipses . . . like hearing a story through a strobe light . . . every other phrase 2
2345678901234567890123456789012123456789012345678901
12345678901234567890123456789012123456789012345678901
making its way throughothers, blacked out . . . Mon Dieu! Thats a 2
2
12345678901234567890123456789012123456789012345678901
1 terrible style to use for an essay. The reader will not say,Hmm, an homage 2
2
12345678901234567890123456789012123456789012345678901
to Celine! The response will be more along the lines of, This clown doesnt 2
12345678901234567890123456789012123456789012345678901
2345678901234567890123456789012123456789012345678901
2
12345678901234567890123456789012123456789012345678901
know when to use a period. As you can see, some styles are just too radical 2
12345678901234567890123456789012123456789012345678901
1 to use on the essay format. If you are gunning for a 6, you probably have 2
2
12345678901234567890123456789012123456789012345678901
2
12345678901234567890123456789012123456789012345678901
enough
of
a
writing
background
to
know
when
youve
crossed
the
line
2345678901234567890123456789012123456789012345678901
12345678901234567890123456789012123456789012345678901 2
2
12345678901234567890123456789012123456789012345678901
2
1 from strong voice into experimental text creation.
12345678901234567890123456789012123456789012345678901 2
12345678901234567890123456789012123456789012345678901
On a personal note, I like to make jokes while writing. (I wont go so far 2
2
12345678901234567890123456789012123456789012345678901
2
12345678901234567890123456789012123456789012345678901
and
claim
they
are
funny.)
I
could
have
written
an
essay
without
humor,
2
12345678901234567890123456789012123456789012345678901
2
12345678901234567890123456789012123456789012345678901
but
I
didnt
see
the
point
in
doing
so.
While
the
entire
text
wasnt
a
12345678901234567890123456789012123456789012345678901 2
2345678901234567890123456789012123456789012345678901
12345678901234567890123456789012123456789012345678901
stand-up routine, I told about one joke per paragraph, for both essays. My 2
2
12345678901234567890123456789012123456789012345678901
12345678901234567890123456789012123456789012345678901
conclusion on the first essay was even a joke. I did this because thats the 2
12345678901234567890123456789012123456789012345678901 2
1 kind of writer I am. My score on the Analytical portion was a 6, which is 2
2
12345678901234567890123456789012123456789012345678901
12345678901234567890123456789012123456789012345678901
good news for you because you probably wouldnt want to be reading a 2
2
12345678901234567890123456789012123456789012345678901
2
12345678901234567890123456789012123456789012345678901
book from someone who scored a 3. You can say I received a 6 because:
12345678901234567890123456789012123456789012345678901 2
2345678901234567890123456789012123456789012345678901
2
12345678901234567890123456789012123456789012345678901
1. Im a professional writer.
2
1
2
12345678901234567890123456789012123456789012345678901
2
12345678901234567890123456789012123456789012345678901
2.
I
used
my
own
voice,
and
this
helped
distinguish
my
essay
from
2
12345678901234567890123456789012123456789012345678901
2345678901234567890123456789012123456789012345678901
2
1
others.
2
12345678901234567890123456789012123456789012345678901
12345678901234567890123456789012123456789012345678901 2
2
250 12345678901234567890123456789012123456789012345678901
123456789012345678901234567890121234567890123456789012

Chapter 6: Analytical Writing Review

Note

Take It to the Next Level

123456789012345678901234567890121234567890123456789012
12345678901234567890123456789012123456789012345678901 2
2
12345678901234567890123456789012123456789012345678901
3. I told jokes and the bored readers thought my jokes were funny, 2
12345678901234567890123456789012123456789012345678901
2
12345678901234567890123456789012123456789012345678901
or at least helped break up the monotony of their task.
2
12345678901234567890123456789012123456789012345678901
12345678901234567890123456789012123456789012345678901 2
12345678901234567890123456789012123456789012345678901
There might an element of truth to all three points, but the second point is 2
2
12345678901234567890123456789012123456789012345678901
2
12345678901234567890123456789012123456789012345678901
the
one
you
should
concern
yourself
with.
Dont
feel
you
have
to
shackle
2
12345678901234567890123456789012123456789012345678901
2
12345678901234567890123456789012123456789012345678901
the
way
you
write
in
order
to
conform
to
the
standard
essay.
Make
all
the
2
12345678901234567890123456789012123456789012345678901
2345678901234567890123456789012123456789012345678901
1 standard essay points as necessary, and then garnish them with sentences 2
2
12345678901234567890123456789012123456789012345678901
2
12345678901234567890123456789012123456789012345678901
that
are
your
own.
2
12345678901234567890123456789012123456789012345678901
2
12345678901234567890123456789012123456789012345678901
2345678901234567890123456789012123456789012345678901
1 While the strategy of using your own voice is a good one, its not 2
2
12345678901234567890123456789012123456789012345678901
12345678901234567890123456789012123456789012345678901
something that is easily taught. For more concrete assistance, look no 2
2
12345678901234567890123456789012123456789012345678901
2
12345678901234567890123456789012123456789012345678901
further than the last technique, which pretty much reeks of realpolitik.
2345678901234567890123456789012123456789012345678901
2
1
2
12345678901234567890123456789012123456789012345678901
2
12345678901234567890123456789012123456789012345678901
2
12345678901234567890123456789012123456789012345678901
2
12345678901234567890123456789012123456789012345678901
3.
Just
Prepare
Ahead
of
Time
2345678901234567890123456789012123456789012345678901
12345678901234567890123456789012123456789012345678901 22
12345678901234567890123456789012123456789012345678901
You know you have to write 2 essays, and you know where all the sample 2
12345678901234567890123456789012123456789012345678901
12345678901234567890123456789012123456789012345678901
topics are located. It doesnt take a rocket scientistor person aspiring to 2
2
12345678901234567890123456789012123456789012345678901
2
12345678901234567890123456789012123456789012345678901
1 go to rocket scientist graduate schoolto put two and two together and 2
2
12345678901234567890123456789012123456789012345678901
realize you can look over the topics and prepare ahead of time.
2
12345678901234567890123456789012123456789012345678901
2
12345678901234567890123456789012123456789012345678901
2345678901234567890123456789012123456789012345678901
12345678901234567890123456789012123456789012345678901
The question is not whether you should prepare ahead of time, but how 2
2
12345678901234567890123456789012123456789012345678901
2
12345678901234567890123456789012123456789012345678901
much
preparation
do
you
wish
to
do?
Writing
out
roughly
500
sample
12345678901234567890123456789012123456789012345678901 22
12345678901234567890123456789012123456789012345678901
essays would not be the best use of your time. Besides, other people have
2
12345678901234567890123456789012123456789012345678901
12345678901234567890123456789012123456789012345678901
already done this for you. If youre interested in seeing what some 2
12345678901234567890123456789012123456789012345678901 22
12345678901234567890123456789012123456789012345678901
1 sample essays look like, you check out ARCO GRE CAT Answers to the 2
12345678901234567890123456789012123456789012345678901
Real Essay Questions. You dont want to use the essays as they appear in 2
2
12345678901234567890123456789012123456789012345678901
the book, since this is wrong for many different reasons. Instead, looking 2
12345678901234567890123456789012123456789012345678901
2
12345678901234567890123456789012123456789012345678901
over the sample essays will give you an idea of what kinds of supporting 2
12345678901234567890123456789012123456789012345678901
2345678901234567890123456789012123456789012345678901
2
12345678901234567890123456789012123456789012345678901
details you could use for each essay. It will also give you a chance to 2
12345678901234567890123456789012123456789012345678901
1 look over different essay formats. Seeing different ways that supporting 2
2
12345678901234567890123456789012123456789012345678901
2
12345678901234567890123456789012123456789012345678901
details
are
used
should
help
you
brainstorm
ideas
of
your
own,
while
2345678901234567890123456789012123456789012345678901
12345678901234567890123456789012123456789012345678901 22
12345678901234567890123456789012123456789012345678901
1 reviewing different formats should make it easier for you to organize 2
2
12345678901234567890123456789012123456789012345678901
your own essay.
2
12345678901234567890123456789012123456789012345678901
2345678901234567890123456789012123456789012345678901
12345678901234567890123456789012123456789012345678901 22
12345678901234567890123456789012123456789012345678901 2
12345678901234567890123456789012123456789012345678901 2
12345678901234567890123456789012123456789012345678901
2
1 These books are by Petersons as well, although there might be similar
2
12345678901234567890123456789012123456789012345678901
titles by other publishers. The books listed here are the ones rated highest
2
12345678901234567890123456789012123456789012345678901
2
12345678901234567890123456789012123456789012345678901
by
the
author
and
publisher,
but
thats
merely
a
coincidence.
2
12345678901234567890123456789012123456789012345678901
12345678901234567890123456789012123456789012345678901 2
2345678901234567890123456789012123456789012345678901
12345678901234567890123456789012123456789012345678901 22
12345678901234567890123456789012123456789012345678901 2
12345678901234567890123456789012123456789012345678901
The importance of spending about 5 minutes brainstorming before writing 2
12345678901234567890123456789012123456789012345678901
1 your essay is covered on pages 229230 earlier in this chapter. This 2
2345678901234567890123456789012123456789012345678901
12345678901234567890123456789012123456789012345678901 22
1 brainstorming will be a lot better and take less time if you have already
2
12345678901234567890123456789012123456789012345678901
2
12345678901234567890123456789012123456789012345678901
thought
about
the
topic
previously.
So
even
if
you
dont
want
to
use
the
2
12345678901234567890123456789012123456789012345678901
2345678901234567890123456789012123456789012345678901
1 sample essay book, you should still look over some sample topics and 2
12345678901234567890123456789012123456789012345678901 2
12345678901234567890123456789012123456789012345678901 2
12345678901234567890123456789012123456789012345678901 2 251
123456789012345678901234567890121234567890123456789012

http://www.xtremepapers.net

Part III: Section Review

123456789012345678901234567890121234567890123456789012
12345678901234567890123456789012123456789012345678901 2
2
12345678901234567890123456789012123456789012345678901
mentally sketch out how you would approach them. Take about 5 minutes 2
12345678901234567890123456789012123456789012345678901
2
12345678901234567890123456789012123456789012345678901
for each topic and ask yourself, Would I agree or disagree with the topic 2
12345678901234567890123456789012123456789012345678901
2
12345678901234567890123456789012123456789012345678901
sentence? What type of supporting details would I like to use? You dont 2
12345678901234567890123456789012123456789012345678901
2345678901234567890123456789012123456789012345678901
1 have to write your answers down, although you can if you like. The main 2
2
12345678901234567890123456789012123456789012345678901
2
12345678901234567890123456789012123456789012345678901
goal
is
to
acquaint
yourself
with
the
topic
beforehand,
and
come
up
with
a
2
12345678901234567890123456789012123456789012345678901
2
12345678901234567890123456789012123456789012345678901
rudimentary
idea
about
how
you
would
want
to
approach
an
essay
on
the
2345678901234567890123456789012123456789012345678901
2
1
2
12345678901234567890123456789012123456789012345678901
subject.
This
can
help
you
in
several
ways:
2
12345678901234567890123456789012123456789012345678901
2
12345678901234567890123456789012123456789012345678901
2
12345678901234567890123456789012123456789012345678901
1.
If
you
have
a
choice
between
a
topic
youve
thought
about
and
2345678901234567890123456789012123456789012345678901
2
1
2
12345678901234567890123456789012123456789012345678901
one
that
you
havent,
the
decision
to
choose
the
one
youve
2
12345678901234567890123456789012123456789012345678901
2
12345678901234567890123456789012123456789012345678901
thought
about
should
come
very
rapidly.
2
12345678901234567890123456789012123456789012345678901
2345678901234567890123456789012123456789012345678901
2
1
12345678901234567890123456789012123456789012345678901
2. If you get a topic youve seen before, you wont waste any time 2
2
12345678901234567890123456789012123456789012345678901
deciding whether to agree or disagree since you already thought 2
12345678901234567890123456789012123456789012345678901
2
12345678901234567890123456789012123456789012345678901
about how to approach that topic. This should also make coming 2
12345678901234567890123456789012123456789012345678901
2
12345678901234567890123456789012123456789012345678901
up with supporting details much quicker and easier, leaving you 2
12345678901234567890123456789012123456789012345678901
2
12345678901234567890123456789012123456789012345678901
with more time to get words into the computer.
2
12345678901234567890123456789012123456789012345678901
2345678901234567890123456789012123456789012345678901
12345678901234567890123456789012123456789012345678901 2
1
3. Even if you get two topics that you havent thought about 2
2
12345678901234567890123456789012123456789012345678901
2
12345678901234567890123456789012123456789012345678901
already,
your
brainstorming
skills
will
be
greatly
improved
with
2
12345678901234567890123456789012123456789012345678901
2345678901234567890123456789012123456789012345678901
12345678901234567890123456789012123456789012345678901
all the practice. This should translate to less time needed to decide 2
2
12345678901234567890123456789012123456789012345678901
2
12345678901234567890123456789012123456789012345678901
whether
to
agree
or
disagree,
as
well
as
greater
ease
at
coming
up
12345678901234567890123456789012123456789012345678901 2
2
12345678901234567890123456789012123456789012345678901
with good supporting details.
2
12345678901234567890123456789012123456789012345678901
12345678901234567890123456789012123456789012345678901 2
12345678901234567890123456789012123456789012345678901
Doing the 5-minute brainstorm exercise on several topics is something 2
2
12345678901234567890123456789012123456789012345678901
1 everyone should do. If you want to be more gung-ho, there are other things 2
2345678901234567890123456789012123456789012345678901
12345678901234567890123456789012123456789012345678901 2
2
you can do, but these might not be the optimal use of your time.
12345678901234567890123456789012123456789012345678901
2
1
12345678901234567890123456789012123456789012345678901 2
2
12345678901234567890123456789012123456789012345678901
12345678901234567890123456789012123456789012345678901
You could do research to come up with hard facts to use on every essay, 2
2
12345678901234567890123456789012123456789012345678901
2
12345678901234567890123456789012123456789012345678901
but
its
not
a
good
idea
for
two
reasons.
For
one
thing,
it
would
take
a
2
12345678901234567890123456789012123456789012345678901
2
12345678901234567890123456789012123456789012345678901
massive
amount
of
time
for
very
little
payoff.
Also,
if
you
did
include
2345678901234567890123456789012123456789012345678901
12345678901234567890123456789012123456789012345678901 2
2
12345678901234567890123456789012123456789012345678901
1 something very specific, it might tip the reader off to the fact that you 2
12345678901234567890123456789012123456789012345678901
prepared beforehand. Theres nothing illegal about this, but its not going 2
2
12345678901234567890123456789012123456789012345678901
2345678901234567890123456789012123456789012345678901
12345678901234567890123456789012123456789012345678901
to earn you any Brownie points. Its much better to appear like you just 2
2
12345678901234567890123456789012123456789012345678901
12345678901234567890123456789012123456789012345678901
happened to get that particular topic, and you just happened to come up 2
12345678901234567890123456789012123456789012345678901 2
2
1 with the set of supporting details that you did.
2
12345678901234567890123456789012123456789012345678901
2
12345678901234567890123456789012123456789012345678901
2
12345678901234567890123456789012123456789012345678901
2
12345678901234567890123456789012123456789012345678901
2
12345678901234567890123456789012123456789012345678901
The
Second
Essay
2345678901234567890123456789012123456789012345678901
12345678901234567890123456789012123456789012345678901 2
2
12345678901234567890123456789012123456789012345678901
The second essay doesnt give you as much writing wiggle room as the 2
12345678901234567890123456789012123456789012345678901
2
12345678901234567890123456789012123456789012345678901
1 first. That essay is more of a writing assignment, while the second essay is 2
2345678901234567890123456789012123456789012345678901
2
12345678901234567890123456789012123456789012345678901
1 closer to an exercise in logical thinking. Sure, you still have to write, but 2
12345678901234567890123456789012123456789012345678901
you dont have to formulate an argument from scratch. You just have to 2
2
12345678901234567890123456789012123456789012345678901
2
12345678901234567890123456789012123456789012345678901
comment
on
an
argument
given
to
you.
2345678901234567890123456789012123456789012345678901
2
1
12345678901234567890123456789012123456789012345678901 2
12345678901234567890123456789012123456789012345678901 2
2
252 12345678901234567890123456789012123456789012345678901
123456789012345678901234567890121234567890123456789012

www.petersons.com

http://www.xtremepapers.net

Chapter 6: Analytical Writing Review

Take It to the Next Level

123456789012345678901234567890121234567890123456789012
12345678901234567890123456789012123456789012345678901 2
2
12345678901234567890123456789012123456789012345678901
12345678901234567890123456789012123456789012345678901 2
2
12345678901234567890123456789012123456789012345678901
Other Ways to Prep for the Essay Beforehand
2
12345678901234567890123456789012123456789012345678901
12345678901234567890123456789012123456789012345678901 2
2
12345678901234567890123456789012123456789012345678901
2
12345678901234567890123456789012123456789012345678901
1. Prepare a Quote List. As stated earlier on page 248, placing a
2
12345678901234567890123456789012123456789012345678901
2
12345678901234567890123456789012123456789012345678901
relevant quote is a nice addition to any essay. When
2
12345678901234567890123456789012123456789012345678901
2
12345678901234567890123456789012123456789012345678901
brainstorming on a sample topic, ask yourself, Is there a
2345678901234567890123456789012123456789012345678901
2
1
quote I can think of that would help prove my point? If you
2
12345678901234567890123456789012123456789012345678901
2
12345678901234567890123456789012123456789012345678901
cant think of one right away, you could always do some
2
12345678901234567890123456789012123456789012345678901
2
12345678901234567890123456789012123456789012345678901
searching
in
a
book
of
quotations
like
Bartletts.
Even
if
this
2345678901234567890123456789012123456789012345678901
2
1
2
12345678901234567890123456789012123456789012345678901
exercise doesnt land you the quote you want, looking
2
12345678901234567890123456789012123456789012345678901
2
12345678901234567890123456789012123456789012345678901
through
a
book
of
quotes
is
often
pretty
cool.
2
12345678901234567890123456789012123456789012345678901
2345678901234567890123456789012123456789012345678901
2
1
2
12345678901234567890123456789012123456789012345678901
2.
Have
Some
Snazzy
Vocabulary
Words
Ready
to
Go.
You
can
2
12345678901234567890123456789012123456789012345678901
2
12345678901234567890123456789012123456789012345678901
compile
a
list
of
advanced
words
that
could
be
brought
into
2
12345678901234567890123456789012123456789012345678901
2345678901234567890123456789012123456789012345678901
2
12345678901234567890123456789012123456789012345678901
play on your essay. For example, in the paragraph where you
2
12345678901234567890123456789012123456789012345678901
2
12345678901234567890123456789012123456789012345678901
show
that
you
can
acknowledge
the
other
sides
point
of
view,
12345678901234567890123456789012123456789012345678901 22
12345678901234567890123456789012123456789012345678901
you might have a sentence like, Although this might happen,
2
12345678901234567890123456789012123456789012345678901
2
1
the chances of this occurring are actually very small. Instead
2
12345678901234567890123456789012123456789012345678901
2
12345678901234567890123456789012123456789012345678901
of small, use infinitesimal. It means the same thing, but the
2
12345678901234567890123456789012123456789012345678901
2345678901234567890123456789012123456789012345678901
2
12345678901234567890123456789012123456789012345678901
GRE readers will note your use of advanced vocabulary.
2
12345678901234567890123456789012123456789012345678901
2
12345678901234567890123456789012123456789012345678901
Other good words like specious, verify, quandary (seen earlier
2
12345678901234567890123456789012123456789012345678901
in this chapter), ratiocination, and blinkered can crop up in
2
12345678901234567890123456789012123456789012345678901
2
12345678901234567890123456789012123456789012345678901
many point-of-view essays.
2
12345678901234567890123456789012123456789012345678901
12345678901234567890123456789012123456789012345678901 22
12345678901234567890123456789012123456789012345678901
If you are already compiling a list of vocabulary words for the
2
1
2
12345678901234567890123456789012123456789012345678901
GRE Verbal section, you dont need to make two lists. Simply
2
12345678901234567890123456789012123456789012345678901
2
12345678901234567890123456789012123456789012345678901
keep
those
advanced
words
in
mind
when
writing
your
GRE
12345678901234567890123456789012123456789012345678901 2
2
12345678901234567890123456789012123456789012345678901
essays.
2
12345678901234567890123456789012123456789012345678901
2
12345678901234567890123456789012123456789012345678901
12345678901234567890123456789012123456789012345678901 2
2
12345678901234567890123456789012123456789012345678901
Two of the three strategies you used on the first essay work on the second 2
12345678901234567890123456789012123456789012345678901
2345678901234567890123456789012123456789012345678901
2
12345678901234567890123456789012123456789012345678901
one, but the Tweaking the Essay Format technique doesnt really apply 2
12345678901234567890123456789012123456789012345678901
1 to the second essay. With only 30 minutes, you need to attack the 2
12345678901234567890123456789012123456789012345678901 2
12345678901234567890123456789012123456789012345678901
argument and uncover the flaws. A basic essay with distinct paragraphs 2
2
12345678901234567890123456789012123456789012345678901
2
12345678901234567890123456789012123456789012345678901
attacking
specific
points
is
the
best
way
to
do
this.
If
you
can
add
a
quote
2
12345678901234567890123456789012123456789012345678901
2
12345678901234567890123456789012123456789012345678901
to
help
make
your
point,
bully
for
you.
However,
adding
a
complex
12345678901234567890123456789012123456789012345678901 2
2345678901234567890123456789012123456789012345678901
12345678901234567890123456789012123456789012345678901
organizational layer might obscure the points you are trying to make, so 2
2
12345678901234567890123456789012123456789012345678901
2
12345678901234567890123456789012123456789012345678901
keeping
it
simple
works
best.
If
you
want
to
take
a
risk
and
write
the
entire
12345678901234567890123456789012123456789012345678901 22
1 second essay as a Socratic dialogue, go right ahead, but this approach will
2
12345678901234567890123456789012123456789012345678901
2
12345678901234567890123456789012123456789012345678901
likely
be
too
radical
for
most
readers.
A
simple
format
showing
complex
2
12345678901234567890123456789012123456789012345678901
2
12345678901234567890123456789012123456789012345678901
thoughts
is
better
than
a
complex
format
obscuring
complex
thoughts.
12345678901234567890123456789012123456789012345678901 2
2345678901234567890123456789012123456789012345678901
12345678901234567890123456789012123456789012345678901 22
1 Using your own voice can work on the second paragraph, but in a limited
2
12345678901234567890123456789012123456789012345678901
12345678901234567890123456789012123456789012345678901
manner. Again, for the second essay your overall goal is to expose flaws in 2
2
12345678901234567890123456789012123456789012345678901
the original argument, not display your writing chops. If you can show a 2
12345678901234567890123456789012123456789012345678901
2
12345678901234567890123456789012123456789012345678901
little personality, by all means do so. It would certainly look good if 2
12345678901234567890123456789012123456789012345678901
12345678901234567890123456789012123456789012345678901 2 253
123456789012345678901234567890121234567890123456789012

http://www.xtremepapers.net

Part III: Section Review

www.petersons.com

http://www.xtremepapers.net

Tip

123456789012345678901234567890121234567890123456789012
12345678901234567890123456789012123456789012345678901 2
2
12345678901234567890123456789012123456789012345678901
elements of your writing style displayed in the first essay appear in the 2
12345678901234567890123456789012123456789012345678901
2
12345678901234567890123456789012123456789012345678901
second essay as well. That will show the GRE readers that the writing is 2
12345678901234567890123456789012123456789012345678901
2
12345678901234567890123456789012123456789012345678901
definitely yours, and not something you crafted ahead of time with the 2
12345678901234567890123456789012123456789012345678901
2345678901234567890123456789012123456789012345678901
2
1 help of others.
2
12345678901234567890123456789012123456789012345678901
2
12345678901234567890123456789012123456789012345678901
2
12345678901234567890123456789012123456789012345678901
The
third
strategy,
preparing
beforehand,
is
the
one
that
works
best
for
the
2
12345678901234567890123456789012123456789012345678901
2345678901234567890123456789012123456789012345678901
1 second essay. It is easily the best thing you can do to write a better second 2
2
12345678901234567890123456789012123456789012345678901
2
12345678901234567890123456789012123456789012345678901
essay.
The
second
essay
requires
you
to
acquire
and
display
a
certain
2
12345678901234567890123456789012123456789012345678901
2
12345678901234567890123456789012123456789012345678901
skilldetermining
the
flaws
of
a
short
argumentand
the
foremost
way
2345678901234567890123456789012123456789012345678901
2
1
2
12345678901234567890123456789012123456789012345678901
to
get
better
at
this
skill
is
through
practice
on
real
topics.
2
12345678901234567890123456789012123456789012345678901
2
12345678901234567890123456789012123456789012345678901
2
12345678901234567890123456789012123456789012345678901
2345678901234567890123456789012123456789012345678901
1 To help you analyze arguments, you can peruse the book ARCO GRE 2
2
12345678901234567890123456789012123456789012345678901
2
12345678901234567890123456789012123456789012345678901
CAT
Answers
to
the
Real
Essay
Questions.
This
will
give
you
an
idea
of
2
12345678901234567890123456789012123456789012345678901
2
12345678901234567890123456789012123456789012345678901
how
to
create
and
write
about
alternative
scenarios
that
undermine
the
2345678901234567890123456789012123456789012345678901
12345678901234567890123456789012123456789012345678901 2
2
12345678901234567890123456789012123456789012345678901
conclusions and supporting details of an argument.
2
12345678901234567890123456789012123456789012345678901
12345678901234567890123456789012123456789012345678901 2
12345678901234567890123456789012123456789012345678901 2
2
12345678901234567890123456789012123456789012345678901
1 You do not need to write out entire essays, although you can write out 2
12345678901234567890123456789012123456789012345678901
some if you feel like it. Focus instead on honing your brainstorming skills 2
2
12345678901234567890123456789012123456789012345678901
12345678901234567890123456789012123456789012345678901
to the point where you can take any 100-word argument, easily spot the 2
2345678901234567890123456789012123456789012345678901
12345678901234567890123456789012123456789012345678901 2
12345678901234567890123456789012123456789012345678901
conclusion and supporting details, and then start churning out alternatives 2
2
12345678901234567890123456789012123456789012345678901
that show the logical flaws. This can be a completely mental exercise 2
12345678901234567890123456789012123456789012345678901
2
12345678901234567890123456789012123456789012345678901
where you spend roughly 5 minutes just thinking about every argument.
2
12345678901234567890123456789012123456789012345678901
12345678901234567890123456789012123456789012345678901 2
12345678901234567890123456789012123456789012345678901
You have 240 arguments to work on, but you probably wont need that 2
2
12345678901234567890123456789012123456789012345678901
1 many to get proficient at finding the flaws in an argument. Once you have 2
2345678901234567890123456789012123456789012345678901
12345678901234567890123456789012123456789012345678901 2
2
12345678901234567890123456789012123456789012345678901
1 this skill under your belt, the hardest part of the second essay is done. 2
12345678901234567890123456789012123456789012345678901
When test day comes, you might get lucky and encounter an argument 2
2
12345678901234567890123456789012123456789012345678901
2345678901234567890123456789012123456789012345678901
12345678901234567890123456789012123456789012345678901
youve worked on, but even if you dont, it should be easy for you to read 2
2
12345678901234567890123456789012123456789012345678901
1 the text and begin formulating alternatives. You then place as many of 2
2
12345678901234567890123456789012123456789012345678901
2
12345678901234567890123456789012123456789012345678901
these
alternatives
into
a
basic
essay
format
as
time
allows,
and
give
2345678901234567890123456789012123456789012345678901
12345678901234567890123456789012123456789012345678901 2
2
12345678901234567890123456789012123456789012345678901
1 yourself a few minutes at the very end to check over your grammar and 2
2
12345678901234567890123456789012123456789012345678901
perhaps tighten up a few sentences.
2
12345678901234567890123456789012123456789012345678901
2345678901234567890123456789012123456789012345678901
12345678901234567890123456789012123456789012345678901 2
A second essay with a host of strong alternatives is an excellent 2
12345678901234567890123456789012123456789012345678901
2
12345678901234567890123456789012123456789012345678901
complement to a first essay that shows some originality in content and 2
12345678901234567890123456789012123456789012345678901
1 structure. While this combo cant guarantee you a 6nothing can, for that 2
2
12345678901234567890123456789012123456789012345678901
2
12345678901234567890123456789012123456789012345678901
matterit does present a strong case to the GRE readers that you are 2
12345678901234567890123456789012123456789012345678901
2
12345678901234567890123456789012123456789012345678901
someone who really knows her stuff on the GRE Analytical section.
2
12345678901234567890123456789012123456789012345678901
2345678901234567890123456789012123456789012345678901
12345678901234567890123456789012123456789012345678901 2
12345678901234567890123456789012123456789012345678901 2
12345678901234567890123456789012123456789012345678901 2
12345678901234567890123456789012123456789012345678901 2
2
1
2
12345678901234567890123456789012123456789012345678901
12345678901234567890123456789012123456789012345678901 2
2
12345678901234567890123456789012123456789012345678901
12345678901234567890123456789012123456789012345678901 2
2
12345678901234567890123456789012123456789012345678901
12345678901234567890123456789012123456789012345678901 2
12345678901234567890123456789012123456789012345678901 2
12345678901234567890123456789012123456789012345678901 2
2
254 12345678901234567890123456789012123456789012345678901
123456789012345678901234567890121234567890123456789012

PART

IV
Three Practice GREs
Preface to Practice Tests
Practice Test 1

258

Practice Test 2

282

Practice Test 3

305

Closing Remarks

256

329

PART IV
http://www.xtremepapers.net

Chapter

7
Preface to Practice Tests
123456789012345678901234567890121234567890123456789012
2
12345678901234567890123456789012123456789012345678901
2
12345678901234567890123456789012123456789012345678901
2
12345678901234567890123456789012123456789012345678901
Putting
Ideas
and
Concepts
into
Practice
2345678901234567890123456789012123456789012345678901
12345678901234567890123456789012123456789012345678901 2
12345678901234567890123456789012123456789012345678901
NOTE: Dont start on any of the following three tests until you have read 2
2
12345678901234567890123456789012123456789012345678901
2
12345678901234567890123456789012123456789012345678901
this section.
12345678901234567890123456789012123456789012345678901 2
2
12345678901234567890123456789012123456789012345678901
1 The study materials in this book should have your head swirling with 2
2
12345678901234567890123456789012123456789012345678901
information. Although this might lead to strange dreams featuring QC 2
12345678901234567890123456789012123456789012345678901
2
12345678901234567890123456789012123456789012345678901
problems, on the whole its a positive thing. The following 3 practice tests 2
12345678901234567890123456789012123456789012345678901
12345678901234567890123456789012123456789012345678901
will give you a chance to put some of the knowledge flitting around in your 2
2
12345678901234567890123456789012123456789012345678901
2
12345678901234567890123456789012123456789012345678901
noggin
to
good
use.
2
12345678901234567890123456789012123456789012345678901
2345678901234567890123456789012123456789012345678901
12345678901234567890123456789012123456789012345678901 2
12345678901234567890123456789012123456789012345678901
These tests should be used to give you a chance to test-drive the new 2
2
12345678901234567890123456789012123456789012345678901
2
12345678901234567890123456789012123456789012345678901
strategies
and
ideas
previously
presented
within
this
book.
Its
important
2
1
2345678901234567890123456789012123456789012345678901
12345678901234567890123456789012123456789012345678901
you dont just jump to these questions and start answering them willy-nilly 2
2
12345678901234567890123456789012123456789012345678901
1 before reading all that came before. If you do this, odds are good you will 2
12345678901234567890123456789012123456789012345678901 2
12345678901234567890123456789012123456789012345678901
only help reinforce the faults you had before you started studying for the 2
2
12345678901234567890123456789012123456789012345678901
12345678901234567890123456789012123456789012345678901
GRE. To truly maximize your GRE score, these tests must be approached 2
12345678901234567890123456789012123456789012345678901 2
12345678901234567890123456789012123456789012345678901
with the attitude that you will use them to try out the various different 2
2
12345678901234567890123456789012123456789012345678901
2345678901234567890123456789012123456789012345678901
12345678901234567890123456789012123456789012345678901
points and ideas covered so far. Doing this will help you absorb the 2
2
12345678901234567890123456789012123456789012345678901
2
1 material discussed and make you a better GRE test-taker.
12345678901234567890123456789012123456789012345678901 2
12345678901234567890123456789012123456789012345678901
Make sure you take only one test at a time, and then look over the 2
2
12345678901234567890123456789012123456789012345678901
2
12345678901234567890123456789012123456789012345678901
explanations
to
the
answers.
Everyone
will
want
to
read
the
explanations
2
12345678901234567890123456789012123456789012345678901
2
12345678901234567890123456789012123456789012345678901
for
the
questions
that
they
miss,
but
its
also
a
good
idea
to
look
the
12345678901234567890123456789012123456789012345678901 2
2345678901234567890123456789012123456789012345678901
12345678901234567890123456789012123456789012345678901
answers for the other questions as well. The explanations can give you 2
2
12345678901234567890123456789012123456789012345678901
2
12345678901234567890123456789012123456789012345678901
ideas
about
different
ways
you
can
approach
a
problem;
learning
about
12345678901234567890123456789012123456789012345678901 2
1 these different approaches will earn you points toward your Superior 2
2
12345678901234567890123456789012123456789012345678901
2
12345678901234567890123456789012123456789012345678901
Test-Taker
merit
badge.
Many
people
who
score
poorly
on
standardized
2
12345678901234567890123456789012123456789012345678901
2
12345678901234567890123456789012123456789012345678901
tests
do
so
because
they
are
locked
into
the
mindset
that
there
is
only
one
12345678901234567890123456789012123456789012345678901 2
2345678901234567890123456789012123456789012345678901
2
12345678901234567890123456789012123456789012345678901
1 way to answer every problem. Its as if every problem is a locked door, and 2
2
12345678901234567890123456789012123456789012345678901
only a single correct key can unlock it.
2
12345678901234567890123456789012123456789012345678901
2
12345678901234567890123456789012123456789012345678901
2345678901234567890123456789012123456789012345678901
2
1
12345678901234567890123456789012123456789012345678901 2
12345678901234567890123456789012123456789012345678901 2
12345678901234567890123456789012123456789012345678901 2
123456789012345678901234567890121234567890123456789012
256

http://www.xtremepapers.net

Note

Chapter 7: Preface to Practice Tests

123456789012345678901234567890121234567890123456789012
12345678901234567890123456789012123456789012345678901 2
2
12345678901234567890123456789012123456789012345678901
12345678901234567890123456789012123456789012345678901 2
2
12345678901234567890123456789012123456789012345678901
Its doesnt matter if you take these tests under timed conditions or not,
2
12345678901234567890123456789012123456789012345678901
2
12345678901234567890123456789012123456789012345678901
although
a
good
plan
would
be
to
take
the
first
two
untimed
and
the
third
2
12345678901234567890123456789012123456789012345678901
2345678901234567890123456789012123456789012345678901
2
1 one timed. Your real practice with timed GRE sections will come with the
2
12345678901234567890123456789012123456789012345678901
2
12345678901234567890123456789012123456789012345678901
exams,
not
the
CD,
since
these
mimic
the
adaptive
format.
2
12345678901234567890123456789012123456789012345678901
2
12345678901234567890123456789012123456789012345678901
2345678901234567890123456789012123456789012345678901
2
1
2
12345678901234567890123456789012123456789012345678901
If
you
get
nothing
else
from
this
book,
you
should
learn
that
this
mode
of
2
12345678901234567890123456789012123456789012345678901
2
12345678901234567890123456789012123456789012345678901
thinking
is
incorrect.
There
are
skeleton
keys
(general
facts
you
can
use
to
2
12345678901234567890123456789012123456789012345678901
2345678901234567890123456789012123456789012345678901
1 come to the right answer), lock-picking equipment (strategies like 2
2
12345678901234567890123456789012123456789012345678901
2
12345678901234567890123456789012123456789012345678901
estimating
and
process
of
elimination),
and
if
worse
comes
to
worse,
you
2
12345678901234567890123456789012123456789012345678901
2
12345678901234567890123456789012123456789012345678901
can
just
try
kicking
the
door
down
with
your
boot
(guessing
as
time
runs
2345678901234567890123456789012123456789012345678901
2
1
12345678901234567890123456789012123456789012345678901
out). These wont work every time, but theyll work more times than you 2
2
12345678901234567890123456789012123456789012345678901
2
12345678901234567890123456789012123456789012345678901
might think.
2
12345678901234567890123456789012123456789012345678901
2345678901234567890123456789012123456789012345678901
2
12345678901234567890123456789012123456789012345678901
Do your best, and remember that the goal is to learn something from every 2
12345678901234567890123456789012123456789012345678901
2
12345678901234567890123456789012123456789012345678901
missed question, and even from some that you answered correctly.
2
12345678901234567890123456789012123456789012345678901
12345678901234567890123456789012123456789012345678901 22
12345678901234567890123456789012123456789012345678901 2
1
2
12345678901234567890123456789012123456789012345678901
2
12345678901234567890123456789012123456789012345678901
2
12345678901234567890123456789012123456789012345678901
2345678901234567890123456789012123456789012345678901
12345678901234567890123456789012123456789012345678901 22
12345678901234567890123456789012123456789012345678901 2
12345678901234567890123456789012123456789012345678901 2
12345678901234567890123456789012123456789012345678901 2
12345678901234567890123456789012123456789012345678901 2
12345678901234567890123456789012123456789012345678901 2
12345678901234567890123456789012123456789012345678901 2
12345678901234567890123456789012123456789012345678901 2
12345678901234567890123456789012123456789012345678901 2
1
2
12345678901234567890123456789012123456789012345678901
2
12345678901234567890123456789012123456789012345678901
12345678901234567890123456789012123456789012345678901 2
12345678901234567890123456789012123456789012345678901 2
2
12345678901234567890123456789012123456789012345678901
2
12345678901234567890123456789012123456789012345678901
2
12345678901234567890123456789012123456789012345678901
12345678901234567890123456789012123456789012345678901 2
2
12345678901234567890123456789012123456789012345678901
2
12345678901234567890123456789012123456789012345678901
2345678901234567890123456789012123456789012345678901
12345678901234567890123456789012123456789012345678901 22
12345678901234567890123456789012123456789012345678901 2
1
12345678901234567890123456789012123456789012345678901 2
12345678901234567890123456789012123456789012345678901 2
2
12345678901234567890123456789012123456789012345678901
2
12345678901234567890123456789012123456789012345678901
2
12345678901234567890123456789012123456789012345678901
2
12345678901234567890123456789012123456789012345678901
12345678901234567890123456789012123456789012345678901 2
2345678901234567890123456789012123456789012345678901
12345678901234567890123456789012123456789012345678901 22
12345678901234567890123456789012123456789012345678901 2
12345678901234567890123456789012123456789012345678901 2
12345678901234567890123456789012123456789012345678901 2
1
2
12345678901234567890123456789012123456789012345678901
2
12345678901234567890123456789012123456789012345678901
2
12345678901234567890123456789012123456789012345678901
2
12345678901234567890123456789012123456789012345678901
12345678901234567890123456789012123456789012345678901 2
2345678901234567890123456789012123456789012345678901
12345678901234567890123456789012123456789012345678901 22
1
2
12345678901234567890123456789012123456789012345678901
12345678901234567890123456789012123456789012345678901 2
2
12345678901234567890123456789012123456789012345678901
12345678901234567890123456789012123456789012345678901 2
12345678901234567890123456789012123456789012345678901 2
12345678901234567890123456789012123456789012345678901 2
12345678901234567890123456789012123456789012345678901 2 257
123456789012345678901234567890121234567890123456789012

http://www.xtremepapers.net

Practice Test

1
1234567890123456789012345678901212345678901234567890123456789012123456
3456789012345678901234567890121234567890123456789012345678901212345 6
12
3456789012345678901234567890121234567890123456789012345678901212345 6
12
3456789012345678901234567890121234567890123456789012345678901212345
6
12
Quantitative
2
6
123456789012345678901234567890121234567890123456789012345678901212345
6
123456789012345678901234567890121234567890123456789012345678901212345
6
123456789012345678901234567890121234567890123456789012345678901212345
28
Questions45
Minutes
6
123456789012345678901234567890121234567890123456789012345678901212345
6
123456789012345678901234567890121234567890123456789012345678901212345
General
Information
3456789012345678901234567890121234567890123456789012345678901212345
123456789012345678901234567890121234567890123456789012345678901212345 66
1
3456789012345678901234567890121234567890123456789012345678901212345
12
1. The test has 28 questions. If you decide to take this exam under timed conditions, you have 6
3456789012345678901234567890121234567890123456789012345678901212345
6
12
3456789012345678901234567890121234567890123456789012345678901212345
6
12
45 minutes to complete the section.
2
3456789012345678901234567890121234567890123456789012345678901212345
123456789012345678901234567890121234567890123456789012345678901212345 66
123456789012345678901234567890121234567890123456789012345678901212345
2. All numbers used are real numbers.
6
123456789012345678901234567890121234567890123456789012345678901212345
123456789012345678901234567890121234567890123456789012345678901212345 66
123456789012345678901234567890121234567890123456789012345678901212345
3. All angle measurements can be assumed to be positive unless otherwise noted.
6
123456789012345678901234567890121234567890123456789012345678901212345
123456789012345678901234567890121234567890123456789012345678901212345 66
123456789012345678901234567890121234567890123456789012345678901212345
4. All figures lie in the same plane unless otherwise noted.
6
123456789012345678901234567890121234567890123456789012345678901212345
6
1
2
3456789012345678901234567890121234567890123456789012345678901212345
123456789012345678901234567890121234567890123456789012345678901212345
5. Drawings that accompany questions are intended to provide information useful in 6
6
123456789012345678901234567890121234567890123456789012345678901212345
6
1
answering the question. The figures are drawn closely to scale unless otherwise noted.
123456789012345678901234567890121234567890123456789012345678901212345 6
3456789012345678901234567890121234567890123456789012345678901212345 6
12
3456789012345678901234567890121234567890123456789012345678901212345 6
12
3456789012345678901234567890121234567890123456789012345678901212345
6
12
Directions: For questions 114, Column A and Column B will have one quantity each.
6
123456789012345678901234567890121234567890123456789012345678901212345
3456789012345678901234567890121234567890123456789012345678901212345
6
12
Your goal is to compare the quantities in each column and choose
3456789012345678901234567890121234567890123456789012345678901212345 6
12
2
3456789012345678901234567890121234567890123456789012345678901212345
6
123456789012345678901234567890121234567890123456789012345678901212345
A if the quantity in Column A is greater
6
123456789012345678901234567890121234567890123456789012345678901212345
6
1
B if the quantity in Column B is greater
6
123456789012345678901234567890121234567890123456789012345678901212345
6
123456789012345678901234567890121234567890123456789012345678901212345
C
if
the
two
quantities
are
equal
2
3456789012345678901234567890121234567890123456789012345678901212345
123456789012345678901234567890121234567890123456789012345678901212345 66
123456789012345678901234567890121234567890123456789012345678901212345
D if the relationship between the two quantities cannot be determined from the
6
123456789012345678901234567890121234567890123456789012345678901212345
6
123456789012345678901234567890121234567890123456789012345678901212345
information
in
the
problem
6
1
2
3456789012345678901234567890121234567890123456789012345678901212345
123456789012345678901234567890121234567890123456789012345678901212345 66
123456789012345678901234567890121234567890123456789012345678901212345
On some questions, information about one or both quantities will be given. This informa6
123456789012345678901234567890121234567890123456789012345678901212345
6
123456789012345678901234567890121234567890123456789012345678901212345
tion
will
be
centered
above
the
two
columns.
6
1
2
3456789012345678901234567890121234567890123456789012345678901212345
123456789012345678901234567890121234567890123456789012345678901212345 66
123456789012345678901234567890121234567890123456789012345678901212345 6
123456789012345678901234567890121234567890123456789012345678901212345 6
123456789012345678901234567890121234567890123456789012345678901212345 6
1
3456789012345678901234567890121234567890123456789012345678901212345 6
12
123456789012345678901234567890121234567890123456789012345678901212345 6
3456789012345678901234567890121234567890123456789012345678901212345 6
12
123456789012345678901234567890121234567890123456789012345678901212345 6
3456789012345678901234567890121234567890123456789012345678901212345 6
12
123456789012345678901234567890121234567890123456789012345678901212345 6
123456789012345678901234567890121234567890123456789012345678901212345 6
123456789012345678901234567890121234567890123456789012345678901212345 6
123456789012345678901234567890121234567890123456789012345678901212345 6
1234567890123456789012345678901212345678901234567890123456789012123456
258

http://www.xtremepapers.net

TEST 1
1234567890123456789012345678901212345678901234567890123456789012123456
123456789012345678901234567890121234567890123456789012345678901212345 6
3456789012345678901234567890121234567890123456789012345678901212345 6
12
6
123456789012345678901234567890121234567890123456789012345678901212345
Column A
Column B
Column A
Column B
3456789012345678901234567890121234567890123456789012345678901212345
6
12
3456789012345678901234567890121234567890123456789012345678901212345
6
12
1.
h56
4.
|x| 5 3, |y| 5 4
6
123456789012345678901234567890121234567890123456789012345678901212345
3456789012345678901234567890121234567890123456789012345678901212345 6
12
2
3456789012345678901234567890121234567890123456789012345678901212345
6
1
The number of
The number of
y2x
1
3456789012345678901234567890121234567890123456789012345678901212345
6
12
3456789012345678901234567890121234567890123456789012345678901212345
6
12
minutes
in
h
hours
degrees
in
the
sum
3456789012345678901234567890121234567890123456789012345678901212345 6
12
3456789012345678901234567890121234567890123456789012345678901212345
6
12
of the angles of
6
123456789012345678901234567890121234567890123456789012345678901212345
3456789012345678901234567890121234567890123456789012345678901212345
6
12
a
square
3456789012345678901234567890121234567890123456789012345678901212345 6
12
3456789012345678901234567890121234567890123456789012345678901212345 6
12
3456789012345678901234567890121234567890123456789012345678901212345 6
12
2
1 3456789012345678901234567890121234567890123456789012345678901212345 6
3456789012345678901234567890121234567890123456789012345678901212345 6
12
3456789012345678901234567890121234567890123456789012345678901212345 6
12
3456789012345678901234567890121234567890123456789012345678901212345
6
12
5. The sum of the
3456789012345678901234567890121234567890123456789012345678901212345
6
12
49 1 =8
=
2
3456789012345678901234567890121234567890123456789012345678901212345
6
1
three smallest
3456789012345678901234567890121234567890123456789012345678901212345
6
12
3456789012345678901234567890121234567890123456789012345678901212345
6
12
prime
numbers
3456789012345678901234567890121234567890123456789012345678901212345 6
12
3456789012345678901234567890121234567890123456789012345678901212345
6
12
2.
3456789012345678901234567890121234567890123456789012345678901212345
6
12
3456789012345678901234567890121234567890123456789012345678901212345 6
12
3456789012345678901234567890121234567890123456789012345678901212345 6
12
3456789012345678901234567890121234567890123456789012345678901212345 6
12
3456789012345678901234567890121234567890123456789012345678901212345 6
12
2
6
123456789012345678901234567890121234567890123456789012345678901212345
1 3456789012345678901234567890121234567890123456789012345678901212345 6
3456789012345678901234567890121234567890123456789012345678901212345 6
12
3456789012345678901234567890121234567890123456789012345678901212345 6
12
3456789012345678901234567890121234567890123456789012345678901212345 6
12
2
6
123456789012345678901234567890121234567890123456789012345678901212345
6.
6
123456789012345678901234567890121234567890123456789012345678901212345
6
123456789012345678901234567890121234567890123456789012345678901212345
6
123456789012345678901234567890121234567890123456789012345678901212345
6
123456789012345678901234567890121234567890123456789012345678901212345
3456789012345678901234567890121234567890123456789012345678901212345
123456789012345678901234567890121234567890123456789012345678901212345 66
123456789012345678901234567890121234567890123456789012345678901212345
6
123456789012345678901234567890121234567890123456789012345678901212345
a2
The hypotenuse
6
123456789012345678901234567890121234567890123456789012345678901212345
6
1
squared minus
2
3456789012345678901234567890121234567890123456789012345678901212345
123456789012345678901234567890121234567890123456789012345678901212345 66
123456789012345678901234567890121234567890123456789012345678901212345
itself
6
1
123456789012345678901234567890121234567890123456789012345678901212345 6
3456789012345678901234567890121234567890123456789012345678901212345 6
12
3456789012345678901234567890121234567890123456789012345678901212345 6
12
3456789012345678901234567890121234567890123456789012345678901212345
6
12
a2 1 ab 1 a
180
6
123456789012345678901234567890121234567890123456789012345678901212345
3456789012345678901234567890121234567890123456789012345678901212345
6
12
a
3456789012345678901234567890121234567890123456789012345678901212345
6
12
2
3456789012345678901234567890121234567890123456789012345678901212345
123456789012345678901234567890121234567890123456789012345678901212345 66
123456789012345678901234567890121234567890123456789012345678901212345 6
1
123456789012345678901234567890121234567890123456789012345678901212345 6
6
123456789012345678901234567890121234567890123456789012345678901212345
3. A square has a side that is h inches long.
3456789012345678901234567890121234567890123456789012345678901212345
6
12
3456789012345678901234567890121234567890123456789012345678901212345
6
12
2
3456789012345678901234567890121234567890123456789012345678901212345
6
12
The number of
h
3456789012345678901234567890121234567890123456789012345678901212345
6
12
inches along the
6
123456789012345678901234567890121234567890123456789012345678901212345
2
3456789012345678901234567890121234567890123456789012345678901212345
6
123456789012345678901234567890121234567890123456789012345678901212345
perimeter of the
6
123456789012345678901234567890121234567890123456789012345678901212345
6
123456789012345678901234567890121234567890123456789012345678901212345
square
6
123456789012345678901234567890121234567890123456789012345678901212345
6
1
2
3456789012345678901234567890121234567890123456789012345678901212345
123456789012345678901234567890121234567890123456789012345678901212345 66
123456789012345678901234567890121234567890123456789012345678901212345 6
123456789012345678901234567890121234567890123456789012345678901212345 6
123456789012345678901234567890121234567890123456789012345678901212345 6
1
3456789012345678901234567890121234567890123456789012345678901212345 6
12
123456789012345678901234567890121234567890123456789012345678901212345 6
3456789012345678901234567890121234567890123456789012345678901212345 6
12
123456789012345678901234567890121234567890123456789012345678901212345 6
3456789012345678901234567890121234567890123456789012345678901212345 6
12
123456789012345678901234567890121234567890123456789012345678901212345 6
123456789012345678901234567890121234567890123456789012345678901212345 6
123456789012345678901234567890121234567890123456789012345678901212345 6
123456789012345678901234567890121234567890123456789012345678901212345 6 259
1234567890123456789012345678901212345678901234567890123456789012123456

http://www.xtremepapers.net

Part IV: Three Practice GREs

1234567890123456789012345678901212345678901234567890123456789012123456
123456789012345678901234567890121234567890123456789012345678901212345 6
3456789012345678901234567890121234567890123456789012345678901212345 6
12
6
123456789012345678901234567890121234567890123456789012345678901212345
Column A
Column B
Column A
Column B
3456789012345678901234567890121234567890123456789012345678901212345
6
12
3456789012345678901234567890121234567890123456789012345678901212345
6
12
7.
Jackie, Beth, and Allison each have
10. A triangle has one side of length 4 and
6
123456789012345678901234567890121234567890123456789012345678901212345
3456789012345678901234567890121234567890123456789012345678901212345
6
12
different
cars
and
park
in
a
three
spot
another
of
length
10.
2
3456789012345678901234567890121234567890123456789012345678901212345
6
1
3456789012345678901234567890121234567890123456789012345678901212345
6
12
parking lot behind their house.
3456789012345678901234567890121234567890123456789012345678901212345
6
12
5
The
length
of
the
3456789012345678901234567890121234567890123456789012345678901212345 6
12
3456789012345678901234567890121234567890123456789012345678901212345
6
12
The number of
third side of the
7
6
123456789012345678901234567890121234567890123456789012345678901212345
3456789012345678901234567890121234567890123456789012345678901212345
6
12
possible
different
triangle
3456789012345678901234567890121234567890123456789012345678901212345 6
12
3456789012345678901234567890121234567890123456789012345678901212345
6
12
arrangements of
3456789012345678901234567890121234567890123456789012345678901212345
6
12
2
3456789012345678901234567890121234567890123456789012345678901212345
6
1
their cars in the
3456789012345678901234567890121234567890123456789012345678901212345
6
12
3456789012345678901234567890121234567890123456789012345678901212345
6
12
three
spots
3456789012345678901234567890121234567890123456789012345678901212345 6
12
3456789012345678901234567890121234567890123456789012345678901212345 6
12
123456789012345678901234567890121234567890123456789012345678901212345 6
3456789012345678901234567890121234567890123456789012345678901212345 6
12
3456789012345678901234567890121234567890123456789012345678901212345 6
12
3456789012345678901234567890121234567890123456789012345678901212345 6
12
3456789012345678901234567890121234567890123456789012345678901212345
6
12
11. The median of the
The mean of the
2
6
123456789012345678901234567890121234567890123456789012345678901212345
6
123456789012345678901234567890121234567890123456789012345678901212345
even
integers
10
even
integers
10
6
123456789012345678901234567890121234567890123456789012345678901212345
6
123456789012345678901234567890121234567890123456789012345678901212345
through
18
through
18
6
123456789012345678901234567890121234567890123456789012345678901212345
3456789012345678901234567890121234567890123456789012345678901212345
6
123456789012345678901234567890121234567890123456789012345678901212345
inclusive
p
inclusive
6
1 8. Twice the circum3456789012345678901234567890121234567890123456789012345678901212345
6
12
ference
of
a
circle
3456789012345678901234567890121234567890123456789012345678901212345 6
12
3456789012345678901234567890121234567890123456789012345678901212345
6
12
divided by two
3456789012345678901234567890121234567890123456789012345678901212345
6
12
3456789012345678901234567890121234567890123456789012345678901212345
6
12
times its diameter
3456789012345678901234567890121234567890123456789012345678901212345 6
12
3456789012345678901234567890121234567890123456789012345678901212345 6
12
3456789012345678901234567890121234567890123456789012345678901212345 6
12
2
6
123456789012345678901234567890121234567890123456789012345678901212345
6
123456789012345678901234567890121234567890123456789012345678901212345
6
123456789012345678901234567890121234567890123456789012345678901212345
6
123456789012345678901234567890121234567890123456789012345678901212345
12.
1 3456789012345678901234567890121234567890123456789012345678901212345 6
2
3456789012345678901234567890121234567890123456789012345678901212345
123456789012345678901234567890121234567890123456789012345678901212345 66
123456789012345678901234567890121234567890123456789012345678901212345 6
1
123456789012345678901234567890121234567890123456789012345678901212345 6
3456789012345678901234567890121234567890123456789012345678901212345
6
12
9. One is less than a, and a is less than b.
3456789012345678901234567890121234567890123456789012345678901212345
6
12
3456789012345678901234567890121234567890123456789012345678901212345
6
12
2
6
123456789012345678901234567890121234567890123456789012345678901212345
a
b
1
a
1
3456789012345678901234567890121234567890123456789012345678901212345
6
12
3
b
2
4
2
12a
2
3456789012345678901234567890121234567890123456789012345678901212345
6
12
2
a
6
3b
2
3456789012345678901234567890121234567890123456789012345678901212345
123456789012345678901234567890121234567890123456789012345678901212345 66
123456789012345678901234567890121234567890123456789012345678901212345 6
1
123456789012345678901234567890121234567890123456789012345678901212345 6
123456789012345678901234567890121234567890123456789012345678901212345 6
3456789012345678901234567890121234567890123456789012345678901212345 6
12
3456789012345678901234567890121234567890123456789012345678901212345 6
12
3456789012345678901234567890121234567890123456789012345678901212345 6
12
3456789012345678901234567890121234567890123456789012345678901212345 6
12
123456789012345678901234567890121234567890123456789012345678901212345 6
2
6
123456789012345678901234567890121234567890123456789012345678901212345
The length of GH
The slope of GH
6
123456789012345678901234567890121234567890123456789012345678901212345
6
123456789012345678901234567890121234567890123456789012345678901212345
times
24
6
123456789012345678901234567890121234567890123456789012345678901212345
1 3456789012345678901234567890121234567890123456789012345678901212345 6
2
3456789012345678901234567890121234567890123456789012345678901212345
123456789012345678901234567890121234567890123456789012345678901212345 66
123456789012345678901234567890121234567890123456789012345678901212345 6
123456789012345678901234567890121234567890123456789012345678901212345 6
123456789012345678901234567890121234567890123456789012345678901212345 6
1
3456789012345678901234567890121234567890123456789012345678901212345 6
12
123456789012345678901234567890121234567890123456789012345678901212345 6
3456789012345678901234567890121234567890123456789012345678901212345 6
12
123456789012345678901234567890121234567890123456789012345678901212345 6
3456789012345678901234567890121234567890123456789012345678901212345 6
12
123456789012345678901234567890121234567890123456789012345678901212345 6
123456789012345678901234567890121234567890123456789012345678901212345 6
123456789012345678901234567890121234567890123456789012345678901212345 66
260 123456789012345678901234567890121234567890123456789012345678901212345
1234567890123456789012345678901212345678901234567890123456789012123456

GS D F S D G

www.petersons.com

http://www.xtremepapers.net

TEST 1
1234567890123456789012345678901212345678901234567890123456789012123456
123456789012345678901234567890121234567890123456789012345678901212345 6
3456789012345678901234567890121234567890123456789012345678901212345 6
12
16.
6
123456789012345678901234567890121234567890123456789012345678901212345
Column A
Column B
y
3456789012345678901234567890121234567890123456789012345678901212345
6
12
3456789012345678901234567890121234567890123456789012345678901212345
6
12
13.
A and B are distinct digits.
6
123456789012345678901234567890121234567890123456789012345678901212345
3456789012345678901234567890121234567890123456789012345678901212345 6
12
2
3456789012345678901234567890121234567890123456789012345678901212345
6
1
|A 1 A 1 B|
28
3456789012345678901234567890121234567890123456789012345678901212345
6
12
3456789012345678901234567890121234567890123456789012345678901212345 6
12
3456789012345678901234567890121234567890123456789012345678901212345 6
12
3456789012345678901234567890121234567890123456789012345678901212345 6
12
2
1 3456789012345678901234567890121234567890123456789012345678901212345 6
3456789012345678901234567890121234567890123456789012345678901212345
6
12
x
3456789012345678901234567890121234567890123456789012345678901212345
6
12
0
3456789012345678901234567890121234567890123456789012345678901212345 6
12
3456789012345678901234567890121234567890123456789012345678901212345 6
12
123456789012345678901234567890121234567890123456789012345678901212345 6
3456789012345678901234567890121234567890123456789012345678901212345 6
12
3456789012345678901234567890121234567890123456789012345678901212345 6
12
3456789012345678901234567890121234567890123456789012345678901212345
6
12
14. The local pet store has 73 cats and dogs.
3456789012345678901234567890121234567890123456789012345678901212345
6
12
2
3456789012345678901234567890121234567890123456789012345678901212345
6
1
40 of them are female, and 35 are
3456789012345678901234567890121234567890123456789012345678901212345
6
12
The linear equation that accurately
3456789012345678901234567890121234567890123456789012345678901212345
6
12
dogs.
There
are
9
less
male
cats
3456789012345678901234567890121234567890123456789012345678901212345
6
12
corresponds
to
line
r
is
3456789012345678901234567890121234567890123456789012345678901212345
6
12
than
male
dogs.
2
3456789012345678901234567890121234567890123456789012345678901212345
123456789012345678901234567890121234567890123456789012345678901212345 66
123456789012345678901234567890121234567890123456789012345678901212345
A. y 1 2 5 22x
6
123456789012345678901234567890121234567890123456789012345678901212345
The number of
The number of
6
123456789012345678901234567890121234567890123456789012345678901212345
B.
y
2
2
5
22x
6
123456789012345678901234567890121234567890123456789012345678901212345
female
cats
male
dogs
6
123456789012345678901234567890121234567890123456789012345678901212345
C. y 1 2 5 2x
6
1
3456789012345678901234567890121234567890123456789012345678901212345
6
12
D.
y
2
2
5
2x
3456789012345678901234567890121234567890123456789012345678901212345 6
12
3456789012345678901234567890121234567890123456789012345678901212345
6
12
E. y 1 2 5 x
3456789012345678901234567890121234567890123456789012345678901212345
6
12
3456789012345678901234567890121234567890123456789012345678901212345 6
12
3456789012345678901234567890121234567890123456789012345678901212345 6
12
3456789012345678901234567890121234567890123456789012345678901212345
6
12
Directions: Select the best answer for each
Questions 1718 refer to the following graph.
3456789012345678901234567890121234567890123456789012345678901212345
6
12
2
3456789012345678901234567890121234567890123456789012345678901212345
6
123456789012345678901234567890121234567890123456789012345678901212345
of the following questions.
6
123456789012345678901234567890121234567890123456789012345678901212345
123456789012345678901234567890121234567890123456789012345678901212345 66
123456789012345678901234567890121234567890123456789012345678901212345
6
1 15. If 2x2 5 2x 2 15, then what does x
3456789012345678901234567890121234567890123456789012345678901212345 6
12
3456789012345678901234567890121234567890123456789012345678901212345
6
12
equal?
6
123456789012345678901234567890121234567890123456789012345678901212345
123456789012345678901234567890121234567890123456789012345678901212345 6
3456789012345678901234567890121234567890123456789012345678901212345
6
12
I. 23
3456789012345678901234567890121234567890123456789012345678901212345
6
12
3456789012345678901234567890121234567890123456789012345678901212345 6
12
6
123456789012345678901234567890121234567890123456789012345678901212345
II. 3
3456789012345678901234567890121234567890123456789012345678901212345
6
12
3456789012345678901234567890121234567890123456789012345678901212345
6
12
III.
5
2
3456789012345678901234567890121234567890123456789012345678901212345
123456789012345678901234567890121234567890123456789012345678901212345 66
123456789012345678901234567890121234567890123456789012345678901212345 6
1
A. I only
6
123456789012345678901234567890121234567890123456789012345678901212345
6
123456789012345678901234567890121234567890123456789012345678901212345
B.
II
only
2
3456789012345678901234567890121234567890123456789012345678901212345
123456789012345678901234567890121234567890123456789012345678901212345 66
123456789012345678901234567890121234567890123456789012345678901212345
C. I and III only
6
123456789012345678901234567890121234567890123456789012345678901212345
6
123456789012345678901234567890121234567890123456789012345678901212345
17. Which month had the fewest reported
D.
II
and
III
only
6
1
2
3456789012345678901234567890121234567890123456789012345678901212345
6
123456789012345678901234567890121234567890123456789012345678901212345
incidences of flu?
E. I, II, and III
6
123456789012345678901234567890121234567890123456789012345678901212345
6
123456789012345678901234567890121234567890123456789012345678901212345
A. January 1962
6
123456789012345678901234567890121234567890123456789012345678901212345
6
1
B.
June
1962
2
3456789012345678901234567890121234567890123456789012345678901212345
123456789012345678901234567890121234567890123456789012345678901212345 66
123456789012345678901234567890121234567890123456789012345678901212345
C. January 1963
6
123456789012345678901234567890121234567890123456789012345678901212345
6
123456789012345678901234567890121234567890123456789012345678901212345
D.
February
1963
6
1
2
3456789012345678901234567890121234567890123456789012345678901212345
6
123456789012345678901234567890121234567890123456789012345678901212345
E. April 1963
6
1
3456789012345678901234567890121234567890123456789012345678901212345 6
12
123456789012345678901234567890121234567890123456789012345678901212345 6
3456789012345678901234567890121234567890123456789012345678901212345 6
12
123456789012345678901234567890121234567890123456789012345678901212345 6
123456789012345678901234567890121234567890123456789012345678901212345 6
123456789012345678901234567890121234567890123456789012345678901212345 6
123456789012345678901234567890121234567890123456789012345678901212345 6 261
1234567890123456789012345678901212345678901234567890123456789012123456

http://www.xtremepapers.net

Part IV: Three Practice GREs

1234567890123456789012345678901212345678901234567890123456789012123456
123456789012345678901234567890121234567890123456789012345678901212345 6
3456789012345678901234567890121234567890123456789012345678901212345 6
12
18. Which two-month period had the greatest
Questions 2123 are based on the following
6
123456789012345678901234567890121234567890123456789012345678901212345
3456789012345678901234567890121234567890123456789012345678901212345
6
12
number of reported child incidences of flu?
graphs.
3456789012345678901234567890121234567890123456789012345678901212345
6
12
123456789012345678901234567890121234567890123456789012345678901212345 6
3456789012345678901234567890121234567890123456789012345678901212345
6
12
A. FebruaryMarch 1962
6
123456789012345678901234567890121234567890123456789012345678901212345
3456789012345678901234567890121234567890123456789012345678901212345
6
12
B.
MarchApril
1962
3456789012345678901234567890121234567890123456789012345678901212345 6
12
3456789012345678901234567890121234567890123456789012345678901212345
6
12
C. AprilMay 1962
3456789012345678901234567890121234567890123456789012345678901212345
6
12
2
3456789012345678901234567890121234567890123456789012345678901212345
6
1
D. AprilMay 1963
3456789012345678901234567890121234567890123456789012345678901212345
6
12
3456789012345678901234567890121234567890123456789012345678901212345
6
12
E.
MayJune
1963
3456789012345678901234567890121234567890123456789012345678901212345 6
12
3456789012345678901234567890121234567890123456789012345678901212345 6
12
123456789012345678901234567890121234567890123456789012345678901212345 6
3456789012345678901234567890121234567890123456789012345678901212345
6
12
19.
3456789012345678901234567890121234567890123456789012345678901212345
6
12
3456789012345678901234567890121234567890123456789012345678901212345 6
12
3456789012345678901234567890121234567890123456789012345678901212345 6
12
123456789012345678901234567890121234567890123456789012345678901212345 6
3456789012345678901234567890121234567890123456789012345678901212345 6
12
3456789012345678901234567890121234567890123456789012345678901212345 6
12
3456789012345678901234567890121234567890123456789012345678901212345 6
12
3456789012345678901234567890121234567890123456789012345678901212345 6
12
2
6
123456789012345678901234567890121234567890123456789012345678901212345
6
123456789012345678901234567890121234567890123456789012345678901212345
6
123456789012345678901234567890121234567890123456789012345678901212345
6
123456789012345678901234567890121234567890123456789012345678901212345
6
123456789012345678901234567890121234567890123456789012345678901212345
If the area of the top of the rectangular
3456789012345678901234567890121234567890123456789012345678901212345
123456789012345678901234567890121234567890123456789012345678901212345 66
1
box is 15, what is the surface area of the
3456789012345678901234567890121234567890123456789012345678901212345
12
21. In which quarter was the ratio of criminal 6
entire figure?
3456789012345678901234567890121234567890123456789012345678901212345
6
12
3456789012345678901234567890121234567890123456789012345678901212345
6
12
to
civil
cases
the
greatest?
2
3456789012345678901234567890121234567890123456789012345678901212345
6
123456789012345678901234567890121234567890123456789012345678901212345
A. 69
6
123456789012345678901234567890121234567890123456789012345678901212345
st
6
123456789012345678901234567890121234567890123456789012345678901212345
A.
1
quarter,
1970
B.
72
6
123456789012345678901234567890121234567890123456789012345678901212345
nd
6
123456789012345678901234567890121234567890123456789012345678901212345
B. 2 quarter, 1970
C.
78
6
123456789012345678901234567890121234567890123456789012345678901212345
rd
6
123456789012345678901234567890121234567890123456789012345678901212345
C. 3 quarter, 1970
D.
85
6
123456789012345678901234567890121234567890123456789012345678901212345
th
6
123456789012345678901234567890121234567890123456789012345678901212345
D. 4 quarter, 1970
E.
90
6
1
st
2
3456789012345678901234567890121234567890123456789012345678901212345
6
123456789012345678901234567890121234567890123456789012345678901212345
E. 1 quarter, 1971
6
123456789012345678901234567890121234567890123456789012345678901212345
6
1 20. What is the median minus the mode of the
6
123456789012345678901234567890121234567890123456789012345678901212345
22.
Which
of
the
following
best
approximates
3456789012345678901234567890121234567890123456789012345678901212345
6
12
following number set {4, 16, 7, 2, 7, 10}?
2
3456789012345678901234567890121234567890123456789012345678901212345
6
123456789012345678901234567890121234567890123456789012345678901212345
the ratio of female lawyers in Canada in
6
123456789012345678901234567890121234567890123456789012345678901212345
A. 23
6
1
1970
practicing
Civil
Law
to
Criminal
3456789012345678901234567890121234567890123456789012345678901212345
6
12
B.
0
3456789012345678901234567890121234567890123456789012345678901212345
6
12
Law?
2
3456789012345678901234567890121234567890123456789012345678901212345
6
123456789012345678901234567890121234567890123456789012345678901212345
C. 3
6
123456789012345678901234567890121234567890123456789012345678901212345
6
1
A.
1:2
D.
5
123456789012345678901234567890121234567890123456789012345678901212345 6
6
123456789012345678901234567890121234567890123456789012345678901212345
B. 4:1
E. 9
3456789012345678901234567890121234567890123456789012345678901212345
6
12
3456789012345678901234567890121234567890123456789012345678901212345
6
12
C. 1:3
3456789012345678901234567890121234567890123456789012345678901212345 6
12
3456789012345678901234567890121234567890123456789012345678901212345
6
12
D. 5:1
6
123456789012345678901234567890121234567890123456789012345678901212345
2
3456789012345678901234567890121234567890123456789012345678901212345
6
123456789012345678901234567890121234567890123456789012345678901212345
E. 1:4
123456789012345678901234567890121234567890123456789012345678901212345 66
123456789012345678901234567890121234567890123456789012345678901212345 6
123456789012345678901234567890121234567890123456789012345678901212345 6
1
3456789012345678901234567890121234567890123456789012345678901212345 6
12
3456789012345678901234567890121234567890123456789012345678901212345 6
12
3456789012345678901234567890121234567890123456789012345678901212345 6
12
3456789012345678901234567890121234567890123456789012345678901212345 6
12
123456789012345678901234567890121234567890123456789012345678901212345 6
2
6
123456789012345678901234567890121234567890123456789012345678901212345
1 3456789012345678901234567890121234567890123456789012345678901212345 6
3456789012345678901234567890121234567890123456789012345678901212345 6
12
123456789012345678901234567890121234567890123456789012345678901212345 6
3456789012345678901234567890121234567890123456789012345678901212345 6
12
123456789012345678901234567890121234567890123456789012345678901212345 6
123456789012345678901234567890121234567890123456789012345678901212345 6
123456789012345678901234567890121234567890123456789012345678901212345 66
262 123456789012345678901234567890121234567890123456789012345678901212345
1234567890123456789012345678901212345678901234567890123456789012123456

www.petersons.com

http://www.xtremepapers.net

TEST 1
1234567890123456789012345678901212345678901234567890123456789012123456
123456789012345678901234567890121234567890123456789012345678901212345 6
3456789012345678901234567890121234567890123456789012345678901212345 6
12
26. The average fifth grader scored a 15 on
23. Which field of law in Canada had the
6
123456789012345678901234567890121234567890123456789012345678901212345
3456789012345678901234567890121234567890123456789012345678901212345
6
12
the GST test. The average sixth grader
greatest number of practitioners in 1970?
3456789012345678901234567890121234567890123456789012345678901212345
6
12
6
123456789012345678901234567890121234567890123456789012345678901212345
scored
18
on
the
GST
test.
There
are
half
3456789012345678901234567890121234567890123456789012345678901212345
6
12
A.
Criminal
Law
2
3456789012345678901234567890121234567890123456789012345678901212345
6
1
as many sixth graders as fifth graders.
3456789012345678901234567890121234567890123456789012345678901212345
6
12
B.
Civil
Law
3456789012345678901234567890121234567890123456789012345678901212345
6
12
What
is
the
overall
average
score?
3456789012345678901234567890121234567890123456789012345678901212345
6
12
C.
Corporate
Law
3456789012345678901234567890121234567890123456789012345678901212345 6
12
2
3456789012345678901234567890121234567890123456789012345678901212345
6
1
A. 15.5
D. Real Estate Law
3456789012345678901234567890121234567890123456789012345678901212345
6
12
3456789012345678901234567890121234567890123456789012345678901212345
6
12
B.
16
E.
Public
Interest
Law
3456789012345678901234567890121234567890123456789012345678901212345 6
12
3456789012345678901234567890121234567890123456789012345678901212345
6
12
C. 16.5
6
123456789012345678901234567890121234567890123456789012345678901212345
3456789012345678901234567890121234567890123456789012345678901212345
6
12
24.
D.
17
3456789012345678901234567890121234567890123456789012345678901212345 6
12
3456789012345678901234567890121234567890123456789012345678901212345
6
12
E. 17.5
3456789012345678901234567890121234567890123456789012345678901212345
6
12
2
3456789012345678901234567890121234567890123456789012345678901212345
6
1
3456789012345678901234567890121234567890123456789012345678901212345
6
12
27.
3456789012345678901234567890121234567890123456789012345678901212345
6
12
3456789012345678901234567890121234567890123456789012345678901212345 6
12
3456789012345678901234567890121234567890123456789012345678901212345 6
12
3456789012345678901234567890121234567890123456789012345678901212345 6
12
3456789012345678901234567890121234567890123456789012345678901212345 6
12
3456789012345678901234567890121234567890123456789012345678901212345 6
12
3456789012345678901234567890121234567890123456789012345678901212345 6
12
3456789012345678901234567890121234567890123456789012345678901212345 6
12
2
6
123456789012345678901234567890121234567890123456789012345678901212345
6
1 3456789012345678901234567890121234567890123456789012345678901212345
In
the
figure
above
the
non-shaded
region
3456789012345678901234567890121234567890123456789012345678901212345 6
12
3456789012345678901234567890121234567890123456789012345678901212345
6
12
is a quarter circle. If the area of rectangle
3456789012345678901234567890121234567890123456789012345678901212345
6
12
2
3456789012345678901234567890121234567890123456789012345678901212345
6
123456789012345678901234567890121234567890123456789012345678901212345
ABCD is the sum of 12 and AB 5 4, what
6
123456789012345678901234567890121234567890123456789012345678901212345
6
123456789012345678901234567890121234567890123456789012345678901212345
is the length of arc BE 1 ED?
123456789012345678901234567890121234567890123456789012345678901212345 66
123456789012345678901234567890121234567890123456789012345678901212345
3
6
123456789012345678901234567890121234567890123456789012345678901212345
6
123456789012345678901234567890121234567890123456789012345678901212345
A. p 1 1
Which
function
is
graphed
above?
6
2
123456789012345678901234567890121234567890123456789012345678901212345
6
123456789012345678901234567890121234567890123456789012345678901212345
2
6
1
B.
2p
1
1
A.
y
5
x
2
1
2
3456789012345678901234567890121234567890123456789012345678901212345
6
123456789012345678901234567890121234567890123456789012345678901212345
2
6
123456789012345678901234567890121234567890123456789012345678901212345
B.
x
5
y
2
1
6
1
5
2
6
123456789012345678901234567890121234567890123456789012345678901212345
C. p 1 1
C. y 5 x 1 1
3456789012345678901234567890121234567890123456789012345678901212345
6
12
2
2
2
3456789012345678901234567890121234567890123456789012345678901212345
6
123456789012345678901234567890121234567890123456789012345678901212345
D. x 5 y 1 1
6
123456789012345678901234567890121234567890123456789012345678901212345
2
2
D.
6p
1
2
6
1
E. y 5 x 1 1
3456789012345678901234567890121234567890123456789012345678901212345
6
12
E.
8p
1
2
3456789012345678901234567890121234567890123456789012345678901212345 6
12
2
3456789012345678901234567890121234567890123456789012345678901212345
123456789012345678901234567890121234567890123456789012345678901212345 66
123456789012345678901234567890121234567890123456789012345678901212345 6
1 25. What is the length of the longest distance
123456789012345678901234567890121234567890123456789012345678901212345 6
6
123456789012345678901234567890121234567890123456789012345678901212345
between two corners of a rectangular box
3456789012345678901234567890121234567890123456789012345678901212345
6
12
3456789012345678901234567890121234567890123456789012345678901212345
6
12
with
sides
2,
3,
and
4?
3456789012345678901234567890121234567890123456789012345678901212345 6
12
3456789012345678901234567890121234567890123456789012345678901212345 6
12
6
123456789012345678901234567890121234567890123456789012345678901212345
A. 2=7
2
6
123456789012345678901234567890121234567890123456789012345678901212345
6
123456789012345678901234567890121234567890123456789012345678901212345
B. 5
6
123456789012345678901234567890121234567890123456789012345678901212345
6
123456789012345678901234567890121234567890123456789012345678901212345
6
1 3456789012345678901234567890121234567890123456789012345678901212345
C. =29
2
3456789012345678901234567890121234567890123456789012345678901212345
123456789012345678901234567890121234567890123456789012345678901212345 66
123456789012345678901234567890121234567890123456789012345678901212345 6
123456789012345678901234567890121234567890123456789012345678901212345
D. =33
6
123456789012345678901234567890121234567890123456789012345678901212345
6
1
2
3456789012345678901234567890121234567890123456789012345678901212345
6
123456789012345678901234567890121234567890123456789012345678901212345
E. 6
6
1
3456789012345678901234567890121234567890123456789012345678901212345 6
12
123456789012345678901234567890121234567890123456789012345678901212345 6
3456789012345678901234567890121234567890123456789012345678901212345 6
12
123456789012345678901234567890121234567890123456789012345678901212345 6
123456789012345678901234567890121234567890123456789012345678901212345 6
123456789012345678901234567890121234567890123456789012345678901212345 6
123456789012345678901234567890121234567890123456789012345678901212345 6 263
1234567890123456789012345678901212345678901234567890123456789012123456

http://www.xtremepapers.net

Part IV: Three Practice GREs

1234567890123456789012345678901212345678901234567890123456789012123456
123456789012345678901234567890121234567890123456789012345678901212345 6
3456789012345678901234567890121234567890123456789012345678901212345 6
12
28.
6
123456789012345678901234567890121234567890123456789012345678901212345
3456789012345678901234567890121234567890123456789012345678901212345 6
12
3456789012345678901234567890121234567890123456789012345678901212345 6
12
123456789012345678901234567890121234567890123456789012345678901212345 6
3456789012345678901234567890121234567890123456789012345678901212345 6
12
123456789012345678901234567890121234567890123456789012345678901212345 6
3456789012345678901234567890121234567890123456789012345678901212345 6
12
3456789012345678901234567890121234567890123456789012345678901212345 6
12
3456789012345678901234567890121234567890123456789012345678901212345 6
12
3456789012345678901234567890121234567890123456789012345678901212345 6
12
2
1 3456789012345678901234567890121234567890123456789012345678901212345 6
3456789012345678901234567890121234567890123456789012345678901212345 6
12
3456789012345678901234567890121234567890123456789012345678901212345 6
12
3456789012345678901234567890121234567890123456789012345678901212345 6
12
3456789012345678901234567890121234567890123456789012345678901212345 6
12
2
1 3456789012345678901234567890121234567890123456789012345678901212345 6
3456789012345678901234567890121234567890123456789012345678901212345 6
12
3456789012345678901234567890121234567890123456789012345678901212345 6
12
3456789012345678901234567890121234567890123456789012345678901212345 6
12
3456789012345678901234567890121234567890123456789012345678901212345 6
12
2
1 3456789012345678901234567890121234567890123456789012345678901212345 6
3456789012345678901234567890121234567890123456789012345678901212345
6
12
The two figures in the circle are squares,
3456789012345678901234567890121234567890123456789012345678901212345
6
12
3456789012345678901234567890121234567890123456789012345678901212345
6
12
each
with
one
corner
at
the
center
of
the
3456789012345678901234567890121234567890123456789012345678901212345 6
12
2
3456789012345678901234567890121234567890123456789012345678901212345
6
123456789012345678901234567890121234567890123456789012345678901212345
circle and one corner touching the
6
123456789012345678901234567890121234567890123456789012345678901212345
6
123456789012345678901234567890121234567890123456789012345678901212345
circumference.
If
the
radius
of
the
circle
is
123456789012345678901234567890121234567890123456789012345678901212345 66
123456789012345678901234567890121234567890123456789012345678901212345
4, what is the area of the shaded region?
6
123456789012345678901234567890121234567890123456789012345678901212345
6
1
3456789012345678901234567890121234567890123456789012345678901212345
6
12
A.
2(p
2
1)
3456789012345678901234567890121234567890123456789012345678901212345 6
12
3456789012345678901234567890121234567890123456789012345678901212345
6
12
B. 6(p 2 1)
3456789012345678901234567890121234567890123456789012345678901212345
6
12
3456789012345678901234567890121234567890123456789012345678901212345
6
12
C. 8(p 2 1)
3456789012345678901234567890121234567890123456789012345678901212345 6
12
3456789012345678901234567890121234567890123456789012345678901212345
6
12
D. 12(p 2 1)
3456789012345678901234567890121234567890123456789012345678901212345
6
12
2
3456789012345678901234567890121234567890123456789012345678901212345
6
123456789012345678901234567890121234567890123456789012345678901212345
E. 16(p 2 1)
6
123456789012345678901234567890121234567890123456789012345678901212345
123456789012345678901234567890121234567890123456789012345678901212345 66
123456789012345678901234567890121234567890123456789012345678901212345 6
1 Verbal
3456789012345678901234567890121234567890123456789012345678901212345 6
12
3456789012345678901234567890121234567890123456789012345678901212345 6
12
6
123456789012345678901234567890121234567890123456789012345678901212345
30 Questions30 Minutes
6
123456789012345678901234567890121234567890123456789012345678901212345
3456789012345678901234567890121234567890123456789012345678901212345 6
12
2
3456789012345678901234567890121234567890123456789012345678901212345
6
123456789012345678901234567890121234567890123456789012345678901212345
Antonyms
6
123456789012345678901234567890121234567890123456789012345678901212345
6
1
3456789012345678901234567890121234567890123456789012345678901212345 6
12
3456789012345678901234567890121234567890123456789012345678901212345
6
12
Directions: Each question contains a word printed in capital letters, followed by five
3456789012345678901234567890121234567890123456789012345678901212345
6
12
3456789012345678901234567890121234567890123456789012345678901212345
6
12
answer
choices.
Choose
the
answer
choice
that
contains
the
word
or
phrase
most
nearly
123456789012345678901234567890121234567890123456789012345678901212345 6
6
123456789012345678901234567890121234567890123456789012345678901212345
OPPOSITE in meaning to the word in capital letters.
6
123456789012345678901234567890121234567890123456789012345678901212345
2
3456789012345678901234567890121234567890123456789012345678901212345
123456789012345678901234567890121234567890123456789012345678901212345 66
123456789012345678901234567890121234567890123456789012345678901212345 6
123456789012345678901234567890121234567890123456789012345678901212345 6
123456789012345678901234567890121234567890123456789012345678901212345 6
1 1. ZEAL :
2. HETEROGENEOUS :
3456789012345678901234567890121234567890123456789012345678901212345
6
12
3456789012345678901234567890121234567890123456789012345678901212345 6
12
3456789012345678901234567890121234567890123456789012345678901212345
6
12
A. enthusiasm
A. orthodox
3456789012345678901234567890121234567890123456789012345678901212345
6
12
6
123456789012345678901234567890121234567890123456789012345678901212345
B. apathy
B. abnormal
2
3456789012345678901234567890121234567890123456789012345678901212345
123456789012345678901234567890121234567890123456789012345678901212345 66
123456789012345678901234567890121234567890123456789012345678901212345
C. antagonism
C. uniform
6
123456789012345678901234567890121234567890123456789012345678901212345
6
123456789012345678901234567890121234567890123456789012345678901212345
D. pride
D. translucent
6
1
2
3456789012345678901234567890121234567890123456789012345678901212345
6
123456789012345678901234567890121234567890123456789012345678901212345
E. neutrality
E. deficient
6
1
3456789012345678901234567890121234567890123456789012345678901212345 6
12
123456789012345678901234567890121234567890123456789012345678901212345 6
3456789012345678901234567890121234567890123456789012345678901212345 6
12
123456789012345678901234567890121234567890123456789012345678901212345 6
123456789012345678901234567890121234567890123456789012345678901212345 6
123456789012345678901234567890121234567890123456789012345678901212345 66
264 123456789012345678901234567890121234567890123456789012345678901212345
1234567890123456789012345678901212345678901234567890123456789012123456

www.petersons.com

http://www.xtremepapers.net

TEST 1
1234567890123456789012345678901212345678901234567890123456789012123456
123456789012345678901234567890121234567890123456789012345678901212345 6
3456789012345678901234567890121234567890123456789012345678901212345 6
12
3. FEALTY :
6
7. OPPROBRIUM :
123456789012345678901234567890121234567890123456789012345678901212345
3456789012345678901234567890121234567890123456789012345678901212345 6
12
3456789012345678901234567890121234567890123456789012345678901212345
6
12
A. disloyalty
A. contempt
6
123456789012345678901234567890121234567890123456789012345678901212345
3456789012345678901234567890121234567890123456789012345678901212345
6
12
B.
turbidity
B.
platitude
2
3456789012345678901234567890121234567890123456789012345678901212345
6
1
3456789012345678901234567890121234567890123456789012345678901212345
6
12
C.
probity
C.
commendation
3456789012345678901234567890121234567890123456789012345678901212345 6
12
3456789012345678901234567890121234567890123456789012345678901212345
6
12
D. fidelity
D. prescience
3456789012345678901234567890121234567890123456789012345678901212345
6
12
2
3456789012345678901234567890121234567890123456789012345678901212345
6
1
E. substantiality
E. ennui
3456789012345678901234567890121234567890123456789012345678901212345
6
12
3456789012345678901234567890121234567890123456789012345678901212345 6
12
3456789012345678901234567890121234567890123456789012345678901212345 6
12
3456789012345678901234567890121234567890123456789012345678901212345
6
12
4. ARCHAIC :
8. TYRO :
2
6
1 3456789012345678901234567890121234567890123456789012345678901212345
3456789012345678901234567890121234567890123456789012345678901212345
6
12
A. lusterless
A. miscreant
3456789012345678901234567890121234567890123456789012345678901212345
6
12
3456789012345678901234567890121234567890123456789012345678901212345
6
12
B.
odious
B.
curmudgeon
3456789012345678901234567890121234567890123456789012345678901212345 6
12
2
3456789012345678901234567890121234567890123456789012345678901212345
6
1
C. clichd
C. virtuoso
3456789012345678901234567890121234567890123456789012345678901212345
6
12
3456789012345678901234567890121234567890123456789012345678901212345
6
12
D.
customary
D.
dilettante
3456789012345678901234567890121234567890123456789012345678901212345 6
12
3456789012345678901234567890121234567890123456789012345678901212345
6
12
E. novel
E. misogynist
3456789012345678901234567890121234567890123456789012345678901212345
6
12
3456789012345678901234567890121234567890123456789012345678901212345 6
12
3456789012345678901234567890121234567890123456789012345678901212345 6
12
3456789012345678901234567890121234567890123456789012345678901212345
6
12
5. ASSUAGE :
9. SOPORIFIC :
3456789012345678901234567890121234567890123456789012345678901212345
6
12
2
3456789012345678901234567890121234567890123456789012345678901212345
123456789012345678901234567890121234567890123456789012345678901212345 66
1
A. pillory
A. stimulant
3456789012345678901234567890121234567890123456789012345678901212345
6
12
B. satiate
B. exigent
3456789012345678901234567890121234567890123456789012345678901212345
6
12
3456789012345678901234567890121234567890123456789012345678901212345
6
12
C.
blandish
C.
torpor
2
3456789012345678901234567890121234567890123456789012345678901212345
123456789012345678901234567890121234567890123456789012345678901212345 66
123456789012345678901234567890121234567890123456789012345678901212345
D. ameliorate
D. sophomoric
6
123456789012345678901234567890121234567890123456789012345678901212345
6
123456789012345678901234567890121234567890123456789012345678901212345
E.
provoke
E.
jejune
123456789012345678901234567890121234567890123456789012345678901212345 66
123456789012345678901234567890121234567890123456789012345678901212345 6
123456789012345678901234567890121234567890123456789012345678901212345 6
123456789012345678901234567890121234567890123456789012345678901212345
6. MERCURIAL :
6
123456789012345678901234567890121234567890123456789012345678901212345
6
1
2
3456789012345678901234567890121234567890123456789012345678901212345
6
123456789012345678901234567890121234567890123456789012345678901212345
A. emotionally stable
6
123456789012345678901234567890121234567890123456789012345678901212345
6
1
B. bitterly rancorous
123456789012345678901234567890121234567890123456789012345678901212345 6
3456789012345678901234567890121234567890123456789012345678901212345
6
12
C. embarrassingly sordid
3456789012345678901234567890121234567890123456789012345678901212345
6
12
D. constantly morose
3456789012345678901234567890121234567890123456789012345678901212345
6
12
6
123456789012345678901234567890121234567890123456789012345678901212345
E. extremely melodramatic
3456789012345678901234567890121234567890123456789012345678901212345
6
12
3456789012345678901234567890121234567890123456789012345678901212345 6
12
2
3456789012345678901234567890121234567890123456789012345678901212345
123456789012345678901234567890121234567890123456789012345678901212345 66
123456789012345678901234567890121234567890123456789012345678901212345 6
1
Analogies
6
123456789012345678901234567890121234567890123456789012345678901212345
123456789012345678901234567890121234567890123456789012345678901212345 6
2
3456789012345678901234567890121234567890123456789012345678901212345
123456789012345678901234567890121234567890123456789012345678901212345 66
123456789012345678901234567890121234567890123456789012345678901212345
Directions: In each question, there is an initial pair of words or phrases that are related in
6
123456789012345678901234567890121234567890123456789012345678901212345
6
123456789012345678901234567890121234567890123456789012345678901212345
some
manner.
Select
the
answer
choice
containing
the
lettered
pair
that
best
expresses
a
6
1
2
3456789012345678901234567890121234567890123456789012345678901212345
6
123456789012345678901234567890121234567890123456789012345678901212345
relationship similar to the initial pair.
6
123456789012345678901234567890121234567890123456789012345678901212345
123456789012345678901234567890121234567890123456789012345678901212345 66
123456789012345678901234567890121234567890123456789012345678901212345 6
1
3456789012345678901234567890121234567890123456789012345678901212345 6
12
3456789012345678901234567890121234567890123456789012345678901212345
6
12
10. PRATTLE : BABBLE ::
11. FAWN : FLATTER ::
3456789012345678901234567890121234567890123456789012345678901212345
6
12
3456789012345678901234567890121234567890123456789012345678901212345 6
12
6
123456789012345678901234567890121234567890123456789012345678901212345
A. battle : fight
A. praise : criticize
3456789012345678901234567890121234567890123456789012345678901212345
6
12
B. divulge : tell
B. digest : eat
6
123456789012345678901234567890121234567890123456789012345678901212345
3456789012345678901234567890121234567890123456789012345678901212345
6
12
C. burgeon : grow
C. engage : withdraw
6
123456789012345678901234567890121234567890123456789012345678901212345
3456789012345678901234567890121234567890123456789012345678901212345
6
12
D.
bolster
:
disapprove
D.
misconstrue
:
grasp
2
3456789012345678901234567890121234567890123456789012345678901212345
6
1
6
123456789012345678901234567890121234567890123456789012345678901212345
E. misquote : lie
E. mettle : disassociate
6
123456789012345678901234567890121234567890123456789012345678901212345
123456789012345678901234567890121234567890123456789012345678901212345 6 265
1234567890123456789012345678901212345678901234567890123456789012123456

http://www.xtremepapers.net

Part IV: Three Practice GREs

1234567890123456789012345678901212345678901234567890123456789012123456
123456789012345678901234567890121234567890123456789012345678901212345 6
3456789012345678901234567890121234567890123456789012345678901212345 6
12
12. DENOUEMENT: STORY ::
15. ERSATZ : ARTIFICAL ::
6
123456789012345678901234567890121234567890123456789012345678901212345
3456789012345678901234567890121234567890123456789012345678901212345 6
12
3456789012345678901234567890121234567890123456789012345678901212345
6
12
A. prologue : novel
A. natural : polluted
6
123456789012345678901234567890121234567890123456789012345678901212345
3456789012345678901234567890121234567890123456789012345678901212345
6
12
B.
sunset
:
day
B.
abstruse
:
facile
2
3456789012345678901234567890121234567890123456789012345678901212345
6
1
3456789012345678901234567890121234567890123456789012345678901212345
6
12
C.
intermission
:
play
C.
caustic
:
astute
3456789012345678901234567890121234567890123456789012345678901212345 6
12
3456789012345678901234567890121234567890123456789012345678901212345
6
12
D. sidewalk : bistro
D. nascent : incipient
3456789012345678901234567890121234567890123456789012345678901212345
6
12
2
3456789012345678901234567890121234567890123456789012345678901212345
6
1
E. traffic : thoroughfare
E. ribald : nave
3456789012345678901234567890121234567890123456789012345678901212345
6
12
3456789012345678901234567890121234567890123456789012345678901212345 6
12
3456789012345678901234567890121234567890123456789012345678901212345 6
12
3456789012345678901234567890121234567890123456789012345678901212345
6
12
13. FEDERALISM : POLITICAL ORDER ::
16. DIATRIBE : EULOGY ::
2
6
1 3456789012345678901234567890121234567890123456789012345678901212345
3456789012345678901234567890121234567890123456789012345678901212345
6
12
A. class : student
A. recommendation : commendation
3456789012345678901234567890121234567890123456789012345678901212345
6
12
3456789012345678901234567890121234567890123456789012345678901212345
6
12
B.
dictatorship
:
anarchy
B.
impugnation
:
chastisement
3456789012345678901234567890121234567890123456789012345678901212345 6
12
2
3456789012345678901234567890121234567890123456789012345678901212345
6
1
C. colleague : cohort
C. rebuke : reproof
3456789012345678901234567890121234567890123456789012345678901212345
6
12
3456789012345678901234567890121234567890123456789012345678901212345
6
12
D.
insomnia
:
sleep
disorder
D.
verity
:
axiomatic
3456789012345678901234567890121234567890123456789012345678901212345 6
12
3456789012345678901234567890121234567890123456789012345678901212345
6
12
E. extrinsic : external
E. remonstration : encomium
3456789012345678901234567890121234567890123456789012345678901212345
6
12
3456789012345678901234567890121234567890123456789012345678901212345 6
12
3456789012345678901234567890121234567890123456789012345678901212345 6
12
3456789012345678901234567890121234567890123456789012345678901212345
6
12
14. SPURIOUS : TRUE ::
17. HAGIOGRAPHY: VENERATES ::
3456789012345678901234567890121234567890123456789012345678901212345
6
12
2
3456789012345678901234567890121234567890123456789012345678901212345
123456789012345678901234567890121234567890123456789012345678901212345 66
1
A. sporadic : anomalous
A. polemic : attacks
3456789012345678901234567890121234567890123456789012345678901212345
6
12
B. ingenious : innovative
B. vendetta : scares
3456789012345678901234567890121234567890123456789012345678901212345
6
12
3456789012345678901234567890121234567890123456789012345678901212345
6
12
C.
hilarious
:
witty
C.
complaint
:
angers
2
3456789012345678901234567890121234567890123456789012345678901212345
123456789012345678901234567890121234567890123456789012345678901212345 66
123456789012345678901234567890121234567890123456789012345678901212345
D. insipid : flavorful
D. report : obscures
6
123456789012345678901234567890121234567890123456789012345678901212345
6
123456789012345678901234567890121234567890123456789012345678901212345
E.
base
:
dishonest
E.
truism
:
enunciates
123456789012345678901234567890121234567890123456789012345678901212345 66
123456789012345678901234567890121234567890123456789012345678901212345 6
123456789012345678901234567890121234567890123456789012345678901212345 6
123456789012345678901234567890121234567890123456789012345678901212345 6
123456789012345678901234567890121234567890123456789012345678901212345
Sentence Completions
6
1
2
3456789012345678901234567890121234567890123456789012345678901212345
123456789012345678901234567890121234567890123456789012345678901212345 66
123456789012345678901234567890121234567890123456789012345678901212345 6
1
Directions: Each sentence below contains one or two blanks, with each blank corre6
123456789012345678901234567890121234567890123456789012345678901212345
3456789012345678901234567890121234567890123456789012345678901212345
6
12
sponding to an omitted word. Find the answer choice that has the word or set or words
2
3456789012345678901234567890121234567890123456789012345678901212345
123456789012345678901234567890121234567890123456789012345678901212345 66
123456789012345678901234567890121234567890123456789012345678901212345
that best fits the meaning of the sentence as a whole.
6
1
3456789012345678901234567890121234567890123456789012345678901212345 6
12
3456789012345678901234567890121234567890123456789012345678901212345 6
12
3456789012345678901234567890121234567890123456789012345678901212345 6
12
3456789012345678901234567890121234567890123456789012345678901212345 6
12
6
123456789012345678901234567890121234567890123456789012345678901212345
18. Although __________ continues on minor
19. Recent advances in research methodolo6
123456789012345678901234567890121234567890123456789012345678901212345
6
123456789012345678901234567890121234567890123456789012345678901212345
points, the __________ themes of the
gies in neuroscience have created excite2
3456789012345678901234567890121234567890123456789012345678901212345
123456789012345678901234567890121234567890123456789012345678901212345 66
123456789012345678901234567890121234567890123456789012345678901212345
Generals plan have been accepted.
ment in academic circles and have
6
123456789012345678901234567890121234567890123456789012345678901212345
__________ greater interest in funding
6
123456789012345678901234567890121234567890123456789012345678901212345
A.
discussion.
.peripheral
6
1
new
research
projects
in
the
field.
2
3456789012345678901234567890121234567890123456789012345678901212345
6
123456789012345678901234567890121234567890123456789012345678901212345
B. debate. .central
6
123456789012345678901234567890121234567890123456789012345678901212345
6
123456789012345678901234567890121234567890123456789012345678901212345
C.
dissension.
.cultural
A.
stymied
123456789012345678901234567890121234567890123456789012345678901212345 66
1
D. discovery. .main
B. intensified
3456789012345678901234567890121234567890123456789012345678901212345
6
12
3456789012345678901234567890121234567890123456789012345678901212345
6
12
E.
agreement.
.basic
C.
magnified
3456789012345678901234567890121234567890123456789012345678901212345 6
12
3456789012345678901234567890121234567890123456789012345678901212345
6
12
D. engendered
6
123456789012345678901234567890121234567890123456789012345678901212345
2
3456789012345678901234567890121234567890123456789012345678901212345
6
123456789012345678901234567890121234567890123456789012345678901212345
E. multiplied
6
1
3456789012345678901234567890121234567890123456789012345678901212345 6
12
123456789012345678901234567890121234567890123456789012345678901212345 6
3456789012345678901234567890121234567890123456789012345678901212345 6
12
123456789012345678901234567890121234567890123456789012345678901212345 6
123456789012345678901234567890121234567890123456789012345678901212345 6
123456789012345678901234567890121234567890123456789012345678901212345 66
266 123456789012345678901234567890121234567890123456789012345678901212345
1234567890123456789012345678901212345678901234567890123456789012123456

www.petersons.com

http://www.xtremepapers.net

TEST 1
1234567890123456789012345678901212345678901234567890123456789012123456
123456789012345678901234567890121234567890123456789012345678901212345 6
3456789012345678901234567890121234567890123456789012345678901212345 6
12
20. Although the boom appeared to still be in
6
23. The professor became __________ after
123456789012345678901234567890121234567890123456789012345678901212345
3456789012345678901234567890121234567890123456789012345678901212345
6
12
full swing, in the latter years of the
attempting to explain the important
3456789012345678901234567890121234567890123456789012345678901212345
6
12
6
123456789012345678901234567890121234567890123456789012345678901212345
decade,
most
economic
indicators
pointed
concept
multiple
different
ways,
but
her
3456789012345678901234567890121234567890123456789012345678901212345 6
12
2
3456789012345678901234567890121234567890123456789012345678901212345
6
1
to __________ times ahead.
visible frustration only made the student
3456789012345678901234567890121234567890123456789012345678901212345
6
12
3456789012345678901234567890121234567890123456789012345678901212345
6
12
less
likely
to
__________
the
material.
3456789012345678901234567890121234567890123456789012345678901212345
6
12
A.
leaner
3456789012345678901234567890121234567890123456789012345678901212345 6
12
2
3456789012345678901234567890121234567890123456789012345678901212345
6
1
B. profitable
A. mystified. .assimilate
3456789012345678901234567890121234567890123456789012345678901212345
6
12
3456789012345678901234567890121234567890123456789012345678901212345
6
12
C.
healthy
B.
belligerent.
.understand
3456789012345678901234567890121234567890123456789012345678901212345 6
12
3456789012345678901234567890121234567890123456789012345678901212345
6
12
D. daring
C. stupefied. .comprehend
6
123456789012345678901234567890121234567890123456789012345678901212345
3456789012345678901234567890121234567890123456789012345678901212345
6
12
E.
aberrant
D.
vexed.
.grasp
3456789012345678901234567890121234567890123456789012345678901212345 6
12
3456789012345678901234567890121234567890123456789012345678901212345
6
12
E. agitated. .eschew
3456789012345678901234567890121234567890123456789012345678901212345
6
12
21.
Though
often
crude
and
overly
2
3456789012345678901234567890121234567890123456789012345678901212345
6
1
3456789012345678901234567890121234567890123456789012345678901212345
6
12
__________, early theories of biogenesis
3456789012345678901234567890121234567890123456789012345678901212345
6
12
3456789012345678901234567890121234567890123456789012345678901212345
6
12
were
important
for
the
__________
of
the
3456789012345678901234567890121234567890123456789012345678901212345 6
12
2
3456789012345678901234567890121234567890123456789012345678901212345
6
123456789012345678901234567890121234567890123456789012345678901212345
question of lifes origin.
6
123456789012345678901234567890121234567890123456789012345678901212345
123456789012345678901234567890121234567890123456789012345678901212345 66
123456789012345678901234567890121234567890123456789012345678901212345
A. dramatic. .institutionalization
6
123456789012345678901234567890121234567890123456789012345678901212345
6
123456789012345678901234567890121234567890123456789012345678901212345
B. hypothetical. .clarification
6
1
3456789012345678901234567890121234567890123456789012345678901212345
6
12
C.
inscrutable.
.illumination
3456789012345678901234567890121234567890123456789012345678901212345 6
12
3456789012345678901234567890121234567890123456789012345678901212345
6
12
D. schematic. .extermination
3456789012345678901234567890121234567890123456789012345678901212345
6
12
3456789012345678901234567890121234567890123456789012345678901212345
6
12
E.
simplistic.
.advancement
3456789012345678901234567890121234567890123456789012345678901212345 6
12
3456789012345678901234567890121234567890123456789012345678901212345 6
12
3456789012345678901234567890121234567890123456789012345678901212345
6
12
22. The lawyers argumentation was
2
6
123456789012345678901234567890121234567890123456789012345678901212345
6
123456789012345678901234567890121234567890123456789012345678901212345
__________ and factually misleading, but
6
123456789012345678901234567890121234567890123456789012345678901212345
6
123456789012345678901234567890121234567890123456789012345678901212345
these
problems
in
his
closing
argument
1 3456789012345678901234567890121234567890123456789012345678901212345 6
2
3456789012345678901234567890121234567890123456789012345678901212345
6
123456789012345678901234567890121234567890123456789012345678901212345
were masked by his __________ and
6
123456789012345678901234567890121234567890123456789012345678901212345
6
1
superior
mastery
of
language.
123456789012345678901234567890121234567890123456789012345678901212345 6
3456789012345678901234567890121234567890123456789012345678901212345 6
12
3456789012345678901234567890121234567890123456789012345678901212345
6
12
A. spurious. .elusiveness
3456789012345678901234567890121234567890123456789012345678901212345
6
12
6
123456789012345678901234567890121234567890123456789012345678901212345
B.
debonair.
.equivocations
3456789012345678901234567890121234567890123456789012345678901212345 6
12
3456789012345678901234567890121234567890123456789012345678901212345
6
12
C. duplicitous. .dissembling
3456789012345678901234567890121234567890123456789012345678901212345
6
12
3456789012345678901234567890121234567890123456789012345678901212345
6
12
D.
specious.
.eloquence
123456789012345678901234567890121234567890123456789012345678901212345 6
6
123456789012345678901234567890121234567890123456789012345678901212345
E. spectacular. .suaveness
6
123456789012345678901234567890121234567890123456789012345678901212345
2
3456789012345678901234567890121234567890123456789012345678901212345
123456789012345678901234567890121234567890123456789012345678901212345 66
123456789012345678901234567890121234567890123456789012345678901212345 6
123456789012345678901234567890121234567890123456789012345678901212345 6
123456789012345678901234567890121234567890123456789012345678901212345 6
1
3456789012345678901234567890121234567890123456789012345678901212345 6
12
3456789012345678901234567890121234567890123456789012345678901212345 6
12
3456789012345678901234567890121234567890123456789012345678901212345 6
12
3456789012345678901234567890121234567890123456789012345678901212345 6
12
123456789012345678901234567890121234567890123456789012345678901212345 6
2
6
123456789012345678901234567890121234567890123456789012345678901212345
6
123456789012345678901234567890121234567890123456789012345678901212345
6
123456789012345678901234567890121234567890123456789012345678901212345
6
123456789012345678901234567890121234567890123456789012345678901212345
1 3456789012345678901234567890121234567890123456789012345678901212345 6
2
3456789012345678901234567890121234567890123456789012345678901212345
123456789012345678901234567890121234567890123456789012345678901212345 66
1
3456789012345678901234567890121234567890123456789012345678901212345 6
12
123456789012345678901234567890121234567890123456789012345678901212345 6
3456789012345678901234567890121234567890123456789012345678901212345 6
12
123456789012345678901234567890121234567890123456789012345678901212345 6
123456789012345678901234567890121234567890123456789012345678901212345 6
123456789012345678901234567890121234567890123456789012345678901212345 6
123456789012345678901234567890121234567890123456789012345678901212345 6 267
1234567890123456789012345678901212345678901234567890123456789012123456

http://www.xtremepapers.net

Part IV: Three Practice GREs

1234567890123456789012345678901212345678901234567890123456789012123456
123456789012345678901234567890121234567890123456789012345678901212345 6
3456789012345678901234567890121234567890123456789012345678901212345 6
12
Reading Comprehension
6
123456789012345678901234567890121234567890123456789012345678901212345
3456789012345678901234567890121234567890123456789012345678901212345 6
12
3456789012345678901234567890121234567890123456789012345678901212345 6
12
123456789012345678901234567890121234567890123456789012345678901212345 6
3456789012345678901234567890121234567890123456789012345678901212345
6
12
Directions: Questions follow each passage. After reading the passage, determine the best
6
123456789012345678901234567890121234567890123456789012345678901212345
3456789012345678901234567890121234567890123456789012345678901212345
6
12
answer for each question based on the passages content.
3456789012345678901234567890121234567890123456789012345678901212345 6
12
3456789012345678901234567890121234567890123456789012345678901212345 6
12
3456789012345678901234567890121234567890123456789012345678901212345 6
12
2
1 3456789012345678901234567890121234567890123456789012345678901212345 6
3456789012345678901234567890121234567890123456789012345678901212345 6
12
3456789012345678901234567890121234567890123456789012345678901212345
6
12
Passage 1
presence of dark matter if the galaxys
3456789012345678901234567890121234567890123456789012345678901212345
6
12
3456789012345678901234567890121234567890123456789012345678901212345
motion requires gravitational interactions 6
12
Line
The
nature
and
structure
of
dark
matter
2
3456789012345678901234567890121234567890123456789012345678901212345
6
1
greater than those afforded by the
3456789012345678901234567890121234567890123456789012345678901212345
6
12
is a primary topic in contemporary
3456789012345678901234567890121234567890123456789012345678901212345
6
12
galaxys light matter.
3456789012345678901234567890121234567890123456789012345678901212345
6
12
cosmology,
the
sub-discipline
of
as3456789012345678901234567890121234567890123456789012345678901212345
6
12
(40)
Our
understanding
of
dark
matter
2
3456789012345678901234567890121234567890123456789012345678901212345
6
1
tronomy in which the universe is studied
3456789012345678901234567890121234567890123456789012345678901212345
6
12
does not extend much beyond this
3456789012345678901234567890121234567890123456789012345678901212345
6
12
(5)
as
a
whole.
Dark
matter
has
aroused
3456789012345678901234567890121234567890123456789012345678901212345
6
12
rudimentary
model
of
matter
that
is
3456789012345678901234567890121234567890123456789012345678901212345
6
12
great
interest
in
cosmological
circles,
2
3456789012345678901234567890121234567890123456789012345678901212345
6
123456789012345678901234567890121234567890123456789012345678901212345
gravitationally interactive but electro6
123456789012345678901234567890121234567890123456789012345678901212345
because
current
theories
of
the
evolution
6
123456789012345678901234567890121234567890123456789012345678901212345
magnetically
non-interactive.
Since
most
6
123456789012345678901234567890121234567890123456789012345678901212345
of
the
universe
require
the
hypothesis
6
123456789012345678901234567890121234567890123456789012345678901212345
(45)
astronomical
probing
of
the
universe
6
123456789012345678901234567890121234567890123456789012345678901212345
that dark matter constitutes nearly
6
1
employs
light-sensitive
instruments,
3456789012345678901234567890121234567890123456789012345678901212345
6
12
(10)
three-fourths
of
the
matter
in
the
3456789012345678901234567890121234567890123456789012345678901212345
6
12
knowledge
of
dark
matter
comes
only
3456789012345678901234567890121234567890123456789012345678901212345
6
12
universe. This hypothesis, if true, makes
2
3456789012345678901234567890121234567890123456789012345678901212345
123456789012345678901234567890121234567890123456789012345678901212345
indirectly. Thus, we are left to theoretical 6
6
123456789012345678901234567890121234567890123456789012345678901212345
understanding dark matter of signal
6
123456789012345678901234567890121234567890123456789012345678901212345
hypotheses
about
dark
matter
until
more
6
123456789012345678901234567890121234567890123456789012345678901212345
importance to understanding the
6
123456789012345678901234567890121234567890123456789012345678901212345
(50)
fecund methods of investigating dark
6
123456789012345678901234567890121234567890123456789012345678901212345
universe.
6
123456789012345678901234567890121234567890123456789012345678901212345
matter are developed.
(15)
Dark matter is matter that does not
6
123456789012345678901234567890121234567890123456789012345678901212345
6
123456789012345678901234567890121234567890123456789012345678901212345
interact with electromagnetic waves
6
1
2
3456789012345678901234567890121234567890123456789012345678901212345
6
123456789012345678901234567890121234567890123456789012345678901212345
24. The authors tone toward the future
that is, it does not emit or absorb light.
6
123456789012345678901234567890121234567890123456789012345678901212345
6
1
advancement
of
our
understanding
of
In contrast, light or regular matter does
6
123456789012345678901234567890121234567890123456789012345678901212345
3456789012345678901234567890121234567890123456789012345678901212345
6
12
dark
matter
is
interact
with
electromagnetic
waves
and
2
3456789012345678901234567890121234567890123456789012345678901212345
123456789012345678901234567890121234567890123456789012345678901212345 66
123456789012345678901234567890121234567890123456789012345678901212345
6
1 (20) therefore does emit and absorb light.
A. deeply pessimistic.
3456789012345678901234567890121234567890123456789012345678901212345
6
12
When
astronomers
look
into
space
with
3456789012345678901234567890121234567890123456789012345678901212345
6
12
B. moderately skeptical.
2
3456789012345678901234567890121234567890123456789012345678901212345
6
123456789012345678901234567890121234567890123456789012345678901212345
electromagnetic sensitive telescopes (or
6
123456789012345678901234567890121234567890123456789012345678901212345
C. guardedly hopeful.
6
1
other
forms
of
light
sensitive
telescopes),
6
123456789012345678901234567890121234567890123456789012345678901212345
D. tenaciously idealistic.
6
123456789012345678901234567890121234567890123456789012345678901212345
they
can
detect
light
matter
but
not
dark
2
3456789012345678901234567890121234567890123456789012345678901212345
6
123456789012345678901234567890121234567890123456789012345678901212345
E. basically theoretical.
6
123456789012345678901234567890121234567890123456789012345678901212345
(25)
matter.
This
is
how
dark
matter
earned
123456789012345678901234567890121234567890123456789012345678901212345 66
123456789012345678901234567890121234567890123456789012345678901212345
its moniker. Astronomers are not able to
6
1
2
3456789012345678901234567890121234567890123456789012345678901212345
6
123456789012345678901234567890121234567890123456789012345678901212345
see it with conventional telescopes.
6
123456789012345678901234567890121234567890123456789012345678901212345
6
123456789012345678901234567890121234567890123456789012345678901212345
The existence of dark matter in a
123456789012345678901234567890121234567890123456789012345678901212345 66
1
particular region of space must be
3456789012345678901234567890121234567890123456789012345678901212345
6
12
3456789012345678901234567890121234567890123456789012345678901212345
6
12
(30)
inferred from the dark matters gravita3456789012345678901234567890121234567890123456789012345678901212345 6
12
3456789012345678901234567890121234567890123456789012345678901212345
6
12
tional interaction with light matter. Like
6
123456789012345678901234567890121234567890123456789012345678901212345
light
matter,
dark
matter
gravitationally
2
3456789012345678901234567890121234567890123456789012345678901212345
123456789012345678901234567890121234567890123456789012345678901212345 66
1
attracts all matter and so sustains a
3456789012345678901234567890121234567890123456789012345678901212345
6
12
6
123456789012345678901234567890121234567890123456789012345678901212345
gravitational
field.
As
an
astronomer
3456789012345678901234567890121234567890123456789012345678901212345 6
12
2
3456789012345678901234567890121234567890123456789012345678901212345
6
1 (35) studies a galaxy, he or she can infer the
123456789012345678901234567890121234567890123456789012345678901212345 6
123456789012345678901234567890121234567890123456789012345678901212345 66
268 123456789012345678901234567890121234567890123456789012345678901212345
1234567890123456789012345678901212345678901234567890123456789012123456

www.petersons.com

http://www.xtremepapers.net

TEST 1
1234567890123456789012345678901212345678901234567890123456789012123456
123456789012345678901234567890121234567890123456789012345678901212345 6
3456789012345678901234567890121234567890123456789012345678901212345 6
12
25. According to the passage, dark matter
crowd of narratives the causes, inner
6
123456789012345678901234567890121234567890123456789012345678901212345
3456789012345678901234567890121234567890123456789012345678901212345
6
12
workings, and final consequences of the 6
3456789012345678901234567890121234567890123456789012345678901212345
12
I.
gravitationally
interacts
with
light
6
123456789012345678901234567890121234567890123456789012345678901212345
(15)
Revolution all have differing assign3456789012345678901234567890121234567890123456789012345678901212345
6
12
matter.
2
3456789012345678901234567890121234567890123456789012345678901212345
6
1
ments.
3456789012345678901234567890121234567890123456789012345678901212345
6
12
II.
does
interact
with
electromagnetic
3456789012345678901234567890121234567890123456789012345678901212345
6
12
To
the
flagship
conservative,
Edmund
3456789012345678901234567890121234567890123456789012345678901212345
6
12
waves.
3456789012345678901234567890121234567890123456789012345678901212345
6
12
Burke,
writing
contemporaneously
with
2
3456789012345678901234567890121234567890123456789012345678901212345
6
1
III. is similar to light matter in its
3456789012345678901234567890121234567890123456789012345678901212345
6
12
the
events,
the
Revolution
was
a
sudden
3456789012345678901234567890121234567890123456789012345678901212345
6
12
gravitational
properties.
3456789012345678901234567890121234567890123456789012345678901212345
6
12
(20)
and
radical
manipulation
of
the
French
3456789012345678901234567890121234567890123456789012345678901212345 6
12
2
3456789012345678901234567890121234567890123456789012345678901212345
6
1
A. I only
nation by a small class of philosophes,
3456789012345678901234567890121234567890123456789012345678901212345
6
12
3456789012345678901234567890121234567890123456789012345678901212345
6
12
B. III only
literati,
and
the
newly-monied.
In
Burkes
3456789012345678901234567890121234567890123456789012345678901212345 6
12
3456789012345678901234567890121234567890123456789012345678901212345
12
C. I and II only
view, these so-termed aspirants to power 6
6
123456789012345678901234567890121234567890123456789012345678901212345
3456789012345678901234567890121234567890123456789012345678901212345
6
12
D. I and III only
successively
harnessed
methods
of
mass
3456789012345678901234567890121234567890123456789012345678901212345 6
12
3456789012345678901234567890121234567890123456789012345678901212345
6
E. I, II, and III
12
(25)
persuasion to incite revolt and to
3456789012345678901234567890121234567890123456789012345678901212345
6
12
2
3456789012345678901234567890121234567890123456789012345678901212345
6
123456789012345678901234567890121234567890123456789012345678901212345
ultimately destroy the basic institutions
6
123456789012345678901234567890121234567890123456789012345678901212345
26.
It
can
be
inferred
from
the
third
para123456789012345678901234567890121234567890123456789012345678901212345
of French society and in their place erect 6
6
123456789012345678901234567890121234567890123456789012345678901212345
graph that
123456789012345678901234567890121234567890123456789012345678901212345
a historical novelty, the newly christened 6
123456789012345678901234567890121234567890123456789012345678901212345 66
1
French Republic. In an instructive
A. light matter sustains a gravitational
3456789012345678901234567890121234567890123456789012345678901212345
6
12
(30)
contrast, Alex de Tocqueville, a French
3456789012345678901234567890121234567890123456789012345678901212345
6
12
field.
3456789012345678901234567890121234567890123456789012345678901212345
6
12
provincial
aristocrat,
writing
a
few
2
3456789012345678901234567890121234567890123456789012345678901212345
6
123456789012345678901234567890121234567890123456789012345678901212345
B. regular matter is more stable than
6
123456789012345678901234567890121234567890123456789012345678901212345
decades after Burke, saw the Revolution 6
123456789012345678901234567890121234567890123456789012345678901212345
dark matter.
6
123456789012345678901234567890121234567890123456789012345678901212345
as a logical consequence and unfolding of 6
123456789012345678901234567890121234567890123456789012345678901212345
C. astronomers infer the presence of light
6
123456789012345678901234567890121234567890123456789012345678901212345
the reforms set in motion by the ancien
6
123456789012345678901234567890121234567890123456789012345678901212345
matter if a galaxys dark matter does
6
123456789012345678901234567890121234567890123456789012345678901212345
(35)
regime.
On
this
account,
the
Revolution
not fully account for gravitational
6
123456789012345678901234567890121234567890123456789012345678901212345
6
1
was
a
continuation
of
the
administrative
interactions.
2
3456789012345678901234567890121234567890123456789012345678901212345
123456789012345678901234567890121234567890123456789012345678901212345 66
123456789012345678901234567890121234567890123456789012345678901212345
revolution begun by Louis XVI. In this 6
D. dark matter is of primary importance
1
6
123456789012345678901234567890121234567890123456789012345678901212345
light, the French Republic was the
in contemporary cosmology.
3456789012345678901234567890121234567890123456789012345678901212345
6
12
2
3456789012345678901234567890121234567890123456789012345678901212345
123456789012345678901234567890121234567890123456789012345678901212345
reasonable outgrowth of the evolution of 6
E. theoretical hypotheses about dark
6
123456789012345678901234567890121234567890123456789012345678901212345
6
1
(40)
French
society
that
stretched
far
back
in
matter
are
becoming
more
refined.
3456789012345678901234567890121234567890123456789012345678901212345 6
12
3456789012345678901234567890121234567890123456789012345678901212345
6
12
the eighteenth century, as opposed to a
3456789012345678901234567890121234567890123456789012345678901212345
6
12
3456789012345678901234567890121234567890123456789012345678901212345
6
12
Passage
2
monstrous
and
cataclysmic
break
with
123456789012345678901234567890121234567890123456789012345678901212345 6
6
123456789012345678901234567890121234567890123456789012345678901212345
Frances past.
6
123456789012345678901234567890121234567890123456789012345678901212345
Line Though the discipline of historiography
2
3456789012345678901234567890121234567890123456789012345678901212345
6
123456789012345678901234567890121234567890123456789012345678901212345
Both of these figures, through their
purports to be an empirical endeavor free
6
123456789012345678901234567890121234567890123456789012345678901212345
6
123456789012345678901234567890121234567890123456789012345678901212345
(45)
respective historical fame, have come to
of the biases of political persuasion and
6
123456789012345678901234567890121234567890123456789012345678901212345
6
1
embody loaded historical mythoi
temperament,
the
actual
writing
of
2
3456789012345678901234567890121234567890123456789012345678901212345
6
123456789012345678901234567890121234567890123456789012345678901212345
themselves, but the contrast between
6
123456789012345678901234567890121234567890123456789012345678901212345
(5)
history is often reduced to the role of
6
123456789012345678901234567890121234567890123456789012345678901212345
their understanding and interpretation of 6
123456789012345678901234567890121234567890123456789012345678901212345
ideologys
handmaiden.
Histories
of
the
6
1
the events of the French Revolution is
3456789012345678901234567890121234567890123456789012345678901212345
6
12
French Revolution serve as a prime
3456789012345678901234567890121234567890123456789012345678901212345
6
12
(50)
still illuminating with respect to histori3456789012345678901234567890121234567890123456789012345678901212345
6
12
example
of
this
historiographic
tendency.
3456789012345678901234567890121234567890123456789012345678901212345
6
12
ography.
The
complexity
of
large-scale
6
123456789012345678901234567890121234567890123456789012345678901212345
A
survey
of
the
histories
written
on
the
2
3456789012345678901234567890121234567890123456789012345678901212345
123456789012345678901234567890121234567890123456789012345678901212345
historical events resists simplification into 6
6
1 (10) French Revolution produces an impres3456789012345678901234567890121234567890123456789012345678901212345
6
12
incontrovertible
datum
in
the
service
of
6
123456789012345678901234567890121234567890123456789012345678901212345
sive
array
of
narratives
of
late
eighteenth
3456789012345678901234567890121234567890123456789012345678901212345
6
12
one
narrative
over
another.
This
obtuse
2
3456789012345678901234567890121234567890123456789012345678901212345
6
1
century France. Within this diverse
6
123456789012345678901234567890121234567890123456789012345678901212345
123456789012345678901234567890121234567890123456789012345678901212345 66
123456789012345678901234567890121234567890123456789012345678901212345 269
1234567890123456789012345678901212345678901234567890123456789012123456

http://www.xtremepapers.net

Part IV: Three Practice GREs

1234567890123456789012345678901212345678901234567890123456789012123456
123456789012345678901234567890121234567890123456789012345678901212345 6
3456789012345678901234567890121234567890123456789012345678901212345 6
12
(55)
reality complicates the historians
29. In line 28, the author states that in Burkes 6
123456789012345678901234567890121234567890123456789012345678901212345
3456789012345678901234567890121234567890123456789012345678901212345
6
12
attempt to simply ply her trade, but
view, the French Republic was a histori- 6
3456789012345678901234567890121234567890123456789012345678901212345
12
6
123456789012345678901234567890121234567890123456789012345678901212345
nonetheless is a facet of historiography
cal novelty. This implies that Burke
3456789012345678901234567890121234567890123456789012345678901212345
6
12
2
3456789012345678901234567890121234567890123456789012345678901212345
6
1
that must be addressed if history is to be
believed the French Republic
3456789012345678901234567890121234567890123456789012345678901212345
6
12
3456789012345678901234567890121234567890123456789012345678901212345
6
12
written
with
intellectual
honesty.
3456789012345678901234567890121234567890123456789012345678901212345
6
12
A.
bore
close
resemblance
to
its
historical
3456789012345678901234567890121234567890123456789012345678901212345 6
12
2
3456789012345678901234567890121234567890123456789012345678901212345
6
1
predecessor.
3456789012345678901234567890121234567890123456789012345678901212345
6
12
3456789012345678901234567890121234567890123456789012345678901212345
6
12
27. The passage would most appropriately be
B.
shared
some
resemblance
to
its
3456789012345678901234567890121234567890123456789012345678901212345 6
12
3456789012345678901234567890121234567890123456789012345678901212345
6
12
titled
historical predecessor.
6
123456789012345678901234567890121234567890123456789012345678901212345
3456789012345678901234567890121234567890123456789012345678901212345
6
12
C.
bore
no
resemblance
to
its
historical
A. The French Revolution: Two
3456789012345678901234567890121234567890123456789012345678901212345
6
12
3456789012345678901234567890121234567890123456789012345678901212345
6
12
predecessor.
Competing
Views.
3456789012345678901234567890121234567890123456789012345678901212345 6
12
2
3456789012345678901234567890121234567890123456789012345678901212345
6
1
D. was the logical outgrowth of its
B. Contemporary Issues in Historiogra3456789012345678901234567890121234567890123456789012345678901212345
6
12
3456789012345678901234567890121234567890123456789012345678901212345
6
12
historical predecessor.
phy.
3456789012345678901234567890121234567890123456789012345678901212345 6
12
3456789012345678901234567890121234567890123456789012345678901212345
6
12
E. had minimal institutional continuity
C. Challenges in Intellectually Honest
3456789012345678901234567890121234567890123456789012345678901212345
6
12
3456789012345678901234567890121234567890123456789012345678901212345
6
12
with
its
historical
predecessor.
Historiography.
3456789012345678901234567890121234567890123456789012345678901212345 6
12
3456789012345678901234567890121234567890123456789012345678901212345
6
12
D. Methodologies in Intellectual
3456789012345678901234567890121234567890123456789012345678901212345
6
12
30.
The
author
employs
the
idea
that
there
are
2
3456789012345678901234567890121234567890123456789012345678901212345
6
123456789012345678901234567890121234567890123456789012345678901212345
Historiography.
6
1
diverse
narratives
(line
1213)
of
the
3456789012345678901234567890121234567890123456789012345678901212345
6
12
E.
Historiographys
Dark
Secret.
3456789012345678901234567890121234567890123456789012345678901212345
6
12
French
Revolution
to
support
the
conten3456789012345678901234567890121234567890123456789012345678901212345 6
12
2
3456789012345678901234567890121234567890123456789012345678901212345
6
123456789012345678901234567890121234567890123456789012345678901212345
tion that historiography
6
123456789012345678901234567890121234567890123456789012345678901212345
28. The second paragraphs role in the
6
123456789012345678901234567890121234567890123456789012345678901212345
passage as a whole is to
6
123456789012345678901234567890121234567890123456789012345678901212345
A.
is
merely
a
function
of
the
historians
123456789012345678901234567890121234567890123456789012345678901212345 66
123456789012345678901234567890121234567890123456789012345678901212345
biases.
A. present a view contrary to the one
6
123456789012345678901234567890121234567890123456789012345678901212345
6
123456789012345678901234567890121234567890123456789012345678901212345
B.
can be influenced by the historians
presented
in
the
introductory
para123456789012345678901234567890121234567890123456789012345678901212345 66
1
ideology.
graph.
3456789012345678901234567890121234567890123456789012345678901212345
6
12
3456789012345678901234567890121234567890123456789012345678901212345
6
12
C.
is not a science.
B.
conceptually
link
the
first
and
third
6
123456789012345678901234567890121234567890123456789012345678901212345
D. is the discipline of sophisticated story 6
123456789012345678901234567890121234567890123456789012345678901212345
paragraphs.
3456789012345678901234567890121234567890123456789012345678901212345
6
12
telling.
3456789012345678901234567890121234567890123456789012345678901212345
6
12
C. illustrate with examples the thesis of
3456789012345678901234567890121234567890123456789012345678901212345
6
12
E. can be manipulated by diverse
6
123456789012345678901234567890121234567890123456789012345678901212345
the
passage.
3456789012345678901234567890121234567890123456789012345678901212345
6
12
interests.
3456789012345678901234567890121234567890123456789012345678901212345
6
12
D.
offer
a
complimentary
interpretation
2
3456789012345678901234567890121234567890123456789012345678901212345
123456789012345678901234567890121234567890123456789012345678901212345 66
123456789012345678901234567890121234567890123456789012345678901212345
to the one given in the first paragraph.
6
1
6
123456789012345678901234567890121234567890123456789012345678901212345
E. attack the topic sentence of the first
123456789012345678901234567890121234567890123456789012345678901212345 6
2
3456789012345678901234567890121234567890123456789012345678901212345
6
123456789012345678901234567890121234567890123456789012345678901212345
paragraph.
6
123456789012345678901234567890121234567890123456789012345678901212345
123456789012345678901234567890121234567890123456789012345678901212345 66
123456789012345678901234567890121234567890123456789012345678901212345 6
1
3456789012345678901234567890121234567890123456789012345678901212345 6
12
3456789012345678901234567890121234567890123456789012345678901212345 6
12
3456789012345678901234567890121234567890123456789012345678901212345 6
12
3456789012345678901234567890121234567890123456789012345678901212345 6
12
123456789012345678901234567890121234567890123456789012345678901212345 6
2
6
123456789012345678901234567890121234567890123456789012345678901212345
6
123456789012345678901234567890121234567890123456789012345678901212345
6
123456789012345678901234567890121234567890123456789012345678901212345
6
123456789012345678901234567890121234567890123456789012345678901212345
1 3456789012345678901234567890121234567890123456789012345678901212345 6
2
3456789012345678901234567890121234567890123456789012345678901212345
123456789012345678901234567890121234567890123456789012345678901212345 66
1
3456789012345678901234567890121234567890123456789012345678901212345 6
12
123456789012345678901234567890121234567890123456789012345678901212345 6
3456789012345678901234567890121234567890123456789012345678901212345 6
12
123456789012345678901234567890121234567890123456789012345678901212345 6
123456789012345678901234567890121234567890123456789012345678901212345 6
123456789012345678901234567890121234567890123456789012345678901212345 66
270 123456789012345678901234567890121234567890123456789012345678901212345
1234567890123456789012345678901212345678901234567890123456789012123456

www.petersons.com

http://www.xtremepapers.net

Answers and Explanations


123456789012345678901234567890121234567890123456789012
2
12345678901234567890123456789012123456789012345678901
2
12345678901234567890123456789012123456789012345678901
The
following
notes
are
intended
to
show
you
one
way
in
which
the
2345678901234567890123456789012123456789012345678901
2
1
2
12345678901234567890123456789012123456789012345678901
correct answer can be determined. If you found the right answer using 2
12345678901234567890123456789012123456789012345678901
2
12345678901234567890123456789012123456789012345678901
another method, good for you! The more strategies you have at your 2
12345678901234567890123456789012123456789012345678901
2345678901234567890123456789012123456789012345678901
2
12345678901234567890123456789012123456789012345678901
disposal, the better off you will be.
2
12345678901234567890123456789012123456789012345678901
12345678901234567890123456789012123456789012345678901 22
12345678901234567890123456789012123456789012345678901 2
12345678901234567890123456789012123456789012345678901 2
12345678901234567890123456789012123456789012345678901 2
1Quantitative
2
12345678901234567890123456789012123456789012345678901
2
12345678901234567890123456789012123456789012345678901
1. C For Column A, the number of minutes in 6 hours (since h 5 6) 2
12345678901234567890123456789012123456789012345678901
2345678901234567890123456789012123456789012345678901
2
12345678901234567890123456789012123456789012345678901
can be found by multiplying: (6)(60 minutes) 5 360 minutes, or 2
12345678901234567890123456789012123456789012345678901
2
12345678901234567890123456789012123456789012345678901
360. For Column B, you must know that a square has four right 2
12345678901234567890123456789012123456789012345678901
2
12345678901234567890123456789012123456789012345678901
angles, all of which equal 90 degrees. This means Column B 2
12345678901234567890123456789012123456789012345678901
2
12345678901234567890123456789012123456789012345678901
works out to: (4)(90) 5 360.
2
12345678901234567890123456789012123456789012345678901
12345678901234567890123456789012123456789012345678901 22
1
This first problem is fairly straightforward, and even though 2
12345678901234567890123456789012123456789012345678901
12345678901234567890123456789012123456789012345678901
there was a variable involved, definite real numbers were found 2
2
12345678901234567890123456789012123456789012345678901
2
12345678901234567890123456789012123456789012345678901
for
both
columns.
Be
aware
that
this
will
change
as
the
2
12345678901234567890123456789012123456789012345678901
2345678901234567890123456789012123456789012345678901
12345678901234567890123456789012123456789012345678901
questions get harder. On later problems, the use of variables will 2
2
12345678901234567890123456789012123456789012345678901
2
1
lead
to
column
quantities
that
have
more
than
one
possible
2
12345678901234567890123456789012123456789012345678901
12345678901234567890123456789012123456789012345678901
answer, so even though these problems will resemble this earlier 2
2345678901234567890123456789012123456789012345678901
12345678901234567890123456789012123456789012345678901 22
12345678901234567890123456789012123456789012345678901
one, finding the correct answer will not be as straightforward.
2
1
2
12345678901234567890123456789012123456789012345678901
2.
D
Since
they
are
a
simple
shape
with
a
ton
of
special
cases
and
12345678901234567890123456789012123456789012345678901 2
2345678901234567890123456789012123456789012345678901
2
12345678901234567890123456789012123456789012345678901
associated rules, triangles make many appearances on the 2
12345678901234567890123456789012123456789012345678901
2
12345678901234567890123456789012123456789012345678901
standard GRE. On question 2, the triangle has a right angle, 2
12345678901234567890123456789012123456789012345678901
1
meaning the Pythagorean theorem can be applied. Start with the 2
2
12345678901234567890123456789012123456789012345678901
2
12345678901234567890123456789012123456789012345678901
basic
Pythagorean
theorem:
2
12345678901234567890123456789012123456789012345678901
2
12345678901234567890123456789012123456789012345678901
2
2
2
2
12345678901234567890123456789012123456789012345678901
1
b
5
c
a
2345678901234567890123456789012123456789012345678901
12345678901234567890123456789012123456789012345678901 22
12345678901234567890123456789012123456789012345678901
2
12345678901234567890123456789012123456789012345678901
a2 1 b2 2 b2 5 c2 2 b2 (This subtracts b2from both sides)
2
12345678901234567890123456789012123456789012345678901
2
1
2
2
2
a
5
c
2
b
2345678901234567890123456789012123456789012345678901
12345678901234567890123456789012123456789012345678901 22
1
12345678901234567890123456789012123456789012345678901
Column A can be written as either a2 or c2 2 b2. As for Column 2
2
12345678901234567890123456789012123456789012345678901
2
12345678901234567890123456789012123456789012345678901
B,
the
hypotenuse
is
length
c,
so
the
square
of
the
hypotenuse
2345678901234567890123456789012123456789012345678901
2
1
2
2
12345678901234567890123456789012123456789012345678901
minus
itself
would
be
c
2
c.
Comparing
this
to
Column
A,
you
12345678901234567890123456789012123456789012345678901 22
12345678901234567890123456789012123456789012345678901
123456789012345678901234567890121234567890123456789012
271

http://www.xtremepapers.net

Part IV: Three Practice GREs

123456789012345678901234567890121234567890123456789012
12345678901234567890123456789012123456789012345678901 2
2
12345678901234567890123456789012123456789012345678901
can see that both have an initial c2 that is then subtracted by 2
12345678901234567890123456789012123456789012345678901
2
12345678901234567890123456789012123456789012345678901
either b2(Column A) or c (Column B). The problem boils down 2
12345678901234567890123456789012123456789012345678901
2
12345678901234567890123456789012123456789012345678901
to how c compares with b2. Frankly, theres no way to know. 2
12345678901234567890123456789012123456789012345678901
2345678901234567890123456789012123456789012345678901
2
1
The variable b could be smaller than one, in which case its 2
12345678901234567890123456789012123456789012345678901
12345678901234567890123456789012123456789012345678901
square would be even smaller. Depending on which numbers 2
2
12345678901234567890123456789012123456789012345678901
2
12345678901234567890123456789012123456789012345678901
you
plug
in,
Column
A
or
Column
B
might
be
greater.
2345678901234567890123456789012123456789012345678901
2
1
2
12345678901234567890123456789012123456789012345678901
2
12345678901234567890123456789012123456789012345678901
3.
D
The
perimeter
of
the
square
is
4h,
but
you
do
not
know
how
this
2
12345678901234567890123456789012123456789012345678901
2
2
12345678901234567890123456789012123456789012345678901
relates
to
h
.
If,
h
5
1
then
Column
B
is
greater,
but
if
h
5
10,
2345678901234567890123456789012123456789012345678901
2
1
2
12345678901234567890123456789012123456789012345678901
then
Column
A
is
greater.
Note
that
if
you
confuse
the
area
of
a
2
12345678901234567890123456789012123456789012345678901
2
12345678901234567890123456789012123456789012345678901
square
with
its
perimeter,
you
will
find
the
two
columns
are
2
12345678901234567890123456789012123456789012345678901
2345678901234567890123456789012123456789012345678901
2
1
equal, leading you to pick the incorrect answer (C).
2
12345678901234567890123456789012123456789012345678901
2
12345678901234567890123456789012123456789012345678901
4. D You might jump to say that the answer is (C), but that would be a 2
12345678901234567890123456789012123456789012345678901
2
12345678901234567890123456789012123456789012345678901
little hasty. The absolute value sign (those two bars on either side) 2
12345678901234567890123456789012123456789012345678901
2
12345678901234567890123456789012123456789012345678901
means that x could be either 3 or 23, and that y could equal either 2
12345678901234567890123456789012123456789012345678901
2
12345678901234567890123456789012123456789012345678901
4 or 24. Remember that |y 2 x| |y| 2 |x| 5 1. We actually do not 2
12345678901234567890123456789012123456789012345678901
2345678901234567890123456789012123456789012345678901
12345678901234567890123456789012123456789012345678901
know what |y 2 x| equals. It could be seven, or it could be one. The 2
2
1
2
12345678901234567890123456789012123456789012345678901
answer,
then,
is
(D).
2
12345678901234567890123456789012123456789012345678901
2
12345678901234567890123456789012123456789012345678901
2345678901234567890123456789012123456789012345678901
12345678901234567890123456789012123456789012345678901
5. A For people rusty with their math terminology, this question 2
2
12345678901234567890123456789012123456789012345678901
2
12345678901234567890123456789012123456789012345678901
looks
much
more
difficult
that
it
really
is.
The
three
smallest
12345678901234567890123456789012123456789012345678901 2
12345678901234567890123456789012123456789012345678901
prime numbers are 2, 3, and 5, and their sum is 10. As for 2
2
12345678901234567890123456789012123456789012345678901
2
12345678901234567890123456789012123456789012345678901
Column
B,
the
square
root
of
49
is
7.
You
probably
dont
know
12345678901234567890123456789012123456789012345678901 2
12345678901234567890123456789012123456789012345678901
the square root of 8 off the top of your head, but you do know 2
2
1
2345678901234567890123456789012123456789012345678901
12345678901234567890123456789012123456789012345678901
that the square root of 9 is 3. This means the square root of 8 2
2
12345678901234567890123456789012123456789012345678901
1
must be something less than 3. Seven 1 something less than 3 5 2
12345678901234567890123456789012123456789012345678901 2
12345678901234567890123456789012123456789012345678901
something a little less than 10. Therefore, Column A is greater. 2
2
12345678901234567890123456789012123456789012345678901
2
12345678901234567890123456789012123456789012345678901
6. A In looking at the figure, we can reason that a 1 b 5 180 since 2
12345678901234567890123456789012123456789012345678901
2
12345678901234567890123456789012123456789012345678901
the lines are parallel. Thus, we need to manipulate Column A so 2
12345678901234567890123456789012123456789012345678901
2345678901234567890123456789012123456789012345678901
12345678901234567890123456789012123456789012345678901
that we can compare it to a 1 b. In the numerator, we can factor 2
2
12345678901234567890123456789012123456789012345678901
2
1
out
an
a
and
then
eliminate
it
in
the
following
manner:
12345678901234567890123456789012123456789012345678901 2
12345678901234567890123456789012123456789012345678901 2
2
12345678901234567890123456789012123456789012345678901
2
12345678901234567890123456789012123456789012345678901
a2 1 ab 1 a a(a 1 b 1 1) a(a 1 b 1 1)
2
12345678901234567890123456789012123456789012345678901
5
5
5a1b11
2
12345678901234567890123456789012123456789012345678901
a
a
a
12345678901234567890123456789012123456789012345678901 2
2345678901234567890123456789012123456789012345678901
2
12345678901234567890123456789012123456789012345678901
This leaves us with a 1 b 1 1, which is 1 greater than Column 2
12345678901234567890123456789012123456789012345678901
2
12345678901234567890123456789012123456789012345678901
Bs a 1 b 5 180. Therefore, Column A is greater.
2
12345678901234567890123456789012123456789012345678901
2
1
2345678901234567890123456789012123456789012345678901
12345678901234567890123456789012123456789012345678901
7. B A good way to attack a problem like this is to sketch it out. If J is 2
2
12345678901234567890123456789012123456789012345678901
2
12345678901234567890123456789012123456789012345678901
space
1
then
there
are
two
possibilities:
JBA
and
JAB.
If
(B)
is
in
12345678901234567890123456789012123456789012345678901 2
1
space 1, BJA and BAJ. If (A) is in space 1, AJB and ABJ. That is 2
2
12345678901234567890123456789012123456789012345678901
2
12345678901234567890123456789012123456789012345678901
six
distinct
arrangements.
Can
you
get
anymore?
Try
to
put
J
in
2
12345678901234567890123456789012123456789012345678901
2
12345678901234567890123456789012123456789012345678901
space
2,
AJB
or
BJA.
You
already
have
both
of
those
counted.
2
12345678901234567890123456789012123456789012345678901
2345678901234567890123456789012123456789012345678901
2
1
12345678901234567890123456789012123456789012345678901 2
12345678901234567890123456789012123456789012345678901 2
2
272 12345678901234567890123456789012123456789012345678901
123456789012345678901234567890121234567890123456789012

www.petersons.com

http://www.xtremepapers.net

ANSWERS
123456789012345678901234567890121234567890123456789012
12345678901234567890123456789012123456789012345678901 2
2
12345678901234567890123456789012123456789012345678901
Try any other arrangement, and you will see that it is a repeat. 2
12345678901234567890123456789012123456789012345678901
2
12345678901234567890123456789012123456789012345678901
(B) is greater.
2
12345678901234567890123456789012123456789012345678901
12345678901234567890123456789012123456789012345678901 2
12345678901234567890123456789012123456789012345678901
8. C This one is tricky, but not if you recall that p is defined as the 2
2
12345678901234567890123456789012123456789012345678901
2
12345678901234567890123456789012123456789012345678901
ratio
of
the
circumference
divided
by
the
diameter.
If
you
didnt
2
12345678901234567890123456789012123456789012345678901
2
12345678901234567890123456789012123456789012345678901
recall
this,
you
still
could
have
tried
to
find
an
equation
that
2
12345678901234567890123456789012123456789012345678901
2345678901234567890123456789012123456789012345678901
2
1
C
2
12345678901234567890123456789012123456789012345678901
.
As
for
related
all
three
entities,
such
as
C
5
2pr
5
pd

p
5
2
12345678901234567890123456789012123456789012345678901
d
2
12345678901234567890123456789012123456789012345678901
2
12345678901234567890123456789012123456789012345678901
C
2C
2345678901234567890123456789012123456789012345678901
1
5 5 p. The 2
Column B it can be manipulated like this,
2
12345678901234567890123456789012123456789012345678901
2d
d
2
12345678901234567890123456789012123456789012345678901
columns
are
equal,
so
the
answer
is
(C).
2
12345678901234567890123456789012123456789012345678901
2
12345678901234567890123456789012123456789012345678901
2345678901234567890123456789012123456789012345678901
2
1 9. D This requires straightforward arithmetic work.
2
12345678901234567890123456789012123456789012345678901
2
12345678901234567890123456789012123456789012345678901
2
12345678901234567890123456789012123456789012345678901
Column
B
Column
A
2
12345678901234567890123456789012123456789012345678901
2345678901234567890123456789012123456789012345678901
2
12345678901234567890123456789012123456789012345678901
2
2
12345678901234567890123456789012123456789012345678901
a
b
1
a
1
2
12345678901234567890123456789012123456789012345678901
b
2
4
3
2
12a
2
2
12345678901234567890123456789012123456789012345678901
2
a
6
3b
12345678901234567890123456789012123456789012345678901 22
12345678901234567890123456789012123456789012345678901
ab
a
1
2
1
5
2 12
5 3b 2
2 12
2
12345678901234567890123456789012123456789012345678901
2
6 3b
2
12345678901234567890123456789012123456789012345678901
2
12345678901234567890123456789012123456789012345678901
2345678901234567890123456789012123456789012345678901
2
12345678901234567890123456789012123456789012345678901
ab
2
12345678901234567890123456789012123456789012345678901
5
2 1 2 12
2
12345678901234567890123456789012123456789012345678901
2
12345678901234567890123456789012123456789012345678901 22
12345678901234567890123456789012123456789012345678901 2
12345678901234567890123456789012123456789012345678901
You can cancel out the 212, since both columns share it. 2
12345678901234567890123456789012123456789012345678901
12345678901234567890123456789012123456789012345678901
Column A and B share the first term, but Column B is the first 2
2
12345678901234567890123456789012123456789012345678901
1
term 21. Since we are dealing with positive numbers here, that 2
2345678901234567890123456789012123456789012345678901
12345678901234567890123456789012123456789012345678901 22
12345678901234567890123456789012123456789012345678901
makes Column A greater.
2
1
2
12345678901234567890123456789012123456789012345678901
10.
B
The
only
way
that
your
triangle
could
be
a
triangle
is
if
the
third
2
12345678901234567890123456789012123456789012345678901
2345678901234567890123456789012123456789012345678901
2
12345678901234567890123456789012123456789012345678901
side is greater than 6. If it is not, then the three sides would not 2
12345678901234567890123456789012123456789012345678901
2
1
connect, making an enclosed figure with 3 angles and 3 sides. 2
12345678901234567890123456789012123456789012345678901
12345678901234567890123456789012123456789012345678901
This is readily apparent if you attempt to make a triangle with 2
2
12345678901234567890123456789012123456789012345678901
2
12345678901234567890123456789012123456789012345678901
sides
1,
4,
and
10.
Thus,
(B)
is
greater.
12345678901234567890123456789012123456789012345678901 2
12345678901234567890123456789012123456789012345678901 2
12345678901234567890123456789012123456789012345678901
11. C The number set is {10, 12, 14, 16, 18}, so, the median is 14. The 2
2
12345678901234567890123456789012123456789012345678901
2
12345678901234567890123456789012123456789012345678901
mean
of
any
evenly
spaced
number
set
is
always
the
median,
so
2
12345678901234567890123456789012123456789012345678901
2
12345678901234567890123456789012123456789012345678901
the
mean
is
also
14
(you
could
also
just
sum
the
numbers
and
12345678901234567890123456789012123456789012345678901 2
2345678901234567890123456789012123456789012345678901
2
12345678901234567890123456789012123456789012345678901
divide by 5).
2
12345678901234567890123456789012123456789012345678901
12345678901234567890123456789012123456789012345678901 22
12345678901234567890123456789012123456789012345678901
12. B This one is greatly simplified if you realize that you are
2
1
looking at a right triangle here, and you know two of its sides, 2
2345678901234567890123456789012123456789012345678901
12345678901234567890123456789012123456789012345678901
2
12345678901234567890123456789012123456789012345678901
namely 3 and 4. Using the Pythagorean theorem or recognizing 2
12345678901234567890123456789012123456789012345678901
2
12345678901234567890123456789012123456789012345678901
it is a 3:4:5 triangle, you can determine the third side is 5. That 2
1
2345678901234567890123456789012123456789012345678901
2
12345678901234567890123456789012123456789012345678901
is the value of Column A. For Column B, the slope of GH can 2
1
12345678901234567890123456789012123456789012345678901
be determined by simply counting the rise (4) and the run (3) 2
2
12345678901234567890123456789012123456789012345678901
2
12345678901234567890123456789012123456789012345678901
2345678901234567890123456789012123456789012345678901
2
1
12345678901234567890123456789012123456789012345678901 2
12345678901234567890123456789012123456789012345678901 2
12345678901234567890123456789012123456789012345678901 2 273
123456789012345678901234567890121234567890123456789012

GS D F S

D G
D

http://www.xtremepapers.net

Part IV: Three Practice GREs

123456789012345678901234567890121234567890123456789012
12345678901234567890123456789012123456789012345678901 2
2
12345678901234567890123456789012123456789012345678901
4
2
12345678901234567890123456789012123456789012345678901
12345678901234567890123456789012123456789012345678901
and putting the former over the latter, 2 . Now multiply it, 2
2
12345678901234567890123456789012123456789012345678901
3
2
12345678901234567890123456789012123456789012345678901
4
16
1
2
12345678901234567890123456789012123456789012345678901
2
24
5
5
5
,
which
is
greater
than
5.
~
!
2345678901234567890123456789012123456789012345678901
2
1
3
3
3
2
12345678901234567890123456789012123456789012345678901
2
12345678901234567890123456789012123456789012345678901
13. B The greatest that Column A could be is 9 1 9 1 8 5 26, since 2
12345678901234567890123456789012123456789012345678901
2
12345678901234567890123456789012123456789012345678901
A and B are distinct digits, so the answer then is (B).
2
12345678901234567890123456789012123456789012345678901
2
12345678901234567890123456789012123456789012345678901
2
12345678901234567890123456789012123456789012345678901
14.
A
A
good
way
to
approach
this
problem
is
make
a
table
filling
in
2
12345678901234567890123456789012123456789012345678901
2
12345678901234567890123456789012123456789012345678901
the
information
you
do
know
and
from
that
trying
to
determine
2345678901234567890123456789012123456789012345678901
2
1
2
12345678901234567890123456789012123456789012345678901
the
information
you
need
to
answer
the
question.
2
12345678901234567890123456789012123456789012345678901
2
12345678901234567890123456789012123456789012345678901
2
12345678901234567890123456789012123456789012345678901
If
there
are
73
total
animals
and
35
dogs,
then
there
are
38
cats.
2345678901234567890123456789012123456789012345678901
2
1
2
12345678901234567890123456789012123456789012345678901
If
there
are
40
females,
then
there
are
33
males.
If
x
is
the
2
12345678901234567890123456789012123456789012345678901
12345678901234567890123456789012123456789012345678901
number of male cats, then the number of male dogs is x 19, 2
2
12345678901234567890123456789012123456789012345678901
2
12345678901234567890123456789012123456789012345678901
since it states there are 9 less male cats than dogs.
2
12345678901234567890123456789012123456789012345678901
2
12345678901234567890123456789012123456789012345678901
The total number of male animals is 33, so x 1 (x 1 9) 5 33 2
12345678901234567890123456789012123456789012345678901
2
12345678901234567890123456789012123456789012345678901
2x 5 24 x 5 12. There are 12 male cats and 21 (x 1 9) male 2
12345678901234567890123456789012123456789012345678901
2
12345678901234567890123456789012123456789012345678901
dogs. The total number of cats is 38, so that means there are 26 2
12345678901234567890123456789012123456789012345678901
2
12345678901234567890123456789012123456789012345678901
female cats.
2
12345678901234567890123456789012123456789012345678901
2345678901234567890123456789012123456789012345678901
12345678901234567890123456789012123456789012345678901 2
12345678901234567890123456789012123456789012345678901
15. C There are two ways to find the answer. One way is to plug in the 2
2
12345678901234567890123456789012123456789012345678901
2
12345678901234567890123456789012123456789012345678901
answer
choices
and
see
which
ones
work
or
solve
the
binomial
12345678901234567890123456789012123456789012345678901 2
2
12345678901234567890123456789012123456789012345678901
expression. You can also solve the equation:
2
12345678901234567890123456789012123456789012345678901
2
12345678901234567890123456789012123456789012345678901
2
2
12345678901234567890123456789012123456789012345678901
5
22x
2
15

2
x
2
1
2345678901234567890123456789012123456789012345678901
2
12345678901234567890123456789012123456789012345678901
2 x2 1 x2 5 22x 2 15 1 x2
2
12345678901234567890123456789012123456789012345678901
2
1
2
2
12345678901234567890123456789012123456789012345678901
0 5 x 2 2x 2 15
2
12345678901234567890123456789012123456789012345678901
2345678901234567890123456789012123456789012345678901
2
12345678901234567890123456789012123456789012345678901
0 5 ~x 2 5!~x 1 3!
2
12345678901234567890123456789012123456789012345678901
2
1
x
5
23,
5
2
12345678901234567890123456789012123456789012345678901
2
12345678901234567890123456789012123456789012345678901
2345678901234567890123456789012123456789012345678901
12345678901234567890123456789012123456789012345678901
16. E The first thing to do is to convert each answer choice into 2
2
12345678901234567890123456789012123456789012345678901
2
1
y
5
mx
1
b
form.
From
the
graph,
you
can
figure
that
the
slope
12345678901234567890123456789012123456789012345678901 2
12345678901234567890123456789012123456789012345678901
is 21, and the y-intercept is 22. Now you can simply look for 2
2
12345678901234567890123456789012123456789012345678901
2
12345678901234567890123456789012123456789012345678901
the
equation
with
these
characteristics,
and
(E)
is
it.
2
12345678901234567890123456789012123456789012345678901
2
12345678901234567890123456789012123456789012345678901
12345678901234567890123456789012123456789012345678901
17. C Eyeball the graph to see which months have both adult and 2
2345678901234567890123456789012123456789012345678901
2
12345678901234567890123456789012123456789012345678901
children graphs near the zero line in the middle. Both Januaries 2
12345678901234567890123456789012123456789012345678901
2
12345678901234567890123456789012123456789012345678901
do, but 1963 is clearly a smaller total number. Thus, (C) is 2
12345678901234567890123456789012123456789012345678901
2
1
correct.
2
12345678901234567890123456789012123456789012345678901
2
12345678901234567890123456789012123456789012345678901
2
12345678901234567890123456789012123456789012345678901
18.
B
If
you
notice
that
MarchApril
1962
is
the
only
time
the
child
2
12345678901234567890123456789012123456789012345678901
2
12345678901234567890123456789012123456789012345678901
flu
incidence
exceeds
100,
this
one
will
be
straightforward:
2345678901234567890123456789012123456789012345678901
12345678901234567890123456789012123456789012345678901 2
2
1
answer choice (B).
2
12345678901234567890123456789012123456789012345678901
12345678901234567890123456789012123456789012345678901 2
12345678901234567890123456789012123456789012345678901
19. C If the area of the top of the box is 15, then the third side is of 2
2
12345678901234567890123456789012123456789012345678901
12345678901234567890123456789012123456789012345678901
length 5 since this length times 3 must yield an area of 15. Now 2
12345678901234567890123456789012123456789012345678901 2
2
274 12345678901234567890123456789012123456789012345678901
123456789012345678901234567890121234567890123456789012

S D

www.petersons.com

http://www.xtremepapers.net

ANSWERS
123456789012345678901234567890121234567890123456789012
12345678901234567890123456789012123456789012345678901 2
2
12345678901234567890123456789012123456789012345678901
that you know all three lengths you just need to add up the areas 2
12345678901234567890123456789012123456789012345678901
2
12345678901234567890123456789012123456789012345678901
of all six sides. Four of the sides have an area of 15, which sums 2
12345678901234567890123456789012123456789012345678901
2
12345678901234567890123456789012123456789012345678901
to 60. The two end sides have an area of 9, which sums to 18. 2
12345678901234567890123456789012123456789012345678901
2345678901234567890123456789012123456789012345678901
2
1
Add these together and you get (C).
2
12345678901234567890123456789012123456789012345678901
2
12345678901234567890123456789012123456789012345678901
2
12345678901234567890123456789012123456789012345678901
20.
B
Begin
by
determining
the
median
and
the
mode.
The
mode
is
2
12345678901234567890123456789012123456789012345678901
2345678901234567890123456789012123456789012345678901
1
simply the number that occurs the most times in the set. That is 2
2
12345678901234567890123456789012123456789012345678901
2
12345678901234567890123456789012123456789012345678901
clearly
7.
The
median
is
the
number
that
is
in
the
middle
if
you
2
12345678901234567890123456789012123456789012345678901
2
12345678901234567890123456789012123456789012345678901
line
them
up
from
least
to
greatest.
The
two
middle
numbers
are
2345678901234567890123456789012123456789012345678901
2
1
2
12345678901234567890123456789012123456789012345678901
7,
so
halfway
between
them
is
still
just
7.
The
median
minus
the
2
12345678901234567890123456789012123456789012345678901
2
12345678901234567890123456789012123456789012345678901
mode
then
is
7
2
7
5
0.
2
12345678901234567890123456789012123456789012345678901
2345678901234567890123456789012123456789012345678901
2
1
12345678901234567890123456789012123456789012345678901
21. B Look to the number chart to find this one. The ratio we are 2
2
12345678901234567890123456789012123456789012345678901
looking for is criminal/civil. Remember that you are looking for 2
12345678901234567890123456789012123456789012345678901
2
12345678901234567890123456789012123456789012345678901
the quarter that has the greatest ratio and not the quarter with 2
12345678901234567890123456789012123456789012345678901
2
12345678901234567890123456789012123456789012345678901
the largest numbers. Look for numbers whose difference is great, 2
12345678901234567890123456789012123456789012345678901
2
12345678901234567890123456789012123456789012345678901
and then compare. The 2nd quarter in 1970 is the answer, 2
12345678901234567890123456789012123456789012345678901
2345678901234567890123456789012123456789012345678901
2
12345678901234567890123456789012123456789012345678901
choice (B).
2
1
2
12345678901234567890123456789012123456789012345678901
2
12345678901234567890123456789012123456789012345678901
22.
B
You
need
to
be
careful
on
this
one.
It
has
a
number
of
steps,
but
2
12345678901234567890123456789012123456789012345678901
2345678901234567890123456789012123456789012345678901
12345678901234567890123456789012123456789012345678901
none of them are hard. First, estimate how many female 2
2
12345678901234567890123456789012123456789012345678901
2
12345678901234567890123456789012123456789012345678901
criminal
lawyers
there
are
(approximately
10,000).
Now
12345678901234567890123456789012123456789012345678901 22
12345678901234567890123456789012123456789012345678901
estimate how many female civil lawyers there are (approxi- 2
12345678901234567890123456789012123456789012345678901
12345678901234567890123456789012123456789012345678901
mately 20,000). So the ratio is, 20,000 to 10,000, which reduces 2
2
12345678901234567890123456789012123456789012345678901
2
12345678901234567890123456789012123456789012345678901
to 2:1 or (B).
2
1
2345678901234567890123456789012123456789012345678901
12345678901234567890123456789012123456789012345678901 22
23. C As the graph headings make clear, (A) and (B) are not fields of
12345678901234567890123456789012123456789012345678901
2
1
law but are types of law. With that obstacle out of the way, you 2
12345678901234567890123456789012123456789012345678901
2
12345678901234567890123456789012123456789012345678901
merely need to add up the bar graph pairs to see which is the 2
12345678901234567890123456789012123456789012345678901
2
12345678901234567890123456789012123456789012345678901
greatest.
2
12345678901234567890123456789012123456789012345678901
2
12345678901234567890123456789012123456789012345678901
2
12345678901234567890123456789012123456789012345678901
24.
A
Since
you
know
the
area
of
the
rectangle
and
one
of
the
sides,
2345678901234567890123456789012123456789012345678901
12345678901234567890123456789012123456789012345678901 22
12345678901234567890123456789012123456789012345678901
you can deduce the length of the second side ~BC 5 3!. This 2
1
12345678901234567890123456789012123456789012345678901
side also happens to be the radius of the quarter circle. Thus the 2
2
12345678901234567890123456789012123456789012345678901
2345678901234567890123456789012123456789012345678901
12345678901234567890123456789012123456789012345678901
circumference of the full circle is C 5 2p(3) 5 6p. But you only 2
2
12345678901234567890123456789012123456789012345678901
2
12345678901234567890123456789012123456789012345678901
1
3
2
12345678901234567890123456789012123456789012345678901
5
need
a
quarter
of
the
circumference,
which
is
6p
p.
Now
2
1
4
2
2345678901234567890123456789012123456789012345678901
2
12345678901234567890123456789012123456789012345678901
CD 5 4 and EC 5 3, so ED 5 1. So, (A) is the answer.
2
12345678901234567890123456789012123456789012345678901
12345678901234567890123456789012123456789012345678901 22
12345678901234567890123456789012123456789012345678901 2
1
2
12345678901234567890123456789012123456789012345678901
2
12345678901234567890123456789012123456789012345678901
2
12345678901234567890123456789012123456789012345678901
2
12345678901234567890123456789012123456789012345678901
12345678901234567890123456789012123456789012345678901 2
2345678901234567890123456789012123456789012345678901
12345678901234567890123456789012123456789012345678901 22
1
2
12345678901234567890123456789012123456789012345678901
12345678901234567890123456789012123456789012345678901 2
2
12345678901234567890123456789012123456789012345678901
12345678901234567890123456789012123456789012345678901 2
12345678901234567890123456789012123456789012345678901 2
12345678901234567890123456789012123456789012345678901 2
12345678901234567890123456789012123456789012345678901 2 275
123456789012345678901234567890121234567890123456789012

SD

http://www.xtremepapers.net

Part IV: Three Practice GREs

123456789012345678901234567890121234567890123456789012
12345678901234567890123456789012123456789012345678901 2
2
12345678901234567890123456789012123456789012345678901
25. C The longest distance between two corners in a rectangular 2
12345678901234567890123456789012123456789012345678901
2
12345678901234567890123456789012123456789012345678901
boxthough sounding difficult to findis actually rather easy 2
12345678901234567890123456789012123456789012345678901
2
12345678901234567890123456789012123456789012345678901
to compute. Simply square all three sides, sum them, then take 2
12345678901234567890123456789012123456789012345678901
2345678901234567890123456789012123456789012345678901
2
1
the square root of this sum and you have it. Here it would be:
2
12345678901234567890123456789012123456789012345678901
2
12345678901234567890123456789012123456789012345678901
2
12345678901234567890123456789012123456789012345678901
2
2
2
2 13 14 5
2
12345678901234567890123456789012123456789012345678901
2345678901234567890123456789012123456789012345678901
2
1
2
12345678901234567890123456789012123456789012345678901
4
1
9
1
16
5
29
2
12345678901234567890123456789012123456789012345678901
2
12345678901234567890123456789012123456789012345678901
whose square root is (C). The other way to do this problem is to 2
12345678901234567890123456789012123456789012345678901
2345678901234567890123456789012123456789012345678901
2
1
draw the figure and start using the Pythagorean theorem. You 2
12345678901234567890123456789012123456789012345678901
2
12345678901234567890123456789012123456789012345678901
need to find the diagonal of one of the rectangular sides, and 2
12345678901234567890123456789012123456789012345678901
2
12345678901234567890123456789012123456789012345678901
then create another triangle using that diagonal, the remaining 2
12345678901234567890123456789012123456789012345678901
12345678901234567890123456789012123456789012345678901
side, and a hypotenuse, which just happens to be the diagonal 2
2
12345678901234567890123456789012123456789012345678901
2
12345678901234567890123456789012123456789012345678901
from
one
corner
of
the
box
to
another.
You
end
up
with
the
2
12345678901234567890123456789012123456789012345678901
2345678901234567890123456789012123456789012345678901
2
12345678901234567890123456789012123456789012345678901
same answer, (C).
2
12345678901234567890123456789012123456789012345678901
12345678901234567890123456789012123456789012345678901 2
12345678901234567890123456789012123456789012345678901
26. B The overall average is the sum of all the scores of all the classes 2
2
12345678901234567890123456789012123456789012345678901
12345678901234567890123456789012123456789012345678901
divided by the total number of students. You do not have either 2
2
1
2
12345678901234567890123456789012123456789012345678901
of
those
numbers,
so
you
have
to
arrive
at
the
overall
average
in
2
12345678901234567890123456789012123456789012345678901
12345678901234567890123456789012123456789012345678901
some other fashion. A weighted average would do the trick here. 2
2345678901234567890123456789012123456789012345678901
12345678901234567890123456789012123456789012345678901 2
12345678901234567890123456789012123456789012345678901
Since there are twice as many fifth graders, you weight their 2
2
12345678901234567890123456789012123456789012345678901
12345678901234567890123456789012123456789012345678901
average with a coefficient of two and the sixth grade average 2
12345678901234567890123456789012123456789012345678901 2
2
12345678901234567890123456789012123456789012345678901
with a coefficient of one.
2
12345678901234567890123456789012123456789012345678901
12345678901234567890123456789012123456789012345678901 2
2
12345678901234567890123456789012123456789012345678901
~2!~15! 1 ~1!~18! 5 48 5 16
2
1
2345678901234567890123456789012123456789012345678901
2
12345678901234567890123456789012123456789012345678901
3
3
2
12345678901234567890123456789012123456789012345678901
2
1
12345678901234567890123456789012123456789012345678901
27. C On this graph there are two x values for every y value; thus, it is 2
2
12345678901234567890123456789012123456789012345678901
the x variable that should be squared. Furthermore, when x 5 0 2
12345678901234567890123456789012123456789012345678901
2
12345678901234567890123456789012123456789012345678901
on this graph, y 5 1, so you need an equation that reflects this. 2
12345678901234567890123456789012123456789012345678901
2
12345678901234567890123456789012123456789012345678901
That is true of (C).
2
12345678901234567890123456789012123456789012345678901
2345678901234567890123456789012123456789012345678901
12345678901234567890123456789012123456789012345678901 2
12345678901234567890123456789012123456789012345678901
find the area of the circle since you know the radius (a 2
2
1 28. E You can
2
2
12345678901234567890123456789012123456789012345678901
5
16p).
If
you
find
the
area
of
the
two
squares
then
you
5
gpr
12345678901234567890123456789012123456789012345678901 2
2345678901234567890123456789012123456789012345678901
12345678901234567890123456789012123456789012345678901
can subtract from the area of the circle to find the area of the 2
2
12345678901234567890123456789012123456789012345678901
2
12345678901234567890123456789012123456789012345678901
shaded
region.
The
diagonal
of
each
square
is
the
radius
of
the
12345678901234567890123456789012123456789012345678901 2
1
circle, so this number is 4. From this you can deduce that the 2
2
12345678901234567890123456789012123456789012345678901
12345678901234567890123456789012123456789012345678901
sides of the square are 2=2. Thus the area of each square is 8. 2
2
12345678901234567890123456789012123456789012345678901
2
12345678901234567890123456789012123456789012345678901
Incidentally,
you
know
that
the
squares
are
congruent
because
12345678901234567890123456789012123456789012345678901 2
2345678901234567890123456789012123456789012345678901
12345678901234567890123456789012123456789012345678901
the length of their diagonal is the same. This makes the area in 2
2
12345678901234567890123456789012123456789012345678901
2
12345678901234567890123456789012123456789012345678901
the
two
squares
16.
The
area
of
the
circle
minus
the
area
of
the
12345678901234567890123456789012123456789012345678901 2
2
1
squares is 16p 2 16, which is equivalent to (E).
2
12345678901234567890123456789012123456789012345678901
12345678901234567890123456789012123456789012345678901 2
2
12345678901234567890123456789012123456789012345678901
12345678901234567890123456789012123456789012345678901 2
2
12345678901234567890123456789012123456789012345678901
12345678901234567890123456789012123456789012345678901 2
12345678901234567890123456789012123456789012345678901 2
12345678901234567890123456789012123456789012345678901 2
2
276 12345678901234567890123456789012123456789012345678901
123456789012345678901234567890121234567890123456789012

www.petersons.com

http://www.xtremepapers.net

ANSWERS
123456789012345678901234567890121234567890123456789012
12345678901234567890123456789012123456789012345678901 2
2
12345678901234567890123456789012123456789012345678901
2
12345678901234567890123456789012123456789012345678901
Verbal
2
12345678901234567890123456789012123456789012345678901
2
12345678901234567890123456789012123456789012345678901
Antonyms
2
12345678901234567890123456789012123456789012345678901
2
12345678901234567890123456789012123456789012345678901
2345678901234567890123456789012123456789012345678901
1 1. B To have zeal is to have great enthusiasm. The opposite of zeal is 2
2
12345678901234567890123456789012123456789012345678901
2
12345678901234567890123456789012123456789012345678901
apathy,
choice
(B).
2
12345678901234567890123456789012123456789012345678901
2
12345678901234567890123456789012123456789012345678901
2345678901234567890123456789012123456789012345678901
1 2. C Heterogeneous is the antonym of homogeneous. Homogeneous 2
2
12345678901234567890123456789012123456789012345678901
2
12345678901234567890123456789012123456789012345678901
means
to
be
uniform
in
kind
or
quality;
similar.
Choice
(C)
2
12345678901234567890123456789012123456789012345678901
2
12345678901234567890123456789012123456789012345678901
comes
closest
to
this.
2345678901234567890123456789012123456789012345678901
2
1
2
12345678901234567890123456789012123456789012345678901
12345678901234567890123456789012123456789012345678901
3. A Fealty is loyalty. Thus, if you know the definition, this antonym 2
2
12345678901234567890123456789012123456789012345678901
is straightforward, disloyalty, choice (A).
2
12345678901234567890123456789012123456789012345678901
2345678901234567890123456789012123456789012345678901
2
1
12345678901234567890123456789012123456789012345678901
4. E Something archaic is old or antiquated. A good pre-guess then 2
2
12345678901234567890123456789012123456789012345678901
2
12345678901234567890123456789012123456789012345678901
would
be
new.
In
looking
down
the
list
of
answer
choices,
only
2
12345678901234567890123456789012123456789012345678901
2345678901234567890123456789012123456789012345678901
12345678901234567890123456789012123456789012345678901
(E), novel, fits our pre-guess. In this context, novel is not a noun, 2
2
12345678901234567890123456789012123456789012345678901
2
12345678901234567890123456789012123456789012345678901
which
you
can
tell
since
all
choices
and
the
stem
word
are
12345678901234567890123456789012123456789012345678901 22
12345678901234567890123456789012123456789012345678901
adjectives. The adjective novel means new, original, or unique.
2
12345678901234567890123456789012123456789012345678901
2
1
2
12345678901234567890123456789012123456789012345678901
5.
E
To
assuage
is
to
pacify
or
to
satiate.
Choices
(B)
and
(D)
are
2
12345678901234567890123456789012123456789012345678901
12345678901234567890123456789012123456789012345678901
synonyms, not antonyms. The opposite of assuaging an issue is 2
2345678901234567890123456789012123456789012345678901
12345678901234567890123456789012123456789012345678901 22
12345678901234567890123456789012123456789012345678901
to provoke it, answer choice (E). Note that even if you didnt
2
12345678901234567890123456789012123456789012345678901
12345678901234567890123456789012123456789012345678901
know what the stem word meant, if you had two answer choices 2
12345678901234567890123456789012123456789012345678901 22
12345678901234567890123456789012123456789012345678901
that mean almost the same thing, (B) and (D), its a good guess
2
12345678901234567890123456789012123456789012345678901
that neither of them will be right since theres no way they could 2
12345678901234567890123456789012123456789012345678901
2
12345678901234567890123456789012123456789012345678901
both be right.
2
1
2345678901234567890123456789012123456789012345678901
12345678901234567890123456789012123456789012345678901 22
12345678901234567890123456789012123456789012345678901
1 6. A If you did not know the definition of mercurial, you might have 2
12345678901234567890123456789012123456789012345678901
recalled that mercury was once used in thermometers to gauge 2
2
12345678901234567890123456789012123456789012345678901
2345678901234567890123456789012123456789012345678901
12345678901234567890123456789012123456789012345678901
temperatures. The mercury goes up and down in a thermometer, 2
2
12345678901234567890123456789012123456789012345678901
2
1
and
in
similar
fashion,
mercurial
means
to
have
an
unstable
or
2
12345678901234567890123456789012123456789012345678901
2
12345678901234567890123456789012123456789012345678901
fast-changing
mood.
The
opposite
of
this
is
(A).
2345678901234567890123456789012123456789012345678901
12345678901234567890123456789012123456789012345678901 22
12345678901234567890123456789012123456789012345678901 2
1 7. C Opprobrium means severe scorn, so you are looking for a word
12345678901234567890123456789012123456789012345678901 2
12345678901234567890123456789012123456789012345678901
that means something like approval or praise. Choice (C), 2
2
12345678901234567890123456789012123456789012345678901
2
12345678901234567890123456789012123456789012345678901
commendation, fits the bill.
2
12345678901234567890123456789012123456789012345678901
2
12345678901234567890123456789012123456789012345678901
8.
C
This
is
a
tough
word.
Tyro
means
beginner
or
novice.
Thus
a
12345678901234567890123456789012123456789012345678901 2
2345678901234567890123456789012123456789012345678901
2
12345678901234567890123456789012123456789012345678901
good pre-guess would be experienced. None of the words fit 2
12345678901234567890123456789012123456789012345678901
2
12345678901234567890123456789012123456789012345678901
with our pre-guess except (C), virtuoso. A virtuoso is a master of 2
12345678901234567890123456789012123456789012345678901
2
1
a medium or instrument.
2
12345678901234567890123456789012123456789012345678901
2
12345678901234567890123456789012123456789012345678901
2
12345678901234567890123456789012123456789012345678901
9.
A
Something
that
is
soporific
is
sleep
inducing.
The
opposite
of
2
12345678901234567890123456789012123456789012345678901
2
12345678901234567890123456789012123456789012345678901
this
then
would
be
something
that
would
wake
you
up.
2345678901234567890123456789012123456789012345678901
12345678901234567890123456789012123456789012345678901 22
1
Stimulant, choice (A), fits.
2
12345678901234567890123456789012123456789012345678901
12345678901234567890123456789012123456789012345678901 2
2
12345678901234567890123456789012123456789012345678901
12345678901234567890123456789012123456789012345678901 2
12345678901234567890123456789012123456789012345678901 2
12345678901234567890123456789012123456789012345678901 2
12345678901234567890123456789012123456789012345678901 2 277
123456789012345678901234567890121234567890123456789012

http://www.xtremepapers.net

Part IV: Three Practice GREs

123456789012345678901234567890121234567890123456789012
12345678901234567890123456789012123456789012345678901 2
2
12345678901234567890123456789012123456789012345678901
Analogies
2
12345678901234567890123456789012123456789012345678901
2
12345678901234567890123456789012123456789012345678901
2
12345678901234567890123456789012123456789012345678901
10.
A
The
stem
relationship
is
definitional:
To
prattle
is
to
babble.
To
12345678901234567890123456789012123456789012345678901 2
12345678901234567890123456789012123456789012345678901
battle is to fight, so (A) fits. Do any of the others? No. To digest 2
2
12345678901234567890123456789012123456789012345678901
2
12345678901234567890123456789012123456789012345678901
is
to
eat
does
not
work
because
after
you
eat
something,
you
2
12345678901234567890123456789012123456789012345678901
2
12345678901234567890123456789012123456789012345678901
then
digest
it.
These
events
come
in
sequence,
but
they
dont
2
12345678901234567890123456789012123456789012345678901
2345678901234567890123456789012123456789012345678901
2
1
mean the same thing.
2
12345678901234567890123456789012123456789012345678901
2
12345678901234567890123456789012123456789012345678901
2
12345678901234567890123456789012123456789012345678901
11.
C
Here
the
relationship
between
the
stem
words
is
one
of
degree.
To
2
12345678901234567890123456789012123456789012345678901
2345678901234567890123456789012123456789012345678901
1
fawn is to flatter excessively. In similar fashion, to burgeon is to 2
2
12345678901234567890123456789012123456789012345678901
12345678901234567890123456789012123456789012345678901
grow quickly. Choice (B) does not share the same relationship of 2
2
12345678901234567890123456789012123456789012345678901
12345678901234567890123456789012123456789012345678901
degree because it is a relationship of definition (to divulge some- 2
2345678901234567890123456789012123456789012345678901
2
1
12345678901234567890123456789012123456789012345678901
thing is to tell it). Choice (E) might have also seemed attractive, 2
2
12345678901234567890123456789012123456789012345678901
2
but remember to misquote is not the same thing as lying.
12345678901234567890123456789012123456789012345678901
2
12345678901234567890123456789012123456789012345678901
2345678901234567890123456789012123456789012345678901
2
12345678901234567890123456789012123456789012345678901
12. B The denouement is the end/falling action of the story. Which 2
12345678901234567890123456789012123456789012345678901
12345678901234567890123456789012123456789012345678901
answer pair has the same relationship? Choice (B), a sunset, is 2
2
12345678901234567890123456789012123456789012345678901
2
12345678901234567890123456789012123456789012345678901
the
end
of
a
day.
If
you
didnt
know
the
meaning
of
12345678901234567890123456789012123456789012345678901 2
1
denouement, you should be very wary of answer choices (A) and 2
2
12345678901234567890123456789012123456789012345678901
2
12345678901234567890123456789012123456789012345678901
(C),
which
have
second
words
meant
to
catch
people
shopping
2
12345678901234567890123456789012123456789012345678901
2345678901234567890123456789012123456789012345678901
2
12345678901234567890123456789012123456789012345678901
for connections.
2
12345678901234567890123456789012123456789012345678901
12345678901234567890123456789012123456789012345678901 2
12345678901234567890123456789012123456789012345678901
13. D Federalism is a form/kind of political order. Going through the 2
2
12345678901234567890123456789012123456789012345678901
12345678901234567890123456789012123456789012345678901
answer choices, a class is composed of students, a dictatorship is 2
2
12345678901234567890123456789012123456789012345678901
12345678901234567890123456789012123456789012345678901
not a kind of anarchy, a colleague might be in a cohort, 2
12345678901234567890123456789012123456789012345678901 2
1
insomnia is a kind of sleep disorder, and something extrinsic is 2
2
12345678901234567890123456789012123456789012345678901
external to a thing. Clearly, (D) is the only pair that shares the 2
12345678901234567890123456789012123456789012345678901
2
12345678901234567890123456789012123456789012345678901
same relationship.
2
12345678901234567890123456789012123456789012345678901
2
12345678901234567890123456789012123456789012345678901
2345678901234567890123456789012123456789012345678901
12345678901234567890123456789012123456789012345678901
14. D What is the stem relationship? To be spurious is to not be true. 2
2
12345678901234567890123456789012123456789012345678901
2
1
Thus,
we
have
a
relationship
of
opposites.
The
only
answer
2
12345678901234567890123456789012123456789012345678901
2
12345678901234567890123456789012123456789012345678901
choice
that
shares
this
relationship
is
(D),
since
to
be
insipid
is
to
2345678901234567890123456789012123456789012345678901
12345678901234567890123456789012123456789012345678901 2
2
12345678901234567890123456789012123456789012345678901
not be flavorful.
2
1
12345678901234567890123456789012123456789012345678901 2
12345678901234567890123456789012123456789012345678901
15. D Remember, always begin with the stem relationship. It is 2
2
12345678901234567890123456789012123456789012345678901
2
12345678901234567890123456789012123456789012345678901
If
you
did
not
know
definitional,
as
something
ersatz
is
artificial.
2
12345678901234567890123456789012123456789012345678901
12345678901234567890123456789012123456789012345678901
the definition of ersatz, you could have eliminated (A) and (C) 2
12345678901234567890123456789012123456789012345678901 2
12345678901234567890123456789012123456789012345678901
since neither has a necessary relationship. Of choices (B), (D), 2
2
12345678901234567890123456789012123456789012345678901
12345678901234567890123456789012123456789012345678901
and (E), only (D)to be nascent is to be incipienthas the 2
2
12345678901234567890123456789012123456789012345678901
2
definitional relationship.
12345678901234567890123456789012123456789012345678901
2345678901234567890123456789012123456789012345678901
12345678901234567890123456789012123456789012345678901 2
2
12345678901234567890123456789012123456789012345678901
16. E A diatribe is harsh verbal attack, and a eulogy is a speech of 2
12345678901234567890123456789012123456789012345678901
12345678901234567890123456789012123456789012345678901
praise or remembrance. Thus, the words are antonyms. The 2
2
1
2345678901234567890123456789012123456789012345678901
12345678901234567890123456789012123456789012345678901
stem relationship is a diatribe is the opposite of a eulogy. To 2
2
1
2
12345678901234567890123456789012123456789012345678901
remonstrate
is
to
reprove,
or
stand
in
opposition
to,
while
an
12345678901234567890123456789012123456789012345678901 2
12345678901234567890123456789012123456789012345678901
encomium is warm praise. Thus a remonstration is the opposite 2
2
12345678901234567890123456789012123456789012345678901
2
12345678901234567890123456789012123456789012345678901
of
an
encomium.
12345678901234567890123456789012123456789012345678901 2
2
278 12345678901234567890123456789012123456789012345678901
123456789012345678901234567890121234567890123456789012

www.petersons.com

http://www.xtremepapers.net

ANSWERS
123456789012345678901234567890121234567890123456789012
12345678901234567890123456789012123456789012345678901 2
2
12345678901234567890123456789012123456789012345678901
17. A The stem relationship is one of definition, a hagiography 2
12345678901234567890123456789012123456789012345678901
2
12345678901234567890123456789012123456789012345678901
venerates its subject. In (A), a polemic attacks its subject. A 2
12345678901234567890123456789012123456789012345678901
2
12345678901234567890123456789012123456789012345678901
vendetta may or may not scare its object. The same is true for a 2
12345678901234567890123456789012123456789012345678901
2345678901234567890123456789012123456789012345678901
2
1
complaint. The other choices do not have a strong relationship. 2
12345678901234567890123456789012123456789012345678901
12345678901234567890123456789012123456789012345678901
Choice (A) is the answer. If you have trouble with the 2
2
12345678901234567890123456789012123456789012345678901
2
12345678901234567890123456789012123456789012345678901
vocabulary,
you
could
look
at
(C)
and
(D)
and
eliminate
these
2345678901234567890123456789012123456789012345678901
2
1
2
12345678901234567890123456789012123456789012345678901
choices
for
having
weak
links.
2
12345678901234567890123456789012123456789012345678901
2
12345678901234567890123456789012123456789012345678901
2
12345678901234567890123456789012123456789012345678901
2345678901234567890123456789012123456789012345678901
2
1
2
12345678901234567890123456789012123456789012345678901
Sentence Completions
2
12345678901234567890123456789012123456789012345678901
2
12345678901234567890123456789012123456789012345678901
18.
B
On
two
blank
questions,
its
easiest
to
consider
one
blank
at
a
2
12345678901234567890123456789012123456789012345678901
2345678901234567890123456789012123456789012345678901
2
1
time. Attacking the second blank first makes most sense since 2
12345678901234567890123456789012123456789012345678901
12345678901234567890123456789012123456789012345678901
you know that it is in contrast to minor points (the trigger word 2
2
12345678901234567890123456789012123456789012345678901
2
12345678901234567890123456789012123456789012345678901
Although
should
tip
your
hand).
What
is
a
contrast
to
a
minor
2345678901234567890123456789012123456789012345678901
12345678901234567890123456789012123456789012345678901 22
12345678901234567890123456789012123456789012345678901
point? A major one. You should go look for something like 2
12345678901234567890123456789012123456789012345678901
12345678901234567890123456789012123456789012345678901
major or important. That eliminates (A) and (C). Into the first 2
2
12345678901234567890123456789012123456789012345678901
12345678901234567890123456789012123456789012345678901
blank, debate clearly fits best of the remaining choices, as a 2
2
1
2
12345678901234567890123456789012123456789012345678901
debate
is
more
likely
to
have
themes
than
an
agreement
or
2
12345678901234567890123456789012123456789012345678901
2
12345678901234567890123456789012123456789012345678901
discovery.
So,
(B)
is
the
answer.
2345678901234567890123456789012123456789012345678901
12345678901234567890123456789012123456789012345678901 22
12345678901234567890123456789012123456789012345678901 2
12345678901234567890123456789012123456789012345678901
19. D The blank is a word that goes along with created excitement, and
2
12345678901234567890123456789012123456789012345678901
12345678901234567890123456789012123456789012345678901
so it is definitely positive. What is a positive verb that flows with 2
12345678901234567890123456789012123456789012345678901 22
12345678901234567890123456789012123456789012345678901
greater interest? Created is a good pre-guess. Check the answer
2
12345678901234567890123456789012123456789012345678901
choices, and (D), engendered, fits your pre-guess quite well.
2
12345678901234567890123456789012123456789012345678901
2
1
2345678901234567890123456789012123456789012345678901
12345678901234567890123456789012123456789012345678901
20. A The although at the beginning of the sentence clues you into 2
2
12345678901234567890123456789012123456789012345678901
2
1
the
fact
that
this
is
a
contrast
sentence.
The
good
times
spoken
of
12345678901234567890123456789012123456789012345678901 2
12345678901234567890123456789012123456789012345678901
in the present are contrasted with what is in the blank. What is a 2
2
12345678901234567890123456789012123456789012345678901
2
12345678901234567890123456789012123456789012345678901
contrast
with
good
times?
Tough
times.
Choice
(A)
works
well.
12345678901234567890123456789012123456789012345678901 2
2
12345678901234567890123456789012123456789012345678901
2
12345678901234567890123456789012123456789012345678901
21.
E
It
should
be
easy
to
eliminate
some
answer
choices
with
the
first
2345678901234567890123456789012123456789012345678901
12345678901234567890123456789012123456789012345678901 22
12345678901234567890123456789012123456789012345678901
blank since it has to fit with crude. Choice (C) certainly does not 2
1
12345678901234567890123456789012123456789012345678901
fit and possibly (A) also. The second blank is a positive word 2
2
12345678901234567890123456789012123456789012345678901
2345678901234567890123456789012123456789012345678901
12345678901234567890123456789012123456789012345678901
since the logic of the sentence is that though the early theories 2
2
12345678901234567890123456789012123456789012345678901
12345678901234567890123456789012123456789012345678901
were crude, they were still helpful. Thus we can eliminate (A) 2
12345678901234567890123456789012123456789012345678901 22
1
and (D). Choice (E) works best.
2
12345678901234567890123456789012123456789012345678901
2
12345678901234567890123456789012123456789012345678901
22. D Here the first blank seems more assailable. You know that it has 2
12345678901234567890123456789012123456789012345678901
12345678901234567890123456789012123456789012345678901
to pair with factually misleading. Choices (B) and (E) do not, so 2
2
12345678901234567890123456789012123456789012345678901
2345678901234567890123456789012123456789012345678901
12345678901234567890123456789012123456789012345678901
eliminate them. As for the second blank, you know two things: 2
2
12345678901234567890123456789012123456789012345678901
2
12345678901234567890123456789012123456789012345678901
It
pairs
with
superior
mastery
of
language,
and
it
masks
the
12345678901234567890123456789012123456789012345678901 22
1
problems mentioned in the first part of the sentence. Thus, it 2
12345678901234567890123456789012123456789012345678901
12345678901234567890123456789012123456789012345678901
must be something fairly positive. Choices (A) and (C) do not 2
2
12345678901234567890123456789012123456789012345678901
2
12345678901234567890123456789012123456789012345678901
meet
these
requirements,
but
(D)
does.
2
12345678901234567890123456789012123456789012345678901
2345678901234567890123456789012123456789012345678901
2
1
12345678901234567890123456789012123456789012345678901 2
12345678901234567890123456789012123456789012345678901 2
12345678901234567890123456789012123456789012345678901 2 279
123456789012345678901234567890121234567890123456789012

http://www.xtremepapers.net

Part IV: Three Practice GREs

123456789012345678901234567890121234567890123456789012
12345678901234567890123456789012123456789012345678901 2
2
12345678901234567890123456789012123456789012345678901
23. D A pre-guess on the second blank would be a good place to start. 2
12345678901234567890123456789012123456789012345678901
2
12345678901234567890123456789012123456789012345678901
Something like understand works. Checking the answer choices, 2
12345678901234567890123456789012123456789012345678901
2
12345678901234567890123456789012123456789012345678901
we can eliminate (E). The first blank goes along with the 2
12345678901234567890123456789012123456789012345678901
2345678901234567890123456789012123456789012345678901
2
1
professor being visibly frustrated. Choice (B) is out because it 2
12345678901234567890123456789012123456789012345678901
12345678901234567890123456789012123456789012345678901
does not fit the context. Choices (A) and (C) are okay choices, 2
2
12345678901234567890123456789012123456789012345678901
2
12345678901234567890123456789012123456789012345678901
but
(D)
definitely
works
best
in
the
first
blank.
2345678901234567890123456789012123456789012345678901
2
1
2
12345678901234567890123456789012123456789012345678901
2
12345678901234567890123456789012123456789012345678901
2
12345678901234567890123456789012123456789012345678901
2
12345678901234567890123456789012123456789012345678901
Reading Comprehension
2345678901234567890123456789012123456789012345678901
2
1
2
12345678901234567890123456789012123456789012345678901
24. C The authors tone toward the advancement of our understand- 2
12345678901234567890123456789012123456789012345678901
2
12345678901234567890123456789012123456789012345678901
ing of dark matter is most clearly seen in the last sentence of the 2
12345678901234567890123456789012123456789012345678901
2345678901234567890123456789012123456789012345678901
2
1
passage. There, the author looks forward to a time when more 2
12345678901234567890123456789012123456789012345678901
12345678901234567890123456789012123456789012345678901
fecund methods of investigating dark matter are developed. 2
2
12345678901234567890123456789012123456789012345678901
2
12345678901234567890123456789012123456789012345678901
That
is
an
expression
of
hope,
but
not
unqualifiedly
so
(just
read
2345678901234567890123456789012123456789012345678901
12345678901234567890123456789012123456789012345678901 2
12345678901234567890123456789012123456789012345678901
the first part of that sentence to see that). Thus, (C) is the best 2
2
12345678901234567890123456789012123456789012345678901
2
12345678901234567890123456789012123456789012345678901
answer.
12345678901234567890123456789012123456789012345678901 2
12345678901234567890123456789012123456789012345678901 2
1 25. D Here, you just have to go through each proposition and see 2
2
12345678901234567890123456789012123456789012345678901
2
12345678901234567890123456789012123456789012345678901
which
ones
the
passage
relates
to.
2
12345678901234567890123456789012123456789012345678901
2345678901234567890123456789012123456789012345678901
12345678901234567890123456789012123456789012345678901 2
12345678901234567890123456789012123456789012345678901
Option Ithe passage makes clear that this is true. Check 2
2
12345678901234567890123456789012123456789012345678901
the third and fourth paragraph for verification of this.
2
12345678901234567890123456789012123456789012345678901
12345678901234567890123456789012123456789012345678901 2
2
12345678901234567890123456789012123456789012345678901
Option IIthe passage states the exact opposite of this.
2
12345678901234567890123456789012123456789012345678901
12345678901234567890123456789012123456789012345678901 2
12345678901234567890123456789012123456789012345678901
Option IIIlines 3133, Like light matter, dark matter 2
2
1
2345678901234567890123456789012123456789012345678901
12345678901234567890123456789012123456789012345678901
gravitationally attracts all matter. III then is true. Choice 2
2
12345678901234567890123456789012123456789012345678901
2
1
(D)
then
is
correct.
12345678901234567890123456789012123456789012345678901 2
2
12345678901234567890123456789012123456789012345678901
12345678901234567890123456789012123456789012345678901
26. A To start, reread the entire third paragraph. You have to go 2
2
12345678901234567890123456789012123456789012345678901
12345678901234567890123456789012123456789012345678901
through the answer choices to see which is a correct inference 2
2
12345678901234567890123456789012123456789012345678901
12345678901234567890123456789012123456789012345678901
from the third paragraph. For choice (A), the third paragraph 2
2345678901234567890123456789012123456789012345678901
2
12345678901234567890123456789012123456789012345678901
reads, Like light matter, dark matter gravitationally attracts all 2
12345678901234567890123456789012123456789012345678901
2
1
matter and so sustains a gravitational field. If dark matter 2
12345678901234567890123456789012123456789012345678901
2
12345678901234567890123456789012123456789012345678901
sustains a gravitational field and dark matter is like light matter 2
12345678901234567890123456789012123456789012345678901
2
12345678901234567890123456789012123456789012345678901
in this respect, then it can be inferred that light matter sustains a 2
12345678901234567890123456789012123456789012345678901
12345678901234567890123456789012123456789012345678901
gravitational field. For choice (B), the third paragraph does not 2
2
12345678901234567890123456789012123456789012345678901
2345678901234567890123456789012123456789012345678901
12345678901234567890123456789012123456789012345678901
mention anything about the comparative stability of light and 2
2
12345678901234567890123456789012123456789012345678901
2
12345678901234567890123456789012123456789012345678901
dark
matter.
Choice
(C)
has
it
backward,
as
astronomers
infer
12345678901234567890123456789012123456789012345678901 2
1
the existence of dark matter from insufficient light matter. 2
2
12345678901234567890123456789012123456789012345678901
2
12345678901234567890123456789012123456789012345678901
Choice
(D)
is
true,
but
is
stated
in
the
first
paragraph,
not
the
2
12345678901234567890123456789012123456789012345678901
2
12345678901234567890123456789012123456789012345678901
third.
For
(E),
the
third
paragraph
does
not
discuss
theoretical
12345678901234567890123456789012123456789012345678901 2
2345678901234567890123456789012123456789012345678901
12345678901234567890123456789012123456789012345678901
hypotheses about dark matter. Choice (A) is the correct answer. 2
2
1
2
12345678901234567890123456789012123456789012345678901
12345678901234567890123456789012123456789012345678901
27. C On a question like this, you have to go through the answer 2
2
12345678901234567890123456789012123456789012345678901
12345678901234567890123456789012123456789012345678901
choices to see which is best. The title, of course, should reflect 2
2
12345678901234567890123456789012123456789012345678901
12345678901234567890123456789012123456789012345678901
the tone and content of the passage, so you are looking for the 2
2
280 12345678901234567890123456789012123456789012345678901
123456789012345678901234567890121234567890123456789012

www.petersons.com

http://www.xtremepapers.net

ANSWERS
123456789012345678901234567890121234567890123456789012
12345678901234567890123456789012123456789012345678901 2
2
12345678901234567890123456789012123456789012345678901
title that best does this. Choice (A) could be the title of the 2
12345678901234567890123456789012123456789012345678901
2
12345678901234567890123456789012123456789012345678901
second paragraph, but not of the entire passage. Choice (B) 2
12345678901234567890123456789012123456789012345678901
2
12345678901234567890123456789012123456789012345678901
sounds appealing but there is no indication in the passage that 2
12345678901234567890123456789012123456789012345678901
2345678901234567890123456789012123456789012345678901
2
1
the issues discussed are just ones for the present time. Choice (C) 2
12345678901234567890123456789012123456789012345678901
12345678901234567890123456789012123456789012345678901
captures the tone and content of the passage and so is a likely 2
2
12345678901234567890123456789012123456789012345678901
2
12345678901234567890123456789012123456789012345678901
candidate.
Choice
(D)
is
also
appealing
except
for
the
phrase
2345678901234567890123456789012123456789012345678901
2
1
2
12345678901234567890123456789012123456789012345678901
Intellectual
Historiography,
which
does
not
appear
in
the
2
12345678901234567890123456789012123456789012345678901
2
12345678901234567890123456789012123456789012345678901
passage.
Choice
(E)
is
much
too
extreme
in
its
tone.
Choice
(C)
2
12345678901234567890123456789012123456789012345678901
2345678901234567890123456789012123456789012345678901
2
1
is the best choice.
2
12345678901234567890123456789012123456789012345678901
2
12345678901234567890123456789012123456789012345678901
2
12345678901234567890123456789012123456789012345678901
28.
C
Reread
the
second
paragraph
(or
at
least
skim
it)
and
consider
2
12345678901234567890123456789012123456789012345678901
2345678901234567890123456789012123456789012345678901
1
its relation to the rest of the passage. Then, proceed to the 2
2
12345678901234567890123456789012123456789012345678901
12345678901234567890123456789012123456789012345678901
question. Notice that the second paragraph contains two 2
2
12345678901234567890123456789012123456789012345678901
12345678901234567890123456789012123456789012345678901
contrasting examples. The answer choice should reflect this. The 2
2345678901234567890123456789012123456789012345678901
12345678901234567890123456789012123456789012345678901 22
12345678901234567890123456789012123456789012345678901
basic thesis of the passage is that there are multiple narratives in
2
12345678901234567890123456789012123456789012345678901
historiography of the same events. The content of the second 2
12345678901234567890123456789012123456789012345678901
2
12345678901234567890123456789012123456789012345678901
paragraph illustrates this thesis. Thus, (C) is correct.
2
12345678901234567890123456789012123456789012345678901
2
1
2
12345678901234567890123456789012123456789012345678901
29.
C
Go
and
read
the
relevant
text
and
be
careful
to
have
in
mind
2
12345678901234567890123456789012123456789012345678901
2
12345678901234567890123456789012123456789012345678901
what
Burkes
view
is,
which
is
that
the
Revolution
produced
a
2345678901234567890123456789012123456789012345678901
12345678901234567890123456789012123456789012345678901 22
12345678901234567890123456789012123456789012345678901
result that was a radical departure from the past. On this view 2
12345678901234567890123456789012123456789012345678901
12345678901234567890123456789012123456789012345678901
the phrase historical novelty clearly implies the newness and 2
2
12345678901234567890123456789012123456789012345678901
12345678901234567890123456789012123456789012345678901
uniqueness of the French Republic. Choice (C) fits this well. 2
2
12345678901234567890123456789012123456789012345678901
2
12345678901234567890123456789012123456789012345678901
Choice
(E)
might
have
seemed
attractive
but
recall
that
12345678901234567890123456789012123456789012345678901 22
1
institutional continuity was never mentioned in the passage. 2
12345678901234567890123456789012123456789012345678901
12345678901234567890123456789012123456789012345678901
That should make you wary of that choice. Choice (D) is the 2
2
12345678901234567890123456789012123456789012345678901
2
12345678901234567890123456789012123456789012345678901
view of de Tocqueville, not Burke. So, (C) is the best choice.
2
12345678901234567890123456789012123456789012345678901
2345678901234567890123456789012123456789012345678901
2
12345678901234567890123456789012123456789012345678901
30. B This question relates to the main idea and tone of the passage. 2
12345678901234567890123456789012123456789012345678901
2
1
The correct answer will reflect both.
2
12345678901234567890123456789012123456789012345678901
2
12345678901234567890123456789012123456789012345678901
2345678901234567890123456789012123456789012345678901
12345678901234567890123456789012123456789012345678901
Main idea: Historiography can be influenced by ideology 2
2
12345678901234567890123456789012123456789012345678901
2
1
and
so
intellectually
honest
historiography
should
be
12345678901234567890123456789012123456789012345678901 2
2
12345678901234567890123456789012123456789012345678901
mindful of this.
2
12345678901234567890123456789012123456789012345678901
2
12345678901234567890123456789012123456789012345678901
2
12345678901234567890123456789012123456789012345678901
Tone:
Guarded
optimism
that
with
care
historiography
can
2
12345678901234567890123456789012123456789012345678901
2
12345678901234567890123456789012123456789012345678901
be
intellectually
honest
2345678901234567890123456789012123456789012345678901
12345678901234567890123456789012123456789012345678901 22
12345678901234567890123456789012123456789012345678901 2
12345678901234567890123456789012123456789012345678901
Choice (A) is too extreme. Choice (B) fits well with both. As for
2
12345678901234567890123456789012123456789012345678901
1
(C), though the passage states, historiography purports to be 2
2345678901234567890123456789012123456789012345678901
2
12345678901234567890123456789012123456789012345678901
an empirical endeavor free of . . . biases the passage neither 2
12345678901234567890123456789012123456789012345678901
2
12345678901234567890123456789012123456789012345678901
defines what it is to be a science nor states that historiography is 2
12345678901234567890123456789012123456789012345678901
2
1
not one. Choice (D) is cynical, and the tone of the passage is not 2
12345678901234567890123456789012123456789012345678901
2
12345678901234567890123456789012123456789012345678901
(just read the last sentence of the passage). Choice (E) is also 2
12345678901234567890123456789012123456789012345678901
2
12345678901234567890123456789012123456789012345678901
cynical, more so than the passage is. So, (B) is the best choice.
2
12345678901234567890123456789012123456789012345678901
2345678901234567890123456789012123456789012345678901
2
1
12345678901234567890123456789012123456789012345678901 2
12345678901234567890123456789012123456789012345678901 2
12345678901234567890123456789012123456789012345678901 2 281
123456789012345678901234567890121234567890123456789012

http://www.xtremepapers.net

Practice Test

2
1234567890123456789012345678901212345678901234567890123456789012123456
3456789012345678901234567890121234567890123456789012345678901212345 6
12
3456789012345678901234567890121234567890123456789012345678901212345 6
12
3456789012345678901234567890121234567890123456789012345678901212345
6
12
Quantitative
2
6
123456789012345678901234567890121234567890123456789012345678901212345
6
123456789012345678901234567890121234567890123456789012345678901212345
6
123456789012345678901234567890121234567890123456789012345678901212345
28
Questions45
Minutes
6
123456789012345678901234567890121234567890123456789012345678901212345
6
123456789012345678901234567890121234567890123456789012345678901212345
General
Information
3456789012345678901234567890121234567890123456789012345678901212345
123456789012345678901234567890121234567890123456789012345678901212345 66
1
3456789012345678901234567890121234567890123456789012345678901212345
12
1. The test has 28 questions. If you decide to take this exam under timed conditions, you have 6
3456789012345678901234567890121234567890123456789012345678901212345
6
12
3456789012345678901234567890121234567890123456789012345678901212345
6
12
45 minutes to complete the section.
2
3456789012345678901234567890121234567890123456789012345678901212345
123456789012345678901234567890121234567890123456789012345678901212345 66
123456789012345678901234567890121234567890123456789012345678901212345
2. All numbers used are real numbers.
6
123456789012345678901234567890121234567890123456789012345678901212345
123456789012345678901234567890121234567890123456789012345678901212345 66
123456789012345678901234567890121234567890123456789012345678901212345
3. All angle measurements can be assumed to be positive unless otherwise noted.
6
123456789012345678901234567890121234567890123456789012345678901212345
123456789012345678901234567890121234567890123456789012345678901212345 66
123456789012345678901234567890121234567890123456789012345678901212345
4. All figures lie in the same plane unless otherwise noted.
6
123456789012345678901234567890121234567890123456789012345678901212345
6
1
2
3456789012345678901234567890121234567890123456789012345678901212345
123456789012345678901234567890121234567890123456789012345678901212345
5. Drawings that accompany questions are intended to provide information useful in 6
6
123456789012345678901234567890121234567890123456789012345678901212345
6
1
answering the question. The figures are drawn closely to scale unless otherwise noted.
123456789012345678901234567890121234567890123456789012345678901212345 6
3456789012345678901234567890121234567890123456789012345678901212345 6
12
3456789012345678901234567890121234567890123456789012345678901212345 6
12
3456789012345678901234567890121234567890123456789012345678901212345
6
12
Directions: For questions 1628, Column A and Column B will have one quantity each.
6
123456789012345678901234567890121234567890123456789012345678901212345
3456789012345678901234567890121234567890123456789012345678901212345
6
12
Your goal is to compare the quantities in each column and choose
3456789012345678901234567890121234567890123456789012345678901212345 6
12
2
3456789012345678901234567890121234567890123456789012345678901212345
6
123456789012345678901234567890121234567890123456789012345678901212345
A if the quantity in Column A is greater
6
123456789012345678901234567890121234567890123456789012345678901212345
6
1
B if the quantity in Column B is greater
6
123456789012345678901234567890121234567890123456789012345678901212345
6
123456789012345678901234567890121234567890123456789012345678901212345
C
if
the
two
quantities
are
equal
2
3456789012345678901234567890121234567890123456789012345678901212345
123456789012345678901234567890121234567890123456789012345678901212345 66
123456789012345678901234567890121234567890123456789012345678901212345
D if the relationship between the two quantities cannot be determined from the
6
123456789012345678901234567890121234567890123456789012345678901212345
6
123456789012345678901234567890121234567890123456789012345678901212345
information
in
the
problem
6
1
2
3456789012345678901234567890121234567890123456789012345678901212345
123456789012345678901234567890121234567890123456789012345678901212345 66
123456789012345678901234567890121234567890123456789012345678901212345
On some questions, information about one or both quantities will be given. This informa6
123456789012345678901234567890121234567890123456789012345678901212345
6
123456789012345678901234567890121234567890123456789012345678901212345
tion
will
be
centered
above
the
two
columns.
6
1
2
3456789012345678901234567890121234567890123456789012345678901212345
123456789012345678901234567890121234567890123456789012345678901212345 66
123456789012345678901234567890121234567890123456789012345678901212345 6
123456789012345678901234567890121234567890123456789012345678901212345 6
123456789012345678901234567890121234567890123456789012345678901212345 6
1
3456789012345678901234567890121234567890123456789012345678901212345 6
12
123456789012345678901234567890121234567890123456789012345678901212345 6
3456789012345678901234567890121234567890123456789012345678901212345 6
12
123456789012345678901234567890121234567890123456789012345678901212345 6
3456789012345678901234567890121234567890123456789012345678901212345 6
12
123456789012345678901234567890121234567890123456789012345678901212345 6
123456789012345678901234567890121234567890123456789012345678901212345 6
123456789012345678901234567890121234567890123456789012345678901212345 6
123456789012345678901234567890121234567890123456789012345678901212345 6
1234567890123456789012345678901212345678901234567890123456789012123456
282

http://www.xtremepapers.net

TEST 2
1234567890123456789012345678901212345678901234567890123456789012123456
123456789012345678901234567890121234567890123456789012345678901212345 6
3456789012345678901234567890121234567890123456789012345678901212345 6
12
1.
4. If 3c 1 6d 5 24 and 4a 13d 5 18, what
6
123456789012345678901234567890121234567890123456789012345678901212345
3456789012345678901234567890121234567890123456789012345678901212345
6
12
does a equal in terms of c?
3456789012345678901234567890121234567890123456789012345678901212345
6
12
123456789012345678901234567890121234567890123456789012345678901212345 6
3456789012345678901234567890121234567890123456789012345678901212345
6
12
9 3
6
123456789012345678901234567890121234567890123456789012345678901212345
2 c
A.
3456789012345678901234567890121234567890123456789012345678901212345
6
12
2 4
3456789012345678901234567890121234567890123456789012345678901212345
6
12
3456789012345678901234567890121234567890123456789012345678901212345 6
12
3456789012345678901234567890121234567890123456789012345678901212345
6
12
3
3
6
123456789012345678901234567890121234567890123456789012345678901212345
c1
B.
3456789012345678901234567890121234567890123456789012345678901212345
6
12
BCDE is a parallelogram. What is its
8
2
3456789012345678901234567890121234567890123456789012345678901212345
6
12
area?
3456789012345678901234567890121234567890123456789012345678901212345
6
12
3
12
3456789012345678901234567890121234567890123456789012345678901212345
6
12
c
1
C.
2
3456789012345678901234567890121234567890123456789012345678901212345
6
1
11
11
A. =10
3456789012345678901234567890121234567890123456789012345678901212345
6
12
3456789012345678901234567890121234567890123456789012345678901212345
6
12
1
3456789012345678901234567890121234567890123456789012345678901212345
6
12
B. 10
3456789012345678901234567890121234567890123456789012345678901212345
6
12
D. c 1 4
2
2
3456789012345678901234567890121234567890123456789012345678901212345
6
1
C. 10=2
3456789012345678901234567890121234567890123456789012345678901212345
6
12
3456789012345678901234567890121234567890123456789012345678901212345
6
12
3
D.
10=5
3456789012345678901234567890121234567890123456789012345678901212345
6
12
c
1
5
E.
3456789012345678901234567890121234567890123456789012345678901212345
6
12
4
E.
2
3456789012345678901234567890121234567890123456789012345678901212345
6
123456789012345678901234567890121234567890123456789012345678901212345
10=10
6
123456789012345678901234567890121234567890123456789012345678901212345
123456789012345678901234567890121234567890123456789012345678901212345 66
123456789012345678901234567890121234567890123456789012345678901212345
Questions 56 refer to the following graph.
2. If &x 5 [x 2 (x 2 1)](x 1 2) then
6
123456789012345678901234567890121234567890123456789012345678901212345
6
123456789012345678901234567890121234567890123456789012345678901212345
&(&(2)) 5
6
1
3456789012345678901234567890121234567890123456789012345678901212345 6
12
3456789012345678901234567890121234567890123456789012345678901212345
6
12
A. 4
3456789012345678901234567890121234567890123456789012345678901212345
6
12
2
3456789012345678901234567890121234567890123456789012345678901212345
6
123456789012345678901234567890121234567890123456789012345678901212345
B. 6
6
123456789012345678901234567890121234567890123456789012345678901212345
6
123456789012345678901234567890121234567890123456789012345678901212345
C.
12
123456789012345678901234567890121234567890123456789012345678901212345 66
123456789012345678901234567890121234567890123456789012345678901212345
D. 18
6
123456789012345678901234567890121234567890123456789012345678901212345
6
123456789012345678901234567890121234567890123456789012345678901212345
E. 24
123456789012345678901234567890121234567890123456789012345678901212345 66
123456789012345678901234567890121234567890123456789012345678901212345 6
1 3. The price of Company Xs stock started
3456789012345678901234567890121234567890123456789012345678901212345 6
12
3456789012345678901234567890121234567890123456789012345678901212345
6
12
the day at $40 per share, rose to $90 per
6
123456789012345678901234567890121234567890123456789012345678901212345
6
123456789012345678901234567890121234567890123456789012345678901212345
share,
and
then
fell
to
$45
per
share.
3456789012345678901234567890121234567890123456789012345678901212345 6
12
2
3456789012345678901234567890121234567890123456789012345678901212345
6
123456789012345678901234567890121234567890123456789012345678901212345
Compared to the price at the beginning of
6
123456789012345678901234567890121234567890123456789012345678901212345
6
1
the
day,
the
final
price
of
the
stock
was
a
3456789012345678901234567890121234567890123456789012345678901212345 6
12
3456789012345678901234567890121234567890123456789012345678901212345 6
12
3456789012345678901234567890121234567890123456789012345678901212345
6
12
A. 10% increase.
3456789012345678901234567890121234567890123456789012345678901212345
6
12
6
123456789012345678901234567890121234567890123456789012345678901212345
B.
10%
decrease.
6
123456789012345678901234567890121234567890123456789012345678901212345
5. How many millions of years ago did the
6
123456789012345678901234567890121234567890123456789012345678901212345
C. 12.5% increase.
2
3456789012345678901234567890121234567890123456789012345678901212345
6
123456789012345678901234567890121234567890123456789012345678901212345
extinction rate reach its highest value?
6
123456789012345678901234567890121234567890123456789012345678901212345
D. 12.5% decrease.
6
123456789012345678901234567890121234567890123456789012345678901212345
A. 200
6
123456789012345678901234567890121234567890123456789012345678901212345
E. 50% decrease.
6
1
B.
245
2
3456789012345678901234567890121234567890123456789012345678901212345
123456789012345678901234567890121234567890123456789012345678901212345 66
123456789012345678901234567890121234567890123456789012345678901212345
C. 500
6
123456789012345678901234567890121234567890123456789012345678901212345
6
123456789012345678901234567890121234567890123456789012345678901212345
D.
560
6
1
2
3456789012345678901234567890121234567890123456789012345678901212345
6
123456789012345678901234567890121234567890123456789012345678901212345
E. 600
6
123456789012345678901234567890121234567890123456789012345678901212345
123456789012345678901234567890121234567890123456789012345678901212345 66
123456789012345678901234567890121234567890123456789012345678901212345 6
1
3456789012345678901234567890121234567890123456789012345678901212345 6
12
123456789012345678901234567890121234567890123456789012345678901212345 6
3456789012345678901234567890121234567890123456789012345678901212345 6
12
123456789012345678901234567890121234567890123456789012345678901212345 6
3456789012345678901234567890121234567890123456789012345678901212345 6
12
123456789012345678901234567890121234567890123456789012345678901212345 6
123456789012345678901234567890121234567890123456789012345678901212345 6
123456789012345678901234567890121234567890123456789012345678901212345 6
123456789012345678901234567890121234567890123456789012345678901212345 6 283
1234567890123456789012345678901212345678901234567890123456789012123456

http://www.xtremepapers.net

Part IV: Three Practice GREs

1234567890123456789012345678901212345678901234567890123456789012123456
123456789012345678901234567890121234567890123456789012345678901212345 6
3456789012345678901234567890121234567890123456789012345678901212345 6
12
6. In which era did the extinction rate and
6
9.
123456789012345678901234567890121234567890123456789012345678901212345
3456789012345678901234567890121234567890123456789012345678901212345
6
12
G
number of families have the same value
3456789012345678901234567890121234567890123456789012345678901212345
6
12
6
123456789012345678901234567890121234567890123456789012345678901212345
the
most
number
of
times?
3456789012345678901234567890121234567890123456789012345678901212345 6
12
2
3456789012345678901234567890121234567890123456789012345678901212345
6
1
3456789012345678901234567890121234567890123456789012345678901212345
6
12
A.
Pre-Cambrian
3456789012345678901234567890121234567890123456789012345678901212345 6
12
3456789012345678901234567890121234567890123456789012345678901212345
6
12
B. Paleozoic
3456789012345678901234567890121234567890123456789012345678901212345
6
12
2
3456789012345678901234567890121234567890123456789012345678901212345
6
1
C. Mesozoic
3456789012345678901234567890121234567890123456789012345678901212345
6
12
F
3456789012345678901234567890121234567890123456789012345678901212345
6
12
D.
Cenozoic
3456789012345678901234567890121234567890123456789012345678901212345 6
12
3456789012345678901234567890121234567890123456789012345678901212345
6
12
E. Silurian
6
123456789012345678901234567890121234567890123456789012345678901212345
3456789012345678901234567890121234567890123456789012345678901212345
6
12
D
E
3456789012345678901234567890121234567890123456789012345678901212345 6
12
3456789012345678901234567890121234567890123456789012345678901212345
6
7. Which of the following numbers would
12
3456789012345678901234567890121234567890123456789012345678901212345
6
12
The
area
of
square
B
is
16
meters
squared
have
a
remainder
of
zero
if
it
were
divided
2
3456789012345678901234567890121234567890123456789012345678901212345
6
1
3456789012345678901234567890121234567890123456789012345678901212345
6
12
DE
5
EG
What
is
the
area
of
square
and
by 3 or 6 and 9?
3456789012345678901234567890121234567890123456789012345678901212345
6
12
3456789012345678901234567890121234567890123456789012345678901212345
6
12
C
in
meters
squared?
3456789012345678901234567890121234567890123456789012345678901212345
6
12
A.
1,024
2
3456789012345678901234567890121234567890123456789012345678901212345
123456789012345678901234567890121234567890123456789012345678901212345 66
123456789012345678901234567890121234567890123456789012345678901212345
A. 1
B. 1,323
6
123456789012345678901234567890121234567890123456789012345678901212345
6
123456789012345678901234567890121234567890123456789012345678901212345
B. 2
C.
1,506
123456789012345678901234567890121234567890123456789012345678901212345 66
123456789012345678901234567890121234567890123456789012345678901212345
C. 4
D. 1,632
6
1
3456789012345678901234567890121234567890123456789012345678901212345
6
12
D. 6
E.
1,764
3456789012345678901234567890121234567890123456789012345678901212345
6
12
E. 9
3456789012345678901234567890121234567890123456789012345678901212345
6
12
2
3456789012345678901234567890121234567890123456789012345678901212345
123456789012345678901234567890121234567890123456789012345678901212345 66
123456789012345678901234567890121234567890123456789012345678901212345
8. In the number set {15, 10, 7, 12, 10, 18} a
6
123456789012345678901234567890121234567890123456789012345678901212345
equals the mean, b equals the mode, c
6
123456789012345678901234567890121234567890123456789012345678901212345
Questions
1012
refer
to
the
following
graphs.
6
123456789012345678901234567890121234567890123456789012345678901212345
equals
the
median,
and
d
equals
the
range.
123456789012345678901234567890121234567890123456789012345678901212345 66
123456789012345678901234567890121234567890123456789012345678901212345
Which of the following statements is true?
6
123456789012345678901234567890121234567890123456789012345678901212345
123456789012345678901234567890121234567890123456789012345678901212345 66
1
A. a , b 5 c , d
3456789012345678901234567890121234567890123456789012345678901212345
6
12
3456789012345678901234567890121234567890123456789012345678901212345
6
12
B.
a
.
b
,
c
.
d
123456789012345678901234567890121234567890123456789012345678901212345 6
6
123456789012345678901234567890121234567890123456789012345678901212345
C. a 5 b 5 c . d
3456789012345678901234567890121234567890123456789012345678901212345
6
12
2
3456789012345678901234567890121234567890123456789012345678901212345
6
123456789012345678901234567890121234567890123456789012345678901212345
D. a . b , c 5 d
6
123456789012345678901234567890121234567890123456789012345678901212345
6
1
E.
a
,
b
,
c
,
d
3456789012345678901234567890121234567890123456789012345678901212345 6
12
3456789012345678901234567890121234567890123456789012345678901212345 6
12
3456789012345678901234567890121234567890123456789012345678901212345 6
12
3456789012345678901234567890121234567890123456789012345678901212345 6
12
123456789012345678901234567890121234567890123456789012345678901212345 6
123456789012345678901234567890121234567890123456789012345678901212345 6
123456789012345678901234567890121234567890123456789012345678901212345 6
3456789012345678901234567890121234567890123456789012345678901212345 6
12
3456789012345678901234567890121234567890123456789012345678901212345 6
12
3456789012345678901234567890121234567890123456789012345678901212345 6
12
3456789012345678901234567890121234567890123456789012345678901212345 6
12
6
123456789012345678901234567890121234567890123456789012345678901212345
10. Which county and crop had the smallest
2
6
123456789012345678901234567890121234567890123456789012345678901212345
6
123456789012345678901234567890121234567890123456789012345678901212345
percentage
change
from
1970
to
1990?
6
123456789012345678901234567890121234567890123456789012345678901212345
6
123456789012345678901234567890121234567890123456789012345678901212345
6
1 3456789012345678901234567890121234567890123456789012345678901212345
A.
County
X,
Corn
2
3456789012345678901234567890121234567890123456789012345678901212345
123456789012345678901234567890121234567890123456789012345678901212345 66
123456789012345678901234567890121234567890123456789012345678901212345
B. County Y, Corn
6
123456789012345678901234567890121234567890123456789012345678901212345
6
123456789012345678901234567890121234567890123456789012345678901212345
C.
County
X,
Wheat
6
1
2
3456789012345678901234567890121234567890123456789012345678901212345
6
123456789012345678901234567890121234567890123456789012345678901212345
D. County Y, Wheat
6
1
3456789012345678901234567890121234567890123456789012345678901212345
6
12
E. None of the above
123456789012345678901234567890121234567890123456789012345678901212345 6
3456789012345678901234567890121234567890123456789012345678901212345 6
12
123456789012345678901234567890121234567890123456789012345678901212345 6
123456789012345678901234567890121234567890123456789012345678901212345 6
123456789012345678901234567890121234567890123456789012345678901212345 66
284 123456789012345678901234567890121234567890123456789012345678901212345
1234567890123456789012345678901212345678901234567890123456789012123456

www.petersons.com

http://www.xtremepapers.net

TEST 2
1234567890123456789012345678901212345678901234567890123456789012123456
123456789012345678901234567890121234567890123456789012345678901212345 6
3456789012345678901234567890121234567890123456789012345678901212345 6
12
11. How much on average would ten acres of
6
14.
123456789012345678901234567890121234567890123456789012345678901212345
3456789012345678901234567890121234567890123456789012345678901212345
6
12
corn in County Y in 1980 produce?
3456789012345678901234567890121234567890123456789012345678901212345
6
12
123456789012345678901234567890121234567890123456789012345678901212345 6
3456789012345678901234567890121234567890123456789012345678901212345
6
12
A. 100 bushels
6
123456789012345678901234567890121234567890123456789012345678901212345
3456789012345678901234567890121234567890123456789012345678901212345
6
12
B.
120
bushels
3456789012345678901234567890121234567890123456789012345678901212345 6
12
3456789012345678901234567890121234567890123456789012345678901212345
6
12
C. 150 bushels
3456789012345678901234567890121234567890123456789012345678901212345
6
12
2
3456789012345678901234567890121234567890123456789012345678901212345
6
1
D. 200 bushels
3456789012345678901234567890121234567890123456789012345678901212345
6
12
3456789012345678901234567890121234567890123456789012345678901212345
6
12
E.
220
bushels
3456789012345678901234567890121234567890123456789012345678901212345 6
12
3456789012345678901234567890121234567890123456789012345678901212345 6
12
123456789012345678901234567890121234567890123456789012345678901212345 6
3456789012345678901234567890121234567890123456789012345678901212345
6
12
12. What is the ratio of bushels of corn
3456789012345678901234567890121234567890123456789012345678901212345
6
12
3456789012345678901234567890121234567890123456789012345678901212345
6
produced per acre to bushels of wheat
12
3456789012345678901234567890121234567890123456789012345678901212345
6
12
For
the
5-sided
polygon
above
a
1
b
5
produced
per
acre
in
County
Y
in
1980?
2
3456789012345678901234567890121234567890123456789012345678901212345
6
1
3456789012345678901234567890121234567890123456789012345678901212345
6
12
160
degrees
and
d
2
b
5
30.
How
many
3456789012345678901234567890121234567890123456789012345678901212345
6
12
A.
1:1
3456789012345678901234567890121234567890123456789012345678901212345
6
12
degrees
is
c
1
e
1
b?
3456789012345678901234567890121234567890123456789012345678901212345
6
12
B.
2:1
2
3456789012345678901234567890121234567890123456789012345678901212345
123456789012345678901234567890121234567890123456789012345678901212345 66
123456789012345678901234567890121234567890123456789012345678901212345
A.
70
C. 3:1
6
123456789012345678901234567890121234567890123456789012345678901212345
B. 125
6
123456789012345678901234567890121234567890123456789012345678901212345
D.
3:2
6
123456789012345678901234567890121234567890123456789012345678901212345
C.
350
6
123456789012345678901234567890121234567890123456789012345678901212345
E. 3:4
6
1
D. 300
3456789012345678901234567890121234567890123456789012345678901212345
6
12
3456789012345678901234567890121234567890123456789012345678901212345
6
12
E.
325
3456789012345678901234567890121234567890123456789012345678901212345
6
12
13. How many distinct integers less than 100 can
2
3456789012345678901234567890121234567890123456789012345678901212345
123456789012345678901234567890121234567890123456789012345678901212345 66
123456789012345678901234567890121234567890123456789012345678901212345
be formulated with the digits 2, 4, and 6?
6
123456789012345678901234567890121234567890123456789012345678901212345
15. What is the sum of the consecutive
6
123456789012345678901234567890121234567890123456789012345678901212345
A.
9
6
123456789012345678901234567890121234567890123456789012345678901212345
integers
from
15
to
55
(inclusive)?
6
123456789012345678901234567890121234567890123456789012345678901212345
B. 11
6
123456789012345678901234567890121234567890123456789012345678901212345
6
123456789012345678901234567890121234567890123456789012345678901212345
A. 1405
C.
12
123456789012345678901234567890121234567890123456789012345678901212345 66
1
B. 1435
D. 14
3456789012345678901234567890121234567890123456789012345678901212345
6
12
3456789012345678901234567890121234567890123456789012345678901212345
6
C. 1465
12
E.
15
6
123456789012345678901234567890121234567890123456789012345678901212345
D. 1500
6
123456789012345678901234567890121234567890123456789012345678901212345
3456789012345678901234567890121234567890123456789012345678901212345
6
12
E.
1587
2
3456789012345678901234567890121234567890123456789012345678901212345
123456789012345678901234567890121234567890123456789012345678901212345 66
123456789012345678901234567890121234567890123456789012345678901212345 6
1
3456789012345678901234567890121234567890123456789012345678901212345 6
12
3456789012345678901234567890121234567890123456789012345678901212345 6
12
2
6
123456789012345678901234567890121234567890123456789012345678901212345
6
123456789012345678901234567890121234567890123456789012345678901212345
1 3456789012345678901234567890121234567890123456789012345678901212345 6
123456789012345678901234567890121234567890123456789012345678901212345 6
123456789012345678901234567890121234567890123456789012345678901212345 6
3456789012345678901234567890121234567890123456789012345678901212345 6
12
3456789012345678901234567890121234567890123456789012345678901212345 6
12
3456789012345678901234567890121234567890123456789012345678901212345 6
12
3456789012345678901234567890121234567890123456789012345678901212345 6
12
123456789012345678901234567890121234567890123456789012345678901212345 6
2
6
123456789012345678901234567890121234567890123456789012345678901212345
6
123456789012345678901234567890121234567890123456789012345678901212345
6
123456789012345678901234567890121234567890123456789012345678901212345
6
123456789012345678901234567890121234567890123456789012345678901212345
1 3456789012345678901234567890121234567890123456789012345678901212345 6
2
3456789012345678901234567890121234567890123456789012345678901212345
123456789012345678901234567890121234567890123456789012345678901212345 66
123456789012345678901234567890121234567890123456789012345678901212345 6
123456789012345678901234567890121234567890123456789012345678901212345 6
123456789012345678901234567890121234567890123456789012345678901212345 6
1
3456789012345678901234567890121234567890123456789012345678901212345 6
12
123456789012345678901234567890121234567890123456789012345678901212345 6
3456789012345678901234567890121234567890123456789012345678901212345 6
12
123456789012345678901234567890121234567890123456789012345678901212345 6
3456789012345678901234567890121234567890123456789012345678901212345 6
12
123456789012345678901234567890121234567890123456789012345678901212345 6
123456789012345678901234567890121234567890123456789012345678901212345 6
123456789012345678901234567890121234567890123456789012345678901212345 6
123456789012345678901234567890121234567890123456789012345678901212345 6 285
1234567890123456789012345678901212345678901234567890123456789012123456

http://www.xtremepapers.net

Part IV: Three Practice GREs

1234567890123456789012345678901212345678901234567890123456789012123456
123456789012345678901234567890121234567890123456789012345678901212345 6
3456789012345678901234567890121234567890123456789012345678901212345 6
12
6
123456789012345678901234567890121234567890123456789012345678901212345
Column A
Column B
Column A
Column B
3456789012345678901234567890121234567890123456789012345678901212345
6
12
3456789012345678901234567890121234567890123456789012345678901212345
6
12
16. The circumference The perimeter of a
20. The circumference The perimeter of a 6
123456789012345678901234567890121234567890123456789012345678901212345
3456789012345678901234567890121234567890123456789012345678901212345
6
12
of a circle with
of a circle with a
rectangle with an
square that has one
6
123456789012345678901234567890121234567890123456789012345678901212345
3456789012345678901234567890121234567890123456789012345678901212345
6
12
1
a diameter
diameter of 7
area of 24
3456789012345678901234567890121234567890123456789012345678901212345
6
12
side
equal
to
1
3456789012345678901234567890121234567890123456789012345678901212345
6
12
equal
to
2
2
3456789012345678901234567890121234567890123456789012345678901212345 6
12
2
3456789012345678901234567890121234567890123456789012345678901212345
6
1
3456789012345678901234567890121234567890123456789012345678901212345 6
12
3456789012345678901234567890121234567890123456789012345678901212345 6
12
3456789012345678901234567890121234567890123456789012345678901212345 6
12
3456789012345678901234567890121234567890123456789012345678901212345 6
12
2
1 3456789012345678901234567890121234567890123456789012345678901212345 6
3456789012345678901234567890121234567890123456789012345678901212345 6
12
3456789012345678901234567890121234567890123456789012345678901212345 6
12
3456789012345678901234567890121234567890123456789012345678901212345 6
12
3456789012345678901234567890121234567890123456789012345678901212345
6
12
21. z is a one-digit even integer. The square
2
6
1 3456789012345678901234567890121234567890123456789012345678901212345
3456789012345678901234567890121234567890123456789012345678901212345
6
12
17.
root of z is also an integer.
3456789012345678901234567890121234567890123456789012345678901212345
6
12
1 1
2 1
3456789012345678901234567890121234567890123456789012345678901212345
6
12
2
3456789012345678901234567890121234567890123456789012345678901212345
6
12
3 4
3 5
z
1
z
1
z
z
2
3456789012345678901234567890121234567890123456789012345678901212345
123456789012345678901234567890121234567890123456789012345678901212345 66
123456789012345678901234567890121234567890123456789012345678901212345
2 2
2 1
6
123456789012345678901234567890121234567890123456789012345678901212345
6
123456789012345678901234567890121234567890123456789012345678901212345
3
4
5
3
123456789012345678901234567890121234567890123456789012345678901212345 66
123456789012345678901234567890121234567890123456789012345678901212345 6
1
3456789012345678901234567890121234567890123456789012345678901212345 6
12
3456789012345678901234567890121234567890123456789012345678901212345 6
12
3456789012345678901234567890121234567890123456789012345678901212345 6
12
2
6
123456789012345678901234567890121234567890123456789012345678901212345
6
123456789012345678901234567890121234567890123456789012345678901212345
6
123456789012345678901234567890121234567890123456789012345678901212345
22.
$16,000
is
invested
at
5%
simple
6
123456789012345678901234567890121234567890123456789012345678901212345
6
123456789012345678901234567890121234567890123456789012345678901212345
annual
interest.
3456789012345678901234567890121234567890123456789012345678901212345
6
123456789012345678901234567890121234567890123456789012345678901212345
18.
AB 5 CD
6
123456789012345678901234567890121234567890123456789012345678901212345
6
123456789012345678901234567890121234567890123456789012345678901212345
Interest
accrued
Six
hundred
dollars
123456789012345678901234567890121234567890123456789012345678901212345 66
1
after eight months
3456789012345678901234567890121234567890123456789012345678901212345
6
12
3456789012345678901234567890121234567890123456789012345678901212345 6
12
123456789012345678901234567890121234567890123456789012345678901212345 6
6
BC
10t
123456789012345678901234567890121234567890123456789012345678901212345
3456789012345678901234567890121234567890123456789012345678901212345 6
12
2
3456789012345678901234567890121234567890123456789012345678901212345
123456789012345678901234567890121234567890123456789012345678901212345 66
123456789012345678901234567890121234567890123456789012345678901212345 6
1
3456789012345678901234567890121234567890123456789012345678901212345 6
12
3456789012345678901234567890121234567890123456789012345678901212345 6
12
2
6
123456789012345678901234567890121234567890123456789012345678901212345
6
123456789012345678901234567890121234567890123456789012345678901212345
1 3456789012345678901234567890121234567890123456789012345678901212345 6
123456789012345678901234567890121234567890123456789012345678901212345 6
123456789012345678901234567890121234567890123456789012345678901212345 6
3456789012345678901234567890121234567890123456789012345678901212345 6
12
3456789012345678901234567890121234567890123456789012345678901212345
6
12
19.
3 2 2x . 5
3456789012345678901234567890121234567890123456789012345678901212345
6
12
3456789012345678901234567890121234567890123456789012345678901212345
6
12
2
x
2
123456789012345678901234567890121234567890123456789012345678901212345 6
2
3456789012345678901234567890121234567890123456789012345678901212345
123456789012345678901234567890121234567890123456789012345678901212345 66
123456789012345678901234567890121234567890123456789012345678901212345 6
123456789012345678901234567890121234567890123456789012345678901212345 6
123456789012345678901234567890121234567890123456789012345678901212345 6
1
3456789012345678901234567890121234567890123456789012345678901212345 6
12
3456789012345678901234567890121234567890123456789012345678901212345 6
12
3456789012345678901234567890121234567890123456789012345678901212345 6
12
3456789012345678901234567890121234567890123456789012345678901212345 6
12
123456789012345678901234567890121234567890123456789012345678901212345 6
2
6
123456789012345678901234567890121234567890123456789012345678901212345
1 3456789012345678901234567890121234567890123456789012345678901212345 6
3456789012345678901234567890121234567890123456789012345678901212345 6
12
123456789012345678901234567890121234567890123456789012345678901212345 6
3456789012345678901234567890121234567890123456789012345678901212345 6
12
123456789012345678901234567890121234567890123456789012345678901212345 6
123456789012345678901234567890121234567890123456789012345678901212345 6
123456789012345678901234567890121234567890123456789012345678901212345 66
286 123456789012345678901234567890121234567890123456789012345678901212345
1234567890123456789012345678901212345678901234567890123456789012123456

S DS D
S DS D

S DS D
S DS D

www.petersons.com

http://www.xtremepapers.net

TEST 2
1234567890123456789012345678901212345678901234567890123456789012123456
123456789012345678901234567890121234567890123456789012345678901212345 6
3456789012345678901234567890121234567890123456789012345678901212345 6
12
6
123456789012345678901234567890121234567890123456789012345678901212345
Column A
Column B
Column A
Column B
3456789012345678901234567890121234567890123456789012345678901212345
6
12
3456789012345678901234567890121234567890123456789012345678901212345
6
12
2
2
23.
The ratio of a to b is three fourths.
26.
x 2 4y 1 7 5 7
6
123456789012345678901234567890121234567890123456789012345678901212345
3456789012345678901234567890121234567890123456789012345678901212345 6
12
2
3456789012345678901234567890121234567890123456789012345678901212345
6
1
3456789012345678901234567890121234567890123456789012345678901212345 6
12
3456789012345678901234567890121234567890123456789012345678901212345 6
12
3456789012345678901234567890121234567890123456789012345678901212345 6
12
3456789012345678901234567890121234567890123456789012345678901212345 6
12
2
1 3456789012345678901234567890121234567890123456789012345678901212345 6
3456789012345678901234567890121234567890123456789012345678901212345 6
12
3456789012345678901234567890121234567890123456789012345678901212345 6
12
3456789012345678901234567890121234567890123456789012345678901212345 6
12
3456789012345678901234567890121234567890123456789012345678901212345 6
12
2
1 3456789012345678901234567890121234567890123456789012345678901212345 6
3456789012345678901234567890121234567890123456789012345678901212345 6
12
3456789012345678901234567890121234567890123456789012345678901212345 6
12
3456789012345678901234567890121234567890123456789012345678901212345
6
12
a = b
3456789012345678901234567890121234567890123456789012345678901212345
6
12
2
3456789012345678901234567890121234567890123456789012345678901212345
6
1
3456789012345678901234567890121234567890123456789012345678901212345 6
12
3456789012345678901234567890121234567890123456789012345678901212345 6
12
3456789012345678901234567890121234567890123456789012345678901212345 6
12
3456789012345678901234567890121234567890123456789012345678901212345 6
12
2
6
123456789012345678901234567890121234567890123456789012345678901212345
6
123456789012345678901234567890121234567890123456789012345678901212345
6
123456789012345678901234567890121234567890123456789012345678901212345
2
2
2
2
c
1
a
a
1
b
6
123456789012345678901234567890121234567890123456789012345678901212345
The area of
Seventy-five
6
123456789012345678901234567890121234567890123456789012345678901212345
3456789012345678901234567890121234567890123456789012345678901212345
6
123456789012345678901234567890121234567890123456789012345678901212345
sector 1
percent of sector 2
6
1
3456789012345678901234567890121234567890123456789012345678901212345 6
12
3456789012345678901234567890121234567890123456789012345678901212345 6
12
3456789012345678901234567890121234567890123456789012345678901212345 6
12
2
6
123456789012345678901234567890121234567890123456789012345678901212345
6
123456789012345678901234567890121234567890123456789012345678901212345
6
123456789012345678901234567890121234567890123456789012345678901212345
6
123456789012345678901234567890121234567890123456789012345678901212345
6
123456789012345678901234567890121234567890123456789012345678901212345
3456789012345678901234567890121234567890123456789012345678901212345
6
123456789012345678901234567890121234567890123456789012345678901212345
27.
C and D are integers, and C . D.
6
123456789012345678901234567890121234567890123456789012345678901212345
123456789012345678901234567890121234567890123456789012345678901212345 66
123456789012345678901234567890121234567890123456789012345678901212345
The greatest
The least common
The number of
C2 2 CD 1 2C
6
1 24.
2
3456789012345678901234567890121234567890123456789012345678901212345
6
123456789012345678901234567890121234567890123456789012345678901212345
common factor of multiple of 8 and 6
integers from D to
4C 2 3C
6
123456789012345678901234567890121234567890123456789012345678901212345
6
24 and 120
1
C
inclusive
123456789012345678901234567890121234567890123456789012345678901212345 6
3456789012345678901234567890121234567890123456789012345678901212345 6
12
3456789012345678901234567890121234567890123456789012345678901212345 6
12
3456789012345678901234567890121234567890123456789012345678901212345 6
12
123456789012345678901234567890121234567890123456789012345678901212345 6
3456789012345678901234567890121234567890123456789012345678901212345 6
12
3456789012345678901234567890121234567890123456789012345678901212345 6
12
2
6
123456789012345678901234567890121234567890123456789012345678901212345
6
123456789012345678901234567890121234567890123456789012345678901212345
1 3456789012345678901234567890121234567890123456789012345678901212345 6
123456789012345678901234567890121234567890123456789012345678901212345 6
123456789012345678901234567890121234567890123456789012345678901212345 6
3456789012345678901234567890121234567890123456789012345678901212345
6
12
25.
m and n are integers.
28.
{x, y, x 1 4, z, y 1 3}
3456789012345678901234567890121234567890123456789012345678901212345
6
12
3456789012345678901234567890121234567890123456789012345678901212345
6
12
This
number
set
is
ordered
from
least
to
(m 1 n)(m 1 n)
(m 1 2n)(m 2 n)
3456789012345678901234567890121234567890123456789012345678901212345
6
12
6
123456789012345678901234567890121234567890123456789012345678901212345
greatest,
and
x,
y,
and
z
are
distinct
2
3456789012345678901234567890121234567890123456789012345678901212345
123456789012345678901234567890121234567890123456789012345678901212345 66
123456789012345678901234567890121234567890123456789012345678901212345
integers.
6
123456789012345678901234567890121234567890123456789012345678901212345
123456789012345678901234567890121234567890123456789012345678901212345 66
1
z25
x
3456789012345678901234567890121234567890123456789012345678901212345
6
12
3456789012345678901234567890121234567890123456789012345678901212345 6
12
3456789012345678901234567890121234567890123456789012345678901212345 6
12
3456789012345678901234567890121234567890123456789012345678901212345 6
12
123456789012345678901234567890121234567890123456789012345678901212345 6
2
6
123456789012345678901234567890121234567890123456789012345678901212345
1 3456789012345678901234567890121234567890123456789012345678901212345 6
3456789012345678901234567890121234567890123456789012345678901212345 6
12
123456789012345678901234567890121234567890123456789012345678901212345 6
3456789012345678901234567890121234567890123456789012345678901212345 6
12
123456789012345678901234567890121234567890123456789012345678901212345 6
123456789012345678901234567890121234567890123456789012345678901212345 6
123456789012345678901234567890121234567890123456789012345678901212345 6
123456789012345678901234567890121234567890123456789012345678901212345 6 287
1234567890123456789012345678901212345678901234567890123456789012123456

http://www.xtremepapers.net

Part IV: Three Practice GREs

1234567890123456789012345678901212345678901234567890123456789012123456
123456789012345678901234567890121234567890123456789012345678901212345 6
3456789012345678901234567890121234567890123456789012345678901212345 6
12
6
123456789012345678901234567890121234567890123456789012345678901212345
Verbal
3456789012345678901234567890121234567890123456789012345678901212345
6
12
3456789012345678901234567890121234567890123456789012345678901212345 6
12
6
123456789012345678901234567890121234567890123456789012345678901212345
30 Questions30 Minutes
3456789012345678901234567890121234567890123456789012345678901212345
6
12
2
3456789012345678901234567890121234567890123456789012345678901212345
6
1
3456789012345678901234567890121234567890123456789012345678901212345
6
12
Antonyms
3456789012345678901234567890121234567890123456789012345678901212345 6
12
3456789012345678901234567890121234567890123456789012345678901212345 6
12
3456789012345678901234567890121234567890123456789012345678901212345 6
12
2
6
1 3456789012345678901234567890121234567890123456789012345678901212345
Directions: Each question contains a word printed in capital letters, followed by five
3456789012345678901234567890121234567890123456789012345678901212345
6
12
3456789012345678901234567890121234567890123456789012345678901212345
6
12
answer choices. Choose the answer choice that contains the word or phrase most nearly
3456789012345678901234567890121234567890123456789012345678901212345 6
12
3456789012345678901234567890121234567890123456789012345678901212345
6
12
OPPOSITE in meaning to the word in capital letters.
6
123456789012345678901234567890121234567890123456789012345678901212345
3456789012345678901234567890121234567890123456789012345678901212345 6
12
3456789012345678901234567890121234567890123456789012345678901212345 6
12
3456789012345678901234567890121234567890123456789012345678901212345 6
12
3456789012345678901234567890121234567890123456789012345678901212345 6
12
2
6
1 3456789012345678901234567890121234567890123456789012345678901212345
1. PRAGMATIC :
5. GARRULOUS :
3456789012345678901234567890121234567890123456789012345678901212345
6
12
3456789012345678901234567890121234567890123456789012345678901212345
6
12
A. reticent
A. trepidatious
3456789012345678901234567890121234567890123456789012345678901212345
6
12
3456789012345678901234567890121234567890123456789012345678901212345
6
12
B.
quixotic
B.
decorious
2
3456789012345678901234567890121234567890123456789012345678901212345
123456789012345678901234567890121234567890123456789012345678901212345 66
123456789012345678901234567890121234567890123456789012345678901212345
C. lavish
C. irksome
6
123456789012345678901234567890121234567890123456789012345678901212345
6
123456789012345678901234567890121234567890123456789012345678901212345
D.
practical
D.
gregarious
123456789012345678901234567890121234567890123456789012345678901212345 66
123456789012345678901234567890121234567890123456789012345678901212345
E. hopeful
E. taciturn
6
1
3456789012345678901234567890121234567890123456789012345678901212345 6
12
3456789012345678901234567890121234567890123456789012345678901212345 6
12
3456789012345678901234567890121234567890123456789012345678901212345
6
12
2. COGENT :
6. ENERVATE :
2
6
123456789012345678901234567890121234567890123456789012345678901212345
6
123456789012345678901234567890121234567890123456789012345678901212345
A. unintelligent
6
A. disabuse
123456789012345678901234567890121234567890123456789012345678901212345
6
123456789012345678901234567890121234567890123456789012345678901212345
B. culpable
B. strengthen
6
123456789012345678901234567890121234567890123456789012345678901212345
3456789012345678901234567890121234567890123456789012345678901212345
6
123456789012345678901234567890121234567890123456789012345678901212345
C. thoughtful
C. maximize
6
123456789012345678901234567890121234567890123456789012345678901212345
6
123456789012345678901234567890121234567890123456789012345678901212345
D. unconvincing
D. maltreat
6
123456789012345678901234567890121234567890123456789012345678901212345
6
1
E.
mysterious
E.
articulate
2
3456789012345678901234567890121234567890123456789012345678901212345
123456789012345678901234567890121234567890123456789012345678901212345 66
123456789012345678901234567890121234567890123456789012345678901212345 6
1
6
123456789012345678901234567890121234567890123456789012345678901212345
3. OBDURATE :
7. MANUMIT :
3456789012345678901234567890121234567890123456789012345678901212345
6
12
2
3456789012345678901234567890121234567890123456789012345678901212345
123456789012345678901234567890121234567890123456789012345678901212345 66
123456789012345678901234567890121234567890123456789012345678901212345
A. persuadable
A. enslave
6
1
3456789012345678901234567890121234567890123456789012345678901212345
6
12
B. gullible
B. indemnify
3456789012345678901234567890121234567890123456789012345678901212345 6
12
2
3456789012345678901234567890121234567890123456789012345678901212345
6
123456789012345678901234567890121234567890123456789012345678901212345
C. litigious
C. ostracize
6
123456789012345678901234567890121234567890123456789012345678901212345
D. manageable
D. simplify
6
1
6
123456789012345678901234567890121234567890123456789012345678901212345
E.
stubborn
E. delete
6
123456789012345678901234567890121234567890123456789012345678901212345
2
3456789012345678901234567890121234567890123456789012345678901212345
123456789012345678901234567890121234567890123456789012345678901212345 66
123456789012345678901234567890121234567890123456789012345678901212345 6
123456789012345678901234567890121234567890123456789012345678901212345
4. EBULLIENT :
6
123456789012345678901234567890121234567890123456789012345678901212345
6
1
2
3456789012345678901234567890121234567890123456789012345678901212345
6
123456789012345678901234567890121234567890123456789012345678901212345
A. stoic
6
123456789012345678901234567890121234567890123456789012345678901212345
6
123456789012345678901234567890121234567890123456789012345678901212345
B. stationary
123456789012345678901234567890121234567890123456789012345678901212345 66
1
C. stern
3456789012345678901234567890121234567890123456789012345678901212345
6
12
3456789012345678901234567890121234567890123456789012345678901212345
6
12
D. turgid
3456789012345678901234567890121234567890123456789012345678901212345 6
12
3456789012345678901234567890121234567890123456789012345678901212345
6
12
E. static
6
123456789012345678901234567890121234567890123456789012345678901212345
2
3456789012345678901234567890121234567890123456789012345678901212345
123456789012345678901234567890121234567890123456789012345678901212345 66
1
3456789012345678901234567890121234567890123456789012345678901212345 6
12
123456789012345678901234567890121234567890123456789012345678901212345 6
3456789012345678901234567890121234567890123456789012345678901212345 6
12
123456789012345678901234567890121234567890123456789012345678901212345 6
123456789012345678901234567890121234567890123456789012345678901212345 6
123456789012345678901234567890121234567890123456789012345678901212345 66
288 123456789012345678901234567890121234567890123456789012345678901212345
1234567890123456789012345678901212345678901234567890123456789012123456

www.petersons.com

http://www.xtremepapers.net

TEST 2
1234567890123456789012345678901212345678901234567890123456789012123456
123456789012345678901234567890121234567890123456789012345678901212345 6
3456789012345678901234567890121234567890123456789012345678901212345 6
12
8. ADUMBRATE :
9. PANEGYRIC :
6
123456789012345678901234567890121234567890123456789012345678901212345
3456789012345678901234567890121234567890123456789012345678901212345 6
12
3456789012345678901234567890121234567890123456789012345678901212345
6
12
A. to cease
A. assertion
6
123456789012345678901234567890121234567890123456789012345678901212345
3456789012345678901234567890121234567890123456789012345678901212345
6
12
B.
to
blandish
B.
elegy
2
3456789012345678901234567890121234567890123456789012345678901212345
6
1
3456789012345678901234567890121234567890123456789012345678901212345
6
12
C.
to
foreshadow
C.
dirge
3456789012345678901234567890121234567890123456789012345678901212345 6
12
3456789012345678901234567890121234567890123456789012345678901212345
6
12
D. to make explicit
D. dissertation
3456789012345678901234567890121234567890123456789012345678901212345
6
12
2
3456789012345678901234567890121234567890123456789012345678901212345
6
1
E. to loosen
E. obloquy
3456789012345678901234567890121234567890123456789012345678901212345
6
12
3456789012345678901234567890121234567890123456789012345678901212345 6
12
3456789012345678901234567890121234567890123456789012345678901212345 6
12
3456789012345678901234567890121234567890123456789012345678901212345
6
12
Analogies
2
6
1 3456789012345678901234567890121234567890123456789012345678901212345
3456789012345678901234567890121234567890123456789012345678901212345 6
12
3456789012345678901234567890121234567890123456789012345678901212345 6
12
3456789012345678901234567890121234567890123456789012345678901212345
6
12
Directions: In each question, there is an initial pair of words or phrases that are related in
3456789012345678901234567890121234567890123456789012345678901212345
6
12
2
3456789012345678901234567890121234567890123456789012345678901212345
6
1
some manner. Select the answer choice containing the lettered pair that best expresses a
3456789012345678901234567890121234567890123456789012345678901212345
6
12
3456789012345678901234567890121234567890123456789012345678901212345
6
12
relationship similar to the initial pair.
3456789012345678901234567890121234567890123456789012345678901212345
6
12
3456789012345678901234567890121234567890123456789012345678901212345 6
12
2
3456789012345678901234567890121234567890123456789012345678901212345
123456789012345678901234567890121234567890123456789012345678901212345 66
123456789012345678901234567890121234567890123456789012345678901212345 6
123456789012345678901234567890121234567890123456789012345678901212345 6
123456789012345678901234567890121234567890123456789012345678901212345
10. ECLIPSE : OVERSHADOW ::
14. HALCYON : PEACEFUL ::
6
123456789012345678901234567890121234567890123456789012345678901212345
123456789012345678901234567890121234567890123456789012345678901212345 66
1
A. set : rise
A. tectonic : magnetic
3456789012345678901234567890121234567890123456789012345678901212345
6
12
3456789012345678901234567890121234567890123456789012345678901212345
6
12
B.
outstrip
:
flail
B.
tempestuous
:
placid
3456789012345678901234567890121234567890123456789012345678901212345 6
12
2
3456789012345678901234567890121234567890123456789012345678901212345
6
123456789012345678901234567890121234567890123456789012345678901212345
C. debauch : debase
C. surreptitious : clandestine
6
123456789012345678901234567890121234567890123456789012345678901212345
6
123456789012345678901234567890121234567890123456789012345678901212345
D.
overcome
:
outstretch
D.
boisterous
:
rueful
123456789012345678901234567890121234567890123456789012345678901212345 66
123456789012345678901234567890121234567890123456789012345678901212345
E. pervert : correct
E. terrifying : relaxing
6
123456789012345678901234567890121234567890123456789012345678901212345
123456789012345678901234567890121234567890123456789012345678901212345 66
123456789012345678901234567890121234567890123456789012345678901212345
11. EMPHATIC : INSIST ::
6
15. DESOLATE : VERDANT ::
123456789012345678901234567890121234567890123456789012345678901212345
6
1
2
3456789012345678901234567890121234567890123456789012345678901212345
6
123456789012345678901234567890121234567890123456789012345678901212345
A. contentious : argue
A. destitute : opulent
6
123456789012345678901234567890121234567890123456789012345678901212345
6
1
B.
absorbed
:
engrossed
B.
rococo
:
ornate
123456789012345678901234567890121234567890123456789012345678901212345 6
3456789012345678901234567890121234567890123456789012345678901212345
6
12
C. raucous : mitigate
C. astringent : severe
3456789012345678901234567890121234567890123456789012345678901212345
6
12
3456789012345678901234567890121234567890123456789012345678901212345
6
12
D.
delusional
:
rage
D.
tropical
:
humid
123456789012345678901234567890121234567890123456789012345678901212345 6
3456789012345678901234567890121234567890123456789012345678901212345
6
12
E. intentional : forget
E. technical : detailed
3456789012345678901234567890121234567890123456789012345678901212345
6
12
2
3456789012345678901234567890121234567890123456789012345678901212345
123456789012345678901234567890121234567890123456789012345678901212345 66
123456789012345678901234567890121234567890123456789012345678901212345 6
1 12. ASTRONOMER : STARS ::
16. HARBINGER : PRESAGE ::
6
123456789012345678901234567890121234567890123456789012345678901212345
6
123456789012345678901234567890121234567890123456789012345678901212345
:
calculates
A.
account
:
financial
records
A.
computer
2
3456789012345678901234567890121234567890123456789012345678901212345
123456789012345678901234567890121234567890123456789012345678901212345 66
123456789012345678901234567890121234567890123456789012345678901212345
B. television : broadcasts
B. librarian : books
6
123456789012345678901234567890121234567890123456789012345678901212345
6
123456789012345678901234567890121234567890123456789012345678901212345
C.
liaison
:
interrogates
C.
historian
:
past
6
1
2
3456789012345678901234567890121234567890123456789012345678901212345
6
123456789012345678901234567890121234567890123456789012345678901212345
D. messenger : relates
D. police officer : laws
6
123456789012345678901234567890121234567890123456789012345678901212345
6
123456789012345678901234567890121234567890123456789012345678901212345
E.
broker
:
absconds
E.
magician
:
tricks
123456789012345678901234567890121234567890123456789012345678901212345 66
1
3456789012345678901234567890121234567890123456789012345678901212345 6
12
3456789012345678901234567890121234567890123456789012345678901212345
6
12
17. ASSEVERATE : AVER ::
13. INOCULATE : DISEASE ::
3456789012345678901234567890121234567890123456789012345678901212345
6
12
3456789012345678901234567890121234567890123456789012345678901212345 6
12
6
123456789012345678901234567890121234567890123456789012345678901212345
A. beseech : request
A. defraud : conspirator
3456789012345678901234567890121234567890123456789012345678901212345
6
12
B. reject : install
B. detest : agitator
6
123456789012345678901234567890121234567890123456789012345678901212345
3456789012345678901234567890121234567890123456789012345678901212345
6
12
C. answer : reply
C. delight : friend
6
123456789012345678901234567890121234567890123456789012345678901212345
3456789012345678901234567890121234567890123456789012345678901212345
6
12
D.
enrage
:
mollify
D.
defend
:
enemy
2
3456789012345678901234567890121234567890123456789012345678901212345
6
1
6
123456789012345678901234567890121234567890123456789012345678901212345
E. approach : accost
E. defame : title
6
123456789012345678901234567890121234567890123456789012345678901212345
123456789012345678901234567890121234567890123456789012345678901212345 6 289
1234567890123456789012345678901212345678901234567890123456789012123456

http://www.xtremepapers.net

Part IV: Three Practice GREs

1234567890123456789012345678901212345678901234567890123456789012123456
123456789012345678901234567890121234567890123456789012345678901212345 6
3456789012345678901234567890121234567890123456789012345678901212345 6
12
Sentence Completions
6
123456789012345678901234567890121234567890123456789012345678901212345
3456789012345678901234567890121234567890123456789012345678901212345 6
12
3456789012345678901234567890121234567890123456789012345678901212345 6
12
123456789012345678901234567890121234567890123456789012345678901212345 6
3456789012345678901234567890121234567890123456789012345678901212345
6
12
Directions: Each sentence below contains one or two blanks, with each blank corre6
123456789012345678901234567890121234567890123456789012345678901212345
3456789012345678901234567890121234567890123456789012345678901212345
6
12
sponding to an omitted word. Find the answer choice that has the word or set or words
3456789012345678901234567890121234567890123456789012345678901212345 6
12
that best fits the meaning of the sentence as a whole.
3456789012345678901234567890121234567890123456789012345678901212345
6
12
3456789012345678901234567890121234567890123456789012345678901212345 6
12
2
3456789012345678901234567890121234567890123456789012345678901212345
6
1
3456789012345678901234567890121234567890123456789012345678901212345 6
12
3456789012345678901234567890121234567890123456789012345678901212345 6
12
3456789012345678901234567890121234567890123456789012345678901212345 6
12
18. Mr. Thomas had, in his local community,
3456789012345678901234567890121234567890123456789012345678901212345
6
21. The __________ feuds between rival
12
2
6
1 3456789012345678901234567890121234567890123456789012345678901212345
earned the reputation as being somewhat
factions of the companys board destroyed 6
3456789012345678901234567890121234567890123456789012345678901212345
12
3456789012345678901234567890121234567890123456789012345678901212345
6
12
of a miser, but a review of his financial
the effectiveness of the companys
3456789012345678901234567890121234567890123456789012345678901212345
6
12
3456789012345678901234567890121234567890123456789012345678901212345
6
12
records
revealed
he
was
actually
a
leadership.
2
3456789012345678901234567890121234567890123456789012345678901212345
6
1
3456789012345678901234567890121234567890123456789012345678901212345
6
12
__________.
3456789012345678901234567890121234567890123456789012345678901212345
6
12
A.
fatalistic
3456789012345678901234567890121234567890123456789012345678901212345 6
12
3456789012345678901234567890121234567890123456789012345678901212345
6
12
A. maverick
B. infernal
3456789012345678901234567890121234567890123456789012345678901212345
6
12
3456789012345678901234567890121234567890123456789012345678901212345
6
12
B.
hooligan
C.
hegemonic
3456789012345678901234567890121234567890123456789012345678901212345 6
12
3456789012345678901234567890121234567890123456789012345678901212345
6
12
C. maestro
D. internecine
3456789012345678901234567890121234567890123456789012345678901212345
6
12
2
3456789012345678901234567890121234567890123456789012345678901212345
6
123456789012345678901234567890121234567890123456789012345678901212345
D. spendthrift
E. flagging
6
1
3456789012345678901234567890121234567890123456789012345678901212345
6
12
E.
micro-manager
3456789012345678901234567890121234567890123456789012345678901212345
6
12
22. Unlike his most recent predecessor, the
3456789012345678901234567890121234567890123456789012345678901212345
6
12
2
3456789012345678901234567890121234567890123456789012345678901212345
6
123456789012345678901234567890121234567890123456789012345678901212345
19. Recent demographic shifts in the populanewly elected mayor did not appoint
6
123456789012345678901234567890121234567890123456789012345678901212345
6
123456789012345678901234567890121234567890123456789012345678901212345
tion of Country X have resulted in many
friends and associates to high posts in the 6
123456789012345678901234567890121234567890123456789012345678901212345
6
123456789012345678901234567890121234567890123456789012345678901212345
traditional political alliances becoming
city government, but eschewed such
6
123456789012345678901234567890121234567890123456789012345678901212345
123456789012345678901234567890121234567890123456789012345678901212345
__________ as nascent political parties
__________ in favor of a __________-based 6
6
123456789012345678901234567890121234567890123456789012345678901212345
6
123456789012345678901234567890121234567890123456789012345678901212345
__________
large
swathes
of
the
electoral
appointment
criteria.
6
1
2
3456789012345678901234567890121234567890123456789012345678901212345
6
123456789012345678901234567890121234567890123456789012345678901212345
landscape.
6
123456789012345678901234567890121234567890123456789012345678901212345
A.
nepotism.
.value
6
1
6
123456789012345678901234567890121234567890123456789012345678901212345
A.
obsolete.
.secure
B.
cronyism.
.merit
3456789012345678901234567890121234567890123456789012345678901212345 6
12
2
3456789012345678901234567890121234567890123456789012345678901212345
6
123456789012345678901234567890121234567890123456789012345678901212345
B. dislodged. .provide
C. favoritism. .work
6
123456789012345678901234567890121234567890123456789012345678901212345
6
1
C.
unhinged.
.procure
D.
jingoism.
.resum
3456789012345678901234567890121234567890123456789012345678901212345 6
12
3456789012345678901234567890121234567890123456789012345678901212345
6
12
D. solvent. .release
E. recidivism. .experience
3456789012345678901234567890121234567890123456789012345678901212345
6
12
3456789012345678901234567890121234567890123456789012345678901212345
6
12
E. elusive. .gain
6
123456789012345678901234567890121234567890123456789012345678901212345
23. Though customarily marked by deliberate 6
123456789012345678901234567890121234567890123456789012345678901212345
6
123456789012345678901234567890121234567890123456789012345678901212345
20. Many see the attempt to sway public
and __________ prose, the authors most
3456789012345678901234567890121234567890123456789012345678901212345
6
12
3456789012345678901234567890121234567890123456789012345678901212345
6
12
__________,
though
ostensibly
a
process
recent correspondence was convoluted
3456789012345678901234567890121234567890123456789012345678901212345
6
12
3456789012345678901234567890121234567890123456789012345678901212345
6
12
of
popular
persuasion,
to
be
__________
and
__________.
123456789012345678901234567890121234567890123456789012345678901212345 6
2
3456789012345678901234567890121234567890123456789012345678901212345
6
123456789012345678901234567890121234567890123456789012345678901212345
since it has been undertaken employing
6
123456789012345678901234567890121234567890123456789012345678901212345
A.
terse.
.tiresome
6
123456789012345678901234567890121234567890123456789012345678901212345
questionable
pretences.
6
123456789012345678901234567890121234567890123456789012345678901212345
B.
genuine.
.superfluous
6
1
2
3456789012345678901234567890121234567890123456789012345678901212345
6
123456789012345678901234567890121234567890123456789012345678901212345
A. consent. .disreputable
C. acidic. .prosaic
6
123456789012345678901234567890121234567890123456789012345678901212345
6
123456789012345678901234567890121234567890123456789012345678901212345
B. opinion. .undemocratic
D. urbane. .ubiquitous
123456789012345678901234567890121234567890123456789012345678901212345 66
1
C. disfavor. .impractical
E. succinc. .prolix
3456789012345678901234567890121234567890123456789012345678901212345
6
12
6
123456789012345678901234567890121234567890123456789012345678901212345
D. ill-will. .treacherous
3456789012345678901234567890121234567890123456789012345678901212345 6
12
6
123456789012345678901234567890121234567890123456789012345678901212345
E. disinterest. .crude
3456789012345678901234567890121234567890123456789012345678901212345
6
12
2
3456789012345678901234567890121234567890123456789012345678901212345
6
1
123456789012345678901234567890121234567890123456789012345678901212345 6
123456789012345678901234567890121234567890123456789012345678901212345 66
290 123456789012345678901234567890121234567890123456789012345678901212345
1234567890123456789012345678901212345678901234567890123456789012123456

www.petersons.com

http://www.xtremepapers.net

TEST 2
1234567890123456789012345678901212345678901234567890123456789012123456
123456789012345678901234567890121234567890123456789012345678901212345 6
3456789012345678901234567890121234567890123456789012345678901212345 6
12
Reading Comprehension
6
123456789012345678901234567890121234567890123456789012345678901212345
3456789012345678901234567890121234567890123456789012345678901212345 6
12
3456789012345678901234567890121234567890123456789012345678901212345 6
12
123456789012345678901234567890121234567890123456789012345678901212345 6
3456789012345678901234567890121234567890123456789012345678901212345
6
12
Directions: Questions follow each passage. After reading the passage, determine the best
6
123456789012345678901234567890121234567890123456789012345678901212345
3456789012345678901234567890121234567890123456789012345678901212345
6
12
answer for each question based on the passages content.
3456789012345678901234567890121234567890123456789012345678901212345 6
12
3456789012345678901234567890121234567890123456789012345678901212345 6
12
3456789012345678901234567890121234567890123456789012345678901212345 6
12
2
1 3456789012345678901234567890121234567890123456789012345678901212345 6
3456789012345678901234567890121234567890123456789012345678901212345 6
12
3456789012345678901234567890121234567890123456789012345678901212345
12
Passage 1
cellular life, it was believed, was a fixture 6
3456789012345678901234567890121234567890123456789012345678901212345
6
12
3456789012345678901234567890121234567890123456789012345678901212345
6
across all biological organisms. Archae12
Line
The
existence
and
novelty
of
the
domain
2
3456789012345678901234567890121234567890123456789012345678901212345
6
1
ans, in recent decades, have repeatedly
3456789012345678901234567890121234567890123456789012345678901212345
6
12
Archaean has demanded many long3456789012345678901234567890121234567890123456789012345678901212345
6
12
demonstrated that the previously
3456789012345678901234567890121234567890123456789012345678901212345
6
12
standing
hypotheses
in
biology
to
be
3456789012345678901234567890121234567890123456789012345678901212345
6
12
(40)
maintained
thermal
threshold
for
life
was
2
3456789012345678901234567890121234567890123456789012345678901212345
6
1
reconfigured. The most basic challenge
3456789012345678901234567890121234567890123456789012345678901212345
6
12
far too low. So-called extremophilic
3456789012345678901234567890121234567890123456789012345678901212345
6
12
(5)
that
the
discovery
of
Archaeans
presented
3456789012345678901234567890121234567890123456789012345678901212345
6
12
Archaeans
have
been
discovered
to
thrive
3456789012345678901234567890121234567890123456789012345678901212345
6
12
to
biologists
was
classification.
Prior
to
2
3456789012345678901234567890121234567890123456789012345678901212345
6
123456789012345678901234567890121234567890123456789012345678901212345
in temperatures as high as 160 degrees
6
123456789012345678901234567890121234567890123456789012345678901212345
the
discovery
of
the
existence
of
Archae6
123456789012345678901234567890121234567890123456789012345678901212345
centigrade.
Such
discoveries
have
6
123456789012345678901234567890121234567890123456789012345678901212345
ans,
life
was
classified
into
five
king6
123456789012345678901234567890121234567890123456789012345678901212345
(45)
required
a
broadening
of
biologys
6
123456789012345678901234567890121234567890123456789012345678901212345
doms: Animalia, Plantae, Fungi, Proto6
1
conceptions
concerning
what
environ3456789012345678901234567890121234567890123456789012345678901212345
6
12
(10)
zoa,
and
Bacteria.
But
Archaeans,
being
3456789012345678901234567890121234567890123456789012345678901212345
6
12
ments
are
hospitable
to
life.
3456789012345678901234567890121234567890123456789012345678901212345
6
12
such a biological novelty, did not readily
2
3456789012345678901234567890121234567890123456789012345678901212345
123456789012345678901234567890121234567890123456789012345678901212345 66
123456789012345678901234567890121234567890123456789012345678901212345
submit to this system of classification. In
6
123456789012345678901234567890121234567890123456789012345678901212345
24. This passage could best be titled
6
123456789012345678901234567890121234567890123456789012345678901212345
fact, an innovation of biological classifi123456789012345678901234567890121234567890123456789012345678901212345 66
123456789012345678901234567890121234567890123456789012345678901212345
cation was needed to properly place
A. Biological Designations in Transfor- 6
123456789012345678901234567890121234567890123456789012345678901212345
(15)
Archaeans in the panorama of life.
6
123456789012345678901234567890121234567890123456789012345678901212345
mation.
6
123456789012345678901234567890121234567890123456789012345678901212345
Previously, kingdom had been the most
6
1
B.
The
Archaean
Diversity
of
Life.
2
3456789012345678901234567890121234567890123456789012345678901212345
6
123456789012345678901234567890121234567890123456789012345678901212345
general biological designation, but to
6
123456789012345678901234567890121234567890123456789012345678901212345
C.
The
Archaean
Challenge
to
Biological
6
1
properly include the Archaeans a new
6
123456789012345678901234567890121234567890123456789012345678901212345
Paradigms.
3456789012345678901234567890121234567890123456789012345678901212345
6
12
biological
classification,
the
domain,
was
2
3456789012345678901234567890121234567890123456789012345678901212345
123456789012345678901234567890121234567890123456789012345678901212345
D. Discovering Thermophilic Archaeans. 6
6
123456789012345678901234567890121234567890123456789012345678901212345
6
1 (20) formulated. Now kingdoms are classified
E.
Classifying
Thermophiles.
3456789012345678901234567890121234567890123456789012345678901212345
6
12
as
a
sub-group
of
a
particular
domain.
3456789012345678901234567890121234567890123456789012345678901212345 6
12
2
3456789012345678901234567890121234567890123456789012345678901212345
6
123456789012345678901234567890121234567890123456789012345678901212345
Thus, domain has become the most
6
123456789012345678901234567890121234567890123456789012345678901212345
6
1
general
biological
designation
for
life.
123456789012345678901234567890121234567890123456789012345678901212345 6
6
123456789012345678901234567890121234567890123456789012345678901212345
A second arena in which Archaeans
3456789012345678901234567890121234567890123456789012345678901212345
6
12
3456789012345678901234567890121234567890123456789012345678901212345
6
12
(25)
have
challenged
accepted
hypotheses
and
3456789012345678901234567890121234567890123456789012345678901212345 6
12
3456789012345678901234567890121234567890123456789012345678901212345
6
12
broadened biological horizons is in the
6
123456789012345678901234567890121234567890123456789012345678901212345
2
3456789012345678901234567890121234567890123456789012345678901212345
6
123456789012345678901234567890121234567890123456789012345678901212345
realm of environmental extremes. It was
6
123456789012345678901234567890121234567890123456789012345678901212345
6
123456789012345678901234567890121234567890123456789012345678901212345
generally accepted not more than a few
123456789012345678901234567890121234567890123456789012345678901212345 66
1
decades ago that no life could exist in
3456789012345678901234567890121234567890123456789012345678901212345
6
12
3456789012345678901234567890121234567890123456789012345678901212345
6
12
(30)
temperatures much hotter than 60
3456789012345678901234567890121234567890123456789012345678901212345 6
12
3456789012345678901234567890121234567890123456789012345678901212345
6
12
degrees centigrade. This limit was set
6
123456789012345678901234567890121234567890123456789012345678901212345
because
it
was
thought
that
the
molecular
2
3456789012345678901234567890121234567890123456789012345678901212345
123456789012345678901234567890121234567890123456789012345678901212345 66
1
integrity of vital cellular components
3456789012345678901234567890121234567890123456789012345678901212345
6
12
6
123456789012345678901234567890121234567890123456789012345678901212345
could
not
be
maintained
beyond
such
3456789012345678901234567890121234567890123456789012345678901212345 6
12
2
3456789012345678901234567890121234567890123456789012345678901212345
6
1 (35) temperatures. The thermal capacity of
123456789012345678901234567890121234567890123456789012345678901212345 6
123456789012345678901234567890121234567890123456789012345678901212345 6
123456789012345678901234567890121234567890123456789012345678901212345 6 291
1234567890123456789012345678901212345678901234567890123456789012123456

http://www.xtremepapers.net

Part IV: Three Practice GREs

1234567890123456789012345678901212345678901234567890123456789012123456
123456789012345678901234567890121234567890123456789012345678901212345 6
3456789012345678901234567890121234567890123456789012345678901212345 6
12
25. The passage implies that extremophilic
6
Passage 2
123456789012345678901234567890121234567890123456789012345678901212345
3456789012345678901234567890121234567890123456789012345678901212345
6
12
Archaeans
3456789012345678901234567890121234567890123456789012345678901212345
6
12
Line
Theoretical
Economics
faces
considerable
123456789012345678901234567890121234567890123456789012345678901212345 6
3456789012345678901234567890121234567890123456789012345678901212345
12
I. are able to maintain molecular
challenges in developing reliable models 6
6
123456789012345678901234567890121234567890123456789012345678901212345
3456789012345678901234567890121234567890123456789012345678901212345
6
12
integrity
of
cellular
components
past
for
public
choice
scenarios.
These
3456789012345678901234567890121234567890123456789012345678901212345 6
12
3456789012345678901234567890121234567890123456789012345678901212345
12
the formerly accepted thermal
scenarios involve aggregating individual 6
3456789012345678901234567890121234567890123456789012345678901212345
6
12
2
3456789012345678901234567890121234567890123456789012345678901212345
1
(5)
threshold of life.
preferences into a public choice outcome. 6
3456789012345678901234567890121234567890123456789012345678901212345
6
12
3456789012345678901234567890121234567890123456789012345678901212345
6
12
II.
have
been
known
to
exist
in
moderate
An
election
in
a
democratic
society
is
a
3456789012345678901234567890121234567890123456789012345678901212345 6
12
3456789012345678901234567890121234567890123456789012345678901212345
6
12
environments for some years, but
well-considered example of a public
6
123456789012345678901234567890121234567890123456789012345678901212345
3456789012345678901234567890121234567890123456789012345678901212345
6
12
their
extremophilic
properties
have
choice
scenario.
Formulating
helpful
and
3456789012345678901234567890121234567890123456789012345678901212345 6
12
3456789012345678901234567890121234567890123456789012345678901212345
12
only recently been discovered.
insightful models of political elections is 6
3456789012345678901234567890121234567890123456789012345678901212345
6
12
2
3456789012345678901234567890121234567890123456789012345678901212345
6
1
(10)
III. are able to live and thrive in temperaa perpetual issue in politico-economic
3456789012345678901234567890121234567890123456789012345678901212345
6
12
3456789012345678901234567890121234567890123456789012345678901212345
6
12
tures as hot as 150 degrees centigrade.
discourse.
3456789012345678901234567890121234567890123456789012345678901212345 6
12
3456789012345678901234567890121234567890123456789012345678901212345
12
Many public-choice election scenario 6
A. I only
3456789012345678901234567890121234567890123456789012345678901212345
6
12
3456789012345678901234567890121234567890123456789012345678901212345
6
12
models begin with the individual as the
B. II only
3456789012345678901234567890121234567890123456789012345678901212345
6
12
basic building block of the aggregate
3456789012345678901234567890121234567890123456789012345678901212345
6
12
C.
III
only
3456789012345678901234567890121234567890123456789012345678901212345
6
12
(15)
outcome.
On
this
methodology,
the
2
3456789012345678901234567890121234567890123456789012345678901212345
6
123456789012345678901234567890121234567890123456789012345678901212345
D. I and II only
6
1
primary issue is determining a proper
3456789012345678901234567890121234567890123456789012345678901212345
6
12
E.
I
and
III
only
3456789012345678901234567890121234567890123456789012345678901212345
6
12
decision-matrix
for
the
individual
in
the
3456789012345678901234567890121234567890123456789012345678901212345 6
12
2
3456789012345678901234567890121234567890123456789012345678901212345
6
123456789012345678901234567890121234567890123456789012345678901212345
society under consideration. The deci6
123456789012345678901234567890121234567890123456789012345678901212345
26.
The
passage
states
that
a
kingdom
is
6
123456789012345678901234567890121234567890123456789012345678901212345
sion-matrix
is
a
theoretical
construct
123456789012345678901234567890121234567890123456789012345678901212345 66
123456789012345678901234567890121234567890123456789012345678901212345
A. a sub-group within a domain.
(20)
whichwhen given the appropriate input 6
123456789012345678901234567890121234567890123456789012345678901212345
6
123456789012345678901234567890121234567890123456789012345678901212345
B. the most general biological designation.
for a particular situationoutputs the
6
123456789012345678901234567890121234567890123456789012345678901212345
C. composed of sub-groups of domains.
6
123456789012345678901234567890121234567890123456789012345678901212345
choice
of
a
rational
individual
in
the
6
1
D.
a
newly
formulated
biological
2
3456789012345678901234567890121234567890123456789012345678901212345
6
123456789012345678901234567890121234567890123456789012345678901212345
specified situation. The integrity of the
6
123456789012345678901234567890121234567890123456789012345678901212345
classification.
6
1
decision-matrix
is
the
central
concern
of
6
123456789012345678901234567890121234567890123456789012345678901212345
E.
an
Archaean
domain.
3456789012345678901234567890121234567890123456789012345678901212345
6
12
(25)
the
model-maker.
If
the
decision-matrix
2
3456789012345678901234567890121234567890123456789012345678901212345
123456789012345678901234567890121234567890123456789012345678901212345 66
123456789012345678901234567890121234567890123456789012345678901212345
appropriately produces the choices of
6
1 27. It can be inferred from the passage that the
3456789012345678901234567890121234567890123456789012345678901212345
12
individuals in specific situations, then the 6
3456789012345678901234567890121234567890123456789012345678901212345
6
12
phrase thermal threshold in line 40 is
3456789012345678901234567890121234567890123456789012345678901212345
6
12
aggregate model will be useable. How3456789012345678901234567890121234567890123456789012345678901212345
6
12
123456789012345678901234567890121234567890123456789012345678901212345
A. the critical point temperature at which
ever, if the decision-matrix inadequately 6
123456789012345678901234567890121234567890123456789012345678901212345 6
6
123456789012345678901234567890121234567890123456789012345678901212345
(30)
the metabolic pathways of extremoreflects the decision-making process of
3456789012345678901234567890121234567890123456789012345678901212345
6
12
philes become functional.
individuals, the aggregate model will be
3456789012345678901234567890121234567890123456789012345678901212345
6
12
3456789012345678901234567890121234567890123456789012345678901212345
6
12
B. the temperature at which the molecuunreliable.
3456789012345678901234567890121234567890123456789012345678901212345
6
12
6
123456789012345678901234567890121234567890123456789012345678901212345
lar
integrity
of
cellular
components
of
Decision-matrices
are
often
con2
3456789012345678901234567890121234567890123456789012345678901212345
123456789012345678901234567890121234567890123456789012345678901212345 66
123456789012345678901234567890121234567890123456789012345678901212345
an organism are compromised.
structed on the assumption that individu- 6
123456789012345678901234567890121234567890123456789012345678901212345
6
123456789012345678901234567890121234567890123456789012345678901212345
(35)
C.
the
thermal
capacity
of
extremophiles.
als are rational in decision-making and
6
1
2
3456789012345678901234567890121234567890123456789012345678901212345
6
123456789012345678901234567890121234567890123456789012345678901212345
D. an environmental extremity indicator.
that they predicate decisions upon
6
123456789012345678901234567890121234567890123456789012345678901212345
6
123456789012345678901234567890121234567890123456789012345678901212345
E.
a
biological
constant
across
all
of
self-interest.
Furthermore,
self-interest
is
123456789012345678901234567890121234567890123456789012345678901212345 66
1
nature.
generally construed to mean selfish
3456789012345678901234567890121234567890123456789012345678901212345
6
12
6
123456789012345678901234567890121234567890123456789012345678901212345
interests.
A
serious
flaw
in
such
a
3456789012345678901234567890121234567890123456789012345678901212345 6
12
6
123456789012345678901234567890121234567890123456789012345678901212345
(40)
constructional is its disallowal of
3456789012345678901234567890121234567890123456789012345678901212345
6
12
2
3456789012345678901234567890121234567890123456789012345678901212345
6
1
altruistic motivation in the decision6
123456789012345678901234567890121234567890123456789012345678901212345
123456789012345678901234567890121234567890123456789012345678901212345 66
292 123456789012345678901234567890121234567890123456789012345678901212345
1234567890123456789012345678901212345678901234567890123456789012123456

www.petersons.com

http://www.xtremepapers.net

TEST 2
1234567890123456789012345678901212345678901234567890123456789012123456
123456789012345678901234567890121234567890123456789012345678901212345 6
3456789012345678901234567890121234567890123456789012345678901212345 6
12
matrix. It can easily be empirically
6
29. What is a common assumption of
123456789012345678901234567890121234567890123456789012345678901212345
3456789012345678901234567890121234567890123456789012345678901212345
6
12
verified that human choice is not
decision-matrix construction that the
3456789012345678901234567890121234567890123456789012345678901212345
6
12
6
123456789012345678901234567890121234567890123456789012345678901212345
predominated
by
altruism,
but
such
author
does
not
dispute?
3456789012345678901234567890121234567890123456789012345678901212345 6
12
2
3456789012345678901234567890121234567890123456789012345678901212345
6
1 (45) verification does not warrant the
3456789012345678901234567890121234567890123456789012345678901212345
6
12
A.
Self-interest
should
be
construed
as
3456789012345678901234567890121234567890123456789012345678901212345
6
12
complete
exclusion
of
altruism
from
3456789012345678901234567890121234567890123456789012345678901212345
6
12
selfish
interest.
3456789012345678901234567890121234567890123456789012345678901212345
6
12
decision-matrices.
Therefore,
altruistic
2
3456789012345678901234567890121234567890123456789012345678901212345
6
1
B. Altruism is a basic motivator of
3456789012345678901234567890121234567890123456789012345678901212345
6
12
motivators
should
be
possible
inputs
for
3456789012345678901234567890121234567890123456789012345678901212345
6
12
human
preference.
3456789012345678901234567890121234567890123456789012345678901212345
6
12
any
decision-matrix
schema
that
attempts
3456789012345678901234567890121234567890123456789012345678901212345
6
12
C.
Individuals
are
rational
in
decision2
3456789012345678901234567890121234567890123456789012345678901212345
6
1 (50) to model human preference
3456789012345678901234567890121234567890123456789012345678901212345
6
12
making.
3456789012345678901234567890121234567890123456789012345678901212345
6
12
determination.
3456789012345678901234567890121234567890123456789012345678901212345
6
12
D.
Altruism
is
an
irrational
predicate.
3456789012345678901234567890121234567890123456789012345678901212345 6
12
2
3456789012345678901234567890121234567890123456789012345678901212345
1
E. Inputs should be expanded to include 6
3456789012345678901234567890121234567890123456789012345678901212345
6
12
3456789012345678901234567890121234567890123456789012345678901212345
6
12
self-interest motivators.
3456789012345678901234567890121234567890123456789012345678901212345 6
12
3456789012345678901234567890121234567890123456789012345678901212345
6
12
28. The intent of the author in writing this
3456789012345678901234567890121234567890123456789012345678901212345
6
12
30.
The
second
paragraph
in
the
passage
3456789012345678901234567890121234567890123456789012345678901212345
6
12
passage
is
to
3456789012345678901234567890121234567890123456789012345678901212345 6
12
3456789012345678901234567890121234567890123456789012345678901212345
12
A. elaborates on the issues introduced in 6
3456789012345678901234567890121234567890123456789012345678901212345
6
12
A. offer a radical critique of contempo2
3456789012345678901234567890121234567890123456789012345678901212345
6
123456789012345678901234567890121234567890123456789012345678901212345
the first paragraph.
6
1
rary Theoretical Economics.
3456789012345678901234567890121234567890123456789012345678901212345
6
12
B.
presents
views
in
contrast
to
those
B. explain the basic mechanisms of
3456789012345678901234567890121234567890123456789012345678901212345
6
12
3456789012345678901234567890121234567890123456789012345678901212345
6
12
introduced
in
the
first
paragraph.
modeling
public
choice
scenarios.
2
3456789012345678901234567890121234567890123456789012345678901212345
123456789012345678901234567890121234567890123456789012345678901212345 66
123456789012345678901234567890121234567890123456789012345678901212345
C. presents the thesis of the passage.
C. contrast two competing views on
6
123456789012345678901234567890121234567890123456789012345678901212345
6
123456789012345678901234567890121234567890123456789012345678901212345
D.
argues
for
the
validity
of
the
thesis
of
decision-matrix
construction.
123456789012345678901234567890121234567890123456789012345678901212345 66
123456789012345678901234567890121234567890123456789012345678901212345
the passage.
D. explain why in public choice scenarios
6
123456789012345678901234567890121234567890123456789012345678901212345
6
123456789012345678901234567890121234567890123456789012345678901212345
E.
describes the intellectual issues
decision-matrices
should
include
123456789012345678901234567890121234567890123456789012345678901212345 66
1
pertaining to the thesis of the passage.
altruistic motivators.
3456789012345678901234567890121234567890123456789012345678901212345
6
12
3456789012345678901234567890121234567890123456789012345678901212345
6
12
E.
criticize
the
assumption
that
society
is
123456789012345678901234567890121234567890123456789012345678901212345 6
6
123456789012345678901234567890121234567890123456789012345678901212345
composed of aggregates of rational
3456789012345678901234567890121234567890123456789012345678901212345
6
12
2
3456789012345678901234567890121234567890123456789012345678901212345
6
123456789012345678901234567890121234567890123456789012345678901212345
decision-makers.
6
123456789012345678901234567890121234567890123456789012345678901212345
6
1
3456789012345678901234567890121234567890123456789012345678901212345 6
12
3456789012345678901234567890121234567890123456789012345678901212345 6
12
3456789012345678901234567890121234567890123456789012345678901212345 6
12
3456789012345678901234567890121234567890123456789012345678901212345 6
12
123456789012345678901234567890121234567890123456789012345678901212345 6
123456789012345678901234567890121234567890123456789012345678901212345 6
123456789012345678901234567890121234567890123456789012345678901212345 6
3456789012345678901234567890121234567890123456789012345678901212345 6
12
3456789012345678901234567890121234567890123456789012345678901212345 6
12
3456789012345678901234567890121234567890123456789012345678901212345 6
12
3456789012345678901234567890121234567890123456789012345678901212345 6
12
123456789012345678901234567890121234567890123456789012345678901212345 6
2
6
123456789012345678901234567890121234567890123456789012345678901212345
6
123456789012345678901234567890121234567890123456789012345678901212345
6
123456789012345678901234567890121234567890123456789012345678901212345
6
123456789012345678901234567890121234567890123456789012345678901212345
1 3456789012345678901234567890121234567890123456789012345678901212345 6
2
3456789012345678901234567890121234567890123456789012345678901212345
123456789012345678901234567890121234567890123456789012345678901212345 66
123456789012345678901234567890121234567890123456789012345678901212345 6
123456789012345678901234567890121234567890123456789012345678901212345 6
123456789012345678901234567890121234567890123456789012345678901212345 6
1
3456789012345678901234567890121234567890123456789012345678901212345 6
12
123456789012345678901234567890121234567890123456789012345678901212345 6
3456789012345678901234567890121234567890123456789012345678901212345 6
12
123456789012345678901234567890121234567890123456789012345678901212345 6
3456789012345678901234567890121234567890123456789012345678901212345 6
12
123456789012345678901234567890121234567890123456789012345678901212345 6
123456789012345678901234567890121234567890123456789012345678901212345 6
123456789012345678901234567890121234567890123456789012345678901212345 6
123456789012345678901234567890121234567890123456789012345678901212345 6 293
1234567890123456789012345678901212345678901234567890123456789012123456

http://www.xtremepapers.net

Answers and Explanations


123456789012345678901234567890121234567890123456789012
2
12345678901234567890123456789012123456789012345678901
2
12345678901234567890123456789012123456789012345678901
As
before,
the
following
notes
are
intended
to
show
you
one
way
in
which
2345678901234567890123456789012123456789012345678901
2
1
2
12345678901234567890123456789012123456789012345678901
the correct answer can be determined. Try to concern yourself with the 2
12345678901234567890123456789012123456789012345678901
2
12345678901234567890123456789012123456789012345678901
methods used to arrive at the right answer, and not focus all your attention 2
12345678901234567890123456789012123456789012345678901
2345678901234567890123456789012123456789012345678901
2
12345678901234567890123456789012123456789012345678901
on whether you got the question right or wrong.
2
12345678901234567890123456789012123456789012345678901
12345678901234567890123456789012123456789012345678901 2
12345678901234567890123456789012123456789012345678901 2
12345678901234567890123456789012123456789012345678901 2
12345678901234567890123456789012123456789012345678901 2
2
1Quantitative
2
12345678901234567890123456789012123456789012345678901
2
12345678901234567890123456789012123456789012345678901
1. B The area of a parallelogram is base times height (like with a 2
12345678901234567890123456789012123456789012345678901
2345678901234567890123456789012123456789012345678901
2
12345678901234567890123456789012123456789012345678901
triangle). The obvious base to choose is ED and then attempt to 2
12345678901234567890123456789012123456789012345678901
2
12345678901234567890123456789012123456789012345678901
find its height, or AE. The triangle is a 45-45-90 triangle, and 2
12345678901234567890123456789012123456789012345678901
12345678901234567890123456789012123456789012345678901 2
12345678901234567890123456789012123456789012345678901
since the hypotenuse is 2=2, the other two sides are 2. So the 2
2
12345678901234567890123456789012123456789012345678901
2
12345678901234567890123456789012123456789012345678901
height is 2 and the base is 5, which makes the area 10, or (B).
12345678901234567890123456789012123456789012345678901 2
1 2. B Here, you just need to be careful and work through the problem 2
2
12345678901234567890123456789012123456789012345678901
2
12345678901234567890123456789012123456789012345678901
step by step.
2
12345678901234567890123456789012123456789012345678901
12345678901234567890123456789012123456789012345678901 2
2
12345678901234567890123456789012123456789012345678901
&(2) 5 [2 2 (2 21)](2 1 2) 5 (2 2 1)(4) 5 (1)(4) 5 4,
2
12345678901234567890123456789012123456789012345678901
2
12345678901234567890123456789012123456789012345678901
2
12345678901234567890123456789012123456789012345678901
and
now
plug
this
result
in:
2
12345678901234567890123456789012123456789012345678901
2
12345678901234567890123456789012123456789012345678901
2345678901234567890123456789012123456789012345678901
2
12345678901234567890123456789012123456789012345678901
&(&(2)) 5 &(4) 5 [4 2 (4 2 1)](4 1 2)
2
12345678901234567890123456789012123456789012345678901
2
1
5 (4 2 3)(6)
2
12345678901234567890123456789012123456789012345678901
12345678901234567890123456789012123456789012345678901 2
2345678901234567890123456789012123456789012345678901
2
12345678901234567890123456789012123456789012345678901
5 (1)(6)
2
12345678901234567890123456789012123456789012345678901
2
12345678901234567890123456789012123456789012345678901
5
6
12345678901234567890123456789012123456789012345678901 2
2
1
2
12345678901234567890123456789012123456789012345678901
Choice (B) is the answer.
2
12345678901234567890123456789012123456789012345678901
2
12345678901234567890123456789012123456789012345678901
3. C You can solve this one quickly if you realize that the stock price 2
12345678901234567890123456789012123456789012345678901
2
12345678901234567890123456789012123456789012345678901
increased and that it did so more than 10 percent. If you do not 2
12345678901234567890123456789012123456789012345678901
2
12345678901234567890123456789012123456789012345678901
see this right off the bat, you can always just work the equation 2
12345678901234567890123456789012123456789012345678901
2
12345678901234567890123456789012123456789012345678901
for percentage change to get the answer:
2
12345678901234567890123456789012123456789012345678901
2345678901234567890123456789012123456789012345678901
12345678901234567890123456789012123456789012345678901 2
2
1
5
1
2
12345678901234567890123456789012123456789012345678901
100%
5
100%
5
12.5%
~
!
~
!
2
12345678901234567890123456789012123456789012345678901
40
8
2
12345678901234567890123456789012123456789012345678901
2345678901234567890123456789012123456789012345678901
2
1
The answer is (C).
2
12345678901234567890123456789012123456789012345678901
12345678901234567890123456789012123456789012345678901 2
12345678901234567890123456789012123456789012345678901 2
123456789012345678901234567890121234567890123456789012

S D

SD

294

http://www.xtremepapers.net

ANSWERS
123456789012345678901234567890121234567890123456789012
12345678901234567890123456789012123456789012345678901 2
2
12345678901234567890123456789012123456789012345678901
4. B You need to have an equation that has only the variables a and c 2
12345678901234567890123456789012123456789012345678901
2
12345678901234567890123456789012123456789012345678901
in it. If you multiply both sides of 4a 1 3d 5 18 by two we get 2
12345678901234567890123456789012123456789012345678901
2
12345678901234567890123456789012123456789012345678901
8a 1 6d 5 36. You can subtract this from the other equation to 2
12345678901234567890123456789012123456789012345678901
2345678901234567890123456789012123456789012345678901
2
1
eliminate the d variable; (3c 1 6d 5 24) 2 (8a 1 6d 5 36) yields 2
12345678901234567890123456789012123456789012345678901
2
12345678901234567890123456789012123456789012345678901
3c 2 8a 5 212. Now you only need to solve for a:
2
12345678901234567890123456789012123456789012345678901
2
12345678901234567890123456789012123456789012345678901
2345678901234567890123456789012123456789012345678901
2
1
3
3
2
12345678901234567890123456789012123456789012345678901
c
1
,
which
is
(B).
3c
2
8a
5
212

8a
5
3c
1
12

a
5
2
12345678901234567890123456789012123456789012345678901
8
2
2
12345678901234567890123456789012123456789012345678901
2
12345678901234567890123456789012123456789012345678901
5.
D
The
extinction
rate
is
the
solid
line,
and
its
highest
value
is
its
2345678901234567890123456789012123456789012345678901
2
1
2
12345678901234567890123456789012123456789012345678901
highest peak. The highest peak is in the early Paleozoic era 2
12345678901234567890123456789012123456789012345678901
2
12345678901234567890123456789012123456789012345678901
approximately 560 million years ago.
2
12345678901234567890123456789012123456789012345678901
2345678901234567890123456789012123456789012345678901
2
1
2
12345678901234567890123456789012123456789012345678901
6.
B
The
extinction
rate
and
the
number
of
families
have
the
same
2
12345678901234567890123456789012123456789012345678901
2
12345678901234567890123456789012123456789012345678901
value
when
the
two
lines
cross.
You
are
looking,
then,
for
the
era
2
12345678901234567890123456789012123456789012345678901
2345678901234567890123456789012123456789012345678901
12345678901234567890123456789012123456789012345678901
in which the lines cross the most number of times. They do so 2
2
12345678901234567890123456789012123456789012345678901
2
12345678901234567890123456789012123456789012345678901
four
times
in
the
Paleozoic
era,
which
is
more
than
any
other,
12345678901234567890123456789012123456789012345678901 22
12345678901234567890123456789012123456789012345678901
making (B) the answer.
2
12345678901234567890123456789012123456789012345678901
2
1
12345678901234567890123456789012123456789012345678901
7. E You can eliminate the odd number right off the bat, because a 2
2
12345678901234567890123456789012123456789012345678901
12345678901234567890123456789012123456789012345678901
number has to be even in order to be divisible by 6. Now you 2
2345678901234567890123456789012123456789012345678901
2
12345678901234567890123456789012123456789012345678901
just have to go through the remaining choices one by one:
2
12345678901234567890123456789012123456789012345678901
12345678901234567890123456789012123456789012345678901 22
12345678901234567890123456789012123456789012345678901
1,024: Digits sum to 7, which is not a multiple of 3.
2
12345678901234567890123456789012123456789012345678901
12345678901234567890123456789012123456789012345678901 22
12345678901234567890123456789012123456789012345678901
1,506: Digits sum to 12, which is a multiple of 3 but not 9. 2
12345678901234567890123456789012123456789012345678901
12345678901234567890123456789012123456789012345678901 22
1
1,632: Digits sum to 12, and so same as previous.
2
12345678901234567890123456789012123456789012345678901
2
12345678901234567890123456789012123456789012345678901
12345678901234567890123456789012123456789012345678901
1,764: Digits sum to 18, which is a multiple of 3 and 9. For 2
12345678901234567890123456789012123456789012345678901 2
12345678901234567890123456789012123456789012345678901
6, the digits must sum to a multiple of 3 and the original 2
2
12345678901234567890123456789012123456789012345678901
number must be even.
2
12345678901234567890123456789012123456789012345678901
12345678901234567890123456789012123456789012345678901 2
2
12345678901234567890123456789012123456789012345678901
10 1 10 1 15 1 7 1 12 1 18 72
2
12345678901234567890123456789012123456789012345678901
5
5
12,
and
so
The
mean
is
8.
D
2345678901234567890123456789012123456789012345678901
2
12345678901234567890123456789012123456789012345678901
6
6
2
12345678901234567890123456789012123456789012345678901
2
1
a
5
12.
The
mode
is
10,
and
so
b
5
10.
As
for
the
median,
order
12345678901234567890123456789012123456789012345678901 2
12345678901234567890123456789012123456789012345678901
the numbers least to greatest {7, 10, 10, 12, 15, 18}. Since you 2
2
12345678901234567890123456789012123456789012345678901
12345678901234567890123456789012123456789012345678901
have an even number of numbers, the median is the average of 2
2
12345678901234567890123456789012123456789012345678901
12345678901234567890123456789012123456789012345678901
the two middle numbers. Thus, the median is 11, so c 5 11. The 2
12345678901234567890123456789012123456789012345678901 2
12345678901234567890123456789012123456789012345678901
range is simply the greatest number minus the least, which 2
2
12345678901234567890123456789012123456789012345678901
2
equals 11, so d 5 11. So, a . b , c 5 d, which is (D).
12345678901234567890123456789012123456789012345678901
2
12345678901234567890123456789012123456789012345678901
12345678901234567890123456789012123456789012345678901 2
2345678901234567890123456789012123456789012345678901
12345678901234567890123456789012123456789012345678901 22
12345678901234567890123456789012123456789012345678901 2
12345678901234567890123456789012123456789012345678901 2
12345678901234567890123456789012123456789012345678901 2
1
2
12345678901234567890123456789012123456789012345678901
12345678901234567890123456789012123456789012345678901 2
2
12345678901234567890123456789012123456789012345678901
12345678901234567890123456789012123456789012345678901 2
2
12345678901234567890123456789012123456789012345678901
12345678901234567890123456789012123456789012345678901 2
12345678901234567890123456789012123456789012345678901 2
12345678901234567890123456789012123456789012345678901 2
12345678901234567890123456789012123456789012345678901 2 295
123456789012345678901234567890121234567890123456789012

http://www.xtremepapers.net

Part IV: Three Practice GREs

123456789012345678901234567890121234567890123456789012
12345678901234567890123456789012123456789012345678901 2
2
12345678901234567890123456789012123456789012345678901
9. C To begin with, you know that the sides of square B are equal to 2
12345678901234567890123456789012123456789012345678901
2
12345678901234567890123456789012123456789012345678901
4 since its area is 16. If you can find the length of the 2
12345678901234567890123456789012123456789012345678901
2
12345678901234567890123456789012123456789012345678901
non-hypotenuse sides of triangle A, then you could find the 2
12345678901234567890123456789012123456789012345678901
2345678901234567890123456789012123456789012345678901
2
1
answer. Since you know the hypotenuse you can use the 2
12345678901234567890123456789012123456789012345678901
12345678901234567890123456789012123456789012345678901
Pythagorean theorem to find the other sides of the triangle; 2
2
12345678901234567890123456789012123456789012345678901
2
2
12345678901234567890123456789012123456789012345678901
2
2
2
2
2345678901234567890123456789012123456789012345678901
1
a 1 b 5 ~=72! 2a 5 72 a 5 36 a 5 6. The 2
2
12345678901234567890123456789012123456789012345678901
12345678901234567890123456789012123456789012345678901
length of the triangle side that B and C share is 6, which means 2
2
12345678901234567890123456789012123456789012345678901
that one of the sides of square C is 2 (since the B side is 4). 2
12345678901234567890123456789012123456789012345678901
2345678901234567890123456789012123456789012345678901
2
1
Therefore, the area of C is 4, or (C).
2
12345678901234567890123456789012123456789012345678901
2
12345678901234567890123456789012123456789012345678901
2
12345678901234567890123456789012123456789012345678901
10.
B
Eye
the
graph
to
see
which
of
the
four
options
changed
little
2
12345678901234567890123456789012123456789012345678901
2345678901234567890123456789012123456789012345678901
1
from its 1970 average to its 1990 average. Looking closely, you 2
2
12345678901234567890123456789012123456789012345678901
2
12345678901234567890123456789012123456789012345678901
can
see
that
the
average
production
of
corn
in
County
Y
was
the
2
12345678901234567890123456789012123456789012345678901
2
12345678901234567890123456789012123456789012345678901
same
in
1970
and
1990.
Thus,
the
percentage
change
is
0.
That
2345678901234567890123456789012123456789012345678901
12345678901234567890123456789012123456789012345678901 2
2
12345678901234567890123456789012123456789012345678901
must be the smallest, so the answer is (B).
2
12345678901234567890123456789012123456789012345678901
12345678901234567890123456789012123456789012345678901 2
12345678901234567890123456789012123456789012345678901
11. C First note that the graph is given in terms of bushels per acre. 2
2
12345678901234567890123456789012123456789012345678901
1
Reading the graph then, County Y in 1980 produced 15 bushels 2
2
12345678901234567890123456789012123456789012345678901
12345678901234567890123456789012123456789012345678901
of corn per acre. Thus, 10 acres on average would produce 150 2
2
12345678901234567890123456789012123456789012345678901
2
12345678901234567890123456789012123456789012345678901
bushels, so (C) is correct.
2
12345678901234567890123456789012123456789012345678901
2
12345678901234567890123456789012123456789012345678901
12. D In 1980 County Y produced 15 bushels of corn per acre on 2
12345678901234567890123456789012123456789012345678901
12345678901234567890123456789012123456789012345678901
average, and it produced 10 bushels of wheat on average. Thus, 2
2
12345678901234567890123456789012123456789012345678901
2
12345678901234567890123456789012123456789012345678901
the ratio is 15:10, or 3:2, which is answer choice (D).
2
12345678901234567890123456789012123456789012345678901
2
12345678901234567890123456789012123456789012345678901
2
12345678901234567890123456789012123456789012345678901
13.
C
It
is
best
to
write
the
possibilities
out
and
count
them
up.
In
the
2345678901234567890123456789012123456789012345678901
12345678901234567890123456789012123456789012345678901 2
12345678901234567890123456789012123456789012345678901
ones column the possibilities are: 2, 4, 6. In the tens, the 2
2
1
2
12345678901234567890123456789012123456789012345678901
possibilities
are:
22,
24,
26,
42,
44,
46,
62,
64,
66.
That
2
12345678901234567890123456789012123456789012345678901
2345678901234567890123456789012123456789012345678901
12345678901234567890123456789012123456789012345678901
exhausts the possibilities. You get 12 when you count them up, 2
2
12345678901234567890123456789012123456789012345678901
2
1
so (C) is the answer.
2
12345678901234567890123456789012123456789012345678901
2
12345678901234567890123456789012123456789012345678901
2345678901234567890123456789012123456789012345678901
14.
C
First,
the
sum
of
the
interior
angles
of
an
n-sided
polygon
is
12345678901234567890123456789012123456789012345678901 2
2
12345678901234567890123456789012123456789012345678901
(n 2 2)(180), which makes the sum of the interior angles of a 2
1
2
12345678901234567890123456789012123456789012345678901
5-sided polygon 540 degrees. You can find the sum of a 1 d by 2
12345678901234567890123456789012123456789012345678901
2345678901234567890123456789012123456789012345678901
2
12345678901234567890123456789012123456789012345678901
summing the two equations a 1 b 5 160 and d 2 b 5 30. The 2
12345678901234567890123456789012123456789012345678901
2
12345678901234567890123456789012123456789012345678901
end result is a 1 d 5 190. The remaining angles must sum to 2
12345678901234567890123456789012123456789012345678901
1
350 for the sum of the interior angles to be 540. The answer, 2
2
12345678901234567890123456789012123456789012345678901
2
12345678901234567890123456789012123456789012345678901
then,
is
(C).
2
12345678901234567890123456789012123456789012345678901
2
12345678901234567890123456789012123456789012345678901
2
12345678901234567890123456789012123456789012345678901
15.
B
The
sum
of
a
set
of
consecutive
integers
is
the
average
times
the
2345678901234567890123456789012123456789012345678901
12345678901234567890123456789012123456789012345678901 2
2
12345678901234567890123456789012123456789012345678901
55 1 15
2
12345678901234567890123456789012123456789012345678901
5
35.
The
number
of
number
of
terms.
The
average
is
2
12345678901234567890123456789012123456789012345678901
2
2
1
terms
is
(remember,
it
is
inclusive)
55
2
15
1
1
5
41.
Now
you
2345678901234567890123456789012123456789012345678901
12345678901234567890123456789012123456789012345678901 2
2
1
just have to multiply these together. Their product is 1,435, 2
12345678901234567890123456789012123456789012345678901
2
12345678901234567890123456789012123456789012345678901
or (B).
2
12345678901234567890123456789012123456789012345678901
2345678901234567890123456789012123456789012345678901
2
1
12345678901234567890123456789012123456789012345678901 2
12345678901234567890123456789012123456789012345678901 2
2
296 12345678901234567890123456789012123456789012345678901
123456789012345678901234567890121234567890123456789012

www.petersons.com

http://www.xtremepapers.net

ANSWERS
123456789012345678901234567890121234567890123456789012
12345678901234567890123456789012123456789012345678901 2
2
12345678901234567890123456789012123456789012345678901
16. D The circumference of the circle is 2p ' 6.28. The perimeter of 2
12345678901234567890123456789012123456789012345678901
2
12345678901234567890123456789012123456789012345678901
the square is just 4 times the length of the side, which is equal to 2
12345678901234567890123456789012123456789012345678901
2
12345678901234567890123456789012123456789012345678901
6. Column A is greater.
2
12345678901234567890123456789012123456789012345678901
2345678901234567890123456789012123456789012345678901
2
1
2
12345678901234567890123456789012123456789012345678901
17.
B
Here
you
just
need
to
work
through
simplifying
each
fraction.
2
12345678901234567890123456789012123456789012345678901
2
12345678901234567890123456789012123456789012345678901
2
12345678901234567890123456789012123456789012345678901
Column
A
Column
B
2345678901234567890123456789012123456789012345678901
2
1
2
12345678901234567890123456789012123456789012345678901
2
12345678901234567890123456789012123456789012345678901
1
1
2
1
1
2
2
12345678901234567890123456789012123456789012345678901
2
12345678901234567890123456789012123456789012345678901
3
4
3
5
12
15
2345678901234567890123456789012123456789012345678901
2
1
5
5
2
12345678901234567890123456789012123456789012345678901
4
2
2 2
2 1
2
12345678901234567890123456789012123456789012345678901
2
12345678901234567890123456789012123456789012345678901
12
15
3 4
5 3
2
12345678901234567890123456789012123456789012345678901
2345678901234567890123456789012123456789012345678901
2
1
2
12345678901234567890123456789012123456789012345678901
12
15
1
2
1
2
2
12345678901234567890123456789012123456789012345678901
5
5
5
5
5
1
2
12345678901234567890123456789012123456789012345678901
12
4
4
15
2
2
2
12345678901234567890123456789012123456789012345678901
2345678901234567890123456789012123456789012345678901
12345678901234567890123456789012123456789012345678901 22
12345678901234567890123456789012123456789012345678901
As you can see, Column B is greater.
2
12345678901234567890123456789012123456789012345678901
12345678901234567890123456789012123456789012345678901 22
12345678901234567890123456789012123456789012345678901
18. A Use the information given about the line segments to write down
2
12345678901234567890123456789012123456789012345678901
1
equations that might be helpful in comparing the two columns. 2
2
12345678901234567890123456789012123456789012345678901
12345678901234567890123456789012123456789012345678901
AB 5 CD yields: r 1 s 5 s 1 3t r 5 3t. You can plug this 2
2
12345678901234567890123456789012123456789012345678901
12345678901234567890123456789012123456789012345678901
directly into BC 5 2t 1 3r 5 2t 1 3(3t) 5 11t. You can infer 2
2
12345678901234567890123456789012123456789012345678901
that both r and t are positive numbers since they only differ by a 2
12345678901234567890123456789012123456789012345678901
2
12345678901234567890123456789012123456789012345678901
multiple of three and at least one of them has to be positive since 2
12345678901234567890123456789012123456789012345678901
2345678901234567890123456789012123456789012345678901
2
12345678901234567890123456789012123456789012345678901
BC, a distance, is positive. Column A is greater.
2
12345678901234567890123456789012123456789012345678901
12345678901234567890123456789012123456789012345678901 22
12345678901234567890123456789012123456789012345678901
2
1 19. D To begin, you need to solve the inequality for x, 22x . 2 x , 21.
12345678901234567890123456789012123456789012345678901
You know x is less than 21. What does that tell you about x2? 2
2
12345678901234567890123456789012123456789012345678901
2
12345678901234567890123456789012123456789012345678901
Not
much,
certainly
nothing
definite
in
relation
to
2
since
x
could
12345678901234567890123456789012123456789012345678901 2
2
12345678901234567890123456789012123456789012345678901
be 21.1 or it could be 210. Thus, (D) is correct.
2
12345678901234567890123456789012123456789012345678901
2
12345678901234567890123456789012123456789012345678901
12345678901234567890123456789012123456789012345678901
20. D The circumference of the circle has to be something around 7p 2
2
12345678901234567890123456789012123456789012345678901
12345678901234567890123456789012123456789012345678901
' 22. The perimeter of the rectangle, on the other hand, is not 2
2345678901234567890123456789012123456789012345678901
12345678901234567890123456789012123456789012345678901 22
12345678901234567890123456789012123456789012345678901
definite. The sides could be 2 and 12, and so the perimeter
2
1
12345678901234567890123456789012123456789012345678901
would be 28, or the sides could be 4 and 6 and the perimeter 2
2
12345678901234567890123456789012123456789012345678901
would be 20. In one instance Column A is greater and in the 2
12345678901234567890123456789012123456789012345678901
2
12345678901234567890123456789012123456789012345678901
other Column B is greater.
2
12345678901234567890123456789012123456789012345678901
2
12345678901234567890123456789012123456789012345678901
2
12345678901234567890123456789012123456789012345678901
21.
B
The
first
thing
to
do
is
determine
the
value
of
z.
The
1-digit
even
2345678901234567890123456789012123456789012345678901
12345678901234567890123456789012123456789012345678901 22
12345678901234567890123456789012123456789012345678901
integers are 2, 4, 6, and 8. The square root of z is also an integer. 2
12345678901234567890123456789012123456789012345678901
12345678901234567890123456789012123456789012345678901
That is only true of 4. Thus, z equals 4. Four squared is 16, and 2
2
1
2345678901234567890123456789012123456789012345678901
2
12345678901234567890123456789012123456789012345678901
three times 4 is 12. Thus, Column B is greater.
2
12345678901234567890123456789012123456789012345678901
12345678901234567890123456789012123456789012345678901 22
12345678901234567890123456789012123456789012345678901 2
1
2
12345678901234567890123456789012123456789012345678901
12345678901234567890123456789012123456789012345678901 2
2
12345678901234567890123456789012123456789012345678901
12345678901234567890123456789012123456789012345678901 2
2
12345678901234567890123456789012123456789012345678901
12345678901234567890123456789012123456789012345678901 2
12345678901234567890123456789012123456789012345678901 2
12345678901234567890123456789012123456789012345678901 2
12345678901234567890123456789012123456789012345678901 2 297
123456789012345678901234567890121234567890123456789012

S DS D
S DS D

S DS D
S DS D

http://www.xtremepapers.net

Part IV: Three Practice GREs

123456789012345678901234567890121234567890123456789012
12345678901234567890123456789012123456789012345678901 2
2
12345678901234567890123456789012123456789012345678901
22. B Five percent simple annual interest means that after one years 2
12345678901234567890123456789012123456789012345678901
2
12345678901234567890123456789012123456789012345678901
time, 5 percent of interest will have accrued. After eight months 2
12345678901234567890123456789012123456789012345678901
2
12345678901234567890123456789012123456789012345678901
(which is two thirds of a year) three fourths of the interest will 2
12345678901234567890123456789012123456789012345678901
2345678901234567890123456789012123456789012345678901
2
1
have accrued. You need to multiply the 16,000 times 5 percent 2
12345678901234567890123456789012123456789012345678901
2
12345678901234567890123456789012123456789012345678901
2
2
12345678901234567890123456789012123456789012345678901
16,000
0.05
5
533.33.
times
three
fourths,
which
is
~
!~
!
2
12345678901234567890123456789012123456789012345678901
3
2345678901234567890123456789012123456789012345678901
2
1
2
12345678901234567890123456789012123456789012345678901
Column
B
is
greater,
so
the
answer
is
(B).
2
12345678901234567890123456789012123456789012345678901
2
12345678901234567890123456789012123456789012345678901
2
12345678901234567890123456789012123456789012345678901
3
a 3
2345678901234567890123456789012123456789012345678901
1 23. C You know that 5 a 5 b, and that the area of a sector is 2
2
12345678901234567890123456789012123456789012345678901
b 4
4
2
2
12345678901234567890123456789012123456789012345678901
.
For
Column
the
sector
angle
measure
divided
by
360
times
pr
2
12345678901234567890123456789012123456789012345678901
2
12345678901234567890123456789012123456789012345678901
A,
the
area
of
sector
1
is.
2345678901234567890123456789012123456789012345678901
2
1
2
12345678901234567890123456789012123456789012345678901
a
3
b
2
12345678901234567890123456789012123456789012345678901
2
2
2
12345678901234567890123456789012123456789012345678901
pr ! 5
pr !
~
~
2
12345678901234567890123456789012123456789012345678901
360
4 360
2345678901234567890123456789012123456789012345678901
12345678901234567890123456789012123456789012345678901 2
2
12345678901234567890123456789012123456789012345678901
For Column B, you need the area of the sector 2 times 0.75, 2
12345678901234567890123456789012123456789012345678901
2
12345678901234567890123456789012123456789012345678901
which is
2
12345678901234567890123456789012123456789012345678901
12345678901234567890123456789012123456789012345678901 2
2
1
b
3
b
2
12345678901234567890123456789012123456789012345678901
0.75!
pr2!5
pr2!
~
~
~
2
12345678901234567890123456789012123456789012345678901
360
4 360
2
12345678901234567890123456789012123456789012345678901
2345678901234567890123456789012123456789012345678901
2
12345678901234567890123456789012123456789012345678901
Column A and B are equal and (C) is the answer.
2
12345678901234567890123456789012123456789012345678901
12345678901234567890123456789012123456789012345678901 2
12345678901234567890123456789012123456789012345678901
24. C Remember the factors of a number multiply to give that number 2
2
12345678901234567890123456789012123456789012345678901
12345678901234567890123456789012123456789012345678901
and the multiples of a number are that number times the 2
2
12345678901234567890123456789012123456789012345678901
2
12345678901234567890123456789012123456789012345678901
integers.
For
Column
A,
begin
by
checking
to
see
if
120
is
evenly
12345678901234567890123456789012123456789012345678901 2
1
divisible by 24 because that would clearly make 24 the greatest 2
2
12345678901234567890123456789012123456789012345678901
2
12345678901234567890123456789012123456789012345678901
common
factor.
Five
times
24
gives
120
and
so
24
is
the
greatest
12345678901234567890123456789012123456789012345678901 2
12345678901234567890123456789012123456789012345678901
common factor. For Column B, if the number does not quickly 2
2
12345678901234567890123456789012123456789012345678901
2345678901234567890123456789012123456789012345678901
12345678901234567890123456789012123456789012345678901
come to mind then, write out multiples of each to find the least 2
2
12345678901234567890123456789012123456789012345678901
2
1
common
multiple.
The
LCM
is
24
so
the
answer
is
again
(C).
2
12345678901234567890123456789012123456789012345678901
2
12345678901234567890123456789012123456789012345678901
2
2
2345678901234567890123456789012123456789012345678901
12345678901234567890123456789012123456789012345678901
25. D Column A is equivalent to m 1 2mn 1 n and Column B is 2
2
12345678901234567890123456789012123456789012345678901
equivalent to m2 1 mn 2 2n2. You have to compare these two 2
1
2
12345678901234567890123456789012123456789012345678901
expressions. Since the first term in each is the same, you can 2
12345678901234567890123456789012123456789012345678901
2345678901234567890123456789012123456789012345678901
2
12345678901234567890123456789012123456789012345678901
eliminate them from consideration. If n equals zero then both 2
12345678901234567890123456789012123456789012345678901
2
12345678901234567890123456789012123456789012345678901
expressions are zero and so equal, but if n is non-zero then there 2
12345678901234567890123456789012123456789012345678901
1
certainly are situations when the two expressions are unequal. 2
2
12345678901234567890123456789012123456789012345678901
2
12345678901234567890123456789012123456789012345678901
Thus,
the
answer
is
(D).
2
12345678901234567890123456789012123456789012345678901
2
12345678901234567890123456789012123456789012345678901
2
12345678901234567890123456789012123456789012345678901
26.
C
First
solve
the
equation
to
see
if
you
can
glean
any
useful
2345678901234567890123456789012123456789012345678901
12345678901234567890123456789012123456789012345678901 2
2
12345678901234567890123456789012123456789012345678901
information about the relationship of x and y:
2
12345678901234567890123456789012123456789012345678901
2
12345678901234567890123456789012123456789012345678901
2
2
2
2
2
1
x 2 4y 1 7 5 7 x 5 4y x 5 2y
2345678901234567890123456789012123456789012345678901
12345678901234567890123456789012123456789012345678901 2
2
1
If x 5 2y, then side b is equal to side c. This means that we can 2
12345678901234567890123456789012123456789012345678901
2
12345678901234567890123456789012123456789012345678901
eliminate c2 and b2 from the columns, and all we are left with is 2
12345678901234567890123456789012123456789012345678901
2345678901234567890123456789012123456789012345678901
2
1
a2 in each. Ergo, the answer is (C).
2
12345678901234567890123456789012123456789012345678901
12345678901234567890123456789012123456789012345678901 2
2
298 12345678901234567890123456789012123456789012345678901
123456789012345678901234567890121234567890123456789012

SD

S D

S DS D

S D

S DS D

www.petersons.com

http://www.xtremepapers.net

ANSWERS
123456789012345678901234567890121234567890123456789012
12345678901234567890123456789012123456789012345678901 2
2
12345678901234567890123456789012123456789012345678901
27. B The number of integers from D to C is simply C 2 D, but since 2
12345678901234567890123456789012123456789012345678901
2
12345678901234567890123456789012123456789012345678901
it is inclusive we must include D and so add one, C 2 D 1 1. 2
12345678901234567890123456789012123456789012345678901
2
12345678901234567890123456789012123456789012345678901
Now you just need to see if we can manipulate Column B so that 2
12345678901234567890123456789012123456789012345678901
2345678901234567890123456789012123456789012345678901
2
1
you can compare it to Column A.
2
12345678901234567890123456789012123456789012345678901
2
12345678901234567890123456789012123456789012345678901
2
12345678901234567890123456789012123456789012345678901
2
C 2 CD 1 2C C(C 2 D 1 2)
2
12345678901234567890123456789012123456789012345678901
2345678901234567890123456789012123456789012345678901
2
1
5
5C2D12
2
12345678901234567890123456789012123456789012345678901
4C 2 3C
C
2
12345678901234567890123456789012123456789012345678901
2
12345678901234567890123456789012123456789012345678901
You
can
now
directly
compare
Column
A
and
B,
and
Column
B
2
12345678901234567890123456789012123456789012345678901
2345678901234567890123456789012123456789012345678901
1
is greater since they share the C minus D term but Column B 2
2
12345678901234567890123456789012123456789012345678901
2
12345678901234567890123456789012123456789012345678901
is
12.
2
12345678901234567890123456789012123456789012345678901
2
12345678901234567890123456789012123456789012345678901
2345678901234567890123456789012123456789012345678901
1 28. C You might suspect (D) here, but look closely at the number set. 2
2
12345678901234567890123456789012123456789012345678901
The only way that y and y 1 3 could have the two intervening 2
12345678901234567890123456789012123456789012345678901
2
12345678901234567890123456789012123456789012345678901
numbers in the set is if they are all consecutive integers:
2
12345678901234567890123456789012123456789012345678901
2345678901234567890123456789012123456789012345678901
12345678901234567890123456789012123456789012345678901 22
12345678901234567890123456789012123456789012345678901
(x 1 4) 5 (y) 1 1
2
12345678901234567890123456789012123456789012345678901
2
12345678901234567890123456789012123456789012345678901
(z) 5 (y) 1 2
2
12345678901234567890123456789012123456789012345678901
12345678901234567890123456789012123456789012345678901 22
1
Solve for x and z in terms of y:
2
12345678901234567890123456789012123456789012345678901
2
12345678901234567890123456789012123456789012345678901
2
12345678901234567890123456789012123456789012345678901
x
5
y
1
1
2
4
5
y
2
3
2345678901234567890123456789012123456789012345678901
12345678901234567890123456789012123456789012345678901 22
12345678901234567890123456789012123456789012345678901
z5y12
2
12345678901234567890123456789012123456789012345678901
12345678901234567890123456789012123456789012345678901 22
12345678901234567890123456789012123456789012345678901
So, Column A, z 2 5, must equal (y 1 2) 2 5, or y 2 3. Since 2
12345678901234567890123456789012123456789012345678901
12345678901234567890123456789012123456789012345678901
x 5 y 2 3, both columns are equal, and the correct answer 2
2
12345678901234567890123456789012123456789012345678901
2
12345678901234567890123456789012123456789012345678901
is
(C).
2
1
2345678901234567890123456789012123456789012345678901
12345678901234567890123456789012123456789012345678901 22
12345678901234567890123456789012123456789012345678901 2
1
2
12345678901234567890123456789012123456789012345678901
Verbal
2
12345678901234567890123456789012123456789012345678901
2345678901234567890123456789012123456789012345678901
12345678901234567890123456789012123456789012345678901 22
12345678901234567890123456789012123456789012345678901
2
1 Antonyms
2
12345678901234567890123456789012123456789012345678901
2
12345678901234567890123456789012123456789012345678901
1.
B
To
be
pragmatic
is
to
focus
on
the
practical.
The
opposite
of
this
2345678901234567890123456789012123456789012345678901
12345678901234567890123456789012123456789012345678901 22
12345678901234567890123456789012123456789012345678901
is something like idealistic. Looking through the answer choices,
2
1
12345678901234567890123456789012123456789012345678901
(B), quixotic, fits the pre-guess. If you are unfamiliar with this 2
12345678901234567890123456789012123456789012345678901 2
12345678901234567890123456789012123456789012345678901
term, pick up a copy of Cervantes literary classic Don Quixote. 2
2
12345678901234567890123456789012123456789012345678901
2
12345678901234567890123456789012123456789012345678901
2. D A cogent argument is compelling and well-constructed. You are 2
12345678901234567890123456789012123456789012345678901
2
12345678901234567890123456789012123456789012345678901
looking for the opposite of this. Unintelligent might attract you, 2
12345678901234567890123456789012123456789012345678901
2
12345678901234567890123456789012123456789012345678901
but unconvincing is much nearer the mark.
2
12345678901234567890123456789012123456789012345678901
2
12345678901234567890123456789012123456789012345678901
2
12345678901234567890123456789012123456789012345678901
3.
A
To
be
obdurate
about
an
issue
is
to
stubbornly
maintain
your
2345678901234567890123456789012123456789012345678901
12345678901234567890123456789012123456789012345678901 22
12345678901234567890123456789012123456789012345678901
position. Flexible or changeable is a good pre-guess for the 2
12345678901234567890123456789012123456789012345678901
12345678901234567890123456789012123456789012345678901
antonym. Choice (A) fits the pre-guess well because to be 2
2
1
2345678901234567890123456789012123456789012345678901
2
12345678901234567890123456789012123456789012345678901
persuadable is to be capable of change.
2
1
2
12345678901234567890123456789012123456789012345678901
12345678901234567890123456789012123456789012345678901
4. A Ebullient means to be boiling over with emotion. The opposite 2
2
12345678901234567890123456789012123456789012345678901
12345678901234567890123456789012123456789012345678901
of this is emotionless. Stern approximates our pre-guess, but 2
2
12345678901234567890123456789012123456789012345678901
2
12345678901234567890123456789012123456789012345678901
stoic hits it on the head.
12345678901234567890123456789012123456789012345678901 2 299
123456789012345678901234567890121234567890123456789012

http://www.xtremepapers.net

Part IV: Three Practice GREs

123456789012345678901234567890121234567890123456789012
12345678901234567890123456789012123456789012345678901 2
2
12345678901234567890123456789012123456789012345678901
5. E A garrulous person is extremely talkative, even inconsiderately 2
12345678901234567890123456789012123456789012345678901
2
12345678901234567890123456789012123456789012345678901
so. Thus, you are looking for an adjective that means not 2
12345678901234567890123456789012123456789012345678901
2
12345678901234567890123456789012123456789012345678901
talkative. Decorious means to act with decorum, or good 2
12345678901234567890123456789012123456789012345678901
2345678901234567890123456789012123456789012345678901
2
1
manners, and so might seem attractive, but taciturn is a better 2
12345678901234567890123456789012123456789012345678901
2
12345678901234567890123456789012123456789012345678901
choice. It means tight-lipped.
2
12345678901234567890123456789012123456789012345678901
2
12345678901234567890123456789012123456789012345678901
2345678901234567890123456789012123456789012345678901
1 6. B To enervate is to weaken the strength of something. The 2
2
12345678901234567890123456789012123456789012345678901
2
12345678901234567890123456789012123456789012345678901
opposite
is
to
make
stronger.
Looking
through
the
answer
2
12345678901234567890123456789012123456789012345678901
2
12345678901234567890123456789012123456789012345678901
choices,
(B)
obviously
jumps
out.
It
is
correct.
2345678901234567890123456789012123456789012345678901
2
1
2
12345678901234567890123456789012123456789012345678901
12345678901234567890123456789012123456789012345678901
7. A To manumit someone is to free them from slavery. The antonym 2
2
12345678901234567890123456789012123456789012345678901
12345678901234567890123456789012123456789012345678901
is to make someone a slave. Enslave, choice (A), clearly fits the 2
2345678901234567890123456789012123456789012345678901
2
1
2
12345678901234567890123456789012123456789012345678901
pre-guess and it is correct.
2
12345678901234567890123456789012123456789012345678901
2
12345678901234567890123456789012123456789012345678901
8.
D
Adumbrate
means
to
foreshadow
vaguely,
and
so
its
opposite
2
12345678901234567890123456789012123456789012345678901
2345678901234567890123456789012123456789012345678901
2
12345678901234567890123456789012123456789012345678901
would be something like to clearly state. To make explicit, 2
12345678901234567890123456789012123456789012345678901
2
12345678901234567890123456789012123456789012345678901
choice (D), fits the pre-guess and is correct.
2
12345678901234567890123456789012123456789012345678901
12345678901234567890123456789012123456789012345678901 2
2
12345678901234567890123456789012123456789012345678901
1 9. E A panegyric is a speech of effusive praise. You are therefore 2
12345678901234567890123456789012123456789012345678901
looking for something like criticize. Obloquy means abusive 2
2
12345678901234567890123456789012123456789012345678901
2
12345678901234567890123456789012123456789012345678901
or
condemnatory
language.
That
certainly
is
the
opposite
of
2345678901234567890123456789012123456789012345678901
12345678901234567890123456789012123456789012345678901 2
2
12345678901234567890123456789012123456789012345678901
praise, and so (E) is the answer.
2
12345678901234567890123456789012123456789012345678901
12345678901234567890123456789012123456789012345678901 2
12345678901234567890123456789012123456789012345678901 2
2
12345678901234567890123456789012123456789012345678901
Analogies
2
12345678901234567890123456789012123456789012345678901
12345678901234567890123456789012123456789012345678901 2
2
12345678901234567890123456789012123456789012345678901
1 10. C The relationship between the stem words is one of definition, as 2
2345678901234567890123456789012123456789012345678901
12345678901234567890123456789012123456789012345678901
to eclipse is to overshadow. Going through the answer choices, 2
2
12345678901234567890123456789012123456789012345678901
2
1
to
set
is
not
to
rise;
to
outstrip
is
not
to
flail;
to
debauch
is
to
12345678901234567890123456789012123456789012345678901 2
12345678901234567890123456789012123456789012345678901
debase; to overcome is not necessarily to outstretch; to pervert is 2
2
12345678901234567890123456789012123456789012345678901
2
12345678901234567890123456789012123456789012345678901
certainly
not
to
correct.
12345678901234567890123456789012123456789012345678901 2
2
12345678901234567890123456789012123456789012345678901
12345678901234567890123456789012123456789012345678901
11. A Here the stem relationship is one of degree: To be emphatic is to 2
2345678901234567890123456789012123456789012345678901
2
12345678901234567890123456789012123456789012345678901
insist strongly. Choice (B) might seem appealing here, but be 2
12345678901234567890123456789012123456789012345678901
2
1
careful because it has the order of degree reversedto be very 2
12345678901234567890123456789012123456789012345678901
2
12345678901234567890123456789012123456789012345678901
absorbed is to be engrossed. (A) is the correct choiceto be 2
12345678901234567890123456789012123456789012345678901
2
12345678901234567890123456789012123456789012345678901
contentious is to argue excessively.
2
12345678901234567890123456789012123456789012345678901
2
12345678901234567890123456789012123456789012345678901
2
12345678901234567890123456789012123456789012345678901
12.
C
An
astronomer
studies
the
stars.
That
is
a
good
relationship.
2345678901234567890123456789012123456789012345678901
12345678901234567890123456789012123456789012345678901 2
12345678901234567890123456789012123456789012345678901
Applying it to the answer choices, an accountant keeps the 2
2
12345678901234567890123456789012123456789012345678901
2
12345678901234567890123456789012123456789012345678901
financial
records;
a
librarian
manages
books;
a
historian
studies
2
1
2345678901234567890123456789012123456789012345678901
12345678901234567890123456789012123456789012345678901
history; a police officer administers the laws; and a magician 2
2
12345678901234567890123456789012123456789012345678901
2
12345678901234567890123456789012123456789012345678901
performs
tricks.
12345678901234567890123456789012123456789012345678901 2
2
1
12345678901234567890123456789012123456789012345678901
13. D The relationship between the stem pair is that you inoculate to 2
2
12345678901234567890123456789012123456789012345678901
12345678901234567890123456789012123456789012345678901
protect against a disease. Of the answer choices, only (D), you 2
2
12345678901234567890123456789012123456789012345678901
defend to protect against an enemy, shares this relationship.
2
12345678901234567890123456789012123456789012345678901
2345678901234567890123456789012123456789012345678901
2
1
12345678901234567890123456789012123456789012345678901 2
12345678901234567890123456789012123456789012345678901 2
2
300 12345678901234567890123456789012123456789012345678901
123456789012345678901234567890121234567890123456789012

www.petersons.com

http://www.xtremepapers.net

ANSWERS
123456789012345678901234567890121234567890123456789012
12345678901234567890123456789012123456789012345678901 2
2
12345678901234567890123456789012123456789012345678901
14. C Halcyon means peaceful and calm, and so the relationship 2
12345678901234567890123456789012123456789012345678901
2
12345678901234567890123456789012123456789012345678901
between the stem words is one of definitionhalcyon means 2
12345678901234567890123456789012123456789012345678901
2
12345678901234567890123456789012123456789012345678901
peaceful. Choices (B) and (E) are antonyms not synonyms, and 2
12345678901234567890123456789012123456789012345678901
2345678901234567890123456789012123456789012345678901
2
1
(A) and (D) do not share a necessary relationship. Choice (C) 2
12345678901234567890123456789012123456789012345678901
12345678901234567890123456789012123456789012345678901
does share a necessary relationshipsurreptitious means clan- 2
2
12345678901234567890123456789012123456789012345678901
2
12345678901234567890123456789012123456789012345678901
destine.
Choice
(B)
is
there
to
catch
people
shopping
for
words
2345678901234567890123456789012123456789012345678901
2
1
2
12345678901234567890123456789012123456789012345678901
like
peaceful.
2
12345678901234567890123456789012123456789012345678901
2
12345678901234567890123456789012123456789012345678901
2
12345678901234567890123456789012123456789012345678901
15.
A
The
stem
words
are
oppositesa
desolate
place
is
the
opposite
2345678901234567890123456789012123456789012345678901
2
1
2
12345678901234567890123456789012123456789012345678901
of
a
verdant
place.
Choices
(B)
and
(C)
are
synonyms
and
so
do
2
12345678901234567890123456789012123456789012345678901
2
12345678901234567890123456789012123456789012345678901
not
fit
the
stem
relationship,
and
(D)
and
(E)
do
not
have
a
2
12345678901234567890123456789012123456789012345678901
2345678901234567890123456789012123456789012345678901
1
strong relationship (a tropical place is often humid, a technical 2
2
12345678901234567890123456789012123456789012345678901
12345678901234567890123456789012123456789012345678901
manual may or may not be detailed.) Choice (A) shares the stem 2
2
12345678901234567890123456789012123456789012345678901
12345678901234567890123456789012123456789012345678901
relationshipa destitute area is the opposite of an opulent area. 2
2345678901234567890123456789012123456789012345678901
12345678901234567890123456789012123456789012345678901 22
12345678901234567890123456789012123456789012345678901
16. D Here, the relationship is again one of definitiona harbinger 2
12345678901234567890123456789012123456789012345678901
2
12345678901234567890123456789012123456789012345678901
presages something (presage means to foretell, or portend). A 2
12345678901234567890123456789012123456789012345678901
2
12345678901234567890123456789012123456789012345678901
computer can calculate something; a television shows things 2
1
12345678901234567890123456789012123456789012345678901
that have been broadcast; a liaison does not necessarily 2
2
12345678901234567890123456789012123456789012345678901
2
12345678901234567890123456789012123456789012345678901
interrogate
someone;
a
messenger
relates
something;
and
a
2345678901234567890123456789012123456789012345678901
12345678901234567890123456789012123456789012345678901 22
12345678901234567890123456789012123456789012345678901
broker does not necessarily abscond with something.
2
12345678901234567890123456789012123456789012345678901
12345678901234567890123456789012123456789012345678901 22
12345678901234567890123456789012123456789012345678901
17. A Asseverate means to aver or affirm earnestly, and so the stem 2
12345678901234567890123456789012123456789012345678901
12345678901234567890123456789012123456789012345678901
relationship is to asseverate is to aver earnestly. Choice (A) 2
2
12345678901234567890123456789012123456789012345678901
12345678901234567890123456789012123456789012345678901
shares this relationshipto beseech is to request earnestly. Do 2
2
1
12345678901234567890123456789012123456789012345678901
any of the other answer pairs share it? Approach and accost 2
2
12345678901234567890123456789012123456789012345678901
12345678901234567890123456789012123456789012345678901
might, but the order of degree is reversedto accost is to 2
12345678901234567890123456789012123456789012345678901 2
2
12345678901234567890123456789012123456789012345678901
approach forcefully.
2
12345678901234567890123456789012123456789012345678901
2
12345678901234567890123456789012123456789012345678901
12345678901234567890123456789012123456789012345678901 2
2
12345678901234567890123456789012123456789012345678901
Sentence Completions
2
12345678901234567890123456789012123456789012345678901
2345678901234567890123456789012123456789012345678901
12345678901234567890123456789012123456789012345678901 22
12345678901234567890123456789012123456789012345678901
1 18. D The word but in the sentence is the key clue. It tells you that the 2
12345678901234567890123456789012123456789012345678901
first and second parts of the sentence are in contrast to each 2
2
12345678901234567890123456789012123456789012345678901
2345678901234567890123456789012123456789012345678901
12345678901234567890123456789012123456789012345678901
other. In the first part we are told that Mr. Thomas had earned 2
2
12345678901234567890123456789012123456789012345678901
12345678901234567890123456789012123456789012345678901
the reputation as a miser, and so in the second part you should 2
12345678901234567890123456789012123456789012345678901 22
1
expect to find a strong contrast to this. Which answer choice
2
12345678901234567890123456789012123456789012345678901
makes a strong contrast to miser? Choice (D), spendthrift, 2
12345678901234567890123456789012123456789012345678901
2
12345678901234567890123456789012123456789012345678901
which means one who spends money wastefully, fits the bill.
2
12345678901234567890123456789012123456789012345678901
12345678901234567890123456789012123456789012345678901 2
2345678901234567890123456789012123456789012345678901
12345678901234567890123456789012123456789012345678901
19. A Consider the first blank first. Shifts in the demographics of a 2
2
12345678901234567890123456789012123456789012345678901
2
12345678901234567890123456789012123456789012345678901
country
would
at
least
challenge
traditional
political
alliances
12345678901234567890123456789012123456789012345678901 22
1
and at most destroy them. Since the sentence speaks of nascent 2
12345678901234567890123456789012123456789012345678901
12345678901234567890123456789012123456789012345678901
political parties, you can infer that the traditional ones are at 2
2
12345678901234567890123456789012123456789012345678901
2
12345678901234567890123456789012123456789012345678901
least
being
challenged.
Solvent
and
elusive
are
not
good
answer
2
12345678901234567890123456789012123456789012345678901
2345678901234567890123456789012123456789012345678901
1
choices for the first blank. As the old political parties are being 2
2
12345678901234567890123456789012123456789012345678901
12345678901234567890123456789012123456789012345678901 22
12345678901234567890123456789012123456789012345678901 301
123456789012345678901234567890121234567890123456789012

http://www.xtremepapers.net

Part IV: Three Practice GREs

123456789012345678901234567890121234567890123456789012
12345678901234567890123456789012123456789012345678901 2
2
12345678901234567890123456789012123456789012345678901
challenged, the new ones then are gaining ground. Gain then is a 2
12345678901234567890123456789012123456789012345678901
2
12345678901234567890123456789012123456789012345678901
good pre-guess to the second blank. Provide does not match this 2
12345678901234567890123456789012123456789012345678901
2
12345678901234567890123456789012123456789012345678901
pre-guess. Between procure and secure, secure is the better 2
12345678901234567890123456789012123456789012345678901
2345678901234567890123456789012123456789012345678901
2
1
choice since procure intimates buying, and one does not 2
12345678901234567890123456789012123456789012345678901
2
12345678901234567890123456789012123456789012345678901
purchase large swathes of the electoral landscape.
2
12345678901234567890123456789012123456789012345678901
2
12345678901234567890123456789012123456789012345678901
2345678901234567890123456789012123456789012345678901
1 20. B Begin with a pre-guess on the first blank. What is a word that goes 2
2
12345678901234567890123456789012123456789012345678901
2
12345678901234567890123456789012123456789012345678901
with
public
and
can
be
persuaded?
Opinion.
Looking
down
the
2
12345678901234567890123456789012123456789012345678901
2
12345678901234567890123456789012123456789012345678901
first
blank
answer
choices,
(B)
obviously
matches
this
pre-guess,
2345678901234567890123456789012123456789012345678901
2
1
2
12345678901234567890123456789012123456789012345678901
but
(A)
and
possibly
(C)
and
(D)
do
as
well.
The
word
ostensibly
2
12345678901234567890123456789012123456789012345678901
2
12345678901234567890123456789012123456789012345678901
clues
you
to
the
fact
that
the
second
blank
is
a
strong
contrast
to
2
12345678901234567890123456789012123456789012345678901
2345678901234567890123456789012123456789012345678901
1
process of popular persuasion. Which of the answer choices 2
2
12345678901234567890123456789012123456789012345678901
12345678901234567890123456789012123456789012345678901
matches this conception? Undemocratic does. Thus, (B) is a strong 2
2
12345678901234567890123456789012123456789012345678901
12345678901234567890123456789012123456789012345678901
candidate for the best answer choice. Reading the other answer 2
2345678901234567890123456789012123456789012345678901
12345678901234567890123456789012123456789012345678901 2
12345678901234567890123456789012123456789012345678901
choices, (A) is the reasonable selection, but (B) is better since both 2
2
12345678901234567890123456789012123456789012345678901
answer blanks more specifically fit the logic of the sentence.
2
12345678901234567890123456789012123456789012345678901
12345678901234567890123456789012123456789012345678901 2
2
12345678901234567890123456789012123456789012345678901
1 21. D You are looking for a word that modifies feuds between rival 2
12345678901234567890123456789012123456789012345678901
factions of a company. Internal is a good pre-guess. Internecine 2
2
12345678901234567890123456789012123456789012345678901
2
12345678901234567890123456789012123456789012345678901
means
fighting
within
a
group,
and
so
matches
the
pre-guess.
It
2345678901234567890123456789012123456789012345678901
12345678901234567890123456789012123456789012345678901 2
2
12345678901234567890123456789012123456789012345678901
is the best choice, (D).
2
12345678901234567890123456789012123456789012345678901
12345678901234567890123456789012123456789012345678901 2
12345678901234567890123456789012123456789012345678901
22. B The sentence tells us that the first blank can be described as the 2
2
12345678901234567890123456789012123456789012345678901
2
12345678901234567890123456789012123456789012345678901
practice
of
appointing
friends
and
associates
to
high
posts.
A
12345678901234567890123456789012123456789012345678901 2
12345678901234567890123456789012123456789012345678901
good pre-guess would be favoritism. Looking at the answer 2
2
1
2345678901234567890123456789012123456789012345678901
12345678901234567890123456789012123456789012345678901
choices, (D) and (E) can definitely be eliminated. Choice (A) also 2
2
12345678901234567890123456789012123456789012345678901
1
can be eliminated because nepotism is showing favor to ones 2
12345678901234567890123456789012123456789012345678901 2
12345678901234567890123456789012123456789012345678901
own family, not to friends and associates. The second blank is a 2
2
12345678901234567890123456789012123456789012345678901
contrast to the first since the favoritism of the predecessors is 2
12345678901234567890123456789012123456789012345678901
2
12345678901234567890123456789012123456789012345678901
being eschewed in favor of the new way. Both (B) and (C) are 2
12345678901234567890123456789012123456789012345678901
2
12345678901234567890123456789012123456789012345678901
contrasts to favoritism, and so you must ask which one better 2
12345678901234567890123456789012123456789012345678901
2
12345678901234567890123456789012123456789012345678901
fits the flow of the sentence. Merit, based makes more sense than 2
12345678901234567890123456789012123456789012345678901
2
12345678901234567890123456789012123456789012345678901
work, based, and so (B) is a better choice.
2
12345678901234567890123456789012123456789012345678901
2345678901234567890123456789012123456789012345678901
12345678901234567890123456789012123456789012345678901 2
12345678901234567890123456789012123456789012345678901
23. E In this sentence, the authors customary style of correspondence 2
2
12345678901234567890123456789012123456789012345678901
2
12345678901234567890123456789012123456789012345678901
is
being
contrasted
with
the
style
in
her
most
recent
letter.
2
1
2345678901234567890123456789012123456789012345678901
12345678901234567890123456789012123456789012345678901
Customarily her style is deliberate, but in this letter it is 2
2
12345678901234567890123456789012123456789012345678901
2
12345678901234567890123456789012123456789012345678901
convoluted.
Therefore,
in
the
first
blank,
you
are
looking
for
a
12345678901234567890123456789012123456789012345678901 2
1
word that pairs well with deliberate and contrasts well with 2
2
12345678901234567890123456789012123456789012345678901
12345678901234567890123456789012123456789012345678901
convoluted. In the second blank, you are looking for a word that 2
2
12345678901234567890123456789012123456789012345678901
12345678901234567890123456789012123456789012345678901
pairs well with convoluted and contrasts well with deliberate. 2
12345678901234567890123456789012123456789012345678901 2
12345678901234567890123456789012123456789012345678901
Genuine, acidic, and urbane are not good pairs with deliberate 2
2
12345678901234567890123456789012123456789012345678901
12345678901234567890123456789012123456789012345678901
or contrasts with convoluted. As for the second blank, tiresome 2
2
12345678901234567890123456789012123456789012345678901
does not pair as well with convoluted as prolix does (which 2
12345678901234567890123456789012123456789012345678901
2345678901234567890123456789012123456789012345678901
2
1
means to be wordy). Choice (E) is best.
2
12345678901234567890123456789012123456789012345678901
12345678901234567890123456789012123456789012345678901 2
2
302 12345678901234567890123456789012123456789012345678901
123456789012345678901234567890121234567890123456789012

www.petersons.com

http://www.xtremepapers.net

ANSWERS
123456789012345678901234567890121234567890123456789012
12345678901234567890123456789012123456789012345678901 2
2
12345678901234567890123456789012123456789012345678901
Reading Comprehension
2
12345678901234567890123456789012123456789012345678901
2
12345678901234567890123456789012123456789012345678901
2
12345678901234567890123456789012123456789012345678901
24.
C
A
question
about
the
proper
title
of
the
passage
is
looking
for
12345678901234567890123456789012123456789012345678901 2
12345678901234567890123456789012123456789012345678901
the basic point or thrust of the passage. What is the point of this 2
2
12345678901234567890123456789012123456789012345678901
2
12345678901234567890123456789012123456789012345678901
passage?
It
relates
how
the
discovery
and
study
of
the
Archaeans
2
12345678901234567890123456789012123456789012345678901
2
12345678901234567890123456789012123456789012345678901
has
led
to
the
reformulation
of
central
concerns
in
the
field
of
2
12345678901234567890123456789012123456789012345678901
2345678901234567890123456789012123456789012345678901
1
biology. Choice (A) is too narrow because it excludes the 2
2
12345678901234567890123456789012123456789012345678901
2
12345678901234567890123456789012123456789012345678901
discussion
of
the
extremophiles.
Choice
(B),
although
better
2
12345678901234567890123456789012123456789012345678901
2
12345678901234567890123456789012123456789012345678901
than
(A),
is
still
too
narrow.
Choice
(C)
is
a
good
restatement
of
2345678901234567890123456789012123456789012345678901
2
1
2
12345678901234567890123456789012123456789012345678901
the
thrust
of
the
passage,
since
reformulation
of
central
concerns
2
12345678901234567890123456789012123456789012345678901
2
12345678901234567890123456789012123456789012345678901
in
biology
is
a
good
approximation
of
Challenge
to
Biological
2
12345678901234567890123456789012123456789012345678901
2345678901234567890123456789012123456789012345678901
1
Paradigms. Choices (D) and (E) are too narrow in that they 2
2
12345678901234567890123456789012123456789012345678901
12345678901234567890123456789012123456789012345678901
exclude the classification issues, and notice that the passage 2
2
12345678901234567890123456789012123456789012345678901
2
12345678901234567890123456789012123456789012345678901
discusses extremophiles and not thermophiles. (C)s the one.
2345678901234567890123456789012123456789012345678901
12345678901234567890123456789012123456789012345678901 22
12345678901234567890123456789012123456789012345678901
25. E On questions of this form you must go through each proposition 2
12345678901234567890123456789012123456789012345678901
2
12345678901234567890123456789012123456789012345678901
individually:
2
12345678901234567890123456789012123456789012345678901
12345678901234567890123456789012123456789012345678901 22
1
Option IThe passage states that prior to the discovery of 2
12345678901234567890123456789012123456789012345678901
12345678901234567890123456789012123456789012345678901
extremophiles the thermal threshold was set at 60 degrees 2
2
12345678901234567890123456789012123456789012345678901
2345678901234567890123456789012123456789012345678901
12345678901234567890123456789012123456789012345678901
centigrade because it was thought that the molecular 2
2
12345678901234567890123456789012123456789012345678901
2
12345678901234567890123456789012123456789012345678901
integrity
of
vital
cellular
components
could
not
be
main12345678901234567890123456789012123456789012345678901 22
12345678901234567890123456789012123456789012345678901
tained beyond such temperatures. It can be inferred that if 2
12345678901234567890123456789012123456789012345678901
12345678901234567890123456789012123456789012345678901
extremophiles can live and thrive past 60 degrees centigrade, 2
2
12345678901234567890123456789012123456789012345678901
12345678901234567890123456789012123456789012345678901
they can maintain molecular integrity past the previous 2
2
1
2
12345678901234567890123456789012123456789012345678901
thermal threshold.
2
12345678901234567890123456789012123456789012345678901
2
12345678901234567890123456789012123456789012345678901
Option IIThe passage does not mention extremophiles 2
12345678901234567890123456789012123456789012345678901
2
12345678901234567890123456789012123456789012345678901
existing in moderate environments, so II cannot be inferred 2
12345678901234567890123456789012123456789012345678901
2
12345678901234567890123456789012123456789012345678901
from the passage.
2
12345678901234567890123456789012123456789012345678901
2
12345678901234567890123456789012123456789012345678901
2
12345678901234567890123456789012123456789012345678901
Option
IIIThe
passage
states
that
extremophiles
can
live
2345678901234567890123456789012123456789012345678901
12345678901234567890123456789012123456789012345678901 22
12345678901234567890123456789012123456789012345678901
and thrive at 160 degrees and so it can be inferred that they 2
1
2
12345678901234567890123456789012123456789012345678901
can live and thrive in temperatures as hot as 150 degrees.
2
12345678901234567890123456789012123456789012345678901
2345678901234567890123456789012123456789012345678901
12345678901234567890123456789012123456789012345678901 22
12345678901234567890123456789012123456789012345678901
Since I and III are inferable, the answer is (E).
2
12345678901234567890123456789012123456789012345678901
12345678901234567890123456789012123456789012345678901 22
1 26. A The passage states that, kingdoms are classified to be a
2
12345678901234567890123456789012123456789012345678901
sub-group of a particular domain (lines 2021). Choice (A) 2
12345678901234567890123456789012123456789012345678901
2
12345678901234567890123456789012123456789012345678901
states exactly that.
2
12345678901234567890123456789012123456789012345678901
12345678901234567890123456789012123456789012345678901 2
2345678901234567890123456789012123456789012345678901
12345678901234567890123456789012123456789012345678901
27. B The passage uses thermal threshold to mean the temperature at 2
2
12345678901234567890123456789012123456789012345678901
2
12345678901234567890123456789012123456789012345678901
which
biological
organisms
begin
to
break
down
due
to
heat.
12345678901234567890123456789012123456789012345678901 22
1
Which of the answer choices matches this usage? Choice (A) 2
12345678901234567890123456789012123456789012345678901
12345678901234567890123456789012123456789012345678901
does not, and it uses terms not in the passage. Choice (B) 2
2
12345678901234567890123456789012123456789012345678901
2
12345678901234567890123456789012123456789012345678901
matches
up.
As
for
(C),
the
passage
clearly
uses
the
term
thermal
2
12345678901234567890123456789012123456789012345678901
2345678901234567890123456789012123456789012345678901
1
threshold to apply to all kinds of life and not just extremophiles, 2
2
12345678901234567890123456789012123456789012345678901
12345678901234567890123456789012123456789012345678901 22
12345678901234567890123456789012123456789012345678901 303
123456789012345678901234567890121234567890123456789012

http://www.xtremepapers.net

Part IV: Three Practice GREs

123456789012345678901234567890121234567890123456789012
12345678901234567890123456789012123456789012345678901 2
2
12345678901234567890123456789012123456789012345678901
and so this choice is out. Choice (D) uses terms not in the 2
12345678901234567890123456789012123456789012345678901
2
12345678901234567890123456789012123456789012345678901
passage and that should make you skeptical. Choice (E) is tricky 2
12345678901234567890123456789012123456789012345678901
2
12345678901234567890123456789012123456789012345678901
because the passage does speak of the former thermal threshold 2
12345678901234567890123456789012123456789012345678901
2345678901234567890123456789012123456789012345678901
2
1
as being thought of as a constant across nature. Yet, the 2
12345678901234567890123456789012123456789012345678901
12345678901234567890123456789012123456789012345678901
threshold changed when extremophiles were discovered. Choice 2
2
12345678901234567890123456789012123456789012345678901
2
12345678901234567890123456789012123456789012345678901
(B)
is
unquestionably
a
strong
answer,
and
(E)
is
problematic
2345678901234567890123456789012123456789012345678901
2
1
2
12345678901234567890123456789012123456789012345678901
because
the
threshold
has
fluctuated.
Therefore,
(B)
is
a
better
2
12345678901234567890123456789012123456789012345678901
2
12345678901234567890123456789012123456789012345678901
choice.
2
12345678901234567890123456789012123456789012345678901
2345678901234567890123456789012123456789012345678901
2
1
2
12345678901234567890123456789012123456789012345678901
28.
D
The
entire
passage
culminates
in
the
conclusion
of
the
author
in
2
12345678901234567890123456789012123456789012345678901
2
12345678901234567890123456789012123456789012345678901
the
last
sentence
of
the
passage,
Therefore,
altruistic
motiva2
12345678901234567890123456789012123456789012345678901
2345678901234567890123456789012123456789012345678901
1
tors should be possible inputs for any decision-matrix schema 2
2
12345678901234567890123456789012123456789012345678901
12345678901234567890123456789012123456789012345678901
that attempts to model human preference determination. The 2
2
12345678901234567890123456789012123456789012345678901
12345678901234567890123456789012123456789012345678901
passage builds to this conclusion and so the clear intent of the 2
2345678901234567890123456789012123456789012345678901
12345678901234567890123456789012123456789012345678901 2
12345678901234567890123456789012123456789012345678901
author is to make this point. Which answer choice reflects this 2
2
12345678901234567890123456789012123456789012345678901
intent? Choice (D) does.
2
12345678901234567890123456789012123456789012345678901
12345678901234567890123456789012123456789012345678901 2
2
12345678901234567890123456789012123456789012345678901
1 29. C To answer this question you have to go to the place in the 2
12345678901234567890123456789012123456789012345678901
passage where the assumptions of decision-matrices are dis- 2
2
12345678901234567890123456789012123456789012345678901
2
12345678901234567890123456789012123456789012345678901
cussed
(lines
3137).
The
passage
clearly
states
that
in
the
2345678901234567890123456789012123456789012345678901
12345678901234567890123456789012123456789012345678901 2
12345678901234567890123456789012123456789012345678901
construction of decision-matrices it is assumed that individuals 2
2
12345678901234567890123456789012123456789012345678901
2
12345678901234567890123456789012123456789012345678901
are
rational
in
decision-making.
The
author
does
not
dispute
12345678901234567890123456789012123456789012345678901 2
2
12345678901234567890123456789012123456789012345678901
this assumption. That is answer choice (C), and it is correct.
2
12345678901234567890123456789012123456789012345678901
12345678901234567890123456789012123456789012345678901 2
2
12345678901234567890123456789012123456789012345678901
1 30. A Looking at the structure of the passage, the first paragraph 2
2345678901234567890123456789012123456789012345678901
12345678901234567890123456789012123456789012345678901
presents the challenge of modeling public choice scenarios, the 2
2
12345678901234567890123456789012123456789012345678901
1
second paragraph explains the details of a particular methodol- 2
12345678901234567890123456789012123456789012345678901 2
12345678901234567890123456789012123456789012345678901
ogy for such modeling, and the third paragraph discusses 2
2
12345678901234567890123456789012123456789012345678901
problems with the methodology presented in the third para- 2
12345678901234567890123456789012123456789012345678901
2
12345678901234567890123456789012123456789012345678901
graph. Given this basic structure, we can inspect the answer 2
12345678901234567890123456789012123456789012345678901
2
12345678901234567890123456789012123456789012345678901
choices to see which best describes the role of the second 2
12345678901234567890123456789012123456789012345678901
2
12345678901234567890123456789012123456789012345678901
paragraph. Choice (A) matches the tone and content of our 2
12345678901234567890123456789012123456789012345678901
2
12345678901234567890123456789012123456789012345678901
description of the second passage, and it is correct.
2
12345678901234567890123456789012123456789012345678901
2345678901234567890123456789012123456789012345678901
12345678901234567890123456789012123456789012345678901 2
12345678901234567890123456789012123456789012345678901 2
12345678901234567890123456789012123456789012345678901 2
12345678901234567890123456789012123456789012345678901 2
2
1
2
12345678901234567890123456789012123456789012345678901
2
12345678901234567890123456789012123456789012345678901
2
12345678901234567890123456789012123456789012345678901
2
12345678901234567890123456789012123456789012345678901
12345678901234567890123456789012123456789012345678901 2
2345678901234567890123456789012123456789012345678901
12345678901234567890123456789012123456789012345678901 2
12345678901234567890123456789012123456789012345678901 2
12345678901234567890123456789012123456789012345678901 2
12345678901234567890123456789012123456789012345678901 2
2
1
2
12345678901234567890123456789012123456789012345678901
12345678901234567890123456789012123456789012345678901 2
2
12345678901234567890123456789012123456789012345678901
12345678901234567890123456789012123456789012345678901 2
2
12345678901234567890123456789012123456789012345678901
12345678901234567890123456789012123456789012345678901 2
12345678901234567890123456789012123456789012345678901 2
12345678901234567890123456789012123456789012345678901 2
2
304 12345678901234567890123456789012123456789012345678901
123456789012345678901234567890121234567890123456789012

www.petersons.com

http://www.xtremepapers.net

Practice Test

3
1234567890123456789012345678901212345678901234567890123456789012123456
3456789012345678901234567890121234567890123456789012345678901212345 6
12
3456789012345678901234567890121234567890123456789012345678901212345 6
12
3456789012345678901234567890121234567890123456789012345678901212345
6
12
Quantitative
2
6
123456789012345678901234567890121234567890123456789012345678901212345
6
123456789012345678901234567890121234567890123456789012345678901212345
6
123456789012345678901234567890121234567890123456789012345678901212345
28
Questions45
Minutes
6
123456789012345678901234567890121234567890123456789012345678901212345
6
123456789012345678901234567890121234567890123456789012345678901212345
General
Information
3456789012345678901234567890121234567890123456789012345678901212345
123456789012345678901234567890121234567890123456789012345678901212345 66
1
3456789012345678901234567890121234567890123456789012345678901212345
12
1. The test has 28 questions. If you decide to take this exam under timed conditions, you have 6
3456789012345678901234567890121234567890123456789012345678901212345
6
12
3456789012345678901234567890121234567890123456789012345678901212345
6
12
45 minutes to complete the section.
2
3456789012345678901234567890121234567890123456789012345678901212345
123456789012345678901234567890121234567890123456789012345678901212345 66
123456789012345678901234567890121234567890123456789012345678901212345
2. All numbers used are real numbers.
6
123456789012345678901234567890121234567890123456789012345678901212345
123456789012345678901234567890121234567890123456789012345678901212345 66
123456789012345678901234567890121234567890123456789012345678901212345
3. All angle measurements can be assumed to be positive unless otherwise noted.
6
123456789012345678901234567890121234567890123456789012345678901212345
123456789012345678901234567890121234567890123456789012345678901212345 66
123456789012345678901234567890121234567890123456789012345678901212345
4. All figures lie in the same plane unless otherwise noted.
6
123456789012345678901234567890121234567890123456789012345678901212345
6
1
2
3456789012345678901234567890121234567890123456789012345678901212345
123456789012345678901234567890121234567890123456789012345678901212345
5. Drawings that accompany questions are intended to provide information useful in 6
6
123456789012345678901234567890121234567890123456789012345678901212345
6
1
answering the question. The figures are drawn closely to scale unless otherwise noted.
123456789012345678901234567890121234567890123456789012345678901212345 6
3456789012345678901234567890121234567890123456789012345678901212345 6
12
3456789012345678901234567890121234567890123456789012345678901212345 6
12
3456789012345678901234567890121234567890123456789012345678901212345
6
12
Directions: For questions 114, Column A and Column B will have one quantity each.
6
123456789012345678901234567890121234567890123456789012345678901212345
3456789012345678901234567890121234567890123456789012345678901212345
6
12
Your goal is to compare the quantities in each column and choose
3456789012345678901234567890121234567890123456789012345678901212345 6
12
2
3456789012345678901234567890121234567890123456789012345678901212345
6
123456789012345678901234567890121234567890123456789012345678901212345
A if the quantity in Column A is greater
6
123456789012345678901234567890121234567890123456789012345678901212345
6
1
B if the quantity in Column B is greater
6
123456789012345678901234567890121234567890123456789012345678901212345
6
123456789012345678901234567890121234567890123456789012345678901212345
C
if
the
two
quantities
are
equal
2
3456789012345678901234567890121234567890123456789012345678901212345
123456789012345678901234567890121234567890123456789012345678901212345 66
123456789012345678901234567890121234567890123456789012345678901212345
D if the relationship between the two quantities cannot be determined from the
6
123456789012345678901234567890121234567890123456789012345678901212345
6
123456789012345678901234567890121234567890123456789012345678901212345
information
in
the
problem
6
1
2
3456789012345678901234567890121234567890123456789012345678901212345
123456789012345678901234567890121234567890123456789012345678901212345 66
123456789012345678901234567890121234567890123456789012345678901212345
On some questions, information about one or both quantities will be given. This informa6
123456789012345678901234567890121234567890123456789012345678901212345
6
123456789012345678901234567890121234567890123456789012345678901212345
tion
will
be
centered
above
the
two
columns.
6
1
2
3456789012345678901234567890121234567890123456789012345678901212345
123456789012345678901234567890121234567890123456789012345678901212345 66
123456789012345678901234567890121234567890123456789012345678901212345 6
123456789012345678901234567890121234567890123456789012345678901212345 6
123456789012345678901234567890121234567890123456789012345678901212345 6
1
3456789012345678901234567890121234567890123456789012345678901212345 6
12
123456789012345678901234567890121234567890123456789012345678901212345 6
3456789012345678901234567890121234567890123456789012345678901212345 6
12
123456789012345678901234567890121234567890123456789012345678901212345 6
3456789012345678901234567890121234567890123456789012345678901212345 6
12
123456789012345678901234567890121234567890123456789012345678901212345 6
123456789012345678901234567890121234567890123456789012345678901212345 6
123456789012345678901234567890121234567890123456789012345678901212345 6
123456789012345678901234567890121234567890123456789012345678901212345 6
1234567890123456789012345678901212345678901234567890123456789012123456
305

http://www.xtremepapers.net

Part IV: Three Practice GREs

1234567890123456789012345678901212345678901234567890123456789012123456
123456789012345678901234567890121234567890123456789012345678901212345 6
3456789012345678901234567890121234567890123456789012345678901212345 6
12
6
123456789012345678901234567890121234567890123456789012345678901212345
Column A
Column B
Column A
Column B
3456789012345678901234567890121234567890123456789012345678901212345
6
12
3456789012345678901234567890121234567890123456789012345678901212345
6
12
2
1.
3
2
2
6
123456789012345678901234567890121234567890123456789012345678901212345
3
1
7
=
3456789012345678901234567890121234567890123456789012345678901212345 6
12
2
3456789012345678901234567890121234567890123456789012345678901212345
6
1
3456789012345678901234567890121234567890123456789012345678901212345 6
12
3456789012345678901234567890121234567890123456789012345678901212345 6
12
3456789012345678901234567890121234567890123456789012345678901212345 6
12
3456789012345678901234567890121234567890123456789012345678901212345 6
12
2
1 3456789012345678901234567890121234567890123456789012345678901212345 6
3456789012345678901234567890121234567890123456789012345678901212345 6
12
3456789012345678901234567890121234567890123456789012345678901212345 6
12
3456789012345678901234567890121234567890123456789012345678901212345 6
12
3456789012345678901234567890121234567890123456789012345678901212345 6
12
2
1 3456789012345678901234567890121234567890123456789012345678901212345 6
3456789012345678901234567890121234567890123456789012345678901212345
6
12
2.
x 1 2y 5 9
3456789012345678901234567890121234567890123456789012345678901212345
6
12
3456789012345678901234567890121234567890123456789012345678901212345 6
12
3456789012345678901234567890121234567890123456789012345678901212345
6
12
x5y
5. The figure above is an isosceles triangle.
6
123456789012345678901234567890121234567890123456789012345678901212345
3456789012345678901234567890121234567890123456789012345678901212345 6
12
3456789012345678901234567890121234567890123456789012345678901212345
6
2y
6
12
c
=2b
3456789012345678901234567890121234567890123456789012345678901212345
6
12
3456789012345678901234567890121234567890123456789012345678901212345 6
12
2
3456789012345678901234567890121234567890123456789012345678901212345
123456789012345678901234567890121234567890123456789012345678901212345 66
123456789012345678901234567890121234567890123456789012345678901212345 6
123456789012345678901234567890121234567890123456789012345678901212345 6
123456789012345678901234567890121234567890123456789012345678901212345 6
123456789012345678901234567890121234567890123456789012345678901212345 6
123456789012345678901234567890121234567890123456789012345678901212345 6
1
3456789012345678901234567890121234567890123456789012345678901212345 6
12
3456789012345678901234567890121234567890123456789012345678901212345 6
12
3456789012345678901234567890121234567890123456789012345678901212345 6
12
2
6
123456789012345678901234567890121234567890123456789012345678901212345
3.
x is a positive integer.
6.
b2a.1
6
123456789012345678901234567890121234567890123456789012345678901212345
6
123456789012345678901234567890121234567890123456789012345678901212345
x
a
a
b
0.90
6
123456789012345678901234567890121234567890123456789012345678901212345
1
x
x
x
6
123456789012345678901234567890121234567890123456789012345678901212345
12
b
3456789012345678901234567890121234567890123456789012345678901212345
6
123456789012345678901234567890121234567890123456789012345678901212345
3
x
6
123456789012345678901234567890121234567890123456789012345678901212345
123456789012345678901234567890121234567890123456789012345678901212345 66
123456789012345678901234567890121234567890123456789012345678901212345 6
1
3456789012345678901234567890121234567890123456789012345678901212345 6
12
3456789012345678901234567890121234567890123456789012345678901212345 6
12
123456789012345678901234567890121234567890123456789012345678901212345 6
123456789012345678901234567890121234567890123456789012345678901212345 6
3456789012345678901234567890121234567890123456789012345678901212345 6
12
3456789012345678901234567890121234567890123456789012345678901212345 6
12
3456789012345678901234567890121234567890123456789012345678901212345 6
12
6
123456789012345678901234567890121234567890123456789012345678901212345
4.
7.
|x| . 1
3456789012345678901234567890121234567890123456789012345678901212345
6
12
3456789012345678901234567890121234567890123456789012345678901212345 6
12
2
3456789012345678901234567890121234567890123456789012345678901212345
6
123456789012345678901234567890121234567890123456789012345678901212345
x(x 2 1)
0
6
123456789012345678901234567890121234567890123456789012345678901212345
6
1
123456789012345678901234567890121234567890123456789012345678901212345 6
123456789012345678901234567890121234567890123456789012345678901212345 6
3456789012345678901234567890121234567890123456789012345678901212345 6
12
3456789012345678901234567890121234567890123456789012345678901212345 6
12
3456789012345678901234567890121234567890123456789012345678901212345 6
12
3456789012345678901234567890121234567890123456789012345678901212345 6
12
6
123456789012345678901234567890121234567890123456789012345678901212345
1
2
6
123456789012345678901234567890121234567890123456789012345678901212345
d1c
(b2 1 2bc 1 c2)2
6
123456789012345678901234567890121234567890123456789012345678901212345
6
123456789012345678901234567890121234567890123456789012345678901212345
6
123456789012345678901234567890121234567890123456789012345678901212345
1 3456789012345678901234567890121234567890123456789012345678901212345 6
2
3456789012345678901234567890121234567890123456789012345678901212345
123456789012345678901234567890121234567890123456789012345678901212345 66
123456789012345678901234567890121234567890123456789012345678901212345 6
123456789012345678901234567890121234567890123456789012345678901212345 6
123456789012345678901234567890121234567890123456789012345678901212345 6
1
3456789012345678901234567890121234567890123456789012345678901212345 6
12
123456789012345678901234567890121234567890123456789012345678901212345 6
3456789012345678901234567890121234567890123456789012345678901212345 6
12
123456789012345678901234567890121234567890123456789012345678901212345 6
3456789012345678901234567890121234567890123456789012345678901212345 6
12
123456789012345678901234567890121234567890123456789012345678901212345 6
123456789012345678901234567890121234567890123456789012345678901212345 6
123456789012345678901234567890121234567890123456789012345678901212345 66
306 123456789012345678901234567890121234567890123456789012345678901212345
1234567890123456789012345678901212345678901234567890123456789012123456

SD

www.petersons.com

http://www.xtremepapers.net

TEST 3
1234567890123456789012345678901212345678901234567890123456789012123456
123456789012345678901234567890121234567890123456789012345678901212345 6
3456789012345678901234567890121234567890123456789012345678901212345 6
12
6
123456789012345678901234567890121234567890123456789012345678901212345
Column A
Column B
Column A
Column B
3456789012345678901234567890121234567890123456789012345678901212345
6
12
3456789012345678901234567890121234567890123456789012345678901212345
6
12
15
11
12
13
8. A certain swim-team is composed of
11.
4
4 14 14 1 6
123456789012345678901234567890121234567890123456789012345678901212345
3456789012345678901234567890121234567890123456789012345678901212345
6
12
414
divers, individual swimmers, and relay
6
123456789012345678901234567890121234567890123456789012345678901212345
3456789012345678901234567890121234567890123456789012345678901212345
6
12
swimmers, and each member only
3456789012345678901234567890121234567890123456789012345678901212345
6
12
3456789012345678901234567890121234567890123456789012345678901212345
6
12
participates
in
one
event.
The
team
has
3456789012345678901234567890121234567890123456789012345678901212345 6
12
2
3456789012345678901234567890121234567890123456789012345678901212345
6
1
50 members, 20 percent of which are
3456789012345678901234567890121234567890123456789012345678901212345
6
12
3456789012345678901234567890121234567890123456789012345678901212345
6
12
divers.
3456789012345678901234567890121234567890123456789012345678901212345 6
12
3456789012345678901234567890121234567890123456789012345678901212345 6
12
6
123456789012345678901234567890121234567890123456789012345678901212345
Four times the
The number of
3456789012345678901234567890121234567890123456789012345678901212345
6
12
3456789012345678901234567890121234567890123456789012345678901212345
6
12
number
of
divers
relay
racers
12. Six students compete for first, second,
3456789012345678901234567890121234567890123456789012345678901212345
6
12
3456789012345678901234567890121234567890123456789012345678901212345
6
12
and
third
place
finishes
in
a
ski
race.
2
3456789012345678901234567890121234567890123456789012345678901212345
6
1
3456789012345678901234567890121234567890123456789012345678901212345 6
12
3456789012345678901234567890121234567890123456789012345678901212345
6
12
120
Number of
3456789012345678901234567890121234567890123456789012345678901212345
6
12
3456789012345678901234567890121234567890123456789012345678901212345
6
12
possible
outcomes
2
3456789012345678901234567890121234567890123456789012345678901212345
123456789012345678901234567890121234567890123456789012345678901212345 66
123456789012345678901234567890121234567890123456789012345678901212345
for first, second,
6
123456789012345678901234567890121234567890123456789012345678901212345
6
123456789012345678901234567890121234567890123456789012345678901212345
and
third
place
123456789012345678901234567890121234567890123456789012345678901212345 66
123456789012345678901234567890121234567890123456789012345678901212345
6
1 9. Kathy drives 20 kilometers due north
3456789012345678901234567890121234567890123456789012345678901212345
6
12
from
town
A
to
town
B.
Then
she
drives
3456789012345678901234567890121234567890123456789012345678901212345 6
12
3456789012345678901234567890121234567890123456789012345678901212345
6
12
30 kilometers due west to town C.
3456789012345678901234567890121234567890123456789012345678901212345
6
12
3456789012345678901234567890121234567890123456789012345678901212345 6
12
3456789012345678901234567890121234567890123456789012345678901212345
6
12
Distance between
40
3456789012345678901234567890121234567890123456789012345678901212345
6
12
town A and C in
3456789012345678901234567890121234567890123456789012345678901212345
6
12
2
3456789012345678901234567890121234567890123456789012345678901212345
6
123456789012345678901234567890121234567890123456789012345678901212345
kilometers
6
123456789012345678901234567890121234567890123456789012345678901212345
13.
C
and
D
are
digits
and
C
.
D.
123456789012345678901234567890121234567890123456789012345678901212345 66
123456789012345678901234567890121234567890123456789012345678901212345 6
1
CD
3456789012345678901234567890121234567890123456789012345678901212345
6
12
3456789012345678901234567890121234567890123456789012345678901212345
6
12
2 DC
6
123456789012345678901234567890121234567890123456789012345678901212345
123456789012345678901234567890121234567890123456789012345678901212345 6
3456789012345678901234567890121234567890123456789012345678901212345
6
12
EF
3456789012345678901234567890121234567890123456789012345678901212345
6
12
3456789012345678901234567890121234567890123456789012345678901212345
6
12
E1F
9
6
123456789012345678901234567890121234567890123456789012345678901212345
3456789012345678901234567890121234567890123456789012345678901212345 6
12
3456789012345678901234567890121234567890123456789012345678901212345
6
10. A sock drawer contains 75 socks. There
12
3456789012345678901234567890121234567890123456789012345678901212345
6
12
are an equal number of unpaired black,
3456789012345678901234567890121234567890123456789012345678901212345
6
12
6
123456789012345678901234567890121234567890123456789012345678901212345
white, and gray socks in the drawer.
6
123456789012345678901234567890121234567890123456789012345678901212345
123456789012345678901234567890121234567890123456789012345678901212345 6
2
3456789012345678901234567890121234567890123456789012345678901212345
6
123456789012345678901234567890121234567890123456789012345678901212345
The probability of
One sixth
6
123456789012345678901234567890121234567890123456789012345678901212345
6
123456789012345678901234567890121234567890123456789012345678901212345
grabbing
two
gray
123456789012345678901234567890121234567890123456789012345678901212345 66
1
socks in two
3456789012345678901234567890121234567890123456789012345678901212345
6
12
14.
n is a positive integer.
3456789012345678901234567890121234567890123456789012345678901212345
6
12
attempts
3456789012345678901234567890121234567890123456789012345678901212345
6
12
n
1
1
!
n!
1
1
~
!
3456789012345678901234567890121234567890123456789012345678901212345 6
12
6
123456789012345678901234567890121234567890123456789012345678901212345
n! 1 1
~n 1 1!!
3456789012345678901234567890121234567890123456789012345678901212345
6
12
3456789012345678901234567890121234567890123456789012345678901212345 6
12
3456789012345678901234567890121234567890123456789012345678901212345 6
12
3456789012345678901234567890121234567890123456789012345678901212345 6
12
123456789012345678901234567890121234567890123456789012345678901212345 6
2
6
123456789012345678901234567890121234567890123456789012345678901212345
1 3456789012345678901234567890121234567890123456789012345678901212345 6
3456789012345678901234567890121234567890123456789012345678901212345 6
12
123456789012345678901234567890121234567890123456789012345678901212345 6
3456789012345678901234567890121234567890123456789012345678901212345 6
12
123456789012345678901234567890121234567890123456789012345678901212345 6
123456789012345678901234567890121234567890123456789012345678901212345 6
123456789012345678901234567890121234567890123456789012345678901212345 6
123456789012345678901234567890121234567890123456789012345678901212345 6 307
1234567890123456789012345678901212345678901234567890123456789012123456

http://www.xtremepapers.net

Part IV: Three Practice GREs

1234567890123456789012345678901212345678901234567890123456789012123456
123456789012345678901234567890121234567890123456789012345678901212345 6
3456789012345678901234567890121234567890123456789012345678901212345 6
12
17. Which two y-intercepts could sum to zero? 6
123456789012345678901234567890121234567890123456789012345678901212345
3456789012345678901234567890121234567890123456789012345678901212345
6
12
Directions:
Select
the
best
answer
for
each
3456789012345678901234567890121234567890123456789012345678901212345
6
12
A.
Line
1
and
4
6
123456789012345678901234567890121234567890123456789012345678901212345
of
the
following
questions.
3456789012345678901234567890121234567890123456789012345678901212345
6
12
B.
Line
1
and
3
2
3456789012345678901234567890121234567890123456789012345678901212345
6
1
3456789012345678901234567890121234567890123456789012345678901212345
6
12
C.
Line
2
and
3
3456789012345678901234567890121234567890123456789012345678901212345
6
12
15.
3456789012345678901234567890121234567890123456789012345678901212345
6
12
D.
Line
2
and
4
3456789012345678901234567890121234567890123456789012345678901212345 6
12
2
3456789012345678901234567890121234567890123456789012345678901212345
6
1
E. Line 4 and 5
3456789012345678901234567890121234567890123456789012345678901212345
6
12
3456789012345678901234567890121234567890123456789012345678901212345 6
12
3456789012345678901234567890121234567890123456789012345678901212345 6
12
3456789012345678901234567890121234567890123456789012345678901212345
6
12
18.
2
6
1 3456789012345678901234567890121234567890123456789012345678901212345
3456789012345678901234567890121234567890123456789012345678901212345 6
12
3456789012345678901234567890121234567890123456789012345678901212345 6
12
3456789012345678901234567890121234567890123456789012345678901212345 6
12
3456789012345678901234567890121234567890123456789012345678901212345
6
12
The figures above are similar triangles.
2
6
1 3456789012345678901234567890121234567890123456789012345678901212345
3456789012345678901234567890121234567890123456789012345678901212345
6
12
What
is
the
value
of
a?
3456789012345678901234567890121234567890123456789012345678901212345 6
12
3456789012345678901234567890121234567890123456789012345678901212345 6
12
3456789012345678901234567890121234567890123456789012345678901212345
6
12
5
2
6
123456789012345678901234567890121234567890123456789012345678901212345
A.
6
123456789012345678901234567890121234567890123456789012345678901212345
3
The two circles above have the same
6
123456789012345678901234567890121234567890123456789012345678901212345
6
123456789012345678901234567890121234567890123456789012345678901212345
B.
2
radius.
Their
circumference
is
6p
each.
6
123456789012345678901234567890121234567890123456789012345678901212345
3456789012345678901234567890121234567890123456789012345678901212345
6
123456789012345678901234567890121234567890123456789012345678901212345
What is the distance between opposite
6
C. 3
1
3456789012345678901234567890121234567890123456789012345678901212345
6
12
corners
of
the
rectangle?
3456789012345678901234567890121234567890123456789012345678901212345
6
12
9
3456789012345678901234567890121234567890123456789012345678901212345
6
12
D.
2
3456789012345678901234567890121234567890123456789012345678901212345
6
123456789012345678901234567890121234567890123456789012345678901212345
A. =160
2
6
123456789012345678901234567890121234567890123456789012345678901212345
123456789012345678901234567890121234567890123456789012345678901212345 66
123456789012345678901234567890121234567890123456789012345678901212345
E. 5
B. =165
6
123456789012345678901234567890121234567890123456789012345678901212345
123456789012345678901234567890121234567890123456789012345678901212345 66
123456789012345678901234567890121234567890123456789012345678901212345
C. =180
6
123456789012345678901234567890121234567890123456789012345678901212345
Questions 1617 refer to the following graph.
6
123456789012345678901234567890121234567890123456789012345678901212345
6
1
D.
2
3456789012345678901234567890121234567890123456789012345678901212345
6
123456789012345678901234567890121234567890123456789012345678901212345
=210
6
123456789012345678901234567890121234567890123456789012345678901212345
6
1
6
123456789012345678901234567890121234567890123456789012345678901212345
E. =220
3456789012345678901234567890121234567890123456789012345678901212345 6
12
2
3456789012345678901234567890121234567890123456789012345678901212345
123456789012345678901234567890121234567890123456789012345678901212345 66
123456789012345678901234567890121234567890123456789012345678901212345 6
1
19. A class has x number of students in it.
3456789012345678901234567890121234567890123456789012345678901212345
6
12
3456789012345678901234567890121234567890123456789012345678901212345
Three students join the class, and the class 6
12
2
3456789012345678901234567890121234567890123456789012345678901212345
6
123456789012345678901234567890121234567890123456789012345678901212345
size increases by 20 percent. How many
6
123456789012345678901234567890121234567890123456789012345678901212345
6
1
students are in the class now?
6
123456789012345678901234567890121234567890123456789012345678901212345
123456789012345678901234567890121234567890123456789012345678901212345 6
2
3456789012345678901234567890121234567890123456789012345678901212345
6
123456789012345678901234567890121234567890123456789012345678901212345
A. 12
6
123456789012345678901234567890121234567890123456789012345678901212345
6
123456789012345678901234567890121234567890123456789012345678901212345
B.
15
123456789012345678901234567890121234567890123456789012345678901212345 66
1
C. 18
3456789012345678901234567890121234567890123456789012345678901212345
6
12
3456789012345678901234567890121234567890123456789012345678901212345
6
12
D.
20
3456789012345678901234567890121234567890123456789012345678901212345 6
12
3456789012345678901234567890121234567890123456789012345678901212345
6
12
E. 21
6
123456789012345678901234567890121234567890123456789012345678901212345
16.
Which
line
has
the
greatest
slope?
2
3456789012345678901234567890121234567890123456789012345678901212345
123456789012345678901234567890121234567890123456789012345678901212345 66
123456789012345678901234567890121234567890123456789012345678901212345 6
123456789012345678901234567890121234567890123456789012345678901212345
A. Line 1
6
123456789012345678901234567890121234567890123456789012345678901212345
6
1
B. Line 2
2
3456789012345678901234567890121234567890123456789012345678901212345
123456789012345678901234567890121234567890123456789012345678901212345 66
1
C. Line 3
3456789012345678901234567890121234567890123456789012345678901212345 6
12
6
123456789012345678901234567890121234567890123456789012345678901212345
D. Line 4
3456789012345678901234567890121234567890123456789012345678901212345
6
12
E.
Line
5
2
3456789012345678901234567890121234567890123456789012345678901212345
6
1
123456789012345678901234567890121234567890123456789012345678901212345 6
123456789012345678901234567890121234567890123456789012345678901212345 66
308 123456789012345678901234567890121234567890123456789012345678901212345
1234567890123456789012345678901212345678901234567890123456789012123456

www.petersons.com

http://www.xtremepapers.net

TEST 3
1234567890123456789012345678901212345678901234567890123456789012123456
123456789012345678901234567890121234567890123456789012345678901212345 6
3456789012345678901234567890121234567890123456789012345678901212345 6
12
20. A square has a side of length 4. What is
Questions 2325 refers to the following graph. 6
123456789012345678901234567890121234567890123456789012345678901212345
3456789012345678901234567890121234567890123456789012345678901212345
6
12
the distance from the center of the square
3456789012345678901234567890121234567890123456789012345678901212345
6
12
6
123456789012345678901234567890121234567890123456789012345678901212345
to
one
of
its
corners?
3456789012345678901234567890121234567890123456789012345678901212345 6
12
2
3456789012345678901234567890121234567890123456789012345678901212345
6
1
3456789012345678901234567890121234567890123456789012345678901212345
6
12
A.
3456789012345678901234567890121234567890123456789012345678901212345
6
12
2
z
=
3456789012345678901234567890121234567890123456789012345678901212345 6
12
3456789012345678901234567890121234567890123456789012345678901212345
6
12
B. 2=2
6
123456789012345678901234567890121234567890123456789012345678901212345
3456789012345678901234567890121234567890123456789012345678901212345
6
12
3456789012345678901234567890121234567890123456789012345678901212345 6
12
3456789012345678901234567890121234567890123456789012345678901212345
6
C. 4=2
12
3456789012345678901234567890121234567890123456789012345678901212345
6
12
2
3456789012345678901234567890121234567890123456789012345678901212345
6
1
3456789012345678901234567890121234567890123456789012345678901212345
6
12
D. 6=2
3456789012345678901234567890121234567890123456789012345678901212345 6
12
3456789012345678901234567890121234567890123456789012345678901212345 6
12
3456789012345678901234567890121234567890123456789012345678901212345
6
12
E. 8=2
2
6
1 3456789012345678901234567890121234567890123456789012345678901212345
3456789012345678901234567890121234567890123456789012345678901212345 6
12
3456789012345678901234567890121234567890123456789012345678901212345 6
12
3456789012345678901234567890121234567890123456789012345678901212345
6
12
21. If 3 pounds of peanuts, which cost 34
3456789012345678901234567890121234567890123456789012345678901212345
6
12
2
3456789012345678901234567890121234567890123456789012345678901212345
6
123456789012345678901234567890121234567890123456789012345678901212345
cents per pound, are mixed with 5 pounds
6
123456789012345678901234567890121234567890123456789012345678901212345
6
123456789012345678901234567890121234567890123456789012345678901212345
of
cashews,
which
cost
90
cents
per
123456789012345678901234567890121234567890123456789012345678901212345 66
123456789012345678901234567890121234567890123456789012345678901212345
pound, how many cents per pound is the
6
123456789012345678901234567890121234567890123456789012345678901212345
6
1
mixture of the two?
3456789012345678901234567890121234567890123456789012345678901212345 6
12
3456789012345678901234567890121234567890123456789012345678901212345
6
12
A. 67
3456789012345678901234567890121234567890123456789012345678901212345
6
12
23. According to the table, what is the
2
3456789012345678901234567890121234567890123456789012345678901212345
6
123456789012345678901234567890121234567890123456789012345678901212345
B. 69
6
123456789012345678901234567890121234567890123456789012345678901212345
relationship between the frequency and
6
123456789012345678901234567890121234567890123456789012345678901212345
C. 71
6
123456789012345678901234567890121234567890123456789012345678901212345
the energy level of electromagnetic
6
123456789012345678901234567890121234567890123456789012345678901212345
D.
73
6
123456789012345678901234567890121234567890123456789012345678901212345
radiation?
6
123456789012345678901234567890121234567890123456789012345678901212345
E.
75
6
123456789012345678901234567890121234567890123456789012345678901212345
A. As one increases, the other increases. 6
123456789012345678901234567890121234567890123456789012345678901212345
1
B. As one increases, the other decreases. 6
3456789012345678901234567890121234567890123456789012345678901212345
6
12
22. A board is 6 feet long, 6 inches wide, and
3456789012345678901234567890121234567890123456789012345678901212345
6
12
C. There is no direct relationship.
6
123456789012345678901234567890121234567890123456789012345678901212345
2 inches tall. What is its surface area in
6
123456789012345678901234567890121234567890123456789012345678901212345
D.
The
energy
level
is
twice
as
much
as
3456789012345678901234567890121234567890123456789012345678901212345
6
12
feet squared? (One foot is 12 inches.)
2
3456789012345678901234567890121234567890123456789012345678901212345
6
123456789012345678901234567890121234567890123456789012345678901212345
the frequency level.
6
123456789012345678901234567890121234567890123456789012345678901212345
11
6
1
E.
The
frequency
level
is
twice
as
much
3456789012345678901234567890121234567890123456789012345678901212345
6
12
A.
2
3456789012345678901234567890121234567890123456789012345678901212345
6
12
as
the
wavelength
level.
2
3456789012345678901234567890121234567890123456789012345678901212345
123456789012345678901234567890121234567890123456789012345678901212345 66
123456789012345678901234567890121234567890123456789012345678901212345
1
6
1
B. 6
6
123456789012345678901234567890121234567890123456789012345678901212345
24. Which kind of radiation has the shortest
2
123456789012345678901234567890121234567890123456789012345678901212345 6
2
3456789012345678901234567890121234567890123456789012345678901212345
6
wavelength?
123456789012345678901234567890121234567890123456789012345678901212345
C. 7
6
123456789012345678901234567890121234567890123456789012345678901212345
6
123456789012345678901234567890121234567890123456789012345678901212345
A.
Gamma
Rays
6
123456789012345678901234567890121234567890123456789012345678901212345
49
6
1
D.
B.
X-rays
2
3456789012345678901234567890121234567890123456789012345678901212345
6
123456789012345678901234567890121234567890123456789012345678901212345
6
6
123456789012345678901234567890121234567890123456789012345678901212345
C.
Ultra-violet
6
123456789012345678901234567890121234567890123456789012345678901212345
E. 9
6
123456789012345678901234567890121234567890123456789012345678901212345
D.
Infrared
6
1
2
3456789012345678901234567890121234567890123456789012345678901212345
6
123456789012345678901234567890121234567890123456789012345678901212345
E. Radio
6
123456789012345678901234567890121234567890123456789012345678901212345
123456789012345678901234567890121234567890123456789012345678901212345 66
123456789012345678901234567890121234567890123456789012345678901212345 6
1
3456789012345678901234567890121234567890123456789012345678901212345 6
12
123456789012345678901234567890121234567890123456789012345678901212345 6
3456789012345678901234567890121234567890123456789012345678901212345 6
12
123456789012345678901234567890121234567890123456789012345678901212345 6
3456789012345678901234567890121234567890123456789012345678901212345 6
12
123456789012345678901234567890121234567890123456789012345678901212345 6
123456789012345678901234567890121234567890123456789012345678901212345 6
123456789012345678901234567890121234567890123456789012345678901212345 6
123456789012345678901234567890121234567890123456789012345678901212345 6 309
1234567890123456789012345678901212345678901234567890123456789012123456

http://www.xtremepapers.net

Part IV: Three Practice GREs

1234567890123456789012345678901212345678901234567890123456789012123456
123456789012345678901234567890121234567890123456789012345678901212345 6
3456789012345678901234567890121234567890123456789012345678901212345 6
12
25. If electromagnetic radiation has a fre27. How many liters of a solution that is 20
6
123456789012345678901234567890121234567890123456789012345678901212345
3456789012345678901234567890121234567890123456789012345678901212345
6
12
22
quency of 10 Hz, what kind of radiation
percent water by volume must be added to 6
3456789012345678901234567890121234567890123456789012345678901212345
12
6
123456789012345678901234567890121234567890123456789012345678901212345
is it?
3 liters of a solution that is 40 percent
3456789012345678901234567890121234567890123456789012345678901212345
6
12
2
3456789012345678901234567890121234567890123456789012345678901212345
6
1
water by volume to create a solution that 6
3456789012345678901234567890121234567890123456789012345678901212345
12
A.
Gamma
Rays
3456789012345678901234567890121234567890123456789012345678901212345
6
12
is 25 percent water by volume?
3456789012345678901234567890121234567890123456789012345678901212345
6
12
B.
X-rays
3456789012345678901234567890121234567890123456789012345678901212345 6
12
2
3456789012345678901234567890121234567890123456789012345678901212345
6
1
C. Ultra-violet
A. 5
3456789012345678901234567890121234567890123456789012345678901212345
6
12
3456789012345678901234567890121234567890123456789012345678901212345
6
12
D.
Infrared
B.
6
3456789012345678901234567890121234567890123456789012345678901212345 6
12
3456789012345678901234567890121234567890123456789012345678901212345
6
12
E. Radio
C. 7
6
123456789012345678901234567890121234567890123456789012345678901212345
3456789012345678901234567890121234567890123456789012345678901212345
6
12
D.
8
3456789012345678901234567890121234567890123456789012345678901212345 6
12
3456789012345678901234567890121234567890123456789012345678901212345
6
26. If a, b, and c are all positive odd integers,
12
E. 9
3456789012345678901234567890121234567890123456789012345678901212345
6
12
then
which
of
the
following
must
be
odd?
2
3456789012345678901234567890121234567890123456789012345678901212345
6
1
3456789012345678901234567890121234567890123456789012345678901212345
6
12
28. There are 150 students in math courses at 6
3456789012345678901234567890121234567890123456789012345678901212345
12
I.
ab
2
bc
3456789012345678901234567890121234567890123456789012345678901212345
6
12
Kronhorst High School: 73 are in geom3456789012345678901234567890121234567890123456789012345678901212345
6
12
II.
abc
2
3456789012345678901234567890121234567890123456789012345678901212345
6
123456789012345678901234567890121234567890123456789012345678901212345
etry, 62 are in algebra, and 52 are in
2
6
123456789012345678901234567890121234567890123456789012345678901212345
III.
c
1
c
6
123456789012345678901234567890121234567890123456789012345678901212345
neither.
How
many
students
are
in
both
123456789012345678901234567890121234567890123456789012345678901212345 66
123456789012345678901234567890121234567890123456789012345678901212345
A. I only
geometry and algebra?
6
123456789012345678901234567890121234567890123456789012345678901212345
6
1
B.
II
only
3456789012345678901234567890121234567890123456789012345678901212345
6
12
A.
32
3456789012345678901234567890121234567890123456789012345678901212345
6
12
C.
I
and
II
only
3456789012345678901234567890121234567890123456789012345678901212345
6
12
B.
33
2
3456789012345678901234567890121234567890123456789012345678901212345
6
123456789012345678901234567890121234567890123456789012345678901212345
D. I and III only
6
123456789012345678901234567890121234567890123456789012345678901212345
C.
35
6
123456789012345678901234567890121234567890123456789012345678901212345
E. I, II, and III
6
123456789012345678901234567890121234567890123456789012345678901212345
D. 36
123456789012345678901234567890121234567890123456789012345678901212345 66
123456789012345678901234567890121234567890123456789012345678901212345
E. 37
6
123456789012345678901234567890121234567890123456789012345678901212345
123456789012345678901234567890121234567890123456789012345678901212345 66
123456789012345678901234567890121234567890123456789012345678901212345 6
1 Verbal
3456789012345678901234567890121234567890123456789012345678901212345 6
12
3456789012345678901234567890121234567890123456789012345678901212345 6
12
6
123456789012345678901234567890121234567890123456789012345678901212345
30 Questions30 Minutes
6
123456789012345678901234567890121234567890123456789012345678901212345
3456789012345678901234567890121234567890123456789012345678901212345 6
12
2
3456789012345678901234567890121234567890123456789012345678901212345
6
123456789012345678901234567890121234567890123456789012345678901212345
Antonyms
6
123456789012345678901234567890121234567890123456789012345678901212345
6
1
3456789012345678901234567890121234567890123456789012345678901212345 6
12
3456789012345678901234567890121234567890123456789012345678901212345
6
12
Directions: Each question contains a word printed in capital letters, followed by five
3456789012345678901234567890121234567890123456789012345678901212345
6
12
3456789012345678901234567890121234567890123456789012345678901212345
6
12
answer
choices.
Choose
the
answer
choice
that
contains
the
word
or
phrase
most
nearly
123456789012345678901234567890121234567890123456789012345678901212345 6
6
123456789012345678901234567890121234567890123456789012345678901212345
OPPOSITE in meaning to the word in capital letters.
6
123456789012345678901234567890121234567890123456789012345678901212345
2
3456789012345678901234567890121234567890123456789012345678901212345
123456789012345678901234567890121234567890123456789012345678901212345 66
123456789012345678901234567890121234567890123456789012345678901212345 6
123456789012345678901234567890121234567890123456789012345678901212345 6
123456789012345678901234567890121234567890123456789012345678901212345 6
1 1. CORROBORATE :
2. INEXACTITUDE :
3456789012345678901234567890121234567890123456789012345678901212345
6
12
3456789012345678901234567890121234567890123456789012345678901212345 6
12
3456789012345678901234567890121234567890123456789012345678901212345
6
12
A. pledge
A. crude
3456789012345678901234567890121234567890123456789012345678901212345
6
12
6
123456789012345678901234567890121234567890123456789012345678901212345
B.
deny
B.
precise
2
3456789012345678901234567890121234567890123456789012345678901212345
123456789012345678901234567890121234567890123456789012345678901212345 66
123456789012345678901234567890121234567890123456789012345678901212345
C. harbor
C. inexpert
6
123456789012345678901234567890121234567890123456789012345678901212345
6
123456789012345678901234567890121234567890123456789012345678901212345
D. detain
D. industrious
6
1
2
3456789012345678901234567890121234567890123456789012345678901212345
6
123456789012345678901234567890121234567890123456789012345678901212345
E. affirm
E. articulate
6
1
3456789012345678901234567890121234567890123456789012345678901212345 6
12
123456789012345678901234567890121234567890123456789012345678901212345 6
3456789012345678901234567890121234567890123456789012345678901212345 6
12
123456789012345678901234567890121234567890123456789012345678901212345 6
123456789012345678901234567890121234567890123456789012345678901212345 6
123456789012345678901234567890121234567890123456789012345678901212345 66
310 123456789012345678901234567890121234567890123456789012345678901212345
1234567890123456789012345678901212345678901234567890123456789012123456

www.petersons.com

http://www.xtremepapers.net

TEST 3
1234567890123456789012345678901212345678901234567890123456789012123456
123456789012345678901234567890121234567890123456789012345678901212345 6
3456789012345678901234567890121234567890123456789012345678901212345 6
12
3. FOMENT :
6. PULCHRITUDE :
6
123456789012345678901234567890121234567890123456789012345678901212345
3456789012345678901234567890121234567890123456789012345678901212345 6
12
3456789012345678901234567890121234567890123456789012345678901212345
6
12
A. quell
A. ostentatious
6
123456789012345678901234567890121234567890123456789012345678901212345
3456789012345678901234567890121234567890123456789012345678901212345
6
12
B.
agitate
B.
elegant
2
3456789012345678901234567890121234567890123456789012345678901212345
6
1
3456789012345678901234567890121234567890123456789012345678901212345
6
12
C.
appropriate
C.
estranged
3456789012345678901234567890121234567890123456789012345678901212345 6
12
3456789012345678901234567890121234567890123456789012345678901212345
6
12
D. estimate
D. comedic
3456789012345678901234567890121234567890123456789012345678901212345
6
12
2
3456789012345678901234567890121234567890123456789012345678901212345
6
1
E. exonerate
E. uncomely
3456789012345678901234567890121234567890123456789012345678901212345
6
12
3456789012345678901234567890121234567890123456789012345678901212345 6
12
3456789012345678901234567890121234567890123456789012345678901212345 6
12
3456789012345678901234567890121234567890123456789012345678901212345
6
12
4. CALLOW :
7. STOLID :
2
6
1 3456789012345678901234567890121234567890123456789012345678901212345
3456789012345678901234567890121234567890123456789012345678901212345
6
12
A. picturesque
A. imaginative
3456789012345678901234567890121234567890123456789012345678901212345
6
12
3456789012345678901234567890121234567890123456789012345678901212345
6
12
B.
mature
B.
sterile
3456789012345678901234567890121234567890123456789012345678901212345 6
12
2
3456789012345678901234567890121234567890123456789012345678901212345
6
1
C. timid
C. venal
3456789012345678901234567890121234567890123456789012345678901212345
6
12
3456789012345678901234567890121234567890123456789012345678901212345
6
12
D.
canny
D.
emotional
3456789012345678901234567890121234567890123456789012345678901212345 6
12
3456789012345678901234567890121234567890123456789012345678901212345
6
12
E. particular
E. abrasive
3456789012345678901234567890121234567890123456789012345678901212345
6
12
3456789012345678901234567890121234567890123456789012345678901212345 6
12
3456789012345678901234567890121234567890123456789012345678901212345 6
12
3456789012345678901234567890121234567890123456789012345678901212345
6
12
5. BACCHANALIAN :
8. PREVARICATE :
3456789012345678901234567890121234567890123456789012345678901212345
6
12
2
3456789012345678901234567890121234567890123456789012345678901212345
123456789012345678901234567890121234567890123456789012345678901212345 66
1
A. petulant
A. speak honestly
3456789012345678901234567890121234567890123456789012345678901212345
6
12
B. orgiastic
B. speak hastily
3456789012345678901234567890121234567890123456789012345678901212345
6
12
3456789012345678901234567890121234567890123456789012345678901212345
6
12
C.
analgesia
C.
speak
slowly
2
3456789012345678901234567890121234567890123456789012345678901212345
123456789012345678901234567890121234567890123456789012345678901212345 66
123456789012345678901234567890121234567890123456789012345678901212345
D. pious
D. speak with vulgarities
6
123456789012345678901234567890121234567890123456789012345678901212345
6
123456789012345678901234567890121234567890123456789012345678901212345
E.
shrewd
E.
speak
with
pathos
123456789012345678901234567890121234567890123456789012345678901212345 66
123456789012345678901234567890121234567890123456789012345678901212345 6
123456789012345678901234567890121234567890123456789012345678901212345 6
123456789012345678901234567890121234567890123456789012345678901212345 6
123456789012345678901234567890121234567890123456789012345678901212345
Analogies
6
1
2
3456789012345678901234567890121234567890123456789012345678901212345
123456789012345678901234567890121234567890123456789012345678901212345 66
123456789012345678901234567890121234567890123456789012345678901212345 6
1
Directions: In each question, there is an initial pair of words or phrases that are related in
6
123456789012345678901234567890121234567890123456789012345678901212345
3456789012345678901234567890121234567890123456789012345678901212345
6
12
some manner. Select the answer choice containing the lettered pair that best expresses a
2
3456789012345678901234567890121234567890123456789012345678901212345
123456789012345678901234567890121234567890123456789012345678901212345 66
123456789012345678901234567890121234567890123456789012345678901212345
relationship similar to the initial pair.
6
1
3456789012345678901234567890121234567890123456789012345678901212345 6
12
3456789012345678901234567890121234567890123456789012345678901212345 6
12
3456789012345678901234567890121234567890123456789012345678901212345 6
12
3456789012345678901234567890121234567890123456789012345678901212345 6
12
6
123456789012345678901234567890121234567890123456789012345678901212345
9. LONG-WINDED : BREVITY ::
11. CANDID : FRANK
6
123456789012345678901234567890121234567890123456789012345678901212345
6
123456789012345678901234567890121234567890123456789012345678901212345
:
kind
A.
oversized
:
large
A.
sincere
2
3456789012345678901234567890121234567890123456789012345678901212345
123456789012345678901234567890121234567890123456789012345678901212345 66
123456789012345678901234567890121234567890123456789012345678901212345
B. erudite : learned
B. attractive : repulsive
6
123456789012345678901234567890121234567890123456789012345678901212345
6
123456789012345678901234567890121234567890123456789012345678901212345
C.
duplicitous:
timorous
C.
lean
:
excess
6
1
2
3456789012345678901234567890121234567890123456789012345678901212345
6
123456789012345678901234567890121234567890123456789012345678901212345
D. pernicious : hopeful
D. wealthy : money
6
123456789012345678901234567890121234567890123456789012345678901212345
6
123456789012345678901234567890121234567890123456789012345678901212345
E.
measured
:
mercurial
E.
spent
:
used
123456789012345678901234567890121234567890123456789012345678901212345 66
1
3456789012345678901234567890121234567890123456789012345678901212345 6
12
3456789012345678901234567890121234567890123456789012345678901212345
6
12
12. AUSTERE : DECORATED ::
10. LAWYER : COURT ::
3456789012345678901234567890121234567890123456789012345678901212345
6
12
3456789012345678901234567890121234567890123456789012345678901212345 6
12
6
123456789012345678901234567890121234567890123456789012345678901212345
A. paltry : exorbitant
A. actor : play
3456789012345678901234567890121234567890123456789012345678901212345
6
12
B. offensive : tactful
B. janitor : school
6
123456789012345678901234567890121234567890123456789012345678901212345
3456789012345678901234567890121234567890123456789012345678901212345
6
12
C. ingenious : creative
C. manager : team
6
123456789012345678901234567890121234567890123456789012345678901212345
3456789012345678901234567890121234567890123456789012345678901212345
6
12
D.
lackluster
:
ominous
D.
prison
:
guard
2
3456789012345678901234567890121234567890123456789012345678901212345
6
1
6
123456789012345678901234567890121234567890123456789012345678901212345
E. powerful : elusive
E. bureaucrat : bureaucracy
6
123456789012345678901234567890121234567890123456789012345678901212345
123456789012345678901234567890121234567890123456789012345678901212345 6 311
1234567890123456789012345678901212345678901234567890123456789012123456

http://www.xtremepapers.net

Part IV: Three Practice GREs

1234567890123456789012345678901212345678901234567890123456789012123456
123456789012345678901234567890121234567890123456789012345678901212345 6
3456789012345678901234567890121234567890123456789012345678901212345 6
12
13. PHYSICIAN : HUMAN BODY ::
15. TURGID : BOMBASTIC ::
6
123456789012345678901234567890121234567890123456789012345678901212345
3456789012345678901234567890121234567890123456789012345678901212345 6
12
3456789012345678901234567890121234567890123456789012345678901212345
6
12
A. horticulturist : plant
A. lazy : lethargic
6
123456789012345678901234567890121234567890123456789012345678901212345
3456789012345678901234567890121234567890123456789012345678901212345
6
12
B.
coach
:
player
B.
rotund
:
portly
2
3456789012345678901234567890121234567890123456789012345678901212345
6
1
3456789012345678901234567890121234567890123456789012345678901212345
6
12
C.
teacher
:
student
C.
bemused
:
distract
3456789012345678901234567890121234567890123456789012345678901212345 6
12
3456789012345678901234567890121234567890123456789012345678901212345
6
12
D. veterinarian : cow
D. catatonic : cataclysmic
3456789012345678901234567890121234567890123456789012345678901212345
6
12
2
3456789012345678901234567890121234567890123456789012345678901212345
6
1
E. biologist : evolution
E. histrionic : sordid
3456789012345678901234567890121234567890123456789012345678901212345
6
12
3456789012345678901234567890121234567890123456789012345678901212345 6
12
3456789012345678901234567890121234567890123456789012345678901212345 6
12
3456789012345678901234567890121234567890123456789012345678901212345
6
12
14. CRAVEN : TREPIDATIOUS ::
2
6
1 3456789012345678901234567890121234567890123456789012345678901212345
3456789012345678901234567890121234567890123456789012345678901212345
6
12
A. miniscule : minute
3456789012345678901234567890121234567890123456789012345678901212345
6
12
3456789012345678901234567890121234567890123456789012345678901212345
6
12
B.
migratory
:
transitory
3456789012345678901234567890121234567890123456789012345678901212345 6
12
2
3456789012345678901234567890121234567890123456789012345678901212345
6
1
C. ageless : ancient
3456789012345678901234567890121234567890123456789012345678901212345
6
12
3456789012345678901234567890121234567890123456789012345678901212345
6
12
D.
agile
:
acute
3456789012345678901234567890121234567890123456789012345678901212345 6
12
3456789012345678901234567890121234567890123456789012345678901212345
6
12
E. exultant : joyful
3456789012345678901234567890121234567890123456789012345678901212345
6
12
3456789012345678901234567890121234567890123456789012345678901212345 6
12
3456789012345678901234567890121234567890123456789012345678901212345 6
12
3456789012345678901234567890121234567890123456789012345678901212345 6
12
Sentence Completions
3456789012345678901234567890121234567890123456789012345678901212345
6
12
2
6
123456789012345678901234567890121234567890123456789012345678901212345
1 3456789012345678901234567890121234567890123456789012345678901212345 6
3456789012345678901234567890121234567890123456789012345678901212345 6
12
3456789012345678901234567890121234567890123456789012345678901212345
6
12
Directions: Each sentence below contains one or two blanks, with each blank corre3456789012345678901234567890121234567890123456789012345678901212345
6
12
2
3456789012345678901234567890121234567890123456789012345678901212345
6
123456789012345678901234567890121234567890123456789012345678901212345
sponding to an omitted word. Find the answer choice that has the word or set or words
6
123456789012345678901234567890121234567890123456789012345678901212345
6
that best fits the meaning of the sentence as a whole.
123456789012345678901234567890121234567890123456789012345678901212345
123456789012345678901234567890121234567890123456789012345678901212345 66
123456789012345678901234567890121234567890123456789012345678901212345 6
123456789012345678901234567890121234567890123456789012345678901212345 6
123456789012345678901234567890121234567890123456789012345678901212345 6
123456789012345678901234567890121234567890123456789012345678901212345
16. The demolition crew did not successfully
18. The publicity surrounding the mismanage- 6
123456789012345678901234567890121234567890123456789012345678901212345
6
1
__________
the
building
since
two
walls
ment of the companys capital __________ 6
2
3456789012345678901234567890121234567890123456789012345678901212345
123456789012345678901234567890121234567890123456789012345678901212345
6
123456789012345678901234567890121234567890123456789012345678901212345
remained intact.
the companys credibility with investors,
6
1
6
123456789012345678901234567890121234567890123456789012345678901212345
but
the
company
remained
solvent
largely
3456789012345678901234567890121234567890123456789012345678901212345
6
12
A.
raze
2
3456789012345678901234567890121234567890123456789012345678901212345
6
123456789012345678901234567890121234567890123456789012345678901212345
on account of its __________ market
6
123456789012345678901234567890121234567890123456789012345678901212345
B.
construct
6
1
share.
3456789012345678901234567890121234567890123456789012345678901212345
6
12
C.
erect
3456789012345678901234567890121234567890123456789012345678901212345 6
12
2
3456789012345678901234567890121234567890123456789012345678901212345
6
123456789012345678901234567890121234567890123456789012345678901212345
D. incapacitate
A. belabored. .steady
6
123456789012345678901234567890121234567890123456789012345678901212345
6
1
E.
deactivate
B.
weakened.
.evolving
123456789012345678901234567890121234567890123456789012345678901212345 6
6
123456789012345678901234567890121234567890123456789012345678901212345
C. aided. .climbing
3456789012345678901234567890121234567890123456789012345678901212345
6
12
3456789012345678901234567890121234567890123456789012345678901212345
6
12
17. The ethicist argued that the person in
D.
lessened.
.volatile
3456789012345678901234567890121234567890123456789012345678901212345
6
12
question was not __________ for the
3456789012345678901234567890121234567890123456789012345678901212345
6
12
E. damaged. .stable
6
123456789012345678901234567890121234567890123456789012345678901212345
mans
injury
since
it
was
an
accident.
2
3456789012345678901234567890121234567890123456789012345678901212345
123456789012345678901234567890121234567890123456789012345678901212345 66
123456789012345678901234567890121234567890123456789012345678901212345 6
123456789012345678901234567890121234567890123456789012345678901212345
A. culpable
6
123456789012345678901234567890121234567890123456789012345678901212345
6
1
B.
attributable
2
3456789012345678901234567890121234567890123456789012345678901212345
123456789012345678901234567890121234567890123456789012345678901212345 66
123456789012345678901234567890121234567890123456789012345678901212345
C. capable
6
123456789012345678901234567890121234567890123456789012345678901212345
6
123456789012345678901234567890121234567890123456789012345678901212345
D.
reasonable
6
1
2
3456789012345678901234567890121234567890123456789012345678901212345
6
123456789012345678901234567890121234567890123456789012345678901212345
E. actionable
6
1
3456789012345678901234567890121234567890123456789012345678901212345 6
12
123456789012345678901234567890121234567890123456789012345678901212345 6
3456789012345678901234567890121234567890123456789012345678901212345 6
12
123456789012345678901234567890121234567890123456789012345678901212345 6
123456789012345678901234567890121234567890123456789012345678901212345 6
123456789012345678901234567890121234567890123456789012345678901212345 66
312 123456789012345678901234567890121234567890123456789012345678901212345
1234567890123456789012345678901212345678901234567890123456789012123456

www.petersons.com

http://www.xtremepapers.net

TEST 3
1234567890123456789012345678901212345678901234567890123456789012123456
123456789012345678901234567890121234567890123456789012345678901212345 6
3456789012345678901234567890121234567890123456789012345678901212345 6
12
19. Though the official had hoped his speech
6
22. Though the glass was __________ when
123456789012345678901234567890121234567890123456789012345678901212345
3456789012345678901234567890121234567890123456789012345678901212345
6
12
would calm the citizenry, his assurances
bought, years of poor maintenance had
3456789012345678901234567890121234567890123456789012345678901212345
6
12
6
123456789012345678901234567890121234567890123456789012345678901212345
did
not
________
the
crowds
fears
rendered
it
nearly
__________,
stifling
3456789012345678901234567890121234567890123456789012345678901212345 6
12
2
3456789012345678901234567890121234567890123456789012345678901212345
6
1
concerning the likelihood of the natural
almost all of the sunlight that strained to
3456789012345678901234567890121234567890123456789012345678901212345
6
12
3456789012345678901234567890121234567890123456789012345678901212345
6
12
disaster
recurring.
cross
its
interior.
3456789012345678901234567890121234567890123456789012345678901212345 6
12
3456789012345678901234567890121234567890123456789012345678901212345 6
12
6
123456789012345678901234567890121234567890123456789012345678901212345
A. alter
A. permeable. .dark
3456789012345678901234567890121234567890123456789012345678901212345
6
12
3456789012345678901234567890121234567890123456789012345678901212345
6
12
B.
assuage
B.
clear.
.obscure
3456789012345678901234567890121234567890123456789012345678901212345 6
12
3456789012345678901234567890121234567890123456789012345678901212345
6
12
C. increase
C. lucid. .obstructed
6
123456789012345678901234567890121234567890123456789012345678901212345
3456789012345678901234567890121234567890123456789012345678901212345
6
12
D.
bolster
D.
penetrable.
.impregnable
3456789012345678901234567890121234567890123456789012345678901212345 6
12
3456789012345678901234567890121234567890123456789012345678901212345
6
12
E. gauge
E. translucent. .opaque
3456789012345678901234567890121234567890123456789012345678901212345
6
12
2
3456789012345678901234567890121234567890123456789012345678901212345
6
1
3456789012345678901234567890121234567890123456789012345678901212345
6
12
20. It is a commonly noted behavior for a
23. Historiography, at least of historic
3456789012345678901234567890121234567890123456789012345678901212345
6
12
3456789012345678901234567890121234567890123456789012345678901212345
6
12
younger
sibling
to
________
the
behavior
personages,
in
the
last
century
tended
3456789012345678901234567890121234567890123456789012345678901212345 6
12
2
3456789012345678901234567890121234567890123456789012345678901212345
6
123456789012345678901234567890121234567890123456789012345678901212345
of an older sibling, taking on their
toward hagiography, but today
6
123456789012345678901234567890121234567890123456789012345678901212345
6
123456789012345678901234567890121234567890123456789012345678901212345
mannerisms
or
sense
of
style.
__________
distance
and
a
__________
123456789012345678901234567890121234567890123456789012345678901212345 66
123456789012345678901234567890121234567890123456789012345678901212345
tone are basic requirements of profes6
123456789012345678901234567890121234567890123456789012345678901212345
A. antagonize
6
1
sional
history
writing.
3456789012345678901234567890121234567890123456789012345678901212345
6
12
B.
ridicule
3456789012345678901234567890121234567890123456789012345678901212345 6
12
3456789012345678901234567890121234567890123456789012345678901212345
6
12
C. improve
A. critical. .reserved
3456789012345678901234567890121234567890123456789012345678901212345
6
12
3456789012345678901234567890121234567890123456789012345678901212345
6
12
D.
emulate
B.
minute.
.extravagant
3456789012345678901234567890121234567890123456789012345678901212345 6
12
3456789012345678901234567890121234567890123456789012345678901212345
6
12
E. vex
C. impartial. .subdued
3456789012345678901234567890121234567890123456789012345678901212345
6
12
2
3456789012345678901234567890121234567890123456789012345678901212345
6
123456789012345678901234567890121234567890123456789012345678901212345
D. holistic. .daring
6
123456789012345678901234567890121234567890123456789012345678901212345
21. Since the visitor was unfamiliar with the
6
123456789012345678901234567890121234567890123456789012345678901212345
E. even. .careful
6
123456789012345678901234567890121234567890123456789012345678901212345
regions
customs
he
was
unaware
of
the
6
1
2
3456789012345678901234567890121234567890123456789012345678901212345
6
123456789012345678901234567890121234567890123456789012345678901212345
__________ of his actions, and so he was
6
123456789012345678901234567890121234567890123456789012345678901212345
6
1
__________
to
the
fact
that
he
had
123456789012345678901234567890121234567890123456789012345678901212345 6
3456789012345678901234567890121234567890123456789012345678901212345
6
12
offended his host.
3456789012345678901234567890121234567890123456789012345678901212345
6
12
3456789012345678901234567890121234567890123456789012345678901212345 6
12
6
123456789012345678901234567890121234567890123456789012345678901212345
A. impropriety. .oblivious
3456789012345678901234567890121234567890123456789012345678901212345
6
12
3456789012345678901234567890121234567890123456789012345678901212345
6
12
B.
curtness.
.ignorant
2
3456789012345678901234567890121234567890123456789012345678901212345
123456789012345678901234567890121234567890123456789012345678901212345 66
123456789012345678901234567890121234567890123456789012345678901212345
C. blandness. .blithe
6
1
6
123456789012345678901234567890121234567890123456789012345678901212345
D. boldness. .unaware
123456789012345678901234567890121234567890123456789012345678901212345 6
2
3456789012345678901234567890121234567890123456789012345678901212345
6
123456789012345678901234567890121234567890123456789012345678901212345
E. audacity. .blind
6
123456789012345678901234567890121234567890123456789012345678901212345
123456789012345678901234567890121234567890123456789012345678901212345 66
123456789012345678901234567890121234567890123456789012345678901212345 6
1
3456789012345678901234567890121234567890123456789012345678901212345 6
12
3456789012345678901234567890121234567890123456789012345678901212345 6
12
3456789012345678901234567890121234567890123456789012345678901212345 6
12
3456789012345678901234567890121234567890123456789012345678901212345 6
12
123456789012345678901234567890121234567890123456789012345678901212345 6
2
6
123456789012345678901234567890121234567890123456789012345678901212345
6
123456789012345678901234567890121234567890123456789012345678901212345
6
123456789012345678901234567890121234567890123456789012345678901212345
6
123456789012345678901234567890121234567890123456789012345678901212345
1 3456789012345678901234567890121234567890123456789012345678901212345 6
2
3456789012345678901234567890121234567890123456789012345678901212345
123456789012345678901234567890121234567890123456789012345678901212345 66
1
3456789012345678901234567890121234567890123456789012345678901212345 6
12
123456789012345678901234567890121234567890123456789012345678901212345 6
3456789012345678901234567890121234567890123456789012345678901212345 6
12
123456789012345678901234567890121234567890123456789012345678901212345 6
123456789012345678901234567890121234567890123456789012345678901212345 6
123456789012345678901234567890121234567890123456789012345678901212345 6
123456789012345678901234567890121234567890123456789012345678901212345 6 313
1234567890123456789012345678901212345678901234567890123456789012123456

http://www.xtremepapers.net

Part IV: Three Practice GREs

1234567890123456789012345678901212345678901234567890123456789012123456
123456789012345678901234567890121234567890123456789012345678901212345 6
3456789012345678901234567890121234567890123456789012345678901212345 6
12
Reading Comprehension
6
123456789012345678901234567890121234567890123456789012345678901212345
3456789012345678901234567890121234567890123456789012345678901212345 6
12
3456789012345678901234567890121234567890123456789012345678901212345 6
12
123456789012345678901234567890121234567890123456789012345678901212345 6
3456789012345678901234567890121234567890123456789012345678901212345
6
12
Directions: Questions follow each passage. After reading the passage, determine the best
6
123456789012345678901234567890121234567890123456789012345678901212345
3456789012345678901234567890121234567890123456789012345678901212345
6
12
answer for each question based on the passages content.
3456789012345678901234567890121234567890123456789012345678901212345 6
12
3456789012345678901234567890121234567890123456789012345678901212345 6
12
3456789012345678901234567890121234567890123456789012345678901212345 6
12
2
1 3456789012345678901234567890121234567890123456789012345678901212345 6
3456789012345678901234567890121234567890123456789012345678901212345 6
12
3456789012345678901234567890121234567890123456789012345678901212345
12
Passage 1
tance of the components neglected by the 6
3456789012345678901234567890121234567890123456789012345678901212345
6
12
3456789012345678901234567890121234567890123456789012345678901212345
semi-empirical method. Furthermore, the 6
12
Line
The
introduction
of
electronic
computers
2
3456789012345678901234567890121234567890123456789012345678901212345
6
1
ab initio practitioners criticized the
3456789012345678901234567890121234567890123456789012345678901212345
6
12
into the field of quantum chemistry in the
3456789012345678901234567890121234567890123456789012345678901212345
6
12
semi-empirical method as only able to
3456789012345678901234567890121234567890123456789012345678901212345
6
12
1950s
had
an
immediate
and
ultimately
3456789012345678901234567890121234567890123456789012345678901212345
6
12
(40)
sustain
the
current
theoretical
paradigm
2
3456789012345678901234567890121234567890123456789012345678901212345
6
1
polarizing effect on the discipline.
3456789012345678901234567890121234567890123456789012345678901212345
6
12
and not able to explore possible inad3456789012345678901234567890121234567890123456789012345678901212345
6
12
(5)
Quantum
chemistry
is
primarily
con3456789012345678901234567890121234567890123456789012345678901212345
6
12
equacies
in
the
current
framework.
The
3456789012345678901234567890121234567890123456789012345678901212345
6
12
cerned
with
solving
Schrdingers
2
3456789012345678901234567890121234567890123456789012345678901212345
123456789012345678901234567890121234567890123456789012345678901212345
semi-empirical practitioners, on the other 6
6
123456789012345678901234567890121234567890123456789012345678901212345
Equationthe fundamental equation of
6
123456789012345678901234567890121234567890123456789012345678901212345
hand,
criticized
the
ab
initio
methodol6
123456789012345678901234567890121234567890123456789012345678901212345
the
quantum
realmfor
different
6
123456789012345678901234567890121234567890123456789012345678901212345
(45)
ogy
as
overly
laborious
and
ultimately
6
123456789012345678901234567890121234567890123456789012345678901212345
theoretical scenarios. The difficulty in
6
1
less
productive.
Since
the
ab
initio
3456789012345678901234567890121234567890123456789012345678901212345
6
12
(10)
this
endeavor
is
that
Schrdingers
3456789012345678901234567890121234567890123456789012345678901212345
6
12
methodology
solved
all
computational
3456789012345678901234567890121234567890123456789012345678901212345
6
12
Equation does not readily submit to
2
3456789012345678901234567890121234567890123456789012345678901212345
123456789012345678901234567890121234567890123456789012345678901212345
components to Schrdingers Equation, it 6
6
123456789012345678901234567890121234567890123456789012345678901212345
straightforward solutions. Elaborate and
6
123456789012345678901234567890121234567890123456789012345678901212345
demanded
considerable
amounts
of
time
6
123456789012345678901234567890121234567890123456789012345678901212345
extensive calculations are often necessary
6
123456789012345678901234567890121234567890123456789012345678901212345
(50)
and effort, so fewer research projects
6
123456789012345678901234567890121234567890123456789012345678901212345
for solving Schrdingers Equation,
123456789012345678901234567890121234567890123456789012345678901212345
could be pursued with this methodology. 6
(15)
which explains why the introduction of
6
123456789012345678901234567890121234567890123456789012345678901212345
123456789012345678901234567890121234567890123456789012345678901212345
Ultimately, both methodologies have 6
electronic computing held great promise
6
1
3456789012345678901234567890121234567890123456789012345678901212345
12
proved important to quantum chemistry. 6
as a research tool in theoretical quantum
3456789012345678901234567890121234567890123456789012345678901212345
6
12
The semi-empirical method has been
6
123456789012345678901234567890121234567890123456789012345678901212345
chemistry.
6
123456789012345678901234567890121234567890123456789012345678901212345
(55)
essential for exploring questions within
3456789012345678901234567890121234567890123456789012345678901212345
6
12
As
the
discipline
incorporated
the
use
2
3456789012345678901234567890121234567890123456789012345678901212345
6
123456789012345678901234567890121234567890123456789012345678901212345
the standard paradigm while the ab initio 6
123456789012345678901234567890121234567890123456789012345678901212345
6
1 (20) of electronic computers, two research
method has probed the boundaries of our 6
3456789012345678901234567890121234567890123456789012345678901212345
12
methodologies
emerged.
The
first,
termed
3456789012345678901234567890121234567890123456789012345678901212345
6
12
understanding of quantum chemistry.
3456789012345678901234567890121234567890123456789012345678901212345
6
12
ab initio, relied entirely upon the
3456789012345678901234567890121234567890123456789012345678901212345
6
12
6
123456789012345678901234567890121234567890123456789012345678901212345
electronic
computer
to
solve
the
knotty
123456789012345678901234567890121234567890123456789012345678901212345 6
6
123456789012345678901234567890121234567890123456789012345678901212345
mathematics of Schrdingers Equation,
3456789012345678901234567890121234567890123456789012345678901212345
6
12
3456789012345678901234567890121234567890123456789012345678901212345
6
12
(25)
and
it
employed
no
empirical
data
save
3456789012345678901234567890121234567890123456789012345678901212345 6
12
3456789012345678901234567890121234567890123456789012345678901212345
6
12
fundamental constants of nature. The
6
123456789012345678901234567890121234567890123456789012345678901212345
2
3456789012345678901234567890121234567890123456789012345678901212345
6
123456789012345678901234567890121234567890123456789012345678901212345
second, the so-called semi-empirical
6
123456789012345678901234567890121234567890123456789012345678901212345
6
123456789012345678901234567890121234567890123456789012345678901212345
method, simplified the computational
123456789012345678901234567890121234567890123456789012345678901212345 66
1
hurdles by neglecting possibly negligible
3456789012345678901234567890121234567890123456789012345678901212345
6
12
3456789012345678901234567890121234567890123456789012345678901212345
6
12
(30)
components of the calculations and by
3456789012345678901234567890121234567890123456789012345678901212345 6
12
3456789012345678901234567890121234567890123456789012345678901212345
6
12
extensively employing empirical data.
6
123456789012345678901234567890121234567890123456789012345678901212345
The
semi-empirical
method
quickly
2
3456789012345678901234567890121234567890123456789012345678901212345
123456789012345678901234567890121234567890123456789012345678901212345 66
1
became very fruitful, but the ab initio
3456789012345678901234567890121234567890123456789012345678901212345
6
12
6
123456789012345678901234567890121234567890123456789012345678901212345
practitioners
were
skeptical
of
this
3456789012345678901234567890121234567890123456789012345678901212345 6
12
2
3456789012345678901234567890121234567890123456789012345678901212345
6
1 (35) research because of the possible impor123456789012345678901234567890121234567890123456789012345678901212345 6
123456789012345678901234567890121234567890123456789012345678901212345 66
314 123456789012345678901234567890121234567890123456789012345678901212345
1234567890123456789012345678901212345678901234567890123456789012123456

www.petersons.com

http://www.xtremepapers.net

TEST 3
1234567890123456789012345678901212345678901234567890123456789012123456
123456789012345678901234567890121234567890123456789012345678901212345 6
3456789012345678901234567890121234567890123456789012345678901212345 6
12
24. The ab initio method was criticized for
26. The semi-empirical method was empirical 6
123456789012345678901234567890121234567890123456789012345678901212345
3456789012345678901234567890121234567890123456789012345678901212345
6
12
in that it employed
3456789012345678901234567890121234567890123456789012345678901212345
6
12
I.
being
too
demanding
with
regard
to
123456789012345678901234567890121234567890123456789012345678901212345 6
3456789012345678901234567890121234567890123456789012345678901212345
12
labor.
A. strictly empirical means in obtaining a 6
6
123456789012345678901234567890121234567890123456789012345678901212345
3456789012345678901234567890121234567890123456789012345678901212345
6
12
II.
its
inability
to
critique
the
current
result.
3456789012345678901234567890121234567890123456789012345678901212345 6
12
3456789012345678901234567890121234567890123456789012345678901212345
6
12
paradigm.
B. the fundamental constants of nature
3456789012345678901234567890121234567890123456789012345678901212345
6
12
2
3456789012345678901234567890121234567890123456789012345678901212345
6
1
III. being a slower producer of new
in obtaining a result.
3456789012345678901234567890121234567890123456789012345678901212345
6
12
3456789012345678901234567890121234567890123456789012345678901212345
6
12
research.
C.
extensive
empirical
data
in
obtaining
3456789012345678901234567890121234567890123456789012345678901212345 6
12
3456789012345678901234567890121234567890123456789012345678901212345
6
12
a result.
6
123456789012345678901234567890121234567890123456789012345678901212345
A. I only
3456789012345678901234567890121234567890123456789012345678901212345
6
12
D.
only
empirically
verifiable
scenarios
in
3456789012345678901234567890121234567890123456789012345678901212345
6
12
B. II only
3456789012345678901234567890121234567890123456789012345678901212345
6
12
modeling
quantum
outcomes.
3456789012345678901234567890121234567890123456789012345678901212345
6
12
C. III only
2
3456789012345678901234567890121234567890123456789012345678901212345
6
1
E. restricted amounts of empirical data
3456789012345678901234567890121234567890123456789012345678901212345
6
12
D. I and II only
3456789012345678901234567890121234567890123456789012345678901212345
6
12
in obtaining results.
3456789012345678901234567890121234567890123456789012345678901212345
6
E. I and III only
12
3456789012345678901234567890121234567890123456789012345678901212345 6
12
2
3456789012345678901234567890121234567890123456789012345678901212345
6
123456789012345678901234567890121234567890123456789012345678901212345
27. The authors intent was to
6
123456789012345678901234567890121234567890123456789012345678901212345
25.
According
to
the
passage,
what
was
the
123456789012345678901234567890121234567890123456789012345678901212345 66
123456789012345678901234567890121234567890123456789012345678901212345
primary methodological challenge in
A. explain the effects of the introduction 6
123456789012345678901234567890121234567890123456789012345678901212345
6
123456789012345678901234567890121234567890123456789012345678901212345
working
with
Schrdingers
Equation?
of electronic computers on quantum
6
1
3456789012345678901234567890121234567890123456789012345678901212345
6
12
chemistry.
3456789012345678901234567890121234567890123456789012345678901212345
6
12
A.
The
difficulty
of
correctly
applying
3456789012345678901234567890121234567890123456789012345678901212345
6
12
B.
point
out
the
harmfulness
of
scientific
2
3456789012345678901234567890121234567890123456789012345678901212345
6
123456789012345678901234567890121234567890123456789012345678901212345
Schrdengers Equation in the
6
123456789012345678901234567890121234567890123456789012345678901212345
rivalries.
6
123456789012345678901234567890121234567890123456789012345678901212345
quantum world
6
123456789012345678901234567890121234567890123456789012345678901212345
C.
argue
that
the
semi-empirical
method
6
123456789012345678901234567890121234567890123456789012345678901212345
B. The complexity of the calculations
6
123456789012345678901234567890121234567890123456789012345678901212345
was more productive.
6
123456789012345678901234567890121234567890123456789012345678901212345
required to solve Schrdingers
6
123456789012345678901234567890121234567890123456789012345678901212345
D. illustrate the importance of cooperaEquation
6
123456789012345678901234567890121234567890123456789012345678901212345
6
1
tion in scientific disciplines.
C.
The
inability
of
electronic
computers
2
3456789012345678901234567890121234567890123456789012345678901212345
123456789012345678901234567890121234567890123456789012345678901212345 66
123456789012345678901234567890121234567890123456789012345678901212345
E. lay out the differences between the ab
to solve Schrdingers Equation
6
1
initio and semi-empirical methods.
6
123456789012345678901234567890121234567890123456789012345678901212345
D.
The
rivalry
between
the
ab
initio
3456789012345678901234567890121234567890123456789012345678901212345 6
12
2
3456789012345678901234567890121234567890123456789012345678901212345
6
123456789012345678901234567890121234567890123456789012345678901212345
practitioners and the semi-empirical
6
123456789012345678901234567890121234567890123456789012345678901212345
Passage
2
6
1
practitioners
3456789012345678901234567890121234567890123456789012345678901212345 6
12
3456789012345678901234567890121234567890123456789012345678901212345
12
Line Though nearly fifty years of research has 6
E. The difficulty of obtaining reliable
3456789012345678901234567890121234567890123456789012345678901212345
6
12
3456789012345678901234567890121234567890123456789012345678901212345
6
12
been
done
since
the
Khartoum
Hospital
empirical
data
123456789012345678901234567890121234567890123456789012345678901212345 6
6
123456789012345678901234567890121234567890123456789012345678901212345
and Shaheinab excavations in Central
6
123456789012345678901234567890121234567890123456789012345678901212345
2
3456789012345678901234567890121234567890123456789012345678901212345
6
123456789012345678901234567890121234567890123456789012345678901212345
Sudan, a coherent and rigorous frame6
123456789012345678901234567890121234567890123456789012345678901212345
6
123456789012345678901234567890121234567890123456789012345678901212345
(5)
work for understanding the Mesolithic123456789012345678901234567890121234567890123456789012345678901212345 66
1
Neolithic (8000 B.C.E.4000 B.C.E.)
3456789012345678901234567890121234567890123456789012345678901212345
6
12
transition in the Central Nile has not
3456789012345678901234567890121234567890123456789012345678901212345
6
12
3456789012345678901234567890121234567890123456789012345678901212345
6
12
been achieved. A. J. Arkell, the primary
3456789012345678901234567890121234567890123456789012345678901212345
6
12
6
123456789012345678901234567890121234567890123456789012345678901212345
excavator
at
these
sites,
developed
a
2
3456789012345678901234567890121234567890123456789012345678901212345
123456789012345678901234567890121234567890123456789012345678901212345 66
123456789012345678901234567890121234567890123456789012345678901212345
(10)
model of cultural evolution based on two 6
123456789012345678901234567890121234567890123456789012345678901212345
123456789012345678901234567890121234567890123456789012345678901212345
basic ceramic typologiesthe wavy and 6
6
1
2
3456789012345678901234567890121234567890123456789012345678901212345
6
123456789012345678901234567890121234567890123456789012345678901212345
dotted linefound at these sites.
6
1
3456789012345678901234567890121234567890123456789012345678901212345
6
12
Predictably,
the
wavy
line
type
are
123456789012345678901234567890121234567890123456789012345678901212345 6
3456789012345678901234567890121234567890123456789012345678901212345
6
12
ceramics whose outer surface are
6
123456789012345678901234567890121234567890123456789012345678901212345
123456789012345678901234567890121234567890123456789012345678901212345 6
123456789012345678901234567890121234567890123456789012345678901212345 6
123456789012345678901234567890121234567890123456789012345678901212345 6 315
1234567890123456789012345678901212345678901234567890123456789012123456

http://www.xtremepapers.net

Part IV: Three Practice GREs

1234567890123456789012345678901212345678901234567890123456789012123456
123456789012345678901234567890121234567890123456789012345678901212345 6
3456789012345678901234567890121234567890123456789012345678901212345 6
12
(15)
decorated with wavy lines, while the
(55)
chronological index for the region? Does 6
123456789012345678901234567890121234567890123456789012345678901212345
3456789012345678901234567890121234567890123456789012345678901212345
6
12
dotted line ceramics are decorated with
their geographical dispersion tell us of
3456789012345678901234567890121234567890123456789012345678901212345
6
12
6
123456789012345678901234567890121234567890123456789012345678901212345
the
same
grooves
of
the
wavy
line
but
possible
cultural
uniformities
across
these
3456789012345678901234567890121234567890123456789012345678901212345 6
12
2
3456789012345678901234567890121234567890123456789012345678901212345
6
1
with indentions, or dots, further lining
diverse regions? The answers these to
3456789012345678901234567890121234567890123456789012345678901212345
6
12
3456789012345678901234567890121234567890123456789012345678901212345
6
12
the
grooves.
questions
about
the
past
hopefully
lie
3456789012345678901234567890121234567890123456789012345678901212345 6
12
3456789012345678901234567890121234567890123456789012345678901212345
12
(20)
Arkells model designates the wavy
(60)
somewhere in the not-too-distant future. 6
6
123456789012345678901234567890121234567890123456789012345678901212345
3456789012345678901234567890121234567890123456789012345678901212345
6
12
line
type
as
markers
for
Khartoum
3456789012345678901234567890121234567890123456789012345678901212345 6
12
3456789012345678901234567890121234567890123456789012345678901212345
6
12
Mesolithic culture and the dotted line
3456789012345678901234567890121234567890123456789012345678901212345
6
12
28. The main idea of the passage is
2
3456789012345678901234567890121234567890123456789012345678901212345
6
1
type as markers for Khartoum Neolithic
3456789012345678901234567890121234567890123456789012345678901212345
6
12
A. Nilotic and Saharan archaeology has 6
3456789012345678901234567890121234567890123456789012345678901212345
12
culture.
Yet
this
model
became
problem3456789012345678901234567890121234567890123456789012345678901212345
6
12
failed to provide an adequate frame3456789012345678901234567890121234567890123456789012345678901212345
6
12
(25)
atic
as
excavations
beyond
the
Central
2
3456789012345678901234567890121234567890123456789012345678901212345
6
1
work
for
understanding
the
Me3456789012345678901234567890121234567890123456789012345678901212345
6
12
Nile
began
to
unearth
wavy
and
dotted
3456789012345678901234567890121234567890123456789012345678901212345
6
12
solithic-Neolithic
transition.
3456789012345678901234567890121234567890123456789012345678901212345
6
12
line
pottery
across
the
Sahara-Sahel
belt
3456789012345678901234567890121234567890123456789012345678901212345
6
12
B.
Arkell
and
Canerva
were
incorrect
in
2
3456789012345678901234567890121234567890123456789012345678901212345
6
123456789012345678901234567890121234567890123456789012345678901212345
in strata and in sequences inconsistent
6
123456789012345678901234567890121234567890123456789012345678901212345
their
respective
attempts
to
system6
123456789012345678901234567890121234567890123456789012345678901212345
with Arkells designations. It appeared a
6
123456789012345678901234567890121234567890123456789012345678901212345
atize
their
research.
6
123456789012345678901234567890121234567890123456789012345678901212345
(30)
new model was needed to account for
123456789012345678901234567890121234567890123456789012345678901212345
C. wavy line ceramics clearly are not the 6
6
1
not only chronological development but
3456789012345678901234567890121234567890123456789012345678901212345
6
12
chronological
predecessors
of
dotted
geographical dispersal as well.
3456789012345678901234567890121234567890123456789012345678901212345
6
12
3456789012345678901234567890121234567890123456789012345678901212345
6
12
line
ceramics.
To
address
these
issues,
I.
Canerva
2
3456789012345678901234567890121234567890123456789012345678901212345
123456789012345678901234567890121234567890123456789012345678901212345 66
123456789012345678901234567890121234567890123456789012345678901212345
D. dotted and wavy line pottery has not 6
put forth a classification strategy for
123456789012345678901234567890121234567890123456789012345678901212345
123456789012345678901234567890121234567890123456789012345678901212345
been successfully incorporated into a 6
(35)
Nilotic and Saharan ceramics based on
6
123456789012345678901234567890121234567890123456789012345678901212345
6
123456789012345678901234567890121234567890123456789012345678901212345
larger theory that explains their
technique. Arkells model classified by
6
123456789012345678901234567890121234567890123456789012345678901212345
6
123456789012345678901234567890121234567890123456789012345678901212345
importance, thus leaving important
motif,
the
ceramics
outer
decorations,
123456789012345678901234567890121234567890123456789012345678901212345 66
1
further research open.
but Canerva focused on the techniques
3456789012345678901234567890121234567890123456789012345678901212345
6
12
3456789012345678901234567890121234567890123456789012345678901212345
6
12
E.
research on Nilotic and Saharan
employed,
such
as
method
of
hardening,
6
123456789012345678901234567890121234567890123456789012345678901212345
ceramics has matured over the past
6
123456789012345678901234567890121234567890123456789012345678901212345
(40)
and
technologies
needed,
such
as
tools,
in
3456789012345678901234567890121234567890123456789012345678901212345
6
12
five
decades.
2
3456789012345678901234567890121234567890123456789012345678901212345
6
123456789012345678901234567890121234567890123456789012345678901212345
making the ceramics. This strategy
6
123456789012345678901234567890121234567890123456789012345678901212345
6
1
initially
promised
to
provide
artifact
3456789012345678901234567890121234567890123456789012345678901212345
6
12
29.
Arkells
model
was
problematic
in
that
3456789012345678901234567890121234567890123456789012345678901212345
6
12
markers
for
the
cultural
evolution
from
2
3456789012345678901234567890121234567890123456789012345678901212345
123456789012345678901234567890121234567890123456789012345678901212345 66
123456789012345678901234567890121234567890123456789012345678901212345
the Nile basin to the western edges of the
A. the discovery of wavy and dotted line 6
1
123456789012345678901234567890121234567890123456789012345678901212345
(45)
Sahara in the Mesolithic-Neolithic. This
pottery in the Central Nile conflicted 6
6
123456789012345678901234567890121234567890123456789012345678901212345
2
3456789012345678901234567890121234567890123456789012345678901212345
6
123456789012345678901234567890121234567890123456789012345678901212345
approach, though, became mired in
with his findings.
6
123456789012345678901234567890121234567890123456789012345678901212345
123456789012345678901234567890121234567890123456789012345678901212345
questions related to determination of
B. dating discrepancies were discovered 6
123456789012345678901234567890121234567890123456789012345678901212345 66
1
technique, as the reverse inference from
within his account.
3456789012345678901234567890121234567890123456789012345678901212345
6
12
motif to technique became dubious.
C. wavy and dotted line pottery were
3456789012345678901234567890121234567890123456789012345678901212345
6
12
3456789012345678901234567890121234567890123456789012345678901212345
6
12
(50)
At this point major questions remain
discovered outside the Central Nile
3456789012345678901234567890121234567890123456789012345678901212345
6
12
6
123456789012345678901234567890121234567890123456789012345678901212345
unanswered
concerning
wavy
and
dotted
that
did
not
conform
to
his
model.
2
3456789012345678901234567890121234567890123456789012345678901212345
123456789012345678901234567890121234567890123456789012345678901212345 66
123456789012345678901234567890121234567890123456789012345678901212345
line ceramics in Mesolithic-Neolithic
D. pottery fragments were discovered
6
123456789012345678901234567890121234567890123456789012345678901212345
6
123456789012345678901234567890121234567890123456789012345678901212345
Nilotic
and
Saharan
regions.
Do
these
outside
the
Central
Nile
as
far
west
as
6
1
2
3456789012345678901234567890121234567890123456789012345678901212345
6
123456789012345678901234567890121234567890123456789012345678901212345
ceramic typologies provide any sort of
the Western Sahara.
6
1
3456789012345678901234567890121234567890123456789012345678901212345
6
12
E.
ceramics
that
chronologically
pre123456789012345678901234567890121234567890123456789012345678901212345 6
3456789012345678901234567890121234567890123456789012345678901212345
6
12
ceded the wavy line pottery were
6
123456789012345678901234567890121234567890123456789012345678901212345
6
123456789012345678901234567890121234567890123456789012345678901212345
discovered
in
the
Central
Nile.
123456789012345678901234567890121234567890123456789012345678901212345 66
316 123456789012345678901234567890121234567890123456789012345678901212345
1234567890123456789012345678901212345678901234567890123456789012123456

www.petersons.com

http://www.xtremepapers.net

TEST 3
1234567890123456789012345678901212345678901234567890123456789012123456
123456789012345678901234567890121234567890123456789012345678901212345 6
3456789012345678901234567890121234567890123456789012345678901212345 6
12
30. It can be inferred from the passage that
6
123456789012345678901234567890121234567890123456789012345678901212345
3456789012345678901234567890121234567890123456789012345678901212345
6
12
the author would agree with which one of
3456789012345678901234567890121234567890123456789012345678901212345
6
12
6
123456789012345678901234567890121234567890123456789012345678901212345
the
following?
3456789012345678901234567890121234567890123456789012345678901212345 6
12
2
3456789012345678901234567890121234567890123456789012345678901212345
6
1
3456789012345678901234567890121234567890123456789012345678901212345
6
12
A.
The
work
of
Arkell
and
Canerva
has
3456789012345678901234567890121234567890123456789012345678901212345 6
12
3456789012345678901234567890121234567890123456789012345678901212345
6
12
been thoroughly debunked.
3456789012345678901234567890121234567890123456789012345678901212345
6
12
2
3456789012345678901234567890121234567890123456789012345678901212345
6
1
B. Future research into Mesolithic3456789012345678901234567890121234567890123456789012345678901212345
6
12
3456789012345678901234567890121234567890123456789012345678901212345
6
12
Neolithic
Khartoum
should
be
done.
3456789012345678901234567890121234567890123456789012345678901212345 6
12
3456789012345678901234567890121234567890123456789012345678901212345
6
12
C. Motif should be considered over
6
123456789012345678901234567890121234567890123456789012345678901212345
3456789012345678901234567890121234567890123456789012345678901212345
6
12
technique
when
researching
ceramics.
3456789012345678901234567890121234567890123456789012345678901212345 6
12
3456789012345678901234567890121234567890123456789012345678901212345
6
12
D. The material composition of a
3456789012345678901234567890121234567890123456789012345678901212345
6
12
2
3456789012345678901234567890121234567890123456789012345678901212345
6
1
ceramic is an important clue in
3456789012345678901234567890121234567890123456789012345678901212345
6
12
3456789012345678901234567890121234567890123456789012345678901212345
6
12
archaeological research.
3456789012345678901234567890121234567890123456789012345678901212345 6
12
3456789012345678901234567890121234567890123456789012345678901212345
6
12
E. Material culture is more readily
3456789012345678901234567890121234567890123456789012345678901212345
6
12
3456789012345678901234567890121234567890123456789012345678901212345
6
12
studied than non-material culture.
3456789012345678901234567890121234567890123456789012345678901212345 6
12
3456789012345678901234567890121234567890123456789012345678901212345 6
12
3456789012345678901234567890121234567890123456789012345678901212345 6
12
2
6
123456789012345678901234567890121234567890123456789012345678901212345
1 3456789012345678901234567890121234567890123456789012345678901212345 6
3456789012345678901234567890121234567890123456789012345678901212345 6
12
3456789012345678901234567890121234567890123456789012345678901212345 6
12
3456789012345678901234567890121234567890123456789012345678901212345 6
12
2
6
123456789012345678901234567890121234567890123456789012345678901212345
6
123456789012345678901234567890121234567890123456789012345678901212345
6
123456789012345678901234567890121234567890123456789012345678901212345
6
123456789012345678901234567890121234567890123456789012345678901212345
6
123456789012345678901234567890121234567890123456789012345678901212345
3456789012345678901234567890121234567890123456789012345678901212345
123456789012345678901234567890121234567890123456789012345678901212345 66
123456789012345678901234567890121234567890123456789012345678901212345 6
123456789012345678901234567890121234567890123456789012345678901212345 6
123456789012345678901234567890121234567890123456789012345678901212345 6
1
3456789012345678901234567890121234567890123456789012345678901212345 6
12
3456789012345678901234567890121234567890123456789012345678901212345 6
12
123456789012345678901234567890121234567890123456789012345678901212345 6
123456789012345678901234567890121234567890123456789012345678901212345 6
3456789012345678901234567890121234567890123456789012345678901212345 6
12
3456789012345678901234567890121234567890123456789012345678901212345 6
12
3456789012345678901234567890121234567890123456789012345678901212345 6
12
123456789012345678901234567890121234567890123456789012345678901212345 6
3456789012345678901234567890121234567890123456789012345678901212345 6
12
3456789012345678901234567890121234567890123456789012345678901212345 6
12
2
6
123456789012345678901234567890121234567890123456789012345678901212345
6
123456789012345678901234567890121234567890123456789012345678901212345
1 3456789012345678901234567890121234567890123456789012345678901212345 6
123456789012345678901234567890121234567890123456789012345678901212345 6
123456789012345678901234567890121234567890123456789012345678901212345 6
3456789012345678901234567890121234567890123456789012345678901212345 6
12
3456789012345678901234567890121234567890123456789012345678901212345 6
12
3456789012345678901234567890121234567890123456789012345678901212345 6
12
3456789012345678901234567890121234567890123456789012345678901212345 6
12
123456789012345678901234567890121234567890123456789012345678901212345 6
2
6
123456789012345678901234567890121234567890123456789012345678901212345
6
123456789012345678901234567890121234567890123456789012345678901212345
6
123456789012345678901234567890121234567890123456789012345678901212345
6
123456789012345678901234567890121234567890123456789012345678901212345
1 3456789012345678901234567890121234567890123456789012345678901212345 6
2
3456789012345678901234567890121234567890123456789012345678901212345
123456789012345678901234567890121234567890123456789012345678901212345 66
123456789012345678901234567890121234567890123456789012345678901212345 6
123456789012345678901234567890121234567890123456789012345678901212345 6
123456789012345678901234567890121234567890123456789012345678901212345 6
1
3456789012345678901234567890121234567890123456789012345678901212345 6
12
123456789012345678901234567890121234567890123456789012345678901212345 6
3456789012345678901234567890121234567890123456789012345678901212345 6
12
123456789012345678901234567890121234567890123456789012345678901212345 6
3456789012345678901234567890121234567890123456789012345678901212345 6
12
123456789012345678901234567890121234567890123456789012345678901212345 6
123456789012345678901234567890121234567890123456789012345678901212345 6
123456789012345678901234567890121234567890123456789012345678901212345 6
123456789012345678901234567890121234567890123456789012345678901212345 6 317
1234567890123456789012345678901212345678901234567890123456789012123456

http://www.xtremepapers.net

Answers and Explanations


123456789012345678901234567890121234567890123456789012
2
12345678901234567890123456789012123456789012345678901
2
12345678901234567890123456789012123456789012345678901
This
is
the
last
sample
test
you
will
take
before
moving
onto
the
computer
2345678901234567890123456789012123456789012345678901
2
1
2
12345678901234567890123456789012123456789012345678901
adaptive exams. Hopefully, you have tested out some new strategies and 2
12345678901234567890123456789012123456789012345678901
2
12345678901234567890123456789012123456789012345678901
ideas on the previous two tests and are now comfortable with them.
2
12345678901234567890123456789012123456789012345678901
2345678901234567890123456789012123456789012345678901
12345678901234567890123456789012123456789012345678901 2
12345678901234567890123456789012123456789012345678901 2
12345678901234567890123456789012123456789012345678901 2
12345678901234567890123456789012123456789012345678901 2
2
12345678901234567890123456789012123456789012345678901
Quantitative
2
12345678901234567890123456789012123456789012345678901
2
1
1. B The only way to do a problem like this is to simplify the 2
12345678901234567890123456789012123456789012345678901
2
12345678901234567890123456789012123456789012345678901
expressions.
2
12345678901234567890123456789012123456789012345678901
2345678901234567890123456789012123456789012345678901
12345678901234567890123456789012123456789012345678901 2
2
12345678901234567890123456789012123456789012345678901
2
For Column B, 3 5 9.
2
12345678901234567890123456789012123456789012345678901
12345678901234567890123456789012123456789012345678901 2
2
12345678901234567890123456789012123456789012345678901
For Column A, =32 1 72 5 =9 1 49 5 =58 , 8 since 2
12345678901234567890123456789012123456789012345678901
12345678901234567890123456789012123456789012345678901 2
2
12345678901234567890123456789012123456789012345678901
=64 5 8.
2
12345678901234567890123456789012123456789012345678901
2
1
2345678901234567890123456789012123456789012345678901
2
12345678901234567890123456789012123456789012345678901
Column B is greater.
12345678901234567890123456789012123456789012345678901 2
1 2. C To be able to compare 2y to 6, you need to solve for y. You can 2
12345678901234567890123456789012123456789012345678901 2
2
12345678901234567890123456789012123456789012345678901
do this by substituting y for x in the first equation. This yields:
2
12345678901234567890123456789012123456789012345678901
2
12345678901234567890123456789012123456789012345678901
2
12345678901234567890123456789012123456789012345678901
x 1 2y 5 9
2
12345678901234567890123456789012123456789012345678901
2
12345678901234567890123456789012123456789012345678901
2345678901234567890123456789012123456789012345678901
2
12345678901234567890123456789012123456789012345678901
y 1 2y 5 9
2
12345678901234567890123456789012123456789012345678901
2
1
3y
5
9

12345678901234567890123456789012123456789012345678901 2
2
12345678901234567890123456789012123456789012345678901
y53
2
12345678901234567890123456789012123456789012345678901
2
12345678901234567890123456789012123456789012345678901
2
12345678901234567890123456789012123456789012345678901
This
means
2y
5
6.
Choice
(C)
then
is
the
answer.
2
12345678901234567890123456789012123456789012345678901
12345678901234567890123456789012123456789012345678901 2
2345678901234567890123456789012123456789012345678901
12345678901234567890123456789012123456789012345678901
3. D A good way to attack a problem like this is try a few values for 2
2
12345678901234567890123456789012123456789012345678901
12345678901234567890123456789012123456789012345678901
x. At its smallest x 5 1, and Column A would equal two thirds, 2
12345678901234567890123456789012123456789012345678901 2
1
or 0.67. If x 5 2, Column A equals eight ninths, and if x 5 3, 2
2
12345678901234567890123456789012123456789012345678901
12345678901234567890123456789012123456789012345678901
Column A equals twenty-six twenty-sevenths. As you can see, 2
2
12345678901234567890123456789012123456789012345678901
12345678901234567890123456789012123456789012345678901
Column A increases as x increases. In the first case, Column B 2
2
12345678901234567890123456789012123456789012345678901
was greater; in the third, Column A was. Thus the answer is (D). 2
12345678901234567890123456789012123456789012345678901
12345678901234567890123456789012123456789012345678901 2
2
12345678901234567890123456789012123456789012345678901
12345678901234567890123456789012123456789012345678901 2
2
12345678901234567890123456789012123456789012345678901
12345678901234567890123456789012123456789012345678901 2
12345678901234567890123456789012123456789012345678901 2
12345678901234567890123456789012123456789012345678901 2
12345678901234567890123456789012123456789012345678901 2
123456789012345678901234567890121234567890123456789012
318

http://www.xtremepapers.net

ANSWERS
123456789012345678901234567890121234567890123456789012
12345678901234567890123456789012123456789012345678901 2
2
12345678901234567890123456789012123456789012345678901
4. A First, simplify the expression in Column A,
2
12345678901234567890123456789012123456789012345678901
2
12345678901234567890123456789012123456789012345678901
2
12345678901234567890123456789012123456789012345678901
1
1
2
12345678901234567890123456789012123456789012345678901
2
2 2
2 2
b 1 2bc 1 c ! 5 @~b 1 c! # 5 b 1 c.
~
2
12345678901234567890123456789012123456789012345678901
2345678901234567890123456789012123456789012345678901
2
1
2
12345678901234567890123456789012123456789012345678901
b
1
c
5
180
.
0.90
2
12345678901234567890123456789012123456789012345678901
2
12345678901234567890123456789012123456789012345678901
2
12345678901234567890123456789012123456789012345678901
5.
D
Remember
that
an
isosceles
triangle
is
not
necessarily
a
right
2345678901234567890123456789012123456789012345678901
2
1
2
12345678901234567890123456789012123456789012345678901
triangle.
Since
the
triangle
is
not
a
right
triangle,
you
have
no
2
12345678901234567890123456789012123456789012345678901
2
12345678901234567890123456789012123456789012345678901
way
of
comparing
b
with
c.
Thus,
the
answer
is
(D).
2
12345678901234567890123456789012123456789012345678901
2345678901234567890123456789012123456789012345678901
2
1
12345678901234567890123456789012123456789012345678901
6. D Begin by manipulating the exponents to more readily compare 2
2
12345678901234567890123456789012123456789012345678901
2
12345678901234567890123456789012123456789012345678901
the columns:
2
12345678901234567890123456789012123456789012345678901
2345678901234567890123456789012123456789012345678901
2
1
a b
a1b
2
12345678901234567890123456789012123456789012345678901
Column A: x x 5 x
2
12345678901234567890123456789012123456789012345678901
2
12345678901234567890123456789012123456789012345678901
a
2
12345678901234567890123456789012123456789012345678901
x
a
2
b
2345678901234567890123456789012123456789012345678901
2
12345678901234567890123456789012123456789012345678901
Column B: b 5 x
2
12345678901234567890123456789012123456789012345678901
x
12345678901234567890123456789012123456789012345678901 22
12345678901234567890123456789012123456789012345678901 2
12345678901234567890123456789012123456789012345678901
and you know that b 2 a . 1 a 2 b , 21. This means that
2
12345678901234567890123456789012123456789012345678901
1
the x in Column B is raised to a negative power, but how much 2
2
12345678901234567890123456789012123456789012345678901
12345678901234567890123456789012123456789012345678901
does this tell us about its value in comparison to Column A? 2
2
12345678901234567890123456789012123456789012345678901
a 1 b could also be negative, which would mean that Column A 2
12345678901234567890123456789012123456789012345678901
2
12345678901234567890123456789012123456789012345678901
was also raised to a negative, and a greater negative at that. Or 2
12345678901234567890123456789012123456789012345678901
2
12345678901234567890123456789012123456789012345678901
a 1 b could be positive, which would mean that the x in Column 2
12345678901234567890123456789012123456789012345678901
2345678901234567890123456789012123456789012345678901
2
12345678901234567890123456789012123456789012345678901
A would be raised to a positive.
2
12345678901234567890123456789012123456789012345678901
12345678901234567890123456789012123456789012345678901 22
12345678901234567890123456789012123456789012345678901
In the former case, Column B could be greater, depending on the 2
1
12345678901234567890123456789012123456789012345678901
value of x, and in the latter case, Column A would be greater, 2
2
12345678901234567890123456789012123456789012345678901
2
12345678901234567890123456789012123456789012345678901
again
depending
on
the
value
of
x.
As
you
can
see,
there
are
a
12345678901234567890123456789012123456789012345678901 2
2
12345678901234567890123456789012123456789012345678901
variety of possible outcomes.
2
12345678901234567890123456789012123456789012345678901
2
12345678901234567890123456789012123456789012345678901
2
12345678901234567890123456789012123456789012345678901
7. A Distribute the x in Column A, which yields x 2 x. You can 2
2
12345678901234567890123456789012123456789012345678901
12345678901234567890123456789012123456789012345678901
reason that this difference is greater than zero. xs absolute value 2
2345678901234567890123456789012123456789012345678901
2
12345678901234567890123456789012123456789012345678901
12345678901234567890123456789012123456789012345678901
is greater than 1, and x2 is positive and greater than x. Column 2
2
1
A, then, is greater.
2
12345678901234567890123456789012123456789012345678901
12345678901234567890123456789012123456789012345678901 2
2345678901234567890123456789012123456789012345678901
12345678901234567890123456789012123456789012345678901
8. A Twenty percent of 50 is 10, and so there are 10 divers. Four times 2
2
12345678901234567890123456789012123456789012345678901
2
12345678901234567890123456789012123456789012345678901
this
number
is
40.
How
many
relay
racers
are
there?
Since
each
12345678901234567890123456789012123456789012345678901 22
1
member only participates in 1 event, and since there are 10 divers, 2
12345678901234567890123456789012123456789012345678901
12345678901234567890123456789012123456789012345678901
there are 40 members who are either divers or individual swim- 2
2
12345678901234567890123456789012123456789012345678901
2
12345678901234567890123456789012123456789012345678901
mers.
We
know
there
must
be
at
least
1
individual
swimmer,
or
12345678901234567890123456789012123456789012345678901 2
2345678901234567890123456789012123456789012345678901
12345678901234567890123456789012123456789012345678901
the team would not be composed of individual swimmers; that 2
2
12345678901234567890123456789012123456789012345678901
2
12345678901234567890123456789012123456789012345678901
means
there
can
at
most
be
39
relay
racers.
Thus,
Column
A
12345678901234567890123456789012123456789012345678901 22
1
is greater.
2
12345678901234567890123456789012123456789012345678901
12345678901234567890123456789012123456789012345678901 2
2
12345678901234567890123456789012123456789012345678901
12345678901234567890123456789012123456789012345678901 2
2
12345678901234567890123456789012123456789012345678901
12345678901234567890123456789012123456789012345678901 2
12345678901234567890123456789012123456789012345678901 2
12345678901234567890123456789012123456789012345678901 2
12345678901234567890123456789012123456789012345678901 2 319
123456789012345678901234567890121234567890123456789012

http://www.xtremepapers.net

Part IV: Three Practice GREs

123456789012345678901234567890121234567890123456789012
12345678901234567890123456789012123456789012345678901 2
2
12345678901234567890123456789012123456789012345678901
9. B Since the directions are due north and due west, the three towns 2
12345678901234567890123456789012123456789012345678901
2
12345678901234567890123456789012123456789012345678901
form a right triangle, and we can use the Pythagorean theorem 2
12345678901234567890123456789012123456789012345678901
2
12345678901234567890123456789012123456789012345678901
to determine the distance between A and C.
2
12345678901234567890123456789012123456789012345678901
2345678901234567890123456789012123456789012345678901
2
1
2
12345678901234567890123456789012123456789012345678901
2
2
12345678901234567890123456789012123456789012345678901
c 5 400 1 900 5 1,300 c 5 =1,300 5 10=13
2
12345678901234567890123456789012123456789012345678901
2
12345678901234567890123456789012123456789012345678901
The
question
now
becomes,
How
do
you
compare
the
value
of
2345678901234567890123456789012123456789012345678901
2
1
2
12345678901234567890123456789012123456789012345678901
12345678901234567890123456789012123456789012345678901
c with 40? Notice that =13 , =16 5 4 and so ten times 2
2
12345678901234567890123456789012123456789012345678901
12345678901234567890123456789012123456789012345678901
the square root of 13 is less than 40. Ergo, Column B is greater. 2
2345678901234567890123456789012123456789012345678901
2
1
2
12345678901234567890123456789012123456789012345678901
10. B The probability of the first event, grabbing the first gray sock, is 2
12345678901234567890123456789012123456789012345678901
2
12345678901234567890123456789012123456789012345678901
one third. The probability of grabbing the second gray sock is 2
12345678901234567890123456789012123456789012345678901
2345678901234567890123456789012123456789012345678901
2
1
not as straightforward. Now there are only 24 gray socks and 2
12345678901234567890123456789012123456789012345678901
12345678901234567890123456789012123456789012345678901
still 25 black and white socks. Thus, the probability is 2
2
12345678901234567890123456789012123456789012345678901
2
12345678901234567890123456789012123456789012345678901
something
a
little
bit
less
than
one
third.
To
get
the
probability
2345678901234567890123456789012123456789012345678901
12345678901234567890123456789012123456789012345678901 2
12345678901234567890123456789012123456789012345678901
of both events occurring, we have to multiply the two 2
2
12345678901234567890123456789012123456789012345678901
2
12345678901234567890123456789012123456789012345678901
probabilities
together.
One
third
times
something
a
little
smaller
12345678901234567890123456789012123456789012345678901 2
12345678901234567890123456789012123456789012345678901
than one third is going to be smaller than one sixth, the value in 2
2
1
2
12345678901234567890123456789012123456789012345678901
Column
B.
2
12345678901234567890123456789012123456789012345678901
2
12345678901234567890123456789012123456789012345678901
2345678901234567890123456789012123456789012345678901
12345678901234567890123456789012123456789012345678901
11. A Obviously it is going to be difficult to compare these by 2
2
12345678901234567890123456789012123456789012345678901
12345678901234567890123456789012123456789012345678901
multiplying out the exponents. There must be another way, and 2
2
12345678901234567890123456789012123456789012345678901
12345678901234567890123456789012123456789012345678901
there is 415 5 4 3 4 14 5 414 1 414 1 414 1 414, which gives us 2
2
12345678901234567890123456789012123456789012345678901
four terms in Column A to compare with four terms in Column 2
12345678901234567890123456789012123456789012345678901
2
12345678901234567890123456789012123456789012345678901
B. The terms in Column B are less than all the respective terms in 2
12345678901234567890123456789012123456789012345678901
2
1
Column A, and so (A) is the answer.
2
12345678901234567890123456789012123456789012345678901
2
12345678901234567890123456789012123456789012345678901
2
12345678901234567890123456789012123456789012345678901
12.
C
Six
students
are
competing
for
first
place,
and
so
there
are
six
12345678901234567890123456789012123456789012345678901 2
12345678901234567890123456789012123456789012345678901
possibilities for first. But since there will always be a first before 2
2
12345678901234567890123456789012123456789012345678901
2
12345678901234567890123456789012123456789012345678901
a
second,
only
five
can
be
second.
And
since
there
will
be
a
first
12345678901234567890123456789012123456789012345678901 2
12345678901234567890123456789012123456789012345678901
and a second before a third, only four can be third. Multiply 2
2
12345678901234567890123456789012123456789012345678901
2345678901234567890123456789012123456789012345678901
12345678901234567890123456789012123456789012345678901
these together to get the total number of possibilities, (6)(5)(4) 2
2
12345678901234567890123456789012123456789012345678901
2
1
5
120.
The
answer
is
(C).
You
may
recall
the
formula
for
these
12345678901234567890123456789012123456789012345678901 2
2
12345678901234567890123456789012123456789012345678901
n!
12345678901234567890123456789012123456789012345678901
where P is the 2
kinds of permutation problems as P 5
2
12345678901234567890123456789012123456789012345678901
~n 2 c!!
2
12345678901234567890123456789012123456789012345678901
total number of possible outcomes, n is the number of 2
12345678901234567890123456789012123456789012345678901
2
12345678901234567890123456789012123456789012345678901
participants, and c is the number of places.
2
12345678901234567890123456789012123456789012345678901
2
12345678901234567890123456789012123456789012345678901
2
12345678901234567890123456789012123456789012345678901
13.
C
Consider
the
subtraction
in
the
ones
column
and
tens
column,
2
12345678901234567890123456789012123456789012345678901
2
12345678901234567890123456789012123456789012345678901
respectively.
In
the
ones,
since
C
.
D,
you
have
to
borrow
ten
2345678901234567890123456789012123456789012345678901
12345678901234567890123456789012123456789012345678901 2
12345678901234567890123456789012123456789012345678901
from the tens column. This means F 5 D 1 10 2 C. As for the 2
2
12345678901234567890123456789012123456789012345678901
2
12345678901234567890123456789012123456789012345678901
tens
column,
E
5
C
2
1
2
D
since
you
borrowed
one
from
the
2
1
2345678901234567890123456789012123456789012345678901
12345678901234567890123456789012123456789012345678901
column. Putting these two results together, E 1 F 5 (C 2 1 2 D) 2
2
1
2
12345678901234567890123456789012123456789012345678901
1
(D
1
10
2
C)
5
9.
The
answer
then
is
(C).
12345678901234567890123456789012123456789012345678901 2
2
12345678901234567890123456789012123456789012345678901
12345678901234567890123456789012123456789012345678901 2
12345678901234567890123456789012123456789012345678901 2
12345678901234567890123456789012123456789012345678901 2
2
320 12345678901234567890123456789012123456789012345678901
123456789012345678901234567890121234567890123456789012

www.petersons.com

http://www.xtremepapers.net

ANSWERS
123456789012345678901234567890121234567890123456789012
12345678901234567890123456789012123456789012345678901 2
2
12345678901234567890123456789012123456789012345678901
14. D The numerator in Column A might at first blush appear greater, 2
12345678901234567890123456789012123456789012345678901
2
12345678901234567890123456789012123456789012345678901
but be careful. If n 5 1 then n! 1 1 5 (n 1 1)!, in which case 2
12345678901234567890123456789012123456789012345678901
2
12345678901234567890123456789012123456789012345678901
Column A and B are equal. If n is a large number, though, 2
12345678901234567890123456789012123456789012345678901
2345678901234567890123456789012123456789012345678901
2
1
n! 1 1 , (n 11)!, in which case Column A would be greater. 2
12345678901234567890123456789012123456789012345678901
2
12345678901234567890123456789012123456789012345678901
Thus the answer is (D).
2
12345678901234567890123456789012123456789012345678901
2
12345678901234567890123456789012123456789012345678901
2345678901234567890123456789012123456789012345678901
1 15. D Since the triangles are similar the ratios of the side lengths are 2
2
12345678901234567890123456789012123456789012345678901
2
12345678901234567890123456789012123456789012345678901
proportional,
and
you
can
use
this
to
find
a.
You
do
not
know
2
12345678901234567890123456789012123456789012345678901
2
12345678901234567890123456789012123456789012345678901
the
length
of
the
side
on
the
larger
triangle
that
corresponds
2345678901234567890123456789012123456789012345678901
2
1
2
12345678901234567890123456789012123456789012345678901
with
side
a,
and
so
you
need
to
determine
the
third
side
of
the
2
12345678901234567890123456789012123456789012345678901
2
12345678901234567890123456789012123456789012345678901
larger
triangle.
Using
the
Pythagorean
theorem,
you
can
2
12345678901234567890123456789012123456789012345678901
2345678901234567890123456789012123456789012345678901
1
determine that the third side is equal to 6. Using similar triangles 2
2
12345678901234567890123456789012123456789012345678901
2
12345678901234567890123456789012123456789012345678901
then,
2
12345678901234567890123456789012123456789012345678901
2
12345678901234567890123456789012123456789012345678901
2345678901234567890123456789012123456789012345678901
2
12345678901234567890123456789012123456789012345678901
36 9
6 a
2
12345678901234567890123456789012123456789012345678901
5

36
5
8a

a
5
5
,
2
12345678901234567890123456789012123456789012345678901
8
6
8
2
12345678901234567890123456789012123456789012345678901 22
12345678901234567890123456789012123456789012345678901 2
12345678901234567890123456789012123456789012345678901
which is answer choice (D).
2
1
2
12345678901234567890123456789012123456789012345678901
16. D The greatest slope means the line that has the steepest ascent left 2
12345678901234567890123456789012123456789012345678901
2
12345678901234567890123456789012123456789012345678901
to right. If a line ascends right to left, then its slope is negative. 2
12345678901234567890123456789012123456789012345678901
2
12345678901234567890123456789012123456789012345678901
Line 4, choice (D), has the steepest slope left to right.
2
12345678901234567890123456789012123456789012345678901
2
12345678901234567890123456789012123456789012345678901
2
12345678901234567890123456789012123456789012345678901
17.
C
Recall
that
the
y-intercept
is
simply
the
y-value
where
a
line
2345678901234567890123456789012123456789012345678901
12345678901234567890123456789012123456789012345678901 22
12345678901234567890123456789012123456789012345678901
crosses the y-axis. If the y-intercepts of two lines sum to zero, 2
12345678901234567890123456789012123456789012345678901
12345678901234567890123456789012123456789012345678901
then they either both must have y-intercepts of zero, or they 2
2
1
2345678901234567890123456789012123456789012345678901
12345678901234567890123456789012123456789012345678901
must have y-intercepts equidistant from the x-axis, one with a 2
2
12345678901234567890123456789012123456789012345678901
2
1
positive
value
and
the
other
a
negative
value.
Line
2
and
line
3
12345678901234567890123456789012123456789012345678901 2
2
12345678901234567890123456789012123456789012345678901
cross the y-axis equidistant from the x-axis.
2
12345678901234567890123456789012123456789012345678901
2
12345678901234567890123456789012123456789012345678901
12345678901234567890123456789012123456789012345678901
18. C To begin with, since you know the circumference of the circles, 2
2
12345678901234567890123456789012123456789012345678901
you can determine the radius of the circles, which is 3 (C 5 2pr). 2
12345678901234567890123456789012123456789012345678901
2345678901234567890123456789012123456789012345678901
2
12345678901234567890123456789012123456789012345678901
If the radius is 3, then the height of the rectangle is 6, and the 2
12345678901234567890123456789012123456789012345678901
2
1
length of the rectangle is 12. The distance between two opposite 2
12345678901234567890123456789012123456789012345678901
2
12345678901234567890123456789012123456789012345678901
corners then is the hypotenuse of a right triangle with sides 6 2
12345678901234567890123456789012123456789012345678901
2
12345678901234567890123456789012123456789012345678901
and 12. Using the Pythagorean theorem we can determine the 2
12345678901234567890123456789012123456789012345678901
2
12345678901234567890123456789012123456789012345678901
12345678901234567890123456789012123456789012345678901
hypotenuse, =36 1 144 5 =180. Choice (C) then is the 2
2345678901234567890123456789012123456789012345678901
12345678901234567890123456789012123456789012345678901 22
12345678901234567890123456789012123456789012345678901
answer.
2
12345678901234567890123456789012123456789012345678901
2
12345678901234567890123456789012123456789012345678901
19.
C
Three
students
constitute
a
20-percent
increase
for
the
class.
2
1
2345678901234567890123456789012123456789012345678901
2
12345678901234567890123456789012123456789012345678901
This translates mathematically to
2
12345678901234567890123456789012123456789012345678901
12345678901234567890123456789012123456789012345678901 22
12345678901234567890123456789012123456789012345678901
3
3
2
1
2
0.20 5 x 5
5 15,
12345678901234567890123456789012123456789012345678901
2
12345678901234567890123456789012123456789012345678901
x
0.20
2
12345678901234567890123456789012123456789012345678901
2
12345678901234567890123456789012123456789012345678901
which
gives
you
the
number
of
original
students.
Add
3
to
2
12345678901234567890123456789012123456789012345678901
2345678901234567890123456789012123456789012345678901
2
1
get 18.
2
12345678901234567890123456789012123456789012345678901
12345678901234567890123456789012123456789012345678901 22
12345678901234567890123456789012123456789012345678901 321
123456789012345678901234567890121234567890123456789012

http://www.xtremepapers.net

Part IV: Three Practice GREs

123456789012345678901234567890121234567890123456789012
12345678901234567890123456789012123456789012345678901 2
2
12345678901234567890123456789012123456789012345678901
20. B Drawing a square with a diagonal creates a 45-45-90 triangle, 2
12345678901234567890123456789012123456789012345678901
2
12345678901234567890123456789012123456789012345678901
since the diagonal bisects the 90 degree angles. This means that 2
12345678901234567890123456789012123456789012345678901
12345678901234567890123456789012123456789012345678901 2
12345678901234567890123456789012123456789012345678901
the diagonal is 4=2. But you are not looking for the length 2
2
12345678901234567890123456789012123456789012345678901
12345678901234567890123456789012123456789012345678901
from corner to corner. You are looking for the length from 2
2
12345678901234567890123456789012123456789012345678901
12345678901234567890123456789012123456789012345678901
center to corner, which would be half the length of the diagonal, 2
2
12345678901234567890123456789012123456789012345678901
2
12345678901234567890123456789012123456789012345678901
or 2=2, choice (B).
2
12345678901234567890123456789012123456789012345678901
2
12345678901234567890123456789012123456789012345678901
2
12345678901234567890123456789012123456789012345678901
21.
B
Three
pounds
of
peanuts
at
34
cents/pound
costs
102
cents.
Five
2
12345678901234567890123456789012123456789012345678901
2345678901234567890123456789012123456789012345678901
1
pounds of cashews at 90 cents/pound costs 450 cents. Added 2
2
12345678901234567890123456789012123456789012345678901
12345678901234567890123456789012123456789012345678901
together they cost 552 cents, and there are 8 pounds total. 552 2
2
12345678901234567890123456789012123456789012345678901
2
12345678901234567890123456789012123456789012345678901
cents divided by 8 pounds is 69 cents/pound, or choice (B).
2345678901234567890123456789012123456789012345678901
2
1
2
12345678901234567890123456789012123456789012345678901
22. D The first thing to do is to convert the measurements in inches 2
12345678901234567890123456789012123456789012345678901
2
12345678901234567890123456789012123456789012345678901
1
1
2
12345678901234567890123456789012123456789012345678901
into
feet.
Six
inches
is
a
foot.
Two
inches
is
a
foot.
The
2345678901234567890123456789012123456789012345678901
2
12345678901234567890123456789012123456789012345678901
2
6
2
12345678901234567890123456789012123456789012345678901
2
12345678901234567890123456789012123456789012345678901
surface
area
of
a
three-dimensional
rectangle
is
the
sum
of
areas
12345678901234567890123456789012123456789012345678901 2
2
12345678901234567890123456789012123456789012345678901
of the sides.
2
12345678901234567890123456789012123456789012345678901
2
1
2
12345678901234567890123456789012123456789012345678901
Top and bottom:
2
12345678901234567890123456789012123456789012345678901
2
12345678901234567890123456789012123456789012345678901
2345678901234567890123456789012123456789012345678901
2
12345678901234567890123456789012123456789012345678901
1
2
12345678901234567890123456789012123456789012345678901
6
5
6,
2
2
12345678901234567890123456789012123456789012345678901
2
12345678901234567890123456789012123456789012345678901 2
12345678901234567890123456789012123456789012345678901 2
2
12345678901234567890123456789012123456789012345678901
two ends:
2
12345678901234567890123456789012123456789012345678901
12345678901234567890123456789012123456789012345678901 2
2
12345678901234567890123456789012123456789012345678901
1 1
1
2
1
5
2

,
2345678901234567890123456789012123456789012345678901
2
12345678901234567890123456789012123456789012345678901
6 2
6
2
12345678901234567890123456789012123456789012345678901
2
1
2
12345678901234567890123456789012123456789012345678901
two sides:
2
12345678901234567890123456789012123456789012345678901
2345678901234567890123456789012123456789012345678901
12345678901234567890123456789012123456789012345678901 2
2
12345678901234567890123456789012123456789012345678901
1
2
1
5
2

6
2
2
12345678901234567890123456789012123456789012345678901
6
2
12345678901234567890123456789012123456789012345678901
2345678901234567890123456789012123456789012345678901
12345678901234567890123456789012123456789012345678901 2
2
12345678901234567890123456789012123456789012345678901
The sum of these is choice (D).
2
1
2
12345678901234567890123456789012123456789012345678901
23.
A
Looking
at
the
frequency
column,
the
frequency
increases
as
you
12345678901234567890123456789012123456789012345678901 2
2345678901234567890123456789012123456789012345678901
2
12345678901234567890123456789012123456789012345678901
ascend the column. The same thing is true for the energy level 2
12345678901234567890123456789012123456789012345678901
2
12345678901234567890123456789012123456789012345678901
columnas you ascend the column the energy level increases. 2
12345678901234567890123456789012123456789012345678901
1
Thus, (A) is the answer. If you want further confirmation of 2
2
12345678901234567890123456789012123456789012345678901
2
12345678901234567890123456789012123456789012345678901
thissimply
pick
a
certain
frequency
and
read
the
energy
level
2
12345678901234567890123456789012123456789012345678901
2
12345678901234567890123456789012123456789012345678901
at
that
frequency.
Then
pick
another
frequency
and
read
the
12345678901234567890123456789012123456789012345678901 2
2345678901234567890123456789012123456789012345678901
12345678901234567890123456789012123456789012345678901
energy level at that frequency. If the frequency increasedthen 2
2
12345678901234567890123456789012123456789012345678901
2
12345678901234567890123456789012123456789012345678901
so
did
the
energy
and
vice
versa.
12345678901234567890123456789012123456789012345678901 2
2
1
2
12345678901234567890123456789012123456789012345678901
12345678901234567890123456789012123456789012345678901 2
2
12345678901234567890123456789012123456789012345678901
12345678901234567890123456789012123456789012345678901 2
2
12345678901234567890123456789012123456789012345678901
12345678901234567890123456789012123456789012345678901 2
12345678901234567890123456789012123456789012345678901 2
12345678901234567890123456789012123456789012345678901 2
2
322 12345678901234567890123456789012123456789012345678901
123456789012345678901234567890121234567890123456789012

S D

S D

S D

www.petersons.com

http://www.xtremepapers.net

ANSWERS
123456789012345678901234567890121234567890123456789012
12345678901234567890123456789012123456789012345678901 2
2
12345678901234567890123456789012123456789012345678901
24. A On this question, you have to be careful. Notice that the 2
12345678901234567890123456789012123456789012345678901
2
12345678901234567890123456789012123456789012345678901
wavelengths get shorter as they ascend the column (e.g., 10213 2
12345678901234567890123456789012123456789012345678901
2
12345678901234567890123456789012123456789012345678901
meters is shorter than 105 meters). This means that the shortest 2
12345678901234567890123456789012123456789012345678901
2345678901234567890123456789012123456789012345678901
2
1
wavelengths are at the top of the column. Which kind of 2
12345678901234567890123456789012123456789012345678901
12345678901234567890123456789012123456789012345678901
radiation corresponds to this part of the column? Choice (A), 2
2
12345678901234567890123456789012123456789012345678901
2
12345678901234567890123456789012123456789012345678901
gamma
rays,
is
correct.
2345678901234567890123456789012123456789012345678901
2
1
2
12345678901234567890123456789012123456789012345678901
22
2
12345678901234567890123456789012123456789012345678901
Hz
on
the
frequency
column
on
the
far
left.
Then
25.
A
First
find
10
2
12345678901234567890123456789012123456789012345678901
2
12345678901234567890123456789012123456789012345678901
slide
over
to
see
which
kind
of
radiation
this
is.
It
is
gamma
rays,
2345678901234567890123456789012123456789012345678901
2
1
2
12345678901234567890123456789012123456789012345678901
choice
(A).
2
12345678901234567890123456789012123456789012345678901
2
12345678901234567890123456789012123456789012345678901
12345678901234567890123456789012123456789012345678901
26. B On a problem like this, you have to go through the three choices 2
2345678901234567890123456789012123456789012345678901
2
1
2
12345678901234567890123456789012123456789012345678901
and determine which must be odd as follows:
2
12345678901234567890123456789012123456789012345678901
2
12345678901234567890123456789012123456789012345678901
Option
IIs
not
odd
since
the
two
products
are
even
and
an
2
12345678901234567890123456789012123456789012345678901
2345678901234567890123456789012123456789012345678901
2
12345678901234567890123456789012123456789012345678901
even minus an even is even.
2
12345678901234567890123456789012123456789012345678901
12345678901234567890123456789012123456789012345678901 22
12345678901234567890123456789012123456789012345678901
Option IIIs odd since an odd times an odd is even and an 2
12345678901234567890123456789012123456789012345678901
2
12345678901234567890123456789012123456789012345678901
even times an odd is odd.
2
1
2
12345678901234567890123456789012123456789012345678901
2
12345678901234567890123456789012123456789012345678901
Option
IIIIs
even
since
an
odd
squared
is
odd
and
an
odd
2
12345678901234567890123456789012123456789012345678901
2345678901234567890123456789012123456789012345678901
2
12345678901234567890123456789012123456789012345678901
plus an odd is even.
2
12345678901234567890123456789012123456789012345678901
2
12345678901234567890123456789012123456789012345678901
So only option II is odd, which is answer choice (B). If all that 2
12345678901234567890123456789012123456789012345678901
2
12345678901234567890123456789012123456789012345678901
seemed a bit confusing, just assign odd values to the three 2
12345678901234567890123456789012123456789012345678901
2
12345678901234567890123456789012123456789012345678901
variables, plug them in, and see if the results are odd or even.
2
12345678901234567890123456789012123456789012345678901
12345678901234567890123456789012123456789012345678901 22
1 27. E There is a helpful formula for a problem of balancing weaker and
2
12345678901234567890123456789012123456789012345678901
2
12345678901234567890123456789012123456789012345678901
stronger
solutions,
(amount
of
weaker
solution)
times
(percent12345678901234567890123456789012123456789012345678901 2
12345678901234567890123456789012123456789012345678901
age difference of desired and weaker solution) 5 (amount of 2
2
12345678901234567890123456789012123456789012345678901
2345678901234567890123456789012123456789012345678901
12345678901234567890123456789012123456789012345678901
stronger solution) times (percentage difference of stronger and 2
2
12345678901234567890123456789012123456789012345678901
2
1
desired
solution).
In
this
situation,
the
formula
reads,
n(25
2
20)
2
12345678901234567890123456789012123456789012345678901
2
12345678901234567890123456789012123456789012345678901
5
3(40
2
25)

5n
5
45

n
5
9.
Therefore,
(E)
is
the
answer.
2345678901234567890123456789012123456789012345678901
12345678901234567890123456789012123456789012345678901 22
12345678901234567890123456789012123456789012345678901 2
1 28. E There is a formula for neither or both problems like this,
12345678901234567890123456789012123456789012345678901 2
12345678901234567890123456789012123456789012345678901
G1 1 G2 1 Neither 1 Both 5 Total. You can use this equation 2
2
12345678901234567890123456789012123456789012345678901
or reason your way through this problem. If 52 students are in 2
12345678901234567890123456789012123456789012345678901
2
12345678901234567890123456789012123456789012345678901
neither class, then 98 students are in algebra or geometry. This 2
12345678901234567890123456789012123456789012345678901
2
12345678901234567890123456789012123456789012345678901
means that 25 are in algebra but not geometry and 36 are in 2
12345678901234567890123456789012123456789012345678901
2
12345678901234567890123456789012123456789012345678901
geometry but not algebra. So, there are 73 in geometry and 36 of 2
12345678901234567890123456789012123456789012345678901
12345678901234567890123456789012123456789012345678901
them are not in algebra, which means that 37 of those in 2
2
12345678901234567890123456789012123456789012345678901
2345678901234567890123456789012123456789012345678901
12345678901234567890123456789012123456789012345678901
geometry are in algebra. You can obtain the same result using 2
2
12345678901234567890123456789012123456789012345678901
2
12345678901234567890123456789012123456789012345678901
the
formula.
12345678901234567890123456789012123456789012345678901 22
1
2
12345678901234567890123456789012123456789012345678901
12345678901234567890123456789012123456789012345678901 2
2
12345678901234567890123456789012123456789012345678901
12345678901234567890123456789012123456789012345678901 2
2
12345678901234567890123456789012123456789012345678901
12345678901234567890123456789012123456789012345678901 2
12345678901234567890123456789012123456789012345678901 2
12345678901234567890123456789012123456789012345678901 2
12345678901234567890123456789012123456789012345678901 2 323
123456789012345678901234567890121234567890123456789012

http://www.xtremepapers.net

Part IV: Three Practice GREs

123456789012345678901234567890121234567890123456789012
12345678901234567890123456789012123456789012345678901 2
2
12345678901234567890123456789012123456789012345678901
2
12345678901234567890123456789012123456789012345678901
Verbal
2
12345678901234567890123456789012123456789012345678901
2
12345678901234567890123456789012123456789012345678901
Antonyms
2
12345678901234567890123456789012123456789012345678901
2
12345678901234567890123456789012123456789012345678901
2345678901234567890123456789012123456789012345678901
1 1. B To corroborate someones claim is to back it up or validate it, so 2
2
12345678901234567890123456789012123456789012345678901
2
12345678901234567890123456789012123456789012345678901
you
are
looking
for
a
word
that
means
something
like
not
2
12345678901234567890123456789012123456789012345678901
2
12345678901234567890123456789012123456789012345678901
support.
Scanning
the
answer
choices,
only
(B)
fits
the
2345678901234567890123456789012123456789012345678901
2
1
2
12345678901234567890123456789012123456789012345678901
pre-guess.
2
12345678901234567890123456789012123456789012345678901
2
12345678901234567890123456789012123456789012345678901
2
12345678901234567890123456789012123456789012345678901
2.
B
Inexactitude
is
the
opposite
of
exactitude,
or
being
exact,
so
you
2345678901234567890123456789012123456789012345678901
2
1
12345678901234567890123456789012123456789012345678901
are looking for a word that means exact. Precise, choice (B), 2
2
12345678901234567890123456789012123456789012345678901
12345678901234567890123456789012123456789012345678901
certainly means exact, and none of the other answer choices do. 2
2
12345678901234567890123456789012123456789012345678901
2
12345678901234567890123456789012123456789012345678901
3. A To foment means to incite, and so its antonym would be 2
12345678901234567890123456789012123456789012345678901
2
12345678901234567890123456789012123456789012345678901
something like un-incite. Not a real word, but a fine pre-guess 2
12345678901234567890123456789012123456789012345678901
2
12345678901234567890123456789012123456789012345678901
since it puts you on the right path. Answer choice (A), quell, 2
12345678901234567890123456789012123456789012345678901
2
12345678901234567890123456789012123456789012345678901
matches this pre-guess.
2
12345678901234567890123456789012123456789012345678901
2
12345678901234567890123456789012123456789012345678901
2
12345678901234567890123456789012123456789012345678901
4.
B
Callow
has
a
negative-sounding
connotation,
and
so
the
answer
2345678901234567890123456789012123456789012345678901
12345678901234567890123456789012123456789012345678901 2
1
will have a positive-sounding connotation. You can, at least, 2
2
12345678901234567890123456789012123456789012345678901
2
12345678901234567890123456789012123456789012345678901
eliminate
choice
(C),
timid,
on
these
grounds.
Callow
means
2
12345678901234567890123456789012123456789012345678901
2345678901234567890123456789012123456789012345678901
2
12345678901234567890123456789012123456789012345678901
immature. Choice (B), mature, then, is a good antonym.
2
12345678901234567890123456789012123456789012345678901
12345678901234567890123456789012123456789012345678901 2
12345678901234567890123456789012123456789012345678901
5. D Bacchanalian derives from Bacchus, the Greek god of wine. It 2
2
12345678901234567890123456789012123456789012345678901
12345678901234567890123456789012123456789012345678901
means unbridled revelry. The opposite is something like 2
2
12345678901234567890123456789012123456789012345678901
sobriety, or a tame activity. Of the answer choices, pious best 2
12345678901234567890123456789012123456789012345678901
2
12345678901234567890123456789012123456789012345678901
approximates this pre-guess.
2
1
2345678901234567890123456789012123456789012345678901
12345678901234567890123456789012123456789012345678901 2
2
12345678901234567890123456789012123456789012345678901
1 6. E Pulchritude means beauty. This word was used in the first part 2
2
12345678901234567890123456789012123456789012345678901
of the book, so hopefully you remembered it.
2
12345678901234567890123456789012123456789012345678901
2345678901234567890123456789012123456789012345678901
12345678901234567890123456789012123456789012345678901 2
2
12345678901234567890123456789012123456789012345678901
1 7. D To be stolid is to lack emotion. Thus, the antonym here is 2
2
12345678901234567890123456789012123456789012345678901
something like emotional. Choice (D) is exactly that.
2
12345678901234567890123456789012123456789012345678901
2345678901234567890123456789012123456789012345678901
12345678901234567890123456789012123456789012345678901 2
2
12345678901234567890123456789012123456789012345678901
1 8. A If you do not know the definition of prevaricate, you might 2
12345678901234567890123456789012123456789012345678901
guess that it has a negative-sounding connotation, and so the 2
2
12345678901234567890123456789012123456789012345678901
2345678901234567890123456789012123456789012345678901
answer would be positive. Choices (A) and (E) are the strong 2
12345678901234567890123456789012123456789012345678901
2
12345678901234567890123456789012123456789012345678901
positives. Prevaricate means to lie, and so (A) is the correct 2
12345678901234567890123456789012123456789012345678901
2
12345678901234567890123456789012123456789012345678901
answer.
2
1
2345678901234567890123456789012123456789012345678901
12345678901234567890123456789012123456789012345678901 2
12345678901234567890123456789012123456789012345678901 2
12345678901234567890123456789012123456789012345678901 2
2
12345678901234567890123456789012123456789012345678901
2
1 Analogies
2345678901234567890123456789012123456789012345678901
12345678901234567890123456789012123456789012345678901 2
12345678901234567890123456789012123456789012345678901
9. C Someone who is long-winded lacks brevity. Choices (B) and (C) 2
12345678901234567890123456789012123456789012345678901 2
12345678901234567890123456789012123456789012345678901
appear to be possibilities. Attractive is the opposite of repulsive. 2
2
1
Something lean lacks excess. Choice (C) is the better of the two 2
2345678901234567890123456789012123456789012345678901
12345678901234567890123456789012123456789012345678901
2
1
choices.
2
12345678901234567890123456789012123456789012345678901
12345678901234567890123456789012123456789012345678901 2
2
12345678901234567890123456789012123456789012345678901
12345678901234567890123456789012123456789012345678901 2
12345678901234567890123456789012123456789012345678901 2
12345678901234567890123456789012123456789012345678901 2
2
324 12345678901234567890123456789012123456789012345678901
123456789012345678901234567890121234567890123456789012

www.petersons.com

http://www.xtremepapers.net

ANSWERS
123456789012345678901234567890121234567890123456789012
12345678901234567890123456789012123456789012345678901 2
2
12345678901234567890123456789012123456789012345678901
10. E A lawyer is an officer of the court, or a lawyer works in the 2
12345678901234567890123456789012123456789012345678901
2
12345678901234567890123456789012123456789012345678901
court. An actor acts in a play, but a play is an activity whereas a 2
12345678901234567890123456789012123456789012345678901
2
12345678901234567890123456789012123456789012345678901
court is a location or part of a system. A janitor might work at a 2
12345678901234567890123456789012123456789012345678901
2345678901234567890123456789012123456789012345678901
2
1
school. A manager manages a team, a prison has guards, and a 2
12345678901234567890123456789012123456789012345678901
2
12345678901234567890123456789012123456789012345678901
bureaucrat works in a bureaucracy.
2
12345678901234567890123456789012123456789012345678901
2
12345678901234567890123456789012123456789012345678901
2345678901234567890123456789012123456789012345678901
1 11. B The relationship between the stem pair is one of definition; to be 2
2
12345678901234567890123456789012123456789012345678901
2
12345678901234567890123456789012123456789012345678901
candid
is
to
be
frank.
Which
of
the
answer
choices
shares
this
2
12345678901234567890123456789012123456789012345678901
2
12345678901234567890123456789012123456789012345678901
relationship?
Only
(B)to
be
erudite
is
to
be
learned.
Choice
2345678901234567890123456789012123456789012345678901
2
1
2
12345678901234567890123456789012123456789012345678901
(A)
might
have
seemed
appealing
but
realize
that
a
sincere
2
12345678901234567890123456789012123456789012345678901
2
12345678901234567890123456789012123456789012345678901
person
might
be
kind
or
he
or
she
might
be
sincerely
unkind.
2
12345678901234567890123456789012123456789012345678901
2345678901234567890123456789012123456789012345678901
2
1
12345678901234567890123456789012123456789012345678901
12. B Here, the stem relationship is of lack or negationto be austere 2
2
12345678901234567890123456789012123456789012345678901
is to not be decorated. In a similar fashion, to be offensive is to 2
12345678901234567890123456789012123456789012345678901
2
12345678901234567890123456789012123456789012345678901
not be tactful. None of the other stem pairs share the 2
12345678901234567890123456789012123456789012345678901
2
12345678901234567890123456789012123456789012345678901
relationship of negation.
2
12345678901234567890123456789012123456789012345678901
2
12345678901234567890123456789012123456789012345678901
2
12345678901234567890123456789012123456789012345678901
13.
A
Physician
studies/cares
for
the
human
body.
In
the
same
way,
a
2345678901234567890123456789012123456789012345678901
12345678901234567890123456789012123456789012345678901 22
1
horticulturist studies/cares for a plant. But it is also true that 2
12345678901234567890123456789012123456789012345678901
12345678901234567890123456789012123456789012345678901
veterinarians study/care for cows. And certainly biologists study 2
2
12345678901234567890123456789012123456789012345678901
2345678901234567890123456789012123456789012345678901
12345678901234567890123456789012123456789012345678901
evolution. But notice that biologists do not care for evolution. 2
2
12345678901234567890123456789012123456789012345678901
2
12345678901234567890123456789012123456789012345678901
Also
notice
that
a
cow
is
a
specific
kind
of
animal
that
a
12345678901234567890123456789012123456789012345678901 22
12345678901234567890123456789012123456789012345678901
veterinarian cares for while human body and plant are general 2
12345678901234567890123456789012123456789012345678901
2
12345678901234567890123456789012123456789012345678901
terms. So, (A) is the best choice.
2
12345678901234567890123456789012123456789012345678901
12345678901234567890123456789012123456789012345678901 22
1 14. B Craven means cowardly and completely subdued by fear.
2
12345678901234567890123456789012123456789012345678901
Trepidatious means to be apprehensive or fearful. Thus 2
12345678901234567890123456789012123456789012345678901
2
12345678901234567890123456789012123456789012345678901
craven is an extreme of trepidatiousness. This is a relationship of 2
12345678901234567890123456789012123456789012345678901
2
12345678901234567890123456789012123456789012345678901
degree. Only choice (E) shares this relationship of degree 2
12345678901234567890123456789012123456789012345678901
2
12345678901234567890123456789012123456789012345678901
exultant is an extreme of joyfulness. Choice (A) is not as strong 2
12345678901234567890123456789012123456789012345678901
2
12345678901234567890123456789012123456789012345678901
a choice because miniscule and minute both mean very small, 2
12345678901234567890123456789012123456789012345678901
2345678901234567890123456789012123456789012345678901
12345678901234567890123456789012123456789012345678901
and so there is no difference in degree. The same is true for 2
2
12345678901234567890123456789012123456789012345678901
2
1
migratory
and
transitory,
choice
(B).
As
for
(C),
ageless
means
12345678901234567890123456789012123456789012345678901 2
12345678901234567890123456789012123456789012345678901
without age while ancient means to by very old. They do not 2
2
12345678901234567890123456789012123456789012345678901
2
12345678901234567890123456789012123456789012345678901
have
a
relationship
of
degree.
2
12345678901234567890123456789012123456789012345678901
2
12345678901234567890123456789012123456789012345678901
2
12345678901234567890123456789012123456789012345678901
15.
B
If
you
do
not
know
the
definition
of
either
of
these
words,
you
2345678901234567890123456789012123456789012345678901
12345678901234567890123456789012123456789012345678901 22
12345678901234567890123456789012123456789012345678901
can eliminate some of the answer pairs that have a weak 2
12345678901234567890123456789012123456789012345678901
12345678901234567890123456789012123456789012345678901
relationship. Catatonic and cataclysmic do not have a necessary 2
2
1
2345678901234567890123456789012123456789012345678901
12345678901234567890123456789012123456789012345678901
relationship, nor do histrionic (to be overly theatrical) and 2
2
12345678901234567890123456789012123456789012345678901
12345678901234567890123456789012123456789012345678901
sordid. That leaves (A), (B), and (C). Turgid actually means 2
12345678901234567890123456789012123456789012345678901 22
pompous or bombastic, and so the stem relationship is one of
1
2
12345678901234567890123456789012123456789012345678901
definition. To be turgid is to be bombastic. Is to be lazy to be 2
12345678901234567890123456789012123456789012345678901
2
12345678901234567890123456789012123456789012345678901
lethargic? Maybe. To be rotund is to be portly. And to bemuse is 2
12345678901234567890123456789012123456789012345678901
2
12345678901234567890123456789012123456789012345678901
to bewilder, not to distract.
2
12345678901234567890123456789012123456789012345678901
12345678901234567890123456789012123456789012345678901 2
12345678901234567890123456789012123456789012345678901 2
12345678901234567890123456789012123456789012345678901 2 325
123456789012345678901234567890121234567890123456789012

http://www.xtremepapers.net

Part IV: Three Practice GREs

123456789012345678901234567890121234567890123456789012
12345678901234567890123456789012123456789012345678901 2
2
12345678901234567890123456789012123456789012345678901
Sentence Completions
2
12345678901234567890123456789012123456789012345678901
2
12345678901234567890123456789012123456789012345678901
2
12345678901234567890123456789012123456789012345678901
16.
A
What
does
a
successful
demolition
crew
do?
It
demolishes
12345678901234567890123456789012123456789012345678901 2
12345678901234567890123456789012123456789012345678901
buildings, and so you are looking for a word like demolish. 2
2
12345678901234567890123456789012123456789012345678901
2
12345678901234567890123456789012123456789012345678901
Which
of
the
answer
choices
fits
this
pre-guess?
Certainly
not
2
12345678901234567890123456789012123456789012345678901
2
12345678901234567890123456789012123456789012345678901
construct
or
erect.
Incapacitate
and
deactivate
do
not
fit
the
2
12345678901234567890123456789012123456789012345678901
2345678901234567890123456789012123456789012345678901
1
context. Raze means to destroy completely, so it completes the 2
2
12345678901234567890123456789012123456789012345678901
2
12345678901234567890123456789012123456789012345678901
sentence
well.
2
12345678901234567890123456789012123456789012345678901
2
12345678901234567890123456789012123456789012345678901
2345678901234567890123456789012123456789012345678901
1 17. A Begin with a pre-guess. Something like punishable/responsible 2
2
12345678901234567890123456789012123456789012345678901
12345678901234567890123456789012123456789012345678901
works, since it is generally thought that if something is an 2
2
12345678901234567890123456789012123456789012345678901
12345678901234567890123456789012123456789012345678901
accident then one is not fully punishable/responsible. Looking 2
2345678901234567890123456789012123456789012345678901
2
1
12345678901234567890123456789012123456789012345678901
down the list, culpable matches our pre-guess. Reading it back 2
2
12345678901234567890123456789012123456789012345678901
2
into the sentence, it works well.
12345678901234567890123456789012123456789012345678901
2
12345678901234567890123456789012123456789012345678901
2345678901234567890123456789012123456789012345678901
2
12345678901234567890123456789012123456789012345678901
18. E Remember to do a two blank sentence one blank at a time. The 2
12345678901234567890123456789012123456789012345678901
12345678901234567890123456789012123456789012345678901
first blank is clearly a negative term since publicity surrounding 2
2
12345678901234567890123456789012123456789012345678901
2
12345678901234567890123456789012123456789012345678901
mismanagement
would
hurt
a
companys
credibility.
Thus,
we
12345678901234567890123456789012123456789012345678901 2
1
can eliminate choice (C), aided. Belabored, choice (A), also does 2
2
12345678901234567890123456789012123456789012345678901
2
12345678901234567890123456789012123456789012345678901
not
fit
with
the
notion
of
hurt.
On
the
second
blank,
we
can
2
12345678901234567890123456789012123456789012345678901
2345678901234567890123456789012123456789012345678901
12345678901234567890123456789012123456789012345678901
infer that it must be a positive since it is on its account that the 2
2
12345678901234567890123456789012123456789012345678901
2
12345678901234567890123456789012123456789012345678901
company
remained
solvent.
This
eliminates
(D)
and
(B)
also,
12345678901234567890123456789012123456789012345678901 2
2
12345678901234567890123456789012123456789012345678901
since evolving is a more neutral than positive term.
2
12345678901234567890123456789012123456789012345678901
12345678901234567890123456789012123456789012345678901 2
12345678901234567890123456789012123456789012345678901
19. B The though at the beginning of the sentence clues you into the 2
2
12345678901234567890123456789012123456789012345678901
1
fact that the officials hopes were not achieved. Thus, his 2
2345678901234567890123456789012123456789012345678901
12345678901234567890123456789012123456789012345678901 2
assurances must not have had their desired effect. Assurances 2
12345678901234567890123456789012123456789012345678901
2
1
are meant to calm, and that is a good pre-guess. Looking at the 2
12345678901234567890123456789012123456789012345678901
2
12345678901234567890123456789012123456789012345678901
answer choices, assuage, choice (B), is the only choice that 2
12345678901234567890123456789012123456789012345678901
2
12345678901234567890123456789012123456789012345678901
matches the pre-guess.
2
12345678901234567890123456789012123456789012345678901
2
12345678901234567890123456789012123456789012345678901
2
12345678901234567890123456789012123456789012345678901
20.
D
The
sentence
structure
indicates
that
the
word
in
the
blank
is
2345678901234567890123456789012123456789012345678901
12345678901234567890123456789012123456789012345678901 2
12345678901234567890123456789012123456789012345678901
described by the younger sibling . . . taking on mannerisms . . . 2
2
1
2
12345678901234567890123456789012123456789012345678901
What
are
the
younger
siblings
doing?
They
are
copying
their
12345678901234567890123456789012123456789012345678901 2
2345678901234567890123456789012123456789012345678901
12345678901234567890123456789012123456789012345678901
older siblings. Which answer choice matches with copy? 2
2
12345678901234567890123456789012123456789012345678901
2
12345678901234567890123456789012123456789012345678901
Emulate,
choice
(D),
means
to
copy,
and
it
is
the
best
answer
12345678901234567890123456789012123456789012345678901 2
2
1
choice.
2
12345678901234567890123456789012123456789012345678901
2
12345678901234567890123456789012123456789012345678901
12345678901234567890123456789012123456789012345678901
21. A Start with the first blank here. Since the visitor was unfamiliar 2
2
12345678901234567890123456789012123456789012345678901
12345678901234567890123456789012123456789012345678901
with the regions customs and you know that he offended his 2
2345678901234567890123456789012123456789012345678901
2
12345678901234567890123456789012123456789012345678901
host, you should suspect that his actions were culturally 2
12345678901234567890123456789012123456789012345678901
2
12345678901234567890123456789012123456789012345678901
improper. Looking at the answer choices, we can eliminate (C). 2
12345678901234567890123456789012123456789012345678901
2
1
Moving to the second blank, since he was unaware of his poor 2
12345678901234567890123456789012123456789012345678901
2
12345678901234567890123456789012123456789012345678901
actions, it makes sense that he did not realize he had offended 2
12345678901234567890123456789012123456789012345678901
12345678901234567890123456789012123456789012345678901
his hosts. Unaware does not fit the context, and blind does not 2
2
12345678901234567890123456789012123456789012345678901
2345678901234567890123456789012123456789012345678901
1
fit as well as oblivious or ignorant. That leaves choices (A) and 2
2
12345678901234567890123456789012123456789012345678901
12345678901234567890123456789012123456789012345678901 2
2
326 12345678901234567890123456789012123456789012345678901
123456789012345678901234567890121234567890123456789012

www.petersons.com

http://www.xtremepapers.net

ANSWERS
123456789012345678901234567890121234567890123456789012
12345678901234567890123456789012123456789012345678901 2
2
12345678901234567890123456789012123456789012345678901
(B). Reading both back into the sentence, (A) works better. 2
12345678901234567890123456789012123456789012345678901
2
12345678901234567890123456789012123456789012345678901
Curtness does not capture the idea of culture improperness like 2
12345678901234567890123456789012123456789012345678901
2
12345678901234567890123456789012123456789012345678901
impropriety does.
2
12345678901234567890123456789012123456789012345678901
2345678901234567890123456789012123456789012345678901
2
1
2
12345678901234567890123456789012123456789012345678901
22.
E
The
second
blank
is
quite
approachable
since
you
know
that
it
2
12345678901234567890123456789012123456789012345678901
2
12345678901234567890123456789012123456789012345678901
means
allowing
no
light
through
or
something
like
that.
2
12345678901234567890123456789012123456789012345678901
2345678901234567890123456789012123456789012345678901
1
Which answer choices fit this? Opaque, choice (E), really is the 2
2
12345678901234567890123456789012123456789012345678901
2
12345678901234567890123456789012123456789012345678901
only
good
fit.
Translucent
works
well
in
the
first
blank,
and
so
2
12345678901234567890123456789012123456789012345678901
2
12345678901234567890123456789012123456789012345678901
(E)
is
the
best
choice.
2345678901234567890123456789012123456789012345678901
2
1
2
12345678901234567890123456789012123456789012345678901
12345678901234567890123456789012123456789012345678901
23. A Here the sentence is contrasting the way that history was written 2
2
12345678901234567890123456789012123456789012345678901
12345678901234567890123456789012123456789012345678901
in the past with the professional standards of history writing 2
2345678901234567890123456789012123456789012345678901
2
1
12345678901234567890123456789012123456789012345678901
today. In the past, the sentence reads, historiography . . . tended 2
2
12345678901234567890123456789012123456789012345678901
to hagiography. In this usage, hagiography means overly 2
12345678901234567890123456789012123456789012345678901
2
12345678901234567890123456789012123456789012345678901
sympathetic and full of praise. Thus, todays requirements 2
12345678901234567890123456789012123456789012345678901
2
12345678901234567890123456789012123456789012345678901
should be in strong contrast to this. With this in mind, which of 2
12345678901234567890123456789012123456789012345678901
2
12345678901234567890123456789012123456789012345678901
the answer choices is a good modifier for tone in the second 2
12345678901234567890123456789012123456789012345678901
2345678901234567890123456789012123456789012345678901
2
12345678901234567890123456789012123456789012345678901
blank? Reserved and careful are good choices. Looking at the 2
1
12345678901234567890123456789012123456789012345678901
first blank, even, choice (E), does not fit with distance, while 2
2
12345678901234567890123456789012123456789012345678901
2
12345678901234567890123456789012123456789012345678901
critical
does
fit
well.
Therefore,
choice
(A)
is
the
best
choice.
2345678901234567890123456789012123456789012345678901
12345678901234567890123456789012123456789012345678901 22
12345678901234567890123456789012123456789012345678901 2
12345678901234567890123456789012123456789012345678901 2
12345678901234567890123456789012123456789012345678901 2
12345678901234567890123456789012123456789012345678901
Reading Comprehension
2
12345678901234567890123456789012123456789012345678901
2
12345678901234567890123456789012123456789012345678901
24. E The passage reads, The semi-empirical practitioners . . . 2
12345678901234567890123456789012123456789012345678901
2
12345678901234567890123456789012123456789012345678901
criticized the ab initio methodology as overly laborious and 2
1
2345678901234567890123456789012123456789012345678901
12345678901234567890123456789012123456789012345678901
ultimately less productive. That is I and III. II, on the other 2
2
12345678901234567890123456789012123456789012345678901
2
1
hand,
is
actually
a
criticism
of
the
semi-empirical
method.
12345678901234567890123456789012123456789012345678901 2
2
12345678901234567890123456789012123456789012345678901
12345678901234567890123456789012123456789012345678901
25. B The second half of the first paragraph clearly explains that the 2
2
12345678901234567890123456789012123456789012345678901
2
12345678901234567890123456789012123456789012345678901
major
hurdle
in
working
with
Schrdingers
Equation
was
that
2
12345678901234567890123456789012123456789012345678901
2
12345678901234567890123456789012123456789012345678901
it
was
difficult
to
solve.
If
you
have
been
keeping
track
of
the
2345678901234567890123456789012123456789012345678901
12345678901234567890123456789012123456789012345678901 22
12345678901234567890123456789012123456789012345678901
flow of ideas in the passage, it will be easy to locate the pertinent 2
1
2
12345678901234567890123456789012123456789012345678901
part of the text and answer the question. Choice (B) is correct.
2
12345678901234567890123456789012123456789012345678901
2345678901234567890123456789012123456789012345678901
2
12345678901234567890123456789012123456789012345678901
26. C Lines 2731 read, . . . the so-called semi-empirical method, 2
12345678901234567890123456789012123456789012345678901
2
12345678901234567890123456789012123456789012345678901
simplified the computational hurdles by neglecting possibly 2
12345678901234567890123456789012123456789012345678901
2
1
negligible components of the calculations and by extensively 2
12345678901234567890123456789012123456789012345678901
2
12345678901234567890123456789012123456789012345678901
employed empirical data. Thus, it was empirical in that it 2
12345678901234567890123456789012123456789012345678901
12345678901234567890123456789012123456789012345678901
employed extensive empirical data in obtaining a result, which is 2
2
12345678901234567890123456789012123456789012345678901
2345678901234567890123456789012123456789012345678901
2
12345678901234567890123456789012123456789012345678901
choice (C).
2
12345678901234567890123456789012123456789012345678901
12345678901234567890123456789012123456789012345678901 22
12345678901234567890123456789012123456789012345678901
1 27. A A question about the authors intent will include both the main 2
12345678901234567890123456789012123456789012345678901
idea of the passage and the authors tone in expressing the main 2
2
12345678901234567890123456789012123456789012345678901
2
12345678901234567890123456789012123456789012345678901
idea.
What
was
the
main
idea
of
the
passage?
The
introduction
12345678901234567890123456789012123456789012345678901 2
12345678901234567890123456789012123456789012345678901
of electronic computers produced a bifurcation in methodolo- 2
2
12345678901234567890123456789012123456789012345678901
12345678901234567890123456789012123456789012345678901
gies in quantum chemistry. What was the authors tone in 2
12345678901234567890123456789012123456789012345678901 2
12345678901234567890123456789012123456789012345678901 2 327
123456789012345678901234567890121234567890123456789012

http://www.xtremepapers.net

Part IV: Three Practice GREs

123456789012345678901234567890121234567890123456789012
12345678901234567890123456789012123456789012345678901 2
2
12345678901234567890123456789012123456789012345678901
expressing this idea? Primarily explanatory as opposed to 2
12345678901234567890123456789012123456789012345678901
2
12345678901234567890123456789012123456789012345678901
argumentative. Which answer choice captures both of these 2
12345678901234567890123456789012123456789012345678901
2
12345678901234567890123456789012123456789012345678901
elements? Choice (A) does. All the other answer choices are 2
12345678901234567890123456789012123456789012345678901
2345678901234567890123456789012123456789012345678901
2
1
either wrong in tone or wrong in content.
2
12345678901234567890123456789012123456789012345678901
2
12345678901234567890123456789012123456789012345678901
2
12345678901234567890123456789012123456789012345678901
28.
D
You
should
read
with
this
question
in
mind,
because
questions
2
12345678901234567890123456789012123456789012345678901
2345678901234567890123456789012123456789012345678901
1
like this are standard. The passage discusses the inadequacies of 2
2
12345678901234567890123456789012123456789012345678901
2
12345678901234567890123456789012123456789012345678901
current
theories
of
wavy
and
dotted
line
pottery
in
the
2
12345678901234567890123456789012123456789012345678901
2
12345678901234567890123456789012123456789012345678901
Mesolithic-Neolithic
transition,
and
points
to
the
need
for
2345678901234567890123456789012123456789012345678901
2
1
2
12345678901234567890123456789012123456789012345678901
further
research
in
this
area.
The
answer
should
include
both
of
2
12345678901234567890123456789012123456789012345678901
2
12345678901234567890123456789012123456789012345678901
these
aspects.
Choice
(A)
includes
the
former
but
not
the
latter.
2
12345678901234567890123456789012123456789012345678901
2345678901234567890123456789012123456789012345678901
1
Choices (B) and (C) are too specific, while choice (E) is too 2
2
12345678901234567890123456789012123456789012345678901
12345678901234567890123456789012123456789012345678901
general. Only choice (D) contains both aspects of the passage, 2
2
12345678901234567890123456789012123456789012345678901
12345678901234567890123456789012123456789012345678901
the shortcomings of contemporary theories and the need for 2
2345678901234567890123456789012123456789012345678901
12345678901234567890123456789012123456789012345678901 2
2
12345678901234567890123456789012123456789012345678901
more research.
2
12345678901234567890123456789012123456789012345678901
2
12345678901234567890123456789012123456789012345678901
29.
C
The
passage
clearly
states
that
Arkells
model
was
problematized
12345678901234567890123456789012123456789012345678901 2
2
12345678901234567890123456789012123456789012345678901
by the discovery of wavy and dotted line pottery outside of the 2
1
12345678901234567890123456789012123456789012345678901
Central Nile in strata and progressions that conflicted with his 2
2
12345678901234567890123456789012123456789012345678901
2
12345678901234567890123456789012123456789012345678901
account.
That
is
answer
choice
(C).
2345678901234567890123456789012123456789012345678901
12345678901234567890123456789012123456789012345678901 2
12345678901234567890123456789012123456789012345678901 2
12345678901234567890123456789012123456789012345678901
30. B On inference questions like this, you have to bear down and 2
2
12345678901234567890123456789012123456789012345678901
2
12345678901234567890123456789012123456789012345678901
read
through
each
answer
choice
and
compare
it
with
what
you
12345678901234567890123456789012123456789012345678901 2
12345678901234567890123456789012123456789012345678901
know of the author through reading the passage. In choice (A), 2
2
12345678901234567890123456789012123456789012345678901
12345678901234567890123456789012123456789012345678901
the word thoroughly sounds extreme in comparison to the tone 2
2
1
12345678901234567890123456789012123456789012345678901
of the passage and so should give you caution. Choice (B) 2
2
12345678901234567890123456789012123456789012345678901
12345678901234567890123456789012123456789012345678901
sounds like something the author would recommend. As for (C), 2
12345678901234567890123456789012123456789012345678901 2
12345678901234567890123456789012123456789012345678901
the author never makes any specific methodological suggestions 2
2
12345678901234567890123456789012123456789012345678901
of this kind. Choice (D) might be something that the author 2
12345678901234567890123456789012123456789012345678901
2
12345678901234567890123456789012123456789012345678901
would agree with but the author never mentions composition in 2
12345678901234567890123456789012123456789012345678901
2
12345678901234567890123456789012123456789012345678901
the passage, and the same could be said for choice (E) and 2
12345678901234567890123456789012123456789012345678901
2
12345678901234567890123456789012123456789012345678901
material vs. non-material culture. Choice (B) is best.
2
12345678901234567890123456789012123456789012345678901
12345678901234567890123456789012123456789012345678901 2
12345678901234567890123456789012123456789012345678901 2
2
12345678901234567890123456789012123456789012345678901
2
12345678901234567890123456789012123456789012345678901
2
12345678901234567890123456789012123456789012345678901
2
12345678901234567890123456789012123456789012345678901
12345678901234567890123456789012123456789012345678901 2
2345678901234567890123456789012123456789012345678901
12345678901234567890123456789012123456789012345678901 2
12345678901234567890123456789012123456789012345678901 2
12345678901234567890123456789012123456789012345678901 2
12345678901234567890123456789012123456789012345678901 2
2
1
2
12345678901234567890123456789012123456789012345678901
2
12345678901234567890123456789012123456789012345678901
2
12345678901234567890123456789012123456789012345678901
2
12345678901234567890123456789012123456789012345678901
12345678901234567890123456789012123456789012345678901 2
2345678901234567890123456789012123456789012345678901
12345678901234567890123456789012123456789012345678901 2
2
1
2
12345678901234567890123456789012123456789012345678901
12345678901234567890123456789012123456789012345678901 2
2
12345678901234567890123456789012123456789012345678901
12345678901234567890123456789012123456789012345678901 2
12345678901234567890123456789012123456789012345678901 2
12345678901234567890123456789012123456789012345678901 2
2
328 12345678901234567890123456789012123456789012345678901
123456789012345678901234567890121234567890123456789012

www.petersons.com

http://www.xtremepapers.net

Chapter

8
Closing Remarks
123456789012345678901234567890121234567890123456789012
2
12345678901234567890123456789012123456789012345678901
2
12345678901234567890123456789012123456789012345678901
Congratulations!
You
have
just
read
through
a
mountain
of
GRE-related
2
12345678901234567890123456789012123456789012345678901
2345678901234567890123456789012123456789012345678901
2
12345678901234567890123456789012123456789012345678901
information! Consider yourself an expert on a subject, but dont go 2
12345678901234567890123456789012123456789012345678901
12345678901234567890123456789012123456789012345678901
bragging about it at social functions because its not likely to impress 2
2
12345678901234567890123456789012123456789012345678901
2
12345678901234567890123456789012123456789012345678901
people.
Believe
me;
I
speak
from
experience.
12345678901234567890123456789012123456789012345678901 22
1
12345678901234567890123456789012123456789012345678901
If you have studied all the strategies and techniques, learned the formats 2
2
12345678901234567890123456789012123456789012345678901
2
12345678901234567890123456789012123456789012345678901
for
each
section,
and
practiced
on
the
sample
questions,
then
you
should
2345678901234567890123456789012123456789012345678901
12345678901234567890123456789012123456789012345678901 22
12345678901234567890123456789012123456789012345678901
feel pretty good about your chances on the real test. Theres only so much 2
12345678901234567890123456789012123456789012345678901
12345678901234567890123456789012123456789012345678901
preparation you can do, and if youve finished this book, youve done 2
2
12345678901234567890123456789012123456789012345678901
12345678901234567890123456789012123456789012345678901
more than most people taking the test. Be confident about your chances on 2
2
12345678901234567890123456789012123456789012345678901
12345678901234567890123456789012123456789012345678901
the real test, because at the very least, this book has shown you what to 2
12345678901234567890123456789012123456789012345678901 22
1 expect on the GRE. You would be surprised at how peoples scores
2
12345678901234567890123456789012123456789012345678901
improve when a test leaves the realm of mysterious, unexpected exam 2
12345678901234567890123456789012123456789012345678901
2
12345678901234567890123456789012123456789012345678901
and becomes just another standardized test.
2
12345678901234567890123456789012123456789012345678901
2
12345678901234567890123456789012123456789012345678901
2345678901234567890123456789012123456789012345678901
12345678901234567890123456789012123456789012345678901
Thats all the GRE is, really. By now, you should realize that the GRE says 2
2
12345678901234567890123456789012123456789012345678901
1 nothing about your potential as a graduate student or your abilities as a 2
2
12345678901234567890123456789012123456789012345678901
2
12345678901234567890123456789012123456789012345678901
scholar.
Its
little
more
than
a
hoop
you
must
pass
through
to
get
into
2345678901234567890123456789012123456789012345678901
12345678901234567890123456789012123456789012345678901 22
12345678901234567890123456789012123456789012345678901
1 graduate school, testing you in subjects that have a bearing on higher 2
12345678901234567890123456789012123456789012345678901
education in only the vaguest, most general sense. If you do well on the 2
2
12345678901234567890123456789012123456789012345678901
2345678901234567890123456789012123456789012345678901
2
12345678901234567890123456789012123456789012345678901
test, then youre a good standardized test-taker, nothing more.
2
12345678901234567890123456789012123456789012345678901
12345678901234567890123456789012123456789012345678901 22
12345678901234567890123456789012123456789012345678901
1 I would wish you luck on the GRE, but if youve read this book, then you 2
2
12345678901234567890123456789012123456789012345678901
will have made your own luck. Your GRE score will reflect it.
2
12345678901234567890123456789012123456789012345678901
2
12345678901234567890123456789012123456789012345678901
2
12345678901234567890123456789012123456789012345678901
12345678901234567890123456789012123456789012345678901 2
2345678901234567890123456789012123456789012345678901
12345678901234567890123456789012123456789012345678901 22
12345678901234567890123456789012123456789012345678901 2
12345678901234567890123456789012123456789012345678901 2
12345678901234567890123456789012123456789012345678901 2
1
2
12345678901234567890123456789012123456789012345678901
12345678901234567890123456789012123456789012345678901 2
2
12345678901234567890123456789012123456789012345678901
12345678901234567890123456789012123456789012345678901 2
2
12345678901234567890123456789012123456789012345678901
12345678901234567890123456789012123456789012345678901 2
12345678901234567890123456789012123456789012345678901 2
12345678901234567890123456789012123456789012345678901 2
12345678901234567890123456789012123456789012345678901 2
123456789012345678901234567890121234567890123456789012
329

http://www.xtremepapers.net

http://www.xtremepapers.net

PART

V
Appendix
About the CAT

332

PART V
http://www.xtremepapers.net

About the CAT


123456789012345678901234567890121234567890123456789012
2
12345678901234567890123456789012123456789012345678901
2
12345678901234567890123456789012123456789012345678901
2345678901234567890123456789012123456789012345678901
2
1 to Apply for the GRE
How
2
12345678901234567890123456789012123456789012345678901
2
12345678901234567890123456789012123456789012345678901
The
best
way
to
learn
all
the
relevant
facts
about
the
GRE
is
to
go
on
line
2
12345678901234567890123456789012123456789012345678901
2
12345678901234567890123456789012123456789012345678901
to
the
official
GRE
Web
site,
www.gre.org.
There,
you
will
find
a
list
of
2345678901234567890123456789012123456789012345678901
12345678901234567890123456789012123456789012345678901 2
12345678901234567890123456789012123456789012345678901
testing centers in your area, registration information, stuff about score 2
2
12345678901234567890123456789012123456789012345678901
2
12345678901234567890123456789012123456789012345678901
reporting,
and
scads
of
other
GRE-related
trivia.
If
you
dont
want
to
go
12345678901234567890123456789012123456789012345678901 2
2
12345678901234567890123456789012123456789012345678901
1 on line because you dont like computers, you should keep in mind that its 2
12345678901234567890123456789012123456789012345678901
a computer-based test you are about to take. Its time to start familiarizing 2
2
12345678901234567890123456789012123456789012345678901
12345678901234567890123456789012123456789012345678901
yourself. There is a phone number to call during business hours, 2
2345678901234567890123456789012123456789012345678901
12345678901234567890123456789012123456789012345678901 2
12345678901234567890123456789012123456789012345678901
609-771-7670, but the recorded voice there will harangue you to use the 2
2
12345678901234567890123456789012123456789012345678901
2
12345678901234567890123456789012123456789012345678901
Web site. You can eventually get a person, but it normally takes a while.
12345678901234567890123456789012123456789012345678901 2
2
12345678901234567890123456789012123456789012345678901
If you do want to speak to a person, a better approach is to call someone at 2
12345678901234567890123456789012123456789012345678901
2
12345678901234567890123456789012123456789012345678901
a local testing center. Since you may potentially give them your money, 2
12345678901234567890123456789012123456789012345678901
1 they are often more amenable to answering any questions you might have. 2
2
12345678901234567890123456789012123456789012345678901
2
12345678901234567890123456789012123456789012345678901
12345678901234567890123456789012123456789012345678901 2
12345678901234567890123456789012123456789012345678901 2
2
12345678901234567890123456789012123456789012345678901
The
GRE CAT Interface
2
12345678901234567890123456789012123456789012345678901
2
12345678901234567890123456789012123456789012345678901
The three simulated screen shots on pages 333, 335, and 336 show the 2
12345678901234567890123456789012123456789012345678901
2
12345678901234567890123456789012123456789012345678901
GRE CAT interface for the Analytical Writing sections, the Quantitative 2
12345678901234567890123456789012123456789012345678901
2345678901234567890123456789012123456789012345678901
12345678901234567890123456789012123456789012345678901
section, and the Verbal section. Lets first examine the features of the 2
2
12345678901234567890123456789012123456789012345678901
2
1 interface that are common to all exam sections.
12345678901234567890123456789012123456789012345678901 2
12345678901234567890123456789012123456789012345678901 2
2
12345678901234567890123456789012123456789012345678901
2
12345678901234567890123456789012123456789012345678901
2
12345678901234567890123456789012123456789012345678901
The
CAT
Title
Bar
2
12345678901234567890123456789012123456789012345678901
2
12345678901234567890123456789012123456789012345678901
A
dark
title
bar
will
appear
across
the
top
of
the
computer
screen
at
all
2345678901234567890123456789012123456789012345678901
12345678901234567890123456789012123456789012345678901 2
12345678901234567890123456789012123456789012345678901
times during all test sections. (You cannot hide this bar.) The CAT title bar 2
2
12345678901234567890123456789012123456789012345678901
2
12345678901234567890123456789012123456789012345678901
displays
three
items:
2
1
2345678901234567890123456789012123456789012345678901
12345678901234567890123456789012123456789012345678901 2
12345678901234567890123456789012123456789012345678901
Left corner: The time remaining for the current section (hours and 2
12345678901234567890123456789012123456789012345678901 2
2
12345678901234567890123456789012123456789012345678901
minutes)
2
1
2345678901234567890123456789012123456789012345678901
2
12345678901234567890123456789012123456789012345678901
Middle: The name of the test (GRE) and current section number
2
1
2
12345678901234567890123456789012123456789012345678901
2
12345678901234567890123456789012123456789012345678901

Right
corner:
The
current
question
number
and
total
number
of
2
12345678901234567890123456789012123456789012345678901
2345678901234567890123456789012123456789012345678901
2
1
questions in the current section
2
12345678901234567890123456789012123456789012345678901
12345678901234567890123456789012123456789012345678901 2
12345678901234567890123456789012123456789012345678901 2
123456789012345678901234567890121234567890123456789012
332

http://www.xtremepapers.net

About the CAT

123456789012345678901234567890121234567890123456789012
12345678901234567890123456789012123456789012345678901 2
2
12345678901234567890123456789012123456789012345678901
2
12345678901234567890123456789012123456789012345678901
Name and Number
2
12345678901234567890123456789012123456789012345678901
CAT Title Bar
of Text Section
Time elapsed
2
12345678901234567890123456789012123456789012345678901
12345678901234567890123456789012123456789012345678901 2
2
12345678901234567890123456789012123456789012345678901
12345678901234567890123456789012123456789012345678901 2
2
12345678901234567890123456789012123456789012345678901
2
12345678901234567890123456789012123456789012345678901
2
12345678901234567890123456789012123456789012345678901
2
12345678901234567890123456789012123456789012345678901
2345678901234567890123456789012123456789012345678901
2
1
The following appeared in a memo from the manager of UpperCuts, a hair salon
2
12345678901234567890123456789012123456789012345678901
located in a suburb of the city of Apton, to the salon's owner:
2
12345678901234567890123456789012123456789012345678901
2
12345678901234567890123456789012123456789012345678901
"According
to
a
nationwide
demographic
study,
more
and
more
people
today
are
2
12345678901234567890123456789012123456789012345678901
AWA Topic
moving
from
suburbs
to
downtown
areas.
So
in
order
to
boost
sagging
profits
at
2345678901234567890123456789012123456789012345678901
2
1
UpperCuts we should relocate the salon from its current location in Apton's
2
12345678901234567890123456789012123456789012345678901
2
12345678901234567890123456789012123456789012345678901
suburban mall to downtown Apton, while retaining the salon's decidedly upscale
2
12345678901234567890123456789012123456789012345678901
approach in terms of services, products and pricing. After all, HairDooz, our chief
2
12345678901234567890123456789012123456789012345678901
competitor at the mall, has just relocated downtown and is populr among
2345678901234567890123456789012123456789012345678901
2
1
2
12345678901234567890123456789012123456789012345678901
2
12345678901234567890123456789012123456789012345678901
The AWA
2
12345678901234567890123456789012123456789012345678901
2
12345678901234567890123456789012123456789012345678901
Editing
2345678901234567890123456789012123456789012345678901
2
12345678901234567890123456789012123456789012345678901
Screen
2
12345678901234567890123456789012123456789012345678901
12345678901234567890123456789012123456789012345678901 22
12345678901234567890123456789012123456789012345678901 2
12345678901234567890123456789012123456789012345678901 2
12345678901234567890123456789012123456789012345678901 2
1
2
12345678901234567890123456789012123456789012345678901
?
2
12345678901234567890123456789012123456789012345678901
2
12345678901234567890123456789012123456789012345678901
2345678901234567890123456789012123456789012345678901
2
12345678901234567890123456789012123456789012345678901
Next
Confirm
Help
Quit
Time
Exit
2
12345678901234567890123456789012123456789012345678901
button Answer button
Test Selection button
2
12345678901234567890123456789012123456789012345678901
button
button button
2
12345678901234567890123456789012123456789012345678901
12345678901234567890123456789012123456789012345678901 22
12345678901234567890123456789012123456789012345678901 2
12345678901234567890123456789012123456789012345678901 2
12345678901234567890123456789012123456789012345678901 2
12345678901234567890123456789012123456789012345678901
2
1The CAT Toolbar
2
12345678901234567890123456789012123456789012345678901
A series of six buttons appear in a toolbar across the bottom of the 2
12345678901234567890123456789012123456789012345678901
12345678901234567890123456789012123456789012345678901
computer screen at all times during all test sections. (You cannot hide the 2
2
12345678901234567890123456789012123456789012345678901
2
12345678901234567890123456789012123456789012345678901
toolbar.)
Heres
a
description
of
each
buttons
function:
2345678901234567890123456789012123456789012345678901
12345678901234567890123456789012123456789012345678901 22
12345678901234567890123456789012123456789012345678901 2
1
QUIT TEST
2
12345678901234567890123456789012123456789012345678901
2
12345678901234567890123456789012123456789012345678901
Click
on
this
button
to
stop
the
test
and
cancel
your
scores
for
the
2345678901234567890123456789012123456789012345678901
12345678901234567890123456789012123456789012345678901 22
12345678901234567890123456789012123456789012345678901
entire test. (Partial score cancellation is not allowed in any event.) 2
1
12345678901234567890123456789012123456789012345678901
If
you click here, a dialog box will appear on the screen, asking you 2
2
12345678901234567890123456789012123456789012345678901
2345678901234567890123456789012123456789012345678901
12345678901234567890123456789012123456789012345678901
to confirm this operation. Stay away from this button unless youre 2
2
12345678901234567890123456789012123456789012345678901
12345678901234567890123456789012123456789012345678901
absolutely sure you wish your GRE score for the day to vaporize 2
12345678901234567890123456789012123456789012345678901 22
1
and youre willing to throw away your GRE registration fee.
2
12345678901234567890123456789012123456789012345678901
2
12345678901234567890123456789012123456789012345678901
EXIT SECTION
2
12345678901234567890123456789012123456789012345678901
2
12345678901234567890123456789012123456789012345678901
Click
on
this
button
if
you
finish
the
section
before
the
allotted
12345678901234567890123456789012123456789012345678901 2
2345678901234567890123456789012123456789012345678901
12345678901234567890123456789012123456789012345678901
time expires and wish to proceed immediately to the next section. 2
2
12345678901234567890123456789012123456789012345678901
2
12345678901234567890123456789012123456789012345678901
A
dialog
box
will
appear
on
the
screen
asking
you
to
confirm
this
12345678901234567890123456789012123456789012345678901 22
1
operation. Stay away from this button unless youve already 2
12345678901234567890123456789012123456789012345678901
12345678901234567890123456789012123456789012345678901
answered every question in the current section and dont feel you 2
2
12345678901234567890123456789012123456789012345678901
2
12345678901234567890123456789012123456789012345678901
need
a
breather
before
starting
the
next
one!
2
12345678901234567890123456789012123456789012345678901
2345678901234567890123456789012123456789012345678901
2
1
12345678901234567890123456789012123456789012345678901 2
12345678901234567890123456789012123456789012345678901 2
12345678901234567890123456789012123456789012345678901 2 333
123456789012345678901234567890121234567890123456789012
00:28

Computer-Adaptive GRE-Section 2:Analytical Writing 2

Beginning

The manager's argument relies on a series of unproven assumptions and is


therefore unconvincing as it stands. To begin with, the argument assumes that
Apton's demographic trend reflects the national trend. Yet, the mere fact that
one hair salon has moved downtown hardly suffices to |

Cut

Paste

Undo

Test

Section

Quit

Exit

Answer

Time

http://www.xtremepapers.net

Help

Confirm

Next

Part V: Appendix

123456789012345678901234567890121234567890123456789012
12345678901234567890123456789012123456789012345678901 2
2
12345678901234567890123456789012123456789012345678901
TIME
2
12345678901234567890123456789012123456789012345678901
2
12345678901234567890123456789012123456789012345678901
Click on this button to display the time remaining to the nearest 2
12345678901234567890123456789012123456789012345678901
2
12345678901234567890123456789012123456789012345678901
second. By default, the time remaining is displayed (in the upper 2
12345678901234567890123456789012123456789012345678901
2345678901234567890123456789012123456789012345678901
2
1
left corner) in hours and minutes, but not to the nearest second.
2
12345678901234567890123456789012123456789012345678901
2
12345678901234567890123456789012123456789012345678901
2
12345678901234567890123456789012123456789012345678901
HELP
2
12345678901234567890123456789012123456789012345678901
2345678901234567890123456789012123456789012345678901
1
Click on this button to access the directions for the current 2
2
12345678901234567890123456789012123456789012345678901
2
12345678901234567890123456789012123456789012345678901
question
type
(for
example,
Data
Sufficiency
or
Sentence
Comple2
12345678901234567890123456789012123456789012345678901
2
12345678901234567890123456789012123456789012345678901
tion),
as
well
as
the
general
test
directions
and
the
instructions
for
2345678901234567890123456789012123456789012345678901
2
1
2
12345678901234567890123456789012123456789012345678901
using
the
toolbar
items.
2
12345678901234567890123456789012123456789012345678901
2
12345678901234567890123456789012123456789012345678901
2
12345678901234567890123456789012123456789012345678901
NEXT and CONFIRM ANSWER
2345678901234567890123456789012123456789012345678901
2
1
12345678901234567890123456789012123456789012345678901
Click on the NEXT button when youre finished with the current 2
2
12345678901234567890123456789012123456789012345678901
question. When you click on NEXT, the current question will 2
12345678901234567890123456789012123456789012345678901
2
12345678901234567890123456789012123456789012345678901
remain on the screen until you click on CONFIRM ANSWER. 2
12345678901234567890123456789012123456789012345678901
2
12345678901234567890123456789012123456789012345678901
Until you confirm, you can change your answer as often as you 2
12345678901234567890123456789012123456789012345678901
2
12345678901234567890123456789012123456789012345678901
wish (by clicking on a different oval). But once you confirm, the 2
12345678901234567890123456789012123456789012345678901
2345678901234567890123456789012123456789012345678901
2
12345678901234567890123456789012123456789012345678901
question disappears forever and the next one appears in its place. 2
1
12345678901234567890123456789012123456789012345678901
Whenever the NEXT button is enabled (appearing dark gray), the 2
2
12345678901234567890123456789012123456789012345678901
2
12345678901234567890123456789012123456789012345678901
CONFIRM
ANSWER
button
is
disabled
(appearing
light
gray),
2345678901234567890123456789012123456789012345678901
12345678901234567890123456789012123456789012345678901 2
2
12345678901234567890123456789012123456789012345678901
and vice versa.
2
12345678901234567890123456789012123456789012345678901
12345678901234567890123456789012123456789012345678901 2
12345678901234567890123456789012123456789012345678901 2
12345678901234567890123456789012123456789012345678901 2
12345678901234567890123456789012123456789012345678901 2
2
12345678901234567890123456789012123456789012345678901
The Analytical Writing Screen
2
12345678901234567890123456789012123456789012345678901
1 As illustrated in the screen shot on page 333, the Analytical Writing 2
2345678901234567890123456789012123456789012345678901
12345678901234567890123456789012123456789012345678901 2
2
12345678901234567890123456789012123456789012345678901
1 prompt appears at the top of your screen, and your essay response appears 2
12345678901234567890123456789012123456789012345678901
below it as you type your response. (The screen in the figure includes the 2
2
12345678901234567890123456789012123456789012345678901
2345678901234567890123456789012123456789012345678901
12345678901234567890123456789012123456789012345678901
first several lines of a response.) Notice that you have to scroll down to 2
2
12345678901234567890123456789012123456789012345678901
1 read the entire topic and question. You compose your essays using the 2
2
12345678901234567890123456789012123456789012345678901
CAT word processor. (Just ahead, youll look closely at its features and 2
12345678901234567890123456789012123456789012345678901
2345678901234567890123456789012123456789012345678901
2
12345678901234567890123456789012123456789012345678901
limitations.)
2
12345678901234567890123456789012123456789012345678901
2
1
12345678901234567890123456789012123456789012345678901 2
12345678901234567890123456789012123456789012345678901 2
2
12345678901234567890123456789012123456789012345678901
2
12345678901234567890123456789012123456789012345678901
The
Quantitative
and
Verbal
Screens
2
12345678901234567890123456789012123456789012345678901
2
12345678901234567890123456789012123456789012345678901
To
respond
to
multiple-choice
questions,
click
on
one
of
the
ovals
to
12345678901234567890123456789012123456789012345678901 2
2345678901234567890123456789012123456789012345678901
12345678901234567890123456789012123456789012345678901
the left of the answer choices. You cant use the keyboard to select 2
2
12345678901234567890123456789012123456789012345678901
2
12345678901234567890123456789012123456789012345678901
answers.
Notice
that
the
answer
choices
are
not
lettered;
youll
click
on
12345678901234567890123456789012123456789012345678901 2
2
1 blank ovals.
2
12345678901234567890123456789012123456789012345678901
2
12345678901234567890123456789012123456789012345678901
2
12345678901234567890123456789012123456789012345678901
2
12345678901234567890123456789012123456789012345678901
12345678901234567890123456789012123456789012345678901 2
2345678901234567890123456789012123456789012345678901
12345678901234567890123456789012123456789012345678901 2
2
1
2
12345678901234567890123456789012123456789012345678901
12345678901234567890123456789012123456789012345678901 2
2
12345678901234567890123456789012123456789012345678901
12345678901234567890123456789012123456789012345678901 2
12345678901234567890123456789012123456789012345678901 2
12345678901234567890123456789012123456789012345678901 2
2
334 12345678901234567890123456789012123456789012345678901
123456789012345678901234567890121234567890123456789012

www.petersons.com

http://www.xtremepapers.net

X-Ref

About the CAT

123456789012345678901234567890121234567890123456789012
12345678901234567890123456789012123456789012345678901 2
2
12345678901234567890123456789012123456789012345678901
12345678901234567890123456789012123456789012345678901 2
2
12345678901234567890123456789012123456789012345678901
2
12345678901234567890123456789012123456789012345678901
12345678901234567890123456789012123456789012345678901 2
2
12345678901234567890123456789012123456789012345678901
In the sample questions throughout this book, the answer choices were
2
12345678901234567890123456789012123456789012345678901
2
12345678901234567890123456789012123456789012345678901
lettered for easy reference to corresponding explanations.
2
12345678901234567890123456789012123456789012345678901
2
12345678901234567890123456789012123456789012345678901
2
12345678901234567890123456789012123456789012345678901
2345678901234567890123456789012123456789012345678901
2
1
2
12345678901234567890123456789012123456789012345678901
2
12345678901234567890123456789012123456789012345678901
Split screens. For some multiple-choice questions, the screen splits 2
12345678901234567890123456789012123456789012345678901
2
12345678901234567890123456789012123456789012345678901
either horizontally or vertically.
2
12345678901234567890123456789012123456789012345678901
2
12345678901234567890123456789012123456789012345678901
2
12345678901234567890123456789012123456789012345678901
Reading
Comprehension:
The
screen
splits
vertically.
The
left
side
2
12345678901234567890123456789012123456789012345678901
2
12345678901234567890123456789012123456789012345678901
displays
the
passage;
the
right
side
displays
the
question
and
2345678901234567890123456789012123456789012345678901
2
1
2
12345678901234567890123456789012123456789012345678901
answer
choices.
2
12345678901234567890123456789012123456789012345678901
2
12345678901234567890123456789012123456789012345678901
2
12345678901234567890123456789012123456789012345678901
Quantitative
questions
that
include
figures:
The
screen
splits
2345678901234567890123456789012123456789012345678901
12345678901234567890123456789012123456789012345678901 22
12345678901234567890123456789012123456789012345678901
horizontally. The figures appears at the top; the question and 2
12345678901234567890123456789012123456789012345678901
2
12345678901234567890123456789012123456789012345678901
answer choices appear at the bottom.
2
12345678901234567890123456789012123456789012345678901
2
12345678901234567890123456789012123456789012345678901
Vertical Scrolling. For some multiple-choice questions, youll have to 2
1
2
12345678901234567890123456789012123456789012345678901
scroll up and down (using the vertical scroll bar) to view all the 2
12345678901234567890123456789012123456789012345678901
2
12345678901234567890123456789012123456789012345678901
material that pertains to the current question.
2
12345678901234567890123456789012123456789012345678901
2
12345678901234567890123456789012123456789012345678901
2
12345678901234567890123456789012123456789012345678901
Reading
Comprehension:
Passages
are
too
long
for
you
to
see
on
2
12345678901234567890123456789012123456789012345678901
2
12345678901234567890123456789012123456789012345678901
the
screen
in
their
entirety;
youll
need
to
scroll.
2345678901234567890123456789012123456789012345678901
12345678901234567890123456789012123456789012345678901 22
12345678901234567890123456789012123456789012345678901 2
12345678901234567890123456789012123456789012345678901
Quantitative questions that include figures: Some figures 2
12345678901234567890123456789012123456789012345678901
1
especially charts and graphswont fit on the screen in their 2
2
12345678901234567890123456789012123456789012345678901
2
12345678901234567890123456789012123456789012345678901
entirety; you might need to scroll.
12345678901234567890123456789012123456789012345678901 2
12345678901234567890123456789012123456789012345678901 2
2
12345678901234567890123456789012123456789012345678901
01:06
Computer-Adaptive GRE-Section 3:Quantitative
21 of 37
2
12345678901234567890123456789012123456789012345678901
2
12345678901234567890123456789012123456789012345678901
12345678901234567890123456789012123456789012345678901 2
2
12345678901234567890123456789012123456789012345678901
2
12345678901234567890123456789012123456789012345678901
2345678901234567890123456789012123456789012345678901
12345678901234567890123456789012123456789012345678901 22
12345678901234567890123456789012123456789012345678901 2
1
12345678901234567890123456789012123456789012345678901 2
12345678901234567890123456789012123456789012345678901 2
2
12345678901234567890123456789012123456789012345678901
2
12345678901234567890123456789012123456789012345678901
2
12345678901234567890123456789012123456789012345678901
2
12345678901234567890123456789012123456789012345678901
12345678901234567890123456789012123456789012345678901 2
2345678901234567890123456789012123456789012345678901
12345678901234567890123456789012123456789012345678901 22
12345678901234567890123456789012123456789012345678901 2
12345678901234567890123456789012123456789012345678901 2
12345678901234567890123456789012123456789012345678901 2
1
2
12345678901234567890123456789012123456789012345678901
2
12345678901234567890123456789012123456789012345678901
2
12345678901234567890123456789012123456789012345678901
2
12345678901234567890123456789012123456789012345678901
12345678901234567890123456789012123456789012345678901 2
2345678901234567890123456789012123456789012345678901
12345678901234567890123456789012123456789012345678901 22
1
2
12345678901234567890123456789012123456789012345678901
Test Section
2
12345678901234567890123456789012123456789012345678901
? Answer
2
12345678901234567890123456789012123456789012345678901
2345678901234567890123456789012123456789012345678901
2
1
12345678901234567890123456789012123456789012345678901 2
12345678901234567890123456789012123456789012345678901 2
12345678901234567890123456789012123456789012345678901 2 335
123456789012345678901234567890121234567890123456789012
Richard began driving from home on a trip averaging 30 miles per hour. How
many miles per hour must Carla drive on average to catch up to him in exactly
3 hours if she leaves 30 minutes after Richard?
35
55
39
40
60

Quit

Exit

Time

http://www.xtremepapers.net

Help

Confirm

Next

Part V: Appendix

123456789012345678901234567890121234567890123456789012
12345678901234567890123456789012123456789012345678901 2
2
12345678901234567890123456789012123456789012345678901
12345678901234567890123456789012123456789012345678901 2
2
12345678901234567890123456789012123456789012345678901
01:09
Computer-Adaptive GRE-Section 4:Verbal
6 of 41
2
12345678901234567890123456789012123456789012345678901
2
12345678901234567890123456789012123456789012345678901
Questions
6
to
8
Beginning
2
12345678901234567890123456789012123456789012345678901
2345678901234567890123456789012123456789012345678901
2
1
2
12345678901234567890123456789012123456789012345678901
2
12345678901234567890123456789012123456789012345678901
2
12345678901234567890123456789012123456789012345678901
2
12345678901234567890123456789012123456789012345678901
2345678901234567890123456789012123456789012345678901
2
1
2
12345678901234567890123456789012123456789012345678901
2
12345678901234567890123456789012123456789012345678901
2
12345678901234567890123456789012123456789012345678901
2
12345678901234567890123456789012123456789012345678901
2345678901234567890123456789012123456789012345678901
2
1
2
12345678901234567890123456789012123456789012345678901
2
12345678901234567890123456789012123456789012345678901
2
12345678901234567890123456789012123456789012345678901
2
12345678901234567890123456789012123456789012345678901
2345678901234567890123456789012123456789012345678901
2
1
2
12345678901234567890123456789012123456789012345678901
2
12345678901234567890123456789012123456789012345678901
2
12345678901234567890123456789012123456789012345678901
2
12345678901234567890123456789012123456789012345678901
2345678901234567890123456789012123456789012345678901
12345678901234567890123456789012123456789012345678901 2
12345678901234567890123456789012123456789012345678901 2
12345678901234567890123456789012123456789012345678901 2
12345678901234567890123456789012123456789012345678901 2
2
12345678901234567890123456789012123456789012345678901
Test Section
? Answer
2
12345678901234567890123456789012123456789012345678901
2
1
2
12345678901234567890123456789012123456789012345678901
2
12345678901234567890123456789012123456789012345678901
2
12345678901234567890123456789012123456789012345678901
2345678901234567890123456789012123456789012345678901
12345678901234567890123456789012123456789012345678901 2
2
12345678901234567890123456789012123456789012345678901
The CATs Word Processor
2
12345678901234567890123456789012123456789012345678901
2
12345678901234567890123456789012123456789012345678901
During
the
two
GRE
essays
section,
youll
use
the
simple
word
processor
12345678901234567890123456789012123456789012345678901 2
12345678901234567890123456789012123456789012345678901
built into the CAT system. While the word processor includes some 2
2
12345678901234567890123456789012123456789012345678901
2
12345678901234567890123456789012123456789012345678901
features
standard
in
programs
like
Word
and
WordPerfect,
it
also
lacks
12345678901234567890123456789012123456789012345678901 2
2
1 many of these programs features.
2
12345678901234567890123456789012123456789012345678901
2
12345678901234567890123456789012123456789012345678901
12345678901234567890123456789012123456789012345678901 2
12345678901234567890123456789012123456789012345678901 2
2
12345678901234567890123456789012123456789012345678901
2
12345678901234567890123456789012123456789012345678901
Keyboard Commands for Navigation and Editing
2
12345678901234567890123456789012123456789012345678901
2
12345678901234567890123456789012123456789012345678901
Here
are
the
navigational
and
editing
keys
available
in
the
CAT
word
2
12345678901234567890123456789012123456789012345678901
2
12345678901234567890123456789012123456789012345678901
processor:
2345678901234567890123456789012123456789012345678901
12345678901234567890123456789012123456789012345678901 2
2
12345678901234567890123456789012123456789012345678901
Backspace removes the character to the left of the cursor.
2
1
12345678901234567890123456789012123456789012345678901 2
2
12345678901234567890123456789012123456789012345678901
Delete removes the character to the right of the cursor.
2
12345678901234567890123456789012123456789012345678901
2
12345678901234567890123456789012123456789012345678901
2
12345678901234567890123456789012123456789012345678901

Home
moves
the
cursor
to
the
beginning
of
the
line.
2
12345678901234567890123456789012123456789012345678901
12345678901234567890123456789012123456789012345678901 2
2345678901234567890123456789012123456789012345678901
2
12345678901234567890123456789012123456789012345678901
End moves the cursor to the end of the line.
2
12345678901234567890123456789012123456789012345678901
12345678901234567890123456789012123456789012345678901 2
2
12345678901234567890123456789012123456789012345678901
Arrow Keys move the cursor up, down, left, or right.
2
1
2345678901234567890123456789012123456789012345678901
2
12345678901234567890123456789012123456789012345678901
Enter inserts a paragraph break (starts a new line).
2
12345678901234567890123456789012123456789012345678901
12345678901234567890123456789012123456789012345678901 2
2
12345678901234567890123456789012123456789012345678901
Page Up moves the cursor up one page (screen).
2
1
2345678901234567890123456789012123456789012345678901
12345678901234567890123456789012123456789012345678901 2
2
1
Page Down moves the cursor down one page (screen).
2
12345678901234567890123456789012123456789012345678901
12345678901234567890123456789012123456789012345678901 2
2
12345678901234567890123456789012123456789012345678901
12345678901234567890123456789012123456789012345678901 2
12345678901234567890123456789012123456789012345678901 2
12345678901234567890123456789012123456789012345678901 2
2
336 12345678901234567890123456789012123456789012345678901
123456789012345678901234567890121234567890123456789012
The Andean cordillera is made up of
many interwoven mountain ranges, which
include high intermountain plateaus,
basins and valleys. The Northern Andes
(5) contain several broad ecosystems falling
into four altitudinal belts. Its northern subregion is distinguished from the rest of the
region by higher relative humidity and
greater climatic symmetry between the
(10) eastern and western flanks of the range.
The Central Andes are characterized by a
succession of agricultural zones with
varied climatic conditions along the
mountains' flanks and by large, high(15) altitude plateaus, variously called puna or
altiplano, which do not occur in the
Northern Andes. The soil fertility of the
northern altiplano is generally good. The
western Central Andean ranges are
(20) relatively arid with desert-like soils,
whereas the eastern ranges are more
humid and have more diverse soils. The
eastern slopes of the Central Andes in
many ways are similar to the wet forests

Quit

Exit

Time

In the passage, the author's primary concern is


to
describe the climate and topography of
various regions of the Andean cordillera
discuss the factors affecting the climate of
the Andean cordillera
suggest various alternative explanations for
the diversity of climate among the
various regions of the Andean cordillera
examine the effects of topograpy on the
climate and vegetation of the Andean
cordillera
compare and contrast the climate and
topography of the Northern Andes to that
the Central Andes

Help

Confirm

Next

www.petersons.com

http://www.xtremepapers.net

About the CAT

123456789012345678901234567890121234567890123456789012
12345678901234567890123456789012123456789012345678901 2
2
12345678901234567890123456789012123456789012345678901
Certain often-used features of standard word processing programs are not 2
12345678901234567890123456789012123456789012345678901
2
12345678901234567890123456789012123456789012345678901
available in the CAT word processor. For example, no keyboard 2
12345678901234567890123456789012123456789012345678901
2
12345678901234567890123456789012123456789012345678901
commands are available for:
2
12345678901234567890123456789012123456789012345678901
2345678901234567890123456789012123456789012345678901
2
1
2
12345678901234567890123456789012123456789012345678901

TABdisabled
(does
not
function)
2
12345678901234567890123456789012123456789012345678901
2
12345678901234567890123456789012123456789012345678901
2
12345678901234567890123456789012123456789012345678901

Beginning/end
of
paragraph
(not
available)
2345678901234567890123456789012123456789012345678901
2
1
2
12345678901234567890123456789012123456789012345678901
2
12345678901234567890123456789012123456789012345678901
Beginning/end of document (not available)
2
12345678901234567890123456789012123456789012345678901
2
12345678901234567890123456789012123456789012345678901

No
key
combinations
(using
the
CTRL,
ALT,
or
SHIFT
key)
or
other
2345678901234567890123456789012123456789012345678901
2
1
2
12345678901234567890123456789012123456789012345678901
so-called macros are available for editing functions. (Youll use 2
12345678901234567890123456789012123456789012345678901
2
12345678901234567890123456789012123456789012345678901
your mouse for cutting and pasting text.)
2
12345678901234567890123456789012123456789012345678901
2345678901234567890123456789012123456789012345678901
2
1
2
12345678901234567890123456789012123456789012345678901
2
12345678901234567890123456789012123456789012345678901
2
12345678901234567890123456789012123456789012345678901
2
12345678901234567890123456789012123456789012345678901
Mouse-Driven
Navigation
and
Editing
Functions
2345678901234567890123456789012123456789012345678901
12345678901234567890123456789012123456789012345678901 22
12345678901234567890123456789012123456789012345678901
Just as with other word processors, to navigate the editing screen you can 2
12345678901234567890123456789012123456789012345678901
12345678901234567890123456789012123456789012345678901
simply point the cursor to the position at which you wish to begin typing, 2
2
12345678901234567890123456789012123456789012345678901
2
12345678901234567890123456789012123456789012345678901
1 then click. The CAT word processor also includes mouse-driven CUT, 2
2
12345678901234567890123456789012123456789012345678901
PASTE, and UNDO.
2
12345678901234567890123456789012123456789012345678901
2
12345678901234567890123456789012123456789012345678901
2345678901234567890123456789012123456789012345678901
12345678901234567890123456789012123456789012345678901
Selecting text you wish to cut. You select text the same way as with 2
2
12345678901234567890123456789012123456789012345678901
standard word processing programs: either (1) hold down your mouse 2
12345678901234567890123456789012123456789012345678901
2
12345678901234567890123456789012123456789012345678901
button while sweeping the I-beam on the screen over the desired text, or 2
12345678901234567890123456789012123456789012345678901
2
12345678901234567890123456789012123456789012345678901
(2) hold down the SHIFT key and use the navigation keys to select text.
2
12345678901234567890123456789012123456789012345678901
12345678901234567890123456789012123456789012345678901 22
12345678901234567890123456789012123456789012345678901
1 The CUT Button. If you wish to delete text but want to save it to a 2
12345678901234567890123456789012123456789012345678901
temporary clipboard for pasting elsewhere, select that text (see above) then 2
2
12345678901234567890123456789012123456789012345678901
2
12345678901234567890123456789012123456789012345678901
click
on
the
CUT
button.
Cutting
text
is
not
the
same
as
deleting
it.
When
12345678901234567890123456789012123456789012345678901 2
12345678901234567890123456789012123456789012345678901
you delete text (using the DELETE key), you cannot paste it elsewhere in 2
2
12345678901234567890123456789012123456789012345678901
2
12345678901234567890123456789012123456789012345678901
your
document
(but
see
UNDO
below).
12345678901234567890123456789012123456789012345678901 2
2
12345678901234567890123456789012123456789012345678901
12345678901234567890123456789012123456789012345678901
The PASTE button. If you wish to move text from one position to another, 2
2345678901234567890123456789012123456789012345678901
12345678901234567890123456789012123456789012345678901 22
12345678901234567890123456789012123456789012345678901
1 select and cut the text, then reposition your cursor where you want the text 2
2
12345678901234567890123456789012123456789012345678901
to go and click on the PASTE button.
2
12345678901234567890123456789012123456789012345678901
2345678901234567890123456789012123456789012345678901
2
12345678901234567890123456789012123456789012345678901
The UNDO button. Click on this button to undo the most recent delete, 2
12345678901234567890123456789012123456789012345678901
2
12345678901234567890123456789012123456789012345678901
cut, or paste that you performed.
2
12345678901234567890123456789012123456789012345678901
2
1
2345678901234567890123456789012123456789012345678901
12345678901234567890123456789012123456789012345678901
Limitations of CUT and UNDO. The following mouse-driven features are 2
2
12345678901234567890123456789012123456789012345678901
2
12345678901234567890123456789012123456789012345678901
not
available:
12345678901234567890123456789012123456789012345678901 22
1
2
12345678901234567890123456789012123456789012345678901
DRAG-AND-DROP cut-and-paste (not available)
2
12345678901234567890123456789012123456789012345678901
2
12345678901234567890123456789012123456789012345678901
12345678901234567890123456789012123456789012345678901
COPY (not available; to copy you need to cut, then paste, in the 2
2
12345678901234567890123456789012123456789012345678901
same spot)
2
12345678901234567890123456789012123456789012345678901
12345678901234567890123456789012123456789012345678901 2
12345678901234567890123456789012123456789012345678901
MULTIPLE UNDO (the CAT word processor stores only your most 2
2
12345678901234567890123456789012123456789012345678901
2
12345678901234567890123456789012123456789012345678901
recent
delete,
cut,
or
paste,
or
keyboard
entry.)
2345678901234567890123456789012123456789012345678901
2
1
12345678901234567890123456789012123456789012345678901 2
12345678901234567890123456789012123456789012345678901 2
12345678901234567890123456789012123456789012345678901 2 337
123456789012345678901234567890121234567890123456789012

http://www.xtremepapers.net

Part V: Appendix

www.petersons.com

http://www.xtremepapers.net

Note

123456789012345678901234567890121234567890123456789012
12345678901234567890123456789012123456789012345678901 2
2
12345678901234567890123456789012123456789012345678901
The vertical scroll bar. Once you key in ten lines or so, youll have to scroll 2
12345678901234567890123456789012123456789012345678901
2
12345678901234567890123456789012123456789012345678901
to view your entire response. A vertical scroll bar also appears to the right 2
12345678901234567890123456789012123456789012345678901
2
12345678901234567890123456789012123456789012345678901
of the AWA prompt. Be sure to scroll all the way down to make sure youve 2
12345678901234567890123456789012123456789012345678901
2345678901234567890123456789012123456789012345678901
2
1 read the entire prompt.
2
12345678901234567890123456789012123456789012345678901
2
12345678901234567890123456789012123456789012345678901
2
12345678901234567890123456789012123456789012345678901
Spell-checking,
grammar-checking,
fonts,
attributes,
hyphenation.
The
2
12345678901234567890123456789012123456789012345678901
2345678901234567890123456789012123456789012345678901
1 CAT word processor does not include a spell-checker or grammar-checker, 2
2
12345678901234567890123456789012123456789012345678901
2
12345678901234567890123456789012123456789012345678901
nor
does
it
allow
you
to
choose
typeface
or
point
size.
Neither
manual
nor
2
12345678901234567890123456789012123456789012345678901
2
12345678901234567890123456789012123456789012345678901
automatic
hyphenation
is
available.
Attributes
such
as
bold,
italics,
and
2345678901234567890123456789012123456789012345678901
2
1
2
12345678901234567890123456789012123456789012345678901
underlining
are
not
available.
2
12345678901234567890123456789012123456789012345678901
2
12345678901234567890123456789012123456789012345678901
2
12345678901234567890123456789012123456789012345678901
2345678901234567890123456789012123456789012345678901
2
1
2
12345678901234567890123456789012123456789012345678901
As
for
words
that
you
would
otherwise
italicize
or
underline
(such
as
2
12345678901234567890123456789012123456789012345678901
2
12345678901234567890123456789012123456789012345678901
titles
or
non-English
words),
its
okay
to
leave
them
as
is.
The
readers
2
12345678901234567890123456789012123456789012345678901
2345678901234567890123456789012123456789012345678901
2
12345678901234567890123456789012123456789012345678901
understand the limitations of the CAT word processor.
2
12345678901234567890123456789012123456789012345678901
12345678901234567890123456789012123456789012345678901 2
12345678901234567890123456789012123456789012345678901 2
12345678901234567890123456789012123456789012345678901 2
2
12345678901234567890123456789012123456789012345678901
The
2
1 GRE CAT Test-Taking Experience
2
12345678901234567890123456789012123456789012345678901
12345678901234567890123456789012123456789012345678901
When you take a test as important as the GRE, its a good idea to minimize 2
2
12345678901234567890123456789012123456789012345678901
12345678901234567890123456789012123456789012345678901
test anxiety by knowing exactly what to expect on exam dayaside from 2
2
12345678901234567890123456789012123456789012345678901
12345678901234567890123456789012123456789012345678901
the timed test itself. Lets walk you through the various pre-test and 2
2
12345678901234567890123456789012123456789012345678901
post-test procedures and describe the physical testing environment.
2
12345678901234567890123456789012123456789012345678901
2345678901234567890123456789012123456789012345678901
12345678901234567890123456789012123456789012345678901 2
12345678901234567890123456789012123456789012345678901 2
12345678901234567890123456789012123456789012345678901 2
12345678901234567890123456789012123456789012345678901 2
2
1When You Arrive at the Test Center
2
12345678901234567890123456789012123456789012345678901
2
12345678901234567890123456789012123456789012345678901
Heres
what
you
can
expect
when
you
arrive
at
the
test
center:
12345678901234567890123456789012123456789012345678901 2
12345678901234567890123456789012123456789012345678901 2
12345678901234567890123456789012123456789012345678901
The supervisor will show you a roster, which includes the names 2
2
12345678901234567890123456789012123456789012345678901
2
12345678901234567890123456789012123456789012345678901
of
test-takers
scheduled
for
that
day,
and
will
ask
you
to
initial
12345678901234567890123456789012123456789012345678901 2
12345678901234567890123456789012123456789012345678901
the roster next to your name, and indicate on the roster your 2
2
12345678901234567890123456789012123456789012345678901
2345678901234567890123456789012123456789012345678901
2
12345678901234567890123456789012123456789012345678901
arrival time.
2
12345678901234567890123456789012123456789012345678901
2
1
The supervisor will ask you to read a two-page list of testing 2
12345678901234567890123456789012123456789012345678901
2
12345678901234567890123456789012123456789012345678901
procedures and rules. (Ill cover all these rules in the pages 2
12345678901234567890123456789012123456789012345678901
2
12345678901234567890123456789012123456789012345678901
immediately ahead.)
2
12345678901234567890123456789012123456789012345678901
2
12345678901234567890123456789012123456789012345678901
2
12345678901234567890123456789012123456789012345678901

The
supervisor
will
give
you
a
Nondisclosure
Statement.
Youre
2345678901234567890123456789012123456789012345678901
12345678901234567890123456789012123456789012345678901 2
12345678901234567890123456789012123456789012345678901
to read the printed statement, then write the statement (in the space 2
2
12345678901234567890123456789012123456789012345678901
2
12345678901234567890123456789012123456789012345678901
provided
on
the
form)
and
sign
it.
In
the
statement,
you
agree
to
the
2
1
2345678901234567890123456789012123456789012345678901
12345678901234567890123456789012123456789012345678901
testing policies and rules, and you agree not to reproduce or disclose 2
2
12345678901234567890123456789012123456789012345678901
2
12345678901234567890123456789012123456789012345678901
any
of
the
actual
test
questions.
The
supervisor
will
not
permit
you
12345678901234567890123456789012123456789012345678901 2
1
to enter the exam room until youve written and signed the 2
2
12345678901234567890123456789012123456789012345678901
2
12345678901234567890123456789012123456789012345678901
statement.
2
12345678901234567890123456789012123456789012345678901
12345678901234567890123456789012123456789012345678901 2
2
12345678901234567890123456789012123456789012345678901
12345678901234567890123456789012123456789012345678901 2
12345678901234567890123456789012123456789012345678901 2
12345678901234567890123456789012123456789012345678901 2
2
338 12345678901234567890123456789012123456789012345678901
123456789012345678901234567890121234567890123456789012

About the CAT

123456789012345678901234567890121234567890123456789012
12345678901234567890123456789012123456789012345678901 2
2
12345678901234567890123456789012123456789012345678901
Youll probably have to sit in a waiting room until the supervisor 2
12345678901234567890123456789012123456789012345678901
2
12345678901234567890123456789012123456789012345678901
calls your name. A 5- to 10-minute wait beyond your scheduled 2
12345678901234567890123456789012123456789012345678901
2
12345678901234567890123456789012123456789012345678901
testing time is not uncommon. (Taking the GRE is like going to the 2
12345678901234567890123456789012123456789012345678901
2345678901234567890123456789012123456789012345678901
2
1
dentistin more than one respect!)
2
12345678901234567890123456789012123456789012345678901
2
12345678901234567890123456789012123456789012345678901
2
12345678901234567890123456789012123456789012345678901

The
supervisor
will
check
your
photo
identification.
(You
wont
be
2
12345678901234567890123456789012123456789012345678901
2345678901234567890123456789012123456789012345678901
1
permitted to take the test unless you have one acceptable form of 2
2
12345678901234567890123456789012123456789012345678901
2
12345678901234567890123456789012123456789012345678901
photo
identification
with
you.)
2
12345678901234567890123456789012123456789012345678901
2
12345678901234567890123456789012123456789012345678901
2345678901234567890123456789012123456789012345678901
1
The test center will provide a secure locker (free of charge) for 2
2
12345678901234567890123456789012123456789012345678901
2
12345678901234567890123456789012123456789012345678901
stowing your personal belongings during the test.
2
12345678901234567890123456789012123456789012345678901
2
12345678901234567890123456789012123456789012345678901

To
help
ensure
that
nobody
else
takes
any
part
of
the
exam
in
your
2345678901234567890123456789012123456789012345678901
2
1
2
12345678901234567890123456789012123456789012345678901
place, the supervisor will take a photograph of you.
2
12345678901234567890123456789012123456789012345678901
2
12345678901234567890123456789012123456789012345678901
2
12345678901234567890123456789012123456789012345678901

The
supervisor
might
give
you
some
rudimentary
tips
about
2345678901234567890123456789012123456789012345678901
12345678901234567890123456789012123456789012345678901 22
12345678901234567890123456789012123456789012345678901
managing your time during the exam. Just ignore the supervisors 2
12345678901234567890123456789012123456789012345678901
2
12345678901234567890123456789012123456789012345678901
tips, because they might not be good advice for you!
2
12345678901234567890123456789012123456789012345678901
12345678901234567890123456789012123456789012345678901 22
1
Before you enter the testing room, you must remove everything from 2
12345678901234567890123456789012123456789012345678901
2
12345678901234567890123456789012123456789012345678901
your pockets except your photo I.D. and locker key.
2
12345678901234567890123456789012123456789012345678901
2345678901234567890123456789012123456789012345678901
12345678901234567890123456789012123456789012345678901 22
12345678901234567890123456789012123456789012345678901
The supervisor will provide you with several pieces of scratch paper
2
12345678901234567890123456789012123456789012345678901
(stapled together), along with two pencils. These are the only items 2
12345678901234567890123456789012123456789012345678901
2
12345678901234567890123456789012123456789012345678901
youll have in hand as you enter the testing room.
2
12345678901234567890123456789012123456789012345678901
12345678901234567890123456789012123456789012345678901 22
12345678901234567890123456789012123456789012345678901 2
12345678901234567890123456789012123456789012345678901 2
1
2
12345678901234567890123456789012123456789012345678901
Testing Procedures and Rules
2
12345678901234567890123456789012123456789012345678901
2
12345678901234567890123456789012123456789012345678901

If
you
want
to
exit
the
testing
room
for
any
reason,
you
must
raise
your
12345678901234567890123456789012123456789012345678901 2
12345678901234567890123456789012123456789012345678901
hand and wait for the supervisor to come in and escort you from the 2
2
12345678901234567890123456789012123456789012345678901
2
12345678901234567890123456789012123456789012345678901
room.
(You
wont
be
able
to
pause
the
testing
clock
for
any
reason.)
12345678901234567890123456789012123456789012345678901 2
2
12345678901234567890123456789012123456789012345678901
2
12345678901234567890123456789012123456789012345678901
No guests are allowed in the waiting room during your test.
2345678901234567890123456789012123456789012345678901
12345678901234567890123456789012123456789012345678901 22
12345678901234567890123456789012123456789012345678901
2
1 No food or drink is allowed in the testing room.
12345678901234567890123456789012123456789012345678901 2
2
12345678901234567890123456789012123456789012345678901
No hats are allowed.
2
12345678901234567890123456789012123456789012345678901
2
12345678901234567890123456789012123456789012345678901
2
12345678901234567890123456789012123456789012345678901

You
must
sign
out
whenever
you
exit
the
testing
room.
2
12345678901234567890123456789012123456789012345678901
12345678901234567890123456789012123456789012345678901 2
2345678901234567890123456789012123456789012345678901
12345678901234567890123456789012123456789012345678901
You must sign in whenever you re-enter the testing room (the supervisor 2
2
12345678901234567890123456789012123456789012345678901
2
12345678901234567890123456789012123456789012345678901
will ask to see your photo I.D. each time).
12345678901234567890123456789012123456789012345678901 22
1 If you need more scratch paper during the exam, just raise your hand
2
12345678901234567890123456789012123456789012345678901
2
12345678901234567890123456789012123456789012345678901
and ask for it. The supervisor will happily replace your bundle with a 2
12345678901234567890123456789012123456789012345678901
2
12345678901234567890123456789012123456789012345678901
fresh one.
2
12345678901234567890123456789012123456789012345678901
2345678901234567890123456789012123456789012345678901
12345678901234567890123456789012123456789012345678901 22
1 The supervisor will replace your tired pencils with fresh, sharp ones
2
12345678901234567890123456789012123456789012345678901
2
12345678901234567890123456789012123456789012345678901
upon
your
request
anytime
during
the
exam
(just
raise
your
hand).
2
12345678901234567890123456789012123456789012345678901
2345678901234567890123456789012123456789012345678901
2
1
12345678901234567890123456789012123456789012345678901 2
12345678901234567890123456789012123456789012345678901 2
12345678901234567890123456789012123456789012345678901 2 339
123456789012345678901234567890121234567890123456789012

http://www.xtremepapers.net

Part V: Appendix

www.petersons.com

http://www.xtremepapers.net

Alert!

123456789012345678901234567890121234567890123456789012
12345678901234567890123456789012123456789012345678901 2
2
12345678901234567890123456789012123456789012345678901
2
12345678901234567890123456789012123456789012345678901
What You Should Know about the CAT Testing
2
12345678901234567890123456789012123456789012345678901
2
12345678901234567890123456789012123456789012345678901
Environment
12345678901234567890123456789012123456789012345678901 2
12345678901234567890123456789012123456789012345678901
Individual testing stations are like library carrels; theyre separated by 2
2
12345678901234567890123456789012123456789012345678901
2
12345678901234567890123456789012123456789012345678901
half-walls.
2
12345678901234567890123456789012123456789012345678901
2
12345678901234567890123456789012123456789012345678901

The
height
of
your
chairs
seat
will
be
adjustable
and
the
chair
will
2
12345678901234567890123456789012123456789012345678901
2345678901234567890123456789012123456789012345678901
2
1 swivel. Chairs at most testing centers have arms.
2
12345678901234567890123456789012123456789012345678901
2
12345678901234567890123456789012123456789012345678901
2
12345678901234567890123456789012123456789012345678901

Computer
monitors
are
generally
of
the
15-inch
variety.
You
can
adjust
2
12345678901234567890123456789012123456789012345678901
2345678901234567890123456789012123456789012345678901
1 contrast. If you notice any flickering, ask the supervisor to move you to 2
2
12345678901234567890123456789012123456789012345678901
2
12345678901234567890123456789012123456789012345678901
another
station.
(You
wont
be
able
to
tell
if
you
monitor
has
color
2
12345678901234567890123456789012123456789012345678901
2
12345678901234567890123456789012123456789012345678901
capability,
because
the
GRE
is
strictly
a
black-and-white
affair.)
2345678901234567890123456789012123456789012345678901
2
1
2
12345678901234567890123456789012123456789012345678901
2
12345678901234567890123456789012123456789012345678901
2
12345678901234567890123456789012123456789012345678901
2
12345678901234567890123456789012123456789012345678901
2345678901234567890123456789012123456789012345678901
12345678901234567890123456789012123456789012345678901
You cant change the size of the font on the screen, unless you specifically 2
2
12345678901234567890123456789012123456789012345678901
12345678901234567890123456789012123456789012345678901
request before the exam begins that a special ZOOMTEXT function be 2
12345678901234567890123456789012123456789012345678901 2
2
12345678901234567890123456789012123456789012345678901
made available to you.
2
12345678901234567890123456789012123456789012345678901
2
1
2
12345678901234567890123456789012123456789012345678901
2
12345678901234567890123456789012123456789012345678901
2
12345678901234567890123456789012123456789012345678901

If
your
mouse
has
two
buttons,
you
can
use
either
button
to
click
your
2345678901234567890123456789012123456789012345678901
12345678901234567890123456789012123456789012345678901 2
2
12345678901234567890123456789012123456789012345678901
way through the exam (both buttons serve the same function). Dont 2
12345678901234567890123456789012123456789012345678901
2
12345678901234567890123456789012123456789012345678901
expect that nifty wheel between buttons for easy scrolling, because 2
12345678901234567890123456789012123456789012345678901
2
12345678901234567890123456789012123456789012345678901
youre not going to get it. For all you gamers and laptop users, trackballs 2
12345678901234567890123456789012123456789012345678901
2
12345678901234567890123456789012123456789012345678901
are available, but only if you request one before you begin the test.
2
12345678901234567890123456789012123456789012345678901
2
1
2345678901234567890123456789012123456789012345678901
12345678901234567890123456789012123456789012345678901
Testing rooms are not soundproof. During your test, you might hear 2
2
12345678901234567890123456789012123456789012345678901
2
1 talking and other noise from outside the room.
12345678901234567890123456789012123456789012345678901 2
2
12345678901234567890123456789012123456789012345678901
12345678901234567890123456789012123456789012345678901
Expect the supervisor to escort other test-takers in and out of the room 2
2
12345678901234567890123456789012123456789012345678901
2
12345678901234567890123456789012123456789012345678901
during your test. Do your best to ignore this potential distraction.
2
12345678901234567890123456789012123456789012345678901
2
12345678901234567890123456789012123456789012345678901

If
the
testing
room
is
busy,
expect
to
hear
lots
of
mouse-clicking
during
2345678901234567890123456789012123456789012345678901
12345678901234567890123456789012123456789012345678901 2
2
12345678901234567890123456789012123456789012345678901
1 your test. Because the room is otherwise fairly quiet, the incessant 2
2
12345678901234567890123456789012123456789012345678901
mouse-clicking can become annoying!
2
12345678901234567890123456789012123456789012345678901
2345678901234567890123456789012123456789012345678901
12345678901234567890123456789012123456789012345678901 2
2
12345678901234567890123456789012123456789012345678901
Earplugs are available upon request.
2
12345678901234567890123456789012123456789012345678901
12345678901234567890123456789012123456789012345678901 2
2
1 Expect anything in terms of room temperature, so dress in layers.
2
12345678901234567890123456789012123456789012345678901
2
12345678901234567890123456789012123456789012345678901
12345678901234567890123456789012123456789012345678901
Youll be under continual audio and video surveillance. To guard 2
2
12345678901234567890123456789012123456789012345678901
12345678901234567890123456789012123456789012345678901
against cheating, and to record any irregularities or problems in the 2
2345678901234567890123456789012123456789012345678901
2
12345678901234567890123456789012123456789012345678901
testing room as they occur, the room is continually audiotaped and 2
12345678901234567890123456789012123456789012345678901
2
12345678901234567890123456789012123456789012345678901
videotaped. (Look for the cameras or two-way mirrors, then smile and 2
12345678901234567890123456789012123456789012345678901
2
1 wave!)
2
12345678901234567890123456789012123456789012345678901
12345678901234567890123456789012123456789012345678901 2
2
12345678901234567890123456789012123456789012345678901
12345678901234567890123456789012123456789012345678901 2
2
12345678901234567890123456789012123456789012345678901
12345678901234567890123456789012123456789012345678901 2
12345678901234567890123456789012123456789012345678901 2
12345678901234567890123456789012123456789012345678901 2
2
340 12345678901234567890123456789012123456789012345678901
123456789012345678901234567890121234567890123456789012

Alert!

About the CAT

123456789012345678901234567890121234567890123456789012
12345678901234567890123456789012123456789012345678901 2
2
12345678901234567890123456789012123456789012345678901
12345678901234567890123456789012123456789012345678901
Before You Begin the TestThe Computer Tutorial 2
2
12345678901234567890123456789012123456789012345678901
2
12345678901234567890123456789012123456789012345678901
Okay,
the
supervisor
has
just
escorted
you
into
the
inner
sanctum
and
to
12345678901234567890123456789012123456789012345678901 2
12345678901234567890123456789012123456789012345678901
your station, and has wished you luck. (Some supervisors have been 2
2
12345678901234567890123456789012123456789012345678901
12345678901234567890123456789012123456789012345678901
known to encourage test-takers to have fun!) Before you begin the test, 2
2
12345678901234567890123456789012123456789012345678901
12345678901234567890123456789012123456789012345678901
the CAT System will lead you through a tutorial which includes five 2
2
12345678901234567890123456789012123456789012345678901
2
12345678901234567890123456789012123456789012345678901
sections (each section steps you through a series of screens):
2
12345678901234567890123456789012123456789012345678901
2
12345678901234567890123456789012123456789012345678901
1. How to use the mouse (6 screens)
2
12345678901234567890123456789012123456789012345678901
2
12345678901234567890123456789012123456789012345678901
2345678901234567890123456789012123456789012345678901
2
1
2. How to select and change an answer (6 screens)
2
12345678901234567890123456789012123456789012345678901
2
12345678901234567890123456789012123456789012345678901
2
12345678901234567890123456789012123456789012345678901
3.
How
to
scroll
the
screen
display
up
and
down
(6
screens)
2
12345678901234567890123456789012123456789012345678901
2345678901234567890123456789012123456789012345678901
2
1
2
12345678901234567890123456789012123456789012345678901
4. How to use the toolbars (21 screens); here youll learn how to:
2
12345678901234567890123456789012123456789012345678901
2
12345678901234567890123456789012123456789012345678901
Quit the test.
2
12345678901234567890123456789012123456789012345678901
2345678901234567890123456789012123456789012345678901
12345678901234567890123456789012123456789012345678901 22
12345678901234567890123456789012123456789012345678901
Exit the current section.
2
12345678901234567890123456789012123456789012345678901
12345678901234567890123456789012123456789012345678901 22
12345678901234567890123456789012123456789012345678901
Access the directions.
2
12345678901234567890123456789012123456789012345678901
2
1
2
12345678901234567890123456789012123456789012345678901

Confirm
your
response
and
move
to
the
next
question.
2
12345678901234567890123456789012123456789012345678901
2
12345678901234567890123456789012123456789012345678901
2345678901234567890123456789012123456789012345678901
2
12345678901234567890123456789012123456789012345678901
5. How to use the AWA word processor features (14 screens)
2
12345678901234567890123456789012123456789012345678901
2
12345678901234567890123456789012123456789012345678901
Heres what you need to know about the CAT tutorial:
2
12345678901234567890123456789012123456789012345678901
12345678901234567890123456789012123456789012345678901 22
12345678901234567890123456789012123456789012345678901
You wont be able to skip any section or any screen during 2
12345678901234567890123456789012123456789012345678901
2
12345678901234567890123456789012123456789012345678901
the tutorial.
2
12345678901234567890123456789012123456789012345678901
2
1
2345678901234567890123456789012123456789012345678901
12345678901234567890123456789012123456789012345678901
As you progress, the system requires that you demonstrate 2
2
12345678901234567890123456789012123456789012345678901
2
1
competency
in
using
the
mouse,
selecting
and
confirming
answer
12345678901234567890123456789012123456789012345678901 2
12345678901234567890123456789012123456789012345678901
choices, and accessing the directions. So you cant begin taking the 2
2
12345678901234567890123456789012123456789012345678901
12345678901234567890123456789012123456789012345678901
actual test unless youve shown that you know how to use the 2
12345678901234567890123456789012123456789012345678901 2
12345678901234567890123456789012123456789012345678901
system. (Dont worry: no test-taker has ever flunked the CAT system 2
2
12345678901234567890123456789012123456789012345678901
2345678901234567890123456789012123456789012345678901
2
competency test.)
12345678901234567890123456789012123456789012345678901
12345678901234567890123456789012123456789012345678901 22
1
At the end of each tutorial section (series of screens), you can repeat 2
12345678901234567890123456789012123456789012345678901
12345678901234567890123456789012123456789012345678901
that section, at your option. But once you leave a section you cant 2
2
12345678901234567890123456789012123456789012345678901
2
12345678901234567890123456789012123456789012345678901
return
to
it.
2
12345678901234567890123456789012123456789012345678901
2
12345678901234567890123456789012123456789012345678901
12345678901234567890123456789012123456789012345678901 2
2345678901234567890123456789012123456789012345678901
12345678901234567890123456789012123456789012345678901 22
12345678901234567890123456789012123456789012345678901 2
12345678901234567890123456789012123456789012345678901
Dont choose to repeat any tutorial section. Why not? If you do, youll be
2
12345678901234567890123456789012123456789012345678901
2
1 forced to step through the entire sequence of screens in that section again
2345678901234567890123456789012123456789012345678901
12345678901234567890123456789012123456789012345678901 22
12345678901234567890123456789012123456789012345678901
(an aggravating time-waster, especially for the 21-screen section!)
2
12345678901234567890123456789012123456789012345678901
12345678901234567890123456789012123456789012345678901 22
1
2
12345678901234567890123456789012123456789012345678901
12345678901234567890123456789012123456789012345678901 2
2
12345678901234567890123456789012123456789012345678901
12345678901234567890123456789012123456789012345678901 2
2
12345678901234567890123456789012123456789012345678901
12345678901234567890123456789012123456789012345678901 2
12345678901234567890123456789012123456789012345678901 2
12345678901234567890123456789012123456789012345678901 2
12345678901234567890123456789012123456789012345678901 2 341
123456789012345678901234567890121234567890123456789012

http://www.xtremepapers.net

Part V: Appendix

http://www.xtremepapers.net

Alert!

www.petersons.com

Tip

123456789012345678901234567890121234567890123456789012
12345678901234567890123456789012123456789012345678901 2
2
12345678901234567890123456789012123456789012345678901
The Analytical Writing section of the tutorial allows you to practice 2
12345678901234567890123456789012123456789012345678901
2
12345678901234567890123456789012123456789012345678901
using the word processor.
2
12345678901234567890123456789012123456789012345678901
12345678901234567890123456789012123456789012345678901 2
12345678901234567890123456789012123456789012345678901
If you carefully read all the information presented to you, expect to 2
2
12345678901234567890123456789012123456789012345678901
2
12345678901234567890123456789012123456789012345678901
spend
about
20
minutes
on
the
tutorial.
2
12345678901234567890123456789012123456789012345678901
2
12345678901234567890123456789012123456789012345678901
2
12345678901234567890123456789012123456789012345678901
2345678901234567890123456789012123456789012345678901
1 On test day, youll already know how the CAT system works. So step 2
2
12345678901234567890123456789012123456789012345678901
2
12345678901234567890123456789012123456789012345678901
through
the
tutorial
as
quickly
as
you
can,
reading
as
little
as
possible.
2
12345678901234567890123456789012123456789012345678901
2
12345678901234567890123456789012123456789012345678901
You
can
easily
dispense
with
the
tutorial
in
510
minutes
this
way.
2345678901234567890123456789012123456789012345678901
2
1
2
12345678901234567890123456789012123456789012345678901
Remember:
The
less
time
you
spend
with
the
tutorial,
the
less
fatigued
2
12345678901234567890123456789012123456789012345678901
2
12345678901234567890123456789012123456789012345678901
youll
be
during
the
exam
itself.
2
12345678901234567890123456789012123456789012345678901
2345678901234567890123456789012123456789012345678901
2
1
2
12345678901234567890123456789012123456789012345678901
2
12345678901234567890123456789012123456789012345678901
Post-Test
GRE
CAT
Procedures
2
12345678901234567890123456789012123456789012345678901
2
12345678901234567890123456789012123456789012345678901
Okay,
its
been
about
4
hours
since
you
first
entered
the
testing
center,
and
2345678901234567890123456789012123456789012345678901
12345678901234567890123456789012123456789012345678901 2
2
12345678901234567890123456789012123456789012345678901
youve just completed the second of two multiple-choice GRE sections. You 2
12345678901234567890123456789012123456789012345678901
12345678901234567890123456789012123456789012345678901
may think youve finished the CAT, but the CAT has not quite finished with 2
2
12345678901234567890123456789012123456789012345678901
2
12345678901234567890123456789012123456789012345678901
1 you yet! There are a few more hoops to jump through before youre done. 2
2
12345678901234567890123456789012123456789012345678901
2
12345678901234567890123456789012123456789012345678901
1.
Respond
to
a
brief
questionnaire.
The
CAT
will
impose
on
you
a
2
12345678901234567890123456789012123456789012345678901
2345678901234567890123456789012123456789012345678901
12345678901234567890123456789012123456789012345678901
brief questionnaire (presented in a series of screens) about your 2
2
12345678901234567890123456789012123456789012345678901
2
12345678901234567890123456789012123456789012345678901
test-taking
experience
(believe
it
or
not,
these
questions
are
12345678901234567890123456789012123456789012345678901 2
12345678901234567890123456789012123456789012345678901
multiple-choice, just like the exam itself). The questionnaire 2
2
12345678901234567890123456789012123456789012345678901
2
12345678901234567890123456789012123456789012345678901
might ask you, for example:
12345678901234567890123456789012123456789012345678901 2
2
12345678901234567890123456789012123456789012345678901
Whether your supervisor was knowledgeable and helpful
2
1
2345678901234567890123456789012123456789012345678901
12345678901234567890123456789012123456789012345678901 2
2
12345678901234567890123456789012123456789012345678901
Whether the testing environment was comfortable
2
1
12345678901234567890123456789012123456789012345678901 2
12345678901234567890123456789012123456789012345678901
How long you waited after you arrived at the testing site to 2
2
12345678901234567890123456789012123456789012345678901
2
12345678901234567890123456789012123456789012345678901
begin
the
test
12345678901234567890123456789012123456789012345678901 2
2
12345678901234567890123456789012123456789012345678901
2
12345678901234567890123456789012123456789012345678901
Whether you were distracted by noise during your exam
2345678901234567890123456789012123456789012345678901
12345678901234567890123456789012123456789012345678901 2
2
12345678901234567890123456789012123456789012345678901
2. Cancel your test, at your option. The most important question 2
1
2
12345678901234567890123456789012123456789012345678901
youll answer while seated at your testing station is this next one. 2
12345678901234567890123456789012123456789012345678901
2345678901234567890123456789012123456789012345678901
2
12345678901234567890123456789012123456789012345678901
The CAT will ask you to choose whether to:
2
12345678901234567890123456789012123456789012345678901
12345678901234567890123456789012123456789012345678901 2
12345678901234567890123456789012123456789012345678901
Cancel your scores (no scores are recorded; partial 2
2
1
2345678901234567890123456789012123456789012345678901
12345678901234567890123456789012123456789012345678901
cancellation is not provided for) or see your scores 2
2
12345678901234567890123456789012123456789012345678901
2
12345678901234567890123456789012123456789012345678901
immediately.
12345678901234567890123456789012123456789012345678901 2
2
1
2
12345678901234567890123456789012123456789012345678901
2
12345678901234567890123456789012123456789012345678901
2
12345678901234567890123456789012123456789012345678901
2
12345678901234567890123456789012123456789012345678901
12345678901234567890123456789012123456789012345678901 2
2345678901234567890123456789012123456789012345678901
2
12345678901234567890123456789012123456789012345678901
1 If you click on the CANCEL SCORES button, the CAT will then give you 2
12345678901234567890123456789012123456789012345678901
yet another 5 minutes to think over your decision. So you really have 10 2
2
12345678901234567890123456789012123456789012345678901
2
12345678901234567890123456789012123456789012345678901
minutes altogether to make up your mind.
2345678901234567890123456789012123456789012345678901
2
1
12345678901234567890123456789012123456789012345678901 2
12345678901234567890123456789012123456789012345678901 2
2
342 12345678901234567890123456789012123456789012345678901
123456789012345678901234567890121234567890123456789012

Note

Tip

About the CAT

123456789012345678901234567890121234567890123456789012
12345678901234567890123456789012123456789012345678901 2
2
12345678901234567890123456789012123456789012345678901
Once you elect to see your scores, you can no longer cancel themever! So 2
12345678901234567890123456789012123456789012345678901
2
12345678901234567890123456789012123456789012345678901
you should take a few minutes to think it over. The CAT gives you 5 2
12345678901234567890123456789012123456789012345678901
2
12345678901234567890123456789012123456789012345678901
minutes to choose. If you havent decided within 5 minutes, the CAT will 2
12345678901234567890123456789012123456789012345678901
2345678901234567890123456789012123456789012345678901
1 automatically show you your scores (and you forfeit your option to 2
2
12345678901234567890123456789012123456789012345678901
2
12345678901234567890123456789012123456789012345678901
cancel).
2
12345678901234567890123456789012123456789012345678901
2
12345678901234567890123456789012123456789012345678901
2345678901234567890123456789012123456789012345678901
1
3. View and record your scores. If you elect to see your scores, you 2
2
12345678901234567890123456789012123456789012345678901
2
12345678901234567890123456789012123456789012345678901
should
write
them
down
on
your
scratch
paper.
When
you
leave
2
12345678901234567890123456789012123456789012345678901
2
12345678901234567890123456789012123456789012345678901
the
testing
room,
the
supervisor
will
allow
you
to
transcribe
them
2345678901234567890123456789012123456789012345678901
2
1
2
12345678901234567890123456789012123456789012345678901
onto
another
sheet
of
paper
(one
that
you
can
take
home
with
2
12345678901234567890123456789012123456789012345678901
2
12345678901234567890123456789012123456789012345678901
you),
so
that
you
dont
have
to
memorize
them.
2
12345678901234567890123456789012123456789012345678901
2345678901234567890123456789012123456789012345678901
2
1
12345678901234567890123456789012123456789012345678901
4. Direct your scores to the schools of your choice. Once youve 2
2
12345678901234567890123456789012123456789012345678901
elected to see your scores, the CAT will ask you to select the 2
12345678901234567890123456789012123456789012345678901
2
12345678901234567890123456789012123456789012345678901
schools you wish to receive your score report (the CAT provides a 2
12345678901234567890123456789012123456789012345678901
2
12345678901234567890123456789012123456789012345678901
complete list of schools).
2
12345678901234567890123456789012123456789012345678901
2
12345678901234567890123456789012123456789012345678901
2
12345678901234567890123456789012123456789012345678901
2345678901234567890123456789012123456789012345678901
2
12345678901234567890123456789012123456789012345678901
2
1 You can select as many as five schools at this timewithout incurring an
2
12345678901234567890123456789012123456789012345678901
additional fee. This is your last chance for a freebie, so you should take
2
12345678901234567890123456789012123456789012345678901
2
12345678901234567890123456789012123456789012345678901
full
advantage
of
it.
Be
sure
to
compile
your
list
of
five
schools
before
2345678901234567890123456789012123456789012345678901
12345678901234567890123456789012123456789012345678901 22
12345678901234567890123456789012123456789012345678901
exam day.
2
12345678901234567890123456789012123456789012345678901
12345678901234567890123456789012123456789012345678901 22
12345678901234567890123456789012123456789012345678901 2
12345678901234567890123456789012123456789012345678901
Before You Leave the Testing Center
2
12345678901234567890123456789012123456789012345678901
12345678901234567890123456789012123456789012345678901 22
12345678901234567890123456789012123456789012345678901
1 Upon exiting the testing room for the final time, the following will happen: 2
2
12345678901234567890123456789012123456789012345678901
1. The supervisor will collect your pencils and scratch paper, and 2
12345678901234567890123456789012123456789012345678901
2
12345678901234567890123456789012123456789012345678901
will count the number of sheets of paper to make sure you arent 2
12345678901234567890123456789012123456789012345678901
2
12345678901234567890123456789012123456789012345678901
trying to sneak out with any. (Then, if youre lucky, youll be 2
12345678901234567890123456789012123456789012345678901
12345678901234567890123456789012123456789012345678901
allowed to watch while the supervisor ceremoniously rips up your 2
2
12345678901234567890123456789012123456789012345678901
2
12345678901234567890123456789012123456789012345678901
scratch
paper
and
drops
it
in
the
trash
basket!)
2
12345678901234567890123456789012123456789012345678901
2345678901234567890123456789012123456789012345678901
12345678901234567890123456789012123456789012345678901 22
12345678901234567890123456789012123456789012345678901
2. The supervisor will remind you to collect your belongings from 2
1
2
12345678901234567890123456789012123456789012345678901
your locker (if you used one), and turn in your locker key.
2
12345678901234567890123456789012123456789012345678901
2345678901234567890123456789012123456789012345678901
12345678901234567890123456789012123456789012345678901 22
12345678901234567890123456789012123456789012345678901
3. The supervisor will provide you with an ETS pamphlet that
2
12345678901234567890123456789012123456789012345678901
12345678901234567890123456789012123456789012345678901
explains how to interpret your test scores. (You can take this 2
2
1
2
12345678901234567890123456789012123456789012345678901
home with you.)
2
12345678901234567890123456789012123456789012345678901
2
12345678901234567890123456789012123456789012345678901
2
12345678901234567890123456789012123456789012345678901
12345678901234567890123456789012123456789012345678901 2
2345678901234567890123456789012123456789012345678901
2
12345678901234567890123456789012123456789012345678901
The supervisor might also provide you with a postcard-sized invitation to
2
12345678901234567890123456789012123456789012345678901
2
12345678901234567890123456789012123456789012345678901
blow the whistle on anybody you suspect of cheating on the exam (the
12345678901234567890123456789012123456789012345678901 22
1 invitation ends with the assurance: Confidentiality guaranteed).
2
12345678901234567890123456789012123456789012345678901
12345678901234567890123456789012123456789012345678901 2
2
12345678901234567890123456789012123456789012345678901
12345678901234567890123456789012123456789012345678901 2
2
12345678901234567890123456789012123456789012345678901
12345678901234567890123456789012123456789012345678901 2
12345678901234567890123456789012123456789012345678901 2
12345678901234567890123456789012123456789012345678901 2
12345678901234567890123456789012123456789012345678901 2 343
123456789012345678901234567890121234567890123456789012

http://www.xtremepapers.net

You might also like